You are on page 1of 1044

P^ukJ^ee^f^

FUNDAMENTAL

ww
PHYSICS
Flo
Vol.I

e
eree
FFr
Strictly according to new syllabus & examination pattern of CBSE, ISCE and state boards of Punjab, Haryana,
H.R, J & K, Odisha, Karnataka, Kerala, Nagaland, Manipur, Rajasthan, Jharkhand, Bihar, U.P.,
uurr
orr
Uttarakhand, M.R, Chhatisgarh, Assam, Gujarat, West Bengal, A.R etc,
for class XII of+2 stage.
sfo
kks
Yoo
oooo

Dr. K.L GOMBER K.L. GOGIA


eBB

M.Sc.
M-Sc„ M. Phil. (Gold Medalist). Ph.D.
Formerly Head of Physics Department Formerly Head of Physics Department
D.A.V. College, AMBALA CITY S.A. Jain College, AMBALA CITY
Formerly Post Doctoral Research Associate Ex. Member, Board of Studies in
uurr

State University of New York. Buffalo Physics & Science Faculty


' K.U., KURUKSHETRA
ad

NEW YORK, U.S.A.


Yo
dY

/ ●'
innd
Re

; /
Fi

■tf..
2023-2024

' j

Pradeep Publications
JALANDHAR (INDIA)
Contents
Unit -1 [Electrostatics]
Chapters Pages

1. Electrostatic Charges and Fields 1/1—1/134


1.1. Introduction l/l
1/2
1.2. Electric Charge
1.3. Two Kinds of Charges 1/2

1.4. Conductors, Insulators and Dielectrics 1/4

ww
1/5
1.5. Gold Leaf Electroscope (OLE)
1/5
1.6. Origin of Electric Charge in Electrostatics

Floo
1/6
*1.7. Charging by Induction
1/8
1.8. Quantization of Electric Charge

ee
1.9. Additivity of Charge 1/9

reer
rFF
1/10
1.10. Conservation of Charge
1/10
l.ll. Comparison ofCharge and Mass
uur r
1.12. Coulomb’s Law 1/11

1.13. Coulomb’s Law in VectorForm ffoor 1/12


sks
1/14
1.14. UnitsofCharge
YYoo
1.15. Dielectric Constant or Relative Electrical Permittivity 1/14
ooko

1/15
1.16. Forces between Multiple Charges: Principle of Superposition
eBB

1/16
1.17. Continuous Charge Distribution
1.18. Force Due to Continuous Distribution ofCharges 1/17
r

1/18
1.19. Concept of Electric Field
ouur
ad

1/18
1.20. Electric Field Intensity
Y

1/19
1.21. Electric Field Intensity due to a Point Charge
dY

1/20
1.22. Electric Field Intensity due to a Group ofCharges
Re

1/21
1.23. Electric Field Intensity due to Continuous Charge Distribution
nnd
FFii

1/21
1.24. Rectangular Components of Electric Intensity due to a Point Charge
1/22
1.25. Physical Significance ofElectric Field
1.26. Electric Field Lines 1/22
1/23
1.27. Properties ofElectric Field Lines
1/24
1.28. Electric Dipole
1/25
1.29. Dipole Moment
1/26
1.30. Dipole Field
1/26
1.31. Field Intensity on Axial Line ofElectric Dipole
1/27
1.32. Field Intensity on Equatorial Line ofElectric Dipole
1/28
1.33. Electric Field Intensity at any Point due to a Short Electric Dipole
* Not included in CBSE syllabus. This article has been given only for the preparation ofcompetitive examinations.
1.34. Electric Field Intensity at any Point on the axis of a Unifonnly Charged Ring 1/29

1.35. Electric Dipole in a Unifonn Two Dimensional Electric Field 1/30


1.36. Potential Energy of Dipole in a Uniform Electric Field 1/32
1.37. Area Vector 1/33
1.38. Electric Flux 1/33
1.39. Gauss’s Theorem or Gauss’s Law in Electrostatics 1/34
1.40. Simple Verification of Gauss’s Theorem 1/37
1.41. Deduction of Coulomb’s Law from Gauss’s Theorem 1/38

1.42. Field due to an Infinitely Long Straight Uniformly Charged Wire 1/38

1.43. Electric Field Intensity due to a Uniformly Charged Spherical Shell 1/40

1.44. Electric Field Intensity due to a Non-conducting Charged Solid Sphere 1/41

1.45. Electric Field Intensity due to a Thin Infinite Plane Sheet of Charge 1/42

w
1.46. Electric Field Intensity due to Two Thin Infinite Plane Parallel Sheets of Charge 1/43
Solved Examples 1/44

Flo
Revision At A Glance 1/60

e
Multiple Choice Questions 1/62

reee
Assertion-Reason Type Questions 1/65

FFr
Conceptual Problems 1/69
Very Short Answer Questions [I Mark] 1/75

for
ur
Short Answer Questions [2 Marks] 1/78
Short Answer Questions [3 Marks] 1/80
kkss
Long Answer Questions [5 or more Marks] 1/81
Yo
Case-Based Very Short/Short Questions 1/82
oo

Case-Based MCQs andAssertion-Reason Questions 1/83


B

Problems For Practice 1/85


re

NCER TExercises with Solutions 1/97


NCERT Exemplar Problems with Answers, Hints & Solutions 1/102
ou
ad

Competition Focus [NEET/JEE Special] 1/116


YY

2. Electrostatic Potential and Capacitance 2/1—2/140


ndd
Re

2.1. Introduction 2/1


Fi

2.2. Electrostatic Potential Energy 2/1


2.3. Electrostatic Potential 2/2
2.4. Electrostatic Forces are conservative 2/3

2.5. Electrostatic Potential due to a Point Charge 2/4

2.6. Potential at a Point due to Group of Electric Charges 2/5


2.7. Electrostatic Potential at a Point due to an Electric Dipole 2/6
2.8. Equipotential Surfaces 2/8

2.9. Relation Between Electric Intensity and Electric Potential 2/9

2.10. Expression for Electrostatic Potential Energy of a System of Charges 2/10


2.11. Potential Energy of Charges in an External Electric Field 2/12
2.12. Electrostatics of Conductors 2/13
2.13. Electrostatic Shielding 2/15

2.14. Electrical Capacitance 2/16

2.15. Capacity of an Isolated Spherical Conductor 2/17

2.16. Capacitor and Its Principle 2/17

2.17. Parallel Plate Capacitor 2/18

2.18. Relative Permittivity or Dielectric Constant 2/19

2.19. Spherical Capacitor 2/20

2.20. Grouping of Capacitors/Condensers 2/20

2.21. Energy Stored in a Capacitor 2/22

2.22. Total Energy Stored in a Combination of Capacitors 2/24

ww
2.23. Energy Density of a Parallel Plate Capacitor 2/24
2.24. Common Potential 2/25

2.25. Loss ofEnergy on Sharing Charges 2/25

Flo
2.26. Non-polar and Polar Dielectrics 2/26

e
2.27. Dielectric Polarization 2/27

ree
2.28. Dielectric Strength 2/29

Fr
rF
2.29. Capacitance of a Parallel Plate Capacitor with a Conducting Slab 2/29
uurr
2.30. Capacitance of a Parallel Plate Capacitor with a Dielectric Slab 2/30

2.31.
2.32.
Action of Sharp Points
Lightning Conductor
s for 2/32
2/32
kks
Yo
2.33. Van de GraaffElectrostatic Generator 2/32
oooo

Solved Examples 2/33


eB

Revision At A Glance 2/55

Multiple Choice Questions 2/58


r

Assertion-Reason 'fype Questions 2/61


ou
ad

2/64
YY

Conceptual Problems
Very Short Answer Questions [I Mark] 2/71
nndd

ShortAnswer Questions [2 Marks] 2/74


Re

ShortAnswer Questions [3 Marks] 2/82


Fi

LongAnswer Questions [5 or more Marks] 2/83

Case-Based Very Short/Short Questions 2/84

Case-Based MCQs andAssertion-Reason Questions 2/86

Problems For Practice 2/88

NCERTExercises with Solutions 2/107

NCERTExemplar Problems with Answers, Hints & Solutions 2/109

Competition Focus [NEET/JEE Special] 2/119


Unit - 2 [Current Electricity]

3- Current Electricity 3/1—3/214


3.1. Introduction 3/1

3.2. Current Carriers 3/1

3.3. Electromotive Force 3/2

3.4. Difference Between Electromotive Force and Potential Difference 3/2

3.5. Electric Current 3/3

3.6. Electric Current in Conductors 3/5


3/5
3.7. Drift Velocity
3.8. Mobility 3/6

3.9. Relation Between Current and Drift Velocity 3/7

ww
3.10. Ohm’sLaw 3/8

3.11. Electrical Resistance 3/9

3.12. Electrical Resistivity or Specific Resistance 3/9

Floo
3.13. Current Density, Conductance and Electrical Conductivity 3/12

ee
3.14. Classification of Materials in Terms of Conductivity 3/14

3.15. Effect of Temperature on Resistance 3/14

reer
rFF
3.16. Variation of Electrical Resistivity with Temperature 3/16
3.17. Non-Ohmic Devices or Limitations ofOhm’s Law 3/17
uur r
3.18. Thermistors
3.19. Super-Conductivity ffoor 3/17
3/18
sks
*3.20. Meissner Effect 3/19
YYoo
ooko

*3.21. Colour Code for Carbon Resistors 3/19

*3.22. Resistances in Series 3/20


eBB

*3.23. Resistances in Parallel 3/21

3.24. EMF, Internal Resistance and Terminal Potential Difference of a Cell 3/22
r
ouur

3.25. Distinction Between E.M.F. andTenninal Potential Difference of a Cell 3/24


ad

3.26. Grouping ofTwo Unidentical Cells in Series and Parallel 3/24


YY

3.27. Grouping ofNumber of Identical Cells 3/26

3.28. Heating Effect of Current; Joule’s Law 3/29


nndd
Re

3.29. Cause of Heating Effect of Current 3/29


FFii

3.30. Heat Produced by Electric Current 3/29


3.31. Electric Power 3/30

3.32. Electric Energy 3/30

3.33. Some Aspects ofHeating Effects of Current 3/31


3.34. Maximum Power Transfer Theorem 3/32

3.35. Kirchhoff’s Rules 3/32

3.36. Wheatstone Bridge Principle 3/34


3/35
3.37. Temperature Measurement Using Wheatstone Bridge
3/36
3.38. Slide wire Bridge or Meter Bridge
*3.39. Potentiometer and its Principle of Working 3/37

Not included in CBSE syllabus. These articles have been given only for the preparation of competitive examinations.
*3.40. Determination of Potential Difference Using Potentiometer 3/38
*3.41. Comparison of EMFs ofTwo Cells Using Potentiometer 3/39
*3.42. Determination ofintemal Resistance ofa Cell by Potentiometer Method 3/40
*3.43. SensitivenessofPotentiometer 3/40
*3.44. Difference Between Potentiometer and Voltmeter 3/41
Solved Examples 3/41
RevisionAtA Glance 3/74
Multiple Choice Questions 3/77
Assertion-Reason Type Questions 3/82
Conceptual Problems 3/89
Very Short A nswer Questions [/ Mark] 3/102
Short Answer Questions [2 Marks'] 3/110
Short Answer Questions [3 Marks] 3/122

w
Long Answer Questions [5 or more Marks] 3/124
Case-Based Very Short/Short Questions 3/125

Flo
Case-Based MCQs andAssertion-Reason Questions 3/127
Problems For Practice 3/130

reeee
NCERTExercises with Solutions 3/162

FFr
NCERT Exemplar Problems with Answers, Hints & Solutions 3/165
Competition Focus [NEET/JEE Special] 3/175

Unit - 3 [Moving Charges and Magnetism]


for
ur
4. Moving Charges and Magnetism
kkss
4/1—4/102
4.1. Introduction 4/1
Yo
oo

4.2. Oersted’s Experiment 4/1


eB

4.3. Magnetic Field 4/2


4.4. Biot-Savart’s Law 4/4
4.5. Similarities and Dis-Similmties Between the Biot-Savart’s Law for the 4/7
r
ou
ad

Magnetic Field and Coulomb’s Law for Electrostatic Field


4.6. Magnetic Field due to a Straight Wire Carrying Current
YY

4/7
4.7. Magnetic Field at the Centre of the Circular Coil Carrying Current 4/9
ndd

4.8. Magnetic Field at a Point on the Axis of a Circular Coil Carrying Current 4/11
Re

4.9, Magnetic Field due to Various Current Carrying Configurations 4/13


Fi

4.10. Ampere’s Circuital Law 4/15


4.11. Applications ofAmpere’s Circuital Law 4/17
4.12. Magnetic Field due to Infinite Long Straight Wire Carrying Current 4/18
4.13. Magnetic Field due to Current Through a Very Long Circular Cylinder 4/18
4.14. The Solenoid 4/20
4.15. TheToroid 4/22
4.16. Comparison Between Ampere’s Circuital Law and Biot Savart’s Law 4/23
4.17. Motion of a Charged Particle in a Uniform Electric Field 4/24
4.18. Motion of a Charged Particle in a Uniform Magnetic Field 4/25
4.19. Lorentz Force 4/27
* Not included in CBSE syllabus. These articles have been given only for the preparation of competitive examinations.
4/28
4.20. Velocity Filter
4/28
4.21. Cyclotron
4/31
4.22. Force on a Current Carrying Conductor Placed in a Magnetic Field
4.23. Force Between Two Parallel Linear Conductors Carrying Current 4/33
4/34
4.24. Experimental Demonstration ofthe Magnetic Force Between Parallel Currents
4/36
4.25. Comparison of Electrical and Magnetic Forces
4/37
4.26. Torque on a Current Carrying Coil in a Magnetic Field
4/39
4.27. Moving Coil Galvanometer
4/41
4.28. Weston Type Galvanometer
4.29. Shunt 4/42

4.30. Ammeter 4/43

oww
4.31. Voltmeter 4/43
4/45
Solved Examples
Revision At A Glance 4/66
4/70
Multiple Choice Questions

e
4/74
Assertion-Reason Type Questions

ree
rFl
4/83
Conceptual Problems

Fre
Very Short Answer Questions [IMark] 4/93

rr F
Short Answer Questions [2 Marks] 4/98

Short Answer Questions [5 Marks] 4/109


ouur
Long Answer Questions [5 or more Ma}'ks]
Case-Based Very Short/Short Questions
sfoo 4/112
4/112
kks
Case-Based MCQs andAssertion-Reason Questions 4/114
Yo
oooo

Problems For Practice 4/116


eBB

NCER TExercises with Solutions 4/136

NCERT Exemplar Problems withAnswers, Hints & Solutions 4/139

Competition Focus [NEET/JEE Special] 4/150


urr
You
ad

5. Magnetism And Matter 5/1—5/116


dY

5.1. Introduction 5/1


5/1
innd

5.2. TheBarMagnet
Re

5.3. Some Basics of Magnetism 5/1


Fi

5.4. The Magnetic Field Lines 5/3


5/5
5.5. Properties ofMagnetic Field Lines
5/5
5.6. Magnetic Dipole
5.7. Magnetic Field Strength at a Point due to Bar Magnet 5/6

5.8. Current Loop as a Magnetic Dipole 5/8


5/9
5.9. Bar Magnet as an Equivalent Solenoid
5/10
5.10. Torque on a Bar Magnet in a Magnetic Field
5.11. Determination of Magnitude ofMagnetic Field Using a Compass Needle 5/11
5/11
5.12. Potential Energy of a Magnetic Dipole in a Magnetic Field
5.13. The Electrostatic Analogue of Magnetism 5/13

5.14. Gauss’s Law for Magnetism 5/14


5.15. Magnetic Field of Earth 5/15
5.16. Cause of Earth’s Magnetism 5/15
5.17. General Features of Earth’s Magnetic Field 5/15
5.18. Magnetic Elements 5/16
5.19. Variation of Earth’s Magnetic Field 5/19
5.20. Neutral Points 5/20
5.21. The Tangent Law in Magnetism 5/21
5.22. Tangent Galvanometer 5/22
5.23. Bohr Magneton or Magnetic Dipole Moment ofan Atom due to Revolving Electron 5/24
5.24, Some Important Terms Used in Magnetism 5/25
5.25. Relation Between Magnetic Permeability and Magnetic Susceptibility 5/28
5.26. Classification of Magnetic Materials 5/29

w
5.27. Curie Law in Magnetism 5/36
5.28. Hysterisis Curve 5/37
5.29. Energy Dissipation Due to Hysteresis 5/38

Flo
5.30. Uses of Ferromagnetics 5/39

e
Solved Examples

reee
5/41
Revision At A Glance 5/54

FFr
Multiple Choice Questions 5/57
Assertion-Reason 7^pe Questions 5/58

for
ur
Conceptual Problems 5/62
Very Short Answer Questions [IMark] 5/67
kss
Short Answer Questions [2 Marks] 5/70
Yo
Short Answer Questions [3 Marks] 5/73
oo

Long Answer Questions [5 ormore Marks] 5/74


B

Case-Based Very Short/Short Questions 5/75


re

Case-Based MCQs andAssertion-Reason Questions 5/77


Problems For Practice 5/79
ou
ad

NCERTExercises with Solutions 5/89


YY

NCERTExemplar Problems with Answers, Hints <&. Solutions 5/92


Competition Focus [NEET/JEESpecial] 5/101
nd
Re

Unit - 4 [Electromagnetic induction And Alternating Currents]


Fi

6. Electromagnetic Induction 6/1—6/102


6.1. Introduction 6/1
6.2. Magnetic Flux 6/1
6.3. The Experiments of Faraday and Henry 6/4
6.4. Cause of Induced E.M.F. 6/5
6.5. Faraday’s Laws of Electromagnetic Induction 6/6
6.6. Lenz’sLaw 6/7

6.7. Lenz’s Law and Energy Conservation 6/8


6.8. Fleming’s Right Hand Rule 6/8

6.9. Various Methods of Producing Induced E.M.F. 6/9


6.10. Motional Electromotive Force 6/10
6.11. Power Dissipated in a Conductor Moved in Magnetic Field 6/11
6.12. Eddy Currents or Foucault Currents 6/12
6.13. Applications of Eddy Currents 6/13
6.14. Selfinduction 6/14
6.15. Self Inductance of a Long Solenoid 6/17
6.16. GroupingofCoils 6/17
6.17. Mutual Induction 6/18
6.18. Mutual Inductance of Two Long Coaxial Solenoids 6/20
6.19. A.C. GeneratororA.C. Dynamo 6/22
6.20. Multi Phase A.C. Generator 6/25
*6.21. D.C. GeneratororD.C. Dynamo 6/26

w
*6.22, D.C. Motor 6/27
*6.23. Motor Starter 6/29

Flo
Solved Examples 6/30
Revision At A Glance 6/37

reeee
Multiple Choice Questions 6/39

FFr
Assertion-Reason Type Questions 6/43
Conceptual Problems 6/47

for
Very Short Answer Questions [/ Mark] 6/52
ur
Short Answer Questions [2 Marks] 6/55
Short Answer Questions [3 Marks]
kkss
6/59
Long Answer Questions [5 or more Marks]
Yo
6/59
oo

Case-Based Very Short/Short Questions 6/60


eB

Case-Based MCQs andAssertion-Reason Questions 6/62


Problems For Practice 6/64
NCERTExercises with Solutions 6/70
r
ou
ad

NCERT Exemplar Problems with Answers, Hints & Solutions 6/73


Competition Focus [NEET/JEE Special]
YY

6/85
ndd
Re

7. Alternating Currents 7/1—7/132


7.1. Introduction: The Alternating Current 7/1
Fi

7.2. Transient Currents 7/2


7.3. Growth and Decay of Current in an Inductor 7/3
7.4. Charging and Discharging of a Capacitor Through Resistor 7/5
7.5. Mean Value or Average Value ofAlternating Current 7/8
7.6. Mean Value or Average Value ofAlternating E.M.F. 7/9
7.7. Root Mean Square Value ofAltemating Current 7/10
7.8. Root Mean Square Value ofAlternating E.M.F. 7/11
7.9. A.C. Circuit Containing Resistance Only 7/12
7.10. A.C. Circuit Containing Inductance Only 7/13
7.11. A.C. Circuit Containing Capacitance Only 7/15

’* Not included in CBSE syllabus. These articles have been given only for the preparation of competitive examinations.
7.12. A.C. Circuit Containing Resistance, Inductance and Capacitance in Series im

(RLC Circuit)
7.13. Impedance Triangle 7/20

7.14. A.C. Circuit Containing Resistance and Inductance 7/21

7.15. A.C. Circuit Containing Resistance and Capacitance 7/21

7.16. Energy Stored in an Inductor 7/23


7.17. LC Oscillations 7/24

7.18. Analogy Between Mechanical and Electrical Quantities 7/26


7.19. Electric Resonance nil

7.20. Q Factor of Resonance Circuit or Sharpness of Resonance 7/29

oww
7.21. Parallel Resonance Circuit 7/30

7.22. Average Power Associated with Resistance or Non-inductive Circuit 7/31


7/32
7.23. Average Power Associated with an Inductor
7/33
7.24. Average Power Associated with a Capacitor
7/34
7.25. Average Power in RLC Circuit or Inductive Circuit

e
ree
7.26. Power Factor ofAn A.C. Circuit 7/36

rFl
Fre
7.27. Wattless Current or Idle Current 7/37
7/38

rr F
7.28. Real Resistors, Inductors and Capacitors
7/38
7.29. Advantages and Drawbacks ofA.C. Over D.C.
ouur
7/38
7.30. Transformer
7.31. Energy Losses in a Transformer sfoo 7/40
7/41
kks
7.32. Uses of Transformer
Yo
oooo

7.33. Choke Coil 7/42


7/43
Solved Examples
eBB

Revision At A Glance 7/59


7/61
Multiple Choice Questions
urr

7/64
Assertion-Reason Type Questions
You
ad

7/68
Conceptual Problems
7/75
dY

Very Short Answer Questions [1 Mark]


[IMarks] nil
ShortAnswer Questions
innd

[3Marks] 7/80
Re

ShortAnswer Questions
[5 or more Marks] 7/81
LongAnswer Questions
Fi

7/82
Case-Based Very Short/Short Questions
7/84
Case-Based MCQs andAssertion-Reason Questions
Problems For Practice 7/86

NCERTExercises with Solutions 7/102


7/104
NCERTExemplar Problems with Answers, Hints & Solutions
7/114
Competition Focus [NEET/JEE Special]
USEFUL INFORMATION
A. DERIVED SI UNITS

Physical quantity Name Symbol

ooww
frequency hertz Hz

energy Joule J

force newton N

e
ree
rFl
power watt W

Fre
pascal Pa

rrF
pressure

electric charge coulomb C


ouur
electric current sffoo
ampere A
okks
Yo
electric potential volt V
ooo
Y
BB

electric resistance ohm Q


rr e

capacitance farad F
ouu
ad
Y

magnetic flux weber Wb


dY

inductance henry H
innd
Re

magnetic induction tesla T


FFi

B. FUNDAMENTAL CONSTANTS
Constant Symbol Value [with estimated error]
Speed of light c 3xl0“ms'

Gravitational constant G 6-67xlO-”NmV^


Acceleration of free fall 9-81ms-2

Atmospheric pressure at sea level atm 1-OlxloW^


Fundamental Constants (Contd.)
Constant Symbol Value [with estimated error]
Absolute zero K -273-15‘’C

Boltzmonn's constant k l-38xlO"^^JK-

Avogodro's number N 6-023x1 O^^mol*'

Universal gas constant


R 8-31 Jmor’k’’
Stefan's constant a 5'67xlO"’Wm"^K"^

ww
Permittivity of free space So e-SSxlO-^^C^N-’m'^

Permeability of free space 4tc X10'^TmA"' (WbA"^ m"’)

FF loo
Electronic charge e l-60x]0“”C

ee
9-lxlO'^’kg

ee r
Electron rest mass nit

rFrF
1'76xlO”Ckg
-1
Specific charge elm
rur
Proton rest mass Wp ffoor 1-6726x10'^^kg
ks
Neutron rest mass l'6749xl0-^^kg
YYoou
ookos

Atomic mass unit u l-67xlO-^^kg = 931MeV


BBo

Planck's constant 6-63xl0-^'‘js


re

Electronic Radius rt 2-8xl0-''m


ouur
ad

1'09678x1 OV’
Y

Rydberg constant RH

Bohr Magneton 9-27x10-^-^Am^


dndY

|4b
Re
FFini

C. CONVERSION FACTORS
1 mile = 1-61 km
1 radian = 57'30'’ = 57M8'
1 amu = l'66xl0^^kg = 931MeV
1 electron volt [eV] = l-6xl0-’’j
lo9eX = 2 3026 log,QX
Ji «3'1416, r = 9-8696
1 emu of charge = 3 X 10^*^650 of charge
1 emu of current = 3 X lO^^esu of current
1 emu of current = 10 ampere
1 emu of potential = 10‘^olt
1 esu of potential = 300 volt
w
e
re
o
rw F
ullo
FF
ee
srr
roF
k
uor
ofof
kos
YYo
CHAPTER 1.
Boo
rY

ELECTROSTATIC CHARGES AND FIELDS


eerB

CHAPTER 2.
u

ELECTROSTATIC POTENTIAL AND CAPACITANCE


d
oou
ad
nY
nid
Re
FFi
1

ELECTROSTATIC

w
CHARGES AND FIELDS

Flo
WW-.-VWJ ■ dJ- ,.»S‘V.V..v

reeee
1.1. INTRODUCTION

FFr
Many of us have the experience of seeing a spark or hearing a crackle, when we take off our synthetic
shirts or nylon sweaters, particularly in dry weather. Sometimes, we feel the sensation of an electric shock

for
ur
while opening the door of our car or by holding the iron bar of a bus, after sliding from our seat. The reason
for these experiences is discharge of electric charges through our body, which were accumulated due to
kkss
rubbing of insulating surfaces. Another common example of electric discharge is the lightning that we see in
Yo
the sky during thunderstorms.
oo

We know that when a glass rod is rubbed with a piece of silk, the rod acquires the property of attracting
eB

light objects like bits of paper, straw, pith balls, dry leaves or even dust particles towards it. The glass rod is
said to be electriifed or charged. Similarly, a plastic comb gets electrified on passing through dry hair.
r

The metallic bodies of cars and trucks also get charged because of friction between them and the air
ou
ad

rushing past them. This charge being large can produce even a spark. Such a spark can be dangerous in case
YY

of a petrol tanker. That is why petrol tankers often have a metal chain dragging along the ground. The charge
produced leaks to the ground through this chain. Now a days, the tyres of cars and trucks are made by adding
ndd
Re

some carbon compound to the rubber. This facilitates the charge built up on the body of the vehicle to leak to
the ground.
Fi

We learn from above that electric charges are produced due to friction between two insulating
bodies, which are rubbed against each other. The charges on insulating bodies cannot move on their own.
That is why they are called static charges.

The branch of Physics, which deals with the study of charges at rest (i.e., static charges), the
1 forces betweenthe static charges,fieldsandpotentialsdue to these charges is called Electrostatics
I if'!; or Static Electricity or Frictional Electricity.

Historically, this phenomenon was discovered around 600 BC by a Greek philosopher ‘Thales of Miletus \
The name electricity was taken from Greek word ‘Elecktron’.
Note that most of the early experiments on electrostaticswork best on a dry day, because excessive
moisture provides a pathway for charge to leak off a charged object.
1/1
1/2
"Pnaxiee^ it Fundamental Physics (XII)
1.2. ELECTRIC CHARGE

Electric charge is a characteristic that accompanies fundamental particles, wherever they exist.
According to William Gilbert, charge is something possessed by material objects that makes it
possible for them to exert electrical force and to respond to electrical force.
The three most common elementary particles are electrons, protons and neutrons having masses

= 9*10940 X 10“^* kg ; = 1*67262 x 10"^^ kg and = 1*67493 x 10"^^ kg

w
Because of their mass, these particles attract one another by gravitational forces. Thus, an electron
attracts another electron at a distance of 1 cm, with a gravitational force

6*67x10-^‘x(9*1x10-3>)2

e
p _ _ = 5*5x 10-^'^N
r- (10-2)2

re
o
rw
However, an electron repels another electron at a distance of 1 cm with a force = 2*3 x 10-2“^ N. This
force is called electric force. We observe that electric force is very large compared to the gravitational force.

F
The electrons must have some additional property (other than their mass), which is responsible for the

ullo
FF
electric force between them. This additional property of electron, which gives irse to electric force between
two electrons is called electric charge. Just as masses are responsible for the gravitational force, charges

ee
are responsiblefor the electric force.

srr
roF
Two protons placed at a distance of 1 cm also repel each other with the same force = 2*3 x 10“2'^ N. It

k
shows that protons also have charge which in magnitude, must be equal to charge on electron. Two neutrons
placed at a distance of 1 cm attract each other with a force = 1*9 x 10^ N, which is the gravitational force
uor
ofof
due to their masses. However, electric force between two neutrons is zero. Therefore, neutrons have no
charge, though they have mass.
Y
kos
Yo
rBB
oo

1.3. TWO KINDS OF CHARGES


eY

Let us look at some simple facts, which were established from years of efforts and careful experiments :
re

(0 A glass rod rubbed with a piece of silk brought close to a suspended glass rod rubbed with silk repels
u

the latter as shown in Fig. 1.1 {a).


(«■) The two pieces of silk cloth with which the glass rods were rubbed also repel each other. However,
d
ou
o
ad

each glass rod attracts the silk piece with which it was rubbed.
nY

{Hi) Two ebonite/amber rods rubbed with cat’s fur repel eachother as shown in Fig. 1.1(/?). However, each
ebonite rod attracts the cat’s fur with which it was rubbed,
nid
Re

(/v) An ebonite rod rubbed with cat’s fur attracts a glass rod rubbed with a piece of silk as shown in Fig.
F
Fi

1.1(c). However, the charged glass rod repels the cat’s fur.
(v) When a glass rod rubbed with a piece of silk is FIGURE 1.1
made to touch two small pith balls (or
9 Silk ? Silk ? Silk
polystyrene balls) suspended by silk/nylon
threads, the balls repel eachother, as shown in gb onite
Fig.
/ ^-5
(vi) Similarly, when an ebonite/amber rod rubbed ■^55-*^
with cat’s fur is made to touch two small pith £)00 eoo
balls suspended by silk/nylon threads, the balls O o Q
repel eachother, as shown in Fig. 1.1(c).
(vii) However, a pith ball (or polystyrene ball) touched
with charged glass rod attracts another pith ball
touched with charged ebonite rod, as shown in
O
Fig. I.!(/●).
ELECTROSTATIC CHARGES AND FIELDS 1/3

A careful analysis of these facts observed leads us to the following DO YOU KNOW ?
conclusions :
● If an electrically charged rod is
(0 The bodies like glass rods, ebonite/amber rods, silk, fur etc. acquire
brought near normal flow of
electric charge on rubbing. water from a tap, the flow gets
(//) The pith balls or polystyrene balls acquire electric charge by actual slightly diverted towards the rod
contact with a charged body. as shown in Fig. 1.2. This is due
(Hi) The charge developed on glass rod rubbed with silk is different to force of attraction exerted by
from the charge developed on ebonite rod rubbed with fur. This is the charged rod.
because a charged glass rod repels another charged glass rod, but FIGURE 1.2

attracts a chtu’ged ebonite rod. Hence, there are two kinds of charges. X

oww
When a glass rod is rubbed with silk, the rod acquires one kind of
charge and the silk acquires second kind of charge. This is true for
any pair of objects that are rubbed to be electrified. ii
Further, like charges repel each other and unlike charges attract

e
FFrlo
each other,

re
(iv) When an electrified glass rod is brought in contact with the silk piece, with which it was rubbed, they

ree
F
no longer attract eachother. They also do not attract other light objects as they did on being electrified.
It means the charges acquired on rubbing are lost when the two oppositely charged bodies are brought

rF
in contact with eachother. It leads us to the conclusion that charges acquired by the objects on rubbing
against eachother must be equal and opposite. That is why they neutralise or nullify eachother’s

fsoor
ouur
effect. Du Fay was the first to show two kinds of charges. An American scientist Benjamin Franklin
skf
named the two kinds of charges as positive and negative. By convention, charge acquired by glass
rod or cat’s fur is called positive. And the charge acquired by ebonite/amber rod or silk cloth is
ooko
YYo
called negative.
Hence, there are two kinds of charges. The property which differentiates the two kinds of charges is
Bo
reB

called the polarity of charge.


If a body possesses an electric charge, it is said to be charged or electrified. When it has no charge, it is
said to be neutral.
uur
ooY
ad

Further, note that only rubbed area of non conducting body gets charged, and this charge does not
move to other parts of the body. The charge is static on rubbed portion only.
dY
innd
Re

Curiosity Questions
r
FFi

Q.l. What role does electrostatics play in a xerox copying machine ?


Ans. A xerox copying machine Is one of the many industrial applications of the forces of attraction
and repulsion between charged bodies. Particles of black powder, called toner, stick to a tiny
carrier bead of the machine on account of electrostatic forces. The negatively charged toner
particles are attracted from carrier bead to a rotating drum, where a f5ositively charged image of
document being copied has formed. A charged sheet of paper then attracts the toner particles
from the drum to itself. They are then heat fused in place to produce the photo copy.
Q. 2. What are some other applications of electrostatics ?
Ans. (/) Electric capacitors or condensers, which have variety of applications are based on the
principle of electrostatics.
(ii) High electric potential sources like van-de-graaf generator are based on principle of
electrostatics.
(Hi) Pollution Is controlled by electrostatic precipitators of fly ash.
(/v) Spray paintings and powdered coatings are based on the principle of electrostatics.
(v) Power factor condors, which reduce energy losses are based on the principle of electrostatics.
1/4 Fundamental Physics (XIOBZsSQ

o
{vi) Electrostatic shielding protects sensitive instruments from external electric fields.

Y
(vii) Natural phenomena like lightning and thunder storms are explained on the basis of
eiectrostatics.
(w//) Electrostatics plays a great role in the reproduction of flowers.
Q. 3. When your friend chews a winter green life saver in a dark room, you see a faint fiash of
bfue iight from his mouth. How ?

FF

o
Ans. This display of light is often called sparking. It occurs due to electric discharge of the electrostatic

r
charges produced in chewing the winter green life saver.

1.4. CONDUCTORS. INSULATORS AND DIELECTRICS

YMost of the substances in nature are divided into two categories, namely, conductors and insulators.

o
ll
A substance which can be used to conduct electric charge from one end to the other is called a

ow
conductor. Silver is one of the best conductors. Other examples of conductors are copper, iron, aluminium,

Fd
mercury, coal etc. Earth is a good conductor. Human body is also a good conductor of electricity. The liquid
conductors include, salt solutions, acids, alkalis etc.

kB

f
In metallic conductors, there are very large number of free electrons which act as carriers of charge.

r
Infact, in a metal, the outer (valence) electrons part away from their atoms and are free to roam about in the

ree
body of the metal, but they cannot leave the metal under normal circumstances.The free electrons form a
kind of electron gas, they collide with one another ; and also with the ions ; and move randomly in different
directions. In an ex- temal electric field, the free electrons drift against the direction of the field. The residual
n
atoms made up of nuclei and the bound electrons remain held in their fixed positions. They constitute the
u
or
ur
bound charges in the conductor as they cannot move. In electrolytic conductors, the charge carriers are both,
sf
the positive and negative ions.
The insulators are the materials which cannot conduct electricity, Le., they are poor conductors of
e

electricity. Common examples of insulators are glass, rubber, plastic, ebonite, mica, wax, paper, wood etc.
Yo
oo
ri

They are called insulators, because they prevent charge from going to places where it is not desired. Such
substances possess almost no free electrons.
eB
o

Infact, in an insulator, each electron is attached or bound to a particular atom and is not free to move in
the body of the insulator. As each electron stays near its ‘‘parent' nucleus or within its atom or molecule, and
F

cannot move far away from it, therefore, an insulator does not possess freely movable charges. Hence it fails
ou
ad

to conduct electricity.
Insulators are also called Dielectrics. Obviously, dielectrics cannot conduct electricity. However,
Y

when an external electric field is applied on them, induced charges appear on the surface of the dielectric.
Hence we may define dielectrics as the insulating materials which transmit electric effects without conducting.
nd
Re

When some charge is transferred to a conductor, it gets distributed readily over the entire surface of the
conductor. In contrast, if some charge is put on an insulator, it stays at the same place.
Fi

A nylon or plastic comb gets electrified on combing dry hair or on rubbing. But a metallic rod does not
get electrified on rubbing. This is because charges on metal leak through our body to the ground as both are
good conductors of electricity.
Retain in Memory
When a charged body is brought in contact with the earth, all the excess charge on the body
disappears by causing a momentary current to pass to the ground through our body. This process
of sharing charges with the earth is called grounding or earthing. Earthing near the mains supply
of a building is done by burying the neutral (earth) metal wire deep into earth and there, it is
connected to a thick metal plate. The electric wiring in our houses has three wires ; live, neutral
and earth. The first two carry electric current from the power station and the third (earth) is
connected to the buried metal plate. Metallic bodies of appliances like TV., refrigerator, electric
iron etc. are connected to earth wire. If a live wire were to touch the metallic body, charge flows
to earth without damaging the appliance and without causing injury to us.
ELECTROSTATIC CHARGES AND FIELDS 1/5

1.5. GOLD LEAF ELECTROSCOPE (OLE)


A gold leaf electroscope (GLE) is an instrument which is used for detecting the presence of electric
charge and its polarity (i.e., ± sign of charge). The instrument can also be used for measuring potential
difference. < 1
The essential parts of a gold leaf electroscope are shown in Fig.
1.3. LL are two extremely thin gold foils attached to lower end of a metal
rod R fitted in a glass jar G through an insulating stopper S of cork or
rubber etc. D is a metal disc at the free end of the metal rod. The sensitivity
of the instrument is increased by pasting two tin foils F, F on the inner
side of glass jar opposite to the gold leaves.
As the gold leaves are extremely thin conducting foils which have
low mass per unit area and are flexible, therefore, they respond very

ww
quickly to small electrostatic forces. Thin aluminium foils can also serve
the same purpose. When a charged rod is touched with the metal disc D,
the same charge is transferred to the gold leaves through the metal rod.
The leaves repel eachother and diverge as shown in Fig. 1.3. By measuring

Flo
F F
the divergence of leaves, the amount of charge on the body can be

ee
estimated.

rere
Nature of electric charge on the body can be determined using a gold leaf electroscope, which is

r FF
already charged.
When charge on the charged body is of opposite sign to that of the charge on GLE, then on touching the
uurr
foor
body with the disc of GLE, divergence of leaves would decrease.
When charge on the charged body is of same sign as that of charge on GLE, then on touching the
ks s
charged body with the disc of GLE, the divergence of leaves would increase.
Yoo
oook

1.6. ORIGIN OF ELECTRIC CHARGE IN ELECTROSTATICS


eBB

It is known that all matter is made up of atoms and/or molecules, the basic unit being an atom. We also
know that every atom consists of a central core called the atomic nucleus, around which negatively charged
electrons revolve in circular orbits. Every atom is electrically neutral, containing as many electrons as the
rr

number of protons in the nucleus. Thus, even though normally, the materials are electrically neutral, they do
ouu
ad

contain charges, but their charges are exactly balanced.


YY

The vast amount of charge in an object is usually hidden as the object contains equal amounts of
positive charge and negative charge. yVith such an equality or balance of charge, the object is said to be
nndd

electrically neutral, Le., it contains no net charge.


Re

If the positive and negative charges are not in balance, then there is a net charge. Thus, an object is
FFii

charged if it has a charge imbalance or some net charge. Hence, to electrify or charge a neutral body, we
need to add or remove one kind of charge. When we say that a body is charged, we always refer to excess
charge or deficit charge.
In solids, some of the electrons are less tightly bound in the atom. These are the charges, which are
transferred from one body to the other.
When we rub two insulating substances against each other, we provide energy to overcome friction
between them. This energy is used in removing electrons from one substance and transferring them to the
other. The transfer takes place from the material in which electrons are held less tightly (with lower work
function) to the material in which electrons are held more tightly (with higher work function), i.e.,
electrons are transferred from the material whose work function u lower to the material whose work
function is higher. Consequently, the material which loses electrons acquires a positive charge and material
which gains electrons acquires an equal negative charge. For example, when we rub a glass rod with silk,
electrons are transferred from glass rod to silk. The glass rod becomes positively charged and silk acquires an
equal negative charge. Thus, charging by rubbing is due to actual transfer of electrons.
1/6 T^rcidee^ ^ Fundamental Physics (XII)PKIBl
The cause of charging is actual transfer of electrons from one material to the other. The insulating
material with lower work function loses electrons and becomes positively charged and vice-versa.
Further, as an electron has a mass, howsoeversmall it may be, therefore there does occur some change
in mass on charging. A positively charged body has lost some electrons and hence its mass reduces slightly.
On the other hand, a negatively charged body has gained some electrons and hence its mass increases
slightly.
Note that in rubbing two insulating bodies, the number of electrons that are transferred, is a very small
fraction of the total number of electrons in the material bodies. Hence, the charge acquired by friction is a
very small fraction (« 10^ coulomb) of the total positive and negative charge content of the bodies.
Further, as only the less tightly bound electrons in a material body can be transferred from it to another
by rubbing, only under suitable conditions, we have to stick to certain pairs of materials to observe charging
on rubbing the bodies.

ww
In Table 1. 1, we have listed the pairs of objects which get charged on rubbing against each other. They
have been divided into two classes, one acquiring positive charge and the other acquiring negative charge on
rubbing.

Floo
TABLE 1.1. Objects acquiring two kinds of charges on rubbing

ee
POSITIVE CHARGE NEGATIVE CHARGE

eer
1. Glass rod 1. Silk cloth

rFrF
2. Fur or woolen cloth 2. Ebonite, Amber, Rubber rod
3. Woolen coat 3. Plastic seat
rur
4. Woolen caipet
5. Nylon or Acetate
s ffoor
4. Rubber shoes
5. Cloth

6. Dry hair 6. Comb


osk
YYoou
oook

Obviously, any two charged objects, in the same column repel each other and any two charged objects
eBB

from different columns attract each other.

Further, different substances have been arranged in a series, called IHboelectric Series. When any two
of the substances in series are rubbed together, the one occurring earlier in the series acquires positive charge
r
ouur

and the other occurring later in series acquires negative charge. Some of the substances in triboelectricseries
ad

are : I. Fur2. Flannel 3. Wool 4. Glass 5. Paper6. Cotton?. Silk 8. Wood 9. Metals 10. Rubber 11. Orion 12.
Y

Polyethylene 13. Teflon 14. Ebonite, etc.


Thu.s gla.ss acquires a positive charge when rubbed with silk and it acquires a negative charge
nd dY
Re

when rubbed with fur.


FFini

1.7. CHARGING BY INDUCTION

We know that a body can be charged by putting it in contact with another charged body either directly or
by means of a conductor. For example, when a charged ebonite rod is in contact with a pith ball or connected
to it by a copper wire, it transfers some of its negative charge to the pith ball. This is charging by conduction.,
Le.y charging by actual contact.
When a charged body A is brought near an uncharged metallic body B, then some charge of opposite
sign appears on the near face of body B and an equal charge of same sign appears on the rear face of body B.
The total charge on body B is zero. In this process, body A does not lose any charge as it is not in direct
contact with body B. This phenomenon of charging body B without actual contact is called charging by
induction.

Thus, phenomenon of charging an uncharged conducting body, by bringing a charged body


near it, without making a direct contact between the two bodies is called charging by induction.
The steps involved in charging a metallic sphere by induction are shown in Fig. 1.4.
ELECTROSTAHC CHARGES AND FIELDS 1/7

w
(a) To begin with, a metallic sphere B on an insulating stand is uncharged. Fig. 1.4(a).

e
{b) When a positively charged glass rod A is brought near the uncharged metallic sphere, free electrons

row
re
of the sphere are attracted and start piling up at the near end. This end therefore, becomes negatively charged
and the farther end of the sphere becomes positively charged to the same extent, due to deficit of electrons as
the sphere on the whole remains neutral. Fig. 1 A(b). The redistribution of charge is almost instantaneous and

eeF
ullo
stops as soon as net force on free electrons in the metallic sphere becomes zero,

FF
(c) When the sphere is grounded, Le., it is connected to earth by a conducting wire, electrons flow from
the ground to the sphere and neutralise the positive charge on the farther end of the sphere. The negative

srr
charge at the near end of the sphere remains bound there due to attractive force of glass rod. Fig. 1.4(c).

roF
(d) When the sphere is disconnected from the ground, the negative charge continues to be held on the

k
near end. Fig. 1 A{d).
uor
ofof
(e) When the glass rod is removed, the negative charge spreads uniformly over the sphere B,
Fig. 1.4(c).
kos
Similar steps are involved when a negatively charged rod is used for charging the sphere positively by
YYo
eerBB
oo

induction.
rY

Let us now understand how we charge two spheres by induction. Fig. 1.5(a) shows two metal spheres
A and B supported on insulating stands, held in contact with eachother.
u

Let a positively charged glass rod be brought near the sphere A. Free electrons in both the spheres are
attracted towards the rod. Therefore, left surface of left sphere A has an excess of negative charge and right
oou
ad
d

surface of right sphere B has an excess of positive charge. Note that all the electrons in the spheres have not
nY

accumulated on the left surface of sphere A. This is because as negative charge starts building up at the left
face of A, further electrons are repelled by these. An equilibrium is reached almost instantly under the action
nid
Re
FFi

o o o

o o
1/8 '4- Fundamental Physics (XII) KSnw
offeree of attraction of the rod and the force of repulsion due to the accumulated negative charges. Fig.
1.5(/>) shows this equilibrium situation.
Further, the accumulated charges Would remain on the surfaces as shown till the glass rod is held near
the sphere A. If the rod were removed, the charges would return to their original neutral state in the absence
of any outside force.
Separate the spheres A and B by a small distance, while the glass rod is still held near the sphere A. The
two spheres carry opposite charges as shown in Fig. 1.5(c). They attract each other.
Remove the glass rod. The stams of charges on the spheres is shown in Fig. 1.5(d).
When the spheres are now separated widely, the charges on them get uniformly distributed as shown in
Fig. 1.5(e).
This is how two metal spheres get oppositely charged by induction. Note that in this process of electric
induction, the positively charged glass rod does not lose any charge. This is contrary to charging by

ww
conduction, ue., charging by actual contact where the charged glass rod loses some charge.

1.8. QUAriTIZATlON OF ELECTRIC CHARGE

FF loo
The quantization of electric charge is the property by virtue of which, the charge on a body is

ee
integral multiple of a basic unit of charge of an electron/proto n, represented by e.

Fe r
Thus, charge ^ of a body is always given by q-ne ...(1)

where n is any integer, positive or negative. The basic unit of charge is the charge that an electron or proton

Frre
carries. By convention, charge on an electron is taken to be negative. Therefore, charge on an electron is
rur
written as (- e) and charge on a proton is (+ e). fforo
The value of the basic unit of charge or elementary charge is c - 1-6 X 10 coulomb ...(2)
ks
It is one of the important constants of nature.
YYouo
okso

If a body contains electrons and protons, the total amount of charge on the body is
BBoo

q = n2 {e) + n^ (- e) = («2 - 'll) ^


As /ij, «2 are integers, their difference must also be an integer. Thus, the charge on anybody is always an
r ee

integral multiple of e, and can be increased or decreased also in steps of e.


Thus, any charged body or charged particle can possess charge equal to ± 1 e,±2e,±3e and so on, j.e.,
ad
oouur

the possible values of charge are


Y

(? = ±le = ±lxl-6x 10-^9 C = ± 1-6 X C


±2e = ±2xl-6x 10-^^C = ±3-2x 10-*^C
dnYd

q-
Re

^ = ±3e = ±3x l-6x 10-^^C = ±4.8x lO-’^C


FFini

and so on. The values of charge lying in between these values are not possible.
The cause of quantization is that only integral number of electrons can be transferred from one
body to another. For example, when one electron is transferred, the charges acquired bv the two bodies will
be^ = ±le = ±l-6x 10“^^ C. Similarly, when n electrons are transferred, the charge* acquired by the two
bodies will \>tq = ±ne = ±n'.x 1-6 x 10"*^ C.
The quantization of charge was first suggested by the experimental laws of electrolysis discovered by
Faraday. It was actually demonstrated experimentally by Millikan in 1912. Thus, quantization of charge is
an experimentally verified law in all domains of nature. Like charge; energy and angular momentum are also
quantized.
IMPORTANT NOTE —
Recent discoveries have shown that protons and neutrons are made up of quarks. These new particles
have charges ± c/3 and ± 2e/3. However, these particles are not known to exist in free slate. That is
why their charges are not taken as elementary charges. The basic unit of charge which has independent
existence is ‘c’ only. Various quarks and antiquarks alongwi± their charges are listed in Table 1.2.
ELECTROSTATIC CHARGES AND FIELDS 1/9

TABLE 1.2. Various quarks and andquarks alongwith their charges


S. No. QUARK ANTI-QUARK
1. up (tt) = + 2e/3 anti-up (u) = -2e/3
2. down (d) = - e/3 anti-down (d) = e/3
2e 2e
3. charm (c) = + — anti charm (c ) =
3 3
— €
4. strange (5) = - - anti strange (^ ) = -
3 3
2e 2e

ooww
5.
top (0 = + Y anti-top (t )
3
£ — ^
6. bottom (b) - — anti-bottom (b)~-
3 3

Retain in Memory

e
ree
rFl
Quantization of charge is meaningful only at the microscopic level, where the charges involved

Fre
are of the order of a few tens or hundreds of e, i.e., they can be counted. Such charges appear in

rrF
discrete lumps and quantization of charge cannot be ignored.
However, at the macroscopic level, we deal with charges of a few microcoulomb. A charge of
ouur
sffoo
magnitude 1 |iC contains electrons whose number n = ~
q_ lxlQ~^C
e ~1-6xI0-’9C
= 10^^, which is very
okks
large. At this scale, the fact that charge can increase or decrease only in units of e is not visible.'
Yo
ooo

The grainy nature of charge is lost and it appears to be continuous.


BB

Is a charge of 5-8 x C possible ?


rr e

5-8x10"*®
Sol. From q- ne. n = i =
Youu

= 36-25
ad

e l-6xl0"‘^
dY

As n is not an integer, this value of charge is not possible.


What is the charge on a body &x>m which one million electrons are removed ?
innd
Re

Sol. Here, n = 10® e = 1-6 x 10"*^ C


Fi

As electrons are removed, charge acquired is positive


F

<? = ne = 10® X 1-6 x 10"*^ C = 1*6 x C


1.9. ADDITIVITY OF CHARGE

Additivity of charge is a property by virtue of which total charge of a system is obtained simply
by adding algebraically all the charges present anywhere on the system.
It means charges are scalars like the mass of a body and are added by simple laws of Mathematics.
If a system contains « charges q^, q2, q^,...» then the total charge of the system is
9 = 9] + 92 + 93 + -■ +
Charge has magmtude only, but no direction, similar to the mass. However, mass of a body is always
positive, but charge can be either positive or negative. Therefore, proper signs have to be used while
adding the charges in a system.yoT example, if a system contains chaige + q,-2q,-¥3qand + 5q, then the
total charge of the system is = + ^- 2g + 3^ + 5^ = + 7^
1/10 ‘P%et,eUe^ Fundamental Physics (XII) IT/aWi
1.10. CONSERVATION OF CHARGE

Conservation of charge is the property by virtue of which total charge of an isolated system
*

always remains constant or conserved.


Within an isolated system consisting of many charged bodies, charges may get redistributed due to
interactions among the bodies, but the total charge of the system shall remain the same.
For example, when we rub two insulating bodies, what one body gains in charge, the other body loses
the same amount of charge. Thus, it is not possible to create or destroy net charge carried by any isolated
system. However, charge carrying particles may be created or destroyed in a process. For example, a neutron
turns into a proton and an electron. The proton and electron thus created have equal and opposite charges.
The total charge is zero before and after the creation. Thus, charges can be created or destroyed in equal
and unlike pairs only.

w
Following examples illustrate the property of conservation of charge.
(/) In the phenomenon of pair production, a y ray photon materialises into an electron and a positron
having total charge-e + e = 0, which is the initial charge on a photon.

Flo
y = e~ + e (pair production)
(ii) In annihilation of matter, an electron and a positron annihilate eachother to produce two y-ray

ee
photons with zero charge. Charge is thus conserved.

Fr
€~ + e* = y + y (annihilation)
(iii) In all radioactive decays, charge number is always conserved.
For example, in radioactive decay of U-238, the nucleus is transformed into Th-234 with the emission

for
ur
of an alpha particle, which is a helium nucleus, i.e.,
92
U238 » 90Th234 + 2He^^ (radioactive decay)
ks
(iv) In all nuclear transformations, charge number is always conserved. For example :
Yo
+ 2He^
oo

s 1
IMPORTANT NOTE
eB

Note that in applying the conservation of charge principle, we must add the charges algebraically, with
regard to their signs.
r
ou
ad

1.11. COMPARISON OF CHARGE AND MASS


Y

We are familiar with role of mass in gravitation, and we have just studied some features of electric
charge. We can compare the two as shown in Table 1.3.
nd
Re

TABLE 1.3. Comparison of charge and mass


Fi

CHARGE MASS

1. Electric charge on a body may be positive, 1. Mass of a body is a positive quantity.


negative or zero.
2. Charge carried by a body does not depend upon 2. Mass of a body increases with its velocity as
velocity of the body. mo
m = , where c is velocity of light in

vacuum, m is the mass of the body moving with


velocity v and is rest mass of the body.
3. Charge is quantired. 3. The quantization of mass is yet to be established.

*An isolated system is that which is free from external influence i.e., no charged matter is allowed to enter
or leave the volume of space occupied by the isolated system.
ELECTROSTATIC CHARGES AND FIELDS 1/11

CHARGE MASS

4. Electric charge is always conserved. 4. Mass is not conserved as it can be changed into
energy and vice-versa.
5. Force between charges can be attractive or 5. The gravitational force between two masses is
repulsive, according as charges are unlike or like always attractive.
charges.
6.
The force between two charges follows inverse 6. The force between two masses also follows
square law. inverse square law.
7. Charge cannot exist without mass. 7. Mass can exist without charge.
8.
Unit of charge is a derived unit {1 C = 1 As). 8. Unit of mass is a fundamental unit.
9. An accelerated charge emits radiation. 9. Accelerated mass emits no radiation.

ww
1.12. COULOMB'S LAW

Coulomb performed several experiments to measure the extent of force between two point charged

Flo
bodies.*

According to Coulomb’s law,

ee
rere
The force of interaction between any two point charges is directly proportional to the product of

r FF
the charges and inversely proportional to the square of the distance between them.
uurr
The force acts always along the line joining the two charges.
Suppose two point charges foor
and ^2 are separated in vacuum by a distance r.
ks s
1^1 I 1^2 '
Yoo
According to Coulomb’s law, F oc
or
...(1)
ooook

r2 r2
where k is electrostatic force constant ^ '
eBB

The value of electrostatic force constant k depends on the nature of medium separating the charges,
and on the system of units.
rr

When the charges are situated in free space (air/vacuum), then in cgs system, it = 1.
ouu
ad

In SI, it = 9 X 10® N C"2


YY

1
We write. it =
4tC€ ...(2)
nndd
Re

where e q is called absolute electrical permittivity of the free space.


Fii

From (1), the magnitude of force is F =


1 I I I ?2 I
...(3)
4tC€
0 r2
Units, Dimensions and Value of € »
1
From (3), ^0 =
47tF

1 C.C
As SI unit of charge is coulomb (C), therefore, Units of €g= — - = c2N-*m-2
N m

(AT) (AT)
Dimensions of €q = = [M-‘ L-3 T^ a2]
(MLT-2) (l2)

*When linear sizes of charged bodies are much smaller than the distance separating them, the size may be
ignored. The charged bodies are then treated as point charged bodies.
1/12 ‘P'uxdee^ ^ Fundamental Physics (XII) IIIMjin

1 1
From (2), it = or
^0 = 4nk ...(4)
4n&
0

1
^0 =
4x3-14x9x10^ €0 = 8-85 x10-‘2c2n-1j^-2 ...(5)

1.13. COULOMB'S LAW IN VECTOR FORM

As stated already, according to Coulomb’s law, the force of interaction F between any two point charges
and ^2 directly proportional to the product of the charges, and inversely proportional to the square of the
distance (r) between them, i.e., f I M ^2 ’ OC

r2

ww
.
t =
^1^1 TI 1^2 I
r2

Floo
where k is electrostatic force constant.
As force is a vector, it is better to write Coulomb’s law in

ee
the vector notation. In Fig. 1.6, let

ee r
rFrF
= OA = position vector of charge
rur
= OB - position vector of charge ^2 ffoor
ks
YYoou
.-. The vector leading from «?i to ^2 ...(6)
ookos

AB =r,2=r2-r,
BBo

In the same way, vector leading from qi^o q^ is BA =r2, = r,-r2 ...(7)
re

The magnitude of r^2 ''12 magnitude of r2j is r2j.


ouur
ad

As the direction of a vector is specified by a unit vector along the vector, we define
Y

'12 .r^ _ '21


A
and
hi ~
nd dY

21 “
Re

'■12 ^21

FFini

If ,F12 = force on q^ due to q2, and F21 = °n ^2 due to then as is clear from Fig. 1.6,
Coulomb’s force law between two point charges and ^2 at Tj and T2 in vacuum is expressed
1 9, ^2
as
^21 “ 47l€n AB^ ’ along AB
0

1 ^ 1 ^2 i ^1 ^2
or ^21 “ * ^ xn12 xr,
12 ...(8)
4tc€ r2 4tU€ 0 47E€ 0
0 '12 ^12 r-7p
^2 M '

It should be clearly understood, that eqn. (8) is valid for any sign of q^ and ^2. whether positive or
negative.

If ^2 same sign {either both positive or both negative) \ q^q2>0 \ F2j is along rj2 » which
denotes repulsion for like charges ; Fig. 1.7(a).
ELECTROSTATIC CHARGES AND FIELDS 1/13

If ^j and are of opposite sign {i.e., one is positive FIGURE 1.7


-;rF2i
and other is negative), q,^ q2<Q \ F21 is along -r
which denotes attraction between unlike charges, Fig. 0

1.7(b).
w <^2
Thus, eqn. (8) takes care of both the cases of like and
unlike charges correctly. o

The force F^2 > on charge q^ due to charge ^2 is Fi2


Fi2
obtained from eqn. (8), by simply interchanging 1 and 2, ^1
i.e..
1

w
^12 =
-^xr2i--K
- F
47C£ 21
-.(9)
0 12

Therefore, force on q^ due to ^2 is equal and opposite to the force on ^2 due to qy Thus, Coulomb's law

Flo
is in accordance with Newton’s third law of motion.

reee
Note that eqns. (8) and (9) give us the forces between two charges q^ and ^2 io vacuum only.
IMPORTANT NOTES

FFr
1. Coulomb’s law is universal. The force which binds the electrons to the nucleus to form an atom is
calculated from this law. The same law applies to van der waal forces which unite the atoms to form
urr
for
molecules. The three states of matter viz. solids, liquids and gases are also explained on the basis of
this law.
kkss
2. Coulomb’s law has been verified over distances ranging from nuclear dimensions (~ 10“*^ m) to
Yo
macroscopic distances (— 10^^ m). Further, the law is applicable only to point charges. This is the
oooo

limitation of Coulomb’s Law.


eB

3. Coulomb s law of electrostatic force between two charges corresponds to Newton’s law of gravitational

force between two masses, i.e., f - ^ ^ , where G is universal gravitational constant


r

r^
ou
ad

= 6-67 X 10"* ● Nm2/kg2. This value is much smaller compared to the value of electrostatic force
constant, /: = 9 x 10^ Nm^/C^. That is why electrostatic forces are far more stronger than the gravitational
YY

forces. This is evident from the fact that a charged glass rod attracts a piece of paper against the
nndd

gravitational pull of earth on the paper.


Re

Further, whereas electrostatic force may be attractive or repulsive depending on the sign of charges,
Fi

gravitational force is always attractive.


4. Characteristics of Electrostatic force or Coulomb’s force
(/) The Coulomb force is a central force as it acts along a line joining the two point charges.
(ii) The Coulomb force is a conservative force as the work done in taking a charge from one point to
another in the electric field of another charge is independentof the path followed.
(in) Electrostatic force between two charges is spherically symmetric.
(iv) Electrostatic force between two charges obeys inverse square law.
(v) Electrostatic force between two charges is attractive for unlike charges and it is repulsive for like charges.
(vi) Electrostatic force between two charges is not affected by the presence or absence of any other charge.
(v/7) Coulomb’s law governing electrostatic force between two charges correspondsto Newton’s Law of
gravitation.
(viii) Coulomb’s law is in accordance with Newton’s third law of motion Le.. forces of action and reaction
are equal and opposite.
1/14 U Fundamental Physics (XII) vifumt

^amt^Nprpbiem How is force between two charges affected when each charge is doubled
and distance between them is also doubled ?

Sol. As F OC
I ‘ ?2 ' F becomes
(2) (2) .
time = 1 time, i.e., force remains the same.
r2 (2)2

1.14. UNITS OF CHARGE

The SI unit of charge is coulomb.


We can define unit charge from eqn. (3). Suppose = q2 = q , r= I m and F = 9 x 10^ N
From (3), 9 x 10^ = 9 x 10^ or = 1 or g = ± 1 (coulomb), Hence,

Unif charge in SI (Le. one coulomb) is that much charge which when placed in vacuum at a distance of

ww
one metre from an equal and similar charge would repel it with a force of 9 x 10^ newton.
The cgs unit of charge is 1 electrostatic unit (e.s.u.) of chaise or stat coulomb.
It is also called one franklin (Fr), in honour of an American scientist Franklin for his contributions to

Flo
the study of electrostatics.

e
As charge on an electron is 4-8 x 10“**^ stat coulomb, therefore,

reree
^SxlO"**^

r FF
1-6 X 10“*^ coulomb = 4-8 x 10 stat coulomb or 1 coulomb = stat coulomb
1-6x10"^9
uurr
1 coulomb = 3 X 10^ stat coulomb ...(10)
foor
Yet another unit of charge is electromagnetic unit (e.m.u.) of chaise, where
ks s
Yoo
1 e.m.u. of charge = 3 x 10*^ e.s.u. of charge (stat coulomb) = 10 coulomb.
ooook
eBB

1.15. DIELECTRIC CONSTANT OR RELATIVE ELECTRICAL PERMITTIVITY


When the charges are situated in a medium other than free space (vacuum or air), the force between
rr

them is given by
ouu
ad

1
Fm X ...(11)
YY

47ie r2
where e is called absolute electrical permittivity of the intervening medium.
nndd
Re

The force between the same two charges held the same distance apart in vacuum is
1
Fii

^0 = .(12)
47EG 0 r2

^0 _ e = € or iir ...(13)
Dividing (12) by (11), we get
Fm ^0

where €^is called relative electrical permittivity of the medium. It is also called dielectric constant
of the medium and is denoted by K.
From (13), we may define
'Dielectric constant of a medium is the ratio of absolute electrical permittivity of the medium to
the absolute electrical permittivity offree space. Also,
Dielectric constant of a medium may be defined as the ratio of force of interaction
between two point charges separated by a certain distance in airivacuum to the force of attraction/
repulsion between the same two point charges, held the same distance apart in the medium.
ELECTROSTATIC CHARGES AND FIELDS 1/15

The value of K depends only on the nature of medium.


For example, for vacuum, K = 1-00000 ; for air, K = 1-006 ; for hydrogen, K~ 1-00026 ; for glass,
A: = 3 to4 ; for mica, = 3 to 6 ; for water, A:= 81 and so on.
From (13), € = Sa

1
Using it in (11), F.m
47CSn0 K -p K

Thus, force between two given charges held a given distance apart in water (K = 81) is only 1/81
of the force between them in air/vacuum.

oww
I sampio'.probiom^ Two equal like charges in air repel eachother with a force F. By what
percentage should each charge be reduced so that the force between them in a medium of dielectric
constant 2 reduces by 28% ?
Sol. Let ^ j = ^2 = ^ r be the distance between them.

ee
F = ...(/)

FFrlo
4tU€ 47ie r2

r
0 0

rF
ee
Let the charge be reduced to q^' = q^= q'
a
Kqq' _ 2q

rF
ouru F' =
47CGq P ^^^0

fosor
72
/^ = (100-28)% =
Now,
s kf 100
ooko
F' _ 2^ 72
Yo
Y
T~ q^ 100
Bo
reeB

_ 36 , q' _ ^
ooY
uur

Percentage reduction in charge


ad

{q-q^ xl00 =
(g-6g/10) X100 = 40%
dY

q q
nidn
Re

1.16. FORCES BETWEEN MULTIPLE CHARGES : PRINCIPLE OF SUPERPOSITION


The mutual electric force between two charges is given by Coulomb’s law. However, when we have to
FFi

calculate the force on a charge due to several stationary charges, we use the principle of superposition in
addition to Coulomb’s law.
According to superposition principle, totalforce on any charge due to a number ofother charges
at rest is the vector sum ofall the forces on that charge due to other charges, taken one at a time. The
forces due to individual charges are unaffected by the presence or absence of other charges.

Suppose point charges q^, q2> q^ ●●●● ate situated at points with position vectors , r2» r^, . ...r
n

respectively w.r.t. the origin O of the rectangular co-ordinate system XYZ, Fig. 1.8.
In general, total force Fq on a test charge qQ at position Tq due to all the n discrete charges can be
written as

...(14)
^0 “^01 ■‘■^02 ■*‘^03 — + On
1/16
Fundamental Physics (XII) VOL.5

The component forces are shown in Fig. 1.8.


Here, .f01 - force on due ioq^, = force on q^ due to ^2


and so on, ,FOn = force on q^ due to q^.
1 A

According to Coulomb’s law, .F01 “ 10


47te r2
0 'io
This is the force on due to q^, even though other charges are
present.
Similarly, force on q^ due to ^2* ®ven when other charges are
present is given by

oww
1 % ^2 ^
E and so on
02 “
47ce 2 '220
0 ^20
1
Mn;nO
47C€
0
r2
nO

e
Putting these values in eqn. (14), we get total force on charge as

ree
rFl
Fre
1
^0 ^1 r,^
^ "T, % ^2
F.0 /U/V + r,„+....+ pnO

rr F
“ 10 20 30
4 Tie
0 L '10 r}.
20
r,230 r2nO
ouur
^0 =
I

4 Tie
t = n

I ^r. iO
sfoo
0 1=1 'io
kks
Yo
oooo

t=n
%
^0 = y
^ .3 'io
eBB

0 «=i ^0
urr

F=-^y
You
ad

i?-r |3 I
dY

The direction of net force Ff0 on charge q^ due to n discrete charges can be determined using polygon
innd

law of vectors.
Re

Retain in Memory
Fi

We can use superposition principle for computing (/) net force («) net field (Hi) net flux (iv) net

potential as well as net potential energy at the observation point P due to any configuration of
charges.

1.17. CONTINUOUS CHARGE DISTRIBUTION —


As charge can exist only as integral multiple of basic charge (e), therefore, charge distribution is always
discrete, on account of atomicity of charge. However, it is impractical to work in terms of discrete charges
always. For example, on the surface of a charged conductor, we cannot specify the charge distribution in
terms of the locations of the microscopic charged constituents. However, we can consider a small area element
AS on the surface of the conductor. This area element is very small on the macroscopic scale, but big enough
to include a very large number of electrons. If AQ is the amount of charge on this element, we define surface
AQ
charge density (a) at the area element by a =
AS.
1/17
ELECTROSTATIC CHARGES AND FIELDS

We can repeat the process at different points on the surface of the conductor and thus arrive at a
continuous function O, called the surface charge density. At the microscopic level, charge distribution is
discontinuous, as there are discrete charges separated by intervening space, where there is no charge. Therefore,
O represents macroscopic surface charge density which is a smoothed out average of the microscopic charge
density over an area element AS, which is small macroscopically but large microscopically.
On the same basis, when charge is distributed along a line, straight or curved, we define
AQ
linear charge density, X =
AL
where Al is a small line element of wire on the macroscopic scale that includes a large number of microscopic
charged constituents and AQ is the charge contained in that line element. The units of X are C/m.
AQ
The volume charge density is defined in a similar manner as p =

w
where AQ is the charge included in the macroscopically small volume element AV that includes a large
number of microscopic charged constituents. The units of p are C/n?.

Flo
Note that the notion of continuous charge distribution is similar to the continuous mass distribution
in mechanics. For example, when we talk of density of a liquid, we are referring to its macroscopic density

ree
treating it as a continuous fluid and ignoring its discrete molecular constitution.

rF
1.18. FORCE DUE TO CONTINUOUS DISTRIBUTION OF CHARGES
The force due to a continuous charge distribution can be obtained in almost the same way as for a
ur
system of discrete charges.
fo
Suppose a continuous charge distribution in space has a volume charge density p. With respect to any
s
ok
suitably chosen origin O, let the position vector of any point in the charge distribution be r. . The volume
Yo
Bo

charge density p is a function of ? , i.e., it may vary from point to point. Divide the charge distribution into
small volume elements of size AV. Therefore, charge in this volume element is AQ = p ● AV
re

charge distribution with position vector OP — Tq .


Consider any general point P inside or outside the
ou
ad

Using Coulomb’s law, force due to charge element AQ on a small test charge at P is
Y

dF = 1 % (Ag) p ^ % pAV p,
4TE€ r'2 4tC€ 0 r'2
nd
Re

where / is the distance between the charge element and point P ; and r' is unit vector directed from charge
Fi

element to the point P. As is clear from Fig. 1 -9(a), P = (^ - ^)


1/18
'P’Mdeefi, '4. Fundamental Physics (XII) VOL.I

By the superposition principle, total force due to entire volume charge distribution is obtained by summing
over the forces due to different volume elements.
F =
%
4ne ...(16)
0 allAV

When AV^ 0, the sum becomes an integral and total force can be written as

%
F = pdV p
r'2 ...(17)

w
Proceeding as above, we can write total force due to continuous line distribution of charge as shown in
Fig. \.9(b) as

% f ^

e
F =
7^^

row
47ce ...(18)

re
0 L

And total force due to continuous surface distribution of charge as shown in Fig. 1.9(c) as

FFllo
F
% ads
F =
4tC€ 7^^ ...(19)

u
ree
0 s

sFr
1.19. CONCEPT OF ELECTRIC FIELD

kro
When an electric charge Qq is held in the vicinity of another charge Q, then the charge qQ experiences a.
uor
force of attraction or repulsion. We say that this force is due to interaction of electric fields of two charges Q
offo
and qQ. Thus, electric field of charge Q may be conceived as the space property by virtue of which, the
presence of charge Q modifies the space around Q leading to the development of a force on any charge qn
kos
Y
held in this space. We may define.
Yo
eerBB
oo

Electric field due to a given charge as the field that permeates the space around the charge, in
rY

which electrostatic force of attraction or repulsion due to the charge can be experienced by any
other charge.
u

If a test charge experiences no force at a point, the electric field at that point (due to some other charge
ou
ad
do

some where) must be zero.


nY

The concept of field was introduced first of all by Faraday.


The charge Q, which is producing the electric field is called a source charge and the charge q^ which
nid

tests the effect of source charge is called a test charge. The source charge can be a single charge or a group
Re

of discrete charges or any continuous distribution of charge.


F
Fi

IMPORTANT NOTE

Note that test charge acts as a detector of the electric field. It is an infinitesimally small charge so that
it affects least the electric field of source charge.

1.20. ELECTRIC FIELD INTENSITY

The electric ifeld intensity at any point is the strength of electric field at that point It is defined
as the force experienced by unit positive charge placed at that point
If F is the force acting on a small test charge + at any point ~r , then electric field intensity at this
point is given by

E(t) =
%
ELECTROSTATIC CHARGES AND FIELDS 1/19

The SI unit of electric field intensity is newton per coulomb (N/C)


Electric field intensity is obviously a vector. The | FIGURE 1.10

direction of E is the same as the direction of F .

E is along the direction in which the test charge + qQ


would tend to move if free to do so.
For a positive source charge, the electric field will be directed radially outwards from the charge, Fig.
1.10(<3). If the source charge is negative, the electric field vector at each point is radially inwards, as shown in

w
Fig. \.\0{b).
As the test charge + qQ may have its own electric field, it may modify the electric field of the source
charge. Therefore, to minimise this effect and ultimately remove it, we rewrite electric field intensity at r as

e
row
F(7)

re
E{r) = Limit ...(20)
%

FFllo
F
As the magnitude of test charge qQ decreases, electric field intensity £ (7) at a point is defined more
and more accurately. On account of discrete nature of charge, the minimum possible value of test charge qQ is

u
ree
1-6 X 10"^^ coulomb (which is the unit charge). It cannot be zero.
However, on the macroscopic scale, it is as good as taking the limit qQ—^0. If the small test charge qQ

sFr
is positive, the measured value of electric intensity will be somewhat less than the actual value of electric

kro
intensity. Howeve,r if the small test charge qg is negative, the measured value of electric intensity will be
uor
somewhat more than the actual value of electric intensity.
offo
Values of some electric fields are given in Table 1.4.
kos
TABLE 1.4. Values of some electric fields
YYo
eerBB
oo

Field Location Value of E (N/C) or (V/m)


rY

1. At the surface of uranium nucleus. 3 X 10^’


11
2. Within a hydrogen atom. 5 X 10
uu

3. Electric breakdown of dry air. 3 X 10^


10^
ad
doo

4. Near a charged comb.


5. In a copper wire of house hold circuits. 10-2
nY

Sample Problem Calculate forcc on an electron in a uniform electric field of 5 x lO'* N/C due
nid
Re

north.
FFi

Sol. Here, = 1 e = - 1-6 x 10"^^ C, E = 5 x 10"^ N/C, due north


F=qQE = - 1-6 X 10-^^x 5 x 10'^N=-8 x 10“*^ N due north = 8 x 10"^^ N due south.
1.21. ELECTRIC FIELD INTENSITY DUE TO A POINT CHARGE
Suppose we have to calculate electric field intensity at any point P
due to a point charge Q at O, where OP = rg. Imagine a small positive test
charge qQ at P, Fig. 1.11. According to Coulomb’s law, force on charge qQ
at P is

F =
1 ggg A

where Pg is unit vector directed from Q towards ^g.


1
As E = ...(21)
4 ^
%
1/20
7\<*dcefi.'4. Fundamental Physics (XU) VOL.I

In general, the electric field vectors at several points around a positive FIGURE 1.12
point charge are directed radially outwards as shown in Fig. 1.12.
A^ow, suppose the point charge + Q is situated at A, where .

We have to calculate electric field intensity {£) at P, where OP = 7^ Fig


1.13. ° ♦

According to Coulomb’s law, force on a small test charge {+ q^) at P is


F = 1 Q% A 1 Qq^
4;c6 r2 % %
4;c€
0 % 0 '/o
FIGURE 1.13
F =
1 0?,0

ww
FF loo
■»
where
='io =

ree
F
As E- —

rreeF
%
r ur
1 Q
E =
%-ri) fofroF ...(22)
ks
Obviously, it is along AP produced.
YYouo
kos o

1.22. ELECTRIC FIELD INTENSITY DUE TO A GROUP OF CHARGES


BBoo

The electric field intensity at any point due to a group of point charges is equal to the vector sum of the
r ee

electric field intensities due to individual charges at the same point. For this, we first calculate the electric
field intensity at the given point P due to individual charges, as if other charges were not there and then add
ad
oouur

them vectorially, Le.,


£-£i + £2+£3+-.£’„
Y

i=n
1
d
Re

i
E =
dnY

r.
4 Tie
0
^ r? ‘
/=] 'i
...(23)
FFini

Here, r,- is the distance of the point P from the ith charge Qj and .r is unit vector directed from Q- to the
point .P

Now, suppose the point charges Qj, Q2, Q3, ....j2n located in vacuum at positions ri,r2,r3, r
n

respectively. We have to calculate electric field intensity ~e at any point P whose position vector is 'b . As
net force at P is vector sum of the forces due to individual charges, therefore, we can write
i=n

F =
% Y —Q
4 Tie ^ I i3
0 1=1 I Tq- .r P
i=n
1

% Sr 'b I
ELECTROSTATIC CHARGES AND FIELDS 1/21

Note that £ is a vector quantity that varies from one point to another point in space. Its value depends
on the positions of the source charges. The direction of E due to a group of discrete charges is determined
using polygon law of vectors.

1.23. ELECTRIC FIELD INTENSITY DUE TO CONTINUOUS CHARGE DISTRIBUTION


In the previous chapter, we have studied three kinds of continuous charge distributions in Article 1.17.
As explained already, total force on a small test charge + at due to linear charge
distribution of density X(^) is
F =
% Wr^)dl

ww
As E = F !q 0

FF loo
Electric field intensity at location ^ due to linear charge distribution X(r.) is

ee
1 'k(ri)dl ..(24)

er
E =

Fr reF
Similarly, we can write expressions for electric field intensity at position Tq due to surface charge
rur
distribution (5) and volume charge distribution (V) at fforo
as
ks
o(r)dS
YYouo
1
okso

E = (25)
0 S l7^-r 13
47ce
^'0
BBoo

p(r)dV
r ee

1
E = ...(26)
ad
oouur
Y

1.24. RECTANGULAR COMPONENTS OF ELECTRIC INTENSITY DUE TO A POINT CHARGE


dnYd
Re

Suppose a point charge + g is held at O, the origin of co-ordinate system shown in Fig. 1.14. We have
FFini

to calculate rectangular components of electric field intensity (£) at any point P (at, y, z), where
■> -1 ^ A A
OP = r = ix + jy + kz

r =1 7^1 = + z^ ...(27)

Force on unit + charge at P i.e.


1 Q A
E = ~ r
...(28)
4;t€n0

£ =
Q Qjtx+jy + kz)
4lt£o " 4neQ (x^ + y^ + z^)3^^
..(29)

If £X , £ y , £ z are the components of E along the three


co-ordinate axes, then
1/22
"PnAdee^ <2. Fundamental Physics (XII) VOL.I

E = iE^ +
or
...(30)
Comparing (29) and (30), we get

E
Qx
^ Atz.€q{x^ + ...(31)

E Qy
^ 4tC€q (x^ + +^2)3^2 ...(32)

E
Qz
^ 4tcGq(x^ + >’^ ...(33)

We find from (28) that as seen from O, the position of point charge + Q, the electric field is spherically

w
symmetric, i.e., ai x y = zE^ = E^, = E^, i.e., at equal distances from the charge, field intensity is equal.
Further, electric intensity at any point varies inversely as the square of the distance of the point from the

Flo
cnarge.

e
1.25. PHYSICAL SIGNIFICANCE OF ELECTRIC FIELD

rree
In Physics, the term ‘field’ generally refers to a quantity that is defined at every point in space. The

r FF
electric field at a point in space around a system of charges gives us the force a unit positive test charge would
experience, when placed at that point, without disturbing the system. The field is independent of the test
uurr
for
charge that we place at a point to determine the electric field. This field varies from point to point. As force
is a vector quantity, therefore, electric field is a vector field.
kss
ooook

From the knowledge of electric intensity E at any point Ir , we can readily calculate the magnitude
Yo

and direction offorce experienced by any charge held at that point, i.e.,
eB

F{7) = q^E{7)
rr

This is the physical significance of electric field.


ou
ad
Y

1.26. ELECTRIC FIELD LINES


dY

Electric field lines map pictorially, the electric field around a configuration of electric charges. The
Re
innd

concept was developed by Faraday and is a non mathematical way of visualizing electric fields around charged
configurations. We may define.
Fi

Aw electric field line as a path, straight or curved in electric field, such that tangent to it at any
point gives the direction of electric field intensity at that point.
Infact, it is the path along which a unit positive charge actually moves in the electrostatic field, if free to
do so. In Fig. 1.15, AB is an electrostatic line of force. The tangent to the line at any point P gives us the

direction of electric intensity Ep at P. Similarly, tangent to AS at 0 gives us the direction of Eq . The field
lines are in three dimensional space, though Fig. 1.15 shows them only in a plane.
Fig. 1. 16 shows some lines of force due to single positive point charge. These are directed radially
outwards. The lines of force extend to infinity. On the contrary, lines of force due to single negative point
charge are directed radially inwards. Fig. 1.17.
Fig. 1.18 shows lines of force due to a pair of equal and opposite charges, {q, - q), forming an electric
dipole*. They show clearly the mutual attraction between the opposite charges.
For details, refer to Art. 1.28
ELECTROSTATIC CHARGES AND FIELDS 1/23

charge

ww
The lines of force due to two equal positive point charges {q, q) are shown in Fig. 1.19.
As is clear from Fig. 1.19, the field lines due to two charges and +^2) lateral pressure on each
other, which is responsible for repulsion between the like charges. Further, in Fig. 1.19, point F represents the
neutral point, where net electric field intensity is zero.

Flo
When the charges are equal, P lies at the centre of the line joining the charges. However, when the

e
ree
charges are unequal, the neutral point P is closer to the smaller charge as shown in Fig. 1.20.

FFr
Fig. 1.21 shows field lines for a section of an infinitely large sheet of positive charge.
uurr FIGURE 1.21

orr
+

sfo
kks
+
Yo
oooo

+
eB

+
rr
ou
ad
YY

1.27. PROPERTIES OF ELECTRIC FIELD LINES


nndd
Re

1. Electric field lines are continuous curves. They start from a positively charged body and end at a
Fi

negatively charged body. No electric lines of force exist inside the charged body. Thus, electrostatic field
lines are continuous but do not form closed loops.
If there is a single charge, the electric field lines may start or end at infinity. For example, due to a single
positive charge, field lines would start from the charge and end at infinity. Due to a single negative charge,
field lines would start from infinity and end at the negative charge.
We shall study in unit 3 that magnetic lines of force are continuous (or endless) closed loops as
against the electric lines offorce which are continuous but not closed loops.
2. Tangent to the electric field line at any point gives the direction of electric FIGURE 1.22

field intensity at that point.


3. No two electric field lines offeree can intersect eachother. This is because
at the point of intersection P, we can draw two tangents PA and PB to the two lines
of force. Fig. 1.22.
This would mean two directions of electric field intensity at the same point, which is not possible.
Hence no two electric lines of force can cross each other.
1/24 ‘Pnadeefi- '4. Fundamental Physics (XII) WMBI
4. The electric field lines are always normal to the surface of a conductor, both while starting or ending
on the conductor. Therefore, there is no component of electric field intensity parallel to the surface of the
conductor.

5. The electric field lines contract longitudinally, on account of attraction between unlike charges.
6. The electric field lines exert a lateral pressure on account of repulsion between like charges.
7. Electric field lines pass through dielectrics.
The magnitude of the field is represented by the density offield lines. Near the charge, lines are closer,
—>

so the density of field lines is more and hence electric field E is stronger near the charge. Away from the
charge, the field lines are well separated, the density of field lines is smaller and hence electric field £ is
weaker, away from the charge. We may, therefore, define

w
Electric field intensity at a point is equal to number offield lines crossing normally a unit area
around that point.

Flo
As field lines are three dimensional and all figures are drawn on the plane of the paper (in two dimensions),
therefore, to estimate the density of field lines, we have to consider the number of field lines per unit cross-

ee
sectional area perpendicular to the lines. We know that in three dimensions, the solid angle A(2 subtended by

Fr
a small perpendicular plane area AS at a distance r from apex can be written as aq = —.

Wi’r/i a charge Qat the apex O of the cone, number of radial field lines in a given solid angle is the

or
ur
same. In Fig. 1.23, let us consider two points Pj and at distances rj and r2 respectively from the charge.
sf
At F,, area subtended by the solid angle A£2 is r^ .AQ . At F2, area subtended by the solid angle AQ is
r2-AQ.
ok
Yo
The number of field lines cutting these area
Bo

elements are the same. Let it be n.

Number of field lines cutting unit area


re

element at Fj = electric intensity at Fj, i.e..


ou

n
ad

E.=- , and number of field lines cutting unit


* r,2.AQ
Y

area element at F2 = electric intensity at F2, i.e..


nd
Re

n
£2 =
r^.lsD.
Fi

2
1
Clearly ; =i i.e., E oe — i.e., electric field intensity is more near the charge and vice-versa,
F r2 r^
1

1.28. ELECTRIC DIPOLE

An electric dipole consists of a pair of equal and opposite point charges separated by some
small distance.

In Fig. 1.24, we have shown two equal and opposite point charges
FIGURE 1.24
(± q) separated by a small distance '2a'. It represents an electric dipole. 2 a is
called length of dipole. It can be taken as a vector whose direction is from p

negative charge to positive charge of dipole. -q 0 + q


H- 2a -H
The middle point of locations of - ^ and + 9 is called the centre of the
dipole. It is represented by O.
ELECTROSTATIC CHARGES AND FIELDS 1/25

The total charge of the electric dipole = - q + q = 0. FIGURE 1.25

w
This does not mean that the field of the electric dipole is zero. MOLECULE OF WATER /
i ● )HYDROGEN
Atoms or molecules of ammonia, water, alcohol, carbon dioxide,
HCl etc. are some of the examples of electric dipoles, because in
their cases, the centres of positive and negative charge distributions NEGATIVEI /
/N''POSITIVE

r
are separated by some small distance. Fig. 1.25 shows a molecule of SIDE loXYGEN^/^Q*g;, SIDE
water {H2O) with three nuclei represented by dots. The electric dipole
● ) HYDROGEN
moment p points from the (negative) oxygen side to the (positive)

r
hydrogen side of the molecule as explained in Art. 1.29.

llowu
o
F
1.29. DIPOLE MOMENT

Dipole moment (p) is a measure of the strength of electric dipole. It is a vector quantity whose

s
FF
magnitude is equal to product of the magnitude of either charge and the distance between

ro
them.

rek
er
p = q(.2a) \p\=q{2a) .(34)

Fo
i.e.j or

ro
By convention, the direction of P is from negative charge to positive charge.

of
Y
o
YY
The SI unit of dipole moment is coulomb-metre (C-m).
The dimensional formula of dipole moment is M ® (AT) L = [Af ® L* A' T’ ]
sf
ou
oB
If charge q gets larger, and the distance 2a gets smaller and smaller, keeping the product I pi -q'x2a
ok
= constant, we get what is called an ideal dipole or point dipole. Thus, an ideal dipole is the smallest dipole
Br

having almost no size.


ud
ee

Physical significance of electric dipoles


The study of electric dipoles is important for electrical phenomena in matter. Matter, as we know,
consists of atoms or molecules which are electrically neutral. In a molecule, there are positively charged
Yn
ur
oo

nuclei and negatively charged electrons. If the centre of mass of positive charges coincides with the centre of
ad

mass of negative charges, the molecule behaves as a non-polar molecule. On the contrary, if the centre of
di

mass of positive charges does not coincide with the centre of mass of negative charges, the molecule behaves
as a polar molecule and it possesses some intrinsic or permanent dipole moment. In the absence of an
FF
Re

external electric field, the dipole moments of different molecules in a piece of matter are randomly oriented,
in

so that net dipole moment of the piece is zero. When an external electric field is applied, the polar molecules
tend to align themselves along the field and some net dipole moment develops. The piece of matter is said to
have been polarised.
When non-polar molecules are subjected to the action of an external electric field, the centres of mass of
positive and negative charges in the molecule get displaced in opposite directions. Thus, the external field
induces some dipole moment in the molecule in the direction of the field. The induced dipole moments of
different molecules in the sample add up vectorially to produce some net total dipole moment.

IsaynyieMrpbiemj Charges ± 20 nC are separated by 5 nun. Calculate the magnitude and


direction of dipole moment.
Sol. Here = q2 = q = ±20 n C = ±20'.x 10“^ C
2 a = 5 mm = 5 x 10“^ m

p = 9 (2 a) = 20 X 10“^ X 5 X lO'^ = 10"*® C.m


The direction of ~p is from negative charge to positive charge.
1/26 ‘PficicCeep, j. Fundamental Physics (XII) kV»1H
1.30. DIPOLE FIELD

The dipole field is the electric field produced by an electric dipole. It is the space around the dipole in
which the electric effect of the dipole can be experienced. The electric field of the pair of charges (- q and
+ q) at any point in space can be found from Coulomb’s law and the superposition principle. To calculate
dipole field intensity at any point, we imagine a unit positive charge held at that point. We calculate force on
this charge due to each charge of the dipole and take vector sum of the two forces. This gives us dipole field
intensity at that point. We shall show that though the total charge of electric dipole is zero, the field of the
electric dipole is not zero. This is because the charges q and - q are separated by some distance. The electric
fields due to these charges, when added, donot cancel out. However, at distances much larger than the separation
of two charges, i.e., r»la, the fields due to q and - q largely cancel out. Therefore, at large distances from
( 1 ) 1
the dipole, the dipole field falls off more rapidly like £ »= — than like E OC
for a point charge.
I .

ww
1.31. FIELD INTENSITY ON AXIAL LINE OF ELECTRIC DIPOLE

Floo
Consider an electric dipole consisting of two point charges - q and + q separated by a small distance 2a.
—>

We have to calculate electric intensity £ at a point P on the axial line of the dipole, and at a distance

ee
OP = r from the centre O of the dipole. Fig. 1.26.

reer
FIGURE 1.26
—>

rFF
If £j is the electric intensity at P due to charge -q + q +1
E

- q, at A, then
uur r
0
_! q__ 1 A
l£ i
4tc6o Ap2
4

47CGy (/- + a)2


M-
ffoor 2a
B
M
E2
sks
M- r -M
It is along PA
YYoo
ooko

Suppose £^ is the electric intensity at P due to charge + qaiB. then


eBB

I q 1 q
I £2! =
47ceQ Bp2 4tcGq {r~a)^
r

It is along BP produced.
ouur
ad

As £j and £., are collinear vectors acting in opposite directions and I £2 ^ * therefore, the
YY

resultant intensity £ at Z’ will be difference of two, acting along BP produced.


nndd
Re

1 q 1 q q 1 1
I £ I = I £2 I -1 £1 I =
FFii

4tuGq {r-a9 4tcGq (r + a)^ 47CG


0 L {r-af {r + a9
q {r + a)^ - {r - a)^ q Aar qxlaxlr
4tcg
0 L (r2-«2)2 4tcGq (r2 4tcGq (r^ —a^y
\p\ Ir
But ^x2a =1 p\, the dipole moment l£l = ...(35)
47CGQ'(r2_^2)2
If dipole is short, 2a<< r.
—>

I £1 =
I p\ 2r 21 p\
...(36)
4tiG(j / 47CGQr^
The direction of £ is along BP produced, Le., along the direction of p .
I
Clearly, I £1 OC

r3
ELECTROSTATIC CHARGES AND FIELDS 1/27

132. FIELD INTENSITY ON EQUATORIAL LINE OF ELECTRIC DIPOLE


Consider an electric dipole consisting of two point charges - q and + q separated by a small distance
AB = 2a with centre at O and dipole moment, p-q (2 a). Fig. 1.27.
We have to find electric intensity £ at a point P on the equatorial line of the dipole, where OP = r.

If £, is electric intensity at P due to charge - ^ at A, then FIGURE 1.27

D
1 Q F
l£.1 I =
0
AP2
But AP2 = OP'^ + OA^ = r2 +
R
1 \

l£.1 1 = ...(37) \

47ceo (r2+fl2)

w
\
C \
> / \
/ \
£j is represented by pc ■ /
/ \
\
r

FF llowo
Let ZPBA = ^PAB = 6. / V
/ \
/ \

has two rectangular components :


/ N

ree
\

£] cos 6 along PRWBA and £j sin 0 along PE X BA A4


A® P 0
d

rF
-q O +q
If £"2 is electric intensity at P, due to charge + ^ at B, then 2a H
ur
1 q 1 q
1^21 = 47C6n BP'^
0 47CGq (r2+fl2) forfFreo (38)
ks
> —^

£2 is represented by PD (along BPD). £2 rectangular components,


YYour o
oo

£2 cos 0 along PR 11 BA and £2 sin 0 along PF (opposite to PE)


BoBks

From (32) and (33), as I £j I = I £2 I £j sin 0 along PE and £2 sin 0 along PF cancel out
r ee

—^

.'. Resultant intensity at P is given by I £ I = £j cos 0 + £2 cos 0


oour u
ad
Y

I £ I = 2 £.I cos 0
(*.● I £, I = I £2 I)
—* 2 2 OA 2
nd

q q a
Re

I £1 = —— cos 0 =
47C€q (r^ 4tzg^ (r^ +a^)l^AP 4tC€o (r2+fl2)^^2^^2
FFind Yi

qx2a
4kEq (r^
Ipl
But ^x2fl = I £ I, the dipole moment I £1 = ...(39)
47C€q (r^
>

The direction of £ is along PR || £A (/.e., opposite to p ).

In vector form, we can rewrite (34) as £= Z£


47ceg (r^
Obviously, £ is in a direction opposite to the direction of p .
1/28 '<1 Fundamental Physics (XII)

If the dipole is short, 2a<<r 1£1 = ...(40)


4tc€^0
1
Clearly, \E\ OC

E
axiai
From eqns. (36) and (40), we learn that = 2

w
c
equatorial

i.e., at a given distance from the centre of dipole, electric field intensity on axial line is twice the electric
intensity on equatorial line. Further, both the magnitude and the direction of dipole field depend not only on

e
the distance r, but also on the angle between the position vector r and dipole moment p .

re
o
Note. Position on Axial Line of an electric dipole is also known as End on position. And position on

rw
equatorial line of an electric dipole is also known as Broadside on position.

F
1.33. ELECTRIC FIELD INTENSITY AT ANY POINT DUE TO A SHORT ELECTRIC DIPOLE

ullo
FF
In Fig. 1.28, AB represents a short electric dipole of moment P along ab . C> is the centre of dipole.

ee
srr
We have to calculate electric field intensity E at any point K, where OK = r, ZBOK = 6.

roF
k
The dipole moment p can be resolved into two rectangular
uor
components :
ofof
(p cos 0) along Aj B| and (p sin 0) along A2B2 J-Aj S|.
kos
Field intensity at K on the axial line of Aj fij.
YYo
B oo

2 p cos 0
l£.1 1 =
rY

47t€.>
eerB

0
u

Let it be represented by along OK.


Field intensity at K on equatorial line of A2 B2,
oou
d
ad

psin 0
nY

I £2! =
47t€n0
nid
Re

Let it be represented by KM I ^2 Az ‘ Complete the rectangle KLNM. Join KN .


FFi

According to 11 gm law, KN represents resultant intensity (£) at AT due to the short dipole.

As KN ^^KiFTki^
r n2
2 p cos 0 p sin 0
I £1 = .^£,2 + £| = 47ce«
+
47ce«0
-—--^4cos^ 0 + sin^ 0
0

l£l = -—-Jzcos^ 0 + 1
——- ^3 cos^ 0 + (cos^ 0 + sin^ 0) = 47te«r3
47C€.,0 0

i.e.. , Jl ^ pV3cos^0
47te„
+1 ..(41)
a

Let ZLKN=a
ELECTROSTATIC CHARGES AND FIELDS 1/29

In AKLN, tan a - LN ^KM _ psin 6 or


1
tana = —tan0 a can be calculated.
KL KL 2/?cos 0 2

Particular cases :

1. When the point K lies on axial line of dipole.


0 - 0°, cos 0 = cos 0® - 1

I £ I = ^—Jscos^O^+l = 2p 1
T and tan a = - tan 0® = 0,. a = 0®
4tc€.0 3 2

i.e., resultant intensity is along the axial line.


2. When the point K lies on equatorial line of dipole.
0 = 90®, cos 0 - cos 90® - 0

ww
— -^cos2 90°+1 = 4ji€
P p 1
E = r and tan a = — tan 90® = ■», « = 90°
ri
4tC€
0 =0'-' 2

Floo
i.e., direction of resultant field intensity is perpendicular to the equatorial line (and hence antiparallel to
axial line of dipole).

ee
We have already proved these results in Arts 1.31. and 1.32.

reer
rFF
1.34. ELECTRIC FIELD INTENSITY AT ANY POINT
ON THE AXIS OF A UNIFORMLY CHARGED RING
uur r
Consider a circular loop of wire of negligible thickness, radius a and centre O held perpendicular to the
plane of the paper. Let the loop carry a total charge + q distributed uniformly over its circumference. We have ffoor
sks
to determine electric field intensity at any point P on the axis of the loop, where OP = r, Fig. 1.29.
YYoo
Consider a small element AB of the loop. Let length of element AB = dl and C be the centre of the
ooko

element.
eBB

FIGURE 1.29
Charge on the element AB is
dq = —^
2na
dl ...(42) a
\
v +
"
r

+
dEcos0
ouur

_ a,
► X
ad

Electric Field intensity at P due to the charge Q ' 0


T dE
element AB is
YY

/ c
? /+ Y''
1 dq
\dE dE sin0
4;C€^0 CP^
nndd
Re

along PC at Z0 with the axis OX.


FFii

1 dq
\dE\ = ...(43)
4te€q (r^ +a^)
dE can be resolved into two rectangular components.
dE cos 0 along PX, the axis of the loop, and dE sin 0 along PY, perpendicular to the axis.
For a pair of diametrically opposite elements of the loop, components of electric field intensity
perpendicularto the axis will cancel, whereas the components along the axis of the loop will add. As the loop
can be considered to be made up of a large number of pairs of diametrically opposite elements, therefore,
^ i/£sin 0 = 0
—> —^
Hence the resultant electric field intensity £ at is \ E \ = ^ dE cos 0
OP r 1 dq r
In tX)PC, cos 0 =
CP (r2+fl2)l/2
l£l=I
4it€Q
1/30 Fundamental Physics (XII)

1 qdl r

Using (42), we get I £ I = ^ 4ne


0 V 'l%a J(r^ +a^
(}r qr{2na)
I di =
4ti:£q 2jia(r^ + a^)^^^ whole loop 47CGo 27lQ(r2+fl2)3/2

\E\ =
..(44)
4tc€q (r^
The direction of E is along PX, the axis of the loop.

Special Cases. (/) When P lies at the centre of the loop,

w
r = 0, therefore, from (44), E - 0

FF llowo
(«) When r»a {i.e. P lies far off from the loop), neglecting in comparison to in (44), we get

ree
I £1 = , along PX
4ti€„0 47ie=0 '●

rF
This is the expression for £ at a distance r from a point charge q. Hence a circular loop of charge
ur
behaves as a point charge when the observation point (P) is at very large distance from the loop, compared
to the radius of the loop, for fFreo
ks
iiii) Electric field intensity due to a uniformly charged ring will be maximum. When
YYour o
oo

dE d qXr q -3r^
= 0 or = 0 or + (r^ +<3^) =0
dr dr 471€
0 (r2+«2)5/2
Bo Bks
r ee

3r2
or = (r2+a2)-3/2 or 3r^ = (r^ + a^)
oour u

(r2+a2)5/2
ad

FIGURE 1.30
Y

or r = ± a/V2. EA
nd
Re

Thus electric field intensity due to a uniformly charged ring will be I


-alj2 i
a ■►r
FFind Y i

from its centre on either side on the axis of T


maximum at a distance r - O alj2
■h \
;

the irng. The variation of electric field intensity due to a charged irng with
the distance r from the centre of the irng is shown in Fig. 1.30.

1.35. ELECTRIC DIPOLE IN A UNIFORM TWO DIMENSIONAL ELECTRIC FIELD


Consider an electric dipole consisting of two equal and opposite point charges - ^ at A and at B
separated by a small distance AB = 2a, having dipole moment I ^1 = ^x2a. Fig. 1.31.
Let this dipole be held in a uniform external electric field £ at an angle 0 with the direction of £ .
—> —^

Force on charge + qai A =q E, along the direction of £ .

Force on charge - q at B = q E, direction opposite to £ .


ELECTROSTATIC CHARGES AND FIELDS 1/31

Since E isuniform, therefore, net force on the


dipole is (qE - qE) = 0. However, as the forces are
equal, unlike and parallel, acting at different points,
therefore, they form a couple which rotates the dipole
in the anticlockwise direction, as shown in Fig. 1.31.
Thus, the couple tends to align the dipole axis along
the direction of field E .

Draw AC X E

perpendicular distance between the forces


- arm of couple - AC

w
As torque = moment of the couple
X = force X arm of couple.
x = Fx AC = F X AB sind = Fx2 a sin Q = (q E)2 a sin Q = (q x2 a) E sin Q

FF llowo
As qx2 a~ p, therefore, X = p £ sin 0 ...(45)

ree
^
In vector form, we can rewrite this eqn. as X = px£ ...(46)

rF
The direction of x is given by and is perpendicular to p and £ ,i.e., perpendicular
to the plane of the paper and outwards.
ur
forfFreo
Special ca.ses. As stated already, this torque tries to align the electric dipole in the direction of the field.
When ^ is along £ , 0 = 0°, x = p £ sin 0° = 0.
ks
YYour o
The dipole is in stable equilibrium.
oo

However, when dipole is held in a direction opposite to £ , the torque would turn the dipole through
180°. As such, the dipole will be in an unstable equilibrium.
BoBks
r ee

The torque will be maximum, when 0 = 90°.

Max. value of torque = p £ sin 90° = p £


oour u
ad

Obviously, units of x are N-w and its dimensional formula is [M* L^T"^].
Y

Note I. When electric dipole is in a uniform electric field (along p ), Fig. 1.32(a)
nd
Re

Net force = q E- q E = 0
FFind Yi

Net torque - p £ sin 0 = 0, as 0 = 0° FIGURE 1.32

II. When electric dipole is in a non-uniform c=C> E'


electricfield o ●-
-q p +q
(a) Field is increasing along p , Fig. 1.32(b) force on -q <J net force = 0 force on +q=qE
= -qE
Net force on dipole is along £ 0 E
o
Net torque = p £ sin 0 = 0, as 0 = 0°, i.e., p -q p +q
less force more force
is along £ on -q
<J <> =<> on +q
net force

(b) Field is decreasing along p , Fig. 1.32(c) E

Net force on dipole is opposite to p o ●-


-q p +q
more force less force
Net torque = p £ sin 0 = 0, as 0 = 0°, i.e. p is on -q
<J on +q
net force
along £ .
1/32 “PtizeUe^ d. Fundamental Physics (XII) P781W1
I sarnpie Pirobiem^ Two Charges of ± 1000 C are separated by 2 mm. The dipole so formed is
held at an angle of 30^ with a uniform electric field of 15 x 10*^ NIC. Calculate the torque acting on the
dipole.
Sol. Here, ^ = ± 1000 |i C = ± 1000 x 10"^C =+ICT^ C, 2 a = 2 mm = 2 x 10"^ 0 = 30®
£=15x 10'*/V/C,T = ?
x = p EsinQ = ^ (2 a) £ sin 0 = 10“^ X 2 X 10“^ x 15 x lO'* sin 30®
1
t = 2x15x10-2 X- ^0.15

1.36. POTENTIAL ENERGY OF DIPOLE IN A UNIFORM ELECTRIC FIELD

Potential energy of dipole is the energy possessed by the dipole by virtue of its particular

ww
position in the electric held.

Suppose an electric dipole of moment p is oriented at an Z0 with the direction of uniform external

FF loo
electric field E , Fig. 1.31. We know, the torque x acting on the dipole is x = p £■ sin 0

ee
It tries to rotate the dipole to align it with E .

ee r
rFrF
Small amount of work done in rotating the dipole through a small angle d0 against the torque is
i/W = xdQ = pE sin 0 d0
rur
.*. Total work done in rotating the dipole from orientation 0j to 02 is
02 ffoor
02
ks
W = pEsmQdQ = pE\- cos 0] = -p E [cos 02-cos 0j] ..(47)
YYoou
0 0,
ookos

Potential energy of dipole. U = W--p E (cos 02 - cos 0j) ...(48)


BBo

Particular cases.
re

1. When the dipole is initially aligned along the electric field, i.e., 0j= 0®, and we have to set it at angle
ouur
ad

0 with E , i-e., 02 = 0-
From (48), W = -pE (cos 0 - cos 0®)
Y

W = -p£(cos 0- 1) (49)
dndY
Re

This work done is stored in the dipole in the form of potential energy.
FFini

2. When the dipole is initially at right angle to E. he., 0j= 90°, and we have to set it at angle 0 with
E , I.C., ©2 = 0
from (48), W=-pE (cos 0 - cos 90°) = -pE cos 0

.*. Potential energy of dipole. U=W-~pEco& 0 ...(50)

U=^-p.E .(51)

Obviously, potential energy of an electric dipole is a scalar quantity. It is measured in joule.

Retain in Memory
To calculate potential energy of an electric dipole at any angle 0, we must start from the position
of zero potential energy (i.e., 0| = 90®). In equation (48), .PE. - W--pE (cos ©2 - cos 0|), we
shall put 0j = 90® and ©2 = 0. Never take 0| = 0®, as it is not the position of zero potential energy.
Therefore, .PE. -W = -pE (cos 0 - cos 90®) = -pE cos 0.
ELECTROSTATIC CHARGES AND FIELDS 1/33

a dipole consists of an electron and a proton separated by a distance of 5 x


10"® m. The dipole is aligned in a uniform electric field of 1*44 x 10“* N/C. Calculate potential energy of
dipole to hold it at 60" with the direction of electric field.
Sol. Here ? = l-6x I0-i9C;2a = 5x 10-^ m.
0t = 0", £ = 1-44 X 10“* N/C, PE = ?, 02 = 60"
P.E. = -pE (cos 02 - cos Oj) = - 9 X 2u £ (cos 02 - cos 0|)
= - l-6x 10“'^x5 X 10-9 X 1-44 X i()4(cos 60"-cos 0°)
1
= _8x1-44 —1 xlO"2^J
2

.PE. = 5-76 X 10“24 J

ww
1.37. AREA VECTOR

Area is a scalar quantity. But in some of the problems, it is convenient to treat it as a vector. The

FF loo
question is how to associate some direction with the area of a curved surface. Let us divide the given closed
area into a large number of very small area elements. Each small area element may be treated as planar. As

ree
normal to the plane specifies the orientation of the plane, therefore, the direction of a planar area vector is
along its normal. But a normal can point in two directions, inwards or outwards. By convention, the vector

eF
associated with every area element of a closed surface is taken to be in the direction of the outward drawn
normal

Frre
r ur
FIGURE 1.33
Thus, an area element vector a5 a point on a closed
surface can be written as fforo AS = fi (AS) AS = n (AS)
ks
r
A5 = ^(A5) ...(52)
YYouo
■ > n
koso

where A5 is magnitude of the area element and « is a unit CE


BBoo

vector in the direction of outward drawn normal at that point, \AreaAS


Area AS
r ee

as shown in Fig. 1.33.

1.38. ELECTRIC FLUX


ad
oouur
Y

Electric flux over an area in an electric field is a measure of the number of electric field lines
crossing this area.
dnYd
Re

We know that the number of electric field lines crossing a unit area placed normal to the field at a
FFini

point is a measure of strength of electric field £ at


that point. If we place a small planar element of area
—>

A£ normal to £ at this point, [Fig. 1.34(tj)], the


number of electric field lines crossing this area
elementis proportionalto E (AS). Note that it is not
proper to say that number of field lines crossing the
area is equal to £ (AS). The number of field lines is
after all, a matter of how many field lines we choose to
draw. What is physically significant is the relative
number offield lines crossing a given area at different
points.

If we tilt the area element by angle 0 [or we tilt £ w..rt. area element by angle 0, Fig. 1.34(/?)], the
number of field lines crossing the area will be smaller. As projection of area element normal to £ is
1/34 '<t Fundamental Physics (XII) LWI

A5 cos 0 (or component of E normal to area element is £cos 0). therefore, number of electric field lines
crossing area A5is proportional to £ (Mcos 0). This is shown in Fig. 34(b).
—^ —>

Hence electric flux A(f> through an area element AS in an electric field £ is defined as

A(|)==£.A5 = £(A5) COS0 ...(53)

which is proportional to the number of field lines cutting the area element.

Here, 0 is smaller angle between £ and ● For a closed surface, 0 is the angle between £ and
outward normal to the area element.

w
Now, A<1)-£.A5 =£ A5 cos 0 = £ (A5 cos 0), i.e., £ times the projectionof area normal to £.

Also, A(|) = £. AS = £ AS cos 0 = (£ cos 0) AS, i.e., component of £ along normal to the area element

Flo
times the magnitude of area element. When E is normal to area element, 0 = 0®, electric flux is maximum.

eeee
When E is along the area element, 0 = 90°, electric flux is zero. When 0 > 90°, cos 0 is negative. Therefore,

Fr
electric flux is negative.

To calculate the total electric flux through any given surface, we have to divide the given surface into

for
ur
small area elements ; calculate the flux at each element and add them up with proper sign. Therefore, total

electric flux through a surface Sis ~'Z E.AS


ks
Yo
However, when we take the limit AS 0, the summation can be written as integral and we obtain the
oo

exact value of electric flux, i.e..


B

(])£. = () E.ds = 0 Eds cos 0


re
ou

The circle on the integral sign indicates that the surface of integration is a closed surface.
ad

Note that electric flux over a closed surface S can be positive ; negative or zero, according as
Y

0 < 90°; e > 90° and 0 = 90".


nd
Re

Electric flux is a scalar quantity.


Fi

Units of (])£■ = unit of £ x unit of S = NC' x = Nm^ CT' .


Dimensional formula of^^ = [L^] [AT]"' = [M' A"']
1.39. GAUSS'S THEOREM OR GAUSS'S LAW IN ELECTROSTATICS

According to this theorem.

The surface integral of electrostatic field E produced by any sources over any closed surface
S enclosing a volume V in vacuum i.e., total electric flux over the closed surface S in vacuum,
is i/e Q times the total charge (Q) contained inside S, ue.,

Q
= y E .ds = ...(54)
S ^0
ELECTROSTATIC CHARGES AND FIELDS 1/35

The charges inside S may be point charges or even continuous charge


distributions. There is no contribution to total electric flux from the
charges outside S. Further, the location of Q inside S does not affect the
value of surface integral.
The surface chosen to calculate the surface integral is called
Gaussian surface*. While selecting such a surface, we shall avoid charges
on S itself.

Proof of Gauss’s theorem (for spherically symmetric surfaces


only)
Suppose an isolated positive point charge q is situated at the centre
O of a sphere of radius r. Fig. 1.35.
According to Coulomb’s law, electric field intensity at any point P on the surface of the sphere is

ww
A

E = —^
r
...(55)
4tC6
0
r2

Flo
where r is unit vector directed from O to P.

e
Consider a small area element ds of the sphere around P. Let it be represented by the vector ds = nds

rree
A

r FF
where n is unit vector along outdrawn normal to the area element.
A
uurr
^4 .(4.)
Electric flux over the area element d^^= E.ds =
4neo for
kss
E .ds =
q ds r.n
A A
ooook

4tC€
Yo
0

As normal to a sphere at every point is along the radius vector at that point, therefore, r. n = 1
eB

E .ds =
q ds
4tC€^0
rr
ou
ad

Integrating over the closed surface area of the sphere, we get total normal electric flux over the entire
Y

q
spherical surface, (|)£= ^ E = >j ds = 4tC€
dY

X total area of spherical surface


5 A%€^r^ 5 =0 '●
Re
innd

q
(47t r^) - —
Fi

4tc€.^0 ^0
q
Hence, - —
, which proves Gauss’s theorem.
s
^0

If there are point charges, q\, q'>, ^3 q^ lying inside the surface, each will contribute to the electric
flux, independent of the others {superposition principle),
q 1 I
+ + ...—
= —(?,+92+93+ -+9„) = — S %
^0 ^0 ^0 ^0 ^0 ^0 /=!

*Gaussian surface around a point charge or a continuous charge distribution is an imaginary closed surface
at every point of which electric intensity is same. As Gaussian surface is an imaginary geometric surface, it may
lie in empty space or it may be embedded in a solid body.
1/36 ‘P'tadee^’4- Fundamental Physics (XlHEfTSTW

Q
^0
(=«

where Q = X algebraic sum of all the charges inside the closed surface.
j=i

Hence, total electric flux over a closed surface in vacuum is — time the total charge within the
surface regardless of how the charges may he distributed. ^0
If the medium surrounding the charge has a dielectric constant K, then

Q _ Q Q Q
h = where A:=e^=e/eo <l>£^ = —

ooww
Ke e
0

If there is no net charge within the closed surface i.e., when Q = 0, 4>£ = 0
It means that the total electric flux through a closed surface is zero if no charge is enclosed by the
surface or when algebraic sum of all the charges enclosed by the surface is zero.

e
It can be shown that charges situated outside the closed surface make no contribution to total

ere
electric flux over the surface.

rFl
Fre
Retain in Memory

rrF
1. Gauss’s theorem holds good for any closed surface, regardless of its shape or size.

sffoo
ouur
2. The surface that we choose for the application of Gauss’s law is called the Gaussian surface.
However, we usually choose a spherical Gaussian surface, because this choice has three simplifying
kosk
features : (0 The dot product E-ds ~ E ds cos 0® = £ (ds\ because at all points on spherical
YYo
oo

Gaussian surface, 9 = 0°. («) Magnitude of E is constant at all points on the spherical Gaussian
BB

surface. Therefore, E can be brought out of the integral sign. (Hi) The remaining integral is merely
the surface area of the sphere (=4n r^), which is obtained without actually doing the integration.
rre

3. Take care to see that the Gaussian surface chosen does not pass through any discrete charge. This
ouu
Y

is because electric field is not well defined at the location of the charge.
ad

4. In the situation when the surface is so chosen that there are some charges inside and some outside
dY

—>

the electric field E (whose flux is calculated) is due to all the charges, both inside and outside
innd
Re

the closed surface. However, for electric flux, the term (q) represents only the total charge inside
the closed surface.
FFi

5. Gauss’s theorem is used most commonly for symmetric charge configurations.


1
6. Gauss’s theorem is based on inverse square dependence of E on distance i.e., Eoc~ .
^ J
7. From Gauss’s theorem, we can calculate the number ofelectric lines offeree that radiate outwards
from one coulomb of positive charge in vacuum. As
1
, therefore, when q - 1 coulomb, 4>£ - 8-85 X 10-^2 = M3xi0i^
^0

The total electric flux emanating from a closed surface enclosing an


alpha particle (e = electronic charge) is (Kerala CEE 2011)
Sol. Charge of a particle, q~2e
ELECTROSTATIC CHARGES AND FIELDS 1/37

According to Gauss’s theorem,


le
electric flux emanating from closed surface surrounding the alpha particle is 4> = — ~
^0 ^0
Sample Problem Q sj and $2 are two hollow concentric spheres enclosing charges Q and 3 Q
respectively as shown in Fig. 1.36. What is the ratio of electric flux through Sj and 52? What would be
electric flux through 5]? If air Inside 5j is replaced by a medium of dielectric constant 3?
Q
Sol. Electric flux through 5 1’ ^1 =
^0

Electric flux through 52, <1>2 =

ww
^0 ^0

When air inside 5] is replaced by a medium of e ^ - 3,

Floo
Q_ Q Q
then electric flux through 5^ = (f>j = 3€

ee
0

reer
1.40. SIMPLE VERIFICATION OF GAUSS'S THEOREM

rFF
To verify Gauss’s theorem, let us calculate electric flux through a closed cylindrical surface containing
uur r
ffoor
no charge. Let the cylinder be held in an external uniform electric field of intensity E along the axis of the
cylinder, as shown in Fig. 1.37.
sks
YYoo
ooko

FIGURE 1.37
eBB

n A A ^
r

/ -V
ouur

4
ad

1 2
Y

U
dY
Re
nnd

Suppose (J)| and ^2 represent the electric flux through the surfaces 1 and 2 of area and S2 of the
FFii

cylinder and 4)3 is the electric flux through the curved surface 3 of the cylinder.
Now, outward normal to surface 1 is opposite to E and outward normal to surface 2 is along E ■
(j)j = - £■ jj and <}>2 = £ ^2
As r j = ^2 = ●y = area of circular cross section of the cylinder, therefore,
—>

Also, normal to surface 3 at every point is perpendicular to £ .


4>3 = £ S3 cos 90° - 0
Hence, <J)l + 4»2 + 4>3 = 0, i.e., total electric flux over the closed cylindrical surface containing no
charge is zero. This verifies Gauss’s theorem.

A uniform electric field exists in space. Find the electric flux of the field
through curved surface area of the cylinder with its axis parallel to the field.
1/38 4 Fundamental Physics (XII)

Sol. As normal to curved surface area of the cylinder is at 90° to electric field.

()) = E.ds = £d5cos90° = zero


5 S

1.41. DEDUCTION OF COULOMB'S LAW FROM GAUSS'S THEOREM

Consider an isolated positive point charge q at O. Imagine a sphere of radius r with centre O, Fig. 1.38.
—'*

The magnitude of electric field intensity E at every point on the surface of the sphere is the same and it is
directed radially outwards. Further, the direction of vector ^ representing a small area element ds on the
—>

surface of the sphere is along E only, i.e., 0 = 0°


According to Gauss’s theorem,

ww
O E.ds =-^ or ct Edscos0°= —
^0 s ^0

FF loo
s

E^ ds = -^

ree
or or E (4tc r^) = —
s ^0 ^0

eF
E = -^

Frre
r ur
4ti£o
fforo
This is the electric field intensity at any point P distant r from an isolated point charge q at the centre of
ks
the sphere. If another point charge q^ were placed at P, then force on q^ would be
YYouo
koso
BBoo

F =
r ee

...(56)
47t€o
ad
oouur

This is Coulomb’s law.

IMPORTANT APPLICATIONS OF GAUSS’S THEOREM


Y

1.42. FIELD DUE TO AN INFINITELY LONG STRAIGHT UNIFORMLY CHARGED WIRE


dnYd
Re

Consider an infinitely long thin wire with uniform linear charge density %. To calculate field due to this
FFini

wire at any point P, we consider a pair of line elements Fj and P2 of the wire at equal distances on either side
of an arbitrary origin O.

Electric fields E^ and E2 at point P due to the two


line elements are shown in Fig. 1.39. Their components
normal to the radius vector OP = r cancel out, being equal
and opposite. However, the components along OP add.
Therefore, re.sultant electric field is radial. This is true for
any such pair of line elements. Therefore, total field at any
point P is radial. As the wire is infinite, electric field does
depend upon distance of point from the wire, but not on
the position of P. Thus, electric field at every point in the
plane cutting the wire normally is radial; and its magnitude
depends only on the radial distance r.
1/39
ELECTROSTATIC CHARGES AND FIELDS

Let us consider a right circular closed cylinder of FIGURE 1.40

radius r and length / with the infinitely long line of charge t


its axis. Fig. 1.40. The magnitude of electric intensity
i.
as A A
n

£ at every point on the curved surface of the cylinder is


i
‘ ‘ n

P
the same, because all such points are at the same distance rr
from the line charge.
n \
«r
ds
I
A . n

Also, £ and unit vector n along outward normal to I + + + +


+ i

curved surface are in the same direction, so that 6 = 0°. w


.-. Electric flux over the curved surface of the cylinder, n
,n

(fc ~E.n d5 = (£ £ (1) cos0°</^ H- / ●M

5 5 S

w
= E n ds - E {2% rl)
S

Flo
where (2 tc rl) is area of the curved surface of the cylinder. A

is 90°.
On the ends of the cylinder, angle between electric field intensity £ and outward normal n

eeee
Therefore, these ends make no contribution to electric flux of the cylinder.

Fr
.-. total electric flux over the whole cylinder, (|)£ = £ (2 tc rl)
Charge enclosed in the cylinder = linear charge density x length

for
ur
q = Xl
q XI
According to Gauss’s theorem, <j)^ = £(2Tcr/) = —
ks
^0 ^0
Yo
oo

X
£ = ...(57)
B
re

1
Clearly, £ oc —
r
ou
ad

If > 0, the direction of electric field at every point is radially outwards.


Y

If A. < 0, the direction of electric field at every point is radially inwards.


Retain In Memory
nd
Re

1. In the above derivation, electric field £ is due to charge on the entire wire of infinite length.
Fi

However, the charge enclosed by the cylindrical Gaussian surface is only Ax/.
2. The assumption that the wire is infinitely long is crucial. £ is taken normal to the curved part of
the cylindrical Gaussian surface only on account of this assumption.
3. If the wire is not infinitely long, we have to take into consideration the end effects.

f^irnp>i<^^PVQblerni:●; An infinite line charge produces a field of 9 x 10'* N C ^ at a distance of


0‘02 m. Calculate the linear charge density.
Sol. Here, £ = 9 x lO'* N C"*, r = 0-02 m, A = ?
2A £x47tGrt0 r 1 0-02
A
A = = 9xl0^x = 10“’ Cm-*
As £ =
27ce«0 r 4tc€„0 r 2 9x10^^ 2
1/40
'a. Fundamental Physics (XII) VOL.i
1.43. ELECTRIC FIELD INTENSITY DUE TO A UNIFORMLY CHARGED SPHERICAL SHELL
(a) Field outside the shell

Consider a thin spherical shell of radius R with centre 0. Let a charge +q be distributed uniformly
over the surface of the shell. To calculate electric field intensity at any point P, where OP = r, imagine a
sphere S, with centre O and radius r. Fig. 1.41(a). The surface of this sphere is a Gaussian surface at
every point of which electric intensity E is the same, directed radially outwards (as is unit vector n, so that
9 ~ (F).
According to Gauss’s theorem,

j>E.d:=jE.nds^^^0 or Eiids = -S-


5 S s ^0

ww
^0

FF loo
E= L ...(58)
47ce«

ree
0

This is exactly the field produced by a charge q placed at the

eF
centre O. Hence we conclude thatfor points outside the spherical

Frre
shell, the field due to uniformly charged shell is as if the entire
r ur
charge of the shell is concentrated at the centre of the
At a point on the surface of the shell
fforo
shell
ks
YYouo
r = R, from (58), £~ = ^ /4 7t € Q - Maximum
kos o

If CT is surface density of charge on the shell, then


BBoo

q = AnR^ .<3
r ee

, E
„ ^kR^o a
ad

- constant, Le., £ = — ...(59)


oouur

4tc€q R} ^0
Y

{b) Field inside the shell


dnYd

In Fig. lA\{b), the point P where we have to find electric


Re

FIGURE 1.42
intensity is inside the shell. The Gaussian surface is the surface of a
FFini

sphere $2 passing through P and with centre at O. The radius of


I: max
sphere 5*2 is r < R.
The electric flux through the Gaussian surface, as calculated
above is £ x 4 7C As charge inside a spherical shell is zero, the
Gaussian surface encloses no charge. The Gauss’s theorem gives
£x4Tcr2 = — = 0 £ = 0 for r < £. O
r
^0

Hence, the field due to a uniformly charged spherical shell Distance from centre (r)
is zero at all points inside the shell
The variation of electric field intensity £ with distance from the centre of a uniformly charged spherical
shell is shown in Fig. 1.42.
1/41
ELECTROSTATIC CHARGES AND FIELDS

Retain in Memory
1. When a uniform spherical shell carries charge Q, electric field at any point inside the shell is zero.
Fig. 1.43.
2. The shell behaves as if the entire charge is concentrated at its centre.
3. Electric intensity at any point on the surface of the shell is maximum.
4. Outside the shell, E varies inversely as the square of distance from the centre.
FIGURE 1.43

^surface ~

^inside " 0

oww
0

E decreasing E decreasing

5. Obviously, a charged particle inside the spherical shell will not experience any force. However,

e
outside the shell, charged particle will be attracted or repelled.

FFrlo
re
1.44. ELECTRIC FIELD INTENSITY DUE TO A NON-CONDUCTING CHARGED SOLID SPHERE

ree
F
Suppose a non conducting solid sphere of radius R and centre O has uniform volume density of charge p.
any point P inside the solid sphere, where OP = r. With

rF
We have to calculate electric field intensity E at
O as centre and r as radius, imagine a sphere S, which acts as a Gaussian surface, Fig. 1.44. At every

fsoor
ouur
point on the surface of 5, magnitude of E is same, directed radially outwards.
skf
If q' is the charge enclosed by the sphere 5, then according to Gauss’s law
ooko
f— »A r n'
Yo
O E .ds = .('> E.n ds ~ EO ds = —
Y

Bo

s S S
reB

where e is electrical permittivity of the material of the insulating sphere.

£(47ir2) = -^ or E= ...(60)
uur
ooY

4riter^
ad

Now, charge inside S.Le. q' = volume of 5 x volume density of charge


dY

/ ^ 3
^' = -7cr^xp
innd
Re

4 jir^p rp ...(61)
FFi

From (60), , i.e..


3e
3 47C€r^ 3e
Clearly, £« r

i.e. electric intensity at any point inside a non-conducting charged solid sphere varies directly as the
distance of the point from the centre of the sphere. FIGURE 1.45

At the centre of the sphere, r = 0, .’. E-0


At the surface of the sphere, r = R

=> maximum.
36

We have already proved that outside the sphere, E l/r^.


All these results are plotted in Fig. 1.45, which represents
the variation of electric field intensity E with distance (r) from
the centre of a nonconducting uniformly charged solid sphere.
1/42
T^^'uidee^ ^ Fundamental Physics (XII) VOL.I

1.45. ELECTRIC FIELD INTENSITY DUE TO A THIN INFINITE PLANE SHEET OF CHARGE
Consider a thin, inifnite plane sheet of charge. Let a be the surface density of charge on the sheet {i.e,
charge/area). We have to calculate electric field intensity £ at any point P, (at a perpendicular distance) r
from the sheet.

From symmetry, we find that £ on either side of the sheet must be perpendicular to the plane of the
sheet, having same magnitude at all points equidistant from the sheet.
Let us imagine a cylinder of cross-sectional
FIGURE 1.46
area ds around P and length 2 r, piercing through
the sheet. At the two cylindrical edges P and Q, A

+
A

A
+
E and outward normal n are parallel to each other.
, .

'+ +
+ -I-
Fig. 1.46. E r -I- T\ E

w
electric flux over these edges I+
,+ / +
A
n
+
+ -T7
~2E .n ds = 2E ds +. ▼ n
r —

Flo
+i
+
On the curved surface of the cylinder, E and

eeee
A

outward normal n are ± to each other. Therefore, no contribution to electric flux is made by the curved

Fr
surface of the cylinder.
.-. Total electric flux over the entire surface of the cylinder, <^^ = 2 Eds

for
ur
As area of the sheet intercepted by the cylinder is ds, therefore.
Total charge enclosed by the cylinder = ads
ks
ads
According to Gauss’s law in electrostatics, - 2Eds
Yo
= ~

^0 ^0
oo

a
E =
B

2e ...(62)
0
re

We observe that E is independent of r, the distance of the point from the plane charged sheet.
If the sheet carries positive charge, ct > 0. The electric field is uniform, norma! and outwards from the
ou
ad

sheet. If the sheet carries negative charge, a < 0. The field is uniform and along the inward normal to the
Y

plane sheet.
Note that for a planar sheet which is large, but not infinite, eqn. (62) is only approximately true in the
nd
Re

middle regions of sheet, away from the ends ; and at distances not far away from the sheet.
If the infinite plane sheet has uniform thickness, the surface
Fi

density of charge a is uniform and same on both the surfaces of the sheet.
The electric field intensity at any point P due to each surface is
a
= £2 = — . Both E^ and E2 are perpendicular to the plane of the sheet
2e
0

and directed away. Fig. 1.47. Therefore, according to superposition


principle, net electric field intensity at P due to an infinite charged plane
sheet of uniform thickness is

£ = £, + E2
E=-^ a
E=^
2e 0 2e
0 ^0
ELECTROSTATIC CHARGES AND FIELDS 1/43

Note that the sheet is supposed to be of infinite size so that the 'edge effect' due to distortion of electric
field at the edges (of a finite sheet) can be ignored,
^an^io ■problem' A charge of 17*7 X 1(H C is distributed over a large sheet of area 400
Calculate the electric field intensity at a distance of 10 cm from it
Sol. Here, g = 17-7 X 10^ C, A = 400 m2, £ = 7, r= 10 cm = 10"^ m
In case of a large plane sheet, distance of the point (= r) does not matter.
a q/A 17-7xl0'^
E = = 2-S X 10^ N/C
2g 2e
2e^A 2x(8-85x10"^2)x400

oww
0 0

1.46. ELECTRIC FIELD INTENSITY DUE TO TWO THIN


INFINITE PLANE PARALLEL SHECTS OF CHARGE
FIGURE 1.48
Let A and B be two thin infinite plane charged sheets held

ee
A B
parallel to each other. Fig. 1.48. Suppose

FFrlo
I + + II HI

r
CT| = Uniform surface density of charge on A ●*■+ +
c ++ +

rF
+ 4. + n
02 == Uniform surface density of charge on B I ++++ ' ^2 ^1 I++ 4-‘
+

ee
Ei + +
El
I +
If E^, E2 are electric field intensities at a point due to E2
+
I
I ■*■-*■ +
E2
I -^+ + I I
ouru
rF
charged sheets A and B respectively, then I ■*■1 I++ + ,

ffosor
0
1
^1 = and
2e
os k
0

The arrangement shows three regions I, II and III. We apply superposition principle to calculate the net
ook
Yo
field intensity in the three regions. As a matter of convention, a field pointing from left to right is taken as
Y

positive and the one pointing from right to left is taken as negative.
Bo
reeB

We assume that 0] > 02 > 0


-0 0 -1
1 2 _
oouY

In region I, E/ = -Ej- E^ (0,+02> ...(63)


ur

2e 2g 2e
0 0 0
ad

Similarly, in region U, (<^1 ...(64)


dY

2e 2e 2g
0 0 0
nidn

0
I <^2 1
Re

and, in region in, + ^2 “ 2e 0 2e 2e


(0, + 02 ) ...(65)
0 0
FFi

Special cases; Suppose 0, = 0, and 02 = - 0


i.e., two thin infinite plane sheets with equal and opposite uniform surface densities of charge are held
parallel to each other.
From (63), Ef=0
From (65), Euj=0 FIGURE 1.49

20 _ 0 — o\

From (64), ^// -


o
= constant. +
2e
0 ^0 + +

Thus field intensity inbetween such sheets having equal


a
+ + Eii=t
and opposite uniform surface densities of charge becomes ^0
+ + >●
E, = 0 E„, = 0
constant i.e., a uniform electric field is produced inbetween two + +

such sheets, Fig. 1.49. Also, E does not depend upon the + +

distance between the thin sheets. +

This is how uniform electric fields are produced in practice.


1/44
‘P'tadecft- Fundamental Physics (XII)

I TYPE I. QUANTIZATION OF CHARGE


I Solution. Here, « = 10^ electrons/sec
Charge given/sec,
Formula used. q~±ne q = ne=\Q^.y 1-6 x 10"*^ C
Units used, q and e are in coulomb, n is a number = I-6x lO-'^C
Standard value, e = 1-6 x 10"^^ coulomb
Total charge. Q = 1 C
1
Example fl Which is bigger, a coulomb or Time required = — sec

charge on an electron ? How many electronic q 1-6x10-10


charges form one coulomb of charge ? 6-25x10^
= 6-25 X =

w
(Pb. Board 2011) year
3600x24x365
Solution. A coulomb of charge is bigger than = 198'18 year
the charge on an electron.

Flo
Magnitude of charge on one electron, ^Example Q Two bodies A and B carry
e = 1-6 X 10”'^ coulomb charges - 3-00 jiC and - 0-44 jiC. How many

ee
Number of electronic charges in one coulomb, electrons should be transferred from A to ^ so that

Fr
they acquire equal charges?
q 1
n = -~ = 0*625 X 10^1^ Solution. Here, = - 3 00 ^iC
e 1.6xl0”l^ and
^2 = - 0-44 |iC

for
ur
Note that one coulomb is too big a unit of charge. Let n electrons be transferred from A to B, when
A and B would carry same charge.
Example @ How much positive and
ks
Charge on A = Charge on B
negative charge is there in a cup of water ?
Yo
- 3-00 + «e = - 0-44 - ne
oo

(NCERT Solved Example)


2ne- 3-00 - 0-44 = 2-56 (|iC)
Solution. Suppose the cup contains 250 cc of
B

2-56
water (H2O).
re

n ~

Mass of 250 cm-^ of water = 250 g. 2e

Molecular weight of water = 2 + 16 = 18 Taking e = 1-6 X 10-1^ C= 1-6x10-13^0


ou
ad

Number of molecules in 18 g of water 2-56


Y

= 6-023 X 10^3
n =
= 0-8x 10l3
2x1-6x10-13
Number of molecules in 250 g of water = 8x10*2
nd
Re

6-023 X 10^3 X 250


I I
Fi

TYPE II. COULOMB’S LAW


18
As each molecule of water contains 10 electrons,
therefore, total number of electrons. Formulae used, /v0 =
1
l^_l^
47ce
0
r2
10x6-023x1023x250
n - = 8-365 X 1025
18
p - * 1^1 11 ^2* 1 I 11 <?2 I
As ^ = ne, therefore, 4JI€ r2 4KGoe, r2
q = 8-365 X lO^^ x 1-6 x IQ-i^C Units used. q^,q2QiQ \n coulomb, F in newton and
= 1*338 x 10^ C r in metre.

1
Example |] If a body gives out 10^ Standard Values. = 9xl0^Nm2c-2
47ce
electrons every second, how much time is required 0

to get a total charge of 1 C from it ?


€o = 8-85x 10-12 q2^-1
(NCERT Solved Example)
ELECTROSTATIC CHARGES AND FIELDS 1/45

Example § A free pith ball F of 10 g Solution. As is clear from Fig. 1.51, forces on
carries a positive charge of 5 x 10“* C. What must the particle at O due to (- 2 - 2 ^); (- 3 - 3
be the nature and magnitude of charge that should and (+4q, + 4q) are equal and opposite. They cancel
be given to another pith ball Q fixed 7 cm below out in pairs. However, forces due to + 1 q and -1 q
the former ball, so that upper ball is stationary ? add up. Therefore, net force on the particle at O is
(Hr. Board 2001) F =
1
X
(7 q){q) + Jq(q)
Solution. Here, m = 10 g = 10 x 10“^ kg, 4Tce
0 (1/2)2
ijj = 5 X 10“^ C, r= 7 cm = 7 X 10“~ m
Second ball Q must carry positive charge so that
9x10^x14(2x10-^)2
1/4
force of repulsion balances the weight of ball P.
When ball remains stationary. Fig. 1.50, = 36 X 14 X 4 X 10"^ N

ww
F = mg = 2016 N
1
= mg Example Q Coulomb’s law for electro
47tG

Flo
0
static force between two point charges and

e
FIGURE 1.50 Newton’s law for gravitational force between two

ere
F stationary point masses, both have inverse square
dependence on the distance between the charges/

FFr
0, 1
masses (a) compare the strength of these forces
uurr by determining the ratio of their magnitudes

orr
(0 for an electron and a proton (m) for two protons
7 cm
mg sfo
(6) estimate the accelerations for electron and
proton due to electrical force of their mutual
kks
attraction when they are 1 A apart.
©
Yoo
^2
oooo

(NCERT Solved Example)


Solution, (fl) (0 For an electron and proton
eBB

9x10^ x5xl0 ^xq^


= 10 X 10-3 9.g 1 exe

(7x10-2)2 I F„
47te r2 ’
urr

10x1Q-3x9-8x(7x10~2)2
ad

Gm^.ntp
Yo

^2 =
9x10^x5x10"^
dY

r2
= 1-067 X 10-^ C
1 ^2
innd
Re

Example A particle carrying charge


I F I 4ti€o Gm m
+ q is held at the centre of a square of each side 8 0 P
Fi

one metre. It is surrounded by eight charges


arranged on the square as shown in Fig. 1.51. If g 9x10^ (1-6 xl0"‘^)2
- 2 pC, what is the net force on the particle? ~ 6-67x10-“ x9x10-3’ x1-66x10"27
FIGURE 1.51 = 2-3 X 1039
-2q *7q -3q
(«■) Similarly, for two protons,
IFJ 1 c2
I F
g
I
47CEo G.m^m^
*4q ■■ O ■ +4q
9x10^(1-6x10"^^)2
6-67x10
,-n
x(1-66x10"2’^)2
-Zq -7q ■2q = 1-3 X 10^
1/46
“Pn^zdecft- 'a. Fundamental Physics (XII)
{b) Force of mutual attraction between an Spheres A and B are touched by uncharged spheres
electron and a proton. C and D respectively, as shown in Fig. 1,52(^). C
and D are then removed and B is brought closer
F =
1
_ 9x10^(1-6x10"^^)2 to A to a distance of 5*0 cm between their centres,
4ne
0 (10-10)2 as shown in Fig. 1.52(c). What is the expected
= 2-3 X 10"® N repulsion of A on the basis of Coulomb’s Law ?
f 2-3 X10-® Spheres A and C and spheres B and D have
Acceleration of electron = — Identical sizes. Ignore the sizes of A and B in
m 9x10-31 comparison to separation between their centres.
= 2-5 X Vp- m/s^ (NCERT Solved Example)

oww
F 2-3 X10"^ Solution. Let the original repulsive force
Acceleration of proton = — = between A and B be
m
P
1-66x10-27
= 1-3 X 10'® m/s2 F=i^
r2
ExampioQ A charged metallic sphere A

ee
As A and C have same size, charges are shared

FFrlo
is suspended by a nylon thread. Another charged equally. Again, as B and D have same size, their

r
metallic sphere B carried by an insulating handle
charges are also shared equally.

rF
is brought close to A such that the distance between

ee
their centres Is 10 cm as shown in Fig. 1.52(a). The As charges on A and B are halved,and distance
resulting repulsion of A is noted (for example, by between them is also halved from 10 cm to 5 cm,

rF
ouru therefore,
shining a beam of light and measuring the
deflection of its shadow on a calibrated screen).
k,(qj2)(qj2) kg,q^ ^
FIGURE 1.52

fosor
s kf
ooko
'Example Two electrons and a positive
YYo
charge q are held along a straight line. At what
position and for what value of q will the system be
Bo
reeB

in equilibrium ? Check whether it is stable,


o unstable or neutral equilibrium.
Solution. Let two electrons of charges - e each
ooY
uur

14- 10cm ■M
be held at A and B. The third charge + q must be
ad

placed at the centre O of AB. The forces on + q, due


dY

to two electrons being equal and opposite, cancel


eachother and it is in equilibrium.
nidn

For the charge (- e) at A to be in equilibrium,


Re

o Fig. 1.53, force on charge at A due to - e charge at


FFi

B + force on charge at A due to + ^ charge at O =


Zero

FIGURE 1.53

-e -e
+q
A '6 B

1 (-e) i-e) 1 q _i-e) = 0


4 Tie 0 a;2 (x/2)2
1 c2 1 q (e)x4
or
O 47C€ 4 Tie 2
X
0 0

H—5cm M or e =4 q or q-e!4
ELECTROSTATIC CHARGES AND FIELDS 1/47

If charge at O is moved slightly towards A. it


would not return to O on its own and shall continue _ ^0 9io
to move towards A. Hence equilibrium is unstable. 4 Tie 0
Tj Tj r_
Example [E Two point chaises 4 pC and
1 pC are separated by a distance of 2 m in air. 1-0x10-8 8x10-8 27x10-8
. + — ■ ‘ ■
Find the point on the line joining the charges at (0-10)2 (q.20)2 (0-30)2
which net electric field of the system is zero. = 9xl0^xl -8
1000x10
[CBSE 2017(C'l +....

Solution. Here, = 4 pC = 4 x 1(T^ C (1-00)2


^2 = 1 pC = 10"^ C = 9 X 10^ X 10-6 [1 + 2 + 3 + + 10]
d-2m = 9x 10^ X 55 = 4-95 X 10® N

ww
FIGURE 1.54 Example Find the magnitude of the
A P B resultant force on a charge of 1 pC held at P due
to two charges of + 2 x lO"® C and - lO”® C at A

FF loo
-4-

4 pC E2 El 1 pC and B respectively.

ee
♦H- >1
r (2-r) Given AP = 10 cm and BP = S cm.

er
ZAPB = 90% Fig. 1.55.

Fr reF
Let electric field of the system be zero at P,
where AP = r. Fig. 1.54.
rur
At P, = E2
^1 xl
fforo
ks
47ce«0 r2 47ceQ(2-r)2
YYouo
okso

4 1 2 1
BBoo

or

r2 (2-r)2 r 2-r
r ee

r = 4 - 2r Solution. Here, F = ?,
3r = 4 Charge at P, = 1 pC = 10"6 C
ad
oouur

r = 4/3m Charge at A, ijj = + 2 x lO”® C


Y

Charge at B, ^2 “ “ ^
AP = 10 cm = 0-1 m, BP = 5 cm = 0-05 m
TYPE III. SUPERPOSITION PRINCIPLE
dnYd
Re

ZAPB = 90°
->

Force at P due to charge at A,


Formula used. Pg = ^01 ^02 + ^03
FFini

1
Units used. Force is in newton, when charges are F. =
1 , along AP produced
in coulomb and distance is in metre. 47TGQ Ap2
9x10^x2x10-8x10-6
= 18 X 10-2 N
Exjample m Ten positively charged par (0-1)2
ticles are kept fixed on x-axis at points jr = 10 cm,
20 cm, 30 cm,...., 100 cm. The first particle has Force at P due to ^2 charge at B,
charge 1-0 x 10^ C, second 8 x 10"® C, third 27 x 1 ^2^
, along PB
10“® C, and so on. Tenth particle will have charge 47t6n0 Bp2
1000 X 10^ C. Find the magnitude of electric force
acting on a 1C charge placed at the origin. 9x10^x10-8x10-6
= 36 X 10-2 N
Solution. By superposition principle, force on (0-05)2
charge I C placed at origin
^0 = ^01 ^02 + —● ^ ^010 As angle between and F2 is 90°,
1/48
“Pfuidec^ Fundamental Physics (XII) VOL.I

Resultant force, p - ^p^ + p^ Total force on Q = 3Qq ^Qq


= 0
47C€n/^ 47ten/2
= V08xIO"3)2+ (36x10-3)2 0 0

= 18 X 10"3 X 2-236 ‘Example 01 Consider the charges q, g and


= 4-0 X 10-2 - q placed at the vertices of an equilateral triangle
of each side 1. What is the force on each charge ?
Exam
pte m Consider three charges (NCERT Solved Example)
93 ®®ch equal to q at the vertices of an Solution. As is clear from Fig. 1.57, force on
equilateral triangle of side 1. What is the force on =^at A
a charge Q (with the same sign as q) placed at the
centroid of the triangle ?
(NCERT Solved Example)

w
Solution. As shown in Fig. 1.56, draw
ADIBC.

Flo
/^/3
AD = AB cos 30° —
2

ee
Fr
for
ur
-» A
s
ok
Yo
qq
Bo

where p =
4tc€..0 /2
re

and
/j = unit vector along BC
ou
ad

Distance AO of the centroid O from A Force on (^2 = q) at


Y

/ -»

3 3 2 ^2 ^2I'*’ ■^23 “ ^ '2 »


nd
Re

.% Force on <2 at O due to charge ^ at A where


T2 = unit vector along AC
Fi

1
Fl = Qq _ ^Qq
Force on ^3 = - ^ at C,
(//V3)2 47te./2’0
^longAO
Similarly, force on Q due to charge ^2 = 9 at B ^3 “ ^31"^ ^32

^ 4tcg„/2
0
along bo = [^Ff +F^+2 Fj F2 cos 60° ] n
and force on Q due to charge ^3 = ^ at C
= ^Fn
F - 36? A

2 47te„;2’ along CO where n = unit vector along the direction


Angle between forces F2 and ^3 = 120° bisecting ZBCA.
We can show that
By parallelogram law, resultant of F2 and

4tC€«/2
along OA Fj + Fj+Fj = 0
0
1/49
ELECTROSTATIC CHARGES AND FIELDS

Solution. Here, m = 1= 10 ^ kg,

I I
TYPE IV. ELECTRIC FIELD AND RELATION
BETWEEN ELECTRIC INTENSITY AND FORCE
9 = + 6 fiC = + 6 X 10-^ C
£ = 400 NC"*, downwards.
Formulae used. As is clear from Fig. 1.58,
i g
1. I£l = 2. F=q^E FIGURE 1.58

3. For calculating field due to any number of charges


and continuous charge distributions, principle of -T

super-position is applied.
Units used. The source charge q is in coulomb, so + 6|iC

oww
is test charge q^. The distance r is in metre. Electric
intensity E is in newton/coulomb and force F is in
mg
newton.

Standard value.
1
= 9xl0^Nm2cr- r=£g + mg = ^£ + mg
4tC€ 0 = 6 X 10-^ X 400 + 10"^ X 9-8

e
= 122 X 10^N= 1-22 X lO-^N

ree
rFl However, when q = -6 jiC, force due to electric

Fre
Example m An oil drop of 12 excess elec field is upwards,

rrF
trons is held stationary under a constant electric
field of 2*55 x 10** NC"^ in Millikan’s oil drop r + Fg = mg
experiment. The density of the oil is T26 g cm”3. or r= mg - Fg = 9-8 X 10-3 _ 24 X iq-4
ouur
Estimate the radius of the drop,
(g = 9*81 ins“2 ; e = 1-60 x 1Q~^^ C)
sffoo = 74 X 10-^ N
Example m An electron falls through a
kks
Solution. Here, n = 12, distance of 1*5 cm in a uniform electric field of
oooo
Yo
£ = 2-55 X 10^ NC"’, value 2 x 10*^ N/C. When the direction of electric
field is reversed, a proton falls through the same
eBB

p = 1-26 gm cm“3 = 1-26 x 10^ kg m"3 ; distance. Compare the time of fall in each case.
r=? Contrast the situation with that of ‘free fall under
urr

As the droplet is stationary, gravity’. [CBSE 2018 (C)]


You

weight of droplet = force due to electric field (NCERT Solved Example)


ad

Solution. («) For electron, Refer to Fig. 1.59


dY

3
= Ene ;
FIGURE 1.59
innd
Re

3 Ene
r3 = e'
FFi

47tpg

3x2-55xl0^xl2xl-6xlQ-'^
4 x3-14xl-26x103x9-81
= 0-94 X 10“
18 - 1-5 cm = 1-5 X 10“^ m, £] = 2 X 10^ N/C
r = (0-94 X 10“^^)'^ = 9-81 X lO""^ m = (-) 1-6 X coulomb, m| =9 x lO^^i kg,
Example [0 A small sphere of mass 1 g Acceleration, = _5_^9oA
carries a charge of+6 |iC. The sphere is suspended mj mj
by a string in an electric field of 400 NC“* acting 16x10"*^ x2xl0^
downwards. Calculate tension in the string. What
9x10-3*
will be the tension if charge on the sphere were
-6|xC?
= 3-55 X 10^3 n;j/s2
1/50
‘Pn4xde^^'<i Fundamental Physics (XII)rowiMi
1
2xl0“^xl0^xl0
From yj = Mj /j + — ci j - 2000 m/s
10-5

yi=°+\<h‘! As Vj. remains constant, therefore, net velocity


after 10 s
2x1-5x10-2
or t
1
= 2^ = = 2-9 X 10"® sec. 2 =7(1000)2+(2000)2
3-55x1015
ih) Forproton, RefertoFig. 1.60, when electric = lOOoVSm/s
field is reversed.
Displacement along Ar-axis after 10 s
FIGURE 1.60 x = v^t= 1000 X 10 = 10000 m
Displacement along y-axis after 10 s

w
Proton 1 .
Lf2
y = U^Xt + - m

Flo
1 2x10-^x105
= 0 + -x (102)= 10000 m
2 10x10"^

eeee
Charge Qq = + 1-6 x 10"'^ C,
r«2= 1-67 X 10-22 kg. Net displacement = -yjx^ + y2

Fr
F>,
Acceleration, ^ = 7(10000)2+ (10000)2 =10(M)0V2 m

for
ur
Example m Two point charges + 6q and
1-6x10-1^x2x10'^ ~ S q are placed at the vertices B and C of an
a
= 1-92 X 10l2 ni/s2
ks
2 “
1-67x10-22 equilateral triangle ABC of side a. Obtain an
Yo
expression for magnitude and direction of
oo

Similarly, 2y2 ^ 2x1-5x10-2 resultantelectric field at the vertex A due to these


B

; 1-92x10'2 two charges. fCBSE 2014 (C)]


= 1-25 X 10-2 s Solution. As is clear from Fig. 1.61.
re

t
1 2-9x10-^
= 2-3 X 10-2
ou

and
ad

^2 1-25x10-2
Y

We observe that acceleration of electron


- 10’5 m/s2 and acceleration of proton « 10i2 m/s2.
nd

The value of ‘g’ in free fall is only 9-8 m/s2


Re

= lO’ m/s2, which is negligible. Therefore, effect of


Fi

acc. due to gravity can be ignored.


Example [Q A charged particle of charge
2 pC and mass 10 milligram, moving with a
velocity of 1000 m/s enters a uniform electric field
of strength lO^ N/C directed perpendicular to its
direction of motion. Find the velocity and
displacement of the particle after 10 s.
E
1 6^x1
- = 6E
(CBSE Sample Paper 2011) BA ~
4 Tie
Solution. Here, ^ = 2 pC = 2 x IQ-^ C,
a
0

w = 10 mg = IQ-5 kg 1 8?
f = 8£
= 1000 m/s, Ey = 1Q2 N/C 4tcg 0 a

QEyt
Vy = Uy+ayt =Q + where E
m 4jc€«0 fl2
ELECTROSTATIC CHARGES AND HELDS 1/51

Magnitude of resultant field is

^net " ^ ^BA ^ AC


cos 120“ = ^+{-2Ef+2E(2E)\-^
E^ = E^ =
= ^(6 £)2+(8£)2+2x6£x8Ej^-- 4jt€Q
If P is angle which E^ makes with AC, then
qM
£„ = eVm = 4tc€(j tanP =
£g^sin0 £ sin 120“
2£ + £cosl20“
Let the resultant field make an angle P with AC.
sin 120“ £^3/2 1
p = 30'

oww
tanp = — 1
£
AC+ £g^ cos 120° £ 2
2
6£V3/2 3^3 Example ai Two identical point charges,
1 5
q each, are kept 2 m apart in air. A third point

ee
8£+6£ —
^ 2 charge Q of unknown magnitude and sign is placed

FFrlo
on the line joining the charges such that the system

r
P = tan remains in equilibrium. Find the position and

rF
ee
5
nature of 0. (CBSE 2019)
Example ^ Two point charges + q and Solution. In Fig. 1.63, two identical point

rF
ouru
- 1 q are placed at the vertices B and C of an charges, q each are placed at A, B, where AB = 2 m.
equilateral triangle ABC of side a. Obtain tbe Let a third point charge Q be placed at P on the line
expression for magnitude and direction of
fosor
AB such that AP = x. Therefore, BP = (2- x).
resultant electric field at the vertex A due to these
s kf
[CBSE 2014 (C)l
ooko
FIGURE 1.63
two charges.
YYo
Solution. In Fig. 1.62, imagine a charge of + 1 Q 9
A B
C at A.
Bo

>1P
reeB

k- ■►I
2 m
ooY

The charge Q will be in equilibrium when it is


uur

repelled/attracted equally by the two charges at A and


ad

B. Therefore, it must be at the centre of AB. it may be


dY

positive or negative.
nidn

Again, charge ^ at £ will be in equilibrium, when


Re

kQq kqq = 0
FFi

or — + - = 0 or 0 = - <?/4
l2 22 1 4

I
TYPE V. electric DIPOLE, DIPOLE MOMENT,
DIPOLE HELD
1 qx\
^BA “ — = £, along BAE
47ce 0 a Formulae used.

-2q 1. Dipole moment \ p \ = qx2a ^ where


and £AC ~ = -2£, along AC
2a - distance between two point charges of dipole.
4tc6q
e = 120°
2. Dipole field intensity on axial line of dipole
2lplr
Resultant electric field at A
1^1 =
47C€o(r^-a^)’
£
R
1/52
U Fundamental Physics (XII) VOL.I

where r is distance of the point from the centre of 1


p _ qx2a
£ =
the dipole.
47C€n 4tc€.
3. Dipole field intensity on equatorial line of dipole 0

l?l 9x109x20x10-^x10-2
1^2 i =
4;ceQ(r2 +a2j3/2 (1/2)3
4. Dipole field intensity at any point due to a short E=1.44x10'*N/C

dipole is I £ I - 47t€r.0
■^3cos2 9 + 1, Example^ What IS the magnitude of
electric intensity due to a dipole of moment 2 x

oww
and tana =
I
— tan 0,
10"* C-m at a point distant 1 m from the centre of
2 dipole, when line joining the point to the centre of
where 0 is the angle which the line joining the point dipole makes an angle of 60° with dipole axis ?
to centre of dipole makes with the axis of dipole. Solution. Here, p = 2x 10“® C-m, r = 1 m,

e
9 =60°,£=?
And a is the angle at which E is inclined to the

re
line joining the point to the centre of dipole.
E= —tJScos^ 0 + 1

FFrllo
Units used, q in coulomb, 2a in metre, I p 1 in 47ceQ r3 '

reF
coulomb metre (C-m), r in metre, £j, £3 are in

e
newton/coulomb 2x10-^x9x109
V3(cos60°)2 -1-1
uoru 13

osFr
Example m Two charges each of 1 p. C £ = 238-1 A^/C

but opposite in sign are 1 cm apart. Calculate


electric field at a point distant 10 cm from the mid
point on axial line of the dipole. fkfor Example^ Two charges ± 10 p C are
placed 5-0 mm apart Determine the electric field
okso
Solution. Here, ^ = 1 p C = 10“^ C, at (fl) a point P on the axis of dipole 15 cm away
from its centre on the side of the positive charge,
Y
Yo
2 a = 1 cm = 10"2 m, r = 10 cm = 10~‘
oo
BB

m
as shown in Fig. 1.64 and at (b) a point Q, 15 cm
On the axial line of dipole. away from O on a line passing through O and
2lp Ir normal to the axis of the dipole as shown in Fig.
Y

2qx2axr
r ree

E = 1.64. (NCERT Solved Example)


oouu

47ieQ (r-2 ^^2 _^2j2


ad

FIGURE 1.64
9x109x2x10-^x10-2x10-'
Yd

Q
(10-2-0-25x10^)2
nidn

''

18
Re

10-^(1-0-0025)2
FFi

ISxlO'*
E = = 18 X 10^ N/C /15 cm>
0-9975x0-9975 I I \

-1 Opc A^ ! Uiopc
Example^ Two charges + 20 p C and 0 B '
- 20 p C are held 1 cm apart Calculate the electric K-5 mm-W

field at a point on the equatorial line at a distance M—16 cm —M

of 50 cm from the centre of the dipole.


Solution. Here, ^ = ±20pC = ±20xl0^C, Solution. Here, ^ = ±10pC = ± IQ-^ C
2a = 5-0 mm = 5 x 10“^ m
2a = 1 cm= 10“2m, r = 50cm = —m r = OP = 15 cm = 15 X I0“2 m
2
As 2 a < < r, therefore, intensity on equatorial I p I =qx2a = 10“^ x 5 x 10"3
line of short dipole is = 5 X 10-® C-m
ELECTROSTATIC CHARGES AND FIELDS 1/53

(a) As P lies on axial line of dipole, field of 10"^ N/C making an angle of 30“ with the
direction of the field. Determine the torque exerted
21 pir by the electric field on the dipole.
E. =
1
along BP
47t£Q {Hr. Board 2002)
As a < < r, Solution. Here, p = 4 x 10~^ C.m,
£ = 10-3 Q 30°
E.=^ =
2x5x10-8x9x10^
T = p£ sin e = (4 X 10-3) iq-3 sin 30“
47C€q r3 (15x10-2)3
= 2 X 10“* N-m
= 2-67 X 10^ NIC, along BP
ib) As Q lies on equatorial line of dipole. Example ^ In a certain region of space,

I?i electric field is along the Z-direction throughout.
I p I
The magnitude of electric field is, however, not
2 4neg (r2+a^)3/2 4jcGq r3

ww
constant, but increases uniformly along the positive
(v a<<r) Z>direction, at the rate of lO^ NCT^ per metre. What
are the force and torque experienced by a system
= -£,
2 *
=-x2.67x103
2
=1-33x103n/C having a total dipole moment equal to 10“^ C-m

Flo
in the negative Z-direction ?

e
is along a line parallel to BA as shown in

eree
Solution. Consider an electric dipole with - q
Fig. 1.64. charge at A and + q charge at B, placed along Z-axis,

FFr
so that its dipole moment is in negative Z direction,,

I
uurr
TYPE VI. TORQUE ON DIPOLE AND POTENTIAL i.e., Pj = -10-2 C-m, Fig. 165

orr
ENERGY OF DIPOLE IN ELECTRIC HELD

Formulae used. 1. Torque, x = p£ sin 6, where 0 is


sfo FIGURE 1.65
kks
Yoo
E
oooo

angle between p and E


2. Potential energy of dipole,
eBB

A (-<l)
U = ~pE (cos 02 - cos 0j)
^

where 0] is initial angle between p and E , and X


urr

02 is final angle between p and E .


ad
Yo

Units used, p in coulomb-metre, E in newton/


dY

coulomb ; 0, 0j, 02 in degrees ;


X in newton-metre and U in joule.
innd

The electric field is along + direction of Z-axis,


Re

such that
Example^ An electric dipole of moment
Fi

5 X 10~8 C-m is aligned in a uniform electric field clE


= 103 NC-' m-1
of 1-44 X 10“* N/C. Calculate potential energy of dz
the dipole to hold the dipole at 60“ with the dE
direction of electric field. From, F = qdE - qx xdz
dz
Solution. Here, p = 5 x IQ-^ C-m, 9i= 0“,
£ = 1-44 X lO'* N/C, 02 = 60“, PE. = ? = (^ X dz) X
dE dE

RE. =-pE (cos 02 - cos 0i) dz~^ dz


= -5x 1Q-8x 1-44x 10‘*(cos60“-cos0“) £ = - 10-2 X 103 = -10"2 N
£.£.=-7-2x10-^ (0-5-1) i.e., the force on the dipole is along negative
= 3-6 X 10-* J direction of Z-axis.

Example^ An electric dipole of dipole Now, 0= 180“.


moment 4 x IQ-^ Cm is placed in a uniform electric torque on dipole X = p £ sin 180“ = 0
1/54
'4. Fundamental Physics (XII) VOL.I

TYPE VII. ELECTRIC FLUX AND (/fV) On introducing dielectric medium inside S2,
GAUSS’S THEOREM IN ELECTROSTATICS electric flux through Si will not change.
Formulae used.
Example Rli] Consider a Uniform electric

1. E.ds = E s cos 0 , where 6 is angle field E = 3x10^ / NC Calculate the fiux of this
S field through a square surface of area 10 cm square
which normal to the area makes with E . (0 when its plane is parallel to Y-Z plane.
(11) when the normal to its plane makes an
Q angle of 60° with X-axis. (CBSE 2013)

w
2. (j)^ = 0 E^cis - , where Q is total charge (i.e.
^0
Solution. Here, £ = 3x10^/ N/C;
algebraic sum of all the charges) inside the closed A - (0-1 m)^ - 10“2
surface.
(0 As normal to area is in the direction of electric
Units used. E is in NC"*; 5 in ; <J)g in NC"* m^.

e
roow
field, therefore, 0 = 0°

re
(|> = E4 cos 0 = 3 X 10^ X 1(T^ cos 0°
Example 5j and $2 are two concentric = 30 Nm^ C-1
spheres enclosing charges Q and 2 Q respectively (ii) In this case, 0 = 60°

reF
uFFll
as shown in Fig.1.66. - EA cos 0 = 3 X 10^ X 10“^ cos 60°

e
= 15 Nm^ C-*
Example m The electric field components

sFr
foro
due to a charge inside the cube of side O'l m are
ofk
uor
= a j:, where a = 500 (N/C) Ey = 0, = 0.
Calculate the flux through the cube and the charge
inside the cube. [CBSE Sample Paper 2019-20,
kos
CBSE (OD) 2008, 2018 (C)]
Y
Yo
(0 What is the ratio of electric flux through
reeBB
oo

Solution. As is clear from Fig. 1.67, flux is only


Sj and $2 ? along X-axis.
(11) How will the electric flux through the
uurY

sphere 5j change, if a medium of dielectric FIGURE 1.67

constant K is introduced in the space inside in


place of air ?
ad
doo

(Hi) How will the electric flux through sphere


nY

Sj change, if a medium of dielectric constant K is 0-1 m->


■►X
introduced in the space inside 52 in place of air ?
nid
Re

(CBSE 2014) -4-0-1 m-^


Solution. ^0 From Gauss’s theorem
FFi

Q
electric flux through 5j is (j)j =
^0 Through the left face
<^i= A cos 180°
electric flux through S2 is <^2 ~ 2Q+Q^3Q = 500x0-1 X 10"2 (- l) = -0-5
^0 ^0
Through the irght face
3 4>2 = ■ A cos 0°
'2
= 500 (0-2) X 10"2x 1 = l-O
When a medium of dielectric constant K is
Net flux through the cube
introduced in the space inside 5], then
—> = <t>, + <|)2 = - 0-5 + 1 = 0-5 Nm^ C“*
Charge inside the cube = € q ({>
= 8-85 X 10-*2 X 0-5 = 4-425 x 10"*2 C.
ELECTROSTATIC CHARGES AND FIELDS 1/55

Exam pie A Uniformly charged Conduc Net flux through the cube = +
ting sphere of 2*4 m diameter has a surface charge = aa^^‘^-j2 - aa
5/2

density of 180*0 p.C/m^ (/) Find the charge on the


sphere. (i7) What is the total flux leaving the <J)=aa5/2 (V2-I) =800(0-1)5/2 (V2-I)
surface of the sphere ? (CBSE 2009)
= 1*05 N m2 Cr^
2-4
Solution. Here, R = — = 1-2 ar (b) By Gauss’s theorem,
2 ,-12
g = Sq(j) = 8-85 X 10 X 1-05
a = 180 0 pC/m2 = 180 x 10-^ Cm"2 = 9*27 X 10-12 Q
(0 Charge on the sphere q = 4 n G -Example An electric field is uniform,
= 4 X 3-14 X (1-2)2 X 180 X 10"^
and in the positive Ar-direction for positives ; and
= 3*53 X 10-3 Q
uniform with the same magnitude, but in the
3*53x10-3 negative x-direction for negative x. It is given that
(>i) Electric flux, =~

w
^0 8*85xI0-‘2 £ = 200/'n/C forx>0and £=-200?N/C for
= 3-9 x 10** Nm2 cr* X < 0. A right circular cylinder of length 20 cm

Flo
Example ES The electric field compo and radius 5 cm has its centre at the origin and its
nents in Fig. 1.68 are = ax*/2, Ey = E^ = Q in axis along the x-axis so that one face is at

reeee
which a = 800 N/C-m^2, Consider the cube shown X = + 10 cm and the other is at x - - 10 cm.

in Fig. 1.67. Calculate (a) the flux through the (a) What is the net outward flux through each flat

FFr
cube, and (b) the chaise within the cube. Assume face ? (b) What is the flux through the side of the
that a = 0*1 m. [CBSE 2018 (O) cylinder ? (c) What is the net outward flux through
(NCERT Solved Example)
for
the cylinder ? (d) What is the net charge inside
ur
the cylinder ? (NCERT Solved Example)
FIGURE 1.68
Solution, (o) Here, / = 20 cm, r = 5 cm = 0 05 m.
kkss
As is clear from Fig. 1.69, on the left face L,
Yo
oo

E and AS are parallel.


eB

FIGURE 1.69
a , A
R R Y
O
X
r
ou
ad

a ' a
a
5 cm
Z
YY

Solution. (") Here, = ax^^, Ey=0,E^=0 4 i


O*"-
R
a = 800N/C-m^/2.^^0-I m. V7
n
ndd
Re

As the electric field has only x component, x=-10cm'^ 20 cm ■►x=10 cm


Fi

therefore, E.AS =([)£. = 0 for each of four faces of


cube _L to Y-axis and Z-axis. Flux is there only for
A

left face L and right face R of the cube shown in Fig. (|)^ = £’.A5 = -200/.AS
1.68. = - 200 [jc (0*05)2] cos 0
At the left face, x = a = - 1*57 N m2 C-l
1/2 —» —*
E^=aa Again, on right face, E and AS are parallel.
^1=E^.AS =aa^/2(^2jcos 180° = -aa5/2 4>^ = £.A?=-t-200?.A5
At the right face, x = a-i-fl = 2a = 200 [7C (0*05)2] cos 0°
1/2
= a (2 a) = l-57Nm2cr^

= ^. as' = a (2 fl)‘/2 {fl2) cos 0° For any point on the curved surface of the

= a^5/272 cylinder, E A-AS .*. <!> = £’.A5=0


1/56 Fundamental Physics (XII){SSm

(c) Net outward flux through the cylinder, Ze-Zer^lR^


<t>= 1-57 + 1-57 + 0 From (h), E (r) =
= 314Nm2c-l 4jte«0 47te«0

By Gauss's Theorem, Ze f 1 r \
-12
^ = eg 4* = 8-85 X 10 X3-14
4tC€ 0 V, r2
*= 2-78 X 10-1* Q

exampia^ An early model for an atom Example §0 \ hoUow cylindrical box of


considered it to have a positively charged point length 1 m and area of cross section 25 cm^ is
nucleus of charge Ze, surround^ by a uniform placed in a three dimensional co-ordinate system
density of negative charge upto a radius R. The as shown in Fig. 1.71. The electric field in the
atom as a whole is neutral. For this model, what is
the electric field at a distance r from the nucleus ?
region is given by £ = 50x f , where E is in
NC“1, and x is in metre. Find

w
(NCERT Solved Example)
(i) Net flux through the cylinder
Solution. The charge distribution for this model
(fi) Charge enciosed by the cylinder.
of atom is shown in Fig. 1.70.

Flo
(CBSE 2013)
Charge on nucleus = + Ze
As atom is neutral, total negative charge = - Ze FIGURE 1.71

ee
4 y
Charge

Fr
.*. Negative charge density, p =
Volume
^ A
ni rv^

for
FIGURE 1.70
ur
0, » ■►X

.-●Pi K y e2
^m
ks
r h+ ♦I
?2 z ^m
Yo
oo

-IR
+Ze Solution. Here, ds = 25 cm^
B

= 25 X 10^ m^, along X-axis


re

A
ou
ad

On the left end of the cylinder,


Y

-Ze -3Ze
P = ..(0 = 50 X 1 NC“', along X-axis
4itR^
nd
Re

3 (|)j =£j ds cos 180°


Imagine a Gaussian surface - which is a = 50 X 25 X 10^ (-1)
Fi

spherical surface of radius r with centre at the nucleus = -0-125 NC-’


(not shown). On the right end of the cy'inder,
According to Gauss’s theorem in electrostatics.
£2 = 50 X 2 = 100 NC“^ along X-axis
4> = £ (r)x47cr^ = — m
(f)2 = £2i/^cos0°= 180x25x 10^(1)
^0
= 0-250 NCr*
(/■) When point Pj is outside,
Net flux through the cylinder
i.e.,r>R,q=Ze-Ze^0 :.E{r) = Q.
(ii) When point P2 is inside, i.e., r < R, Charge (}) = (i)j + (|}2 =-0-125 + 0-250
enciosed by Gaussian surface. = 0-125 NC-*
-12
47cr^ z-3 ^ = Eg (J) = 8-85 X 10 X 0-125

q' = Ze + — p = Ze-Ze ...using (0 = 1-106 X 10-*2 C


R^
ELECTROSTATIC CHARGES AND FIELDS 1/57

TYPE VIII. APPLICATIONS OF -3-8 X10-”^


GAUSS’S THEOREM
Linear charge density, X = — -
/ 2-2
= - 1-73 X ICr'^Cm-^
Formulae used. 1. Electric intensity due to a line
X X 2X
charge, £ = As £ =
27I€
0^ 2tC€o f 47teo
2. Electric intensity due to thin infinite plane sheet
£ =
2(-l-73xlQ-'^)x9xI0^
of charge, £ = a/2 s 0 3-6x10-3
When sheet of charge is thick, £ = a/e q - 8-6 X 10^ N/C
3. Electric intensity between two thin infinite plane
sheets carrying equal and opposite charge densities T
'Example A large plane sheet of charge
£ = o/e 0 having surface charge density 5 x 10"*^ cm“^ lies

ww
4. Electric intensity at a point on the surface or in XY plane. Find electric flux through a circular
area of radius 1 cm. Given normal to the circular
Q
outside a charged spherical shell. £ = area makes an angle of 60° with Z-axis.
4jc€«0 ’
Solution. Here, a = 5 x 10“^^ cm“3, <|) = ?,

Flo
where r = /? on the surface r = 1 cm = 10“3 m, 0 = 60“

e
eree
5. Electric intensity at a point inside a charged solid a
= £ (AS) cos 0 — 71 cos 60“
'●p 2€

FFr
non conductingsphere, E = 0/
3e
uurr
Units used. £ is in NC‘ ; X is in C m-', a is in _5xlQ-^^x3-14(10-^)3xl/2

orr
C m"^ and p is in C m“3 2x8-85x10-'2
sfo
= 4-44 x 10-’ Nm^ C~^
Example M Two large, thin metal plates
kks
Yoo
Example ES In Fig. 1.72, calculate the
oooo

are parallel and close to each other. On their inner


faces, the plates have surface charge densities of total flux of the electrostatic field through the
eBB

opposite signs and of magnitude 19 x Cm“^, spheres and £2. The wire AB shown here has a
What is E (a) to the left of the plates (A) to the linear charge density X given by X = cfc, where x is
right of the plates (c) between the plates ? the distance measured along the wire from end A.
urr

Here, o = 19 x IQ-^^
ad
Yo

Solution, (a) On the left, the fields of the two


dY

plates are equal and opposite. So £ = Zero.


(b) On the right, the fields of the two plates are
innd
Re

equal and opposite, SoE - Zero,


(c) Between the plates, the fields due to both
Fi

plates are in same direction, so the resultant field is

£ = -5
a a 19x10-32
2€
0
26
0 ^0 "8-85x10“'3
= 2-14 X 10-10 Solution. Charge on an element of length dx of
wire A£
Example 03 A plastic rod of length 2*2 m
dq = Xdx = kx dx
and radius 3*6 mm carries a negative charge of Total charge on wire AB,
3*8 X 10“^ C spread uniformly over its surface. I
What is the electric field near the mid-point of the 1
rod, at a point on its surface ? g=fdq=\kxdx = k — = -it/3
^ J ^ J0 [2 Jo 2
Solution. Here, / = 2-2 m,
r = 3-6 mm = 3-6 x 10“3 m. Q
Total flux through -Vi -
^ = - 3-8 X 10-3 c, £ = ? ^0
1/58 ^ Fundamental Physics (XII)

_ Q+Q Force on this charge exerted by the charge + e


Total flux through 82=^^- qje)
^0 ^2 = directed away from (e)
4TceQ {a-xf '
e+it/2 For the charge ^ to be in equilibrium F| = F2
^0 i.e.
q{Ae) qie)
tP- 4;teQ {a-xp
I
0
TYPE IX. TYPICAL EXAMPLES
4 1 2 1
or or

Example [i: A particle of mass m carrying


{a-xp x a-x

or X = 2fl - 2x
a charge - 9] is moving around a charge + ^2 along
3x - 2a X = 2al3
a circular path of radius r. Prove that period of or

revolution of charge - about + 92*^ given by Hence the charge q should be held at a

w
distance 2u/3 from charge (+ 4 e).
T =
167t^ €q mr^ Example EE] An inflnite number of
1

Flo
charges each equal to 4 )xC are placed along X-
Solution. Here, force of attraction between axis atx = lm,x = 2m,x = 4m,x = 8m and so on.

ee
charges = centripetal force Find the total force on a charge of 1C placed at
the origin.

Fr
2
1 mv
Solution. Here, ^ = 4 p.C = 4 x 10"^ C, = 1C
4tC6 r2 r
0
From the principle of superposition, total force

for
ur
1 acting on 1C charge at the origin due to all the given
So v = charges is
■y4n€ 0 mr
s
1 1 1
ok
Time period of revolution
Yo
F =
4tC€ 0 ^2
n ^22 ^32
Bo

2nr 47C€^0 mr
T = = 27tr
V
1 1 1
^1^2 = 9 X 10^ X 4 X 10“^ X I
re

-7 + —7 + —7 +
P 2P 42
167C^€n-mr
3
As sum of infinite geometric progression
ou
ad

7’ = i,
\ «I«2 a
I _4
Y

1-r 1 "3
Example E0 Two fixed point charges + 4e 1--
nd

4
Re

and +e units are separated by a distance a. Where


4
should the third point charge be placed for it to F = 9x 10^x4x 10"^x - =4‘8x 10‘*N
Fi

be in equilibrium ? (Chhatisgarh Board 2012)


Solution. Let a point charge q be held at a Example ^ A pendulum bob of mass 80
distance x from the charge + 4 c. Fig. 1.73. milligram and carrying a charge of 2 x C is
at rest in a horizontal uniform electric Held of 2 x
FIGURE 1.73
lO** Vm~^ Find the tension in the thread of the
F2
4
+q Fi pendulum and the angle it makes with the vertical.
+4e - —^
+e
Solution. Here, m = 80 mg = 80 x 10“^ kg,
(a-x)—M
M-
^ = 2x Ur^C, £ = 2x lO^V/m
X

.'. Distance of q from charge + e-(a-x) Let T be the tension in the string and 9 be the
Force on this charge exerted by the charge + 4 e angle it makes with the vertical, Fig. 1.74.
q(4e) In equilibrium, T sin 9 = ^ £ <0
is ,F1 = , directed away from (4 e)
4jceA0 x^
and T cos 9 = mg
ELECTROSTATIC CHARGES AND FIELDS 1/59

At A, Field intensity due to charge


1 giXl . along PA
47ueQ AP'^
9x10^x10"^
= 36000 N/C
(0-05)2
Field intensity due to charge ^2
_ 9x10^x10-^ ; along AQ = 36000 N/C
(0-05)2

ww
Net field intensity at A = 36000 + 36000
= 72000 N/C = 7*2 x 10** N/C along AQ
Dividing, we get At B, Field intensity due to charge qt

Flo
1 ?i

e
tan 6 =
qE - — : along PB produced

eree
mg 47ueQ
_ 9x10^x10“^

FFr
= 36000 N/C
2xl0"^x2xl0‘^ (0-05)2
uurr tan0 = = 0-5102
80x10-® x9-8

orr
Field intensity due to charge ^2

From (i),
9 = 27'* sfo
1
, along BQ
4tcGq bQ^
kks
Yoo
oooo

qE _2xlQ-^x2xl0^ 9x10^x10-^
= 4000 N/C
sin 6 sin IT (0-15)2
eBB

= 8-8 X 10“^ N Net field intensity at B = 36000 - 4000


= 32000 N/C
Example ES Two point charges of 10"* C
uurr

= 3*2 X lO'* N/C, along PB produced


and - 10“® C are placed 0*1 m apart. Calculate
ad

At C, Field intensity due to charge q^,


Yo

electric field intensity at A, B and C shown in Fig.


dY

1 ^1
1.75. »along PC.
(NCERT Solved Example)
47t€n0 PC2
innd
Re

FIGURE 1.75
9x10^x10-®
^1 = = 9 X 102 N/C
Fi

(0-1)2
►R
/ 60° Field intensity due to charge
1 ^2
/
along CQ.
0.1 m/
/
'\0.1 m 4neQ QC^ ’
9x10^x10-8
qi=+10 ®c/ = 9 X 10^ N/C.
\
-10“®C ^2 =
B P Q
(0-1)2
A
M-
0.05 m 0.05 m 0.05 m
-H Both, E^ and £2 equal in magnitude, acting
at 60° to CR parallel to BA.
Solution. The charges -f 10“8 C and - 10“8 C Resultant intensity at R
are held at P and Q respectively, PQ = 0-1 m. Fig. = £[ cos 60° + £2 cos 60° = 2 £j cos 60°
1.75. 1
= 2x9xl0^x
Also, PA=PB = 0-05 m 2

QA = 0-05 m and CP = C0 = 0-1 m = 9 X 103 N/C


1/60 ‘P'un.eUe^ 4- Fundamental Physics (XII) iWI

It is represented by CR parallel to BA. The Total charge on the wire


—^ >

components of fj and in directions ± to CR r


Kx^
0
Kl}
9 = Kxdx —
cancel out. 2 2
0 0

Example^ Fig. 1.76 shows a closed Electric flux through the Gaussian hollow
surface surrounding some electric charges
(a) what is the net electric flux through the surface is
KJ}
surface ? (6) Is the electric flux directed inward ^0 2e 0
or outward from the surface ?

Example ^ Two large thin metal plates

ww
are parallel and close to eachother. On their inner
fac^, the plates have surface charge densities of
opposite signs and of magnitude 17*7 x 10"^ CAn^.
What is electric field intensity E (a) in the outer

Flo
region of the first plate and between the plates ?

e
(CBSE Sample Paper 2022-23)

rree
Solution. Here, surface charge density of plate

r FF
Solution, (a) Here, total charge inside the A = + 17-7 X 10-22
surface : Surface charge density of plate
uurr
^ - 6-7 pC + 3-2 pC = - 3-5 pC
-3-5x10-^
for
B = ~ 17-7 X 10-22 am2
In the outer region of plate A, electric field
kss
* ^
As <t»£=r intensity £ = 0
^0 8-85x10-^2
ooook
Yo
FIGURE 1.77
= - 3-95 X 10^ Nm2 CT*
eB

(b) As is negative, it is directed Inwards.

Example £0 \ wire AB of length L has I II B Ill


rr

A
ou
ad

linear charge density X - Kxj where x is measured


from the end A of the wire. This wire is enclosed
Y
dY

by a Gaussian hollow surface. Find the expression


for electric flux through the surface. Inbetween the plates o = + 17-7 x 10 22 C7m2
Re
innd

[CBSE 2017 (C)l Permittivity of free space g q


= 8-85 X 10'*2n-^ c2m-2
Fi

dq
Solution. Here, X = — = Kx\ ([) = ? 17-7xl0“22
dx
●●● £ = — = 2-0x 10-‘°N/C
dq = Kxdx ^0 8-85x10->2

1. Electrostatics stands for static electricity involving study of charges at rest. These charges are developed on
insulators when they are rubbed against each other. The charges are produced due to actual transfer of
electrons.
2. Two important properties of charge are Quantization and Conservation. The quantization means that charge
can exist only as integral multiple of electronic charge. (- e\ i.e., q = ±ne, where n is an integer.
The conservationof charge means that on an isolated system, total electric charge always remains constant.
Obviously,the charges can be created only in equal and unlike pairs.
Et£CTROSTAnC:CHARQEStAND FIELDS 1/61

3. Like charges repel and unlike charges attract each ptiier,^th a fp^ F. governed; by law
—> 1 9i > ●'.Hi'
p2\ - "j ^^“3 hi (when > 0), i.e., charges are like charges
47tSo r

and F - ^2 r2i (when qy ^2 < 0)» » charges are unlike charges.


4ne„r3
where ^2 the charges held r distance apart in vacuum ; F2J is force on ^2 hy and 1^2 ^ vector

ooww
1
pointing q^ to ^2- = 9 X10^ N C”^, where e q is absolute electrical permittivity of free space.
47CSj
When the same charges are separated by the same distance in a medium of relative permittivity the
1
force between the charges becomes : =

e
12
4jC€o/: r3

re
rFFl
4. To find total force on a given charge due to any number of charges q^, ^2

ree
9n* we use superposition

F
principle, according to which
^0 =-^01 + ^02+^03 + Fon

rF
i.e. vector sum of forces due to individual charges would give us the total force.

sffoor
5. Electric Field
ouur
kosk
● F
Elecdic field intensity at any position (r) is defined as E(r)= Limit —
^0
YYo
oo

where F is total force experienced by a small positive test charge ^0-Th® direction of E is the direction
BB

along which the positive test charge would try to move, if fiee to do so.
rre

1 q A
Electric field intensity at a point distant r from a point charge 9 in air is e =
ouu

4jcSo
Y
ad

Electric field intensity due to a group of charges is obtained by applying superposition principle. The SI
dY

unit of F is NC^.
innd

6. An electric line offeree is a path, straight or curved, such that tangent to the path at any point gives us the
Re

direction of electric field intensity at that point. No two electric lines of force can intersect eachother.
FFi

7. Two equal and opposite charges separated by a small distance ate said to form an electric dipole. Many of
the atoms/molecules are electric dipoles.
Every dipole is associated with a dipole moment p whose magnitude is equal to the product of the
magnitude of either charge (^) and the distance 2 ? between the charges, i.e., p = ^ x (2 ?)

The direction of P is from negative charge to positive charge. It is measured in coulomb-metre. Every
dipole has an electric field around it
I 2r
Field; intensity at any point on axial line of electric dipole is, I FI = —— along p
4jc€o (r2-fl2)2 ’
where r is distance of the point from the centre of the dipole.
I 1
If the point lies on equatorial line of dipole at the same distance r, then I FI = —— , opposite
4jteo (r2+fl|2)3/2
to p
1/62 "PnaxCee^ ^ Fundamental Physics (XII)

8. When an electric dipole of moment P is held at an angle 0 with the direction of a uniform electric field E,

a torque t acts on the dipole, given by : x = p x £ = sin 6, in magnitude.


This torque tries to align the dipole in the direction of the field, reducing angle 0 to zero.

To orient the dipole at any other angle with E , work has to be done on the dipole, which is stored as P.E.
^ -*
of dipole given by : P.E. = -p£cos0 = - p.E
9. Electric Flux over an area in an electric field represents the total number of electric field lines crossing this
area. It is represented by ([>£●
Electric flux is a scalar quantity, and is measured in Nnp- C“^
According to Gauss’s theorem in electrostatics, total normal electric flux over a closed surface S in vacuum
is 1/€q time the total charge (Q) contained inside the surface,

ww
f ^ Q
= o E.ds = —
s ^0

Floo
Coulomb’s law of force can be obtained from Gauss’s theorem. This theorem can be used for calculating
electric field intensity due to highly symmetric charge distributions.

ee
eer
rFrF
QUESTIONS

Based on NCERT Book


rur
I. Multiple Choice Questions
s ffoor FIGURE 1(Q).3
1. According to Coulomb’s law, which is the correct
osk
YYoou
relation for the following figure ? I 1
oook

I I

2 1
■►X o >X
eBB

FIGURE 1(Q).1 O +
2
4 6 4 6

-E -E

^2 (6)
r

Fi2 ^21 (a)


ouur
ad

(«) >0 (b) <0


Y

—I

I 4 6
«l \—►X O
^4-^X
nd dY

o + +
(C) ^1^2 = 0 (d) I > — >o 2
Re

2 I I

^2 I j
I
FFini

(CBSE Sample Paper 2022-23) (c) id)


2. The electric potential V as a function of distance X
is shown in Fig. 1(Q).2. (CBSE Sample Paper 2022-23)
18
3. An object has a charge of 1 C and gains 5-0 x 10
FIGURE 1(Q).2
electrons. The net charge on the object becomes
(fl)-0'80C (b) + 0-80 C
(c)+ 1-80C (d) + 0-20 C
(CBSE 2022)
4. A negatively charged object X is repelled by another
charged object Y. However, an object Z is attracted
X to object Y. Which of the following is the most
possibility for object Z ?
(a) positively charged only
The graph of the magnitude of electric field (£) as
a function of X is (b) negatively charged only
ELECTROSTATIC CHARGES AND FIELDS 1/63

(c) neutral or positively charged {a)2E {b)E


{d)EIA (CBSK 2022)
{d) neutral or negatively charged (CBSE 2022) (c) EH
5. In a chamber, three spheres carry charges + 3 e, 10. A square sheet of side ‘a’ is lying parallel to XF
+ 5 e and - 3 e. They came in contact for a moment plane at z = o. The electric field in the region is
and got separated. Which one of the following are
possible values for the final charges on the
~E=cz^k . The electric flux through the sheet is
spheres ?
(a) ib) Zero
(a) + 5 e, - 4 e, + 5 e
(/?) + 6 e, + 6 e, - 7 c
1 4
(c) - 4 e, + 3-5 e, + 5-5 e (c) c {d) -fl^c (CBSE 2022)

oww
id) + 5 e,-Z e,-¥l e (CBSE 2022)
11. An electric dipole placed in a non-uniform electric
6. The electric potential on the axis of an electric field will experience
dipole at a distance r from its centre is V. Then the ib) only a torque
(a) only a force
potential at a point at the same distance on its
(c) both, force and torque

e
equatorial line will be
(CBSE 2022)

re
FFrlo
{a)lV {b)-V {d) neither force nor torque
(c) V/2 {d) Zero 12. Let A^j be the number of electric field lines going

rF
ee
(CBSE Siimplf Paper 2022-23) out of an imaginary cube of side a, that enclosed
an isolated point charge 2 q and A^2
7. A potential difference of200 V is maintained across corresponding number for an imaginary sphere of

rF
ouru
a conductor of resistance 100 The number of
radius a that encloses an isolated point charge 3 q.
electrons passing through it in 1 second is

fosor
18 Then N\!N2 is
(a) 1-25 X 10^^ {b) 2-5 X 10 skf
18 16 I 2
(c) 1-25 X 10 (d) 2-5 X 10 (a) - (^>3
ooko
K
(CBSE 2022)
Yo
9
Y
8. Which of the diagrams correctly represents the (CBSE 2022)
{d)n
Bo

electric field between two charged plates if a


reeB

neutral conductor is placed inbetween the plates ? 13. Which statement is true for Gauss Law ?

FIGURE 1(Q).4
(a) All the charges whether inside or outside the
ooY

Gaussian surface contribute to electric flux


uur
ad

+ + +
(b) Electric flux depends upon the geometry of the
Gaussian surface
dY

1 (c) Gauss’s theorem can be applied to non-uniform


nind

] [Z Z] electric field
Re

(a) (6) (d) The electric field over the Gaussian surface
FFi

remains continuous and uniform at every


+ +
±] ±]
point
14. In the process of charging, a positively charged
body
(a) loses mass (i?) gains mass
ic) id) (c) neither of the two
(d) depends on conditions
(CBSE 2022)

9. The magnitude of electric field due to a point 15. Electric flux of an electric field E through an area
charge 2 ^ at a distance r is E. Then the magnitude
dA is given by
of electric field due to a uniformly charged thin
spherical shell of radius R with total charge ^ at a E
(«) — (b) E-dA
distance — (r » R) will be dA
2
1/64
“Pn/idee^ 4 Fundamental Physics (XII) VOL.I

(c) Conservation of mass


EdA ExdA
ic) id) id) Quantisation of charge
^0 ^0 24. SI unit of surface integral of electric field is
(CBSE 2622) ia) Nm^ C-2 ib) Nm-2
16. A charge Q is placed at the centre of the line joining ic) Nm^ C"> id) Nm C~2
two charges q and q. The system of three charges 25. The relation between electric intensity (£) and
will be in equilibrium if Q is electric flux <j) is
ia)-qn ih)-qll
r a

ia) ^ = OE-ds = — ib) <^ = ~


(i/) - q/4 (CBSE 2022) ^0 ^0

17. Two point charges + 16 ^ and - 4 q are located at

ww
(c) (}) = — id) none of these
j: = 0 and x = L The location of the point on x-axis ^0
at which resultant electric field due to these charges
is zero is 26. What does q^ + q2 = 0 signify ?

FF loo
ia)L ib)6L ia) Electric potential ib) Electric intensity
ic)4L id)2L (CBSE 2022) (c) Electric dipole id) None of these

ree
18. An electric dipole of dipole moment 4 x 10"^ Cm 27. 1 stat-Coulomb = Coulomb
kept in a uniform electric field of 10”^ N/C ia) 3 X 10^

reeF
ib) 3x10-^
experiences a torque of 2 x 10^ N-m. The angle
which the dipole makes with the electric field is (c)ixlO^ id) -xlO-^
ur
ia) 30°
(c) 60°
(i>)45°
id) 90° (CBSE 2020)
fofroF
28. When a plastic comb is passed through dry hair,
ks
the charge acquired by the comb is
19. Which of the following is a vector quantity ?
ia) always negative
kos o
YYouor
ia) Electric charge ib) Electric flux
ic) Electric field
ib) always positive
id) Electric potential
BBoo

ic) sometimes negative


20. Electric field lines contract longitudinally. This is
r ee

because of id) none of the above

(a) attraction between unlike charges 29. The cause of quantization of electric charge is
ad
oouur

ia) transfer of electrons


ib) repulsion between like charges
Y

(c) both ia) and ib) ib) transfer of protons


id) neither (a) nor ib) (c) transfer of integral number of electrons
nYd
Re

21. Charge on a conducting sphere resides id) none of the above


FFindi

ia) inside the sphere 30. Charge on a body which carries 200 excess
electrons is
ib) at the centre of the sphere
(a) - 3-2 X 10“^^ C ib) 3-2 x ICT*® C
(c) outside the sphere
(c)-3-2x Ur'^^C id) 3-2 X 10‘ 17 c
id) on the outer surface of the sphere
31. Electric field due to a single charge is
22. Four charges + 5Q,-3Q, + 2Q,-4Qaie enclosed
inside some surface. What will be the outgoing flux ia) asymmetric ib) cylindricallysymmetric
through the surface (c) spherically symmetric
(a) 14V-m ib) Zero id) none of the above
ic) 4 V-m id)-4 V-m 32. At a given distance from the centre of short electric
23. Charge on a body is integral multiple of ± e where dipole, field intensity on axial line is k times the
e is electronic charge. This is because of field intensity on equatorial line, where k =
ia)2 ib)3
ia) Conservation of charge
ib) Quantisation of charge ic)4 id) 1
ELECTROSTATIC CHARGES AND FIELDS 1/65

33. Electric field due to an electric dipole is 41. For a point charge, the graph between electric field
versus distance is given by :
(n) spherically symmetric
{b) cylindrically symmetric FIGURE

(c) asymmetric
(d) none of the above
34. When an electric dipole is held at an angle in a
uniform electric field, the net force F and torque T
on the dipole are
(a)F=0,T = 0 (/>) F 0, X 0

oww
(c)F=0,x^0 {d)F^0,'C = 0
35. The dimensional formula of electric flux is

id) A~^\ {/?)

e
(c) (d)

FFrlo
re
36. A closed surface in vacuum encloses charges - q 42. Four charges + SQ ~3 Q + 5 Q and - 10 2 are kept
inside a closed surface. What will be the outgoing
and + 3 Another charge - 2 q lies outside the

ree
F
surface. Total electric flux over the surface is flux through the surface,
(a) 26V-m (fc) 0 V - m
(b) 2 q/GQ

rF
(a) zero
(c) lOV-m id) 8 V - m
3? 43. On charging a neutral balloon, its size-

fsoor
ouur
(c) - (d) 4 q/eQ
^0 fa) Increases
skf
37, The number of electric lines of force radiating from (b) Decreases
ooko
a closed surface in vacuum is 1-13 x 10**. The (c) Remains same
Yo
Y
charge enclosed by the surface is (d) No relation between charge and size
Bo

44. Electric field lines contract lengthwise. It shows


reB

(a) 1C (i.)lliC
(c) 0-1 C (^/)0-l (a) Repulsion between same charges
(b) Attraction between opposite charges
38. Two charges 3- x 10'^ C and 5 x 10"* C are placed
uur
ooY

(c) No relation between force & contraction


ad

at a distance 10 cm from each other. Find the value


of electrostatic force acting between them. (d) Electric field lines do not move on straight
dY

path
(a) 13-5 X 10" N ib) 40 X 10'* N
45. A charge q is placed at the point of interrection of
innd
Re

(c) ISOxlO^N (^0 13-5 X 10*0 N body diagonals of a cube. The electric flux passing
39. What is the S.I. unit of electric flux ? through any one of its face is
FFi

3q
«.)
N
- xrn^ ib) Nxm^ (a) ^
6e
(b) —
0

6q
N N'^ ic) ^ id) -A
jxC^
36
ic) id) ^0 0
m m
(CBSE Sample Paper 2020)
40. What is the value of minimum force acting between
two charges each of charge e placed at I m apart II. Assertion-Reason Type Questions
from each other ?
Direction. For question numbers 46 to 53, two
(a) Ke^ ib) Ke statements are given, one labelled Assertion (A)
and the other labelled Reason (R). Select the
Ke Ke^
correct answer to these questions from the codes
(a), (A), (c) and (d) as given below :
1/66
‘P’uidcefr ^ Fundamental Physics (XII) VOLJ

(a) Both, A and R are true, and R is correct 49. Assertion. Two identical charges repel each other
explanation of A. with a force equal to 10 mg wt, when they are 0-6 m
(b) Both, A and R are true, but R is not the correct apart in air. The value of each charge is 2 pC and
explanation of A. g = 10 ms"^.
(c) A is true, but R is false. Reason. Like charges repel each other.
(d) A is false, and R is also false. 50. Assertion. If sum of two charges and ^2 is
46. Assertion. The ratio of the electric force between then the force between and ^2will be maximum

two protons to that between two electrons is of the


if the ratio Qlq^ is 2.
order of one. Reason. The coulomb force between the two
charges is
Reason. The like charges repel each other.
1
47. Assertion. A force of 40 N acting between two F =
'}
charges certain distance apart in air. If the space 471s
0
r~

ww
between them is filled with a medium of relative
51. Assertion. When a conducting medium is placed
permittivity 8, then the force between the same two
between two charges then the electric force
charge is 5 N. between them will become infinite.
Reason. The coulomb’s force between two charges

Flo
Reason. The addition of conducting medium
qi and ^2 placed distance r apart in medium of between two charges reduces the distance to zero.

e
relative permittivity e ^ is

rere
52. Assertion. The number of electric lines of force
emanating from 1 pC charge in vacuum is M3x 10*.

r FF
1 ^2
F =
,2 Reason. It follows from Gauss’s Law in electro
statics.
uurr
48. Assertion. The coulomb force is the dominating
force in the universe. for
53. Assertion. The number of electrons in one coulomb
of charge is 6-25 x 10“**.
kss
Reason. The coulomb force is weaker than the
Reason. This follows from the relation q = ne^,
ooook

gravitational force.
Yo
where symbols have usual meaning.
eBB

ANSWERS

I. Multiple Choice Questions


rr
ou
ad

l-ib) 2. (a) 3. (d) 4. (c) 5. (b.c) 6. (d) 7. ia) 8. id) 9. ia) 10. (c)
11. (c) 12. (b) 13. (d)
Y

14. ia) 15. (b) 16. id) 17. (a) 18. ia) 19. (c) 20. ia)
dY

21. id) 22. (b) 23.(b) 24. ic) 25. (a) 26. (c) 27. id) 28. ia) 29. (c) 30. (c)
31. (c) 32. (a) 33. ib) 34. ic) 35. (c)
Re

36. ib) 37. ia) 38. ia) 39. ia) 40. ia)
innd

41. ib) 42. (b) 43. (a) 44. ib) 45. (fl)
Fi

II. Assertion-Reason Type Questions

46. ib) 47. ia) 48. id) 49. ib) 50. ia) SI. id) 52. id) S3, id)

HINTS/EXPLANATIONS For Difficult Questions

I. Multiple Choice Questions


2. We find that from x = 0 to x = 2, =—= 4 .
dx 2
1. In the Fig. I(Q).l shown, ,F12 and -F21 are the
dV
forces of attraction. Therefore, charges ^ j, ^2 niust As E = -
dx '
so, E is negative constant.
be of opposite sign, i.e., ^2 <
From X = 2 to 4, V= constant. Therefore, £ = 0
and so on. Choice id) is correct.
ELECTROSTATIC CHARGES AND FIELDS 1/67

Stotal ” ^object
- A A
7 ">
4) = E-A = {cz} k)a~ ● k = cz a-
= 1-0 + 5 X 10'® (- 1-6 X 10-'^)
At, z = a
Gtotal =1-0 +(-0-8) = 0-20 c 4
4. As X is repelled by Y, therefore, charge on Y must ({) = ca^ = ca
be negative. As Z is attracted by Z must carry 11. In a non-uniform electric field, an electric dipole
no charge or positive charge. Choice (c) is will experience both, force and torque. Thus
correct. option (c) is true.
5. Here, total charge = Qi + Q2+ 12. According to Gauss’s theorem,
= 3 e + 5 e ~3 e = 5 e 2q 3q
N,= N^ =
As all the three spheres came in contact

oww
^0 ^0
simultaneously and got separated, the charge must
N
be conserved !
X

Ni 3q 3
i.e..
2i + 02 ^3 - 5 e
13. For Gauss’s Law, the electric field over the

ee
Now, 6e+6e-7e=5e
Gaussian surface remains continuous and uniform
As - 4 e + 3-5 e + 5'5 e = 5 e
at every point. Option (d) is true.

FFrlo
r
Thus, choice (b) and (c) are correcL 14. A body gets positively charged when it loses some

rF
6. From the knowledge of theory, electric potential electrons. It involves loss of mass. Choice (a) is

ee
at any point on the equatorial line of electric dipole correct.
is zero.
ouru —^ ^

rF
y ne 15. By definition, d^= E-dA
7. I =—
R t 16. As is clear from Fig. 1(Q).7, charge qdXB will be

ffosor
in equilibrium if net force at B is zero.
.'. n = l2^ = Y. L 200x1
os k
e ~R e 100xl-6x!0'*^ FIGURE 1(Q).7

= 1-25 X 10'^
ook
q Q q
Yo
Y
A B
8. Electric field lines always start from positive H- lx ■H
Bo

charge and end at negative charge. Therefore ♦1


reeB

K
option (d) is the correct choice.
9. Electric field due to a uniformly charged thin
oouY

q^q ^ Qq or
ur

spherical shell at a distance r is {1x9


ad

£ =
\ Q _ 2q 17. Let the resultant electric field due to the three
Y
d

47ceQ 47ieQr^ charges be zero at F, at a distance x from charge


-4^, Fig. 1(Q).8.
nidn

When charge becomes q and distance is r/2, then


Re

q Aq FIGURE 1(Q).a
r = = 2E
F
Fi

4JiGo(r/2)^ 47te,^0 16^ -Aq q

10. Herez = a [Refer to Fig. 1(Q).6] j = 0 x=L p


N- X
>l
FIGURE 1(Q).6

Z
kq.y\6 _ k{Aq)
a
{x + lS- ~
4
a = — or 4x=2x + 2L
x+ L X
x-L
♦Y
18. Here, p = 4 X 10"^ cm, £ = 10 ^ N/C
T = 2 X 10-® Nm, e = ?
As x=pE sin 0
2x 10-® = 4x 10*5 X 10“^ sine
1/68
7>>teieUep^ 4. Fundamental Ph}rsics‘ (XII) QVOL.i

sin0 = -
1
0 = 30®
34. When an electric dipole is held at an angle in a
2 uniform electric field, there will be equal and
19. Out of the given quantities, electric field is a vector. opposite forces on the positive and negative
charges of the dipole. Due to it, the resultant force
20. Electric field lines contract longitudinally because
on the dipole will be zero. But these forces will
of attraction between unlike diarges.
21. On a charged conducting sphere, charge resides form a torque to rotate the dipole to align in the
direction of electric field.
on the outer surface of the sphere.
4tc Fr^
22. Electric flux = M _5g-3e+26-4Q 35. Electric flux, (j> = -^ = 9

^0 ^0 Sq q^lAnFr^ q
= Zero
(A/Lr-2)xL2
23. This property of electric charge is due to Dimensions of <{> = = [ML^
AT
quantisation of charge.

ww
Q
24 . Surface integral of electric field = JscosO. 36. Electric flux over a closed surface, <|) = —
s ^0
SI unit of surface integral of electric field is where Q is the total charge enclosed by the closed

Flo
= (N/Q (m2) X 1 surface. Here Q = -q+3 q = 2q

ee
= Nm^C-^ (j) = 2^/eo

rere
25. According to Gauss’s theorem in electrostatics. 37. No. of electric lines of force radiating from a

r FF
closed surface in vacuum s electric flux linked
(|> = 4 E‘ds = — Q
with the closed surface, ({► = —= M3 x 10^*
^ ^0
uurr
26. q\ + qi~^ ^2 = - ^1
Le. the two charges are equal and unlike. They
foor G=M3x10“x€o^°
= M3 X lO^i X (8-85 X 10"^2) „iq
ks s
signify an electric dipole. 38. Here, ?, = 3 x KT^ C, qfj = 5 x 10^ C,
Yoo
oook

27. As, 1 C = 3 X 10^ stat coulomb r = 10 cm = 10"^ m


eBB

.’. 1 stat coulomb = —^^C


3x10^
= -xl0-*^C
3
F = -M2_ _9x10^x(3x1Q-^)x(5x10^)
47C€or2 (10-^)2
28. When a plastic comb is passed through dry hair,
rr

= 135 X lO^O N = 13-5 X 10“ N


ouu

the electrons pass from hair to comb. Due to it,


ad

39. Electric flux, ({> = F X


the comb acquire native charge.
YY

SI unit of electric flux = {NIQ x


29. The cause of quantization of electric charge is due
to transfer of integral number of electrons.
nndd

cxe
Re

40. Force F = F = F
30. Charge on a body = 200 x (- 1-6 x 10~^^) C r2 l2
= -3*2xlO-“C
Fii

1
31. Knowledge based question. 41. From the knowledge of theory, e OC — .

r
32. Field intensity at a point on the axial line of a
Therefore, choice (h) is correct
1 2p
short dipole at distance r, F^ = x-4- T.q 8Q-3Q + SQ-10Q
42. As, <|> = = Zero
4jC6 0 ^ ^0 ^0
Field intensity at a point on the equitorial line at 43. On charging a neutral balloon, its size increases
1 because of repulsive force between the charges
distance r, F^ = 4ice 0 r3
on die surface of balloon.

45. A cube has six faces. The total flux <|> = ^/€q is
divided equally among the six faces. Therefore,
—A = 2 or Fy| = 2 E£ through any one face of the cube,
1 _ g
33. Knowledge based question.
6^0
ELECTROSTATIC CHARGES AND FIELDS 1/69

II. Assertion-Reason lype Questions


10-1X (0-6)2 = 4x10-12
46. The magnitude of charge on proton is same as
or ,2 = 9x10^
that on electron. Accordingto coulomb’s law
^ = 2 X 10-^ C = 2 nC
Thus Assertion is true. Here Reason is true but
p- . As ^] = ^2 “ ^
r2 can not explain the Assertion.
(for protons and electrons) 50. Given, + ^2 = 2 or <?2 = ((2 - 4)
F 1 4 ^2
protons K exelr^ Coulomb force, F =
= 1
4 It e r2
p
electrons
K exetr^ 0

Thus Assertion is true. Here Reason is also true 1 gj (6-^i)


but can not explain the Assertion. 471 € r2

ww
0
47. Here, Reason is true,

I 4^1 1 Q~(i\

FF loo
F_. = 40N ...(O
r2
47ceo6, r2 4tC€ 0
air

ee
1 dF
4^2 For F to be maximum, = 0

ee r
and .Fmed
r2 dq^

rFrF
47C8o€^
dF 1
rur
1
Ml _i [2-2.?,]=0
47te.^0 x8
X
r2 8 ffoordq^ 4jte
0
ks
or
e-2^l = 0 or Q = lq\ ●●● 2/91 = 2
1
YYoou
=5N Thus both Assertion and Reason are true and
ookos

8
Reason is the correct explanation of Assertion.
BBo

Thus Assertion is true and Reason is the correct 51. When a conducting medium is inserted between
explanation of Assertion. two charges then e ^. =««.
re

48. Every mass in the universe is governed by


1
gravitational force of attraction. Hence x^
oouur
ad

F =
gravitational force is dominating force but it is a 471 60^, r2
Y

weakest force as compared to other forces in


nature. So, both Assertion and Reason are false. 1
X^
dndY

= 0.
Re

1
4TC Xoo r2
49. F = 431.
FFini

47t€
0
r2 This both Assertion and Reason are false.
52. Both the Assertion and Reason are false.
(9xl0^)x.?x^ 53. In 1 C charge, number of electrons = 6-25 x lO’^.
or
(10x10-2x10) = The relation given in Reason q = is also false.
(0-6)2

Sol. Electrons are loosely bound to atoms and can be


I. Electric charge and its properties readily exchanged during rubbing. Protons are
1. The electric charge of any body is actually a firmly bound inside the nucleus. TTiey cannot be
easily detached. This is the reason why electric
surplus or deficit of electrons. Why not
charge of any body is just a surplus or deficit of
protons ? electrons.
1/70
4. Fundamental Physics (XII) PKiBi
2. A charge Q is kept in (Hi) When A and B are placed at a large distance,
the inner cavity and a the positive and negative charges on each flow
charge 2 Q is given to and neutralise, so that no charge is left on A or B.
the inner shell. A
4. (a) A comb run through one’s hair attracts
charge 3 Q is given to
small bits of paper. Why ? What happens if
the outermost shell, as
the hair are wet or if it is a rainy day ?
shown in Fig. 1(Q).9.
Find the charges at the (b) Ordinary rubber is an insulator. But the
surfaces A, B and C. special rubber tyres of aircrafts are made
slightly conducting. Why is this necessary ?
Sol. Due to induction, - Q charge is induced on the
inner surface A of inner shell and + Q charge is (c) Vehicles carrying inflammable materials
induced on the outer surface B of inner shell. Thus, usually have metallic ropes touching the
charge on surface A = - Q. ground during motion. Why ?

w
Now charge on surface B = 2 Q + Q = 3 Q (d) A bird perches on a bare high power line,
and nothing happens to the bird. A man
Due to induction, -3 Q charge is induced on the
standing on the ground touches the same line
inner surface of outer shell C and + 3 Q charge is

Flo
induced on the outer surface of outer shell C. and gets a fatal shock. Why ?
Thus, total charge on surface C=3Q + 3Q = 6Q. (NCERT Solved Example)

ee
3. A glass rod rubbed with silk is brought close Sol. (a) This is because the comb gets charged by

Fr
to two uncharged metallic spheres in contact friction. If the hair are wet, or if it is a rainy day,
with each other, inducing charges on them as friction between the hair and the comb reduces.
shown in Fig. l(Q).10(a). The comb does not get charged, and it will not

or
ur
Describe what happens when attract small bits of paper.
sf
(i) the spheres are slightly separated and {b) The special rubber tyres of air crafts are made
slightly conducting so that electricity generated
ok
(ii) the glass rod is subsequently removed and
Yo
finally on account of friction between the tyres and the
Bo

runway goes to earth,


(Hi) the spheres are separated far apart
Sol. A glass rod rubbed with silk acquires positive (c) When a vehicle moves, its body gets charged
re

charge. When this rod is brought close to two on account of friction due to air. The tyres also
uncharged metallic spheres A and B in contact, accumulate the charge on account of friction
ou
ad

negative charge is induced on the left side of between the tyres and the road. The metallic ropes
sphere A and an equal positive charge is induced from the vehicle touching the ground enable the
Y

on the right side of sphere B, Fig. 1(Q).10(a). accumulated charges to flow to earth. This would
otherwise be hazardous to the inflammable
nd

(/) When the spheres are slightly separated, the


Re

materials.
induced charges are shown in Fig. 1(Q).10(h).
Fi

{d) When a bird perches on single bare high power


line, nothing happens to him as no current flows
through his body because the circuit is not
complete. In case of a person standing on ground,
the circuit is completed with the ground through
his body. Current flows through his body resulting
in a fatal shock.

o o 5. An insulating rod carries some net charge, and


a copper sphere is neutral. The rod and the
sphere do not touch. Can there be force of
(ii) As the glass rod is removed subsequently and attraction/repulsion between the two?
separation is too small, the distribution of charges Sol. When the charged rod is brought near the neutral
on A and B remains the same as shown in Fig. sphere, the sphere gets oppositely charged by
KQ).\0(b). induction. Therefore, the sphere and the rod will
ELECTROSTATIC CHARGES AND FIELDS 1/71

attract each other. However, force of repulsion 2. Both forces are proportional to product of
between the two is not possible. charges (or masses) of interacting particles.
3. Both are central forces.
6. Neutral metal objects, especially in industry,
4. Both are conservative forces.
are often coated with electrically charged paint
or powder particles. How do these particles II. Coulomb’s Law
stick on the metal objects?
Sol. The electrically charged paint/powder particles 9. If two objects repel one another, you know both
charge the neutral metal surface oppositely by the carry either positive charge or negative charge.
How would you determine whether these
process of induction. The particles stick on the
chaises are positive or negative?
metal objects on account of force of attraction
between unlike charges.
Sol, To determine the polarity of charge on the two

ooww
objects, we bring one of the objects near a
7. Figure l(Q).ll shows two protons (symbol p)
positively charged glass rod. If the object is
and one electron (symbol e) on a straight line repelled away from the rod, it must be positively
AB. What is the direction of net electrostatic
charged (as like charges repel). However, if the
force on the central proton? object is attracted towards the glass rod, it must

e
be negatively charged (as unlike charges attract).

ree
FIGURE 1(Q).11
B 10. Is coulomb’s law in electrostatics valid in all

rFl
Fre
situations ?

Sol. Coulomb’s law is not applicable in all situations.

rrF
It is appliable under the following conditions :
e
A
I. The electric charges must be at rest.
sffoo
ouur
Sol. In Fig. l(Q).ll, the central proton is repelled by 2. The electric charges must be point charges, i.e.,
oksk
the other proton, along BA ; and it is attracted by size of charges must be smaller than separation
between the charges.
Yo
electron again along BA. Therefore, net
oo

electrostatic force on central proton is along BA. 3. The separation between the charges must be
Y

greater than nuclear size (10“'^ m) because if


BB

8. Give some points of dissimilarity and similarity


distance is < 10~’^ m, strong nuclear forces
between electrostatic forces and gravitational
rre

dominate over the electrostatic forces.


forces.
ouu

III. Electric field


Sol. {a) Dissimilarity
Y
ad

1. Electrostatic forces are between two charges 11. A point charge + Qis placed in the vicinity of a
dY

and gravitational forces are between two conducting surface. Draw the electric field lines
masses.
between the surface and the charge.
innd
Re

2. Electrostatic forces may be attractive or [CBSE 2017 (C)]


repulsive. But gravitational forces are always
Fi

● Sol. The electric field lines are always normal to the


F

attractive. surface of a conductor, both while starting or


3. Electrostatic force between two protons is ending on the conductor. This is shown in Fig.
10^ times stronger than gravitational force I(Q).12.
between them.
FIGURE 1(Q).12
4. Electrostatic forces operate over distances
which are not large. Gravitational forces
operate over very large distances.
5. Electrostatic force depends on the medium
between the two charges. Gravitational force
does not depend upon the medium between
the two masses.

(b) Similarity Conductor


Surface
1. Both the forces obey inverse square law.
1/72 Pn<uU^p,'4. Fundamental Physics (XII)QSI93
12. Draw the pattern of electric field lines when a a potential difference V as shown in Fig.
point charge - g is kept near an uncharged 1(Q).14. The forces on the two protons are
conducting plate. (CBSE 2019) identical. Is this statement true ?
Sol. An explained in the above question, the pattern of
electric field lines will remain the same as shown in FIGURE 1(Q).14
Fig. 1(Q).12 except that all arrows showing the
●B
direction are reversed as the charge is negative.
13. Fig. 1(Q).13 shows the electric field lines for ●A

two point charges separated by a distance.


What are the signs of and q^t Can you Sol. True. This is because field between two parallel
determine the ratio q^lq^ plates is uniform (£), charge on each proton is
same {q). Therefore, force on each proton F-qE
is same.

ww
17. Fig. 1(Q).1S shows tracks of three charged
particles in a uniform electrostaticfield. Give
the signs of the three charges. Which particle

Floo
has the highest charge to mass ratio ?

ee
eer
rFrF
+ +
rur
Sol. As the electric field lines emanate from q^ and
some of them end at ^2- therefore q^ must be a
s ffoor o
osk
positive charge and ^2 must be a negative charge.
YYoou
(a) Suppose two particles have identical curved
oook

As the number of lines leaving q^ is twice the


number of field lines terminating on ^2* therefore, trajectories. Which of the following are
eBB

magnitude of q-^ must be twice the magnitude of necessarily true ?


^2, = (1) they have same charge (u), they have same
r

14. In defining electric field due to a point charge, mass {Hi) the charges have the same sign,
ouur
ad

the test charge has to be vanishingly small. (iv) they have the same ehn ratio.
Y

How this condition can be justified, when we (b) You are given the initial velocity f of a beam
know that charge less than that of electron or particle and the length of the capacitor /. What
nd dY
Re

a proton is not possible. other measurement would enable one to find


elm ?
Sol. This is true that charge less than electron or proton
FFini

is not possible but in macroscopic situations, Sol. The charge on particles 1 and 2 must be negative
source charge is much larger than charge on and charge on particle 3 must be positive. We shall
electron or proton, so the limit ^0 ^ show that charge to mass n tio {elm) is highest in
charge is justified. case of particle 3.
15. Can electric field lines of force form closed If we assume that the three charged particles have
loops ? Give reason for your answer. entered the electric field with the same velocity,
Sol. No. The direction of electric field is from positive then the deflection is proportional to e/m i.e.,
to negative charge. So line of force can be charge to mass ratio of the particle.
regarded starting from positive charge and ending As deflection of particle 3 is maximum, the {e/m)
on a negative charge. That is why, electric lines of this particle must be the highest.
of force can not form closed loops. {a) When two particles have identical curved
16. Two protons A and B are placed inbetween the trajectories, their charges must have the same sign
plates of a parallel plate capacitor charged to and the particles must have the same ehn ratio.
ELECTROSTATIC CHARGES AND FIELDS 1/73

(b) If h is the vertical displacement of particle at Sol. Here, only the charge - 2 Q is enclosed by the
the end of capacitor plates as shown in Fig. sphere of radius 3 a so. By Gauss theorem
1(Q).16, then, from
-2Q
^E =
FIGURE 1(Q).16 ^0
+ + + +
20. (a) Define electric flux. Is it a scalar or a
vector ? A point charge ^ is at a distance dJ2
h directly above the centre of a square of side d,
as shown in Fig. 1(Q).17. Use Gauss’s Law to
M- ●N obtain the expression for the electric flux
through the square.

ww
1
s = ut +-ai^ FIGURE 1(Q).17
2
(i

Flo
I f eEY I
L(±]Ell

e
h=0 + -
2 m A^J J

ree
Fr
2v^h

rF
elm =
e-P
uurr d

As E, I and v are known, by measuring h, we can


calculate (e/m). for
(A) If the point charge is now moved to a
s
kks
The relation shows that for given E, I and v ; distance d from the centre of the square and
Yo
oooo

deflection (A) (e/m). This is what we have stated the side of the square is doubled, explain how
above. the electric flux will be affected.
eB

(CBSE 2018)
IV. Electric dipole
Sol. (a) Electric flux over an area in an electric field
18. A point charge placed at any point on the axis
r

is a measure of th* number of electric field lines


ou
ad

of an electric dipole at some large distance crossing this area. It is a scalar quantity.
YY

experiences a force F. What will be the force


According to Gauss’s theorem, electric flux
acting on the point charge when its distance passing through six faces of the cube is
nndd

from the dipole is doubled.


Re

Sol. As electric field on axial line varies as


^0
Fi

1 F 1 1
The given square is one face of the cube.
^“7
or — «>« — OC

Therefore, electric flux through the square is


So, when distance is doubled, force reduces to
F/8.
6 6e
0
V. Electric Flux and Gauss’s Theorem
(A) When point charge is moved to a distance d
19. Two charges of magnitude - 2 Q and + Q are from the centre of the square and the side of the
located at points (a, 0) and (4 a, 0) respectively. square is doubled there will be no change in
What is the electric flux due to charges through electric flux through the square. It will remain
a sphere of radius ’3 a’ with its centre at the
origin. (CBSE 2013) tlie same, /.e., <]) =
6e 0
1/74 TkeuCeefr'^ Fundamental Physics^ (XH) VOL.I

Q. 21. (a) A conductor A with a cavity as shown in 9 = 0, Le., chaige inside the cavity is zero.
Pig* 1(Q)*18 (a) is given a charge Q. Show Hence the entire charge Q on the conductor
that the entire charge must appear on the must appear on the outer surface of the
outer surface of the conductor, conductor.

(b) Another conductor B vnth chaise q is (b) The conductor B carrying a charge +q
inserted into the cavity keeping B insulated inserted in the cavity induces a charge - q on
from A. Show that the total charge on the the surface of cavity and +q on the outside
outside surface of A is (Q+q), DBIg. 1(Q).18 surface of the conductor A, Fig. 1(Q). 18(£>).
(*)]. As the outer surface of A originally had a chaige
(c) A sensitive instrument is to be shielded Q, the total chaige on it would become (Q+q).
from the strong electrostatic fields in its (c) To shield a sensitive instrument from strong
environment Surest a possible way. electrostatic fields, the instrument must be put

ww
in a metallic cover.

Q. 22. (a) Consider an arbitrary electrostatic field


configuration. A small test charge is placed

Flo
at a null point (i.e. where E = 0) of the

e
configuration. Show that the equilibrium of

reree
the test charge is necessarily unstable,

r FF
(b) Verify this result for the simple
configuration of two charges of the same
uurr
foor magnitude and sign placed a certain distance
apart.
ks s
Sol. (a) To prove this, suppose the equilibrium of
Yoo
ooook

test charge is stable. When the test chaige is


displaced from the null point in any direction,
eBB

it must experience a restoring force towards


the null point. This indicates that there is net
inward flux of electric field through a closed
rr
ouu
ad

surface around the null point. But, according


to Gauss’s law, the electric flux through a
YY

surface not enclosing any charge must be zero.


Hence the equilibrium cannot be stable.
nndd
Re

Sol. (a) We know, the net field inside a charged (b) The middle point of the line joining two
Fii

—>
like charges is a null point. When we displace
conductor is zero, i.e., E =0, inside. Let us a test charge slightly along the line, a restoring
choose a Gaussian surface lying wholly within force tends to bring the test charge back to the
the conductor and enclosing the cavity. Fig. centre. But when we displace the test charge
1(0). 18(a). According to Guass’s law. normal to the line, the net force on the test

sld:=^ = 0 charge takes it further away from the null point.


^ ^0 (v E = 0,inside) Hence the equilibrium cannot be stable.
ELECTROSTATIC CHARGES AND FIELDS 1/75

it B^i ● Very Short Answer


ill
>... ● Short Answer
● Long Answer

VERY SHORT ANSWER QUESTIONS Carrying 1 mark

I. Electric charge and its properties (b) length (c) time (rf) charge (e) charge density
and (/) phase ?
1. What is the cause of charging ?
(Jharkhand Board 2011)
Ans. Mass, length, time and charge density depend
on state of rest or motion of a body. However,
Ans. The cause of charging is actual transfer of charge and phase are invariant.
electrons from one body to the other. 11. Is the force acting between two point charges
2. An isolated conducting sphere is given a and ^2 Itept at some distance in air,

w
positive charge. Does its mass increase, attractive or repulsive when :
decrease or remain the same ?
(0 > 0 (li) q^q2 < 0 (CBSE 2007, 2003)
Ans. Its mass decreases slightly as it loses some Ans. (/) When ^,^2 > 0, the force is repulsive,

FF llowo
electrons.
(/i) When < 0, the force is attractive.
3, What is meant by quantization of charge ?

ree
(Bihar Board 2011) II. Coulomb’s Law

Ans. Charge on any body or particle can be integral 12. Does Coulomb’s law of electric force obey

rF
multiple of charge on an electron (- e), i.e., Newton’s third law of motion ?
q = ± ne, where « = 1, 2, 3, Ans. Yes, it obeys. Forces exerted by two charges on
ur
4. What do you mean by conservation of electric eachother are always equal and opposite.
charge ?
Ans. Conservation of electric charge means that the
for Ffre o
13. Is the electric force between two electrons
greater than the gravitational force between
ks
total charge on an isolated system remains them ? If so, by what factor ?
YYour o
oo

unchanged with time. Ans. Yes, electric force between two electrons is
5. Is the total charge of the universe conserved ? greater than gravitational force between them by
a factor of
Ans. Yes, charge conservation is a global
Bo kBs

phenomenon. 14. Electrostatic forces are much stronger than


r ee

6. Name any two basic properties of electric gravitational forces. Give one example.
oour u

charge. Ans. A charged glass rod can lift a piece of paper


ad

(Bihar Board 2012)


Ans. (0 Quantization of charge («) Conservation of against the gravitational pull of earth on this
Y

charge. piece.
7. What is the value of charge on an electron ? 15. What is the dimensional formula for e q ?
nd
Re

Is a charge less than this value possible ? Ans. [M-* L-3 T^ A^].
(Manipur Board 2011) 16. Write down the value of absolute permittivity
FFind Y i

of free space. (Pb. Board 1996)


Ans. e = - 1-6 X 10“*^ C. No, charge less than this
value does not exist. Ans. Absolute permittivity of free space s 0 = 8-85 X
10-'2 C^N-^ m-2.
8. Give two points of distinction between charge
and mass. 17. What is the relevance of large value of K
(= 81) for water ? (Chhatisgarh Board 2011)
Ans. Charge can be positive, negative or zero. But
Ans. It makes water a great solvent. This is because
mass is a positive quantity. Further, electric
binding force of attraction between oppositely
charge is always conserved. But mass is not
charged ions of the substance in water Incomes
conserved as it can be changed into energy and 1/81 of the force between these ions in air.
vice-versa.
18. Force of attraction between two point charges
9. Can ever photons have a charge ? If not why ? placed at a distance d is F. What distance
Ans. No, photons can never have a charge. This is apart should they be kept in the same medium
because charge cannot exist without rest mass. so that force between them is F/3 ?
10. Which of the following quantities depends on (CBSE 2011)
state of rest or motion of a body ; (a) mass
Ans. As F« 1/r^ New distance = y/3r .
1/76 ‘P'uidee^ Fundamental Physics (XII)
19. In coulomb’s law, on what factors does the 28. Why is electric Held intensity inside a charged
value of electrostatic force constant K conductor zero ?
depend ? {Pb. Board 2011) Ans. This is because there are no electric lines of force
Ans. The value of K depends on nature of medium inside the body of a conductor.
separating the charges and on the system of units. 29. Two point charges of 3 ilC each are 100 cm
apart At what point on the line joining the
m. Superposition Principle charges will the electric intensity be zero ?
Ans. At the centre of the two charges.
20. When two charges and qi are kept at some
30. What is nature of symmetry of Held due to a
distance apart, force acting between these
point chaise ?
charges is F. If a third charge q^ is placed
Ans. The field due to point charge has spherical
quite close to ^2* will happen to the force
symmetry.
between q-^ and q^ ? 31. When is an electric line of force straight ?

ww
Ans. As force b7etween any two charges doesn’t Ans. Electric line of force is straight in the field of a
depend upon the presence of any other charge, single charge.
so force between F, and F2 will remain F only. 32. When is an electric line of force curved ?

Flo
21. Does the coulomb force that one charge exerts Ans. Electric line of force is curved in the field of

ee
on another, change if other charges are more than one point charges.
brought nearby ?

rere
V. Electric dipole

rFF
Ans. No, the coulomb force due to one charge on
another charge is not changed. 33. Define electric dipole moment. Write its SI
(Jharkhand Board 2011 ;
uur r
unit.
IV. Eletric field
foor Raj. Board 2011 ; CBSE 2011)
Ans. Electric dipole moment is equal to product of
ks s
22. A point charge q is placed at the origin. How
magnitude of either charge and distance between
Yoo
does the electric field due to the charge vary
ook

them. Its SI unit is C-m.


with distance r from the origin ?
BBo

Ans. Electric field varies inversely as square of 34. What is an ideal dipole ?
distance from the point charge. Ans. For a given dipole moment I p I ~q{2d),
re

23. Write the dimensional formula of electric when the charge q gets larger and larger, while
ouur

field. the separation (2 a) between two charges gets


ad

Ans. smaller and smaller, we obtain an ideal dipole.


YY

Thus, an ideal dipole has almost no size.


24. A proton is placed in a uniform electric field
35. In which orientation, a dipole placed in a
directed along the positive .r-axis. In which
nndd
Re

direction will it tend to move ? (CBSE 2011) uniform field is in (i) stable (n) unstable
equilibrium ? (CBSE 2010)
Fii

Ans. As proton is positively charged, it will tend to


move along + x axis, i.e., along the direction of Ans. If 0 is angle between p and E , then 6 = 0® for
electric field. stable equilibrium, 9 = 180® for unstable
25. Name any four vector fields. equilibrium.
Ans. Electric field, magnetic field, gravitational field 36. How does a torque affect the dipole in an
electric field ?
and flow field (of a liquid) are vector fields.
26. What is the use of the concept of electric field Ans. Torque tries to align the dipole along the field.
intensity ? 37. Which rule gives you the direction of torque ?
Ans. It enables us to find force on any charge in the Ans. The direction of torque is given by right hand
screw rule.
field as F = qE 38. What happens when an electric dipole is held
27. Name the physical quantity whose SI unit is in a non uniform electric field ?
NC-* ? (Pb. Board 2011) Ans. It experiences some net force and some net
Ans. Electric field intensity. torque.
ELECTROSTATIC CHARGES AND FIELDS 1/77

39. At what points, dipole field intensity is 46. Fig. 1(Q).19 shows three point charges + 2g,
parallel to the line joining the charges ? - q and + 39. What is the electricflux due to
Ans. At any point on axial line or equatorial line of this configuration through the surface S ?
(CBSE 2010)
dipole.
40. When does an electric dipole placed in a non-
uniform electric fleld experience a zero torque
but non-zero force.

Ans. When the dipole is placed parallel to the non-


uniform electric field.

41. Will an electric dipole have translational


motion when placed in a non-uniform electric
2q-q q
field ? Give reason for your answer. Ans. =

w
Ans. Yes, in a non-uniform electric field, an electric ^0 ^0
dipole experiences unequal forces at its ends. 47. What is the relation between electric intensity
and electric flux ?
The two forces, add up to give a resultant force,

Flo
which gives a translatory motion to the dipole. Ans. The surface integral of electric field intensity

e
over a closed surface in free space (/.e., electric
VI. Electric Flux and Gauss’s Theorem

ree
flux) is 1/€q times the total charge q enclosed

FFr
42. Name the principle which is mathematical
by the surface, i.e., o E.ds = q! .€ 0
equivalent of coulomb’s law and super
urr
position principle. 48. What is the number of electric lines of force
Ans. Gauss’s theorem of electrostatics.
43. If the radius of the Gaussian surface enclosing for
that radiate outwards from one coulomb of
charge in vacuum ?
kkss
a charge q is halved, how does the electric flux Ans. From ({) = —, when ^ = I C,
Yo
through the Gaussian surface change ?
ooo

^0
(CBSE 2008)
1
B

= 1-13x10^*
q 8-85x10"*2
Ans. As (|) = —, so flux does not depend upon the
re

^0 49. What is the SI unit of surface integral of


radius of Gaussian surface, it will remain electric field ?
ou
ad

unchanged. Ans. N C~^ or volt-metre, as the surface integral


YY

44. A charge q is placed at the centre of a cube of of electric field represents electric flux.
side I what is the electric flux passing through 50. An electric dipole of dipole moment 20 x
ndd

two opposite faces of the cube ? (CBSE 2012) 10"^ C /«is enclosed by a closed surface. What
Re

is the net electric flux coming out of the


Fi

surface ?
Ans. Flux through each face = —
6e 0 Ans. Net electric flux coming out of closed surface is
zero, because net charge on electric dipole is
.-. flux thorugh two opposite faces zero.

a. ^ 9 51. Two plane sheets of charge densities -i- a and


6e
0
6e
0
6e 3e - a are kept in air as shown in Fig. 1(Q).20.
0 0
What are the electric fleld intensities at points
45. Two concentric spherical shells of radii R and A and B ? (Kerala Board 2012)
2 R are given charges gj and Q2 respectively. FIGURE 1(Q).20
The surface charge densities of the outer ● A
surfaces are equal. Determine the ratio +0

(CBSE 2013)
● B
-<J

Qi _ 4nR^a
Ans. Here. = 1:4
47t(2/?)2c Ans. At A, £ = 0, and At B, E-—
^2
^0
1/78 Fundamental Physics (XII)B&!9]

SHORT ANSWER QUESTIONS Carrying 2 marks

I. Electric charge and its properties 5. An attractive force of 5 TV is acting between


two charges of + 2 p. C and -2]iC placed at
1. A copper sphere of mass 2 g contains nearly some distance. If the charges are mutually
2 X 10^ atoms. The charge on the nucleus of touched and placed again at the same
each atom is 29 e. What fraction of the
distance, what will be the new force, between
electrons must be removed from the sphere them ?
to give it a charge of + 2 p C ? Ans. On touching, charges neutralise. Therefore,
Ans. Total number of electrons in the sphere F = 0
= 29x2x1022
6. Two point charges of + 2 p C and + 6 p. C

ooww
No. of electrons removed
repel each other with a force of 12 N. If each
is given an additional charge of - 4 p C, what
q 2xl0~^ = 1-25x10*2 will be the new force ?
~ e~ 1-6x10-'^
Ans. ^, = + 2 p C, ^2 ” ^ ^ ^

e
Fraction of electrons removed
g,' = + 2- 4 = -2pC;i?2' = + 6- 4 = 2pC,

re
F' = ?
1-25x10*2

rFFl
ree
= 2-16 x

F
29x2x1022 F'^ (g,)(<?2> _ (-2) (2) _ 1
F “ ^,^2 (2) (6) 3

rF
2. Give four properties of electric charges.
Ans. (/) Like charges repel and unlike charges attract -F _ -12
F' =

fsfoor
= - 4 iV (attractive)
ouur
each other,
3 " 3
(i/) Charge is quantized. 7. What is the importance of expressing
kosk
(Hi) Charge is conserved, coulomb’s law in vector form ?
(iv) Charge on a body is not affected by its motion.
Yo
A A
oo

Ans. As in vector form


Y

II. Coulomb's Law


BB

It shows two charges exert equal and opposite


3. Dielectric constant of a medium is unity. What forces on each other. So Newton’s third law is
rre

will be its permittivity ? obeyed.


Ans. We know that dielectric constant of a medium is
ouu
Y

As Coulombian force acts along and F21


ad

i.e., along the line joining the centres of two


dY

0
charges, so they are central forces.
G =/:€()= 1 X 8-854 X 10-*2
innd

= S-854 X 10"*2 C2 N-* m-2 111. Superposition Principle


Re

4. Two small balls having equal positive charge 8. Two identical metallic spheres A and B, each
FFi

q coulomb are suspended by two insulating carrying a charge q repel each other with a
strings of equal length I metre from a hook force F. A third metallic unchargedsphere C
fixed to a stand. The whole set up is taken in of the same size is made to touch the spheres
a satellite into space where there is no gravity. A and B alternately and then removed away.
What is the angle between the two strings and What is the force of repulsion between A and
the tension in each string ? B ?
Ans. In a satellite, there is condition of weightlessness. Ans. Let r be the distance between two spheres A and
Therefore, mg - 0. On account of electrostatic B. Then force of repulsion between spheres A
force of repulsion between the balls, the strings and B is
would become horizontal. Therefore, angle 1 gxg
F =
between the strings = 180®. 47t6 r2 -(1)
0
Also, tension in each string = force of repulsion
When uncharged sphere C of same size is placed
1
T = in contact with sphere A, Fig. l((2).21(a), each
2^ sphere acquires a charge
4kGq (21)
1/79
ELECTROSTATIC CHARGES AND FIELDS

FIGURE 1(Q).21
V. Electric dipole
Charge = 0 gl2 ql2 12. What does (^, + 92) = ® signify ?
® * © * @© © Ans. + ^2 = ^1 = “ ^2- Such a system of point
charges is called an electric dipole. It is a pair of
q ql2 3^/4 ZqlA equal and opposite point charges separated by a
small distance.
13. An electric dipole is placed at rest in a uniform
ql2 ZqlA electric field, and released. How will it move ?
0 ©=B O Ans. A torque will develop and align the electric
A
dipole in the direction of the electric field, if the
dipole is not aligned already. The dipole shall
^+0 _ q not move as net force on the dipole is zero.
2 2 14. Define the term electric dipole momenL Is it

w
When the sphere C is placed in contact with scalar or vector ?
sphere S, Fig. l(Q).21(h), the charge is Ans. Electric dipole moment (p) is the product of
redistributed equally between B and C. Thus, either charge (± q) and the distance (2 a) between

Flo
charge on B or C is the charges, i.e.,p = q (2 a). It is a vector quantity,
1 <? 3^ directed from - ^ to + ^.

ee
- ^+- 15. What is the direction of field intensity at a
2 2 4

Fr
point (i) on axial line of dipole and (u) on
Refer to Fig. l(Q).21(c), Net force of repulsion equatorial line of dipole ?
between spheres A and B is Ans. For a point on the axial line of dipole, the

for
ur
1 (^/2)(3^/4) 3 ^ direction of electric field intensity £ is along a
F' = ;; = — r
4 Tie
0
8 line parallel to the axis of dipole directed along
ks
—»
Yo
the direction of dipole moment P ■
rV. Electric field
oo

For a point on the equatorial line of dipole,


9. Why no two electric lines of force can
—>
B

the direction of electric field £ is along a line


intersect each other ? (Hr. Board 2007)
parallel to the axis of dipole directed opposite
re

Ans. This is because at the point of intersection, we —»

can draw two tangents to the two lines of force. to the direction of dipole moment P ■
ou
ad

This would mean two directions of electric field 16. What is the nature of symmetry of the electric
intensity at the point of intersection, which is field due to (/) point charge and {ii) electric
Y

not possible. dipole ?


10. Give two properties of electric field lines. Ans. The electric field due to a point charge has
nd
Re

Ans. 1. Electric field lines are continuous curves. They spherical symmetry with point charge at the
start from a positively charged body and end at centre. It is so because, at equal distances from
Fi

a negatively charged body. They are continuous the point charge, field intensity is equal.
but do not form closed loops as there are no lines The electric field due to a dipole has a
of force inside the charged body. cylindrical symmetry. The axis of the cylinder
2. Tangent to the electric field line at any point passes through the dipole axis. It is so because,
gives the direction of electric field intensity at the electric field due to dipole will be same at
that point. every point on the surface of a right circular
11. A charged particle is free to move in an cylinder with electric dipole as the axis.
electric field. Will it always move along an 17. When an electric dipole is suspended in a
electric line of force ? uniform electric field, then under what con
Ans. If charged particle was initially at rest, it will ditions the dipole is in (z) stable equilibrium
and (ii) unstable equilibrium.
move along the direction of electric field. If
initial velocity of the charged particle makes a Ans. The dipole in an electric field will be in stable
certain angle with electric field, then the charged equilibrium if the following conditions are
satisfied :
particle will not move along the line of force.
1/80
Fundamental Physics (XII) VOL.I

(/) 'fhe resultant force on dipole is zero, i.e., there Ans. Electric flux over an area in an electric field
is no translatory motion of dipole. («) The torque represents the total number of electric lines of
on dipole is zero, i.e., there is no rotatory motion force crossing this area. SI units of electric flux
of dipole. (Hi) The potential energy of dipole is are N m^ C"^
minimum.

It will be so when dipole is aligned along the When R is reduced to half, electric flux through
direction of electric field. the surface remains the same. This is because
The dipole will be in unstable equilibrium if electric flux through the surface depends only
(0 the resultant force on dipole is zero, (ii) the on charge enclosed.
torque on the dipole is zero. (Hi) the potential 23. Draw a graph to show the variation of electric
energy of dipole is maximum. intensity E with perpendicular distance r from
It will be so when dipole is aligned opposite to an infinite line of charge density X C/m. Find
the direction of electric field.
work done in bringing a charge q from
18. Show that when an electric dipole is placed

ww
perpendicular distance rj to T2 (r2 > rj).
in a uniform electric field E , potential energy (CBSE 2018)
£/is given by U =-^-E. Ans. As is known, electric intensity E at a

Flo
Ans. Refer to Art. 1.36. perpendicular distance r from an infinite line of
19. An electric dipole is held at an angle 6 in a charge density X C/m is

ee
uniform electric field £. Will there be any

rere
X
(/) net translating force («) torque acting on £ =

r FF
it ? Explain. 27ceQ r
Ans. (0 There will be no net translating force. The variation of E with r is as shown in Fig.
uurr
(ii) Torque on the diple ,● x= pE sin 6. It will
foor
align the dipole along the field. Once the dipole
is aligned, x = 0. For details, refer to Art. 1.35.
1(Q).22. Work done in bringing charge q from
distance rj to T2 is
ks s
20. Charge is inside the Gaussian surface ;
Yoo
ooook

charge ^2 just outside the surface. Does the


electric flux through the surface depend on
eBB

qj ? Does it depend on q2 ? Explain.


Ans. Electric flux through the surface depends on
(as q^ is inside the Gaussian surface). But it does
rr
ouu

not depend on ^2- H being outside Gaussian


ad

surface. ^2 does not contribute to electric flux.


YY

21. Using Gauss’s law, derive an expression for


the electric field intensity at any point near a X
dW~I.Fdr = qEdr = q dr
nndd

uniformly charged thin wire of charge/length


Re

2tc€o r
= X C/m. (CBSE 2008)
^2
Fii

Ans. Refer to Art. 1.35. Xq


W = dr
22. Define electric flux. Write its SI unit. A charge 27C€«r
0
q is enclosed by a spherical surface of radius
R. If the radius is reduced to half, how would
Xq Xq I
the electric flux through the surface W =
[logr]Jn = log
change ? 2ne 0
(CBSE 2009, 2018 (C) ] 0 1

SHORT ANSWER QUESTIONS Carrying 3 marks

1. What do you understand by (a) quantisation of 3. Define SI unit of charge, electromagnetic unit
charge and (b) conservation of charge. Explain of charge and electrostatic unit of charge. State
each with illustrations.
the relation between them. [Art 1.14]
{HP Board 2013) [Arts. 1.8 and 1.10]
4. What is meant by Dielectric constant and
2. Explain Coulomb’s law in vector form and discuss
absolute electrical permittivity of medium. How
the importance of this study. [Art. 1.13]
1/81
ELECTROSTATIC CHARGES AND FIELDS

are they related ? State SI units of dielectric 8. Give two properties of electric lines of force.
constant and absolute electrical permittivity. Sketch them for an isolated positive point charge
[ArL 1.15] and an electric dipole.
5. Explain super position principle. Apply it to (Kerala B4)ard 2012) [Art. 1.26]
obtain total force on a point charge due to n 9. State and prove Gauss’s law in electrostatics.
discrete charges. [Art. 1.16] (Bihar Board 2012, Jharkhand Board 2011,
6. Establish relation between electric field strength J & K Board 2011)
and force. [Art 1.20] [Art 1.39]
7. Explain what is meant by an electric line of 10. Slate Gauss’s Theorem in electrostatics and
force ? Give its two important properties ? deduce coulomb’s law from Gauss’s theorem.
(Raj. Board 2011)
(Pb. Board 2011) [Art 1.41]
[Arts. 1.26 & 1.27]

w
LONG ANSWER QUESTIONS Carrying 5 or more marks ’

1. How will you show that there are two kinds of 7. Derive an expression for force and torque acting

Flo
charges ? Explain with proper illustrations. on an electric dipole in a uniform two dimensio
nal electric field. In which situation, torque on

reeee
[Art 1.3]
the dipole is (i) maximum and (/i) minimum.
2. What do you understand by the phenomenon of (CBSE 2017, HI* Board 2013. WB Board 2012)

FFr
charging by induction. Give some illustrations [Art 135]
to support your answer. [Art. 1.7]
8. Derive an expression for potential energy of an
3. What is meant by continuous charge
for
electric dipole in a uniform electric field. In
ur
distribution ? Apply superposition principle to which situation, the potential energy of dipole
obtain total force on a point charge due to is (0 maximum and (t’l) minimum ?
kkss
(0 continuous line distribution of charge (Raj. Board 2011) [Art 136]
Yo
oo

(ii) continuous surface distribution of charge and 9. Using Gauss’s theorem, derive an expression
(ill) continuous volume distribution of charge. for electric field intensity at a point due to
eB

(0 a line of charge, [Art. 1.42]


[Art. 1.17 & 1.18]
(/i) a uniformly charged thin spherical shell.
4. Derive an expression for electric field intensity
r

(CBSE 2011, 2013, CBSE (F) 2017,


ou

at a point due to
ad

Jharkhand Board 2012, Kerala Board 2012.


(i) A point charge [Art. 1.21] Manipur Board 2011, Pb. Board 2011)
YY

(ii) A group of charges [Art. 1.22] [Art 1.43]


(ill) Continuous charge distribution {Hi) a charged solid sphere.
ndd
Re

[Art 1. 23] (Chhatisgarh Board 2011) [Art. 1.44]


Fi

5. Derive general expression for rectangular (iV) an infinite plane sheet of charge.
components of electric intensity due to point (CBSE 2017, 2016. Uttarakhand Board 2012,
charge in space. [Art 1. 24] Manipur Board 2012. Raj. Board 2011)
[Art. 1.45]
6. Derive an expression for dipole field intensity
at any point on (0 axial line of dipole (v) two parallel sheets of charge with charge
densities, o and - o.
(CBSE 2017, Raj. Board 2012) [Art 1. 31]
(CBSE 2017, 2009) [Art. 1.46]
(li) equatorial line of dipole 10. State Gauss’s Law. Using this law, obtain the
(CBSE 2019, 2013, MP Board 2011) expression for the electric field due to an
[Art 1. 32] infinitely long straight conductor of linear
(ill) at any point [Art 1.33] charge density X. [CBSE Sample Paper
2019-20, CBSE 2018, 2017 (C)]
(iv) axis of uniformly charged irng
[Arts. 1.39 and 1.42]
[Art 1.34]
1/82
Fundamental Physics (XII) VOL.I

CASE 1. The vast amount of charge in an object Read the above passage carefully and answer
is usually hidden as the object contains equal amounts the following very short answer and short
of positive charge and negative charge. When a glass answer questions.
rod is rubbed with a piece of silk, some electrons from
5. A cube of side of 0-1 m encloses charges of 3 pC
glass are transferred to silk piece. Therefore, glass rod and - 5 pC. What is electric flux from any one
acquires positive charge and silk piece acquires an equal face of the cube ?
negative charge. The charge acquired by a body is an 6. In the above question, a charge of + 2 pC is held
integral multiple of basic unit of charge of an electron/ outside the cube. What is the electric flux from
proton represented by (e), i.e., q = ne. This property is any one surface of the cube ?
called quantisation of charge. Further, total charge on 7. Point charges of 1 pC, 2 pC, 3 pC and 4 pC are

ww
an isolated system always remains constant or held outside a conducting sphere and another
conserved. This property is called conservation of
charge of 10 pC is held inside the sphere. What
charge. We can compare the role of charge to the role is the total electric flux from the sphere ?
of mass in gravitation.

Flo
8. Algebraic sum of point charges present inside a
Read the above paragraph carefully and hollow sphere is zero. A point charge of- 10 pC

e
ree
answer the following very short and short is now held outside the hollow sphere. The total
an.swer questions : electric flux from the sphere will be.

Fr
rF
1. Which of the following is not an insulator ? glass,
CASE 3. From the knowledge of electric intensity
uurr
ebonite, paper and copper ?
2. How is the vast amount of charge in every object
hidden ? for
(£) at any point ( r ), we can calculate the magnitude
and direction of force experienced by any charge
s
held at that point
kks
3. What are the two basic properties of electric
Yo
charge ?
oooo

4. What is the charge acquired by a body from which f {7) = q^E{7)


eB

one million electrons have been removed ? This is the physical significance of electric field.
As against the electric lines of force which are
CASE 2. According to Gauss’s theorem in
continuous but not closed loops, magnetic lines of force
r

electrostatics, total normal electric flux over a closed


ou

are continuous, endless closed loops.


ad

surface S in vacuum is l/Sg times the total charge Q


YY

contained inside 5, i.e.. An electric dipole consists of a pair of equal and


opposite point charges separated by some small
nndd

-* ->
Q distance. The dipole moment (p) is a measure of
Re

E-ds = —
s ^0
strength of electric dipole ~p =q{l7). The direction
Fi

The charges enclosed may be point charges or

even continuous charge distributions. Further, location of (p) is from negative charge to positive charge. On
of Q inside the closed surface does not affect the electric axial line of electric dipole
flux.
I p \2r
An imaginary closed surface at every point of l£l =

which electric intensity is same is called Gaussian 47C6Q(r2-a2):


surface. It may be an empty space or it may be
embedded in a solid body. The direction of E is opposite to direction of
If the medium surrounding the charges has a p . On equatorial line of electric dipole,
dielectric constant K, then

Q 1 £1 = I p I
Ke 0 4KEQ(r^
ELECTROSTATIC CHARGES AND FIELDS 1/83

Read the above passage carefully and answer


-*

11. Electric field £ on axial line of electric dipole


the following very short and short questions. -»

9. A charge is held in an electric field of intensity is £ oc r . What is the value of n ?

£ . What is the force on the charge ? 12. Give the magnitude and direction of electric
intensity due to an electric dipole at any point on
10. The algebraic sum of charges on an electric dipole
equatorial line of electric dipole.
is zero. Why does it have an electric field ?

ANSWERS

1. Copper is not an insulator. 7. As charges outside the sphere do not affect the
electric flux, therefore.
2. The vast amount of charge in an object is usually
hidden as the object contains equal amounts of q _ lOxlQ-^ _ 10"^

w
positive charge and negative charge. The object Yr -
^ e0 ^0 ^0
is said to be electrically neutral, i.e., it contains
no charge. 8. As algebraic sum of charges inside a hollow

Flo
sphere is zero, = 0. Further, presence of a
3. Two basic properties of electric charge are : charge of- 10 pC outside the hollow sphere does

eeee
(/■) Quantisation of charge not affect the electric flux through the sphere.
(j7) Conservation of charge. Therefore, = 0.

Fr
4. Q=ne=\0^x 1-6 x 10^*® C = 1-6 x 10"*^ C
9. £ = ^,£
As electrons have been removed, charge on the

for
ur
body is positive.
10. The electric dipole has an electric field because
charges of the dipole do not lie at the same point.
5. A cube has six faces. According to Gauss’s
11. Electric field on axial line of electric dipole is
ks
theorem in electrostatics, from one face of the
Yo
cube. ■ . Therefore, n = - 3.
proportional to
oo

^^__l^(3-5)pC
B

'£ -
6 s.,0 6
12. On equatorial line of electric dipole,
^0
re

p
l£l =
-2x10*^ 1x10-^
47l£o(r2+a2)3^2
ou
ad

6€
0
3i 0
£ is in a direction opposite to the direction of
Y

6. Any charge present outside the cube does not


affect the electric flux from the cube. P ■
nd
Re

CASE-BASED MCQs AND ASSERTION-REASON QUESTIONS


Fi

CASE 1. According to Coulomb’s law, the force Sq is called absolute electric permittivity of free
of interaction between any two point charges is directly space.

proportional to the product of charges and inversely Based on the above paragraph, answer
proportional to the square of distance between them. questions no. 1 to 4 :
The force acts always along the line joining the two
1. Units of Eq are
charges.
(a) Nm-2
11^2 > (b) C-2 Nm2
(c) N-* m -2
Here, k is electrostatic force constant.
(d) Nm-
In CGS system, = 1. In SI, = 9 x 10^ Nm^
2. Value of absolute electrical permittivity of free
1
k = , where space is
4ne
0
1/84
Fundamental Physics (XII) VOL.I

(a) 8-85 X 10-^2 q2^-i ^-2


(6) 8-85 X 10^2 q2 j^-i ^-2 dipole moment is p = ^(2a). Directionof p is from
(c) 9 X 10^ C2 N-' m-2 negative charge to positive charge, p is measured in
(cO 9 X 10-9 C2 j^-l j^-2 Coulomb-meter.
For Question No. 3 and 4, we have given two Based on the above paragraph, answer
statements each, one labelled as Assertion (A) questions no. 5 to 8 :
and other labelled as Reason (R).
5. Can two electric lines of force intersect each
Choose the correct option out of the four options other ?
given below:
(a) No, never (b) Yes, always
(a) Both A and R are true and R is correct
(c) sometimes (d) cannot predict
explanation of A.
(b) Both A and R are true and R is not the correct 6. Dipole moment p is equal to

ww
explanation of A.
(a) q (2 a) ib) q/2 a
(c) A is true but R is false.

Floo
(d) A is false and R is also false. 2a
(c) (d) none of these
3. Assertion. The value of electrostatic force

ee
constant depends on nature of medium separating
the charges and also on the system of units. For Question No. 7 and 8, we have given two

reer
statements each, one labelled as Assertion (A)

rFF
Reason. In SI, = 9 x 10^ Nm^ C"2. In CGS
and other labelled as Reason (R).
system k~l.
Choose the correct option out of the four options
uur r
4. Assertion. Coulomb’s law corresponds to
Newton’s law of gravitation. ffoor
given below:
(a) Both A and R are true and R is correct
sks
Reason. According to Newton’s law of
explanation of A.
YYoo
gravitation
ooko

(b) Both A and R are true but R is not a correct


explanation of A.
eBB

r2 (c) A is true, but R is false.


The symbols have standard meanings. (d) A is false and R is also false.
r

7. Assertion. Electric dipole moment is a scalar.


ouur

CASE 2. An electric line of force is a path, straight


ad

or curved, such that tangent to the path at any point


Y

gives us electric field intensity at that point. No two Reason. p = q(2a).


electric lines of force can intersect each other.
dY

8. Assertion. No two electric lines of force can


Re

Two equal and opposite charges separated by a intersect each other.


nnd

small distance are said to form an electric dipole. Its Reason. Because they lie far apart.
FFii

ANSWERS

l.(c) 2. (a) 3. (a) 4. (a) 5. (a) 6. (a) 7. (d) 8. (c)

HINTS/EXPLANATIONS For Difficult Questions

1. From F = 3. Both, the Assertion and Reason are true, and


4tc6«0 r2 Reason is correct explanation of the Assertion.
4. Both, the assertion and Reason are true and
C2 Reason is correct explanation of the Assertion.
^2
^0 =
4tu Fr^ Nm^ 7. As p = 9 X (2 a ). Hence Reason is false. Here
2a
= C2 N-^ m-2. is also a vector, therefore Electric dipole
moment is a vector. Thus Assertion is also false,
2. 6(j = 8-85x 10-‘2c2n-‘ m"2 8. Assertion is true, but the Reason is false.
1/85
ELECTROSTATIC CHARGES AND FIELDS

negligible mass. If 0 is the angle which each


TYPE I. QUANTIZATION OF CHARGE string makes with the vertical when equilibrium
has been reached, show that
1. A person combs his hair on a dry day. The comb
causes 10^ electrons to leave the person’s hair = A mg sin^ 0 tan 0 (4 n e q)
and stick to the comb. Calculate the charge the 12. Two panicles, each having a mass of 5 g and
comb carries. [Ans. - 1-6 X 1Q3 C] charge 10'^ C stay in limiting equilibrium on a
2. Estimate the number of free electrons in 36 g of horizontal table with a separation of 10 cm
water and the negative charge possessed by them. between them. The coefficient of friction between
Given : Avogadro’s number = 6-023 x 10^^ and each particle and the table is the same. Find the
molecular weight of water = 18. value of this coefficient. [Ans. = 0-18]

ww
[Ans. 12-046 x - 1-93 x 10® C] 13. Two point charges ^2 = 3 x 10~® C and =5x
3. What is the total charge on 75 0 kg of electrons ? 10“® C are located at (3, 5, 1) and (1, 3, 2) m.
[Ans. - 1-33 x 10*3 q
Find Fj2 and Fjj using vector form of

Flo
4. How many mega coulombs of positive (or nega Coulomb’s law. Also, find their magnitude.
tive) charge are in 1 -00 mole of neutral molecular

ee
hydrogen gas (H2) ? [Ans. 0-1927 MC] -*

[Ans. fj2 = -^21 = -(5xlO"3){2?+2/-fc)N

rere
5. Calculate the total positive or negative charge on

rFF
a 3-11 g copper penny. Given, Avogadro’s number = 1-5 X 10~3 N, in magnitude]
= 6-023 X 10^3 per gram mole; for copper, atomic
uur r
14. Two small charged spheres contain charges + <?,
charge - 29 and atomic mass - 63-5.
[Ans. ± 1-37 X C]
foor and + ^2 respectively. A charge dq is removed
from sphere carrying charge q^ and is transferred
sks
to the other. Find charge on each sphere for
Yoo
TYPE II. COULOMB’S LAW
maximum electric force between them.
oko

6. A charge of magnitude Q is divided into two parts


BBo

1
q and {Q - q) such that the two parts exert Ans. -1-92)
re

maximum force on each other. Calculate the ratio


Q/q- [Ans. 2]
ouur

TYPE m. SUPERPOSITION PRINCIPLE


7. Two identical metal spheres A and B have equal
ad

and similar charges. They repel each other with


15. Three point charges of + 3 ^iC, - 5 p,C and
YY

a force 103 N, when they are placed 10 cm apart


in a medium of dielectric constant 7. Determine - 5 )iC are kept on the vertices of an equilateral
nndd

the charge on each sphere.[Ans. 28-3 x 10^ C] triangle of side 20 cm as shown in Fig. 1(Q).23.
Re

8. Calculate the Coulomb force between two alpha


What should be the sign and magnitude of charge
FFii

particles separated by a distance of 3-2 x 10"*3 m to be placed at the mid point (Af) of side BC of
m air. [Ans. 90 N, repulsive)] triangle ABC so that charge A remains in
equilibrium ?
9. What equal charges would have to be placed on
earth and moon to neutralize their gravitational
attraction ? Given, mass of earth = 10^3 kg, mass
of moon = 10^3 kg. [Ans. 8-6 X 10*3 cj
10. The electrostatic force of repulsion between two
positively charged ions carrying equal charges is
3-7 X 10“^ N, when they are separated by a
distance of 5 A. How many electrons are missing
from each ion ? [Ans. 2]
11. Two small spheres each of mass ‘m’ kg and charge
q coulomb are suspended from a point by
[Ans. 6-49 |xC, positive]
insulating threads each of I metre length, but of
1/86
"Pn^ideefi. ^ Fundamental Physics (XII) VOL.I

16. Four equal point charges each 16 are placed 24. Two electric charges + ^ and 4 ^ are placed at a
on four corners of a square of side 0-4 m.
distance 6a apart on a horizontal plane. Find the
Calculate force on any one of the changes. position of the point on the line joining the two
[Ans. 27-56 N] charges where the electric field is zero.
17. Two opposite comers of a square carry Q charge [Ans. 2 a from + q]
each and the other two opposite comers of the 25. In Fig. 1(Q).25, the electric field lines on the left
same square carry q charge each. If the resultant have twice the separation of those on the right,
force on q is zero, how are Q and q related ? (a) If the magnitude of the field at A is 40 N/C,
what force acts on a proton at A ? {b) What is the
[Ans. q = -2V2Q] magnitude of the field at ?
18. Equal charges each of 20 C are placed at x = 0,
FIGURE 1(Q).25
2, 4, 8, 16 cm on X-axis. Find the force
experienced by the charge at x = 2 cm.

w
[Ans. 1-2 X 10^ N]
19. Charges = 1-5 mC, ^2 = 0-2 mC and iA
= - 0-5 mC, are placed at points A, B, C

Flo
respectively as shown in Fig. 1(Q).24. If rj =
\-2m and T2 = 0-6 m, calculate magnitude of

ee
resultant force on ^2- [Ans. 3-1 X 10^ N] [Ans. (fl) 6-4 X 10“^* N {b) 20 N/C|

Fr
26. Eight identical point charges of q coulomb each
are placed at the comers of a cube of each side 0-1
m. Calculate electric field at the centre of the cube.

for
ur
Calculate the field at the centre when one of the
comer charges is removed.
[Ans. Zero ; (1-2 x 10*2 towards the
ks
Yo
corner without charge]
oo

27. A charge of 4 x 10“^ C is distributed uniformly


B

TYPE IV. ELECTRIC FIELD over the circumference of a conducting ring of


radius 0-3 m. Calculate the field intensity at a point
re

AND RELATION BETWEEN


ELECTRIC INTENSITY AND FORCE on the axis of the ring at 0-4 m from its centre,
and also at the centre. [Ans. 115-2 N/C ; zero]
ou
ad

20. A small ball of paper has mass 9 x 10“^ kg and TYPEV. ELECTRIC DIPOLE,
Y

carries a charge of 5 [i C. When it is held over DIPOLE MOMENT, DIPOLE FIELD


another charged ball of paper at a distance of
nd

2 cm above it, the two balls stay in equilibrium. 28. Calculate the magnitude of the force, due to an
Re

What is the charge on the second ball ? electric dipole of dipole moment 3-6 x lO”^^ C-m,
Fi

[Ans. 7-84 x 10"*2 C] on an electron 25 nm from the centre of the dipole,


21. A water droplet of radius 1 micron in Millikan oil along the dipole axis. Assume that this distance is
drop apparatus is first held stationary under the large relative to the dipole’s charge separation.
influence of an electric field of intensity [Ans. 6-6 X 10-*S N]
5-1 X 10‘*NC“^How many excess electrons does 29. Charge = + 6-0 n C is on y-axis at y = + 3 cm
it carry ? Take e = 1-6 x 10~*^C, g = 9-8 ms“2 and and charge ^2 = - 6 0 n C is on y-axis at y = - 3 cm.
density of water = 10^ kg m“2. [Ans. 5] Calculate force on a test charge qQ= 2 n C placed
22. A particle of mass m and charge q is thrown at a on X-axis at .x = 4 cm. "
[Ans. - 51-8 j^ fiN]
speed u against a uniform electric field £. How
30. Two charges -1- ^ and - ^ are located at points
much distance will it travel before coming to A (0,0, - 2) and B (0,0,2) respectively. How much
momentary rest ? [Ans. m «2/2 q£] work will be done in moving a test charge from
23. A particle of mass m and charge q is released from point P (4, 0, 0) to Q (- 5, 0, 0) ?
rest in uniform electric field of intensity E. (CBSE 2009) .2ero]
Calculate the kinetic energy it attains after moving 31. Two point charges each of 5 )aC but opposite in
a distance x between the plates. [Ans. qEx] sign are placed 4 cm apart. Calculate the electric
ELECTROSTATIC CHARGES AND FIELDS 1/87

field intensity at a point distant 4 cm from the mid 38. A spherical Gaussian surface encloses a charge of
point on the axial line of dipole. 8-85 X 10** C (i) Calculate the electric flux passing
(Pb. Board 2002) [Ans. 10* NC"*] through the surface («) If the radius of Gaussian
surface is doubled, how would the flux change ?
TYPE VI. TORQUE ON DIPOLE
(CBSE 2007, 2001)
AND POTENTIAL ENERGY OF
DIPOLE IN ELECTRIC FIELD [Ans. lO'* Nm^ C“*, No change]
39. A rectangular surface of sides 10 cm and 15 cm is
32. An electric dipole of dipole moment 4 x 10”^ C.m placed inside a uniform electric field of
is placed in a uniform electric field of 10^ NC”* 25 Vm**, such that normal to the surface makes
making an angle of 30“ with the direction of field. an angle of 60“ with the direction of electric Field.
Determine the torque exerted by the electric field Find the flux of electric field through the
on the dipole. (Hr. Board 2001)
rectangular surface. [Ans. 1*8 x 10~^ Nm^ C“^]
[Ans. 2 x 10“* Nm]

ww
A A A

33. An electric dipole of length 10 cm having charges 40. If the electric field is given by (6 / + 4y + 4k),
+ 6 X 10"^ (f, placed at 30° with respect to a calculate the electric flux through a surface of area
uniform electric field experiences a torque of 20 units lying in F- Z plane. (Ans. 120 units]

Flo
41. The electric field in a certain region of space is
magnitude 6yj3N-m ■ Calculate (/) magnitude of

ee
electric field (i() the potential energy of dipole. (5 i + 4j ~ 4/:)x 10^ N/C. Calculate electric

rere
flux due to this field over an area of
[Ans. 0) 2 X 10^ NC-’ (ii) - 18 J]

rFF
34. An electric dipole of length 4 cm, when placed (2 f - 7)x 10“2 m2 . [Ans. 6 x lO^ NC“^ m^]
with its axis making an angle of 60“ with a uniform
uur r
42. A charge of 2 x 10*^ C is placed on a comer of a

electric field, experiences a torque of 4->^Nm. foor cube of side 1 m. Find the electric flux passing
through this cube. [Ans. 28-25 NC"l m^]
ks s
Calculate the potential energy of the dipole, if it 43. Five thousand lines of force enter a certain volume
Yoo
has a charge ± 8 nC. (CBSE 2014) [Ans. - 4 J]
ook

of space and three thousand lines emerge from it.


TYPE VII. ELECTRIC FLUX What is the total charge in coulomb within this
BBo

AND GAUSS’S THEOREM volume? [Ans. - 1-77 X 1(H C]


IN ELECTROSTATICS
re

44. A positive charge of 17-7 [iC is placed at the centre


35. Given a uniform electric field of a hollow sphere of radius 0-5 m. Calculate the
ouur

flux density through the surface of the sphere.


ad

E = 5 X 10^ 7nC~K (CBSE 2003) [Ans. 6-4 x 10^ NC"!]


YY

Find the flux of this field through a square of TYPE VIII. APPLICATIONS
10 cm on a side, whose plane is parallel to Y-Z
nndd

OF GAUSS’S THEOREM
Re

plane. What would be the flux through the same


square if plane makes an angle of 30“ with X-axis ? 45. An infinite line charge produces a field of 9 x 10“*
FFii

(CBSE 2014) [Ans. 50 Nm^ C~^ ; 25 Nm^ C"M NC”’ at a distance of 4 cm. Calculate the linear
A
36. A uniform electric field E = E i N/C for x > 0 charge density. (Hr. Board 2001)
^ A
[Ans. 2 X ir’ Cm”*]
and E = -E^ i N/C for x < 0 exists. A right
circular cylinder of length / cm and radius r cm 46. An elecU-on is revolving around a long line charge
has its centre at the origin and its axis along having charge density 2 x 10”* Cm*’. Find the
X-axis. Find out the net outward flux. What is the kinetic energy of the electron, assuming that it is
net charge within the cylinder ? [CBSE 2008 (C)] independent of the radius of electron’s orbit.
[Ans. 2 7C (10^) Nm^ C-^
[Ans. 2-88 x ir^’ J]
47. A large plane sheet of charge having surface
2ni^E^€Q(lQr^) C] charge density 5 x 10"^ Cm”2 lies in XY plane.
—y A A A
Find the electric flux through a circular area of
37. If the electric field is given by £ = 8 i + 4 y + 3 /t
radius 01 m, if the normal to the circular area
NC”’, calculate the electric flux through a surface
makes an angle of 60“ with the Z-axis.
of area 100 m^ lying in X-Y plane.
[Ans. 300 Nm2 C”^] [Ans. 4-44 x 10^ Nra^ (T']
1/88 <t Fundamental Physics (XII) P7awi

48. A particle of mass 9 x 10"^ g is kept over a large suspended in oil in electric field of intensity 3600
horizontal sheet of charge density 5x10"^ C m”^. V/m acting in the upward direction ?
What charge should be given to the particle so [Ans. 1*1 X ir^ C]
that it does not fall on release?
54. Two charges of - 4 |i C and + 4 |i C are placed at
[Ans. 3-12 X 10“*^ C] the points A (1,0,4) and B (2, -1,5) located in an
TYPE IX. TYPICAL PROBLEMS electric field £ = 0-20 f V/cm. Calculate the
torque acting on the dipole.
49. In a cartesian co-ordinate system, charge =-2 [Ans. M3 X 10-^ N-m]
X 10“^ C is at jc = 0, y = 0, charge =1x C 55. An infinite number of charges each equal to q, are
is at ;r = 10 m and y = 0 and charge -- \ x placed along X-axis at x = 1, x = 2, x = 4, .r = 8,

oww
10“^ C is at jc = 0, y = - 5 cm. Find the magnitude and so on.
of resultantant force on [Ans. 19-4 N]
(i) Find the electric field at a point x = 0 due to
50. Two identical helium
this set up of charges.
filled balloons A and B
(//) What will be the electric field if in the above
fastened to a weight of

e
set up, the consecutive charges have opposite

FFrlo
5 gram by threads float

re
in equilibrium as signs.
q
Ans. ——

ree
shown in Fig. 1(Q).26.

F
0
’Stce 0
Calculate the charge on

rF
each balloon, assuming 56. A metal ball suspended from a long thread is held
that they carry equal between the plates of a capacitor ; as shown in

fsoor
charges. [Ans. ± 5-6 X 10-’ C] Fig- l(Q)-27. How will the oscillations of this
ouur
51. Identify X in the following nuclear reactions (in pendulum change, if the ball and the plates of two
skf
the first reaction, n represents a neutron): capacitor are charged ?
ooko
{a),H'+4Be^
YYo
FIGURE 1(Q).27
»X;
Bo

-I-
(c) 7N*5 + ,Hl > 2He^ + X. ///////
reB

[Ans. (a) (b) (c)


-I-
52. Four particles, each having a charge q are placed
uur
ooY

on the four comers A. B, C, D of a regular penta A B A B


ad

gon A£CZ)£. The distance of each comer from the +


centre is a. Find the electric field at the centre of
dY

the pentagon. [Ans. 9/4 7t e Q along OE] -H


53. A copper ball of density 8-6 g/cc and 1 cm in
innd
Re

mg
diameter is immersed in oil of density 0-8 g/cc.
What is the charge on the ball, if it remains just [Ans. Time period of ball will decrease]
FFi

For Difficult Questions

1. Here, n = e = 1-6 x KT*® C hydrogen atom contains one electron and each
^ = ne= H)22x l-6x 10-'®= l-6x lO^C oxygen atom contains 8 electrons. So, each
Due to excess of electrons, charge on the comb molecule of water has 10 electrons.

= - 1*6 X 10^ C No. of free electrons in 36 g of water


2. Here, no. of molecules in 36 g of water = 12-046 X 10^3 X 10
= 12-046 X
6-023 xlO^^
x36 = 12-046 X 1023
18 Negative charge, q = ne
= 12-046 X lO^^x 1-6 X 10-'® C
A water (H2O) molecule is made up of two
hydrogen atoms and one oxygen atom. Each = 1-92736 X 10^ C
ELE^rBOSTATIC CHARGES AND RELDS 1/89

7. Here ^ = 103 N
total mass
3. Number of electrons (n) = AT s= 7, r = 10 cm = 010 m
mass of each electron

F =
1 <1^
75-0 25x10^1
Au&qK -P
9x10- -31 3

103 = 9x^2 103x 7 X (0-10)2


7 (0-10)
2 or q^ = 9xl0^
q = ne =
yxl03lx(-l-6xl0-^^) or 28-3 x 10“^ C
= -l-33xlO^C 8. Charge on each a particle =-I-2 e.
4. Number of hydrogen molecules in 1-00 mole
9. Use Gm^m^ ^ qq

ww
= 6-023x 1023. r2 r2
Each molecule has two electrons/two protons 10. Here, = ^2 = 9 = ?
n = 2 x 6-023x 1023 F = 3-7 X 10“® N, r = 5 A = 5 X lO'^O m
« = ne = 2 X 6-023 x 1Q23 x 1-6 x lO"^’ C

Flo
n = ?

e
= 19-27 xl04c = 0-1927 MC _kq^

ere
As F =
5. Number of atoms in penny r2 r2

FFr
V=
6-023x1023 q - yjP'F/k
uurr X3-11
63-5

orr
1
As each atom contains 29 protons and 29 sfo (5x10-^0)2x3-7x10-9
V9xl09
electrons
kks
25x3-7
Total positive/negative charge on the penny xlO-^9^ =3-2x l(r*9c
Yo
“V 9
oooo

g = ±ne = ±n'x29e
From q=tne
eBB

_ ^ 6-023 x1Q23x 3-11 X 29 X 1-6 X 10-^9 Q 3-2x10 ,-19


63-5
^ 1-6x10-»9
rr

= ± 1-37 X 10® C 11. As is clear from Fig. 1(Q).28,


ou
ad
YY

6. Here, F = 1 qiQ-q)
4tC€o r2
nndd
Re

As j2 and r are fixed. So F is a function of q


Fi

dF
Force F will be maximum, only if = 0
dq

1
or
^[«((2-9)] = 0
4jcSq dq
F mg
or
-^[9(fi-9)]
dq
=0 AC OC

AC
or 9(-l) + (fi-9) = 0 F -mg. .(0
OC
or i2-2^ = 0 or Q^2q
qiq)
= mg. tan 6
e=2 4)ceo(AB)2
q
Now, AB = 2AC = 2/sin6
1/90 T^uuUefi^ i Fund VOL.I

This force will be maximum, when


= mgtanO

w
dF
4iieQ (2/sin 0)2 = 0
d(dq)
or 92s4i}|^/2sin2 0tan0.4n€o
which was to be proved.
12. Here, q^ = q2= 10~2 C, r = 10 cm = 10~* m OX q^-q2-2dq-0 or <3^ =-j (?|-^2)

r
m = 5 g = 5 X 10“^ kg, p = ? Charge on first sphere = q^-dq
For limiting equilibrium,
F = p /? = p. mg

r
9,92

wo
and charge on second sphere = q2 + dq

llou
= pmg
4jce«0 r2

F
9,92 _ 10~2x10-2x9x1Q9 = 92+^(9i-92)=|(9i+92)
^ " 4jc^ r^mg ~ (10-1 )2 x 5 x lO’^ x9-8

FF
s
= 018 15.

o
13. Here, i?2 = 3 x 10*^ C, ^, = 5 x 10-^ C

k
er
^ = (3r+ 5 j+ k), and ^ = (i+3y+ 2k)

e
For
ro
'* '*12 “ r2~rj = (Si + Sy+k)-(i+3j+2k)

of
A A A

o
kY
Y
= (2/+27-k)
ur
I q2l = V2^+2^+(-l)^ =3m
sf
B
A A A
Yo
A ^ fj2 _2i+2J—k As shown in Fig. 1((2).29, force exerted on charge
roo

^12 = — - 3 + 3 pC by charge at B
Ir.. I
eB
e
d

12
ru

1 ^1^2
F,1 =
● ● F 47ce 0 r2
21 “ ji hi
n
o

hi
ou
ad

_9xl0^x3xl0-^x5xl0-^
_ (9x1q9) (5xlQ-^)(3xl0-^)(2r-f 2j"-k)
iY

(0-2)2
32 3
= 3-375 N along AB
F
nd
Re

= 5 X10-3 (2 i+ 2 j- k) N. Force exerted on + 3 pC charge by charge at C


Fi

. 9x10^x3x10-6x5x10-6
I I = V[22 + 22 +(-1)2 ] 5x10-3 ^2 =
(0-2)2
= 1-5 X ir2 N
= 3-375 N along AC
^12-= -^1. = -(5X10-3) (2i+ 2j-k)N Resultant force of F, and F2
14. After transfer of charge
charge on one sphere = q^-dq
F = ^F,2 + F22 + 2 F,F2 cos 60“
= 5-845 N
Charge on second sphere = q2 + dq
Coulomb’s force between the two spheres is For charge A to be in equilibrium positive charge
must be placed at M, to exert force along MA
F =
k(qi-dq) iq2+dq)
r2 AM =V(0-2)2-(0-1)2
k

^ =-jj (9,92 + 9i ^^9 - 92 ^ - ^9^ ) = 0-lxV3 m


ELg^QSTATIC^CHABGES AMD HELDS 1/91

Force on charge + 3 |lC will be zero if

9xl0^xgx3xlQ-^ = 5-845
(0-lxV3)2
q = 6*49 nC
16.
FIGURE 1(Q).30
Fa
F2,

oww
0-4 m 94
Fi^ A

As net force on q is zero, therefore,

e
0-4 m 0-4 m

re
FFrllo
B C
QqS
92 0-4 m 93

reF
4n€n0 2x4ji£„jc^

e
Refer to Fig. 1(Q).30,
uoru 0

AB = BC = CD = AD = 0-4 m.
q =-1Sq

osFr
Force exerted by q^ on qi = force exerted by ^2 18. Force on charge at x = 2 cm due to charge at
on^l
X = 0 cm and x = 4 cm are equal and opposite.
„ _ 9x109x16x10-^x16x10-6
fkfor They cancel.
Net force on charge at x = 2 cm is resultant of
kso
repulsive forces due to two charges at x = 8 cm
ooo
Y
Yo
= 14-4 N and X = 16 cm.
BB

Force exerted by on qxq 1 1


-- F = -—^x

P _ 9x109x16x10-6x16x10-6 4TC6o . (0-08 - 0-02)2 (0-16 - 0-02)2


Y
r ree

= 7-2N
^ (0-4)2+ (04)2
oouu

1 1
F= 9 X 109(20 X 10-^)2
Fj and F3 are perpendicular to each other, (0-06)2 (0-14)2
ad

resultant force
Yd

F=l*2x lO^N

F = ^F^ + F^ = 3/(14-4)2 + (144)2 = 20-36 N 19. Refer to Fig. 1(Q).16.


nidn
Re

Total force on ^| = F + F2 = 20-36 + 7-2 F - ^»^2


' 4iceor2
FFi

= 27*56 N
17. Let each side of square be x. Fig. 1(Q).31 1-5 X10-3 X 0-2 X10-3 X 9 X109
.-. Diagonal, BD = ylx^ +x^ = x^f2 (1-2)2
= 1-875 X 103 N .along AB produced
/=; = F2 = — ,
4jceQx2 jr ._3i32-

F -
0-2 X10-3 X Q.5 X lpr-3 X 9 X 1Q9
^ 4JceQ(x-^/2)2 2x4jceQx2 (0-6)2
As Fj and F2 are ± to each other, their resultant = 2-5 X 103 iV along BC J. AB.
force

Resultant force on ^2 = ●^F,2 +


= = F, ^ = 3*1 X 103 N
1/92 P’uuCee^ 4'^ Fund^ental* Physic VOL.I

20. ^1 =5^iC = 5x 10^C,^2=‘^


r = 2cm = 2x 10"^m (6^^4
iw = 9 X 10"^ kg.
X = 2a from + q
j.- gig2 = mg
25. (a) F = = 1-6 X x 40 = 6*4 x ir^* N
4itSor^ (b) As separation of electric field lines at F is twice
the separation at A, Fig. 1(Q).19,
9xlO’xSxlOr»g^ = 9x10-5x98 1
E„ =-x40N/C = 20N/C
(2x10-2)2 B 2

oww
^2 = 7*84 X 10-12 C 26. Length of each side a = 0*1 m =:10 cm
21. Here, r = 1 micron = 1(T® m, E = 5*1 x 10^ N/C, distance ofthe centre ofthe cube from each comer,
n = ?

From qE = mg r =
aS 10^ =5^
2
cm

e
n = 47cr2pg

FFrlo
(n«)£ = -Jtr^pg,
|

re
= 5^3x10-2 m.
3 eE
When identical point charges +q each are at the

rFee
eight comers of the cube, net field intensity at the
^4 22 (1Q-^)2x103x9>8 _g centre is zero (cancelling out in four pairs). When

rF
” 3^7 1^x10-19xS-lxlO'^ ” one of the comer charges is removed, three pairs
ouru
22. From woric energy theorem.
of E values cancel. Due to charge on seventh

fsoor
comer.
1
skf
-ma2-0
F X j = Change in K.E. =-
2^ =9x10’ (5-^x10-2)2
1 q
E =
ooko
4TC€o r2
Yo
1 2
Y
qExs =
= (l-2xl0i2 4y)NC-l
Bo
reeB

mu
2 It is towards the comer without charge directed
s = radially away from + 9.
2qE 27. ^ = 4x 10-9c,a = 0-3m,r= 04m,E?:?
ooY
uur

23. As F = qE
ad

9x10^x4x10-^ x04
F qE
p qr ^
47C€o(r2+a2)3/2 ' (O42 +0-32)3/2
dY

a- — = —
m m

As v2-«2 = 2oj,and« = 0 v2 = 2as


nind

14-4
Re

E = = 115-2N/C
^ IqEx
=> \P--1 — x = —— (0-5)5
FFi

V m / m
At the centre of ring, r = 0, .*. E= 0
1 2 _ 1 (2qEx\ q(2p)
K.E. - - mv = —m = qEx 28. F = qE^
2 2
4JC6Q r3
m

24. Let electric field be zero at F, where AP = x, Fig.


-29
1(Q).32. 9 X10^ X (1-6 X10"*®) X 2 X 3-6 X10 N

FIGURE 1(Q).32
(25x10"®)5
●H
= 6-6 X irl5 N
K 6a
—>
A« 4 > B
*4q 30. As is clear fix>m Fig. 1(Q).33, dipole moment P
*q Eb p Ea
!◄—X— i6a-x) H is along BA .Test charge is to be moved from P
to Q along negative X-axis. As electric intensity
As ... K(4q) is perpendicular to displacement, ,
x2 (6a-x)2 .*. W = ^E ds cos 90® = Zcto
ANP fields 1/93

FIGURE 1(Q).33 When plane is II to Y-Z plane, 6 = 0®


Z (j) = £4 cos 0 = (5 X ICP) X 10"^ cos 0*
= 50Nm2C“V
-q-B(0, 0,2^ When plane makes an angle of 30® with X-axis,
0 = 60®
P
Q (-5. 0. 0) <|)' = £4 cos 0 = 5 X 10^ X 10"^ cos 60®
o
-X = 25Nm2c-i
P(4, 0, 0)
36. Net outward flux.
\2
Y‘ +q. ( r f +0
A(0, 0,-2) + £,.it

oww
31. Here, ^ = 5pC = 5xl(Hc (t>£ = 2 jcr2£, (1(H) Nm^ C"l
2fl = 4 cm = 4 X 10“^ m, £ = ? ? = eo<)£ = 2«r2e(,£,(10-^)C
r = 4 cm = 4 X 10"^ m
37. £=8?+47+3ik NC-^ ?=100ftm2

e
2pr 2(q)i2a)r

FFr lo
^axial (!>£ = £-?=(8/ + 47>3j?)-(100ife)

re
47C€Q(r2-a2)2 4tc6q (r^-^2)2 = 300Nm2c-l

ree
F
:
_ 9 X 1Q9 X 2 (5x IQ-^) (4 x 10"^) x 4 x 10"^
[a042-0-022]2 = 10<Nm2c-l

rF
o^al = 10*NC-l
^ Gq 8-85x10-12
(I'O flux will remain the same as charge enclosed

ffsoor
32. Here, X = p£ sin 0 = 4 x 1(T^ x 10“^ x sin 30®
ouur
by the surface is same as in case (i).
= 2 X 1(H Nm
39. Here,£=25 Vm-‘,
kosk
33. Here, 2 a = 10 cm = 0-1 cm, ^ = ± 6 x 1(T^ C 5 = 10 X 15 cm^ = 150 X 10~* m^
YYo
0 = 30®, X = 6 ^ N-ot, £ = ? re. = ? 0 = 60®,(|) = ?
oo

<j) = £s cos 0 = 25 X 150 X 10"^ cos 60®


BB

^ . As X = p, £ sin 0 = ^ (2 a) £ sin 0
(|) = 1-8 X 10-1 Nm^C-l
X 6V3
rre

*. £.=
?(2a)sin0 6xl0"2 x0-lsin30® 40. (Uf ==E.ds = (6?-f 3;-f4ik).20?
oYuu

= 120 units
ad

- £ = 2V3xlO‘*NC-^ Note direction of area element is along normal


dY

to YZ plane, i.e., along x-axis.


1/ = -p £ cos 0 = - ^ (2 a) £ cos 0
42. As is clear from Fig. 1((J).34, eight cubes will
= - 6 X 10-3 X 0-1 X 2 ^ lO^cos 30®
innd

be required to form a Gaussian surface-such that


Re

f/ = -18J the given charge of 2 x 10“® C appears at the


FFi

centre of the bigger cube.


34. Here 2 a = 4 cm = 4 x 10t"2 m, 0 = 60®,
FIGURE 1(Q).34
x = 4>SNm, 9 = ±8nC = 8x lO-^C
Now,p = qx2a = (Sx 10"^) x (4 x 10"2) Cm
X = p£ sin 0 or £ = x/p sin 0
/; -

P£ = -p£cos0 =-px XCOS0 K


psin0

I =-tcot0 =^.^cot60® Thus electric flux passing through given small


cube
= ^V3xl/->^ =-4J
U±]-l 2x10-9
35. Here. £ = 5x103/iVCT^ 8 € 8 8-85x10-*2
A = (10 cm)2 = (IQ-^ m)2 = 10-2 m2 = 28-25 NC-i m2
1/94 7>nadeefi>'^ Fundamental Physics (XII)BQ19I
43. Here, number of lines of force emerging from
the volume = <j> = 3000 - 5000 = - 2000 (^ = Escosd= (nr^) cos 0

From (j) = — 5x10"^


^0 —X (0-1)2 COS 60®
“ 2x8-85x10-^2 7
9 = €q 4> = 8-85 X 10-‘2 (- 2000)
= -l-77xl0-*C <l) = 4-44xl05Nm2c-l
44. Here, q = 17-7 = 17*7 x lO"^ C, r = 0-5 m 48. Here, m = 9 x 10“^ g = 9 x 10"^ kg
17>7xl0~^
a = 5xl0-5Cm-2,g = ?
q _
Total flux, ^ = — In equilibrium, F=qE = mg
^0 8-85xl0r>2 \
( ^
= 2 X 10<^ Nm2 Cr^
totalflux _ (|) i^eoj = mg

ww
Flux density = 2eQmg
area 47cr^ 9 =
a
2x10^
= 6-4x105nC-1 2 X 8-85 X 10"^2 X 9 X lO"^ X 9-8

Flo
4x314(0-5)2 5x10-5
45. Here, £ = 9 x 10^ NC"', r = 4 cm = 4 x 10“2 m

e
9 = 3*12x10-^C

rree
A, = ?
49. FIGURE 1(Q).3S

r FF
X
As £ = F2a F
2ice«r
0
uurr
(10.0)
.^1
X^lne^rE =4jteQ ^ E for (0.0) 92
{
kss
1 4x10-2
x9xl0^
ooook
Yo
= kX
9x10^ 2 (0.-5): :?3
£ = 2xl(r2Cm-^
eBB

46. Due to a line charge.


X 2X Refer to Fig. 1((J).35.
rr

E =
Fj is force on qi due to ^2
2tc€q r 4jieQr
ou
ad

Force exerted by this field on electron 9xio^x2xi(r^xi(r^


Y

F,1 = = 18N
(10)^
dY

e-2X
F = qE =
4iceor F2 is force on q^ due to q^
Re
innd

Force exerted by Electric field=Centripetal force 9xl0^x2xl0-^xlxl0“^


= 7-2N
Fi

mv
2
e-2X mv
2 (5)2
F =
' As Fj and F2 are perpendicular to each other,
or
r
47ceor r

resultant force.
2 _ e-2X
So V

4m€q m = V(18)^+(7-2)2 =i!MN


eX
50. InFig. 1(Q).36, A and Bare two balloons attached
1 1 6‘2X
K.E. = -mu2 = — m to weight W by strings OA = OB = I m.
2 2
V
4ji€nm
J 47CGo AB = 0-6 m, C is centre of AB.
= 9 X 10’ X 1-6 X 10-® X 2 X 10-« AC = BC = 0-3m

= 2-88 X 10-12 J OC = ylOA^-AC^


47. Here, o = 5 x 10^ C m-2, r = 0-1 m
= V12-O-32 = 0-954
e = 60®.(j)=?
ELECTROSTATIC CHARGES AND FIELDS 1/95

Force of repulsion between the balloons. As 6+ I =Z Z=7

F =
Hence, 7A^**^
(c) Lei
4tc€q(AB)- As I5+ I = 4 + A A= 12
As 7+l=2 + Z .-.Z = 6

Hence,
52. In Fig. 1(Q).37, suppose four particles each of
charge q are placed at the corners A, B, C and D
of regular pentagon ABCDE.
FIGURE 1(Q).37

w
FF loow
ee
As the balloons float in equilibrium,

Fr
If we were to pul the same charge q at E also,
Using triangle law of vectors, we get
then by symmeti7, net field at O would be zero.
F w
Hence combined field at O due to charges at A,

rer
ur
AC OC' B, C, D is equal and opposite to the field, at O
where w = upthrust on each balloon.
As >v + w = ly,
EO).
fofr Fo
due to charge at E. (which is <//4 7t €„ along
ks
Hence the combined field at O due to four
YYouro
w = H72
charges at A, B, C. D = q/4 tc e along OE.
s oo

AC W
F = — X — 53. Here, density of copper ball, pj = 8*6 g/cc
BBook

OC 2 = 86(X) kg/m
r ee

qxq AC
X
irtf> radius of ball, r = -cm = 0-5 X l0-2m
2
4%e^iAB)^ OC 2
oouru
ad

density of oil, P2 = 0-8 g/cc = 8(K) kg/nv^


Y

AC mg Charge, ^=?, £=3600 V/m.


= 4 7t € 0 (AB)- X OC
X
2 The ball will remain first suspended in oil, when
Ynd

force due to electric field (F) is equal to apparent


Re

(0-6)-x0-3x(5xl0-'^x9-8) weight of the ball in the oil, IFig. l(Q).38J


FFindi

9x10^x0-954x2
FIGURE 1(Q).38
0-36 X 0-3 X 49 F = 9Ea
xl()-'-
9 X 0-954 X 2

q = ± 0-56 X lO-^ C = ± 5-6 x 10'^ C


51. Using the law of conservation of mass number
and charge number,
-:App'.wt;
(a) Let
As 1 +9 = A-i- 1 A = 9
As 1 -t-4 = Z+0 Z = 5 ie., F = qE— apparent weight
Hence, is is = actual wt. - upthrust (wl. of oil displaced)
(b) Let 4 4 4
As 12+ 1 =A A= 13 -nr^p.yg = I -Po)
1/96 <*. Fundamental Ph3nsics (XH) VOL.I

(») If consecutive charges have opposite signs,


Aen at X = 0,
<1- *1 *
E
Electric field, £' =
4jc€o Ll^ 4 16 64
i X ^(05
_ 3 7
X10-2 )3 x9-8(8600 - 800) 1
3600
4ji€o
^ = MxlO-®C
54. AsA(l,0,4),B(2,-l,5) = —S—
Sjtso

oww
2 fl = ^ = [(2 -1) t + (-1 - 0) (5 - 4)k ] 56. Suppose charge given to the ball is + q. Let plate
A of capacitor be given positive charge and plate
= [i-j+k] B be given an equal negative charge, so as to
produce a horizontal electric field E.
q = ±4x 10^C,E = 0-20 iVIcm^lO iV/m

e
The equilibrium position of the ball will change

re
As X = pxE = q(2a)xE iiom O to A making angle 0 with vertical.

FFrlo
The ball experiences two forces : qE along
? = 4xl0“^(i‘-j + it)x20f

rF
horizontal and mg along vertical such that

ee
X =8xl0-5(it+^ qE
ouru
rF
tanO =
Magnitude of torque, mg

ffosor
The resultant force on the ball
X = 8x10"^ +l2 = M3 X 10“* N-m
os k
55. (0 Electric field at x = 0 F' = mg' = V(m«)^ + (9E)^
ook
Yo
1
±+±+±+ g' = ilg^ + (qEAn)^
Y
E =
4tcGo y 2^ 4^ n
Bo

Time period of vibration of ball


reeB

_ g /
H/2
t' = 2jc = 2jc
oouY

4ti€o [1 4 16 J -Jp~+(qE^
ur
ad

1
^ 9 As g'> g, therefore z'< /
£ = —2
dY

4jc€q 11-1/4 3ns, Le., time period of vibration of the ball will
decrease.
nidn
Re
F
Fi

♦Note that it is a geometrical progression, whose sum =


1-r
ELECTROSTATIC CHARGES AND FIELDS 1/97

' J ●j-li'-j

WITH
7e SOLUTIONS

Q. 1. What is the force between two small charged spheres having charges of 2 x 10"^ C and 3 x 10”^ C
placed 30 cm apart in air ?
Sol. Here, = 2 X 10“^ C, ^2 = 3 X lO""^ C, r = 30 cm = 30 X m, F = ?

As F =
<ii <}2 ^ 9xlO^(2xlQ~'^)(3xlO~'^) = 6 X 10-3 N
47C€n0 (30x10"2)2

ww
Q. 2. The electrostatic force on a small sphere of charge 0*4 ji C due to another small sphere of charge
- 0*8 |l C in air is 0*2 N. (a) What is the distance between the two spheres ? (b) What is the force on
the second sphere due to the first ?

Flo
Sol. Here, ^, = 04 p. C = 0-4 x 10"^ C, ^2 = ~ ^ C = - 0-8 x 10“^ C. F= 0-2 N ; r = ?

ee
rere
As F =
9i<?2 ^1 ^2 _ (04xlQ~^)(0-8xl0~^)x9xl0^

r FF
or
47ie« 47ce^0 F 0-2
0

r2 = 16x9x 10-^ or r = 4x3xl0^m = 0*12m


uurr
foor
Force on second sphere due to the first is the same, i.e., 0*2 N
Check that the ratio ke^IGm^ nip is dimensionless. Look up a table of Physical Constants and
ks s
Q.3.
Yoo
determine the value of this ratio. What does the ratio signify ?
ooook

ke^ [Nm^C-^][C^]
eBB

Sol. = 1= (M°L°T°) The given ratio is dimensionless.

using k = 9x 10^Nm2c-^e= l-6x C, G = 6-67 x lO"*^ Nm^kg-^


rr
ouu
ad

ke^
= 2-29x1039
g = 9-1 X 10"3* kg, and nip = 1-66 x 10"^^ kg, we get.
YY

m
Gm„e m„p
nndd
Re

This is the ratio of electrostatic force to gravitational force between an electron and a proton,
Q. 4. (a) Explain the meaning of the statement ‘electric charge of a body is quantised.’
Fii

(b) Why can one ignore quantisation of electric charge when dealing with macroscopic, i.e., large
scale charges ?
Sol. (a) Refer to Art. 1.8.

(b) As electric charge is quantised, q=±ne where e = 1 -6 x 10-^9c


At macroscopic level, one deals with charges that are enormous compared to the magnitude of charge (e).
For example, when ^ = 1 |X C = 10"^ C
q 10-^
n = — « 6-2x10^2
e 1-6x10->9
i.e., a charge of I p C contains 6-2 x 10*^ electrons. The fact that charge can increase or decrease only in
units of 'e' when 10^^ electronic charges are involved has almost no significance. Thus at the macroscopic
level, graininess of electric charge or quantisation of electric charge is of no practical consequence. It can
be ignored and electric charge can be considered as continuous.
1/98 Fundamental Physics (XiniTTSIMl
Q. 5. When a glass rod is rubbed with a silk cloth, charges appear on both, A similar phenomenon is
observed with many other pairs of bodies. Explain how this observation is consistent with the law
of conservation of charge.
Sol. When a glass rod is rubbed with a silk cloth, charges appear on both. These charges are equal in magnitude
and opposite in sign, so that algebraic sum of the charges produced on both is zero. The net charge on the
two bodies was zero even before rubbing them. Thus we find that charges can be created only in equal and
unlike pairs. This is consistent with the law of conservation of charge.
Q. 6. Four point charges = 2 p. C ; - 5 p C, ^ ^ ^ P C are located at the corners
of a square ABCD of side 10 cm. What is the force on a charge of I p C placed at the centre of the
square ?
FIGURE 1(N).1
Sol. In Fig. 1(N).I, length of diagonal of the square
D (-5pc) C (2pc)
= AC = BD = Vlf^TTo^ = lO V2 cm ●
■7

w
10V2 lO cm.
OA = OB = OC = OD = —
iT f^\
\ /

Forces of repulsion on I p C charge at O due to 2 p C ¥ 10cm

Flo
charge, at A and C are equal and opposite. Therefore, they /0\
cancel. \"b

ee
Similarly, forces of attraction on I p C charge at O, due to ’ S

Fr
- 5 p C charges at B and D are also equal and opposite.
Therefore, these also cancel. A(2mc) B (-5pc)
Hence the net force on the charge of I p C at O is zen>.

or
ur
Q. 7. (a) An electrostatic Held line is a continuous curve. That is a field line cannot have sudden breaks.
sf
Why not ?
(b) Explain why two field lines never cro.ss each other at any point.
ok
Yo
Sol. {a) An electrostatic field line is a continuous curve, because it represents the actual path of a unit positive
charge, which experiences a continuous force. It cannot have sudden breaks because the moving test
Bo

charge never jumps from one position to the other.


(/>) Refer to Art 1.27.
re

Q. 8. Two point charges = 3 p C and - 3 p C are located 20 cm apart in vacuum (a) What is the
electric field at the mid point O of the line AB joining the two charges ? (b) If a negative test charge
ou

of magnitude 1*5 x 10~^ C is placed at the point, what is the force experienced by the test charge ?
ad

Sol. Here, (/^=3pC = 3x IQ-^C, - 3 p C = - 3 xlfi-^ C, A5 = 20cm,Fig. 1(N).2.


Y

r=OA=OB= 10cm= 10“‘ m.E^?


Electric field is the force on unit positive charge (+ 1C) at O.
nd
Re

FIGURE 1(N).2

2xr/^xl
Fi

E = +3nC E -3gC
4jc€o0 r-
A O B
M- 20 cm -H

„ 2x3xlO-^x9xlO^
{10-1)2 = 5*4x10^ N/C, along OB FIGURE 1(N).3

Force on negative charge q = - 1-5 x 1()-'^C at O will be Z

F=qE^~ I-5x IO-^(54x 10^)N B (0.0.+15)


F = -8*l X ir^N ; along BA
Q. 9. A system has two charges = 2*5 x 10“^ C and q/f^- 2*5 x
10~' C l<»cated at points A (0,0,-15 cm) and B (0,0, +15 cm)
respectively. What are the total charge and electric dipole
moment of the system ?
Sol. Two charges q^ and are located at points A (0,0,-15) and B
(0, 0, 1.5) on Z-axis, as shown in Fig. I(N).3. They form an
electric dipole. A (0.0. -15)
ELECTROSTATIC CHARGES AND RELDS 1/99

Total ch^e, q - q^ + qg = 2-5 x 10"’ - 2-5 x 10"^ = 0


Also, = 15 + 15 = 30 cm = 30 X 10-2 m
Electric dipole moment,
p = either charge xAB- 2-5 x 10“2 x (30 x I0"2) = 7*5 x 10“* C-m
The direction of p is along BA,/,e., along negative Z-axis.
Q. 10. An electric dipole with dipole moment 4 x 10^’ C m is aligned at 30** with the direction of a uniform
electric field of magnitude 5 x 10* NC~*. Calculate the magnitude of the torque acting on the dipole.
Sol. Here,p = 4x lO-^Gm, e = 30*,£*5x 104nC-',t = ?
X = p£ sin 0 = (4 X 10"^) X (5 X Id**) sin 30®
x = 4x5xl0"^x-=1(Hn -m
2

ww
Q. 11. A polythene piece rubbed with wool is found to have a negative charge of 3 x 10*^ C.
(a) Estimate the number of electrons transferred (from which tO which ?)
(b) Is there a transfer of mass from wool to polythene ?

Flo
Sol. (a) Here, ^ = - 3 x 10“2 C, Charge on one electron, c = - 1 -6 x 10“'^ C

e
-3x10"'^

eree
q
Number of electrons transferred to polythene piece from wool, n = — = = 1*875 X 10*2
-l-bxior'^

FFr
(b) Yes, there is a transfer of mass from wool to polythene piece,
As
uurr mass of each electron = 9 x 10“^' kg

orr
Mass transferred to polythene = 1-875 x lO'^ x 9 x IQr^* kg = T687 x 10“** kg
sfo
Q. 12. (a) Two insulated charged copper spheres A and B have their centres separated by a distance of
50 cm. What is the mutual force of electrostatic repulsion if the cha^e oii each is 6*5 x 10~^ C ? The
kks
Yoo
radii of A arid B are negligible .compared to the distance of separation,
oooo

(b) What is the force of repulsion if each sphere is charged double the above amount, and the
distance between them is halved ?
eBB

Sol. (a) Here, ^, = ^2 = 6-5 x 10"^ C, r = 0-5 m


1 _9 x109x(6:5XI0~^)2
uurr

F =
= 1*521 X ir2 N
47t€ 0 r2 (0-5)2
ad
Yo

(b) Now 9|, ^2 both are doubled and r is halved,


dY

As F = 1 ^1^2 .%. Fbecomes 16 times, i.e., F' = 16 F= 16 x 1-521 x 10“2 N = 0*24 N


4JI€ 0 r2 .
innd
Re

Q. 13. The given Fig. 1(Q).15 shows tracks of three charged particles, in a uniform electrostatic field. Give
the signs of the three charges. Which particle has the highest charge to mass ratio ?
Fi

Sol. Refer to Conceptual Problem 17, Page 1/72.


Q. 14. Consider a uniform electric field E = 3 x 10^ / N/C. (a) What is the flux of this Held through a

square of 10 cm on a side whose plane is parallel to the yz plane ? (b) What is the flux through the
same square if the normal to its plane m^es a 60® angle with the x-axis ?
Sol. Here, £ = 3 x 10^ / N/C, i.e., field is along + direction of X-axis
surface area, s = (10 cm)2 = 1Q2 cm2 _ jq2 ^ jQr4 ^2 jQr2 ,^2
(a) When plane is parallel to yz plane, 0 = 0®
<!)£ = £i cos 0 = 3 x 10^ X 10-2 cos 0“ = 30 NC-* m2
(b) When normal to the plane makes an angle of 60® with X-axis, then 0 = 60®

(J>^ = £.v cos 0 = 3 X 10^ X 10“2 cos 60® = 30x—2 =15 NC“* iri2
1/100 “P>uuieefr'A Fimdamehtal'Physii^'CXII)^ VOL.I
Q. IS. What is the net flux of the uniform electric field of Q.^14 above through a cube of side 20 cm
oriented so that its faces are parallel to the <co>ordinate planes ?
Sol. Net flux over the cube is zero, because the number of lines entering the cube is the same as the number of
lines leaving the cube.
Q. 16. Careful measurement of the electric field at the surface of a hlack hox indicates that the net outward
flux through the surface of the box is 8*0 x 10^ Nm^/C (a) What is the net charge inside the box ?
(b) If the net outward flux through the surface of the box were zero, could you conclude that there
were no charges inside the box ? Why or why not ?
Sol. (a) = 8-0 X 103 NCr* m^, ^ ?
As <t>£ = 9 = So cl)£ = (8-85 X 10-*2) (8-0 xlO^) = 0 07 x 1(H C = 0 07 \i C
^0
(h) When (|>£ = 0, ^ = 0. Infact, Xq = 0, i.e., algebraic sum of the charges inside the box must be zero or

ww
diere may be no charge inside the box. FIGURE 1(N).4
Q. 17. A point chaise +10 p C is at a distance of 5 cm directly above
the centre of a square of side 10 cm as shown in Fig. 1(N).4.
What is the magnitude of the electric flux through the square ?

Flo
f- .u

(Hint Think <^the square as one face of a cube with edge 10 cm)

e
Sol. In Fig. 1(N).4, ABCD is a square of side 10 cm. The point charge +

ree
10 p C is at a distance of 5 cm. directly above the centre of ABCD. +9 f

Fr
We can imagine the square ABCD as one of the six faces of a cube D 5 cm

rF
C
of side 10 cm. The charge + $ is at the centre of this cube.
uurr
According to Gauss’s theorem, total electric flux through all the six
faces of the cube = q/e q
^ 1 q
for A 10 cm

1 10x10“^ = l*88xl0SNm2c-l
s B
kks
.*. Electric flux through the square = -— = ——Ty
^ ^ ^ 6 Sq 6 8-85x10"^2
Yo
oooo

Q. 18. A point charge of 2*0 p C is at the centre of a cubic Gaussian surface 9*0 cm on edge. What is the net
electric flux through the surface ?
eB

Sol. Here, ^ = 2*0 p C = 2 x KT® C, / = 9-0 cm.


2x10-^
r

^E=—= 12 = 2*26xlO®Nm2c-l
ou

^ Sq 8-85x10-‘2
ad
YY

Q. 19. A point charge causes an electric flux of -1*0 x 10^ N m^/C to pass through a spherical Gaussian
surface of 10*0 cm radius centred on the charge, (a) If the radius of the Gaussian surface were
nndd

doubled, how much flux would pass through the surface ? (b) What is the value of the point charge ?
Re

Sol. Here, (j)^ = - 10 x lO^ N m^/C, r = 10 0 cm.


Fi

(a) If the radius of Gaussian surface were doubled, flux passing through the surface would remain the
same. This is because flux depends only on charge q present inside the surface.

(b)As <1>£ = — /. 9 = 6q = (8-85 X 10-^2) (_ 1.0 X lO^) = - 8*85 x C


^0
Q. 20. A conducting sphere of radius 10 cm has an unknown chaise. If the electric field 20 cm from the
centre of the sphere is 1*5 x 10^ N/C and points radially inwards, what is the net charge on the
sphere ?
Sol. Here, radius of sphere = 10 cm.
Distance of point from the centre of the sphere, r = 20 cm = 0-2 m
: Electric field, £ = - 1-5 x 10^ N/C (Neg. sign for inward field)
Charge, q = l

(0-2)2 (-15x 10^) =-|xl0"®C =_6^7x10t^C


1
As = .*. ^ = (4jcGq r2)£ =
4jc6o r2 9x109
ELECTROSTATIC CHARGES AND FIELDS 1/101

Q. 21. A uniformly charged conducting sphere of 2*4 m diameter has a surface charge density of
80*0 p. C/m^. (a) Find the charge on the sphere (b) What is the total electric flux leaving the surface
of the sphere ?
Sol. Here, D = 2r = 2-4 m, r = 1 -2 m. o = 80-0 Orn^ = 80-0 x 10“^ Qm^.
22
Charge on the sphere, Q=<sx4nr^ = 80xi0“^x4x —x(l-2)^ = 1-45 x ir^ c

Q 1-45x10"^
Electric flux, =— -12
= 1-6 X 10* NC-^ m^
8-85x10

Q. 22. An inflnite line charge produces a fleld of 9 x 10^ N/C at a distance of 2 cm. Calculate the linear
charge density.
Sol. Here, £ = 9x10'^ N/C, r = 2 cm = 2 x m, >, = ?
X 1
As E = X - 2nGr.0 rE = x2x10-2x9x10^ =ir’Cm-^

w
2TC€/^
0
r 2x9x10^
FIGURE 1(N).S
Q. 23. Two large thin metal plates are parallel and close to each
other. On their inner faces, the plates have surface charge

Flo
densities of opposite signs and magnitude 17*0 x 10~^^ C/nr.

e
ree
I II Ill
What is £ : in the outer region of the first plate, (b) in the +o

FFr
outer region of the second plate, and (c) between the plates ?
See Fig. 1(N).5.
Sol. Here, a = 17-0 x 10-22 q ^-2
urr
for
As discussed in theory, in region I, to the left of the plates, £ = 0. And in region III to the right of the plates,
a 17-0x10-22
= 1-92 X 10-1® ffQ-l
kkss
again, £ = 0. However, in region II, inbetween the plates, £ = —
^0 8-85x10“‘2
Yo
ooo
B
re
ou
ad
YY
ndd
Re
Fi
1/102 “Pn^deep. ^ Fundamental Physics (XII) PWl

. V WITH ANSWERS,
■I 9f HINTS AND SOLUTIONS

m I
iW sBimvy

MULTIPLE CHOICE QUESTIONS-I

1. In Fig. 1(N).6, two positive charges 92 ^"<1 ^3 (a) Fig. (0 (b) Fig. (//)

ww
fixed along the>>-axi$, exert a net electric force (c) Fig. (m) id) Fig. (/V)
in the + x direction on a charge q ^ fixed along 3. The electric flux through the surface
the 3T>axis. If a positive charge Q is added at

Floo
(x, 0), the force on q^
FtGURE 1(N).6

ee
eer
‘/2 ‘i2

rFrF
Q
♦ .r ■►.T
rur
9l (T. 0)

^3
ffoor
s
O O
osk
YYoou
oook

(a) shall increase along the positive .v-axis


eBB

(b) shall decrease along the positive .r-axis


(c) shall point along the negative .t-axis
(d) shall increase but the direction changes
r

(rt) in Fig. UN).8(/v’) is the largest


because of the intersection of Q with ^2 ‘>nd %
ouur
ad

2. A point po.sitive charge is brought near an ih) in Fig. l{N).8(/;7) is the least
(c) in Fig. l(N).8(/7)issameasinFig. 1(N).8(j77)
Y

isolated conducting sphere, Fig. 1(N).7. The


electric field is be.st given by but is smaller than that in Fig. l(N).8(/v)
nd dY

id) is the same for all the figures


Re

FIGURE 1(N).7
4. Five charges ^2» ^3» and are fixed at
their positions as shown in Fig. 1(N).9. 5 is a
FFini

o
Gau-ssian surface. The Gauss’s law is given
by (f E.ds = ^
s ^0
a
FIGURE 1(N).9
Gaussian Surface

a
Which of the following statements is correct ?
ELECTROSTATIC CHARGES AND FIELDS 1/103

(a) Eon the LHS of the above equation will have (fl) The dipole will not experience any force
a contribution from ^i, <?5 and while q on (b) The dipole will experience a force towards
the RHS will have a contribution from ^2 and right
^4 only (c) The dipole will experience a force towards
{b) E on the LHS of tlie above equation will have left
a contributionfrom all charges while q on
the RHS will have a contribution from ^2 and (</) The dipole will experience a force upwards
^4 only 6. A point charge + g, is placed at a distance d
(c) E on the LHS of the above equation will have from an isolated conducting plane. The field
a contribution from all charges while q on at a point P on the other side of the plane is
RHS will have a contribution from ^ and (a) directed perpendicular to the plane and away
95 only from the plane
{d) Both, £ on the LHS and q on the RHS will (b) directed perpendicular to the plane but
have contributions from ^2 and q^ only

ww
towards the plane
5. Figure 1(N).10 shows electric field lines in
which an electric dipole p is placed as shown. (c) directed radially away from the point charge
Which of the following statements is correct ? (d) directed radially towards the point charge
7. A hemisphere is uniformly charged positively.

Flo
FIGURE 1(N).10
The electric field at a point on a diameter

e
away from the centre is directed

ree
(a) perpendicular to the diameter

Fr
(b) parallel to the diameter

rF
(c) at an angle titled towards the diameter
uurr
(d) at an angle tilted away from the diameter
s for
kks
MULTIPLE CHOICE QUESTIONS-II
Yo
oooo

If ^ EJs = 0 over a surface, then (b) 0 E.ds = 0 if the charge is outside the
eB

8.
s s
surface
(a) the electric field inside the surface and on it
r

is zero
ou

(c) 0 E.ds could not be defined


ad

(b) the electric field inside the surface is


YY

S
necessarily uniform —*

(c) the number of flux lines entering the surface (t/) (E £.t/j’ = — if charges of magnitude q
nndd

must be equal to the number of flux lines s ^0


Re

leaving it were inside the surface


Fi

(if) all charges must necessarily be outside the 11. Consider a region inside which there are
surface various types of charges but the total charge
9. The Electric field at a point is is zero. At points outside the region
(a) always continuous (a) the electric field is necessarily zero
(b) continuous if there is no charge at that point (b) the electric field is due to the dipole moment
(c) discontinuous only if there is a negative of the charge distribution only
charge at that point
(c) the dominant electric field is OC —^ ^ for large
(d) discontinuous if there is a charge at that point
10. If there were only one type of charge in the r, where r is the distance from a origin in
universe, then this region
id) the work done to move a charged particle
id) (.) E.ds=}^0 on any surface along a closed path, away from the region,
5 will be zero.
1/104
“pfuidce^'a. Fundamental Physics fXinPZSTMl
12. Refer to the arrangement of charges in Fig. charge q is placed at the centre of the ring,
l(N).ll and a Gaussian surface of radius K Fig. 1(N).12. Then
with Q at the centre. Then
FIGURE 1(N).12
FIGURE
+ + + Q
Gaussian Surface
i s +
\ +

V \ +

\+
+i 9
R +'

w
/ +
/+
5Q R/2 +
+'
-2Q

{a) total flux through the surface of the sphere (a) lfq>0 and is displaced away from the centre

e
-Q in the plane of the ring, it will be pushed back

row
re
IS towards the centre,
%
~Q (b) lfq<0 and is displaced away from the centre
ib) field on the surface of the sphere is in the plane of the ring, it will never return

FFllo
F
(c) flux through the surface of sphere due to 5 Q to the centre and will continue moving till it
hits the ring,

u
IS zero

ree
(d) field on the surface of sphere due to - 2 Q is (c) If ^ < 0, it will perform SHM for small dis
same everywhere placement along the axis,

sFr
13. A positive charge Q is uniformly distributed (d) q at the centre of the ring is in an unstable equi

kro
along a circular ring of radius R. A small test librium within the plane of the ring for q > 0.
uor
offoANSWERS
kos
l.(fl) 2.{a) 3. id) 4.(b) S.(c) 6. (a) 7. (a) 8. (c, d) 9.{b,d)
YYo
10. (b, d) 11. (c. d) 12. (a, c) 13. (a. b, c)
eerBB
oo
rY

HINTS FOR DIFFICULT MULTIPLE CHOICE QUESTIONS


uu

Multiple Choice Questions -1


ad
doo

1. As ^2. q-i are positive charges and net force on


nY

FIGURE 1(N).13
q^ is along + x direction, therefore q^ must be
negative as shown in Fig. 1(N).13(a).
+?2
nid
Re

When a positive charge Q is added at (x, 0), it


will attract (-● q^) along -i- x direc- tion. Fig.
FFi

Q
l(N).13(i). Therefore, force on qi will ¥ ^X
increase along the positive A^-axis. O
{~)q^ {X, 0)
2. When a point positive charge is brought near
an isolated conducting sphere, there developes
some negative charge on left side of the sphere
and an equal positive charge on the right side
o
of the sphere. Electric lines of force emanating
from the point positive charge end normally on the left side of the sphere. And due to positive charge on the
right side of the sphere, the electric lines of force emanate normally from the right side. The electric field
is best given by Fig. l(N).7(i).
3. The electric flux through a surface depends only on amount of charge enclosed by the surface. It does not
depend on size and shape of the surface, as per Gauss’s theorem in electrostatics. Therefore, electric flux
through the surface is the same for all the figures, 1(N).8. Choice (d) is correct.
1/105
EkiE^RQSTAnC CHARGES ARD FIELDS
^ -4
4. According to Gauss’s theorem in electrostatics, ^ E.ds Here, E is due to all the charges q^, ^2*

0

^3, ^4 and q^. As ^ is charge enclosed by the Gaussian surface, therefore, q = Q2 + ?4- Choice (^) is correct
5. In Fig. 1(N).10, spacing between electric lines of force increases from left to right. Therefore, E on left is
greater that E on right. Force on + ^ charge of dipole is smaller and to the right. Force on - ^ charge of
dipole is bigger and to the left. Hence, the dipole will experience a net force towards the left
6. When a point charge + ^ is placed at a distance (d) firom an isolated FIGURE 1(N).14
conducting plane, some negative charge developes on the surface of the
plane towards the charge and an equal positive charge developes on opposite
side of the plane. Hence, the field at a point P on the other side of the plane
is directed perpendicular to the plane and away from the plane as shown in
●P
Fig. 1(N)-14. Choice (a) is correct ©

ww
7. When the point is situated at a point on diameter away from the centre of +q
hemisphere charged uniformly positively, the electric field is perpendicular
. , . , to the diameter. The components of electric intensity parallel to the

Floo
j ; : diameter cancel out.

Multiple Choice Questions - II

ee
reer
. -4 r .

8. y E.ds represents electric flux over the closed surface. When ^ E.ds = 0, it means the number of

rFF
' c 0
flux lines entering the surface must be equal to number of flux lines leaving it. Further, as d E.ds = —
uur r
ffoor
where q is charge enclosed by the surface. When ^ E.ds = 0, \q = 0, i.e., net charge enclosed by the
sks
surface must be zero. Therefore, all other charges must necessarily be outside the surface. This is because
YYoo
charges outside the surface do not contribute to the electric flux. Choices (c) and (d) are correct
ooko

9. From the general knowledge of theory, electric field at a point is continuous if there is no charge at that
point. And the field is discontinuous if there is charge at that point Choices iff) and (d) are correct
eBB

10. As per Gauss’s theorem in electrostatics, E ds = — * where q is charge enclosed by the surface. If the
^ ■ ^0
r
ouur
ad

f —^
charge is outside the surface, ^inside = 0- Therefore, ^ E.ds = 0
YY

' Hence, choices (6) and (d) are correct


11. When diere are various types of charges in a region, but the total charge is zero, the region can be supposed
nndd
Re

to contain a number of electric dipoles. Therefore, at points outside the region (which may be anywhere
FFii

w.r.t electric dipoles), the dominant electric field oe


ir
r3
for large r. Choice (c) is correct
Further, as electric field is conservative, work done to move a charged particle along a closed path, away
from the region will be zero. Choice (d) is also correct
12. In Fig. l(N).ll, charge enclosed by the Gaussian surface = Q-2Q = -Q. Therefore, as per Gauss’s
theorem, total flux through the surface of the sphere = - Q/^q. Further, as charge 5 g lies outside the
surface, it makes no contribution to electric flux though the given surface. Choices (a) and (c) are correct
13. As is known from theory, at the centre of the ring, £ = 0 when a positive charge (q > 0) is displaced away
from the centre in the plane of the ring, say to the right force of repulsion on q, due to charge on right half
increases and due to charge on left half decreases. Therefore, charge q is pushed back towards the centre.
Choice (a) is correct
When charge q is negative {q < 0), force is of attraction. Therefore, charge q displaced to the right continues
moving to the right till it hits the ring. Choice (ft) is correct
1/106
^ Fundamental Physics (XII) W«TI

Qr
Along the axis of the ring, at a distance r from the centre, £ =
47l€Q(r^
If charge ^ is negative {q < 0). it will perform SHM for small displacement along the axis. Choice (c) is
correct.

VERY SHORT ANSWER QUESTIONS


14. An arbitrary surface enclo.ses a dipole. What Is the electric flux through this surface ?
A ns. As net charge on a dipole i.s (- ^ + ^) = 0. therefore, when an arbitrary surface encloses a dipole, as per
Gauss's theorem, electric flux through the surface

^ EM = — = Zero

w
^0
15.
A metallic spherical shell has an inner radius /?, and outer radius A charge Q is placed at the

Flo
centre of the sphericalcavity. What will be surfacecharge density on (/) the inner surface,and («) the

e
outer .surface ?

reee
Ans. When a charge + Q is placed at the centre of spherical cavity, shown

FFr
in Fig. 1(N).I5, charge induced on the inner surface of shell = - Q
and charge induced on the outer surface of the shell = + Q.

for
ur
.●. Surface charge density on the inner surface = -Q and
4kR?'
I
kss
Yo
Suifacc charge density on the outer suriace = +Q
oo

4nRj
B

16. The dimensions of an atom are of the order of an Angstrom. Thus there must be large electric fields
re

between the protons and electrons. Why, then is the electrostatic field inside a conductor zero ?
Ans. As is known, electrostatic field is caused by excess charges. On the inside of an isolated conductor, there is
ou
ad

no excess charge. Therefore, electrostatic field inside a conductor is zero.


YY

17. If the total charge enclosed by a surface is zero, does it imply that the electric field everywhere on the
surface is zero ? Conversely, if the electric field everywhere on a surface is zero, does it imply that net
nd
Re

charge inside is zero.


Ans. When the total charge enclosed by a surface is zero, it does not imply that electric field everywhere on the
Fi

(■-»-» a
suriace is zero. As O E .tls — — 0, therefore, the field may be normal to the surface. The converse
^0
is. however tme, i.e., when £ = 0 FIGURE 1(N).16 FIGURE 1(N}.17
everywhere on a surface, net
charge inside is zero. 4-
+ + + + + +
18. Sketch the electric field lines
+ +

for a uniformly charged + +


●4 >
hollow cylinder shown in Fig. + +
+ + 4 4
1(N).I6.
+ +
Ans. For a uniformly charged hollow + +

cylinder, the electric field lines + + + +


are as shown in Fig. I(N).17. Side View Top View
ELECTROSTATIC CHARGES AND FIELDS 1/107

19. What will be the total flux through the faces of the cube. Fig. 1(N).18 FIGURE 1(N).18
with side of iength ^ if a charge q is placed at (a) A : a corner of the cube
{b) B : mid-point of an edge of the cube (c) C: centre of a face of the cube
(d) D : mid-point of B and C.
Ans. (a) In Fig. 1(N).I8, when a charge q is placed at comer A of the cube, it is
being shared equally by 8 cubes. Therefore, total flux through the faces of C
t
the given cube = ^/8 Sq- D*
(b) When the charge q is placed at B, middle point of an edge of the cube, it
is being shared equally by 4 cubes. Therefore, total flux through the faces of B
the given cube = q/4 Eq-
(r) When the charge q is placed at C, the centre of a face of the cube, it is A

being shared equally by 2 cubes. Therefore, total flux through the faces of
the given cube = qll €().
id) Similarly, when charge q is placed at D, the mid point of B and C, it is being shared equally by 2 cubes.

ww
Therefore, total flux through the faces of the given cube = qll e ()●

Floo
SHORT ANSWER QUESTIONS
20. A paisa coin is made up of Al-Mg alloy and weighs 0.75 g. It has a square shape and Us diagonal

ee
measures 17 mm. It is electrically neutral and contains equal amounts of positive and negative charges.

reer
TVeating the paisa coin made up of only Al, find the magnitude of equal number of positive and

rFF
negative charges. What conclusion do you draw from this magnitude ?
Ans. Here, mass of a paisa coin = 0-75 g
uur r
Atomic mass of aluminium = 26-9815 gram
Avogadro’s number = 6 023 x 10-^
ffoor
sks
6-023 X IO--’
YYoo
Number of aluminium atoms in one paisa coin N = xO-75 = 1-6742 X I0~
ooko

26-9815
eBB

As charge number of Al is 13, each atom of Al contains 13 protons and 13 electrons.


Magnitude of positive and negative charges in one paisa coin = N Ze
= 1-6742 X I022 X 13 X 1-60 X 10^'^ C = 3-48 x 10'‘C = 34.8 KC
r
ouur

Thus, we find that ordinary neutral matter contains enormous amount of ± charges.
ad

21. Consider a coin of Example 20 above. It Is electrically neutral and containsequal amountsof positive
YY

and negative charge of magnitude 34.8 kC. Suppose that these equal charges were concentrated in
two point charges separated by (i) 1 cm Vz x diagonal of the one paisa coin), (ii) 100 m (●' length of
nndd

a long building), and {Hi) 10^ m (radius of the earth). Find the force on each such point charge in
Re

each of the three cases. What do you conclude from these results ?
FFii

Ans. Here, q = ± 34-8 KC = ± 348 x 10^ C, r, = 1 cm = 10"^ m ; r-> = 100 m ; = 10*^ m. /■’,=?. F, = ?; F3 = ?
According to Coulomb’s law.

\q\^ 9x10^(3-48x10-^)-
^1 = = l ()9x lO^'^N,
4it€^r^ (10-2)2

l^|2 9x10^(348x10'*)2
^2 = = 1-09 X 10'*' N,
4neQr~ (100)2

'7I- 9x10^(3-48xI0‘^)2
= 1-09 X U)2 N
471^0 f (10^)2
We observe that when ± charges in ordinary neutral matter are separated as point charges, they exert an
enormous force. Hence, it is not easy to disturb electrical neutrality of matter.
1/108 a Fundamental Physics (XII)
22. Fig. 1(N).19 represents a crystal unit of cesium chloride, CsCl.
The cesium atoms, represented by open circles are situated at the
corners of a cube of side 0.40 nm, whereas a Cl atom is situated
at the centre of the cube. The Cs atoms are deficient in one electron
while the Cl atom carries an excess electron.
(0 What is the net electric field on the Cl atom due to eight Cs
atoms ?

(u) Suppose that the Cs atom at the corner A is missing. What is


the net force now on the Cl atom due to seven remaining Cs
atoms ?
Ans. (0 In Fig. 1(N).19, Chlorine atom at the centre of the cube is attracted
equally by eight cesium atoms at the eight corners of the cube.

ooww
Symmetry shows that these forces would cancel out in pairs.
Therefore, net electric field on Cl atm due to eight Cs atoms is zero.
(if) Removing a Cs atom at the comer A is equivalent to adding a singly charged negative Cs ion at A. Net

force on the Cl atom at A would be F = - where r = distance between Cl ion and Cs ion, i.e.,

e
2 ’
47teQr

re
rFFl
ree
r = V(0*20)2 + (0-20)2 + (0-20)2 ^ 10"9 = 0-346 x ir^

F
m

rF
.2 9x10^(1-6x10“'^)2 = 1-92 x 10-’ N
F =
4ics„r2 (0-346x10-^)2

fsfoor
ouur
The direction of this force is from comer A to Cl" ion at the centre.
kosk
23. Two charges q and - 3 ^ are placed fixed on x-axis separated by distance ‘d*. Where should a third
charge 2 ^ be placed such that it will not experience any force ?
YYo
Let the charge 2 ^ be placed at P, at a distance x from A where charge q is placed. Fig. 1 (N).20. The charge
oo

Ans.
BB

2 q will not experience any force, when force of repulsion on it due to q is balanced by force of attraction
on it due to - 3 at B, where AB-d
rre

('^q)q 2?(3g)
i.e.. or (x + (Pp- = 3x^ or 1:^-2x d - = 0.
oYuu

4kg 0r.x^ Ane^ix + d)'^


ad

2d±j4d^+Sd'^ d , Sd
dY

FIGURE 1(N).20
x = = —+
4 2 2 2q 9 -3q
innd
Re

P A B
d + ^l3d h* ♦K d
X =
(ignoring - sign) I
FFi

The position P of the charge is on the left side of charge qa.\ A.


24. Fig. 1(N).21 shows the electric field lines around
three point charges A, B and C.
(a) Which charges are positive ?
(b) Which charge has the largest magnitude ?
Why ?
(c) In which region or regions of the picture
could the electric field be zero ? Justify your
answer.

(/) near A, (ii) near B, (Hi) near C, (iv) nowhere.


Ans. (a) In Fig. I(N).2I, the electric lines of force
emanate from A and C. Therefore, charges A and
C must be positive.
ELECTROSTATIC CHARGES AND FIELDS 1/109

(b) As maximum number of electric lines of force are associated with charge C, it must have the largest
magnitude,
(c) There cannot be a neutral point between a positive and a negative charge, Neutral point can be there
only between two like charges, i.e., A and C. As magnitude of charge ^4 is smaller than the magnitude of
charge C, therefore, neutral point would lie closer to charge A.
25. Five charges, q each are placed at the corners of a regular pentagon of
side ‘fl’, Fig. 1(N).22.
(a) (i) What will be the electric Held at O, the centre of the pentagon ?
(ii) What will be the electric field at O if the charge from one of the
corners (say A) is removed ?

oww
(m) What will be the electric field at O if the charge ^ at A is replaced
by-^ ?
(^) How would your answer to (a) be affected if pentagon is replaced
by n*$ided regular polygon with charge q at each of its corners ?
Ans. (a) (/) In Fig. I(N).22, the electric field at 0, the centre of pentagon would

ee
be zero.

FFrlo
07) When charge q from comer A is removed, electric field at 0 would be

r
qxl

rF
^l = along OA.

ee
4jcGQr2
(Hi) If charge ^ at A is replaced by - q, it is equivalent to adding charge -2qatA. Therefore, electric field

rF
ouru
at O would be

ffosor
2q
along OA.
47le„r2
os k
(b) When pentagon is replaced by n sided regular polygon with charge q at each of its comers, the electric
ook
field at 0 would continue to be zero.
YYo
Bo
reeB

LONG ANSWER QUESTIONS

26. In 1959, Lyttleton and Bondi suggested that the expansion of the Universe could be explained if
oouY

mattercarrieda net charge. Supposethat the Universeis made up of hydrogen atoms with a number
ur

density iV, which is maintained a constant. Let the charge on the proton be : = - (1 + y) e where e
ad

is the electronic charge,


dY

(a) Find the critical value of y such that expansion may start.
(^) Show that the velocity of expansion is proportional to the distance from the centre.
nidn
Re

Ans. (fl) Suppose the universe is a sphere of radius R and its constituent hydrogen atoms are distributed uniformly
in the sphere.
FFi

As hydrogen atom contains one proton and one electron, therefore, charge on each hydrogen atom,
e^ = ep + e = -(l+y)e + e = -ye = (ye)
If E is electric field intensity at distance /?, on the surface of the sphere, then according to Gauss’s theorem,

O E.ds = -^ , i.e., E(4nR^)=-4 uR'^N lye \


f a

^0 ^0

^ 1 NlyelR ...(0
3 ■ Gq
Now, mass of each hydrogen atom = nip - mass of a proton.
If Gff is gravitational field at distance R on the surface of the sphere, then

- 4 7t /?“ G;; = 4 7C G . 1 kR^ N or ~nGm


3 P
NR ...(ii)
1/110 “Pnadce^ <*● Fundamental Physics (XII)

Gravitational force on this atom is — Qn?NR


F\^m^xGR = 3 P
...{in)

I Ny^e^R
and coulomb force on hydrogen atom at R is F2 = {ye)E=- using (/)
^0
The expansion would start when coulomb repulsion {F-^i on hydrogen atom is larger than the gravitational
force of attraction (F,) on the hydrogen atom. The critical value of y to start expansion would be when
1 Ny^e^R 4n
F2 = F,. i.e..
3 -GmlNR
n2
m (1-66x10-27)2 = 79-8x 10-^*
or
y2 = (47C€o) G
p x(6-67xl0"“)
e 9x10^ (1-6x10-’9)2

w
y =Vv9-8x 10-38 =8.9 X 10-*9 ~ iq-18
This is the critical value ofy corresponding to which expansion of universe would start.

Flo
\ N '-p e~ R All
{b) Net force experiencedby the hydrogen atom, F = F2 - Fj = - Gm^NR
3 p
^0

e
rree
If acceleration of hydrogen atom is represented by (P'Rldfi, then

r FF
m ^ = F ]_ Ny^ e- R- 4n Gm-NR =
I Ny2^2 Gn? N R
P dt^ 3 Gq 3 P
3~i; T p
uurr
d'^R 1 1 Ny-e- 4tc
GnrN R
for
kss
3^^ T
= a-R ...(/V)
dt^ m p
P
ooook
Yo

2 - L 1
where
— G mlP N
eB

a
3
m
p .3 ^0
The general solution of eqn. (/v) is R=Ae^‘ + Be-^‘
rr

As we are looking for expansion, B = 0. R = Ae^'


ou
ad
Y

dR
velocity of expansion, v = — = Ae^"^ (a) = aA =aF
dY

dt
Re
innd

Hence, v oc
R, which was to be proved.
27. Consider a sphere of radius R with charge density distributed as p (r) = kr for r<R
Fi

= 0 for r > F.

(a) Find the electric field at all points r.


(/>) Suppose the total chaise on the sphere is 2e, where e is the
electron charge. Where can two protons be embedded such that
the force on each of them is zero. Assume that the introduction of
the proton does not alter the negative charge distribution.
Ans. The given charge density distribution of the sphere of radius R is
p (r) = it r for r < F
= 0 for r > F

The electric field is obviously radial, as shown in Fig. 1(N).23.


For points r < F
let us consider a spherical Gaussian surface 5[ of radius r. Then on
the surface,
ELECTROSTATIC CHARGES AND FIELDS 1/111

r ^ 1
() E.ds = pdV
^0

4 1
As V =-nr^, dV = - n3r-dr = 4nr^dr and p(r) = /tr
3 3

1
() E.ds = —4nk rr^ dr
^0 0

4nk 1
iE)4nr- = or E = kr- ...(0
4 4e 0
^0

ww
The direction of £ is radially outwards (for positive charge density)
For points r>£, let us considera spherical Gaussian surface ^2 of radius r. Then on the surface, when r = R
1

Flo
() E .ds = — p.dV
^0

ee
R

rere
4iik
r^dr = 4itk ^ k R*

rFF
E(4Kr~) = or £ =
^0 0 ^0 ^ 4^0
—>
uur r
The direction of £ is radially outwards (for positive charge density).
foor
(b) From symmetry, we find that the two protons must be on the opposite
sides of the centre, along a diameter of the sphere as shown in Fig. 1(N).24.
ks s
Yoo
Proceeding as above, charge on the sphere.
ook

R R
R^
BBo

c/ = pdV =j (kr) 4Kr'^dr =4-^1^ 4


re

0 0

2e
ouur

k= —T ...m
ad

71/?'^
YY

If protons 1 and 2 are embedded at distance r from the centre of the sphere as shown, then attractive force
on proton 1 due to charge distribution is
nndd
Re

kr^
F| = - e £ = -e ...using (/)
FFii

4s
0

e~
Repulsive force on proton 1 due to proton 2 is ^2 ~
47ceQ(2r)^
kr^
Net force on proton I F=F^+F2=-e 4e
0
16ice«0 r^

er ^ 2e .2
Using (i<0, F =
4e«0 kR'^ 167ce«r'^
"0

er^ .2e R^ R
This net force on proton 1 will be zero, when or = or r =
1/4
4Er.KR^
0
167ce«r2
0
8 (8)

This is the distance of each of the two protons from the centre of the sphere.
1/112
^tade€fr d Fundamental Physics CXH) VOL.I

28. IWo fixed, identical conducting plates (a and P), each ofsurface area 5 are diaiged to-Q and respectively,
where Q>q>0.X third identical plate (y), free to move is located on the other side of the plate with
charge q at a distance Fig. 1(N).25. The third plate is released and collides with the plate p. Assume
the collision is elastic and the time of collision is sufficient to redistribute charge amongst P and y*
(a) Find the electric field acting on the plate y before collision.
0) Find the charges on P and y after the collision,
(c) Find the velocity of the plate y after the collision and at a distanced from the plate p.
Ans. The given set up is shown in Fig. 1(N).26
RGURE 1(N).2S
Q yA
(a) The electric field at plate ydue to plate a is = - : to the left
5(2eo) ’

E g
The electric field at plate y due to plate p is
2 s(2eq) ; to the irght.

ww
a

Hence, the net electric field at plate y before collision is

FF loo
g-Q
£= + £2 ” to the left if Q>q
S(2eo)’

ee
(b) During collision, plates p and y are together. Their potentials become same.

ee r
Suppose chaige on plate p is and chaige on plate y is ^2- At any point O,

rFrF
inbetween the two plates. Fig. 1(N).26, the electric field must be zero. <—d—►
-O 9 Q
rur
HQ
Electric field at O due to plate a = ; to the left
5(2€q) ffoor
ks
Electric field at O due to plate p = ; to the irght
YYoou

5(2eo) ’
ookos

FIGURE KN1.26
a
Pr—y
BBo

Electric field at O due to plate y = Si ; to the left


re

5(26o) ’
oouur
ad

6+?2 _ ^1 91 92
As the electric field at O is zero, therefore. O
S(2eo)“S(2eo)
Y

C + 92 = or
fi = 9i”92 ...(0
dndY
Re

As there is no loss of charge on collision, 6 + 9 = 9i + 9a ...00 Q

On solving (0 and (iO. we get 9] = (0 + q!2) = charge on plate p


FFini

92 = (9/2) = charge on plate y.


(c) After collision, at a distance d from plate p, let the velocity of plate y be u.

After the collision, electric field at plate y is £2 = -6 , (6+9/2)^ 9/2 to the irght.
2^0*^ 2€qS 2€qS
6-9
Just before collision, electric field at plate y is £,=
2eoS
(6-9)6
If £| is force on plate y before collision, then F,=£,6 =
2e^S

Similarly, force £2 on pinte y after collision. F ~E


EtEOTROSTATIC CHARGES AND FIELDS 1/113

Total work done by the electric field in round trip movement of plate y
m-‘iyd+'W2f'ld (Q-qnf d
W=(Fj+F2)rf =
2eo5 2eo<5
1i 2
If m is mass of plate y, the KE gained by plate y - “
1

— nrtp- = W = ig-qnfd
According to work eneigy principle, ^ 2^0 5
xl/2
d
v-{Q-qH)

29. There is another useful system of units, besides the Sl/mksA system, called the cgs (centimeter-gram*

ww
second) system. In this system, Coloumb’s law is given by F = ^ r
where the distance r is measured in cm (= IQr^ m), F in dyne (= 1(F® N) and the charges in electrostatic

Flo
units (es units), where 1 es unit of charge = — x 10"^C

ee
[3]
The number [3] actu^y arises from the speed of light in vacuum which is now taken to be exactly

rere
given by c = 2.99792458 x 10* m/s. An approximate value of e then is c = [3] x 10* m/s.

rFF
(i) Show that the coulomb law in cgs units yields 1 esu of charge = 1 (dyne)^ cm.
uur r
Obtain the dimensions of units of chaise in terms of mass 3f, length L and time T, Show that it is
given in terms of fkactional powers of M and L. foor
(u) Write 1 esu of charge = xC, where x is a dimenionless number. Show that this gives
sks
Yoo
1 ^ IQ-^ Njn^
ook

With x = i-xl0-^ we have


4ic€q X* C* 131
BBo

Nm^
—^=1312x10** Nm^
re

or = (2.99792458)* X10® (exactly).


4jt€o C* 4jceo C*
ouur
ad

Ans. (OFrorn
YY

r*

1 dyne =
(1 esu of charge)*
nndd
Re

(1 cm)*
1 esu of charge = (1 dyne)‘'* x 1 cm. = F*^ . L = (ML7^*)*^ L or 1 esu of charge = Af
FFii

1 3
Thus esu of charge is represented in terms of fractional powers : — of Af and 2
(it) Let 1 esu of charge s x C, where x is a dimensionless number.
Coulomb force on two charges, each of magnitude 1 esu separated by 1 cm is 1 dyne = 10“^ N.
This situation is equivalent to two charges of magnitude x C separate by 10“* m.
1 X* 1 IQ-^ Nm*
F = = 1 dyne = 10"^ N 4jceo X* C*
47t€o (10-2)*
Nm* Nm*
Taking x =
1
o, we get. -i- = 10"®xl3l*xl0‘* = 9x10®
131x10® 4jceo C* C*

1
Ifl 3 1^2-99792458, we get = 8-98755 X 10® Nm* C*
47teo
1/114
4 Fundamental Physics (XII) VOL.I

30. Two charges - q each are fixed separated by distance


2d. A third charge q of mass m placed at the mld^pointis
displaced slightly by x (x « dli;perpendiciilar to the line
joining the two fixed charges as shown in Fig. 1(N).27.
Show that q will perform simple harmonic oscillation of
time period.
nl/2

Ans. In Fig. 1(N).28, two charges - q each are held at A and B


wljere AB = 2 d. A third charge + q of mass m is placed at
O, middle point of AB. Let it be displaced perpendicular to
AB, through a small distance OP = jc. Forces of attraction at
P towards A and B .^ each

ww
7(g)
whereAP = BP = r
4ns„r^’
Horizontal components of those forces cancel out Vertical components along PO add.

Flo
If ZAPO - 0, the net force on q along PO is P' = 2 P cos 0

e
rree
2q^x .
4iteor^ U

r FF
4jceQ(d? +
uurr
2q^x
When X « d, F‘ - K x, where K =. .

for
Ane^d}
kss
i.e., force on charge^, is proportional to its displacement from the centre O ^d is directed towards O.
ooook

Hence, motion of charge would be simple harmonic, where


Yo

nl/2

.f
eB

<0 = and T = —
m
= 2kJ— =2k m.AKeQ d^ 0

CO K
rr

31. Total charge -Qis uniformly spread along length of a


ou
ad

FIGURE 1(N).29
ring of radius R, A small test charge ●¥ q of mass m is
Y

kept at the centre of the ring and is given a gentle push AZ


dY

along the axis of the ring, Axis of ring


(a) Show that the particle executes a simple harmonic
Re
innd

oscillation.
{b) Obtain its time period.
Fi

O R
Ans. In Fig. 1(N).29, we have shown - Q charge distributed
uniformly over a ring of radius R with centre Oi A and B are
two points on the ring at the ends of a diameter.
If we take line elements of charge at A and B, having unit FIGURE 1(N).30
Q AZ
length, then charge on each element = -
2%R
Axis of irng
A small test charge + ^ of mass m kept at the centre O of the
ring is given a gentle push to P where OP = z
If AP = BP = r = (z2 + r2)V2^ and ZAPO = 0, Fig. I(N).30,
then force on q due to the two line elements
1 I
dP = 2
2b«J ' 4n6„ rJCOS0
ELECTROSTATIC CHARGES AND FIELDS 1/115

Total force on the charge due to entire ring


1
F = -
Qq
{%R). ]_ z Qqz
liR
4jtGQ r 4JIGQ +
Qq
When z<<R, F = - —^= -kz where K = = constant
4JceQ R^
Clearly, force on q is proportional to negative of its displacement. Therefore, motion of q is simple harmonic.

(0 =
K
and T
27t. V
= 2n
m47t€QF^
m (O Qq

w
47C€n0: m .
= 2tc
^ Qq

Flo
eeee
Fr
for
ur
ks
Yo
oo
eB
r
ou
ad
YY
nd
Re
Fi
1/116
7\<uUefi- ^ Fundamental Physics (XII)

NEET/JEE
SPECIAL

[QTiTuitimatS

ww
A"'
● MCQs In Physics for MEET
Pradeep's Stellar Series.... ● MCQs In Physics for JEE (Main) IHH

Floo
H Multiple Choice Questions (with one correct Answe^

ee
reer
rFF
I. Electrostatic Charges 4. Two point charges A and B, having charges + Q
and - Q respectively, are placed at certain distance
1. Four charges, each equal to - Q, are placed at
uur r
apart and force acting between them is F. If 25%
four comers of a square and a charge + ^ is at its
centre. If the system is in equilibrium, the value
ffoor
charge of A is transferred to B, then force between
sks
of q is the charges becomes
YYoo
9F
(i>) ^(1 + 2^2)
ooko

ia)F (b)
(a) 4
(I + 2V2) 16
4
eBB

16F 4F

(c) -|(1 + 2V2) id) |(1 + 2V2) {d) — ■ (NEET 2019)

5. A tiny spherical oil drop carrying a net charge


r

(AIEEE 2004) q
ouur

is balanced in still air with a vertical uniform


ad

2. Three identical spheres, each having a charge q


8l7C
and radius R are kept in such a way that each electric field of strength — xlO^ Vm When
Y

touches the other two. The magnitude of electric


nddY

force on any sphere due to other two is given as : the field is switched off, the drop is observed to
Re

\2 fall with terminal velocity 2 x ms“'. Given,


1
g = 9.8 ms“^, viscosity of the air -1.8xl(T^ Ns
FFiin

(a) 4ne {b)


0 V 4Tceo 4[r m“^ and the density of oil = 9000 kg m”^, the
magni- tude of q is

(c) 47t6«
I f«/ f id) 47t€A 2
(a) 1.6xlO-'®C ib) 3.2 X 10-1® c
0
4 R R (c) 4.8 X 10-1® C id) 8.0 X 10-1® c
0
(IIT 2010)
3. An infinite number of charges each of charge
6. Two identical charged spheres suspended from a
I pC, are placed on the X-axis with co-ordinates
x= 1,2,4, 8
common point by two massless strings of length /,
. If a charge of 1 C is kept at
OC

are initially at a distance xix«t) apart because


the origin then what is the net force acting on 1 C of their mutual repulsion. The charges begin to
charge ? leak from both the spheres at a constant rate. As a
(fl) 9(X)0 N ib) 12000 N result, the spheres approach each other with a
(c) 24000 N (d) 36000 N velocity v. Then v varies as a function of distance
(DCE 2004) X between the spheres as
ANSWERS

h(b) 2.(c) 3. (W 4.(b) 5. id)


ELECTROSTATIC CHARGES AND FIELDS 1/117

.-1/2
(a) V X {h) V <* a: (c) Acquire a dipole moment only in the presence
(c) V «: a:
-1
{d) V X
1/2
of electric field due to displacement of charges
(NEET 2016) (d) Acquire a dipole moment only when magnetic
field is absent (NEET 2021)
7. Two pith balls carrying equal charges are suspended
from a common point by strings of equal length, the 11. Two point charges - q and + q are placed at a
equilibrium separation between them is r. Now the distance L The magnitude of electric field intensity
strings are rigidly clamped at half the height. The at a distance R {R » L) varies as
equilibrium separation between the balls, now 1
becomes ;
(«) ib)
r} R^
FIGURE 1(CF).1
1 1
(c) id) (NEET 2022)
R^ R^

II. Electric Field

ww
12. Consider a uniform spherical charge distribution
of radius R^ centred at the origin O. In this

FF loo
distribution, a spherical cavity of radius /?2,
centred at P with distance OP = a = R]-R2 (Fig.
1(CF).2) is made. If the electric field inside the

ee
\2
( 1
ia) ib)
cavity at position ~r is E{r), then the correct

er
.3 - IV2J

FrreF
statement is
r (2r\ -)
ic) id)
IV3 ia) E is uniform, its magnitude is independent
rur
(AIPMT 2013)
8. Two charges, each equal to q are kept aix = ~a and
fforo
of /?2> but its direction depends on r
ks
a: = « on the X-axis. A particle of mass m and and (b) E is uniform, its magnitude depends on /?2
YYouo
—>
okso

q
charge *** placed at the origin. If charge and its direction depends on r
BBoo

is given a small displacement (y « a) along the


y-axis, the net force acting on the particle is
r ee

proportional to :
ia) y ib) -y
ad
oouur

1
ib) - id) --
Y

y y
(JEE Main 2013)
dnYd
Re

9. The capacitance of a parallel plate capacitor with


air as medium is 6 pF. With the introduction of a
FFini

dielectric medium, the capacitance becomes 30 jiF.


The permittivity of the medium i.s: (c) E is uniform, its magnitude is independent
(So = 8-85 X 10-'2c2 N"' m-2) of a , but direction depends on a
-4
ia) l-77xlO-'2c2N-'m-2 id) E is uniform and both, its magnitude and
ih) 0-44 X lO-'t^C^ N"* m"2 direction depend on a (JEE Advanced 2015)
ic) 5-00 C2 N-' m-2 13. A long cylindrical shell carries positive surface
id) 0-44 X IQ-'^C^N-' m‘2 (NEET 2020) charge o in the upper half and negative surface
10. Polar molecules are the molecules charge - c in the lower half. The electric field
lines around the cylinder will look like figure
(fl) Having a permanent electric dipole moment
given in (figures are schematic and not drawn to
ih) Having zero dipole moment scate).

ANSWERS

6. ib) 7. (c) 8. (a) 9. ih) 10. (t/) 11. ih) 12. (</)
1/118
Fundamental Physics rxiHPisrm

(c) charge + q executes simple harmonic motion


while charge - q continues moving in the
direction of its displacement.
(</) charge - q executes simple harmonic motion
while charge + q continues moving in the
direction of its displacement.
(JEE Advanced 2015)
17. A hollow metal sphere of radius R is uniformly
charged. The electric field due to the sphere at a
distance r from the centre
(a) increases as r increases for r < /? and for
r>R

ww
{b) zero as r increases for r < /? ; decreases as r
(JEE Main 2015) increases for r > /?
14. Two point charges + 8 ^ and - 2 ^ are located at a: (c) zero as r increases for r < /? ; increases as r
= 0 and x = L respectively. The location of a point

FF loo
increases for r> /?
on X-axis at which the net electric field due to
these two point charges is zero is : {d) decreases as r increases for r < /? and for

ee
iq)2L r<R. (NEET 2019)
{b)UA

ee r
(c)ZL {d)AL (AIEEE 2005) 18, Two parallel infinite line charges with linear

rFrF
15. If potential (in volts) in a region is expressed as charge densities + X c/m and - X. c/m are placed
^ y> s) = 6.ty - y + lyz, the electric field at a distance of 2 /? in free space. What is the
rur
(in N/C) at point (1, 1,0) is : electric field midway between the two line

(a) ~i6i +9 j +k)


A

(b) ~(3i +5j +3 k)


A* ffoor charges ?
ks
A A A A A 2X
(c) ~{6i +5j+2k) (d) -(2i+3j + k) (a) Zero ib) N/C
YYoou
7t /?
ookos

0
(AIPMT 2015)
BBo

16. The figures below depict two situations in which X X


two infinitely long static line charges of constant ic) N/C (d) N/C.
re

R 2n 6. R
positive line charge density X are kept parallel to 0 0

each other. In their resulting electric field, point (NEET 2019)


ouur
ad

charges q and - q are kept in equilibrium between 19. Let there be a spherically symmetric charge
them. The point charges are confined to move in
Y

distribution with charge density varying as


the X direction only. If they are given a small
/5 r\
displacement about their equilibrium positions, P(x)=Pq — upto r = /?, and p (r) = 0 for
dndY
Re

then the correct statement(s) is (are) V4 R J


r> R, where r is the distance from the origin. The
FFini

FIGURE 1(CF).4
electric field at a distance r(r<R) from the origin
is given by
X X X X 4ppr/5 r
(o) (b)
^x Px
3Go 1,4 R
-9 \

3eo U R^
(c) id)
' 36pU R
(AIEEE 2010)

id) both charges execute simple harmonic motion. 20. A thin semi-circular ring of radius r has a positive
charge q distributed uniformly over it, Fig.
ib) both charges will continue moving in the
direction of their displacement. 1(CF).5. The net field £ at the centre 0 is
ANSWERS
13.(n) 14. (n) 15. (r) 16, (c) 17, (/?) 18. (r) 19. (a)
ELECTROSTATIC CHARGES AND FIELDS 1/119

24. Three charges +(2, ^, + Q are placed respectively


FIGURE 1(CF).5 at distance 0, d/2 and d from the origin on the X-
axis. If the net force experienced by + (2 placed
at X = 0 is zero, the value of q is
(a)-Q/4 (b) + Q/2
(c) + Q/4 (d)-Q/2
(JEE Main 2019)
O
III. Electric Flux
A A

(a) - J (b) - J 25. The electrical field in a certain region is acting


4 -p 2 71“ Eq r~
0
radially outward and is given by E = Ar.A charge
contained in sphere of radius ‘a’ centred at origin
s ]
A

(c) ■y J (d) of the field will be given by :

ww
2%- £q r- 0
{a)AGQa^ (/)) 4 7t Eq A
(AIEEE 2010)
(c) e 0 A (d) 4 jt 6q A

FF loo
Qr (AlPMT 2015)
21. Let p(r) = J be the charge density distri
kR 26. Three infinitely charged sheets are kept parallel

ee
bution for a solid sphere of radius R and total to xy plane having charge densities as shown in Fig.
charge Q. For a point P inside the sphere at a I(CF).6. Then the value of electric field at P is

ee r
distance rj from the centre of the sphere, the

rFrF
magnitude of electric field is FIGURE 1(CF).6
rur
Q Qrf z = 3a +CT

(a)
4neQrf
(b)
0
ffoor P*
ks
z = 0 -2a

(d) zero (AIEEE 2009)


YYoou
ic) z--a -a

3kGqR'^
ookos

22. Two positive charges of magnitude 'q' are placed


BBo

at the ends of side 1 of a square of side '2a'. Two ■ -4a ^ 4a ^


re

(a) k (b) — k
negative charges of the same magnitude are kept ^0
^0
at the other comers. Starling from rest, if a charge
oouur
ad

Q moves from the middle of side 1 to the centre of


square, its kinetic energy at the centre of square is -2a kp 2a p
Y

(d) — k (HT 2005)


(c)
1 1. ^0 ^0
(a) 1- (b) zero
dndY

47t€ 0 V5 27. A disc of radius a/4 having a uniformly distributed


Re

1 'i charge 6 C is placed in X-Y plane with its centre


1 2qQ
FFini

(c) 1 + at (- a/2, 0, 0). A rod of length a carrying a


47t€.^0 a V5 uniformly distributed charge 8 C is placed on the
1 2qQ(.1 2 'I X-axis from.t=a/4 to.x = 5a/4. Two point charges
(d) (AIEEE 2011)
4 TIG 0 a Vs a a
-1C and 3 C are placed at — , —, 0 and
23. For a uniformly charged ring of radius R, the 4 J
electric field on its axis has the largest magnitude ^ — ,0 respectively. Consider a cubical
4 ’
4 J
at a distance h from its centre. Then value of h is ' a a
surface formed by six surfaces :c = ±—, y - ±—.
(«) /?/Vs (i') ff/V2
(c)R . r42 2 = ±^. The electric flux through the cubical
surface is
(JEE Main 2019)

ANSWERS

20. (c) 21. (f) 22. (a) 23.(6) 24. («) 25.(6) 26. (c)
1/120
Fundamental Physics (XII)mwiBl
FIGURE 1(CF).7 (0, - fl/4,0), + 3 ^ at (0,0,0) and - ^ at (0, + a/4,
rf-
V^i 0)> Fig- 1(CF).9. Choose the wrong option(5).
y
FIGURE 1(CF).9
-7C
X- ♦
3C a/4 5a/4
(-a/2)
U-".

(a) - 2 C/s 0 (*) 2 C/s 0


(c) lO C/s 0 (tO 12 C/6 0 (IIT 2009)
28. Suppose the charge of a proton and an electron
differ slightly. One of them is - e, the other is

ww
{e + Ac). If the net of electrostatic force and
gravitational force between two hydrogen atoms
placed at a distance d (much greater than atomic
size) apart is zero, then Ac is of the order of [Given (a) The net electric flux crossing the plane

FF loo
mass of hydrogen m;, = 1 -67 x 10“^^ kg] X = ■¥ all is equal to the net electric flux
(a) 10-20 C (b) 10-22 c crossing the plane x^-aH

ee
(c) 10-22 C (d) 10^2 c (6) The net electric flux crossing the plane

ee r
(NEET 2017) y = + a/2 is more than the net electric flux

rFrF
-4
crossing the plane y = - o/2
29. Consider an electric field E = EqX, where Eq is
(c) The net electric flux crossing the entire region
rur
a constant. The flux through the shaded area {as
shown in the Fig. 1(CF).8 due to this field is
FIGURE 1(CF).6
ffoor is^
^0
ks
z
(d) The net electric flux crossing the plane z =
YYoou
A
ookos

[a, 0, a) (a, a, a) + a/2 is equal to the net electric flux crossing


the plane x = + all (IIT 2012)
BBo

31. An electric dipole has a fixed dipole moment


re

pi which makes angle 9 with respect to x-axis.


(0,0,0) (0,a,0) When subjected to an electric field = £ f, it
oouur
ad

experiences a torque jf = xk. When subjected to


X
Y

(«) IEqu'^ (b) V2£(,a2 another electric field ^ = SE^ j , it experiences


dndY
Re

£,.a2
0 torque Tj = -T^. The angle 9 is
(c) £002 id)
I2 (IIT 2011) (a) 60° ib) 90“
FFini

30. A cubical region of side a has its centre at the (c) 30“ id) 45“
origin. It encloses three fixed point charges - ^ at (JEE Main 2017)

□ Multiple Choice Questions (with One or More than One Correct Answers) .
32. A few electric field lines for a system of two
charges and Q2 fixed at two different points
on AT-axis are shown in the [Fig. 1(CF).10]. These
lines suggest that
(«) I Gi I > I 02 I
(b)IQ,KIQ2l

ANSWERS

27. (a) 28. (c) 29. (c) 30. (b) 31. (a)
ELECTROSTATIC CHARGES AND FIELDS 1/121

(c) at finite distance to the left of (2i> th© electric volume charge density p = Air, where A is a
field is zero constant and r is distance from the centre. At the
id) at a finite distance to the right of Q2, the centre of the spheres is a point charge Q. The value
electric field is zero (IIT 2010) of A, such that the electric field in the region
between the spheres will be constant is
33. A spherical metal shell A of radius and a solid
metal sphere B of radius Rg < are kept far Q
(a)
apart and each is given charge + Q. Now they are 2na^
connected by a thin metal wire. Then :
Q
inside ” ® ib)QA>QB ib)
R
(c) ^ = £

^A 2Q
(C)
(d) on surface < Eg on surface (IIT 2011) 7c(a^ -b^)
34. A positively charged metal ring of radius R is fixed

ww
2Q
in the xy-plane with its centre at origin 0. A id) (JEE Main 2016)
na^
negatively charged particle P is released from rest
at the point (0, 0, Zq) where Zq > 0. Then, the 36. Let £, (r), £2 ^3 respective

FF loo
motion of P is; electric fields at a distance r from a point charge
(<j) Periodic for all the values of Zq, satisfying Q, an infinitely long wire with constant linear

ee
0<Zq< QO
charge density X, and an infinite plane with
uniform surface charge density a. If £1 (r^) - £2

ee r
(b) Simple harmonic for all the values of Zq

rFrF
satisfying 0<Zq^R (rp) = £3 (rQ) at a given distance Tq, then
(c) approximately simple harmonic provided
(fl) Q = 4o7ir2
rur
Zq«R
(d) such that P crosses O and continues to move
along the negative Z-axis towards Z = — 00
ffoor
(fc) rQ-X/2KO
(c) £i(ro/2) = 2£2(ro/2)
ks
35. The region between two concentric spheres of (d) £2 (r^2) = 4 £q (ry/2) (JEE Advanced 2014)
YYoou
radii a and b respectively. Fig. l(CF).ll has
ookos

dQ/IVIultlple Choice Questions (Based on the given Passage/Comprehens]^)]


BBo
re

Each comprehension given below is followed by some multiple choice questions. Each question has one
ouur

correct option. Choose the correct option.


ad

37. Charges + 6 - 2 ^ and +3 ^ are enclosed by a


Y

According to Gausses surface in vacuum. The total electric flux over


theorem in electrostatics, total electric flux the surface is
dndY
Re

over a closed surface 5 in vacuum Is 1/e 0 ia)6qleQ ib)-2qleQ


times the total charge iQ) contained inside 5 3q
FFini

r -* -* O
ic) id) 1 ql^Q
ue., = 0 E-ds = — ^0
ss ^0 38. A charge - 7 ^ is situated outside the surface in
the above question. What will be the total electric
flux in that case ?
The charges enclosed may be distributed any
way. If the medium surrounding the charge (a) Zero {b)-1q/^Q
has a dielectric constant K. then ic) 7 q/eQ id) 14 ^/€o
39. Charges a- 2 q, - 5 q and 8 9 are enclosed by a
e_ Q surface in a medium of dielectric constant 6. The
Ke
0
total electric flux over the surface will be
5q 6q
Charges situated outside the surface make id) ib)
6g 5 G
no contribution to electric flux. 0 0

ANSWERS

32. .{a.d) 33. {a,b,c,d) 34. {a,c) 35. {a) 36.(c) 37.{d) 38.(c)
1/122
“Px<uieep.'4, Fundamental Physics fXlI^roSTWl
15^ 1 1
(c) Zero (d) (a) — ib)
6 € 6 €
0 ^0 0

40. A cube of side 1 metre encloses a charge of I


coulomb in vacuum. What is the electric flux from 6 ^0
any one surface of the cube ? (c)
%

ng Matching Type Questions

DIRECTIONS. In each of the following questions, match column I and column II, and select the correct
match out of the four given choices.

41. Four charges Q^, Q2, and ^4 of same magnitude are fixed along the x axis at .r = - 2 a, - a, + a and + 2

ww
a. respectively. A positive charge q is placed on the positive y axis at a distance b>0. Four options of the
signs of these charges are given in List I. The direction of the forces on the charge q is given in List II. Match
List I with List II and select the correct answer using the code given below the lists.

Flo
FIGURE 1(CF).12

?{0. b)

ee
rere
rFF
uur r
Qi
{-2a, 0) foor
Q2
(-a. 0)
O3
(+0,0)
O4
(+2o, 0)
ks s
List-I List-II
Yoo
(A) (?]. ^3' Qa positive (p) +x
ook

(B) 0|. 02 positive; Qy 04 negative (q) -X


BBo

(C) 0|’ 04 positive; Q^, negative (r) +y


re

(D) 01. 03 positive ; 0j, 04 negative (^) -y (JEE Advanced 2014)


ouur

(o) A-r, B-/7. Cs. D-q {b) As, B-q, C-r, D-p (c) A-r, B-p, C-q, D-5 (d) As, B-q, C-p, D-r
ad
YY

Bi Matrix-Match Type Questions


nndd
Re

DIRECTIONS. This question contains statements given in two columns, _ p q_^ s


Fii

which have to be matched. The answers to these questions have to be


properly bubbled. If the correct matches are A-r, As ; B-p, B-^ ; C-p, C-r ''liS.S) lOjOi
and D-^, then the correctly bubbled matrix will look like the one shown
here :
8
o o Soil©!
42. Match Column I and Column II: ^©/©i©.©)
Column I Column II 0
© © :O.0
(a) unit of electric charge (p) Nm-C-'
(b) unit of electric intensity iq) coulomb
(c) unit of electric flux (r) stat coulomb
id) charges outside a closed surface (s) NC-'
(/) do not affect electric flux
ANSWERS

39. (a) 40.(b) 41.(a) 42. {a)-q,r ; {b)-s ; (c)~f); (d)-r


ELECTROSTATIC CHARGES AND FIELDS 1/123

V!. Integer Type Questions A B C D

0j-(P)l0j{0
DIRECTIONS. The answer to each of the following questions is a single digit
integer, ranging from 0 to 9. If the correct answers to the question numbers A, oooo
B, C and D (say) are 4, 0, 9 and 2 respectively, then the correct darkening of
bubbles should be as shown on the side : @ ©©@
43. An infinitely long uniform line charge distribution of charge © © ©©
FIGURE 1(CF).13
per unit length X lies parallel to y-axis in the Y-Z plane at z. @000
s ® © © ®
Z = — a. Fig. 1(CF).13. If the magnitude of the flux of —L—H
●/C
@000
a
2
the electric field through the rectangular surface ABCD
©©©©

ww
XL
lying in the x-y plane with its centre at the origin is —
0 ®. © ® ®
(6y permitivitly of free space), then the value of n is ©.©® ®

Floo
X

(JEE Advanced 2015)


FIGURE 1(CF).14

ee
44. Two charges of values 2 p C and - 50 p C are placed at a distance of 6 cm
from each other. The distance of the point (in cm) from the bigger charge

reer
rFF
where the electric intensity will be zero is :
45. A point charge q = \ C and mass 1 kg is projected with speed 10 m/s in the
uur r
perpendicular direction of uniform electric field £ = lOO V/m. The value R

of latus rectum of the path followed by charged particle (in metre) is :


46. An infinitely long solid cylinder of radius R has a uniform volume charge
ffoor RI2
P
.V
sks
density p. It has a spherical cavity of radius R/2 with its centre on the axis 2R
YYoo
ooko

of the cylinder, as shown in Fig. 1(CF).I4. The magnitude of the electrical x‘

field at the point P, which is at a distance 2 R from the axis of this cylinder
eBB

23pR . The value of k is


is given by the expression (IIT 2012)
I6ke 0
r

VII,
ouur
ad

Assertion-Reason Type Questions


YY

FOR MEDICAL STUPENTS 48. Assertion. When charges are shared between any
two bodies, no charge is really lost but some loss
nndd
Re

DIRECTIONS. In each of the following questions of energy does occur.


read the two statements and choose if
FFii

Reason. Some energy disappears in the form of


A. both. Assertion and Reason are true and the
heat, sparking etc.
Reason is correct explanation of the Assertion. (a) A (b) B (c) C (d) D
B. both. Assertion and Reason are true, but Reason
(AIIMS 2014)
is not the correct explanation of the Assertion.
C. Assertion is true, but the Reason is false.
49. Assertion. A charge q is placed on a height h/4
above the centre of a square of side b. The flux
D. both, Assertion and Reason are false.
associated with the square is independent of side
47. Assertion. Dielectric polarisation means formation length.
of positive and negative charges inside the dielectric. Reason. Gauss’s law is independent of size of
the Gaussian surface.
Reason. Free electron are formed in this process.
(a) A ib) B (c) C (d) D {a) A {b) B (£■) C ((/) D
(AIIMS 2014) (AIIMS 2015)
ANSWERS
43. 6 44. 5 45. 2 46. 6 47. (r) 48. (a) 49. (a)
1/124 <t Fundamental Physics (XII) LWI
50. Assertion. Insulators do not allow flow of current 53. Statement-1. The number of electrons in one
through themselves. coulomb is 6-25 x lO’^.
Reason. They have no free charge carriers. Statement-2, q - ne, where symbols have their
(a) A (b) B (c) C (d)D usual meaning.
(a) A (b) B (c) C (d) D
51. Assertion. If a point charge q is placed in front
54. Statement-1. Units of electric dipole moment are
of an infinite grounded conducting plane surface,
C-m and units of torque are N-m
the point charge will experience a force.
Statcment-2. p = q {2a) and x = force x distance
Reason. This force is due to the induced charge (a) A (b) B (c) C {d)D
on the conducting surface which is at zero
55. Statement-1. When charges are shared between
potential
any two bodies, no charge is really lost, but some
(a) A (b) B (c) C (if) D loss of energy does occur.
52. Assertion. Work done in moving a charge Statement-2. Some energy disappears in the form
between any two points in an electric field is of heat, sparking etc.

ww
independent of the path followed by the charge, (a) A (b) B (c) C {d)D
between these points. 56. Statement-1. The number of electric lines of force
Reason. Electrostatic forces are non conservative emanating from 1 p C charge in vacuum is

Flo
(a) A {b) B (c) C {d)D
M3 X 10®.
Statement-2. This follows from Gauss’s theorem

ee
FOR ENGINEERING STUDENTS in electrostatics,

rere
(a) A (b)B (c)C (t/)D

rFF
DIRECTIONS. In each of the following questions
read the two statements and choose if 57. Statement-1. For a charged particle moving from
point P to point Q, the net work done by an
uur r
A. Statement-1 is true ; Statement-2 is true ; State
ment-2 is correct explanation of Statement-1.
B. Statement-1 is true ; Statement-2 is true ;
foor electrostatic field on the particle is independent
of the path connecting point P to point Q.
ks s
Statement-2. The net work done by a conservative
Staiement-2 is not a correct explanation of
Yoo
Statement-1. force on an object moving along a closed loop is
ook

zero,
C. Statement-1 is correct and Statement-2 is false.
(a) A (b)B {c)C {d)D
BBo

D. Statement-1 is false and Statement-2 is true.


(AIEEE 2009)
re

A A o
ouur
ad

For Difficult Questions


YY

B’Multiple Choice Questions (with one Correct Answ«^


nndd
Re

1. Let us consider charge at A to be in equilibrium KQ^


FFii

Now ^BA - ^DA ~ 2


a

KQ^
and ^CA ~
{■yjla)'^ 2fl2
KQq ^K2Qq
Now ^AO ~
CV2o/2)2 a
2

for equilibrium, all forces along AO should be


equal to forces along CA,

f^AO = ^CA + ^BA cos 45° + cos 45°


^AO = ^CA 2 f^BA oos 45°
ANSWERS
50. (a) 51. (fl) 52. (c) 53. {a) 54. (a) 55. (a) 56. (a) 57, (b)
ELECTROSTATIC CHARGES AND FIELDS 1/125

FIGURE 1(CF).17
For Difficult Questions
A B
► 4
+ Q -Q
K2Qq Are\ 2*re\, 1

a” a~ V2 Qa »■- '4 Qb
r
>t

9 =
4
^ 0 30 ^ Q -30
2. Force on sphere C due to A.^ac = Force on sphere

C due to B. F BC ~
,r30V 30'I
Fig. 1(CF).16. k —
4ne^ (2R)- ’ / V 4 ^
r2

ww
£' = -
16 16

5. Here, £ = x 10^Vm"' v = 2x !0"^ms"',

Flo
7

Ti = 1-8 X 10”^ N sm"^ ; p - 900 kg m”^.

e
When drop is balanced in still air under the effect

rere
of electric field, then

r FF
4 1
^£ = ^7lr-^pg or = 3£
— Tcr^g ...(0
uurr
Now and Fq^ are inclined at 60® to each other,
therefore resultant force on C is
for
When the electric field is switched off, let the drop
fall with terminal velocity v, then
kss
i

2r^(p-g)g
ooook

9r)u 2
~ ^BC ^ ^AC ^BC COS 60®
Yo
V = or r =
9n 2(p-a)g
= Vf2 + £^ + 2f2cos60®
eBB

= V3£ [Taking = ^bc~^


1 4 9tiu 2
^ = -x-^^P^x
2(p-o)g
rr

1
ou
ad

7 4
'4n€(, 4 x-xjix900 x 9.8
Y

8171x10^ 3
dY

3. Net force on charge I C at the origin, 3

’9xl.8xl0~^x2xl0"3l2
Re

ICxl^iC ^ ICxl^iC ^ ICxlpC ^


innd

F = K .oo N X
l2 22 42 2x900x9.8
Fi

On solving, = 8*0 x 1(T*’ C


= 9x10^x10-^ 1 + 1 + —+ -L + ,00 N 6. As is clear from Fig. 1(CF).18,
4 16 64

In brackeit, we have geometrical progression


a
whose sum =
1-r

F =9x10^ = 9000x^3 ~ 12000N


L1-1/4J
4. Accordingto coulomb’s Law

F = -
r2
when 25% of charge of A is transferred to B, then
1/126 Fundamental Physics (XII) fTOMl

For Difficult Questions

tan0 = F _ kq- ...(/)


●>
mg X- mg

xJ2 X
Also, tan0 = ...(»)
2/
-jc^/4

w
X
From (0 and (//), —
-

X- mg 21

mgx^
C
=^=5

e
2kl 9. K =

row
C 6

re
0

dq
G =Xgo = 5x8-85x 10-‘-
oc 3^1/2 .V = 0-44 X 10-1® N-1 m"^

FFllo
F
dt dx dt 2
10. In a polar molecule, the centre of positive charge
does not coincide with the centre of negative

u
ree
As — is constant, therefore, v «
dt charge. Hence, the polar molecules have a
permanent electric dipole moment of their own.

sFr
1. If 9 is angle which each pith ball in equilibrium, makes
with the vertical, then as is clear from Fig. I (CF). 19. 11. From the knowledge of theory, electric field intensity

kro
(£) on axial line and equatorial line of electric dipole
uor
1
offo varies with distance /? as £ «
kos
Y
12. Here, total electric field, £ = £j+ E-,
Yo
eerBB
oo

P p * ^
OA + AP = ^ {OA+ AP)
rY

3e 3g 3e
0 0 0
u
ou
ad
do
nY

tan0 =
r/2 _ F _kq^lr^ mg
or y =
y mg mg 2kq^
nid
Re

or y OC
FFi

^. / \l/3 J/3 nI/3


r y y/2 1
r
\
y
● I ^ )
r
£ = -P-('^)
3e
= _£
3e
a

0 0

8. In Fig. 1(CF).20, let OP = y « a.


Net force on charge (- q/2) at P due to the two given Hence E is uniform and, both its magnitude
charges is and direction depend on a .
^nct = 2 £ cos 0, where ZAPO = ZBPO = 0 13. As lower and upper half carry equal and opposite
kq^ charges, hence electric field around the cylinder
__ 2 kg {q/2) y
will be identical to that of an electric dipole.
Choice (a) is correct.
"kqb 14. Let the net electric field due to two given charges
F nei ~, where y « a
— be zero at P, where OP = x.
a
ELECTROSTATIC CHARGES AND FIELDS 1/127

}. ●rrrr:;TT- 17. Here, charge Q is distributed uniformly over the


For Difficult Ouostions
surface of hollow metal sphere
(0 For r<R (inside the sphere)
FIGURE 1(CF).22 q = 0. Therefore, = Zero.
+8^ -2q P
» X FIGURE 1(CF).24
A B
Ebp Eap
(x = 0) (x = l)
L- t

w
I
/
\

For net electric field to be zero I


I
S

= B BP ~
K%q _ Kj^q) \

~ {x-Lf
\
S

e

row
1

re
2
or («) For r> R (outside the sphere)
^2 {x-L9 x x- L The charge behaves as if concentrated at the centre
or 7x-lL=x or X = 2L of the sphere.

eeF
ullo
15. Given, V=6.ry-y + 2yz !

FF
E or E OC —
oui
-> A 4ne„ r
out
r2
E=- k
Choice (b) is correct

srr
18. In Fig. 1(CF).25, at the centre of plates,

roF
5 A 0 A

k
£ = — —[6xy-y + 2yzli +—[^-^~y + ‘2-yz]j FIGURE 1(CF).25
3y
uor
+
B
ofof
— [6xy-y + 2yz]/: ■
^ 1
+ X I

2
kos
Y
Bz
Yo
eerBB

El
oo

= -{(6y)i + {6x-\ + 2z)j + {2y)k}


rY

E2
^.1,0 = - [(6 X1) / + (6 X1 - 1 + 2x 0) (2 X1) it ]
u

A A A

= -(6/ + 5y+2A) M- ♦1
ou

2 R
o
ad
d

16. Refer to Figs. 1(CF).23 (a) and (b), when point


Electric field due to line charge (1)
nY

charges + q and - ^ are displaced a little towards


positive x-direction, then + q charge will X A

E.I = i {NIC)
experience resisting force which will tend to bring
nid

2;t R
Re

it back to its original position. As a result of it the


Electric field due to line charge (2) is
charge + q will execute SHM. The charge - q will
FFi

X
experience a force in the direction of displace £■> = i {NIC)
ment, hence charge - q will move in that direction. 2n 6a0 R
Therefore, option (c) is true. X A X A

●●● E i +
net = £, -I- Ej - 2n €a0 R 2k 6a0 /?
FIGURE 1(CF).23
X A

i NIC
ne^R
X X X X
19. Due to spherically
E E E E symmetric charge
-►X -►X
-q
distribution, the charge
on the spherical
surface of radius r in
Fig. l(CF).26(when r
O O < R) is
1/128 ‘P'uicCee^ <» Fundamental Physics (XII) LWI
180“
For Difficult Questiona X.rA0sin0, a,
●> (-/)
4jte 0 r-

r f (S
, = jpdV' = j p„ 14- -—R) Anx’dx I A

0 -[-cos0|o (-/■)
47t6o r
\»-
5 X 4tc 5 1 TCr"*' 1
— [cos 180” - cos 0” J ]
- 7C -
3 /? 4 >0 R )
47ce«0 r
Electric field intensity at a point on this spherical
surface is
1 {qhzr) A A

i-i-ny =
E =
1 q 1 Pofs
x-^ -Jtr’
1 Ttr
4 > 4jte
0

47ten0 47t€ 0 rH3 R ) 21. In Fig. 1(CF).28,

w
dotted sphere of radius
. pQ^rS r' T] is the Gaussian
~4g^[3 r) surface. According to

Flo
Gauss’s theorem,
20. Let X be the linear charge density on the

eeee
semicircular ring. Then X = q!nr. Take an element Exit
of the ring of length A / as shown in Fig. 1 (CF).27.

Fr
^0
I
£ cos 0“ = — p (r) X dV

or
ur
sf ^0

Q\^ (4-nr,^3
£ (4 71 r,2) X1 = — 1
€(,71 £"*(3
ok
Yo

£ =
0.47irj'* - Qri^
Bo

3G(j n /?■* (4 Tc r,2) 37te „ /?**


22, Refer to Fig. 1 (CF).29 below.
re

AC=AD=a
ou

AE = AF = ^a'^ + (2af = -js


ad

Charge on element, dq = XAI= X (r A0).


Y

BC = BD = BE=BF =-Jz a
Electric field intensity at O due to charge on
nd

FIGURE 1(CF).29
1 XrA0 , ..
Re

element is dE = --— along OA


4 Tie 0 Side 1 2a
Fi

Ya
I
I

Two rectangular components of dE ared£cos0


along .v-direction and dE sin 0 along negative
y-direction. If we find the electric field at O due
2a
Xb \
to charge on all other elements of the ring and
resolve them into two rectangular components,
we note that the components along X-direction
cancel out in pairs and components along negative
y-direction add up. Therefore, total electric field Potential at point A is
intensity at O due to charge on the entire ^ 1 ,
semicircular irng is ''^=2 4ne^0 aa
i'l_2r_!
4nef.a-JS
180“ y V ” j

£= J d£sin0(-_;) 1 2<?(.1- 1 )
0 471E/V0 a I sr
ELECTROSTATIC CHARGES AND FIELDS 1/129

According to Gauss’s law,


For Difficult Questions

<1) = — or q=eQ(^
Potential at point B is ^0
q= A (4 n a^) - 4 K A.
V =2f-^ ^ 2f_! 1 = 0
26. For infinitely charged plane sheet, electric field
« 471^0 ^ a
4ns„ V2 a

1 'i is given by £ = —
I 2q 2e 0
1-
4 Tie
0
a V5
K.E. at the centre of square of charge Q Due to charge + o, E, = — . downwards
‘ 2e^0
= Work done on charge Q from A to 5 is

W=QiV^-Vs) =
1 2qQ( 1- 1
Due to charge - 2 a, =
2<t
, downwards
[ Vs

ww
4TC€ 0 a 2e
0

23. As is known from theory o

Due to charge, “ ^3 “ ^ ’
downwards
qh

Flo
E = 0

4tC€o {/l2+£2)3/2 Total electric field at P,

e
2<y a 2a

reree
dE
E will be maximum, when = 0. , downwards
dh 2e 2e 2e
0 ^0

r FF
0 0

d qh
uurr
= 0

foor
or
^0
dh [4jc Eg (/l2 +£2)3/2 27. Now charge on disc contributing to electric flux
6C
ks s
q -3h^ through cubical surface = — = 3C
+ (/l2 + £2)-3/2 =0
Yoo
or
2
4n e (/j2 +£2)5/2
ooook

0 L
Charge on rod contributing to electric flux through
3/j2 8C
eBB

= 2C
or = (/i2 + £2)-3/2 cubical surface =
4
(/l2 + £2)5/2
-1C charge also contributes to electric flux but
rr

£2 £ 3 C charge is outside the cubical surface, so it


ouu

3/j2 = /i2 + £2 /l2 = or h = +


ad

or or
2 V2 does not contribute to electric flux.
Therefore, ^enclosed = 3C + 2C-7C = -2C
YY

Hence, choice (b) is correct


24. The three charges are placed as shown in Fig. ^enclosed _
Hence electric flux, =
nndd

1(CF).30.
Re

^0 ^0
FIGURE 1(CF).30
Fii

28. Here, electrostatic force = gravitational force


+ Q Q -Q
k (e +Ae)2 G m,, m,,
x = 0 x = dl2 x = d d'^

As net force on + Q charge at x = 0 is zero, therefore, - G muniL 6-7X10-" (1-67x10-27)2


(e + Ae)2 = ^ =
k 9xi0^
or kQq ^ KQQ = 0 or q ^ Q 6-7 X 1-67x1-67
{d!2)^ d- d^l4 d^ xlO-7^
9
or q-~ Qf4 e + Ae = (2-076 x lO"^^)'^ = 1-44 x
25. Flux linked with sphere = E dV Hence Ae is of the order of 10"^^ C
A

Since electric field is radial. It is always 29. Here. E = 0


x
perpendicular to the surface of the sphere. A

So 4) = £A cos 0“ = £4 = Ar (4 Jt r2) Let x^ y and z be the unit vectors along x, y


(J) = A 4 71 (as r - a) and z axis respectively.
1/130
‘Pt<uUe^ Fundamental Physics (XII) EZSX9]

For Difficult Questions


The net electric flux crossing the given region,
according to Gauss’s theorem is
Refer to Fig. 1(CF).31, the shaded area, J.q _ ~q+2>ci-q _ q
^0 ^0 ^0
Choice (c) is correct.
Again, as + 3 is at the origin 0. and charges
- <y, - 9 are placed symmetrically about the planes
z = + all and = - all so net electric flux crossing
these planes is equal. Choice (</) Is correct.
The only wrong option is {b).
31. Here, = -T,

ww
- —*
^ ^

A= PQx PS From X = pxE


A A A A A A

A = (0.t+ay + 0z)x(nx+Oy+nz)
..A _ A

= {a^x-a- z)

Flo
e
Electric flux through the shaded area.

eree
<^= E.A

FFr
30. In the Fig. 1(CF).10,
uurr
we have shown a cubical region of side a.

orr
—» —» A A r~
It encloses three fixed point charges : '^PyJ +V3£; )
- ^ at A (0, - fl/4, 0), + 3 (? at 0 (0, 0, 0) and
sfo
- .7 at S (0, (7/4, 0) x-JS E k + p^, E {-k)
= Px
kks
Yoo
As + 3 7 is at the origin O, and charges - q,- q
0=Ek (V3 p^ - p^.)
oooo

are placed symmetrically about planes a = + all


and A= - all, therefore, electric flux through them ^ = V3 i.e.. tane = ^ 0 = 60'
eBB

or
is same. Choice (a) is correct Px

mD
urr

Multiple Choice Questions (with One or More than One Correct Answers);
ad

34. Electric field at point (0, 0, Zq)


Yo

32. Number of field lines from is greater than that


dY

from Q2. Therefore IC] I > I G2 ●● Also, as electric KQZ^ 0


field is less for Q2, at finite distance to the right i.e.. E =
{/?-+Z2)3/2 FIGURE 1(CF).33
innd

of Q2, electric field would be zero.


Re

33. In a conductor, electric field is zero. Therefore, P(-q) (0. 0, 2o)


E acts away from 0
Fi

E^ inside = Zero
KQ H Force on (- q) at P
After connection, V^ = Vg ^ +Q
R -f(QqZ^
B F=-qE =
(/?2+z2)3/2
0
As > Rg > Qg
QB R
B F acts towards O.
So, motion of particle P is periodic (for 0 < Z(, < «>) for
O Ra QB c
A _
R
B
« R, F oc Zq and such motion is approximately
A ~
AkEqR^I’ Ans^Rgl ^B ^A simple harmonic.

on surface Rg 35. Let us consider a sphere of radius r and thickness


Eg on surface Og/e ^B dr lying in the region between two concentric
spheres.
As Rg<Rj^
According to Gauss’s theorem, charge inside
on surface < Eg on surface alone contributes to electric field/flux.
ELECTROSTATIC CHARGES AND FIELDS 1/131

X X
For Difficult Queiitions
^2^'b> = ; ^2(^o/2) =
27C60/5 TlSgr^,
b
A

kQ
k Q+
a
4-Kr'^dr—
r ^3<'b> = ^
Z€o
; £3 ('0^2)=^
a />2
a Given, E^ (rg) = E2 {r^) ~ £3 (rg)
Q X o

\r^t = Q + 4nA
ib^-a^)
or Q— = Q + 4nA —
L 2 2 4n&f^r^ 2e 0
X X
(f.2 ^ Hence, Q = 2kc and = o or K
0 "
Q —-1 =2nAib^ «2) 71 ,r
0
Tta

Q{b^-a'^) = 2iiA{b^ -a^) fi|fY]='»e|('b) = 2£-2('b28)

ww
Q
2
A=
a 27Cfl^
'b
Also, £, = 2£o(ro) = 2£3(V2)

Floo
Q 4Q 2 2
36. Here, £j (tq) = £,(ro/2) =
4neQrf
\

47cey Thus, option (c) is correct.

ee
on Multiple

eer
Choice Questions (Based on the given Passage/Comprehens ion)

rFrF
^ci 2q-5q + Sg_ 5g
39. (1)^ =
rur
I.q _+6q-2q + 3q _ 6^0 6€o
37. =
^0 ^0 ^0 ffoor 0

40. Total electric flux from all the surfaces of the cube
s
7^
= _£_ = _L
osk
38. (I)^ = . This is because charges present
YYoou
^0 ^0 ^0
oook

outside the closed surface do not affect the electric As the cube has six faces, therefore, electric flux
eBB

flux.
1
from any one surface of the cube =
6 6q
09
r

Matching Type Questions


ouur
ad
Y
nd dY

9 9,
41. (A)
Re

(B)
FFini

+Qi +Q2 +0.3 +04 +01 +02 -03 “04


Resultant force along + y direction Resultant force along + x direction

(C) (D)

+01 -02 -03 +04 +01 -02 +03 -04


Resultant force along -y direction Resultant force along - .r direction
Thus correct option is (a).
1/132 4 Fundamental Physics (XII) PZgTWl

For Difficult Questions

ei Matrix-Match Type Questions

42. SI unit of electric charge is coulomb and electrostatic unit of charge is stat coulomb. NC"* is the unit of

electric intensity. Unit of electric flux ^ is Nm^ C"*, charges present outside a closed surface do not
affect the electric flux over the surface.

VI.
Integer Type Questions

w
43. Here, E =
X X X ^all
Ldx

Flo
or =
2 Tie
o''
2jie
2 3fl^
x^+
2ne
0 .t2
+ 1
3u2
OV

eeee
4 3a^/4 4

d(j) = £ sin 6 ● A cos 0 = £ sin 0 ● A

Fr
X ■j3a/2
X Sa/2 Ldx

3u2
X

3<j2
xLdx 2;t€
0 x^
+ 1
irv3uf

or
ur
2ne«.. x^ + ^2 + 3fl2/4 2
0^ 4 4
sf
a/2
iXSa/DLdx (|) = d(j) =
X 1
Ldx
ok
^2
Yo
3fl2 2k€ 0 a
-all + 1
2ji e 0 ;c2 +
3fl2/4 2
Bo

...(/)
re

X
X V3 a
2 Let X =

Sa/2 ^
or
Ldx
ou

27ce 2 3fl2
ad

0
+
4 dx
Y

or dx = -dy or ==dy
V3o/2
FIGURE 1(CF).34
nd

+ 1/-^
Re

XL dy
From (0, 0 =
0 -1/-^ y2 +1
Fi

2tC€
r

jc = -^ corresponds to y = -^
Z Z7 x = + -2 corresponds to y = -1=
Esin 6 v3
a
from j: =

X Sa/2 XL 1
Ldx -1
x2tan
2n € 0 2 +
3fl2 2Jte 0
4 3fl2
X
XL
3fl2 4
x2x K _ ^L
4 2jce 6 6g
0 0
ELECTROSTATIC CHARGES AND FIELDS 1/133

/. y^ = lx
For Difficult Questions
which is the equation of a parabola. Compare with
XL the standard equation of parabola : = 4 or, we
Comparing with i|) = , we get n = 6 find 4 a s 2 m, which is the length of latus rectum
0 of parabola.
44. In Fig. 1(CF).35, let the electric intensity be zero 46. Due to symmetry of charge distribution, the elec
at P, where BP -x cm and AP = (6 - a) cm. tric field at a point outside the cylinder is Ir to
—►

FIGURE 1(CF).38 theaxisofcylinder, outwards. Let f, be the elec


2pc -50pc
tric field intensity at P due to charge on solid
As- B
P4- cylinder.
X
♦I
Draw a cylindrical gaussian surface of radius 2 R

ooww
14 6 cm 41
and height R as shown in Fig. 1(CF).37, so that
point P lies on the curved surface of gaussian
JLSL-Lh. cylinder.
AP^ BP^
FIGURE 1(CF).37

e
2 50

ree
(6-x)2 x^

rFl
Fre
I 5 ds

rF
6-a: X
E
AT = 30 - 5 A

ffoor
ouur
2R
6 a: = 30, AT =: 5 cm R r4 4 P_^Ei
ks s
45. Here, ^ = I C, m = 1 kg ds
u= I0m/s,£= 100 V/m.
oook
Yo
In Fig. I(CF).36, E is along OX and velocity v is
Y
BBo

along OY.
rr e
ouu

Electric flux over the gaussian cylindrical surface


ad
Y

f 1
dY

ss ® E^-ds = — xcharge enclosed by the surface


^0
innd
Re

1
FFi

Let P (a:, y) be the position of point charge at any 1


time t. = — kR^ ^
^0
y =v t

V y
or or E.=—
I (0
^0 4kR^ 46 0
V 10

Acceleration along X-axis


Charge on cavity of radius R/2,
qE 1x100 ,2
=-— = —:— = 100m/.s-^
m
q = ~n(R/2)^p ^nR^ p/6
A ^ 2 1 , ry f y^
As A = — a,r‘ AT = -i-xl00/^=50 Electric field intensity at P due to charge on
2 ^ 2 10 2
spherical cavity is
1/134 Fundamental Physics (XII)BSm

Hence net electric field, E-E^-E-y


For Difficult Questions

/?p /?p _ 23p/?


1 H 1 7i/?^p/6 4e
0
96e
0
96e
0
^2 = X

4tc€q (2/?)^ 4jc€ 0 4/?2


As per question, =
_ 23p/? _ 23p/?
96e 16/:€
__Rp ...(«)
0 0

96e 0 16 /: = 96 or /: = 6

VII. Assertion-Reason Type Questions

ooww
FOR MEDICAL STUDENTS
1 <i
Electric flux through one face = —
47. When the dielectric is subjected to an electric 6€
0
field, its molecules get polarised, i.e., the centre
which is independent of side length b.
of gravity of positive and negative charged

e
molecules get separated from each other. Due to 50. Insulators do not have free charge carriers. That

ree
it, a net negative charge appears on one face and is why flow of current through them is not

rFl possible.

Fre
equivalent positive charge appears on opposite
face of dielectric. Hence, Assertion Is true. Here, 51. The conducting surface being connected to earth

rrF
Reason is false as no free electron is produced in is at zero potential. The point charge in front
this process. induces an equal and opposite charge on the
ouur
48. Both, Assertion and Reason are true and Reason
is the correct explanation of Assertion, because
on sharing the charges by two bodies, there is
sffoo conducting surface. This gives ri.se to force of
attraction.
okks
52. Assertion is true, but reason is false. Electrostatic
conservation of charge, but there is .some loss of
Yo
forces are rather conservative forces.
ooo

energy in the form of heat.


Y

FOR ENGINEERING STUDENTS


BB

49. Here, both Assertion and Reason are true and


Reason is the correct explanation of the Assertion
53. From q = ne.
rr e

because electric flux linked with a gaussian


surface is 1C
ouu

n = — = 6-25x10’^
ad
Y

1 e l-6xlO“‘^C
= () E.ds
dY

^0 Both the statements are true and Statement-2 is

X charge enclosed by the surface correct explanation of the Statement-1.


innd
Re

If a charge q is placed at the centre 0 of the cube 54. Both the statements are true and Statement-2 is
then electric flux through all the six faces of the correct explanation of the Statement-1.
FFi

cube is = — . 55. Charge is conserved. Energy too is conserved, if


^0 we were to take into account the loss of energy in
the form of heat etc.
FIGURE 1(CF).38
56. From Gauss’s theorem in electrostatics.

-6
IxlO
/j/4 = M3xl0^
-12
^0 8-85x10

Both the statements are true and statement 2 is


a
correct explanation of statement 1.
57. Statement-1 and 2 both are true, but Statement-2

■4-
is not a correct explanation of statement-1.
b
ELECTROSTATIC
POTENTIAL AND CAPACITANCE

w
2.1. INTRODUCTION

Flo
In the previous chapter, we have learnt the concept of electric field due to a point charge and due to a

ee
distribution of charges. It is represented in terms of a vector quantity E , called electric field intensity. In this
chapter, we shall learn that electric field can also be represented in terms of a .scalar quantity V, called

Fr
electrostatic potential. This concept is of great practical value.
Basically, electrostatic potential of a charged body represents the degree of electrificationof the

for
ur
body. It determines the direction of flow of charge between two charged bodies placed in contact with eachother.
The charge always flows from a body at higher potential to another body at lower potential. The flow of
charge stops as soon as the potentials of the two bodies become equal.
ks
Yo
Further, we studied Coulomb’s law for calculating electric field intensity. In this chapter, an alternative
oo

law called Gauss's Law is described for calculating electric field intensity of highly symmetric charge
distributions. This law enables us to understand more complicated problems in electrostatics.
eB

2.2. ELECTROSTATIC POTENTIAL ENERGY


r

Like the potential energy of a mass in a gravitational field, we can define electrostatic potential energy
ou
ad

of a charge in an electrostatic field,


Y

^r the sake of simplicity, let us assume that electrostatic


field E is due to charge + Q placed at the origin O. Let a small
nd
Re

test charge + be brought from a point A to a point B, against


the repulsive force on it due to charge + Q, Fig. 2.1. Wc shall
Fi

assume that the test charge + ^ is so small that it does not disturb
the configuration of charge + Q at the origin. Further, we assume
that an external force applied is just sufficient to counter

the repulsive electric force F^ on the test charge q so that net


force on test charge q is zero and it moves from AtoB without
any acceleration.
In this situation, work done by external force is negative of work done by the electric force and gets
fully stored in the charge q in the form of its potential energy.
On reaching if the external force applied on q were removed, the electric force will take the test
charge + q away from source charge + Q. The stored potential energy at B is used to provide kinetic energy to
the charge q in such a way that the sum of kinetic and potential energies at every point is conserved.
2/1
2/2 Fundamental Physics (XII)BSm

Work done by external force in moving the test charge + q from A to B through a distance AB = dr is

w
AB =1^, ext'
d? = -\K,d7
E
d)
A A

This work done against electrostatic force gets stored as potential energy.
Note that charge q possesses a certain amount of electrostatic potential energy at every point in the
electric field. Therefore, work done in moving the charge q from A to fl increases its potential energy by an
amount equal to electrostatic potential energy difference between points B and A

i.e.. AU^Us-^a-^ab ...(2)

Hence, we define,

ww
Electrostatic potential energy difference between two points B and A as the minimum work required
to be done by an external force in moving without acceleration a test charge q from A to B.
This is true for electrostatic field of any charge configuration.

Floo
Note that work done by an electrostatic field in moving a given charge from one point to another

ee
depends only on the positions of initial and final points. It does not depend on the path chosen in going
from one point to the other.

reer
rFF
Eqn. (2) defines electrostatic potential energy difference in terms of physically meaningful quantity
‘work’. Therefore, it is only the difference of potential energy which is significant. The actual value of potential
uur r
energy at a point is not relevant. Thus, there is a freedom in choosing the point where potential energy is zero.
ffoor
For a charge distribution of finite extent, we choose zero electrostatic potential energy at infinity. Therefore,
sks
when point A is at infinity, we get from (2)
YYoo
.(3)
ooko

Hence we may define


eBB

Electrostatic Potential energy of a charge q at a point (B) in an electrostatic field due to any
charge configuration as the work done by the external force (equal and opposite to electric
r
ouur

force) in bringing the charge q from infinity to that point without any acceleration.
ad
Y

23. ELECTROSTATIC POTENTIAL


dY

Electrostatic potential is characteristic of electric field associated with a given charge configuration.
Re
nnd

To make this characteristic independent of the magnitude of te.st charge, we define


FFii

Electric potential difference between two points B and A in an electrostatic field as the amount
of work done in carrying unit positive test charge without acceleration from A to B (against the
electrostatic force of the field) along any path between the two points.
Fig. 2.2 shows that work done on a test charge q
by the electrostatic field due to any given charge
configuration depends only on the initial point A and
final point B. Work done is independent of the path
chosen in going from A to B.
If V4 and Vg are the electrostatic potentials at A
and B respectively, then from eqn. (2),

^B-^A^^AB
-Vg-VA^AV ...(4)
ELECTROSTATIC POTENTIAL AND CAPACITANCE 2/3

The dimensional formula for electrostatic potential difference is

AV =
V/
AB _ [ML2t-2]
9 [AT]
SI unit of electrostatic potential difference is volt. That is why potential difference is sometimes referred
to as voltage:
IJ
1V = = 1 JC"' =1 NmC“‘
1C

Thus, Electrostatic Potential difference between any two points in an electrostatic field is said to
be one volt, when one joule of work is done in moving a positive charge of one coulomb from
one point to the other against the electrostatic force of the field without any acceleration.
As explained already in case of electrostatic potential energy, it is potential difference between any two
points, which is physically significant,... and not the actual value of potential at a point.

w
We can, however, define electric potential at a point by choosing the potential to be zero at infinity. If
we take point A at infinity, = 0. From (4),

Flo
W
__ ' OOjS

e
...(5)

rree
q

r FF
■ Hence, Electrostatic potential at any point in a region of electrostatic field is the minimum work
● done in carrying a unit positive charge (without acceleration) from infinity to that point.
uurr
for
Obviously, electrostatic potential at a point is a scalar quantity. Its SI unit is volt
Electrostatic potential at a point is said to be one volt, when one joule of work is done in moving one
kkss
coulomb of positive charge from infinity to that point against the electrostatic force of the field without
Yo
oooo

acceleration.

l^ampi_51g'fgSiomll Jhe potential difference between two points is 10 volt. How much work
eB

will be done in carrying a charge of 200 |i C from one point to the other ?
Sol. As work done = charge x potential
rr

W= (200 X 10"^) X 10 = 2 X 10-3 J


ou
ad
YY

2.4. ELECTROSTATIC FORCES ARE CONSERVATIVE


To prove that electrostaticforces are conservativein nature, we show that work done in moving a unit
nndd
Re

positive test charge over a closed path in an electric field is zero. We know that along any path L, Fig. 2.3,
work done in carrying unit positive charge from A to B.
Fi

WAB

%
Similarly, work done in carrying unit positive charge from B to
A along any other path V
W
BA

%
Adding the two, we get total work done in carrying unit positive charge over a closed path A ^ -> A is
, '^BA '^ABA = (V's-K4) + (K4-V'B) = Zero
% % ^0
Hence, no work is done in moving a unit positive test charge over a closed path in an electric field.
Hence, electrostatic field is a conservative field and electrostatic forces are conservative forces in nature.
2/4 ^>usuiCee^ <t Fundamental Physics (XII) OTSTWI

Mathematically, we can write this result as o £.d/ = 0 .(6)

/>. line integral of electric field over a closed path in the electric field is always zero.

2.5. ELECTROSTATIC POTENTIAL DUE TO A POINT CHARGE

Suppose we have to calculate electrostatic


potential at any point P due to a single point charge
+ ^ at O : where OP - r, Fig. 2.4.
By definition, electro.static potential at P is the
amount of work done in carrying a unit positive charge
from <» to P.

An work done is independent of the path, we

ww
choose a convenient path along the radial direction from
infinity to the point P without acceleration.
At some intermediate point A on this path, where

Floo
OA = X, the elecUxtstatic force on unit positive charge is

ee
1
£ = along OA produced ...(7)
4K€^X^

reer
rFF
Small amount of work done in moving a unit positive charge from A to £ where
uur r
AB = dx ‘s

dW = E.dx^Edx cos m° = -Edx


ffoor ...(8)
sks
YYoo
Total work done in moving unit + charge from oo
to the point P is
ooko

iT
r
1
x-2 dx
w - ^
= J- Edx^
q
eBB

w —
4tC€ 0 x^ 47te^0 ●' 4neQ L jc
q
r
ouur
ad

47ce 0 L r
47t€() rOO

By definition, this is the electrostatic potential at P due to charge q


YY
nndd

...(9)
Re

47i:eQ r
FFii

When q is positive, electro.static potential V is po.sitive and when q is negative, electrostatic potential V
is negative, i.e.. A positive charge produces a positive electrostatic potential and a negative charge produces
a negative electro.static potential.

At r — oo.
OO

i.e.. electrostatic potential is zero at infinity.


Eqn. (9) shows that at equal distances from a point charge
q, value of V is same. Hence, electrostatic potential due to a
single charge is spherically symmetric.
Fig. 2.5 shows the variation of electrostatic potential V
1
with distance. Le., V PC — ; and also the variation of electric
r
1
intensity E with distance, i.e., E OC —— ,

r2
ELECTROSTATIC POTENTIAL AND CAPACITANCE 2/5

If the charge is negative, the variation of £:and Vwith r is shown in Fig. 2.6.

w
Again, the variation of electric potential Vwith (\/r) is shown in Fig.2.7(fl). When 0 > 0, i.e., charge is
positive. The same variation is shown in Fig.2.7(/?), when Q<0, U., charge is negative.

Flo
Sample Problem What IS the electrostatic potential at the surface of a gold nucleus of
diameter 13-2 fermi. Z for gold is 79 ?

reeee
!3-2
Sol. Here, T=? , r= — =-6-6 fermi = 6-6 x 10 -15 m, q=Ze~19x l-6x Kr'^C

FFr
V =
7 9x10^x79x1-6x10-'^
= 1-7 X 10^ volL
4ns 6-6x10-'^
for
ur
o'*
kkss
2.6. POTENTIAL AT A POINT DUE TO GROUP OF ELECTRIC CHARGES
Yo
Suppose there are a number of point charges ^2*
oo

FIGURE 2.8
73’ 7n distances r,, r2, r^, respectively from
eB

the point P, where electric potential is to be calculated. r


Fig. 2.8. 1
●■2 ^2
Now, potential at P due to charge
r

'’3
ou
ad

p.
'^3
1 ^1
YY

V.1 =
-■s
47teg r,
^’4
ndd

Similarly, values of potential at P due to other charges


Re

are
Fi

I I ^3
^2 = 47TG ,^3 = , and so on V/[ = 7,
...(10)
0 '2 4tI6o
Using superposition principle, we obtain resultant potential at P due to total charge configuration as the
algebraic sum of the potentials due to individual charges.
V=V'i + V2+V’3+....
1 7| 1 1
72 73 1 7n II
+

0 [ 1^ ^2 'l
4ns
0 'i 4ne„ rj 47ie„ 4nSn0 rn 47te r
II

1 i=n ^

4ne
X ^ .Ml)
0 /=i 7
2/6 Fundamental Physics (XII) iwi

If r,.r2,r3. ^ are position vectors of the charges ^3» ●— respectively, then electrostatic
potential at point P, whose position vector is ^ would be
1
V =
47t€
I ...(12)
» 0 <
I

Note that if we have to calculate electric potential due to a continuous charge distribution characterised
by volume charge density p (r), we divide the entire volume into a large number of small volume elements,
each of size A V. Charge on each element = p AV. Calculate potential due to each volume element and sum (or

ooww
integrate) over all such contributions.
2.7. ELECTROSTATIC POTENTIAL AT A POINT DUE TO AN ELECTRIC DIPOLE
Let an electric dipole consist of two equal and unlike point charges - ^ at A and + ^ at B, separated by a

e
small distance AB - 2a, with centre at O. The dipole moment

ere
rFl
...(13)
I ^ I = ^ X 2a
FIGURE 2.9

Fre
Let us take the origin at the centre of the dipole. We have to calculate r'l

rF
i!\
/ / I

electric potential at any point P where OP = ~r and ZBOP = 0, Fig. 2.9.


/ /
I
/ /
I

foor
ouur
/ I
/ / I

Let the distance of P from charge - ^ at A be r,, i.e., AP - r, and / /


I
sksf I

distance of P from charge + ^ at B be T2, i.e., BP = r2-


/ I
/

W ' t

Electrostatic Potential at P due to - ^ charge at A / '^2


oko
I
I r
Yo
/
/ / I
/ / (
Y
-(J / I t
BBo

V.I = / / I

4ns„ ,r
/ I I
/ / I
/ I I
rre

and Electrostatic potential at P due to + ^ charge at B (-q) / i(+q)


A*
Q
ouu

M- ●H
Y

2a
ad

0 '2
dY

Therefore, applying superposition principle


potential at P due to the dipole,
innd
Re

V= V2+V|
FFi

1 1
V=^ ...(16)
4jcGo
Now, by geometry, 'i ^ + 2 ar cos 0, and

= r^ + a^ + 2ar cos (18O®-0) = - 2 ar cos 0

We may rewrite r?1 - r 2 1 + it- + ?£ cos 0


r2 r

a ^2
If a < < r, — is small, —r can be neglected.
r r^
2a
r?=r^
I
1+ COS0
r
ELECTROSTATIC POTENTIAL AND CAPACITANCE 2/7

nI/2
2a
r\ = r 1 + —
r
cos 0
n-1/2
1 1 2a
1 + COS0
r

x-l/2
i (,1
-
2a cos 9
Similarly,
^2 ''I /●

-1/2 x-l/2
1 2a 1 f 2a
Putting these values in (16), we get V = —— 1- cos 0 1 + — cos 0
4k€ r r
0 L'-V r

Using Binomial theorem and retaining terms upto the first order in a/r, we get

ww
V= ^— 1
a a
+ —COS0- 1- COS0
4 7C r[ r r
Jj

Flo
a a
1 + — cos 9 -1 + — cos 0

ee
4Tt£^, r , r r

rere
rFF
V =
qx2a cos 0
4Tie,,0
uur r
V =
/>cos0
4tc€„
ffoor ...(15)
sks
0
Yoo
Asp cos 0 = p-r , where r is unit vector along the position vector Qp = ~r ,
oko

Electrostatic potential at P due to a short dipole (a < < r) is


BBo
re

...(16)
4ti€q r2
ouur
ad

P
YY

On the dipole axis. 0 = 0° or 71 V =±


4 TIG
0
r2
nndd

Positive sign for 0 = 0° and negative sign for 0 = ti.


Re

71
FFii

At any point on the equatorial plane, 0 = — : cos 0 - cos — = 0 V=0


3 2

i.e., electrostatic potential at any point on the equatorial plane of dipole is zero.
Retain in Memory
When we compare electrostatic potential due to a point charge with electrostatic potential due to
a dipole, we find that
(0 whereas potential due to point charge varies inversely as the distance from the charge, i.e.,
1
Voc- , potential due to dipole falls off more rapidly, i.e., V oc _, i.e., electrostatic potential
r

varies inversely as the square of distance from the centre of the dipole.
(«) The potential due to a dipole depends not only on r, but also on the angle between the position
vector and dipole moment vector p .
2/8 “P^uicUe^ 4 Fundamental Physics (XII)
2.8. EQUIPOTENTIAL SURFACES

An equipotential surface is that surface at every point of which electric potential is the same.
By definition, potential difference between two points B and A = work done in carrying unit positive test
charge from A to B, i.e., Vg- = W^g
If points A and B lie on an equipotential surface, then Vg =
Hence, no work is done in moving the test charge from one point of equipotential surface to the
Other.
If dl is the small distance over the equipotential surface through which unit positive charge is carried,
then dW= ~E.dl =£d/cos6 = 0 /. cos 0 = 0 or 0 = 90°, i.e., E ^.dl

Electric field intensity ~E is always normal to the equipotential surface, i.e., for any charge

ww
configuration, equipotential surface through a point is normal to the electric field at that point,
q
For a single charge q, the potential is given by V =
471^0 r

Flo
This shows that V is constant if r is constant. Hence, equipotential surfaces of a single point charge

ee
are concentric spherical surfaces centred at the charge, as shown in Fig. 2.10(a) by solid curves.

rere
For a uniform electric field, say. along the X-axis, the equipotential surfaces are planes normal to the

rFF
X-axis, i.e., planes parallel to the Y-Z plane as shown in Fig. 2.10(/>).
uurr
foor
ks s
Yoo
ook
BBo
re
ouur
ad
YY
nndd
Re
Fii

The equipotential surfaces for an electric dipole and for two identical positive charges are shown in Fig.
2.10(c) and Fig. 2.10(d) respectively.
The drawings of equipotential surfaces give us an alternative visual picture of electric field around a
charge configuration.
ELECTROSTATIC POTENTIAL AND CAPACITANCE 2/9

Some of the important properties of equipotential surfaces are :


{/) No work is done in moving a test charge from one point to another point on an equipotential
surface.
(i7) Electric field is always perpendicular to the equipotential surface.
(iii) No two equipotential surfaces can intersect eachother.
(/V) Equipotential surfaces are closer in regions of strong electric field and they are farther apart in the
regions of weak electric fields.

2,9. RELATION BETWEEN ELECTRIC INTENSITY AND ELECTRIC POTENTIAL

w
Fig. 2.11 shows cross sections of a set of equipotential surfaces. Let the potential difference between each
pair of adjacent surfaces be cIV. Lxt a small test charge (+ q) at P move through a distance dr from one equipotential
surface to the other. The electric field E at P is perpendicular to the equipotential surface through point P.

e
roow
Work done by electric field on the lest charge

re
= -qdV ...(/)

(Neg. sign for decrease in potential in the direction of £ )

reF
uFFll
e
Also, this work done = F.dr = q E.dr

sFr
= q E dr cos 6 ...(«)

foro
Here, 6 is angle between E und dr .
ofk
uor
From (0 and (ii).
kos
dV
- q dV = q E dr cos 6 or £ cos 0 = -
YYo
dr
reeBB
oo

Now, E cos 9 is component of £ in the direction of dr .


uurY

dV
If dr is along ,y-axis, we may write £ .(17)
dr
ad
doo

Negative sign shows that the direction of electric field E is the direction of decreasing potential.
nY

dV dV dV
It implies that £^ = , £ and £, =
nid

dz
Re

^ dx y dy
Further, the magnitude of electric field is given by change in magnitude of potential per unit
FFi

displacement normal to the equipotential surface at the point. This is called potential gradient, ie.,
\dV\
I £l = - = - (potential gradient)
dr

Retain in Memory
1. In the region of strong electric fields, equipotential surfaces are close together and in the region
of weak electric fields, equipotential surfaces are far apart. This follows from
dV dV 1
£ = “ ,i.e.,dr = -- or dr oc

dr E

2. Note that electric potential is a scalar quantity, but electric potential gradient is a vector, as it is
numerically equal to electric field intensity.
2/10
'P>u^(!Uef.i'<i. Fundamental Physics (XII) VOL. I

r^;Smi?lolf?rqbibm || A uniform electric field of 10 N/C exists in the vertically downward


direction. Find the increase in the electric potential as one goes up through a height of 50 cm.
Sol. Here, £■ = - 10 N/C (neg. sign for downward direction)
1
dV = ? = 50 cm = — m.
2

dW 1
As £ = - dV = -Edr = -(-iO)x- = 5 volt
dr

0 The electric potential V at any point (x, y, z) in space is given by V=4


volt. Calculate electric intensity at the point (1 m, 0, 2 m).
Sol. Here, V = 4x^, E=l,x=lm,y = 0, z = 2m
dV
E = -
8x = -8(l) = -8Vm * along negative direction of A;-axis.

w
Curiosity Question

Flo
f Q. What do you understand by EGG and EEG ? What Is their basis ?

reeee
Ans. EGG stands for eiectro-cardio graph and EEG stands for electro-encephalo graph.

FFr
Both are based on the fact that human body is a good conductor of eiectricity, but not a perfect
conductor. Therefore, muscie activity leads to smaii differences in electric potential from one
point of the body to another.

for
As human heartbeats, potential differences in the range of 1 miili voit deveiop. These potential
ur
differences are detected and dispiayed with an instrument known as EGG.
kkss
Further, nerve impulses in the brain iead to potentiai differences in the range of 0-1 miilivoit.
They are detected and displayed with an instrument known as EEG.
J
Yo
oo

2.10. EXPRESSION FOR ELECTROSTATIC POTENTIAL ENERGY OF A SYSTEM OF CHARGES


eB

The electrostatic potential energy is the energy possessed by a system of point charges by virtue of their
positions. When two charges are infinite distance apart, their fields do not extend uplo each other. Their
r

electrostatic potential energy is zero, because no work is done in moving a charge at infinite distance where
ou
ad

electric field is zero. Hence we may define


YY

Electrostatic potential energy of a system of point charges as the total amount of work done in
bringing the various charges to their respective positions from inifnitely large mutual separations.
ndd
Re

(fl) Electrostatic potential energy of a system of two point charges


Fi

FIGURE 2.12

Suppose a point charge is held at a point P| with position vector +Q2


in space. Another point charge ^2 is at infinite distance from <7,. This is to be (AT INFINITY)
f
brought to the position ?2 (^), where P^p2 = ^2 ● Pi§- 2.12.
/

/
Now, electrostatic potential at P2 due to charge at Fj is
V =
47t6ofi2
1

Pi(n)
I 2
P2(>’2)
'12
By definition, work done in carrying charge (?2 from to P2
W = potential x charge

w = X
^2
2/11
ELECTROSTATIC POTENTIAL AND CAPACITANCE

This is stored in the system of two point charges and qj in the form of electrostatic potential energy U.
Thus

^1 ...(18)
V =^W =
4jt£o rij
{b) Electrostatic potential energy of a system of N point charges
It is equal to the total amount of work done in assembling all the charges at the given positions from
infinity.
In bringing the first charge q^ to position P^ (1^), no work is done, because all other charges are still at
infinity, and there is no field, i.e.. W,1 = 0

When we bring charge ^2 from infinity to P2 (r^) at a distance rj2 from q^, work done is

ww
^1
W2 = [potential due to q{\ x q2 =
4jceo rj2
to P3 (;J), work has to be done against electrostatic forces of both and ^2

Flo
In bringing q^ from 00

e
1
Sl+Si.

ree
W3 = [potential due to q^ and ^2! ^ charge q^ - 4ne
X^3
0 ^3 ^*23

Fr
rF
1
uurr
for
47tG '13 ^23
0

Similarly, in bringing q^ from 00


to /’4 (7^), work has to be done against electrostatic forces of g,, ^2
s
kks
and qy
Yo
oooo

1 ^1 ^4 . ^2 ^4 . ^3 ^4
47t€ ''14 '24 ^34
eB

Electrostatic Potential energy of a system of four charges.


1
1
^2 ^ ^3 ^ ^2 ^3
r

U=w= W, + W2+W2+ W4 =0 +
J 4ice„ [ r,3
ou
ad

^23 .
YY

47te
nndd

0 ^14 ^24 ^34 .


Re

1
9i^2^'?I^3 . ^1^4 . ^2^3 . ^2^4 . ^3 ^4
Fi

.(19)
47TG '12 '*13 ^14 ^23 ^24 ^34 .
0

Proceeding in this way, we can write electrostatic potential energy of a system of N point charges at
7^ as
q,r2.

I ^
1
U = ...(20)
"^^^0 all pairs 'O*
Note that in this summation, we should include only one term for each pair of charges. Therefore, the
above expression can be rewritten as
. j=N k=N 1 ...(21)
U =
I I I fc=i
4;t6 0
O’*
j*k
2/12
Fundamentai Physics (XII) yoL.i

The factor 1/2 is needed because in this summation set up, we count each pair of charges twice. For
example, whenj= 1, A: = 2 andy=2,/:= 1. we get contribution from the same pair of charges, 1,2. As we have
to include only one term for each pair, the factor 1/2 is a must.
, j^N ( k=N
Rewriting (21) as u =
I 47t:e
j=i k=l 0 'jk
[
I j^N
...(22)
2 7=1
^ ^J''J
k=N
1 ^k
where Vj — ^

ooww
47ce = Potential at r* due to all other charges.
A=i 0 0*
k^j

Equation (22) represents the relation between electrostatic potential energy and electrostatic
potential:

e
Units of electrostatic potential energy

ree
rFl
The SI unit of potential energy is joule. One joule is the energy stored in moving a charge of one

Fre
coulomb through a potential difference of one volt.

rF
Another convenient unit of energy is electron volt (eV). which is the kinetic energy gained or lost by an
electron in moving through a potential difference of one volt.
foor
ouur
1 electron volt = I 1-6 x lO"'*^ coulomb x I V = 1-6 x joule
ks sf
Some other related units of energy are
ook

1 milli electron volt = 1 meV= 10"^ eV = l-6x 10'~“J


YYo

I kilo electron voll= I keV = 10^<?V= l-6x 10"'^ J


BBo

I Mega or Million electron volt = 1 MeV - 10^eV= l-6x


rre

1 Giga electron volt = 1 GeV= \Cp eV = l-6x 10"''*J


ouu

I Trillion electron volt = 1 TeV = IO'“eK= l-6x 10"’^J.


Y
ad

Problem Three charges + ^, 2 ^ and -4 ^ are placed on the three vertices of an equi
dY

lateral triangle of each side 0*1 m. Calculate electrostatic potential enci^ of the system, take q = 10"^ C
(Uttarakhand Board 2013)
innd
Re

1
Sol. Potential Energy of .system of charges =
FFi

47te
[q{2q) + lq{-Aq) + {-A q) q]
0

10^2 9x10^x10(10*'^)2
= -9x lO-^J
4jre^, r 01

2.11. POTENTIAL ENERGY OF CHARGES IN AN EXTERNAL ELECTRIC FIELD

(fl) Potential energy of a single charge in an external field


We have already obtained an expression for potential energy of a system of charges. In this case, the
source of the electric field, Le., charges and their locations were specified.
We have now to determine potential energy of a charge (or charges) in an external field E which is not
produced by the given charges whose potential energy we have to calculate. The sources which produce
external field E are often unknown and they are of no interest to us.
ELECTROSTATIC POTENTIAL AND CAPACITANCE 2/13

The external electric field E and the corresponding external potential V may change from point to
point. If V (7^) is external potential at any point P of position vector 7^, then by definition, work done in
bringing a unit positive charge from infinity to the point P is equal to V.
Therefore, work done in bringing a charge q from infinity to the point P in the external field = . V (r)
This work is stored in the charged particle in the form of its potential energy.
Potential energy of a single charge ^ at 7* in an external field = q .V (r)
(^) Potential energy uf a sy.stem of two charges in an external field
Suppose ^2 ^ h respectively, in a uniform external
electric field of intensity ~E ■ To calculate the potential energy of the system of these two charges in the

ww
external field, we find that

Work done in bringing charge q^ from infinity to position is = q^ . V (7^) where V (7^) is

Floo
potential at ^ due to external field.
Again, work done in bringing charge ^2 from infinity to position against the external field is

ee
reer
^2 = ^2 ● ’ where V (J^) is potential at ^ due to external field.

rFF
While bringing ^2 from infinity to position , work has also to be done against the field due to ^ j. This
uur r
is W3 =
__M2
47ce
where rj2 is the distance between q^ and ^2*
ffoor
sks
0 '12
YYoo
By the superposition principle, we add up the work done and find P.E. of the system = total work done
ooko

in assembling the charge configuration


eBB

U=W^ + W2+W^

^2
U =q^ .V (r^)+q^ .V (/^) +
r
ouur

4ne„r,2
ad
Y

Retain in Memory
nddY

Potential energy of any system of charges is defined relative to a point, where its value is zero.
Re

1.
Normally, this point would be at infinite distance from the charges.
FFiin

2. When two charges q^ and </2 ^^re of same sign, they


repel eacholher. Work done to bring them to their
respective positions is positive. Therefore, potential
energy of a system of two charges of .same sign is
positive as shown in Fig. 2.13(«).
When the two charges are of opposite sign, they
attract eacholher. Work done to bring them to their
respective positions is negative. The variation of P.E
(U) with distance is .shown in Fig. 2.13(b).

2.12. ELECTROSTATICS OF CONDUCTORS


In Art. 1.4, we have learnt that a conductor is a .substance that can be used to carry or conduct electric
charges from one place to the other. All metals, for example, are good conductors of electricity. This is
because conductors contain a large number of free charge carriers (i.e., free electrons).
2/14 'P'utdee^ ^ Fundamental Physics (XII) P;«iwi
Some of the important results regarding electrostatics of conductors are discussed below :
1. In electrostatic equilibrium, electric field is zero everywhere inside the conductor. Suppose a
conductor ABCD is held in an external electric field of intensity , Fig. 2.14. Free electrons in the conductor
move from AB to CD.
FIGURE 2.14
As a result, some net negative charge appears on CD and
an equal positive charge appears on AB. These are called D e'◄■ A

induced charges. They produce an induced electric field of


intensity which opposes the flow of free electrons from AB
+ _(E^ = 0^
to CD. The flow, therefore, stops as soon as E becomes equal

to Eq ■ As the applied field and induced electric field inside ^ +

ww
the conductor are equal and opposite, therefore, net electric field in +
the interior of the conductor is zero. Obviously, there will be no C B
electric field lines inside the conductor held in an external electric

Flo
field. Eo

e
2. Electric field just outside a charged conductor is perpendicular to the surface of the conductor

rree
at every point. Under electrostatic conditions, once the charges on a conductor are rearranged, the flow of

r FF
charges stops. Therefore, component of electric field along the tangent to the surface of the conductor must
be zero.
uurr
i.e. E cos 0 = 0, where 0 is the angle which electric field
intensity makes with the tangent to the surface. for
kss
AsE^O, cos 0 = 0 or 0 = 90®
ooook

Hence the field is perpendicular to the surface of the


Yo

conductor at every point as shown in Fig. 2.15. If charge on


eB

conductor is positive (a >0), e is along the outdrawn normal.


If charge on conductor is negative (a <0), e is along normal
rr

drawn inwards.
ou
ad

3. Net charge in the interior of a conductor is zero.


Y
dY

c 0
According to Gauss’s theorem ; o E.ds = —
^0
Re
innd

Inside a conductor, £ = 0 q = 0 i.e. net charge inside a conductor is zero.


Fi

If some excess charge (positive or negative) were placed inside a conductor by any means, the coulomb
repulsive forces between the charges force them as far apart as possible. These forces will cause them to
migrate quickly on the outer surface of the conductor. Hence any excess charge on an isolated conductor in
electrostatic equilibrium resides entirely on the outer surface of the conductor.
4. Electrostatic potential is constant throughout the volume of the conductor and has the same
^ >
value as on its surface. As electric field £ = 0, inside the conductor, no work is done in moving a small test
charge within the conductor. Therefore, there is no potential difference between any two points inside the
conductor, i.e., electrostatic potential is constant throughoutthe volume of the conductor.
—> .
Further, £ is perpendicular to the surface of the conductor. Therefore, E has no tangential component
on its surface, no work is done in moving a test charge on the surface of the conductor. Hence, there is no
potential difference between any two points on the surface of the conductor. Hence, the electrostatic potential
is constant throughout the volume of the conductor and has the same value as on its surface.
ELECTROSTATIC POTENTIAL AND CAPACITANCE 2/15

5. Electric field at the surface of a charged conductor is E = — n, where c is surface charge density
A
and n is a unit vector normal to the surface in the outward direction.
Let us consider a short cylinder of small area of cross section ds and negligible height partly inside and
partly outside the surface of a conductor of surface charge density c, as shown in Fig. 2.16.
Just inside the surface, £ = 0.

Just outside, the field £ is normal to the surface. The contribution to


the total flux through the cylinder comes only from the outside circular
cross section of the cylinder. Over the small area ds, taking £ to be constant,
electric flux = ± £ (ds), positive for a > 0 and negative for o < 0.
As charge enclosed by the element -a ds

ww
cds ^ a
.●. By Gauss’s theorem, £ (ds) = or £ = —
^0 ^0
O A
As electric field is normal to the surface, we can write £ = n,

Flo
^0

ee
which is true for both signs of o. For a > 0, electric field is normal
to the surface and outwards as shown in Fig. 2.16. The reverse is also

rere
rFF
true.

6. Surface density of charge is different at different points of an irregularly shaped conductor.


When a conductor is placed in an external electric field, charges on the conductor rearrange on account of
uur r
foor
flow of negative charges (Le., electrons). Therefore, the local charge density c (i.e. charge/area) will be
different at different points on the surface of the conductor, a will be positive at places where the charge is
ks s
positive and a will be negative at places where the charge on the conductor is negative. Further, at points
Yoo
where radius of curvature of surface is small, area is smaller, therefore, a will be higher and vice-versa.
ook
BBo

2.13. ELECTROSTATIC SHIELDING


re

Electrostatic shielding/screening is the phenomenon of protecting a certain region of space


from external electric field.
ouur
ad

We know electricfield inside a conductor is zero. Therefore, to protect delicate instruments from external
electric fields, we enclose them in hollow conductors. Such hollow conductors are called Faraday Cages.
YY

They need not be earthed. In a thunderstorm accompanied by lightening, it is safer to be inside a car or a bus
than to be in the open ground or under a tree etc. The metallic body of the car or bus provides electrostatic
nndd
Re

shielding from the lightening. The fundamental fact is that electric field inside a cavity is zero, whatever be
the size and shape of the cavity and whatever be the charge on the conductor and the external fields in which
FFii

it might be placed. That is, whatever be the charge and field configuration outside, any cavity in a conductor
remains protected from outside electrical influence.
Further, an earthed conductorAB can also FIGURE 2.17 FIGURE 2.18

act as a screen against the electric field. For


example, in Fig. 2.17, when AB is not earthed,
field of the charged body C due to electrostatic rN
induction continues beyond Afi. However, when
AB is earthed, the induced positive charge flows
to earth and the field in the region beyond AB
disappears, Fig. 2.18. A high voltage generator
is usually enclosed in such a cage which is
B A B
earthed. This would prevent the electrostatic + + 4- + + ,+ +
field of the generator from spreading out of the
cage.
2/16 “Ptadeep- ^ Fundamental Physics (XII)

DO YOU KNOW ?
1. Human body is a good conductor of electricity, />., charge can flow easily through the human body.
2. Grounding or Earthing. It is the method by which charges are shared between a charged body and earth.
Metallic bodies of all electrical appliances are earthed from the safety point of view of the appliance as

w
well as the user.

The electrostatic shielding has been used in protecting sensitive instruments from outside electrical
influence. The phenomenon has also been used in the design of T.V. Cables.

e
2.14. ELECTRICAL CAPACITANCE

e
wr
lloo
Electrical capacitance of a conductor is related to its ability to store the electric charge or energy.

r
When a conductor is given some charge, its electric potential increases. If a charge Q given to the

F
conductor raises its potential by V, then it is found that Q V or Q = CV, where C is a constant of

u
oc

proportionality and is called electrical capacitance of the conductor. From C = QI,V ...(23)

r FF
we may define
Capacity of a conductor as the ratio of charge given to the conductor to the rise in its potential.

Frrs
ee
uo
If we plot a graph between Q (charge given) and V (the potential to

k
FIGURE 2.19

o
which capacitor is raised), we get a straight line as shown in Fig.2.19. It
implies that changing Q by a certain factor changes V by the same factor.

foo
fr
Therefore, the capacity of a conductor depends neither on Q nor on .V It
depends on the shape and size of the conducto.r Furthe,r the capacityof
kso
Y
the conductor does not depend upon the nature of material of the
B
eBrY

conducto.r However, the capacity depends on the nature of medium (e.g.


Yo
oo

its relative permittivity, e^) in which the conductor is located.


If V= {, C=Q ... from eqn. (23). Therefore,
e

Electrical capacity ofa conductor is numerically equal to the charge required to raise its potential
uru

by unity. Capacitance is a positive quantity only.


od
o
ad

In SI, the unit of capacitance is farad.


n
Y

I coulomb (C) .
I farad {F) = , i.e..
I volt (V)
di
Re
FF

a conductor is said to have a capacity of one farad, when a charge of one coulomb raises its
in

potential by one volt.


Smaller units of capacitance are I micro farad, i.e.,
I |lF= 10-6 farad.
I nanofarad, i.e., 1 nF = 10“^ farad,
1 micro microfarad, i.e., 1 fi [x F = 10“’^ farad.
|X )X F is also called picofarad ipF).
DlmensiunK of capacitance

From C=^ g _ Q _Q^ (AT)^ = [a/-'l-2 T‘^A^]


V W/Q W ML^ T-2
A potential difference of 250 volt is applied across the plates of a capacitor
of 10 pF. Calculate the charge on the plates of the capacitor.
Sol. Here, V=250W, C = 10 p F = 10 x IQ-'^ F = lO"” F
e = CV' = 10-‘' X 250 = 2-5 X 10-^ C
ELECTROSTATIC POTENTIAL AND CAPACITANCE 2/17

2.15. CAPACITY OF AN ISOLATED SPHERICAL CONDUCTOR


Suppose 0 is the centre and r is radius of an isolated spherical conductor. Fig. 2.20. Let a charge + ^ be
given to the sphere. This charge spreads uniformly over the outer surface of the sphere whether the sphere is
hollow or solid. Therefore, potential at every point on the surface of the sphere is same. As the sphere
behaves as if the entire charge were concentrated at the centre of the sphere, therefore, potential at any point
on the surface of the sphere in free space.

4ner.0 r
...(24)

As C=^ . C =
<?47CSq r C = 47t€nr ...(25)
0
V
When r is in metre, C is in farad.

ww
Clearly, C «= ,r i. e., larger the radius of the sphere, greater is its capacitance.
Note that a capacitor is normally made of two conductors. We have calculated here the
capacity of a single spherical conductor, assuming the other conductor is at inifnity.

FF loo
Capacity of Earth
We can calculate electrical capacity of earth, taking it to be a conducting sphere of radius,

ree
r = 6400 km = 64 X 10^ m

rree F
I
As C = 4 7t e Q r .-. C = (64x10^) =0-711 X 10-3farad = 0-711 ^ iq-3 x 10^ p F = 711 jiF
9x10^
ur
Thus the capacity of earth, which is the biggest sphere we come across is only 711 p F. Therefore,
fofr oF
in routine, capacitance will be in nanofarad (nF) and picofarad (pF). Farad is too big a unit of capacitance.
Problem Diameter of a spherical conductor is 1 metre. What is its capacity ?
ks
YYouro
(Uttarakhand Board 2012)
soo

Sol. Here, r = 1/2 m = 0-5 m, C = ?


BBook

0-5
C = 47ie 0r,r = farad = 55-5 x 10 farad = 55*5pF
r ee

9x10^
oouru

2.16. CAPACITOR AND ITS PRINCIPLE


ad

A capacitor is an arrangement for storing large amounts of electric charge and hence electric
Y

energy in a small space.


Ynd

Usually, a capacitor consists of a system of two conductors separated by an insulating medium. Often
Re

the two conductors are charged by connecting them to the two terminals of a battery.
FFindi

To understand the principle of a capacitor, let us consider an insulated metal plate A. Let some positive
charge be given to this plate, till its potential becomes maximum. No further charge can be given to this plate
as it would leak out.
FIGURE 2.21
Now, consider another insulated metal plate B held
A B A B
near the plate A. By induction, a negative charge is
+ + + +
produced on the nearer face of B and an equal positive +

charge develops on the farther face of B, Fig. 2.21(a). + + + + +

The induced negative charge tends to decrease the + + + + +


potential of A and the induced positive charge tends to +
+ .+ + +
increase the potential of A. As the induced negative charge
is closer to A, it is more effective. The overall potential + + + + +

of A reduces and hence some more charge can be given + + + + +

to A to raise its potential to the same maximum value.


Thus, capacity of the conductor A has increased, by .'f e O
bringing another uncharged conductor B in its vicinity.
2/18 “Pn^icUc^ Fundamental Physics (XII)
Now, connect the plate B to earth. The induced positive charge on B being free, flows to earth, Fig.
2.2\{b). The induced negative charge on B, however, stays on as it is bound to positive charge on A. Due to
the induced negative charge on B, potential of A is greatly reduced. Thus, a large amount of charge can be
given to A to raise it to the maximum potential.
Hence, we conclude that the capacitance of an insulated conductor is increased considerably by
bringing near it an uncharged conductor connected to earth. This is the principle of the capacitor.
Such an arrangement of two conductors separated by a dielectric medium is said to form a capacitor
or condenser.

If the conductors are plane, the capacitor is called a parallel plate capacitor. When the conductors
are spherical, the capacitor is called a spherical capacitor and when the conductors are cylindrical, it is
called a cylindrical capacitor. Sometimes, capacitors are also named on the basis of dielectric medium
used in them. For example, electrolytic capacitors, paper condenser, mica condenser, ceremic condensers

w
etc.

As a capacitor is used to store large amounts of charge and hence electric energy, it can safely absorb
large amounts of charge. Similarly, large amounts of charge can be released by the capacitors.

Flo
A capacitor with fixed capacitance is symbolically shown as ( — — ), while a capacitor with variable

ee
capacitance is represented as ( — These symbols are used for capacitors of all geometries.

Fr
Retain in Memory

for
ur
Capacitors or condensers are used in many electrical appliances, like electric fans, electric motors,
in oscillator circuits, in radio tuning, in ignition system of engines in vehicles, in filter circuits etc.
In the microelectronic age we live, microscopic capacitors form the memory banks of computers.
s
ok
Yo
Bo

2.17. PARALLEL PLATE CAPACITOR

This is the capacitor which is used most commonly. It FIGURE 2.22


re

consists of two thin conducting plates 1 and 2, each of area A


held parallel to each other, suitable distance d apart. The plates 1 Surface charge 2
Area A density \
ou
ad

are separated by an insulating medium like air, paper, mica, glass +a -cr

etc. One of the plates 1 is insulated and the other plate 2 is earth + +
Y

connected. Fig. 2.22. + +

When a charge + (2 is given to the insulated plate 1, then a


nd

+ +
Re

E
charge - 0 is induced on the nearer face of plate 2 and charge + + +
Q is induced on the farther face of plate 2. As plate 2 is earthed,
Fi

+
the charge + Q being free, flows to earth. +
-- d--

Plate 1 has surface charge density, a = —


A

and plate 2 has a surface charge density, - ct = —^


A
As discussed in Art. 1.46,
In the regions on left of plate 1 and on right of plate 2, the electric field = 0
a 1 Q
In the region between the plates, separated by air/vacuum, electric intensity is, E =
^0 6q a
Taking this electric field localised between the two plates as uniform throughout, the potential difference
V between the plates is simply the electric field times the distance between the plates, i.e.,
V = Exd = JLfi,
e. A
0
ELECTROSTATIC POTENTIAL AND CAPACITANCE 2/19

The capacity C of parallel plate capacitor is given by


Q
C = ^ Q_ i.e., C =
0
A
...(26)
V Qd d ' d
G« A
0

Retain in Memory
1. From eqn. (4), we find that capacitance of a capacitor is determined geometrically from the
dimensions of the capacitor.
2. The capacitance does not depend upon the charge on the capacitor or FIGURE 2.23

potential difference between the plates of the capacitor.


3. Infact, net charge on the capacitor plates is ^ + (- ^) = Zero. However,

ww
electric field between the plates of capacitor is not zero. The electric field
between the plates of capacitor is as shown in Fig.2.23.
+
We observe that E is uniform in the central portion of the capacitor and it
is non uniform at the edges. This is called edge ejfect or fringing. This non

Flo
uniformity of electric field at the boundaries of the plates becomes negligible,

ee
when distance between the plates is very small, compared to the length of

rere
the plates.

rFF
4. When plates of capacitor are separated by a dielectric medium of relative permittivity e ^ = /T, its
capacity
uur r
C
m
6 A
d
foor
d
= e^C^ = KC 0* i.e., C-.
m
= /f c0 ...(27)
sks
i.e., capacity becomes fC times the capacity with air/vacuum as dielectric.
Yoo
oko

5. Charged capacitors have electrostatic forces between the walls. Force of attraction between the
plates of capacitor
BBo

W/A)Q
re

F = EQ =
A
l"o ./
^0 0
ouur
ad
YY

Problem' Calculate the capacitance of a parallel plate condenser of two plates


100 cm X 100 cm each separated by 2 mm thick glass sheet of K = 4.
nndd

Sol. Here, A = 100 cm x 100 cm - 10"^ cm^ - 1 m^, d = l mm = 2 x 10“^ m, /C = 4, C = ?


Re
FFii

K^r.A
0 4x8-85x10"^2x1
C = = 1-77 X 10-* farad
d 2x10-3

2.18. RELATIVE PERMITTIVITY OR DIELECTRIC CONSTANT

In Art. 1.15, we have defined relative permittively or dielectric constant of a medium :


Dielectric constant of a medium is the ratio of the electrostaticforce of interaction between two
given point charges held certain distance apart in vacuum/airto theforce of interaction between A
the same two charges held the same distance apart in the material medium.
Faraday observed that when an insulating material is fully occupying the space between the charged
plates of a capacitor, its capacitance increases. The factor by which capacitance is multiplied depends on the
nature of the dielectric introduced, and is called relative electrical permittivity (e ^ or dielectric constant (K)
of the material. We may write
capacitance of a capacitor with dielectric inbetween the plates
G^or K =
capacitance of the same capacitor with vacuum/air inbetween the plates
2/20 Fundamental Physics fXIItPTSTW

Cm
e or K== ...(28)
r
c0

A 6 A
0
Thus, C...
m
=KCr.
0
= ex
r
d d
The values of or K, for common dielectric materials are given in Table 2.1.

TABLE 2.1. Dielectric constant for some common dielectrics

MATERIAL S;.or K MATERIAL e r.or /C

ww
1. Vacuum 1-0000 9. Paraffined paper 2-1-2-3
2. Air (1 atm) 1 0006 10. Porcelain 6-7

3. Air (100 atm) 1-0548 11. Germanium 16

4. Hydrogen 1-00026 12. Glass 3-4

Flo
5. Turpentine 2-23 13. Alcohol 25

e
ere
6, Teflon 2-1 14. Transformer oil 2-24
7. Amber 2-8 15. Water 81

FFr
8. Mica 6-7 16. All metals OO
uurr
orr
sfo
Problem The capacity of a capacitor becomes 10 p F when air between the plates is
replaced by a dielectric slab of F = 2. What is the capacity of the condenser with air inbetween the plat^ ?
kks
Sol. Here, C,„=10pF, Cq = ?, K = 2
Yo
oooo

Cm C 10
As K = C
m

Y = 5hF
eBB

0 “
C0 K
rr

2.19. SPHERICAL CAPACITOR


ou
ad

Note. A cylindrical capacitor consists of two co-axial cylinders of same length L and radii and
YY

The outer cylinder of radius is earthed and inner cylinder of radius is given some charge.
The capacity of cylindrical air capacitor is given by
nndd
Re

2ne^0 L *

C = ...(29)
Fi

log (r,b /ra')

2.20. GROUPING OF CAPACITORS/CONDENSERS

In many electrical circuits, capacitors are to be grouped suitably to obtain the desired capacitance. Two
most common modes of grouping of condensers are : Series grouping and Parallel grouping
(n) Ciipucitors in Serle.s
The capacitors are said to beconnectedin series between two points, when we can proceed from
one point to the other only through one path.

In Fig. 2.24, three condensers of capacities Cj, C2, C3 are connected in series. Vis the potential difference
applied across the combination.

♦Obviously, flow of negative charges (electrons) is in the opposite direction i.e. from negative to positive
electrode.
ELECTROSTAUC POTENTIAL AND CAPACITANCE 2/21

Let + Q units of charge be given to the left FIGURE 2.24


plate of C|. The elctric field of this positive charge
(+ Q) pulls negative charge upto the right plates + + +

of Cj. Therefore, right plate of C] acquires - Q +Q + +Q + +Q +

charge. As this - Q charge comes from the left + + 4-

plate of C2 through the wire connecting them, + + +

therefore, left plate of C2 acquires + Q charge. Ci ^2 C3


The right plate of C2 acquires - Q charge as
explained for capacitor C,. Similarly, capacitor
.V, K ■V2. -V3.

C3 is charged with + Q charge on left plate and + ●V-


- Q charge on the right plate. Thus, each capacitor
receives the same charge of magnitude Q. As their
capacities are different, therefore, potential
differences across the three capacitors are

ww
different :

Q
K . V22 = — , V3^ =-2C3

Floo
I
C, ...(30)

If Cj is the total capacitance of the combination, (i.e., capacitance of a single capacitor that would

ee
acquire charge Q when potential difference across its terminals is V), then

reer
Q

rFF
v =
C (31)
s
uur r
As total potential drop V across the combination is the sum of potential drops Vj, Vj, V3 across Cj, C>,
C3 respectively, therefore V = V^ + V2 + ffoor
sks
Q Q . Q ( \ I 1
— +
-=—+ ^ ^
YYoo
C.V C
or ...(32)
C2 ^3 C C2 C3
ooko

1 c ^^2
eBB

i—n
1 1
For n capacitors connected in series, total capacitance would be
C
=I C.
r

s /=! i
ouur

i.e., the reciprocal of equivalent capacitance of any


ad

number of capacitors joined in series is equal to sum of the


YY

reciprocals of individual capacitances.


Obviously, is less than the least of Cj, C2, C3 C_.
nndd

n
Re

(h) Capacitors in parallel


FFii

The capacitors are said to be connected in parallel


between any two points, if we can proceedfrom one
point to the other along different paths.
In Fig. 2.25, three capacitors of capacitances C|, C2, C3
are connected in parallel.
Let V be the potential difference applied across the
combination.

As potential difference across the three condensers is the


same {i.e. V), therefore, each capacitor will store charge
according to its capacity, say Q^, Q2 and O3, such that
Q^ = C^V.Q2 = C2V.Q^ = C^V
If Q is the total charge on the parallel net work, then as
total charge is the sum of charges on all capacitors.
2/22 P^uuiec^ ^ Fundamental Physics (XII)

(2 = !2i + ^2 23 ...(33)
DO YOU KNOW ?
Let Cp be the equivalent capacitance in parallel {i.e., capacity of a ● In a series combination of
single capacitor which would acquire the same total charge Q under the
capacitors, charge can move
same potential difference V).
along only one route. That is
Q = CpV why charge on each capacitor
Putting these values in (11), we get is same.

Cp v= q v+ C2V+ C3(c, + C2 + c^) v ● In a parallel combination, two


...(34)
plates of every capacitor are
q - Cl + C2 + C3 connected to the same battery.
In general, when n capacitors are connected in parallel. Therefore, potential difference
across each capacitor is same.
Charge on each would depend
i=n

then
c> ...(35)
upon capacity of the

ww
1=1 condenser.

● Further, note that battery


Le., equivalent capacitance of any number of capacitors joined in

Floo
produces charges on only the
parallel is equal to sum of the individual capacitances. Obviously, two plates to which it is
capacitance in parallel combination increases. Here, > the largest of

ee
connected. Charges that are
C), C2, C3, , ,Cn' produced on the other plates

reer
rFF
are due merely to the shifting
sampie^Pro^'ijafgS Three capacitors of 1 jiF, 2 p,F and
of charges already there.
3 pF are joined in series. How many times will the capacity become
uur r
when they are joined in parallel ?
Sol. Here, C, = 1 pF, C2 = 2 pF, C3 = 3 pF
ffoor
sks
The combined capacity (C^) in series is given by
YYoo
ooko

L-i i —pF
eBB

q c, q C3-r2‘"3-_
-- — + -
" 11

Total capacity in parallel combination,


r
ouur
ad

Cp = C| + C2 + C3= 1 + 2 + 3 = 6 pF
YY

C
P _
— = 11 Hence, C„p = 11 C,»
nndd
Re

C. 6/11
FFii

2.21. ENERGY STORED IN A CAPACITOR


We know that a capacitor is a system of two conductors carrying charges FIGURE 2.26

Q and - Q held some distance apart, Fig. 2.26. To calculate the energy stored +Q -Q

in this charge configuration, suppose the conductors 1 and 2 are initially +


un charged. Let positive charge be transferred from conductor 2 to conductor
+
1 in very small instalments of dq each till conductor 1 gets charge + Q. By
charge conservation, conductor 2 would acquire charge - Q. +
E
At every stage of charging, conductor 1 is at a higher potential than +

conductor 2. Therefore, work is done externally in transferring each instalment +

of charge. +
At any intermediate stage, suppose charge on conductor 1 is + ^ and 2
charge on conductor 2 is -
.●. Potential difference between conductors 1 and 2 is q!C, where C is the capacity of the capacitor.
ELECTROSTATIC POTENTIAL AND CAPACITANCE 2/23

Small amount of work done in giving an additional charge dq to the conductor 1 is dW = — xdq
c

9=0 19=0
1 sL 1 02
Total work done in giving a charge Q to conductor 1, W =
Cl 2 J?=o C 2
q=0
As electrostatic force is conservative, this work is stored in the form of potential energy (C/) of the
condenser.

1
u =w = -
2 C

1 (CV)2 = -CV2
1

ooww
Put Q = cv (/ = - ...(36)
2 C 2

Put CV=Q U = iQv

ee
1 1 1
Hence, U = - = -CV2 = -QV ...(37)
2 C 2

r
rFFl
ree
F
When Q is in coulomb, V is in volt, C is in farad, Energy U is in joule. FIGURE 2.27

rF
If we plot a graph between charge given and potential of the capacitor,
we get a straight line. OA as shown in Fig.2.27.

fsfoor
ouur
Energy stored in the capacitor = area under V - Q graph
1 1 1 Q
kosk
u = -X base (05) X height (A5) = —xOxV = —Ox— -
2 2 2 C 2C
YYo
oo

Retain in Memory
BB

1.
The potential energy stored in the capacitor is independent of the manner in which charge
rre

configuration of the capacitor is built up.


2. The potential energy of charged capacitor can be viewed as stored in the electric field between
oYuu

the plates.
ad

3. The potential energy of capacitor lies in the dielectric medium between the plates.
dY

4. The potential energy of capacitor is obtained at the cost of chemical energy stored in the battery
— used for charging the capacitor.
innd
Re

5. Energy spent by the battery in charging the capacitor = QV, whereas energy stored in the capacitor
FFi

The rest of the energy is dissipated/lost in charging the capacitor.

What capacitance is required to store an energy of 100 kWh at a potential


difference of 10** V ?
Sol. Here, C = ?, 1/ =: 100 kWh = 100 X 10^ X 3600 J = 3-6 x 10^ J
V= 10^ volt
1
From U = -CV^
2

2^ _ 2x3-6x10^ = 7-2F
1/2 " (lO**)^
2/24 Fundamental Physics (XU)EEI5]
2.22. TOTAL ENERGY STORED IN A COMBINATION OF CAPACITORS
In series combination of capacitors, Q is constant., i.e., every capacitor carries the same charge Q.

Total energy, U = - i ^— +—+—+


2 2 ■ 2 C1 C2 C3

U = I
...(38)
1C,1 2C2 2C3
(/ = C/| + t/2 + C/3 +
In parallel combination of capacitors, V is constant, i.e.. potential difference across each capacitor is same.

Total energy, t/= —C + ^2^3 ,)y2 ^ir


2 * 2 2 3 ^
^

ww
1 P 2

C/= C/j + C/2 + C/3 +● ...(39)

FF loo
total energy stored in series combination or parallel combination of capacitors is equal to sum
of the energies stored in individual capacitors.

ee
2.23. ENERGY DENSITY OF A PARALLELPLATE CAPACITOR

Fr
Energy density (u) is defined as the total energy stored per unit volume of the condenser.
ur
rre
— CV^
i.e., u =
total energy (C/) _ 2
volume (u) Ad
fofr oF
using C =
6.0 A

d
and V - Ed, we get
ks
YYouro
\ (
Ifen A E^d'^ 1 1
soo

E^, le. u = -e. £2 ...(40)


2^0
u = —
2 d )[ Ad 2 0
BBook

Here, E is the strength of electric field in the space between the plates of the capacitor.
r ee

SI unit of energy density is J m“^.


energy _ MI? T ~ = [ML-^T~^]
oouru
ad

Dimensional formula of energy density is u =


volume I?
Y

Curiosity Question
Ynd
Re

r Q. What Is a defibrillator ? Explain briefly.


FFindi

Ans. A defibrillator is a device used to save the life of a person suffering a heart stroke.
Due to rapid, unco-ordinated twitching of the heart muscles, the heart undergoes ventricular
fibrillaticri. To save the person’s life, regular beating of the heart is to be restored by delivering
a jolt to the heart using a defibrillator.
Suppose, a 70 p F capacitor of a defibrillator is charged to a potential of 5000 V. The energy

stored = ^ (70x10"®) (5000)2 J = 375 j typically, about 200 J of energy is allowed to


pass through person’s body in a pulse lasting 2 milli second. Therefore, power delivered to the
200
body =
2x10"
J = 10® W =100 kW, which is much larger than the power delivered by the
battery.
Thus energy stored in the capacitor discharged at a much higher power provides the much
needed jolt to save the person’s life. J
ELECTROSTATIC POTENTIAL AND CAPACITANCE 2/25

2.24. COMMON POTENTIAL

When two capacitors charged to different potentials are connected by a conducting wire, charge flows
from the one at higher potential to the other at lower potential. This flow of charges continues till their
potentials become equal. The equal potential of the two capacitors is called common potential. Obviously, no
charge is lost in the process of sharing of charges by the two capacitors.
Suppose C], C2 are capacities of two condensers charged to potentials and V2 respectively. Total
charge before sharing = C| Kj + C2 V2
If V is the common potential on sharing charges, then total charge after sharing
= C| V+C2V=(C^ + C2)V
As no charge is lost in the process of sharing, therefore (C| + C2) V = C, V| + C2 V2

or v = ...(41)

w
C1+C2
total charge
i.e., common potential =

Flo
total capacity
From (19). Cl Vj + C2V2 = CjV+C2V C, Vj-C, ^=02^-02^2
or

ee
i.e., charge lost by one capacitor = charge gained by the other capacitor

Fr
This is NOT true for potentials, Le,, potential lost by one is not equal to potential gained by the
other, as their capacities are different.

for
ur
IMPORTANT NOTE

Note that in the derivation of this relation (41), we have assumed that the two capacitors are connected
ks
in parallel with their positive plates at one point and negative plates at another point. However, when
Yo
the two capacitors are connected in series, i.e., with positive plate of one capacitor to the negative
oo

C V -C V
plate of other capacitor, eqn. (41) will be modified to F =
eB

^1 ■'■*^2
Thus total charge in this ca.se, would be algebraic sum of the charges on the two capacitors.
r
ou
ad

2.25. LOSS OF ENERGY ON SHARING CHARGES


Y

When charges are shared between two bodies, no charge is lost. However, some energy is dissipated in
the form of heat etc., i.e., some energy is lost on .sharing charges. To calculate the loss of energy, suppose two
nd
Re

capacitors of capacitances C|, C2 charged respectively to potentials Vj, V2 ^re connected together and their
Fi

charges are shared. The charge flows from one capacitor at higher potential to the other at lower potential till
their potentials become equal. The common potential reached is given by

V = ...(42)
C, +C2
1 9 I 9
Total energy before sharing charges, t/j = — Cj V, + — C2V2

Total energy after sharing charges,

1
(CiV;+C2Y2)^
Using (42), t/2=-(q+C2)
(C,+C2)2 2(Cj+C2)
2/26 ^>utdee^ ^ Fundamental Physics (XU)

Let us calculate : C/ I -t/2^ =lq


2 ^ v2^1r
* 2^2\/2_
2(Cj+q)
q (q + q) + q (c, + q) - (c, V, + f
2(q+q)
q2 v,2 + c, C2 v,2 + q C2 ^2^ + c| - q^ 1^2 „ c\ v} - 2 q C2 Vj ^2
2(q +C2)
CiC;(K,^+V|-2V,V^)
2(C|+C2)

_C,C,(V,-V,)^
£/,-C/2 = , which is positive ...(43)
2(q+C2)

w
i.e., U^-U2>0 or t/, > U2 or (/2<

Flo
This proves that total energy after contact is somewhat less than total energy before contact. Hence
there occurs a loss of energy = (C/| - U2) on sharing charges. This loss of energy appears in the form of

reee
sparking and heat produced in the connecting wire.

Retain in Memory

FFr
When charges are shared between any two bodies, their potentials become equal. The charges acquired
urr
for
are In the ratio of their capacities. No charge is really lost, but some loss of energy does occur.

A 600 pF capacitor is charged by a 200 V supply. It is then disconnected


kkss
from the supply and is connected to another uncharged 600 pF capacitor. How much electrostatic
Yo
ooo

energy is lost in the process ?


Sol. Here, C, = C2 = 600 = 600 x F = 6 x 10^^® /=; q = 200 V, V2 = 0
eB

qC2(Vj~V'2)2 _ 6x10~^Q x6x1Q-^Q (200 - 0)^ = 6 X 10“^ J


Energy lost = 2x12x10-^0
2(q + C2)
r
ou
ad

sampielprobiem!^ A 400 pF capacitor, charged by a 100 volt d.c supply is disconnected


YY

from the supply and connected to another uncharged 400 pF capacitor. Calculate the loss of energy.
Sol. Here, C, = 400 pF = 400 X 10-‘2f = 4x lO-^^F, V= 100 volt
nndd
Re

Energy stored, V = -!-.7V=-Cy2 =ix4xl0-*f’(100)2 ^2x 10-6J


Fi

2 2 2
When this charged capacitor is connected to another uncharged capacitor of same capacity, half the
charge {q!2) will flow to the unchanrged capacitor.
1 1 1 1
Final energy, V' = — ^ y = -x-qV =-
-x2xlO-®J = iO-6j
2 2 2 2 2

Loss of energy - U - U' = 2x 10“0 - 10“^ = 10“® J


2.26. NON-POLAR AND POLAR DIELECTRICS

As discussed in Art. 1.4, dielectrics are insulating materials which transmit electric effects without
actually conducting electricity.
Further, we know that in every atom, there is a positively charged nucleus and a negatively charged
electron cloud surrounding it. The two oppositely charged regions have their own centres of charge. The
centre of positive charge is the centre of mass of positively charged protons in the nucleus. The centre of
negative charge is the centre of mass of negatively charged electrons in the atom/molecule.
ELECTROSTATIC POTENTIAL AND CAPACITANCE 2/27

Dielectrics are of two types : Non-polar dielectrics and Polar Dielectrics.


Non-polar dielectrics like hydrogen, nitrogen, oxygen, CO2,
benzene, methane etc. are made of non-polar molecules. In such
molecules, the centre of mass of positive charge coincides with the
centre of mass of negative charge in the molecule. Each molecule
has zero dipole moment in its normal state. Such molecules are
symmetric, in shape as shown in Fig. 2.28(a).
Polar dielectrics like water, alcohol, NH3, HCl etc. are made
of polar molecules. In such molecules, (under normal conditions
with no external field) the centres of mass of positive and negative
charges do not coincide because of the asymmetric shape of the
molecules as shown in Fig. 2.2S(b). Therefore, each molecule
has some spontaneous (or intrinsic) or permanent electric dipole

ww
moment.

2.27. DIELECTRIC POLARIZATION

Flo
When a non-polar dielectric is held in an external electric field Eq , the FIGURE 2.29
centre of positive charge (protons) in each molecule is pulled in the direction of

ee
rere
Eq (towards negative plate) and the centre of negative charge (electrons) is

r FF
pulled in a direction opposite to Eq (towards positive plate). Therefore, the two
uurr
foor
centres of positive and negative charges in the molecule are separated. The
molecule gets distorted. Fig. 2.29, and is said to be polarised as a tiny dipole
moment is imparted to each molecule. Infact, the force due to electric field pulling FIGURE 2.30
ks s
the two centres apart and the force of mutual attraction between the centres of
Yoo
oook

-+ - +
positive and negative charges reach a balance or equilibrium and the molecule
gets polarised. -+ - +
eBB

A dielectric with polar molecules also develops a net dipole moment in an -+ - +

external field, but for a different reason. When no external field is applied, the
E = 0 ■►E
rr

different permanent dipoles of such a dielectric are oriented randomly due to


ouu

0 Non polar molecules


ad

thermal agitation, so the total dipole moment is zero. When an external electric
field is applied, the individual dipole moments tend to align with the field. When
YY

-+ - + -+ - +

summed over all the molecules, there appears some net dipole moment in the \
* -+ - +

direction of the external field, i.e., the dielectric is polarised. The extent of
nndd
Re

-+ - +
polarisation depends on the relative strength of two mutually opposing factors. The
applied external electric field tending to align the dipoles with field and thermal
Fii

E = 0

energy tending to disrupt the alignment. Fig. 2.30 shows how a dielectric develops a Polar molecules

net dipole moment in an external electric field.


Thus each molecule becomes a tiny electric dipole, with a dipole moment parallel to the external field,
and proportionalto it.
Induced dipole moment p acquired by the molecule may be written as
P = aeoEo .(44)
where a is a constant of proportionality and is called atomic/molecular polarizability.
Here, e q has been used to give the dimensions of volume to a.
From (44), we can write the dimensions of a :

a -
dimensions of p a =
Cm
= m
3

(dimensions of Sq) (dimensions of £q) (C^ N-^ m-2)(/VC"M


Thus atomic polarizability has the dimensions of volume. For most of the atoms, a is of the order of
10-29' to 10“^^ m^ ; which is also the order of atomic volume.
2/28 <t Fundamental Physics (XII) CTSIWI

Let us now consider a non-polar dielectric slab A5CD placed in an electric field maintained between
the two plates.
Suppose all its atoms are uniformly polarized in the direction of Eq and x is the displacement between
the centres of the ± charges in the atom. Dipole moment of each atom p = qx.
If N is the number of atoms per unit volume, then dipole moment per unit volume = total dipole moment
density P = Np or P = N qx
This total dipole moment density P is called electric polarization. It measures the extent of polarisation
of the dielectric. The SI units of P are C-m (nr^) = which represent a,-.
If we consider any small volume element in the interior of the slab, FIGURE 2.31
shown dotted. Fig 2.31, each volume element has no net charge though it
has net dipole moment. This is because positive charge of one dipole sits Eo
close to the negative charge of the adjacent dipole. However, at the surfaces B C

ww
t
of the dielectric normal to the electric field, there is net charge density. I
-+ -+ -+
As seen in Fig. 2.31, the negative ends of the dipoles remain unneutralized
I
at the surface AB and positive ends of the dipoles remain unneutralizedat I

FF loo
the surface CD. These are the induced charges or polarization charges -+ -+ -+
' <4
due to external field. They set up an electric field E^ opposite to E^ .

ee
I
+ I
Ep
Effective electric field in a polarized dielectric is

ee r
I

-+ -+ -+

rFrF
...(45) I
I
Here E is called the reduced value of the electric Held. I
rur
Further,
a.
I _ ife, P
ffoor
...(46)
-+
I
-+ -+


gI, A I
ks
I
where P is total dipole moment density. -+ -+ -+
Obviously, the value of E depends upon the nature of the
YYoou
i
A
ookos

D
dielectric slab introduced. It is found that
BBo

the ratio of external electric field applied (Eq) to the reduced value of electric field (E) is
re

constant for the material. This constant is denoted by K and is called dielectric constant qJ //|g
material of the slab
oouur
ad

E
Y

0
i.e., = K ...(47)
E
dndY
Re

Electric susceptibility, it is found that the electric polarization P is directly proportional to the reduced
value of electric field (£),
FFini

i.e.. PocE P-e %£■ = %€ E


or ...(48)
where %* is a constant, called electric susceptibility of the dielectric. Here, e has been used to keep %
dimensionless. The electric susceptibility describes the electrical behaviour of a dielectric. It has different
values for different dielectrics. For vacuum, X = 0
Note that eqn. (48) corresponds to eqn. (44), where Gq is replaced by g ; £q is replaced by E and a is
replaced by X-

From (45), (46) and (48) E = E^-


^

e 0 or E^Eq-xE
or
Ef^=E+xE = E(l+x) or EqIE = I + X
Using (47), K^X.±X.. ...(49)
This is the relation between dielectric constant and electric susceptibility of the material.

*X is pronounced as ‘chi’.
ELECTROSTATIC POTENTIAL AND CAPACITANCE 2/29

2.2a DIELECTRIC STRENGTH

The maximum electric field that a dielectric medium can withstand without breaking down of
its insulating property is called its dielectric strength.
This is of great practical importance.

From V =
Q
—, we find that a capacitor can hold large amounts of charge Q at a relatively small potential
C*

V, only when its capacity is large. High potential difference (V) implies a strong electric field that would

oww
ionise the surrounding air, accelerate the charges so produced to oppositely charged plates, thereby neutralizing
partly the charge on capacitor plates. It means the charge on capacitor plates would leak away due to reduction
in insulating power of the insulating medium.
Hence, for a capacitor to store large amounts of charge, its capacitance should be large enough, so that

e
the potential difference and hence the electric field do not exceed the break down limits.

re
The dielectric constant and dielectric strength of some common dielectrics are given in Table 2.2.

FFrllo
For reasons of safety, the maximum electric field actually applied is only 10 % of the dielectric

reF
strength of the material.

e
TABLE 2.2. Dielectric constant and Dielectric strengtib of some common dielectrics
ouru
osrF
S. No. Dielectric Dielectric constant Dielectric strength (kV/mm)

1.
2.
Natural rubber
Mica ffor
k
2-7-5-0
6-7
18-24
kso
80
3. Polystyrene 2-55 20-28
ooo
YYo
4. Polythene 2-3 20-160
BB

5. Teflon 2-1 16-20


6. PVC 5-6 30
r ree

7. Quartz glass 3-78


Y

8. Pyrex glass 4-6


oouu

9. Alumina 8-9-5
ad

10. Zircon 7-1-9-1


Yd

11. air 1 0006 0-8


12. Vacuum 1-00000
nidn

00
Re

2.29. CAPACITANCE OF A PARALLEL PLATE CAPACITOR WITH A CONDUCTING SLAB


FFi

The capacitance of a parallel plate capacitor of plate area A and plate separation d with vacuum/air
inbetween is

A
0
C
FIGURE 2.32
d
+ + Q
Suppose ± Q art the charges on the capacitor + +
plates, Fig. 2.32.
Eo
When a conducting slab of area A and thickness
t<d IS introduced between the plates, the free charges CONDUCTING SLAB
in the conductor flow. The charges ± Q appear on the d t

two faces of the slab as shown reducing ^e electric + E = 0 A- +


field in the interior of slab to zero. The original
uniform electric field £0 exists over a distance - V - V
Eo
V V Q
{d - r), so that potential difference between the plates ±
2/30 Fundamental Physics (XIl)EEIBl

Q
V = £oW-0 = —W-0 = Ae
(^-0
^0 0

As c=^
V

GAe ^ _ €r>0 A gp A/^ _ Cq 0


C = \ ’
j.e.,
C=7 ...(50)
Q{d-t) d 1
t
1-
/ 'l 1--
t

\ d ) d d) d)

Clearly, C > Cq, i.e.. Capacitance increases on introducing a conducting slab inbetween the plates.
When / = d, from (50), C = «», i.e., capacity becomes infinite.

ww
2.30. CAPACITANCE OF A PARALLEL PLATE CAPACITOR WITH A DIELECTRIC SLAB

The capacitance of a parallel plate capacitor of plate area A and plate separation d with vacuum/air

Floo
inbetween is
6^ A

ee
d
...(51)

reer
FIGURE 2.33

rFF
Suppose ± G are the charges on the capacitor + + + + + Q
plates which produce a uniform electric field £g in the
uur r
I

space between the plates, Fig. 2.33. Eo


When a dielectric slab of thickness t < d is ffoor DIELECTRIC SLAB
introduced between the plates, the molecules in the slab
sks
d t
YYoo
get polarized in the direction of Eq. The electric E=Eo- Ep
±Ji
ooko

+ +
polarization vector P in the direction of Eq induces an
electric field Ep opposite to Eq. Therefore, the effective Eo
eBB

Q
field inside the dielectric is E = Eq- Ep
Outside the dielectric, field remains Eq only.
r

Therefore, potential difference between the two plates is V = Eq


ouur
ad

E E
^0
Y

0
But ^ = €r or K £ = .*. V = E.{d-t) +
E K K
dY
Re

t
y = £.0 d-t-^ —
nnd

K]
FFii

As
(3 Q V -
Q
d-t + —
t
^0 = Ae Ae ATJ
^0 0 0 ‘-
.'. Capacitance of the capacitor with dielectric inbetween is

Ae A €„0 A
0 0 , I.e., c = ...(52)
1 I
d-t 1 d-t i —
K K K

Clearly, C > Cq, i.e., on introduction of a dielectric slab inbetween the plates of a capacitor, its
capacitance increases.
Particular cases

(0 When t = d, i.e., dielectric slab fits in completely between the plates of capacitor, then from eqn. (52)
ELECTROSTATIC POTENTIAL AND CAPACITANCE 2/31

£«0 A Ke^0 A
C = =/CC 0
1 d
d-d 1- —
K

The capacity becomes K times.


2KC0 2d 3KC0
(/01ft = d/2, then from (31), C = —, then from (31), C =
K +\ 3 K +2

I Find the capacitance of a system of three parallel plates, each of area A

w
metre^ separated by distances dj and ^2 metre respectively. The space between them is filled with
dielectrics of relative dielectric constants and ^2*
dielectric constant of free space is e q.
Sol. A system of three parallel plates is forming two capacitors of
FIGURE 2.34
capacities C| and C2 connected in series as shown in Fig. 2.34.

e
roow
re
C1 = and C2 =
d
1 ^2

reF
Ki K2
The equivalent capacity of the system is

uFFll
e
1-J_ J- or c =
qc2
C ~ C.I C2 C,+C2

sFr
*\
\ / d2

foro
A
K^K^A ^ ^0
0 V 0
K
ofk d,d
uor
1 d1 1“2
K,i
j \ 2 ;
C =
K
^2 A(K^d^+K^d^)
kos
1
€«0 A
Y
d, d.
Yo
reeBB
oo

Retain in Memory
uurY

1. That capacity of condenser increases both, when a conducting slab or an insulating slab is introduced
between the plates of the condenser. In the former case, £ = 0 inside the conductor and in the latter
case, E < Eq, inside the insulator. Thus, potential difference. Exd, decreases in both the cases
ad

and hence capacity increases.


doo
nY

1
Note that maximum charge induced on the dielectric/conducting slab, =-Q 1
' V ^
nid
Re

1. When a dielectric slab is introduced inbetween the plates of a charged capacitor with battery
connected across the plates:
FFi

(0 Potential V remains constant, {V = Vq) (j7) Capacity C increases, (C = ATCq)


(Hi) Charge Q increases, {Q = CV) (iv) Electric field E decreases (E = E^fK)
f 1

(v) Energy U increases, {/ = - CV^ = KU 0


2

3. However, when battery across the plates of charged condenser is off and dielectric slab is introduced
between its plates:
(0 Charge Q remains constant, {Q - Qq) (//) Capacity C increases, (C = ATCq)
V )
(Hi) Potential V decreases, v = Q (iv) Electric field E decreases £ = — = -^
C K ) I ^ Kd
0
(v) Energy U decreases. U =
2C K
2/32 ‘P'uuUep- a Fundamental Physics (XII)

2.31. ACTION OF SHARP POIMTS


When a spherical conductor of radius r carries a charge q, the surface density of charge is given by
charge q
a = =
...(53)
area 47cr-^
For a pointed end, r is very very small, therefore a is very large. Electric field intensity E = ct/Sq
becomes large. As a result, air around the sharp end becomes ionized and a bluish glow, called corona is
seen. Though air does not transfer electric charges easily, ionized air is a good conductor and hence the
charges leak from the surface into the surrounding air.
FIGURE 2.35

2.32. LIGHTNING CONDUCTOR


CLOUD
A lightning conductor is used to save big buildings from the damage by

ww
the lightning flashes.
♦+
A lightning conductor consists of a number of pointed conductors fixed I f+
to the top of a building and connected to a thick copper wire. This wire runs /

Flo
down the side of the building and ends on a metal plate buried in the ground,
Fig. 2.35.

ee
When a negatively charged cloud passes over the building, it induces a

rere
rFF
positive charge on the pointed ends of lightning conductor and an equal negative
charge at the metal plate. Due to discharging action of sharp points (corona
uurr
discharge), a positively charged electric wind is set up, which neutralises the
foor
negative charge on the cloud. The negative charge on the metal plate is
immediately neutralized in the earth. If the discharge of the cloud occurs, it
GROUND
ks s
passes through the conducting copper strip and the building is saved from the
Yoo
ook

damage.
BBo

2.33. VAN DE GRAAFF ELECTROSTATIC GENERATOR


FIGURE 2.36
re

A van de graaff generator is a device used for building up high


potential differences of the order of a few million volts. Such high C
ouur
ad

potential differences are used to accelerate charged particles like


electrons, protons, ions etc. needed for various experiments of
YY

Nuclear Physics.
nndd

It was designed by Van de graaff in the year 1931.


Re

SOURCE +
Principle. This generator is based on
Fii

(i) the action of sharp points, i.e., the phenomenon of corona


4
discharge. V

(«) the property that charge given to a hollow conductor is 4


4-

transferred to outer surface and is distributed uniformly over it. 4 V


Construction. The essential parts of Van de graaff generator
are shown in Fig. 2.36. 5 is a large spherical conducting shell of P1 P2
radius equal to a few metres. This is supported at a suitable height V

(of several metres above the ground) over the insulating pillars pj,
P2- A long narrow belt of insulating material like, silk, rubber or 4- 4

rayon is wound around two pulleys and P2* ground


+ 4
+ 1 Pi V
level and P2 is at the centre of 5. The belt is kept moving continuously
over the pulleys with the help of a motor (not shown). and B2 are
two sharply pointed metal combs fixed as shown. 5j is called the ] TARGET (T)
spray comb and B2 is called the collecting comb.
2/33
ELECTROSTATIC POTENTIAL AND CAPACITANCE

The positive ions to be accelerated are produced in a discharge tube D. The ion source lies at the head
of the tube inside the spherical shell. The other end of the lube carrying the target nucleus is earUieu.
The generator is enclosed in a steel chamber C filled with nitrogen or methane at high pressure in order
to minimise leakage of charge from spherical conductor.
Working. The spray comb S] is given a positive potential (= 10^ volt) w.r.t. the earth by high tension
source H.T. Due to discharging action of sharp points, a positively charged electric wind is set up. which
sprays positive charge on the belt (corona discharge). As the belt moves, and reaches the comb., a negative
charge is induced on the sharp ends of collecting comb B2 and an equal positive charge is induced on the
farther end of B2. This positive charge shifts immediately to the outer surface of 5. Due to discharging action
of sharp points of 62» ^ negatively charged electric wind is set up. This neutralises the positive charge on the
belt. The uncharged belt returns down, collects the positive charge from Bj, which in turn is collected by B2.
This is repeated. Thus, the postive charge on S goes on accumulating.
Now, the capacity of spherical shell C = 4 te € q /?, where R is radius of the shell.

w
Q Q
As V V =
C 47ceQ R

Flo
Hence the potential V of the spherical shell goes on increasing with increase in Q.
The breakdown field of air is about 3 x 10^ V/m. The moment the potential of spherical shell exceeds

ee
this value, air around 5 is ionised and leakage of charge starts. The leakage is minimised by housing the

Fr
generator assembly inside a steel chamber filled with nitrogen or methane at high pressures.
If q is the charge on the ion to be accelerated and V is the potential difference developed across the ends
of the discharge tube, then energy acquired by the ions = qV. The ions hit the target with this energy and carry
for
ur
out the artificial transmutation, etc.
Uses
ks
1. The beams of accelerated charged particles are often used to trigger nuclear reactions.
Yo
2. These beams are also used in the treatment of cancer.
oo

3. The beams of accelerated charged particles are used in variety of experiments in Atomic Physics and
eB

Nuclear Physics.
Limitations

1, The Van de graaf generator cannot accelerate uncharged particles like neutrons.
r
ou
ad

2. The modem day accelerators which produce much higher energies have almost replaced such generators.
Y
nd
Re

Solution. In Fig. 2.37, a charge q - % mC

I I
TYPE 1. POTENTIAL DIFFERENCE
= 8 X 10“^ C is located at the origin O. Charge to be
Fi

AND WORK DONE


carried is
WAB
Formula used. Vg - =
%
where is work done in carrying q^ charge from
A to B.

Units used. Charge qQ is in coulomb, work is


in joule and potential difference ( is in volt.
.Exa
A charge of 8 mC is located at
the origin. Calculate the work done in taking
a small charge of 2 X 10"® C from a point
P (0,0,3 cm) to a point Q (0, 4 cm, 0) via a point
R (0,6 cm, 9 cm).
2/34
Fundamental Physics (XII) VOL.I

= - 2 X 10 ^ C, from P to Q,
where OP = rp = 3 cm = 3 x 10"^ m = 2_J_
and Oj2 = rg = 4cm = 4x 10"^m (22'P. “4^l“2
As electrostatic forces are conservative forces, Clearly, V2 > Vj. Hence charge flows from
work done is independent of the path. Therefore, there sphere of radius 2 /? to sphere of radius R.
is no relevance of point R. Example B Electric field intensity at a
?^o_f J I,' point B due to a point charge Q kept at point A is
4jtg 24 NC"^ and electric potential at B due to the
same charge is 12 JC-^. Calculate the distance
1
= 9x10^ x8xI0"3(-2x10-^)
1
AB and magnitude of charge. [CBSE 2003 (C)]
.4x10"2 3x10"2
Q
1 \ Solution. Here, E = -y =24NC
-1
, and
= -144x10“^ xl()2 1. =l-2joule 47U€
0

w
12

I
Q

I
V = = 12JC-^
type II. ELECTRIC POTENTIAL 4tc£„0 r

FF llowo
Formulae used.
V 12
I. Potential at a point due to single charge, Dividing, we get — = r = — = 0-5 m = AB

ree
E 24

V =—^— From V = Q

rF
47tE„ r 4te€o r
2. Potential at a point due to group of N charges. 1
ur
i=N G = X 0-5x12 = 0-667 X 10-^ c
V =
47tE
I for Ffre o 9x10^
0 ,=i 'i-
Example 0 (a) Calculate the potential
ks
at
3. Potential at a point due to electric dipole,
YYour o
a point P due to a charge of 4 x 10"'^ C located
oo

V =
pcosB 9 cm away,
4jie^0 (b) Hence obtain the work done in bringing
a charge of 2 x 10“^ C from infinity to the point P.
Bo kBs
r ee

Units used, charges q, Q are in coulomb, dipole


moment, p = g (2 a) in C-m, distance r in metre, Does the answer depend on the path along which
potential V in volt. the charge is brought ? (NCERT Solved Example)
oour u
ad

Solution, (a) Here, ^ = 4 x 10"^ C,


Y

Example S Two metallic spheres of radii r = 9cm = 9x 10~^m; V=1


R and 2 R are charged so that both of these have V = —^— = 9x10^ (4x10-'^)
nd
Re

same surface charge density, o. If they are 47t€o 9x10-2


connected to eachother with a conducting wire,
FFind Y i

= 4 x 10** volt
in which direction will the charge flow and why ? (b) W=qxV=(2x 10"^)x4x 10**
(CBSE 2016)
= 8x10-5j
Solution. Let R^ = /?, /?2 = 2R.
The answer does not depend upon the path
If 2, and Q2 are the respective charges, then as along which the charge is brought because
a =
g. _ g2 electrostatic forces are conservative forces.

4 7C R 2 4 71 ^ Example An infinite number of charges


R?1 \2 each numerically equal to q and of the same sign are
01 R 1
placed along the x-axis at a: = 1, at = 2, jc = 4,
02 2R 4 a: = 8 and so on. Find electric potential at a: 0.
01 02 Solution. Using superposition principle, we may
As =
^2 = write electric potential at the origin (x - 0) due to
47C€o /?, ’ 47ceQ /?2 ' various charges as
ELECTROSTATIC POTENTIAL AND CAPACITANCE 2/35

1 Q Q Q FIGURE 2.39
V =
4 Tie 0 1 2 4 8 q
h
1 1 1 1 1m o 2m
A
- + - + -^ + ^ + B

4tc6qLi 2 2^ 2^ B T
O
a
As sum of infinite G.P. series, S =
l-r
1m
where a is first term and r is common ratio.
q H
1 1 2g o 2m
A
O
4tC€ 0 {1-1/2)J 4tc€ 0
Solution. Here, ^ = + 1 )X C = 10“^ C,
;Exam r|«;I3 Two charges 3 x 10“® C and

ww
AO -2 m, BO - 1 m
- 2 X 1(H C are located 15 cm apart. At what point As shown in Fig. 2.39(a),
on the line joining the two charges is the electric
potential zero ? Take the potential at infinity to g r 1 1'

Floo
be zero. (NCERT Solved Example) 47C€n0 Lao bo.

ee
Solution. Let the location of positive charge be
taken as the origin O, Fig. 2.38 ; and line joining the 1

reer
= 9x10^x10“^ -1 =-4500 V

rFF
two charges as .x-axis. Let P be required point, where 2
potential is zero. Let OP = a: cm.
If position of B is changed as shown in Fig. 239(b),
uur r
FIGURE 2.38 value of p.d. (V^ - Vg) shall remain the same. This is
ffoor
0 P A because electric potential is a scalar quantity and its value
does not change with change in orientation of the
sks
3*10"®C-^ -2 »10"®C
YYoo
15 cm- charges.
ooko

Potential at P is ; Example^ Four point charges + 1 pC,


eBB

+1 pC, -1 pC and -1 pC are placed at the corners


3xlQ-^ -2x10-®
V = A, C and D of a square of each side 0-1 m
4neQ (xxlO”^) 4tiGq (15-x)x10“^ (i) Calculate electric potential at the centre O of
r
ouur

the square (ii) If E is middle point of BC, what Is


ad

= 0
work done in carrying an electron from 0 to E7
Y

3 2
— = 0, which gives x = 9 cm. Solution. In Fig. 2.40,
X \5-x
nd dY

AB = BC=CD = DA-a-0-\m
Re

If P lies on extended line OA, the condition for


V2
FFiin

3 BD = AC =
^ = 0,
= a
zero electric potential would be —
X jc-15
1 a
which gives a: = 45 cm OA = OC^OB = OD = -a
Hence electric potential is zero at 9 cm and 45 cm
2 iT
away from positive charge on the side of negative
charge.
Potential at infinity is taken zero in the formula
used.

;Exarripj[b^ A charge ^ = +1 pC is held at


0 between two points A and B such that AO
= 2 m and BO - Im, Fig. 239(a). Calculate the
value of potential difference (V^ - Vg). What will
be the value of potential difference (V^ - Vg) if
position of JJ is changed as shown in Fig. 2.39(d) ?
2/36
Fundamental Physics (XII) VOL.I

Potential at O, =
1
X
inserted between the plates, while the voltage
0
4 Tie supply remains connected ?
0

(CfiSE Sample Paper 2022*23)


1x10"^ . 1x10“^ 1x10“^ 1x10“^ Solution.
(a)
OA OB OC OD

10"^ 1 1 1 1
4 Tie
0 Lfl a / V2 a! -y/l a! -Jl
= 0
Now, BE = CE = a/2

Again, AE = DE = 4dC^ + CE^

ww
5a^ a sr
14 2
1

FF loo
Potential at E, = X
4 Tie
0

ree
1x10“^ 1x10“^ 1x10-6 1x10-6

rreeF
AE BE CE DE

10-6 2 2 2 2
ur
4 Tie
0 10
+
a a a
= 0.
fofroF
Work done in carrying an electron of charge
ks
(- e) from O to £
kos o
YYouor

W^-e[V^-Vo]=-e[0-0] = Zcro.
BBoo

TYPE 111 POTENTIAL GRADIENT


r ee

AND ELECTRIC FIELD

dV
ad

Formulae used. E=-


oouur

. It follows that
dr
Y

BV BV
E . F
E
X
Bx ’ ^ By ' 2
d

Bz
Re

(b) Here, A = 6 X lO-^m^,


dnY

Units used. , E. and E are in N/C or V/m, dV


FFini

is in volt and dr is in metre. c/ = 3 mm = 3x10"^ m

A 8-85x10-'2 x6x10'3
Example Q (a) Draw equipotential (0 C = 0

surfaces for (1), an electric dipole (it) two identical d 3 X10-3


positive chaises placed near eachother. = 17*7 X 10“^^ farad
(b) In a parallels plate capacitor with air bet (ii) Charge on each plate
ween the plates, each plate has an area 6 x 10"3
Q=CV=17-7x 10-'2x 100 = 17*7 X 10"*® C
m^, and the separation between the plates is 3 mm.
(Hi) New charge Q' = KQ = 6x 17-7 x 10"^^ C
(i) Calculate the capacitance of the capacitor,
= 1-062 X 10-* C
(li) If the capacitor is connected to 100 V
supply, what would be the charge on each plate ? Example [0 Xwo identical plane metallic
(Hi) How would charge on the plate be surfaces A and B are kept parallel to each other in
affected if a 3 mm thick mica sheet of AT = 6 is air separated by a distance of 1-0 cm as shown in
Fig. 2.42. Surface A is given a positive potential of
ELECTROSTATIC POTENTIAL AND CAPACITANCE 2/37

10 V and the outer surface of B is earthed, TYPE IV. ELECTROSTATIC


(i) What is the magnitude and direction of uniform POTENTIAL ENERGY
electric field between points Y and Z ? (ii) What is
work done in moving a charge of 20 |xC from point Formulae used. 1. Electrostatic potential energy of
X to Y, where X is situated on surface A ? a system of two point charges, u =
47tE„r,2
FIGURE 2.42
A B
2. Electrostatic potential energy of a system of «
1
●X
point charges, u =
4TC€
- X
0 all pairs 0*
Y z
Units used. All charges in coulomb, distance r in
metre, energy U in joule.

ww
Solution. Magnitude of electric field,
Example m Three charges -q^Q^-q are
placed at equal distances on a straight line. If the
dV lOV potential energy of the system of these charges is
= 1(P Vm"*

Flo
E =
dr lxl0“^m zero, then what is the ratio Qlq ?
(CBSE Sample Paper 2022-23)

ee
The direction of electric field is from Y to Z.

rere
As surface A is an equipotential surface, Solution. As shown in Fig. 2.44, potential

rFF
therefore pot. diff. between X and Y is, i.e., AV = 0 energy of the system of three charges is
Work done = q . AV = 0, i.e., no work is done
uur r
FIGURE 2.44

in moving a charge from X to Y. foor -4

X
Q
+
X
-4
ks s
Example m Three points A, B, C lie in a
Yoo
uniform electric field £ of 5 x 10^ N/C, as shown
ook

K(-q)Q . KQ{-q) , k{-q)(-g) = 0


in Fig. 2.43. Find the potential difference between
BBo

A and C.
X X (2x)
re

Kq^ 2KqQ
FIGURE 2.43 2x X
ouur
ad

^ B Q 1
<7 = 4 Q or
YY

>
q 4
5 cnf I 3 crry Example m Two positive point charges
nndd
Re

of 0’2 |iC and 0*01 pC are placed 10 cm apart


FFii

Calculate the work done in reducing the distance


to 5 cm.
(CBSE (F) 2009)
Solution. Here q^ = 0-2 x 10"^ C,
Solution. In Fig. 2.43, as B and C are in same q-j - 0 01 X 10“^ C, T2 - 0 05 m, r^ = 0-10 m
vertical line, therefore = Vq Work done = Change in RE. = Final RE.
Now, potential difference between A and C - Initial RE.
= potential difference between A and B 1 1
4| 42
VAC = VAB = -^ (Ax)
47C€o [^2 'iJ
Now, Ax=AB = ^IaC^-BC^ = -32 = 9 X 10^ X 0-2 x 10-^ X 0 01 X 10“^
= 4 cm = 4 X 10"^ m 1 1
V = -5x 10^x4x 10-2
AC~
X
005 0-10
= - 200 V = 1-8 X 10^ J
2/38 '®*‘***^®^ Fundamental Physics (XII)CQISI
Example m Two point charges 20 x Example [0 Four charges are arranged at
10^ C and - 4 x 10^ C are separated by a distance the comers of a square ABCD of side d, as shown
of 50 cm in air. Find (i) the point on line joining in Fig. 2.46. Find the work required to put together
the charges, where the electrostatic potential is this arrangement {b) A charge is brought to the
zero (ii) Also, calculate the electrostatic potential centre E of the square, the four charges being held
energy of the system. (CBSE (OD) 2008) fixed at the corners. How much extra work is
Solution. needed to do this ? (NCERT Solved Example)
FIGURE 2.45 FIGURE 2.46
A P B d
4 Ar^ -q
B
20x10“®C -4x10“® C

ooww
h- 50-x -*●!
50 cm
^0
d
Suppose the point of zero potential is located at E

a distance x metre from the charge 20 x 10“^ C, Fig.


2.45.

e
D C

re
-q +q
1 20x10"^ 4x10“^

rFFl
then, V = = 0

ree
F
47ue X 50-x Solution, (a) Here, AB-BC = CD = DA = d
0

X - 0-41 m = 41 cm from 20 x 10“^ C charge

rF
AC=^BD = ^d^+d^ =d^
1
RE. of the system (/ =
fsfoor Total work done = RE. of system of 4 charges
ouur
471^0 ^
1
kosk
_ 9xl0^x20xl0"®x(-4xl0-^) 47te 0
0-50 all pairs
YYo
oo

= -l*44J 1
BB

[q (~q) + i-q) (+q)


Example m Two particles have equal 4ice^0 d
masses of 5*0 g each and opposite charges of
rre

+ 4'0 X 10~® C and - 4*0 x 10^^ C. They are released + q(-q) + (-q) (+?)] +
ouu

from rest with a separation of 1*0 m between them. 47teQ d V2


Y
ad

Find the speeds of the particles when the


dY

separation is reduced to 50 cm. -4^2 2^2


Solution. Here, m, = m2 = 5-Og = 5 x 10"^ kg , 4tU€qC? 47tGQ^/V2
=4 0x 10"^ C. ^2 = -4-0x 10“^C
innd
Re

1
= -:^(4-V2)
FFi

r| = 1-0 m, T2 = 50 cm = — m
2 4tc€q d
From symmetry, v^=V2 = v = 7 (^) Extra work needed = charge x potential at E
As increase in K.E. = loss in RE. - final .PE. -
due to charges at four comers
initial .PE.

(q-q+q-q)
2x-mv^ = ^1 11^2
1
.●. L_1 W = q^ - zero
2 4ns
0 L^2 ^l. 4ne
d^
0
1 2 J
5 X 10-31,2 _ 9 X 10^ (4 X 10-3)2 1/2
-1

jExampib|0 (a) Determine the electro


= 9x16x10-' (2-1)
static potential energy of a system consisting of
14-4
V
2 _
= 2-88x1Q3 two charges 7 p. C and - 2 p C (and with no external
5x10-3 field) placed at (- 9 cm, 0, 0) and (9 cm, 0, 0)
V - 53‘7 m/s respectively.
ELECTROSTATIC POTENTIAL AND CAPACITANCE 2/39

(b) How much work is required to separate


the two charges infinitely away from each other ?
(c) Suppose that the same system of charges
is now placed in an external electric field
E = A X 1/r^, where A = 9 x 10® C m"^.
What would the electrostatic energy of the
configuration be ? (NCERT Solved Example)
9,
Solution, (a) U =
47ieo r
7xlO“^x(-2xiO-^)x9xlO^
(9 + 9)x10“2
= - 0-7 J

w
Angle between Fj and F2 ZA'AB'
(b) Work required = U2~ U^ = 0 - U = 0'7 3
= 180“-60“
(c) In the electric field, total P.E.

Flo
= 120“
^2
47ie
- + <?: y ]0) + Q2^(^) Resultant force on charge ^ at A is

ee
0

-dV R =
Vf,2 + F2+2F,F2 cos 120“

Fr
From E -
dr

f ^ dr
^ =-
^ 2kq^ -1
■>
2kq^ Akq'-

for
V= -Edr = -— + 2x
ur
X
" /2 2
+
J r
/2 /2
Total P.E. =
qx ^2 Aq^ Aq^
kq^ V4+16-8 = Vl2 kq^
ks
47l6r. r '2 /? = ; along AD
Yo
0
/2 /2 ’
oo

9x10® (7x10“^) 1
= (-0-7) +
eB

009 Here, k -
4jte„ ■
9x10® (-2x10-^) (b 1 Work done to separate the charges
r

009
= potential energy of the system
ou
ad

= - 0-7 + 70 - 20 = 49-3 J

-0 all Ipairs ^
Y

1
Example [0 {q) Three point charges
4ke
~ A q and 2 q are placed at the vertices of an y/-'
nd
Re

equilateral triangle ABC of side / as shown in Fig.


2.47. Obtain the expression for magnitude of the
1 q{2q) , 2q{-Aq) _ q{-Aq)
Fi

resultant electric force acting on charge q. 47C€n L I I /

(A) Find out the amount of work done to


separate the charges at infinite separation. -\Qq^
47cen0 I
(CBSE 2018)
Solution, {a) As is clear from Fig. 2.47, force Negative sign for work required to be done.
on charge ^ at A due to charge 2q at C is
Example m Three point charges + 0,
= alongA4' + 2 Q and - 3 g are placed at the vertices of an
equilateral triangle ABC of side /. If these charges
Force on charge q at A due to charge - 4 ^ at are displaced to the mid points Aj, and Cj
Sis
respectively, find the amount of work done in
shifting the charges to their new positions.
alongAB'
(CBSE (OD) 2015)
2/40
‘P'uideefi. A Fundamental Physics (XII)
Solution. As is clear from Fig. 2.48, FtGURE 2.49

Q 4
A

w
e
row
re
F - along AC
AjF| - SjCj - AjC,
_AB _ I

eeF
FFllo
Resultant of Fj and F2 is
~ 2 ~2

u
Initial potential energy of the system, R =
a

sFrr
U. =
t
I \Q{2Q) , 2Q{-2>Q) ^ Q{~^Q)' R is along AC produced.
4tcg / / /

kro
0 Resultant force on charge Q at C is
uor
V:« =
-7G^ offo F' = F + F
kQQ
47tG.^0 / (cz>^)2
kos
Y
_AB _ I
Yo
Now AjFi =F|Cj = C,A ~ 2 “2
eerBB
oo

Therefore, final potential energy of the system. Q


rY

R' =
1
(/ g(20,2Q(-3Q) _ Q(-30 47l6nfl‘
0 L 2
/ “ 4tC€
u

//2 1/2 1/2


0 (b) Potential energy of the system
ou
ad
do

1 14
£/ = (1/4ji£„) X
nY

47TS
0 all pains 0^
Work done, IV = - t/- I
^+ + gg
nid

<IQ
Re

14 7g2 7g^ 4tC€


0 L
<7 a a a a
FFi

4tI€., / 47CG..0 I 4716.,0 I


0 1 4gg , Q^ + g^
4 JIG
Example ^ Four point charges 0 q, Q
a a
0 L

and q are placed at the corners of a square of side


a as shown in Fig. 2.49. Find (a) resultant electric
force on charge 0 and (b) potential energy of this
I TYPE V. CAPACITANCE

system. Formulae used.


(CBSE 2018)
Solution, (n) As is clear from Fig. 2.49, 1. Capacity C - 0V
2. Capacity of a sphericiU conductor, C = 4 jc g q r
Fj = , along DC Units used. C is in farad ; 0 is in coulomb ;
V is in volt; r is in metre.
1
F2 a
along FC Standard Value. = 9x10^ Nm^ C"2
4^
ELECTROSTATIC POTENTIAL AND CAPACITANCE 2/41

pie m Eight identical spherical


I I
Exam
TYPE VI. PARALLEL PLATE CAPACITOR
drops, each carrying charge 1 nC are at a potential
of 900 V each. All these drops combine together to Formulae used. 1. Capacity of a parallel plate
form a single large drop. Calculate the potential
of this large drop. Assume no wastage of any kind.
Take capacitance of a sphere of radius r as
capacitor with air as dielectric, C = ^
proportional to r. (CBSE Sample Paper 2015) 2. Capacity of a parallel plate capacitor with
Solution. Here, C ^ roi C = kr €«0 A
insulating medium as dielectric, C = K
Charge on each small drop, d

q = CV=(krx900)C Units used. C in farads, ^ in m^ ; </ in metre


Let R be radius of large drop Standard Value. Eq = 8-85 x 10”’^ C^ N"* m"“

ooww
As volume of large drop = Volume of 8 small drops,
Example^ Two parallel plate air
-kR^ or/? = 2r.
3 3 capacitors have their plate areas 100 and 500 cm^
respectively. If they have the same charge and
Capacitance of large drop, C = kR = 2kr

e
potential and the distance between the plates of

ere
Potential of large drop the first capacitor is 0*5 mm, what is the distance

rFl between the plates of second capacitor ?

Fre
q' 8^ 8 x/crx 900
= 3600 V Solution. Here, two parallel plate capacitors

rrF
c c 2kr
have same charge q and same potential V, so they
Example m A capacitor of unknown have equal capacitances as

sffoo
ouur
capacitance is connected across a battery of V C = ^/V. C, = C2
volts. The charge stored in it is 360 pC. When
kosk
^0 A ^0 A A,
potential across the capacitor is reduced by d
or d2=~ r/j
^2 A
Yo
120 V, the charge stored in it becomes 120 pC. I
oo
Y

Calculate (/) the potential V and the unknown Now, AI = 100 cm^, A2 = 500 cm^.
BB

capacitance C. {ii) What will be the charge stored d, = 0-5 mm - 0 05 cm


rre

in the capacitor, if the voltage applied had


500x005
increased by 120 V? (CBSE 2013) - 0-25 cm = 2*5 mm
ouu

100
Y
ad

Solution. If capacity is C pF, then as per the


statement.
dY

Cx V=360 ...(0
Example ^ A parallel plate capacitor has
plate area 25*0 cm^ and a separation of
innd

C(V- 120)= 120


Re

2*0 mm between the plates. The capacitor is


Dividing, we get connected to a battery of 12*0 V (a) Find the
FFi

charge on the capacitor (b) The plate separation


C(y-i20) _ 120 _ 1 is decreased to 1*00 mm. Find the extra charge
CxV ~ 360 ~3 given by the battery to the positive plate.
3 V-360= V Solution. Here, A = 25*0 cm^ = 25-0 x 10^ m^,
or V=180 volt d = 2*0 mm = 2 x 10"^ m, V= 12 volt
(a) Charge on the capacitor, q = 7
From (/), C =
360 _ 360
= 2pF A
y ~ 180 q‘ = CV=-^ y
d
New voltage, V' = V+ 120= 180+ 120
= 300 volt
8-85x10-^2(25.0x10^)x12
New charge. 2x\0~^
Q' = CY = 2x300 = 600 pC = 1*35 X 10"‘® C
2/42 ^>uicUe^ <1 Fundamental Physics (XII) iWl
{b) When plate separation is decreased to half, Example^ The thickness of air layer
capacity becomes twice. The charge {q' = CV), between two coatings of a spherical capacitor is
becomes twice. Hence extra charge given by the
2 cm. The capacitor has same capacitance as the
battery = q'-q = 2q-q-q - 1*35 x C
sphere of 1*2 m diameter. Find the radii of its
Example^ A parallel plate capacitor is surfaces.
to be designed with a voltage rating 1 kV using a 4;re„0 ab
material of dielectric constant 3 and dielectric Solution. Here, = R
0
strength about 10^ Vm~^ [Dielectric strength is b-a
the maximum electric field a material can tolerate
ab
without break down, i.e., without starting to = R
b-a
conduct electricity through partial ionisation. For
safety, we should like the field never to exceed say 1-2
As b-a = 2 cm R = m = 60 cm
10% of the dielectric strength]. What minimum 2

w
area of the plates is required to have a capacitance
ab
of 50 pF? — = 60 or ab =\20
Solution. Here, V = 1 kV = 1000 volt; 2
As (b + a)2 = (b- a)2 + 4ab = (2)2 + 4x120 = 484

Flo
K-e^ = 3 ; Dielectric strength = 10^ V/m b + a = 22
As electric field at the most should be 10% of

eeee
2 + a + a = 22 [●.● b-a = 2 cm]
dielectric strength, due to reasons of safety,

Fr
E = 10% of 10"^ V/m = 10^ V/m. A = ?, a s 10 cm, 6 = 12 cm
C = 50 pF = 50x 10-‘2f Example W Calculate the capacitance of

for
ur
a spherical capacitor consisting of two concentric
As, E = -
V
V _ 10^
d = - = 10 ^m spheres of radii 0*50 m and 0*60 m. The material
d £ ~ 10^ filled in the space between the two spheres has a
ks
dielectric constant of 6.
Yo
Now, C = Solution. Here, C = ?, = 0-50 m,
oo

d
= 0-60 m, K= 6
B

Cd 50x10-*2x10-3
re

C = KCr.0 = K
8-85x10-'2x3
% - r
a
ou

= 1*9 X 10-3 ^2
ad

6 (0-5)(0-6)
= 2 X 10-5 F
Y

9x10^ (0-6-0-5)
TYPEVH. SPHERICAL AND
CYLINDRICAL CAPACITORS Example ^ What is the capacitance of a
nd
Re

1 m long hifi cable where the central conductor is


Formulae used. 1. Capacity of a spherical air
Fi

1 mm in diameter and the shield is 5 mm in


diameter ?
capacitor, C = 4 Tce 0 a

Solution. Here, C = ?, £ = 1 m
2. Capacity of a cylindrical air capacitor, 1
r_ = - mm - 0-5 mm = 5 x 10"^ m.
a
2ner.0 L 2
C = a
5

.ojf J
%
— mm = 2-5 mm = 2-5 x 10-3
^
For a cylindrical capacitor.
m

Units used, , £ are in metre ; 2ti€«0 £ 47C€rt0 £


C =
€q is in m~^ ; C is in farad.
Standard value.
1
= 9x lO^Nm^C^
iog^ r.
4tC€ 0 ;
ELECTROSTATIC POTENTIAL AND CAPACITANCE 2/43

Ixl Solution. Suppose a parallel combination of n


2-5x10-3
capacitors is connected in series with a series
9x10^ x21og^ combination of (7 - n) capacitors.
5x10“^ ^ Capacitance of parallel combination,
1 10-9 Cj = 2 rt )iF
C =
18xl09log^5 18x1-609 Capacitance of (7 - n) series combination.
= 3-45x 10“’'F =34-5pF
<^2 =
Example PH The outer cylinder of a
cylindrical capacitor of length 0*15 m and radii As these two combinations are in series, and
1*61 cm and 1*5 cm is earthed while inner cylinder 10
of this capacitor is given a charge of 8 |iC. Find Cy = —
11
pF, therefore, from
the capacitance and potential of inner part of the

w
11
capacitor. L-_L + -i--J 1-n

Solution. Here, / = 0-15 m ; C


s
C
1 Cl 2n 2 10
- 1-61 X 10“^ m ; = 1-50 x 10"^ m

Flo
Multiplying both sides by lO/i, we get
^ = 8-0xl0-^C;C = ?,F=? 5 + 5 n (7 - «) = 11 «

reeee
For cylindrical capacitor, 5 + 35 n - 5 = 11 n
2ne^0 I

Fr
C =
or 5 - 24 n - 5 = 0

% («-5)(5n + 1) = 0
2-303 log
r

for
Rejecting negative value, we get n = 5
ur
Hence parallel combination of 5 capacitors
27t(8-854xl0-’2)(0-15) must be connected in series with series
kss
1-61x10-2 combination of 2 capacitoi^.
Yo
2-303 log 10
oo

1-50x10-2 Example SO Find the charge appearing


eB

C = M8 X 10-^® F on each of the three capacitors shown in Fig. 2.50.


8-0x10-^
V = B. = 6*78 X lO'^ V
r

FIGURE 2.50
C 1-18x10-’°
ou
ad

I
YY

TYPE VIII. grouping OF CAPACITORS


B C
Formulae used. 12V-± ~4nF
nd
Re

4jiF
1. Series combination :
Fi

J_-_L ^ ^
—— + ^—— +

1 C, C3 Solution. Capacitors B and C are in parallel.


2. Parallel combination ;
Therefore, their total capacity = 4 + 4 = 8pF
Cp = Cj + C2 -F C3 +. The total capacity of 8 p F and capacitor A which
3. Q = CV are in series w.r.t. battery is given by
Units used. Cj, C2, C3... ; C^, Cp all in farad, V in
volt, Q in coulomb. =_+_=_ C=4pF
8 8 4 "

Exampie§H Seven capacitors, each of Total charge = V = 4 x 12 = 48 p C


capacitance 2 pF are to be connected in a This is the charge on A, and also on the
configuration to obtain an elective capacitance combination of B and C. As capacities of B and C are
of (10/11) pF. Suggest a suitable combination. equal, the total charge is shared equally between them.
2/44 7\etd€^'^ Fundamental Physics fXlI>PTSTWi
48 Calculate the equivalent capacitance
/. Charge on B = charge onC= — = 24pC between the points P and Q, If a battery of 10 volt
is connected between these points, what will be
Example Find the charge supplied by the charge on each capacitor ?
the battery in the arrangement shown in Fig. 2.51. FIGURE 2.53

FIGURE 2.51 C1

S.O^F e.OjiF C2
s
A B
P R Q
C3 C5
h
10V
C4

w
Solution. As is clear from Fig. 2.51, left plate Solution. The given circuit can be redrawn as
shown in Fig. 2.54.
of capacitor A is + and right plate of capacitor A is

Flo
negative. Again left plate of B is negative and right FIGURE 2.54

plate of B is positive.

eee
Ci C2
R
The two capacitors are, therefore, in parallel.

Fr
Their combined capacity is
C5
= C, + C2 = 5-0 + 6-0 = 11 p F

for
ur
Charge supplied, P S
^ = 11 X 10 = 110pC C3 C4
ks
Exsm
p*e Find the capacitance of the Clearly, the capacitors form Wheatstone bridge
Yo
oo

combination shown in Fig. 2.52 between A and B. arrangement.


eB

FIGURE 2.52 ^1 _ ^3
As C| — C2 — C3 — C4
2iiF 2nF 2mF C2 c.
) /I A
r

Hence the bridge is balanced. The points R and


ou
ad

S are at the same potential. No charge can accumulate


YY

:=FZ2iiF on C5, which thus becomes ineffective.


IjiF ImF imf
Between P and Q, two series combinations (Cj,
nd
Re

B C2) and (C3, C4) are connected in parallel. If C and


C" are the equivalent capacitances of these
Fi

Solution. Starting from the left side of network, combinations, then from
two 2 pF capacitors are in series. Therefore, J---L J_-i 1- - ; <r = 2pF
c'" c, C2 ~ 4 4 " 4
Cs 2 2 ^ Similarly, C" = 2 p F
This 1 pF and third 1 pF capacitor are in Hence equivalent capacitance between P and Q
parallel. Therefore, Cp = 1 + I = 2 pF and so on. is C=<r + C" = 2 + 2 = 4 pF
Proceeding in this way, finally two 2 pF As pot. di ff. between P and 0 is V = 10 volt and
capacitors are in series. Therefore, capacitance of the equivalent capacitance of C] and C2 in series is
combination between A and B is 1 p F. 2pF.

Example^ In Fig. 2.53, the values of .●. Charge on each of capacitors Cj and C2 is
= 2x 10 = 20 pC
capacitances are as follows :
Similarly, charge on each of capacitors C3 and
Cj = C2 = C3 = C4 = 4 p F, Cg = 5 p F C4 is also 20 pC. Charge on Cg = 0
ELECTROSTATIC POTENTIAL AND CAPACITANCE 2/45

Example A network of four 10 fiF Fig. 2.56. What is the magnitude and nature of
capacitors is connected to a 500 V supply, as shown charge on plate 1 and 4 respectively ?
in Fig. 2.55. Determine the (a) equivalent FIGURE 2.56
capacitance of the network and (b) charge on each
capacitor. (CBSE 2010) 1 2 3 4 5
(NCERT Solved Example)
m
+ 4-
+ - +
+ — - +

A B C D
FIGURE 2.55
+
+
C
+
+

C2 Solution. The five plates make up four identical


capacitors A, B, C, D, each of capacity (SqAIcI),

ww
+ + + +
connected in parallel.
Cl C3
+ + + +
As plate 1 is connected to + of battery and is a
part of one capacitor only, therefore, charge on it is

FF loo
C4
+ A
0
+
^1 = + V

ee
A
D
+
+
d

Fe r
500 V
However, plate 4 is connected to negative of
battery, and is common to two identical capacitors C

Frre
Solution, (a) In Fig. 2.55, Cj, C2, C3 are
and D in parallel, therefore,
rur
connected in series

I 1
+— +
1 3
fforo
^4 = -(<?c + ^d) = -2^] =
— 2Gj,Lv
A
ks
Cs ,C1 d
^2 10
YYouo
Example m Find equivalent capacity
okso

10
between A &B, Fig. 2.57. (Bihar Board 2012)
BBoo

This combination is in parallel with C4.


r ee

FIGURE 2.57
Equivalent capacitance, C = + C4
3pF 3pF 3iiF
= 153 + 10= 13-3 pF
ad

K L
oouur

A M
Y

(b) Charge on C,, C2, C3 is same = Q, say.


Let charge on C4 be Q'. 2pF 2pF 3|rF
dnYd
Re

A + ^ + ^ = 500
FFini

C
I C2 C3 B N
P 0
1 3pF 3|iF 3pF
Q = 500
Cs
Solution. In Fig. 2.57, if C[ is capacity of
10 1 111,^ , c
or Q = 500C^ =500x — [iC = 1-7 X 10-3 C LMNO, — = - + - + -=1, Ci = lliF
C^ 3 3 3
Q' This is in parallel with 2jxF capacitor in LO.
Also, = 500, O' = C4 X 500 = 10 X 500 p C
^4 C2 = 1 + 2 = Spf ■
= 5 X 10"3 C Similarly, capacity across KP,
Example EQ Five identical capacitor C3 = 1 + 2 = 3pF.
plates each of area A are arranged such that Capacity between A and B = C4 is given by
adjacent plates are at a distance d apart The plates 1 111,
are connected to a source of EMF (V) as shown in
<^4
2/46 “P>vxiUep- 4^ Fundamental Physics (XII)

Example ^0 hi Fig. 2.58, find equivalent Electrostatic energy stored


capacity between A and (Uttarakhand Board 2012)
t; = ic,v2
2 '
=ixl2xl0-‘2
2
(50)2
FIGURE 2.56

2pF t/=I-5xlO-«J
A*-
Now, C2 = 6pF = 6x
8pF 1^IF 1pF
L-_L J_-J_ i-A-i
c.~~~
q C2 ~n'^6~i2~ 4
4pF 4|iF
B C, = 4pF = 4x 10-^2 p

w
Charge on each condenser.
Solution. The equivalent circuit is shown in Fig. ?l = 92 = q'V = 4xl(r‘2x50
2.59.
= 2 X 10-^® C

Flo
FIGURE 2.59

eeee
2x1Q-‘Q 50
2pF volt
1
C 12x10-‘2 ” 3

Fr
A 1

8pF _ ^2 _ 2xiq-^Q _ m volt


for
ur
2pF
^^2 = 6x10-12“ 3
^2
8pF
Example nil An electrical technician
ks
Yo
B
requires a capacitance of 2 )iF in a circuit across a
oo

potential difference of 1 kV. A large number of 1


B

pF capacitors are available to him, each of which


ia.i = l, can withstand a potential difference of not more
re

C, 2 2
than 400 V. Suggest a possible arrangement that
Ci = lpF requires minimum number of capacitors.
ou
ad

Solution. Here, total capacitance, C = 2 p F,


1 _ 1 1 _ 2
Y

and Potential diiference, V= 1 kV = 1000 volt


Capacity of each capacitor, Cj = 1 p F,
nd
Re

8 Maximum potential difference across each,


... = V' = 400 volt
Fi

Let n capacitors of 1 p F each be connected in


Between A and B ; Cj, C2 are in parallel. series in a row and m such rows be connected in
*■* ^AB = C] + C2 = 1 + 4 parallel as shown in Fig. 2.60.
= 5pF FIGURE 2.60

Exampi«;§Q A 12 pF capacitor is Ci Ci Cl

connected to a 50 V battery. How much


electrostatic energy is stored in the capacitor ? If
another capacitor of 6 pF is connected in series
with it with the same battery connected across the
combination, find the charge stored and potential
difference across each capacitor. (CBSE 2017) 1000V#-

Solution. Here, C, = 12 pF = 12 x 10"'2 F


As potential difference across each row = 1000
V = 50 volt.
volt
ELECTROSTATIC POTENTIAL AND CAPACITANCE 2/47

Potential difference across each capacitor ^4 = C4V4 = (100 X 10-^2) 200 = 2 X 10-« C
1000 Pot. diff. across 200 pF = 300 - 200 = 100 V
= V' = 400
n This must be the pot. diff. across C^
1000
i.e.
Vj = 100 V
= 2.5
n =
400 = Cl Vi =(100 X 10-^2) X 100 = 10"* C
As n has to be a whole number (not less than Also, pot. diff. across C2, C3 in series = 100 V

w
2.5), therefore, n = 3 100
= 50V
Capacitance of each row of 3 condensers of ●●2 3 2
1 \x F each, in series = 1/3.
= C2V2 = (200 X 10-^2j X 50 = 10-* C,
Total capacitance of m such rows in parallel
^3 = C3V3 = (200 X 10-‘2) X 50 = 10“* C

e
m m
= 2(pF) or m = 6.

row
3

re
TYPE IX. ENERGY STORED IN A
total number of capacitors = nxm = 3x6 CAPACITOR AND LOSS OF
= 18 ENERGY ON SHARING CHARGES

FFllo
F
Hence 1 p. F capacitors should be connected
Formulae used.
in six parallel rows, each row containing three

u
ree
capacitorsin series.
Example m Obtain equivalent capaci
1. Energy, U = iQv = ieiacv^

sFr
2 2 C 2

kro
tance of the following net work, Fig. 2.61. For a 2. In series as well as parallel combination of
300 V supply, determine the charge and voltage
uor
capacitors, total energy stored is sum of the energies
across each capacitor. (CBSE 2008) offo stored in individual capacitors.
3. Common Potential,
kos
FIGURE 2.61

V'=illi2_ =
YYo
eerBB

Ci = 100 pF
oo

Cj +C2 C^+C^
rY

4. Loss of energy on sharing charges,

_C,Cj(F,-V2)2
uu

+
C2=200pF C3 = 200pF 300V
2(C,+C2)
ad
doo

Units used. Charge Q in coulomb, potential V in


nY

C4=100 pF volt, capacity C in farad and energy U in joule.


nid
Re

Solution. The equivalent circuits of network in Example Find the ratio of potential
Fig. 2.71 are shown in Fig. 2.62.
FFi

difl’erences that must be applied across the parallel


In Fig. 2.62 (c), as 100 pF and 200 pF capacitors and series combination of two capacitors C| and
are in series, pot. diff. across C4 is in the ratio 2:1. C2 with their capacitances in the ratio 1:2 so that
300x2 energy stored in the two cases becomes the same.
V = 200V
4 ~
2+ 1 (CBSE 2016)

FIGURE 2.62

Ci=100 pF Ci
C4 C4 C4
C2 C3 100 pF _
100 pF 100 pF 100 pF 200 pF
200 pF 200 pF 100 pF
k- 300V 300V 300V

e o o
2/48 4- Fundamental Physics (XII) LV»iWI

1 ^ energy stored in the two cases becomes the same.


Solution. Here - = — or = 2 C,
2 ^ (CBSE 2010)
^2
Cp — C| + C2 — Cj + 2 Cj — 3 Cj Solution. Here

C
C,
1 Z
C,x2C, 2
^
C2 3
^ ” Cj +C2 ~ Cj +2C, ~ 3 ' If Cj = C, C2 = 3 C
If Vj is p.d applied across parallel combination Cp = C, +C2 = C+ 3C = 4C
and V2 is p.d applied across series combination, then
as
£,=£2
^ C,C2 C(3C) _3C
^ Cj+C2 C + 3C 4
1
- C = -c If Vj and V2 potential differences applied
2 P * 2^2 across the parallel and series combination, then as

ww
y, 2 1 1
I _
-X -c V.2 =-c
3 3C I 9 3 2 p I 2^2
^2

Flo
Example m A capacitor is charged to V,
1 _ Sl- |3C/4 _ S
potential Vj. The power supply is disconnected and y 4C 4

e
^^2

reree
capacitor is connected in parallel to another
uncharged capacitor. Calculate common potential

r FF
Example E0 A parallel plate capacitor is
of the combination of capacitors. Show that total
energy of the combination is less than sum of
charged by a battery to a potential difference V.
uurr
energies stored in them before they are connected.
(CBSE 2015)
foor It is disconnected from battery and then connected
to another uncharged capacitor of the same
capacitance. Calculate the ratio of the energy
kss
Solution. Let C| and C2 be the capacitances of stored in the combination to the initial energy on
Yoo
ooook

the two capacitors and V be their common potential. the single capacitor. (CBSE 2019)
total charge _ ^1+^2 Solution. Let C be the capacity of parallel plate
eBB

As V =
total capacity Cj + C2 capacitor. When it is charged to a potential difference
V, initial energy of the capacitor.
c,y,
rr

(v V2 = 0)
ouu

V = 1
ad

C1+C2 C1+C2 £, =-CV^


’ 2
Y

1 2
dY

When it is connected to another uncharged


Initial energy stored, t/, = — Cj Vj
capacitor of the same capacitance, the charge is shared
Re
innd

Final energy after connection, equally between the two. Potential of each capacitor
becomes V72.

V2^\<-Cl+C2)V^
Fi

.●. Energy stored in the combination,

1 (C,V.)2 1
u.=-{a+c^) ‘ ^ -
£2=-(C + C)^-
= 3.cy2
2 2 ' 2 (q+c2)2 4

f/,C, ^2 1/4CV^ 1
2 c, + C2 Cj +C2 £
1
l/2cy2 ” 2
Clearly, U2<Uy
which was to be proved. Example 03 A 4 ft F capacitor is charged
by a 200 V supply. It is then disconnected from
Example]^ Find the ratio of potential
the supply and is connected to another uncharged
difference that must be applied across the parallel
2 p F capacitor. How much electrostatic energy of
and series combination of two capacitors C] and the first capacitor is dissipated in the form of heat
C2 with their capacitance in the ratio 1 : 3 so that and electromagnetic radiation ?
ELECTROSTATIC POTENTIAL AND CAPACITANCE 2/49

Solution. Here, Ci=4[iF = 4x F, When key K is brought in contact with terminal


Vj = 200 volt B. The entire energy stored is dissipated as heat
energy.
Initial electrostatic energy stored in Cj is
As 500 .Q and 330 Q resistances are in series,
t/, =lc, V,- =-x4x 10-^x200x200 heat produced is in the ratio 500 : 330.
Heat produced in 500 ohm resistance
= 8 X 10“^ joule 500 500x10
= 6-024 J
When 4 p F capacitor is connected to uncharged 500+330
xU =
830
capacitor of 2 p F, charge flows and both acquire a
common potential
Example^ A900pFcapacitorischarged
total charge
V = by 100 V battery.
total capacity How much electrostatic energy is stored by

w
4x10-^x200 800 the capacitor ? The capacitor is disconnected from
V =
-6
volt the battery and connected to another 900 pF
6
C,+C2 (4 + 2)10 capacitor. How much is the electrostatic energy

Flo
Final electrostatic energy of both capacitors stored in the system ? (NCERT Solved Example)

e
Solution. Here, C = 900 pF = 900 x 10"'^ F

rree
1 ■) 1 . 800 800
U^=-(q +C2)V- =-x.6xl0-^x — X
6
= 9x10-’OF.V = 100V

r FF
= 5.33 X 10"- joule
Energy stored,

U,’ =-C\~ =ix9xlO-io (100)2


uurr
Energy dissipated in the form of heat and
electromagnetic radiation
t/, - t/2 = 8 X 10-“ - 5.33 X 10"2
2
for
2
= 4-5 X 10-^ J
kss
= 2.67 X 10"^ joule On connecting to another 900 pF capacitor,
ooook
Yo

Example [H In the circuit shown in Fig. charge is shared equally.


1 1
2.63, the key K was initially in contact with the -O and V' = -V
eB

terminal A. What amount of heat will be generated 2 2


in 500resistance, when the key K is brought in Total energy of the system
rr

V^ = 2x^Q'r = ^Qx^V
contact with terminal B ?
ou
ad

FIGURE 2.63
Y
dY

500nF
= ifiovl = -x4-5xl0-^J
2l2^ J 2
Re
innd

= 2-25 X 10-^ J
I
Fi

3300 Thus half the energy is lost in the form of heat


and electromagnetic radiation.
B
- 5000
Example]^ Keeping the voltage of the
A V
\
K
charging source constant, what would be the
percentage change in the energy stored in a
parallel plate capacitor, if the separation between
1
200V its plates were to be decreased by 10% ?
Solution. Energy stored in a parallel plate
Solution. When key K is in contact with terminal
capacitor is
A, the condenser gets charged. Energy stored in
condenser y2
u = -cv^ = ...(0
2 2 d
f/ = - cy2 = i (500 X10"^ )x 200x200
1

2 2 When separation between the plates is decreased


= lOjoule. by 10%,
2/50
Fundamental Physics (XII) VQL.I

10
d' = d-
100
d = Q-9d C2=C; = lx3^iF-3^iF’
Change in energy. p.d across Cj' = 6 V,
1
(C/'-C/) = _e AV^ J__l charge on C,'= 3)o.Fx6V = 18 |xC
2 ° 0-9 1 0-9 9
% age change in energy When switch 5 is left open, charge ^2 on C2 is
still 6 p,C, but its potential changes to
100
xIOO = = 1M%
U 9 ^2 = 2V
c'
^2 Syl'
TYPE X. EFFECT OF INTRODUCING A
CONDUCTOR/DIELECTRIC SLAB Example m A dielectric slab of thickness
BETWEEN THE PLATES OF A
1.0 cm and dielectric constant 5 is placed between

w
PARALLEL PLATE CAPACITOR
the plates of a parallel plate capacitor of plate area
0*01 and separation 2*0 cm. Calculate the
Formulae used. 1. Capacity of a parallel plate

Flo
capacitor with a conducting slab of thickness t change in capacity on introduction of dielectric.
inbetween the plates, What would be the change, if the dielectric slab

reeee
6n0 A were conducting ?
C =
Solution. Here, r= 1.0 cm = 10"^ m,

FFr
d~t
= A = 0.01 = 10“^
2. Capacity of a parallel plate capacitor with
dielectric slab of thickness t inbetween the plates, d = 2 cm = 2x10"^ m
for
ur
0
A Capacity with air inbetween the plates
c =
1
kss
d-t 1- —
C _€q a 8-85x10-'2x10-2
K
0" d
Yo
2x10"2
oo

In both cases, t < d


12
Units used. = 4.425 X 10- farad
C is in farad; A in ; t/, r in metre.
eB

Standard Value. eQ=8-85x 10“*^ C^ N“'m“^ Capacity with dielectric slab inbetween the
plates
Example ^ Fig. 2.64, shows two identical
r

A 8-85x10-'2x10-2
ou
ad

0
C =
capacitors Cj and Cj, each of 1 pF capacitance 1
\ “
YY

connected to a battery of 6 V. Initially, switch S is d-t 1-- 2x10-2-10-2 I--


closed. After some time, the switch S is left open
K
l 5
= 7.375 X 10-12 farad
ndd

and dielectric slabs of A = 3 are inserted to fill


Re

completely the space between the plates of two Capacity with conducting slab inbetween the
Fi

capacitors. How will (/) the charge and («) potential plates
difference between the plates of the capacitors be €« A
0 8-85x10-12x10-2
affected after the slabs are inserted ? (CBSE 2011) C' =
d-t 2x10-2-1x10-2
FIGURE 2.64
8-85xl0-i'i
C' = = 8.85x 10-12 farad.
3", 10-2
Increase in capacity on introduction of
6V-i- IpFiCi 1|iF=tC2 dielectric

= C-Co = 7.375 x 10-12-4.425 X lQ-12


= 2.95 X 10-12 farad
Solution. When switch S is closed,
Increase in capacity on introduction of
Vj = ^2 = 6V conducting slab
= ^2 = CV = 1 pF X 6V = 6 pC = C - Cq= 8.85 X 10-12-4.425 X 10-12
When dielectric slab of A" = 3 is inserted, = 4.425 X 10-12 faraj
ELECTROSTATIC POTENTIAL AND CAPACITANCE 2/51

I I
Example A slab of material of dielectric TYPICAL EXAMPLES
TYPE XI,
constant K has the same area as the plates of a
/

Example 0] An infinitely long positively


paraUel plate capacitor, but has a thickness I - A »

charged wire has a linear charge density X Cm"'.


where d is the separation of the plates. How is the An electron is revolving around the wire as its
capacitance changed when the slab is inserted

w
centre with a constant velocity in a circular plane
between the plates ? (NCERTSolved Example) perpendicular to the wire. Deduce the expression
Solution. Here, t - - 5d for KE of electron. Plot a graph of K.E. as a
4 function of charge density X. (CBSE 2013)

e
0
A 0
A Solution. The electostatic force exerted by the
From (7 =
infinitely long line charge provides the necessary

e
I 3 f 1

o
rw
d-t 1 d--d 1
4 centripetal force to the revolving electron.

r
K
2
€«0 A €« A X

eF
mv
0 eE - , but E=

ullo
27t€^0 r
d ^ 'id
r

FF
4 4/f 4 K
eX mxP- 2 _
eX
or V

srre
( 4K 27ce«0 r r 27te«0 m
= C0

oF
d a: + 3 K + 3

k
1 1 eX eX
KE ~-mv^ =-m
Example A parallel plate capacitor is
fofr
2nEr,m 4tc€ 0
uor
2 2
0
maintained at a certain potential difference.
When a 3 mm thick slab is introduced between
o As KEo^X, therefore, graph of KE as a function
of charge density will be a straight line as shown in
kos
the plates, in order to maintain the same potential
YYo
Fig. 2.65.
rBB

difference, the distance between the plates is


oo
Y

increased by 2*4 mm. Find the dielectric constant


of the slab.
r ee

Solution. Here, t - 3 mm, x - 2-4 mm, K-7


u

_£o
With air as dielectric, ,C
Yd
oou

0“ d
ad

If d] is new distance of separation between the


id n

plates when dielectric is introduced, then


A
0
Re

Co =
in
FF

1
d,-t \ ! Example ^ Two charges 4 pC and - 4|iC
1
K are placed at (- 3,0,0) and (3,0,0) cm respectively
1 9x10®
As C = Cn, therefore, d,-t
1
1
K
=d in an external field given by E = —^— C m"^. r

1
Find the energy of the system in this external field.
or / 1 =d,-d
1
= x Solution. Here, -4[iC-4x 10"® C ;
K
rj=-3cm = -3x 10“^ m

As / = 3 mm and x = d^- d = 2-4 mm ^2 = - 4 pC = - 4 X 10"® C ;


... 3 1
1
=2-4
r2 = 3 cm = 3 X 10"^ m

K ''12 = 3 - (- 3) cm = 6 cm = 6 x 10“^ m
dV
1- = 0-8
From E = dV=E.{dr)oxV=E{r).
dr ’
K 3
^1^2
1
— = 1-0-8 = 0-2, K = 5 Energy = q^V^-¥ ^2 ^2 +
K 471 '-12
2/52
'a Fundamental Physics (XII) VOL.I

by a factor /i, if it is enclosed within an earthed


= ^j X £ X rj + ^2 ^ ^2 4 TIG concentric sphere, the ratio of radii of the spheres
0 'l2 being n!{n - 1).
4xl0“^x9xl0^ Xr,
Energy = I Solution. The capacitance of an isolated
r.2 conducting sphere of radius a is C = 4 n Ena
0
I
When surrounded by an earthed sphere of radius
^(-4xi0“^)x9xl0^xr2 b, its capacitance becomes
ab
C' = 47tE 0
a-b
^ 9x10^ (4xlQ-^)(-4xlO-^) C ab b I
6x10-2 C
= n
a(b~a) b-a 1—
a
1-
«-l
36 36 16x9

ww
Energy — b n
= -2'4J
3x10-2 3x10-2 60
Example Fig. 2.67 shows two identical
Example ^ An electron is fired directly parallel plate capacitors connected to a battery
with a switch 5 closed. The switch is now opened

Flo
towards the centre of a large metal plate that has and the free space between the plates of the

e
excess negative charge with surface charge density capacitors is now filled with a dielectric of

ree
= 2*0 X 1Q-® C/m2. If the initial kinetic energy of dielectric constant 3. Find the ratio of the total
electron is 100 eV, and if it is to stop due to

Fr
electrostatic energy stored in both capacitors

rF
repulsion just as it reaches the plate, how far from before and after the introduction of dielectric.
uurr
the plate must it be fired ?
Solution. Here, o = - 2-0 x 10*^ C/m2
Initial K.E. = /:«=
0 100 eV
s for FIGURE 2.67

s
kks
= lOOx 1-6 X
Yo
= 1-6 X IO-'2j
oooo

6
Let the electron be fired at a distance s from the
eB

plate. Fig. 2.66. From work energy theorem, work


done = F X .y = loss in K.E.
(- eE) s = Fn0
Solution. Initially, when the switch S is closed,
r

both the capacitors have same potential difference


ou
ad

FIGURE 2.66 (V) across them.


YY

Metal Plate
■&K-
Initial energy stored in both the capacitors is

£/, =t/^+£/g=lcv2+ici/2=CV2
nndd
Re

When the dielectric {K = 3) is introduced, the


Fi

Electron
Ko capacitance of each capacitor becomes 3 C. The p.d.
across A is still V' as it is .still connected acro.ss the
Now, electric field due to the plate (having some
thickness), E ~ a/e 0 battery, with switch S open, P.d across B attains new

a
value but charge remains constant
—e s = K
0 q=CV='iCx V' or V'= W3.
^0
So final energy ^ /
S' =
^0 ^0 _ 8-85x10-‘2x1-6x1Q-‘2
\2 ●
-1-6x10-'^ (-2x10“^)
^(30v2+i(3C)|^^
-e<j

= 4-43 X 10--*
= ^V2
m 3

CV2
Example m Assuming an expression for u.
I
= 3:5
the potential of an Isolated conductor, show that V
f -cv^
the capacitance of such a sphere will be increased 3
ELECTROSTATIC POTENTIAL ANQ CAPACITANCE 2/53

A parallel plhte capacitor charge on each condenser, q = CV =3x4


contains a mica sheet! of thickness = 10^ m = 12|lC
and one fibre sheet of thickness - O'SxlO"^. jExampielEfI The area of each plate of
Values of K for mica and fibre are 8 and 2*5 parallel plate air capacitor is 150 cm^. The distance
respective^. Fibre breaks down in electric field between its plates is 0*8 mm. It is chained ot a
of 6*4 X10” Ym~^. What maximum voltage can be pot difff. of 1200 volt What will be its energy ?
applied to the capacitor ? What will be the energy when it is filled with a
Solution. Let a be the surface charge density medium ofK=3 and then charged, ff it is charged
of capacitor plates. first as an air capacitor and then filled with this
c dielectric, what will happen ot energy ?
formica E,1 = Solution. Here, A = 150 cm^ = 150 x 10~* m^,

ooww
d = 0-8mm = 8xlO-^m,Ko=1200V,q)= 1 Eq= ?
CT
and for fibre = —— 885x10-^^x150x10^
8x10-^
= 1-66 X 10-*® F

ee
or
K
1
//„\j = -2 C, i2 X1-66 X10-*® X (1200)2

r
Asi?2 = 6-4x lO^V/m ”

rFFl
ree
F
= 1*2 X 10^ J
E, =
^2 2-5
X64x10^ = 2x lO^V/m
XE2 = With dielectric medium, capacity becomes

rF
1
K 8
1V| o C = ^Cq = 3 X 1-66 X KT'O farad
Maximum voltage for capacitor
fsfoor
When charged to same potential, V = 1200 V,
ouur
V = E,dj + E2^ = 2 X 10^ + 3-2 x lO^
Energy, U = icv^ =|(KC„)V2
kosk
= 5200 V

fExampie nfi] In the drcuit shown in Fig. = 3xl-2xl(r‘ = 3H>xlO-‘j


YYo
oo

2.68, each capacitor has a capacity of 3 pF. When capacitor is charged first as air capacitor,
BB

Clalculate the quantity of charge on each capacitor. then on filling with the dielectric, its .potential
V 1200
rre

FIGURE 2.68 becomes v = — = = 400 volt, because capa-


K 3
oYuu

h^h city becomes 3 times whereas charge remains the same.


ad

3gF4Q 3gF New energy of capacitor


dY

f\ A A A A A B
I 1 fv ^
=-cv3=-(*:c„)(^]
innd

4a
Re
FFi

3pF 3pF
1 CqV^ _b2xlQ-^ = 4 X 10“® joule
2 K 3
HWV c
10V lExSIffigi^ Two identical metal plates are
Solution. Total resistance in the circuit ABCD,
gjven charges qi and ^2 (< 9i) erspectively, ff they
are now brought dose otgether ot form a {Murallel
i? = 4+ 1 = 5Q. plate capadtor with capadtance C, what will be the
^ 10 ^ * potential difference between the plates ?
.*. Current / = — = — = 2A
R 5 Solution. Let A be the area of each metal plate.
Potential diff. across AandE=/x4=2x4 = 8V If they are distance d apart, then the capacitance of
the parallel plate capacitor so formed will be
As two capacitors of 3 p F each' are in series,
A
8 0
C =
pot diff. across each condenser = — = 4 V d
2
2/54 ^ Fundamental Physics (XII)
If £j, £2 electric field intensities due to the As one mole of substance contains 6 x 10^^
two plates (in the space between the plates), then molecules, therefore, total dipole moment of all the
E _ ^2^^ molecules, p = 10"^^ x 6 x 10^^ = 6 x 10^ C m. As
/s, - 2e and £2 = 2e polarisation is 100%, therefore, initial potential
0 0
energy
Net electric field between the two plates
U^ = -p E cos (T = ~6x 10"^ X 10^ = - 6 J
1
£ = £i - £2 = When 6 = 60°, final potential energy
2s 2s 2s. A
0 0 0 £2 = - £ E cos 60°
The potential difference between the plates of
the capacitor. = -6xl0-^xl0^x- = -3J
2

oww
V^Exd =
Change in .PE. = - 3 - (- 6) = 3 J
0 This loss in .PE. is the energy released in the
form of heat in aligning the dipoles.
V = ^

ee
2{e^A!d) 2C Example^ A Student requires a

FFrlo
capacitor of 3 p.F in a circuit across a potential of

r
Example H A capacitor of capacitance 1000 V. A large number of 2 pF capacitors are

rF
ee
Cj = 1‘0 microfarad withstands the maximum available to him, each of which can withstand a
voltage Vj s= 6*0 kilo volt While another capacitor potential difference of not more than 300 V. How

rF
of capacitance C2 = 2*0 microfarad withstands the
ouru should the student arrange these capacitors so that
maximum voltage V2 = *^'0 kilovolt What maxi he may use minimum number of condensers ?
mum voltage will the system of these two capa
citors withstand if they are connected in series ? ffosor Solution. Let the student use minimum number
os k
Solution. Here
of N capacitors in m rows in parallel and n condensers
in series in each row.
C, = 1-0 pF, = 6-0 kV = 6 X 10^ V
ook
Yo
N - m X n ...(f)
Y

Charge on first capacitor


Bo

=CiKi= l-0x6x I0VC = 6000pC As each capacitor in a row can withstand 300 V,
reeB

Similarly, charge on 2nd capacitor therefore, voltage across each row


= 300/1= 1000 V
<?2 = C2 V2 = 2-0 X 4 X 10^ p C = 8000 p C
oouY
ur

In series combination, charge on each capacitor 1000


ad

n = = 3-3
must be the same. As max. charge on Cj is 6000 p C ; 300
dY

therefore, max. charge on C2 must also 6000 p C.


There should be four capacitors in each row.
Hence maximum voltage for the combination is
nidn

Capacitance of each row


Re

6000 6000
V' = Vj' + V2' = = 9000 V = 9 kV C
FFi

10 2-0 therefore,
■y
=—
4 j4 = 0-5nF
Example!^ Amoleculeofasubstancehas
As m such rows are in parallel,
a permanent electric dipole moment of magnitude
10~^^ C m. A mole of this substance is polarized at m = ^ =^=6
low temperature by applying a strong electrostatic Cs 0-5 p£
field of magnitude 10^ V m“^. The direction of the
From (0, N = 6 X 4 = 24
field is suddenly changed by an angle of 60°.
Estimate the heat released by the substance in This is the minimum number of capacitors
aligning its dipoles along the new direction of the required by the student.
field. For simplicity, assume 100% polarisation of
sample. (NCERT Solved Example) Example ^ A capacltor is filled with two
dielectrics of same dimensions, but of dielectric
Solution. Here, dipole moment of each
molecule = 10“^^ C m constants 2 and 3 respectively. Find the ratio of
ELECTROSTATIC POTENTIAL AND CAPACITANCE 2/55

capacitances in the two arrangements shown in These capacitors are joined in parallel, therefore
Fig. 2.69(a) and (b). equivalent capacity
A
c' = C3 + C4 = ...(«)
2d

C AK^ X
2d
Now,
d (/fj + K^) Gg A(/Cj + ^^2)
4 ^1^2 4x2x3 _ 24
{K^+K^9 (2 + 3)2 “25
Solution. As is clear from Fig. 2.69(a) Example ^ A parallel plate capacitor of
plate area (1 x 2) and plate separation 5*0 mm
AK
^0 ^^2

w
0 1 . is charged to 10 kV in air. Find charge density
c.I = ; C2 = d/2
d/2 and field displacement.
Solution. Here, A = 1 x 2 = 2
These capacitors are joined in series, therefore,

Flo
equivalent capacity d=5 mm = 5 x 10“^ m

y= 10kV= 10“*^, p = ?

ee
'2Sq A:,Ay2€Q K^A field displacement = ?

Fr
C = qq d_ / \
d
A 8-85x10
-12
x2
2€o a q = CV' = -L y = xlO"*
c, +C2
j— (.K^+K^) d 5x10"5

or
ur
Q = 3-54 X 10-5 Q
sf
c =
2gq a a:, K2 ...(0 Q 3-54x10-5
d(K^+K^)
ok
Charge density, 9 = ~
Yo
A 2
In Fig. 2.69(b), = 1-77 X 10-5 Qj^-2
Bo

€^K,(A/2) ^ e^K^{A/2) Field displacement = charge density


^3 ■—5
=
re

d = 1-77 X 10-5
ou
ad
Y

1. Electrostatic potential difference between two points B and A in an electrostatic field is the amount of
nd

work done in carrying unit positive test charge from A to B (against the electrostatic force of the field)
Re

B
Fi

-» —♦
AB _
along any path between the two points, i.e., VdH - V.A = E.dl
% A

B
B
If A is at 00, - 0, we may write, = E.dl
%

This means electrostatic potential at a point in the electrostatic field is the amount of work done in moving
a unit positive test charge from infinity to that point, against the electrostatic forces, along any path.
Electrostatic potential is a scalar and it is measured in volt.
Electric potential at a point distant r from a point charge q is.

47CEor
2/56
TDfutdee^ 'a, Fundamental Physics (XII) P7CT1
Electric potential at a point due to a group of charges and due to continuous charge distribution can be
calculated similarly using the superposition principle.
Further, electric field intensity at any point is negative rate of change of potential with distance at the point.
dV
i.e.. l£l=-
dr

2. An equipotential surface is that at every point of which electric potential is the same. Equipotential surfaces
are always perpendicular to the field lines.
3. Electrostatic potential energy of a system of point charges is the total amount of work done in bringing the
various charges to their respective positions from infinitely large mutual separations.
Electric potential energy of a system of two point charges is given by :

U =
4;teo/-,2

w
SI unit of t/is joule. Another convenient unit of energy is electron volt{eW), where 1 eV = 1.6 x 10"’^Joule
4, Conductors are the materials which possess a large number of free charge carriers whereas Insulators or

Flo
dielectrics possess a negligibly small number of free charge carriers.

reeee
When a conductor is placed in an external electric field, net electric field in the interior of a conductor is
zero, there is no charge in the interior of the conductor, i.e., charge always resides on the outer surface of

FFr
the conductor. The surface of the conductor is an equipotential surface and electric field over a charged
conductoris perpendicularto the surface of the conductorat every point.

for
5. Capacitance (C) of a conductor is the ratio of charge (Q) given to conductor and the potential (V) to which
ur
it is raised, i.e., C = Q/V. The value of C depends on size, shape and relative position of two coatings of the
capacitor. It also depends on nature of medium separating the two coatings.
kkss
Capacity of an isolated spherical conductor of radius r in vacuum is,
Yo
oo

C = 4 n Eq r
eB

Capacity of a parallel plate capacitor with air as dielectric is


a
0
r

C =
ou
ad

d
YY

where A is area of each plate and d is distance between them.


When the insulating medium separating the two conductors in a capacitor is other than air, its capacity
ndd
Re

becomes K times the capacity in air. Here, K is dielectric constant or relative electrical permittivity of the
medium. For mica, K = 3 to 6 ; for paper K = 3.5, for water, AT = 81, for ebonite, K = 2.7 and for glass, K
Fi

varies from 3 to 4.

6. When cond'^nsers are grouped in series, total capacity is given by

J___L J_ 1
c, “c/c, C3
+ ....

In parallel grouping, total capacity is sum of the individual capacities, i.e., Cj, = Cj + C2 + C3....
7. Work done in charging a capacitor is stored in it in the form of electrostatic energy, given by

I I
u = -CV^ =
2 2 C

when Q is in coulomb, V in volt and C in farad, energy E is in joule.


ELECTROSTATIC POTENTIAL AND CAPACITANCE 2/57

8. When two capacitors charged to different potentials are connected by a conducting wire, charge flows from
the one at higher potential to the other at lower potential till their potentials become equal. The equal
potential is called common potential (V), where

^ total charge
totalcapacity Cj + C2 C1+C2
It should be clearly understood that in sharing charges, there is absolutely no loss of charge. Some energy
is, however, lost in the process which is given by ;

t/ _{/ =_L_2U
C,C-(K-K,)2
IL.
‘ 2 2(C,+C2>

ww
9. Dielectrics are of two types : Non polar dielectrics and polar dielectrics. The non polar dielectrics (like
N2, O2, benzene, methane) etc. are made up of non polar atoms/molecules, in which the centre of positive
charge coincides with the centre of negative charge of the atom/molecule.

Floo
Hlae polar dielectrics (like H2O, CO2-, NH^, HCl etc.) are made up of polar atoms/molecules, in which the
centre of positive charge does not coincide with the centre of negative charge of the atom/molecule.

ee
A non polar dielectric can be polarised by applying an external electric field on the dielectric.

reer
rFF
The effective electric field (E) in a polarised dielectric is given by £ = Eq-
uur r
ffoor
where Eq is strength of external field applied and E^ is intensity of induced electric field set up due to
sks
polarisation.
YYoo
10. Capacitance of a parallel plate capacitor with a conducting slab of thickness t is given by :
ooko

^0 ^
eBB

C' =
d-t

Capacitance of a parallel plate capacitor with a dielectric slab of thickness t is given by :


r
ouur
ad

A
0
Y

( 1 ^
d-t 1--
l K)
nddY
Re

Clearly, capacity increases on introducing a conducting slab as well as dielectric slab between the plates of
FFiin

an air capacitor.
11. A Van de graaffgenerator is a powerful machine used for generating high positive potentials » 10^ volt. It
is based on the action of sharp points and the property that charge always resides on the outer surface of a
hollow conductor. Such high positive potentials are needed for accelerating positive ions.
2/58 ^●uuUe^'4- Fundamental Physics (XlllESsIS

QUESTIONS

Based on NCERT Book

w
I. Multiple Choice Questions 8. N/C is the SI unit of

(a) electric charge (b) electric potential


1. The electric potential on the axis of an electric
dipole at a distance r from its centre is V. Then the (c) electric intensity (d) none of these
potential at a point at the same distance on its 9. Due to a single charge, electric field is

wr
e
equatorial line will be (a) spherically symmetric
{b)-V (fc) cylindrically symmetric

r
(a)2V

oo
(c) V/2 (d) Zero (c) asymmetric (d) none of these

F
llu
(CBSE Sample Paper 2022-23) 10. Three capacitors 2 pF, 3 pF and 6 pF are joined in
series with eachother. The equivalent capacitance
2. A car battery is charged by a 12 V supply, and

roFF
IS
energy stored in it is 7-20 x 10^ J. The charge passed
(a)5pF (b) 1 pF

ees
through the battery is
(b) 5-8 X lO^J (c)2pF (d) 1/2 pF

rr
(a) 6-0 X lO'^ C

Fk
(c) 8-64 X J (d) l-6x lO^C 11. The electric potential at a point on the axis of a

oo
short electric dipole at a distance x from the mid
(CBSE 2022)
point of dipole is proportional to
3. Equipotentials at a large distance from a collection
of charges, whose total sum is not zero are
fofr
so I 1
YY
(.b)
(a) ^4
ou
oY
(a) spheres (b) planes
BrB

1 1
(c) ellipsoids id) parabosides
ok
(CBSE 2022)
(CBSE 2022)
4. If net electric flux through a closed surface is zero, 12. A variable capacitor is connected to a 200 V battery.
ruee

we can infer If its capacitance is changed from 2 pF to X pF,


ooud

the decrease in energy of the capacitor is


(a) no net charge is enclosed by the surface 2 X 10"^ J. The value of X is
(b) uniform electric field exists within the surface (fc)2pF
ad

(fl) 1 pF
Yn

(c) charge is present inside the surface (c)3pF (^4pF (CBSE 2022)
(d) electric potential varies from point to point 13. Let be the magnitude of force between two small
di

inside the surface. (CBSE 2022) spheres charged to a constant potential in free space
Re
FF

5. The basic cause of charging is and F2 be the magnitude of force between them in
in

(a) actual transfer of protons a medium of dielectric constant K. Then F1/F2 is


(b) actual transfer of neutrons 1
(a) - ib)K
(c) actual transfer of electrons K

id) none of the above 1

6. A body carries 500 excess electrons. Charge on the


(c)K^ (^0
K-
body is 14. The capacity of a condenser increases from 6 pF
(a)-8x 10-’’^ C ib)%x lO'^C to 30 pF when a medium of dielectric constant K
ic) 1-6 X 10-1^ C id) 1-6 X lO'^C is introduced between the plates. The permittivity
of the medium is
7. The SI unit of electric flux is
ia) 0-44 X lO-'f^C^N-* m-^
ia) NC"' ib) NC“2 (b) 0-44 X 10-^ C2 N'* m"2
N -1 N 2 ic) 0-44 X lO'^C^N"' m -2
(C)
m
rc (^ C*" id) None of these
ELECTROSTATIC POTENTIAL AND CAPACITANCE 2/59

15. The speed acquired by a free electron when (a) remains unaffected (b) reduced to l/6th
accelerated from rest through a potential diff. of (c) becomes 6 times (d) none of the above
100 Vis
25. When a number of capacitors are connected in
(a) 6 X 10^ m/s (b) 3 X 10^ m/s series between two points, all the capacitors
(c) 4 X 10^ m/s (d)2x 10^ m/s possess same
(CBSE 2022) (a) capacity {b) potential
16. Electrostatic potential V at a point, distant r from a (c) charge {d) none of the above
charge q in air is 26. When a number of capacitors are connected in
(a) q/i^ ib) q^lr parallel between two points, the equivalent
(c) qlr id) q^lP- capacitance
17. Work done in carrying an electron from AxoB lying {a) increases ib) decreases
on an equipotential surface of one volt potential is (c) remains the same {d) none of the above

w
(a) 1 eV ib) 10 eV 27. A condenser is charged to double its initial
(c) 1 volt id) Zero potential. The energy stored in the condenser
becomes x times, where x =

Flo
18. The correct relation between electric intensity E
and electric potential V is (a) 2 {b)A

e
{d) 1/2

reee
dV dV ic)l
(a) E = - ib) £ = 28. When two capacitors charged to different potentials

FFr
dr dr
are connected by a conducting wire, what is not
dE dE
(c) V=- id)V = — true ?
dr dr

for
ur
(a) Charge lost by one is equal to charge gained by
19. A charge of 10 p. C lies at the centre of a square. the other
Work done in carrying a charge of 2 p C from one (b) Potential lost by one is equal to potential gained
kkss
comer of square to the diagonally opposite comer by the other
Yo
IS
oo

(c) Some energy is lost


ia) 20 J ib)5 3
(d) Both the capacitors acquire a common potential
B

(c)Zero (fO 20 p J 29. The relation between dielectric constant K and


re

20. A uniform electric field of 100 N/C exists in


electric susceptibility x of a material is
vertically upward direction. The decrease in electric
(a)K=x (b)K=l+x
ou

potential as one goes up through a height of 5 cm


ad

(c)x = ^+l (d)K2 = {l+x)(l-X)


YY

IS

ia) 20 V ib) 120 V 30. When charge is supplied to a conductor, its


potential depends upon
(c)5V id) Zero
nd

(a) the amount of charge


Re

21. When a conductor is held in an electric field, the


field inside the conductor is always (b) geometry and size of conductor
Fi

ia) positive ib) negative


(c) both (a) and (b) (d) only on (a)
(c) constant id) zero 31. A parallel plate capacitor is charged by a battery.
Once it is charged, battery is removed. Now a
22. Charge on a capacitor is doubled. Its capacity
dielectric material is inserted between the plates
becomes k times, where
of the capacitor. Which of the following does not
ia)k=2 ib)k=l
change ?
1
(c) * = - id)k = A
(a) Electric field between the plates
(b) Potential difference across the plates
23. Electrical capacity of earth is (c) Charge on the plates
(fl)lF ib)l\iF id) Energy stored in the capacitor.
(c)71l pF (rf)9x lO^pF 32. A dipole is placed parallel to electric field. If W is
24. When air inbetween the plates of a capacitor is the work done in rotating the dipole from 0“ to
replaced by mica of dielectric constant 6, its 60°, then work done in rotating it from 0° to 180°
capacity IS
2/60 ‘P%etdcefi^ d- Fundamental Physics (XII) PTSTin
(a) 2 W (h) 3 V/ kq
(a) zero (h)
(c)4W {d) wn a V2
33. Thevarialionofpoienlial Vwith rand electric field
kq kq
with r for a point charge is correctly shown in the (f) q- (cl)
2a-
graphs.
a~

38. Two conducting spheres A and B of radii a and h


respectively are at the same potential. The ratio of
FIGURE 2(Q).1
surface charge densities of A and B is
{a) UJ
ib) m
o3 {«) -
> a

(c)
o~

h^- (d) ^ a

w
r —> r —►
39. Work done to bring a unit positive charge un
0) lij UJ accelerated from infinity to a point inside electric
e9 00
Held is called :

Flo
> >

(a) Electric field {b) Electric potential

e
(r) Capacitance {d) Electric flux

reee
\

r —► r
40. Electric potential due to a point charge -q at

FFr
distance x from it is given by :
34. A charge Q is supplied to a metallic conductor.
Which is true ?
(a) Kq!x~ 0) Kqlx

for
ur
{a) Electric field inside it is same as on the surface (c) -Kqlxr id) -Kqlx
41. Electric field is always :
(b) Electric potential inside is zero
kss
(c) Electric potential on the surface is zero (fl) Parallel to equipotential surface
Yo
{d) Electric potential inside is constant (/?) Perpendicular to equipotential surface
oo

35. A parallel plate capacitor C has a charge Q. The (c) It can be perpendicular and parallel as well
B

actual charges on the plates are id) It does not depend on distribution of charge
re

0) (2. <2 0) QI2,QI2 42. Electric field and electric potential inside a charged
spherical shell :
ou
ad

ic) Q,-Q («) V=0 0) E=0;V^0


YY

36. (c) £;?i0; V=0 0) E^0;V^0


Three capacitors of capacitances 1 pF, 2pF and 3pF
43. Shape of equipotential surface in uniform electric
are connected In series and a potential difference
nd

field will be :
Re

of 11 V is applied across the combination, then


the potential difference across the plates of I pF (n) Spherical normal to electric field
Fi

capacitor is (h) Random


0) 2 V 0) 4 V (c) Circular normal to electric field
(c) I V 0) 6 V
{d) Equidistant planes normal to electric field
37. The potential at the centre of the square is- 44. On reducing potential across a capacitor, its
capacitance
(a) Decreases
(b) Increases
(c) Remains constant
(flf) First increases then decreases
45. Energy stored in a charged capacitor is given by :
(a) U=CV!2 0) U=CV^/2
0) 2CV^ {d) VC^!2
ELECTROSTATIC POTENTIAL AND CAPACITANCE 2/61

46. If n number of equal capacitors each of capacitance Reason. Potential energy of a system of two
C are connected in series, then equivalent charges and ^2 placed distance rj2 apart is given
capacitance will be given as : by
(a) n X C (b) Chi
U =
(c) n + C id) n^C 4 Tie /I?
0 '12
47. Capacitance of parallel plate capacitor when there
50. Assertion. One electron volt (eV) is the KE gained
is no medium between the plates is Cq. If capacitor
by an electron when accelerated under a potential
is now completely filled with dielectric matter of difference of 1 volt.
constant K, the capacitance would be :
Reason. eV is the SI unit of energy in electrostatics.
(a) {b) KCf^ 51. Assertion. Three charges + q,+ 2 q and - 4 ^ are
(c) K^C0 (d) 2 KCq placed on the three vertices of an equilateral

ww
triangle of each side 0 ! m. If ^ =10“^ C, then PE
II. Assertion-Reason Type Questions of the system of charges is - 9 x 10"^ J.
Reason. PE of the system of charges is.

FF loo
Direction. For question numbers 48 to 55, two
statements are given, one labelled Assertion (A) 1

ee
U =
and the other labelled Reason (R). Select the 4kg U all y

ee r
correct answer to these questions from the codes

rFrF
pairs
(a), (A), (c) and (d) as given below :
52. Assertion. Electrical capacity of a solid spherical
(a) Both, A and R are true, and R is correct
rur
conductor depends upon the radius of conductor
explanation of A.
{b) Both, A and R are true, but R is not the correct
ffoor
and dielectric constant of the material of the
conductor.
ks
explanation of A.
Reason. The dimensions of the capacitance of a
YYoou
(c) A is true, but R is false. conductor is L-^ 7^* A^J.
okos
BBoo

(d) A is false, and R is also false. 53. Assertion. Taking earth to be a sphere of radius
48. Assertion. Electrostatic force constant and 6400 km : the capacity of earth is 711 pF.
r ee

dielectric constant represent the same thing. Reason. Capacity of a sphere of radius /? = 4 tcSq R.
Reason. Electrostatic force constant 54. Assertion. In a series or parallel combination of
oouur
ad

capacitors, charge on each capacitor is same.


Y

k = = 9x lO^Nm^C-^ Reason. The charge is equally shared by capacitors


4kg
0 wheather connected in scries or parallel.
dndY
Re

= K, dielectric constant 55. Assertion. There is no loss of energy in the


FFini

charging process of capacitor.


49. Assertion. Electrostatic potential energy of a
system of two charges is equal to total amount of Reason. Energy spent by the battery in charging a
workdone in bringing the two charges to their capacitor is QV which is the energy stored in the
respective positions from infinite separation. capacitor.

ANSWERS

I. Multiple Choice Questions


l.(d) 2. (a) 3. (a) 4. (a) 5. (c) 6. (a) l.(d) 8. (c) 9, (a) 10. (b)
11. (£0 12. (a) 13. (b) 14. (a) 15. (a) 16. (c) 17. (d) 18. (a) 19. (c) 20. (c)
21. (d) 22. (b) 23. (c) 24. (c) 25. (c) 26. (a) 27. {b) 2S. (b) 29. (b) 30. (c)
31. (c) 32. (c) 33.(1?) 34. id) 35. (c) 36. ((/) 37. (a) 38. (a) 39. (/?) 40. (d)
41. (Z?) 42. (b) 43. id) 44. (c) 45. (/?) 46. (/?) 47. (Z?)
2/62 ^ Fundamental Physics (Xll)

II. Assertion-Reason Type Questions

48. (^0 49. (c) 50. (c) 51. (a) S2.(b) 53. (a) 54, (,d) 55. (d)

HINTS/EXPLANATIONS For Difficult Questions

I. Multiple Choice Questions Decrease in energy of capacitor

1. As is known, electric potential on equatorial line


of dipole is zero.
(2-;c)x2x 10'* = 2 X 10-2

ww
2. Here, V= 12 volt,
energy stored = W = 7*20 x 10^ J 2-;c=10-^=l(pF)
Charge passed, 2 = ? ;c = 2 - 1 = 1 pF
13. If F| = force between two small spheres charged

Flo
ly 7-20 xIQ5
Q =- = 0-6x lO^C to a constant potential in free space

e
V 12

eree
= 6 x lO'* C
and ^2 = force between the same two spheres in a
medium of dielectric constant K, then

FFr
3. From a collection of charges, whose total sum is E
uurr
not zero, equipotentials at a large distance as the -i-=K

orr
total charge will act as a point charge, must be ^2
spheres only.
4. When net electric flux through a closed surface
sfo
14. Here, Q = 6 |XF and C„ - 30 pF
Dielectric constant of the medium
kks
Yoo
is zero, i.e., ~ = 0, so Q = 0. Hence no
oooo

K = C„ ^ 30 pF ^ ^
m

C0 6mF
eBB

net charge is enclosed by the surface.


5. The basic cause of charging is actual transfer of The permittivity of the medium is
electrons. e=K€Q = 5x 8-85 x 10"‘2 N"^ m"2
uurr

6. 9 = nc = 500 (- 1-6 X lO"*^) C = - 8 x 10”^’ C € = 0-44 X ir^® c2 N-1 m-2


ad

7. = E.ds = (N/C) m2
Yo

1
15. From, —mv^ = eW
dY

8. N/C is SI unit of electric intensity. 2


9. Due to a single charge electric field is spherically
innd

2 X1-6 X10-’’X100
Re

2eV
symmetric. V =

10. Here, C, = 2 ^iF, C2 = 3 |iF, C3 = 6 p.F, =? y m 9x10-2’


Fi

J___L 1 1 I3‘2
+ — xlO’'’ = 6x lO^’m/s
C C1 ^2 C3 9

17. Work done = charge x potential difference


1 I 1 3+2-t-l 6
“ + “■ + — —
6
= (- e) X 0 = zero.
2 3 6 6
19. Diagonally opposite comers of the square are at
C, = lpF the same potential. Therefore, potential difference
11. On the axis of a short electric dipole. = 0. Hence work done = Zero.

P 1
-dV
Electric potential, V = ± V OC

20. From E =
4tI€q At2 x2 dr

12. Here, V= 200 volt, C, = 2 pF = 2 x KT^ F = -5V


dV =-E(dr) = -\00x —
C2 = X pF = X X 10-^ F, At/ = 2 X 10-2 J 100
ELECTROSTATIC POTENTIAL AND CAPACITANCE 2/63

22. Capacity is not affected by the amount of charge. 1 11 n


^ C
46. — = —+— n terms = ●— or C
23. Electrical capacity of earth. eq
C., c c c
ft

1 47. Capacity of parallel plate capacitor with no


C=s4JI6q/? = x(6-4xlO^)F
9x10’
medium,
Co = d ’
= 711xlO-^F = 7UjlF
A
24. Capacity of capacitor = /fC = 6 C. with medium cm
25. In series combination, charge on each capacitor is
the same. n. Assertion-Reason lype Questions
27. When V becomes twice, eneigy stored = ~ CV^, 48. and /fT do not represent the same thing. Thus both
becomes 4 times. ^ Assertion and Reason are false.
49. Here Assertion is true but Reason is false as the
31. As battery is removed, charge on the capacitor

ww
does not change. P.E. of the system of two charges is given by,
1
32. W=-p£(cos6(P-cos(P)=-pEfi-ll = -2£2 C/ =
4tC€o

Flo
\2 J
W' = - (cos 180“ - cos 0“) = 2 =4W 50. Here, Assertion is true but Reason is false as SI

e
unit of energy is joule, where 1 eV = 1*6 x 10"*® J.
33. For a point charge, electric potential, V « l/r and

rree
51. Here Reason is true.
Electric intensity, E oc 1/A Thus option (b) is true.

r FF
Now, PE of the system of three charges shown in
1 1 1 1 11 1 Fig. 2(Q).3 is.
uurr
=T+-+-
^1
6 + 3+ 2
^2 ^3
11
1 2 3
for
^ .-. C
kss
6 6 " 11
ooook
Yo

xll = 6 coulomb
eBB

v.=£ = f=«v
rr
ou
ad

1
f/ =
38. ^ = i 4lt€o |_ Tj2 ns 'b .
Y
dY

C2 b
IQ-7 X (2xIQ-^) ^ 10"^ X(-4x 10"^)
Re
innd

c 01 0-1
As their potential is same, ^ = -1—- - — = 9x10®
<l2 b ^ (2xl0~^)x(-4xl0~^)
Fi

0-1 J
Ajtb^ b^ (a^ b = 9 X 10® [2 X 10-*3 - 4 X 10"*3 - 8 X 10"*3]
X- = ^ \bj
- =-
O2 4n c-? a = 9 X 10® [- 10 X l(r*3] =-9 X 10^ J
39. Knowledge based question. Thus Assertion is also true and Reason is the

41. Electric field lines are always perpendicular to true explanation of Assertion.
equipotential surface. 52. Here, both Assertion and Reason are true but
42. Inside a charged spherical shell, the electric field Reason is not the correct explanation of Assertion.
is zero but electric potential is same as on the 53. Here Reson is true. The capacitance of earth
surface of spherical shell. 1
44. The capacitance of capacitor depends upon the C-AkeqR= X (64 X 10^)
(9x10®)
dimensions of the plates of capacitor and is
independent of change in potential. = 711xl(r6F=:711nF
Thus Assertion is true and Reason is the correct
45. Knowledge based question.
explanation of Assertion.
2/64 Fundamental Physics (Xll)CSm

54. Both Assertion and Reason are false as in series 55. Energy stored in capacitor when charged to
combination of capacitor, the charge on each
capacitor is same and potential across each is
different and in a parallel combination of
potential ^ ^ CV^, as some energy is

capacitors, the potential across each capacitor is lost in the process of charging.
same but charge on each is different. Thus both Assertion and Reason are false.

1. Electrostatic Potential, Potential 4. Two large parallel thin plates having uniform
difference related to Electric field charge densities + o and - a are kept in X-Z
plane at a distance d apart. Sketch an

ww
I. Express the unit of electric potential in terms equipotential surface due to electric field
of the base units of S.I. (ICSE 2002) between the plates. If a particle of mass m and
charge - q remains stationary between the

FF loo
Work done
Sol. As electric potential = plates, what is the magnitude and direction of
Charge the field ? (CBSE 2011)

ee
Sol. In Fig. 2(Q).5, we have shown the electric field
= = kg m^ s-3 A"‘

ee r
lines by solid line curves with arrowheads. The

rFrF
AT
corresponding equipotential .surfaces arc shown
2. Is the electrostatic potential necessarily zero by dotted line curves.
at a point where the electric fleld strength is
rur
zero ? Give an example to illustrate your
answer.
ffoor FIGURE 2(Q).5
ks
Sol. No. As we know that the electric Held is equal to
YYoou
negative of potential gradient :
r\
ookos

dV
BBo

E = -
dr
re

.so, even if electric field at a point is zero, the


potential may have .some non zero constant value
ouur
ad

at that point. If E is magnitude of the field between the plates,


Example. Electric field inside a charged then as particle remains stationary,
Y

conducting sphere is zero but potential at any upward force due to electric field = weight of
point inside the sphere is same as that on the particle
dndY
Re

surface of sphere. -qE = mg


3. A point charge Q is placed at point O as shown
FFini

mg
in Fig. 2(Q).4. Is the potential difference £ = -
(^4 - V'«) positive, negative or zero if Q is
(i) positive (h) negative ? The direction of field is downwards, i.e., along
(CBSE 2011, CBSE (F) 2017) negative K-axis.
FIGURE 2(Q).4
5. A metal sphere A of radius a is charged to
Q potential .V What will be its potential if it is
O A B enclosed by a spherical conducting shell B of
radius b and the two are connected by a wire ?
Sol. From Fig. 2(Q).3. Sol. If q is charge on sphere A, Fig. 2(Q).6, then
potential of A is
OA OB

As OA < OB. 4Its,,


0
a
.●. When Q is positive, (- V[j) is positive and
<?=(4tc€q«) V ...(/)
when Q is negative, (V^ - Vg) is negative.
2/65
ELECTROSTATIC POTENTIAL AND CAPACITANCE

(a) A charge q is placed at the centre of the


shell. What is the surface charge density on
the inner and outer surfaces of the shell ?
(b) Is the electric field intensity inside a cavity
(with no charge) zero, even if the shell is not
spherical, but has any irregular shape ?
Explain. (CBSE20I0)
Sol. (a) The charge + Q resides on the outer surface of
the shell. The charge q placed at the centre of the
shell induces charge -qon the inner surface and
charge + ^ on the outer surface of the shell. Fig.
When A is enclosed by a spherical conducting 2(Q).8. Therefore, total charge on inner surface
shell B, and the two are connected by a wire, the of the shell is - ^ and total charge on the outer
entire charge ^ of A shifts to outer surface of B. surface of the shell is (Q + q).

ww
Potential of B will be

q (4ti£q a)V ^ using (/)

FF loo
4ii £a0 b 4ti6a0 b

ee
V = —V
s b

ee r
As sphere A is inside B, therefore, final potential

rFrF
of A
rur
6. Fig. 2(Q).7 shows the variation of electric
potential V with 1/r, where r is the distance from
ffoor
ks
the two charges Qj and Q2. Determine
Q+q
YYoou
q
(i) signs of two charges Qj and Q2
ookos

O: and a

(ii) which of the two charges has a larger 4jir|- ^ 4tC/2“


BBo

magnitude ? Justify.
{b) Electric field intensity inside a cavity with no
re

FIGURE 2(Q).7 charge is zero, even when the shell has any
irregular shape. If we were to take a closed loop,
oouur
ad

part of which is inside the cavity along a field


line, and the rest outside it, then net work done
Y

by the field in carrying a test charge over the


closed loop will not be zero. This is impossible
dndY
Re

■►1/r
for an electrostatic field. Hence electric held
intensity inside a cavity with no charge is always
FFini

zero.

8. Two charges - q and + q are located at points


(0, 0, - a) and (0, 0, a) respectively,
Sol. Electric potential due to a point charge Q at a point (i) What is the electrostatic potential at the
distant r from the charge is points (0, 0, z) and (x, y, 0) ?
(ii) Obtain the dependence of potential on the
Q
F = distance r of the point from the origin, when
4tcSq r - » 1.
y is + only when Q is +. Therefore, from Fig. a

2(Q).7, 2, is + and Q2 is negative. For a given (iii) How much work is done in moving a small
value of r, we find from Fig. 2(Q).7 that V2> V| test charge from the point (5,0,0) to (- 7,0,0)
Qi>Qx along the x-axis. Does the answer change if
7. A spherical conducting shell of inner radius r, path of test charge between the same points is
and outer radius has a charge Q. not along the x-axis ?
2/66
^ Fundamental Physics (XII) VOL.I

(iv) If the above point charges are now placed As work done = Charge (^2 - V,)
In the same positions in a uniform external W - zero

electric field E, what would be the potential As work done by electrostatic field is independent
energy of the charge system in its orientation of the path connecting the two points, therefore,
of unstable equilibrium ? Justify your answer work done will continue to be zero along every
in each case. [CBSE 2018 (C)] path.
Sol. Here, - ^ is at (0,0. - a) and + ^ is at (0, 0, a) {iv) When the dipole is placed in an external
—>
(0 Potential at (0, 0, z) would be
electric field E, and it is in unstable equilibrium,
1 1 then0= 180°.
9
y =
0 z + aj 471 Eq iz-a) Potential energy of dipole = -/>£■ cos 0
= -{^ X 2 fl) £cos 180°

oww
q{z + a - z + a) qla P.E. = qx2aE
4tc€q(z2 47CEq(z2 9. Fig, 2(Q).9 shows a charge array known as an
‘electric quadrupole’. For a point on the axis
P
of the quadrupole, obtain the dependence of
potential on r for r/a > > 1, and contrast your

e
results with that due to an electric dipole and

ree
Potential at (x, y, 0), i.e., at a point 1 to z-axis

rFl
where charges are located, is zero. an electric monopole {i.e. a single charge).

Fre
(//) We have proved that Sol, As is clear from Fig. 2(Q).9, an electric quadruple

rr F
may be regarded as a system of three charges,
V =
pcos 0 + q,-2q and + <7 at A, B and Crespectively.
ouur
r
47C€Q(r^ -fl^cos^0) sfoo FIGURE 2(Q).9
2a
pcos 0
kks
If ~ » 1 , then a<< r V =
B
Yo
oooo

a
47te«0 A --I
+q -q -q +q p
1
eBB

Voc^ M- ●H
r2
Let AC = 2 <j. We have to calculate electric
i.e. potential is inversely proportional to square
urr

of the distance, potential at any point P where BP = r. Using


You

superposition principle, potential at P is given by


ad

(m) Potential at (5, 0, 0) is


j I g '
1
dY

1 V =
V,1 = 4 TIE 0 r+a r r~a.
4tce
0 V(5-0)2+(-a)2
innd
Re

r(r-fl)-2(r + fl)(r-g) + r(r + fl)


1 471E
Fi

0 L
47re.0
7(5-0)2 +fl2 q[r^ -ra-2r‘^ +2a^ +r^ +ra]
-9 9
= zero
47lEQr(r2-a2)
47CEq V25+^ 47IE
0 ^25 + fl2 q.2a^ q.2a^
Potential at (-7, 0, 0) is 47t£y r(r2 -fl2) 4;tEQ d-a^/r'^)
1 r
^2 = when ~»L,r>>a or a<<r
4 TIE 0
V(-7-0)2+fl2 a

2
a
1 — is negligibly small,
47t€
0
7(-7-0)2+fl2 1
aearly. Voc
= zero
4tce.0 /-3 r3
2/67
ELECTROSTATIC POTENTIAL AND CAPACITANCE

1 II. Electrostatic Potential Energy


In case of an electric dipole, V OC , and in case
12. Suggest a configuration of three point charges
of an electric monopole (i.e. a single charge), separated by finite distances that has zero
1 electric potential energy.
V OC
r Sol, The configuration shown in Fig. 2(Q).10 has zero

10. Describe schematically, the equipotential potential energy for three charges.
surfaces corresponding to
(a) a constant electric field in the Z direction
(b) a field that uniformly increases in magni

oww
tude but remains in a constant, say z direction
(c) a single positive charge at the origin,
(d) a uniform grid consisting of long equally
spaced parallel charged wires in a plane.

ee
Sol. By definition, an equipotential surface is that at
every point of which potential is the same. In the Kq-q , Kq{-q) , K{~q)q

FFrlo
= 0

r
u =
four cases given above : r 2r 2r

rF
(a) Equipotential surfaces are planes parallel to

ee
X -Y plane. These are equidistant.
13. Figs. 2(Q).ll(a) and 2(Q).ll(ft) show the field
lines of a single positive and negative charge
ib) Equipotential surfaces are planes parallel to
ouru
rF
respectively,
X - Y plane. As the field increases uniformly, (a) Give the signs of potential difference
distance between the planes {differing by fixed
potential) decreases,
ffosor
(Vp-yg)and(VB-V^).
os k (b) Give the sign of potential energy difference
(c) Equipotential surfaces are concentric spheres of a small negative charge between the points
with origin at the centre. Q and P; A and B.
ook
Yo
(d) Equipotential surfaces have the shape which
Y

changes periodically. At far off distances from the FIGURE 2(Q).11


Bo

grid, the shape of equipotential surfaces becomes


reeB

parallel to the grid itself.


oouY

11. In a Van de graaf type generator, a spherical


ur

metal shell is to be a 15 x 10® volt electrode.


ad

The dielectric strength of the gas surrounding


the electrode is 5 x 10^ Vm"^. What is the
Y
d

minimum radius of the spherical shell


nidn

required ? (CBSE 2008)


Re

[You will learn from this exercise why one


cannot build an electrostatic generator using
FFi

a very small shell, which requires a small (c) Give the sign of the work done by the field
charge to acquire a high potential.] in moving a small positive charge from Q to P.
Sol. Here, V= 15x 10® volt.. Dielectric strength = 5 x (d) Give the sign of the work done by external
10^ Vm“*, Minimum radius, r = ? agency in moving a small negative charge from
B to A.
Max. electric field, £ = 10% dielectric strength
(e) Does the kinetic energy of a small negative
£ =
10
x5xl0^ =5xlO®Vm charge increase or decrease in going from B
100 toA ? (NCERT Solved Example)
1
Sol. (fl) As V OC
Vp> Vgor(yp-Vg)ispositive
_V 15x10® - 3m
As
" E~ 5x10® Also, Vg is less negative than
(V distance of B is more than distance of A)
Obviously, we cannot build an electrostatic Thus, Vb>V^ (Vg-y^) is positive.
generator, using a very small spherical shell.
2/68
Fundamental Physics (XII) VOL.I

(b) A negative charge held at Q would move


III. Electrical
towards P. As charge moves from higher potential
energy to lower potential energy. capacitance and capacitors
(P.E)^>iP.E:)p IS. Two copper spheres of same radii, one hollow
Therefore, sign of potential energy difference of and other solid are charged to same potential.
a small negative charge between Q and P is positive. Which, if any, of the two will have more
charge ? (CBSE 2017)
SiEnilarly, (.PE)^^ > (/^.£)g
Sign of potential energy difference is positive, Sol. Capacitance of a spherical conductor
(c) In moving a small positive charge from Q to C=4ti€o/?, i.e., C«R
P. work has to be done by us against the electric As both spheres have equal radii, they have the
field. Therefore, work done by the field is same capacitance. When charged to same
negative. potential, they will have the same charge.
Ul) In moving a small negative charge from B to 16. Two identical metal plates are given charges

ww
A, work has to be done by us against the field. respectively. If they are now brought
close to form a parallel plate capacitor with
Therefore, work done by external agency is capacitance C, what will be the p.d. between
positive.

Floo
the plates ?
(e) Due to force of repulsion on the negative Sol. Let A be the area of each plate. When the two
charge, velocity decreases and hence K.E.

ee
plates are placed d distance apart, capacitance of
decrea.ses in going from B to A. parallel plate capacitor so formed is

reer
14. Show that the force on each plate of a parallel

rFF
A
0
plate capacitor has a magnitude equal to C =
d
uur r
If £| and £2 are electric fields due to the two
-QE, where Q is the charge on the capacitor
and E is the magnitude of electric field between
ffoor
plates, then the net field between the plates
sks
£ = £j - £2
YYoo
a 1
the plates. Explain the origin of factor - .
ooko

1
2 2€ 2e 2Ae
0 0 0
eBB

Sol. If F is the force on each plate of a parallel plate


p.d between the plates
capacitor, then work done in increasing the
separation between the plates by Ax = F. Ax. 1
r

V = Ed =
This must be the increase in potential energy of (^1 -^2^'xd
ouur
ad

2e„0 A
the capacitor.
Y

Now, increase in volume of capacitor = A.A x


I -^2
If u = energy density = energy stored/volume, then
dY

V =
2e.0 A (^1-^2) =
Re

increase in pot. energy = u.A Ax 2C


nnd

FAx=u.AAx d
FFii

M 17. What is the physical significance of


F = u.A = - 0
£^ A = capacitance ?
12
Sol. Capacitance of an arrangement tells us the
1 V ) €« A potential to which the arrangement will be raised
£ = -€.,0 A— £ V C =
0

21 d) d when a given charge is placed on it. For a given


charge, smaller the potential, larger must be the
I capacitance and vice-versa.
F = ~{CV)E = ~QE
2 2
18. By what factor does the capacity of a metal
The origin of factor 1/2 in force can be explained sphere increase if its volume is tripled ?
by the fact that inside the conductor, field is zero
and outside the conductor, the field is £. Sol. As V' = -nR^
3
Therefore, the average value of the field {i.e.
£/2) contributes to the force. or

t
ELECTROSTATIC POTENTIAL AND CAPACITANCE 2/69

^1/3 FIGURE 2(Q).12

C. R, ,V
I )
\II3
fv
^2
q = c, V.
= C, (3)1/3 ^ 1.44 c,
1 )
19. A parallel plate capacitor of capacitance C is
charged to a potential V by a battery. Without
disconnecting the battery, distance between the
plates of capacitor is tripled and a dielectric

oww
medium of /C = 10 is introduced between the
plates of capacitor. Explain giving reasons how
will the followingbe affected ? In the space between the two spheres, eleciric
(a) Capacitance of capacitor intensity £ exists as shown. Potential difterence

e
(/>) Charge on capacitor between the two spheres,

re
FFrlo
(c) Energy density of capacitor. Q Q
[CBSK 2017 (O) V =

rF
ee
Sol. Without the dielectric and with standard notations,
e.0 A Q Q -r^)

rF
C =
ouru d 4jce<, [r. 4ne 0
^ I ^2
When battery is not disconnected, potential

fosor
Q
difference
As C = ^
skf
V = constant.

Q4Tt;e„ ,r 4ne„r,r2
ooko
r/' = 3c/and K= 10
C =
Yo
(2(f| -r^) ’‘\-h
Y
lOe.A lOfe^A'l = —C
10
Bo

C' =
d' 3rf 3 d 3 IV, Grouping of capacitors
reeB

10 10 21. As shown in Fig. 2(Q).I3, a dielectric material


Charge on capacitor, Q' = CV = — CV = — Q
ooY

of dielectric constant K is inserted in half


uur

3 3
portion between plates of parallel plate
ad

Energy density,
capacitor. If its initial capacitance is C, what is
dY

1 V'2 K 10 the new capacitance ?


p' = -^re 0
32 2^0
_

^ 2 d'^
nind

FIGURE 2(Q).13
Re

20. A spherical capacitor consists of two concentric


FFi

spherical conductors, held in position by


suitable insulating supports. Show that the K
capacitance of this spherical capacitor is given
by c =
'■i-'‘2
Sol. The new arrangement is equivalent to two
where rj and r2 are the radii of outer and inner capacitors connected in parallel,
spheres respectively.
e. A/2 K e„ A/2
Sol. As is clear from Fig. 2(Q). 12, + Q charge spreads 0
; C, =
0

d
uniformly on inner surface of outer sphere of
radius r,. The induced charge - Q spreads 6^0 A K €n0 A
uniformly on the outer surface of inner sphere of Cp = Cj + C2 — Id Id
radius r^- The outer surface of outer sphere is
earthed. Due to electrostatic shielding, £ = 0 for €00 A C
(K + \) =-{K+l)
r <T2 and £ = 0, for r > rj. 2d
2/70
^***^^®^ ^ Fundamental Physics (XII) PRTi
22. A technician has only two capacitors. By using
them singly, in series or in parallel, he is able 6x10-^
■■ ^2 ~ = 2V
to obtain the capacitance of 4, 5, 20 and 25 3x10"^
(^2
micro farad. What are the capacitance of the
two capacitors ? V. Energy stored and loss
Sol. In series grouping, the resultant capacitance is of energy on sharing charges
minimum, i.e..
24. An uncharged capacitor is connected to a
= 4|iF battery. Show that half the energy supplied by
Cj +C2 the battery is lost as heat while charging the
In parallel grouping, the resultant capacitance is capacitor.
maximum, i.e., Sol. Let capacitance of a capacitor = C
Cp — C] + C2 = 25 |i F em/of the battery = V
Charge given to the capacitor, q = CV

w
It implies that the remaining values 5 n F and 20
|X F represent the individual capacitance values Energy supplied by battery = Work done by battery
of two capacitors. = qV
23. Fig. 2(Q).14 shows two identical capacitors Cj 1 1
Energy stored in the capacitor = -CV^ = -qV

FF loow
and C2, each of 1 p. F capacitance, connected to - 2

ee
a battery of 6 V. Initially, switch S is closed. After 1 1
sometime, 5 is left open and dielectric slabs of Energy lost as heat =

Fr
dielectric constant X = 3 are inserted to All
completely the space between the plates of two So, half energy is lost as heat while charging the
capacitors. How will the (i) charge and capacitor.

rer
ur
(u) potential difference between the plates of the 25. A spherical shell of radius h with charge Q is
fofr Fo
capacitors be affected after the slabs are expanded to radius a. Find the work done by
(CBSE 2011) the electrical forces in the process ?
ks
inserted ?
Sol. Work done by electrical forces in the process
YYouro
s oo

FIGURE 2(Q).14 s: final stored energy - initial stored energy


^1.
Q}- 12^ 1
BBook
r ee

6V-=-
Cl C2 2C2 2Cj 2{4n:EQa) 2(4jieQfc)
I^1UF
_ \I r
oouru
ad

SjtEQLa b.
Y

Sol. In Fig. 2(Q).14, when switch S is closed, both Cj 26. Two charged spherical conductors of radii R^
and C2 are charged to 6 V potential, and i?2 when connected by a connecting wire
Ynd
Re

I.e.
Vi=6V acquire charges q^ and ^2 respectively. Find
the ratio of their charge densities in terms of
FFindi

and 1^2 = 6 V
their radii ? (CBSE 2014)
When S is left open,
Sol. Two spherical conductors, when connected by a
Cj is still connected to battery. On introducing wire will share charges till, they have common
slab of dielectric constant, K=Z, potential. Thus,
C[ = KC^ =3pF = 3xlO-^F
Si or

Vj' = Vj = 6V 47ISqT?2 ^2 ^2
Ratio of surface density of charges is
q[ = CjV/ = 3 X 10-6 X 6 ^ 18 ^ iq-6 q
However, C2 is disconnected from battery now, 0| q^/A%R^ q^ R^
when 5 is left open. Due to introduction of slab, C2 q2/4nR^ ^2
C' = ^:C2 = 3 p F = 3 X 10-6 F R1
X
q' = 02 = C2 V2 = 1 X 10-6 X 6 = 6 X 10-6 C /?2 R}I R1
ELECTROSTATIC POTENTIAL AND CAPACITANCE 2/71

But when we are in the house, discharge due to


VI. Dielectric Polarization
lightning may be conducted to the ground through
and Van de-graff generator iron pipes/wall etc. Therefore, we are safer.
27. What is the dielectric constant of a metal ? 31. The safest way to protect yourself from
Sol. The dielectric constant of a metal is infinity. The lightning is to be inside a car. Comment
electric field inside a conductor is zero so the Sol. The body of the car is metallic. It provides
dielectric constant, which is the ratio of applied electrostatic shielding to the person in the car,
electric field to the reduced electric field, will be because electric field inside the car is zero. The
infinite for a metallic conductor.
discharging due to lightning passes to the ground
28. The introduction of dielectric slab between the through the metallic body of the car.
capacitor plates increases the capacitance. 32. A sensitive instrument is to be shifted from the

ooww
Why? strong electrostatic field in its environment.
Sol. A dielectric slab of dielectric constant K reduces Suggest a possible way.
the electric field from E to E/K. This reduces the
Sol. For this, the instrument must be enclosed fully in
potential difference from V to V/K. Hence the a metallic cover. This will provide electrostatic

capacitance increases from C to KC. shielding to the instrument.

e
29. Where is the knowledge of dielectric strength
33. Faraday entered a big metallic cage supported

re
helpful ? on insulating pillars and then charged the cage

rFFl
ree
Sol. Dielectric strength is the maximum strength of

F
by a powerful electric machine. He remained
electric field that can be tolerated by the dielectric
quite safe inside the cage. Do you believe on

rF
without electric breakdown. Its knowledge helps this happening ?
us in designing a capacitor by determining the
Sol. Yes. As the cage is insulated, the entire charge
fsfoor
ouur
maximum potential difference that can be applied remained on the outer surface of the cage. The
across the plates of the capacitor.
potential at all points inside the cage remained
kosk
30. During lightning, you are safer inside a house the same. As there was no potential difference
than under a tree. Why ?
YYo
between any two points in the cage, Faraday could
oo

Sol. When we stand under a tree, we provide an easy not get the shock and remained quite safe inside
BB

path for the lightning to pass through our body. the cage.
rre

● Very Short Answer


ouu

ni
1
● Short Answer
Y
ad

● Long Answer
dY

iVERY SHORT ANSWER QUESTIONS Carrying 1 mark


innd
Re

Ans. At any point on equatorial line of an electric


I. Electrostatic Potential, Potential
FFi

dipole, electric potential is zero. Therefore, work


difference related to Electric field done in moving a charge q from A to 5 is zero.
1. A charge q is moved from a point A above a 2. Defme the unit of electric potential.
dipole of dipole moment p to a point B below Ans. The S.I unit of electric potential is volt. The
the dipole on equatorial plane without electric potential at a point is said to be I volt
acceleration. Find the work done in the when I J of work is done in moving a positive
process. (CBSE 2016) charge of I coulomb from infinity to that point
FIGURE 2(Q).15 against the electrostatic force.
A 3. What are the points at which electric potential
of a dipole has (0 maximum value («) mini
mum value ?
-d -+q Ans. (/) At axial points, the electric potential of a
dipole has maximum positive or negative value.
(ii) At equatorial points, the electric potential of
B a dipole is zero.
2/72
‘P’uideeft. ^ Fundamental Physics (XII)
4. Name the physical quantity which has its unit
joule coulomb"^ Is it a scalar or vector ? Ans. V { r ) =
(Kerala Board 2012) 47l€y I 4tC€q I
Ans. joule coulomb'* is the unit of electric potential. 13. Draw an equipotential surface for a uniform
It is a scalar quantity.
electric field in A'-direction.
5. In a certain 0*1 of space, electric potential
is found to be 5 V throughout. What is the (CBSE 2019, 2008)
electric field in this region ? Ans. For a uniform electric field, say along X-axis,
the equipotential surfaces are planes normal to
dW
Ans. As £ = - the X-axis, i.e., planes parallel to Y-Z plane as
dr shown in Fig. 2(Q).16.
and V is constant throughout, therefore, £ = 0. FIGURE 2(Q).16
6. What is the electrostatic potential due to

ww
electric dipole at an equatorial point ?
Y X-axis
(CBSE 2009)
Ans. Electric potential, Y = 0. E

Flo
7. The middle point of a conductor is earthed >
2
and its ends are maintained at a potential

ee
difference of 220 V. What is the potential at

rere
the two ends and at the middle point ?
14. Can two equipotential surfaces intersect

rFF
Ans. Potential at the middle point is zero. The values each other ? Give reasons.
of potential at the two ends of the conductor must (CBSE 2009)
Ans. No. An equipotential surface is normal to the
uurr
be -i-I lO V and - I lO V, so that pot. diff. across
the two ends is 1 lO - (-110) = 220 V.
8. Do electrons tend to go to regions of high
foor electric field intensity. If two equipdteniial
surfaces intersect, then at the point of
ks s
potential or low potential ? intersection, there will be two directions of
Yoo
electric field intensity, which is not possible.
oook

Ans. As electrons are negatively charged, they have


tendency to go to regions of high potential.
eBB

9. Suppose that the earth has a net charge that II. Electrostatic Potential Energy
is not zero. Is it still possible to adopt the earth 15. An electron is accelerated through a potential
as a standard reference point of potential and
rr

difference V. Write the expression for its final


assign the potential V = 0 to it ?
ouu
ad

speed If it was initially at rest.


Ans. Yes, earth can still be taken as a standard (CBSE 2018 (C)]
YY

reference point for potential V = 0, because Ans. If e is charge on electron, m is mass of electron
capacity of earth is infinitely large. and V is final speed of electron, then as
nndd
Re

10. A hollow metal sphere of radius 5 cm is Kinetic energy acquired = work done on it.
charged such that potential on its surface is
Fii

1
10 V. What is the potential at the centre of therefore, —
t mv-=eV.
the sphere ? 2
(CBSE 2011)
Ans. 10 V, since electric potential is same at all points
V =
2eV
inside the hollow sphere and at the surface of m
the sphere.
16. Define the term potential energy of charge q
II. Electric field due to an electric dipole is at a distance r in an external electric field.
cylindrically symmetric. Comment.
Ans. The potential energy ofcharge^ is the work done
Ans. It means field due to electric dipole will be same
in bringing charge q from infinity to the position
at every point on the surface of a irght circular —^

cylinder imagined with the electric dipole as the r in the external electric field
axis.

12, Write an expression for potential at a point P v(7) = q.vC)


17. What will be the electrostatic potential energy
(r ) due to two point charges
->
and ^2 at ^ of the dipole, when placed at right angle to
and respectively. the field ?
ELECTROSTATIC POTENTIAL AND CAPACITANCE 2/73

Ans. Zero. This is because,


FIGURE 2(Q).17
U = -pEcos Q = -pEcos 90° - Zero.
18. Give expression for ‘potential energy’ of
charge at a distance ‘r’ in an external
electric Held.
Ans. In an external field E, the potential at any point
distance r is given by V = Er.
RE. =q.V= qEr
19. 5 J of work is donein movinga positivecharge
26. The distance between the plates of a parallel
of 0-5 C between two points. What is the
plate capacitor is d. A metal plate of thickness

oww
potential difference between these points ?
dll is placed between the plates, what will be
work done the new capacity ?
Ans. Potential difference = — = 10 V
charge 0-5 Ans. As electric field inside the metal plate is zero, d
20. What is the work done in moving a test charge becomes dll. Hence C becomes twice (from
C = eoA/i/).

e
q through a distance of 1 cm along the
27. Can there be a potential difference between

re
equatorial axis of an electric dipole ?

FFrlo
two conductors of same volume carrying
(CBSE 2009)
equal positive charges ?

rF
ee
Ans. On equatorial axis of a dipole, V = 0
Ans. Yes, because two conductors of same volume
W = q xV= zero.
may have different shapes and hence different

rF
21. Define the term potential energy of charge q
ouru
capacitances.
at a distance r in an external electric field.
28. What is the net charge on a charged

fosor
(CBSE 2009) capacitor ? (Bihar Hoard 2011)
Ans. If V (r) is potential at the point due to the external Ans. Zero, because one plate has positive charge and
skf
electric field, then .PE. = W = q.V (r). the other carries an equal negative charge.
ooko
29. On which factors does the capacitance of a
Yo
III. Electrical
Y
capacitor depend ?
capacitance and capacitors
Bo

Ans. It depends on geometry of the plates, distance


reeB

22. Write two applications of capacitors in between them and nature of dielectric medium
electrical circuits ? separating the plates.
ooY

Ans. (0 Capacitors are used in radio circuits for tuning


uur

IV. Grouping of capacitors


ad

purposes,
(ii) Capacitors are used in power supplies for 30. If two isolated spherical conductors each
dY

smoolhening the rectified current. having a definite capacity are far apart and
23. In what form is the energy stored in a charged are connected to eachother by a fine wire, how
nind

do you calculate the capacity of the combi


Re

capacitance ?
Ans. Energy is stored in the form of electrostatic nation ? In joining them with the wire, have
FFi

potential energy in the electric field between the you connected them in parallel or in series ?
plates of capacitor. Ans. On connecting, both the spheres acquire a
24. Write down the expression for the capacitance common potential V. If C is total capacity, then
of a spherical capacitor. total charge, q = CV = charge is
ab conserved.
Ans. C = 47ie 0 , where a and b are the radii
b-a c = = Cj + C2
of the inner and outer spheres respectively. V V
25. Sketch a graph to show how charge Q given Hence the conductors have been connected in
to a capacitor of capacity C varies with the parallel.
potential difference .V 31. A parallel plate capacitor with air between
Ans. Initially, the graph between V and Q is a straight the plates has a capacitance of 8 pF. The
line. However, beyond a certain stage, some separation between the plates is now reduced
leakage of charge starts and V does not increase by half and the space between them is filled
with increasing Q. This variation is shown in with a medium of dielectric constant 5.
Fig. 2(Q).17. Calculate the new value of capacitance.
2/74
'4, Fundamental Physics (XII) VOL.I

0
A 34. What is the basic difference between a
Ans. C„ = - = 8pF charged capacitor and an electric cell ?
e«0 A Ans. A capacitor supplies electrical energy stored in
C=K.
dll - 2 KCq = 2 X 5 X 8 pF it. A cell supplies electrical energy by converting
chemical energy into electrical energy at constant
= 80pF potential difference.
V. Energy stored and loss of 35. Why is a space ship entering the ionosphere
energy on sharing charges not sufficiently heated, though temperature
at the top of ionosphere is nearly 700 K ?
32. The electric field inside a parallel plate Ans. This is because density of air in ionosphere is so
capacitor is E. Find the amount of work done low that very few molecules only bombard the
in moving a charge q over a closed rectangular space ship.
loop abeda. (CBSE 2014)

w
VI. Dielectric Polarization
Ans. Refer to Fig. 2(Q).18, Work done in moving a
charge q on path abed and Van de-graff generator
36. Define polarization density.

Flo

- qE. {ab) + qE .{bc)-¥ qE .{cd)-\- qE .{da) Ans. The induced dipole moment developed per unit
= qE (ab) cos 90° + qE {be) cos 0° volme in a dielectric slab on placing it in an

ee
+ qE {ed) cos 90“ + qE {da) cos 180“ external electric field is called polarization density.

Fr
=0+ {be) + Q- qE {da) 37. Define dielectric strength of a medium. What
= 0 is its values for vacuum?
(●.● be = da)
Ans. The maximum electric field that a dielectric
for
ur
FIGURE 2(Q).18 medium can withstand without breaking down
+ + + + + + + of its insulating property is called dielectric
ks
b strength. Its value for vacuum is infinity.
Yo
38. Why does the electric conductivity of earth’s
oo

E atmosphere increase with altitude?


eB

Ans. This is because of ionisation caused by highly


d energetic cosmic ray particles from cosmos,
which are hitting the atmosphere of earth.
r

39. Why does the electric field inside a dielectric


ou
ad

33. Where does the energy of a capacitor reside ? decrease when it is placed in an exte.nal
Y

Ans. The energy resides in the dielectric medium electric field?


separating the two plates. Ans. It is because the dielectric gets polarised.
nd
Re
Fi

SHORT ANSWER QUESTIONS Carrying 2 marks

I. Electro.static Potential. Potential


difference related to Electric field Ans.

1. Define an equipotential surface. Draw


equipotential surfaces
(0 in the case of a single point charge,
(ii) in a constant electric field in Z direction.
Why the equipotential surfaces about a single
charge are not equidistant ?
(iw) Can electric field exist tangential to an

equipotential surface ? Give reason.


(CBSE 2016)
2/75
ELECTROSTATIC POTENTIAL AND CAPACITANCE

(0 An equipotential surface is that at every point Ans. (i) p.d. does not depend upon the path along
of which electric potential is the same. Due to a which the test charge moves.
single point charge, equipolnetial surfaces are
concentric spherical surfaces centred at the E = - dv _-(yc-^AK^c-^A
dx 2-6 4
1
charge. Fig. 2(Q).15. As v OC — the equi- or Vjj - = 4 E. Therefore, > ^A
r
(ii) Direction of electric field is in the direction
potential surfaces about a single charge cannot of decreasing potential. So,
be equidistant.
(ii) In a constant electric field in Z-direction,
4. Fig. 2(Q).22 shows the variation of electro
static potential V with distance x for a given
equipotential surfaces lie in XT plane charge distribution.
perpendicular to Z-direction as shown in Fig.

ooww
2(Q).20. . FIGURE 2(Q).22

FIGURE 2(Q).20
A B

X ->
E
V

e
c

ree
z

rFl y y

Fre
rrF
(Hi) As electric field is perpendicular to the From the points marked A, B and C, identify
equipotential surface, the field cannot exist the point at which electric field is (/) zero
sffoo
ouur
tangential to the equipotential surface. (ii) maximum.
2. Justify that electrostatic potential is constant Explain your answer in each case.
oksk
throughout the volume of charged conductor dV
and has same value on its surface as inside it. Ans. As £ = - = Negative slope of V-x graph,
YYo
dx
oo

(CBSE Sample Paper 2011)


dV
BB

dV dV
Ans. Since electric field inside the conductor is zero At A. = 0 ; AtS, <0 ;AtC >0
dx dx dx
and has no tangential component on its surface,
rre

therefore, no work is done in moving a test (i) E is zero at point A.


charge within the conductor or on its surface. It (ii) E is maximum at point B.
ouu
Y
ad

means potential difference between any two 5. The electric potential V at any point x, y, z
points inside or on the surface is zero. Hence, (all in metres) in space is given by V =
dY

electrostatic potential is constant throughout the volts. Calculateelectric field at the point (Ira,
volume of charged conductor and has same value Om, 2m).
innd
Re

on its surface as inside it. Ans. Here V=4yp'


3. A test charge g is moved without acceleration
FFi

dV
E. = -Sx,
from A to C along the path from A to B and dx
then from B to C in electric field E as shown
dV
in Fig. 2(Q).21. (i) Calculate the potential £ = 0,
difference between A and C (ii) At what point dy
[of A and C] is the electric potential more and dV
(CBSE 2012) £ = 0
why ? z
dz
FIGURE 2(Q).21 A

.●. £ = -8xi
(2. 3) B At point (1 m, 0, 2 m).
♦ E A A
-1
£=-8x1/ = -8t Vm
>
6. A metallic solid sphere is placed in a uniform
A(^0) electric field, Fig. 2(Q).23(a). Which path is
(2. 0) C
followed by the lines of force ?
2/76
Fundamental Physics (XII) VOL.I

dV
Ans. Using the relation, E = - , we find that
dr
electric intensity is maximum at P and minimum
at K.

9. Electric charge is distributed uniformly on the


surface of a spherical rubber balloon. Show
how the value of electric intensity and
potential vary (/) on the surface («), inside and
(m) outside ?
Ans. On the surface, E = constant, V = constant. Inside

oww
the surface, E = 0, V = constant = potential on
the surface. Outside the balloon.
1 1
Ans. When a solid conducting sphere is placed in a £ « —5- and V OC
- where r is distance of point
r

uniform electric field E . free electrons of the

e
from the centre of the balloon.

re
FFrlo
conductor move in a direction opposite to E . 10. Draw 3 equipotential surfaces corresponding
to a field that uniformly increases in
This results in the development of some negative

rF
magnitude but remains constant along

ee
charge on the left part of the sphere and an equal positive Z-direcdon. How are these surfaces
positive charge on right part of the sphere. different from that of a constant electric field

rF
Electric lines of force end normally on left part,
ouru
along Z-direction ? (CBSE 2009)
and start normally from the irght part. Therefore,
Ans. The three equipotential surfaces are in XY plane.
fosor
path 4 is correct. Fig. 2(Q).23(i>).
7. Can we create an electric field in which all As magnitude of field increases along positive
skf
Z direction, distance between successive
the lines of force are parallel but their density
ooko
increases continuously in a direction per equipotential surfaces (for given dV) decreases
YYo
as shown in Fig. 2(Q).27. If electric field were
pendicular to the lines of force, Fig. 2(Q).24 ? constant, distance between successive
Bo

FIGURE 2(Q}.24 equipotential surfaces would stay constant.


reeB

FIGURE 2(Q).2S
> >
>
-B FIGURE 2(Q).27
zzizzz:
ooY

>
uur

I
L ►
V

ad

n
ZZZt D:
dY

CO

Ans. No, this is because work done in moving a unit ' § 8


positive charge along the closed path ABCD ;.|i
nind
Re

would no longer be zero, being more along CD, g-w


UJ

less along AB, zero along BC and DA, Fig.


FFi

2(Q).25. This work done has to be zero, because E

electrostatic forces are conservative forces. 11. A test charge is moved without acceleration
8. Fig. 2(Q).26 shows lines of constant potential from A to C over the path ABC as shown in
in an electric field. Out of the three given Fig. 2(Q).28.
points ,P Q, R where is electric field intensity Calculate potential difference between A and
maximum and where is it minimum ? C.
(CBSE 2012)
FIGURE 2(Q),26
FIGURE 2(Q).28

ft E

40V
30V
4^ cL 0
20V 10V
ELECTROSTATIC POTENTIAL AND CAPACITANCE 2/77

4
14. Two protons A and B FIGURE 2(Q).30
Ans. AsisclearfromFig.2(Q).35, AB + BC = AC are placed between two +
●A
dv
parallel plates having a +
+
As £ = - potential difference V as +
dr d d +
shown in Fig. 2(Q).30. +

.-. V^-Vc^Ed.
+
Will these protons +
B*

12. Electric potential at a point O due to a number experience equal or


of point charges equidistant from 0 is Vj unequal force ?
when charges are uniformly distributed and Ans. Electric field E between the two plates is
it is Vi when charges are non uniformly uniform, i.e., E is same in the entire space. As
distributed, Fig. 2(Q).29. Is Vj = Vj ? Justify. F = qE, therefore, two protons A and B will

oww
experience equal forces.
15. Define surface density of charge and potential
of a charged spherical conductor. Establish a
relation between them.
Ans. Surface density of charge is defined as the charge

ee
per unit area of a charged conductor. If r is the

FFrlo
radius of a spherical conductor having charge q,

r
then its surface density of charge.

rF
ee
Ans. If r is radius of the sphere with centre O, then by
definition, G = ...(/)

rF
ouru
V. =4
! , and V2 = 4 Electric potential of the spherical charged
4 Tie 47160 r
ffosor
conductor,
O'-J os k
.-.Vl = V2 y = or q = 4neQrxV
This is because electric potential is a scalar 471^0 f
ook
quantity. It depends only on distance of the point Putting this value in (i), we get
YYo
from the charge and not on direction of its 47i€^rV GrtV
Bo

0 0
location from the charge. a =
reeB

13. A cube of side b has a charge q at each of its 47ir^ r

vertices. Determine the potential and electric 16. A charge + 0 is lying at the centre of a circle.
oouY

field due to this charge array at the centre of What is work done in carrying charge q from
ur

the cube. A to B, where A and B, both lie on the circle.


ad

Sol. We know that the length of diagonal of the cube Ans. The circular path around the charge + (2 is an
dY

equipotential surface. As A and B lie on the circle


of each side b is ^3b^ = £> VS i.e., on the equipotential surface, work done = 0.
nidn

Distance between centre of cube and each


Re

17. Around a point charge of 1 nC, what is the


distance of an equipotential surface of 0-9 V ?
FFi

vertex, r -
2 Ans. Here g = 1 nC = IQ-^ C, V = 0-9 V, r = ?
As y =
As y = 4716^0 r
47160 r
and 8 charges each of value q are present at the 10"^x9xl0^
r = = 10 m
eight vertices of the cube, therefore. 4716.^0 r 0-9
1 8^
^ 47160 II. Electrostatic Potential Energy

18. A proton is released from rest in a uniform


or
electric field. Does the proton’s electric
V371€q b
potential energy increase or decrease ? Does
Further, electric field intensity at the centre due the proton move towards a location with a
to all the eight charges is zero, because the fields higher or a lower electric potential.
due to individual charges cancel in pairs.
2/78
“Pn<i4Uef.i ^ Fundamental Physics (XII)
Ans. The potential energy of the proton decreases (as Ans. Work done in moving a charge along the closed
it gains kinetic energy due to its motion in the rectangular path would be zero, because field in
electric field). Further the proton (being positive) the entire space is uniform and electrostatic
moves from higher to lower electric potential. forces are conservative forces.
19. When a proton approaches another fixed 23. Three charges and g^ are kept
proton, what happens to : respectively at points A, B and C as shown in
(a) the kinetic energy of the approaching Fig. 2(Q).32.
proton Write the expression for electrostatic potential
(b) the electric potential energy of the system energy of the system.
and
(CBSE Sample Paper 2019-20)
(c) the total energy of the system ?

oww
Ans. (a) The kinetic energy of the approaching proton
decreases.

(b) The electric potential energy of the system


increases because distance between the charges

ee
decreases,

FFrlo
(c) The total energy of the system remains the

r
same because of the conservation of energy.

rF
20. Write an expression for potential energy of

ee
Ans. As is known from theory, electrostatic potential
two charges and ^2 ^ and ^ in a energy of the system of three charges can be put

rF
ouru
uniform electric field E . (CBSE 2010) as

Ans. If V (/^) and V (r^) are electric potentials at


fosor RE. = —?li2_ ^2^3 ^3<?i
4Jt€or,2 47C€q r23 47te 0
s kf
^13
points 7^ and where charges and ^2 are
ooko
1
held, then P.E. = SiSi+Mi+M.
YYo
4tI€
0 ^12 ^23 ^13
Potential EneTgy = q^.V (1^) +q2.V (^J)
Bo
reeB

HI. Electrical
+
^1 ^2
capacitance and capacitors
ooY
uur

24. The difference between the radii of the two


ad

Obviously, the first two terms are the result of spheres of a spherical capacitor is increased.
interactions of q^ and ^2 separately with the Will the capacitance Increase or decrease ?
dY

sources of electric field. The third term is due to


mutual interaction between the two charges. Ans. The capacitance C decreases as c = 4ti€q ab
nidn
Re

21. Equipotential surfaces are perpendicular to b-a

field lines. Why ? (CBSE 2014) where a, b are radii of two spheres. So if b-a
FFi

Ans. No work is done in moving a charge from one increases, capacitance C will decrease.
point on equipotential surface to the other. 25. The given graph in Fig. 2(Q).33 shows the
Therefore, component of electric field intensity variation of charge q versus potential
along the equipotential surface is zero it means, difference for two capacitors Cj and €2- The
the electric field intensity is perpendicular to two capacitors have same plate separation,
equipotential surface. Hence, the surface is but the plate area of C2 is double that of Cj.
perpendicular to field lines.
22. A uniform electric field
E exists between two FIGURE 2(Q),31
charged plates as shown + A
+
in Fig. 2(Q).31. What +
would be work done in +
+
moving a charge g along
the closed rectangular + D C
path ABCDA ?
ELECTROSTATIC POTENTIAL AND CAPACITANCE 2/79

Which of the lines in the graph correspond to on them. Which of the two has higher
Cj and C2 and why ? capacitance ? Give reason.
€ A.
Ans. As C = 2—, and plate area of C-> is double
d
n

thanthatofCj,therefore, C2>C|.Now C = —,
which is greater for line A.
The line A of the graph corresponds to C2 and
line B corresponds to Cj.
26. The space between the plates of a parallel
plate capacitor is filled consecutively with two
Ans. As C = QIV and from the figure, for same value

oww
dielectric layers of thickness and ^2 having
relative permittivities £ j and € 2 respectively. of Q ; A has lower value of V and B has higher
If A is area of each plate, what is the capacity value of V. Therefore, A has higher capacitance.
of the capacitor ? 30. Two circular metal plates each of radius
Ans. As effective thickness of dielectric is 1/e times 10 cm are kept parallel to each other at a

e
the actual thickness, therefore, we may write the distance of 1 mm. What kind of capacitor do

rFFlo
re
6.0 A they make ? Mention one application of this
capacity of condenser as C = -j capacitor. If the radius of each of the plates is

ree
F
increased by a factor of ^ and their distance
£1 £2

rF
of separation is decreased to half of its initial
27. An uncharged insulated conductor A is bro> value, calculate the ratio of capacitance in the
two cases. Suggest a possible method by which
fsfoor
ouur
ught near a charged insulated conductor B.
What happens to charge and potential of £ ? the capacitance in second case is increased to
kosk
n times.
Ans. The charge on conductor B remains same, but
the potential of B gets lowered because it induces Ans. These circular metal plates form a parallel plate
Yo
charge of opposite sign on conductor A. capacitor. Such a capacitor is used in filter
oo
Y

circuits.
28. n small drops of same size are charged to V
BB

volt each. They coalesce to form a bigger drop.


£q A_£QTtr^
rre

Calculate the capacity and potential of the C =


d d
bigger drop.
oYuu

Ans. Let r be the radius of each small drop and R be Eq a' _ £q 71 r)^
ad

C = = 4C
the radius of bigger drop. d' dll
dY

As volume of bigger drop = volume of n small


C 1
drops
innd

C 4
Re

4 3
=nx—nr In the second case, capacitance will become n
3 3
FFi

times by introducing a dielectric of specific


R = r inductive capacity K = n inbetween the plates of
As capacity varies directly as the radius. the capacitor.
1/3 31. A slab of materia] of dielectric constant K has
Capacity of bigger drop becomes n times

the capacity of each small drop. the same area as the plates of a parallel plate
capacitor, but has a thickness 3d/4. Find the
total charge
Also, potential of bigger drop = ratio of the capacitance with dielectric inside
capacity it to its capacitance without the dielectric.
(CBSE (F) 2017)
nq nq 2/3 ^
V = = rt
4JtEn0 r Ans. Without the dielectric, capacity of the capacitor
47ceQ/? r
,2/3
times the potential of each small drop. = ,c 0 _€o A
-
d
29. The graph in Fig. 2(Q).34 shows variation of
With dielectric of thickness t = 3d/4, the capacity
voltage V across the plates of two capacitors of condenser becomes
A and B versus increase of charge Q stored
2/80
<t Fundamental Physics (XII) WW1W1

C =
grt0 A Ans. (a) As Q = CV, If V doubles, charge also
1 > gets doubled.
d-t 1- —
K
(b) As t/ =-cy^, If V is doubled, U is
2
€«0 A €«0 A
C = quadrupled.
1- — 36. On charging a parallel-plate capacitor to a
4 K 4 K
potential V, the spacing between the plates is
halved and a dielectric medium of = 10 is
C _ 4 4K
introduced between the plates, without
^0 1 +1 (/i: + 3)
disconnecting the dc source. Explain using
K
suitable expression how the (a) capacitance
IV. Grouping of capacitors (b) electric field (c) energy density of the
capacitor change. (CRSE 2008)
32. Given a battery, how would you connect two Ans. As the d.c. source remains connected, p.d. (V)

w
capacitors, in series or in parallel for them to between the plates of capacitor remains
store the greater (a) total charge (ft) total unchanged even after dielectric is inserted
energy ?

Flo
between the plates.
€ .A
Ans. Total charge q = CV and total energy u = — CV^

reee
2
(fl) Original capacitance Cq = d
Now, V is constant and C„P > CS'

FFr
therefore, parallel combination is required for New capacitance Cq = d/2
= 20C
0

storing greater charge and greater energy.


urr
33. A metal foil of negligible thickness is
introduced between two plates of a capacitor
for
(ft) Changed electric field,

E =
V
= im) = 2E^
kkss
at the centre. What will be the new d/2
capacitance of the capacitor ?
Yo
(c) Original energy density, jj 0 = 1 g £2
ooo

Ans. On introducing a thin metal foil, d is halved.


2 ^
Arrangement is equivalent to two condensers
eB

1
each of capacity 2 C in series. New energy density, IJ
2
— = —+ 1- = 1
r

1 1
■■ 2C 2C C
ou
ad

= 4e
'■ 2 xCoEq
c, = c
YY

Capacity is unchanged. = 4x10^0 =40C,0


34. Two capacitors of capacitance 6 Ji F and 37. The separation between the plates of a
nndd
Re

12 |i F are connected in series with a battery. charged capacitor is to be increased. Explain


The voltage across the 6 p F capacitor is 2 V. when work done will be more in case battery
Fi

Compute the total battery voltage. is removed after charging the capacitor or
battery remains connected.
Ans. In series, charge on Cj = charge on C2 Ans. With increase in separation between the two
6x2=12x V2 plates, capacity C decreases.
12 When battery is removed, charge Q and electric
V2 = —
12
= I volt
o Q
field £ = — would remain constant.
Battery voltage V=V, + V’2 = 2+1=3V ^0 As
0

V. Energy stored and loss of But when battery remains connected, V is


energy on sharing charges constant, Q (= CV) decreases and hence E
decreases. Clearly, more work is required to be
35. If the potential difference across a capacitor done in the first case.
is doubled, what happens to : (a) the charge 38. The battery remains connected to a parallel
on the capacitor and (ft) the energy stored in plate capacitor and a dielectric slab is inserted
the capacitor between the plates. What will be the effect on
ELECTROSTATIC POTENTIAL AND CAPACITANCE 2/81

its (i) capacity (ii) charge, (in) potential chaises Q and - Q. Show that this energy can
difference (iv) electric field, (v) energy stored ? be expressed in terms of electric field as
Justify your answer. (CBSE 2009) 1 .
Ans. When battery remains connected, pot. diff. V — £ Ad, where A is area of each plate
remains constant. Capacity C increases. Q=CV
and d is the separation between the plates.
How will the energy stored in a fully charged
increases. Electric field, E = will
K Kd capacitor change when the separation
1 between the plates is doubled and the
decrease. Energy stored = -CV2
- i
increases. dielectric medium of constant 4 is introduced
between the plates ?
39. In the above question, if battery is removed
Ans. For energy stored in the capacitor, refer to Art.
after charging the condenser & dielectric slab 2.21.
introduced, how are all the five parameters
affected ? (CBSE 2010, 2009) g- _ (CK)^ 1

w
U = = -CV2
Ans. When battery is removed, charge g remains 2C 2C 2
constant, capacity C = KCq increases. Potential
6« A

Flo
V 0
Now, C = and
diff. y = QIC decreases. Electric field ^ = ~r d d
d

ee
decreases. Energy stored = —
1^ decreases. U =-
I e.0 A
(£i/)- = -6n E'^Ad

Fr
2 C 2 d 2 ^
40. Where does the loss of energy in the above When separation between the plates is doubled
question go ? rf' = 2 d, and

for
ur
Ans. In the process, dielectric slab is polarised. The dielectric medium of constant 4 is introduced,
energy lost is spent in polarising the dielectric.
lc = 2C
41. The graph in Fig. 2(Q) shows the variation new capacity, ^ ~ 2
ks
of total energy V stored in the capacitor
Yo
As charge g remains unchanged.
oo

against the value of the capacitance C itself.


Which of the two — the charge on capacitor
I 1^
B

t/' =
or potential used to charge is kept constant 2C 2(2C) 2
re

for this graph ?


VI. Dielectric Polarization
ou

and Van de>graff generator


ad
Y

43. An air capacitor is given a charge of 2 p,C


raising its potential to 200 V. If on inserting a
nd

dielectric medium, its potential fails to 50 V,


Re

what is the dielectric constant of the medium ?


Fi

Ans. As dielectric constant in terms of potential is

given by K =
Ans. It is known that energy stored in capacitor, ^.n 50
g2 1 44. What is the effect of temperature on dielectric
V = = -CV2 constant ?
2C 2
Ans. Dielectric constant varies inversely with
If V were constant, U ^ C. The graph between temperature, i.e., dielectric constant decreases
U and C should be a straight line, which is not. with irse in temperature.
1 45. What are dielectric substances ? Which of
If g is constant, U OC

the following is a dielectric: Silicon, mica,


c'
carbon ?
which is the kind of graph given.
Ans. Dielectric substances are basically insulators.
42. Derive an expression for energy stored in a However, electric effects can be passed through
parallel plate capacitor of capacitance C with them without actual conduction. Mica is a
air as medium between the plates having dielectric.
2/82 A Fundamental Physics (XII) PiWfWi

46. Why does a charged glass rod attract a piece in attraction of the piece of paper towards the
of paper ? glass rod.
Ans. Paper is a dielectric. When a positively charged 47. If dielectric strength of air (minimum Held
glass rod is brought near it, atoms of paper get required for ionisation of a medium) is
polarised, with centre of negative charge of 3 M V/m, can a metal sphere of radius
atoms coming closer to glass rod. Therefore, 1 cm hold a charge of 1 coulomb ? ,
force of attraction (F^) between glass rod and Ans. Here, ^ = 1 C and r = 1 cm = 10“^ m.
piece of paper becomes greater than the force Electric field intensity at the surface of the
of repulsion (F^ between the glass rod and 9x10^ xl
the piece of paper. Fig. 2(Q).36. This results sphere, E = ^ ^ ^ ^
4jceQ -p (10
= 9x 10‘3v/m
As this value of E is much greater than the

ww
dielectric strength of air (3 MV/m = 3 x iO^
V/m), therefore, air around the sphere will get
ionised and the entire charge from the sphere

Flo
will leak out into air. Hence the sphere of radius
1 cm cannot hold 1 coulomb of charge in air.

e
rree
r FF
SHORT ANSWER QUESTIONS Carrying 3 marks

1. Explain the physical meaning of potential and 7. Explain electrostatic shielding with examples.
uurr
potential difference. Find a relation for
electrostatic potential at a point due to a point
for [Art. 2.13]
8. Show that the electric field at the surface of a
kss
charge. [Arts. 2J, 2.5]
2. What do you understand by potential gradient ? charged conductor is E-~n, where o is the
ooook
Yo

Establish a relation between electric field and ^0


eBB

potential gradient. A

surface charge density and n is a unit vector


(AP Board 2012) [Art. 2.9]
normal to the surface in the outward direction.
3. Establish that electrostatic forces are conser-
rr

[Art. 2.12 (5)]


vative. Give two examples of conservative
ou
ad

forces. [Art. 2.4] 9. What is a surface density of charge ? Show that


surface density of charge is different at different
Y

4. What do you understand by electrostatic


dY

potential energy ? Find an expression for points of an irregular shaped conductor.


electrostatic potential energy of a system of two [Art. 2.12 (6)]
Re
innd

point charges. [Arts. 2.2 and 2.10] 10. What is a capacitor ? Define its capacitance.
Fi

5. Depict the equipotential surfaces for Explain the units of capacitance.


(i) a single isolated charge (Kerala Board 2012) [Art. 2.14]
(CBSE 2012) [Fig. 2.10(a)] 11. Define capacitance of a conductor. Obtain an
(/i) a uniform electric field. [Fig. 2.10(b)] expression for capacity ol an isolated spherical
conductor.
(Hi) an electric dipole.
(CBSE Sample Paper 2020, CBSE 2017) (Chhatisgarh Board 2012)
[Fig. 2.10(c)] [Arts. 2.14 & 2.15]
(iv) a system of two identical point charges 12. Explain the principle of a capacitor ?
placed distance d apart. [Art. 2.16]
(CBSE Sample Paper 2020, CBSE 2010) 13. Derive an expression for equivalent capacitance
[Fig. 2.10(d)] of three capacitors of capacitances Cj, C2 and C3
6. Explain the term electric field intensity. Establish when connected (i) in series (Hi) in parallel.
that electric field inside a charged conductor is (Manipur Board 2012, 2011)
zero. [Art 2.12] [Art. 2.20]
ELECTROSTATIC POTENTIAL AND CAPACITANCE 2/83

LONG ANSWER QUESTIONS Carrying 5 or more marks

1. Define electrostatic potential at a point. Write its (b) Find the total energy stored in the parallel
SI. unit. (CBSE Sample Paper 2019-20) combination
[Art. 2.3] (c) Explain the reason for the difference in energy.
2. Derive an expression for potential at a point [CBSE 2018 (C)] [Arts 2.21 and 2.25]
(0 due to point charge (ManipurBoard 2011) 8. Explain dielectric polarization, electric
[Art. 2.5] susceptibility and dielectric constant. Establish a
(if) due to a group of charges [Art. 2.6] relation between them. [Art, 2.27]
(in) due to an electric dipole. 9. Find the expression for capacitance of a parallel
(CBSE 2019, 2017, Uttarakhand Board 2012, plate capacitor when (/) a thin dielectric slab is
Jharkhand Board 2011, HP Board 2011) inserted in between the plates. (//) a thin

ww
[Art. 2.7] conducting slab is inserted in between the plates.
3. Explain the concept of electric potential energy. (in) a dielectric slab fills the entire space between
Derive an expression for potential energy of a the plates. (CBSE (F) 2017)

Flo
system of two point charges. Generalise the [Art. 2.30 and 2.29]
expression for N discrete charges. 10. Describe briefly the process of transferring the

e
[Arts. 2.2 and 2.10] charge between the two plates of a parallel plate

rree
4. Derive an expression for the electric potential at capacitor when connected to a battery. Derive an

r FF
any point F at a distance r from the centre of an expression for the energy stored in a capacitor.
electric dipole, making angle a with its axis. (CBSE, 2019) [Art. 2.21]
uurr
[CBSE 2017 (C)j [Art. 2.7]
5. Explain the underlying principle of working of a
for
11. Explain the principle on which Van-de-Graaff
generator operates. Draw a labelled schematic
kss
parallel plate capacitor. sketch and write briefly its working.
ooook

If two similar plates, each of area A having surface (Raj. Board 2012, Bihar Board 2011,
Yo

charge densities -i- c and - a are separated by a Hr. Board 2011, CBSE 2008) [Art. 2.33]
eBB

distance d in air, write expressions for 12. What is meant by dielectric constant, and polar
(i) the electric field at points between the two and nonpolar dielectrics ? Explain with illustrations.
plates,
(Kerala Board 2012) [Art. 2.26]
rr

(i7) the potential difference between the plates.


ou
ad

13. What do you understand by energy density of a


(in) the capacitance of the capacitor so formed. parallel plate capacitor ? Derive a relation for it.
Y

(CBSE 2016, 2014, HP Board 2013,


dY

[Art. 2.23]
Raj. Board 2011) [Art 2.17]
14. Explain the terms with illustrations
Re
innd

6. Derive an expression for the energy stored in a


(0 Dielectric strength
charged parallel plate capacitor. What is the form
Fi

of this energy and where from it comes ? Find the (ii) action of sharp points. [Art. 2.28 and 2.32]
expression for energy density of a charged 15. What is a lightning conductor ? Explain its
capacitor. working. [Art. 2.32]
(CBSE 2014, 2012, 2008 ; Manipur Board 2012 : 16. Three capacitors of capacitances Cj, C2 and C3
Pb. Board 2011) are connected (/) in series, (ii) in parallel. Show
[Arts. 2.21 and 2.23] that the energy stored in the series combination is
7. A capacitor of capacitance C| is charged to a the same as that in parallel combination.
potential Vj, while another capacitor of [Art. 2.22]
capacitance C2 is charged to a potential difference 17. When two charged condensers having different
V2. The capacitors are now disconnected from their
capacities and different potentials are joined
respective charging batteries and connected in
parallel to eachother.
together, show that there is always some loss of
energy. What is the form of lost energy ?
(a) Find the total energy stored in the two
(Uttarakhand Board 2012) [Art. 2.25]
capacitors before they are connected.
2/84 ^ Fundamental Physics (XII)

CASE-BASED VERY SHORT/SHORT QUESTIONS

CASE 1. Electrostatic potential difference 1


between two points B and A in an electrostatic field {d) Electric potential is scalar and V OC

r
= amountof work done in carrying unit positive charge
3. What is SI unit of electric potential ?
from A to B (against the electrostaticforce of the field)
along any path between the two points, i.e., 4. What is electrostatic P.E. of a system of two point
charges and ^2 separated by distance r ? Name
W
AB
B
any two units of P.E. How are they related ?
V -V = E.dl
% A CASE 2. The resultant potential at any point due
to total charge configuration is
If A is at infinity, then =0

ww
1 (qI ^
V
B
Wc-oB
= -J E.dl
B
e
V =
4lZ 6 I ^
0 1=1 ^ I;
%
Electrostatic potential at a point due to short

Flo
Electrostatic potential is a scalar. It is measured electric dipole of dipole moment p, at a distance r is

ee
in volt.
At any point distant r from a point charge q. p COS0

rere
V =

rFF
q 47100
electric potential V = 4n r
0
When point is on the dipole axis, 0 = 0° or 180°
uur r
Electric field intensity at any point is negadve rate
of change of potential with distance at that point.
ffoor
sks
47teor"
Yoo
-dV
\E\ =
oko

i.e..
dr
When point is on equatorial plane, O = ■“
BBo

Electrostatic potential energy of a system of two


point charges q^ and ^2 distance r apart is.
re

K
cos 0 - cos — = 0 v=o
9i 2
ouur

U =
ad

4tc r Note that due to a point charge q,


0
YY

SI unit of U is joule. Another unit of energy is


electron volt (eV), where 4n
0
r
nndd
Re

1 eV= l-6x 10"*9j


1
Read the above passage carefully and answer V «
FFii

r
the following very short questions and short
questions. 1
and due to an electric dipole, V OC

1. At any point distant r from a point charge q, r2


electric potential is given by An equipotential surface is that surface, at every
2. Choose the correct statement out of the point of which electric potential is the same. Therefore
following: no work is done in moving a test charge from one point
1 of equipotential surface to the other. Equipotential
(a) Electric potential is scalar and V OC
surfaces of a single point charge are concentric spherical
r2
surfaces centred at the point charge.
I
{b) Electric potential is vector and V <x For a uniform electric field, say along X-axis,
r
equipotential surfaces are planes normal to X-axis and
1 parallel to YZ plane, as shown in Fig. 2(Q).37(a) and
(c) Electric potential is vector and V « 2(Q).37(6).
r2
ELECTROSTATIC POTENTIAL AND CAPACITANCE 2/85

Electrostatic RE. of a system of two point charges


and q2 separated by a distance r^2
U = X
^2
^0 ’]2
For a system of N point charges at distances,
/j, .... respectively, electrostatic RE. is given by

1
U =
4k e

oww
0 all ''jk
pairs

Common unit of electrostatic potential energy is


electron volt (eV)
1 eV = 1 -6 X 10 joule

e
FFrlo
1 keV= 10^ eV= 1-6 x 10“'^ J

re
1 MeV = 10^ eV = 1-6 X 10"’^ J and so on.

ree
F
E Some important results regarding electrostatics of
conductors are :

rF
(i) In electrostatic equilibrium, electric field £ = 0

fsoor
ouur
everywhere inside the conductor.
(//) Just outside a charged conductor electric field is
skf
Read the above passage carefully and answer perpendicular to the surface of the conductor.
the following very short questions and short
ooko
{in) Net charge in the interior of a conductor is zero,
Yo
questions.
Y

(/v) Electrostatic potential = constant throughout the


Bo

5. Distance from a point charge is doubled. How is volume of the conductor, and has the same value
reB

potential due to charge affected ? as on its surface.


6. Distance from the centre of an electric dipole is Read the above passage carefully and answer
doubled. How is potential due to the dipole
uur
ooY

the following very short questions and short


ad

affected ?
questions.
7. What is an equipotential surface ? How much
dY

9. For a system of two point charges , <?2 separated


work is done in carrying a test charge from one by a distance rj2, what is RE. ?
point to the other on such a surface ?
innd

10. Distinguish between one milli electron volt and


Re

8. Draw equipotential surfaces due to uniform one million electron volt.


FFi

electric field along .Y-axis. II. What is the general expression for potential
energy of a system of N point charges. What is
CASE 3. Electrostatic potential energy of a the common unit used ?
system of pointchargesis the total amount of work done
12. What is the net charge in the interior of a charged
in bringing the various charges to their respective conductor ? Is the same true for electric
positions from infinitely large separations, without potential ?
acceleration.

ANSWERS

1. V = ~^ 3. SI unit of electric potential is volt.


4n €«0 r 4. Electrostatic potential energy in free space is
I
2. Electric potential is scalar, and V ec ^ U =
r
4tu€o r
2/86 ^'utdeefr 4. Fundamental Physics (XII)

Two units of electrostatic P.E. are electron 10. 1 milli electron volt = 10"^ eV
volt (eV) and joule. = 10-3x1-6x10-*^J
1 eV= l-6x 10-^® J = i-6x 10-22 j
5. When distance from a point charge is doubled.
I million electron volt = 10® eV
1
potential reduces to half, as y «: = 10® X 1-6 X 10-19 J
r

:= 1-6 X 10-12 J
1
6. Due to an electric dipole. V OC ■'

r2
when r is doubled, V becomes one fourth. 11. 6/ = X
all '-Jk
7. Equipoieniial surface is that surface at every point pairs

of which electric potential is the same.

w
Common unit for electrostatic P.E. is electron
W = charge x potential difference
volt (eV).
IV = charge x zero = zero.

Flo
12. Net charge in the interior of a charged conductor
8. Due to uniform electric field along X-axis, equi- is zero.

e
potential surfaces are planes normal to X-axis and

ree
It is not true for electric potential.
parallel to YZ plane as shown in Fig. 2(Q).37.

FFr
Electrostatic potential is constant throughout the
9. P.E. =
^1^2 volume of the conductor = Potential on its
urr
4tC6o ^,2 surface.

for
kkss
CASE-BASED MCQs AND ASSERTION-REASON QUESTIONS
Yo
ooo

CASE 1. Electrical capacitance of a conductor is For (^estion No. 3 and 4, we have gjven two
related to its ability to store electric charge or electric statements each, one labelled as Assertion (A)
B

energy. A charge Q given to a conductor raises its and other labelled as Reason (R).
potential by V. it is found that QocVotQ = CV, where
re

Choose the correct option out of the four options


C is a constant of proportionality and is called electrical given below:
capacitance of the conductor.
ou

(rr) Both, A and R are true and R is correct


ad

SI unit of capacitance is farad. explanation of A.


YY

Q 1 coulomb {b) Both, A and R are true, but R is not a correct


From C = —. 1 farad = explanation of A.
ndd

V 1 volt
Re

(c) A is true, but R is false.


We can show that capacity of an isolated spherical
Fi

conductor of radius r is (d) A is false, and R is also false.


C = 4 71 €q r 3. Assertion. Capacitance is a positive quantity.
When r is in metre, capacity is in farad. Larger Q
Reason. C = —.
the radius of the sphere, greater is its capacity. y
Based on the above paragraph, answer when Q is positive, V is positive ; and vice-versa.
questions no. 1 to 4 :
4. Assertion. Capacity of earth = 711 pF.
1. The SI unit of capacitance is
Reason. Taking earth as a perfect sphere of radius
(fl) coulomb (_b) volt /? = 6-4 X 10® m.
(c) farad (d) none of these
2. Capacity of an isolated spherical conductor of CASE 2. A capacitor is an arrangement for storing
radius R is a large amount of electric charge and hence electric
(fl) 4tc €q R (6) 1/4tc eg /? energy in a small space. The capacitance of an isolated
(c) Q/V id)V/Q conductor is increased considerably by bringing near it
ELECTROSTATIC POTENTIAL AND CAPACITANCE 2/87

an uncharged conductor connected to earth. This is the A


(b) C = ^0
0
principle of capacitor. Such an arrangement of two (a) C =
A
conductors separated by a dielectric medium is said to
6^0 A d
form a capacitor or condenser Depending on the shape (c) C = id)C = 0
d A
of conductors, we get parallel plate capacitor,
spherical capacitor and cylindrical capacitor. Capacity 6. Energy stored in a charged capacitor is
1
A (fl) U = {b) U = -QV
of a parallel plate capacitor is ^ = 0
, where A is 2
d

area of charged plate and d is the distance between the (c) V = Q!V {d)U =
QV'^
C
two plates of capacitor.

ww
For Question No. 7 and 8, we have given two
When plates of a capacitor are separated by a statements each, one labelled as Assertion (A)
dielectric medium of relative permittivity g ^ or AT, then and other labelled as Reason (R).
Choose the correct option out of die four options

Flo
C
given below:

e
m
C„ = KC0 or AT =

eree
C. (a) Both, A and R are true and R is correct
0

explanation of A.

FFr
Energy stored in a capacitor is (b) Both, A and R are true, but R is not a correct
uurr1 explanation of A.
1

orr
u = -QV = -cv2
- =
(c) A is true, but R is false.
2 ^ 2 2C

When condensers are joined in parallel


sfo
(d) A is false, and R is also false.
7, Assertion. Mega Farad is too big a unit of
kks
Yoo
capacity than farad.
oooo

When condensers are joined in series. Reason. Capacity of condensers increases in


eBB

series.
1 1 1 1
C _ “ C1 8. Assertion. For storing larger amount of energy,
<=2 C, condensers must be connected in parallel.
urr

Based on the above paragraph, answer


ad

1
questions no. 5 to 8 : Reason. Energy stored = — CV^
Yo

oc
C,
dY

2
5. With usual notation, capacity of a parallel plate
capacitor is given by and capacity in parallel increases.
innd
Re

ANSWERS
Fi

l.{c) 2. (a) 3. (a) 4. (a) 5.(c) 6.(b) 7.(c) S.(a)

HINTS/EXPLANATIONS For Difficult Questions

1. farad is the SI unit of capacitance. = 711 X icr^F


2. C = 4ti GqR = 711 pF.
3. Both, the Assertion and Reason are true, and 5. C=^nA/d
0
Reason is correct explanation of Assertion.
1
4. Both, the Assertion and Reason are true and 6. U = -QxV
Reason is correct explanation of Assertion.
Capacity of earth = 4 tr g q R 7. Here Assertion is true but Reason is false as
capacity of condensers decreases in series.
6-4 xlO^
8. Both, Assertion and Reason are true and
9x10^
Reason is the correct explanation of Assertion.
2/88 “pnaxU^ '<a. Fundamental Physics (XII)

9. The electric field at a point due to a point charge


TYPE I. POTENTIAL DIFFERENCE
is 30 N/C, and the electric potential at that point
AND WORK DONE
is 15 J/C. Calculate the distance of the point from
1. Two charges - q and + q are located at points the charge and the magnitude of the charge
A (0. 0, - a) and B {0, 0, + a) respectively. How [Ans. 0-5 m ; 0-83 x C]
much work is done in moving a test charge from 10. A point charge of 10“^ C is situated at the origin
of co-ordinates. Find the potential difference
point P (7, 0, 0) to Q (- 3. 0. 0) ?
between the points A (4, 4, 2) and B (1, 2, 2).
(CBSE2009) [Ans. Zero]
[Ans. -15 V]
2. If 100 J of work must be done to move electric
11. Twenty seven charged water droplets each with a

w
charge equal to 4 C from a place where potential diameterof 2 mm and a charge of 10“*^C coalesce
is -10 V to another place where potential is V volt, to form a single drop. Calculate the potential of
find the value of V. [Ans. 15 V]
the bigger drop. (Pb. Board 2001)

Flo
3. If 10 J of work is to be done in moving a charge of [Ans. 81 V]

e
-200 C from A to B, which of the two points is at
12. A charge of 20 p.C produces an electric field. Two

reee
higher potential ? What is the potential difference ? points are 10 cm and 5 cm from this charge. Find

FFr
[Ans.A ;-0-05 V] the values of potentials at these points and
calculate work done to take an electron from one
TYPE II. ELECTRIC POTENTIAL point to the other.

for
ur
[Ans. 1-8 X 10^ V, 3-6 x 10^ V, 2-88 x IQ-^^ J]
4. The electric field at a point due to a point charge
13. Calculate the voltage required to balance an oil
kss
is 20 NC”^ and electric potential at that point is
drop carrying 10 electrons, when located between
10 JC“^ Calculate the distance of the point from
Yo
plates of a capacitor, which are 5 mm apart. Given
oo

the charge and the magnitude of the charge. mass of drop = 3 x 10"*® kg, charge on electron
(CBSE 2006) = 1-6 X 10"^^ C and g = 9-8 m/s^. [Ans. 9*2 V]
B

[Ans. 0-5 ni ; 0*55 x ir’ C]


re

5. Two point charges q and - 2 ^ are kept 'd' distance TYPE III. POTENTIAL GRADIENT
apart. Find the location of the point relative to AND ELECTRIC FIELD
ou
ad

charge q at which potential due to the system of


14. An infinite plane sheet of charge density
YY

charges is zero. (CBSE 2014) 10“^ Cm”^ is held in air. In this situation how far
[Ans. d/3 from charge q] apart are two equipotential surfaces, whose p.d is
nd
Re

6. A charge of 24 jiC is given to a hollow metallic 5V? [Ans. 8*85 mm]


sphere of radius 0-2 m. Find the potential 15. An electric field of 20 N/C exists along the X-
Fi

(0 at the surface of sphere axis in space. Calculate the potential difference


(ii) at a distance of 0-1 cm from the centre of (Vg - where the co-ordinates of A and B are
sphere. [Ans. 1-08 X 10^ V, 1-08 x 10^ V] given by
7. Two point charges, one of + 100 pC and another (/) A (0,0); B (4 m, 2 m)
of - 400 [iC, are kept 30 cm apart. Find the point (ii) A (4 m, 2 m); S (6 m, 5 m)
of zero potential on the line joining the two [Ans. - 80 V ; - 40 V)
charges. [Ans. 6 cm from 100 pC charge] 16. What is potential gradient at a distance of
8. Two point charges 4\iC and - 2 (i C are separated 10”*^ m from the centre of the platinum nucleus ?
by a distance of 1 m in air. Calculate at what point What is the potential gradient at the surface of the
on the line joining the two charges is the electric nucleus ? Atomic number of platinum is 78 and
-15
potential zero ? (CBSE 2007) radius of platinum nucleus is 5 x 10 m.

[Ans. 1-123 X 10^’ ; 4-5 x lO^’ Vm’*]


2
— m from 4[iC 17. If the potential in the region of space around the
point (- Im, 2m, 3m) is given by
ELECTROSTATIC POTENTIAL AND CAPACITANCE 2/89

V = (10 + 5 - 3 7?), calculate the three to 0 084 C cm Find the initial charge and
components of electric field at this point. capacitance of the spherical conductor.
[Ans. = 20 V nT^; = - 20 V m"*; [Ans. 22 C ; 5-56 x F]
18 V nr*]
TYPE VI. PARALLEL PLATE CAPACITOR
TYPE IV. ELECTROSTATIC
POTENTIAL ENERGY
26. A parallel plate air capacitor consists of two
circular plates of diameter 8 cm. At what distance
18. Two charges of magnitude 5 nC and - 2 nC are should the plates be held so as to have the same
placed at points (2 cm. 0, 0) and {x cm, 0, 0) in a capacitance as that of a sphere of diameter
20 cm ? [Ans. 4 mm]
region of space. Where there is no other external
field. If the electrostatic potential energy of the 27. Calculate the area of paper required to construct
system is - 0-5 pJ. What is the value of x ? a parallel plate capacitor of 0 (X)4 p. F, if the

ww
(CBSE2008) [Ans.x = 4cmJ dielectric constant of paper be 2-5 and its
thickness 0 025 mm. [Ans. 4*52 x 10"^ m^]
19. Two identical particles, each having a charge of
28. What distance should the two plates each of area

Floo
2 X 10“^ C and mass of 10 g, are kept at a
separation of 10 cm and then released. What would
0-2 X 0-1 m^ of an air capacitor be placed in order
to have the same capacitance as a spherical
be the speeds of particles when the separation

ee
conductor of radius 0-5 m ? [Ans. 3*18 mm]
becomes large ? [Ans. 6(K) ms“*]

reer
29. The plates of a parallel plate capacitor are 5 mm

rFF
20. Two point charges A and B of value -t- 15 p C and
+ 9 p C are kept 18 cm apart in air. Calculate the apart and 2 m^ in area. The plates are in vacuum.
work done when charge B is moved by 3 cm A potential difference of 1000 volt is applied
uur r
across the capacitor. Calculate {/), the capaci
towards A. [Ans. 1*35 J]
21. Set up an arrangement of three point charges:
ffoor
tance (n) the charge on each plate and (»7)electric
intensity in the space between the two plates.
sks
+ q, + 2 q and x q separated by equal finite
[Ans. 3-54 x ir^ F ; 3-54 x C;
YYoo
distances so that electric potential energy of the
ooko

system is zero. What is x ?


2x105 NCr'l
eBB

TYPE VII. SPHERICAL AND


Iq
at vertices
[Ans. Charges ●¥ q,-¥lq and —^ CYLINDRICAL CAPACITORS
r

of an equilateral triangle ; x = - 2/31 30. A sphere of radius 0 03 m is suspended within a


ouur
ad

22. Three charges - q, Q and - q are placed at equal hollow sphere of radius 0*05 m. If the inner
YY

distances on a straight line. If the potential energy sphere is charged to a potential of 1500 volt and
of the system of three charges is zero, then what outer sphere is earthed. Find the capacitance and
is the ratio of Q: q 7 [Ans. 1 :4]
nndd

the charge of the inner sphere.


Re

[Ans. 8-33 pF, 1*25 x lO"^ C]


TYPEV. CAPACITANCE
FFii

31. A co-axial cable used in transmission line has


inner radius of 0-1 mm and outer radius of
23. N drops of mercury of equal radii and possessing
0-6 mm. Calculate capacitance per metre of the
equal charges combine to form a big drop. cable. [Ans. 71'5 pF/m]
Compare the charge, capacitance and potential
of bigger drop with the corresponding quantities 32. A spherical capacitor has an outer sphere of
(Pb. Board 2001) radius 015 m and the inner sphere of diameter
of individual drops.
0-2 m. The outer sphere is earthed and the inner
[Ans. N;
sphere is given a charge of 6 p C. The space
24. When electrons equal to Avogadro number are between the concentric spheres is filled with a
transferred from one conductor to another, a material of dielectric constant 6. Calculate
potential difference of 10^ V appears between capacitance and potential of inner sphere.
them. Calculate the capacity of the system of two [Ans, 0-2xlO"’F;18xlO‘*VJ
conductors. [Ans. 9-637 x IQ-^ F] 33. A cable consisting of a wire 3 mm in diameter is
25. A charged spherical conductor has a surface insulated with 3 mm thick dielectric of relative
density of 0-07 C cm”^. When the charge is permittivity 10. Calculate the capacitance of 1
increased by 4-4 C, the surface density changes km length of the cable. [Ans. 0-506 |lF]
2/90
“Pnadeeft. 4. Fundamental Physics (XII) IWl

TYPE VIII. GROUPING OF CAPACITORS 40. There are infinite number of capacitors each of
capacitance 1 pF. They are connected in rows,
34. Two capacitors of capacitances 3 pF and 6 pF are such that number of capacitors in first, second,
charged to potentials of 2 V and 5 V respectively. third rows are respectively 1, 2, 4, 8 . The
These two charged capacitors are connected in rows of these capacitors are connected between
series. Find the potential across each of the A and B, as shown in Fig. 2(Q).40. Determine
capacitors now. (CBSE Sample Paper 2004) the equivalent capacitance of the network
[Ans. 12 V ; 6 V] between points A and B.
35. If Cj = 20 pF, Cj = 30 pF and C3 = 15 pF and the
insulated plate of Cj be at a potential of 90 V, one
FIGURE 2(Q).4Q
plate of C3 being earthed : What is the p.d between
the plates of C2, three capacitors being connected
m senes.
(CBSE 2015) [Ans. 20 V]

w
36. For the network shown in Fig.2(Q).38, find the 1pF 1pF
potential difference between points A and B, and
that between B and C in the steady state.

Flo
1pF 1pF

FIGURE 2(Q).38

reeee
^^xF IpF
3pF B 1pF

FFr
A B
3pF±: 4:1pF
for
ur
[Ans. 2 pF]
kkss
1pF
41. Find the equivalent capacitance of the ladder bet
Yo
oo

ween points A and B, Fig. 2(Q).41.


A AA/V AAV C
eB

20 0 100 V 100
FIGURE 2{Q).41
[Ans. = 25 V, = 75 V] 2pF 2pF 2^lF
r

A*
37. Two capacitors have a capacitance of 5 pF when
ou
ad

connected in parallel and 1 -2 pF when connected


^IHF d^^\lF ^l^iF
YY

in series. Calculate their capacitances.


[Ans. 2 pF, 3 pF] B
ndd
Re

38. Connect three capacitors of 3 pF, 3 pF and 6 pF


such that their equivalent capacitance is 5 pF.
Fi

[Ans. Series combination of 3 pF (Ans. 2 pF]


and 6 pF in parallel with 3 pF] 42. Find out the potential difference across the plates
39. Find the equivalent capacitance between the of 1 p F capacitor in Fig. 2(Q).42.
points P and Q as shown in Fig. 2(Q).39. Given
C=18pFand Cj = 12pF.
FIGURE 2(Q).42
6V
FIGURE 2(Q).39

p C C c 3pF
D A F
4pF
C C
-i-C 6pF
Cl ciT c 12pF
1pF 2pF
Q E B G
C C C

[Ans. 11 pF] [Ans. 3-82 V]


ELECTROSTATIC POTENTIAL AND CAPACITANCE 2/91

43. Find the p.d. between points A and B of arrange 48. What is the capacitance of arrangement of 4
ment shown in Fig. 2(Q).43. plates of area A at distance d in air in Fig. 2(Q).47.

FIGURE 2(Q).47

I
E
A II

III B

[Ans. 3 e 0 A/d]
[Ans. - 8 V] 49. A parallel plate capacitor is filled with dielectrics
as shown in Fig. 2(Q).48. What is its capacitance ?

ww
44. The equivalent capacitance of the combination
between A and 5 in the given Fig. 2(Q).44 is 4 |i F.
FIGURE 2(Q).44

Flo
A B
o- o

ee
C
20pF

rere
(j) Calculate capacitance of capacitor C.

rFF
(ii) Calculate charge on each capacitor if a 12 V
battery is connected across A and B.
uur r
(in) What will be the potential drop across each
capacitor ?
[Ans. 5 p. F ; 48 ^ C ; 2-4 V, 9-6 V]
(CBSE 2009) foor
sks
45. The capacities of three capacitors are in the ratio
Yoo
2Srt0 A
ook

of 1 : 2 : 3. Their equivalent capacity in parallel [Ans.


diK^+K^)
is greater than their equivalent capacity in series
BBo

by 60/11 pF. Calculate their individual 50. Calculate the charges which will flow in sections
re

capacitances. [Ans. 1 pF, 2 pF, 3 pFJ


1 and 2 in Fig. 2(Q).49, when key K is pressed.
46. A combination of four identical capacitors is
ouur

shown in Fig. 2(Q).45. If resultant capacitance


ad

FIGURE 2(Q).49
of the combination between the points P and Q
YY

E
is 1 p F, calculate capacitance of each capacitor.
C2
nndd

FIGURE 2(Q).45
Re

,.2
P
FFii

c C C C Ci
1

[Ans. 4 p F E
47. What is the capacitance of arrangement of 4 plates
each of area A at a distance d in air in Fig. 2(Q)-46.
ECjCj
FIGURE 2(Q).46 Ans. ECj;
(C,4-C2>_
.-rMd 51. In the circuit shown in Fig. 2(Q).50, the emf of
A
r .
> B
each battery is £ = 12 volt and the capacitances
● Y'i - .
II- are Cj = 2-0 p F and C2 = 3-0 p F. Find the charges
which flow along the paths 1,2, 3 when key K is
[Ans. 2 Gq A/d] pressed.
2/92
Fundamental Physics (XII) VOL.I

FIGURE 2fQ).50
55. If Cj = 3 pF and C2 = 2 pF, calculate the equi
Cl C2
valent capacitance of the network shown in Fig.
2(Q).54 between points A and B.

FIGURE 2(Q).S4

1 Ci Cl
A

^ C2
I h
Ei E2
B

w
[Ans. 24jiC;-36jiC;I2jiC]
Ci Cl
52. Calculate the equivalent capacitance between the
points A and B in the combination shown in Fig.

Flo
2(Q).51. [Ans. 1 pF]
56. Find the equivalent capacitance of the combi

reeee
FIGURE 2(Q).S1
nation of capacitors between the points
,9UF 8pF A and B as shown in Fig. 2(Q).55. Also calculate

Fr
A the total chaige that flows in the circuit, when a
100 V battery is connected between the points A
and B.

for
ur
FIGURE 2(Q).S5
kss
B
Ci=40hF C2 = WhF
Yo
10pF
oo

[Ans. 13-44)1 FI C3 60mF


eB

53. Calculate the equivalent capacitance between the


points A and B in the combination shown in Fig.
r

2(Q).52. C5
ou
ad

= 10nF C6 = 10mF = 60nF


FIGURE 2(Q).S2
YY

B
nd
Re

A^ F G
B [Ans. 20 )l F ; 2 X 10^ C]
Ci=5pF C2=10pF C3=15pF C4=30)iF 57. Two capacitors C| and C2 are connected to a
Fi

batteiy of 6 V as shown in Fig. 2(Q).56. Find the


charge on each capacitor.
[Ans. 15 )i F]
54. The outer cylinders FIGURE 2(01.56
of two cylindrical FIGURE 2(01.53
Ci=4^F C2 = 6(jF
capacitors of capa
citance 2-2 ]i F each, ■
-n
; fr

are kept in contact


and the inner
■. ,'j' ■ V
cylinders are
6V
connected through a
1
wire. A battery of
e.m.f. 10 V is
connected as shown in Fig. 2(Q).53. Find the total
charge supplied by the battery to the inner
cylinders. [Ans. 44 )i C] [Ans. 24|iC;36)iC]
ELECTROSTATIC POTENTIAL AND CAPACITANCE 2/93

62. Find the total energy stored in capacitors in the


TYPE IX. ENERGY STORED IN A
network shown in Fig. 2(Q).59.
CAPACITOR AND LOSS OF
ENERGY ON SHARING CHARGES

58. A 800 pF capacitor is charged by a 100 V battery.


After sometime, the battery is disconnected. The
capacitor is then connected to another 800 pF
capacitor. What is the electrostatic energy stored ?
(CBSE 2009)
[Ans. 2x 10-^ J]
59. Net capacitance of three identical capacitors in

ww
(CBSE Sample Paper 2019*20)
series is I pF. What will be their net capacitance
[Ans. 3*6 X J]
in parallel ? Find the ratio of energy stored in
two configurations if they are connected to the 63. Net capacitance of three identical capacitors in

Floo
same source. (CBSE 2011)
scries is 1 p F. What will be their net capacitance
if connected in parallel ? Find the ratio of energy

ee
[Ans. 9 pF ; 1: 9] stored in the two configurations, if they are both

reer
connected to the same source. (CBSE 2011)
60. Fig.2(Q).57, shows a network of five capacitors

rFF
connected to a 100 V supply. Calculate the total [Ans. 9 p F ; 1: 9]
charge and energy .stored in the network. 64. Three identical capacitors Cj, C2 and C3 of
uur r
FIGURE 2(Q).57
ffoor
capacitance 6 p F each are connected to a 12V
battery as shown in Fig. 2(Q).60. Find
sks
3pF
YYoo
FIGURE 2{Q).60
ooko

Cl

^MpF ^2pF
eBB

3pF^
12V _=. C3
r

2pF
ouur
ad
Y

100 V C2
nddY
Re

(/) charge on each capacitor


(CBSE Sample Paper 2008)
FFiin

(//) equivalent capacitance of the network


[Ans. 4 X 10-^ C ; 0-02 J] (Hi) energy stored in the network of capacitors.
61. in Fig. 2(Q).58, the energy stored in C4 is 27 J. (CBSE 2009)
Calculate the total energy stored in the system.
[Ans. = ^2 = 36 p C, = 72 p C ; 9 p F ;
El = £2 = 108 p J, £3 = 432 p J]
FIGURE 2(Q).58
65. A 20 p F capacitor is charged by a 30 V d.c.
C2=2hF
supply and then connected across an uncharged
50 p F capacitor. Calculate (/) the final potential
Ci=1mF C3=3mF diff. across the combination (ii) initial and final
P Q R
energies. How will you account for the difference
C4=6hF
in energies ?
[Ans. 8*57 V ; 9 x J, 2-57 x ir^ J]
66. Two parallel plate capacitors X and V have the
[Ans. 594-0 J] same area of plates and same separation between
2/94 Fundamental Physics (XII)

them. X has air between the plates and Kcontains capacitor. After removing the charging battery,
a dielectric medium of e ^ = 4, Fig. 2(Q).61. an uncharged metal plate of thickness f = 0 01 m
is inserted between capacitor plates. Find the p.d
FIGURE 2{Q).61 across the capacitor. (/) before (//) after the
X Y introduction of plates (Hi) what would be the p.d
if a dielectric slab (K = 2) were introduced in
place of metal plate.
[Ans. 1500 V, 1200 V, 1350 V]
■o o-

12V 70. The two circular plates of a parallel plate


capacitor are 8 cm in diameter each and 15 mm
Calculate (i) capacitance of X and Y if equivalent apart. An ebonite plate 0-5 cm thick is introduced
between the plates. Calculate its capacity. If the

ww
capacitance of combination is 4 p. F.
plate were of copper, what would be the new
(/i) pot. diff. between the plates of X and Y.
capacity ? Take/if = 2-5.
(Hi) What is the ratio of electrostatic energy stored [Ans. 3-71 X 10-*2 F ; 4-45 x F]

FF loo
in X and Y ? (CBSE 2009)
71. When a slab of insulating material 4 mm thick is
[Ans. 5 p F, 20 p F, 9-6 V, 2-4 V ; 4 :1]

ee
introduced between the plates of a parallel plate
67. In the circuit shown in Fig. 2(Q).62, the energy capacitor, it is found that the distance between

Fe r
stored in both capacitors is I/j. If switch S is the plates has to be increased by 3-2 mm to restore
opened and a dielectric slab of constant 5 is put

Frre
the capacity to its original value. Calculate
rur
in free spaces of the capacitors, the energy stored dielectric constant of the material. [Ans. 5]
is found to be t/2. Calculate f/j/t/2- fforo
72. The area of parallel plates of an air capacitor is
0-2 m^ and the distance between them is 0-01 m.
ks
FIGURE 2(Q).62
The potential difference between the plates is
YYouo
okso

3000 V. When a 0-01 m thick sheet of an


insulating material is placed between the plates,
BBoo

c
the potential difference decreases to 1000 volt.
c;
r ee

VT
Determine (0 capacitance of capacitor before
placing the sheet (ii) charge on each plate
ad
oouur

(Hi) dielectric constant of material (iv) capa


citance after placing the insulator (v) absolute
Y

[Ans. 5/13] permittivity of the dielectric.


TYPE X. EFFECT OF INTRODUCING [Ans. 1-77 X 10-^® F ; 5-31 x C; 3;
dnYd
Re

A CONDUCTOR/DIELECTRIC SLAB 5*31 X 10"^® F ; 2'66 x 10"^^ F.m]


FFini

BETWEEN THE PLATES OF A 73. A parallel plate capacitor is to be designed with


PARALLEL PLATE CAPACITOR voltage rating of 1 kV using a material of
dielectric constant 3 and dielectric strength of
68. The two plates of a parallel plate capacitor are 4 10^ Vm"^ For safety, we like the field never to
mm apart. A slab of dielectric constant 3 and
exceed 10% of dielectric strength. What
thickness 3 mm is introduced between the plates minimum area of plate is required to have a
with the faces parallel to them. The distance capacitance of 50 pF ? [Ans. 19 cm^]
between the plates is so adjusted that the
2 TYPE XI. TYPICAL PROBLEMS
capacitance of the capacitor becomes — rd of its

original value. What is the new distance between


74. ABCD is a square of side 0-2 m. Charges of 2 x
the plates ? [CBSE 2008 (C)]
I0“® C, 4 X 10'^ C and 8 x 10“^ C are placed at
the comers A, B and C respectively. Calculate work
[Ans. 8 mm] required to transfer a charge of 2 x 10“^ C from
69. An electric field Eg = 3 x 10^ Vm"^ is established comer D to centre of the square.
between the plates 0 05 m apart, of a parallel plate [Ans. 6-27 x 10'’ J]
ELECTROSTATIC POTENTIAL AND CAPACITANCE 2/95

75. A point charge q moves from a point P to a point


S along the path PQRS in a uniform electric field
E along positive direction of x-axis. Calculate
A
work done in this process, when co-ordinates of
,PQ,R,S are {a, b, 0), (2 a. 0, 0). (a, - b, 0) and
(0, 0, 0) respectively. [Ans. - q E a]

76. The electric field outside a charged long straight


1000 B
wire is given by £ = Vm“’. and is directed
r

outwards. What is the sign of the charge on the


wire ? If two points A and B are situated such that [Ans. 2 n F]

w
= 0-2 m and = 04 m, find the value of
80. A capacitor is made of a flat plate of area A and a
(V'b-Ka)- [Ans. + charge ; ~ 693*1 V] second plate having a stair like structure as shown

Flo
in Fig. 2(Q).65. The width of each stair is a and
77. Fig. 2(Q).63 shows a parallel plate capacitor of
the height is b. Find the capacitance of the
plate area A and plate separation d. Its entire space assembly.

ee
is filled with three different dielectric slabs of

Fr
same thickness. Find the equivalent capacitance
FIGURE 2(Q).65
of the arrangement.
b

for
ur
FIGURE 2(Q).63 b
A' a
ks
H- 3a ■H
Yo
dlZ
oo

e(,A(3d^ + 6M + 2ft^)
eB

dl3 Ans. C =
3d{d+b){d-\-2b) _
dl3
81. Find the capacitance of three parallel plates, each
r

of area A and separated by d^ and d2 metre.


ou
ad

B
The inbetween spaces are filled with dielectrics
Y

of relative permittivity ej and €2- The


permittivity of free space is Sg.
nd

3 0 AK^K^K^
Re

Ans. C -
diK^K2 + K^K^ + K^K^) Ans. C =
Fi

78. A slab of material of dielectric constant K has


82. Find the equivalent capacitance between the
the same area as the plates of a parallel plate terminals A and B in the given Fig. 2(Q).66. Take
capacitor but has thickness d/2, where d is the C = 1 |XF.
separation between the plates. Find the
expression for the capacitance when the slab is FIGURE 2(Q).66
inserted between the plates. C C C
A
2K
(CBSE 2013, 2010) Ans. C
0
K + l C ri= :=C

79. Five capacitors of capacitances Cj = C5 = 1 |i F;


B
C2 = C3 = C4 = 2 p, F are connected as shown in
Fig. 2(Q).64. Calculate equivalent capacitance of
[Ans. 0*62 pF]
the system between points A and B.
2/96
^rieicUep. Fundamental Physics (XII)
● \
vVf'-'
A A
o

For Difficult Questions


1. As shown in Fig. 2(Q).67, we have an electric 5. Let the potential be zero at a point P, distant x
dipole held along Z-axis. We have to move a test from charge q as shown in Fig. 2(Q).68.
charge from P (7, 0, 0) to Q (- 3, 0, 0) on the X-
axis i.e. on equatorial line of dipole. FIGURE 2(Q).68
P
As potential on equatorial line of dipole is zero, q
I- +
-2q
H
therefore, work done
M— X —W4 (d-x) ●M
^=%(Vq-Vp) = %(0-0) M- d ■w
= Zero

w
FIGURE 2(Q).67 Now, -2q = 0
X Ane^id-x)

Flo
P (7, 0, 0) Iq

e
Atisq X Ane^id-x)

reee
or 2x=d-x or x = dl3

FFr
-q 6. (0 ^ = 24 ^iC = 24 X 10-6 c, r - 0-2 m.
■►z
A(0, 0,-a) /
O
B(0. 0, a) At the surface of the sphere.

for
ur
1
V = i 9x10^x24x10-6
47ceo r
kkss
Q (-3. 0,0) 0-2
= 1-08 X 106 V
Yo
Y
oo

2. Here, =l00J,q = AC, (») Potential at any point inside the sphere
= potential on the surface
B

V^ = -i0V, Vg=V= ?
potential at a distance of 0-1 cm from the centre
re

As
= 1*08 X 1Q6 V
I00 = 4(V+ 10); 7. Let the point of zero potential be at a distance
ou
ad

x
V = 15 volt from the charge of + 100 jiC. Then
YY

3. Here, == 10 J, ^ = -200 C
1 JOOxlO-6 400x10-6
V = = 0
nd
Re

4jie X 0-30-.X
As 0 L
^AB=^(yB-yA)
Fi

W 10
x = 0 + 06m = 6cm from 100 p,C charge.
V AB _
= -0*05V
B
-200 8. Here. <?i = 4 p C, ^2 = “ ^ p C
Let the potential be zero at a point P, at a distance
Clearly, A must be at higher potential. r [ = X from 4 p C charge. Therefore, T2 = (I - .r).
4. Here,
0
£ = = 20NC“‘
Ane^0 -p Potential at F =
'a
Q
and V = = 10JC-i ?1 _ -?2
4ti€o r
n ^2
V 10
— = r = — = 0*5 m 4 -(-2)
£ 20 X l-x

From Q^AksqKX V x=2-2x

X0-5X10 =0-5Sx 10-^C 3.r = 2,x = —m


2 from 4 p.C charge
9x10^
ELECTROSTATIC POTENTIAL AND CAPACITANCE ^7

9. Here, E= ^ = 30NC-1 .(0 V =


mgd _ mgd 3 x 10~^^ x9-8 x 5 x 10~^
47t€or2 ne 10x1-6x10"^^
= 9*2 volt
V = —^— = 15 JC-* m 14. Electric field intensity of an infinite plane sheet
4jc€or
Divide (/i) by (/), we get
of charge, E = —^
26q
1
Let Ar be the separation, between two equi-

oww
r = - =0-5m
potential surfaces having potential difference AV,
AV
PutinOO, 9x10^-^
1/2
= 15 then E=
Ar

e
a AV 2e„AV
^ = ^xl0^ =0-83xir^C Ar =
0

re
lo 26 Ar CT
0

FFrllo
10. Here, ^ = 10-8 C, J^=4t + 4j + 2k 2x8-85x10-*2x5

reF
= 8*85 X 10-3
10-8

e
so that = V4^ +4^ + 2^ = 6
uoru = 8*85 min

15. As the electric field is along X-axis, therefore,

osFr
, A A A A

rg=li + 2j + 2k ,so that from £ = - dVldr

rg=#722T? = 3. fkfor (0 dV=-£(dr) = -£(x2-Xi)


= -20(4-0)= -80 V
kso
1 1 ■ (/i) dV = - 20 (^2 - :*'i) = - 20 (6-4) = - 40 V
ooo

V -V ^
YYo
16. Here, ^ = 2e = 78 x 1 6 x C
'*”^0 L'a
BB

Potential gradient at a point is numerically equal


to electric freld at that point, i.e.,
= 9x10^x10-8 --- =-15 V
r ree
Y

L6 3j dV
=£= ^
oouu

12. Here, ^ = 20 = 20 x 10-^ C. dr


471 6q r2
ad

If A and B are two points at distances r^ = 10 cm At r=10-‘2m;


Yd

= 0-1 m and r2 = 5 cm = 0-05 m dV 9xl0’x78xl6xi0-^’


= £ =
nidn

20x10-^x9x10’ dr . (10-‘2)2
then V. = —
Re

01
4%Sq ,r = M23 X 10^7 vro-l
= 1*8 X 10^ V Atr = 5 X lO-'^m;
FFi

20xl0"^x9xl0’ dV 9 X10’X 78x1-6x10-*’


=£= ^
47C6o r2 0-05 dr
4jc6o r2 (5x10-*3)2
= 3*6xlO<^V = 4*5xl02iVm-i
17. Herejc = -lm,y = 2m,z = 3m
= (3-6 X 10^ - 1-8 X 10^) x 1-6 X 10-*’ -av
= 2*88x 10r^3j
13. Here, V= ?, n = 10, d = 5 mm = 5 x 10"3 m,
dx = ^ (10x2 + 5y2-3z2) .
= - 20x = - 20 (-1) = + 20 Vm-i
m = 3x 10-*^kg,e= l-6x 10-*’C,
g = 9-8 m/s2 -dV

To balance the oil drop dy


= ~ (10x2 + 5y2_3z2)
= _l()y = _10(2)
(V'] =
F = qE = mg or ~ mg = -20Vm-l
/
2/98 ^ Fundamental Physics (XII) P75TM1
-dV Potential energy of the system
E, =
Bz
= -^ (I0x^ + 5y^~3z^)
oz 1 q -0q) , '^q{xq') , xq{-Vq) = 0
= 6z = 6{3) = 18 Vm-l 4 Tie
qL r r

1 ^1 ^2
18. i/ = 2^2 3q^x = 0
4 Tie
0 r r

x = -2l3
-5x10-^ = 9xl0^x5xl0“^x(-2)xl0-®
22. If j: is the distance between the charges as shown
(.r-2)xl0-2
in Fig. 2(Q).70, then potential energy of the
9x109x5x10-^x-2x10-9 system of three charges is
jc-2 =
-0-5x10-^x10“2 FIGURE 2(Q).70
On solving, a; s 4 cm

w
■;9. Q

19. Here, ^ = 2 x 10^ C, m = 10 g = 10'2 kg, X

r= 10cm = 0-l m.

Flo
Let V: speed of each particle at infinite separation
k{-q)Q , kQ{-q) ^ ki-q)(-q) = 0
X X 2 a-

e
P.E. of two particles at the separation of 10 cm.
kq^ _ IkqQ

rree
= K.E. of two particles at infinite separation 2kqQ . kq^ = 0 or
2;c

r FF
X 2a: X

1 ^1^7 I 7.1 7 e_i


-■ ■ ■ ■ = —mv + — mv
? = 4g or
uurr
4TceQ r 2 2

1
for
23. Let r be the radius of each small drop carrying
charge q and potential V. When N such drops
9xl0^x2xl0"^x2xl0^
kss
V
2 _ 9,^2
4 Tie 0 combine, there is no loss of charge.
ooook

rm
0-lxl0"2
Yo

Q
= 36 X 1Q4 Charge on bigger drop - Q = Nq or — = N
eB

q
V = 600 ms”^
As volume of big drop = Volume of Vsmall drops,
20. Here, = 15 x 10“^ C,Q2 = 9x 10"^ C
rr

'●i = 18 X 10“2 m ; T2 = (18 - 3) 10“2 m 4


~kR^ = Nx — nr^
1 4 1
ou
ad

or R = r
work done = final P.E. - initial .PE. 3 3
Y
dY

As capacitance OC
radius, therefore
^1^2 1 I
w = 2xS2— = Ji5i C R
Re

47ceor2 4TE6or, 47ieo[r2


innd

C r
Fi

1Q2 10^ Nq Nq
= 9 X 10^ X 15 X 10"^ X 9 X 10-^ Potential of big drop =
15 18
47ieQ/? 47ceQV'''^r
W = 1’35 joule = X pot. of small drop
21. Let the three charges be held at the vertices A, B, 24. Here, q = Charge on 6-023 x 10^^ electrons
C of an equilateral triangle of each side = r, Fig. = 6-023 X lO^^x 1-6 X l(r‘^ C = 9-637 X 10^ C
2(Q).69.
9-637x10^
c=l = 9*637 X 10-2 F
V 10^
25. Here, 0| = 0 07 C cm 2 and <J2 = 0-084 C cm -.
Let q be the charge on the spherical conductor
of radius r cm.

q
CT = 0-07 ...(/)
1 “
4Tir2
ELECTROSTATIC POTENTIAL AND CAPACITANCE 2/99

_ ^ + 44 ^ = 6nC = 6xlO-^ C,^: = 6.


and = 0-084 ...00
^ 4icr^
Dividing (ii) by (i), we get ^b-^a
q + 4A _ 0-084 = 1-2 6 0-1x015
0-07 C = = 0*2xl0-®F
or ^ = 22C
9x10^ (015-01)
From (0,
V =
9 ^b-^a
22x7
=5 cm
\4jtx0-07 T4x22x0-07
0-15-0-1
r = 0-05 m K = 9xl0®x6xl0-®x = 18xl0^V

w
0-15 X 0-1
0-05
C = 4icGq r = 9x10^ = 5-56x10-“F
33. Here, \ = l-50mm = 15 x 10^^ m
= r

Flo
Gn A
26.
-^ = 4tiGo/? = 1-5 + 3 = 4-5 mm = 4-5 x 10"^ m

reeee
L=lkm=103m,A:=10;C = ?
gp 1C iXp-)

Fr
or
d4
= 4tcgq In^^LK
C =

^ (0-08)^ 2-3031ogjo V j-« ;


for
ur
= 4 X 10“^ m = 4 mm
16i? 16x0-10
kss
G» A
2wx8-85x10~^^x103x10
Tl. C = KC^ = K-^
Yo
4-5x10-3 '
/
oo

29. Here, d = 5 x 10"3 m, A = 2 m^, V= 1000 Volt, 2-303log


^1-5x10-3 ^
eB

gn0 A 8-85x10-^^ X 2
C = = 3 -54 X 10-^ F = 50-6 X 10-® F = 0-506 |l F
d 5x10-3
34. Here, C| = 3 jlF, C2 = 6 jiF
r

^ = CV= 3-54 X 10-® X 1000 = 3-54 X KH'C


ou
ad

V,=2V,V2 = 5V
Q ^CV Before connecting, Gi = Cj V] = 3 x 2 = 6 (aC
YY

^0 AGq AGq 02 = C2 V2 = 6x5 = 30hC


Total chaige, Q = Q^-vQ2-1-¥ 30 = 36
nd
Re

3-54x10-9x1000
= 2 X 10® NC-1 In series combination, charge is conserved.
2x8-85x10-12
Fi

30. a = 0-03 m, b = 0-05 m, V = 1500 V


Charge on each capacitor, g = 36 p.C
Capacitance of spherical capacitor is
Potential of 3 pF capacitor, = = 12V
4jiGq ab 0-03x0-05 q 3
C = = 8-33 pF
b-a "9x109(0-05 - 0-03) O 36
q^CV= 8-33 X 10-‘2 x 1500 = 1-25 x 10* C Potential of 6 pF capacitor, Vj = — = — = 6 V
Cj 3
C 2tcGq 22 8-85x10-12 35. The equivalent capacitance of the series
31. -
/
/
x=2XyX 7
0-6x10-3' combination is. Fig. 2(Q).71
logc logg 0-1x10-3 1 1 1 1 1 i 1
+
C C1 C2 C3 20 30 15
= 71-5 pF/m
0-2 3 20
c =
32. Here, =
y = 0-lm, r^ = 0-15 m 20’
2/100 ^^oAiCee^'4, Fundamental Physics (XI1)ISB191
37. Let two capacitances be C| and
FIGURE 2(Q).71
CC
\Z1
Cl C2 C3 Cj + C2 = 5 pF and = l-2pF
C.+C2
20pF 30pF ISpF On solving, we get
C, = 2 |lF, C2 = 3 jlF
38. We will connect series combination of 3 )iF and
Total potential diff. V = 90 - 0 = 90 V 6 fiF in parallel with the third capacitor of
3pF
' 20
Total chaige, q = CV = —xl0“^ x90
V ^ / = i + ^ or C5 = 2|iF

ww
^5
= 600 X 10-6 Q
Cp = 2|iF + 3pF = 5pF
Potential diff. between plates of C2, 39. Equivalent capacitance between points F and B
is

Flo
q 600x10-6
^^2 = = 20V

e
18x18
^2 ” 30x10-6 + 18 =27pF

eree
18 + 18
36. The two capacitors of 3 pF each are in parallel.

FFr
Equivalent capacitance between A and B
Their equivalent capacity = 3 + 3 = 6 pF. Again,
uurr 18x27
two capacitors of 1 pF each are also in parallel.

orr
= 12 + = 22-8 » 23 pF
Their equivalent capacity = 1 + 1 = 2 pF 18 + 27

FIGURE 2(Q).72
sfo
Equivalent capacitance between A and E
kks
Yoo
23x18
6pF B 2pF + 18 =28 pF
oooo

23 + 18

Equivalent capacitance between D and E


eBB

28x18
+ 12 =23pF
28 + 18
uurr

1pF
Equivalent capacitance between D and Q
ad
Yo

A AAAr C
dY

23x18
20 0 100
100V + 18 =28 pF
23 + 18
innd

Therefore, the given netwrok reduces to the


Re

Equivalent capacitance between P and Q is


equivalent circuit shown in Fig. 2(Q).72. In the
Fi

28x18
steady state, when all the capacitors are charged, = llllF
there is no current in the circuit. Therefore, 28 + 18

potential drop across both the resistances is zero. 40. Let C[, C2, Cj be the capacitances of first,
Potential diff. across 1 pF capacitor = p.d. second, third row .... respectively. Then
between A and C = 100 V C, = IpF
As 6 pF and 2 pF capacitors are in series, the p.d 1x1 1
of 100 V is divided between them in the inverse c,= 1 + 1
ratio of their capacitances, i.e., in the ratio 2:6 or
1:3. 1 1111^
=-+-+-+-=4

1
^3 1 1 1 1

V^B - P-d- across 6 pF = —xlOO = 25 V 1


C3 = -pF
and Vbc - P-d- across 2 pF = -xl00
- = 75V 1
Similarly = - pF and so on.
8
ELECTROSTATIC POTENTIAL AND CAPACITANCE 2/101

So equivalent capacitance ‘C ’ as they are 16


connected in parallel is P.D. across lfi¥ = — = -8V
2
4
C = Cj + C/2 +
r, 1 1 1
=1+14+ 44. From — = — +—,
20 C
2 4

= sum of geometrical progression


J L=i_± = ±-i

w
C = 5pF
C = —
1
= 2pF C Cy 20 4 20 “20 “s’
1-r 1-1/2
^j=^2 = C,xV = 4x 12 = 48pC
41. Let C be the equivalent capacitance of the infinite
network. It consists of repeating unit of two V'.=iL = 15

e
I = 2*4 volt
capacitors of 1 pF and 2 pF. Let equivalent Cl 20

row
re
capacitance of that unit be C. The network will
appear as shown in Fig. 2(Q).73. V, = ^2’l = l§5 =9*6 volt

FFllo
F
FIGURE 2(Q).73
2mF
46. Careful observation shows that in Fig. 2(Q).45,

u
A
capacitors are connected in series.

ree
47. Suppose the pair of plates A is connected to

sFr
positive terminal of the battery and the pair of
IHFZ^ plates B is connected to negative terminal of the

kro
battery, Fig. 2(Q).46. As is clear from the figure,
uor
B
offo we have two capacitors I and II. Positive plate of
I is connected to positive plate of II and negative
plate to negative therefore, they are in parallel.
kos
Equivalent capacitance of new arrangement must
Y
Yo
● be equal to C. As Cp = C| + C2 Cp = 2 C = 2 GqAld
eerBB
oo

2xC 48. Suppose the pair of plates A is connected to


C = l+
positive terminal of the battery and the pair of
rY

2+C
plates B is connected to negative terminal of the
c2-C-2 = 0 C = 2pFor-lpF battery. As is clear from the Fig. 2(Q).47, we have
u

As capacitance cannot be negative, so equivalent three capacitors, I, II and III. Their positive plates
ou

capacitance is 2 pF.
ad
do

are connected to each other and so are the


42. InFig.2(Q).42, c negative plates. Therefore, these are joined in
nY

P\
=3+6=9pF
parallel.
_L = i + I = Il 36
Cp — Cj + C2 + C3 = 3 C = 3 € Q A(/t/
nid

C =—UF
Re

C, I ●y 4 9 36 13^ 49. The arrangement shown in Fig. 2(Q).48 is equi


FFi

36 36 + 156 192
valent to two condensers of capacities Cj and
C2, connected in series, where
S2 = — + 12 =
13 13 13
pF
1 1 1 13 192 + 96 + 13 301
€q AK^ 2€q A/C,
— — —i 1 _
d/2 d
1 2 192 “ 192 “ 192
192 192 Sq AK2 ^^0 ^^2
HF G = C,V = x6pC ^2 = dll d
301 301

V _ g _ 192x6 = 3-82 volt


1 1 1

‘ C, 301x1 Use
C C ^
43. Here, C, = 2 pF, C2 = 4 pF, £, = 12 V, £2 = 24 V 50. In Fig 2(Q).49, when key K is pressed,
1 1 1 1 1 3 4 g,=EC,
= —+ — = - + _ = C =-pF
c, q C2 2 4 4^ 3^ As C| & C2 are in series
_L=± 1
? = q(£,-£j) = |(l2-24) = -I6HC. ^2 qq
2/102 Pnadee^ 4^ Fundamental Physics (XII) VOL.I

C,C^ 58. Here. = Cj = 800 pF = 8 x KT‘0 F


EC^ Vl = 100 V, V2 = 0
C^ +C2 Q2-ec,=
Cj + Ci
52. In Fig. 2(Q).51, 8 |i F and 10 )i F capacitors are Common potential, V =
in series and the combination is in parallel with q+C2
9 p F capacitor. 8x10~^Qx100
53. In Fig. 2(Q).52, C, = 5 p F, Cj = 10 p F, = 50V
” (800 + 800)10-'2
and C^= 15 p F are in parallel.
1 2
.-. Cp = C, + C2 + C3 = 5 +10+15 = 30pF Energy stored, “/ =
Again, & C4 are in series. Therefore,
= ●!-(800 + 800)10-12(50)2

ww
”30'*‘30 15’ 2

My.= 2xlO-^J
C,= 15pF 59. Let C be the capacity of each condenser,
54. As is clear from Fig. 2(Q).53, the two cylindrical

Flo
capacitors are joined in parallel. J- = l 1 1 = -
_ = lpF .% C = 3pF
c~c^c'^c c

e
ree
Cp = C, + C2 = 2-2 + 2-2 = 44 p F s

As q^CpV 9 = 44xlO = 44pC Cp = C, + C2 + C3 = 3C = 3x3 = 9pF

Fr
rF
56. In Fig. 2(Q).55, C5, Cg are in parallel. Let V be the p.d of same source.
uurr
1
C56 = C5 + C6=10+10 = 20pF. iC V2
Also, C2, C3, C4 are in series
.
1 1
= — + —+
1
s for ii = £L = ^ = i
2 P
kks
^234 ^2 ^3 C4 ".60 " 20 3C 9
Yo
P P
oooo

^234 = 20 p F 60. The equivalent circuit diagram of given circuit is


Now, C234 and C55 are in parallel. shown in Fig. 2(Q).75.
eB

Their combined capacity is FIGURE 2(Q).75


Cr = C234 + C56 = 20 + 20 = 40 p F.
r

3pF 1pF
Now, Cj and O' are in series.
ou
ad
YY

Therefore, capacity of the combination is


C =
C^C 40x40
:=20pF
nndd

q+C'" 40 + 40 3pF 2pF 2pF


Re

Total charge that flows in the circuit, when a


Fi

battery of 100 V is connected between A and F is


^ = CV= (20 X 10“® ) X 100 = 2 X 10“® C 100V
57. The equivalent circuit shown in Fig. 2(Q).74 reveals
that the two capacitors are connected in parallel. Two capacitors, each of 3 pF are in parallel.
Cl = 3 + 3 = 6 mF
FIGURE 2(Q).74
Two capacitors of 1 pF and 2 pF are in parallel.
Ci= 4nF
C2=l+2 = 3pF
As Cl and C2 are in series, therefore
C2= BjiF
6x3
C,J = = 2pF
6+3

C12 is in parallel with 2 pF capacitor.


6V Net capacitance, C = Ci2 + 2 = 2 + 2 = 4pF.
Total charge, 9 = CV = (4 x 10“®) x 100
Charge on Ci is 4 x 6 = 24 p C = 4 X 1(H C
Charge on C2 is ^2 = ^"2 6 x 6 = 36 p C
ELECTROSTATIC POTENTIAL AND CAPACITANCE 2/103

1 63. Let C be the capacity of each of the three


Total energy, u = - CV^ condensers.
2

1 ^J_ J_ J__2
= |(4x10-6)(100)2 =0-02J ■

~ C~ C

61. 1/4=^04^2=271 or
C
C^=^ = lHF .-.C=3pF
2x27 54
V2 = = 9 X 10^ Now, C^ = C+C+C=3C=3x3pF
6x10-6 = 9jlF

oww
When connected to the same source, V is same.
Energy stored in C2,
£ =-C V2
f/.,2 =-x2x10-6x9x106
2
= 91 s 2 ^

Energy sotred in

e
and E„=-CV'^

FFrlo
re
P 2P
O-=-x3x10-6x9x106 = 13-51
^ 2

rFee
E
Total energy stored in C2, C3, C4 _L- 2_f_ -£l-1
= £/2+f/3 + i/4 = 9+13-5 + 27 1
'S'®

rF
-cv^
ouru
= 49-5 J 2 P
64. As is clear from Fig. 2(Q).60, C|, C2 are in series
fsoor
Equivalent capacitance of C2, C3, C4
= 2 + 3 + 6= 11 pF 1 1 1 1 1 1
skf
^2
ooko
As U=^=
YYo
= 49-5
2C 2x11x10-6 = ^2 = X = 3 X 12 = 36 p C ;
^3 = C3 y=6x 12 = 72 pC
Bo

.*. ^ = 49-5 X 22 X 10-6 C2


reeB

(ii) C3 is in parallel with combination of C|


and C2-
U
_ g2 49-5 X 22x10-6
.*. Cp = C3 + C, = 3 + 6 = 9 pF
ooY

= 544-51
uur

1 “
2C,1 2x1x10-6
ad

1 1
Total energy stored in the arrangement (IIO U^ = U2=-qjV = -q2V
dY

= 544-5 + 49-5
1
= 594*0 J = -x36x6 =108p J;
nind
Re

62. In Fig. 2(Q).59, Two 2 pF on right side are in


1/3 = ^C3V2 = 1x6x122 = 432pJ
FFi

series.

2x2
So C.1 = = lpF 65. HereC, = 20pF = 20x 10-6p
2+ 2
y, = 30V,C2 = 50pF = 50x 10-6r
This 1 pF is in parallel with central 1 pF y2 = 0;y=?
C2=l + l = 2pF 20x10-6x30
V =
C2 is in series with 2 pF at bottom
C1+C2 (20 + 50)10-6
2x2
C3 = 2+ 2
= 1 pF 60
= ::rv = 8*57V
7
C3 is in parallel with left 1 pF capacitor
C,,= l + l=2pF , Initial energy, Vj2
U =
lc^^y2 = 1x2x10-6 x(6)2 = 1(20x10-6)(30)2
= 3*6xlO-«J = 9xl(Hjoule
2/104 "Pnaideefi.\ Fundamental Physics (Xn)ISSQD
1 68. Here,= 4mm,K-Z and/ = 3mm
Final energy, U^ Let d be the new distance between the plates.
2
1 /fiO As — X Capacitance with air = Capacitance with
= -(20 + 50)10-^ —
2 A ^ > dielectric
= 2-57 X 10-^ joule 2 e„0 A
Loss of energy = C/j - 1/2 3 d K
= 9 X 10-3-2-57 X 10-3 d'-t + —
2
= 6-43 X 10-3 J
1
This loss of energy appears in the form of heat or -x- =
3 4

ww
and e.m. radiations.

66. Cj is capacity ofX; C2 is capacity of F, Fig. 2(Q).61 or 6 = d'-3-t-l or if = 8mm


C2 = 4C,.
69. (/) P.d across the capacitor before metal plate was
±=J-+_L 1=±+J_=_L

Flo
inserted
C, c/c2 ●● 4 c/4C, 4C,

e
Vq = = 3 X 10^ X 0-05 = 1500 V

ere
C, =5pF; C2 = 20pF (k) After introduction of plate, V = £q (d - /)

FFr
= = 12 = 48pC = 3x10^ (0-05 - 0-01) = 1200 V
(it) When dielectric slab is introduced, K=2
uurr
V.=iL = « = 9<V.

orr
' C, 5 sfo %
V=Eo(d-t) + K
^1 -i2_i?
= = 24V
kks
<^2 20 3x10^x0-01
= 1200-1- = 1350 V
Yo
oooo

2
/
1 ^
eBB

C/2 C, 71. Change in distance, x = t 1


67. In Fig. 2(Q).62, When S is closed, V ^ j
rr

e« A
-CV^+-CV^ =CV^ 72. (i) C = -2—
ou

2 2
ad

d
YY

When S is open and dielectric is introduced.


{it) Qq — CVq j Vq — 3000 volt
Capacity of each condenser = KC = 5 C.
nndd

Condenser on the left is connected to battery. Its


Re

potential remains V. But condenser on irght is £ V 1000


not connected to battery. So on introducing
Fi

Sn0 A
V («v) c =
dielectric its. potential becomes — /
1 ^
d-t 1- —
.●. Total energy^ V

1 1 f V \2
\ (v)e=Xeo = 3x 8-85 x 10-*2 C^ N~‘ m“2
-(5C)V^.-(5o(-) = 2-66 X ir^l Fmi
73. Here, V= 1 JtV = 103 volt, X = 3,
f
1 'I
Dielectric strength = 10^ Vm"'
= i(5C)v2
2 I1 + ^
25 Electric field, £ = 10% x lO"^ V/m = 10^ V/m
A = ?,C=50pF = 50x 10-'2f
C/, = —
^ 2
.5CV^^^=I3cv2
25 5 V
0
As C = and d = —
d £
V
1 _ CV2 5
13
Xg„ AE
^1 C =
0

5 V
ELECTROSTATIC POTENTIAL AND CAPACITANCE 2/105

—> A
CV force on the charge. F = qE i
A =
A A A
0
Work done, W = F.PS = q El .{-a i - hj)
50xl0-‘^xl0^
A = = I9x l(Hm" = -q Ea
3x8-85x10"‘2x10^
Negative sign shows that work is done against
= 19 cm^
the electric field by some external agency moving
the charge.
74. Here,^^ = 2x 10-^C,9g = 4x IQ-^C, 76. As the field is directed outwards, charge on the
,?c=8x lO'^C wire must be positive
AB = BC=CD = DA = 0-2 m. Fig. 2(Q).76.
B r=04»i
1000
V -V = - E.dl dr
FIGURE 2(Q).76 r
A B A r=0-2m

ww
\ /
s
\
\ /
=04 m
E
Vg-V^=-1000[log^r];;^(j.2«.

FF loo
O \
✓ \
✓ \

/
✓ N
\
*
= - 1000 [log, 0-4-log^ 0-2]

ree
D C
0.2m
04
= -1000 log^ 02
AC=BD = Vo-2^ + 0-^ = 0-2

ree F
m

= - 1000 log, 2
0-2-72
ur
= 0-1-72 Vg-V^ = ~ 1000x0-6931
AO = BO = CO = DO =

Potential at point D is
2
m

fofroF= - 693-1 volt

77. This arrangement in Fig. 2(Q).63 is equivalent


ks
to three capacitor connected in series
k «a_ + iB-+«C
os o

V = 577-26 V
YYouor

D
LAD BD CD K,\ ^ 0n A 3£j€qA
C,=
BBook

d/3 d
Potential at point 0 is
r ee

3K^^q a
C,=
V = 890-82 V d
o ~
Lad BO coj
ad
oouur

r
^3- d
Y

Work done = q(VQ- Vq)


= 2x 10-9(890-82-577-26)
L=J_ -L -L - 1
nYd

d 1
= 6-27x 10"^ joule
Re

— -t-
c1 ^2 ^3
75. Here, P (a, b, 0), Q (2 a, 0, 0), R (a, -/?, 0) and
c 3e.0 A K
^2 ^3)
FFindi

S (0, 0, 0), [Fig. 2(Q).77].


C =
AK^K^K^ 0

FIGURE 2(Q).77 d{K^K2 + K^KJ + K^K^)


V

^E‘
78. Without dielectric,

With dielectnc, £ = —
K

Effective displacement in going from F to 5 via


V=£y--i-£.- I ^0
2 = £,)^
any path is

9 2 "2 K 2

= PS=(0-a)? + (0-b)J + (0-0)k


A A
2 K 2K
= —a i - / - bj
2/106
‘P>uidee^'4. Fundamental Physics (XII) VOL.I

-L=i-+-L _£!_+_iL
v„ v„(ic+i) jr+i « q ^1^0 ^ ^2^0 ^
79. In Fig. 2(Q).64, C2, C3 are in series. This \

combination is in parallel with C|. 60-4 e,€2


The combined capacity of Cj, C3, C, is in series
with C4. The combination is in parallel with
q- ~e,vej</,
80. As total area of flat plate is A and it is divided
82. Suppose the equivalent capacity between A and
into three equal parts, forming three capacitors, B is C. As the network is iniinite, therefore
therefore, area of each capacitor = A/3 Starting addition or removal of one set of capacitors from
from left end. Fig. 2(Q).65
the network will not affect our result. It implies

w
that the capacitance between D and £ can be
CI =
^0 n — ^0
~r~' ^”(rf + b)’ taken as C. Effective capacitance of parallel
combination of C and C, Fig. 2(Q).79 is

FF lowo
CT' = C+C'.

ee
(^/ + 2b) FIGURE 2fQ).79

Fr
As the three capacitors are joined in parallel, C
therefore, total capacity of the assembly A D

Cp = Cj + C2 + C3

rer
ur
C' =^C'
Sq A/3 Sq A/3 A/3 fofr Fo
i
0
d d +b (d + 2b)
ks
B E
YYouro
oo

A
Now C" and C are in series.
c„=-^
P 3
BBo ks

1 1 1 1 1
●● C c'^C'
= —+

^ hd + b)(d + 2b) + d(d + 2b) + d(d + by c (c+o


r ee

d(d + b)(d + 2b)


oouru

C+2C
ad

= eQA(3i^ + 6bd + 2bl^)l3d(d + b)id + 2b) C(c+o


Y

81. The system is equivalent to two condensers or C^ + 2CC = C(C+C) = C^ + CC


connected in series as shown in Fig. 2((2).78.
Ynd

or C^ + CC-C^ = 0
Re

-C±Vc^-h4C^ -C±^I^
FFindi

FIGURE 2fQ1.78
C =
§1 & 2 2
d, d.
As capacitance cannot be negative.

.'. C=
2

6,S(, A _62So A
C =
j(2.24-l) = i|^C = a62C
q = d,1 : q = As C = 1 pF, therefore, C = 0*62 jlF
<'2
2/107
ELECTROSTATIC POTENTIAL AND CAPACITANCE

1
ISI r I
WITH
SOLUTIONS
it ^1.

Q. 1. Two charges 5 x lO"* C and -3 x lO"* C are located 16 cm apart. At what points on the line joining
the two charges is the electric potential zero ? T^ke the potential at inflnity to be zero.
Sol. Here .^1 = 5 X 10-^C,^2 = -3 X 10'* C
Let the potential be zero at a distance x cm from the charge = 5 x 10“* C
''1 = xx 10”^ m ; r2 = (16-x)x ICr^m

w
Now, V = gl , ^2 ; 0 =
1
Si+h.
4ne
4n^Q n 4tc£o T2 0 /l ^2.

Flo
5x10"® -(-3xl0~^) 5 _ 3
Sl = -Si.

eeee
or

'i '2 xxl0“^ (16-j:)10“^ X \6-x

Fr
3jc = 80-5;c ; 8 x = 80. a:= 10 cm
Q. 2. A regular hexagon of side 10 cm has a charge 5 p C
at each of its vertices. Calculate the potential at the
for
ur
centre of the hexagon.
Sol. In Fig. 2(N). 1,0 is centre of hexagon ABCDEFA of each
ks
side 10 cm. As is clear from the figure, OAB, OBC etc.
Yo
oo

are equilateral triangles. Therefore, (5pC)F C (5nO


OA = OB = OC=OD = OE = OF
B

- r = 10 cm = 10“l m.
re

As potential is scalar, therefore, potential at O is


ou

9x10^x6x5x10“^
ad

V =6x -1
= 2-7 X 10® V
Y

4tcg«0 r 10

Q. 3. Two charges + 2 p, C and - 2 p C are placed at points A and B, 6 cm apart (a) Identify an equipotential
nd

surface of the system, (ft) What is the direction of the electric field at every point on this surface ?
Re

Sol. (a) The plane normal to AB and passing through its middle point has zero potential everywhere,
Fi

(ft) The direction of electric field at every point on this surface is along normal to the plane in the direction AB.
Q. 4. A spherical conductor of radius 12 cm has a charge of 1*6 x C distributed uniformly on its
surface. What is the electric field (a) inside the sphere (b) just outside the sphere (c) at a point 18 cm
from the centre of the sphere ?
Sol. Here, r = 12 cm = 12 x 10"^ m, ^ = 1-6 x 10“'^ C.
(a) Inside the sphere, E = 0
(ft) Just outside the sphere, (say, on the surface of the sphere)
i-exio-"^
E = I_ = 9xl0^x = 10® N/C
4ne^0 r- (12x10-2)2

(c)At/= 18cm= 18x 10-2m. F =


9x10^x1-6x10 ^ ^
4tc€a0 r
/2
(18xl0“2)2
2/1Q8
Fundamental Physics (XII) VOL.I

Q. 5. A parallel plate capacitor with air between the plates has a capacitance of 8 pF. (1 pF = F)
What will be the capacitance if the distance between the plates is reduced by half and the space
between them is filled with a substance of dielectric constant 6 ?
€« A €« A 6x2sn0 A
Sol.
q=^ = 8pF C2 =x d/2 d
= 12x8 = 96pF
Q. 6. capadtore each of capadtance 9 pF are connected in series, (a) What is the total capadtance
of the combination ? (b) What is the potential difference across each capacitor if the combination is
connected to a 120 V supply.
I
Sol. -L

y 120
p.d. across each capacitor = — = 40V
3

w
Q. 7. Three capadtors of capadtances 2 pF, 3 pF and 4 pF are connected in parallel, (a) What is the total
capadtance of the combination ? (b) Determine the charge on each capacitor, if the combination is
connected to 100 V supply.

Flo
Sol. C^, = 2 + 3 + 4 = 9pF
For each capacitor, V is same = 100 volt

eee
A <?i =Cj V=2x 100 = 200pC, <72 = ^2'^ = 3x 100 = 300pC,<?3 = C3 V = 4x 100 = 400pC

Fr
Q. 8. In a parallel plate capadtor with air between the plates, each plate has an area of 6 x lO-^m^ and
the distance between the plates is 3 mm. Calculate the capadtance of the capacitor. If this capacitor
is connected to a 100 V supply, what is the charge on each plate of the capadtor ?
for
ur
Sol. Here,
” ^=6x 10"^m2 ^/=3mm = 3x 10~^m, C=?, V= 100V,^ = ?
_ _ (8.85xl(Tt2)(6xl0-3)
ks
= 1-77 X 10-** F
3x10--^
Yo
oo

9 = CqV= 1-77 X 10-** X 100= 1-77 X ir’C


Q. 9. Explain what would happen if in the capacitor in Q. 8, a 3 mm thick mica sheet of (dielectric
B

constant = 6) were inserted between the plates (a) while the voltage supply remained connected (b)
re

after the supply was disconnected.


Sol. (a) While the voltage supply remained connected, voltage remains constant.
ou
ad

Capacity increases ioC = KCq = 6 x 1-77 x 10“'' F


charge increases to = O' V = 6 x 1 -77 x KT*' x 10^
Y

C.
{b) After the supply was disconnected, chaige remains constant. New capacity,
<r = XCo = 6 X 1*77 X 10-“ F
nd
Re

1-77x10-^
new voltage y' = JL =
Fi

- = 16-67 V
C 6x1:77x10-"
Q. 10. A 12 pF capadtor is connected to a 50 V battery. How much dectrostatic energy is stored in the
capadtor ?
Sol. Here, C = 12 pF = 12 x lO-'^F, V= 50 volt, // = ?

U =
|CV2 =1(12x10"*2)(50)2 =1-5x10-*J
Q. 11. A 600 pF capadtor is charged by a 200 V supply. It is then disconnected from the supply and is
connected to another uncharged 600 pF capadtor. How much electitistatic energy is lost in the
process ?
Sol. Here, C, =C2 = 600pF = 600x 10-*2p = 6x 10-*°F, V, =200 V, ^2 = 0
Loss of energy _ ^2 ^^1 ~ ^2)^ _ 6xlO~***x6xlO~*Q (200 - 0)2 _ 36x 1(T2** x 4x l(y* = 6xlO-*^J
2(Cj+C2) 2(6x10-*°+6x10-***) 24x10-*°
ELECTROSTATIC POTENTIAL AND CAPACITANCE 2/109

WITH ANSWERS,
HINTS AND SOLUTIONS

E m
IF

MULTIPLE CHOICE QUESTIONS-I

1. A capacitor of 4 p, F is connected as shown in (a) The work done in Fig. (/) is the greatest
the circuit. Fig. 2(N).2. The internal resistance (h) The work done in Fig. (/i) is least
of the battery is 0.5 Q. The amount of charge

ww
(c) The work done is the same in Fig. (/)* Fig-
on the capacitor plates will be
(//■) and Fig. (ut)
FIGURE 2(N).2
(f/) The work done in Fig. (m) is greater than

FF loo
4mF
AWAAr
10Q
Fig. (/i) but equal to that in Fig. (i)
4. The electrostatic potential on the surface of a

ree
charged conducting sphere is 100 V. Two
2.5 V statements are made in this regard :

rree F
S| : At any point inside the sphere, electric
212
intensity is zero.
ur
●vVW

ia)0 (b)4[lC
fofr oF
S2 : At any point inside the sphere, the
electrostatic potential is 100 V.
ks
(c) 16 pC (i/) 8 p C
Which of the following is a correct statement ?
YYouro
2. A positively charged particle is released from
soo

rest in a uniform electric field. The electric (a) S, is true but S2 is false
potential energy of the charge
BBook

(b) Both 5j and S2 are false


(fl) remains a constant because the electric field (c) 5, is true, S2 is also true and S^ is the cause
r ee

is uniform
of S2
(b) increa.ses because the charge moves along
oouru

{(I) 5] is true, S2 is also tnie but the statements


ad

the electric field


are independent
Y

(c) decreases because the charge moves along


the electric field 5. Equipotentials at a great distance from a
collection of charges whose total sum is not
Ynd

id) decreases because the charge moves opposite


Re

to the electric field zero are approximately


FFindi

3. Figure 2(N).3 shows some equipotential lines (fl) spheres


distributed in space. A charged object is (b) planes
moved from point A to point B.
(c) paraboloids
FIGURE 2{N).3 {d) ellipsoids
30V
6, A parallel plate capacitor is made of two
30V 40V

dielectric blocks in series. One of the blocks


has thickness and dielectric constantand
A* ●B A| ● I fB A* ●B the other has thickness ^2 and dielectric
constant k2 as shown in Fig. 2(N)-4. This
arrangement can be thought as a capacitor
10V 20V 30V 40V 50V10V 20V 50V 10V 20V 40V 50V with dielectric slab of thickness d d^
e and effective dielectric constant k. The k is

i.
2/110
Fundamental Physics (XII)LT/*1WI
FIGURE 2(N).4 k.d, +k^d
O 2“ 2 k,d^ + k^d
2“2
(«) (b)
m d^ +d^ kj+k^

d2
¥2^+^2> 2k,k
V'2
id)
(k^d2 + k2d^) k^+k2

MULTIPLE CHOICE QUESTIONS-II

7. Consider a uniform electric Held in the z is opened and K2 is closed (order is impor
direction. The potential is a constant tant). [Take Q^' and Q2 as charges on C| and
(fl) in all space C2 and Vj and V2 as voltage respectively.)

ww
Then
(b) for any x for a given z
(c) for any y for a given z

Floo
FIGURE 2(N).5
(d) on the x-y plane for a given z. K K2
8. Equipotential surfaces <●> <●>

ee
(a) are closer in regions of large electric fields

reer
rFF
compared to regions of lower electric fields
(b) will be more crowded near sharp edges of a
uur r
conductor

(c) will be more crowded near regions of large


E
ffoor
charge densities
sks
YYoo
(d) will always be equally spaced (a) charge on Cj gets redistributed such that
ooko

9. The work done to move a charge along an V, = V2


eBB

equipotential from A to B (b) charge on Cj gets redistributed such that


B
Qi=Q2
(a) cannot be defined as -f E.dl
r

(c) charge on C^ gets redistributed such that C^ V,


ouur
ad

A + C2V2 = C,£
YY

B
(d) charge on C] gets redistributed such that
Qi' + Q2=Q
(/?) must be defined as -J E.dl
nndd
Re

A 12. If a conductor has a potential V 0 and there


are no charges anywhere else outside, then
FFii

(c) is zero
(d) can have a non-zero value. (a) there must be charges on the surface or inside
itself
10. In a region of constant potential
(a) the electric field is uniform (fc) there cannot be any charge in the body of the
conductor
(b) the electric field is zero
(c) there can be no charge inside the region (c) there must be charges only on the surface
id) the electric field shall necessarily change if {d) there must be charges inside the surface
a charge is placed outside the region 13. A parallel plate capacitor is connected to a
11. In the circuit shown in Fig. 2(N).5, initially battery as shown in Fig. 2(N).6. Consider two
key ATj is closed and key K2 is open. Then Ki situations :
2/111
ELECTROSTATIC POTENTIAL AND CAPACITANCE

A : Key K is kept closed and plates of capaci>


FIGURE 2(N).6 tors are moved apart using Insulating handle
K B : Key K is opened and plates of capacitors
<●> are moved apart using insulating handle
Choose the correct option(s)
(a) InA .Q remains same but C changes
C
{b) In B : V remains same but C changes
6
E
(c) In A : y remains same and hence Q changes
(J) In B : (2 remains same and hence V changes.

ANSWERS

\.{d) 2. (c) 3. (c) 4, (c) 5. (a) 6. (c) 7. {b,c,d) 8. {a,b,c) 9. {b.c)

ww
10. {b,c) 11. M 12. {a.b) 13. (c,d)

HINTS FOR DIFFICULT MULTIPLE CHOICE QUESTIONS

FF loo
Multiple Choice Questions -1

ee
1. In Fig. 1(N).2, no current is allowed to flow through 10 O resistance because of capacitor. The current

ee r
drawn by 2 O res.

rFrF
E 2-5
/ = = 1 A.
rur
R + r 2 + 0-5

Terminal pot. diff. of battery, V = E - Ir = 2-5 - I x 0-5 = 2 volt


.●. Charge on capacitor plates, Q = CV = 4 x 2 = 8 pC
ffoor
ks
Choice id) is correct.
YYoou
2. In a uniform electric field, when a positively charged particle is released from rest, it moves along the
ookos

electric field (/.«., from higher potential to lower potential). Therefore, electric potential energy of charge
BBo

decreases. Choice (c) is correct.


3. In Fig. 1(N).3, we observe that in all the three parts, = 20 V and Vg = 40 V. Work done in carrying a
re

charge g from A to B is W = g (Vg-V^) = same in all the three figures. Choice <c) is correct.
oouur
ad

.. _kg
4. As is known from theory, at any point inside a charged conducting sphere, £ = 0 and V - — - potential
Y

on the surface = 100 V


dndY

dV
Re

Infact, E = - = —— (constant) = Zero.


dr dr
FFini

Choice (c) is correct


5. From a collection of charges, whose total sum is not zero, equipotentials at large distances must be spheres
only.
Choice (fl) is correct.
k^A
6. The capacities of two individual condensers are = d
and C2 =
1 ^2
The arrangement is equivalent to two capacitors joined in series. Therefore, the combined capacity (C^) is
given by

J- = -L —
d
1 i _ 1 .^1^2+*^2^1
Ak,k
^1 ^2 fejGo A ^2^0^ 0 ^ k
L "I
k
2 J 0 n

ALk ke^0 A
0 r2 _
or

d^k2 + d^k^ (d, +^2)


2/112
Fundamental Physics (XII) VOL.I

Multiple Choice Questions - II


7. As electric field in the z direction is uniform, equipotential surfaces are in XY plane. Hence the potenUal
for given Z\ is constant (/) on x-y plane (i7) for any x in this plane and (Hi) for any y in this plane.
Hence Choices (ft), (c), (d) are correct

dV 1
8. From E = - , we find that dr ie., equipotential surfaces are closer in regions of large electric
dr

fields compared to regions of lower electric fields. At sharp edges of a conductor, chaige density is more.
Therefore, electric field is stronger. Hence equipotential surfaces are more crowded. Same argument is true
for option (c). The choices (a), (ft), (c) are correct
9. From the knowledge of theory, work done to move a charge along an equipotential from A to B,

ww
W = -j E.dl
A

Flo
e
On equipotential surface, E Ldl

ree
Fr
B

rF
w =
-J E(dl) cos 90® = Zero.
uurr
A

Choices (ft) and (c) are correct

10. In a region of constant potential (V = constant);


s
E = -
dV
for
- 0, /.e., electric field is zero.
kks
dr
Yo
oooo

As £ = 0, there can be no charge inside the ergion.


Choices (ft) and (c) are correct
eB

11. In Fig. 1(N).5, when is closed and K2 is open, Cj is charged to potential E acquiring a total chaige
Q = C,£.
r

When K^ is opened and Xj >s closed, battery is cut off. C, and C2 are in parallel. The chaige on C, is shared
ou
ad
YY

between C, and C2, such that V, = V2.


As there is no loss of chaige, Qi' + Q2=Q
nndd

Choices (a) and (d) are correct


Re

12. From the knowledge of theory, if a conductor has a potential 0 and there are no charges anywhere else
Fi

outside, then there must be charges on the surface of conductor or inside the conductor. However, there
cannot be any chaige in the body of the conductor.
Choices (a) and (ft) are correct
13. In Fig. 1(N).6, when key K is kept closed, condenser C is charged to potential £. When plates of capacitor
Gq A
are moved apart, its capacity C = decreases. As potential of condenser remains same, charge Q =
CE decreases. Choice (c) is correct
Once key K is closed, condenser gets charged : Q = CE. Now, if key K is opened, battery is disconnected,
no more charging can occur, i.e., Q ermains same. As plates of capacitor are moved apart, its capacity
C = Gq a
— decreases. Therefore, its potential V = Q/C increases. Choice (d) is correct

A
ELECTROSTATIC POTENTIAL AND CAPACITANCE 2/113

VERY SHORT ANSWER QUESTIONS


14. Consider two conducting spheres of radii and R2 with R^ > /?2* If the two are at the same potential,
the larger sphere has more charge than the smaller sphere. State whether the charge density of the
smaller sphere is more or less than that of the larger one.
Ans. Here, V,=V2
Q. Qi _R I ...(/)
47: €q/?. 4tC6o /?2 ^2 R.
As /?j >/?o. Q\> Q2
i.e., larger sphere has more charge than the smaller sphere.

Now, and a, -
^2 <^2 _ ^ ^

w
<j

Ri ■ <2,
I “
4kR?I 4nR^ a
1

c
2 _ ^2 1
Using (0,

Flo
a
1 ^1 ' ^2" ^2
As /?j>/?2> therefore, a2>Oj

ee
i.e., charge density of smaller sphere is more than the charge density of larger sphere.

Fr
15. Do free electronstravel to region of higher potential or lower potential ?
Ans. As free electrons are negatively charged, they would move to regions of higher potential.
Can there be a potential difference between two adjacent conductors carrying the same charge ?
for
16.
ur
Ans. Yes, there can be potential difference between two adjacent conductors carrying the same charge. This is
because the sizes of the conductors might be ditferent, and hence their capacities may be different.
s
17. Can the potential function have a maximum or minimum in free space ?
ok
Yo
Ans. No. The potential function cannot have a maximum or minimum in
Bo

free space. This is because in that event.


dV
£ = - = Zero
re

dr
18. A test charge q is made to move in the electric field of a point
ou
ad

charge Q along two different closed paths, Fig. 2{N).7. First


path has sections along and perpendicular to lines of electric
Y

field. Second path is a rectangular loop of the same area as the


first loop. How does the work done compare in the two cases ?
nd
Re

Ans. From the knowledge of theory, we understand that electric field is


conservative. Therefore, work done in moving a lest charge over
Fi

any closed path is zero.

SHORT ANSWER QUESTIONS

19. Prove that a closed cquipotential surface with no charge within itself must enclose an equipotential volume.
Ans. Suppose a closed equipotential surface with no charge within itself does not enclose an equipotential
volume. Therefore, potential just inside the surface would be different from potential at the surface, resulting
in some potential gradient. Therefore, there would be field lines pointing inwards or outwards from the
surface. This is possible only if other end of the lines are at charges inside. As there is no charge inside,
therefore, the entire volume inside the equipotential surface must be at the same potential,
20. A capacitor has some dielectric between its plates, and the capacitor is connected to a DC .source. The
battery is now disconnected and then the dielectric is removed. State whether the capacitance, the
energy stored in it, electric field, charge stored and the voltage will increase, decrease or remain
constant.

t
2/114 "Ptadee^ ^ Fundamental Physics (XII) OTSTWl
Ans. When the capacitor is connected to dc source, it gets charged. The battery is now disconnected. So no more
charge can flow in, i.e., charged stored will stay constant. On removing dielectric, capacitance decreases.

Energy stored =
Q" Q V'
will increase. Pot, V = — would increase E = — will increase.
1C C d

21. Prove that, if an insulated, uncharged conductor is placed near a charged conductor and no other
conductors are present, the uncharged body must be intermediate in potential between that of the
charged body and that of infinity.
Ans. When an insulated uncharged conductor is placed near a charged conductor, the uncharged conductor
acquires some induced charge. Along any path from the charged conductor to uncharged conductor along
the electric field, potential will continually decrease. Another path from uncharged conductor to infinity
will lower the potential further. Hence the required result.
22. Calculate potential energy of a point charge - q placed along the axis due to a charge + Q uniformly
distributed along a ring of radius R. Sketch P.E. as a function of axial distance z from the centre of

ww
the ring. Looking at graph, can you see what would happen if - ^ is displaced slightly from the centre
of the ring (along the axis) ?
Ans. In Fig. 2(N).8(a), we have shown a charge + Q distributed uniformly

FF loo
along a ring of radius R, with centre at 0.
Let X be the linear charge density of the ring. Therefore,

ee
X=QHTiR ...(0

ee r
Considera small element of length dl of the irng with centre at A.

rFrF
Charge on the element, dq = Xdl
rur
At any point P on the axis of ring,
where OP = z, potential due to the charge element, ffoor
ks
dq Xdl
dV =
Ane^iAP)
YYoou
47ce
ookos

Potential at P due to entire ring,


BBo

Xdl
re

V =()

47ceQ +
oouur
ad

X.(2nR) Q
Y

47ce 4TC€
0 0

Potential energy of charge - q held at P


dndY
Re

-qQ
U ^-qV =
FFini

47ie
0

The variation of potential energy with axial distance z from the centre of the ring is shown in Fig. 2(N).8(/?).
If charge - ^ is displaced slightly from the centre of the ring along the axis of the ring, and left, the charge
would perform oscillations. However, by just looking at the graph, we cannot come to a firm conclusion
about the nature of these oscillations.

23. Calculate potential on the axis of a ring due to charge Q uniformly distributed along the ring of
radius R.

Ans. Refer to Q. 22 above for the derivation of potential on the axis of the charged ring.

LONG ANSWER QUESTIONS

24. Find the equation of the equipotentials for an infinite cylinder of radius Tq, carrying charge of linear
density X.
ELECTROSTATIC POTENTIAL AND CAPACITANCE 2/115

Ans. Fig. 2(N).19 shows an infinite cylinder of radius tq carrying charge of linear density From symmetry, we
find that the field lines must be radially outwards. Imagine a cylindrical Gaussian surface of radius r and
length /. According to Gauss’s theorem,
XI FIGURE 2(N).9
o E.ds = -^
^0
XI A’
X
£ (2 7t r /) = — or E = ...(O
^0 2rtSn0 r a 1

T
D
u y Infinite Cylinder
X
— log
, ^ Gaussian Surface
V(r)-V(ro)=- E.dl =

oww
27C6Q ® r
'b
For an equipotential surface of given V (r),
2tC6-
logg — = ^[F(r)-V (/■„)] . r = K.e -2JiEo[l/(r)-V'(ro)]/X ...(«)
0

e
X
'b

re
FFrlo
Hence, equipotential surfaces are cylinders of radius r given by («).
25. Two point charges of magnitude + q and - q are

rF
ee
placed at {- dll, 0,0) and (d/2,0,0), respectively.
Find the equation of the equipotential surface

rF
where the potential is zero.
ouru
Ans. Fig. 2(N).10 shows two point charges + ^ at

A
Id ] (d
-—,0,0 and charge-^ at B — , 0, 0
fosor with
skf
ooko
centre at 0. Let the equipotential surface (with V =
Yo
0) be at a distance x from the origin 0.
Y

The potential at any point P on the surface at a height


Bo
reeB

h from AB is

V: i ^
ooY

47teQ [{x + d /2)2 + /j2]1/2 - d /2)2 + /i2]i/2


uur
ad

q q
IfV=0, then
dY

4jt€Q[(x + d/2)2+/i2]l/2 47tGQ[(;c-d/2)2 + /i2]l/2


2 d^ . 2 d^
nind

(X + dllf + ;j2 = (x - dUf +


Re

or or x^ ■¥ — + ;cd = A:^ + — -xd or2xd-0orx = 0


4 4
FFi

Hence, x s 0 is the equation of the equipotential surface where potential is zero.


26. A parallel plate capacitor is filled by a dielectric whose relative permittivity varies with the applied
voltage (I/) as e = aU where a = 2 A similar capacitor with no dielectric is charged to Uq = 78 V.
It is then connected to the uncharged capacitor with the dielectric. Find the final voltage on the
capacitors.
Ans. On connecting the two given capacitors, let the final voltage be U.
If capacity of capacitor without the dielectric is C, then the charge on this capacitor is - CU
The other capacitor with dielectric has capacity e C. Therefore, charge on it is Qi = ^ CU.
As e = .a U, therefore, Q2~o.CU'^
The initial charge on the capacitor (without dielectric) that was charged is Go-CC/q
From the conservation of charge. Qq-Q\+ Qi

CC/r.0 = C£/ + a Cf/2 or af/2 + t/-C/n0 = 0 U =


-l±.^l + 4at/ 0
2a
2/116 Fundamental Ph3rsics (Xn^mgwn

using a = 2 V"‘ and = 78 volt, we get U = -


-I±yi + 4x2x78 -1±V625
2x2 4

V625-1 24
As i/ is positive, therefore. U = — = 6 volt
4 4

27. A capacitor is made of two circular plates of radius R each, separated by a distance d « R. The
capacitor is connected to a constant voltage. A thin conducting disc of radius r«R and thickness
t« r is placed at a centre of the bottom plate. Find the minimum voltage required to lift the disc if
the mass of the disc is m.
Ans. As shown in Fig. 2(N).l 1, the disc is in touch with the bottom plate. The electric field on the disc is
E=V/d
FIGURE 2(N).11
Therefore, chaige q' transferred to the disc
R

w
0 V 2
q' = av= - V=-
d
V d
/ r«R:d«r

Flo
Force acting on the disc, F = E= e
\
^d ]d \
Disc

ee
2
If the disc is to be lifted, F"^mg, i.e., €q d^ (for minimum V)

Fr
= mg

mgd^
for
V =
ur
V”®o
28. (a) In a quark model of elementary particles, a neutron is made of one up quarks [chaige (2/3) e\ and
ks
two down quarks [charges - (1/3) e\. Assume that they have a triangle configuration with side length
Yo
of the order of 10^^^ m. Clalculate electrostatic potential energy of neutron and compare it with its
oo

mass 939 MeV.


B

(fr) Repeat above exercise for a proton which is made of two up and
one down quark.
re

Ans. Fig. 2(N). 12 shows quark model of a neutron, where r = 10“*^ m. Potential
eneigy of neutron
ou
ad

1
[/ =
Y

47teor
9xl0nr eV
nd
Re

l(r'5 [[ 3j[ 3f[3 3ft 3 3JJ


Fi

U = =-7-68x IO-'-'J

-7-68x10“'4
U = eV =-4-8 X 10^ eV = -0-48 MeV
1-6x10-'9
U 048
m c2 = 939 MeV. = 511 X 10^
n
m_c
n
2 ” 939
Proceed similarly for a proton whose quaik model is shown in Fig. 2(N). 13.
29. TWo metal spheres, one of radius R and the other of radius 2R, both have same surface chaige density s.
They are brought ot contact and separated. What will be new surface charge densities on them ?
Ans. Let a be the surface charge density of each of two metal spheres of radii R and 2 R.
Before contact, charges on the two spheres are
e, =a.4jc/?2 and fi2.= a4jc(2/?)2 = 4(2i

A
ELECTROSTATIC POTENTIAL AND CAPACITANCE 2/117

When the two spheres are brought in contact, their charges are shared till their potentials become equal.
If and Q2 are the final charges on the two .spheres after contact, then as V, = V2
Q{ Q.'
4;c€o R 4Tien (2/?)
Q{^2Q{ ...(/■)
0

As there is no loss of charge in the process, therefore,


fSi ^2 = !2| + 02 = !2| + 4 = 5 2, = 5 (a ■ 4 n: /?-) using (/), Q,' + 2 Q,' = 5 o 4 7t
10
Qi' = - a 4nR^ and Q-y' = 2Q/ = — g4kR^
' 3 .2 1 3

Hence,
er 5 0/ 5
a o and 02 =
1 “
4nR~ 3 4n{2R)~ 6^

ww
30. In the circuit shown in Fig. 2(N).14, initially is closed and K2 is open. What are the charges on
each capacitor.
Then was opened and K2 was closed (order is important). What will be the chaige on each capacitor
now ? [C = 1 |iF]

Flo
FIGURE 2(N).14
Ans, In Fig. 2(N).14, when is closed and K2 is open,

e
C3 is out of contact. Ci=6C K1 K2

ree
03 = 0. ( (*)^*)

Fr
C|, C2 are in series with the battery £ = 9 V,

rF
_l
uurr 1 1 + 2 3
6*^3 6 6 2
C
,y

C, = 2pF
s E = 9VT
for C2=3C C3=3C

0, =£. Q = 9x2=18pC, I
kks
02 = £.q=9x2=18nC ’
Yo
oooo

Later, /C, is opened and K2 is closed, charge on C, remains unchanged, i.e., = 18 |iC
eB

02 18
Charge on C2 is now shared between and C3. As Cj = C3, therefore, 0->' = 03' = —^ = — = 9 pC
31. Calculate potential on the axis of a disc of radius R due to a charge Q uniformly distributed on its .surface.
r
ou
ad

Ans. When a charge Q is distributed uniformly on the surface of a circular disc of radius R, surface charge density
YY

0 FIGURE 2tN).15
a = ...(0
kR'^
nndd
Re

Consider a small element of the circular disc in the form of a circular


strip of radius r and thickness/width d.r Fig. 2(N). 15,
Fi

= Area of this element = 2n rdr


Charge on this element, q = c {2 n r dr)
Potential due to this element at any point P on the axis of the disc,
where OP = z is

<s2nrdr
dV = -^
4tc€«z'
0 4tI€
0 V?+7
Potential at P due to charge on the entire circular disc
R
2 rdr 2kg ●l/?
'^1777
no ,2
V = + z~ -
47C€
0 0 1^77 4rt€ 0 *- Jo 4jC€,,0 .

20
Using (/), VfiW - z
47ce^/?^
0
.
2/118 Fundamental Physics (XII) VOL.l

32. Two charges and 92 are placed at (0,0, dj and (0,0, - d) respectively. FIGURE 2(N).16
Find locus of points where the potential is zero. AZ
Ans. In Fig. 2(N).16, we have shown two charges (0,0, d) and ^2 (0» ~ <0-
■ (0, 0. d)
For potential to be zero at (jic; y, z) we should have
Si = 0
4jte Q-^Jc^ + y^ + Cz-d)^ 47ceQ 4^^ + + (z + d)^ O

...(0
■^jc^ + y^ +(z-d)^ + y^ + (z + d)^ 92 4 {0,0,-d)
Clearly, total potential can be zero when q^, ^2 opposite signs.
Squaring both sides of (0, we get q^ [x^ + y^ + (z + d)^] = ^2 + y^ + (z - d)^]

ww
(2zd) + d2 = 0.
on simplifying, we get

Floo
This is the equation of a sphere with centre at 0, 0, ~ 2d If 9j = - 92, then z = 0

ee
reer
Therefore, locus of points where potential is zero is the plane through mid point of the two charges.

rFF
33. Two charges - q each are separated by distance 2d. A third charge + 9 is kept at mid point O. Find
potential energy of+9 as a function of small distance x fk'om O due to - 9 charges. Sketch P.E. v/s x
uur r
and convince yourself that the charge at O is in an unstable equilibrium. ffoor
Ans. In Fig. 2(N).17, two charges - 9 each are shown at A and B, where AB = 2 d A charge + 9 is kept at mid
sks
point O of AB. OP = is small displacement of + 9 charge. Therefore, potential energy of + 9 charge at P
YYoo
due to the two charges - 9 each is
ooko

FIGURE 2(N).17
1 2d -9 +9 -9
eBB

0 = ...(0
4jcSq d + x d-x 47t€Q (d^-a:^) A
o_^ B

The potential energy («) versus x graph is as shown in Fig. 2(N).28. K 2d ■H


r
ouur

dU -q^(2d)
ad

lx
Now m
dx 4tcGq (d2-jr2)2 FIGURE 2(N).18
Y

U
-29^
nddY

At X = 0, From (0, Uq = and fromCh) ^=0


Re

4jceQ d’ ’ dx
FFiin

.*. X = 0 is an equilibrium point


O
d^U (-ldq^\' 2 8x2 ■^x

dx^ y 4iC6o (d2-x2)3


Now

d'^U
djp-
J-2dq^ \
4jtGo ,(d}-x^f
1
[2(d2-x2)-8x2]

d^U _(-2dq^) 4(2d2)


1 =-
9^
which is less than zero.
At X = 0,
dr^ 4neo d^
Hence, the equilibrium of charge 9 at O is unstable.
ELECTROSTATIC POTENTIAL AND CAPACITANCE 2/119

SPECIAL

^r^ultimatej^pp'aratibn>S^his^uriit\for*Gompetitive:examinatibns ;*students:sh'6uid%?ir^^

ww
● MCQs In Physics for NEET
Pradeep's Stellar Series.... ● MCQs In Physics for JEE (Main)
rc

*^eparateiy/available'fortheseiexamihations; a:. 5!

Flo
e
Multiple Choice Questions (with one correct Answer)

rree
r FF
I. Electric Flux 10
(b) — V/|im and in neg. x direction,
uurr
1. A thin spherical conducting shell of radius R has
a charge q. Another charge Q is placed at the centre
of the shell. The electrostatic potential at a point at a
10
for
(c) — V/^m and in + a: direction
kss
distance R/2 from the centre of the shell is:
5
ooook

(d) - V/|xm and in neg. x direction.


Yo
2Q 2Q 2q
(a) (b)
4K€r.R 4K€^R 4ne,.0 R (AIEEE 2007)
eBB

0 0
4. An electric dipole with dipole moment
2Q
(.c)
4ti€q/? 4neQR U + j) is held fixed at the origin O in the
rr
ou
ad

presence of a uniform electric field of magnitude


id) (AIEEE 2003) £q. If the potential is constant on a circle of radius
Y

4tC€q/? R centred at the origin as shown in Fig. 2(CF).1 ?


dY

The correct statement is/are :


2. Assume that an electric field E =30x^ i exists
Re
innd

in space. Then the potential difference - Vq


where Vg is the potential at the origin and V^, the
Fi

potential at a: = 2 m is
(a) -80 V (b) 80 V
(c) 120 V id) - 120 V
(JEE Main 2014)
3. The potential at a point distant x (measured in
}im) due to some charges situated on the A:-axis is
20
given by v ix) = Volt.
x^-4
The electric field at a: = 5 ji m is given by
5
(a) - volt/iim and in + a: direction,

ANSWERS

l.(c) 2. (a) 3.(c)


2/120 ‘Pnadecfi’ Fundamental Physics (XII)

is permittivity of free space and R » dipole


size. a a^-b^ a
{a) — + c (b) - + c

^oL a
£oL b
,P0
(«)/? =
471 €(, o b-~c~ <5 b^-c^
+ a
(c) — + a

£o[ b
-

c
(b) The magnitude of total electric field on any 0

two points of the circle will be same, (JEE Main 2018)


(c) Total electric field at point A is 9. In a certain region of space with volume 0-2 m^,
—> /— —> A A the electric potential is found to be 5 V throughout.
The magnitude of electric field in this region is :
(a) 0-5 N/C ib) 1 N/C
(d) Total electric field at point B \s Eg = 0. (c) 5 N/C (d) zero (NEET 2020)
(JEE (Advanced) 2019)

ww
10. A short electric dipole has a dipole moment of 16
5. An electric charge 10“^ pC is placed at the origin
(0, 0) of XY Co-ordinate system. Two points A
X 10”^ Cm. The electric potential due to the dipole
at a point at a distance of 0-6 m from the centre of
and B are situated at (V2, -Jl) and (2, 0) the dipole, situated on a line making an angle of

Flo
respectively. The potential difference between the 60° with the dipole axis is :

e
points A and B will be

reree
1
(a) 4-5 V (b) 9V = 9xlO^Nm2/c2

r FF
47ce
(c) zero id) 2 W (AIEEE 2007) 0

6. Three concentric spherical shells have radii a, b (a) 200 V ib) 400 V
uurr
and c(a<b<c) and have surface charge densities
o. - G + o respectively. If Vg and denote
the potentials of the three shells, then for
foor (c) zero id) 50 V
11. The variation of electrostatic potential with radial
(NEET 2020)
ks s
distance r from the centre of a positively charged
c = <7 + /). we have
Yoo
metallic thin shell of radius R is given by the graph ;
ooook

(a) Vc=Ve=^A ib) V^=Vc^Vg


FIGURE 2(CF).3
eBB

(c) Vc=Vg:^V^ id) Vc=Vg^Vj^


(AIPMT 2009)
7. A charge Q is uniformly distributed over a long rod
rr

AB of length L as shown in the Fig. 2(CF).2. The ib)


ouu
ad

electric potential at the point O lying at distance L


from the end A is :
YY

■►r >7*

R R
FIGURE 2(CF).2
nndd
Re

A B Y4
O*
Fii

Q 32
ia) ib)
8716/,0 L 4;t6«0 L

32 Cln2 (NEET 2020)


(c) id)
47t6„0 Lin 2 4716/,0 L 12. Two charged spherical conductors of radius /?j tind
(JEE Main 2013) /?2 are connected by a wire. Then the ratio of surface
8. Three concentric metal shells A, B and C of charge densities of the spheres [Gj/G2l is
respective radii a, b and cia<b<c) have surface R?I R
1
charge densities + a. - g and + g respectively. («) TT ib)
The potential of shell B is R}
ANSWERS

4. ia.d) 5.(c) 6.ib) 7.id) 8.ib) 9. id) 10.(a) 11.(c)


ELECTROSTATIC POTENTIAL AND CAPACITANCE 2/121

R
I
ic) rr id) (NEET 2021)
R
I P2
13. A dipole is placed in an electric field as shown. In
which direction will it move ?

FIGURE 2(CF).4

+q -q

ww
(JEE Main 2015)

Floo
'A.

16. A capacitance of 2 pF is required in an electrical


circuit across a potential difference of 1-0 kV. A

ee
(a) Towards the right as its potential energy will
increase.
large number of 1 pF capacitors are available

reer
which can withstand a potential difference of not

rFF
(t>) Towards the left as its potential energy will more than 300 V.
increase,
The minimum number of capacitors required to
uur r
achieve this is
(c) Towards the irght as its potential energy will
decrease,

(f/) Towards the left as its potential energy will


(a) 24 ffoor
(&)32 (c)2 (^/) 16
(JEE Main 2017)
sks
17. A parallel plate capacitor has a dielectric slab of
YYoo
decrease. (NEET 2021)
ooko

dielectric constant K between its plates that covers


14. Twenty seven drops of same size are charged at
1/3 of the area of its plates, as shown in Fig.
eBB

200 V each. They combine to form a bigger drop. 2(CF).7. The total capacitance of capacitor is C
Calculate the potential of the bigger drop. while that of the portion with dielectric inbetween
(a)1980 V (b) 660 V is C|. When the capacitor is charged, the plate
r

area covered by the dielectric gets charge Qj and


ouur

(c) 1320 V (d) 1520 V


ad

the rest of the area gets charge 02- ^^e electric


(NEET 2021)
field in the dielectric is Ej and that in the other
YY

II. Capacitance and Capacitors portion is £2- Choose the correct option/options,
ignoring edge effects.
nndd
Re

15. In the given circuit, charge <22 *he 2 (iF FIGURE 2(CF).7
capacitor changes as C is varied from I pF to
FFii

3 |iF. Q2 as a function of C is given properly by : Ql

FIGURE 2{CF).5

.9l
1hF Q2 E2
c .P1
2mF
E E, E,
(«) (b)
E, ^2 K

ANSWERS
12.(c) 13.(c) 14.(a) 15.(b) 16. (b)
2/122 ^%euUe^'4- Fundamental Physics fXinPTSTWl
(a) 3 X C/m2 ib) 6 X 10^ C/m2
(c)
Q, _ 3 (^0 — =
C 3+K
(c) 6 X lO-"^ C/m2 (rf) 3 X 10^^ C/m2
^2 a: C,1
(JEE Main 2014)
(JEE Advanced 2014)
21. Three uncharged FIGURE 2(CF).10
18. Two thin dielectric
FIGURE 2(CF).8 capacitors of
slabs of dielectric P Q A
capacitance Cj, C2
constants and K2 + and C3 are

(ATj < K2) are inserted + connected as


between plates of a + Ci
shown in Fig.
parallel plate capacitor, +
2(CF).10 to one
as shown in Fig. + another and to
2(CF).8. The variation +
of electric field £ points A, B and D
+
between the plates with at potentials Vg

ww
distance d as measured Ki K2 and Vq. Then the
from plate P is correctly shown by potential at 0 will
be :

Floo
FIGURE 2(CF).9
EA EA
(a)
C, + C2 + C3

ee
ru

reer
_n “LJ (b)

rFF
C| + Cj + C3
uur r
(c)
0

EA
O
>d
0

EA
o ffoorC, + Cj + C3
sks
id)
YYoo
C^2 "i" ^2^3 ^3^1
ooko

u 22. A capacitor is charged by a battery. The battery is


eBB

removed and another identical uncharged


capacitor is connected in parallel. The total
0
^d electrostatic energy of resulting system
r

o o (a) increases by a factor of 4


ouur
ad

(b) decreases by a factor of 2


(AIPMT 2014)
YY

(c) remains the same


19. A parallel plate condenser with a dielectric of
(d) increases by a factor of 2 (NEET 2017)
constant K between the plates has a capacity C
nndd

23. The expression for the equivalent capacitance of


Re

and is charged to a potential V volt. The dielectric


slab is slowly removed from between the plates the system shown in Fig. 2(CF).ll is (A is the
FFii

and then reinserted. The net work done by the cross-sectional area of one of the plates) :
system in this process is: FIGURE 2(CF).11
1 A
(a) zero (i) -iK-l)CV^ ■*

cv2 (a:-i) d
(c) (d) (K-\)CV^
K
(AIEEE 2009)
d ^ d
20. A parallel plate capacitor is made of two circular
plates separated by a distance 5 mm and with a
dielectric constant 2.2 between them. When the
electric field in the dielectric is 3 x 10“^ V/m, the d* ^ *
charge density of the positive plate will be close ●4 3A
to

ANSWERS

17.(rt) 18. (c) 19.(a) 20. (c) 21.(b) 22. (b)


ELECTROSTATIC POTENTIAL AND CAPACITANCE 2/123

w -f-
FIGURE 2(CF).14
(a) e^A/Sd a

(c) (d) none of these A B


Cl C2
24. A combination of capacitors is set up as shown in
b
Fig. 2{CF).12. The magnitude of electric field,
due to a point charge Q (having a charge equal to C3 C4
sum of the charges on the 4 p.F and 9 |iF 30V
capacitors), at a point distant 30 m from it would
I- I- ^
be equal to

FIGURE 2(CF).12 (a) 5V Qj) 9V


3|iF (c) 10 V (ti) - 13 V

ww
4pF 27. The plates of a parallel plate capacitor with air as
medium are separated by a distance of 8 mm. A
medium of dielectric constant 2 and thickness

Floo
4 mm having the same area is introduced between
2pF
the plates. For the capacitance to remain the same,

ee
the distance between the plates is

reer
+
1 (fl) 8 mm ib) 6 mm

rFF
8V (c) 4 mm {d) 12 mm
(e) 10 mm (Kerala PET 2007)
uur r
28. Four identical capacitors are connected as shown
(a) 240 N/C
(c) 420 N/C
(b) 360 N/C
(d) 480 N/C
ffoor
in Fig. 2(CF).15. When a battery of 6 V is
connected between A and B, the charge stored is
sks
(JEE Main 2016)
found to be 1-5 ^ C. The value of Cj is
YYoo
25. A parallel plate capacitor with air as the dielectric
ooko

has capacitance C. A slab of dielectric constant K


and having the same thickness as the separation
eBB

between the plates is introduced so as to fill one-


fourth of the capacitor as shown in Fig. 2(CF). 13.
r

The new capacitance will be


ouur
ad

FIGURE 2(CF).13
Y

T
nddY

,● K ● 1 Air t
Re

i
FFiin

C
(a) (/sT +3) — (b) (^^ + 2)-
KC
(c)

(CET Karnataka 2007)


(fl) 2-5 F (b) 15|xF
26. Four capacitors with capacitances Cj = 1 ^iF,
(c) 1-5 pF (d) 0-1 \l F
C2 = 1-5 flF, C3 = 2-5 |Jp and C4 = 0-5 |iF are
connected as shown in Fig. 2(CF).14 to a 30 volt 29. A parallel plate capacitor with air between the
source. The potential difference between points plates has a capacitance of 9 pF. The separation
a and b is between its plates is d. The space between the
ANSWERS

23. {d) 24. (c) 25. (a) 26. (d) 27. (e) 28. (d)
2/124 Fundamental Physics fXlIIPZSnn
plates is now Filled with two dielectrics. One of capacitor. Then in the steady state, the charge on
the dielectrics has dielectric constant and the upper plate of the 3 F capacitor is
thickness d!2>, while the other has dielectric
constant ^2 = ^ thickness 2d/3. Capacitance FIGURE 2(CF).17
of the capacitor is now
(fl) l-8/?F {b) 45 pF
(c) 40-5 p¥ id) 20-25 pF +80pC _L
-r4pF
(AIEEE 2008)
30. A condenser of capacity C is charged to a potential
difference of V^. The plates of the condenser are
then connected to an ideal inductor of inductance
2|lF ^ =p3pF
L. The current through the inductor when the
potential difference across the condenser reduces

ww
to V'-, is
I

(a) c(y^-\^]2 {b)


C(V,--Vf )■

Flo
L L
{a) + 32 n C {b) -F 40 p C

ee
t (c) + 48 p C {d) -F 80 n C

rere
C(K,--fV^2^) C(V,^-V2^)^2 (ITT 2012)

rFF
(0 id)
L L 34. The electrostatic force between the metal plates
of an isolated parallel plate capacitor C having a
uur r
(AIPMT (Main) 2010)
31. A 2 pF capacitor is charged as shown in fig.
2(CF).I6. The percentage of its stored energy
foor charge Q and area A, is
{a) Proportional to the square root of the distance
sks
between the plates
dissi- pated after the switch S is turned to position
Yoo
2 is {b) Linearly proportional to the distance between
ook

the plates
BBo

FIGURE 2(CF).16 (c) Independent of the distance between the


plates
re

1 2

{d) Inversely proportional to the distance between


the plates
ouur

(NEET 2018)
ad

35. A parallel plate capacitor of capacitance 20 pF is


YY

V T
2pF 8pF
being charged by a voltage source, whose
potential is changing at the rate of 3V/i'. The
nndd

conduction current through the connecting wires,


Re

and the displacement current through the plates


FFii

(a) 0% (b) 20% of the capacitor would be, respectively


(c) 75% (d) 80% (IIT 2011) (a) zero, 60 |l4
32. A resistor */?’ and 2 pF capacitor in series are (b) 60 pA, 60 pA
connected through a switch to 200 V direct supply. (c) 60 pA, zero
Across the capacitor is a neon bulb that lights up {d) zero, zero (NEET 2019)
at 120 V. Calculate the value of R to make the
36. A parallel plate capacitor having cross-sectional
bulb light up 5 s after the switch has been closed,
area A and separation d has air inbefween the
(logjo 2-5 =0-4)
plates. Now an insulating slab of same area but
(a) 1-7 X I0^i2 (b) 2-7 X 10^ n
thickness dH is inserted between the plates as
(c) 3-3 X lO'^n (d) 1-3 X lO^n shown in Fig. 2(CF).18 havingdielectricconstant
33. In given circuit. Fig. 2(CF).17, a charge of K (= 4). The ratio of new capacitance to its
+ 80 p C is given to the upper plate of the 4 p F original capacitance will be :
ANSWERS

29. (f) 30. (d) 31. (d) 32. (b) 33. U') 34. (c) 35. (b)
ELECTROSTATIC POTENTIAL AND CAPACITANCE 2/125

40. A parallel plate capacitor is of area 6 cm^ and a


FIGURE 2(CF).18 separation 3 mm. The gap is filled with three
<—d—► dielectric materials of equal thickness with
dielectric constants = \0, K2 = \2 and = 14.
The dielectric constant of a material, which when
fully inserted in above capacitor, gives same
capacitance would be

FIGURE 2(CF).20

4
d/2

Ki K2 \ 3 mm
(a) 4; I (b) 2: \
(c) 8 :5 id) 6 : 5 (NEET 2020)

ww
37. A parallel plate capacitor has a uniform electric K ■H
6 cm^
field 'E/ in the space between the plates. If the
distance between the plates is'd' and the area of

Flo
each plate is ‘A’, the energy stored in the capacitor (a) 36 (b) 14
is (£q= permittivity of free space) (c) 12 (d) 4 (JEE Main 2020)

ee
E^Ad 41. A 5 ^iF capacitor is charged fully by a 220 V supply.

rere
(a) (b)-

rFF
^0
0
It is then disconnected from the supply, and is
connected in series to another uncharged 2-5 pF
capacitor. If the energy change during the charge
uur r
(c)e^EAd (^0

(NEET 2021)
foor redistribution is
X

100’;
, then value of X to the nearest
sks
38. The equivalent capacitance of the combination integer is
Yoo
shown in Fig. 2(CF).19 is (a) 3 (^)5
ook

(c)4 (^6
BBo

FIGURE 2(CF).19
(JEE Main 2020)
re

C
42. In the circuit shown in Fig. 2(CF).21, the total
charge is 750 pC and the voltage across capacitor
ouur
ad

C C2 is 20 V. Then the charge on capacitor C2 is


YY

FIGURE 2(CF).21
nndd

3C
Re

(0)- (b)3C C2
<72
FFii

C, *15pF
(c)2C
.of (NEET 2021)

39. Effective capacitance of parallel combination of ^3

two capacitors C| and C2 is 10 pF. When these


capacitorsare individually connected to a voltage C3 = 8pF
source of 1 V. the energy stored in the capacitor C2 +
o o
is 4 times that of C,. If these capacitors are V
connectedin series, theireffectivecapacitancewill
be
(a) 590 pC (/))450pC
(a) 3-2 pF ib) 8-4 pF
(c) l-6pF id) 4-2 pF (c) 650 pC id) 160 pC
(JEE Main 2020) (JEE Main 2020)

ANSWERS

36. (r) 37. id) 38. (c) .39. (r) 40. (c) 41. (c) 42. ia)
2/126 ^ Fundamental Physics (XII) lV»mi

43. Two capacitors of capacitances C and 2 C are 9 T 25 -5


charged to potential differences V and 2 V — CV^
respectively. These are then connected in parallel
in such a way that positive terminal of one is
connected to negative terminal of the other. The
(c) Zero
(rf) |cv2
final energy of this configuration is (JEE Main 2020)

D1 Multiple Choice Questions (with One or More than One Correct Ari8we^)j
44. A uniformly charged solid sphere of radius R has between the plates and work done on the system
potential Vg (measured with respect to «>o) on its respectively, in the process of inserting the slab
surface. For this sphere, the equipotential surfaces then:

. , 3Vq 5Vo , Vo ^ KAV


(a) e=-V- w e=-“

ww
with potentials — ^ and have
2 4 4 4 a d
radius /?j, /?2, R^ and R^ respectively. Then V
(c) E =
(a) /?! = 0 and /?2 > (^4 ~ ^3)

Floo
Kd
(b) and R2 > {R^ - Rj) e.AV 2 /
1 >
0

ee
(c) R, -0 and /?9 < (Ra - R-i) {d)W =
2d K
(d) 2R<R^ (JEE Main 2015)

reer
rFF
45. A charge + ^ is fixed at each of the points x = Xq, 48. Six point charges are kept at the vertices of a
jr = 3 x:o, a: = 5 JCg 00 on the X-axis and a charge regular hexagon of side L and centre O, as shown
- ^ is fixed at each of the points x = 2 Xq,
uur r
1 g
X = 4 atq, = 6 Xq .... to 00. Here Xq is a positive
constant. Take the electric potential at a point due
ffoor
in Fig. 2(CF).22. Given that K =
47ceg 1} ’
sks
1 Q which of the following statement(s) is (are)
YYoo
to charge ^ at a distance r from it to be correct ?
ooko

4tiGq r
Then the potential at the origin due to above FIGURE 2(CF).22
eBB

system of charges is: L


ia)Q (b) 00

t V
' \
r

/ \ p /
ouur

^ln2 / \ /
N
ad

\ \
(c) {d)
/ /
/ \ N
/

47teQXQln2 4ti€oXo / \ f
\
YY

t \ \
I
/ \ / \

46. A parallel plate air capacitor is connected to a / s \


/

A/ \D
\ /

battery. The quantities charge, voltage, electric


s /
+2<7# s
nndd

^-Iq
Re

0/
field and energy associated with capacitor are
s
/
\ \
/ /
\ \

given by Qq, Vq, Eq and Uq respectively. A


t /
FFii

\ I /
N / \
dielectric slab is now introduced to fill the space
/
/ \
/
N / \
/
between the plates with battery still in connection. \
\ /
/
%
/
/

The corresponding quantities, now given by Q, \ / \

V'c
\ / R
,V E and V are related to previous ones as; B ¥-
-9
(a) 2 > Go (i)V> Vg
(c) E>Eq id) U> Uq (a) The electric field at O is 6 K along OD
47. A parallel plate capacitor of plate area A and (b) The potential at 0 is zero
separation d is charged to a potential difference
V and then battery is disconnected. A slab of (c) The potential at all points on the line PR is
same
dielectric constant K is then inserted between the
plates so as to fill space between the plates. If (d) The potential at all points on the line ST is
2j£i W denote charge on each plate, electric field same (IIT 2012)
ANSWERS

43. {d) 44. (c,d) 45. (d) 46. (a.d) 47. (a,c,d) 48. (a,b,c)
ELECTROSTATIC POTENTIAL AND CAPACITANCE 2/127

DQI Multiple Choice Questions (Based on the given Passage/Compre hension)^


Each comprehension given below is followed by some multiple choice questions. Each question has one
correct option. Choose the correct option.

D We may define electro


It should be clearly understood that when a
dielectric slab is introduced inbetween the
static potential at a point in an electrostatic plates of a charged capacitor with battery
fleld as the amount of work done in moving connected across the plates,
a unit positive test charge from infinity to that (/) Capacity C increases, (ii) Potential V
point against the electrostatic forces, along remains constant, (Hi) Charge Q = CV,
any path. Due to a single charge g, potential increases, (iV) Electric field £ decreases,
at a point distant r from the charge is
1
——. The potential can be positive (v) Energy stored V =—CV^ increases.

w
V =
2
47tE(,r
or negative. However, it is a scalar quantity. However, when battery across the plates of

Flo
The total amount of work done in bringing charged capacitor is put off and dielectric
various charges to their respective positions slab is introduced inbetween the plates of the

reeee
from inHnitely large mutual separations gives capacitor, (i) Capacity C increases,
us the electric potential energy of the system (ii) Charge Q remains constant,

FFr
of charges. Whereas electric potential is Q
(Hi) Potential V = — decreases, (iV) Electric
measured in volt, electric potential energy is c
measured in joule. You are given a square of
for
field, E =:V xd decreases, (v) Energy stored
ur
each side 1*0 metre with four charges
+ 1 X 10-* C, - 2 X 10-* C, + 3 X 10-® C and U = decreases.
kkss
2C
-t- 2 X 10“* C placed at the four corners of the
Yo
square. With the help of the passage given Consider a parallel plate air capacitor with
oo

above, choose the most appropriate alter* area of each plate = 150 cm^ and distance
eB

native for each of the following questions : between its plates = 0*8 mm. With the help
of the passage given above, choose the most
appropriate alternative for each of the
49. Potential at the centre of the square is
r

following questions :
ou
ad

(a) 5-09 X lO^V (b) 5-09X lO^V


(d) 8-23 X lO^V
YY

(c) 5-09 V 51. Energy stored in the capacitor, when charged to a


50. Potential energy of the system of four charges is potential difference of 1200 V is
(b) -6-4x 10"^ J
ndd

(a) 12-73 X 10"^ J (a) 1-2 X 10^ J (b) 1-2 X 10^ J


Re

(c) 12-73 X 10'^ J (d) - 12-73 X 10"^ J (c) 3-6x lO^J (d) 3-6 X 10*^1
Fi

52. If the air capacitor is filled with a medium of


The capacity of a K=2 and then charged to the same potential, the
condenser increases both, when a conducting energy stored will be
slab or an insulating slab is introduced (a) l-2x lO^J (b) 3-6x ICr^J
between the plates of the condenser. In the
(c) 3-6x 10'’J (d) 1-2 X lO'*!
former case, electric field £ = 0 inside the
conductor and in the latter case, £ < Eq, 53. If the capacitor is charged first as an air capacitor
inside the insulator. Thus, potential and then filled with this dielectric,energy stored
difference V = E x d decreases and hence will be

capacity C = Q/V increases. (a) 3-6 X Kr’j (b) 1-2x10-^


(c) 4x 10-5 J (d) 4 X 105j
ANSWERS

49.(a) 50.(b) 51.(a) 52. (b) 53.(c)


2/128 ‘Px<xeiecfr'A Fundamental Physics (XlI)B&Qn

09 Matching Type Questions


DIRECTIONS. In each of the following questions, match column I and column II, and select the correct
match out of the four given choices.
54. Column I Column II
A. Electric intensity (f) due to a single charge (/?) CC
Mr
B. Electric intensity (£) due to an electric dipole oc I//
C. Electric potential (V) due to a single charge ir) OC Mr^

I
D. Electric intensity (E) due to a quadrupole (s)
OC
“T , where r is the distance

w
io) A-s,B-r, C-q,D-p {b) A-p,B-q,C-r,D-s (c) A-r, B-j, C-p, D-q {d) As, B-r, C-p, D-q

Flo
Matrix-Match Type Questions

reeee
DIRECTIONS. Each of the following two questions contains statements given in two columns, which
have to be matched. The answers to these questions have to be properly bubbled. If the correct matches

FFr
are A-r, As ; B-p, B-^ ; C-p, C-r and D-^, then the correctly bubbled matrix will look like the one shown
here :

for
ur
p q r s
11

A !
il©l0IO ®
kkss
B
io o o ©
Yo
oo

ci©j!®!© ©
eB

“!© O O ©
irr=T
r
ou
ad

55. Match Column I and Column II:


YY

Column 1 Column II
(A) In series combination of capacitors ip) pot. diff. across each capacitor is same
ndd
Re

(B) In parallel combination of capacitors (^) charge on each capacitor is same


(C) On inserting dielectric slab inbetween plates (r) capacity decreases
Fi

of a capacitor
(D) On replacing mica by air inbetween plates is) capacity increases
of a capacitor
56. Match Column I and Column II :

Column I Column II
(A) same + dimension of mass 0^) Electric charge
(B) negative dimension of mass Electric potential
(C) zero dimension of mass (r) Electrical capacity
(D) dimension of length is ± 2 (s) energy density

ANSWERS
54. id) 55. A-q, A-r ; B-p, B-r, C-r, D-r 56. A-q, As ; B-r, C-p, D-^, D-r
2/129
ELECTROSTATIC POTENTIAL AND CAPACITANCE

VI. Integer Type Questions


DIRECTIONS. The answer to each of the following questions is a single digit integer, ® @@©
ranging from 0 to 9. If the correct answers to the question numbers A, B, C and D ©oo©
(say) are 4, 0, 9 and 2 respectively, then the correct darkening of bubbles should
be as shown on the side :
© © ©@
©© © ©
57. Eight drops of water, each having a charge of 3 x 10"^ C having surface potential 2 V © ® © ®
coalesce to form a single drop. What is the surface potential (in volt) of new drop ?
©® © ©
58. An electric field is described as E = i x+kz.
© © © ©
The potential difference (in volt) between A (0, 0, 0) and B (2, 2, 0) is : ©00©
59. Two balls with charges 5 p C and ! 0 m C are at a distance of 90 m from each other. In 0® 0©

w
order to reduce the distance between them to 45 m, the amount of work to be performed
in joule is;
©0©®

Flo
VII. Assertion-Reason Type Questions

e
reee
FFr
FOR MEDICAL STUDENTS FOR ENGINEERING STUDENTS

DIRECTIONS. In each of the following questions DIRECTIONS. In each of the following questions

for
ur
read the two statements and choose if read the two statements and choose if
A. both. Assertion and Reason are true and the A. Statement-1 is true ; Statement-2 is true ; State-
meni-2 is correct explanation of Statement-1.
kkss
Reason is correct explanation of the Assertion.
B. both, Assertion and Reason are true, but Reason B. Statement-1 is true ; Statement-2is true ; State
Yo
oo

is not the correct explanation of the Assertion. ment-2 is not a correct explanation of Statement-1.
C. Assertion is true, but the Reason is false. C. Statement-1 is correct and Statemenl-2is false.
B

D. both. Assertion and Reason are false. D. Statement-1 is false and Statement-2is true.
re

63. Statement-1. In a serfes combination of


60. Assertion. A spherical equipotential surface is not
capacitors, charge on each capacitor is same.
ou
ad

possible for a point charge.


Statement-2. In such a combination, charge
YY

Reason. A spherical equipotential surface is


possible inside a spherical capacitor, cannot move only along one route.
ndd

(fl) A (b) B (c) C (d) D (a) A (b)B ic)C id)D


Re

(AIIMS 2015) 64. Statement-1. For practical purposes, the Earth is


Fi

61. Assertion. Electrons move away from a region used as a reference at zero potential in electrical
of lower potential to a region of higher potential. circuits.
Reason. Since an electron has a negative charge Statement-2. The electrical potential of a sphere
(a) A (b) B (c) C (d) D of radius. R with charge Q uniformly distributed
62. Assertion. Work done in moving any charge Q
on the surface is given by
through any distance on an equipotential surface 4JteA0 R
IS zero.

(a) A (b) B (c) C (d) D


Reason. An equipotential surface is very smooth.
(IIT 2008)
(a) A (b) B (c) C (d) D

ANSWERS
57. 8 58. 2 59.5 60. id) 61. (a) 62.(c) 63.(a) 64.(a)
2/130
Pn^Ueefi. ^ Fundamental Physics (XII) voL.r

k k

For Difficult Questions

^ Multiple Choice Questions (with o ne


Correct Answer)'

Kq -|l/3
1. Potential due to charge on the shell, Vj = R R =
Po
and potential due to charge at the centre 471 Gq£q_
_KQ_K2Q Choices (a) and (d) are correct.
^ E/1 R As Eq is uniform and electric field due to dipole

ww
is different at different points, so magnitude of
Net potential, V = Vj + V2 total electric field will also be different at different
^f^^KlQ points.
R R

Flo
^ g , 26

e
ine^R AksqR

ree
Fr
2. dV = ~E.dx = -30x^i.dxi =-30x^dx

rF
uurr
Va 2

^0
dV = - 30x^dx
0
s for
kks
r -j-|2
- lOx^
Yo
= -80 V
oooo

JO

20
eB

3. Here, V{x) = = 20(;c2-4)“*.


-4

dV Therefore, choice {b) is incorrect.


r

E = -
2KP KP _ 3KP
ou

dx
ad

Further, as ^,4 = R?' ^ R^~


YY

40;c

^2-4)2 3KP Pn -
(» + J)
nndd

R^ V?
Re

40x4 160 10
Atj; = 4|4m, £ = = — V/p m
(42 _4)2 ■ 144 9 ^ Choice (c) is also wrong.
Fi

Hence, the correct choices are (a) and (d).


E is along the positive x-direction.
5. Here, q = 10"^ ft C is at 0. A(V2,V2). and
4. Here, p = ^ B (2, 0) are as shown in Fig. 2(CF).24.

As the circle is equipotential, electric field at B


along tangent should be zero, Fig. 2(CF).23.
i-e., ErB = 0
-*

a: IP.0 I
and Eq = R^
R
or R^ = 0

^0 E0
4tc €qEq

k
ELECTROSTATIC POTENTIAL AND CAPACITANCE 2/131

FIGURE 2(CF).26
For Difficult Queatlona
L H dx
n
0
= 0A = + (V2)2 = 2 H L H
♦I
r2 = 0B -2
kdq
V -V = - i 0 Potential at 0 due to this element - —
4jceo Tj 47teo T2 Potential at O due to the rod,
(v rj =r2 = 2) 2L
6. As is clear from Fig. 2(CF).25 V=
aa ob Gc L ^ L { X L ^
yA = = -{a-b + c) Q
^ln2
^0 ^0 ^0 ^0 _ L gln2

w
As c =a +b 4 Its 4ite„0 L
0

8. {b): Refer to Fig. 2(CF).27

Flo
^0

reeee
FFr
for
ur
kkss
Yo
oo
eB

2 2
a a a a
b +c b+a+b
r

J ^ol ^
ou
ad

●y
Charge on sphere A = o 4 n
YY

O
— + a
_aa(a + b) _ Opac Charge on sphere B = - o 4 it b>^
€pL^? “sp b b
Charge on sphere C = a 4 7t
ndd
Re

a
2 a a^-b^ + c^
The potential of shell B due to charges on all the
+ C shells is
Fi

SpLc c ^oL c

g4K (-o4Tt62) ^ g47Cc2


o \,{a-\-b){a-b)+ (a + b)^ ^~4ne^b
0 4ii€r,b
0 4tt€jjCj
SqI- c
g -b^
+ c

■e^L b^
a .(a + b) ala
^C = {a-b +a+b) =
€p C ^0
9. Here, volume, v = 0-2 m^
We find that ^
Electric potential, V=5 volt (constant)
Choice (b) is correct. dV = 0
7. Consider a small element of length dx of the rod dV
at a distance x from O, Fig. 2(CF).26. Charge on As E=- E = zero
the element dr

Q 10. Here, /? = 16 x 10“^ Cm, V = ?


dq- — dx r = 0-6 m, 0 = 60°
^ L

I
2/132
‘Pnadcep.^A Fundamental Physics (XII) VOL.I

For Difficult Questions FIGURE 2(CF).29

V =
/JCOS 0 (9xl0^)xl6xl0-^ cos 60'
r2 (06)2
P
<● 4 ---►£
_ 9xl6x 1/2 ~q
= 200 volt.
0-6 X 0-6

II. In case of a thin shell of radius R, which is


positively charged, the charge can be assumed to
be at the centre of the shell. Inside the shell, r <
/?, the potential is constant, Potential energy of electric dipole in external
electric field is

w
47C€„r 4ice.,/?
0 0
U = -]}.£ =-;»£cosl80"= p E
Outside the shell, at any distance r from the centre.
On moving towards right, the strength of electric

Flo
field decreases, therefore potential energy
r
decreases. Net force on electric dipole is towards

reeee
Therefore, choice (c) represents correctly the right and net torque on it is zero. So the dipole
will move towards right.

FFr
variation of V with r.

12. When two charged conductors A and B are 14. Let r and R be the radius of smaller and bigger
connected by a conducting wire, they will share drop. When twenty seven small drops are
for
ur
the charges till they have the same potential. Let combined to form a bigger drop, volume remains
Q\.Q2hQiht charges on conductors A and B after the same, i.e..
kkss
sharing the charges.
Mil 4 1
Yo
3
oo

FIGURE 2(CF).28 3

(3 r)^ = R^
eB

or or R = 3 r

Let q be the charge on each small drop and Q be


the charge on bigger drop formed. According to
r

law of conservation of charge.


ou
ad

e = 27 q.
YY

As, so.
1 gj _ I ^2 Electric potential due to small charged drop,
4JC€q R^ 471€q R^
ndd

1
Re

- = 220V (given)
or
gj Q2 ^ 47C€o r
Fi

47t/?2ey 4tc/?|6 0
Electric potential due to bigger charged drop is
V = I Q_ I 21 q = 9
or = 9x220
4jie,,0 R 4jt€„0 3r 4jce„ r
^0 0

= 1980 V
or
^2
<^2 ^
15. Here C, = I pF, C2 = 2 ^iF, Cp = 2 + 1 = 3 pF
13. Refer to Fig. 2(CF).29, the direction of electric This combination is in series with C
dipole moment ~p is from -ve charge to +ve 3C
C
3+ C
charge. Angle between electric field E and
electric dipole moment 'p is 180°. Total charge, q = E =( c
3 + C

a
ELECTROSTATIC POTENTIAL AND CAPACITANCE 2/133

Ks^A 2s..0 A
0
For Difficult Questions e, = c,v = 3d
; Q2=c^v = 7>d

Charge on 2 |iF capacitor 9l = L


2 3CE ^ 2CE 2E Q^_~ 1
<i2 = ~x = Q, say
3 3+C 3+C 18. Electric field inside the parallel plate capacitor
c having charge Q at place in the absence ol'
dQ
>0.
<r~Q dielectric between the plates is £ =
Q
As
dC
T<0. As
dC- 0

therefore Q-y versus C graph is shown correctly Q


When dielectric is filled, then Ef —
in Fig. 2(CF).6(b). KAs
(I
16. Capacity of each condenser C = 1 pF.
As therefore, option (c) is correcL

ww
Potential difference it can withstand = 300 V

Required capacitance = 2 )iF across a potential


difference of 1-0 kV i.e., 1000 V 19. Initial energy =
2KC

FF loo
Minimum no. of capacitors in series.
0^
KMK) When the slab is removed, U =

ee
X = = 3-3 => 4 2C‘
300

Fr
When dielectric slab is introduced, energy
^ . ... 1 1 I I 1
Capacitance or each row, — = - + - + -+ -
‘ C, I 1 1 I becomes u
0^ = U.-
d 2KC I

rree
I
ur
If m such rows of capacitors are required, then
fofr oF
Net work done by the system = Uj— Uj = zero
c
ks
20. £ = or <j = KE()E
YYouro
Ke
C,.„ = 2 = nt C.. = /H X - or m = 8 0
soo

= 2-2 X (8-85 X 10-'^) X (3 X 10^)


Hence minimum number of capacitors
= 6 X 10"’ C/m2
BBook

= 8 X 4 = 32
21. Let q^, 93 charges on C|, Cj, Cy
r ee

The arrangement is shown in Fig. 2(CF).30 here.


respectively. From Fig. 2(CF).10,
oouru
ad

FIGURE 2(CF).30
-v;, or
9,=(V'a-Vo)C,
4 0 C1
Y

tHHHf
V -V _- 92
or Q2 = {Vb - Vq) C2
Ynd

8 rows
Re
FFindi

93 or
93 - ^0^ ^3
4 capacitors in each row ^^3
As 9] =92+93
17. When capacitor is charged, both pans of capacitor
have common potential difference V. So ●●● (V44 - V^o) ^^1 = ('"b - ^0) ^2 + (V/) - Vo) C3
E
V
I y^c,+v^c,-FV,,C3
7 = ^^ = 1
which gives, Vq =
=
or

^2 C, + C2 + C3
Ksr.A/3
0 :> e,,2A/3 22. If C is capacity and V is potential of battery, then
c, initial energy stored
d ~ d

KSryA
0 2g4vA
0 {K+2)s^A £, =Lcv- = LQl
C = C, +C2 = —
3d
+
3d 3d 2 2 c

C K+2 When uncharged capacitor of capacity C is


CI K
connected in parallel,
Cp = C+C=2C
2/134 <*. Fundamental Physics (XII)

For Difficult Questions


Xe.0 A 3e.0 A
●●● =
Ad Ad
Charge on each capacitor = QH
A C
Total energy E2 =
(0^2)2 ^ (02)2 = (X + 3)
0
= (X + 3)-
Ad 4
2C 1C

20^ _l( 26. In Fig. 2(CF). 14, let q \iC be charge on Cj and
<^2-
4(2C) 2^20
£2 = £1/2 ^ = ju
30 or — + — = 30
i.e., total electrostatic energy of resulting c/cj or j
system decreases by a factor of 2. ^ = 18 fi C

ww
Sa0 a
e„ A
0 r - _60 A V, ~V
q 18
— = I8V
Id ' ^3 =
23. C. =
3rf A a cl 1

As the three condensers are in parallel, Fig.

Floo
2(CF).ll. q'
Similarly, .^ + ^ = 30 or ~ = 30
2-5 0-5
^^3

ee
Sa A €a0 A €a0 A
C^ = C, + C2 + C3 = -^

eer
Id 3d , 25

rFrF
«? =
6a a ■ IIga a
C =-L 11 +—+—
1 1 0
p d 2 3 6d q' 25
rur
= 5
Choice id) is correct.
24. In Fig. 2(CF).12, 3 pF and 9 pF capacitors are in
ffoor ^3 2x2-5
ks
parallel. V^«-^i. = (^^-^i,)-(^^-V',) = 5-18
YYoou
= -13 volt
ookos

C^ = 3 + 9=l2pF 27. With air as dielectric,


This combination is in series with 4 pF capacitor
BBo

6q AxlO^
^0 —
re

C, = 3pF ...(0
C^~ n A~ 3' 8x10-3
In the new set up, let the distance between the
oouur

Charge on 4 jiF and combination 3 pF is


ad

plates be d' mm,


0=CV=3x8 = 24 pC
Y

Thickness of dielectric = 4 mm
This charge distributes itself in the direct ratio of
capacitors 3 pF and 9 pF. Thickness of air layer = (d' - 4) mm
nd dY
Re

(24) X 9 Ax1Q3 Ax103


= 18pC New capacity, - ^0 -!o
FFini

3 +9
4_^(rf'-4) id'-2)
Now 09jj:-+04jj/r= 18+ 24 = 42 pC 2 1

Electric field intensity due to 0 = 42 pC at a


distance of 30 m from it
As r = c /.
^0 Ax103 _€q Ax1()3
id'-2) 8
£ =
kQ _ 9x10^x42x10-^ = 420 N/C
r2 30x30 (f-l^S
<f = 8 + 2 = 10 mm
25.
- _^0 ^
Q 28. As is clear from Fig. 2(CF).I5, three rows of
0" d C
When one fourth of capacitor is filled with capacity Cj ; and Cj are connected in parallel
dielectric of constant K, Fig. 2(CF).13, then between the points A and B.
a: 6a A/4 6a 3A/4 C. 5
0
C.I = and Ca = -2—
d 3 d Total capacity, C = Cj + — + c1 =-c
2 ^
which are in parallel.
ELECTROSTATIC POTENTIAL AND CAPACITANCE 2/135

Initially, the charge stored in 2 )iF capacitor is


For Difficult Questions
Q. = CV=(2x l(T^) V=2Vx 1(H coulomb.
When switch S is turned to position 2, the charge
● l-5xI0“^ = -C,x6 flows and both the capacitors share charges till a
As q = CV 2 ● common potential V(~ is reached. Fig. 2(CF).31.
Therefore,
— xl0-^F = 0*lpF
30 FIGURE 2(CF).31

29. Here, C = = 9pF


(Q,--q)

oww
a
+ + + + + + + q
^^K.A 2pF 8pF
1 0
C,1 = = 3x3x9 = 81 pF
dl3 d

A^2 a a 3x6

ee
2d/3 ~~2 J' ^x9 = 81pF

FFrlo
total charge 2V x 10“^ V

r
As Cj, C2 are connected in series, V- = ^ = — volt.

rF
^ total capacitance (2 + 8) x 10“° 5

ee
1 ! 1 _J_
■■ Cs C,I C 2 ” 81 81 “ 81 Finally, the energy stored in both the capacitors

rF
ouru
1 fV \2 y2
xlO-^J
81 U
f ~ 2
-[(2 + 8)xl0-^) -

ffosor
C = —= 40-5pF 5 5
" 2 V /
os k
30. Here, = CVj and q = CV2 U:-U
I f
% loss of energy, At/ = xlOO
When a charged capacitor is connected to ideal t/..t
ook
Yo
inductor, the discharge of capacitor is oscillatory.
Y

The charge on capacitor at an instant t is given (y2-y2/5)xlQ-6


Bo

XlOO = 80%
reeB

1 y2xl0"6
by, q = qQ sin cot where, to =
4lc' 32. As, V=VQ(l-e-‘^^^) 120 = 200(1-6-5'^'^)
oouY
ur

120 -5/RC
ad

= 1-6
Therefore, sin cot = — or
200
CV.1 y
%
dY

Current through inductor is e-5/RC


120 2
= — or e 5/RC ^1^2-5
nidn

or
200 5 2
Re

T , ■ S
/ = — = — sin cot) Taking log of both the sides, we get
dt dt "
FFi

= ^0 ^
CO cos cot

“ sin^ COt]*'^ ^Iog^6 = log^2-5


5
-|l/2
RC
xl = 2-3026 logio 2-5 = 2-3026 x 0-4
1
= CV, X 1-
■n/lC V.1
R =
5 5

2-3026 X 0-4 X C 2-3026 x 0-4 x (2 x 10”^)


1/2

C{V}-V^) = 2*7 X 10^ a


L 33. In Fig. 2(CF).32, if Q2, Gs are the charges on C2,
31. Initially, the energy stored in 2 |xF capacitor is C3 respectively, then for the isolated system
marked, with dotted boundary.
i(2xl0-6)v2 =y2j^io-6j
1
(/. = _cy2 = Q2 + — 80 = 0
I
2 or
02+^3 = 80 ...(0
2/136
‘Pn^^eieefa.'4, Fundamental Physics (XII)CSm
e„,4
0 FIGURE 2(CF).33
For Difficult Questions C' =
n d
d-t 1- — K >1
K
FIGURE 2(CF).32
0
C' =
+80)XC 1
r C^=4!iF___ d-d!2 \~-
4;
-80|lC
€«, A
0 _€oA
1
Q2 Q3 dJl 2-1+-
4J 8

C2=2^F C3=3|iF c €oA d 8


da
c
17^ e«A 5

ww
0

C
— = 8:5
■■ C

Flo
As Cj and C3 are in parallel, therefore, potential 37. Volume of the space between the two plates of
difference across C2 and C3 is same, i.e.,

e
parallel plate capacitor, V = Axd.

reree
Energy density associated with electric field E in
Q2 _ £2 = —

r FF
or between the two plates of capacitor is
C2 C3 23 C3 3
W = -€n £2
uurr
9
2 °
Pul in (7), -23 + 23-80
foor Total energy stored in the space between the plates
of capacitor will be
ks s
^ 80x3
= 48pC
Yoo

^Gq £2x(/1xJ) E^Ad


5
ooook

U =uxV =
34. For an isolated capacitor, charge Q is constant.
The electrostatic force between the metal plates 38. Refer to Fig. 2(CF).34(a) the points I, 2 and 3
eBB

of 11 plate capacitor is are at the same potential (since they are connected
2^ by the conducting wire), so the capacitor between
f -
rr

plates points 1 and 3 is short circuited. Hence it will not


2^0 A
ouu
ad

store any charge, Le., it becomes ineffective. The


Clearly, F is independent of the distance effective circuit will be as shown in Fig.
YY

between the plates. 2(CF).34(/?). The equivalent capacitance of the


35. Here, C = 20p£=20x IQ-^F parallel combination of two capacitors is,
nndd
Re

= C+ C= 2 C
dV
Fii

Rate of change of potential. = 3 V/s.


dt FIGURE 2(CF).34
From q = CV C

dq dV
— = C-
d! dt
A C 2 B
/c = 20 X 10"^ X 3 = 60 X 10-^ A = 60 pA
As is known, Ia- = = 60 pA.
36. Original capacity. (a)
e„ A 1
0
C = ...(1)
d
2
When insulating slab of same area but thickness B B
dll is introduced between the plates as shown in
Fig. 2(CF).33, the new capacitance would be 3

{b)
ELECTROSTATIC POTENTIAL AND CAPACITANCE 2/137

,-r -
Final value of energy,
For Difficult Questions
\2
39. Here, Cj + C2 = 10 ^IF
Again
...(0 u
2 =^(<^1 =^(5 + 2-5)xl0-^ 220x-
3
£2 = 4 E^ «2 = 00806
X
=4xic. Au = U2-«i = 0 0806 - 0 1210 = -00404 =
2 ^ 2 ■' 100
X = 0-0404 X 100 = 4-04
C2 = 4 Cj
From (0. Hence, to the nearest integer, X = 4
Cl + 4 Cl = 10, Cj = 2 |XF + C2 = 8 liF 42. Here, total charge Q - 750 {iC
For series combination of these capacitors, As voltage across C3 = voltage across C2 = 20 V
Charge on C3 = ^3 = C3 x V = 8 x 20 = 160 |lC

ww
2x8
C =
= 1*6 llF Hence, charge on C2 = (750 - 160) |XC = 590 }iC
^ c,+C2 2+8
+
40. The three capacitors so formed are connected in 43. Here Q^ = CV

Flo
parallel. Therefore, total capacity of the

e
combination,

eree
2V
C = C| + C2 + C3
Q2 = 2Cx2V=4 CV

FFr
0
+ fo^ 2C
d 3d 3d 3d
uurr
orr
The two condensers are
FIGURE 2(CF).35
K^+K2-i-K^ _ 10 + 12+14 connected in parallel as
K =
3 3
= 12# sfo
shown in Fig. 2(CF).35. +
CV

41. Here, Cj = 5 ^iF = 5 x F, Vj = 220 V If V' is common potential,


kks
Yoo
then by charge conser
oooo

Initial energy =^CjV,^ = —x(5xl0 ^)(220)^ vation, we get


eBB

= 0-121 J <?,■ =

C2 = 2-5 |iF = 2-5 X 10-^ F, V2 = 0 02 + l2i = 4/ ■¥

4CV-CV=(C+2QV 4CV
urr

Final common potential


V=V
ad
Yo

V = 1 -
dY

Cj +C2 Final energy of this configuration = - (3C) V


5x220 + 2-5x0
innd

V = = 220 X-volt = - CV^


Re

5 + 2-5 3 2
Fi

Dl Multiple Choice Questions {with One or More than One Correct Answers)
kQ 1
R
44. Here, Vq = R
where k =
4 lie
On solving, =
0 V2
kQ _3kQ _kQ 4
When r> R, ,V When r= /?3, V OT R.:=-R
(r>fi) ^ ‘h 4 4 R 3 3
^3
kQ
Whenr</?, '^(r<R}~ OR'^-r^) V.
_I0_ I kQ _kQ
2R^ When r = R^, V 4 4 R
^4
At centre, r = 0, Vcentre ^kQ^3^ or
R^ = 4R, i.e., R^>2R
2 r 2 0
4 SR
5V.
When r = /?2> Vh 0 ^5kQ ^ kQ -/?2>
/?4 /?3 =4R--R
3
= —
3
>^22
4 4 R 2R}
2/138 7^n4z<Uep. Fundamental Physics (XII) P7STW1

1 1
For Difficult Questions
Work done W=AU =-CV^~-C'V'^
2 2
45. Potential due to positive charges

1 EqAV^ 1 ATeqAi^v
\2

V.1 =
2 d 2 d K
4Tceo Xq 4nGo 3xq V /

1 +-+
1 AV^ 1
47ceo^L 3 J 2d K

oww
potential due to negative charge 48. In Fig. 2{CF).36, L is the length of each side of a
1 1 regular hexagon ABCDEF, at the vertices of which
-q
^2 = six point charges are held.
47C€o 2Xq 4tc€o 4xq

ee
FIGURE 2(CF).36
+

FFrlo
47t€
0 -^0 L 2 4 6

r
/ \ P 7'^

rF
Total potential V = Vj + Vj
/ \

ee
\ ^\
/ s
/ N

a/
4 ,111
1 1 h +2<j/ s
\
ouru
rF
47ueoXo L 2 3 4 \ /D
\
\
\ /
' \2K /
i \
,111
ffosor \/
-R—2/C
1 —+ +
os k
= ln2
2 3 4 +q -<7

9 In 2
ook
Yo
V =
Y
Imagine a unit positive charge at O.
47t;€o -*^0
Bo

Electric intensity at 0 due to + 2 ^ charge at A


reeB

46. Since = CqVq and Q = CVq = Cq Vq 2qxl


= 2K, along OD
Q 4nenl?
oouY

So 0
= e^, i.e., Q>Qo
ur

Qo Electric intensity at 0 due to - 2 ^ charge at D


ad

= 2 K, along OD.
Y
d

1
Now U Electric intensity at O due to + ^ charge at B and
0 2
nidn

- q charge at £ = 2 K, along OE
Re

I 1 Electric intensity at 0 due to - ^ charge at C and


t/= - CV^2 = - e
+ q charge at F = 2 K along OC
FFi

2 2^0
As ZCOE = 120°, therefore, resultantof 2 K along
U OC and 2 K along OE is 2 K along OD.
U
= i.e., V >Uq
0 Net electric intensity atO = 4K + 2K = 6K
along OD
V and £ remain same.
dE
Potential at 0, V = - — (6 A’) =Zero
47. As battery is disconnected, charge remains dr dr
same
A closer look at figure shows that PR is equatorial
6^0 AV line of three electric dipoles (- q, + ^) at £, F ;
Q = cA = ^EqA =Eq ^ A = d (- 2 ^, + 2 at D, A and (- q, + q) at C, B.
Hence potential at all points on PR is zero (i.e.,
As £ = £o _ V same).
K' K ~ Kd The choices (a), {b\ (c) are correct
ELECTROSTATIC POTENTIAL AND CAPACITANCE 2/139

For Difficult Questions

Qn Multiple Choice Questions (Based on the given Passage/Comprehens ion)

49. As is clear firom Fig. 2(CF).37 51. Here, A = 150 cm2 = 150 X 10^ m2
d = 0-8 mm = 8 x 10^ m,
FIGURE 2(CF).37
A B Vq = 1200 V
S /
\ ✓
s
N
N /

r -^0^
\
S ✓
/
d

ooww
» 1-Om
8-85 xl0~’2x 150x10"^
/ \

8x10^
✓ S

✓ N
✓ S

D = l-66x lO-'Op

e
->12 _ 1 =iCgVo2 =1x1-66x10-^0(1200)2

ere
OA = OB=OC = OD = m

rFl
2 Vi

Fre
Potential at O = lxl'66xl0-^0
2
(1200)2

rrF
Zq = 1-2 X 10--* J
V =
47C6,,0 r 52. With dielectric medium, new capacity becomes
sffoo
ouur
C=/i:Cn0 =3x 1-66X lO-^Op
9x10^ (1x10-*-2x10"®+3x10'®+2x10“*)
kosk
1 1
i/Vi Energy, -cy2
U =-
YYo
= 9 X 10^ X 4 X 10-® Vi = 5 ● 09 X 102 V
oo

1
BB

= K.y-CQV^ = 3x1-2x10^1
50. As Potential energy =
47cer.0 r
= 3-6 X 10--* J
rre

Potential energy of the system of charges 53. When the capacitor is charged first as an air
ouu

= 9x lO^x capacitor, and then filled with dielectric, its


Y
ad

potential becomes
(lxl0-®)(-2xl0“®) , (-2xlO-®)(3xlO“®)
dY

Vn0 1200
10 1-0 y = = 400 volt
K 3
innd
Re

^ (3xlQ-®)(2xl0-®) ^ (2xlO-®)(IxlO“®) 1
1-0 10 y =-cy2 -\kc\
FFi

2
+ 9x lO^x

(Ixl0“®)(3xl0“®) (2xIQ-®)(-2xl0~®) 1^0


2 K
S
1-2x10-^
9xl0^(-10*^) = 4xl0~® J
= 0+ = - 6*4 X 10-2 j 3
Vi

09 Matching Type Questions


1 1 1
-r- ; for an electric dipole, E ; for a quadrupole, E —. The electric
54. Due to a single charge, E oc oc —— OC

r3 ’
1
potential (V) for a single charge OC —
. Choice (d) is correct.
r
2/140 'P^deep. 4. Fundamental Physics (XII) LV*1WI

For Difficult Questions

m Matrix-Match Type Questions


55. In series combination of capacitors, charge on 56. Dimensional formulae of various quantities in
each capacitor is same ; and decreases. In column II are
parallel combination of capacitors, pot. diff. (2 = [ATI;
across each capacitor is same and Cp increases. V=[ML^T~^ A~^]
On inserting dielectric slab inbetween plates of a C=[AT* L-'^T^A^]
capacitor, its capacity increases. and C/= [A/Z.-* r-2]
On replacing mica by air inbetween plates of a
The given answer stands verified.
capacitor, its capacity decreases.

ww
VI.
Integer Type Questions
57. Let r be the radius of each small drop and R be (0.0,0)

Flo
the radius of one big drop. (xdx + zdz)

ee
4 1 4 ●, (2.2,0)
— =8x —Ttr^
3 3

rere
21(0.0.0)

rFF
= (2 r)3 or R = 2r V
Z
AB ~
Now g = CV=4neQrV I 2 2 -1(2,2,0)
uur r
r —
<1
4%e,^V
0
9x10^x3x10"^
2
27
foor
— m
2
= Y [(0^ - 2^) (Q2 - q2)] = 2 volt
sks
27
59. Here, ^i=5|iC = 5x 10"^ C
Yoo
/? = 2r = 2x—= 27m.
ook

2
^2 = 10mC = lOx lO-^C
Surface potential of new drop
BBo

rj = 90 m, T2 = 45 m, W = ?
y _ total charge 8^
_ ?2 ‘?i ^2
re

capacity AuSqR W=U2-U^


47t6p T2 47tGo r,
_ 8x3xlQ-^x9xlQ^
ouur

= 8 volt
ad

27 _ 4i42
4ti:e
YY

58. From E =
-dV 0 /2 n
dr
= (5xl0"^)x(10xl0"^)x9xl0^
nndd
Re

dV =-E.d7
A (0.0.0) 1
FFii

dV=- W=450x—= 5J
{i X + k z).{i dx + jdy + kdz) 90
B (2.2.0)

VII.
Assertion-Reason Type Questions
FOR MEDICAL STUDENTS FOR ENGINEERING STUDENTS
60. Here Assertion is wrong and Reason is also 63. Both the statements are true and statement-2 is
wrong. correct explanation of statement-1.
Q
61. As electron has a negative charge. That is why it 64. As V =

moves from a region of lower potential to a region Aheq R


of higher potential. As radius of earth (R) is very large compared to
62. Assertion is true but the reason is false. the size of bodies we deal with, Iherefoe, potential
of earth is taken as zero. Choice (a) is correct..
w
e
orow
re
F
uFFllryj^Du

ree
a.

sFr
?

kro
uor
t;^if"
offo
H) IM '^rw
L X, jt
kos
ML ii
YYo
eerBB
oo
rY

CHAPTER 3.
uu

CURRENT ELECTRICITY
ad
doo
nY
nid
Re
FFi
w
CURRENT ELECTRICITY

Flo
ee
3.1. INTRODUCTION

Fr
In the first unit on ‘Electrostatistics’, we have studied various characteristics of electric charges at rest.
However, most of the practical applications of electricity involve charges in motion. For example, an electric

for
ur
bulb glows when charge flows through it, an electric fan rotates when charge is made to pass through its coil.
Again, our radio and T. V. sets operate on flowing charges, and so on. The flowing charges are said to constitute
electric current. We shall study later that electric current is of two types : direct current (d.c.) and ‘alternating
ks
Yo
current (a.c.). Most of the practical devices use alternating current where charges flow back and forth through
oo

conductors. However, we start studying ‘charges in motion’ through conductors in one direction. In this
B

chapter, we shall confine ourselves to the study of direct current circuits.


The branch of Physics, which deals with the study of charges in motion is called Current Electricity.
re

3.2. CURRENT CARRIERS


ou
ad

The charged entities whose flow in a definite direction constitutes the electric current are called current
Y

carriers. For example,


(i) In solids, which are good conductors, e.g., metals, the current carriers are the valence electrons.
nd
Re

These electrons are not firmly attached to individual atoms, and are free to move throughout the volume of
Fi

the conductor.

In semiconductors like silicon and germanium, current carriers are negatively charged electrons and
positively charged holes.
In insulators like glass, ebonite etc., there are no free charge/current carriers.
(m) In liquids like NaCl, current carriers are positively charged Na"*" ion (called cation) and negatively
charged Cl" ion (called anion). Similarly, in CUSO4 solution, the current carriers are Cu"*^ ion and SO“ ions.
Such liquids through which electricity can pass are called electrolytes. Thus, in an electrolyte, electric current
is due to simultaneous drifting of cations and anions in opposite directions,
(lit) In Gases under normal conditions, there are no charge/current carriers as they are insulators. However,
gases can be ionized by applying high potential difference at low pressures or by exposing them to X-rays. In
that event, current carriers in gases would be the electrons and positive ions of the gas.
3/1
3/2 Fundamental Physics (XII) L^A!WI
3.3 ELECTROMOTIVE FORCE

Electric current is possible in a closed circuit if there is a source of external force (infact a source of
energy) which compels the current carriers (electrons or ions) to move in a definite direction (i.e. from lower
potential energy to higher potential energy.) This external force which makes the current carriers to move in
a definite direction is called electromotive force (emf). The emf is not a force at all. It is a special case of
potential difference. Due to this, it has the nature of workdone per unit charge.
Thus the electromotive force is associated with an arrangement or mechanism which can supply energy
or does work to move the electric charge from lower potential energy to higher potential energy. Such an
arrangement is called a source of emf, which may be a cell, a battery, a generator or dynamo. It is a seat of
power.

E,M.F. of a cell is defined as the maximum potential difference between the two electrodes of

ww
the cell when no current is drawn from the cell or cell is in the open circuit.

It may also be defined as the energy supplied by the cell to drive a unit positive charge once around the

FF loo
complete circuit and is given by

ree

e = () E.dl

rree F
->

where, £ is the electric field (i.e., force per unit charge) and dl is the small path segment vector.
ur
Symbolically a cell is represented by two unequal parallel lines having a small
fofr oF
separation. Fig. 3.1. The taller Line represents the positive electrode P and smaller line
FIGURE 3.1

p N
represents the negative electrode N.
ks
T ^
The SI unit of emf of a cell is volt (V) or joule per coulomb (JC“^). The emf of
YYouro
soo

a cell is said to be one volt, if 1 joule energy is supplied by the cell to drive one coulomb of charge once
around the whole circuit (including the cell).
BBook

The emf of a cell depends upon the nature of electrodes, nature and the concentration of electrolyte used
r ee

in the cell and its temperature.


oouru
ad

3.4. DIFFERENCE BETWEEN ELECTROMOTIVE FORCE AND POTENTIAL DIFFERENCE


Y

ELECTROMOTIVE FORCE POTENTIAL DIFFERENCE


Ynd
Re

1. It is related with the source of current. 1. It is related with any two points of a closed circuit.
FFindi

2. It is a cause of potential difference between two 2. It is an effect.


points of a circuit.
3. It is the amount of workdone in taking a unit 3. It is the amount of workdone in taking a unit
charge once round the complete circuit charge from one point to another.
4. It is also defined as the maximum potential 4. It will exist between any two points in a circuit,
difference between two electrodes of a cell when when current is sent by the cell in the circuit.
no current is drawn from the cell or cell is in
the open circuit
5. It is equal to sum of potential differences across 5. In a closed circuit, the different components of a
all the components of a closed circuit, including circuit will have different potential differences.
to that of cell.
6. It is independent of the external resistance and 6. It depends on the resistance between two points
current drawn from the cell. of circuit and current flowing through it.
CURRENT ELECTRICITY 3/3

3.5. ELECTRIC CURRENT

As is known, the flowing charges are said to constitute electric current. Quantitatively, electric current
is defined as the rate offlow of electric charge through any cross-section of a conductor, i.e.,

Electric current =
total charge flown _ q
time taken t

Oftenly, the currents are not usually steady. Therefore, more generally, we say that if AQ = net charge
flowing across a cross section of a conductor in a particular direction during the time interval A/ [i.e., between
times t and (/ + A/)], then the current
C
/ (t) = Limit
^Q\_dQ
At dt

ooww
dQ
Here, is first derivative of 0 w.r.t. t. Thus, current through a conductor at anytime is defined as
dt
the first derivative of charge with respect to time passing through a cross section of the conductor in a
particular direction.

e
SI unit of electric current is Ampere. It is also called the practical unit of current.

ree
rFl
1 ampere (A) =
1 coulomb (C)
= 1 Cs-^

Fre
1 second (s)

rrF
Thus, current through a conductor is said to be 1 ampere, if one coulomb of charge is flowing per
second through any cross section of the conductor.
ouur
sffoo
1 coulomb (the standard unit of charge) = charge due to 6-25 x 10^^ electrons.
In solid conductor, there are free electrons called conduction electrons, which are called current earners.
okks
They carry the charge through the circuit because they are free to move throughout the atomic network of the
YYo
conductor. In solid conductor, the protons are bound inside the atomic nuclei which are more or less locked
oo

in fixed positions. Under the effect of electric field applied, the conduction electrons move in a fixed direction
BB

and constitutes the current.


rre

The current carrying wire does not have a net electric charge because the number of electrons
(negatively charged electrons) in the wire is equal to number of positive protons in the atomic nuclei of the
ouu
Y
ad

current carrying wire. When electrons flow in a wire, the number of electrons entering per second at one end
of wire is equal to the number of electrons leaving per second at the other end. As a result of it, the net charge
dY

of the wire is normally zero at every moment.


Total current in a circuit is equal to the sum of the currents due to motion of all charge earners. In a
innd
Re

circuit if n j number of electrons move from 5 to A per second through a conductor and 7I2 number of protons
FFi

(or positively charged particles) move from A to 5 per second through the same conductor. If e is the charge
of electron or proton, then the electric current. / - n^e + n^e - (wj ^ nf) e
In case an electron is revolving in a circle of radius r with speed v, period of revolution of electron is
2nr
T =
V

1 V

Frequency of revolution, ^ ~
27ir

Current at any point of the circular path is ;


/ = charge flowing in one revolution x number of revolutions per second
ev
= ev =
Ittr
3/4 Fundamental Physics (XIDQSIS
Direction of Current.

As is known, current carriers in conductors such as metals


FIGURE 3.2
are valence electrons, which carry negative charge. When a source
of potential difference such as a cell is connected to a conductor .
A
Electronic Current
<4 B
AB, Fig. 3.2 free electrons flow from negative end B to positive A
*
end A. They are said to form electronic current which is along .e
* \1 J-
BA, i.e., from lower potential to higher potential. However,
normally water flows from higher level to lower level and heat Conventional Current
flows from a body at higher temperature to another at lower
temperature and so on. To maintain this convention, we define
what is called conventional current; which is equal and opposite

oww
to the electronic current. It would flow along AB from higher
potential to lower potential.
In all our discussions, electric current will mean only the conventional current which flows from positive
end (higher potential) to negative end (lower potential) in the closed circuit.

e
Retain in Memory

rFFlo
re
When cross sectional area is different at different points of the conductor, the magnitude of current

ree
F
through the conductor, / = dQ/dt remains the same, at every cross-sections.

rF
Current is a scalar quantity. Though the electric current represents the direction of flow of positive
charge, yet it is treated as a scalar quantity because current follows the laws of scalars addition and not the
fsfoor
ouur
laws of vectors addition. The angle between the wires carrying currents does not affect the total current in the
kosk
circuit.
FIGURE 3.3
Fig. 3.3 shows a conductor AS with current/q. It C 11
Yo
is splitted at a junction S into two branches SC and
oo

C
Y

I1
BD, carrying currents /j and l2- Since the charge is
BB

conserved, the sum of the magnitudes of the currents A- » A' la B,


rre

●D
in the two branches must be equal to the magnitude of
the current in the original conductor. So O o h
oYuu

■D
h
/0 = /i+/2
ad

The above relation is valid even when there is bending or reorienting the conducting wires in space as
dY

shown in Fig. 3.3(S).

Sample Problem D What is the current flowing through a conductor if 1 million electrons
innd
Re

are

crossing in 1 millisecond through a cross section of it from A to 5 ? Charge on an electron = 1*6 x C.


FFi

Sol. Here, lO^x l-6x 10-^^C = l-6x 10->3c;f = I0“3s


16xl0~*^
Now, current / - — = = 1-6 X 10-^® A
t 10-3
The direction of current in conductor is from B to A.

Sample Problem Q JTig. 3.4 showS a plot of Current / I(A)t


FIGURE 3.4

through the cross-section of a wire over a time interval of 10 s.


Find the amount of charge that flows through the wire during
this time period. [CBSE OD 2015 (C)J
Sol. Amount of charge that flows in 10 seconds is
= area under the I-t graph
1
= -x5x5-l-(10-5)x5= 12-5 + 25 = 37-5 C >f(s)
CURRENT ELECTRICITY 3/5

3.6 ELECTRIC CURRENT IN CONDUCTORS


In conductors, atoms/molecules are so closely packed, that valence electrons are no longer attached firmly to
individual atoms. Some of these electrons are free to move within the bulk material, like molecules in a gas.

When no electric field is applied, free electrons have thermal motion. They collide with fixed ions in the
material and emerge with the same speed. However, the direction of electron velocities after collision
is completely random. As the average thermal random velocity of electrons is zero, therefore, there is no
current.
FIGURE 3.5

Fig. 3.5 shows a solid cylinder across which an electric field is applied A B

by attaching positively charged and negatively charged discs of a dielectric at A A


the two ends. Due to this electric field, free electrons in the conductor are -Q +Q

accelerated towards the positive disc, and a transient current (i.e., conventional \1 \J
current) is set up along BA. This current persists till the charges ± Q on the Solid Conductor

ww
discs are neutralized, and there is no current there after.
If we apply a steady electric field in the body of conductor by connecting a cell or a battery across the
two ends of the conductor, there will be a steady current in the conductor.

FF loo
3.7 DRIFT VELOCITY

ee
A metal conductor contains a large number of free electrons or conduction electrons, whose number

Fe r
density {i.e., no. of electrons per unit volume) is about 10^^ m“^. At room temperature these electrons move
at random within the body of the conductor, like the molecules of a gas. The average thermal speed of the free

Frre
electrons in random motion at room temperature is of the order of 10^ ms~^ The directions of motion of these
r ur
u
fforo
free electrons are so randomly distributed that the average thermal velocity of the electrons is zero. If
are random thermal velocities of n free electrons in the metal conductor, then the average
ks
YYouo
thermal velocity of electrons is
koso

Uj + «3+ ●●●● + «, = 0 ...(2)


BBoo

As a result, there will be no net flow of electrons or charge in one particular direction in a metal conductor,
r ee

hence no current.
ad
oouur

When some potential difference is applied across the two ends of conductor, say a copper wire ; an
electric field is set up inside the conductor. The free electrons in the conductor experience a force in a
Y

direction opposite to that of electric field and are accelerated from negative end to positive end of the conductor.
On their way, the accelerated free electrons suffer frequent collisions against the copper ions/atoms, lose their
dnYd
Re

gained kinetic energy and again get accelerated due to electric field and lose the gained kinetic energy in the
FFini

next collision and so on. The process continues till the electrons reach the positive end of the conductor.
Thus under the effect of electric field applied, the free electrons acquire small velocities by virtue of
acceleration due to electric field in addition to their random thermal velocities. These velocities are in different
directions. Therefore, each free electron describes a curved path between two successive collisions. Fig. 3.6.
The average velocity of all the free electrons is no longer zero. It has some little value towards the positive
end of the conductor, which is called drift velocity. Thus,

Drift velocity is defined as the average velocity with which the free electrons get drifted towards
the positive end of the conductor under the influence of an external electric field applied.
The drift velocity of electrons is of the order of 10'^ ms“*.
If V is the potential difference applied across the ends of a conductor of length /, the magnitude of
electric field set up is E =
potential difference _ V
length I
3/6 “Pn^tdee^ '4. Fundamental Physics (XII) IWI
The direction of this electric field is from positive end FIGURE 3.6
to negative end of conductor. Since the charge on electron is
- each free electron in the conductor experiences a force,
F=~eE +

Here the negative sign shows that the direction of force


is opposite to that of electric field applied. If m is the mass
of an electron, the acceleration of each electron is

eE
a ...(3)
m

Due to this acceleration, the free electron, apart from its thermal velocity, acquires additional velocity
component in a direction opposite to the direction of electric field. However, die gain in velocity of electron
due to electric field is very small and it is lost in the next collision with the ion/atom of the conductor. As a

ww
result, the acceleration of an electron is not proportional to the external electric field applied [with reference
to equation (3)]. The positive ions also experience a force due to electric field but they cannot move as they
are heavy and tightly bound in the metal.

Flo
At any instant of time, the velocity acquired by electron having thermal velocity mJ will be

ee
—^ >

Uj = Mj + a T 1

rere
rFF
where Xj is the time elapsed since it has suffered its last collision with ion/atom of the conductor.
Similarly, the velocities acquired by other electrons in the conductor will be
uurr
V2=U2+aX2, ^3 =^3+
foor v_
n
=M_+
n
ax n '

The average velocity of all the free electrons in the conductor under the effect of external electric field
ks s
is the drift velocity of the free electrons,
Yoo
oook

i -=> ->

Thus, V
U, + U2 + ....+V n
(mj + a Tj) + (m2 + a ^2) +
eBB

d ~
n
n

Mj + «2 + ... + m n (T,+T,+... + T„)


rr

+ a = 0-\- ax= ax [from (2)]


ouu
ad

n n

Tj +X2 + ... + X
YY

where, x -
n
= average time that has elapsed since each electron suffered its last collision
n
nndd

with the ion/atom of conductor and is called average relaxation itme. Its value is of the order of 10”^'^ second.
Re

Putting the value of a in the above relation, we have


Fii

-eEx
V
d
...(4)
m

eE
Average drift speed, V
d
m

j —^
In (4), the negative sign shows that is opposite to the direction of E .

Note that, _Ayej;age Relaxation itme - mean free path of electron/drift speed of electron.
3.8. MOBILITY

Mobility pf charge carrier (}i), responsible for current is defined as the magnitude of drift
velocity of charge per unit electric field applied, Le.
CURRENT ELECTRICITY 3/7

^ _ drift velocity _v^ _ qEiJm


electric field
qx
E E m

where x is the average relaxation time of the charge while drifting towards the opposite electrode, and m is the

w
mass of the charged particle.

Mobility of electron, =
m
e

Mobility of holes (i.e. positive charge carriers present in semiconductors) will be given by, P;, =

wr
where, x^ and X;, are average relaxation time for electrons and holes respectively,

e
and refer to
mass of electron and mass of hole respectively. Charge on either is e.

r
Mobility is positive for both, positive current carriers and negativecurrent carriers, although their drift

oo
velocities are opposite to each other.

F
llu
The convential current in a conducting material TABLE 3.1. Mobility of electrons
due to motion of negative current carriers will be in and holes in some materials
the same direction as that due to positive current

FF
carriers. Therefore the total current in the conducting Materials Electrons Holes

ees
material is the sum of the currents due to positive mobilities mobilities

rr
current carriers and negative current carriers.
uro (cm2 (cm2

Fk
If there are only electrons in a conducting material

oo
Diamond 1800 1200
as current carrier, then the drift velocity of electron is
Silicon 1350 480
Germanium

fofr
3600 1800
SI unit of mobility is m^ s"’ V“^ or m s“^ N“' C
so
Y
InSb 800 450
oY
Mobilities of electrons and holes in some mater GaAs 8000 300
B

ials, at room temperature are given in Table 3.1.


ok
Yo
eBr

3.9. RELATION BETWEEN CURRENT AND DRIFT VELOCITY


rue

Consider a conductor (say a copper wire) of length / and uniform area of cross-sectionA.
oud

Volume of the conductor = Al.

If n is the number density of electrons, i.e., the number of free electrons per unit volume of the
o
ad
Yn

conductor, then total number of free electrons in the conductor = Ain. If e is the charge on each electron,
then total charge on all the free electrons in the conductor. q -Alne
di

Let a constant potential difference V be applied across the ends of the conductor with the help of a
Re
FF

battery.
in

The electric field set up across the conductor is given \>y E= V/l (In magnitude)
Due to this field, the free electrons present in the conductor will begin FIGURE 3.7
to move with a drift velocity towards the left end A, as shown in Fig. 3.7. /
.'. Time taken by the free electrons to cross the conductor, Af <=T=e
/ = l/Vj
Alne I I
q
Hence, current, / = — = or
I = A nev^ ...(5)
t
!/Vj
+

Because, £, so
I — Ane Pg E
Equation (5), gives the relation between the current flowing through the conductor and dril velocity of
the electrons.

Putting the value of =


eEx
m
in (5), we have ^ _ Ane^x E m
3/8 ‘Pn.adee^ Fundamental Physics (XII) iWI

Estimate the average drift velocity of conduction electrons in a copper


wire of cross-sectional area 2*5 x 10“^ carrying a current of 2*7 A. Assume the density of conduction
electrons to be 9 x 10^ m”^. (CBSE OD 2014)
I
Sol. Here, A = 2-5 x 10”^ ; / = 2-7 A ; n = 9 x 10^® m~^ \ I = nAev^ or v d ~
nAe

2-7
V - 0-75 X 10 ^ ms ^ = 0*75 mms ^

w
(9xl0^^)x(2-5xl0"’)x(l-6xl0“*^)

3.10. OHM'S LAW FIGURE 3.8

It is the most fundamental law of electricity.

e
orow
Ohm's law states that the current (I) flowing through a conductor is

re
directly proportional to the potential difference (V) applied across the
ends of the conductor, provided physical conditions of the conductor

F
such as temperature, mechanical strain etc, are kept constant.

uFFll
ree
V „
i.e.. loc V or Voc/ or V=RI or — = R = a constant ...(6)

sFr
I

kro
where R is known as resistance of the conductor. Itdepends upon the length, shape and the nature of material
uor
of the conductor. R also depends on temp, of the conductor. Thus, from Ohm’s law, the variation between
offo
potential difference (V) and current (/) through a conductor is represented by a straight line OA having a
constant slope as shown in Fig. 3.8(<3).
kos
Y
Yo
If the V-I graph for the two conductors A and B is of the type as shown in Fig.3.8(i), then the slope of
eerBB
oo

V
graph = — = R (resistance). It shows that the higher is the slope, the higher is the resistance. So the resistance
rY

of conductor B is higher than that of conductor A.


u

If the I-V graph for two conductors A and B is of the


ou
ad
do

type as shown in Fig.3.8(c), then the slope of gaph


nY

= — = —. It shows that the smaller is the slope, the higher


V R
nid
Re

is the resistance. So the resistance of conductor A is higher


than that of conductor B.
FFi

Deduction of Ohm’s law.

eE
We know that V
d ~
m

V eV
But £= - V
d ~
I ml

eV Ane^ X
Also, ! = Anevj I = Ane V .(7)
ml / V, ml
V ml
or — = ^ = R = a constant for a conductor for a given value of n, I, A and at a given temperature.
/ An ex
It is known as the electrical resistance of the conductor.

Thus, V-RI

This is Ohm’s law.


CURRENT ELECTRICITY 3/9

3.11. ELECTRICAL RESISTANCE

The electrical resistance of a conductor is the obstruction posed by the conductor to the flow of
electric current through it.
Electrical resistance (R) of a conductor is deifned as the ratio of the potential difference (V)
applied across the ends of the conductor to the current (1) flowing through it,
i.e., R = Vn
Ivolt IV
The SI unit of electrical resistance is ohm (Q), where lohm =
1 ampere 1A
Thus, 1 ohm is the electrical resistance of a conductor through which a current of 1 ampere flows when
a potential difference of 1 volt is applied across the ends of the conducto.r

pot.diff. _ work done/charge _ MLr T ^/AT =

ww
Dimensions of electric resistance = A-2]
current current A

Cause of resistance of a conductor. Resistance of a given conducting wire arises on account of frequent

Flo
collisions of free electrons with the ions or atoms of the conductor while drifting towards the positive end of
the conductor. This would depend upon the arrangement of atoms in the conducting material, i.e., on the

e
nature of material (silver, copper etc.) as well as on the length and area of cross section of the conducting

rree
wire.

r FF
Any material which offers the obstruction to the FIGURE 3.9

flow of current through it is called a resistor. Some I


uurr
of the symbols of the various resistors are shown in
Fig.3.9(a), (b) and (c).
Fixed
resistance
for Variable
resistance
A— —WWW-

Variable
resistance
kss
o o o
ooook
Yo

Curiosity Question
eB

r Q. What is the biological connection of resistance and current to the human body ?
Ans. When the skin is dry, resistance of human body is = 10® ohm ; and when the skin is wet, the
rr

body resistance is * 1500 ohm.


ou
ad

Our supply voltage is 220 V. Therefore, when the skin is dry. current through the body,
Y
dY

V _ 220 U _ 220 ^
/ = — A = 2-2 mA and current through the wet body, / = — = 147-0 mA.
R ~ 10® R”1500
Re
innd

As is known, a current of 100 mA to 3 A through the human body is fatal: current of 20 mA to


100 mA causes paralysis ; current of 10 mA to 20 mA causes muscular disorder and a current
Fi

of 2 mA through the body causes tingling. Hence touching a 220 V live wire, especially when
the body is wet, may have serious consequences.
J
3.12. ELECTRICAL RESISTIVITY OR SPECIFIC RESISTANCE

The resistance of a conductor depends upon the following factors :


(i) Length ([). Consider two identical conductors in the form of slabs, each of length / and area of cross-
section A. Let V be the potential difference applied across either slab and I be the current through it. Fig.
3.10(a). The resistance of each slab is R=V/I

Let the two slabs be placed side by side so that the total length of the slabs becomes 2 / as shown in Fig.
3.10(/>). If same potential difference is applied across the ends of two slabs, then the current through them
becomes 7/2. Therefore, the resistance of the combination is
y
R = 2- = 2R
c
112 I
3/10 'P'Ktdecft 'a Fundamental Physics (XII)

FIGURE 3.10

I
z I
*-H2
T / III
I I
>112
» I
-► L .U
III

o ^ V © H- /■ ►I © N- ●/ -H

Thus, doubling the length of a conductor, doubles the resistance. Therefore, the resistance (R) of a
conductor is directly proportional to its length (/), i.e.. Roc I

Note that resistance of a conductor increases due to increase in its length on account of the fact that
electrons have to travel a longer distance through the conductor suffering large number of collisions.
(li) Area of cross-section (A). Divide a conductor slab into two equal parts cutting it length wise, so

ww
that the slab may be considered as a combinationof two identical slabs each of length /, having a cross-
sectional area A/2, Fig. 3.10(c). When the same potential difference V is applied across each of the two half
slabs, the current flowing through each is 7/2, which is half of the current through the full slab. The resistance
of each of the half slab is

Flo
V V

ee
R' = =2-=2R
1/2 I

rere
rFF
It means, halving the area of cross-section of the conductor, doubles the resistance. Therefore, the
resistance (R) of a conductor is inversely proportional to the area of cross-section (A) of the conducto,r i.e.,
Roc i/A
uur r
ffoor
The resistance of a conductor increases due to decrease in cross sectional area on account of the fact
that free electrons find it more difficult to pass through smaller cross sectional area.
sks
(Hi) The resistance of a conductor also depends upon the nature of material and temperature of the
Yoo
oko

conductor.

I pi
BBo

From above; R OC or R=^ ...(8)


A A
re

where p is constant of proportionality and is known as specific resistance or electrical resistivity of


the material of the conductor
ouur
ad

If / = 1, A = 1, then from (8), R= p


YY

specific resistance (or electrical resistivity) of the material of a conductor is defined as the
resistance of a unit length with unit area of cross section of the material of the conductor, Le.,
nndd
Re

it is also defined as the resistance of unit cube of a material of the given conductor.
FFii

Unit of resistivity ; From (8), p-RA/l


ohmxm^
In SI, unit of resistivity = = ohm-m or Q m.
m

RA A-'^xl}
Dimensions of resistivity = = [AfZ?7'-3 A-2]
/ L

Factors affecting electrical resistivity


V ml m I
From (7), we have. R = ~ X —
/ Ane^ X ne
2
X A

I m

Comparing the above relation with the relation, R = P-, we find P = ...(9)
ne^ X

Thus the resistivity of the material of conductor depends upon the followingfactors.
CURRENT ELECTRICITY 3/11

(/) p « 1/n, i.e., the resistivity of the material of conductor is inversely proportional to the number density of
electrons (n), i.e., number offree electrons per unit volume of the conductor. Since the value of n depends upon the
nature of material, hence the resistivity of the conductor depends upon the nature of material.
{«) p « l/x, i.e., the resistivity of the material of conductor is inversely proportional to the average
relaxation time (x) of the free electrons in the conducto.r Since the value of X depends upon temperature of the
conductor (which is discussed in article 3.16), so resistivity of conductor changes with temperature. With the
increase in temperature of metal conductor, x decreases, hence its electrical resistivity increases.
The value of electrical resistivity of different materials varies over a very wide range as shown in Table 3.2:
TABLE 3.2. Resistivity of different materials

NAME OF THE RESISTIVITY NAME OF THE RESISTIVITY AT


MATERIAL AT QfiC (am) MATERIAL 0»C (am)

ww
A. Conductors B. Semiconductors

(a) Metals
Silver 1-6 X 10-^ Carbon 3-5 X 10-5

Flo
Copper 1-7 X kH Germanium 046

Aluminium 2-7 X 10"^ Silicon 2300

ee
Tungsten 5-6 X 10-®

rere
Iron 10 X 10^

rFF
Platinum 11 X 10-^
Mercury 98 X 10-^ C. Insulators
uur r
(b) Alloys
foor Glass
Hard rubber
10l«- 10'*^
10>3_ io>6
ks s
Nichrome
Yoo
100 X 10-^ 10" - 10'5
ook

(alloy of Ni, Fe, Cr) Mica

Manganin 44 X 10-S Wood 10^-10"


BBo

14
Constantan 49 X 10"^ Amber 5 X 10
re

From the given table, we note that the value of electrical resistivity is very low for conductors, very
ouur

high for insulators and in between those of conductors and insulators for semi-conductors.
ad

Note. 1. Since electrical resistivity of copper is low, its electrical conductivity is high, therefore copper
YY

wire is used as connecting wire.


2. Since electrical resistivity of alloys like manganin (Cu - 84% + Mn - 12% + Ni - 4%) and constantan
nndd
Re

(Cu - 60% + Ni - 40%) is high, the lengths required for the wires of given diameter in maldng the standard
FFii

resistances of given values are smaller. It follows from /? = p l/A. For a given value of R and for a given value
of A, as p is high, / is low.
DO YOU KNOW ?
1. Resistance of a conductor depends upon the dimensions {i.e. length and area of cross-section) nature of
the material of conductor and temperature of the conductor but resistivity depends on nature of the material
and temperature but is independent of the dimensions of the conductor.
2. Resistance is a property of object. Resistivity is a property of material.
3. ohm-metre is the unit of resistivity, where as Ohm-meteris an instrument to measure the resistance.

[ Sample Problem j What IS the length of a nichrome wire of radius 0*32 mm, resistance 9*3 a
and resistivity 15 x a m ? If a potential difference of 10 V is applied across the wire, what will be
the current in the wire ?
3/12 4*. Fundamental Physics (XII) ITOWl

Sol. Here, / = ?; r = 0-32x 10"^ m ; /? = 9-3 H ; p = 15 x 10“^ Om ; V= lOV


/? = Ji r2 (9-3) X 3-14 X (0-32 x 10“3 )2 - 0-199 m = 0*20 m
2 or I =
Ttr
P (15x10“^)
Current 7 = 11 = 10
= 1-075 = 1-08 A
R 9-3

3.13. CURRENT DENSITY, CONDUCTANCE AND ELECTRICAL CONDUCTIVITY

Current density (J) at apoint in a conductor is defined as the amount of currentflowing per unit area of
the conductor around that point provided the area is held in a direction normalto the current.

Let / be the current flowing uniformly and FIGURE 3.11

ww
normally through a cross-sectional area A of the
conductor Fig. 3.11(a), then magnitude of the current
density at any point on the cross-section of the ¥ >●

Floo
->●
conductor is A

ee
o

reer
A A

rFF
If the current I is not flowing normally through a cross-sectional area A as shown in Fig.3.1!(/?) then
component of normal area to the direction of current is. A_ = A cos 0 n
uur r
Current density.
A
n
Acos0
I
or ffoor
/ = 7A cos 0 = JA
sks
This relation shows that the current being the dot product of two vectors is a scalar quantity.
YYoo
ooko

Current density is a vector quantity. Its direction is the direction of motion of positive charge.
eBB

Relation between current density and Drift velocity


We know that I - n A ev^
r

Current density, j = —A = nev.d


ouur

...(10)
ad

The SI unit of current density is ampere (metref^ or Am“^


YY

Note. For a particular surface of a conductor, the current is the flux of current density J over that
nndd

->
Re

surface S and is given by I = J.dS


FFii

where dS is elementary surface area vector of an element taken over the particular surface S and
integral is taken over the surface in question.
Conductance (G) of a resistor is the measure ofease with which the charges flow through the conductor.
It is equal to the inverse of its resistance(R) and is denoted by G. Thus
1
Conductance, G = -
R

The SI unit of conductance is ohm ^ or mho or siemen (symbol S).


Electrical conductivity (o) of a conductor is defined as the reciprocal of its resistivity. It is
represented by a, i.e.,

CT= 1/p ...(11)


CURRENT ELECTRICITY 3/13

The SI unit of electrical conductivity is ohm ^ m"^ or mho m“^ or S m


I neh L-^x(AT)^xT
The dimensions of electrical conductivity are a = —
P m M

Factors affecting electrical conductivity (o) of a conductor


m 1
As P = and a = —
ne^x P m

where the different terms have their usual meanings. Thus electrical conductivity depends on the following
factors :

(i) Nature of material of conductor. As number density of electrons in different materials is different

oww
and o « n (number density of electrons) in a conductor, so conductivity of different material is different.
(ii) Temperature of the conductor. When temperature of the conductor increases then the value of
relaxation time x decreases.
As o t, so, conductivity of a conductor decreases. On the other

ee
PC

hand if the temperature of a conductor decreases, then the value of relaxation

FFrlo
time increases and hence the conductivity of the conductor increases. Thus

r
a i, where Tis the temperature of the conductor. The variation of electrical

rF
ee
■►T
conductivity (a) of a conductor with temperature (T) is shown in Fig. 3.12.
ouru
rF
Relation between J, o and E [Microscopic form of Ohm’s law]
n Ae^ xE I ne^xE E

ffosor
eE
We know. I=nAev^ =nAe or
m/ne^ X
ym m
os k A m

1 I , m
or J = -E J =
-andp =^
ook
A ne^
Yo
P
Y
1
Bo

c = -
y = a£
reeB

PJ
It is a microscopic form of Ohm’s law.
oouY

^ ^
ur

As the direction of current density j is the same as that of electric field £, hence
ad
dY

J =aE

This relation is the vector form of Ohm’s law.


nidn
Re

Relation between electrical resistivity and electron mobility.


I
= 0E^^
FFi

and J=-
We know that I = nA evj; v^ = ]xE A P

E I 1
— = nev^ =neilE or P = ...(12)
ne\\,

SampieleTogiernlfl Find the conductance of a conductor of resistance 2 milli ohm.


1
Sol. Here, R = 2 x 10 O ; conductance, G = — - = 500S
R 2x10"^
fi^a'iTipieTgroiaiS'miPj Potential difference of 100 V is applied to the ends of a copper wire one
metre long. Calculate the average drift velocity of the electrons ? Compare it with thermal velocity at
27“C. Consider there is one conduction electron per atom. The density of copper is 9-0 x 10^ kg/m^ ;
Atomic mass of copper is 63*5 g. Avogadro’s number = 6'0 x 10^ per gram—mole. Conductivity of copper
is 5*81 X lO’ m“^ Boltzmann constant = 1*38 x 10“^ JK"^.
3/14 ‘Pn^z^iU^ ^ Fundamental Physics (Xll) orwi
Sol. Here ; V = 100 V, / = 1 m, Af = 63-5 g\ d = 9-0 x 10^ kg/m^ = 9-0 g/iii^
N = 60x 10^^ per gram-mole ; a = 5-81 x 10^ Q*' m“'.
Since 6-0 x 10^^ copper atoms have a mass of 63-5 g, and there is one conduction electron per atom, so

60x10^^
number density of electrons , n = x9-0 = 8-5 X 10^“ cm ^ = 8-5 x 10^^ m“^
63-5

Electric field, £ = — = = 100 Vm“*


/ 1

As
cE (5-81xl0^)x(100)
J = o E = ne Vj V
d ~ = 0*43 ms ^
ne
(8-5x1028)x(1-61x10-'9)

oww
3kr.T
B 3x1-38x10-23x300
Thermal velocity, v rms ^ 1-17 X 10^ m/s
i \ 9-1x10“3*
V
d _
0-43

ee
= 3-67 X 10-^
V
rms
1-17x10^

FFrlo
r
3.14. CLASSIFICATION OF MATERIALS IN TERMS OF CONDUCTIVITY

rF
ee
On the basis of conductivity, the different materials have been classified into following categories.
1. Insulators.

rF
ouru
These are the materials whose electrical conductivity is either very very small or nil.

fosor
Insulators do not allow the flow of charges. When a small potential difference is applied across the two
s kf
ends of an insulator, the current through the insulator is zero.
ooko
Examples of insulators are glass, rubber, wood etc.
YYo
2. Conductors.
Bo

These are the materials whose electrical conductivity is very high.


reeB

Conductors conduct charges very easily. When a small potential difference is applied across the two
ends of conductor, a strong current flows through the conductor. For super-conductor, the value of electrical
ooY
uur

conductivity is infinite and electrical resistivity is zero.


ad

Examples of conductors are all metals like copper, silver, aluminium, tungsten etc.
dY

3. Semiconductors.

These are the materials whose electrical conductivity lies inbetween that of insulators and
nidn
Re

conductors.
FFi

Semiconductors can conduct charges but not so easily as is in case of conductors. When a small potential
difference is applied across the ends of a semiconductor, a weak current flows through semiconductor due to
motion of electrons and holes.

Examples of semiconductors are germanium, silicon etc.


3.15. EFFECT OF TEMPERATURE ON RESISTANCE
We know that resistance of a metallic conductor is given by
m I
R =
ne^x A
1
For a given conductor, R oc

When the temperature of metal conductor is raised, the number density {«) of electrons in metal
conductor does not change appreciably. However, the ions/atoms of the metal vibrate with greater amplitudes
about their mean povsitions (i.e. lattice sites). Due to increase in thermal energy, the frequency of collision
CURRENT ELECTRICITY 3/15

of free electrons with aloms/ions while drifting towards positive end of the conductor also increases. This
reduces the average relaxation time x. Hence, the value of electrical resistance R increases with irse of
temperature.
The resistance R, of a metal conductor at temperature f C is given by
R, = Rq{\ +at + ^t^)
where a and P are constants, Rq is the resistance of conductor at 0°C. The values of a and p vary from
metal to metal. If the temperature fC is not sufficiently large which is so in most of the practical cases, the
above relation may be simplified as
R, = RqO + at) ...(13)

oww
R ~R increase in resistance
I 0
or
R.~Rq
t + Rq at or a =
R^xt original resistance X rise of temp.
0

Thus, temperature coefficient of resistance (a) is defined as the increase in resistance per unit
original resistance per degree Celsius or kelvin irse of temperature.

e
FFrlo
For metals like silver, copper etc., the value of a is positive, therefore, resistance of a metal increases

re
with irse in temperature. The unit of a is or°C~^.

ree
F
For insulators and semiconductors, a is negative, therefore, the resistance decreases with irse in
temperature.

rF
Note : (/) Let a be the temperature coefficient of resistance of a conductor at 0°C. R^ and R2 be the
resistances of that conductor at /j°C and tf.C respectively, then

fsoor
ouur
/?] = Rq(1 + ar|) and R2 -RQ(i + at2)
skf
R.I i + at 1 ^^
/?j (1 + a/2) =/?2 (1 + ocrj) or a =
ooko
or

Rf2~^2h
YYo
R, 1 + 0/2
Bo

(i7) The value of a is different at different temperatures. Temperature coefficient of resistance averaged
reB

over the temperature range /j °C to /2 °C is given by

a =
«2-«l
uur
ooY

where R^, R2 are the resistances at /j °C and /2 °C respectively. ...(14)


ad
dY

The alloys like manganin, eureka and constantan are used for making standard resistance coils*
due to following facts :
(/) These alloys have high value of resistivity.
innd
Re

(») They have small values of temperature coefficient of resistance. That is why their resistance does not
FFi

change appreciably even for several degrees rise in temperature.


(Hi) Their contact potential with copper is small.
(iV) Their resistance is least affected by atmospheric conditions like air, moisture, etc.
jhe resistance of a conductor at 30“C is 3-25 and at 100"C is 3-95 Q.
Calculate the temperature coefficient of resistance of the conductor and the resistance of the conductor
at 0“C.

Sol. /?! = 3-25 a ; /i= 30^*0; R2 = 3-95Q; /2=100°C


a =
^2-^1 3-95-3-25 0-70 0-70
= 0-0034 ‘»cr^
^\h~^2h (3-25x100)-(3-95x30) 325-118-5 206-5
R 3-25 3-25
I
R0 “ = 2-95 a
1 + ar,] l + (0-0034)x30 1-102

*Ocu “ 10"^ “C-i and *a manganin = 10"^ “C"'


3/16 “P,nduCte^ '<i Fundamental Physics (XII)
3.16. VARIATION OF ELECTRICAL RESISTIVITY WITH TEMPERATURE
1 m 1
Electrical Resistivity of a material, p = — 2 [Eqn. (9)] or poc
ne X nz

This shows that the electrical resistivity is related to two parameters of the material, namely the number
density n of free electrons in the material and relaxation time, x. The variation of electrical resistivity of
material with temperature is different for different materials,
(a) In most metals, number density n of free electrons does not change with temperature but an increase
in temperature increases the amplitude of vibrations of lattice ions of the metal and thermal speed of the
electrons which act as carriers of current. Due to it, the collisions of free electrons with ions/atoms while
drifting towards the positive end of the conductor become more frequent, resulting in a decrease in relaxation

oww
time T. Hence, electrical resistivity of conductor increases with increase in temperature. At low temperature,
resistivity increases with a higher power of T.
The temperature dependence of electrical resistivity of a metal can be obtained from (13), as
p oc R. It is given by the relation p = Pg [1 + a (T - Tq)].

e
where p and pg are the values of resistivity at temperatures T and Tq respectively and a is called temperature

ree
coefficient of resistivity.

rFl
Fre
P~pQ _dp 1
Clearly, a = ..(15)

rr F
Po “ ^0 ^ Po ^
Thus temperature coejficient of electrical resistivity is also defined as the fractional change in
ouur
sfoo
electrical resistivity (d p/ Pq) per unit change in temperature (dT). The unit of a is or K~^.
For metals. The value of a is positive, showing that their electrical resistivity increases with increase in
kks
Yo
oooo

temperature.
For many metallic conductors, the value of a is nearly 4 x 10“^ For such metallic conductors, the
eBB

temperature dependence of p at low temperature is non-linear. At low temperatures, the resistivity (p) of a
pure metal increases as a higher power of temperature. The variation of electrical resistivity p of copper with
temperature T is shown in Fig. 3.13(a).
urr

For alloys, the resistivity is high. The resistivity of nichrome (which is alloys of nickel, iron and chromium)
You
ad

has weak temperature dependence as shown in Fig. 3.13(b). The resistivity of aranganin is almost independent
dY

of temperature. Fig. 3.13(c). That is why wire bound standard resistors are made of manganin. At temperature
OK, a pure metal has negligible small resistivity while an alloy (like nichrome) has some residual resistivity.
innd
Re

This concept has been utilized to distinguish a pure metal from an alloy.
Fi

FIGURE 3.13
p %

1.12

1.10

T(K) o 1.00 >T(K) ©


200400600800

(i) For semiconductors like carbon, germanium, silicon etc., with increase in temperature, the number
density n increases exponentially and time of relaxation T decreases. But the increase in n compensates more than
decrease in x. That is why the resistivity of semiconductor decreases exponentially with increase in temperature.
The number density of electrons at temperature T is given by
-£./koT
B
n(T) = nr,e s
CURRENT ELECTRICITY 3/17

where is Boltzmann constant and Eg is the energy gap between conduction band and valence band of the
1 1 -E/kT
P (T) = Pq€
g
resistor. As, p OC
1/n, so — e * or

piT) Po
This relation shows that resistivity of semiconductor decreases rapidlly with increase in temperature.
For semiconductors, the coefficient of resistivity (a) is negative. Therefore, their resistivity decreases with
increase in temperature. For semiconductor, the variation of p with temperature T is shown in Fig. 3.13(d).
(c) For insulators. The coefficient of resistivity (a) is negative. The resistivity p increases exponentially
with decrease in temperature. It becomes infinitely large at temperature near absolute zero, and the electrical
conductivity of insulator is almost zero at OK.
id) For electrolytes, the coefficient of resistivity (a) is negative. With the rise in temperature, the
resistivity p of electrolytes decreases because with the rise in temperature, interionic attractions decrease and

ww
also viscous forces decrease in electrolytes. Due to it, the ions move more freely, i.e., drift velocity of ions
1
increases and the time of relaxation (t) increases. As p , so p decreases with increase in temperature.

Flo
T

ee
3.17. NON-OHMIC DEVICES OR LIMITATIONS OF OHM'S LAW

rere
Those devices which do not obey Ohm’s law are called non-ohmic devices. For example, vacuum

r FF
tubes, semiconductor diode, liquid electrolyte, transistor etc. are all non-ohmic devices.
uurr
The relation; V/I = R, is valid for both ohmic and non-ohmic devices. The value of R is constant for
foor
ohmic conductors but R is not constant for non-ohmic devices, i.e.. Ohm’s law fails when V/I is not constant.

For all non-ohmic devices (where there will be failure of Ohm’s law), V-I graph has anyone of the following
ks s
characteristics: V-I graph is not a straight line. That is why non-ohmic devices are called non-linear devices,
Yoo
oook

(fl) The relation between V and I is non-linear. Fig. 3.14(a).


eBB

(b) The relation between V and I depends on the sign of.V It means, if / is the current for a certain value
of V, then reversing the direction of V, keeping its magnitude fixed, does not produce a current of same
magnitude /, in the opposite direction. Fig. 3.14(b). This happens in case of junction diode which we shall
rr
ouu

study in chapter IX (b).


ad

(c) The relation


YY

between V and I is not

unique, i.e., there is more


nndd
Re

than one value of V for the


same current I. Fig. 3.14(c).
Fii

This happens for the


material GaAs (i.e., in a
light emitting diode).

3.18. THERMISTORS

A thennistor is a heat sensitive device whose resistivity changes very rapidly with change oftemperature.

The temperature coefficient of resistivity of a thermistor is very high, which may be positive or negative.
Thermistors are usually prepared from oxides of various metals such as nickel, iron, copper, cobalt etc.
These compounds are also semi-conductors. The size of thermistor is very small. The thermistors are generaUy
in the form of beads, discs or rods. A pair of platinum leads are attached at the two ends of thermistor for
electric connections. This arrangement is sealed in a small glass bulb.
A thermistor can have a resistance in the range of 0-1 Q to 10^ Q, depending upon its composition. A
thermistor can be used over a wide range of temperatures.
3/18 4^ Fundamental Physics (XII) PZSTWl

A thermistor differs from an ordinary resistance in the following ways :


1. The resistance of thermistor changes very rapidly with change in temperature, therefore, the temperature
coefficient of resistivity of a thermistor is very high.
2. The temperature coefficient of resistivity of a thermistor can be both positive and negative.
Important applications of thermistors
1. Thermistors are used to detect small temperature changes and to measure very low temperature
(« 10 K). A typical thermistor can easily measure a change in temperature of 10”^ °C.
2. Thermistors are used to safe-guard the filament of picture tube of a television set against the variation
of current.

3. Thermistors are used in temperature control units of industry.


4. Thermistors are used in the protection of windings of generators, transformers and motors.

ww
5. Thermistors are used for voltage stablisation and remote sensing.

3.19. SUPER-CONDUCTIVITY

Flo
Prof. K. Onnes, in 1911, discovered that certain metals and alloys at very low temperature lose their
resistance considerably and almost becomes zero. This phenomenon is known as super-conductivity. As the

e
ree
temperature decreases, the resistance of the material also decreases, but when the temperature reaches a
certain critical value (called critical temperature or transition temperature), the resistance of the material

Fr
rr F
completely disappears i.e., it becomes zero. Then the material behaves as if it is a super-conductor and on
uurr
applying small-electric field, there will be flow of electrons without any resistance whatsoever. Thus
superconductivity is the property by virtue of which a metal alloy or oxide etc. shows zero resistance at
a very low temperature.
sfo
The variation of resistance (R) and temperature (7) for mercury is R(n)
FIGURE 3.15
kks
shown in Fig. 3.15. This graph shows that the resistance of mercurydrops
Yo
oooo

abruptly to zero below 4-2 K. The critical temperature is different for


different materials. It has been found that mercury at critical temperature
eB

4-2 K, lead at 7-25 K and niobium at critical temperature 9-2 K become


.super-conductors.
rr

A team of scientists discovered that an alloy of plutonium, cobalt


ou

and gallium exhibits super conductivity at temperatures below 18-5 K.


ad
YY

Since 1987, many superconductors have been prepared with critical


temperature upto 125 K, as listed below : Bi2 Ca2Sr2 CU3OJ0 at 105 K 4.2
T(K)
and Tl2Ca2 Ba2 CU3OJQ at 125 K.
nndd
Re

The super-conductivity shown by materials can be verified by simple experiment. If a current is once set
Up in a closed irng of super-conducting material, it continues flowing for several weeks after the source of
Fi

e.m.f. has been withdrawn.

The cause of super-conductivity is that, the free electrons in super-conductor are no longer independent
but become mutually dependent and coherent when the critical temperature is reached. The ionic vibrations
which could deflect free electrons in metals are unable to deflect this coherent or co-operative cloud of
electrons in super-conductors. It means the coherent cloud of electrons makes no collisions with ions of the
super-conductor and, as such, there is no resistance offered by the super-conductor to the flow of electrons.
Super-conductivity is a very interesting field of research all over the world these days. The scientists
have been working actively to prepare super-conductors at room temperature and they have met with partial
success only.
Applications of super conductors
1. Super conductors are used for making very strong electromagnets.
2. Super conductivity is playing an important role in material science research and high energy particle physics.
3. Super conductivity is used to produce very high speed computers.
4. Super conductors are used for the transmission of electric power.
3/19
CURRENT ELECTRICITY

3.20. MEISSNER EFFECT


In 1933, Meissner and Orchsenfold found that if a conductor is FIGURE 3.16
cooled in a magnetic field to a temperature equal to or lower than the
transition temperature T^, then at the transition temperature, the
magnetic lines of induction are pushed out as has been shown in Fig.
3.16. o

Thus the Meissner effect states that at or below the


transition temperature, there is no magnetic field or
magnetic flux density inside the super-conductor when the T>Tc
specimen is placed inside a uniform magnetic field.

3.21. COLOUR CODE FOR CARBON RESISTORS O


>

w
>
In the market, the commercial resistors are carbon resistors. These
are made by moulding carbon into a cylinder with wire leads at the two
ends. Every such carbon cylinder is encased in a ceramic or plastic T<Tc

Flo
jacket. Carbon resistors are thus compact and cost less. The value of
the resistance of these resistors and their percentage accuracy are

e
ree
indicated by a colour code printed on them. The colour code for carbon
resistances is given in Table 3.3 :

FFr
TABLE 3.3. Colour code of carbon resistors.
urr
or
NUMBER MULTIPLIER COLOUR TOLERANCE
COLOUR LETTER AS AN
AID TO MEMORY
sf
kks
Black B 0 10° Gold 5%
Yo
1 10%
Silver
ooo

Brown B 1 10
2 \0- No colour 20%
Red R
B

0 3 103
Orange
10**
re

Yellow Y 4

Green G 5 10^
10^
ou

B 6
ad

Blue
10^
YY

Violet V 7

G 8 10^
Grey
W 9 10^
ndd

White
Re

Gold 10-*
Silver 10-2
Fi

To remember the value of colour coding used for carbon resistor, the following sentences are found to
be of great help (where bold letters stand for colours)
B B ROY Great Britain Very Good Wife wearing Gold Silver necklace.
Black Brown Rods Of Your Gate Became Very Good FIGURE 3.17

When Given Silver colour. A BC R

3IU)
Way offinding the resistance of carbon resistor from its A
colour coding.
Vi
In the system of colour coding, strips of different colours
First Second Third Tolerance
arc given on the body of the resistor, Fig. 3.17. The colours on digit digit digit
strips are noted from left to right,
(i) Colour of the first strip A from the end indicates the first significant figure of resistance in ohm.
(h) Colour of the second strip B indicates the second significant figure of resistance in ohm.
3/20
U Fundamental Physics (XII) VOL. I

(iii) The colour of the third strip C indicates the multiplier, i.e., the number of zeros that will follow
after the two significant figures.
(iv) The colour of fourth strip R indicates the tolerance limit of the resistance value or percentage
accuracy of resistance.
Note. 1. If fourth coloured strip does not exist then tolerance is to be taken as ± 20%.
2. If any band is omitted, it is of the same colour as the body of resistor.
For the given carbon resistor, let the first strip be yellow, second strip be
red, third strip be orange and fourth be gold. What is its resistance ?
Sol. We know that the numbers for yellow, red and orange are 4, 2 and 3. Gold represents tolerance of
± 5%. Hence the value of the given resistance = 42 x 10^ ± 5% = 4*2 x 10“* £2 ± 5%.
‘Samble .^oblem' E The resistance of the given carbon resistor is 2-4 x 10^ £2 ± 5%. What is

ww
the sequence of colours on the strips provided on resistor ?
Sol. The value of the resistance of given carbon resistor, /? = 2-4 x 10^ Q ± 5% = 24 x 10^ Q ± 5%.
The colours attached to numbers 2, 4 and 5 are red, yellow and green. The colour for 5% is gold.

FF loo
Therefore, the colours of strips in sequence for the given carbon resistor are red, yellow, green and gold.

ee
3.22. RESISTANCES IN SERIES

ee r
FIGURE 3.18

rFrF
Resistors are said to be connected in series, if the same

current is flowing through each resistor when some potential M-


Ri
V -W

difference is applied across the combination. Here I R2 I R3


rur
the A VvWv—^—WWw—►—-WWrf—I B
resistors are connected end to end with one another. ffoor ^ --Vi- V2---W4---V3--►
If the three resistances R^, R2 and R^ are connected end to
ks
L. ”1
end as shown in Fig 3.18(a), they are said to be connected in
YYoou
series. Let Vbe the potential difference applied across A and B
ookos

using the battery e. In series combination, the same current (say


BBo

/) will be passing through each resistance. Let V^, V2, V3 be the O


potential differences across /?j, R2 and R^ respectively. Rs
re

A -wwv B
According to Ohm’law
oouur
ad

V^=IR^, V2=IR2, V^ = IR^ I- "I

Here, V=V^ + V2+V2 = IR,+IR2 + IR.


Y

I I
= I(Ry+R2 + R^) ...(17)
dndY

If R^ is the equivalent resistance of the given series combination of resistances. Fig. 3.1 S(b), then the
Re

potential difference across A and B is.


FFini

F = //?,. ...(18)
From (17) and (18), we have IR, = /(/?, + /?2 + /?3) or

Thus, the equivalent resistance of a number of resistors connected in series is equal to the sum of
individual resistances.
It shows that the value of equivalent resistance of series combination of resistors is always greater than
the resistance of individual resistors. Therefore, the resistors are to be connected in series, if the effective
resistance in the circuit is to be increased.

Retain in Memory
In a series resistor circuit, it should be noted that:
(/) the current is same in every resistor, because there is single path along which current can flow.
(«) the current in the circuit is independent of the relative positions of the various resistors in the series.
(m) the voltage across any resistor is directly proportional to the resistance of that resistor.
3/21
CURRENT ELECTRICITY

(iv) the total resistance of the circuit is equal to the sum of the individual resistances, plus the internal
resistance of a cell if any.
(v) The total resistance in the series circuit is obviously more than the greatest resistance in the circuit,
(vi) Connecting resistors in series is equivalent to increasing the length of the conductor. That is why
resistance of the series combination increases.
3.23. RESISTANCES IN PARALLEL
Any number of resistors are said to be connected in parallel if potential difference across each of
them is the same and is equal to the applied potential difference.
Here, one end of all the resistors is connected to one point and other end of all the resistors is connected
to another point. The source of potential difference is connected across these two points.
' FIGURE 3.19
If three resistances /?2 and are connected as shown

ww
-V- ■M
in Fig. 3.19(a), they are said to be connected in parallel. Let Vhe Ri
the potential difference applied across A and B with the help of a WVW
]1
battery e. Let I be the main current in the circuit from battery. /

FF loo
R2 h
divides itself into three unequal parts because the resistances of ■VAVv ¥ B
these branches are different. If f, I2, h ^he currents through

ree
the resistances /?[, /?2 ^3 respectively. Then, R3
wwv
I3
"I
...(19)

ree F
/ = /j + /2 + /3
Here, potential difference across each resistor is V, therefore s
ur
y = /] /f] =l2^2~h^3
or /=- I - ^ V
fofroF o
I h

’ ^3 =
ks
A B

' T
os o

I-
Putting values in (19), we get
YYouor

, V 1 V 1V
BBook

...(2d) I
/ =
R o
1
r ee

If is the equivalent resistance of the given parallel combination of resistances. Fig. 3.19(6), then
V
ad

...(21)
oouur

V=/Rp or / = R
Y

V _V _V^ _L = i- -L ^ ...(22)
From (20) and (21), we have
nYd

or
Re

R R1 ^2 ^3 ^11 ^2 ^3
p
FFindi

Thus, the reciprocal of equivalent resistance of a number of resistors connected in parallel is


equal to the sum of the reciprocals of the individual resistances.
It shows that the value of equivalent resistance of parallel combination of resistors is always less than
the resistance of individual resistors. Therefore, the resistors are connected in parallel, if the ejfective resistance
in the circuit is to be decreased.
Note. If two resistors of resistances R^ and R2 are connected in parallel, then their effective resistance, Rp is
J__J_ J y?l/?2
or Rp =
Rp~R^^R2 1 V'2 R^+ R2

Retain in Memory
1. In a parallel resistor circuit, it should be noted that:
(i) the potential difference across each resistor is the same and is equal to the applied potential difference.
(it) the current through each resistor is inversely proportional to the resistance of that resisto.r
3/22
^ Fundamental Physics (XII) W*Tl
m total current through the parallel combination is the sum of the individual currents through the
various resistors.
(/V) The reciprocal of the total resistance of the parallel combination is equal to the sum of the
reciprocals of the individual resistances.
(V) The total resistance in parallel combination is less than the least resistanceused in circuit.
(vO Connecting a number of resistors in parallel is equivalent to increasing the cross sectional area
1
of the conducto.r As R oc
—, therefore, total resistance of parallel combination decreases.
A
2. When resistances are connected in series, the current through each ersistanceis same. When the
resistances are in parallel, the pot-diff. across each resistance is the same and not the current.

Sample Problem A wire of resistance 4R is

ww
FIGURE 3.20
bent in the form of a circle Fig. 3.20(a). What is the 2R
effective resistance between the ends of the diameter ? r—^AA/—I
Sol. Here two resistances each of value 2R are connected

FF loo
in parallel as shown in Fig. 3.20(6). So the ersistance between
A B A 2R B

the ends A and S of a diameter of circle is

ee
2Rx2R o

er
R' = = R

rFreF
2R + 2R

3.24. EMF, INTERNAL RESISTANCE AND TERMINAL POTENTIAL


rur
DIFFERENCE OF A CELL
As already defined in Art 3.3, fforo
ks
emf of a cell is the maximum potential difference between two electrodes of the cell when no
YYouo
current is drawn from the cell.
okso

Internal resistance of a cell is defined as the resistance offered by the electrolyte and electrodes
BBoo

of a cell to the passage of electric current through the cell


re

The value of internal resistance of a freshly prepared cell is generally low and its value goes on increasing
as the cell is put to more and more use.
ad
oouur

Internal resistance of a cell depends upon the following factors: (/) Distance between the electrodes, («)
Y

The nature, concentrauon and temperature of the electrolyte, {Hi) The nature of electrodes, (iv) Area of the
electrodes, immersed in the electrolyte. If area increases, internal resistance decreases.
dnYd
Re

Terminal Potential difference of a cell is defined as the potential difference between the two
FFini

electrodes of a cell in a closed circuit {i.e., when some current is drawn from the cell). It is
generally denoted by V and is measured in volt.

When the electric cell (source of emf) is in a closed circuit, the current flows through the circuit.
There is a fall of potential across the internal resistance of the cell. The terminal potential difference
between the two electrodes of the cell becomes less than the emf of the cell by an amount equal to potential
drop across the internal resistance of the cell. Thus, in a closed circuit the terminal potential difference of a
cell is always less than the emf of the cell.
Consider a cell of emf e, and internal resistance r connected to an external resistor of resistance R and a
one
way key K as shown in Fig. 3.21 (a). A high resistance voltmeter V is connected across the two electrodes
(terminals) of the cell.
If the key K is not closed, the circuit remains open. No current flows through the circuit. The reading of
the voltmeter is equal to emf e of the cell, as per definition of the emf of the cell.
When the key K is closed, the current flows through the circuit as shown in Fig. 3.21(6). Now, total
resistance of the circuit = /? + r.
CURRENT ELECTRICITY 3/23

Magnitude of current, FIGURE 3.21

/ = —^ ...(23)
R + r
Potential difference e
e K K
across internal resistance r
= Ir
+
<> + <●>
It is found that the reading -Vs/Wv— ■VvWV—
r r
of voltmeter (V) now is less
I '
than e by I.r So
V=z-Ir (24) R R
■AVvr ■VvWV
A B A B
If 7 = 0, then e o

w
V=E-0(r) = e
Thus, the terminal potential difference of two electrodes of a cell is equal to emfof the cell when no
current is drawn from the cell.

FF llowo
As the point A is at the same potential as the positive electrode and the point B is at the same potential
as the negative electrode of the cell, therefore, the terminal potential difference of a cell is equal to the

ee
potentialdifference across the external resistance R of the circuit i.e. between points A and B, so

Fr
V=IR or /= V//?

e-V (z-V'

er
From (24), XR ...(25)
ur
r =
I

Graph between (a) e and R (b) V and R and (c) V and /.


fofr For
ks
YYour o
FIGURE 3.22
(a) EMF of a cell is independent of external resistance c A
oo

R connected across the cell. So the graph between e and R


is a st. line as shown in Fig. 3.22(a).
BoBks

V
r ee

(b)As V =z-Ir = z-—r (v /= V/R)


R
■►R ■AR
oouru

( f
ad

e o o
or V 1 +- = e or V =
V4
(1 + r/R)
Y

Above relation shows that as R increases V also


Ynd
Re

increases. A graph between V and /? is a curve of the type


as shown in Fig. 3.22(b). When /? = «», then F= e. In this
FFind i

situation the curve becomes parallel to resistance axis, > I

(c) As V=z- /r = - Ir + z
This relation shows that as I increases, V decreases. It is similar to the equation of straight line,
y = rnx + C. Therefore, the graph between V and / is a st. line as shown in Fig. 3.22(c), whose slope
(= - r) tells the internal resistance of the cell and intercept is z (i.e., emf of the cell)
Retain in Memory
1. In an open circuit, terminal voltage across a secondary cell is FIGURE 3.23

equal to its emf. V

2. It is important to note that during charging of a cell, the positive +


Charger
electrode of the cell is connected to positive terminal of battery I -●I
charger and negative electrode of the cell is connected to negative
terminal of battery charger, Fig. 3.23. In this case, net potential
difference of circuit -V-z
3/24 'P'uuUefr ^ Fundamental Physics (XII) arnTWI

This net potential difference is equal to the voltage drop (= If) across the internal resistance (r) of
the cell. Then

V-t = lr or V = e + /r

Hence, the terminal potential dijference becomes greater than the emf of the cell.
3. The difference of emf and terminal potential difference is called lost voltage as it is not indicated
by a voltmeter. It is equal to Ir.

Ls^mpielRcpgieml A voltmeter of resistance 995 O is connected across a cell of emf 3V and


internal resistance 5 Q. Find the potential difference across the voltmeter, that across the terminals of
the cell and percentage error in the reading of voltmeter.
Sol. Here, /? = 995n;e = 3V;r=5^2

ww
3
Current in the circuit, /= — = 3 X 10-3 A
R + r 995 + 5
Potential difference across the voltmeter V = //? = (3 X 10-3) X 995 = 2-985 V

Floo
The potential difference across the terminals of the cell, V = e- Ir = 3 - (3 x 10“^) x 5 = 2*985 V
As the voltmeter used is to measure the emf of the cell but will read 2-985 V. Hence percentage error is

ee
e-V 3-2-985
xI00 = xlOO =0-5%

reer
3

rFF
e

3.25. DISTINCTION BETWEEN E.M.F. AND TERMINAL POTENTIAL DIFFERENCE OF A CELL


uur r
E.M.F. OF A CELL ffoor
TERMINAL POTENTIAL DIFFERENCE
sks
1. The emf of a cell is the maximum potential 1. The terminal potential difference of a cell is the
YYoo
ooko

difference between the two electrodes of a cell difference of potentials between the two
when the cell is in the open circuit. terminals of the cell in a closed circuit.
eBB

2. It is independent of the resistance of the circuit 2. It depends upon the resistance between the two
and depends upon the nature of electrodes and terminals of the cell and current flowing through
the nature of electrolyte of the cell.
r

the cell.
ouur
ad

3. The term emf is used for the source of electric 3. The terminal potential difference is measured
current. between two terminals of the cell.
YY

4. It is a cause. 4. It is an effect.
nndd

5. It is greater than terminal pot. diff. when current 5. It is greater than emf of a cell, when the cell is
Re

is drawn from the cell. being charged.


FFii

3.26. GROUPING OF TWO UNIDENTICAL CELLS IN SERIES AND PARALLEL


(a) Two cells in series
The two cells are said to be FIGURE 3.24
81. ri 62, T2 ^eq
connected in series between two points 4-HI 4 11 4 ●4 4
A and C, when negative terminal of one A I B I I C A I I C
cell is connected to positive terminal o o
of other cell, Fig. 3.24(a).
Let Ej, 62 be the emfs of the two cells and rj, K2 be their internal resistances respectively. Let the cells be
sending the current in a circuit shown in Fig. 3.24(a). Let V^, Vg and Vq be the potentials at points A, B and
C, and I be the current flowing through them.
Potential difference between positive and negative terminals of the first cell is
...(26)
CURRENT ELECTRICITY 3/25

Potential difference between positive and negative terminals of second cell is


^BC ^ ^2 ...(27)
Potential difference between A and C of the series combination of the two cells is
...(28)
If the series combination of two cells is replaced by a single cell between A and C of emf and internal
resistance r_„
eq
as shown in Fig. 2>.lA{b); then
'^AC = ^eq-^^eq ...(29)

w
Comparing (28) and (29), we get %q = El + ^2 ...(30)
and
^eq = '*1 + '‘2 ...(31)
If n cells of emfs 8j, 62 8„ and of internal resistance r^, r2, ...r„ respectively, are connected in series
between A and C, then equivalent emf, ^eq=^^\+^2+-+ %

e
equivalent internal resistance of tlie cells is = '■1 + ^2+-+^n-

row
re
Rules for the series combination of cells are as follows ;

(0 The equivalent emf of a series combination of cells is equal to the sum of their individual emfs.

eeF
{ii) The equivalent internal resistance of a series combination of cells is equal to sum of their

FFllo
individual internal resistances.

u
Note that in the series combination of two cells, if negative terminal of first cell is connected to the
negative terminal of the second ceil between points A and C, as shown in Fig. 3.25, then

srr
roF
^AB ~^A~ = ^1 “ ^ ''i ’ ^BC = ^ ^ ^2

k
Now, ^AC ~ ^AB + ^BC = ^ ~ /^2)
uor
Then equivalent emf of the two cells is, - 8j- £2
ofof
= (e, - £0) - / (r, + r2>
£2. ^2
Ei, n
FIGURE 3.25
kos
Y
But equivalent internal resistance is, ^eq = r, + T2. <■ <—i ●4-
Yo
eerBB

A I B I I C
oo

(b) Two cells in parallel


The two cells are said to be connected in parallel between two points A and C, if positive terminal of
rY

each cell is connected to one point and negative terminal of each cell is connected to the other point, as shown
in Fig. 3.26(a). Let the cells be sending the current in a circuit shown in Fig. 3.26(a). Let £|, £2 be the emfs of
u

the two cells and T}, F2 be their internal resistances respectively. Let f, I2 be the currents from the two cells
ou
do
ad

flowing towards point B| and / be the current flowing out of Then


...(32)
nY

/i+/2 = 7

Let Vg , Vg be the potentials at and B2 respectively and V be the potential difference between B^
nid
Re

and B2. Here, the potential difference across the terminals of first cell is equal to the potential difference
FFi

across the terminals of the second cell. So, for the first cell.
-V
v= B
1
-v„h =e,l -7, ,r or

For the second cell.

V =Vr. -V
_^2
B
1
- ^2 h h or

^2
Putting values in (32), we have

7 =
Ej-V Y+ r£2-V ^2 1 1 1
'i I "2 n '■2 'i ^2
r. + .r
1 2
-W
3/26 "Pt^dce^ 4*. Fundamental Physics (XII) LV»1WI

y =
or
...(33)
r. +r.
1 2 'i+'2
If the parallel combination of cells is replaced by a single cell between 5j and ^2 of emf and internal
resistance [(Fig. 3.26(/>)], then
y = e._-/r ...(34)
eq eq

£| r2 +£2 jr (£]/r| )+ (£2^2)


Comparing (33) and (34), we have £ eq ...(35)

w
/j+r2 (l/rj) + (l/r2)
r
r,r2
and eq ...(36)

1 _ [^ +^2 _ ^ , *

e
or
r
...(37)
'i'‘2 ^1 ^2

re
row
eq

^eq _ e.
^1 r„'2 + £2 r, e, 62
Dividing (35) by (36), we have

eeF
r
'*1^2 n ^2

ullo
eq

FF
If n cells of emfs £|, £2, ...£„ and internal resistances r^, r^, ...r„ are connected in parallel, whose equivalent
emf is £^,^ and equivalent internal resistance is then

srr
1 1 I 1 £-, £
-^ + ... + -n

roF
— = —+ — + ... + — and
r
^1 ^2 r

k
r
eq ^1 ^2 r
n eq n
uor
Note that (/) If the two cells connected in parallel are of the same emf £ and same internal resistance r, then

from (35), £
£/- + er
ofof = £
kos
eq
Y
r + r
Yo
eerBB

L-l+i-? r
oo

From (37), or r
eq 2
rY

r .. r r r
eq

Rules for parallel combination of cells are as follows :


u

(/) The equivalent emf ofparallel combination of cells of same emf is equal to emf of one cell as this
combination only increases the sizes of the electrodes and not the effective emf of all the cells.
ou
d
o
ad

(«) The reciprocal of equivalent internal resistance of parallel combination of cells is equal to sum
of the reciprocals of the internal resistance of each cell.
nY

If three cells of different emfs and internal resistances are connected in parallel, then equivalent
nid

(Ej/q ) 4- (£2/^2) + (^3/^)


Re

emf of cells £
eq
.(l/r^) + (\/r^) + {\/r^)
FFi

-1-1
1 1 1
Equivalent internal resistance of cells, '‘eq
— + — + —

'■1 ^2 'a

3.27. GROUPING OF NUMBER OF IDENTICAL CELLS

There are three types of grouping of identical cells,


(a) Series grouping of cells {b) Parallel grouping of cells FIGURE 3.27

(c) Mixed grouping of cells, h I- h


(a) Series grouping of cells. When n identical cells each of emf £
and internal resistance r are connected to the external resistor of resistance I < I
R as shown in Fig. 3.27, i.e., negative electrode of one cell is connected R
to positive electrode of other cell and so on, then the cells are connected ^vvwv
in series.
CURRENT ELECTRICITY 3/27

Equivalent emf of n cells in series. = e + e +...upto n terms = ne.


Equivalent internal resistance of n cells in series,
= r + r +...upto n terms = nr
Total resistance of the circuit = nr + R

ne
Current in the resistance R is given by, I = ...(38)
nr+R
Special cases :
m e
Case (i). If /? < < nr, then R can be neglected in comparison to nr. Then from (38), / = —
nr r

Thus, the current in the external resistor is the same as current due to single cell.
ne
Case («). If /? > > nr, then nr can be neglected as compared to R. Then from (38),

ww
R
Thus, the current in the external resistor is n times the current due to a single cell.
From above, we conclude that

FF loo
from the series combination of cells current drawn is maximum if the value of external resistance
is very high as compared to the total internal resistance of the cells.

ree
(b) Parallel grouping of cells. When m identical cells each of emf

eF
e and internal resistance r are connected to the external resistor of
resistance R as shown in Fig. 3.28, i.e., positive terminals of all the cells

Frre
r ur
are connected at one point and their negative terminals to other point,
fforo
then the cells are connected in parallel. As the cells are connected in
parallel, their equivalent internal resistance r^ is given by
ks
YYouo
1 1 I m r
koso

— = - + - + ...uptom terms - — or r
r r r r P m
P
BBoo

As R and r^ are in series, so total resistance in the circuit = /? + r!m


r ee

In parallel combination of identical cells, the elective emf in the circuit is equal to the emf due to
a single cell, because in this combination, only the size of the electrodes increases, but not emf.
ad
oouur

e
me
.'. Current in the resistance R is given by, / = ...(39)
Y

i? + r/m mR+r

Special Cases:
dnYd
Re

me
Case (/) If /? < < rim, then R can be neglected as compared to r/m. .'. from (39), I =
FFini

Thus the current in the external resistance is m times the current due to a single cell.
me e
Case (h). If r<<R, then r can be neglected as compared to m R I =
mR R
Thus the current in the external resistance is same as current due to a single cell.
From above, we conclude that

from the parallel combination of cells current drawn is maximum if the external resistance is
very low as compared to the total internal resistance of the cells.

3. Mixed Grouping of cells, if the cells are connected as shown in Fig. 3.29, i.e., some cells are
connected in series to form a row and number of such rows of cells are connected in parallel, they are said to
be connected in mixed grouping. Let there be n cells in series in one row and m such rows of cells in parallel.
Suppose all the cells are identical. Let each cell be of emfe and internal resistance r.
3/28 'a Fundamental Physics (XU) P75TW1

In each row, there are n cells in series, therefore their FIGURE 3.29
total internal resistance = n r ne

Their total emf - nt


Since there are m rows of cells in parallel, therefore,
total internal resistance (rp of all the cells is given by h
1 1 1 m nr
— 1 h uptom terms = — or r
nr nr nr P m
V
Total resistance in the circuit-R + nrim I
I' ’
Since the parallel combination of rows of cells does
not affect the emf of each row of the cells but simply increases R

the sizes of the electrodes, therefore, effective emf of all the


cells - nz

ww
effective e.mf. nz mnz
The current in the external resistance R is given by, I =
total resistance R + nr/m mR + nr

Floo
The current I will be maximum if (mR + nr) is minimum, i.e.
[(■yJmR)^ + {-yfnr)^ - l->jmnRr]-\-2-ylmnRr =minimumor (Vm/? - -yjnr)^ + 2 -yJmnRr - minimum

ee
eer
It will be so if, ■yJmR - -y/nr = 0 r> ^ nr

rFrF
or
●JmR = or mR = nr 01 /? = —
m

i.e., external resistance = total internal resistance of all the cells.


rur
equal to the total internal resistance of all the cells.
s ffoor
Thus in mixed grouping of cells, current drawn is maximum if the value of external resistance is
osk
fS'amp^le! 45 cells each of internal resistance 0*4 £2 and emf 2*0 V are used to send
YYoou

current through an external circuit of resistance 2*0 £2. What is the best mode of grouping them for
oook

maximum current ? Find the current in the external circuit.


eBB

Sol. Here, r = 04 £2 ; e = 2-0 V, /? = 2-0 £2


As internal resistance r (= 04 £2) of a cell is comparable of external resistance R (= 2 0 £2), so for
r

maximum current, the cells are to be connected in mixed grouping.


ouur
ad

Let there be n cells in a row and m rows of cells. Then


Y

mn - 45 ...(/)
For maximum current in the mixed grouping
nd dY
Re

nr nx04 n
— = R or = 2 or — =5 or n -5 m ...00
FFini

m m m

From (0 and («).


m X 5m = 45 or m2 = 9 or m = 3
„ = 5x3= 15
Therefore for maximum current there should be 3 orws of cells and each orw should have 15 cells.
mnz 45x2-0
Maximum current, / max = 7*5 A
mR + nr 3x2 + 15x04

DO YOU KNOW ?
(0 When n identical cells each of emf e and internal resistance r are connected in parallel, the total emf of
the cells is e, as the parallel combination of cells increases only the size of the electrodes and does not
change the emf of the cell,
(ij) When n identical cells each of emf z are connected in series, the total emf = nz.
CURRENT ELECTRICITY 3/29

DO YOU KNOW ?
(Hi) When one cell is wrongly connected in series of n identical cells, each of em/e, it will reduce the total
em/by 2 e ie., the effective emf= ne - 2e.
(iv) When n cells each of internal resistance r are wrongly connected in series, the total internal resistance of
the cells = nr, i.e.. there is no effect on the total internal resistance of the cells,
(v) The cells are connected in series to get more voltage, in parallel to gel more current and in mixed
grouping to get more power.

3.28. HEATING EFFECT OF CURRENT ; JOULE'S LAW


Whenever an electric current is passed through a conductor, it becomes hot after some time. This effect
is known as heating effect of current or Joule heating effect. It forms the basis of working of various

ww
electrical appliances such as electric bulb, electric furnace, electric press, geyser, heat convector, immersion
rod etc.

Joule in 1941, studied experimentally the heating effect of current and came to the conclusion that the amount

FF loo
of heat produced {H) when a current I flows through a conductor of resistance R for a time t is given by
I'^Rt

ee
H oc or // = /2/?r joule = cal. (40)
4-2

Fe r
This equation (40) is known as Joule's law of heating.

Frre
r ur
3.29. CAUSE OF HEATING EFFECT OF CURRENT fforo
When a potential difference is applied across the ends of a conductor, an electric field is set up inside
ks
the conductor. The large number of free electrons present in the conductor get accelerated towards the
YYouo
positive end of the conductor, i.e., in a direction opposite to the electric field applied and acquire kinetic
koso

energy. This is in addition to the kinetic energy due to their thermal motion. These accelerated electrons on
BBoo

their way suffer frequent collisions with the ions or atoms of the conductor and transfer their gained kinetic
energy to them. As a result, the amplitude of thermal vibrations of the crystal lattice increases and consequently
r ee

the temperature of the conductor rises. That is why the conductor gets heated due to flow of electric current
through it. Obviously, the electrical energy supplied by the source of emf is converted into heat energy.
ad
oouur

FIGURE 3.30
Y

3.30. HEAT PRODUCED BY ELECTRIC CURRENT

Consider a conductor AB of resistance R, Fig. 3.30. A AAAAAAr B


dnYd
Re

R
Let

potential difference (in volt) applied across the ends of AB, I- ‘


FFini

' 'I

I = current (in ampere) flowing through AB,


t = time (in second) for which the current is flowing.
.●. Total charge flowing from A to B in time t is
q^It ...(41) [v charge = current X time]
By definition of potential difference, work done in carrying unit charge from A to B = V
Total work done in carrying a charge q from A to B is, W =V .x q = V7/joule = /^/?r joule ('.* V =IR)
This work done is called electric work done. If this electric work done appears as heat, then amount of
heat produced (H) is given by
I'^Rt
H =W= joule = calorie ...(42) [ 1 calorie = 4.18 7]
4.18

Equation (42) is a statement of Joule’s law of heating. Thus, Joule’s law of heating states that the
amount of heat produced in a conducto,r when current is passed through it is directly proportional to
3/30 Fundamental Physics fXll^ITZSTWl

(0 square of current flowing through the conducto,r {//) resistance of the conductorand
(Hi) time for which the current is passed.

3.31. ELECTRIC POWER

The rate at which electric work is done by the source of emf in maintaining the current in
electric circuit is called electric power of the circuit.
If a current I ampere flows through a conductor of resistance R ohm for a time t second under a potential
difference of V volt, then electric work done to maintain the current is given by W = V/r joule.
W Vlt
Electric power, P = — = VI watt or Js"'
t t

Power (in watt) = V (in volt) x I (in ampere)

ww
If, V= 1 K and /= 1 A then P=lwatt. 1 watt = 1 y X 1 A = I ampere-volt
Thus the power of an electric circuit is said to be one watt or 1 ampere volt if one ampere

Flo
current flows in it against a potential difference of 1 volt.

e
Bigger units of power are kilowatt (kW) and megawatt (MW).

reree
where. 1 kW= lO^W and I MW= 10^ W

r FF
Commercial unit of power is horse power (hp), where 1 hp = 746 watt.
uurr
As. V = IR or I- V/R, so
foor
The source of this power is an external source to keep a steady
= y2/p
current through the conductor. In a
kss
simple circuit, where the current is supplied to a conductor by the cells, it is the chemical energy of the cells
Yoo
ooook

which supplies this power.


Note : 1. The rate of transfer of electric energy from a source to a circuit, P ~ VI
eBB

2. The rate of transfer of electric energy to thermal or heat energy in a resistor of resistance R is
P = -pR = y^lR
rr

'Sam p I e' Problem D A bulb of484 £2 is producing light when connected to 220 V supply. What
ouu
ad

is the electric power of the bulb ?


Y

Sol. Electric power, P = V^/R = (220)2/484 = 100 watt.


dY

Sample Problem How much current is drawn by the motor of 0*5 hp from 220 volt supply ?
Re
innd

Sol. Here, P = 0-5 hp = 0-5 x 746 watt.


Fi

/=^ 0-5x746
= 1-7 A
V 220

3.32. ELECTRIC ENERGY

The total electric work done or energy supplied by the source of emf in maintaining the current
in an electric circuit for a given time is called electric energy consumedin the circuit.

Electric energy, W = VIi = .P t Electric energy = electric power x time


SI unit of electric energy is joule, where 1 joule = I volt x 1 ampere x 1 second = I wait x 1 second
The commercial unit of electric energy is called a kilowatt-hour (kWh) or Board of Trade Unit
(BOT) or UNIT of electricity (in brief), where 1 kWh -I kilo watt x 1 hour = 1000 watt x 1 hour

Thus 1 kilo watt hour is the total electric energy consumed when an electrical appliance of
power 1 kilo-watt works for one hour.
CURRENT ELECTRICITY 3/31

} kWh = 1000 Wh = (1000 W) x (60 x 60 s) = 3-6 x 10^ J.


watt X hour
Note that the number of units of electricity consumed = No. of kWh = 1000

Electric energy = VI t = I-Rt - V^t/R


Cost of electric consumption = No. of units of electricity consumed x cost of energy per unit.
[s^mpiel^Toiaierni A bulb of 100 W Is Operating for 6 hours a day. Find the units of energy

w
consumed in 7 days. 100x6x7
Sol. Energy consumed = power x time =100x6x7 1V/j = kWh = 4*2 unit of electricity.
1000

3.33. SOME ASPECTS OF HEATING EFFECTS OF CURREm"

e
roow
1. The wires supplying current to an electric lamp are not practically heated while the filament of

re
lamp becomes white hot.
We know that in series connections, the heat produced due to current in a conductor is proportional to its

reF
resistance {i.e., H «« R). The filament of the lamp and the supply wires are in series. The resistance of the

uFFll
wires supplying the current to the lamp is very small as compared to that of the filament of the lamp. Therefore,

e
there is more healing effect in the filament of the lamp than that in the supply wires. Due to it, the filament of
the lamp becomes white hot whereas the wires remain practically unheated.

sFr
2. Electric Iron (i.e. Electric press), electric heater and heating rod are some of the important

foro
household electrical appliances, based on the heating effect of electric current. In all such appliances, the
fk
uor
heating element used is of nichrome (an alloy of Ni and Cr). The wire of nichrgme is used because
okso
(0 it has high value of specific resistance. (ii) its melting point is high.
Y
(Hi) it can be easily drawn into wires. (iv) it is not oxidised easily when healed in air.
Yo
oo
reeBB

Electric iron, electric heater and healing rod are high power instruments. As electric power, P = VI.
Therefore, for the given voltage ,V Poc I.
uurY

Thus higher the power of the electric appliance, the larger is the current drawn by it. Since the heat
produced, H«« hence the heat produced will be large.
It should be noted that electric power, P = V^/R. Therefore, for the given voltage V, P 1/R. This
ad

oc
doo

shows that the resistance of high electric power instrument is smaller than that of low electric power.
nY

The heater wire must be of high resistivity and of high melting point. Heat produced,
nid

vh V^
Re

H = At. The resistivity is kept high so that the length /, used for the given area of cross section of
R P/
FFi

the wire may be small. Nichrome wire is used in heater due to its high resistivity as compared to platinum,
tungsten and copper.
3. Electric Arc. It consists of two carbon rods with a small FIGURE 3.31

suitable separation between them. When a high voltage (40 to 60


volt) is applied, a spark jumps across the gap and a very bright light B
is emitted in the gap, Fig. 3.31.
4. Incandescent electric lamp. It consists of a metal filament f
of fine wire of tungsten enclosedin a glass bulb B with some inert gas
at suitable pressure, Fig. 3.32. The metal filament used must be of
very high melting point. When voltage is applied across the bulb, the
A
current is passed through the filament. The filament gets heated to
very high temperature. It then becomes white hot (i.e. incandescent
.state) and then starts emitting white light at once.
3/32 ^>uuUe^ 'a Fundamental Physics (XII) PZSTWl
5. Fuse wire. A ftise wire is a safety device generally prepared from
tin-lead alloy (63% tin + 37% lead). It should have high resistivity, low
melting point and suitable current rating corresponding to the load in the
circuit. It means the value of the melting point of the fuse wire should be
slightly above the temperature that fuse wire can attain when current
corresponding to maximum safe value flows through it.
The fuse wire is used in series with the electrical installations and
protects them from the strong currents. All of a sudden, if strong current
flows, the fuse wire melts away, causing the breakage in the circuit, thereby
saving the main intallations from being damaged. Thus, very cheap fuse wire
is capable of saving very costly appliances.
6. EiTlciency of an electric device (ii)

w
Efficiency of an electric device is defined as the ratio of its output power to the input power, i.e.,
output power

Flo
n = -:
input power

e
reee
output mechanical power
In case of an electric motor, efficiency =

FFr
input electric power
Here, Input electric power = output mechanical power + power lost in heat

for
ur
Sample Problem electric motor operates on a 110 V supply and draws a current of 10 A.
If the motor yields a mechanical power of 330 watt, what is the efficiency of the motor ?
kkss
Sol. Here, input power = V7 = 110 x 10 = 1100 W
Yo
Output power = 330 W
oo

output power 330


Efficiency of motor = xlOO = xl00 = 30%
B

input power 1100


re

3.34. MAXIMUM POWER TRANSFER THEOREM


ou
ad
YY

■ ■ It states that the power output across a certain load due to a cell or battery is maximum if the
load resistance is equal to the effective internal resistance of cell or battery.
nd
Re

It means, when the effective internal resistance of cell or battery is equal to external load resistance in a
circuit, the efficiency of a battery or cell is maximum.
Fi

3.35. KIRCHHOFFS RULES

In simple electrical circuits, resistance and potential difference are calculated using Ohm’s Law. In
actual practice we come across complicated electrical circuits consisting of a large number of resistors and
sources of e.m.f. Simple application of Ohm’s Law fails to solve these circuits. Kirchhofr gave two rules for
solving complicated electrical circuits. These are known as Kirchhoff’s Rules. While studying these rules,
we shall use two terms : Junction or a node is a point in the electrical circuit, where two or more conductors
are connected electrically. Loop or Mesh is any closed path in an electrical circuit.
1. Kirchhoff’s First Rule.

a According to this rule, the algebraic sum of various currents meeting at ajunction in a closed
electrical circuit is always zero, Le., 11 = 0.
This rule is also called Current Rule or Junction Theorem.
CURRENT ELECTRICITY 3/33

Sign Convention. The currents flowing towards a junction are taken as positive, and the currents flowing
away from a junction are taken as negative.
For example, in Fig. 3.33,0 is a junction in an electrical circuit, where
five conductors carrying currents /|, I2, ly U meeting.
Applying Kirchhoff’s first rule at junction 0, we get
- /; + /2 - /3 + 4 + /5 = 0. i.e., E/ = 0.
^2 + ^4 + ^5 = ^1 + h
i.e., total current flowing towards the junction is equal to total current
flowing out of the junction. It implies that electric current cannot be stored at
a junction. This rule supports the law of conservation of charges, i.e., the
charges do not remain accumulated at a junction of a circuit.
Proof: Multiplying the above equation with small interval of time t, we have

w
or
<22 + 24 + G5 = C] + G3 [v Q^h]
or
Q2 G4 + 65 ~ 2i “ 63 = 0 or EQ = 0

Flo
Thus the total incoming charges to ajunction are equal to the total outgoing charges from that junction.
It shows that a junction of a circuit can not act as a source or sink of charges, i.e., during a given time, the total

ee
charge entering a junction is equal to the total charge leaving that junction. This is what stated by law of

Fr
conservation of charge.
Note : 1. Kirchhoff’s first rule applies equally well when we consider any point on a current carrying
conductor, then current entering a point is always equal to current leaving that point.
for
ur
2. When motion of charge takes place across ajunction momentum of charge is not conserved. It is due
to accumulation of charges on the surface of wires at ajunction which produces an electric field causing the
ks
variation in the direction of momentum of charge.
Yo
oo

2. Kirchhoff’s Second Rule.


B

Around any closed loop of a network, the algebraic sum of changes in potential must be zero,
i.e., IdV = 0. Or, in a closed electrical loop or mesh, algebraic sum of e.m.fs is equal to algebraic
re

sum of the products of various resistances and the respective currents flowing through them,
ou

i.e., Ee = JJR
ad
Y

This rule is called Voltage Rule or the Loop Theorem.


Sign Conventions
nd
Re

Select any point on a closed electrical loop and traverse the loop once in any direction, clockwise or
anticlockwise.
Fi

(i) If negative terminal of a cell is encountered first, its


e.m.f. is taken as positive and vice-versa. FIGURE 3.34

(h) If direction of current through a resistance is the same Ri


A AAAAAAr
along which the loop is being traversed, then the product of ■4-

resistance and current is taken as positive and vice-versa.


“M
For example: Fig. 3.34 shows a section of electrical circuit.
The current distribution is shown according to Kirchhoff’s first R3
B E
rule. Applying Kirchhoff’s second rule to the closed loop ^3
ABCDEFA, we get
. ,J2
— “ “ '^2 ^2 ^ '^1 ^1’ I.e., I.s = Ij7?

Similarly, for closed path BEDCB £2 = (3/?3 + 'l2/?2 C 4- 0


<2 R2
Similarly, for closed path ABEFA £j — i'^R'^ ^ 62
3/34 '4. Fundamental Physics (XII) 1^7*1 ■ I
Kirchhoffs second rule supports the law of conservation of energy
ft'oof: In a closed circuit (or loop) the charges circulate around a closed loop in a particular direction
due to electrostatic force which is a conservative in nature. The workdone by the electrostatic force on the
circulating charge in a closed loop is zero. Due to it, the net change in the energy of charge, when goes on a
closed loop is zero. It means there is no gain in energy by circulating charges around a closed loop in a
particular direction. This is what stated by law of conservation of energy. Thus Kirchhoff’s loop law supports
the law of conservation of energy.
Comparison between Kirchhoff’s first and second rules

FIRST RULE SECOND RULE

1. According to this law £ / = 0. 1. According to this law AV = 0 or Z£ = YAR.


2. It can be applied to open or closed circuit. 2. It can be applied to a closed circuit.

ww
3. It supports the law of conservation of charges. 3. It supports the law of conservation of energy.
4. It is also known as junction law or current law. 4. It is also known as mesh law or voltage law.

Floo
3.36. WHEATSTONE BRIDGE PRINCIPLE

ee
V/heatstone bridge is an arrangement offour resistances in the form of a bridge which is used

reer
rFF
for measuring one unknown resistance in terms of other three known resistances.

Fig. 3.35 shows a standard Wheatstone bridge. It is used for


uur r
measuring unknown resistances. Resistance P in arm AB and
resistance Q in the arm BC are said to form Ratio Arms. Arm AD
ffoor
sks
contains a known resistance R which can be varied. The unknown
YYoo
resistance S is put in arm DC. A cell 8 and one way key is
ooko

connected between the points A and C. A galvanometer C and a


eBB

tapping key K2 is put between the points B and D.


On closing key first and tapping key K2 later, if
galvanometer shows no deflection, the Wheatstone bridge is said to
r
ouur

be balanced. In that case, we shall get


ad
YY

Q~ S
nndd
Re

This is called Wheatstone Bridge Principle.


FFii

Wheatstone bridge principle states that iffour resistances P, Q, Rand Bare connected to form
a bridge as shown in Fig. 3.35, and bridge is balanced (i.e., on closing key Kj first and key K2
P R
later on if galvanometer shows no deflection) then
Q~ S
Proof. Let I be the total current given out by the cell. On reaching the point A, it is divided into two
parts : /j is flowing through P and (/ - / j) through R. At B, the current /j is divided into two parts, through
the galvanometer G and (/| - I^) through Q. The current through the arm BD and (/ - /|) through AD,
combine to send a current (/ - /j +1^) through S. On reaching the point C, the current (/| -1^) through BC and
(i-i]+ig) through DC combine to give total current/, thus completing the circuit. The values of the currents
at a junction can be veriifed by applying Kirchhoff’s first rule at that Junction.
Applying Kirchhoff’sSecond rule to the closed circuit ABDA, we get
7,P + /^C-(/-/,)R = 0 ...(43)
CURRENT ELECTRICITY 3/35

where G is the resistance of galvanometer. Again applying Kirchhoff’s Second rule to the closed circuit
BCDB, we get ...(44)

The value of R is adjusted such that the galvanometer shows no deflection, i.e., = 0. The bridge is said
to be balanced. Putting /^ = 0 in (1) and (2) we have
...(45)
/,?-(/-/,)/? = 0 or

and or
/iG = (/-/i)5 ...(46)

P R
Dividing (45) by (46), we get
Q S
This proves Wheatstone Bridge principle.
It is important to note that
(0 Wheatstone bridge method is a null method, hence the measurement of unknown resistance is very

ww
accurate.

(i7) Wheatstone bridge method remains unaffected by the internal resistance of the source and resistance
of the galvanometer.

Floo
(in) Wheatstone bridge becomes very sensitive when resistances in all the four arms of bridge are
almost same,

ee
(jv) Wheatstone bridge method can not be used to measure very high or very low resistances.

eer
rFrF
Retain in Memory
The equivalent resistance between points A and B for FIGURE 3.36
rur
the circuit shown in Fig. 3.36 is ffoor Rl
■V'/'/v—r—
R2

R^ (Ry+3R^)
s
R
osk
AB
YYoou
{/?2+3^1> Rl
oook

A B
For solution. Refer to Solved Example 86, page 3/64.
eBB

A/vvv— WVv—^
R2 R1
r
ouur

3.37. TEMPERATURE MEASUREMENT USING WHEATSTONE BRIDGE


ad

Take a platinum wire about 50 cm in length. Wind it on non-conducting


Y

FIGURE 3.37

rod with non inductive winding. Connect this platinum wire in the arm CD of
nd dY

Wheatstone bridge circuit. Make other connections as shown in Fig. 3.37.


Re

Working and Theory


FFini

1. Keep the platinum wire S in ice at 0®C. Make P = Q, adjust the value
of R so that on closing key first and key K2 later on, galvanometer shows
no deflection, i.e., the bridge is balanced. If Rq is the resistance of platinum
wire at 0®C, then
P R RQ Kl
or R = R ...(47)
Q ,R0 0 p
2. Keep the platinum wire into steam at 100°C. Repeat the experiment. Let R' be the resistance of arm
AD for which the bridge is balanced. If/?ioo is the resistance of platinum wire, at 100“C, then
P R' R'Q ...(48)
or R = R'
Q R 100 100“^
3. Keep the platinum wire into the bath whose temperature tX is to be determined. Again repeat the
experiment. Let R" be the resistance of arm AD for which the bridge is balanced. If R, is the resistance of
platinum wire at PC, then

I
3/36
Fundamental Physics (XII)
//

or R.t =
R"Q = R
tt

...(49)
P

If a is the temperature coefficient of resistance of platinum wire, then


R
100
^100 — ^0 (^ ■*■ ® ^ ^(^0) or a =
...(50)
Rr.0 xlOO

Also a =
/?, =/?0 (1 + oc X 0 or
...(51)
R^xt
From (50) and (51), we have
^ _ ^100 ^0
i?oXf

ww
R"-R
or t = XlOO = XlOO
[from (47), (48) and (49)]
R R'~R
100 ^0

Flo
Thus, unknown temperature can be calculated.

ee
3.38. SLIDE WIRE BRIDGE OR METER BRIDGE

rere
A meter bridge is a practical form of Wheatstone bridge which is used for measuring unknown

rFF
resistances. It consists of a wire AC of constantan or manganin of 1 metre length and of uniform area of
cross-section. It is stretched between the copper strips on a horizontal wooden board. A metre scale is also
uur r
fitted on the wooden board parallel to the length of the wire. There is another copper strip fitted on the
foor
wooden board in order to provide two gaps in strips. Across one gap, a resistance box R and in another gap,
the unknown resistance S are connected. A jockey B can be slided on the bridge wire. One terminal of the
ks s
sensitive plvanometer G is connected to jockey B and the other to the terminal D. The positive pole of the
Yoo
battery £ is connected to terminal A and the negative pole of the battery to terminal C through one way key K.
ook

The circuit is now exactly the same as that of the Wheatstone bridge. Fig. 3.38.
BBo
re
ouur
ad
YY
nndd
Re
FFii

Clo.se key K and take out suitable resistance R from resistance box. Adjust the position of jockey on the
wire (say at B) where on pressing the jockey, galvanometer shows no deflection. Note the length
AB (= I cm) of the wire. Find the length BC [ = (100 - /) cm] of the wire. Now as the bridge is balanced,
therefore, according to Wheatstone bridge principle.
P R

Q S
If r is the resistance per cm length of wire, then P = resistance of the length / cm of the wire AB-lr
Q = resistance of the length (100 - /) cm of the wire £C = (100 - /) r.

I
CURRENT ELECTRICITY 3/37

/
Ir R 100-/
or S = XR
(100-/)r S /

Knowing / and R, we can calculate S.


End error in meter bridge. It arises due to the following reasons:
(0 The zero mark of the scale provided along the bridge may not start from the position where the bridge
wire leaves the copper strip and 100 cm mark of the scale may not be at position, where the bridge wire just
touches the other copper strip.
(h) The resistances of the connecting wires and copper strips of meter bridge have not been taken into
account.

Removal of End error. The end error in meter bridge can be eliminated by changing R with S and S
with R and repeating the experiment. The average value of S determined earlier and now is free from end
error of meter bridge.

w
fsampielRrogiemTI In a Wheatstone bridge circuit, P = 5fi,g = 6ti,i? = 10fi and S = SQ. Find
the additional resistance to be used in series with S, so that the bridge is balanced.
Sol. Let the bridge be balanced when additional resistance x is put in series with S. Then,

Flo
Q Q
{S + x) = — R or x = ^R-S = -xl0-5 = 7ft

ee
P P 5

Fr
DO YOU KNOW ?
1. Meter bridge or slide wire bridge is the practical form of Wheat stone bridge.

or
ur
2. The Wheat stone bridge is most sensitive when the resistance of all the four arms of the bridge is the
same, i.e., null point is obtained at the middle of bridge wire.
sf
3. The Meter bridge can not be used to measure very high or very low resistances.
ok
4. The balanced position of Meter bridge is not affected on interchanging the positions of battery and
Yo

galvanometer.
Bo

3.39. POTENTIOMETER AND ITS PRINCIPLE OF WORKING


re

Potentiometer is an apparatus used for measuring potential difference between two points in an
electrical circuit accurately. It is also used for comparing the e.m.fs of two cells and for measuring internal
ou
ad

resistance of a cell. FIGURE 3.39


Y

A potentiometer consists of a long uniform B |?T{iniiihi ru^nTiriiiTTii ihi iliiiifilini Ss


wire generally made of manganin or constantan,
nd
Re

stretched on a wooden board. (Sometimes there are K 400


four or more wires, each one metre long, fixed
Fi

parallel to one another and connected in series by -K

thick copper strips). Its ends are connected to the Ai


300

binding screws A and B. A metre scale is fixed on


the board parallel to the length of the wire. The
X" e 200
potentiometer is provided with a jockey J with the
help of which, the contact can be made at any point
on the wire. Fig. 3.39. A battery e, an ammter Aj, a aI^o 100

key K and a rheostat Rfj are connected in series with +


V
potentiometer wire.

Principle. The working of a potentiometer is based on the fact that the fall ofpotential across
any portion of the wire is directly proportional to the length of that portion provided the wwe IS
of uniform area of cross-section and a constant current is flowing through it.
3/38
Fundamental Physics (XinPZSTW
Suppose A is the area of cross-section and p is specific resistance of the material of the wire. Let K be the
potential difference across the portion of the wire of length / whose resistance is R. If I is the current flowing
through the wire, then from Ohm’s law; V =IR\ As, R = p//A
/
where K =
/p
V = Ip- = Kl,
^A A
or Vocl (if I and A are constant)
/.e., potential difference across any portion of potentiometer wire is
directly proportional to length of the wire of that portion.
Here, V/l - K = is called potential gradient, i^e., the fall of
potential per unit length of wire.
Obviously, the graph between length (/) and potential difference
(V) across it will be a straight line, passing through the origin, as shown

ww
in Fig. 3.40.

sampiej|.^^ogi,e^ A Current of 1‘0 mA is flowing through a potentiometer wire of length 4 m


and of resistance 4 Q, find the potential gradient of potentiometer wire.

Flo
Sol. Here ; / = 1 -0 mA = 10“^ A ;L = 4m,*7? = 4Q

ee
Potential drop across potentiometer wire, V = I R = I0~^x4 V

rere
r FF
oPotential
, . , ^
gradient, K = — =
V 4x10~3 = 10-^ V/m
L 4
uurr
A battery of emf e is connected between the
foor
3.40. DETERMINATION OF POTENTIAL DIFFERENCE USING POTENTIOMETER
FIGURE 3.41
ks s
end terminals A and B of potentiometer wire with
Yoo
1- h 03 <●>
ooook

ammeter A j, resistance box R and key K in series. e K


This circuit is called an auxiliary circuit. The ends
eBB

of resistance /?| are connected to terminals A and


jockey J through galvanometer G. A cell £j and 0 50 100

key Ky are connected across R^ as shown in Fig.


rr

s
ouu

3.41.
ad

A B

Working and Theory. Close key K and take


YY

out suitable resistance R from resistance box so C D


-T—

that the fall of potential across the potentiometer


nndd
Re

wire is greater than the potential dilference to be '-1-0-'


measured. Ki
Fii

It can be checked by pressing, firstly the jockey J on potentiometer wire near end A and later on near end
//, the deflections in galvanometer must be in opposite directions.
Close key AT,. The current flows through /?,. A potential difference is developed across /?,. Adjust the
position of jockey on potentiometer wire where if pressed, the galvanometer shows no deflection. Let it be
when jockey is at J. Note the length AJ (= /) of potentiometer wire. This would happen when potential
difference across /?j is equal to the fall of potential across the potentiometer wire of length /. If K is the
potential gradient of potentiometer wire, then pot. diff. across /?], i.e.,
V=Kl
...(52)
If r is the resistance of potentiometer wire of length L, then current through potentiometer wire is
^ ~ /? + r e
Potential drop across potentiometer wire = Ir = r
R + r
CURRENT ELECTRICITY 3/39

Polential gradient of potentiometer wire, i.e., fall of potential per unit length is
( e r
K = From (10), V = —

,.(53)
/? + r L
^ R "h r ) L,
Hence, V can be calculated.

3.41. COMPARISON OF EMFs OF TWO FIGURE 3.42

CELLS USING POTENTIOMETER


S K
A battery of emf e is connected between the end I- !■ AAAAAVv (●)-l
Rh
terminals A and B of potentiometer wire with rheostat
Rh, ammeter Aj and key K in series. This circuit is
called an auxiliary circuit. Two cells of emfs e^ and £2
to be compared are connected to the circuit as shown
in Fig. 3.42. The positive terminals of both the cells

w
are connected to point A of the potentiometer. Their
negative terminals are connected to two terminals 1
and 2 of two way key, while its common tenninal 3 is

Flo
connected to jockey J through a galvanometer G.

eeee
Close key K and adjust a suitable constant current in the potentiometer wire with the help of rheostat.
Insert the plug in the gap between the terminals 1 and 3 of two way key so that the cell of emf £| is in the

Fr
circuit. Adjust the position of jockey on potentiometer wire, where if pressed, the galvanometer shows no
deflection. Let it be when jockey is at J^. Note the length A/| (= /| say) of the wire. As the galvanometer is
showing no deflection, therefore, there is no current in the arm Aejyj. It means the potential of positive tenninal of
for
ur
cell = potential of the point A, and the |xHential of negative terminal of cell = potential of the point J^.
Therefore, the e.m.f. of the cell (= 6|) is equal to potential difference between the points A and of the
ks
potentiometer wire.
Yo
I.e., ...(54)
oo

where K is the potential gradient across the potentiometer wire.


eB

Now remove the plug from the gap between 1 and 3 and insert the same in the gap between 2 and 3 of
two way key so that cell of emf Eo comes into the circuit. Again, find the position of jockey on potentiometer
wire, where galvanometer shows no deflection. Let it be at J2- Note the length of the wire AJ2 (= ^2 Then
r

proceeding as above.
ou
ad

£2 = KI2 ...(55)
YY

Dividing (54) by (55), we get


£2 h.
nd
Re

Precautionsof experiment
Fi

1. The current in the potentiometer wire from driving cell E must be kept constant during experiment.
2. While adjusting the position of jockey on potentiometer wire, QQ YOU KNOW V
the edge of jockey should not be rubbed on the wire, otherwise area of
cross-section of wire may not ermain uniform and constant. The potentiometer method is a
null method. It is a very accurate
3. The current in the potentiometer wire from driving cell should
method. Its working does not
not be passed continuously for long time as this would cause heating
depend on the accuracy of
effect, resulting in the change in resistance of wire. reading.
galvanometer
4. The emfoi the driving cell must be greater than the cm/of the cells Potentiometer acts like an ideal
whose values are to be compared, otherwise null point will not be obtained voltmeter of infinite resistance,
on potentiometer wire. because when galvanometer
5. The positive terminals of all the cells (£, E| and £2) are to be connected shows no deflection, no current
atzeroendA of potentiometer wire, otherwise null point will not be obtained is drawn from the cell.
on potentiometer wire. All lengths are measured from this end.
3/40 ‘Pnctdee^'^^ Fundamental Physics fXITIPTSTgl
3.42. DETERMINATION OF INTERNAL RESISTANCE
OF A CELL BY POTENTIOMETER METHOD
To find the internal resistance r of a cell of emf
FIGURE 3.43
e using potentiometer, set up the circuit as shown in
Fig. 3.43. K

Close key K and maintain suitable constant h I- 0)


Rh
current in the potentiometer wire with the help of
rheostat Rh. Adjust the position of jockey on the
potentiometer wire where if pressed, the galvanometer
shows no deflection. Let it be when jockey is at /j.
Note the length A7j (= /j) of the potentiometer wire.
Now emf of the cell, e = potential difference across

ww
the length /| of the potentiometer wire, as no current
is being drawn from the cell.
z = Kl 1 (56)

Flo
where K is the potential gradient across the

ee
potentiometer wire.
Take out suitable resistance R from the resistance box in the cell circuit and close key Again find the

rere
rFF
position of the jockey on the potentiometer wire where galvanometer shows no deflection. Let it be at J2-
Note the length of the wire AJ^ (= I2 say). As current is being drawn from the cell, its terminal potential
uur r
difference V and not emf e is balanced across AJ2. Therefore, potential difference between two terminals of
foor
the cell, V = potential difference across the length I2 of the potentiometer wire
sks
i.e.
V=Kl2 (57)
Yoo
oko

1-i
Dividing (56) by (57), we have ...(58)
BBo

V
h
We know that the internal resistance r of a cell of emf e, when a resistance R is connected in its circuit
re

is given by
ouur
ad

r =
e-v
xR = i-ll/? ...(59)
YY

V
nndd

I
h (2 xR
Re

1
Using (58), we get r= -!--l R=
h h
FFii

Thus, knowing the values of /j, I2 and R, the internal resistance r of the cell can be determined.

3.43. SENSITIVENESS OF POTENTIOMETER

The sensitiveness ofpotentiometer means the smallest potential difference that can be measured
with its help.

The sensitiveness of a potentiometer can be increased by decreasing its potential gradient (V/7). The
same can be achieved

(0 by increasing the length of potentiometer wire.


(//) If the potentiometer wire is of fixed length, the potential gradient can be decreased by reducing the
current in the potentiometer wire circuit by increasing the resistance, with the help of rheostat.
CURRENT ELECTRICITY 3/41

3.44. DIFFERENCE BETWEEN POTENTIOMETER AND VOLTMETER

POTENTIOMETER VOLTMETER

1. It measures the emf of a cell very accurately. 1. It measures the emf of a cell approximately.
2. While measuring emf, it does not draw any 2. While measuring emf, it draws some current from
current from the source of known emf. the source of emf.
3. While measuring emf, the resistance of 3. While measuring emf the resistance of voltmeter
potentiometer becomes infinite. is high but finite.
4. Its sensitivity is high. 4. Its sensitivity is low.
5. It is based on null deflection method. 5. It is based on deflection method.
6. It can be used for various purposes. 6. It can be used only to measure emf or potential
difference.

ww
DO YOU KNOW ?
(i) A balance point is obtained on the potentiometer wire if the fall of potential along the potentiometer

Flo
wire due to driving cell is greater than the emf of the cell to be balanced.
(i7) A balance point can not be obtained on the potentiometer wire if the fall of potential along the

ee
potentiometer wire due to driving cell is less than the emf of the cell to be balanced.

rere
{in) The potentiometer is an ideal voltmeter as it measures the emf of a cell very accurately whereas voltmeter

r FF
measures the emf of cell approximately,
(iv) The positive terminals of driving cell and the cell whose emf '\s to be determined must be connected to
uurr
foor
the zero end of the potentiometer wire, otherwise null point is never obtained. Zero end is the end from
where balancing length of potentiometer wire is to be measured.
ks s
Yoo
ooook
eBB
rr

Solution. Here, I = 300 mA = 300 x 10 ^ A ;


I I
ouu
ad

TYPEI. ELECTRIC CURRENT


f = 60 s, n = ?
YY

Formulae used.
Q
(0 I = —
ne
— ev =
ev
ne It (300xl0~^)x60
/ = — or n- —
t t 2nr 1-6x10-'^
nndd

t e
Re

(ii) I =Q/t-{rig + rip) e!t. = 1*125 X lO^O


Fii

where Q is the charge flowing, t is the time, is Example Q In an atom an electron


the number of electrons and rip is the number of revolves around the nucleus in a circular orbit at
positive ions or holes crossing through a cross
section of conductorand e is the charge of electron the rate of 6 x 10*^ revolutions per second.
or proton or hole = 1-6 x 10“^^ C. Calculate the equivalent current in milliampere.
,-19
Take value of electronic charge = 1*6 x 10 c.
V is the frequency of revolution of electron in an
orbit of nucleus of radius r with speed v. Solution. Here, v = 6 x 10*^ s“^ ;
Units used. Q is in coulomb, t in seconds, I in e = l-6x 10-^^C
ampere, v m s
-1
or Hz andv in ms”^
Equivalent current, I = ev
= (1-6 X 10"^^) X 6 X 10'^ = 9*6 X 10^ A
Example 0 How many electrons pass = 9-6 X 10"^ X 10^ mA
through a lamp in one minute, if the current
is 300 mA ? = 0*96 mA
(Pb. Board 2002)
3/42
4. Fundamental Physics (XII)

Example^ In Bohr model of hydrogen Example The charge flowing in


i a

atom, the electron revolves around the nucleus in conductor varies with time as,
a circular orbit of radius 5*0 x 10"^^ m with a speed
2*2 X 10^ ms“^. Find the equivalent current. q =at —bt^ + -cr
"2r 6
(Electronic charge = 1*6 x 10“^’ coulomb) where a, by c are positive constants. Then,
Solution. Here, r = 5*0 x 10~* * m:
find (i) the initial current (ii) the time after which
1; = 2-2 X 10^ ms"’, e = 1-6 X 10“'^ C. the value of current reaches a maximum value
Let V be the frequency of revolution of electron (iii) the maximum or minimum value of current.
in hydrogen, then dq
Solution. (/) Current, i =
1 1 V dt

T {2nr/v) 2nr d (
at - - bt'^ + - ct^
Current, dt[ 2 6

w
ev
^ (l-6xl0“‘^)x(2-2xl0^) =
. 1
a-bt + — cr
2
I = ev = ...(O
2
2nr 22
2x — x50xl0“*'

Flo
When t - 0, initial current, i - a
7

e
di
= M2 X 10-3 A (ii) = — b + ct

ree
dt
Example Q If 0*8 mole of electrons flow

FFr
For i to be maximum or minimum.
through a wire in 55 minutes. What is (a) the total
di b
charge in kilocoulomb that passes through the
urr
— = 0 = -/? + c/ or

wire and (b) the magnitude of the current ?


Avogadro’s number = 6 x 10^3 per mole. for dt

(Hi) Putting this value of t in (/). we have


c
kkss
Solution. No. of electrons in 0-8 mole, b^ b- b^
z.
~bx-+-
^ 1
A^ = 0'8x6x 10-3
Yo
i = a = a 1— = a-
ooo

c 2 c 2c 2c
(a) Charge flowing,
As this value of i is less than that at r = 0, it
B

q = Ne = (0-8 x 6 x lO^'^j ^ {1-6 x lO''^) C must be minimum. So


= 7-68x I0^C = 76-8x lO^C
re

= 76*8 kilo coulomb b^


minimum value of current = a~
ou

2c
ad

76-8x1()3c
(/>) Current = — = 23-3 A
YY

t (55x60) s Example B A steady beam of a-particles


travelling with kinetic energy E = 83*5 keV carries
Exsmpie g In a conductor, 10*^ electrons
ndd

a current of / = 0*2 p.A. Mass of a-particle


Re

move from a point A towards point B in 1 milli


= 6-68 X kg.
second. lO^"* positive ions move from a point B
Fi

towards point A in 1 milli second. What is the (i) If this beam strikes a plane surface at an
current in ampere and its direction ? Charge on angle 0 = 60° with normal to the surface, how many
electron = charge; on positive ion - 1*6 x 10“*’ C. a-particles strike the surface in t = 4 second ?
Solution. Here, = lO'^ ; = lO'**, t~ 10“3 s, (ii) How many a-particles are there in length
/ = 20 cm of the beam ?
charge on electron or ion, q= 1-6 x 10“*’ C.
Solution. Here, / = 0-2 A = 0-2 x 10“^ A,
Current, 1 = £■ = 83-5 keV = 83-5 X I03eV
t
= 83-5 X 1Q3x 1-6 X lO-'^J
10’^ xl-6xl0“”-^10‘^x 1-6x10“'’ = 83-5x 1-6 X
10-3
(0 Charge on a-particle,
10'^xl-6xl0-” g = 2c = 2x l-6x lO-'^C
= 1*6 A
10-3 No. of a-particles crossing per second a cross-
The direction of current is from B to A. section of beam.
CURRENT ELECTRICITY 3/43

I 0-2x10-^
/ = 0-2 A,n = 84x ;e = 1-6 x 10"'^ C.
n = — = 6-25xl0*’s-* I I
2xl-6xl0"'^ V
d ~
n Ae n(n D^/4)e
No. of a-particles striking the surface in 4
second = nt^ 6-25 x lO' * x 4 = 2*5 x 10*^ 0-2

(84xl0^«)xfx^
[Note. There is no significance of angle xl-6xl0"‘^
6 (= 60°) for finding the number of a-partides striking
the surface.] = 1-895 X 10-5 ^ -1
{//) If V is the velocity of a-particle while
1I
Example Q A 60 coulomb of charge flows
2
travelling towards a surface, then E = —mv through a wire in half minute. The radius of the
wire is 1 mm. The wire contains 5 x 10^^ electrons

ww
per cubic centimetre. Calculate the current and
2x83-5x1-6x10"’6
or v =
drift velocity.
6-68x10"2'^ Solution. Here, ^ = 60 C, / = 30 seconds ;

Floo
= 2x 10^ ms' 1 r - 10"^ m ; « = 5 X 10^^/cc = 5 x 10^® m"5
It means a beam of length u = 2 x 10^ m crosses 60

ee
Current, / = — — = 2A
a section in one second. But number of a-particle t 30

reer
passing through a section in one second is,

rFF
/ I
11
n = 6-25 X 10 Drift velocity, = 2
ne A neitr
uur r
n

No. of a-particles in unit length of the beam = —


No. of a-particles in length / of the beam
V ffoor 2

(5xl028)xa6xl0-‘^)x3-142x(10"3)2
sks
YYoo
nl 6-25x10’^ = 7-95 X l(r5 ms“^
ooko

xO-20 = 6-25 X 10^*


V 2x10^
eBB

Example m A current of 3 A is flowing


TYPE II. DRIFT VELOCITY through a wire of length 2 m and cross-sectional
AND ELECTRON MOBILITY area 1 mm^. If wire contains 10^^ electrons/m^,
r
ouur

calculate the average time taken by an electron to


ad

Formulae used
cross the length of the wire.
Y

(i) I =ne A Vj (if) = vJE = qilm Solution. Here, / = 3 A, / = 2 m,


where, n = number density of electrons i.e. number A - 1 mm^ = lO"*^ m^, n = 10^ m"^
dY

of conduction electrons per unit volume of conductor.


Re

I
nnd

A = area of cross section of the conductor, = drift


Drift velocity, =
FFii

velocity of electron, neA

£ = strength of electric field applied = VA, 3


V = potential difference across the conductor of 102^x(1-6x10-‘^)x10-^
length 1.
Units used, n in m"^, A in m^,
-1
in ms ,£in N/C = 1-875 X 10-^ ms-^
or V/m and / in m. I 2
Time taken, t = -A
= 1-07 X lO'* s
1-875x10
Example Q] A copper wire of diameter
1-0 mm carries a current of 0-2 A. Copper has Example m If the free electron density of
8-4 X 10^ atoms per cubic metre. Find the drift copper is 8-6 x 10^ m~^ and resistivity of copper
velocity of electrons, assuming that one charge at room temperature is 1-7 x 10"^ Q cm, find the
carrier of 1-6 x 10“*^ C is associated with each relaxation time for the free electrons of copper.
atom of the copper. Given, mass of electron = 9-1 x 10“5' kg ; and
Solution. Here, D - 10 mm = 1 x 10“5 m. charge of electron = 1-6 x 10“'^ C.
3/44
a. Fundamental Physics (XII) P/aiBi
Solution. Here, m = 8-6 x 10^* m“^ ; Example
Find the current flowing
p= 1-7 X 10-^ticm= 1-7 X 10“^ Qm ; through a copper wire of length 0*2 m, area of cross-
m = 9-1 X 10“^* kg, e = 1-6 X 10“^^ section 1 mm^, when connected to a battery of 4 V.
m
Given that electron mobility = 4*5 x I(T^ V“* S"*
As, p = —^ ; so, T =
ne 'Z
ne^p and charge on electron = 1-6 x C. The number
9-lxlO-^i density of electron in copper is 8*5 x 10^ m"^.
x =
Solution. Here, / = 0-2 m ;
(8-6 X10^8) X (1-6 x 10" )2 X (1-7 x 10“^) A - 1 mm^= 10“^ m- ; V^=4 volt;
= 2*43 X 10"^“* s
p = 4-5x lO-^m^V-’ s"' ;
Example m What is the drift velocity for e = 1-6 X 10“*^ C and n - 8-5 x 10^^ m“^.
the electrons in a conductor when an electric field We know that electric field set up across the
of strength 200 V/m Is applied on it and mobility conductor, E= —
V
— = 20Vnr‘

w
of electrons is 4*5 x 10“^ m^ V“* s~* ? I 0-2
Solution. Here, E = 200 V/m ; Now, current through wire is
p = 4-5x 10-6 m2 v-l S-' I~nAev^=nAe [iE=(B-5 x 10^^) x (10"6)

Flo
Drift velocity, v^=\iE= (4-5 x 10"6) x 200 x (1-6 X 10-'^) x (4-5 X 10-6) X 20

eeee
= 1-22 A
= 9-0 X 10-^ ms-i

Fr
Example [Q If the Current flowing through Example Estimate the average drift
a copper wire of 1 mm diameter is 1*1 A. The density speed of conduction electrons in a copper wire of
of copper is 9 g cm“^ and atomic mass of copper is
for
ur
cross-sectional area 1*0 x 10“^ m^ carrying a
63 u. One free electron is contributed by each atom current of T5 A. Assume that each copper atom
of copper. Find the drift velocity of electron. contributes roughly one conduction electron. The
ks
Avogadro’s number = 6*0 x 10^6 per kg-atom. density of copper is 9-0 x 10^ kg/m^, and its atomic
Yo
Solution. Here, 2 r = I mm = 10'^ m mass is 63*5 u.
oo

I
-xlO'^m ;/= M A (b) Compare the drift speed obtained above
B

or r =
2 with (i) thermal speed of electrons carrying the
re

d = 9g cm-2 = 9 X 10^ kg/m^. current at 27°C and («) speed of propagation of


electric field along the conductor which causes the
Therefore, mass of 1 m^ of copper = 9 x ICP kg
ou

drift motion.
ad

Atomic mass, Af = 63 u = 63 kg.


Avogadro’s number = 6*0 x 10^6 per kg-atoni.
Y

Avogadro’s number,
= 6-0 X 10^6 per kg atom. Boltzmann constant A: = 1-38 x 10“23 j/k ; mass of
electron = 9’1 x 10“^^ kg.
nd

So 6-0 X 1C)26 atoms of copper has a mass = 63 kg


Re

(NCERT Solved Example)


N _Nd
Fi

No. of atoms per m^ of copper = Solution, (a) Here, A = l-O x lO'^ 2 .


m*";
M/d M
/ = 1 -5 A ; £/ = 9-0 X 1Q2 kg/m^
As 1 atom contributes one conduction electron,
therefore, number of conduction electrons per m^ of Atomic mass, M = 63-5 u = 63-5 kg
copper is Avogadro’s number, V = 6-0 x 10^6 per kg atom
Nd _ (60x1026)x(9x1Q2) Number of atoms per unit volume in the given
n =
M 63 copper wire - N ^Nd
= 8-57 X I028 m-2 M/d ~ M
I As one atom contributes one conduction
V
d ~
ne A electron, therefore, number of conduction electrons
M per unit volume or number density of conduction
\2
electrons in the given copper wire is
(8-57x1028)x(1-6x10-19)x lixfi-xlO-2 N j 6-0x1026x9-0x102 = 8-5x1028 m-2
7 1^2 n = —d =
M 63-5
= 10^ m/s
CURRENT ELECTRtCITY 3/45

I Solution. Here, R = 5 0 ; rj = 5 x 10 ^ m ;
V
d ~
neA D = 2-0 X 10"^ m ; p = 2-0 X lO""^ Qm.
RnD^
1-5 R = or / =
nD'^/4 4p
(8-5x1028)x(1-6x10-^9)x(1-0x10-'^)
Let n be the number of turns in the coil. Then
= M X ir^ ms"* total length of the wire used, / = 2 7i rj n
(b) (i) The thermal speed of electron of mass m RD^
I RnD^

w
at temperature T is. or n =

3kT
27ir.1 4px27cq 8pr,
m 5-0 X (2-Ox 10-3)^ = 250
where k is Boltzmann constant. ”8x(20x10'''^)x(5x10-2)

e
roow
Here, r= 300/f;
[Q A wire of 10 H resistance is

re
Example
1-38 X 10-23 J mole'’K"' .
stretched to thrice its original length. What will
m = 9-1 X 10“3> kg be its (i) new resistivity and (») new resistance ?

reF
uFFll
1/2
-23 Solution. (0 Since resistivity depends on the

e
3x(l-38xl0 )x300
V = nature of material of wire and is independent of
9-1x10-3’
dimensions of wire, so the resistivity remains

sFr
= 1-17 X 10^ ms-’ unchanged.

foro
ofk (»’) In both the cases, the volume of the wire is
uor
V 1-1x10-3
d _ = 10"* same.
V i-nxio*-^
lA^IA^A
kos
So V = Al=A'l' or A' =
(ii) The speed of propagation of electric field is l'~3t 3
YYo
equal to the speed of an electromagnetic wave,
reeBB
oo

c = 3 X 10^ m/s. Now


A
uurY

Vj MxIQ-3 = 0-36x10-” =3-6 X 10-’2 pT _p3/_9£/


and R' = = 9R
c 3x10^ A' “ A/3 ~ A
ad

/r = 9xio = 9on
doo

TYPE III. OHM’S LAW,


Example [0 A wire has a resistance of
nY

RESISTANCE, RESISTIVITY
16 a. It is melted and drawn into a wire of half its
nid

Formulae used. (/) V = IR length. Calculate the resistance of new wire. What
Re

m is the percentage change In its resistance ?


FFi

(ii) R = pl/A or p = RA/l m p =- Solution. In both the cases, the volume of the
wire is same. So
where p is the resistivity of the material of the wire
conductor, / is the length and A is the area of cross- V = Al-A'l' or A' = ftL- — = 2A
section of the wire, e is the charge of electron (= 1 -6 x I' ” U2
10"’^ C), n is the number density of electron in the Now «=£'
conductor and t is the average time of relaxation. A
Units used. V in volt, I in amp, R in ohm, / in metre, p/’ px(//2)_ 1 pl__R
A in m2, p in m and n in m"3 and R' =
A'~ 2 A 4 A~ 4
16
Exam pi® A wire of resistance 5*0 Q is R' = — = 4Cl
4
used to wind a coil of radius 5 cm. The wire has a
% decrease in resistance
diameter 2*0 mm and the specific resistance of its
material is 2*0 x 10“’ ilm. Find the number of R-R'
xl00 =
16-4
xlOO =75%
turns in the coil. R 16
3/46
T^n^teUefi^ '4, Fundamental Physics (XII) VOL.I

There are two wires A and B Calculate the number of electrons flowing through
of same mass and of the same material. The it in one second. Given charge on electron,
diameter of wire A is one-third the diameter of e = l-6xlO-l^^C.
wire B. If the resistance of wire A is 30 Q, find the Solution.Here, V =3 volt;/? = 1 -5Q ;
resistance of wire B. e=l-6xlO-^9Q,^ls
Solution. Here, = 30 Q. Let /^ , be the Now, I = — -2--2A and I = Q. = —
length and diameter of wire A and Ig, Dg be the length R 1-5 t t
and diameter of wire B. Let d be the density of the It 2x1
material of wires A and B. or n = —
= 1*25 X 10^^
e 1-6x10-*9
Mass of the wire = volume x density
Find the relaxation time for
= area of cross-section x length x density
As mass of two wires is same so free electrons in copper, if the density of mobile
electrons is 8*4 x 10" m“®. The resistivity ofcopper

w
m =
nbl nOl at room temperature is 1*7 x 10^ Om. Given :
4 ^ mass of electron = 9*11 x 10“^l kg and charge on

Flo
electron = 1*6 x 10“^*^ C.
D\Ia=dIIb Solution. Here, « = 84 x 10^^ m~^ ;

reee
B _ Pj 1 p = 1-7 X 10-* Q m; m = 9-11 x lO'^l kg ;
/
or
e= l-6x 10-l9c;i: = ?

FFr
^A 0| D| 9
m

^ = ixi = i We know that, P = -2


urr
4p/^/(nD2)
Ra 30
d| 9 9 81 or
for
x =
m
e nx
kkss
/Ip
911x10-3‘
Yo
ooo

(l-6x 10-*9 )2 X 84x 102« X1-7 x lO"*


In a discharge tube the = 2-49xir^'»s
eB

number of hydrogen ions (i.e., protons) drifting A copper wire is Stretched to


across a cross-section per second is 1*0 x 10^ ,
make it 0*2% longer. What is the percentage
r

while number of electrons drifting in opposite


ou
ad

direction across another cross-section is 2*5 x 10^^ change in its resistance ?


Solution. The mass m of the wire of length /,
YY

per second. If the supply voltage is 220 V, what is


the effective resistance of the tube ? area of cross section A and density d is given by
nndd

Solution. Here, = 10 x 10^® s“^ m


Re

m = Ald or A = —
n = 2-5 X 10^8 s-l Id
The resistance R of the wire of resistivity p is
Fi

The current due to motion of negative charged given by


electron is equivalent to the current due to motion of
proton in the opposite direction. Therefore, total
current in the direction of motion of proton is A m

/ = total charge flowing per second where k = p dim is a constant of the wire.
= {rip + Xe
= (10 X 10*8 + 2.5 X 10*8) X 1.6 X 10-*9 ” R / ’
= 0-56 A
% increase in ersistance
Effective resistance. dR 2x0-2
220 xl00 = xl00 = 0*4%
= 3*93 X 102 Q R 100
I 0-56
When 5 Vpotential difference
A potential difference of 3 V is applied across a wire of length 0*1 m. The drift
is applied across a conductor of resistance 1*5 Q. speed ofelectrons is 2*5 x 10^ ms“^. If the electron
CURRENT ELECTRICITY 3/47

density in the wire is 8 x 10^ m calculate the It shows that copper wire is 2 i6 times heavier
resistivity of the material of wire. (CBSE OD 2016) than aluminium wire. Since for the same value of
Solution. Here, V = 5 V ; / = 0-1 m ; length and resistance, aluminium wire has lesser mass
- 2*5 X 10"^ ms“^ « = 8 X 10^^ ms“^ than copper wire, therefore aluminium wire is
preferred for overhead power cables. A heavy cable
As, V = /?/ = — x{nAev^) = plnev^ may sag down owing to its own weight.
Pi
TYPE IV. CURRENT DENSITY,
or p = CONDUCTANCE AND CONDUCTIVITY
IneVj
5 Formulae used. {/) J = I/A\

0-1x(8x102^)x(1-6x10-'9)(2-5x10^) 1 1
(ii) G = — and {Hi) ^
= 1*56 X 10“® £2m R

ww
Example ^ Two wires of equal length, where J is the current density, G is the conductance,
one of aluminium and the other of copper have o is the electrical conductivity and is the electron
the same resistance. Which of the two wires is mobility.

Floo
lighter ? Hence explain why aluminium wires are Units used. J in Am“- ; G in £2“* or S and o in
m“* or Sm“'.

ee
preferred for overhead power cables. Given, For
Al, pj = 2*63 X 10“** £2 m, For Ch, p2 = 1'72 x

eer
Example ^ A wire carries a current of

rFrF
10“* £2 m. Relative density of Al = 2*7, of Cu = 8*9.
Solution. Given,/or a/tt/nim’Km wire ; 0*5 A, when a potential difference of 1*5 V is
applied across it. What is its conductance ? If the
rur
= /?; /j = /; Relative density = 2-7
For copper wire^ ffoor
wire is of length 3 m and area of cross-section
5*4 mm^, calculate its conductivity.
s
R2 = R;l2 = l\d2 = ^-9 Solution. Here, / = 0-5 A ; V= 1*5 V, / = 3 m ;
osk
YYoou
Let Aj, A2 be the area of cross section for A = 5-4 mm^= 54 x 10“*^ m^
oook

aluminium wire and copper wire.


V
eBB

l 2-63xl0"*x/ Now, resistance, R = - = 3Q


We know, R, = pj — - ...(0 / “0-5
^1 A.
1 _ -1 =0-33S
r

Conductance, G = -
and mass of the aluminium wire. R~ 3
ouur
ad

OTj = Aj / j X frj =Aj / X 2-7 ...{ii) 1 /


Electrical conductivity, a = —
Y

/ l-72xl0“^x/ p RA
Also,
nd dY

3
Re

= 1-85 X 105 Sm"*


Mass of copper wire, 3x54x10“^
FFini

m2 = A2I2X ^2~ All ^ 8-9 ...(iv)


Since two wires are of equal resistance Example ® Calculate the electric field in
^1 = ^2 a copper wire of cross-sectional area 2’0 mm
square carrying a current of 2 A. The resistivity
2-63xl0“*x/ l-72xl0"*x/
of copper is 1*7 x 10"* £2m.
A
1 Solution. Here, A = 2-0 arm sq = 2 x 2 mm^
= 4 X 10"^ m^.
or ...(V) / = 2A;p = l-7xl0-*£2m.
~A^~^
V IR Ipl/A /p
From (n) and (iv), we have Electric field £ = —
I I I A
m, A2/X8-9 8-9,, A2 _ 8-9^, 1-72 = 2-16
2x(l-7xl0~**) = 8-5 X 10"^ V/m
m
1 A,/x2-7 2-63
4x10“^
[From (v)]
3/48
‘P'uztiee^ '4- Fundamental Physics (XII) ESSBD
Example An aluminium wire of (b) For a steady flow of current, /, = I2
diameter 2*5 mm is connected in series with a or ~ ~ 2yjA2
copper wire of diameter 1*6 mm. A current of
2‘0 A is passed through them. Find (a) current or
4 A 2 ^ or =
'i
density in aluminium wire, (b) drift velocity of V2
electrons in copper wire. Given the number den 2x10"^
sity of conduction electrons in copper is 10^^ m"^. or ~ - V2 X10 ^ m = 1*414 mm
V2
2-5
Solution. For AI, /j = ^ mm = 1.25 mm TYPE V. VARIATION OF
= 1-25 X 10'3m;/ = 2'0A RESISTANCE WITH TEMPERATURE

Area of cross-section, A^ = nr^ Formulae used

= 3-14(1.25x10-3)2 =4.9x10-^ m2 (i) /?, = /?„(!+ at)

w
1-6 ((7)a = (/?2-/?i)//?i it2-h)
For Cu, ^2 = Y - 0-8 mm = 0-8 x 10“3 m; where R^, = resistance of conductor at
= 1029m-3 temperature 0“C and fC respectively,

FF llowo
n
a = temperature coeff. of resistance
Area of cross-section. A2 = TCr^ In relation (//), oc is temperature coeff. of resistance

ee
= 3-14 (0-8 X 10-3)2 = 2-01 X 10-® m2 of the conductor averaged over the temperature

Fr
(a) Current density in aluminium wire, range /j®C to r2“C.
(Hi) a = (i?2 - R])/(R^t2 - /?2f]) where a is the
j = -L 2
= 4.08 X 10^ A m-2 temperature coefficient of resistance at 0®C.

er
ur
A, 4.9x10-^ Units used. Rq, R^, R^ and /?2 is ohm, a in
fofr For
orK-*.
{b) Drift velocity in copper wire.
ks
Example RTi] A tungsten coil has
YYour o
I 2-0 a
oo

^ neA^ 1029x1.6x10“'^x2.01x10-® resistance 16 £2 at 20”C. If the temperature


coefficient of resistance of tungsten is 0*004 ®C"*,
BoBks

= 6-2 X 10-5 ms
-1
calculate the resistance of the coil at 80*^0.
r ee

Example m Current flows through a Solution. Here, /?2o = 16 Q ; = 20“C,


t2 = 80°C, a = 0-004 “C"* = 4 x 10~3
oouru

constricted conductor as shown in Fig. 3.44. The


ad

radius and the current density to the left of /?80 = ?


Y

constriction are 2 mm and 4*2 x 10® Am"2. /?20 ~ ^0 (1 + CC X 20)


and
Ynd

FIGURE 3.44
Re

A
■ ^0 ^ l + qx80 _ l-f(4xl0-3)x80
FFind i

EDt, ^2 R20 l + ax20 l + (4xl0-3)x20


w 1-32

1-08
(a) How much current flows through the
constriction ? (b) If the current density is doubled 1-32 1-32
as it emerges from the right side of the constriction, ^0 - 1-08
xR
20 “
108
xl6 =19*6Q

what is the radius T2 ?


Solution. Here, rj = 2 mm = 2 x ICT^ m ; Example m A conductor has a cross-
yi=4-2x 105 Am-2 section of 15 mm2 resistivity of 7*6 x 10“* £lm
(u) Current flowing through the constriction at 0“C. If the temperature coefficient of resistance
of the material of the conductor is
/] = y,Ai = 7] X Kr^ 5 X 10“3 ®C“^ calculate its resistance for 2 km
= (4-2 X 105) X (22/7) (2 x 10-3)2 == 5-28 A length of conductor when its temperature is 60®C.
CURRENT ELECTRICITY 3/49

Solution. Here, A = 15 mm^ = 15 x 10“^ ; Exsmple m A heating element using


p = 7-6 X 10-8 a m, a = 5 X 10"3 °C'K nichrome connected to a 115 V supply draws a
current of 1-6 A which settles after a few seconds
/q = 2 km = 2000 m ; t^= 0°C, = 60“C, to a steady value of 1-4 A. What is the steady
p/ (7-6x10~8)x2000 temperature of the heating element if the room
^~ A ” 15x10 -6
= 10-12 a
temperature is ? Temperature coefficient of
resistance of nichrome averaged over the
^60 = ^0 temperature range involved is 1-7 x 10“^ °C"*.
= 10-12 [1 + 5 X 10"^ x60] Solution. Here, initial room temperature,
= 10-12 X 1-3 = 13-16 n fi = 30°C ; /i = 1 -6 A, V= 115 V, a = 1-7 X 10^
Example ^ The heating element of an Let t2°C be the final steady temperature when
current settles down to, /2 = 1-4 A.
electric toaster is of nichrome. When a very small y 115

ww
current passes through it, at room temperature Initial resistance, R t = 71-875 Q ;
27“C, its resistance is 75-3 il. When toaster is I ~ /j “ 1-6
connected to a 230 V supply, the current settles Final resistance, R
V 115
= 82-143 Q ;
after a few seconds to a steady value of 2-68 A. 1-4
^2

Flo
What is the steady temperature of nichrome a= l-7x 10^°C-^

e
element ? The temperature coefficient of resistance R

ree
of nichrome over averaged temperature range is /2
1-7 X 10-^°C-i. As, ^2 “ ^1

Fr
(NCERT Solved Example) R, X .a

rF
Solution. Here, /?27 = ^5-3 Q, R. -R i
uurr
82-143-71-875
a=l-7x 10-^"C-^ y = 230V, /=2-68A 1

for
f-30 =
230 Rj i a 71-875x1-7x10^
= 85-82n ;
s
2-68 = 840-3‘*C
kks
Hence, t = 840-3 + 30 = 870-3”C
_ ^2
Yo
oooo

As, ^2 ^1 “ R.xa Example ES A standard coil marked 5 £2


is found to have a resistance of 5-128 £2 at 30"C.
eB

R. ~R 85-82-75-3
f-27 =
I 27 _ = 822‘’C Calculate the temperature at which the marking
R^^xa
27
75-3x1-7x10-^ is correct. The temperature coefficient of
resistance of the material of the coil is -0042 "C“^.
r
ou

/ = 822 + 27 = 849‘‘C
ad

or
Solution. R,'1 =5i2, R =5-128i2,
YY

Example m The resistance of a platinum rj = ?, f2 = 30“C.


wire of platinum resistance thermometer at the
nndd

R
(1 + a f,) 1 + a /1
Re

ice point is 5 i2 and at steam point is 5-23 SI. When I


1

thermometer is inserted in a hot bath, the R


/?g(l + ar2) l + a?2
Fi

resistance of the platinum wire is 5-795 f2.


Calculate the temperature of the bath ? 5 1 +0-0042 xr 1
or
(NCERT Solved Example) 5-128 1 + 0-0042x30

Solution. Here, /?q = 5 Q, /?^oo ” On solving, = 23-3"C


/?, = 5-795 a,/ = ? Example EE The resistances of iron and
R
As, a=
100
copper wires at 20“C are 4-1 £2 and 4-3 £2
/^xlOO respectively. At what temperature will the
resistance be equal ? Temperature coefficient of
5-795-5
t = xl00 = xlOO resistance for iron is 5-0 x 10“^ K“* and for copper
^100 ^ 5-23-5 is 4-0 X 10"^ K“^. Neglect any thermal expansion.
0-795 Solution. For iron, R-^q = 4-1 £1, a = 5-0 x 10“^ K“*
XlOO =345-65‘’C
0-23 From copper, = 4-3 £2, - 4-0 x 1(T^
3/50
Fundamental Physics (XII)EEIHI
Let at temperature f°C, the resistance of iron
Example m A current of 2mA is passed
wire be equal to the resistance of copper wire. So through a colour coded carbon resistor with first,
/?20 [1 + C( (/ - 20)] = +«'(?- 20)] second and third rings of yellow, green and orange
or 4-1 [1 +5-0x l(r^(r-20)]=4-3[l +4-0x 10“^ colours. What is the voltage drop across the
resistor ?
x(r-20)]
Solution. For colour coded carbon resistor, the
(t - 20) X 10"3 [4-1 X 5-0 - 4-3 x 4-0] = 4-3 - 4-1
0-2
numbers attached for yellow, green and orange are 4,
or r-20 = = 60-6 5 and 3 respectively. Therefore, the resistance of the
3-3x10"3 given carbon resistor, R = 45x 10^ .Q.
/ = 20 + 60-6 = 80-6‘’C Voltage V = RI= (45 X 10^) x (2 x lO’^) = 90 V

ooww
Exam
pie m Resistivity of the material of Example A voltage of 5 V is applied
across a colour coded carbon resistor. A current
a conductor of uniform cross-section varies along
its length as p = (1 + a x). Find its resistance if of 5 mA flows through it What are the colours of
second and third band of coded colour resistor ?
its length is L and area of cross-section is A.
Solution. Resistance of the material of a Solution. Here, V = 5V, / = 5mA = 5 x 10“^ A

ee
conductor of length dx at distance x from one end is Resistance of carbon resistor.

r
rFFl
ree
dx dx V 5

F
^^ = P—= PoO + ct-«) — /? = -
I 5x10-3
= io^n= lox lo^a

rF
[Given, p = pQ (I + ax)] Therefore, the colours of second and third bands
The resistance of the material of conductor of will be related to numbers 0 and 2, i.e., black and red.

fsfoor
ouur
length L is
TYPE VII. COMBINATION OF
kosk
R = RESISTANCES IN SERIES AND PARALLEL
Pod + ax)— L+
0 ^ A L 2
Yo
Formulae used.
oo
Y

I
(0 In series, R^ = R^+ R2 + R^ + ...
BB

TYPE VI. COLOUR CODES


OF CARBON RESISTORS (ii) In parallel. L-i_ J_ _L
rre

R. R
p 1 /?2 R^
Formulae used. The number assigned to different
oYuu

colours from 0 to 9 can be recollected by the sentence where R^ is the total resistance of the various
ad

resistances /?j, /?2, R3. .... connected in series and


B.B.ROY Great Britain Very Good Wife Wearing
dY

Gold Silver necklace. Rp is the total resistance in parallel combination.


Black Brown Red Orange Yellow Units used. R^, R2, R^ Rp all in ohm.
innd
Re

0 1 2 3 4
Green Blue Violet Grey White Example ED Two resistances are in the
FFi

5 6 7 9 ratio 1: 4. If these are connected in parallel, their


Multiplier for gold is 10 ^ and for silver is 10“^. total resistance becomes 20 ohm. Find the value
For tolerance or % accuracy, the colour used are ; of each resistance. (Pb. Board 2000)
Gold for ± 5%, Silver for ± 10% and No colour for Solution. Let/?j, be the two given resistance,
± 20%.
Then or R. = — -.(0
4 * 4
Example^ What is the colour of the
third band of a coded resistor of resistance When these resistances are connected in parallel
2-3 X 10^ « ? (Karnataka Board 2012) then

Solution. Resistance of resistor = 2-3 x 10^ Q R


/?l/?2 or 20 =
(/?2/4)/?2 ^2
= 23 X 10* a
(R2/4)+~ T
Therefore, the colour of third band of a colour 100
coded resistor will be related to number 1, i.e., brown. or R-y = 100 Q ; From (i), R. =~ = 25f>
* 4
3/51
CURRENT ELECTRICITY

This resistance is in parallel with 10 Q. The


Example E0 Two conductors of conduc
10x10
tances Gj and G2 are connected in seri^. They effective resistance between A and C -
10 + 10
= 50
are connected in parallel to another conductor of
conductance Gy Determine their equivalent This resistance is in series with resistance 5 O,
conductance. the effective resistance = 5+5 = 100
Solution. Let /?j, R2 and be the resistances This resistance is in parallel with resistor
of conductors of conductances Gj, G2 and G3 10 O in arm AB. Thus equivalent resistance between
respectively. 10x10
A and B = = 50
1 1 10 + 10

oww
1
Here R,1 -
r ’ r ,R3 = (n) The resistance of arm A5C(= 5 + 10= 15 0)
Gi G2 ^3 is in parallel with resistance 10 O of arm AC.
Refer to Fig. 3.45, the effective resistance 15x10
between A and B is Their effective resistance = = 60
15 + 10

e
FIGURE 3.45 This resistance is in series with resistance 7 O

re
R3 of arm CD. Their effective resistance = 6 + 7
= 13 O. This resistance is in parallel with resistance

FFllro
reF
3 O of arm AD.

e
The equivalent resistance between A and D
A
uoru Rl R2
B

sFr
13 + 3 16
/?3X(B, +/?2)

foro
/? = (Hi) The resistance of arm ADC = 3 + 7=100
+ (^j + ^2 ^
fk Resistance of arm ABC = 10 + 5 = 15 O
okso
Effective conductance, These resistance are in parallel.
R^ + /?j + /?2 They are also in parallel with resistance 10 O
Y
1
Yo
G = - of arm AC.
oo
BB

R
/?jX(/?j +R2) Therefore, equivalent resistance between A and
1 1 1 Cis
rYree

^3 G ^2 1 _ 1 1 1 _ = HO
oouu

I or R
G =
1 1 1
R "iO 10 15 30 eq 8 " 4
eq
ad

g/g2^
X
Yd

G3 Example ^ A resIstor of 5 O is connected


nidn

in series with a parallel combination of a number


of resistors each of 5 O. If the total resistance of
Re

(G2 + G,) the combination is 6 O, how many resistors are in


FFi

Example In the given network of resis- parallel ?


tors, Fig. 3.46, find the equivalent resistance Solution. Let n resistors each of 5 Q be
(0 between the points A and B («) between the points connected in parallel. Their effective resistance is
A and D and (m) between the points A and C. 1 5
Solution. (0 The resistors 3 Q and 7 Q are in = - + — + - + ...« times = — or R
R 5 5 5 5 P n
series. Their total resistance = 3 + 7 = 10 p

As this parallel combination of resistances is


connected in series with 5 Q resistance, the total
resistance of the combination is

/? = R_+5 = - + 5 = 6
p n

or -=6-5=1 or n = 5
n
3/52
“Pnauiee^'^ Fundamental Physics (XII) VOL.l

Example E0 A uniform wire is cut into (0 For resistance beh^'cen opposite comers C
three parts with their lengths in the ratio 2:3:6. and G, we have two resistances in parallel each of
The ends of each of these three parts are connected value Rll.
across an ideal battery of 10 V. If a current of 5 A Therefore, the equivalent resistance between
is drawn from the battery, find the initial
resistance of the wire. (/f/2) (/e/2) n
(CBSE 2020) opposite corners is
Solution. Let 11 / be the initial length of the wire ~ 4
of area of cross-section A and resistance R^. Then («) For resistance between adjacent comers A
and H, we have two resistances of /2/h and (n - 1) /?/«in

w
R. =
P(110
1
A parallel. The equivalent resistance is
When length of the wire is cut into tliree parts {Rln){n-\)Rln («-l)
of lengths 2 /, 3 / and 6 /. Let /?,, Rj and R-^ be their (/?/«) +(«-!)/?/« n
2
resistances. Then

e
roow
Example
Calculate the potential

re
R. =
I
P(2/) . ^ _p(3/) and =
difference between points B and D of the network
A ’ ^ A ^ A
of resistances shown in Fig. 3.48.

reF
These three resistances are connected in parallel,

uFFll
then equivalent resistance is

e
FIGURE 3.48
1 I A A 6Q B 4Q
●WW 'VWV

sFr
R R1
eq ^3 P(2/) p(3/) p(6/)

foro
A an i2n c
A r3 + 2-Hl
fk A
uor
A 1 1 1 ●SAW ●W.V
+-+-
D
p/L2 3 6J p/L 6 P^
okso
P^ V V
Y
R
Yo
eq ^ 12 V
oo
reeBB

But R Solution. Here, potential difference across ABC


uurY

eq
/ 5 A = potential difference across ADC = 12V
Current in arm ABC,
Now, initial resistance. R.I =
ad

A A 12
/.I = = 1-2 A
doo

= 11 X 2 = 22 Q 6 +4
nY

Example ^ \ uniform wire of resistance Pol. diff. between A and B,


nid

R is shaped into a regular n sided polygon where


Re

^A-^e = /|X6=l-2x6 = 7-2V


n is even. Find the equivalent resistance between Current in arm ADC,
FFi

(i) opposite corners of polygon (ii) adjacent


12
corners of polygon.
^2 “ 8 + 12
= 0-6 A
Solution. Let the polygon be as shown in Fig.
3.47. The resistance of each side of polygon = R/n. Pot. diff. across A and D,
FIGURE 3.47 ~ Vq = /2 X 8 = 0-6 X 8 = 4-8 V
D E (Va - ^b) - (Va - V'd) = 7-2 - 4-8 = 24 V
or
Vj^-Vs = 2-4\
C Vs-V^=-2-4\
It shows that the point D is at higher potential
G
than that of point B.
B
Example^ In the circuit shown In Fig.
A H 3.49, find the potential difference across

capacitor.
CURRENT ELECTRICITY 3/53

Solution. Here, e = 10 V, r = 0-3 O,


FIGURE 3.49
3n
c /? = ?,/= 1-2 A
B —►—AA/vNAr E
(i-hV' 3|iF
g
Current, I
R + r
30 3n
10
or R = - r = --0-3
30
I 1-2
l-i D /
A F = 8-3 - 0-3 = 8-0 Q
i- ‘ 15V ' 'i

i G
Example ^ A cell of emf e and internal
resistance r gives a current of 0*5 A with an
Solution. When capacitor is fully charged, it external resistance of 12 Q and a current of

ww
draws no current, and hence provides infinite 0*25 A with an external resistance of 25 Q.
resistance, then no current flows in arm E.F The Calculate (a) internal resistance of the cell and
potential difference across the capacitor is equal to (b) emf of the cell. [CBSE 2013 (C)l
the potential difference across C and F. The resistance Solution. Let be external resistance in series

Flo
of arm ABCD is in parallel with the resistance of arm with the cell of emf e and internal resistance r. The

e
AD and they together are in series with resistance of current in circuit is

eree
arm D.F The effective resistance between A and F of
circuit is, / = _!—

FFr
R + r
3x6
uurr R = + 3 =2 + 3 = 5Q In first case, 1 = 0-5 A, R- 12 .Q

orr
3 + 6

15 sfo 0-5 = —^
12 + r
Main current in the circuit, / = — = 3 A
or e = (12 + r) X 0-5 = 6 0 + 0-5 r ...(0
kks
Yoo
Current through arm BCD In second case, I = 0-25 A, R-25 .O
oooo

_ ix3 _3x3 = 1A 0-25 = —^


eBB

“6+3” 9 25 + r
Potential difference across C and £) = 1 x 3 = 3 V or e = (25 + r) x 0-25 = 6-25 + 0-25 r ...(«)
uurr

Current through arm DF, i-3 A From (0 and (»),


Potential difference across D and F=3x3 = 9V
ad

6-0 + 0-5 r =6-25 +0-25 r


Yo

So, potential difference across C and F-3 + 9


dY

or r=lQ.
= 12 V
From (0, e = 6 0 + 0-5 X 1 = 6-5 V
Thus, potential difference across the capacitor
innd

Example ai A uniform wire of resistance


Re

is 12 V
12 £2 is cut into three pieces in the ratio 1:2:3
Fi

and the three pieces are connected to form a

I
TYPE VIII. EMF, TERMINAL P.D. AND
INTERNAL RESISTANCE OF A CELL triangle. A cell of emf 8 V and internal resistance
1 £2 is connected across the highest of the three
Formulae used, (i) e = V + /r, where V - IR. resistors. Calculate the current through each part
Here, e is the emf of a cell ; V is the terminal pot. of the circuit.
diff., r is the internal resistance of the cell, Solution. When 12 £2 resistance wire is cut into
R is the external resistance in circuit, I is the current.
three pieces AB, BC and CA in the ratio 1:2:3, then
Units used, e in volt, V in volt, I in ampere and r resistances of these parts will be.
or R in ohm.
1
-xl2 = 2£2 ; -xl2
- = 4£2
Exam pP? ES a resistor is connected to a 6 6

battery of emf lOV and internal resistance 0*3 £2. and -xl2 = 60
What is the resistance of the resistor to be inserted 6
in the circuit for the circuit current 1*2 A ?
3/54
‘PnaxUep. ^ Fundamental Physics (XII) VOL,I

respectively. As per question, the cell ofemf 8 V and Pot. diff. across voltmeter
internal resistance 1 Q is connected across CA (the = (5-991 X 10-3) iQQo
highest resistor) as shown in Fig. 3.50. = 5-991 V

Thus terminal potential difference of the cell,


V = pot. diff. across voltmeter
= 5-991 V

e-y
%error = xlOO
V

oww
6-5-991
XlOO =0-15%
6

Example S] Potential differences across

ee
the terminals of a cell were measured (in volt)
against different currents (in ampere) flowing

FFrlo
Here 2 Q and 4 Q are in series. Their effective

r
through the cell. A graph was drawn which was a
resistance = 2 + 4 = 6 Q. This is in parallel with 6

F
straight line A5C as shown in Fig. 3.52.

ee
resistance. The effective resistance between A and C,

Fr
ouru R =
6x6
= 3£I
V(Volt) FIGURE 3.52

sor r
6+ 6

Total current from the cell,

/ = -i_ = -
8
= 2A
os kffo
R+r 3+1
ook
Yo
Let /], /2 = currents through arms AC and ABC
Y

respectively. Since the resistance of arm AC


Bo
reeB

- resistance of arm ABC, so = ^2 = 2 2


(Ampere)
oouY

Example ^ The emf of a battery is 6-0 V Determine from the graph (i) emf of the cell
ur

and its internal resistance is 1*5 Q. Its potential («) maximum current obtained from the cell and
ad

difference is measured by a voltmeter of resistance (in) internal resistance of the cell.


d
Y

1000 Q. Calculate the percentage error in the [CBSE 2011 (C)l


reading of emf shown by voltmeter.
nidn

Solution. (/) EMF of the cell is equal to


Re

Solution. Here, e = 6 0 V ; r = 1-5 H ; maximum potential difference across the two


Resistance of voltmeter, R - 1000 £2 electrodes of cell corresponding to zero current. Thus
FFi

Refer to Fig. 3.51, Current in the circuit is emf of the cell, e = 1*4 V.
6 y (») Max. current is drawn from the cell when
/ = —
= 5-991 X 10-3 A the terminal pot. diff. is zero. Therefore
/? + r I000 + 1-5
/^ = 0-28 A
FIGURE 3.51

r
Cell (»0 Internal resistance,
“I
6-OV £ 1-4 y
r
r =
= sn
I—^AAr I 0-28 A
max
1-5Q

I L_. j Example Find the current drawn from


/

a cell of emf 1 V and internal resistance (2/3) Q


@ connected to the network shown in Fig. 3.53.
R = iooon [CBSE 2001 (C)l
CURRENT ELECTRICITY 3/55

I TYPE IX. GROUPING OF CELLS


I
Formulae used.
ne
(0 Cells in series, I -
R + nr

(/i) Cells in parallel, I =
mR + r

mnz
(in) Cells in mixed grouping, I =
mR + nr
where n = no. of cells in series in each row, m = no.
of rows of cells in parallel.

ww
Units used. I in ampere, e in volt, /? or r in ohm.
Solution. The equivalent circuit is shown in Fig.
3.54.
Example ^ Two identical cells of emf

Flo
1*5 V each joined in parallel provide supply to an

e
FIGURE 3.54
external circuit consisting of two resistances of

ree
in
A B
17 Q each joined in parallel. A very high resistance

Fr
voltmeter reads. The terminal voltage of cells to be

rF
in ia
1*4 V. Calculate the internal resistance of each cell.
uurr
Solution. Here, e = 1-5 V, /?j = /?2 = 17
for
F G
ia in V= 1-4 V

Refer to Fig. 3.55, resistance of external circuit,


s
kks
in
R.R
_ 17x17
Yo
V'2
oooo

D C "I R = = 8-5Q
R^+R2 17 + 17
eB

FIGURE 3.55
IV
I WAr 1-5V
r
r
ou
ad

Resistance in arm AB — 1 Q
1-5V
YY

r
Resistance in arm FG

1x1 1x1 I 1
nndd

=-+-=in Ri = i?n
Re

1+1 1+1 2 2 —VWv—

Resistance in arm DC = 1 n
Fi

R2= 17Q
As resistance of these arms are in parallel, their —Wv\—

equivalent resistance is
As the two cells are connected in parallel, their
J__l 1 1^3 effective emf, z~ 1 -5 V,
"l'*’l'^l 1 their effective internal resistance,
1
or R r' = rxr _ r

3 r+r 2’
Total resistance of circuit. (z-V R
Then, r =
1 2
R = R +r = - + - = m
e
3 3 r r 1-5-14
So 8-5
2 14
r ^ ^ *
.●. Current, / = — = - = lA 01X 8-5
R 1
or r = x2 =l*2n
2
3/56 ^●ta^eUep- ^ Fundamental Physics (XII) orsTWl
Example^ A 20 V battery of internal Solution. Here, 8 = 1-5 V, r = 0-5 £2, = 2 £2.
resistance 1 Q is connected to three coils of 12 Q, Since the value of r is comparable to /?, for the
6 n and 4 £2 in parallel, a resistor of 5 £2 and a best mode, the cells are connected in mixed grouping.
reversed battery \emf= 8 V and internal resistance Let there be m number of rows and n ceils in each
- 2 £2) as shown in Fig. 3.56. Calculate the current row. Then nm - 36 ...(0
in each resistor and the terminal potential For maximum current in the mixed grouping
difference across each battery. nr nxO-5
— = R or -2 or n = 4 m
[CBSE 2001 (C)]
m m

FIGURE 3.56 From (0, 4 m X m = 36 or = 9 or m = 3


I
20V and n = 4x3 = 12
I 1ft I
{— i Thus for maximum current, there should be 3
I
rows and 12 cells in series in each row.

ww
I

5ft
(O Maximum current,
8V
2ft
'“I
m m 36x1-5

Floo
^ \ —V/v /
max
= 4-5 A
mR + nr 3x2 + 12x0-5

ee
Solution. Equivalent resistance R' of 12 £2, Example Two cells JJj and E2 in the

reer
6 £2 and 4 £2 connected in parallel is circuit shown in Fig. 3.57, have emfs of 5 V and 9

rFF
V and internal resistances of 0*3 £2 and
J__J_ I _1^_ A = 1 or = 2 £2 1*2 £2 respectively. Calculate the value of current
R'~ 12 6^
uur r
12 2 flowing through the resistance of 3 £2.
Total resistance of circuit, /?-l + 5 + 2 + 2
ffoor (CBSE 2006, 2010)
sks
= 10£2 FIGURE 3.57
YYoo
Effective emfoi circuit = 20 - 8 = 12 V
ooko

5V. 0-3 ft 9V, 1-2 ft


12
eBB

Current in the circuit / = — = 1-2 A


10 El E2

So, current through each battery and 5 £2 resistor 6ft


r

= 1*2 A AAV AAV


ouur
ad

4-5 ft
Pot. diff. across the parallel combination of three
Y

resistors, V = IR' = 1 -2 x 2 = 2-4 V


3ft
dY

2-4
Current in 12 £2 coil = = 0*2 A Solution. Net emf= £2 - = 9 - 5 = 4 V.
Re

12
nnd

6x3
FFii

2-4 Total resistance = 0-3 + 1 -2 + 4-5 + = 8£2


Current in 6 £2 coil = -76 = 0*4 A 6+3
4V
2-4 Current through the circuit, I = = 0-5 A
Current in 4 £2 coil = = 06 A 8Q
4

Terminal pot. diff. across 20 V battery 6x0-5


Current through 3 £2 resistance =
V=e-/r=20- l-2x 1 = 18*8 V 6+3 3^
Terminal pot. diff. across 8 V battery Example ^ (a) Three cells of emfs, 1*5 V,
K = £' + // = 8 + 1-2 x 2 = 10*4 V 2*0 V and 2*5 V are connected in series. Their
m internal resistances are 0*20 £2,0*15 £2 and 0*15 £2
Example HU Thirty six cells each of emf respectively. The battery is connected to an
1*5 V and internal resistance 0*5 £2 are used to external resistor of 5*5 £2 via a very low resistance
send current through an external resistor of ammeter, what would be the reading of ammeter ?
resistance 2 £2. What is the best mode of grouping (b) If the three cells above were joined in
them and the current through the external resistor. parallel, would they be characterised by a definite
CURRENT ELECTRICITY 3/57

emf and internal resistance (independent of their Example nyi 8 cells, each of internal
individual internal resistance) ? If not, how will resistance 0*5 A and emf 1-5 V are used to send a
you obtain currents in difTerent branches of the current through an external resistance of
circuit ? (a) 200 Q (b) 0-002 Q (c) 1-0 Q. How would you
Solution, (a) Here, Ej = 1-5 V ; £2 = 2-0 V ; arrange them to get the maximum current in each
case ? Find the value of current in each case.
£3 = 2-5 V ; = 0-2 £2 ; r2 = 0-15 £2 ;
Solution. Here, total number of cells = 8 ;
r3 = 015Q;K = 5-5£2
r = 0-5a;e= 1-5 V.
Since the cells are joined in series, the effective
emf of all the cells is («) When R = 200 £2 ; then R >> r, so for

oww
maximum current, the cells are to be connected in
e = £| + £2 + £3 = 1 -5 + 2-0 + 2-5 = 6-0 V series in circuit.
Total internal resistance of all the cells is
Total internal resistance of 8 cells = 8 r
r = r| + T2 + T3 = 0-20 + 0-15 + 0-15 = 0-50 £2 Current in circuit.
Total resistance of the circuit = R + r
8e 8x1-5 12-0

ee
= 5-5 + 0-5 = 6-0 £2 I = = 0-059 A
R + Sr 200 + 8x0-5 204

FFrlo
£ 6
(^) When R = 0-002 £2, then /? < < r ; so for

r
Current, / = — = 1A
R + r 60 maximum current, the cells are to be connected in

rF
ee
(^) No, they will not be characterised by a parallel in circuit.
definite emf and internal resistance, since there is no Total internal resistance of 8 cells = r/S
ouru
rF
simple formula for total emf and total internal Total resistance of circuit = /? + r/8
resistance of cells joined in parallel. However, current = 0-002 + 0-5/8 = 0-0645 .Q
in different branches of the circuit can be obtained
ffosor
os k Effective emf of all cells = emf of each cell
using Kirchhoff’s laws. = 1-5 V

Example 0] Two identical cells, whether


ook
1-5
Yo
Y
Current in circuit, / = = 23-26 A
joined together in series or in parallel give the 0-0645
Bo

same current, when connected to external When R= 1 -0 £2, then R is comparable to r.


reeB

resistance of 1 £2. Find the internal resistance of For maximum current, the cells are to be connected
each cell.
in mixed grouping. Let there be m rows of cells in
oouY
ur

Solution. Let £, r be the emf and internal parallel with n cells in series, in each row. Then,
ad

resistance of each cell. External resistance /? = I £2. m n = 8 ...(/)


dY

n r
When the two cells are connected in series
For maximum current, R =
m
Total emf of cells = £ + £ = 2 £
nidn

nxO-5
Re

Total resistance of circuit = R + r + r=l+2r or 1-0 = or « = 2 m


m
2e
FFi

Current in the circuit, /,1 = From(0, m X 2 m = 8 or = 4or m-2.


l + 2r
So «=2x2=4
When the two cells are connected in parallel
Thus. 4 cells in series in a row and 2 such
Effective emf of two cells = emf of single cell = £
Total internal resistance of two cells rows of cells in parallel.
rxr r ne mne
Max. current, / =
r + r 2 R + nrhn mR + nr
Total resistance of the circuit = /? + r/2 = 1 + r!2. 8x1-5
= 3A
£
Current in the circuit. 2xl + 4x0-5
/2 = l + r/2
Example |20 In the given circuit as shown
As per question, i\= I2
2e £ 2t
in Fig. 3.58, in the steady state, obtain the
so expressions for (0 potential drop iii) the charge
l + 2r l+r/2 2+r
and {Hi) the energy stored in the capacitor C.
or l+2r = 2 + r or r=l£2 (CBSE (F) 2015)
3/58 ‘P’uzdee^ '4- Fundamental Physics (XII)

FtGURE 3.56 ne 6x20


Current, / = = 14 A
V R R + nr 8-5 + 6X0015
A F
Terminal voltage, V= / /? = 14 x 8*5 = 11*9 V.
ib) Here, e = 1-9 V; r = 380n.
C
B E /
e _ 19 = 0-005 A
max
r 380

This amount of current cannot start a car because


2V 2R
C 1 D
to start the motor, the current required is 100 A for
few seconds.

Solution. In the steady state, the capacitor C is


the fully charged. So no current flows through the I TYPEX. ELECTRIC POWER
I

w
arm BE of the circuit. Formulae used. Electric power ;
Effective emf of the circuit = 2 V - V = V P = VI = I^R^ —

Flo
Effective resistance of the circuit = 27? + /? R

= 3R where P is lower, V is voltage, I is current and R is


resistance.

ee
y
Current in the circuit, / = Units used. P in watt, V in volt, I in A, and R in

Fr
3/?

Potential difference across B and E Example Rfl How many electrons flow
for
ur
y 4
through the filament of 220 V and 100 W electric
= 2V-/x2/? =2V- x2/? = -y lamp per second. Given, electronic charge
3R 3
= 1-6 x 10-1^ C.
ks
(/) Potential difference across C Solution. Here, V = 220 V, P =100 W, r = 1 s
Yo
oo

V
^ ^ P _ 100 _ 5 ^
B

3 3 V 220 " 11
re

(//) Charge on capacitor ne


/r_ (5/ll)xl
But / = - = or n =
V rv r t e ~ 1-6x10-'^
ou
ad

Q = CV = Cx- = —
3 3 = 2-84 X 10^*
Y

(///) Energy stored in the capacitor


Example A heating element is marked
\2
nd

1 y cy2 210 V, 630 W. What is the current drawn by the


Re

= -c
2 3 18 element when connected to a 210 V dc mains ?
Fi

V /
What is the resistance of an element ?
(a) Six lead-acid type of (NCERT Solved Example)
secondary cells each of emf 2-0 V and internal Solution. Here, y = 210 V. P = 630 W,
resistance 0-015 £2 are joined in series to provide
a supply to a resistance of 8-5 Q. What are the P_630
current drawn by element, / = — = 3A
current drawn from the supply and its terminal V~210
voltage ? V 210
(b) A secondary cell after long use has an emf Resistance of the element, R = — - = 7oa
/
of 1-9 V and a lai^e internal resistance of 380 Q.
What maximum current can be drawn from the Example^ An electric motor operates on
cell ? Could the cell drive the starting motor of a a 50 V supply and draws a current of 12 A. If the
car ? motor yields a mechanical power of 150 W, what
Solution, (a) Here, e = 2-0 V ; n = 6 ; is the efficiency of the motor ?
r=0-0I5a;P = 8-5a (NCERT Solved Example)
CURRENT ELECTRICITY 3/59

Solution. Here V = 50 V, / = 12 A wastage : power transmission of : (i) 20,000 V or


(m) 200 V ? (NCERT Solved Example)
Input power = V7 = 50 x 12 = 600 W
Solution. Let R be the resistance (in ohm) of
Output power = 150 W transmission cables.
Efficiency of motor, Power of electric station,

output power xlOO 150x100 P = 100 MW = 100 X 10^ W = 10^ W


ri = = 25%
input power 600 (/) Kj = 20,000 V,
P 10^
Example m A 1 kW electric heater is Current /.I = — = 5000A
designed to operate from 220 V lines, (i) What is V,I 20,000
its resistance ? (ii) What current does it draw ? Rate of heat dissipation at 20,000 V is
(iii) What is the rate of dissipation of energy in
calories per second ? (iv) If the line voltage drops = i^R = (5000)2/? ^ 25 X 10^ R watt

w
to 200 volt, what power does the heater take in
watts ? [Assume the resistance is constant Actually, (ii) ^2 = 200 V,
it will change because of the change in temperature.]

Flo
P 10^
Solution. Here, power of electric heater, Current - = 5x\0^A
200
’'a

e
P = IkW = 1000 W ; 220 V

reee
(/) Resistance of^heater, Rate of heat dissipation at 200 V is

FFr
y2 (220)2
R = = 48-4 a P^^iIr = (5x\0^)^xR
P 1000
= 25x 10'0/?watt
for
ur
(//) Current drawn by heater.
P 1000
= 4-545 A
AsP^< P2, therefore, there will be lesser power
wastage when the power is transmitted at
kss
V 220
(///) Rate of dissipation of energy, 20,000 V.
Yo
oo

P 1000
= 238-1 cal.s“^

I
H = —
TYPE XI. ELECTRIC ENERGY
B

J 4-2
AND HEATING EFFECT OF CURRENT
Vf _ (200)2
re

(/v) New power of heater = Formulae used. (/) Electric energy,


R 48.4
ou

= 826-45 W
ad

E = Vlt = —r = /2/?z
YY

Example Which of the two has greater R

resistance a 1 kW heater or a 100 W tungsten bulb, (ii) Electric energy spent in heating a liquid from O,
both marked for 230 V ? to 02"C is E= ms (02 - 0])
nd
Re

(NCERT Solved Example) (iii) Units of electricity consumed


Fi

V X / xt
Solution. Here, V = 230 V, *^volt ^ ‘amp ● *hour
Pj = I kW = 1000 W ; p2 = 100 W 1000
(iv) Cost of electricity consumption = units of
^ ^ (230)2 = 52-9Q electricity consumed x cost per unit of electricity
‘ Pj 1000 (v) 1 unit of electricity = 1 kWh = 3-6 x 10^ J
y2 (230)2 = 529n
Units used. Electric energy in joule, m in kg, s in
^2 = 100 Jkg-‘ K-*,(02-0i)in°C or K.
^2
As R2 > R[, so 100 W bulb has greater
resistance. Example W\ The potential difference
Example]^ An electric power station applied across a given resistor is altered so that
the heat produced per second increases by a factor
(100 MW) transmits power to a distant load of 9. By what factor does the applied potential
through long and thin cables. Which of the two difference change ? (CBSK 2017)
modes of transmission would result in lesser power
3/60
’4. Fundamental Physics (XII) W*Ti
1^2 combined in series and the combination connected
Solution. As heat produced, H =VIt =
R to a 200 V dc supply, which heater will produce
more heat ? (NCERT Solved Example)
V?t
I

R and H2 = R Solution. Resistances of the two heaters are


y2 200x200 400
El YL or
!k= &2- 9H I
= 3
R, =-
’ R1 300 3
n

Hi V} H
1
y2 200x200 200
or V2 = 3Vi ^2 = Q
^2 600 3
Hence potential difference changes by a factor

oww
of 3. For series combination, the effective resistance is

Example 01 A 10 V storage battery of = 200Q


negligible internal resistance is connected across a
‘ 2 3 3 3

50 Q resistor made of alloy manganin. How much n


Current,, /, V =200 = 1A
=—

e
heat energy is produced in the resistor in 1 h ? R 200

FFrlo
re
What is the source of this energy ? Power dissipated in 1st heater,
(NCERT Solved Example)

rF
ee
400 400
P,=I^R,1 =l2
X W
Solution. Here, V = 10 V, /? = 50 H,
I
~1 3

rF
/=lh = 60x60s; Power dissipated in second heater,
ouru
Heat energy produced in I h is 200 200
P2 = fiR2=l^X ^
fosor
W
3
V2f _ (10)2 X (60x60)
skf
H =
R 50
= 7200 J Pj = 2 P2 or Pj > P2
ooko
Example]^ A heating coil of 2000 watt
Yo
The source of this energy is the chemical
Y

energy stored in the cell. is immersed in an electric kettle. How much time
Bo

will it take in raising the temperature of 1 litre of


reeB

Example ^ A heater coil is rated 1000 W, water from 4“C to 100"C ? Only 80% part of the
200 V. It is cut into two identical parts. Both parts thermal energy produced is used in raising the
ooY
uur

are connected together in parallel, to the same temperature of water.


ad

source of 200 V. Calculate the energy liberated Solution. Here, P = 2000 W, / = ? (in seconds)
per second in the new combination. (CBSE 2000)
dY

Volume of water = 1 litre = 1000 c.c.


Solution. Here, P = 1000 W, V = 200 V Mass of water, m = volume x density
nind

Resistance of heater coil,


Re

= 1000 X 1 = 1000 gram


V2 200x200 Rise in temperature, = ©2 - 0j = 100 - 4
FFi

/? = = 40Q
P 1000 = 96“C

Resistance of either part = 20 Q We know sp. heat of water, ^ = 1 cal “CT*


Equivalent resistance when both parts are Heat taken by water = m5 (82-Oj)
= 1000 X 1 X 96 = 96000 cal.
20x20
connected in parallel, R' = = ion
Energy spent in heating coil = Pt = 2000 x t
20 + 20
Useful thermal energy produced = 80%
Energy liberated per second in the new
combinationwhen connected to a source of 200 V 80 _ 2000X/X80
= 2000X/X cal.
V2 200x200 100" 100x4-2
= 4000 Js-*
R' 10 As this thermal energy is taken by water,
2000xrx80
BxampI* M Two heaters are marked 200 V, therefore, = 96000
100x4-2
300 W and 200 V, 600 W. If the heaters are
CURRENT ELECTRICITY 3/61

20
96000x100x4-2 =20x10"^ X = 88 X 10"^ Q
r = = 252 seconds.
or
2000 X 80
1/(20x11)
And rate of heat produced,
Example-^ The resistance of each of the =flR^= (10)2 X 22 X 10“2 = 22 W
three wires joined as shown in the Fig. 3.59 is
4 U, and each one can have a maximum power of
and //2 = ^^^2 = O0)2x88x 10"2 = 88W
20 watt (otherwise it will melt). What maximum Thus the wire of longer length gets hotter.
power will the whole circuit dissipate ? Example fH A heater is designed to
4Q FIGURE 3.59 operate with a power of 1000 W in a 100 V line. It
vwwv is connected to two resistances of 10 Q and R Cl,
Fig. 3.60. If the heater is now operating with a

ooww
4C1
I
-AAAAAAt:* power of 62*5 W, calculate the value of R.
A C B
FIGURE 3.60
4H
AA/WW 10Q 1000 W

e
I-W^ HEATER
(CBSE 2003)

re
rFFl
Solution. Let I be the current through a

ree
A/W

F
resistance wire of maximum power 20 watt. R

fR = 20 or /2 X 4 20 or /2 = 5

rF
Effective resistance R between A and C will be
100V

fsfoor
ouur
4x4
R = 20.
P 4+ 4 (CBSE 2003)
kosk
Electric power dissipated as heat by the Solution. Resistance of heater,
resistance between A and C = /^ /?^ = 5 x 2
Yo
y2 ioo2
oo

= 10 watt.
Y

R' = = ioa
BB

Electric power dissipated as heat by the P 1000


resistance between C and B = R = 5 x4 = 20 watt. When heater is in the circuit, let pot. diff. across
rre

Total electric power dissipated as heat by the heater be V, then 62-5 = V^/io or = 625
whole circuit = 10 + 20 = 30 watt.
ouu

or V = 25 volt
Y
ad

Example yn Two uniform wires of same Therefore, current through heater,


dY

= V/R' = 25/10 = 2-5 A.


material each weighting Ig but one having double
the length of the other, are connected in series, Voltage drop across 10 O resistor connected in
innd

carrying a current of 10 A. The length of longer series with the heater = 100 - 25 = 75 volt.
Re

wire is 20 cm. Calculate the rate of consumption 75


FFi

of energy in each of the two wires. Which one wire Main current = = 7-5 A
gets hotter ? Density of the material of wire
= llg/cc, specific resistance of the material current through resistance R = 7-5 -2-5 = 5 A.
= 20 X 10"® n cm. 25
Resistance, ^ ~ = 5n
Solution. Let aj and ^2 cross-
section of shorter and longer wire respectively.
As, mass = volume x density = alp Exampia QQ The resistance of a 220 V and
200 W electric bulb when hot is 12 times the
1 = X 10 X 11 = fl2 ^ 20 X 11
1
resistance at room temperature (= 30®C). If the
1
or = cm2 _ cm
2
working temperature of the filament is 2000**C,
10x11 20x11
find the temperature coefficient of resistance of
10 the filament.
R,1 =20xl0“®x
1/(10x11) Solution. For hot bulb at temperature,
= 20 X 10"® X 10 X 10 X 11 = 22 X 10"2 O t = 2000“C,
3/62
^ttu^ce^’4^ Fundamental Physics (XII) VOL.I

_ (220)- (0 Draw the circuit diagram.


Resistance, R.I = = 242H
P “ 200 (ii) Using KirchholT’s rules, calculate,
Resistance of bulb at room temperature (a) current through each branch of the circuit
{h) potential difference across the 5 Q
»2on resistance. (CBSK 2005)
12 12
Solution. (/) The circuit diagram is shown in

w
Since /?^ =/? [1 + a x (1000 - 30)] the Fig. 3.61.
242 = 20 [1 +a + 1970]
FIGURE 3.61
14-1 = I + ax 1970
1-5V
I1 1 n
13-1 A 4 if"

e
a = = 6-6 X 10-3 “C-*
1970 ii<. 20 V
●2

e
5 4 Ii—vw E

orw
Example^ A copper electric kettle 2Q h

r
' (Ii + h) (●i *2)
weighing 1 kg contains 0*5 kg of water at 20"C. It

F
C 'vW
takes 10 minutes to raise the temperature to 100®C. 5Q
D

ullo
If the electric energy is supplied at 220 V, calculate

FF
the strength of the current, assuming that 20% (ii) (a) Let the currents through the various arms

e
heat is wasted. Specific heat of copper is 0*1. of net work be as shown in figure.

sre
r
Solution. Here, mass of kettle, m^. = 1000 g ; Using Kirchhoff’s second rule for the loop

oF
ABCDEFA, we have

k
mass of water, ~ 500 g ;
5(/,+/2)+1/, = 1-5
sp. heat of copper, = 01 cal g"' ;
oofr
uor
or
6/1 +5/2 = 1-5 ...(0
sp. heat of water, = 1 cal g~' T!-‘
sf
For the loop BCDEB, we have
ko
Y
Rise in temperature = 02 - 0j = 100 - 20
5 (7| + 72) + 2 /2 = 2
rBB
Yo
= 80®C
5 7j + 7 72 = 2
oo

or
...(«)
eY

Heat required = (m^. + m


(©2 - ®!)
Solving equations (/) and («), we get;
K’

= (1000 x 0-1 +500X l)x 80 = 48000 cal


re

Heat produced.
u

7]
34 2 34
Vlt _ 220x7x(10x60)
odu

77 =
o

cal. , r 1 9 10 ,
ad

J 4-2 and
1 2 34 34 34
Y
din

Useful heat produced = (100 - 20) = 80%


(b) Pot. diff. across 5 Q resistance
80 220x7x10x60
10
Re

X cal.
100 = (7,+72)x5 = —x5 = 1-47 V
FF

4-2
in

80 220x7x10x60
= 48000 Example^ Using Kirchhoff’s rules
■■ 100 42
deter- mine the value of unknown resistance R in
On solving, 7 = 1*9 A the circuit shown in Fig. 3.62(a) so that no current
flows through 4 Q resistance. Also find the
I TYPE XII. KIRCHHOFF’S LAWS

Formulae used. (1) U = 0, and (i7) Ze = 177?.


I potentialdifferencebetweenA and/). (CnSK20I2)

FIGURE 3.62
where i is current, R is resistance and e is the emf. F E D F
1 n
—VA
Units used, i in ampere ; 7? in ohm, and e in volt. »

I
I

4£i IQ 4Q
>
Example 0j] Two cells of emfs 1-5 V and
“I 6V
2*0 V internal resistances 1 £2 and 2 Q are connec- I 6V 3.
A
ted in parallel so as to send current in the same 9V
B
3V
C A
9 V
B
3 V
C

direction through an external resistance of 5 o o


CURRENT ELECTRICITY 3/63

Solution. The current 1 flows in the circuit as On solving (/), (ii) and (JU), we get
shown in Fig. 3.66{/>). 1
/, = — A, /- == — A and I = — A
Applying Kirchhoff’s second rule to loop ' 13 2 13 13
AFEBA, we have Pot. diff. between points B and D
1 x/ + 1 x/ = -6 + 9 = 3or/ = 1-5A
1
From the loop BEDCB, we have = /.I x2 =—x2
13 13

//? = 6-3 = 3 or /? = - = lil {//) Pot. diff. across the terminals of G (giving
1-5 current)

oww
V^^ = (l + l)x/ = 2x 1-5 = 3V
Vq = e - /r = 3 - —
13
x3 = 1-615 V

Exampiej^ In the circuit shown in Fig. Pot. diff. across the terminals of H (receiving
3.63, £, F, G and H are cells of emf 2 V, 1 V, current for charging)

e
3 V and 1 V and their internal resistance.s are 20,
6
IQ, 3Q and IQ respectively. Calculate (0 the

re
Vu=^E'
H + Ir' =1 + —xl =1-46 V
13
potential difference between B and D and (ii) the

FFllro
potential difference across the terminals of each Exam pie^ Use Kirchhoff’s rules to

reF
of the cells G and H. ICBSE 2\m (C)|
determine the potential difference between the

e
points A and D when no current flows in the arm
uoru BE of the electric network shown in Fig. 3.65.

sFr
(CBSE 2015)

foro
fk FIGURE 3.65
okso
E D
AAA^
YYo
oo
BB

1 v-± Ri
Solution. (/) The complete network has been
shown in Fig. 3.64 with currents in the vtuious arms. R
rYree

2H ±_3V
oouu

FIGURE 3.64
ad

* C
Yd

A B
6V 4 V
nidn

1
Re

1 Solution. Since no current flows in the arm BE,


so the current I flows along the outer loop as shown
FFi

3Q ^ in Fig. 3.66.

Applying Kirchhoff’s first rule at junction B, FIGURE 3.66

/=/l+/2 F 3 I F I D
Applying Kirchhoff’s second rule to loop AA/v » -r
ABDA, wc get ’T
1 v-±
-2/2-f2/|-1/2 = -2+1 Rl
or
2 - 3 /2 = - 1 ...(«●) R

For loop BCDB, we get 2il ±_3V


- 1 x /-3x/-2/i = 1 -3 = ~2
or 2 = 4 / + 2 /i = 4 (/, + /2) + 2 /i C
A I I
or 1 = 2 (/, + /2) + /| = 3 /| + 2 I2 6 V 4 V
or
3 /, + 2 /2 = 1 ...(III)
3/64
'P%tuUe^ ^ Fundamental Physics (XII)
Using Kirchhoff’s loop rule to the closed circuit
3r/? + /-2
AFEBA, we have / =e
or
/? + 3r
2/-l+3/ + /?ix0-3-6 = 0
or 5/= 10 or / = 2A Equivalent resistance between A and B
Potential difference across A to D while
following the path AFD, we have
e 2>rR + -p r(3I? + r)
R
eq I /? + 3r (/e + 3r)
V^O = 2x2-1+3x2 = 9V
Example^ Calculate the equivalent A battery of 10 V and negli
resistance between the points A and B of the gible internal resistance is connected across the
network shown in Fig. 3.67(a). diagonally opposite corners of a cubical network
Solution. Let a cell of emf e be connected consisting of 12 resistors each of resistance 1II.
Determine equivalent resistance of the network

oww
between points A and B. Let the currents in the various
and the current through the battery.
branches be as shown in Fig. 3.67(6).
(NCERT Solved Example)
FIGURE 3.67
Solution. L&iABCDKLMNbt the skeleton cube
r R
formed by joining twelve equal wires each of

e
AAV -Wv
resistance r. Fig. 3.68. Let the current enter the cube

ree
rFl
A B
at comer A and after passing through all twelve wires,

Fre
r

let the current leave at N, a comer diagonally opposite

rr F
R r to comer A.
VW AAV
e
ouur
Cl—^—WV
D . (^1.” ^2)
■ 1—V^^—►—|E
l2 ’
sfoo 21
FIGURE 3.68

8 c
kks
A B "I
A. I
Yo
oooo

r
¥
21 D
R r
”21
!'■ hL-►—NAA ir-*—VA—'f
eBB

G (I-I1+I2) 21
e

K -61 21
M N
urr

o I 21
You
ad

Applying Kirchhoff’s Second Rule to loop I K


61 "
dY

KACDEBJK

/] r + (/j -/2) /? - e e
innd
Re

or /j (r + i?) - /2 /? = £ ...(0
Fi

For loop CDGHC, we get


7l r + /2r-(/-/i)7? = 0 Note that the current is divided equally amongst
11 (r + 7?) + /2 r = IR ..(«) paths of equal resistances provided their end points
For loop DEFGDy we get are situated symmetrically w.r.t. the exit point.
Howeve,r along paths of different resistances, current
(/i-/2)7?-(/-/i+/2)r-/2r = 0
is divided unequally in the inverse ratio of
/j (7? + r) - /2 (/? + 2 r) = Ir ...(in) resistances.
Solving equations (/7) and iiii), we get For the sake of convenience, let us suppose that
R+ r R- the total current is 6 I. At A, this current is divided
A = 7 and I2 ~ ’'-I
R + ,2r 7? + 3r into three equal parts each (2 7) along AB, AD and AL
Putting values in (7), we get as the resistances along these paths are equal. At the
points B, D and L, each part is further divided into
R+r 1 R-r two equal parts each equal to 7 because resistances
I(r + R)- IR = e
R + 3r R + 3r along the two paths are equal. Thus, the distribution
of current in the various arms of skeleton cube is
CURRENT ELECTRICITY 3/65

shown according to Kirchhoff’s First rule. The current divided into three parts: /along AD and/j each along
leaving the cube at /Vis again 6 /. If £ is the emf of the AB and AL as the resistances along AB and AL are the
battery used, then applying Kirchhoff’s Second rule same but along AD, it is different. At the points B and
to the closed circuit ALKNzA, we get L, current is further divided into two unequal parts.
2 /r + /r + 2 /r = e or 5 /r = e .(0 The distribution of current in the various arms of
where e is the e.m.f. of the cell of negligible skeleton cube is shown according to Kirchhoff’s First
internal resistance. rule. The current leaving the cube at D is again (/+ 2
If R is the resistance of the cube between the /j). Applying Kirchhoff’s Second rule to the closed
diagonally opposite comers A and N, then according mesh ABCDA, we get
to Ohm’s Law, we have /j r + /2 /● + /| r - /r = 0
6 lxR = e ,(w) or 2/i +/2 = / .(0
For closed mesh BC/VM5 ;
From (0 and (//), 6 IR = Sir or R=
I2 r - (/i - /2) r - 2 (/i -l2)r- (/, - /2) r = 0

ww
Here, r = 1 O, so, /? =
or /2 r = 4 (II -I^r
or
/2 = 4/,-4/2 .(«●)

FF loo
voltage of battery
Current through battery = or 5/2 = 4/i or
^2=5/.
total resistance

ee
10 4 14

eFr
5/6
= 12A From (0, / = 2/j+-/j = —/,1 ...(*«)
lExamWaRI^ Twelve equal wires each of

Frre
For closed mesh ADeA, /r = e .(/V)
r ur
resistance r 12 form a skeleton cube. Find the where e is the emf of the cell of negligible
resistance between the corners of the same edge
fforo
internal resistance.
ks
of cube. If R is the total resistance between the two
YYouo
Solution. Lst ABCDKLMN\)q the skeleton cube comers of the same edge of the cube, then applying
koso

formed by joining twelve equal wires each of Kirchhoff’s second mle for the closed mesh ADeA,
resistance r. Fig. 3.69.
BBoo

we get
fl4
r ee

FIGURE 3.69 {I+2Ii)R-e or \ 5


—1^ + 21^ R = e
c 24
or — /,/? = £ .(V)
ad

■J4 (I1-I2)
oouur

itZi: ZL 5
Y

14
I
° .Xirh) From (ill) and (iv), —I,r-e
1 .(VI*)
dnYd
Re

II 1 r From (v) and (vi).


FFini

M
2(1 rl2) N
24 , „ 14 , „ 14 7
^rl2)
— I.R = — /, r or R = —r=—r
-(I + 2I1) 24 12

I2 K (I + 2I1) V lExaimpil^lW Eleven equal wires each of


resistance 2 12 form the edges of an incomplete
cube. Find the total resistance from one end of
I
E
vacant edge to the other end of the same edge.
Solution. Let ABCDFGHK be the open cube
Let the current enter the cube at comer A and formed by joining eleven equal wires each of
after passing through all the twelve wires, let the resistance 212. Fig. 3.70.
current leave at D, other comer of the same edge of Let the current from the cell of emfz enter the
the cube. cube at comer A and after passing through all the
For the sake of convenience, let us suppose that wires leave at K, the other end of the open edge of
the cube.
the total current is (/ + 2 /j). At A, this current is
3/66 Fundamental Physics (XII)BSm

For the sake of convenience, let us suppose that Example Determine current in each
the total current is 2 /. At A, this current is divided
branch of the network shown in the Fig. 3.71.
into two parts : I along AB and / along AD. This is
(NCERT Solved Example)
because the resistances of the two paths are equal.
At the points B and D, each part of current is further
divided into two unequal parts. The distribution of
current in the various arms of skeleton cube is shown
according to Kirchhoff’s first rule. The current
leaving the cube at K is again 2 I. Let R be the

w
equivalent resistance of skeleton cube between edges
A and K.

FIGURE3.70

2(l-li) G

e
C

roow
0-ii)

re
s
/

D
Solution. In the closed circuit ABCA,
U
{/, - 73) 4 + (/, + /2 - /j) 2 + /, X I = 10

reF
..(Ml)

uFFll
I I or
7/1 +2/2-673 = 10 ...(0

e
B H In the closed circuit ADCA,

sFr
I I 73 X 4 + (73 - 72) 2 + 7j X 1 = 10

foro
K
or
7, -2/2 + 673= 10 ...(«)
A
ofk
uor
21 21 ■’ In the closed current ABEDA
8
(7i-/3)4-73x4=-5
kos
4 7,-8 73 = -5 ...(III)
Y
or
Yo
reeBB
oo

On solving (7), (//) and (m), we get


From Ohm’s law. e = 2/7? ...(I)
7i = 2-5 A,
Applying Kirchhoff’s second rule to mesh
uurY

eABHKe, we have I2 = 1*875 A = ly


e = 2x/ + 2x7, + 2x7
Example Qy The Wheat stone bridge
ad

or e=47+27 I ...{«) circuit have the resistances in various arms as


doo
nY

Applying Kirchhoff’s Second rule to mesh of shown in Fig 3.72. Calculate the current through
DFGCD, we have the galvanometer. (NCERT Solved Example)
nid

27i - 2(7-/|)-2x2(7-/i)-2(/-7i) = 0
Re

2 7, -27+2 7i-4 7 +4 7.1 - 27 + 2/, =0


FFi

or 107,-8 7=0 or 87= 10/ I

8 4
or 7, = —7=-7 ...(III)
' 10 5
From (it) and (Hi),
f4 28 7
e=47+2 -/
5 ; 5
...OV)

From (0 and (iv),


28/
277? =
5

28
or 7? = = 2-8 ft
5x2
CURRENT ELECTRICITY 3/67

Solution. In the closed loop ABDA ;


i00/| + 15/g-60/2 = 0
or
20/j+37^-12/2 = 0 .(i)

In the closed loop BCDB ;


10(/i-/p-5(/2 + /p-15/^ = 0
or
10 /[ - 30 /^ - 5 /2 = 0
or
2/|-6/^-/2 = 0 ...(«)
In the closed loop ADCEA ;
60/2 + 5 (/2 + Ig) = 10
or 65 /2 + 5 /^ = 10 or 13 /2 + /g = 2 ...(m) Thus the bridge is a balanced Wheatstone

w
On solving (i), (») and (m), we gel bridge. The resistance 5 H in arm BD is ineffective.
Therefore, the equivalent circuit reduces to the circuit
i-A =4*87mA as shown in Fig. 3.75.

Flo
821
FIGURE 3.75

1 a B 2Q

ee
TYPE XIII. WHEATSTONE BRIDGE ■Wv—► ●Wv

Fr
PRINCIPLE AND SLIDE WIRE BRIDGE A C

20 D
wv i—vw
P R

for
100-/
ur
h
Formulae used, (i) 77
Q = 'T
5 00 S =
R
4 V

where, .P Q, R, S = resistance in four arms of The resistance of arm ABC = 1 + 2 = 3 Q


ks
Resistance of arm ADC = 2 + 4 = 60.
Yo
Wheatstone bridge.
oo

/ = length of meter bridge wire where galvanometer The resistances of these two arms are in parallel.
The equivalent resistance between A and C is
eB

shows no deflection.

Units used. ,P Q, R and S are in ohm, I is in cm.


3x6
r

R = 20
ou

eg 3 + 6
ad

Example EE Calculate the current drawn


, V 4
Y

from the battery by the network of resistors shown Current / = -— - =2A


R 2
in Fig. 3.73. [CBSE 2015 (C), 2009] eg
nd
Re

Example EB Calculate the resistance


FIGURE 3.73 20
Fi

AAAr between the points A and B of the network shown


50 40 in Fig. 3.76.
A JR ■Wv
D

B C

V/v
20
1
4 V

Solution. The equivalent circuit for the given


network is shown in Fig. 3.74, which is a Wheatstone
bridge. Here P=lO,Q = 2 0, R = 20 and 5 = 40
10 20
and i.e..
20~ 40 Q~ S
3/68 ^ Fundamental Physics (XII)
The equivalent resistance across AB is
Solution. Here, - = so the resistance of
1 1
5 5 1 ]__ 4 2 „ R

20 0 is ineffective. The equivalent circuit is as shown R.


eq
2R 2R R~2R R
in Fig. 3.77. If a battery of emf e is connected to AB, then
Now we have, resistances of (5 + 5) Q, e e 2e
(5 + 5) Q and 20 Q in parallel between points A and current through arm AB - R RI2 R
B. Therefore the effective resistance (R) between A eq

and B is Current through arm D£ = 0 (as Wheatstone


bridge is in balanced state)
Exam
In meter bridge, the null

oww
R 10 10 20 20 4
point is found at a distance of 60*0 cm from end
FIGURE 3.77 A, If now a resistance of 5 Q is connected in series
sn C 5^^ with 5, the null point occurs at 50 cm. Determine
● VAV the values of R and 5, Fig. 3.80. (CBSE 2010)

e
sn FIGURE 3.80

FFrlo
A —V/vV
■g—vwv B

re
R S
I-WV-I rAAAr-1
B

ree
-W/v

F
20 n 3]
1)

rF
II
Exampla; Six equal resistors, each of A C

value R are joined together as shown in Fig. 3.78.

fsoor
ouur
Calculate the equivalent resistance across AB. If h <>
skf
a supply of emf 8 is connected across 4B, compute
the current through the arms DE and AB. R 60
ooko
Solution. In first case, — = ...(0
YYo
(CBSE Sample Paper 2003} S 100-60 2
Bo

R 50
FIGURE 3.78
In second case. _ = lor/? = 5 + 5
reB

A RQ 5 + 5 50
5 +5
R D R From (0, = - or25 +10 = 35
uur
ooY

c VA- VA— E 5 2
ad

or 5 = ion and /?= 10 + 5 = 15n


R R R
dY

Exampla^ When two known resistances


B F
R and 5 are connected in the left and right gaps of
innd
Re

a meter bridge, the balance point Is found at a


Solution. The equivalent circuit is shown in Fig. distance from the zero end of the meter bridge
FFi

3.78(a), where resistance R of arm DE is ineffective wire. An unknown resistance X is now connected
because the Wheatstone bridge is balanced. Thus the in parallel to the resistance 5 and the balance point
equivalent circuit reduces to as shown in Fig. 3.79(/>). is found at a distance I2 from the zero end of the
meter bridge wire. Fig. 3.81. Obtain a formula for
X in terms of /j, I2 and 5. (CBSE 2009, 2017)
FIGURE 3.81 X
R
rV»n
\f^'
» O !

S [4
D

A h (IQO-/1) C
B
+
h I- o
K
CURRENT ELECTRICITY 3/69

Solution. When resistance R and 5 are connec When 12 Q is connected in parallel with 5, the
ted to the left and right gaps of meter bridge and equivalent resistance is
bridge is balanced at length /] from zero end, then 125
5
R eq 12 + 5
...(0
5 (100-/,) For the second balanced bridge situation,
When unknown resistance X is connected in R 51-9 51-9

parallel to 5, then effective resistance in right gap is 5 100-51-9 48-1


eq
SX
5' = ...(«)
/?(12 + 5)^51-9 .(«)

oww
S + X or

Now, balance point is obtained at length I2, 125 48-1

R h R
Putting the value of — from (/) in (t/),
5' (100-/2> 5

e
Putting the value of S', we have (12 + 5) „ 33-7 _ 51-9

re
we have
R(S + X) 12 66-3 "48-1

FFrllo
m
SX
(100-/j) On solving we get, 5 = 13’5 Q

rF
ee
Dividing iiii) by (/), we get 33'7 33-7
From(0, /? = —5 = —xl3-5 = 6-86 H
ouru
S + X h (100-/1) 66-3 66-3

rF
X
X
(IOO-/2) /
1 .Example P' £ The Fig. 3.83 shows the
experimental set up of a meter bridge. The null
5
— + 1 = —, /2(100-/,)—
fosor point is found to be 60 cm away from the end A
or
os kf
X
/i (100-/2) with X and Y in positions as shown. When a
resistance of 15 is connected in series with T,
ook
_ S 100/2-/i/2-100/j+/j/2 100(/2-/i)
Yo
Y
the null point is found to shift by 10 cm towards
X
/j (IOO-/2) /j (100-/2) the end A of the wire. Find the position of the null
Bo
reeB

point if a resistance of 30 were connected in


or X = parallel with Y. (CBSE Sample Paper 2IM18)
oouY

100(/2-/j)
ur

FIGURE 3.83
Example ES jjj ^ meter bridge Fig. 3.82, X Y
ad

rW/vn i-WVv-i
the null point is found at a distance of 33*7 cm
Yd

B
from A. If now a resistance of 12 H is connected in
]
nidn

parallel with 5, the null point occurs at 51*9 cm.


Re

Determine the values of R and S.


FFi

(NCERT Solved Example) D


A C
FIGURE 3.82
R S
lAVvn rWvn I- <●>
B
[
% c
Solution. In ifrst case, L- — -1
7 " 40 ” 2
..(0

0 100 X 50
In second case, ...(«)
0 F + 15 50
c K
Dividing (/) by (11), we get
Solution. For the first balanced bridge situation,
X..(y + 15) 3 ,1 15 3
R 33-7 33-7 or + — = -
■d) Y X 2x1 Y 2
5 100-33-7 66-3
or Y=30Q
3/70 ’«* Fundamental Physics (XII)
Example EJil A resistance of (2 draws
From(i), X = - K =-x30 = 45^2
2 2 current from a potentiometer. The potentiometer
When a resistance of 30 H is connected in has a total resistance /?qQ [Fig. 3.84]. A voltage V
parallel with Y, the total resistance in the irght gap of is supplied to the potentiometer. Derive an
bridge will be expression for the voltage fed into the circuit when
the slide contact is in the middle of potentiometer.
r = 3Qxy _ 30x30 = 15n (CBSK 2014. 2017) (NCERT Solved Example)
30 + y "30 + 30
FIGURE 3.84
If the null point is obtained at length I from end V
A of the meter bridge, then
X I 45 /
Ro

ww
or
A '\>VVVrV'\'^ C
15 (100-0 15 (100-0 B

or 300 - 3 / = / or 4 / = 300 or / = 75 cm

Flo
R

I I
A<WW

e
TYPE XIV. POTENTIOMETER

ere
Solution. When slide is in the middle of the
8, /, h_ potentiometer wire, only half of the resistance of

FFr
Formulae used. (0 —i- = -!- («) r = R
h h potentiometer wire (= /?q/2) will be between the points
uurr
A and B. Hence effective resistance (R]) between A

orr
where 8j, 82 = emfs of two cells, and B is
/j, 0 = lengths of the potentiometer wire against
which emfs Ej and 82 are balanced.
sfo j__J_ 1 R^R
R,~ r'^ {RJ2)
or =
R^+2R
kks
Units used. 8j, 82 in volt, R in ohm, I2 in metre.
Yoo
1 0
oooo

R
0
Example PR In a potentiometer, a Total resistance between A and C = R, +
' 2
eBB

standard source of emf 5 V and negligible internal Current through the potentiometer wire will be
resistance maintains a steady current through the 2V
urr

potentiometer wire of length 10 m. Two primary I =


R,1 + RJ2 2R,+R0
cells of emf and £*2 joined together in series
ad

0
Yo

with (0 same polarity and («) opposite polarity. The voltage V| taken from the potentiometer
dY

The combination is connected to the potentiometer will be the product of current / and resistance R 1’
circuit in each case. The balancing length of the ( 2V \ xR
innd
Re

wire in the two cases are found to be 700 cm and i-e., .V1 = IR,1 =
I
100 cm, respectively. Find the values of emf of the 2/?,+/?o
Fi

two cells. (CBSK 2020) 2V R,.R


0
Solution. Here, £ = 5 V, / = 10 m R^+2R
0
+ .R0
Potential gradient of potentiometer wire
R^ + 2R
— = 0-5V/m 2VR 2VR
I 10
2R + R^+2R R.0 + 4R
Case (i). £j + £3 = ki^ = 0-5 x 7-00 = 3-5
...(1)
Example PEI When a resistor of 5 £2 is
Case (ii). E^- E2- W2 = 0-5 x 100 = 0-5 connected across a cell, its terminal potential
...(2) difference is balanced by 140 cm of potentiometer
wire and when a resistance of 8 £2 is connected
Solving (1) and (2), we get
across the cell, the terminal potential difference is
£1 = 2 V, £2 = 1*5 V balanced by 160 cm of the potentiometer wire.
Find the internal resistance of the cell.
CURRENT ELECTRICITY 3/71

Solution. Here, /?] = 5 n ; /, = 140 cm ; Example [m In the potentiometer circuit


R2 = ^0.\It= 160 cm given below, calculate the balancing length L Give
Let emf of the cell be balanced by length / of reason whether the circuit will work, if the driver
the potentiometer wire. Then as per question, cell of emf 5 V is replaced with a cell of 2 V, keeping
all other factors constant. iCliSE 2019)
In the first case.
FIGURE 3.85
/-/ rl
1 1
r=R or = 1-1 ...O') 5V
I 1
/ R 450 0
1 I
l-WvV

oww
In the second case,
10 m
l-ll '■^2
r = /?2 or
=1-1, ...(«)
1. AEl- 4]B

e
Subtracting (//) from (/), we get

re
FFrllo
/
r
I >2 300 mV Rab “ 50 n

reF
R,

e
Solution. Here, e = SY, R = 450 Q, r = 50 Q,
uoru
^2 160-140
= 2*5 0
L= lOm.

osFr
'■=7I
or
140 160
Current in the potentiometer circuit.
5 8

fkfor
R
1 Rl £ 5 1
I = A

Example m A potentiometer wire of R + r 450 + 50 100


okso
length 100 cm has a resistance of 10 O. It is Fall of potential across wire AB = Ir
Y
Yo
connected in series with a resistance and a battery Potential gradient across wire AB,
oo
BB

of emf 2 V and of negligible internal resistance. A Ir


source of emf 10 mV is balanced against a length k = — —X— =0-05 V/m
L 100 10
of 40 cm of the potentiometer wire. What is the
Y
r ree

value of external resistance ? Let the balancing length be / for the cell of emf
oouu

Solution. Resistance of 40 cm length of 300 X 10“^ V. Then


ad

10 300 X 10-^ = cf/ = 0-05x/


Yd

potentiometer wire = x40 = 4Q


100 300x10-^
nidn

or I = = 6 m
Current through potentiometer wire, 0-05
Re

^ _ poLdiff. _ lOmV _ l^xl0"3 V If the driver cell of emf 5 V is replaced with a


FFi

resistance 4Q 4Q cell of emf 2 V, then fall of potential across the


potentiometer wire is
= 2-5x 10"^ A
e' 2
The same current flows through the potentiometer xr = x50 = 200x10'3 V
wire and the external circuit having resistance R. (R + r) 450+50

Total resistance of potentiometer circuit As this value is less than 300 x 10"^ V, so there
= (/? + 10) Q will be no balancing length on potentiometer wire.

I
2V
2-5x10-3 A = TYPE XV. TYPICAL EXAMPLES
(R + \0)Q

R + \0 =
2 Example 01 It is desired to make a
or = 800
2-5x10-3 20*0 Q coil of wire whose temperature coefficient
of resistance is zero. To do this, a carbon resistor
or /? = 800- 10 = 790Q
of resistance is placed in scries with an iron
3/72 “P>uzdeefi.'4> Fundamental Physics (XII) VOL.I

resistor of resistance proportion of iron Total internal resistance of the two cells will be
and carbon are so chosen that jR^ + I{2 = 20 Q for rxr
= - = 0-5 r
all temperatures near 20°C. Find the values of Ri r+r 2
and i?2* Temperature coefficient of resistance for Total resistance of the circuit = G + /? + 0*5 r
carbon, ac = - 0*5 x 10r^/**C and that of iron is Here, current,/= 0*6 A
aFe = 5xlO^/°C. effective emf
Solution. Let A fC be the rise in temperature. As, current =
total resistance
As per question we need,
1-5
Rj (1 + A/) + /?2 (1 + ttpe = 20 0-6 =
G + R + 0’5 r
Since, /?j + = 20, therefore,
/?! Ot^ A r + /?2 ^~ ® or 0-6 (G + R + 0-5 r) = 1-5
or G + /? + 0.5r=1.5/0.6 = 2-5
or /?! a^; = -/?2 0tpg
or G + R = 2*5 - 0*5 r .(«)
or Rj x(-0'5x I(r3) = _/j^x5x KT^

ww
From (i) and (»)> 3-2r = 2-5-0-5r
or
/?i = 10i?2 r = 0.5/1.5 = 0-333 0
or 0-5 = 1-5 r or
So, 10/?2 ^2 “ 20 or 11/?2“20
fm Determine the potential

Flo
or R2 = 20/11 = 1*82 n
difference between the points C and D in Fig. 3.87.
R, = 10x1.82 = 18-2 0

ee
and

rere
!xami e
A galvanometer together FIGURE 3.87

r FF
with an unknown resistance in series is connected = 1nF C2 = 2nF
C
across two identical batteries each of 1-5 V. When
uurr
the batteries are connected in series, the
galvanometer records a current of 1A and when
the batteries are in parallel, the current is 0-6 A.
foor B

Rl = 3£J
A
ks s
R2 = 6fl
What is the internal resistance of the battery ? AAAAA#—
Yoo

oook

D
Solution. Here, emf of each cell, e = L5 V
Let r be the internal resistance of each cell and
eBB

G be the resistance of galvanometer. e=i2V


in
The circuits are shown in Figs. 3.86(a) and (jb).
AAAAAr—►
rr

In circuit (a), effective emf = 1-5 + 1*5 = 3 0 V


ouu
ad

total resistance =G + R + r + r = G + R + 2r
Solution. Since the capacitors are in series,
YY

Current, / = 1 A
charge on each capacitor is the same. At steady state,
total emf the charge on the capacitor is due to terminal potential
nndd

Current, I =
Re

total resistance difference of cell e.

30 When steady state is reached, no current flows


Fii

or 1 = through the capacitors. The current in the circuit is,


G + R + 2r
12
or G + R + 2r=3or G + R = 3-2r ...(0 / = = 1.2 A
1+3+ 6
FIGURE 3.86
1.5V Terminal pot. diff. across A and B,
1.6V 1.6V
V=e-/r=12-1.2xl = 10.8V
1.8 V Capacitors are in series, their effective
-VWA
r r i 1x2
-vswv-
r
capacitance c = = -pF
1+2 3^
m> o A/V^V\r— m> o Charge on each capacitor,
In circuit (b), effective emf = 1*5 V
When cells are in parallel, only the size of 9 = |x10-®x10*8 =7.2x10“®C
the electrodes increases, the total emf does not change. Charge on capacitor C2, q = l’2x 10^ C.
CURRENT ELECTRICITY 3/73

Pot. diff. between A and C Example UUn The resistance of a 200 V


7-2xl0"^ and 100 W electric bulb when hot is 10 times the
= 3-6V resistance when cold. Find its resistance at room
C2 2x10-^ temperature. If the working temperature of the
Pot. diff. between A and D=Vj^-V^= IR2 filament is 2000°C, find the temperature
= 1-2 X 6 = 7-2 V.
coeflicient of resistance of filament
Pot. diff. between C and D = - V^j Solution. Resistance of hot bulb,
= (’^4 -'"d) - - ^c) = 7-2 - 3-6 = 3-6 V 200x200
/?.! = = 40on

oww
Example {ES A network of resistances is p 100

connected to a 16 V battery with internal resis Resistance of bulb at room temperature,


tance of 1 n as shown in Fig. 3.88. (a) Compute the 400
= 4on
equivalent resistance of the network, {b) Obtain 10

e
the current in each resistor and (c) Obtain the
Now, R, = Rq (i + OJ)

re
voltage drop ^AB> ^BC and ^CD'
400 = 40(1 + ax 2000)

FFrllo
(NCERT Solved Example)
or 10 = 1 + a X 2000

reF
FIGURE 3.88 On solving, we get

e
4Q ^2Ci
I—WWv—I
Ilt—VWW ^3
ouru in a = ~ = 4-5 X 10-3

osrF
B D
2000
I
I2'—wvw u”
ecl
ffor
k ^'1
Example [EB The following graph shows
the variation of terminal potential di^erence V,
kso
across a combination of three cells in series to a
16 V
ooo
in resistor versus the current i:
YYo
1 ●wvw
BB

e
FIGURE 3.89

Solution, (a) In the circuit shown, 4 Q and 4 Cl


r ree

are in parallel; 12 H and 6 Q are in parallel. These


Y

two combination of resistances are in series with 1 Cl


oouu

resistance in the circuit. Total resistance of circuit


ad

between A and D is
Yd

4x4 12x6
R = + 1-F = 2+l+4 = 7Cl
nidn

4+ 4 12 + 6
«(A)
Re

16
(h) Total current, I = —~ = 2A
(1) Calculate the emf of each cell.
FFi

{R + r) 7+1
(ii) For what current i, will the power dissi
Current at A is divided equally in each of 4 H
pation of the circuit be maximum ? (CBSE 2008)
resistances in parallel. So,
Solution. (i)Total emf of 3 cells in series = 6-0 V
2
7,=/2=-=IA 60
EMF of each cell = = 2-0 V
Pot. diff. across C and D, 3
VcD = / X ^cz) = 2 X 4 = 8 V (/7)When i = 1-0 A, 3-0/3 = 1-0 V
V. V,
CD _ 8 2
^3 =
CD _ i = lA Internal resistance of a cell
12 12 = 3A;/,= 6 6 3
e-V 2-0-1-0
(c) V^g = /x/?^g = 2x2 = 4V; r = = 10 Cl
i 1-0
VBC =/xl=2x1=2V;
The output power is maximum when external
^CD = IxR CD =2x4=8V resistance = internal resistance = 3 r
3/74 'Pxadeeft Fundamental Physics (XII) PgTW

total emf 3e e
Consider an element of wire of thickness dx at
/
max
total resistance 3r + 3r 2r a distance x from end A of wire. Its radius be a^. Then
{la-d)
2-0 a
X
a + X =a(l + .x)
= 1-0 A I
2x10
Resistance of this element is
Example Kllil The wire AB of a meter
bridge changes linearly from radius a to 2 a from dR =
pdx _ pdx
left end to irght end ; Fig. 3.90(a). Where should the ml Tca^ (1
free end of the galvanometer be connected on AB so
●^i
that the deflection in the galvanometer be zero? pdx
Solution. Let the free end of galvanometer be K =
' Tca^ (l + x)^ \ + x

ooww
na-
I
connected on wire AB at point D, then galvanometer
shows no deflection. Let length of wire from A to D I
pdx P 1
be X, where x = 0 at end A of wire. Let /?j be the and = f —- _ . , 1 + 1
resistance of left part {i.e., AD) of wire of length otj
and /?2 be the resistance of right part {i.e., DB) of

e
For no deflection in galvanometer
wire of length (1 -JCj).

ree
rFl FIGURE 3.90
R
l =l

Fre
or
/?! -/?2
2n 20,
Rl 2

rrF
r*AV.n
c
3 C E
1 P 1 1
ouur
A
0 sffoo TCa
1 + JCj
1
TCa^ l + x^ 1 + 1
okks
On solving, jCj - 33*33 cm
YYo
O
ooo
BB
rr e

1. Current Carriers. The charged particles whose flow in a definite direction constitutes the electric current
are called current carriers, e.g. electrons in conductors, ions in electrolyte, electrons and holes in
ouu
ad
Y

semiconductor.
2. Electromotive force of a cell is defined as the maximum potential difference between the two electrodes of
dY

a cell when the cell is in the open circuit, emf of a cell depends upon nature of electrodes, nature and
concentration of electrolyte used in the cell and its temperature.
innd
Re

3. Electric Current The flow of charge in a definite direction constitutes the electric current,
charge flowing _ q
FFi

Electric current =
time taken t

SI unit of current is ampere (denoted by A). The arrow head marked in circuit represents the direction of convendonal
current, i.e., direction of flow of positive charge, whereas the direction of flow of electrons gives the direction of
electronic current which is opposite to that of conventional current. Current is a scalar quarttUy.
4. Drift Velocity : It is defined as the average velocity with which free electrons get drifted towards the
positive end of the conductor under the influence of an external electric field. Drift velocity of electrons is
eE
given by : V
(I ~
X
m

where e is the charge on electron, m is the mass, E is the electric field applied and x is the average time of
relaxation, i.e., the average time that has elapsed since each electron suffered its last collision with the ion
or atom of the conductor, while drifting towards the positive end of the conductor under the effect of
external electric field applied.
The value of drift velocity of electron is about 10“^ m/s and value of average relaxation time is about 10”**^
second.
CURRENT ELECTRICITY 3/75

5.
Mobility. Mobility of electron (|ig) is defined as the drift velocity (v^) per unit electric field applied, Le.,
\i^ = VdlE
6.
Relation between current and drift velocity/mobility.
I = nAe Vd = Ane \LgE
where n is the electron density or no. of electrons per unit volume of the conductor and A is the area of
cross-section of the conductor. The small value of the drift velocity (~ 10“^ ms~^) produces a large amount
of current due to presence of large number of firee electrons in a conductor (~ 10^ m”^). The propagation of
current in a conductor under the effect of electric field applied is at the speed of light.
7.
Ohm*s law: It states that the current (/) flowing through a conductor is directly proportional to the potential
difference (V) across the ends of the conductor, provided physical conditions of the conductor such as
temperature, mechanical strain etc. are kept constant, i.e., V « / or v=m
where R is known as resistance of the conductor, which depends upon the nature and dimensions of the
conductor. The SI unit of R is ohm.
8.
Resistance of a conductor. It is the obstruction posed by the conductor to the flow of current through it.

w
Resistance of a conductor is due to the collisions of free electrons with the ions or atoms of the conductor
while drifting towards the positive end of the conductor. The resistance of a conductor can be given by the

Flo
R =
m
l_
expression : ne^x A ^ A

eeee
where m is the mass of electron, e is charge of electron, n is the number density of electron, x is the average

Fr
relaxation time, / is the length of conductor and A is its area of cross section, p is the specific resistance of
the conductor.
9. Specific resistance or electrical resistivity of the material of a conductor is defined as the resistance of
for
ur
unit length and unit area of cross-section of the conductor. The SI unit of resistivity is Q m. Resistivity of a
conductor depends upon the nature of the conductor but is independent of the length or area of cross
ks
m
Yo
section of the conductor. In fact resistivity.
oo

10.
Current density (J) at a point is defined as the amount of current flowing per unit area of cross-section of
eB

the conductor, provided the area is held in a direction normal to the current.
r

●^ = — = The SI unit of current density is A m ^


ou
ad

11. Electrical conductivity (a) of a conductor is the inverse of its resistivity (p), i.e., a = 1/p. The SI unit of a
YY

is m~* or S m“*.
12. Effect of temperature on resistance. The resistance of a metal conductor at a temperature fC is given by
nd
Re

/e^ = /?o(l + otf)


Fi

where Rq is the resistance of a conductor at O^C and a is the temperature co-efficient of resistance.
For metals a is positive, i.e., resistance increases with rise in temperature.
For semi conductors and insulators, a is negative, i.e., resistance decreases with rise in temperature.
For alloys like manganin, eureka and constantan, the value of a is very small as compared to that of
conductors. That is why these alloys are used in making standard resistances.

If R^^ and R^ are the resistances of the same conductor at temperature rj“C and r2®C, then
[l + «(^2 ^ is temperature coeff. of resistance averaged over the temperature
range /|"C and t2*C.
13. Non-ohmic conductors. Those conductors which do not obey Ohm’s law are called non-ohmic conductors
e.g. vacuum tube, liquid electrolyte etc.
14. Super-conductors. Those materials which offer least resistance to the flow of current through them are
called super-conductors. Examples : mercury at temp 4.2 K, lead at 7.25 K and niobium at temperature
9.2 K become super-conductors. The conductivity of superconductor is called superconductivity.
3/76 T^nadee^ Fundamental Physics (XII) LV»1WI
15. Colour code for carbon resistors. The number attached from 0 to 9, 10"^ and 10"^ to the various colours
can be recollected by the sentence B.B. ROY Great Britain Very Good Wife wearing Gold, Silver necklace.
Black-0, Brown-1, Red-2, Orange-3, Yellow-4, Green-5, Blue-6, Violet-7, Grey-8, White-9,
Gold-10~', Silver-10“^. The separate strip of gold, silver and no colour shows the accuracy of ± 5%, ±
10% and ± 20% of the given carbon resistor.
16.
Resistance in series. The total resistance (/?^ is given by + /?2 + ■
17.
Resistance in parallel. The total resistance {Rp) is given by L-_L+JL+J_+
R
1 «3
18. Internal resistance of a cell is defined as the resistance offered by the electrolyte and electrodes of a cell
when electric current flows through it. Internal resistance of a cell depends upon : (/) distance between the
electrodes, (») the nature of electrodes (in) nature of electrolyte and (iv) area of the electrodes immersed in

ww
the electrolyte.
19. Terminal potential difference of a cell is defined as the potential difference between the two electrodes of
a cell in a closed circuit. Terminal potential difference of a cell decreases if the current drawn from the cell
increases. Terminal potential difference of a cell (V) is less than the emf of a cell (e) by an amount equal to

Flo
potential drop across the internal resistance of the cell i.e.

ee
£-V e-V'i R
V=z-Ir (■■V=IR)

rere
or r =
/ V

rFF
where R is the external resistance in the circuit and r is the internal resistance of a cell.
20.
Joule’s law of heating. It states that the amount of heat produced in a conductor is directly proportional to
uur r
foor
the : (0 square of the current flowing through the conductor, (ii) resistance of the conductor and (Hi) time
for which the current is passed.
ks s
21. Electric power. It is defined as the rate at which work is done in maintaining the current in electric circuit.
Yoo
Electric power, P = VI = -fR = V^IR watt or joule/second
ook

22.
Electric energy. The electric energy consumed in a circuit is defined as the total work done in maintaining
BBo

the current in an electric circuit for a given time.


Electric energy ~ V It = P t = I-R t = V^t/R
re

SI unit of electric energy is joule (denoted by J)


ouur

where 1 Joule = 1 watt X 1 second = 1 volt x I ampere x 1 second


ad

Commercial unit of electric energy is kilowatt hour (kWh)


YY

where 1 kWh = 1000 Wh = 3-6 x 10® J = one unit of electricity.


23. Kirchhoff’s rules
nndd
Re

First rule: The algebraic sum of the current meeting at a junction is zero, Le., Z i = 0. The current reaching
a function if taken positive then the current leaving the junction is taken negative. This law supports the
Fii

concept that moving charges are not accumulated at a junction, i.e., the law of conservation of charge.
Second rule : In a closed loop, the algebraic sum of the em/s is equal to algebraic sum of the products of
current and resistance in the various arms of the loop.
While traversing a closed loop (in clockwise or anti clock wise direction), if negative pole of the cell is
encountered first then its emf is +ve, otherwise -ive. The product of resistance and current in an arm of the
circuit is taken positive if the direction of current in that arm is in the same sense as one moves in a closed
loop and is taken negative if the direction of current in that arm is opposite to the sense as one moves in the
closed loop. This rule supports the law of conservation of energy.
24. Wheatstone bridge Principle : It states that if four resistances ,P Q, R, S are arranged to form a bridge as
shown in Fig. 2(b).4, on pressing battery key first and then galvanometer key K^, if the galvanometer
shows no deflection, then the bridge is balanced. In that case.
P R

Q S
The practical application of Wheatstone bridge principle is in meter bridge or slide wire bridge and post
office box which are used to find the unknown resistance or specific resistance of the given metallic wire.
CURRENT ELECTRICITY 3/77

25. Expression for unknown temperature using Wheatstone bridge principle is


t = xlOO
R ~R
100 0

where R^, R^qq, Rq are the resistance of platinum wire at t®C, 100“C and 0*C respectively.
26. Principle of potentiometer. It is based on the fact that the fall of potential across any portion of the wire is
directly proportional to the length of that portion provided the wire is of uniform area of cross section and
a constant current is flowing through it., i.e., V «: / (if / and A are constant) or V = Kl,
where K is called potential gradient, i.e., fall of potential per unit length of the given wire.
27. Expression for comparison of emf of two cells by using potentiometer,

^2 ^2

ww
where I2 are the balancing lengths of potentiometer wire for the emfs 6j and of two cells.
28. Expression for the determination of internal resistance of a ceil (r) by potentiometer method

^1-^2'

Floo
r = R
^2 J

ee
where, /j = balancing length of potentiometer wire corresponding to emf of the cell.

reer
rFF
I2 = balancing length of potentiometer wire corresponding to terminal potential difference of the cell when
a resistance R is connected in series with the cell whose internal resistance is to be determined.
uur r
Tr*iiF'^ra ffoor QUESTIONS
sks
NCERT Book
YYoo
Based on
ooko

I. Multiple Choice Questions (fl) n > T2 {b) r,»r2


eBB

1. Dimensions of a block are 1 cm x 1 cm x 100 cm. If


(c) T,=T2 (.d) 7i < T2
4. Masses of three wires of copper are in the ratio
specific resistance of its material is 3 x 10“^ m,
r

1:3:5 and their length.s are in the ratio of 5 : 3 : 1.


ouur

then the resistance between the opposite rectangular


ad

The ratio of their electrical resistances are


faces is
(a) 1:3:5 (*) 5 : 3 : 1
YY

(a) 3x 10'*^ a (b) 3 X lO"”^ Q (c) 1 : 15 : 125 id) 125 : 15 : 1


(c) 3x 10-5 Q (d) 3x 10-5 Q
5. In the Fig. 3{Q).2, a carbon resistor has bands of
nndd
Re

2. The temperature coefficient of resistance for a wire different colours on its body as mentioned in the
is 0-00125 ®C-’. At 27®C its resistance is 1 fi. The figure. The value of the resistance is
FFii

temperature at which the resistance becomes 2 Q is


FIGURE 3(Q).2
(a) 1154 K (b) HOOK

in
(c) 1400 K (d) 1127 K A
3. The voltage V and current / graph for a conductor at
two different temperatures Tj and T2 are shown in u
the Fig. 3(Q).l. The relation between T\ and T2 is Red Yellow Green Silver

(a) 24 X 10*^ ± 5% ib) 35 x 10^ n ± 10%


(c) 5-6 kn (d) 24 x 10^0 ±10%
6. Two wires of same material have length L and 2 L
and cross-sectional area 4 A and A respectively. The
ratio of their specific resistance would be
(a) 1:2 (6) 8 : 1
(c) 1 : 8 (^1:1
3/78 “Pn/uU^ Fundamental Physics (XII)
7. A cell of emf e and internal resistance r is connected 8
across an external resistance R. The graph showing (.) - (i» -V
the variation of P.D. across R (i.e., V) versus R is
4
(0 -V id) 2 W
FIGURE 3(Q).3
11. The reading of the ammeter as per Fig. 3(Q).6 is :

(b) FIGURE 3{Q).6

2Cl
VvW

oww
■►R ■►R
2 V
2Q
Vv^A' A
id) e

2fl

e
re
R ■►R

FFrllo
2Q
●vvw

rF
(CBSK 2020)

ee
8. A cell supplies a current of 0-9 A through a 2 resistor
1
and a current of 0-3 A through 7 Q resistor. The
ouru ia) tA (b) jA

rF
internal resistance of the cell is : 8 4

(a) 2 0 Q (b) 1-5 n 1

fosor
(c) -A id) 2 A
(c) 1011 id) 0-5 .0 2
os kf
9. The current in the adjoing circuit will be : 12. The equivalent resistance of the arrangement of
resistances shown in Fig. 3(Q).7 between the points
ook
FIGURE 3(Q).4 A and B is :
YYo
I
Bo

FIGURE 3(Q).7
reeB

8U
wv
30 n 30
oouY

_ 2 V 160 20 0
VAV ●VAV
ur

30 Q
160
ad

AWSr ■'vWv
I
Yd

A B

1 1
nidn

90
id) — ampere ib) — ampere i-VWv-i
Re

45 15 60
Wv\
180
I 1 <—WsA<-l
FFi

(c) — ampere id) — ampere


10
id) 6 a ib) sn
10. The potential difference between points A and B of
(c) 16 a id) 24 Q
adjoining Fig. 3(Q).5 is
13. In the network of resistors shown in Fig. 3{Q).8, the
equivalent resistance between A and B is :

(a) 54 a ib) 18 a
(c) 36 a id) 9 a
CURRENT ELECTRICITY 3/79

14. The current from the battery in circuit diagram shown 17. Electric current through resistance 10 Cl in the given
in Fig. 3(Q).9 is : circuit is :

FIGURE 3(Q).9 FIGURE 3(Q).12


2Cl 7€l
6V
>ww AA/W 2H

15 V
AAAAf

en in
0-5 n

1
(I1 + I2) ion
AA/SAr
—ww
AAMf
sn ion

I2 2n

ww
(a) I A ib) 2A <■ I AA/W
(c) 15 A (d) 3A 6V
15. In the circuit diagram. Fig. 3(Q).10, the electric

Floo
current through branch BC is :
(a) OA (b) 0-5 A

ee
FIGURE 3(Q).10 (c) 6/11 A (d) 2A

reer
2A B 2Ah 3A 18. Determine the electric current through branch BD of

rFF
4 the electric network:
uur r
C D
ffoor
FIGURE 3(Q).13
sks
YYoo
ooko
eBB

2A 'rSA

E 3A
r
ouur
ad

(fl) 4 A (b) 2A
Y

(c) 5A (d) 10 A
vwv
dY

16. In the given circuit Fig. 3(Q).ll, the current /j is : 5V 2n


Re
nnd
FFii

FIGURE 3(Q).11
(a) 0-6 A (b) OA
30 n
(c) lA id) 10 A
19. In Wheatstone’s bridge, all the four arms have equal
iii. resistance R. If resistance of the galvanometer arm is
40V
40 n also R, then equivalent resistance of the combination
<■ is
I3
l2'r ia) R ib) 2R
40 £2 (c) R/2 id) R/4
HI
20. In a meter bridge experiment, resistance box (with R
80 V = 2 n) is connected in the left gap and the unknown
resistance S in the right gap. If the balancing length
be 40 cm, calculate the value of S
(a) 04 A ib) - 04 A
id) 2 £2 ib) 3 £2
(c) 0-8 A id) -0-8 A
(c) 4 £2 id) 2-5 £2
3/80 ^na^Uep. ’4- Fundamental Physics (XII) W5TW
21. For a cell of emf 2 V, a balance is obtained for
P,h*P,.h.
50 cm of the potentiometer wire. If the cell is shunted (a) ib)
by a 2 n resistor and the balance is obtained across (/,+y ih-12)
40 cm of the wire, then the internal resistance of the
P)'2^-P2'l Pi h ~ P2 k
cell is: (c) id)
((j+y ih-12)
(a) 1£J ib) 0-5 Q
(c) 1-2 n id) 2-5 n 29. If the ammeter in the given circuit shown in the
diagram reads 2 A, the resistance R is :
22. Two cells when connected in series are balanced on

8 m on a potentiometer. If the cells are connected FIGURE 3(Q).15


with polarities of one of the cell is reversed, they 2n R

oww
balance on 2 m. The ratio of the cmf’s of the two
VAAtf WW
cells is :

(a) 3 : 5 ib) 5:3


6 V
(c) 3 : 4 id) 4:3
A
23. A cell of internal resistance 3 and emf 10 V is

ee
connected to a uniform wire of length 500 cm and

FFrlo
resistance 3 tl. The potential gradient in the wire io) 10 ib) 2 .0

r
IS (c) 3:2 id) 4 0

rF
ee
(fl) 30 mV/cm ib) 10 mV/cm (CBSE .Sample Paper 2020)
(c) 20 mV/cm id) 4 mV/cm 30. The heat produced by 100 W heater in 2 ntinutes is

rF
ouru
24. When two resistances /?j and R2 are connected in equal to:
series, they consume 12 W power. When they are (a) 10-5 kJ ib) 16-3 kJ
connected in parallel, they consume 50 W power.
ffosor (c) 12-0 kJ id) 14-2 kJ
os k
What is the ratio of the powers of /?j and /?2 ? (CBSK Sample Paper 2020)
id) 1/4 ib) 4
31. In the circuit shown, P ^ R and the reading of
ook
YYo
ic) 3/2 id) 3
galvanometer is same with switch S open or closed.
25. The potential difference between A and B in the Fig.
Bo

Then
reeB

3(Q).14is :
FIGURE 3(Q).14
oouY
ur

2A 6f2 QO 50
ad

A*—►—AW -V .●V B
dY

12 V 4 V
nidn

(a) 32 V ib) 48 V
Re

ic) 24 V id) 14 V
FFi

26. In a Wheatston’e bridge, P = 2 0, Q = 2 0, R = 20


and S = 3 O. The resistance with which 5 is to be

shunted in order that the bridge may be balanced is :

ia) 10 ib) 20
ia)lQ = h ib)lR = Ic
(c) 4 0 id) 6 0
ic)Ip^lo id)lQ = h
27. In a potentiometer of wire length /, a cell of emf V is
(CBSE 2022)
balanced at a length U3 from the positive end of the
wire. For another cell of emf 1-5 V, the balancing 32. T\vo wires A and B of the same material having
length becomes length in the ratio 1 : 2 and diameter in the ratio
id) //6 ib) la 2 : 3 are connected in series with a battery. The
ic) U3 id) 21/3 ratio of potential differences (V^/Vg) across the two
wires respectivelyis
28. Two wires of equal diameters of resistivity p| and P2
and length /j and I2 respectively, are joined in series. 1 3

The equivalent resistivity of the combination is : W3


CURRENT ELECTRICITY 3/81

4 9 (a) less if the length of the wire is increased


(O- id) - (CBSE 2022)
8 (b) more if the length of the wire is increased
33. A battery of 15 V and negligible internal resistance (c) more if a wire of steel of same dimensions is
is connected across a 50 Q resistor. The amount of used
energy dissipated as heat in the resistor in one
minute is ;
(d) more if the temperature of the wire is increased
(CBSE 2022)
(a) 122 J ib) 270 J
(c) 420 J (d) 720 J (CBSE 2022) 39. The electric power consumed by a 220 V-100 W
bulb when operated at 110 V is:
34. In a potentiometer experiment, the balancing length
with a cell is 120 cm. When the cell is shunted by (a) 25 W (/j) 30 W
a 1 resistance, the balancing length becomes 40 (c) 35 W id) 45 V/ (CBSE 2022)
cm. The internal resistance of the cell is :
40. A potential difference of 200 V is maintained across
(a) ion ib)lil a conductor of resistance 100 fl. The number of

w
(c)3Q (d)2Q. (CBSE 2022) electrons passing through it in 1 s is :
35. Two students A and B calculate the charge flowing (a) 1-25X 10'5 ib) 2-5 X 10^8

Flo
through a circuit. A concludes that 300 C of charge
flows in 1 minute. B concludes that 3-125 x 10*^ (c) 1-25 X 10^^ id) 2-5 X 10^®

reee
electrons flow in 1 second. If the current measured (CBSE 2022)
in the circuit is 5 A, then the correct calculation is
41. A car battery is charged by a 12 V supply and energy

FFr
done by stored in it is 7-20 x 10^ J. The charge passed
ia)A ib)B through the battery is :
urr
(c) both A and B id) neither Aor B
(CBSE 2022)
(a)6-0x lO'^C
for
(c) 8-64 X 10® J
ib) 5-8 X 10^ J
id) 1-6 X 10® J
kkss
36. The resistance of two wires having same length
and same area of cross-section are 2 El and 8 Q (CBSE 2022)
Yo
oooo

respectively. If resistivity of 2 Q wire is 42. Two sources of equal emf are connected in series.
2-65 X 10“* J[i-m, then the resistivity of 8 wire is The combination is, in turn connected to an external
eB

resistance R. The internal resistance of two sources


(a) 10-60X 10-*fim (/?) 8-32 x IQ-* £2 m are and r2 {/'2 > rj). If the potential difference
(c) 7-61 X 10-^ Urn (f/)5-45x I0"*nm
r

across the source of internal resistance r2 is zero,


ou
ad

(CBSE 2022) then R equals to :


YY

37. When a potential difference V is applied across a


conductor at temperature T, the drift velocity of n + ^2
ia) r - ^1 ib) rj - r,
nndd

the electrons is proportional to :


Re

ia)T ib) Jf
Fi

'1^2
iOV id) Vv (CBSE 2022)
(c)- id)
rjT2
38. The given Fig. 3(Q).17 shows /-Vgraph of a copper
wire whose length and area of cross-section are L (CBSE 2022)
and A respectively. The slope of this curve
becomes; 43. The potential difference V applied across a
conductor is increased to 2 V with its temperature
kept constant, the drift velocity of the free electrons
in a conductor will

(a) remains the same

ib) becomes half of its previous value


(c) be doubled of its initial value
id) becomes zero
3/82 “Pfutdec^ '4. Fundamental Physics (XII) kWI
44. The equivalent resistance between A and B is : 48. A 10 m long wire of uniform cross-section and 20
O resistance is used in a potentiometer. The wire
is connected in series with a battery of 5 V along
with an external resistance of480 Q. If an unknown
emf e is balanced at 6-0 m length of the wire, then
the value of unknown emf is ;

FIGURE 3(Q).20

5V
+
480 a
-ww

10 m

♦ 60 m ♦I
(a)3n (b) 5-5 .Q

w
P Q
(c) 7-5 a (d) 9-5 a
45. A constant voltage is applied between the two ends +

Flo
of a uniform metallic wire, heat H is developed in h G
8
it. If another wire of the same material, double the

reee
radius and twice the length as compared to original
(a) 1-2 V (b) 1-02 V
wire is used then the heat developed in it will be :

FFr
(c) 0-2 V (d) 012 V
{a) HU ib)H
49. A uniform wire of resistance 2 /? is bent in the form
ic)2H id) AH
urr
46. Three resistors having values R^, /?2 and are
connected in series to a battery. Suppose R^ carries
for
of a circle. The effective resistance between the
ends of any diameter of the circle is
(a)2R {b) RI2
kkss
a current 2 0 A, R2 has a resistance 3-0 ohm and R^ ic)R (d) R/4 (CBSE 2020)
Yo
dissipates 6-0 watt of power. Then voltage across
oooo

/?3 is :
II. Assertion-Reason T^pe Questions
eB

(a) IV (b)2W
Direction. For question numbers 50 to 62, two
(c)3V id)4\
statements are given, one labelled Assertion (A)
r

47. A straight line plot showing the terminal potential and the other labelled Reason (R). Select the
ou
ad

difference (V) of a cell as a function of current (/) correct answer to these questions from the codes
YY

drawn from it, is shown in Fig. 3(Q).19. The (a), (b), (c) and (</) as given below :
internal resistance of the cell would be then (a) Both A and R are true, and R is the correct
nndd
Re

explanation of A.
(b) Both A and R are true and R is not the correct
Fi

explanation of A.
(c) A is true, but R is false.
(d) A is false, and R is also false.
50. Assertion. Current flows in a conductor only when
there is an electric field within the conductor.
Reason. The electric field decreases the drift
velocity of electrons in the conductor.
51. Assertion. If a wire is stretched to increase its
length n times, then its resistance becomes n~ times.
(ampere) Reason. Resistance of wire is directly proportional
to length.
52. Assertion. When a wire is stretched so that its
(a) 2-8 a (b) 1-4 .Q
diameter is halved, then its resistance becomes 4
(c) 1-2 .0 (d) zero times.
3/83
CURRENT ELECTRICITY

Reason. For the given volume of the wire, 1


resistance of the wire is inversely proportional to Reason. Vj oc

V
square of the diameter. 59. Assertion. The voltage-current graphs of a
53. Assertion. When an insulated wire is bent, its conductor for two different temperatures Tj and
resistivity increases. T2 are shown in Fig. 3(Q).21. The resistance of
Reason. On bending drift velocity of electrons in conductor is greater at T^ than at T2.
the wire increases.

54. Assertion. Two unequal resistances are connected


in parallel across a cell, then the current through
smaller resistance is more.

Reason. Current through a resistor is directly


proportional to the resistance of resistor.
55. Assertion. Induced emf in two coils made of wire

w
of the same length and same thickness, one of
copper and another of aluminium is same. The

Flo
current in copper coil is more than the aluminium
coil. Reason. Greater is the angle of inclination of V-I

e
reee
Reason. Resistance of aluminium coil is more graph with voltage axis larger will be its resistance.
60. Assertion. Every metal does not become

FFr
than that of copper coil. (CBSE 2621)
56. Assertion. The conductivity of metal decreases superconductor at low temperature.
with rise in temperature. Reason. Resistivity of every metal decreases with

for
ur
Reason. With rise in temperature, the thermal decrease in temperature.

speed of the free electrons and amplitude of a 61. Assertion. At some enough high temperature, the
kss
lattice vibration in a metal conductor will increase, resistivity of carbon disappears entirely, making
Yo
resulting increase in the rate of collision between it super-conducting.
oo

free electrons and lattice. Reason. Carbon has positive temperature


B

57. Assertion. A steady current flows in a metallic coefficient of resistivity.


conductor of non-uniform cross-section. 62. Assertion. A copper wire is stretched to decrease
re

Reason. The current density in a metallic the radius by 0-l%, the percentage change in
conductor of non-uniform cross-section is also resistance is 0-2%.
ou
ad

constant.
YY

^R 2Ar
58. Assertion. A potential difference V is applied to Reason. . where R is the resistance
R r

a copper wire of diameter D and length h. On


nd

and r is the radius of wire.


Re

doubling voltage, the drift velocity becomes half.


Fi

ANSWERS

I. Multiple Choice Questions


5. id) 6. (d) 7. {a) 8. (d) 9. (c) 10.(c)
1. (fe) 2. (i>) 3. (a) 4. {d)
15. (a) 16. (6) 17.(c) 18. (6) 19. (a) 20. (6)
11. {b) 12. {b) 13. (d) 14. (a)
25.(6) 26. (d) 27. (6) 28. (a) 29.(a) 30.(c)
21.(6) 22.(6) 23.(6) 24. (c)
34. id) 35.(c) 36. (a) 37. (c) 38.(a) 39.(a) 40. (a)
31. (6) 32. id) 33, (6)
45.(c) 46. (c) 47.(n) 48. id) 49. (6)
41. (a) 42. (6) 43. (c) 44. (c)

II. Assertion-Reason Type Questions


52. id) 5Xid) 54. (c) 55.ia) 56. (a) 57. (c) 5S.id) 59. id)
50. ic) 51. (6)
60.(6) 61. id) 62. id)
3/84
Fundamental Physics (XII) VOL.I

HINTS/EXPLANATIONS For Difficult Questions


I. Multiple Choice Questions 8. As,£ = /(/? + r) /. e = 0-9(2 + r) = 0-3(7 + r)
1. Here, 4 = I cm x 100 cm = 100 cm^ = KT^ ; or 1-8 + 0-9 r = 2-1+0-3 r or 0-6 r= 0-3
1 = 1 cm = 10'^ m ; p = 3 X 10"^ Q m or r = 0-5Q

Resistance, ^ ^ W __ 3 x lO’’x lO'^ 9. R


equivalent
(30 + 30)x3Q 60x30
= 20Q
A 10"2 (30 + 30) +30 ” 90
= 3 X ir’ D
2 1
Current / = —
2. Here, a = 0-00125 ^27 = I ; /?, = 2 Q ; r = ? /?„
— = — ampere
20 10
«!
/?, = /?27ll +a(r-27)J
or
2=1 [1 +000125 (r-27)] 10. The equivalent circuit is as shown in Fig. 3(Q).22.
2-1 FIGURE 3(Q).22

w
t = + 27 = 800 + 27 = %2TC
000125 B
A
WA*—iC
= 827 + 273 =1100 K 5Q 5fl 50

Flo
3, Slope of V-I graph gives the resistance of a conductor.
As slope of graph at temperature T, is greater than at

e
2 V
temperature T2, so the resistance of conductor is

rree
greater at temperature 7, than that at temperature Fj.

r FF
Since resistance of a conductor increases with A ■VSAA»-*-VW\r ^VWv—*c
sn D 5f2 5Q
temperature, so 7j > T2.
uurr
4. r =
A
=^=^=
VH V mid
for
Current in arm ABC is

^1 = 5
kss
+ 5 + 5 15
where V is volume and d is density of the given copper
ooook

wire of mass m. Potential different between A and B is


Yo

/2
V^ = /.(5.3)=^(5.5) = f=iv
eB

R
m

11. The equivalent circuit is as shown in Fig. 3{Q).23.


rr

Hence : i?2 : /?3 “ FIGURE 3(Q).23


ou
ad

m
1 ^2 ^
-Wv
Y

5^ 3^ l2 2.0
dY

= T^T^I=25^3:1/5 ■VAr
Re
innd

= 125 :15 :1 20

5. Nos. attached to colours red, yellow, green and silver c W.- D


Fi

are 2, 4, 6 and ± 10%. The value of resistance 20

= 24xl0^n±10%. ■VW— I A
20
6. Specific resistance of a material is independent of 2 V

length and area of cross-section of material but Effective resistance across C and D is
depends on the nature of material only.
1 1 1 1 3
7. As. V = e - / r = e
y
r or V 1+-^: R
= — + — + — = — or R In
R \ R)
= £
CD 2 2 2 2 CD “ 2
£ Total resistance of circuit.
or
V =
(1 + r/R)
Above relation shows that as R increases. V also ^ - ^CD ^ ^ “ 3 3

increases, and when R becomes infinite, V becomes £

£. Thus graph (a) is true. Current, I = —


R 8/3 8 ~ 4
CURRENT ELECTRICITY 3/85

12. In this arrangement of resistances 8 Q, 16 £2 and 16 15. Current through branch BC


are in parallel, their equivalent resistance is 4 9 Q s! current through branch AB+Current through branch
and 18 12 are also in parallel, their equivalent HB = 2 + 2 = 4A
resistance is 612. The equivalent circuit is shown in 16. Refer to Fig. 3(Q).27, according to KirchhofTs first
Fig. 3(Q).24. The equivalent resistance between A and rule, at junction F,
B is

FIGURE 3(01.27
FIGURE 3(Q).24

412 2012 3012


WsA/~AA/V'v— A ww B

>
40 V
♦ 4012
A B

ooww
612 612 VsAA;—● -♦ c
h *3
l2^
4012
E D
(4 + 20)x(6 + 6) , 24x12
^AB '^(4+20)
“ + (6+6)’’24+12“ 60 V

e
re e
13. The equivalent circuit is shown in Fig. 3(Q).25. The

rFl
equivfdent resistance between A and B is h-h'^h ...(0

Fre
Using KirchhofTs second rule, for loop ABCFA^ we

rrF
FIGURE 3(Q).2S have

312 312 312 30 /j + 40 /j = 40 or 30 /j + 40 (/, +I2) = 40


sffoo
ouur
●—MAAr ● ^AAAr
A B or 7/i + 4/2 = 4
For loop FCDEF, we have
oksk
-40/3-40/2 = -40-80 or /3+/2 = 3
/?^ = 3 + 3 + 3 = 912
Yo
or
(/i+/2) + /2 = 3 or /] + 2 /2 = 3
oo

14. In the given circuit 712, 112 and 1012 are in series.
Y

27,+4/2 = 6 ...(iiO
BB

They are in parallel to the 612 resistor. Their effective or

resistance is. Subtracting (li) from (iii), we get


rre

(7 + l+10)x6 18x6 -5/1=2 or /j =-2/5 = -0*4A


= 4*512
^eff = (7 + l + 10) + 6 17. Two similar cells are in parallel, their effective emf,
ouu

18+6
Y
ad

E = 6 V, their effective internal resistance.


Ihe equivalent circuit is shown in Fig. 3(Q).26. Current
dY

from the battery, 2x2


r - = 112
2+2
15 15
innd

/ = — = 1A
Re

2 + 4-5 + 8 + 0-5 15 The equivalent circuit is shown in Fig. 3(Q).28.


Current through 1012 resistance is.
FFi

FIGURE 3(01.26 FIGURE 3(Q).28

212 6V
112
AAAAr
I WsAr
15V

1012
4-5 a
0-512 AAAAr

/=-i-=«A
1 + 10 11
AAAAr
812
18. The given electric network is a balanced Wheatstone
bridge. So no current flows through BD.
3/86
Fundamental Physics (XH) VOL.I

P K
19. As, ~ ♦ so bridge is balanced, hence the resis- 28. pii and /L =
/t,^ = A
Q S
^ A
tance of galvanometer can be neglected. Here, P and In series.
G are is series, their resistance = 2R : Also /? and 5
^eq-^l+ ^2
are in series, their resistance = 2 /? O'"
A A A
2Rx2R
Now, equivalent resistance = = R
2R + 2R or 0

40 40
'■+'2
20. - = or S = 3Q
S (100 - 40) 60 29. Current, 2=— or
2+R

21. r = R
(h-h) = 2x
50-40 30. Heat produced, // = /»/= 100 x (2 x 60) W-s
= 0-5£2
h L 40 J = 12x kP 1=12-0 kj

ww
31. The reading of galvanometer will not change with
22. ^1 ^8 + 2 10 5 switch S open or closed when bridge is balanced.
* £2 /j-Ij 8-2 6 3 In this state current through R is same as that

FF loo
through G.
zR 10x3

ee
23. Potential gradient = = 1 V/m
{R + r)L (3 + 3)5

er
rFreF
1000 mV
1(X) cm
= 10 m V/cm Since the two wires are in series, they will have
rur
the same current in them.
24. Let P, and ?2 be the powers of resistances /?j and R2. fforo
. ''a Xa
ks
As per question, — + — ^ ■■ V. IR, RB
p/j/(7tD|/4)
YYouo
B B
12
okso

n2
(D B
BBoo

and
P,+P2 = 50 = —X
1
-X -
9
=-
On solving, we get; Pj = 30 W and Pj = 20 W /
B \
D
A J 2 Uj 8
r ee

33. Here, V=15V,P = 50i2,/ = 60s


A = 12= 3
ad
oouur

P2 20 “2
LI . (15)2
Heat energy = —1 = -—— x60 =270J
Y

25. According to KirchhofTs second rule R 50

>-12 + 4 = 2x6 + 2x9 + 2x5 34. Here, /, = 120 cm, /2 = 40 cm, P = 1 £2


dnYd
Re

- V^= 12 + 18+10 + 12-4 = 48 V


A20 - 40^ xl
FFini

26. Let X be the shunt ersistance in parallel with S, for R = =2Q


balancing the Wheatstone’s bridge. So I h \ 40 J
P R 35. Forstudent A,9 = 300C,t = 60s
Q Sxx/(S + x)
Current I.=^ = = 5A
or
2 2 ^ t 60

2 (3xjc)/(3 + a:) For student B, no. of electrons,


3;c
n = 3125xl0'9
or Time r = 1 s

or x = 6Q
Charge flowing,
g = «€ = (3-125 X 10*^) X (16 X 10-1^ C
27. =i or
V U3 II
— = — or L=l.5xi = - q 3125x16
£2 I2 1-5 V ^ 3 2 C!urrent / = 5-0A
1
CURRENT ELECTRICITY 3/87

36. Here, /j =/j, Aj =^3,/?i = 2 Q./?2 = « FIGURE 3(Q).29


R
Then,
1
_ ^Pi 8
''I e V2
h P2 HVWV HWW

/ /O N

or P2 - Pi = (2-65xl0"*)x - R

V, * >
v^> AAAAr

= l(H»0xl0-*nin
The potential difference across the source of
e(yr current of internal resistance r2 is zero, then

oww
eE
37, Drift velocity, = X = — X
m m \d y , e
8 - /t2 = 0 or / = — ...(«)
[V £ = V/d]
2e 8
cV

e
From (0 and (iO
V X, i.e., VjocV ’ (rj+r2 + /?) Tj
^ md

re
FFrlo
On solving. R = (r2-r{)
38. Slope of/-V graph = —- = 1 = A

rF
/

■■"3
efr

ee
eE
V R pL 43. x = - X
m m \d >
It means if L increases slope of graph decreases.

rF
ouru UrfCcV
If specific resistance p increases (when copper wire
is replaced with steel wire), then slope of graph
ffosor
V

decreases. If temperature increases, then resistance Hence, =A = 2 or v. = 2u d


os k 1
R increases, so slope of graph decreases. 1

39. Here, V = 220V,P= 100 W


44. The equivalent circuit is as shown in Fig. 3(Q).30.
ook
YYo

_ (220)2 = 484 0
FIGURE 3(Q).30
Bo

Resistance of bulb, R =
P “ 100
reeB

30 80
AAAAr B
When V' = 110 V, then power consumed A
oouY

60
V'2 ^ (110)2
ur

P' = = 25W AAAAr


ad

R ~ 484 30 0
dY

40. Here, V= 200 V, /? = 100 Q


nidn

V 200
Current, / = — = 2A Here 3 O and 6 O are in parallel, their effective
Re

R 100 resistance is
FFi

ne
3x6
= 20
R,= 3+6
t
Ri1 and 8 O are in series. Their elective ersistance
It 2x1
or n = = 1-25 X 10*® IS
/?2 = 2 + 8=100
e 1-6x10-*®
/?2 and 30 O are in parallel, the equivalent
41. Here, V= 12 V, eneigy stored, E=7-20 x 10^ J resistance is

Charge passing through battery. 10x30 30


R — =7-5 0
10 + 30 4
7-2x10^
= 6-0xl0^C /
12 n _ 1 1
45. Here, ifi=//, —
42. Refer to Fig. 3(<3).29, current in circuit ^2 2 ’ /^ 2
8 + 8 2e y2 y2 y2iir2
/ = Heat produced, H =
-(0
Tj + T2 + R r, + T2 + R R p//7tr2 px/
3/88
U Fundamental Physics (XII) pygnil
r2 II. Assertion-Reason Type Questions
/.e„ H « —

I Ee
50. As, drift velocity, = X E. Hence,
m

JJ / r
«2_ ^ ( 1 Reason is false. Here Assertion is true, because
jJ
x- = 2
H
1
I 2 electric field within the conductor force the free
electrons to follow a definite path which is
responsible for current in the conductor.
or //2 = 2W|=2H
51. Here, Reason is true and Assertion is also true
46. Since three resistors are in series there will be same
but Reason is not the correct explanation of
current / {= 2-0 A) flowing in each resistance. Here

oww
Assertion.
P3 = 6-0 watt.

60 R = p/ _ p/ _ p/2 [v Volume, V-Af]


A ~ V/l~~V
Voltage across E^=~j 2-0
= 3V

_p/'^ _p(n/)2

e
47. From graph, When, / = 0, V = e = 5-6 V ; R' = = n

re
FFrlo
V V V
When, y=0,/ = 2-0A

52. Resistance, /? = Ri. -

rF
ee
5-6 (v V = A0
Internal resistance, r = — = 2*8Q A A
I 2-0

rF
pV _ pV
ouru
48. Here, / =10m;r = 20Q;/? = 480Q;e' = 5V or R =
2- {nD^/4f «“ 1/0^
Current in potentiometer wire is a2

fosor When diameter becomes half, i.e., D' = D!2, then


skf
e' 5 5
I = = 0-01 A 16pV
ooko
R +r 480 + 20 500 /?' = = 16/?
7t^ {Dllf
Yo
Y
Pot. diff. across potentiometer wire PQ = Ir
Thus Assertion is false and Reason is aiso false.
Bo

= 0 01 X 20
reeB

53. Here, Assertion is wrong because resistivity of


= 0-2 V
wire depends on the nature of material of wire and
Thus 10 m wire balances 0-2 V. Therefore 6 m wire is independent of the bending of wire.
ooY
uur

will balance the voltage. On bending the wire, the area of cross-section of
ad

0-2
the wire at bend portion increases. So for the given
dY

V' = X 6 = 0-12 V current, the drift velocity of electrons decreases.


10
Because, v.= —^ 1
nind

49. If a loop is made, the resistances between the nAl or Uj oe — . Thus Reason is
Re

ends ^ A
of any diameter will be in parallel as shown in Fig. false.
FFi

3(Q).3I(a) and (/?). Therefore, the effective


resistance between A and B is 54. When unequal resistances are connected in parallel
across a cell, there will be same potential difference
FIGURE 3(Q).31 y
across each resistance. Now current / = — or
R R
r'W'/v-i
1
/ <= — i.e., current through smaller resistance is
A B A B R

R
more than in larger resistance. Hence Assertion is
true but Reason is false.
o
pi
55. Resistance of a wire. R = — i.e., /? <= p (when /
A
RxR and A are constant), p is greater for aluminium than
R
eff - R + R 2 for copper, so /?^| > /?copper TTtus, Reason is true.
CURRENT ELECTRICITY 3/69

y 60. Both Assertion and Reason are true but the Reason
Now current / = — . i.e., I l/R (when V is
R is not able to explain the Assertion. When
constant).
temperature of metal is decreased, due to crystal
Current in copper wire is more than the imperfections and impurities, every metal does not
aluminium wire. Thus, Assertion is true and become superconductor at low temperature.
Reason is the correct explanation of Assertion. 61. Both Assertion and Reason are false. The carbon
56. The Assertion is True and the Reason is the has negative temperature coefficient of resistivity.
correct explanation of A^ertion because with the For carbon, resistivity,
rise in temperature, there will be increase in the m
-(.Eg/kT)
rate of collision between free electrons and lattice P = 2 with, n = nQe
ne 1
of metal conductor, the mobility of electrons will So, with rise in temperature n will increase, hence
decrease hence the conductivity of metal will p will decrease. However as T => «>, n ^ «q.
decrease.

w
m
57. Here Assertion is true but Reason is false because so that, p= = Pq = a constant ^ 0.
rtQe^x
current density J = — as A is non-uniform, so J is
Therefore, carbon at high temperature will behave

Flo
A as metal but not superconductor.
non-uniform. 62. Both Assertion and Reason are false because

e
reee
58. Both Assertion and Reason are false, because AR 4Ar

FFr
R r
eE e fV
Vj = —X = - X or OC
AR
xlOO
m m \d and % change in resistance = R

for
ur
V 1 4Ar
xlOO
59. From V-I graph, Resistance, /? = y = tan0
kss
r

01
Yo
As, 01 >02 so R\<R2 i.e.R2>R\ = 4x XlOO
oo

100
Hence, Both Assertion and Reason are wrong.
= 0-4%
B
re

i^i '.It
^1 PR4
ou
ad
YY

1. Current and drift velocity easier to send electric currents to definite paths
by the use of insulators than to direct heat flow
1. A wire is carrying a current. Is it charged ? along definite routes using heat insulators.
nd
Re

(Pb. Board 2011) 3. Clarify your elementary notions about current


in a metallic conductor by answering the
Fi

Sol. No. The current in a wire is due to flow of free


electrons in a definite direction. But the number following queries :
of protons in the wire at any instant is equal to (a) The electron drift speed is estimated to be
number of electrons and charge on electron is only a few mm s"^ for currents in the range of
equal and opposite to that of proton. Hence net a few amperes. How then is current established
charge on the wire is zero. almost the instant a circuit is closed ?
2. It is easier to confine electric current to definite (b) The electron drift arises due to the force
paths (by the use of electric insulators) than to experienced by electrons in the electric field
direct heat flow along definite routes using heat inside the conductor. But force should cause
insulators. Why ? acceleration. Why then do the electrons acquire
Sol. The electrical resistivity of insulators {e.g., glass) a steady drift speed ?
differs from metals by a factor of the order of (c) If the electron drift speed is so small, and
10^^. The thermal conductivity of thermal electron’s charge is small, how can we still
insulators differs from thermal metals by a factor obtain large amounts of current in a
of the order of 10^. It is due to this reason, it is conductor ? (CBSE 2015)
3/90
'P'Mdee^ ^ Fundamental Physics (XII)
(d) When electrons drift in a metal from lower (//) In the presence of electric field, the paths of
to higher potential, does it mean that all the the electrons between two successive collisions
“free” electrons of the metal are moving in the with positive ions are curved as each electron is
same direction ?
having two velocities (a) thermal velocity and
(e) Are the paths of electrons straight lines (b) velocity by virtue of force due to electric field.
between successive collisions (with the positive 4. Current I is flowing through a copper wire of
ions of the metal) in the (i) absence of electric radius r with drift velocity of electrons v^. If
field, (ii) presence of electric field. this current is passed through another copper
(NCERT Solved Example) wire of same length and double the radius, what
Sol. (a) When we dose the circuit, the electric field is
will be the drift velocity of free electrons in it ?
set up in the entire closed circuit instantly with Sol. Drift velocity of the electrons for the current /
the speed of electromagnetic wave which causes flowing through a conductor of area of cross-
electron drift at every portion of the circuit. Due section A, having number density of electrons n
and charge on electron e is given by

w
to it, the current is set up in the entire circuit
instantly. The current so set up does not wait for I I
the electrons to flow from one end of the I = n A ev^ or
^d =
n Ae n{nr^)e

Flo
conductor to other end. However, the current does where r is the radius of the wire.
take a little time to reach its steady value. 1

ee
(/?) When a potential difference is applied across Thus, V oc

the ends of a conductor, the electric field is set up

Fr
inside the conductor. Due to it, each free electron (Since /, n and e are same for another wire)
in the conductor experiences a force and is V
d _ r2 1 V
accelerated towards the positive end of the
for v'a
ur
or

conductor, resulting in the increase in its speed


■■ (2r)2 4
until it collides with a positive ion of the metal. Thus drift velocity of electrons in another wire
ks
After the collision, the free electron loses its speed will become one-fourih of the original value.
Yo
but again starts to accelerate by virtue of force on 5. A potential difference V is aplied across a
oo

it due to electric field. Its speed increases again conductor of length 1. How is the drift velocity
eB

only to suffer a collision again and so on. affected when V is doubled and I is halved ?
Therefore, on the average, the free electrons (CBSE 2010)
acquire only a drift speed in the conductor, under Sol. Drift velocity.
r

the effect of electric field,


ou
ad

eE e(V/l) V
(c) In a conductor, we obtain large amount of V
d ~ x = ~—-X
or Vj
OC
YY

current even if the electron drift speed is small m m I

and electron’s charge is small, because the


^d 2V I
electron number density in the conductor is very = 4 or u' =4vj
nd
Re

— X —

large, (of the order of 10^^ m“^). ^d V 1/2


Fi

{d) When a potential difference is applied across It means drift velocity becomes four times the
the ends of a conductor, the drifting of free original drift velocity.
electrons takes place from lower to higher 6. (<z) Write the nature of path of free electrons
potential. These drifting electrons also have in a conductor in the (i) presence of electric
thermal velocity. At an instant, each free electron field (k) absence of electric field.
moves in a direction of resultant velocity of its (b) Between two successive collisions each free
thermal velocity and drift velocity, which is electron acquires a velocity from 0 to u where
different for various free electrons in a conductor.
eE
Hence, ail the free electrons in the metal V =
X. What is the average velocity of a free
conductor are not moving in the same direction. m

(e) (/) In the absence of electric field, the paths of electron in the presence of an electric field ?
electrons are straight lines between successive Do all electrons have same average velocity,
collisions with positive ions of the metal, since (c) How does this average velocity of free
the electrons are moving with their thermal electrons, in the presence of an electric field
velocity like molecules in a gas. vary with temperature? (CBSE 2016)
3/91
CURRENT ELECTRICITY

Sol. (a) (/) In the presence of electric field, the paths / 1


of free electrons in a metal conductor are curved. Sol. Drift speed, = nAe
, Vjoc-
A
(«) In the absence of electric field, the paths of
free electrons in a metal conductor are straight iiAeVj
Current density, J = — = nev.
lines between two successive collisions. A A ^
(b) Average velocity of a free electron in the - a constant

eE J IR
presence of electric field is u = —x Electric field £ = — =
m
I T~ I A~ A
where the symbols have their usual meanings. 1
The different free electrons in a metal conductor i.e.. £o= —
A
move with different velocity in the presence of
Electric current, I = nA Iv^, i.e., I << A

w
electric field,

(c) As the temperature of a conductor increases, Thus current density remains constant in the
the thermal speed of the electrons increases and wire, whose cross-sectional area is increasing

Flo
also the amplitude of vibration of the metal linearly from its one end to the other.
atoms/ions increases. Now the free electrons II. Reslstunce and effect of temperature

ee
collide more frequently with the atoms/ions of
9. Answer the following questions :

Fr
metal. As a result of it, the value of relaxation
lime X decreases. Since average velocity u « x, so (a) A steady current flows in a metallic
conductor of non-uniform cross-section.
the average velocity of free electrons decreases.

for
ur
Explain which of these quantities is constant
7. A potential difference V is applied to a
along the conductor : current, current density,
conductor of length L, diameter D. How are electric field and drift speed ?
ks
the electric field E, the drift velocity and
(b) Is Ohm’s law universally applicable for all
Yo
the resistance R affected when (i) V is doubled
oo

conducting elements ? If not, give examples


(ii) L is doubled (iii) D is doubled ? of elements which do not obey Ohm’s law.
B

V (c) A low voltage supply from which one needs


Sol. We know. Electric field. £ = -
re

L high currents must have low internal resistance.


Why?
ou
ad

eE eV (d) A high tension (HT) supply of say 6 kV


Drift velocity. V^=—X = X
mL must have a very large internal resistance.
Y

Why ?

«=P-=^ Sol. (a) Only current through the conductor of non-


nd
Re

Resistance,
^ A kD^ uniform area of cross-section is constant as the
remaining quantities vary inversely with the area
Fi

(i) When V is doubled, £ becomes double, of cross-section of the conductor.


becomes double and R remains unchanged, {b) Ohm’s law is not applicable for non-ohmic
(ii) When L is doubled, £ becomes half, elements. For example; vacuum tubes, semi
becomes half and R becomes double, conducting diode, liquid electrolyte etc.
(iii) When D is doubled, £ remains unchanged, (c) As, / max = e.m././intemal resistance, so for
Vj is also unchanged and R becomes one-fourth. maximum current, internal resistance should be
least.
8. A wire whose cross-sectional area is increasing
linearly from its one end to the other, is {d) A high tension supply must have a large
connected across a battery of V volts. Which internal resistance otherwise, if accidently the
circuit is shorted, the current drawn will exceed
of the following quantities remain constant in
the wire ? safety limit and will cause damage to circuit.
10. A wire of length has a resistance Rq. It is
(a) Drift speed {b) Current density (c) Electric
current (d) Electric field. (CBSIC 2017) gradually stretched till its length becomes 2 Lq.
3/92
^ Fundamental Physics (XU) VOL.I

(a) Plot a graph showing variation of its


resistance R with its length / during stretching.
(b) What will be its resistance when its length
becomes 2 £,«0 ? (CBSE 2020)
pi
Sol. (a) R = pl/A = [v Volume V=A/orA = V7/]
V/l

●●● R =
p/2 , f.e., R .« -p
V
Sol. We know that resistance, /? = —
Thus graph between R and / will be as shown in /
Fig. 3(Q).32. From Fig. 3.64, for given V, l\> l^ \ therefore,
/?2 < /?2- /? 7*2 so 7*2 < 72, i.e.,
T2 is greater than Tj.

ww
13. Plot a graph showing the variation of current
density (J) versus the electric field (E) for two
conductors of different materials. What

FF loo
information from this plot regarding the
properties of the conducting material, can be

ee
obtained which can be used to select suitable

er
materials for use in making (/) standard

rFreF
resistance and («) connecting wires in electric
circuit ? (CBSE 2015)
rur
Sol. We know that
ffoor 7 = a£ or q-J/E
(i) R^=plJA = plltV ■ where a is the electrical conductivity of the
ks
When /new
becomes 2 £q. conductor.
YYoou
okso

The variation of current density (7) versus the


then, R new = AR 0 electric field (£) for two conductors A and B is
BBoo

V V
shown in Fig. 3{Q).35.
11. Plot a graph showing the variation of resistance
re

of a conducting wire as function of its radius,


keeping the length of a wire and its
oouur
ad

temperature as constant. (CBSE 2013)


Y

Sol. Resistance of a conducting wire of length /, radius


r and resistivity p is given by
dnYd
Re

1
R =
2 So R OC

r2
FFini

nr

Here, the slope of each st. line tells the


Hence the variation of R versus r is of the type as conductivityof that material.
shown in Fig. 3(Q).33.
From graph, Og>a^.
R, FIGURE 3(Q),33
We know that electrical resistivity (p) is reciprocal
of electrical conductivity.

>1 or Pa>Pb
Pfl

♦ r The material to be used in making a standard


resistance, we require that the resistance of the
12. V-I graph for a metallic wire at two di^erent given conducting wire is large. It will be so if its
temperatures Tj and Tj is shown in the Fig. resistivity is large and its conductivity is small.
3(Q).34. Which of the two temperatures is Therefore, the material A is used in making the
higher and why ? (CBSE 2015) standard resistance.
3/93
CURRENT ELECTRICITY

The material to be used in making a connecting Sol. (0 From graph, corresponding to point A, let the
wire, we require that the resistance of the given resistance be Rq and corresponding to point B,
the resistance be R. When temperature changes
connecting wire is small. It will be so if its
resistivity is small and the conductivity is large. from 0 to e^C, the resistance changes from Rq to
Therefore the material B is used in making a R.

connecting wire in the circuit. Thus change in temperature = 0 - 0 = 0"C


14. The voltage current variation of two metallic change in resistance = R-Rq
wires X and Y at constant temperature are Temperature coefficient of resistance,
shown in Fig. 3(Q).36. Assuming that the wires
have the same length and same diameter, a =
change in resistance
explain which of the two wires will have larger original resistance x change in temp.
resistivity. (CBSE 2003)
R-Rq oc-oa
~ ~ OAxOE

ww
FIGURE 3(Q).36

(ii) With the rise of temperature of conductor, the


resistance of a conductor increases because the

Floo
frequency of collision of electrons with
ions/atoms of the conductor increases, resulting

ee
decrease in relaxation time (t) of electrons.

reer
●V
As, R « I/'T, so R increases as t decreases.

rFF
16. Is current density a vector or a scalar quantity?
Sol. Slope of I-V line for a wire represents the Deduce the relation between current density
uur r
conductance of the wire. From graph.
Slope of I-V line for wire X > slope of I-V line for
wire Y.
ffoor
and potential difference across a current
carrying conductor of length /, area of cross-
section A, and number density of free
sks
Conductance of wire X > conductance of wire Y. electrons n. How does the current density, in a
YYoo
ooko

1 conductor vary with


As, resistance = so
(a) increase in potential gradient ?
eBB

conductance

Resistance of wire X < resistance of wire Y.


(b) increase in temperature ?
(c) increase in length ?
r

I I
(d) increase in area of cross-section ?
ouur
ad

or
Px<Pk
(Assume that the other factors remain constant
Y

Thus, wire Y has larger resistivity. in each case). (CBSE Sample Paper 2011)
15. The variation of resistance of a metallic Sol. Current density is a vector quantity. Its direction
nddY
Re

conductor with temperature is shown in Fig. is same as that of motion of positive charge.
3(Q).37. eE
FFiin

T
(i) Calculate the temperature coefficient of Current, I = nAev^ and ~ m
resistance from the graph.
I eE
(m) State why the resistance of the conductor Current density, J =— = nev^ ~ nex — x
A m
increases with the irse in temperature.
ne
FIGURE 3(Q).37
m
.'J
g ...
(a) With increase in potential gradient (V/0. J
increases.
c
3
,52 ib) With increase in temperature, x decreases, so
J decreases,
Qt A ●ID

E (c) With increase in length J decrases.


0 (^0 With increase in area, J remains unchanged
Temperature ("C)
as y is independent of A.
3/94
Fundamental Physics (XII) VQL.I

17. Explain why bending a wire does not affect its 21. A steady current is flowing in a cylindrical
electrical resistance ?
conductor. Is there any electric field within the
Sol. Free electrons in a wire have small value of drift conductor ? If yes, what is its relation with
velocity and hence low value of inertia of motion. current density ?
Due to it, they are able to go around the bends Sol. Yes, the current flows through a conductor only
easily. That is why, the electrical resistance is not when the electric field established within the
affected on bending the wire till the area of cross- conductor exerts force on the free electrons, due

w
section remains the same at the bend.
to which they move in a definite direction,
18. A copper wire of resistance is stretched till accounting for current In the conductor. The
its length is increased to n times of its original electric field E inside the conductor is given by
length. What will be its new resistance ?
^ _ p<nential difference _V _ IR I pi

e
Sol. Original resistance, =P
length / / / A

re
New resistance, R = p l/A.

row
As the volume of wire remains constant, /
~ = J
so /(,Ao = /A=(n/y)A or
A = A(^n { A

eeF
ullo
III. Gn>uping of resistors

FF
nl.
pi
Now R = i^ = p—2. = n
ifpl = «2/? 0
A A^/n 22. A uniform wire is cut into 10 segments
increasing in length in equal steps. The

srr
19. A copper wire of length / and radius r Is nickel
resistance of shortest segment is R and the

roF
plated till its final radius is 2 r. If the resistivity

k
resistance of the other segments increases in
of the copper and nickel are p^ and p„, then steps of 4 £1. If the resistance of the longest
uor
find the equivalent resistance of the wire.
Sol. Let R^ and R^^ be the resistances of copper wire
ofof segment is 2 /?, find the value of R and the
resistance of original wire.
and nickel plated portion of wire respectively.
kos
Sol. Resistance of first segment = R Q
YYo
B

Then R = Pci
Resistance of second segment = /? -i- 4 xl £1
oo

; and R
rY

n{2 r)^-Kr^ Resistance of third segment = /? + 4 x 2 £2


eerB

Here, R^. and /?„ are in parallel, the equivalent Resistance of tenth segment = /? + 4 x 9
u

resistance of the wire is = R + 36Q

As per question, /? + 36 = 2/? or /? = 36 £2


oou
d

R.R. p^//(7tr^)xp^//[7c(2r)^ - Ttr^J


ad

R = Resistance of the origin wire


K. + /?. P I
nY

^ n
= /? + (/? + 4) -K (/; + 8) + (/? + 12) -t-....
nr'^ [71 (2r)2-icr^I
+ {R + 36)
nid
Re

PcPgl = 10/?+ 4(1 + 2 +3+ .... + 9)


= 10 X 36 + 4 X 4.5 = 540 Q
FFi

(P„+3p^)
23. The V-I graphs of two resistors and their series
20. Give relation between drift velocity and electric combination are shown in Fig. 3(Q)38. Which
field.
one of these graphs represents the series
Sol. We know, I = ne Av^ combination of other two ? Give reason for
or
I/A = n e Vj [v l/A=J] your answer. (CBSE Sample Paper 2003)
or
J = nev^ ^ _ current density (7)
electric field (£)

nev,
OT c E = n e or E= ^ = nepv^l

1
= p = resistivity
c
3/95
CURRENT ELECTRICITY

Effective resistance when connected in parallel


Sol. Let for the given voltage V, the values of currents
be /|, /2 and I2 for resistors 1,2 and 3. R
^1 ^2 _ (p, z;A,)(p^zyAp
cff "
V pl I/A| +P2 IIA^
y?2=- and «3 =
/, 2 /2 p, P2 L^/A|
From graph, we note that I^> L[p| A^+p^Aj]
/?2 < i?2 ^ ^1 A,A2
Hence graph 1 represents the series combination
of other two resistances. P| ...(0

24. V-1 graph.s for parallel and series combination Pl A2 + P2 A^


of two metallic resistors are shown in Fig. L
...(«)
3(Q).39. Which graph represents parallel ^eff ” Peff X

A] +A2
combination ? Justify your answer.

w
From (i) and (»), we have
L P| P2^
Pcff><

Flo
{AJ+A2) p, A2+P2A,
PlP2<'^l + ^2>

eee
or
Peff =
Pl ^2 P2

Fr
IV. Internal resistance
and Grunpinj; of cells
for
V
ur
Sol. As slope of V'l graph = — = resistance R.
I
27. A cell of emf E and internal resistance r is
From the given graphs, the slope of 3 is greater connected across a variable resistor R. Plot a
ks
than that of A. So the resi.stance of B is greater graph showing the variation of terminal
Yo
oo

than that of A. As resistance in series combination potential V with resistance R. Predict from the
is more and in parallel combination is less, so B graph the condition under which V becomes
eB

represents series combination and A represents equal of E. (CBSK 2009)


parallel combination of resistors.
Sol. V=E-lr ^E--r (v I=V/R)
r

25. When we switch on the lights one after the


ou

R
ad

other, what is the effect on the resistance of


the electric circuit of the house ? On the E
YY

r
or V 1+ — = E or V =
current flowing in the main circuit ? R (1 + r//?)
Sol. In a house, all lights are in parallel circuits, to a
nd
Re

As R increases V also increa.ses.


fixed supply voltage 220 V. When we switch on
On plotting a graph between V and R, we get a
lights one after the other, the effective resi.stance
Fi

curve of the type as shown in Fig. 3(Q).40.


of the circuit goes on decreasing. As a result of it,
the main current goes on increasing. Va FIGURE 3(Q).40

26. Two metallic rods, each of length L, area of


cross-section A| and A 2 having resistivity p,
and P2 are connected in parallel across a d.c.
battery. Obtain the expression for the effective
resistivity of this combination. (CBSE 2020)
Pl ^ ■►R
Sol. Resistance of first metallic rod, /?|
When graph becomes parallel to resistance axis,
then, V=£ if /? = =«.

_P2L 28. A battery has an emf E and internal resistance


Resistance of second metallic rod, /?2 = r. A variable resistance R is connected across
3/96
‘Pmt.dee^'4. Fundamental Physics (XII)
the terminals of the battery. Find the value of 30. In which respect, does a nearly discharged lead
R such that (a) the current In the circuit is acid secondary cell differ mainly from a freshly
maximum (b) the potential difference across charged cell in Its emf or in its internal
the terminals Is maximum. resistance ?

Sol. A discharged lead acid secondary cell differs from


Sol. (a) Current in the circuit is, I ^; / will be a freshly charged cell mainly in internal resistance
r + R
maximum when, R = min. = 0. and partly in its emf as a nearly discharged lead
E acid secondary cell acquires very high internal
^max, “ r
resistance.

(b) Potential difference across the terminals, V. Joule’s law of heating


ER E 31. A wire connected to a bulb does not glow,

ooww
V = IR~
r +R (rlR + \) whereas the Hlament of the bulb glows when
same current flows through them. Why ?
V will be maximum when — +1 = mimmum. Sol. Filament of bulb and supply wires are connected
R in series. Therefore, the same current flows
through them. Since the resistance of connecting

e
i.e., — = zero, i.e., R ~ wires is negligibly small as compared to the

ree
CC

rFl resistance of filament and heat produced due to

Fre
In this situation, 7=0 and V=E= max. given current is directly proportional to its
29. A number of identical cells, n, each of emf e, resistance (from Joule’s law of heating), therefore,

rrF
internal resistance r, connected in series are the heat produced in the filament is vey large.
charged by a d.c. source of emf using a Hence the bulb glows, but the connecting wires
ouur
resistor R. (0 Draw the circuit arrangement.
(h) Deduce the expression for (a) the charging
sffoo remain practically unheated.
32. An electric motor runs on a d.c. source of emf
okks
current and (ft) the potential difference across e and internal resistance r. Show that the power
Yo
the combination of the cells. output of the source is maximum when the
oo

current drawn by the motor is e/2 r. (NCERT)


Y
BB

[CBSE 2003 (C), 2008]


Sol. Let I be the current drawn by motor. Then power
Sol. (0 The circuit arrangement for charging the cells output of the source, P = El-fi r
rre

is shows in Fig. 3(Q).41.


P will be maximum, when
ouu

FIGURE 3(Q).41
Y
ad

dP
— n cells —►
dl
= 0 or — (87-/2 r) = 0
dt
dY

e r
I-
e r
or 8 - 2 7r = 0 or 7 =—
innd
Re

2r
s' R
33. In the above question, show that power output
FFi

of electric motor is maximum when the back


(ii) Effective emf of the circuit = e' -ne emf is one-half of the source emf, provided the
Total resistance = R + nr resistance of the winding of the motor is
negligible. (NCERT)
Charging current 7 = effective emf _ e' - #ie Sol. Here, emf of d.c. source = e,
total resistance R + nr
resistance of motor 7? = 0
(Hi) Pot. diff. across each charging cell = 8 + 7r Internal resistance of source = r
8 ne e7? + 8V Let e' be the back emf of motor. Therefore,
= 8 + r -
R + nr R + nr
. L ● ● r e -e' e-e'
Pot. diff. across the series combination of n cells current m the circuit, 7 = -
r+R r

\
tR + t'r Power output of the motor,
V = n (8 + 7r) =n
R + nr P = power output of source = r
3/97
CURRENT ELECTRICITY

dP r = P
P is maximum when = 0
dl
or
P~P^ + P^ + P^ R

Hence total electric power consumed is equal to


or —(e/-/^r) = 0 ore=2/r
dt the sum of the powers of the individual
e~e' e , £
appliances.
or
,
/ = ●—
e or
2r r

2r
or e = -
2 37. What is the power transferred per unit volume
into jouie heat in a resistor ?
34. A nichrome heating element across 230 V
supply consumes 1‘S kW power and heats upto
Sol. Power, P^fiR = -f p//A ; (■.● R = pi'A)
a temperature of TSOX. A tungsten bulb across Volume of resistor, V - Al
the same supply operates at a much higher Power transferred per unit volume
temperature of 1600”C in order to be able to \2
emit light. Does it mean that tungsten bulb P _ I^pUA _( -/ p =
necessarily consumes greater power ? “V Al A

w
(NCERT) where, I/A=J = current density.
Sol. No, the steady temperature acquired by a resistor 38. Prove that in series combination of electrical

Flo
(i.e., heating element) depends not only on the appliances, the reciprocal of total power
power consumed but also on the power loss due consumption is equal to the sum of the

reeee
to radiation, which inturn depends on the reciprocal of the powers of the individual
characteristics of a resistor such as surface area,
appliances.

FFr
emissivity, etc.
Sol. Let /*2’ ^3 powers of electrical
VI. Electric power and Electric energy appliances at V volt. If /?], ^3 their
for
ur
resistances, then
35. When current is passed through a heater, the
heat is generated continuously in it but its y2 y2
kkss
R.I = —, R, = — and R, = —
temperature becomes constant after sometime. P ^ P2 ^ P3
Yo
1
Why ?
oo

When the electrical appliances are connected in


Sol. When the temperature of heater becomes more series, then effective resistance, R is
eB

than the temperature of surroundings, it starts


lossing heat to surroundings. Finally a stage is R = Rj + R2 + R2
reached at which the rate of production of heat R
r

R 1
ou

becomes equal to the rate of loss of heal. At this


ad

or
V/2 y2 V2
y y2
stage, the temperature becomes constant.
YY

1
36. Prove that in parallel combination of elec-trical or
1 1 1

appliances, total power consumption is equal V^/R V^/R, V^/R2 vVRj


ndd
Re

to the sum of the powers of the individual


appliances. (CBSE 2011) 1-_L J_ J-
Fi

or
Sol. Consider electrical appliances of powers, Pj, P2, ^3
P3 having resistances /?j, i?2» ^3 connected in 39. A battery has an emf E and internal resis- tance
parallel. Suppose they are operated at main r. A variable resistor R Is connected across the
voltage Vf Let P be the total power and R be the terminals of the battery. Find the value of R
total resistance of the combination of all
appliances. Then according to law of resistances such that the power delivered to the resistor is
maximum.
in parallel.
E
1-J- ^ ^ Sol. Current in the circuit, / - R + r
R " Rl R2 R3 Power delivered to the resistance R is,
Multiplying both sides by V^, we get £^R
y2 y2 y2 y2 P = /2xR =
— 1 1 (R + rf
R R. R2 P3
3/98
Fundamental Physics (XII) VOL.I

Power delivered to the load will be maximum Power is maximum when R = r/2. Then
R
when is maximum, i.e., f E ^
(i? + r)2 P
max.
r/2 =
[r/2 + r/2 2r

4[R(R + r)-2] = 0
OK
Note that in both the cases, maximum power
delivered to the load is the same.
or [(/? + r)-2 + /e(-2)(/? + r)-3] = 0 41. Long distance power transmission is carried
or (R + r)-^[r-R] = 0 on high voltage lines. Why ?

w
As (R + r)~^ ^ 0 for finite value of R, Sol. When current / is transmitted through a power
so r-R = 0 or R = r line of resistance R, Power loss = /^R.

When /? = r, the power transferred to the load is If the power P is transmitted at voltage V, then P

e
_ E^r E'^ = V7 or I = (P/V) Power loss =

roow
R
V2

re
= max.
(r+r)2 4r
For a given power and given line, P and R are

40. For what value of load resistance, the power constant. Hence, power loss <« (1/V^)

F
transfer is maximum when two identical cells

uFFll
each of emf E and internal resistance r are It means if power is transmitted at high voltage,

ree
power loss will be small and vice-versa.
connected (a) in series (b) in parallel.
42. Two tungsten filaments with resistance R| and

sFr
Sol. (a) When cells are connected in series to the
R2 respectively are connected first in series and
external load resistance R, then effective emf

kro
then in parallel in a lighting circuit of negligible
uor
= E + E = 2E.
offo internal resistance. If Rj » R2;
Total resistance of the circuit = R + r+ r
(a) Which lamp will glow more brightly when
= R + 2r. they are connected in series ?
kos
Y
Yo
2E (b) If the lamp of resistance R2 now bums out
eerBB
oo

Current in load, I =
R + 2r rad the lamp Rj alone is plugged in, will net
iilumination increase or decrease ?
urY

Power delivered to the external load is.


(c) Which lamp will glow more brightly when
they are connected in parailel ?
P = I^R =
(d) If the lamp of resistance R| now bums oiit,
ou

R + 2r
ad
do

how will the net illumination produced


\ /

Power delivered to the external load will be


nY

maximum if external resistance = internal change ?


resistance of cells, i.e., R — 2r. Then maximum Sol. (a) When the bulbs are in series, current I =
nid

constant. As P = /^ /?, so P « /?. Hence the lamp


Re

x2
' 2E with higher resistance (Le. Rj) will glow more
power, P.max.
FFi

x2r =
\
2r+2r
/
2r brightly,
(b) When the two bulbs are operating in series,
{b) When cells are connected in parallel to the
external load resistance P, then effective emf total illumination, P =
= E. Total resistance of the circuit (Pj+Pj)
= R+
rxr
When the lamp of resistance Pj gets burned and
r+r 2) bulb of resistance P, alone is operated, then
E V/2
Current in load, I = illumination, P'= Le.. P'>P
(P + r/2) R,1 ’
Power delivered to the external load. So the net illumination will increase.
(c) When the bulbs are in parallel, voltage across
P = I^R = ' E_ R
[p + r/2j each bulb is same. As, P =
P
, so P OC —
P
1
CURRENT ELECTRICITY 3/99

Hence, the lamp with lesser resistance, i.e., R2 Efficiency of transmission line
will glow more brightly,
_ power delivered byline
{d) When both the bulbs are operating in parallel, powersupplied to line
then total Illumination,
power delivered
V2 v'2 power delivered + power loss
R
1 ^2 20
= 0-858 = 85-8%
When lamp of resistance /?j bums out, only lamp 20 + 3-31

oww
of resistance R2 will give light. Then illumination VIII. Kirchhoff’s laws

/>' =
and Wheatstone bridge
IS, , i.e., P' <P
47. Can meter bridge be used for finding the

e
So the net illumination will decrease. resistance of (i) moderate values (ii) high values
(Hi) low values ? Explain.

re
YIl. Aspects of heating effect of current Sol. The meter-bridge is most sensitive when the

FFrllo
resistance of all the four arms of the bridge are of

reF
43. How does the use of fuse wire save the electrical
the same order. It is so for moderate resistances

e
installations ?
only. The meter bridge becomes insensitive for
uoru
Sol. A fuse wire is one which has high resistance and
low melting point. This is connected in series with
too high or too low resistances. Moreover, while

osFr
measuring low resistances, the resistances of the
electrical installations. When the supply voltage
copper strips and connecting wires become
exceeds the safe limit, more heat is produced in
the fuse wire. (.● rate of heat production P = V^t fkfor comparable to the unknown low resistance and
hence cannot be neglected.
okso
R), and it melts. The circuit breaks and the damage
to the electrical installations is saved. 48. Using meter bridge, it is advised to obtain the
Y
Yo
null point in the middle of bridge wire. Why ?
44, Why the brightness of light emitted by a bulb
oo
BB

decreases gradually with its period of use ? [CBSE20II (C), 2013)


Sol. The error in the measured value of unknown
Sol. When the bulb is used, the evaporation of the
Y
r ree

metal from the filament of bulb takes place with resistance 5 using meterbridge will be minimum,
oouu

time which deposits on the inner side of the glass when the null point is obtained at the middle of
wall as black substance. Due to it, the filament of meterbridge wire. In this situation, the end error
ad

the bulb becomes thinner and thinner with use. of the bridge will become ineffective. To
Yd

This in turn increases the resistance of the bulb. understand it, let a be the end resistance of left
(●.● R « l/7cr^). AsP = vVr, so brightness of bulb
nidn

and right copper strips in meterbridge. Let


decreases gradually with its period of use.
Re

unknown resistance S be in the right gap and


45. If one wants more light at large distances, known resistance /? be in the left gap of meter
FFi

assuming the wattage is the same, will one use bridge. If null point is obtained at the middle of
a point source or a cylindrical tube light ? bridge wire {i.e., / = 50 cm) then
Sol. For a point source, the intensity <>« 1/r^ and for a 5 +g 100-50
cylindrical source, the intensity oc 1/r, Thus, the = 1 or S-R
/? + a 50
cylindrical tube light is better than the point source
of light. It means the effect of a gets cancelled. In this
46. A line having a total resistance of 0*4 Q delivers situation the bridge is most accurate and sensitive.
20 kW at 220 V to a small factory. Calculate If the null point is not at the centre, there will be
the efficiency of the transmission. some error because S will then not be equal to R.
Sol. Power loss in the transmission line in the form of 49. In the meter bridge experiment, a student
heat observes a balance point at the point 7, where
AJ = 1, Fig. 3(Q).42. The value of R and X are
2
P = I^R= (Pf
- R =
20.000
xO-4 both doubled and then interchanged. What will
V 220
be the new position of balance point. If in this
= 3306W = 3-31 kW set up, the galvanometer and battery are
3/100 ^ Fundamental Physics (XII)
interchangedat the balance point position, how Since the current in BU arm is zero, so the
wiil the balance point get affected ? Wheatstone bridge is balanced. Therefore,
(CBSE 2011)

FIGURE 3(Q).42 2R~ S


R X
-VW >VW So. R = S=22xl(P^ = 22kQ
t 2X 2R
I A/Vv When the resistances in arms BC and CD are
interchanged and another carbon resistor R' is
A' /'● £
J B connected in place of R, Fig. 3(Q).44. The
condition for balanced bridge will be
O ©

R /
Sol. Refer to Fig. 3{Q).43(<3) ...(/)

ww
’ X (100-/)
When R = 2R and X = 2 X are connected as shown
in Fig. 3(Q).43(/?), let the balance point be at 7],

Floo
where AJx = /'.

ee
2X /' R 100-/'
Then or — = ...(«)
2R (100-/') X /'

ee r
rFrF
/ 100-/'
From (/) and (i7)
’ 100-/ “ /'
rur
On solving, we get /' = 100 -1 ffoor 2R _ R'
R ~ 2R
ks
When galvanometerand battery are interchanged,
there is no change in the balance point position. or vr = 4R = 4x22kn = 88kn
YYoou
ookos

50. In the given Wheatstone bridge, the current in = 88 X 10^ Si


the resistor 312 is zero. Find the value of R, if The sequence of colours will be grey, grey and
BBo

the carbon resistor, connected in one arm of orange.


re

the bridge, has the colour sequence of red, red


IX. Potentiometer
and orange. Fig. 3(Q).43.
ouur
ad

51. What do you understand by sensitiveness of a


Y

potentiometer and how can you increase the


sensitiveness of a potentiometer ?
nd dY
Re

[CBSE 2011 (C)l


Soi. The sensitiveness of a potentiometer means the
FFini

smallest potential difference that can be measured


with its help and for a small change in pot. diif.
to be measured it shows a large change in
balancing length. The sensitiveness of a
potentiometer can be increased by decreasing its
The resistances of BC and CD arms are now potential gradient. This can be achieved,
interchanged and another carbon resistance is (a) by increasing the length of potentiometer wire.
connected in place of R so that the current {b) If the potentiometer wire is of fixed length,
through the arm BD is again zero. Write the the potential gradient can be decreased by
sequence of colour bands of this carbon reducing the current in the potentiometer wire
resistor. Also find the nature of current through circuit by increasing the resistance with the help
it. (CBSE Sample Paper 2011) of rheostat.

Sol. Let S be the resistance of carbon resistor in arm 52. On what factors, does the potential gradient
DC. Its value is S = 22 x 10^ fl. of the potentiometer wire depend ?
CURRENT ELECTRICITY 3/101

Sol. The fall of potential across a potentiometer wire potentiometer wire will become less than the emf
of length I is given by V = IR-I ^ UA of cell 1-5 V to be balanced on the wire. Due to
it, balance point cannot be obtained on
Potential gradient, k = Y- = L2. potentiometer wire.
I A
(in) When the high resistance R is used in the
Thus potential gradient of a wire depends upon secondary circuit, the position of balance point
the following factors: on the potentiometer wire will not be affected
(0 K oc I {i,e. current passing through the poten because no current flows through the cell at the
tiometer wire («) K oc p (i.e. specific resistance balance point.
of the material of the potentiometer wire 55. Fig. 3(Q).46 shows the circuit diagram of a
1 potentiometer for determining the emf e of a
(Hi) K — , where A is the area of cross-section
A cell of negligible internal resistance.
of the potentiometer wire.

ww
FIGURE 3(Q).46
53. What does the no deflection position in the
2 V Rl
galvanometerof potentiometerexperiment tell vw
us about the flow of current ?

Flo
Sol. When the balance point is obtained on the A B

ee
potentiometer wire, no current flows in the

rere
galvanometer circuit because the fall of potential

r FF
across the potentiometer wire between zero end
and balance point position becomes just equal to
the emf of the cell, whereas the current in the
uurr
IV'J
potentiometer wire continues flowing.
54. A potentiometer wire of length 1 m is connected
foor KeyK
kss
to a driver cell of emf 3 V as shown in Fig. (0 What is the purpose of using high resistance
Yoo
ooook

3(Q).45. When a cell of emf 1*5 V is used in the


secondary circuit, the balance point is found (u) How does the position of balance point (J)
eBB

to be 60 cm. On replacing this cell with a cell change when the resistance Ri is increased ?
of unknown emf, the balance point shifts to
80 cm.
(CBSE Sample Paper 2020)
rr

(Hi) Why cannot the point be obtained,


ouu
ad

FIGURE 3(Q).45
(a) When the emf e is greater than 2 V, and
3V
YY

(b) When the key K is closed. (CBSE 2009)


A B Sol. (0 The purpose of high resistance i?2 is to protect
nndd
Re

1-5V the galvanometer by decreasing the curent through


it for positions, which are far away from the
KiH
Fii

balance point.
(0 Calculate unknown emf of the cell. (H) When resistance /?j is increased, the current
(/() Explain with reason, whether the circuit works, through the potentiometerwire circuit decreases.
if the driver cell is replaced with a cell of emf Due to it, the potential gradient across
IV. potentiometer wire decreases. As a result of it,
(Hi) Does the high resistance Ry used in the the balance point shifts towards the end B.
secondary circuit affect the balance point ? (Hi) (a) When emf e is greater than 2 V, the balance
Justify your answer. (CBSE 2008) point cannot be obtained on potentiometer wire
because the potential drop across the
h i-y 80 potentiometer wire will be less than 2 V.
Sol. (i)-r = T or =1-5X- = 2K)V (b) When the key K is closed, the cell emf e gels
(H) When the driver cell of emf 2 V is replaced short circuited. Due to it, the balance point is not
by a cell of emf 1 V, the circuit will not work obtained on the potentiometer wire.
because the fall of potential on the entire
3/102 ‘PnaxUefi. 'a Fundamental Physics (XII)
56. For the circuit shown in the Fig. 3(Q).47, would Sol. (i) When /?] is decreased, the balancing length
the balancing length increase, decrease decreases. It is because as /?]decreases, the
or remain the same if (i) is decreased
current in the potentiometercircuit increases.Due
(u) i?2 is increased, without any change (in each
case) in the rest of the circuit ? Justify your to it, the potential gradient of potentiometerwire
answer in each case. increases. As a result of it the same emf will be

balanced on smaller length of potentiometer wire.


FIGURE 3(Q).47
Ki (ii) When /?2 is increased, the balancing length
Ri
- h I— increases. It is because as /?2 increases, the current

e
decreases. This increases the

oww
A B /
r +
\ ^ )

+ E
terminal pot. diff. V (= e - /r) across the unknown
{g>-J cell. As a result of it, the terminal potential
R2 K2 difference of cell e will be balanced on larger

e
FFrlo
UvvaK*>-J length of potentiometer wire.

re
(CBSE Sample Paper 2013)

ree
F
● Very Short Answer

rF
● Short Answer
* Long Answer

fsoor
ouur
skf
VERY SHORT ANSWER QUESTIONS Carrying 1 mark
ooko
Yo
I. Current and drift velocity 5. How can you keep a constant current inside
Y

a conductor ?
Bo

1. Why do the free electrons in a metal wire,


reB

Ans. A constant current can be kept inside a


flowing by themselves, not cause any current
flow in the wire ?
conductor by maintaining a constant potential
ICBSE (D) 2016 (C)l difference across the two ends of the conductor.
uur
ooY

Ans. In a metal wire, the free electrons due to thermal


6. If the temperature of a good conductor
ad

motion move at random. There average velocity


is zero in one direction. Due to it, there is no increases, how does the relaxation time of
dY

current flow in the wire. electrons in the conductor change ?


2. Is electric current a vector or scalar (CBSE 2002)
innd
Re

quantity ? Explain. Ans. With the increase in temperature, the free


Ans. Electric current is a scalar quantity, because it electrons collide more frequently with the
FFi

does not follow the laws of vectors addition, i.e., ions/atoms of conductor, resulting decrease in
the angle between the wires carrying the current relaxation time.
does not affect the total current in the circuit. 7. Two different wires X and Y of same diameter
3. If the electric current is passed through a but different materials are joined in series
nerve, the man is excited, why ? across a battery. If the number density of
Ans. The man is excited due to transfer of electric electrons in X is twice that in Y, And the ratio
energy to his body. of drift velocity of electrons in the two wires.
4. A steady current is flowing in a cylindrical (CBSE 2010)
conductor. Is there any electric field within
the conductor ? Ans. Drift velocity,
Ans. Yes. A steady current can flow in a cylindrical V
I I
d ~ i.e..
conductor if electric force is acting on free nAe n{TiD^IA)e ' n
electrons which makes the electrons to move in
Since the wires are connected in series, current
a particular direction. It is possible due to
electric field within the conductor. / through both is same. Therefore,
CURRENT ELECTRICITY
3/103

n
1 (v) It should have robust construction and
convenient size.
n 2
13. Specific resistance of copper, silver and
8. The connecting wires are of copper. Why ? constantan are (1-78 x 10"^ Q cm, 10"^ Q cm
Ans. The electrical conductivity of copper is next and 48 x 10"^ Q cm) respectively. Which is
only to silver which is costly. Therefore, it the best conductor and why ?
conducts the current without offering much Ans. The best conductor is silver because electrical
resistance. The copper being diamagnetic* conductivity is inversely proportional to the
material does not get magnetised due to resistivity and resistivity of silver is least.
current through it and hence does not disturb 14. Is the formula V = IR true for non ohmic
the current in the circuit. devices also ?

9. Show variation of resistivity of silicon with Ans. Yes, the formula defines resistance R = V// and

w
temperature in a graph. (CBSE 2014) V
not Ohm’s law, for which, — = R = constant.
Ans. The variation of resistivity of silicon with /

temperature is shown graphically in Fig.

Flo
15. Graph showing the variation of current
3.13(d). versus voltage for a material GaAs is shown

e
reee
10. Two conducting wires X and Y of diameter in Fig. 3(Q).48. Identify the region of
ratio 2 : 1 but different materiais are joined

FFr
in series across a battery. If number density
of electrons in X is twice that in Y, find the
ratio of drift velocity of electrons in the two
for
ur
wires.

Ans. In series, the current through the two wires will


kss
besame.So /x = /y or n^.A^ev^ = nyA^ev^
Yo
oo

V
-T
n
V
A n tzD^
or
B

= — X
V
y
n
A^ nDl 2 2^ 8
re

11. How does the random motion of free (i) negative resistance
electrons in a conductor get affected when a (h) where Ohm’s law is obeyed.
ou
ad

potential difference is applied across its (CBSE 2015)


YY

ends ? [CBSE 2014 (C)]


Ans. (/) DE is the region of negative resistance
Ans. When a potential difference is applied across because the slope of curve in this region is
nd

the two ends of a conductor, the random motion


Re

negative.
of free electrons is partially directed towards
(//) BC is the region where Ohm’s law is obeyed
Fi

higher potential side, i.e., positive end of the


conductor. because in this part the current varies linearly
with the voltage.
II. Resistance and effect of temperature 16. A 4 non insulated resistance wire is bent
in the middle by 180° and both the halves are
12. What should be the properties of standard twisted .with each other. What will be its new
resistances ?
resistance ? [CBSE 2010(C)]
Ans. The properties of standard resistances should
be:
Ans. Here, R = —
A
(i) Its value should not change with time.
(//) It should show negligible variation in _ p(//2) _ Ql _R
R' = 4
- = m
resistance with temperature. 2A ~4A~4~4
(Hi) It should have proper capacity for carrying 17. Two wires of equal length one of copper and
current without overheating, other of manganin have the same resistance.
(iv) It should have low inductance. Which wire is thicker ? (CBSE 2012)
3/104 ‘Pxadcep’’A- Fundamental Physics fXinPTSTWl

Ans. R = p //A or A -sc p if / and /? are constant. Since Ans. In an alloy like nichrome (made of nickel and
p is greater for manganin than for copper, hence chromium), the free electron finds a disordered
manganin wire is thicker than copper wire. arrangement of nickel ions and cromium ions.
Due to it, the electron is scattered by them
18. The current flowing through a conductor is
2 mA at 50 V and 3 mA at 60 V. Is it an ohmic randomly and very frequently. As a result of it,
the value of relaxation time of electron decreases
or nonohmic conductor ?
and hence resistivity increases because p l/x. oc

Ans. The resistance of first case.


23. Define the term ‘mobility’ of charge carriers.
V, 50V Write its S.I. uniL (CBSE 2015)

/,1 2x10-3 A - 25,000 n Ans. Mobility of charge carriers (p,), responsible for
currents is defined as the magnitude of drift
The resistance of second case, velocity of charge per unit electric field applied

w
V.
2 _
60 V drift velocity _ V
d -
V
^'d
i.e.y P=
^ = -r 3x10-3 = 20,000 n electric field E VA F/q
h
S.I. unit of mobility = m^ s'* = ms“* N“^C

Flo
As resistance changes with current, so the given
conductor is non-ohmic. 24. What is the resistance of carbon resistor on

e
reee
which the colour of rings in sequence is black,
19. It is easier to start a car engine on a warm
brown, black and gold.

FFr
day than on a chilly day. Why ?
Ans. Value of carbon resistance = 01 x 10*^ ± 5%
Ans. With increase in temperature (/.e., on a warm
= in±5%.
day) the internal resistance of a car battery
for
ur
decreases. Due to it, the battery can supply large 25. What will be the bands of colours in sequence
current which helps in starting the car engine easily. on carbon resistor, if its resistance is
kss
0-1 n± 5%.
20. Differentiate between electrical resistance
Ans. Resistance = 0-1 Q ± 5% = 01 x 10“’ ± 5%.
Yo
and resistivity of a conductor. (CBSE 2020)
oo

Thus, the colour of bands in sequence is black,


Ans. Electrical resistance of a conductor depends
brown and gold. Tolerance of ± 5% is indicated
B

upon the dimensions and nature of the material by gold irng.


of the conductor. Resistivity of a conductor
re

26. What is the colour code for a resistor of


depends upon the nature of material of resistance 3*5 k Q with 5% tolerance ?
conductor but is independent of the dimensions
ou
ad

of the conductor. Ans. Given, resistance = 3-5 k ± 5%


YY

= 35 X 10^ n ± 5%
21. If potential difference V applied across a
conductor is increased by 2 V, how will the Colour code of given resistor is orange,
nd
Re

drift velocity of the electrons change ? green, red and gold.


Ans. Let R be the resistance of conductor. In first case, 27. What is the most probable cause of super
Fi

current in conductor, / = V/R. conductivity ?


In second case current in conductor. Ans. The cause of super-conductivity is that, the free
electrons in super-conductor are no longer
2V-HV 3V independent but become mutually dependent
/' = = 3/
R R and coherent when critical temperature is
reached. The ionic vibrations which could
As, I = nAevj or Vjoc I deflect the free electrons in metals are unable
('.● n, A, e are constant) to deflect this coherent or cooperative cloud of
electrons in super-conductors. That is why, there
V
d _ is no resistance offered by the super-conductor
^d y = y = 3 or -3u^ to the flow of electrons.

28. The given graph shows the variation of


It means drift velocity becomes tripled. resistance of mercury in the temperature
22. Alloys of metals have greater resistivity than range 0 < 7 < 4 K. Name the phenomeon
that of their constituent metals. Why ? shown by the graph. (CBSE 2003)
CURRENT ELECTRICITY 3/105

R(Q) FIGURE 3(Q).49 R^_ 2R = 4 or /?j : /?2 = 4 :1


Rl R/2
0-16--
31. Explain why resistance becomes more in
0'08-- series combination.

Ans. In series combination of resistors, the effective


0 ■T(K)
2 4 6 length of the conductor increases. As /? « /,
therefore, resistance increases.
Ans. Superconductivity. 32. Explain why resistance becomes less in
29. Two square metal plates are of same thick parallel combination.
ness and material. The side of B is twice
Ans. In parallel combination of resistors, the effective
that of A. These are connected in series, Fig. area of cross-section of the conductor increases.

ww
3(Q).50. If the resistance of A and B are As, R « 1/A, therefore, resistance decreases.
denoted by and Rg, find R^^Rg. 33. Join three resistances of 2 H each such that
FIGURES ,Q).S0 the total resistance of the circuit is 3 Q.

Flo
Ans. Two resistances in parallel and the third

e
A B resistance is connected in series with them.

ree
IV. Internal resistance

Fr
rF
Ans. Let I, d length and thickness of square metal and Grouping of cells
uurr
plate. Then
34. What Is the difference between electromotive

^ A Ixd d B ~
_ px2/ p
2lxd ~ d
s for
force and terminal voltage of a cell ? How
are they related with each other.
kks
(RaJ. Board 2011)
Yo
oooo

Ans. Electromotive force is the maximum potential


difference between two electrodes of a cell in
R pid
eB

B
the open circuit and terminal voltage of a cell is
the potential differencebetween two electrodes
III. Grouping of resistance of a cell in closed circuit. In a closed circuit,
r

when cell supplies a current in the circuit, then


ou

30. Two identical slabs of given metal are joined


ad

electromotive force = terminal voltage + voltage


YY

together in two different ways as shown in


Fig. 3(Q).Sl(a) and (6). What is the ratio of drop across the internal resistance of the cell.
the resistances of these two combinations ? 35. What is the internal resistance of a cell due
nndd
Re

to? (Pb. Board 2011)


ICBSE 2010 (C)l
Ans. Internal resistance of a cell depends upon :
Fi

(0 the nature, concentration and temperature of


electrolyte,
(h) the nature of electrodes,
(m) the distance between the electrodes and
(/v) area of the electrodes immersed in the
electrolyte.
36. On increasing the current drawn from a cell,
Ans, Let I and A be the length and area of cross-section the potential difference of its terminals Is
of the each of the two slabs. Then R = p l/A lowered. Why ?
In Fig. 3(Q).50(fl), /?,1 = p2l
Ans. This is due to internal resistance r of the cell.
-2/?
A We know that terminal potential difference
V = e - / r. If I is increased, V will be lowered.
In Fig. 3(Q).50W, = 37. Can the terminal potential difference of a cell
2A~ 2
exceed its e.m/. ?
3/106 Fundamental Physics (XIOESSIS]

Ans. Yes, when cell itself is being charged, because 42. When cells are connected in parallel, what
terminal potential difference, will be the effect on (i) current capacity
V=e-(-/r) = e + /r. (ii) e.m.f of the cells.
38. A (i) series (h) parallel combination of two Ans. (<) Current capacity from the combination of
given resistors is connected one by one across cells is the total current available from the cell.
a cell. In which case will the terminal Current capacity increases for the parallel
potential difference across the cell have combination of cells.
higher value ? [CBSE 2008 (C)] {ii) The effective e.m.f. of the cells of equal e.m.f.
Ans. Terminal potential difference is equal to the in parallel will be equal to e.m.f. of one cell.
voltage drop across the external resistor 43. Is it possible that there is no potential
connected to cell. As resistance of two resistors
difference between the plates of a cell ? If
in series is more than in parallel, so the terminal yes, under what condition ?
potential difference will have higher value in

ww
series combination of two resistors. Ans. When cell is shon circuited, it gets totally
discharged. The potential difference between the
39. The plot of the variation of potential
difference across a combination of three two plates of the cell becomes zero.

FF loo
identical ceils in series, versus current is as 44. A parallel combination of two cells of emfs
shown in Fig. 3(Q).52. What is the emf and and £2, and internal resistances and t2

ee
internal resistance of each cell ? is used to supply current to a load of
resistance R. Write the expression for the

ee r
(CBSE 2016, 2008)

rFrF
Ans. Total emfoi three cells in series (3 £) current through the load in terms of e^, £2,
Tj and (CBSE Sample Paper 2011)
= pot. diff. corresponding to zero current
rur
Ans. Equivalent resistance of two cells in parallel
.-. 3e = 6V or e = ®=2V
3
ffoor r,/-2
ks
r
eq
3(Q).52
YYoou
ookos

Equivalent emf oi two cells in parallel


BBo

E
_ £| ^2 '*'^2^
re

eq
n + 'z
oouur
ad

Current in circuit,
/(A)
Y

e.r,
1'2 +£2/]
The internal resistance of each cell
dndY

/ =
Re

r = ^ =-=2H R + r
eq R +
r.r.
V2 R{r^ + r^) + r^r^
I 1
FFini

max
('i +
40. Write any two factors on which internal
resistance of a cell depends. (CBSE 2010) V. Joule’s law of heating
Ans. Internal resistance of a cell depends on (/) the
nature of electrolyte {ii) the concentration of 45. What is the law that defines heat produced
electrolyte. by an electric current ?
41. The car-battery is of 12 volt. 8 simple cells Ans. Joule law of heating. It states that the amount
connected in series can give 12 volt. But such of heat produced in a conductor when a constant
cells are not used In starting a car ; why ? current flows through it is directly proportional
Ans. To start a car, high current is required which to (0 the square of the current, {ii) the resistance
cannot be obtained from the series combination of the conductor and {Hi) the time for which
of 8 simple cells, because their internal current is passed.
resistance is of the order of 10 iil, while the 46. Name the physical quantity which has its unit
resistance of the car battery is only of the order joule coulomb”^. Is It a scalar or vector
of 0-1 a
quantity ?
3/107
CURRENT ELECTRICITY
-I
Ans. A physical quantity with unit, joule coulomb 52. Write an expression for the heat produced
:= workdone/charge = potential difference. It is when an electric current is passed through
a scalar quantity. it.

47. An ammeter reads a current of 30 A when it is V~


connected across the terminals of a cell of emf Ans. H = Vlt= I'^Rt = —t in energy units (joules).
R
2 V. Neglecting the meter resistance, find the
amount of heat (in calories) in cell in 20 seconds.
or // = Vlt _ l-Rt ^ Vh in heat units (cal.)
Ans. Here, / = 30 A, V = 2 V, t = 20 s. Let r be the J ~ J ~ JR

w
internal resistance of a cell. Then
53. What is the meaning of 1 unit electric energy
e 2-0 in domestic use ?
= —Q
/ 30 15 Ans. 1 unit of electric energy = I kWh. If an electrical
r~rt appliance of power 1000 watt is operated on

e
Heat produced in cell, H = mains for 1 hour, it will consume 1 unit of
J

row
electricity.

re
(30)^x(l/15)x2Q = 285-7 cals 54. How many joules of energy is equivalent to
4-2 1 kWh?

FFllo
F
VI. Electric power and Electric energy Ans. Energy = 1 kWh = 1000 W x (60 x 60) s

u
= 3-6 X 10^ J

ree
48. What do you understand by maximum power
55. What is the no. of kWh in 1 joule ?
rating of a resistor ?

sFr
Ans. As 3-6 X 10^J = 1 kWh,
Ans. Maximum power rating of a resistor is the

kro
maximum power which it can tolerate in the ^ kWh =2-77 X 10-^ kWh
uor
therefore, 1J =
form of heat without being melt.
49. Which has greater resistance : 1 kW electro-
offo 3-6x10
0

56. If the current in the electric bulb changes by


kos
heater or a 100 W filament bulb, both marked 1%, then by what percentage will the power
Y
Yo
for 220 V ? (CBSE 2001) change ?
eerBB
oo

y2 (220)2 Ans. F = /2 /? or log F = 2 log / + log F;


a
rY

Ans. Resistance of heater, Fj = — 1000 Differentiating it, we get


1
AF 2M
u

V2 (220)2 a
+ 0 (●.● F is constant);
Resistance of bulb, R^ = P I
ou

100
Pi
ad
do

% change in power
F, 100 1
nY

= — or Ft = 10 F| AF 2A/ 1
^2 1000 10 ^ * xl00 = xlOO =2 xl00 = 2%
P I 100
nid
Re

Thus resistance of bulb is greater than heater.


57. A current in a circuit having constant
50. Distinguish between kilowatt and kilowatt
FFi

hour. resistance is tripled. How does this affect


the power dissipation ?
Ans. Kilowatt is the unit of power. 1 kilowatt= 1000
watt. Kilowatt hour is the unit of electric energy, Ans. The power dissipated in a circuit is F = / F.
where Thus, F /2, when F is constant.
I kilowatt hour = 1000 watt x 1 hour When I becomes 3 times, F becomes 9 times,
= 1000 watt X 60 X 60 second = 3-6 x 10^ J i.e.. Power dissipation becomes 9 times.
51. A heating element is marked 210 V, 630 W. 58. Nichrome and copper wires of same length and
What is the value of the current drawn by same radius are connected in series. Current
the element when connected to a 210 V dc I is passed through them. Which wire gets
source ? (CBSE 2013) heated up more ? Justify your answer.
Ans. Here, V = 210 V, F = 630 W ; (CBSE 2017)

, F 630 Ans. In series combination current I is same in


Current i - — = 3A
210 nichrome and copper wires.
3/108
^ Fundamental Physics (XII) VOL.I

Kirchhoff’s second rule supports the law of


Heat produced. H = I'^Rt = /2 / or // « p
A conservation of energy.
(as I, I, A, t are constant). Then 65. What is the principle of working of meter-
bridge ?
^Nich _ PNich >1
H PNich ^ Pcu^ Ans. Wheatstone bridge principle.
Cu Pc.
66. What happens to the balance point if the
So HNich >H,Cu
position of the cell and the galvanometer are
Thus nichrome wire will produce more heat. inter-changed in balanced Wheatstone
bridge ?
VII. Aspects of heating effect of current Ans. There will be no deflection in the galvanometer
59. Three bulbs 40 W, 60 W and 100 W are as the condition of balanced bridge will still hold
connected in series to 220 V 4mains. Which good.
bulb will glow brightly ? (H.P. Board 2001) 67. At what position of the jockey on slide wire

ww
Ans. In series combination, the same current flows bridge, the results are most accurate ?
through each bulb. As resistance
Ans. The results are most accurate, when balance

Floo
R =
1
or Roc —. Therefore, the resistance point is obtained at the mid point of bridge wire.
P P In this case R = S. The end resistances of copper

ee
of 40 W bulb is highest for the given three bulbs. strips produce least effect and the bridge is most
Heat produced H = Rt or Hoc R, so maximum sensitive.

reer
rFF
heat is produced in 40 W lamp. As a result of it 68. Why is a meter bridge also called a slide wire
40 W lamp will glow more brightly than the bridge ?
other two bulbs.
uur r
Ans. This is because during the experiment, the
60. What is the composition of materials used in
the fuse wire ?
ffoor
jockey is to be slided over the bridge wire.
69. When is Wheatstone Bridge most sensitive ?
sks
Ans. 63% tin + 37% lead.
YYoo
61. State the characteristics of fuse wire. Ans. Wheatstone Bridge is most sensitive when the
oooko

value of resistances of the four arms of the


Ans. A fuse wire should have (0 high resistivity
bridge P, Q, R and S are of the same magnitude.
eBB

(ii) low melting point and (Hi) of suitable current


rating corresponding to the load in the circuit. 70. Calculate the equivalent resistance between
62. What do you mean by specification of a bulb points A and B of the network shown in
r

Fig. 3(Q).53.
ouur

or other electric appliances ?


ad

Ans. When the specification of a bulb or any electric FIGURE 3(Q).53


Y

appliance is given (say 220 V, 100 W), then its 2n


resistance R and maximum current allowed can A' ■Wv
nd dY

be estimated. In the above example,


Re

1 n 4n

the resistance of bulb, R = vVP = (220)^/100 ion


B
FFini

= 484Q
>vw
and maximum current, /= F/V = 100/220 = 5/11 A 2Q

VIII. Kirchhoff’s laws Ans. The equivalent circuit is shown in Fig. 3(Q),54.
and Wheatstone bridge
63. Are KirchhofT’s rules applicable to both a.c.
and d.c. ?

Ans. Yes, Kirchhoff’s rules are equally applicable to


a.c. as well as d.c. circuits.
64. State the fundamental concepts on which two
Kirchhoff’s rules are based.
Ans. Kirchhoff’s first rule is based on the fact that
the charges are not accumulated at a Junction,
i.e., the law of conservation of charge.
3/109
CURRENT ELECTRICITY

Ans. Normally, the potentiometer wire is made of an


1
As — =— so the given circuit is a balanced alloy such as nichrome or manganin. Because
2 4’ those alloys have high resistivity and low
Wheatstone bridge. The resistance 10 in arm temperature coefficient of resistance.
CD is ineffective in circuit. We have (1 U + 2 Q) 77. The emf of the driving cell used in the main
and (2 Q + 4 Q) parallel resistances. Thus circuit of the potentiometer should be more
equivalent resistance between A and B than the potential difference to be measured.
3x6
Why ? [CBSE 2014 (C)l
R = 2H
eq 3 +6
Ans. If it is not so, then the balance point will not be
obtained on the potentiometer wire.
71. What do you mean by sentiveness of a 78. Some times balance point may not be
Wheatstone bridge ? obtained on the potentiometer wire. Why ?
Ans. A Wheatstone bridge is said to be sensitive if it Ans. It is possible only if the fall of potential across

ww
shows large deflection in the galvanometer the potentiometer wire is less than the potential
when ever a small change in resistance is taking difference to be balanced by the potentiometer
place in resistance arm. wire.

Flo
72. Why is meter bridge method not suitable for 79. Whether electric field inside potentiometer

ee
measuring high resistances ? wire is constant or variable ?
Ans. For measuring high resistance, the resistances

rere
Ans. Electric field intensity £ = V// = a constant.

rFF
of all other arms of the bridge should also be
80. Why do we prefer a potentiometer to
high in order to ensure the sensitivity of the measure emf of a cell rather than a
uur r
bridge. But this will reduce the current through
the galvanometer. As a result of it, the
galvanometer becomes insensitive.
ffoor voltmeter ? [CBSE 2016 (C)]
Ans. At null point, a potentiometer does not draw
sks
any current from the cell whose emf is to be
Yoo
IX. Potentiometer
determined, whereas a voltmeter always draws
oko

some little current. Therefore, emf measured


73. State the principle of working of a
BBo

potentiometer. [CBSE 2009, 2014 (C)J by voltmeter is slightly less than actual value
re

of emf of the cell.


Ans. Fall of potential across any portion of the wire
is directly proportional to the length of that 81. How can you make a potentiometer of given
ouur
ad

portion of the wire provided the wire is of wire length more sensitive using a resistance
YY

uniform area of cross-section and a constant box ?

current is flowing through it. Ans. This can be achieved by using more resistance
nndd

from the resistance box in the potentiometer


Re

74. Why is a potentiometer named as


potentiometer ? wire circuit. Which in turn will decrease the
FFii

Ans. It is so named because it is used to measure


current in the potentiometer wire and
potential difference.
consequently, the potential gradient of wire will
decrease. As a result of it, the sensitivity of the
75. Why should the potentiometer wire be of potentiometer will increase.
uniform cross-section and composition ?
82. Why do we prefer a potentiometer with a

[CBSE 2014 {C)l [CBSE 2014 (C)l


longer bridge wire ?
Ans. The potentiometer wire should be of uniform Ans. A potentiometer with longer bridge wire will
cross-section and composition so that there is have a small potential gradient. Due to it, it
same resistance per unit length throughout. Only becomes more sensitive. Hence it is preferred.
then, potential difference will be proportional 83. What should be the properties of the material
to length of the potentiometer wire, which is the for the selection of potentiometer wire ?
requirement for the principle of potentiometer.
Ans. The material must have high resistivity and low
76. Of which material is a potentiometer wire
temperature coefficient of resistance.
normally made and why ? [CBSE 2011 (C)]
3/110
'P'uuie^'A Fundamental Physics (XII) VOL.i

84. A resistance R is connected across a cell of Ans. z = l(R+r) and V=IR


emf e and internal resistance r. A
potentiometer now measures the potential £ _ R + r = 1+^
difference between the terminals of the cell V ~ R R

is V. Write the expression for r in terms of e, or


r (e \

V and R. or r = —-1 R
(CBSE 2011) R IV' )

SHORT ANSWER QUESTIONS Carrying 2 marks

i. Current and drift velocity 1= n A ev^ or


A Uj = a constant
1. How is the current conducted in metals ? or oc
1
OC
1
or
Explain. A

w
nr-

Ans. Every metal conductor has large number of free Since in the second wire, the radius of wire
electrons which move at random at room

temperature. Their average thermal velocity at


becomes 2 times hence the drift velocity will

Flo
any instant is zero. When a pot. diff. is applied become one fourth. In case of iron wire, the
across the ends of the conductor, an electric field number density of free electrons will be less in

e
comparison to copper wire and hence the drift

ree
is set up in the conductor. Due to it, the free
electrons of the conductor experience force due velocity will be more in comparison to that

FFr
to electric field and drift towards the positive in copper wire.
end of the conductor, causing the electric current 4. If the resistance of our body is so large
urr
(i.e. conduction current) in the conductor. The
direction of conventional current is opposite to
the direction of motion of the free electrons in for (*= 10 kfl) why does one experience a strong
shock when one accidently touches the line
wire, say a 240 volt supply ?
kkss
the conductor.
Ans. When a person accidently touches the line wire
Yo
2. A conductor of length L is connected to a dc
ooo

source of emf V. If this conductor is replaced of a 240 V supply, a current 1 = 240/10000


by another conductor of same material and = 0-024 A = 24 mA, flows through the body of
B

same area of cross-section but of length 3 L, the person. This current interferes with the nerve
re

how will the drift velocity change ? process related to our heart beating, which is
(CBSE 2011) basically electrical in nature, this in turn
ou

Ans. Drift velocity. accounts for a strong shock.


ad
YY

I _V/R _V/(pL/A) 5. There is an impression among many people


V
d ~ (■.■R=pUA) that a person touching a high power line gets
neA neA neA
stuck with the line. Is that true ? Explain.
ndd
Re

V 1
or V or OC — Ans. This impression is misleading. Infact there is
^ npeL L
no special attractive force that keeps a person
Fi

V
d_ ^ 1 V stuck with a high power line while touching that
3L 3
or wire, whereas a current of few milliampere is
^d enough to disorganize our nervous system. As
3. If the current flowing in a copper wire be a result of it, the affected person loses tempo
allowed to flow in another copper wire of rarily his ability to exercise his nervous control
same length but of double the radius, then to get himself free from the high power line.
what will be the effect on the drift velocity of 6. Currents of the order of 0-1 A through the
the electrons. If the same current be allowed human body are fatal. What causes the
to flow in an iron wire of the same thickness, death : heating of the body due to electric
then ?
current or something else ?
Ans. We know that if two wires are in series, there Ans. The cause of death is not heating due to current
will be same current in each wire. The current /
passing through a person. Though he may
through the wire is related with drift velocity receive bums if the currents are too large. The
Vj by the relation
cause of death is (he interference of the external
CURRENT ELECTRICITY 3/111

currents with our highly sensitive nervous Dimensional formula of conductance,


system which is basically electrical in nature, I [A]
which intum affect the heart beating. Beyond a G =- = [M~' L-'^T^A^]
certain point, this interference becomes fatal. R V [ML^T-^A-']
Dimensional formula of conductivity,
II. Resistance and effect of temperature /
a = — = [Af"'Zr2
7. While making a standard resistance, the coil p RA
is made of manganin. The coil is double
folded and is wound over non-conducting = [A/"' L~^T^A^]
frame. Why ? 10. Three materials A, B and C have electrical
Ans. For manganin, the temperature coefficient of conductivities o, 2 c and 2 a respectively.
resistanceis very low. Due to it, the resistance Their number densities of free electrons are
of manganin wire remains almost unchanged 2 n, n and 2 n respectively. For which material

w
with change in temperature. The resistivity of is a average collision time of free electrons
manganin is high. Therefore, for making a maximum ?
standard resistance of the given value, the 2
1

FF loow
ne X
smaller length of wire is needed. It is due to Ans. Conductivity, =—

ee
these facts, the wire of manganin is used for P m

making standard resistance coil. The coil is

Fr
me a
double folded on itself to avoid the inductive Relaxation time, x = X oc —
n
ne~
effect and it is wound over the non-conducting

rer
a 2a 2<j a 2a a
ur
frame in order to avoid the conductive effect
and the leakage of current.
8. The V-I graph for a conductor makes angle
fofr Fo 2n n 2n 2n n n

Thus, Xg > x^ > x^. So average collision time


ks
0 with V-axIs. Here V denotes voltage and I
YYouro
for material B is maximum.
s oo

denotes current. What is the resistance of this


11. Explain, with the help of a graph, the
conductor ?
variation of conductivity with temperature
BBook

Ans. V-I graph for a conductor is a st. line, inclined for a metallic conductor ? (CBSE 2004)
r ee

to voltage axis, according to Ohm’s law. If 0 is Ans. Conductivity of a metallic conductor is


the angle which V-I graph makes with K-axis,
oouru

ne^x
ad

1
then slope of the graph. a = —
Y

P m
I
tan 0 = — or R = = cot 0.
V R tan0
For a metal, the number density of electrons n
Ynd

is fixed. When temperature of metal increases,


Re

9. Define the units of conductance and conduc the amplitude of vibrations of the atoms or ions
FFindi

tivity. Give their dimensional formulae. of the metal conductor increases. As a result of
Ans. The units of conductance and conductivity are it, the collision of electrons with atoms/ions
simen (denoted by S) and simen metre"' become more frequent, consequently, the relaxa
(denoted by S m"') respectively. We know that tion time X decreases.It means, conductivityof
conductance = 1/resistance. a metall ic conductor decreases with the increase
.'. 1 simen is the conductance of a conductor of of temperature, as shown in Fig. 3(Q).55.
resistance I ohm.

I
We know, electrical conductivity, ^
p RA
I
If / = I m, /? = 1 ii, A = 1 m^ then a = 1 S m
Thus 1 S m"' is the electrical conductivity of a
conductor of length 1 m, area of cross-section
1 m^ and resistance 1 £i or conductance IS.
3/112
U Fundamental Physics (XII) PI41WI
12. Explain how electron mobility changes for a Thermistors are used (/) to detect small
good conductor, when (i) the temperature of temperature changes {ii) to safe guard the
the conductor is decreased at constant filament of picture tube of a television set
potential difference and (ii) applied potential against the variation of current (Hi) in
difference is doubled at constant temperature control units of industry (/v) for
temperature. (CBSE 2019) voltage stabilisation and remote sensing.
Ans. Electron mobility in a conductor is given by 16. Thermistors differ from ordinary resistors.
eE Explain.
— t
m ex Ans. A thermistor differs from an ordinary resistance
\^ = —
E E m in the following ways.
(0 When temperature of the conductor (0 The resistivity and hence the resistance of
decreases, the relaxation time t of the electrons thermistor changes very rapidly with change of
temperature.

ww
in the conductor increases, so mobility |i
increases. («) The temperature coefficient of resistivity of
{ii) Mobility |i is independent of applied thermistoris very high.

Floo
potential difference. (Hi) The temperature coefficient of resistivity
13. A wire is drawn into double its length and of thermistor can be both, positive and negative.

ee
half its original cross-section. What will be 17. What are super-conductors ? Write their two
applications.

reer
increase in its (/) resistance and

rFF
{ii) resistivity ? (HP Board 2013) Ans. As the temperature of certain metals and alloys
decreases, their resistance also decreases. When
Ans. = p (2 /)/(A/2) = 4 p //A = 4 /?; so increase in
uur r
resistance = R^-R = AR-R= 3R. Resistivity the temperature reaches a certain critical value
will not change as it is independent of the
ffoor
called critical temperature, the resistance of
material completely disappears, i.e., it becomes
dimensions of the wire but depends upon the
sks
zero. Then the material behaves as a super
nature of the wire and also on the temperature
YYoo
conductor.
ooko

of the wire.
Thus super-conductors are those material
14. Two students A and B were asked to pick a
eBB

resistor of 15 kO from a collection of carbon


conductors whose resistances disappear at
critical temperature. The critical temperature is
resistors. A picked a resistor with bands of different for different materials.
r

colours: brown, green, orange while B choose


ouur

Super conductors are used (j) in power


ad

a resistor with bands black, green, red. Who


transmission {ii) to produce very high speed
picked the correct resistor ? Explain.
Y

computers.
[CBSE 2013 (C)]
18. A carbon fllament has a resistance of 100 A
dY

Ans. For student A, the numbers attached to brown,


Re

at 0°C. What must be the resistance of copper


nnd

green and orange are 1, 5 and 3. So the value of niament placed in series with carbon so that
FFii

resistor is, = 15 x 10^ 0=15 kO. the combination has the same resistance at
For student B, the numbers attached to black, all temperatures ? Temperature coefflcient
green and red are 0,5 and 2 respectively. So the of resistance of carbon = - 0*0007 ®C“^ and
value of resistor is, = 05 x 10^ O = 500 O. that of copper is 0*004
Thus, student A picked up the correct resistor Ans. If /?Q, R, be the resistance of a wire at 0®C and
of 15 kO. t “C then temperature coefficient of resistance
15. What are thermistors ? Explain their use in R.-Rr.0
brief. IS
a =
R.t
0
Ans. A thermistor is a heat sensitive device whose
resistivity changes very rapidly with change of or change in resistance = R^- Rq = Rq at
temperature. The temperature coefficient of Let R be the resistance of copper placed in series
resistivity of a thermistor is very high, which with carbon at 0®C. If the combination has same
may be positive or negative. A thermistor can resistance at all temperatures, then at
have a resistance in the range of 0-1 O to 10^ O temperaturet °C,
CURRENT ELECTRICITY 3/113

increase in resistance of copper = decrease in


resistance of carbon
R
I _ _ ‘1
/. ’'2 2 f3^ 3
Ans. X-T =tX
8
ocu /? t = /?o ^
^2 h '■i 3 4

or 0 004 xRxt= -0007 x 100 x / For wires connected in parallel


0-0007 X100
R = = 17-5 a
or
0004 Vy — V2 or /j /?j —12 ^2 or
h 3
19. A uniform wire is cut into four segments.
24. At 0®C, the resistance of a conductor B isn
Each segment is twice as long as the earlier times that of conductor A. The temperature
segment. If the shortest segment has a coefficients of resistance of A and B are aj
resistance of 2 Q, find the resistance of
and .02 respectively. For the series
original wire. combination of the two conductors find

w
Ans. Let / be the length of shortest segment. Then (a) the resistance at 0®C (b) the temperature
lengths of four segments will be /, 2 /, 4 /, 8 /. coefficient of resistance.

Their corresponding resistances will be R, 2 R, Ans. Let Rq be the resistance of the conductor at O^C.

Flo
4 R and 8 R. Given R = 2Q.. Then resistance of conductor B at 0“C = n Rq.

e
Resistance of the original wire Resistance of conductor A at 0*C,

ree
=/? + 2/? + 4/? + 8/? R,=/?o(l + ai0)

Fr
rF
= 15 15x2 = 30 a Resistance of conductor B at 6°C,

/?2 “ ^ ^
urr
20. Why is it unsafe to turn a light switch on or
o^ while taking bath ?
Ans. While taking a bath if we play with a switch of
light, the water comes in between current wire
s for
Thus the total resistance of the series
combination at 9®C is

R^ = Ri+R2- Rq (1 + aj 0) + n Rq (1 + .02 9)
kks
Yo
and our body. Due to it, the current flows = Rq [(1 +n) + (a,+ n02)9]
oooo

through water to our body, resulting in a shock.


a, +na
eB

21. Lights of a car become dim when the starter 1


2.0 ...(0
= {\ + n)RQ 1 + l +n
is operated. Why ?
Ans. When the motor starter of a car is operated, it Comparing this relation with
r

draws more current from the battery for the


ou
ad

/?, = /?,o[l + a,8]


operation of car. Therefore, the voltage across
YY

We have, resistance of series combination at 0“C


the light bulb is lowered, hence the light of a
car is dimmed. Rso = (1 + n) i?o
ndd

Temperature coefficient of resistance of the


Re

III. Grouping of resistance


Fi

series combination is 0,= l + n


22. To reduce the brightness of a light bulb,
should an auxiliary resistance be connected 25. Fig. 3{Q).56 represents a part of closed
in series with it or in parallel ? circuit. What is the potential difference
Ans. To reduce the brightness of a light bulb, we between points A and B ?
should decrease the current flowing through the
FIGURE 3<Q).56
bulb, which is possible when an auxiliary 3V
2A 2G 1 n
resistance is connected in series with the bulb.
A. » V/Mr I W/A 'B
23. An electric current is passed through a circuit
containing two wires of the same material Ans. Potential difference across 2fl = 2x2 = 4V ;
connected in paralle. If the length and radii Potential difference across 1 = 1 x 2 = 2 V.
of the wires are in the ratio 2/3 and 4/3, then Given the e.m.f. of the cell is 3 V. Let the positive
find the ratio of the currents passing through electrode of cell be + 3 V and -ve electrode of
the wires. cell at 0 V. Therefore potential ofA = 4 + 3 = 7V:
3/114
'Pn^eidte^'A Fundamental Physics (XII)[QSm
Potential of S = - 2 V, as current is flowing Ans. During charging of the battery, the current flows
from negative terminal of cell to point 5, hence from the positive to negative terminal inside the
B is at lower potential. Therefore potential ceils of battery. Then terminal potential
difference between A and 5 = 7- (-2) = 9V. difference is given by V = e + / r ; Hence V
Alternative method must be greater than 12 V, during charging.
According to Kirchhoff’s second rule ; 30. Three identical cells each of emf 2 V and
<^.4 - -3 =2 X 2+2X 1 =6 unknown internal resistance are connected
in parallel. This combination is connected to
or
V^-F^ = 6 + 3 = 9V
a 5 ohm resistor. If the terminal voltage across
IV. Internal resistance the cells is VS volt, what is the internal
resistance of each cell ?
and Grouping of cells
Ans. Effective emf of three cells in parallel, e = 2 V;
26. What is terminal potential difference of a Effective internal resistance of three cells

w
cell ? Can its value be greater than the emf in parallel, (when internal resistance of
of a cell ? Explain. (CBSE 2013) each cell is r) « r/3 ; Total resistance of circuit
Ans. Terminal potential difference of a cell is defined

Flo
r e
as the potential difference between the two = — + R ; Current, / =
3 (r/3) + /?
electrodes of a cell in a closed circuit. The value

ee
of terminal potential difference of a cell is less 2x5
than the emf of a cell, when current is drawn Terminal voltage, V=IR or 1.5 =

Fr
(r/3)+ 5
from the cell {i.e. during discharging of cell).
The value of terminal potential difference of a or r = 5Q

cell becomes greater than the emf of the cell


for 31. A battery of emf e and internal resistance r
ur
during charging of the cell, i.e., when the sends currents and I2, when connected to
positive electrode of the cell is connected to external resistances /?] and R2 respectively.
ks
positive terminal of battery charger and negative Find the emf and internal resistance of the
Yo
electrode of the cell is connected to negative battery.
oo

terminal of battery charger. Ans. In first case, current is


eB

27. A car has a fresh storage battery of emf 12 V


and internal resistance 5*0 x 10“^ G. If the
starter motor draws a current of 90 A, what
h =
^ or e = /, ...(I)
r
ou

is the terminal voltage of the battery when


ad

g
the starter is on ? In second case, current, /., =
^ R^+r
Y

Ans. V=E-Ir= 12-90x(50x 10-2) = 7-5 V or


e = /2 (R2 + r) ...(/o
28. After long use of car, as given in question 27, From (0 and (/7), we have
nd
Re

the internal resistance of the storage battery /| (/?] + r) = /2 (/?2 + f)


increases to 500 Q. What maximum current
Fi

can be drawn from the battery ? Assume the orr(/| -/2) = /2/?2-/] /?| or r =
emf of the battery to remain unchanged. h-h
Ans. The maximum current can be drawn from a
battery by short circuiting it. Then V = 0 and Putting this value of r in equation (/), we get
W= 12/500 = 2-4 X 10-2 A = 24 m A h ^2
£ = /
Now, the battery will become useless for starting 1
/,-/2
the car. The battery must be recharged before
u.se.

/, R^ - ?2 /?! + I2 ^2 ~ ^ ^1
29. In Ques. 28, if the discharged battery is = I
1

charged by an external emf source, is the 7,-/2


terminal voltage of the battery during
charging greater or smaller than its emf
12 V?
^.-^2
CURRENT ELECTRICITY 3/115

32. Under what conditions will the strength of 34. A schematic rheostat is shown in Fig.
current in a wire will be the same for 3(Q).58(a). Connect a battery to it so that it
connections in series and in parallel of n acts as a potential divider. Also show the
identical cells ? output terminals.
Ans. Let e, r be the emf and internal resistance of FIGURE 3(Q).S8
each cell. WTicn n cells be connected in series,
l—●Output ●-iC
then the current in the circuit having external
resistance R is
A X ► R
m
A B
/
s
R+nr o o 'W/Zv—'
r
When n cells are connected in parallel, then the E

ww
current in the circuit is
Ans. The circuit for rheostat used as potential divider
/
E nt is shown in Fig. 3(Q).58(i>). The out put voltage
^ R + rin nR+ r is obtained across A and C. In this case, the

Flo
m nt ^ e U
output voltage is V =

e
As, l, = lp ; so.
— X

R + tir nR + r R + r I

eree
or R + nr=n R + r where / is the length AB of resistance R and -a is

FFr
or R - n) = r (\-n) or R=r the length of resistance tapped for output, e is
the emf of battery used and r is the internal
uurr
33. A cell of emf e and internal resistance r is

orr
resistance of the cell.
connected across a variable resistor R. Plot a
graph showing variation of terminal voltage
V of the cell versus the current I. Using the
sfo
35. For what basic purpose the cells are
connected (i) in series («) in parallel and
kks
Yoo
plot, show how the emf of the cell and its (lu) in mixed grouping ?
oooo

internal resistance can be determined. Ans. The cells are connected (/) in series, to get
(CBSE 2014) maximum voltage, (ii) in parallel, to get
eBB

maximum current and (Hi) in mixed grouping.


Ans. Terminal pot. diff. is, V = e - /r or V = - r / + e.
to get maximum power.
This equation is similar to a straight line
uurr

36. A 10 V cell of negligible internal resistance is


equation y = nvc + c. Hence the graph between
connected in parallel across a battery of emf
ad

V and / is a straight line AB with a negative slope


Yo

200 V and internal resistance 38 Q as shown


{= - r) as shown in Fig. 3(Q).57.
dY

in Fig. 3(Q).59. Find the value of current in


For point A, / = 0, and V = V^, so the circuit (CBSE 2018)
innd

= e = intercept on 7-axis.
Re

FIGURE 3(Q).59
Fi

10 V

38 Q
I-Ayvw
200 V

For point B, V=0,1 = Ig Ans. Effective voltage of the circuit,


V=200- 10 = 190 V ;ff = 38 Q.
0 = e-/Dr or r = —
y 190
Current, I = — = 5 A
= negative of the slope of V-I graph R 38
3/116 '4. Fundamental Physics (XII) LTOWl
37. A 9 V battery is connected in series with a
resistor. The terminal voltage is found to be
8 V. Current through the circuit is measured
as 5 A. What is the internal resistance of the
battery ? (CBSE Sample Paper 2018)
Ans. Here, e = 9V;V'=8V;/=5A;r = ?
e= V + /r

or 9 = 8 + 5 X r
or r = (9 - 8)/5 = 0-2 SI
38. A cell of emf E and internal resistance r is
connected across a variable resistor R. Plot V. Joule’s law of heating
the shape of graphs showing variation of

ww
terminal voltage V with (i) R and (ii) circuit 39. The temperature of the filament of an electric
current I. (CBSE 2020) bulb is 2700'’C when it glows. It is not burnt
up at such a high temperature. Why ?
Ans. When a cell of EMF E internal resistance r is

FF loo
Ans. It is so because the filament of the electric bulb
connected to external resistor R, then the current has high melting point and it remains safe in an

ee
£ atmosphere of inert gases which protect its
in the circuit, I = oxidation.

ee r
/? + r

rFrF
40. Why an electric bulb becomes dim when an
Terminal voltage of cell, V = IR electric heater in parallel circuit is switched
on ? Why dimness decreases after some
rur
ER E
or y =
R+r (1 + rlR)
...(/) ffoor
time ?

Ans. The electric heater have more power and less


ks
resistance than electric bulb. So electric heater
From above relation we note that as R increases,
when put in parallel circuit of glowing electric
YYoou
okso

y also increases.
bulb, it draws higher current. Some current from
When /? = », y = £. Thus the variation between
BBoo

the bulb is diverted into heater. Due to it, bulb


y and R will be a curve as shown in Fig. 3(Q).60. becomes dim. After sometime, heater element
r ee

becomes hot, its resistance increases. Due to it,


FIGURE 3(Q).60 it takes less current. Now the current through
oouur
ad

the bulb increases and its dimness decreases.


41. The two electric bulbs of same power are
Y

connected in parallel circuit. If one bulb is


glowing and Aen another bulb is switched
dnYd
Re

on, then brightness of the first bulb increases,


decreases or remains unchanged. Explain.
FFini

Ans. When two bulbs have same power, then their


resistance is also same. When second bulb
>R
connected in parallel to first bulb is switched
on, the effective resistance of circuit becomes
half and curent in circuit becomes double. Due
As, V=E-I r = -Ir + E ...{ii)
to it, this double current divides itself equally
This relation shows that as I increases, V in two bulbs. So the current in the first bulb
decreases. The relation {ii) is similar to the remains unchanged. Due to it, the brightness of
equation of straight line, y = mx-¥C. Therefore, this bulb remains unchanged.
the graph between V and / is a straight line. Fig. 42. Three identical resistors, each of resistance
3(Q).61, whose slope (= - r) tells the internal R, when connected in series with a d.c. source,
resistance of the cell and intercept is E{i.e., emf dissipate power X. If the resistors are
of the cell) connected in parallel to the same d.c. source,
how much power will be dissipated ?
CURRENT ELECTRICITY 3/117

Ans. Let V be the emf of the d.c. source in volt. R LI


t' =
3R' 3 X 3/?2 = 9 minutes
be the resistance of each resistor. or

^2 /?2
In series, total resistance = R + R + R = 3 R
46. In question no. 44, if the two coils are
y2 connected in parallel, find the time taken to
Power dissipated = — = X or = 3X
3R R boil the water in the kettle.

In parallel, total resistance, R' is given by Ans. When the coils are connected in parallel, the
effective resistance is.
i. = 1 + 1 + 1 = T or /?' = R/3
R'~ R R R~ R R' =
^1^2 _ IR^-yR^ 2
= T«2
/?j + /?2 2/?2 ^
y2 _ y2 _ 3y2

oww
Power dissipated =
R: ~ R!3~ R (2/3)/?2 or t' = 2 minutes
=3x3X=9X 3

43. The rate of Joule heat is given by P = V x /


where V is the potential difference across the VI. Electric power and Electric energy

e
FFrlo
ends of a conductor and I is the current

re
47. Two bulbs are rated (P^, V) and {P2, V). If they
flowing through it. Does this relation hold for are connected (/) in series and (if) in parallel

ree
a conductor that does not obey Ohm’s law ?

F
across a supply V. Find the power dissipated in
Ans. Yes, because rate of Joule heat is equal to work the two combinations in terms of Pj and P2.

rF
done in maintaining the current through a (CBSE 2019)
conductor in a unit time [i.e. P = VI] which is
Ans. Let /?j, R2 be the resistances of the two bulbs of

fsoor
ouur
independent, whether Voc lor not. It means the
power Pj and P2 respectively. Then,
rate of Joule heat will also be applicable to the
skf
devices which may not obey Ohm’s law. y2 y2
/?,= — and R^ = —
ooko
44. Current is allowed to flow in a metallic wire M ^2
Yo
at a constant potential difference. When the
Y
({') When bulbs are connected in series, then total
Bo

wire becomes hot, cold water is poured on resistance is R^ and total power is P, given by
reB

half portion of the wire. By doing so, its other


y2 y2 y2 1 1 ^
portion becomes still more hot. Explain its Pj — Pj + R2 or
reason.
P R, R2
uur
ooY

Ans. When cold water is poured on half portion of


ad

(ii) When bulbs are connected in parallel, then total


hot wire, it gets cooled there. Its temperature resistance is Rp and total power P, is given by
dY

falls and hence its resistance decreases, due to


1 1 1 y2 y2 y2
it, the current in the wire increases. As heat J- = -L + -L or -i—= —+ — orP = P,+P2
innd

produced, H «« fi, therefore, the other portion Rp P, R2 Rp Ri R2


Re

of the wire becomes more hot.


48. A boy has two wires of iron and copper of
FFi

45. An electric kettle has two coils. When one of


equal length and diameter. He first joins the
these is switched on, the water in the kettle two wires in series and passes electric current
boils in 6 minutes. When the other coll is
through this combination which increases
switched on, the water boils in 3 minutes. If gradually with time. After that he joins them
the two coils are connected in series, find the in parallel and repeat the process of passing
time taken to boil the water in the kettle.
the current in this arrangement also. Which
Ans. Heat produced. H = V^t/JR or /?« r so Pj « 6 wire will glow first in each case and why ?
and /?2 3, therefore, R^/R2 = 6/3 = 2 Ans. We know that resistivity of iron is more than
or /?j = 2 /?2 that of copper.
When coils are connected in series, then net In series combination, the same current I flows
resistance, through iron and copper wires.
R' t'
R — P j + R2 = 3 R2 so
Heat produced, H = Rt - I^ — t i.e.,Ho^ p
/?2 3 A
3/118 Fundamental Physics fXlIlIffSTMl

As Piron > Pcu SO > ^cu- Therefore, iron (/) In series connection, the current is same, so
will start glowing first in series combination. power s: fiR, i.e., P R.
In parallel combination, the voltage V' is same SOW bulb will glow more than lOOW bulb.
across iron and copper wires. So (//) fn parallel connection, power (P) = V^/R,
i.e., P oe l/R. Thus lOOW bulb will glow more
^2 than that of SOW bulb.
Heat produced, H = t = i.e., H oc —
R p//A P S2. Two electric bulbs P and Q have their
As Piron ^ Pcu ^cu resistances in the ratio of 1 : 2. They are
connected in series across a battery. Find the
Therefore, copper will start glowing first in
ratio of the power dissipated in these bulbs.
parallel combination of wires.
(CBSE 2018)
49. In an electric kettle, water boils in 10 minutes
after the kettle is switched on. With the same Ans. Here, Rp/Rq = 1/2 ; As the bulbs are connected
in series, current (/) in them is same. Ratio of

ww
supply voltage if the water is to be boiled in
power losses will be
8 minutes, should the length of the heating
element be increased or decreased ? Explain. Fp _ /2 _1
Ans. For a given supply voltage, heal produced in

Flo
Q Q
lime t is

e
VII. Aspects of heating effect of current

reree
V2 v2 V^AI . t
H= — t = i.e., H oc -
53. What is the difference between heating wire

r FF
R p//A p/ I
and fuse wire ?
H t
h Ans. The heating wire must be of high resistivity and
uurr
^2 h
As Hi - H2, so 1 =
foor
^2 h of high melting point. Fuse wire must be of high
resistivity and low melting point. Healing wire
ks s
8 is generally made of nichrome. The fuse wire is
Yoo
or
= — <1 or l2<ii
ooook

/ 10 made from tin-lead alloy (63% tin + 37% lead).


1 1
54. A house hold circuit has a fuse of 5 A rating.
eBB

It means the length of the heating wire should Calculate the maximum number of bulbs of
be decreased. rating 60 W - 220 V each which can be
50. Two bulbs of resistances 40 Q and 400 O are connected in this house hold circuit
rr
ouu

in series In a circuit fed with supply current (CBSE Sample Paper 2003)
ad

Which one will glow more ? If one of these Ans. Current drawn by one bulb.
YY

bulbs be switched off, will the light in the 60


room increase or decrease ?
= Aa
V 220 11
nndd
Re

Ans. Since the bulbs are connected in series, each will No. of bulbs which can be safely u.sed with 5 A
carry the same current. We know that light of a
Fii

55
fuse = —
bulb is directly proportional to the heal produced — = 18-33
3/11 3
by it and the heat produced, H=fi Ri, i.e., H o<

R. Therefore 4(X) .Q bulb will glow more. When Thus, 18 bulbs can be used safely with 5 A fuse.
one of these bulbs is switched off, the effective
VIII. Kirchhoff*s laws
resistance of the circuit decreases and hence the
current in the circuit increases. Since H oc /2 aiid Wheatstone bridge
therefore, the light in the room will increase. 55. Kirchhoff’s first rule obeys law of
51. Two electric bulbs of SOW and lOOW are conservation of charge. Explain.
given. When they are (/) connected in series Ans. According to Kirchhoff’s first rule, the current
({/) connected in parallel, which bulb will (/]) entering a given junction of the circuit in a
glow more ? certain lime {t) = current leaving the junction
Ans. Resistance of a bulb R = V^/P or R o^HP. Thus during the same time (r). So ; /, / = I2 f or
the resistance of SOW bulb is more than that of qi = q2, i.e., charge entering the junction is equal
lOOW bulb. As glow « power of a bulb. to the charge leaving that junction.
CURRENT ELECTRICITY 3/119

56. What is the end error in meter bridge ? How 60. Following circuit was set up in a meter bridge
do you remove it ? [CBSE 2017 (C)l experiment to determine the value of X of an
Ans. The end error in the meter bridge is due to the unknown resistance.
following reasons ;
FIGURE 3(Q).62
(0 The zero mark of the scale provided along R X
the bridge wire may not start from the position
where the bridge wire leaves the copper strip
● ● ♦

D
and 100 cm mark of the scale may not be at
position, where the bridge wire just touches the
other copper strip.
A C
(ii) The resistances of the connecting wires and B

copper strips of meter bridge have not been


taken into account.

w
The end error can be removed by repeating the
experiment by interchanging the known and h !■

unknown resistances and taking the mean of the

Flo
resistances determined.
(a) Write the formula to be used for finding

e
57. Why are the connecting resistors in a metre

ree
X from the observations.
bridge made of thick copper strips ? (6) If the resistance R is increased, what will

Fr
rF
(CBSE 2013) happen to balancing length ?
urr
Ans. The thick copper strips are used for connecting (CBSE Sample Paper 2018)
resistors as they offer minimum resistance and
hence avoid error due to end resistance. The
same (i.e., error due to end resistance) has not
Ans. {a)
s R

X
for I

(100-/)
or X =
R (100-/)
I
kks
been taken into account in the bridge formula. {b) When R is increased, then / will have to be
Yo
ooo

58. Why is the meter bridge method considered increased to get the balance point on meter
unsuitable for the measurement of very low bridge. Therefore, the jockey point B is to be
eB

resistances ? .shifted towards end C of meter bridge.


Ans. While measuring a very low resistance using
IX. Potentiometer
r

meter bridge, all other resistances used should


ou
ad

have low values in order to ensure sensitivity 61. What is potential gradient ? How is it
YY

of the bridge. This requires the galvanometer measured ? Explain.


should also be of very low resistance and a very
sensitive. Then the end resistances and Ans. The potential gradient is defined as the fall
ndd

of potential per unit length of the


Re

resistances of the connecting wires used in metre


potentiometer wire. Complete the circuit as
bridge become comparable with the resistance
Fi

to be measured. This would introduce error in shown in Fig. 3(Q).63 and note the current
from ammeter A.
the result. Due to it, the measurement will not
be accurate.
FIGURE 3(Q).63
59. What are the advantages of a Wheatstone E

bridge method of measuring resistance over I- i- ^ R.B.

other methods ? K

Ans. Following are the advantages of Wheatstone


I..

bridge method over other methods :


(0 It is a null method. Therefore, the result is
s B
A B
free from the effect of cell resistance of the
circuit. Let L = length of potentiometer wire AB,
{ii) in null method, it is easier to detect a small e = emf of the battery used in the potentiometer
change in deflection in galvanometer than to wire circuit, R — resistance from resistance box
read a deflection in galvanometer correctly. used in circuit, r = resistance of the wire AB.
3/120 ‘Pn<xdeeft. 4. Fundamental Physics (XII)
Total resistance of the circuit = R + r Ans. (0 When length of the wire is doubled, the
g potential gradient across the potentiometer wire
Current in potentiometer wire, i = will decrease. Due to it, the position of zero
R + r
deflection will occur at longer length, (n) The
Fall of potential across potentiometer wire reverse will be true when length is halved.
= ir =
e
r
65. If the current flowing in the wire of the
R + r potentiometer be decreased, what will be
effect on the position of zero deflection in
fall of potential
.●. Potential gradient = potentiometer ? Explain.
length
Ans. If the current in the wire of potentiometer is
/' e r decreased, the potential gradient will decrease
and hence the position of zero deflection will
R+r) L
occur at longer length.

w
62. Can you express the potential gradient in
66. How can you make a potentiometer of a given
terms of specific resistance of the wire ? If
length more sensitive by using a resistance
yes, find the relation.

Flo
box ?
Ans. Yes. Potential gradient.
Ans. The sensitivity of a potentiometer is the smallest

ee
K =
potential difference applied (V) potential difference it can measure. It can be
length of wire(/) increased by reducing the potential gradient, i.e.,

Fr
potential drop per unit length of potentiometer
V ^ jlR) _ I wire. The same is possible by decreasing the
i.e.. K=-
= -(p//A) = ^A
for current flowing in the potentiometer wire, which
I /.
ur
where p is the specific resistance of the material in turn can be achievedby using more resistance
of the wire. from resistance box in series with the
s
potentiometer wire.
ok
63. In a potentiometer experiment, if the area of
Yo
cross-section of the wire increases uniformly 67. Fig. 3(Q).65 shows use of potentiometer to
Bo

measure the internal resistance of a cell.


from one end to another, draw a graph
showing how potential gradient would vary FIGURE 3(Q).65
re

as the length of the wire increases from one E r


end. [CBSE 2014 (C)] I—wwv
ou
ad

Ans. Potential gradient.


Y

1
or X«« —
l l l A A A
nd

B
Re

A
As per question A increases with /, so, the
variation of K versus I when A increases from
Fi

E'
one end to another is of the type as shown in
F
Fig. 3(Q).64. i;k

FIGURE 3(Q).64
U/s/W^
R
K

(/) When the key K is open, how does the


balance point change, if the current from the
driver cell decreases ?
(«) When the key K is closed, how does the
balance point change if R is increased,
keeping the current from the driver cell
64. If the length of the wire be (0 doubled and constant ?
(») halved, what will be effect on the position Ans. (0 When the current from the driver cell
of zero deflection in a potentiometer ? decreases in potentiometer wire, the potential
Explain. gradient across the wire decreases. Due to it.
CURRENT ELECTRICITY 3/121

the balance point is obtained on longer length Ans. Total resistance of potentiometer circuit
of potentiometer wire,
RS
(n) When key K is closed and the value of R is ~R^ + R +S
increased, the lesser current will be drawn from
the cell S'. Terminal voltage of cell E' will Current in the circuit, I =
e

increase, therefore, the balance point will be Rq+RS/(R + S)


obtained on longer length of potentiometerwire.
Potential difference across the wire AB
68. The variation of potential difference V with
length / in case of two potentiometersX and e

K is as shown in the Fig. 3(Q).66. Which one


V = IR^ = xR. 0
Rq + RS/(R + S)
of these two will you prefer for comparing
emfs of the two cells and why ? (CBSE 2006) Potential gradient, K = —
V

Hj RS \
L R^ +

w
R +S^
The emf of the test cell must be less than the
total potential drop (V) across the wire AB. The

Flo
positive terminal of test cell must be connected

e
to end A of the wire.

rree
70. The following circuit shows the use of

r FF
potentiometer to measure the internal
resistance of a cell:
uurr
Ans. A potentiometer is said to be sensitive if fall of
potential per unit length, i.e., potential gradient
(dV/dl) is small. For given length /,
for
(i) When the key K is open, how does the
balance point change, if the current from the
driver cell increases ?
kss
(u) When the key K is closed, how does the
ooook

(dV^ fdV^
Yo
< balance point change if R is decreased,
[dl Jy V dl A keeping the current from the driver cell
eB

constant ?
Therefore, potentiometer Y will be preferred
for comparing emfs of the two cells. FIGURE 3(Q).68
rr

69. A potentiometer wire has a length L and a


ou
ad

resistance Rq. It is connected to a battery and


Y

a parallel resistance combination of R and S


dY

as shown in Fig. 3(Q).67. Find an exprssion A'


J
B
for the potential gradient of the potentio
Re
innd

meter wire. e'


Fi

A
FIGURE 3(Q).67 K

rVA-i e
K
-WsA"
R R

Ans. (/) When the key K is open and current from the
s driver cell increases, the potential gradient
across the potentiometer wire increases. Due to
A B
it, the balance point shifts towards the end A of
potentiometer wire.
What is the maximum emf of a Hest cell’ for
(//) When the key K is closed and R is decreased,
which one can get a balance point on this keeping the current from the driver cell constant,
potentiometer wire ? What precaution should the terminal potential difference of the cell
one take, while connecting this test cell in the decreases. Due to it, the balance point shifts
circuit. [CBSE 2004 (C)l towards the end A of potentiometer wire.
3/122 '4. Fundamental Physics (XII)
71. A potentiometer wire of length 1 m has a VR^l
resistance of 10 £i. It is connected to a 6 V EMF of the primary cell, e -
iRa+R)^^
battery in series with a resistance of 5 fl.
Determine the emf of the primary cell which 6x10x40
= 1*6 V
gives a balance point at 40 cm. (CBSE 2014) (10 + 5)xl00
Ans. Here, L = 1 m = 100 cm ; = 10 Q, V= 6 V, 72. In a potentiometer arrangement for
R = 5Q.,l = 40 cm, 8 = ? determining the emf of a cell, the balance
V point of the cell in open circuit is 350 cm.
Current in potentiometer wire, / = When a resistance of 9 H is used in the
Rq + R external circuit of the cell, the balance point

oww
Potential difference across the potentiometer shifts to 300 cm. Determine the internal
resistance of the cell. (CBSE 2018)
wire
= //?,= Ans. Here, /j - 350 cm ; /? = 9 12 ; /^ = 300 cm
R^^R ^ Internal resistance.

ee
I 350-300
Potential gradient, R = R = x9 =i'sn

FFrlo
r =

{Rq + R)L h 300

r
rF
ee
SHORT ANSWER QUESTIONS Carrying 3 marks

rF
ouru 6. Define resistance of a conductor. What is its cause ?
1. What do you understand by electric current ?
Define its S.l. unit and mention the direction of Explain the factors on which the resistance of a

fosor
electric current in the circuit. Also explain if current
[Art. 3.5]
conductor depends. [Arts. 3.11 and 12]
is a scalar or vector quantity.
s kf
7. Explain current density, conductance and
2. What is drift velocity ? Establish a relation
ooko
electrical conductivity. State their SI units.
between current and drift velocity.
Yo
[Art. 3.13]
Y
(CBSE 2016 S, Uttarakhand Board 2012,
8. {a) Define the term conductivity of a metallic
Bo

MP Board 2011, Jharkhand 2011)


reeB

wire. Write its SI unit.


[Art. 3.7, 3.9]
{b) Using the concept of free electrons in a
3. State Ohm’s law and deduce it from the
conductor, derive the expression for the
ooY

knowledge of drift velocity of tree electrons in


uur

conductivity of a wire in terms of number density


a conductor carrying current. [Arts. 3.10]
ad

and relaxation time. Hence obtain the relation


4. What do you understand by resistance of a between current density and the applied electric
dY

conductor ? Define its SI unit. Show that resis- field E. (CBSE 2018) [Art. 3.13]
nidn

ml
tance of a conductor is given by /? = 9. Define the term ‘resistivity’ and ‘conductivity’
Re

ne^xA and state their SI units. Draw a graph showing


where the symbols have their usual meanings. the variation of resistivity with temperature for a
FFi

[Art. 3.11] typical semiconductor.


5. Define drift velocity and show that the resistivity [Arts. 3.12 and 3.13, Fig. 3.13(</)]
of the material of a wire, is inversely 10. Show, on a plot, variation of resistivity of (/) a
proportional to the relaxation lime for the free conductor and (//) a typical semiconductor as a
electrons in the metal. (CBSE 2016 S) function of temperature.
ml
Hint. Resistance of a conductor, R = Using the expression for the resistivity in terms
ne~xA of number density and relaxation time between
the collisions, explain how resistivity in the case
Also, R=Pl- of a conductor increases while it decreases in a
A
semiconductor, with the irse of temperature.
9I ml m
(CBSE 2019)
So, or p = 2
A ne^xA ne'-x
Ans, A graph showing the variation of resistivity ip)
Hence, p «= 1/x and temperature (7) for a conductor and typical
semiconductor is shown in the Fig. 3(Q).69.
CURRENT ELECTRICITY 3/123

15. What do you understand by internal resistance


and terminal potential difference of a cell ? On
what factors do they depend ? [ Art. 3.24]
16. Establish a relation between E.M.F. and potential
difference of a cell.
(HP Board 2011) [Art 3.24]
17. Two cells of emfs ej and 62 and internal
resistances rj and r2 respectively are connected
in parallel. Obtain expressions for the equivalent
(i) internal resistance and (//) emf of the
combination. [CBSK 2018 (C)] [Art. 3.26(*)J
18. State Joule’s law for heating effect of electric
current. What is the cause of it ? Is it reversible
or not ? [Art. 3.30]

w
The expression for the resistivity is
19. Explain electric power. State its different relations
m and define SI unit of electric power. [Art. 331]
P =

Flo
ne^ X 20. Explain electric energy. Give the various relations
of electric energy and define the commercial unit

reee
where n is the number density of free electrons of electric energy. [Art. 3.32]
and T is the average relaxation time of free 21. State the two Kirchhoff’s rules used in electric

FFr
electrons. networks. How are these rules justified ?
In metal conductors, n is not dependent on (CBSE2015) [Art. 335]
temperature upto an appreciable extent, but the
for
22. State Wheatstone bridge principle and deduce it
ur
value of T decreases with the irse in temperature. using Kirchhoff’s rules.
kkss
1 (Uttarakhand Board 2012, Karnataka Board
As p« - , so p increases with the rise in 2012, Raj. Board 2011) [Art. 336]
Yo
ooo

X
23. State and prove Wheat stone bridge principle.
temperature. Discuss the determination of unknown
eB

For semiconductor, n increases with temperature temperature with its help.


and r decreases. But increase in n is much more (Tamil Nadu Board 2012, Pb. Board 201!)
r

than decrease in r. Therefore, p decreases with [Art. 3.37]


ou
ad

rise in temperature. 24. Draw a circuit diagram of a meter bridge used to


YY

11. What are ohmic and non-ohmic conductors ? determine the unknown resistance /? of a given
Explain the super-conductivity. wire. Hence derive the expression for R in terms
nndd
Re

[Arts. 3.17, 3.19] of the known resistance S.

12. Define the term ‘temperature coefficient of resis [CBSE (OD) 2017 (C)] [Art. 3.38]
Fi

tivity’. Write its S.I. unit. Plot a graph showing 25. Explain the principle of potentiometer. How is a
the variation of resistivity of copper with temp- potentiometer superior to that of a voltmeter ?
erature. [Arts. 3.15 and Fig. 3.13(a)] (MP Board 2011) [Arts. 3.39 and 3.44]
13. Write the mathematical relation for the resistivity 26. Discuss the principle of potentiometer and
of a material in terms of relaxation time, number explain the determination of potential difference
density and mass and charge of charge carriers across a conductor using a potentiometer.
in it. [Arts. 3.39, 3.40]

Explain, using this relation, why the resistivity 27. State the principle of potentiometer. With the help
of the metal increases and that of a semi of circuit diagram, describe a method to find the
conductor decreases with rise in temperature. internal resistance of a primary cell.
(CBSE 2007) [Art. 3.16] (CBSE 2017 (C), Uttarakhand Board 2012,
KhJ. Board 2012, Pb. Board 2011)
14. Explain colour code for carbon resistors with
illustrations. [Art. 3.21] [Arts. 3.39, 3.42]
3/124 '«*. Fundamental Physics (XII) W«1MI
28. State the principle of potentiometer. Draw a Ans. The galvanometer pointer will show one sided
circuit diagram used to compare the emfs of two
primary cells. Write the formula used. How can deflection if the negative terminal of cell ej is
the sensitivity of a potentiometer be increased. connected to terminal X on potentiometer, where
(Kerala Board 2012)
positive terminal of battery e is connected. In this
[Arts. 3.39, 3.41 and 3.43] case, the current in the galvanometer will be in
29. The diagram shows a potentiometer set up. On the same direction, whether jockey touches the
touching the jockey near to the end X of the potentiometer wire at X or at F.
potentiometer wire, the galvanometer pointer In this situation, the battery e and cell £j will be
deflects to left. On touching the jockey near to in series through potentiometer wire. Then
end Y of the potentiometer, the galvanometer current in galvanometer is
pointer again deflects to left but now by a larger
amount. Identify the fault in the circuit and e + e
I
explain, using appropriate equations or otherwise, / = i- = X0
G+/?

ww
how it leads to such a one-sided deflection.
(CBSE Sample Paper 2018} where G is the resistance of galvanometer and R
is the resistance of potentiometer wire in circuit,
FIGURE 3(Q).70
with reference to jockey point and 0 is the

Flo
E
deflection in the galvanometer.

ee
When jockey touches the potentiometer wire near

rere
the point X, the value of R in the circuit is

rFF
N
maximum, so current / is minimum. Due to it,
X[J 0Y the deflection 0 in galvanometer is small. When
uurr
foor jockey touches the potentiometer wire near the
point y, the value of R in the circuit is minimum.
ks s
Due to it, the deflection 0 is large.
Yoo
oook
eBB

LONG ANSWER QUESTIONS Carrying 5 or more marks


rr

1. Define relaxation time of the free electrons


ouu
ad

drifting in a conductor. Establish a relation FIGURE 3(Q).71

between drift velocity and time of relaxation. Use


YY

E1./-1
this relation to deduce the expression for the
electrical resistivity of the material.
nndd
Re

(Karnataka Board 2012 ; CBSE 2012) A B


Fii

[Arts. 3.7 and 3.12]


2. Derive an expression for drift velocity of ^2- '2
electrons in a conductor. Hence deduce ohm’s
law. (CBSE 2017) [Arts. 3,7 and 3.10]
(0 equivalent emf of the combination
3, (a) Explain the term drift velocity of electrons in
(ii) equivalent internal resistance of the
a conductor. Hence obtain the expression for the combination
current through a conductor in terms of drift
velocity. [Arts. 3.7 and 3.9] (Hi) potential difference between points A and B.
(b) Two cells of emfs E^ and £2 and internal (CBSE Sample Paper 2022-23) [Art. 3.26 (6)j
resistances rj and r2 respectively are connected 4. Derive the expression for the current density of a
in parallel as shown in Fig. 3(Q).71. Deduce the conductor in terms of the conductivity and applied
expression for the electric field. Explain with reason how the mobility
of electrons in a conductor changes when the
CURRENT ELECTRICITY 3/125

condition in terms of the resistances of four arms


potential difference applied is doubled, keeping the
temperature of the conductor constant. of Wheatstone bridge.
[CBSE 2017 (C)] [Arts. 3.13 and 3.8] (CBSE 2013 ; Pb. Board 2011)
5. Define resistivity of the material. State its SI unit [Arts. 3.35 and 3.36]
and discuss its variation with temperature in case 15. State the working principle of a potentiometer.
of (0 metals (I'O semiconductors and With the help of the circuit diagram, explain how
(in) insulators. [Arts. 3.12 and 3.16] a potentiometer is used to compare the emf’s of
6. Find the total resistance when the various resis two primary cells. Obtain the required expression
tors are connected (t) in series and (ii) in parallel. used for comparing the emfs.
(Pb. Board 2011) [Arts. 3.22 and 23] Write two possible causes for one sided

oww
7. What is the e.m..f of a cell ? State the factors on deflection in a potentiometer experiment.
which its value depends. Derive a relation (CBSE 2013) [Arts. 3.39 and 3.41]
between e.m.f. £, terminal potential difference 16. Write the principle of potentiometer. How will
V, internal resistance r of a cell and external
resistance R. Prove that e.m.f. is more than
you compare emfs of two cells using a potentio
meter. Explain with a circuit diagram. Write two

ee
potential difference. [Arts. 3.3 and 3.24]
precautions to be taken during this experiment.

FFrlo
8. Discuss the grouping of two unidentical cells in (Tamil Nadu Board 2012, Pb. Board 2011.

r
(i) series and (») parallel and find their equivalent CBSE 2010) [Arts. 3.39 and 3.41]

rF
emf and internal resistance. [Art. 3.26]

ee
17. (a) Obtain the condition under which the current
9. Explain the various types of grouping of identical
cells and find the condition for the maximum flowing in the current detecting device used in
ouru
rF
current in the external resistor connected to the
the circuit shown in Fig. 3(Q).72, becomes zero.
combination of cells (li) in series (ii) in parallel [Art. 3.36]
and (Hi) in mixed grouping. [Art. 3.27]
ffosor (b) Describe briefly the device, based on the
os k
10. What do you understand by heating effect of above condition. Draw a circuit diagram for this
current ? Explain its cause and find the relation device and discuss, in brief, how it is used for
ook
finding as unknown resistance.
YYo
for the heat produced in a conductor for electric
current. [Art. 3.30] [CBSE 2016 (C)]
Bo

11. Explain electric power, electric energy and define


reeB

FIGURE 3(Q).72
their units and give their relations.
P Q
[Art. 3.31, 3.32]
oouY

-VW ■vw
ur

12. Discuss atleasl 3 important aspects of heating


ad

effect of current. [Art. 3.33] —^ Current


detecting
13. (fl) State the two Kirchhoff s rules used in the
dY

nr device
analysis of electric circuit and explain them.
nidn

[Art. 3.35] A/sAr


Re

X R
(b) Derive the equation of the balanced state in a
FFi

Wheatstone bridge using Kirchhoff’s rules.


(CBSE Sample Paper 2022-23) [Art. 3.36] K I (●>
14. State Kirchhoff's rules for an electric network.
[Art. 3.38]
Using Kirchhoff’s rules obtain the balance

CASE-BASED VERY SHORT/SHORT QUESTIONS

CASE 1. The resistance of a conductor is the obstruction offered by it to the flow of current through it. The
resistance of a conductor depends upon the dimensions (length and area of cross-section) and nature of the material
of the conductor. The resistance of a conductor varies with the temperature of the conductor. With the increase in
temperature, the resistance of a metal conductors (alloys) increases. The resistance of semiconductors decreases.
The resistivity of a conductor depends upon the nature of material of the conductor. Its value is low for the
conductors, moderate for semiconductors and very high for insulators.
The temperature coefficient of resistivity is very low for manganin than for copper aluminium.
3/126
Fundamental Physics (XII) VOL.I

Read the above paragraph carefully and FIGURE 3(Q).74


ansvter the following very short and short
answer questions : a b c

1. What is the resistance of a conductor if its length I


and area of cross-section are doubled ?
2. Why is the resistance of copper wire low and of T e R1
R2
manganin wire high, when both the wires are of
d

w
same dimensions ?
r
e
3. Why are the manganin wires, used in making the
standard resistances ?
7 R3
4. The voltage-current variation of two metallic

e
wires X and Y at constant temperature are shown
in Fig. 3(Q).73. Assuming that the wires have

re
row
8 /
the same length and same diameter, explain which
of the two wires have larger resistivity ? CASK 3. A conductor of length lO cm, cross-
sectional area I (turn)- having number of electrons per

eeF
ullo
unit volume 10^‘^ m~^ and of resistivity 6 x Q m is

FF
connected to the battery of emf 6 V. The electrons in
the conductor give up energy at the rate of R per
second to the thermal energy, where / is the current and

srr
roF
R is the resistance of circuit as shown in Fig. 3(Q).75.

k
FIGURE 3(Q).7S
uor
ofof R
kos
Y
I- .
Yo
' I
eerBB
oo
rY

CASE 2. An experiment was set up with the circuit


diagram shown in Fig. 3(Q).74. Given. R^ = 10 Q, 6V

7?2 = /?3 = 5 Q, r = 0 Q and £ = 5 V,


u

Read the above paragraph carefully and


Read the above paragraph carefully and answer the following very short and short
ou
d
o
ad

answer the following very short and short answer questions :


nY

answer questions : 9. What Is the resistance in the circuit ?


5. Locate the points in the circuit, which are at the 10. What is the drift velocity of electrons in the
nid

same potential ?
Re

conductor ?

6. What is the current through branch h}> ? II. How much energy is absorbed by electrons per
FFi

7. What is the power dissipated in R^ ? second from the initial state of no current (ignore
thermal motion) to the suite of drift velocity ?
8. What is the total current supplied by cell in the
circuit ? 12. What is the total kinetic energy of all the electrons
while drifting in the conductor ?
ANSWERS
1. The resistance of a conductor of length /, area of It means, the resistance of the conductor
cross-section A and resistivity p is given by remains unchanged.
2. The resistivity of the material of copper wire is
A low and of manganin wire is high, />.,
Pcu</^ mangumn'
When I' = 2 I and A' = 2 A, then

We know, R = 2LA \so R <x. p (when A and / are


A' (2A) A constant). So < R
cu
manganin*
CURRENT ELECTRICITY 3/127

3. The temperature coefficient of resistivity of


manganin is very low. Consequently, the Current in the circuit, / = —
e
i = lA
manganin wire shows a negligible variation of
resistance with change in temperature. Due to it,
9. Resistance of conductor
the resistance of manganin wire remains almost
same, with change in temperature.
4. In the given graph, the slope of the /-V line for a R = ^
p/ (6xi0-5)x(0-l) = 6ft
wire represents the conductance of the wire. From A (10-^)
graph, slope of I-V line for wire Y > slope of I-V
line for wire X.
Conductance of wire 7> Conductance of wire X. 10. Current in the circuit, / = — = — = IA
R 6

As, resistance = Drift velocity,


Conductance
/ 1

ww
.●. Resistance of wire Y < resistance of wire X V
(I ~ -t9
n Al (10^^)x(10~^)x(l-6xI0 )
pyl pj. / .
or
— Py<Px
A A

FF loo
10-^
le., wire X has larger resistivity than wire Y. = 0-625 X 10-^ ms -1
1-6
5. In the given circuit, the points a, h, c and / are at

ee
the same potential. Also the points/, g, j and h 11. Energy absorbed by electrons per second

er
are at other same potential. = power loss = R= l^x6 = 6Js
-1
= 6 W

rFreF
6. Current through branch bg of the circuit 12. No. of electrons in the conductor = nA I
= current through resistance R^
rur
KE of all the electrons in the conductor, when

~ R.~ H) 2^
ffoor
electrons are moving with drift velocity
ks
1
I
~mv ,xnAl
2 A I
YYoou
okso

( I 2
7. Power dissipated in R^=I^ R^ = — xIO
2J
BBoo

= 2-5 W -31 -4
)x —xlO
re

-(9-1X10
8. Total resistance of the circuit 2 1-6

/?jX(/?2 + ^) 10x{5+5)
oouur
ad

/?. =
I
+ r = + 0 xl029x(10-6)x0-l
/?j + ( /?2 + /?^) 10 + (5+5)
Y

= 2 X 10-*^ J
= 5Q
dnYd
Re
FFini

CASE-BASED MCQs AND ASSERTION-REASON QUESTIONS

CASE I. Electrochemical cell is a device in which If a cell of emf e and internal resistance r is
the chemical energy is converted into electrical energy. connected to external resistance R, then current in the
Due to chemical reaction taking place in the cell, a circuit is

difference of potential is maintained between the two


electrodes of a cell. As a result of it, the current can be / =—^ or e = I(R + r)-IR+Ir=V + Ir
drawn from the cell. R + r

EMF (£) of a cell is maximum potential difterence where V is the terminal potential difference.
between the two electrodes of the cell, when the cell is When current / is drawn from the cell. Thus
in the open circuit.
Terminal potential difference (V') of a cell is z-V e-V (e-V \ R
r = [V V = IR]
defined as the potential difference between two / V/R [ V
electrodes of a cell when cell is in the clo.sed circuit.
3/128
“Pnauiecfr \ Fundamental Physics (XII)
Based on the above paragraph, answer
FIGURE 3(Q),76
questions no. 1 to 4 :
'●l 6l
1. Terminal potential difference of a cell as
V/A—I
compared to emf of the cell is always
(a) lesser
A B
^2 62
(b) greater ■Vs/W
(c) depends upon the condition of circuit for the
cell
Based on the above paragraph, answer
{d) none of the above questions no. 5 to 8 :
2. The emf of a battery is 12 0 V and internal 5. The equivalent emf of this combination is
resistance is 3 Q. Its potential difference is

ww
measured by a voltmeter of resistance 1000 Q. (a) (b)
The percentage error in the reading of emf shown 'i+^2
by voltmeter is ^2 ^1 ^1 ^2

Floo
(C) (d)e^~e2
(a) 0-2% ib) 0-3% rj+T2

ee
(c) 0-4% (d) 0-5%
(CBSE 2022)

reer
For Question No. 3 and 4, we have given two 6. For terminal B to be negative

rFF
statements each, one labelled as Assertion (A)
(a)ei T2>£2 n (6)£j T2<£2 r^
and other labelled as Reason (R).
uur r
(c)£j rj >62 t-2 (^0«2''2 = «1 ''i
Choose the correct option out of the four options
given below:
ffoor (CBSE 2022)
sks
(a) Both A and R are true and R is the correct For Question No. 7 and 8, we have given two
YYoo
statements each, one labelled as Assertion (A)
ooko

explanation of A.
and other labelled as Reason (R).
(b) Both A and R are true and R is not the correct
eBB

explanation of A. Choose the correct option out of the four options


given below:
(c) A is true but R is false,
r

(a) Both A and R are true and R is the correct


ouur

(d) A is false and R is also false.


ad

explanation of A.
3. Assertion. The emf of the cell does not depend
Y

(b) Both A and R are true and R is not the correct


upon the size and shape of the cell. explanation of A.
nddY

Reason. The emf of the cell depends upon the


Re

(c) A is true but R is false.


nature of electrodes and the concentration of the
(d) A is false and R is also false.
FFiin

electrolyte.
7. Assertion. For the given circuit, the current I is
4. Assertion. An external resistance 990 Q is
connected across a cell of emf 5 V and internal
resistance 10 Q. The terminal potential difference n+'2
across the cell is 4-95 V.
Reason. In internal circuit, two cells are in series,
Reason. V=Ir, where V is the terminal potential so total emf=£ j + £2 and total internal resistance
difference across the cell of internal resistance r
is (ri + r2).
and I is the current drawn from the cell.
8. Assertion. If £[ = £2 = 1-5 V, r, = r2 = 1 Q and
CASE 2. A battery is a combination of two or the cells are connected in parallel to the external
resistance of R = 7 Q, then total resistance of the
more cells. In the following figures, a single battery is
circuit is 9 Q.
represented in which two cells of emf and £2 and
internal resistance rj and r2 respectively are connected. Reason. Total resistance = + r2 + R.
CURRENT ELECTRICITY 3/129

ANSWERS

l.(c) l.ib) 3.(b) 4.(c) 5.(c) 6.(1?) 7. (a) S. (d)

HINTS/EXPLANATIONS For Difficult Questions

1. The terminal potential difference of a cell is less Let and Vj^ be the potentials at C and D
than emf of a cell when current is drawn from respectively and V be the potential difference
the cell and is greater than the emf of the cell if between C and D.
the cell is charged by external battery. Here, potential difference across the terminals
2. Here, e = 12-0 V;r= 3-0 Q of first cell is equal to the potential difference
across the terminals of second less. So, for the
Resistance of voltmeter, /? = 1000 Q
first cell

ww
12-0 V + e
1
Cumnt in the circuit, / = 1000 + 3 y=Vc-VD=-£i+/i or /j =
= 11-964 X 10"^ A
For the second cell,

Flo
Potential difference across voltmeter
y=//? = (11-964 X 10-^) X 1000= 11-964 V= V^~Vf}=£2~^2 '’2 ^2 “
'●2

ee
e-V Putting values in (1), we get

rere
% error = xlOO

rFF
-vfl + 1]
y
I =
V + e, _
("12-11-964 X100 = 0-3% '●2 '■1 1^2 nJ In '’2J
uurr
11-964
3. Both Assertion and Reason are true and
/
foor -V f!ilA
n ''2;
ks s
Reason is not the correct explanation of
Yoo
Assertion. or
ri r2/ = €2 ri-Sir2-V(ri + r2)
oook

4. Here. /?= 990 Q. fi = 5 V. r = 10 Q ^2 n ^1 ^2 n


or V = -/ ...(2)
eBB

/ =
e 5
= 5 X 10-3 A
n+nz In ■"'2;
R+r 990 + 10 If the combination of cells is replaced by a single
rr

Terminal potential difference across the cell is cell between C and D of emf £g^ and internal
ouu
ad

resistance Fig. 3(Q).78(b), then


V = /R = (5x 10-3) 990 = 4.950 V y=£-_-/r„ ...(3)
YY

Thus Assertion is true but Reason is false. eq eq

5. Refer to Fig. 3{Q).77(a), at junction C,


nndd

^eq =
Re

FIGURE 3(Q).77
Fii

Thus, option (c) is true.


^1 ei
6. The terminal B to be negative, the direction of
VWV-M
current should be as shown in Fig. 3(Q).78(fe). It
I
4 c D will be so if £, T2 < £2 '"i- option (b) is
A B true.
'■2
total emf
7. Current -
h
o total internal resistance rj +
Thus both Assertion and Reason are true and
Reason is the correct explanation of Assertion.
●4- 8. Both Assertion and Reason are wrong. As total
A B
1x1
r,r2 + 7
o resistance will be = + R
1 + 1
r^ + r^
...(1) -7‘SQ
/ + /| —/2 or /—(/2 —/j)
3/130
Fundamental Physics (XII) VOL.l

TYPE I. ELECTRIC CURRENT TYPE II. DRIFT VELOCITY


AND ELECTRON MOBILITY
1. In Bohr model of hydrogen atom, the electron
revolves around the nucleus in a circular orbit of 8. A conductor of length L is connected to a d.c.
radius 5*1 x 10~” m at a frequency of 6*8 x 10 source of emfe. The drift velocity of electron is
revolutions per second. Find the equivalent Vj. If the length of conductor is tripled by
current at any point on the orbit of the electron. stretched it, keeping e constant, find its drift
[Ans. 1-088 X 10"^ A] velocity. (CBSIC 2015) [Ans. d^/3]
2. A solution of sodium chloride discharges 9. The number of free electrons per 5 cm of ordinary
6-Ox 10^^Na'*'ions and4-5 x lO'^Cr* ions in 2 copper wire is 2 x 10^^. The average drift speed

ww
of electrons is 0-25 mm/s. What is the current
seconds. What is the current passing through the
solution ? flowing ? [Ans. 1-6 A]
[Ans. 8-4 mA]
3. In hydrogen atom, the electron moves in an orbit 10. What is the drift velocity of electrons in a silver
wire of length 1 m, having cross-sectional area

Flo
of radius
-1
5-0 x 10"*^ m with a speed of 2-2 x 10^ 314 X 10^ m^ and carrying a current of 10 A.

ee
ms
. Find the equivalent current. Electronic
charge = 1-6 x 10“*^ C. Given atomic weight of silver = 108, density
[Ans. 1-12 mA]
of silver 10-5 x 10^ kg/m^, charge of electron

rere
rFF
4. If 0-6 mole of eletrons flow through a wire in 50 1-6 X 10”’^ C, Avogadro’s number
minutes. What is (a) the total charge that passes = 6 023 X 10^^ per kg. atom.
through the wire, and (b) the magnitude of the
uur r
[Ans. 3-4 X 10“^ms“^]
current ? Avogadro’s number = 6 x lO^ per mol.
[Ans. (fl) 5-76 x 10^* C (b) 19-2 A]
ffoor 11. Protons in cosmic-rays strike the earth’s upper
sks
atmosphere at a rate, average over the earth’s
5. The charge flowing through a conductor varies surface of 10 protons m“^ s“‘. What does the total
Yoo
with time as ^ = 8r-3/2 + 5r3
oko

current the earth receive from beyond its


Find (/) the initial current (//) time after which atmosphere in the form of incident cosmic ray
BBo

the current reaches a maximum value (Hi) the protons ? The earth’s radius is 6-4 x 10^ m.
re

maximum or minimum value of current. [Ans. 0-82 mA]


[Ans. (i) 8 A (iV) 0-2 s (in) 7-4 A] 12. A uniform copper wire of length I m and cross
ouur
ad

6. Fig. 3(Q).78 shows a plot of current I (in ampere) sectional area 5x10"^ m^ carries a current of
through the cross-section of the wire over a time 1 A. Assuming that there are 8 x 10^^ free
YY

interval of 14 s. Find the amount of charge that electrons per m^ in copper, how long will an
electron take to drift from one end of the wire to
nndd
Re

1(A) RGURE 3(Q).78 the other. Charge on an electron = 1-6 x 10"'® C.


[Ans. 6-4 X 10^ s]
FFii

13. The number density of electrons in copper is


A
8-5 X 10^® m"^. Find the current flowing through a
5
copper wire of length 0-2 m, area of cross-section
1 mm^, when connected to a battery of 4 V. Given
that electron mobility = 4-5 X 10"^
E
4
m^V"' s"' and charge on electron is l-6x 10"'®C.
[Ans. 1-22 A]
passes through the wire (/) during 10 s (ii) during 14. Assuming that there is one free electron per atom
14 s. [Ans. (0 40 C (ii) 50 C] in copper. Density of copper is 8-9 x 10^ kg m“^
7. An electric current of 20 )iA appears in a discharge and atomic weight is 63-5. Avogadro’s number
tube. If the discharge current is due to flow of equal is 6-02 X 10^ per kg-atom. If the current flowing
number of electrons and protons, then how many through the copper wire is 1 -5 A and its diameter
electrons flow across a cross-section of the tube in is 1 -2 mm, find the drift velocity of electron.
2 minutes. [Ans. 7-5 X 10'^] [Ans. 10“^ ms"^]
CURRENT ELECTRICITY 3/131

23. Two wires A and B of the same material have


TYPE III. OHM’S LAW, their lengths in the ratio 5 : 3 and diameter in the
RESISTANCE, RESISTIVIT Y ratio 2:3. If the resistance of wire A is 15 Q, find
the resistance of wire B. [Ans. 4 O]
15. An arc lamp operates at 80 V, 10 A. Suggest a
method to use it with a 240 V d.c. source. 24. A wire of 15 Q resistance is gradually stretched
Calculate the value of the electric component
to double its original lengtli. It is then cut into
two equal parts. These parts are then connected
required for this purpose.
in parallel across a 3-0 volt battery. Find the
[Ans. Resistor of value 16 £2]
current drawn from the battery.
16. A wire 50 cm long and 0-12 mm diameter has a (CBSE 2009) [Ans. 0-2 A]
resistance of 4-0 O. Find the resistance of another
25. Find the time of relaxation between collision and
wire of the same material whose length is 1 -5 m
and diameter is 0-15 mm. [Ans. 1-92 £2] mean free path of electrons in copper at room
17. A uniform wire of length / and radius r has temperature. Given resistivity of copper
resistance 100 £2. It is recast into a thin wire of = 1 -7 X 10“^ n m ; number density of electron in
copper = 8-5 x 10^^ m"^.

w
(/) length 2 / (/i) radius r/2. Calculate the
resistance of new wire in each case. Charge on electron = 1-6 x 10“'^ C,
[Ans. (/) 400 Q («) 1600 Q] mass of electron = 9-1 x 10”^* kg.

Flo
Drift velocity of free electron= 1-6 x 10“^ ms
-1
18. There are two wires of copper and iron of thd
same length but different radii. When equal [Ans. 2-5 X lO-*-* s ; 4*0 x lO'** m]

e
rree
potential difference is applied between the ends
of each wire, the same current flows in them. TYPE IV. CURRENT DENSITY.

r FF
What the ratio of their radii. Specific resistances CONDUCTANCE AND CONDUCTIVITY
of copper-and iron are 1-6 x 10“* £2m and 10 x
uurr
10”^ £2m respectively. [Ans. 2/5]
19. A wire of mass 10 g, radius I mm is compressed
to its length by 10%. Calculate the percentage
for
26. A rheostat has 1000 turns of a wire of radius
04 mm, having resistivity 49 x 10"* £2 m. The
diameter of each turn is 4 cm. What are the
kss
change in its resistance. [Ans. 19%] maximum values of conductance and
ooook
Yo
20. A wire is stretched to increase its length by 5%. conductivity of rheostat wire.
Calculate percentage change in its resistance. [Ans. 8-2 x 10"^ S ; 2-04 x lO** S mr^]
eB

(CBSE 2007) [Ans. 10-25%]


27. Current flows through a constriction conductor
21. A hollow cylinder of length / and of ardii a and b as shown in Fig. 3(Q).80. The diameter and
is filled with a material of resistivity p and is current density to the left of constriction are
rr

connected to a battery of cmf E through an 2-0 mm and 1-3 x 10^ Am"^. (/) How much
ou
ad

ammeter, as shown in Fig. 3(Q).79. Find the current flows through the constriction ? (i7) If
Y

current through ammeter. the current density is tripled as it emerges from


dY

FIGURE 3(Q).79 the right side of the constriction, what is the


Re

diameter of the irght hand side of constriction ?


innd
Fi

FIGURE 3(Q).80

D
u

M [Ans. (i) 4-09 A ; (li) 1-15 mm]


28. A wire carries a current of 2 0 A, when a potential
Ans.
E2kI difference of 3 0 V is applied across it. What is
its conductance? If the wire is of length 3 m and
plog^ (b/a) area of cross-section 5 4 x 10"^ m^, calculate its
22. Calculate the mass of copper required to draw a conductivity. [Ans. 0-67 S ; 3-7 X 10^ Sm"*]
wire 5 km long having resistance of 5 £2. The
29. Calculate the radius of the wire of conductance
density of copper is 8 9 x 10^ kg m"^ and
resistivity of copper is 1-7 x 10"* £2m. 10 £2"’ and length 10 cm whose electrical
[Ans. 756-5 kg] conductivity is 10^ Sm"'. [Ans. 1-78 mm]
3/132 ’a Fundamental Physics (XII) PTSTMl
TYPEV. VARIATION OF TYPEVn. COMBINATION OF RESIS
RESISTANCE WITH TEMPERATURE TANCES IN SERIES AND PARALLEL

38. A wire of uniform cross-section and length / has


30. The resistance of a tungsten filament at 160°C is
13212. What will be its resistance at 400“C ? The
a resistance of 16 Q. It is cut into four equal parts.
Each part is stretched uniformly to length / and
temperature coefficient of resistance of tungsten all the four stretched parts are connected in
is 4-5 X 10-3 °C-^ [Ans, 21512] parallel, calculate the total resistance of the
31. The temperature coefficient of a resistance wire combination so formed. Assume that stretching
is 0-00125‘’C*^. At 300 K its resistance is 112. At of wire does not cause any change in the density
of its material. [Ans. 1612]
what temperature the resistance of wire will be
2f2? [Ans. 1127 K] 39. Two wires X, Y have the same resistivity, but their

ooww
32. A metal wire of diameter 2 mm and length 50 cm cross-sectional areas are in the ratio 2:3 and lengths
has a resistance 0-31 12 at 25®C and 0-51 £2 at in the ratio 1:2. They are first connected in series
and then in parallel to a d.c. source. Find out the
125*C. Find (/) the temperature coefficient of ratio of drift speeds of the electrons in the two wires
resistance (ii). resistance at 0®C and for the two cases. (CBSE 2008) [Ans. 3:2; 2:1]
{Hi) resistivity at 0°C and 25°C.

e
40. What is the equivalent resistance between A and
[Ans. (i) 6-45 x lO"^ (h) 0-267 £2,

ere
H of resistances as shown in Fig. 3(Q).81.
(m) 1-68 X 10-« £2m, 1-948 x 10"* £2m]

rFl FIGURE 3(Q).81

Fre
33. {a) At what temperature would the resistance of B C D
A -●-AVAV

rrF
a copper conductor be double of its value at 0°C ? 2A 0-8A 04A ^3
{b) Does this same temperature hold for all a
R3 R2

sffoo
ouur
copper conductors, regardless of the size and VT o

shape ? Temperature coefficient of resistance of


copper is 4-0 x 10“3/°C.
kosk
H G F E

[Ans. (a) 250“C (6) yes] [Ans. 10 Q]


YYo
oo

41. Calculate the value of the resistance R in the


TYPE VI. COLOUR CODES
BB

OF CARBON RESISTORS
circuit shown in the Fig. 3(Q).82, so that the
current in the circuit is 0-2A. What would be the
potential difference between points A and B ?
rre

34. The temperature coefficient of resistivity for two


materials A and B are 0 003 PC"* and
ouu
Y
ad

0-0068®C"^ respectively. Two resistorsand/?2


made from materials A and B, respectively, have
dY

resistances of 200 £2 and 100 £2 at 0°C. Show on


a diagram, the colour code of a carbon resistance,
innd
Re

that would have a resistance equal to the series


combination of /?j and /?2 at a temperature of
FFi

100®C. (Neglect the ring corresponding to the


tolerance of the carbon resistor).
(CBSE 2012) [Ans. 5 £2,1 volt]
[CBSE 2016 (C)] [Ans. yellow, orange, black]
42. The reading of an ideal ammeter, in the circuit
35. A carbon resistance of 4-7 k £2 is to be marked
shown in Fig. 3(Q).83, equals :
with strips or bands of different colours for its
identification. Write the sequence of colours. FIGURE 3(Q).83

[Ans. yellow, violet and red]


36. A current of 5 mA is passed through a colour K2
R
coded carbon resistor with first, second and third
rings of black, brown and red. What is the voltage
AAAr ■—(a)—VW“
X
drop across the resistor ? [Ans. 0-5 V]
+
37. What is the colour of the third band of a coded
1- \ /I
resistor of resistance 0-34 £2 ? [Ans. silver] e Ki
CURRENT ELECTRICITY 3/133

(01 when key Ky is closed and key K2 is open FIGURE 3(Q).86


(ii) 7/2 when both keys Ky and K2 are closed. <V>
Find the expression for the resistance of X in
terms of the resistances of R and S. 12 V.2n

RS
Ans.
[CBSE 2016 (C)] R-S, “I
I'’

43. Find the equivalent resistance of the circuit 4fi


■VvW
shown in Fig. 3(Q).84, between the points A
andB.
(CBSE 2016) [Ans. 8 V, yes]
46. A cell of emf e and internal resistance r is
connected across a variable load resistance R. It
is found that when 7? = 4 fli, the current is 1A and

ww
when R is increased to 9 Q, the current reduces
to 0-5 A. Find the values of the emf e and internal
resistance r. (CBSE 2015) [Ans. 5 V, 1 ft]

Flo
47. A voltmeter of resistance 990 is connected
across a cell of emf 3 V and internal resistance

e
reree
3 Q. Find the terminal potential difference of the
cell and percentage error in the reading of

r FF
voltmeter. [Ans. 2-99 V, 0-33%]
48. A battery of emf e, and internal resistance r, gives
uurr
foor
[Ans. (4/3) 11]
a current of 0-5 A with an external resistor of
12 and a current of 0-25 A with an external
kss
TYPE VIII. EMF, TERMINAL P.D. AND resistor of 25 12. Calculate (i) internal resistance
Yoo
of the cell and (ii) emf of the cell.
ooook

INTERNAL RESISTANCE OF A CELL


(CBSE 2002, 2013 S) [Ans. 1 C2, 6-5 V]
44. The reading on a high resistance voltmeter, when
eBB

49. 4 cells each of identical emf £, internal resistance


a cell is connected across it, is 2-2 V. When the
r, are connected in series to a variable resistor.
terminals of the cell are connected to a resistance
Fig. 3(Q).87 shows the variation of terminal
rr

of 5 12 as shown in Fig. 3(Q).85, the voltmeter


voltage of the combination with the current
ouu
ad

reading drops to 1-8 V. Find the internal resistance


of the cell. output:
Y
dY

FIGURE 3(Q).B5
Re
innd
Fi

AAA/V (>
R = sn K

10
(CBSE 2010) Ans. —12 (a) What is the emf of each cell used ?
9
(b) For what current from the cells, does
45. A battery of emf 12 V and internal resistance 212 maximum power dissipation occur in the circuit,
is connected to a 4 12 resistor as shown in Fig. (c) Calculate the internal resistance of each cell.
3(Q).86. Show that a voltmeter when placed [CBSE 2006 (C)]
across the cell and across the resistor, in turn gives
[Ans. (fl) 1-4 V (*) 1 A (c) 0-712]
the same reading.

t
3/134
Fundamental Physics fXinwsTMl
50. The following graph shows the variation of
terminal potential difference V, across a TYPE IX. GROUPING OF CELLS
combination of three cells in series to a resistor,
versus the current I. 54. In the two electric circuit shown in Fig.3(Q).91,
determine the readings of ideal ammeter (/^) and
FIGURE 3(Q),88 ideal voltmeter (V).
FIGURE 3(Q).91

<Y>
6V

1 n

(A)

ww
I (ampere)
9V
(a) Calculate the emf of each cell. I-VAV
1 Q
(b) For what current /, will the power dissipation

FF loo
of the circuit be maximum. (CBSE 2008)
o
[Ans. (a) 2 V (6) 1 A]

ee
51. The galvanometer G in the circuit, Fig. 3(Q).89 <s>

ee r
reads zero. Find the value of R. Batteries are

rFrF
6V
resistance less. h-WW
1 n
rur
FIGURE 3(Q).89
500Q
-wwv
G ffoor
ks
9V
12V
YYoou
R ●>—H -WvV
ookos

1 n
2V
1
BBo

O
re

[Ans. 100 il] ICnSE 2015 (C)l


52. A voltmeter with resistance 500 O is used to [Ans. (a) 7-5 V, 1*5 A (6) - 1-5 V, 7-5 A]
oouur
ad

measure the emf of a cell of internal resistance 55. A cell of emf 1-1 V and internal resistance
Y

4 Q. What will be the percentage error in the 0-5 Q is connected to a wire of resistance 0-5 Q.
reading of the voltmeter. Another cell of the same emf is connected in
[Ans. 0-8 %]
series but the current in the wire remains the
dndY
Re

53. In the circuit shown in Fig. 3(Q).90, when switch same. Find the internal resistance of second cell.
5| is closed and ^2 is open, the ideal voltmeter
FFini

shows a reading 18 V. When switch ^2 is closed [Ans. 1 £2]


and is open, the reading of the voltmeter is
56. A set of 4 cells each of emf 2 V and internal
24 V. What will be the reading of voltmeter when resistance 1 -5 £2 are connected across an external
S^ and S2, both are closed ? load of 10 £2 with 2 rows, two cells in each row.
FIGURE 3(Q).90 Calculate the current in each row and potential
difference across 10 £2.
[Ans. 0-175 A, 0-175 A, 3-5 V]
V2 57. Find the minimum number of cells required to
E,r
produce an electric current of 1 -5 A through a
■IC’
resistance of 30 £2. Given that the emf and internal
60 120
>
resistance of each cell are 1-5 V and 10 £2
i I respectively. [Ans. 120]
58. n identical cells are joined in series with two cells
[Ans. 14-4 V] A and B with reverse polarities. EMF of each cell

ft
CURRENT ELECTRICITY 3/135

is Eand internal resistance r. What is the potential drawn by heater and {b) the current passing thro
difference across the cell A or B ? ugh the fuse line. [Ans. (a) (50/11) A {b) 5 A]
f 2 67. A series battery of 10 lead accumulators each of
Ans.2E 1-- em/2 V and internal resistor 0-25 ohm is charged
ny
by a 220 V d.c. mains. To limit the charging
59. A set of 6 cells, each of emf 2 V and internal current, a resistance of 47-5 is used in series in
resistance 1 -5 are connected across an external
the charging circuit. What is (a) the power
load of 10 with 2 rows, 3 cells in each branch. supplied by the mains and (b) power dissipated
Calculate current in each branch and potential as heat ? (c) Account for the difference of power
difference across 10
in (a) and (b).
[Ans. 0-245 A, 0-245 A, 4-9 V] [Ans. (a) 880 W (b) 800 W (c) 80 W stored
60. A student connects a cell, of emf £2 and internal in battery as chemical energy]
resistance r2> with a cell of emf £^ and internal
resistance Tj, such that their combination has a TYFK XI. EI.KCTRIC ENERiJY

ww
net internal resistance less than r|. This AND HKATINd EFFhXT OF (TIRKKN'I’
combination is then connected across a resistance
R. Draw a circuit of the ‘set up’ and obtain an 68. A dry cell of emf 1 -6 V and internal resistance of

Flo
expression for the current flowing through the 0-10 Q is connected to a resistor of resistance
resistance /?. (CRSF 2016) R Q. If the current drawn from the cell is 2 A,

ee
then (/) What is the voltage drop across R ?

rere
Ans.
Ejrj + E^r,
2'! (») What is the rate of energy dissipation in the

rFF
resistor ? [Ans. (/) 1-4 V («) 2-8 W]
ir^xr2)+ Rir^ + r^)
69. Two wires A and B of same material and mass,
uurr
have their lengths in the ratio 1 :3. On connecting
TYPE X. ELECTRIC POWER

61. A generator is supplying power to a factory by


foor them, one at a time to the same source of emf,
the rate of heat dissipated in B is 10 W. What is
ks s
cables of resistance 20 Q. If the generator is the rate of heat dissipated in A ? [Ans. 90 W]
Yoo
generating 50 kW power at 5000 V, what is the
oook

70. An electric bulb is marked to 100 W, 230 V. If


power received by factory ? [Ans. 48 kW] the supply voltage drops to 115 V, what is the
eBB

62. Calculate the amount of heat produced per second heat and light energy produced by the bulb in 20
(in calories) when a bulb of 100 W, 220 V glows, minutes. Calculate the current flowing through
assuming that only 20% of electric energy is it. (NCKRT) [Ans. 30 kj, (5/23) A]
rr

converted into light. 7 = 4-21 cal“’. 71. An electric kettle used to prepare tea, takes 2
ouu
ad

(Hr. Board 2001) [Ans. 19-05 cal] minutes to boil 4 cups of water (1 cup contains
YY

63. An electric motor operates on a 50 V supply and 200 c.c. of water), and the room temperature is
draws a current of 15 A. If the motor yields a 25°C. (<2) If the cost of electricity is Rs. 300 per
nndd

mechanical power of 150 W, estimate the power unit of electricity, calculate the cost of boiling 4
Re

dissipated across its windings. Also find the cups of water, {b) What will be the corresponding
Fii

efficiency of the motor ? [Ans. 600 W, 20%] cost if the room temperature drops to 5®C ?
64. Find the resistance of 240 V-200 watt electric [Ans. (a) 21 paise {b) 27 paisej
bulb when glowing. If this resistance is 10 times 72. Find the heat developed per minute in each of the
the resistance at 0®C and the temperature of the three resistors /?j, /?2 shown in Fig. 3(Q).92.
glowing filament is 2000®C, then find the
FIGURE 3(Q).92
temperature coefficient of resistance of the
filament. [Ans. 288 ; 4-5 x lO"^ “C'^] I 1 Ri=6n

65. Three equal resistances connected in series across


a source of e.m.f. consume 20 watt. If the same
^2 R2=30
resistors are connected in parallel across the same
source of e.m.f., what would be the power I I
dissipated ? [Ans. 180 watt] 12 V R3=in
66. In a house having 220 V line, the following appli h ■VvVW
ances are working (i) a 60 W bulb («) a lOCK) W
heater (Hi) a 40 W radio. Calculate (a) the current [Ans. 640 J; 1280 J; 960 J]
3/136 '4, Fundamental Physics (XII) orSTWI
73. A motor operating at 110 V draws a current of
TYPE XII. KIRCHHOFF’S LAWS
2A. If the heat is developed in the motor at the
rate of 10 cal s"^ what is its efficiency ?
[Ans. 80-9%] 80. Find the potential difference across each cell and
the rate of energy dissipated in R, Fig. 3{Q).94.
74. A room is lighted by 200 W, 124 V incandescent
lamps fed by a generator whose output voltage (CBSE Sample Paper 2011)
is 130 V. The conducting wires from the generator
to the user are made of aluminium wire of total Ans. 11-75 w"
length 150 m and cross-sectional area 15 mm^. 7 7

How many such lamps can be installed ? What is


FIGURE 3(Q).94
the total power consumed by the user ? sp.
resistance of aluminium is 2-9 x 10"* m. 6i = 12 V
[Ans. 12, 2-4 kW] -]l—vw-
r-I =2fi

ww
75. Twenty one electric bulbs are connected in series
with the mains of a 220 V supply. After one bulb R = 4H
is fused, the remaining 20 bulbs are again
connected in series across the same mains. By

Flo
what percentage will the illumination of (0 a bulb £2 = 6 V
change (») all the bulbs change ?

e
-1—vw
[Ans. (0 10-25% («) 5%]

rree
r2 = 1 n
76. Three 60 W, 120 V light bulbs are connected

r FF
across a 120 V power line as shown. Fig. 3(Q).93. 81. Using Kirchhoff’s rules, calculate the potential
difference between B and D in the circuit as
Find (a) the voltage across each bulb
uurr
shown in the Fig. 3(Q).95. [CBSE 2018 (C)]
(b) the total power developed across three bulbs.

FIGURE 3(Q).93
for
kss
A
ooook
Yo
eB

B C
rr
ou
ad

[Ans. (a) = 80 V ; Vj= Vc= 40 V ;


Y
dY

(ft) 40 W]
Ans. — V
77. A house wiring, supplied with a 220 V supply
Re

13
innd

line is protected by a 9 ampere fuse. Find the


maximum number of 60 W bulbs in parallel that 82. Apply KirchhofTs rules to the loops PRSP and
Fi

can be turned on. [Ans. 33] PRQP to write the expressions for the currents
78. (a) Two bulbs rated 25 W - 220 V and /], I2 and in the circuit shown in Fig. 3(Q).96.
100 W - 220V are connected in series to a 440 V
supply. Show with necessary calculations which
bulb if any will fuse ?
(ft) What should happen if the two bulbs were
connected in parallel to the same supply ?
[Ans. (a) 25 W bulb will fuse
(ft) Both the bulbs will fuse]
79. A cell sends a current through a resistance /?j for
time t, next the same cell sends current through
another resistance R2 for the same lime t. If the
same amount of heat is developed in both the
resistances, then find the internal resistance of 39 11
(CBSE 2010) Ans. A; — A;—A
the cell.
[Ans. yjR^] 860 215 172
CURRENT ELECTRICITY 3/137

83. In the circuit shown in Fig. 3(Q).97 the galvano 87. Twelve identical wires each of resistance 6 Q are
meter G shows zero deflection. If the batteries A arranged to form a skeleton cube. A current of
40 m A is led into cube at one comer and out at
and B have negligible internal resistance find the
value of the resistor R. the diagonally opposite corner. Calculate the
potential difference developed across these
FIGURE 3(Q).97 corners and the effective resistance of the

soon
network. [Ans. 0-2 V ; 5 £2]
88. In the circuit Fig 3(Q).100. E, F, G and H are
cells of emf 1, 2, 2 and 4 volt and their internal
12V T R T 2V resistances are 1, 2, 2 and 1 respectively.
Calculate (/) the potential difference between D
and B, (li) the potential difference across the
terminals and each of the cells G and H.

ooww
(CBSE F 2013) [Ans. 100 £2]
84. Find the currents /j, I2 and I2 through the three
resistorsof the circuit as shown in Fig. 3{Q).98.
FIGURE 3(Q).98

e
2V 2V 2V

re
A B C
h D

rFFl
ree
I2 ’’I3

F
sn sn 5Q

rF
H h E

fsfoor
G F
ouur
2V 2 V 2V [Ans. (0 0*29 V (k) 3-14 V ; 3*43 V]
89. In the given circuit Fig. 3(Q). 101, assuming point
kosk
[Ans. Zero in each resistor] A to be at zero potential, use Kirchhoff’srules to
85. Eleven identical wires each of resistance 5 £2 are
determine the potential at point B.
Yo
joined to form the edges of an incomplete cube.
oo

(CBSE 2011) [Ans. 2V]


Y

Find the total resistance from one end of vacant


BB

FIGURE 3(Q).101
edge to the other end of the same cube. If a battery 4V
rre

of emf 2 V is connected across the vacant edge 1A B

of cube, find the current supplied by the battery. 3A


oYuu

[Ans. 7 £2,1/7 A]
ad

86. Using Kirchhoff’s rules, write the expressions for R 2£2 Rl


dY

the currents /j, I2 and in the circuit diagram


shown, Fig. 3(Q).99. (CBSE 2010)
innd
Re

1 H
FIGURE 3(Q).99 A C 3A
2V
Ei =2V ri =40
FFi

I1
I \VSA*“ 90. Find the potential difference across in the
circuit shown in Fig. 3(Q).102.
FIGURE 3(Q).102
h E2=''V r2 = 30 El Rl
■VW/- ■VAW-i

R3
A ■VAAAr B
E3 = 4V r3 = 20
h
* I W//- R2
E2

2 7 9
Ans. = = —A,/, =—A
13 ^ 13 ^ 13 J Ans.
3/138 ‘Pn^uUep- ^ Fundamental Physics (XII)
95. Find the value of the unknown resistance X, in
TYPE XIII. WHEATSTONE BRIDGE
the following circuit, if no current flows through
PRINCIPLE AND SLIDE WIRE BRIDGE
the section AO, Fig. 3(Q). 105. Also, calculate the
91. (0 In a meter bridge, the balance point is found to current drawn by the circuit from the battery of
be at 30 cm from the end A when the resistance R emf 6 V and negligible internal resistance.
in left gap of bridge is of 12 Q. Find resistance S
in the right gap of bridge,
(ii) If the cell and the galvanometer are
interchanged at the balance point, would it effect
the flow of current through the galvanometer.
(Hi) Calculate the balance point of the bridge if R
and S are interchanged.
[Ans. (0 28 O (k) no elTect (Hi) 70 cm]
92. P, Q, R and 5 are four resistance wires of

w
resistances 3, 3, 3 and 4 ohms respectively. They
are connected to form the four arms of Wheatstone
bridge circuit. Find out the resistance with which

Flo
5 must be shunted in order that bridge may be

e
balanced. [Ans. 12 ft]

ree
93. In the circuit Fig. 3(Q).103, a meter bridge is (CBSE 2009) [Ans. 4 ; 1 A]

FFr
shown in its balanced state. The meter bridge wire 96. The resistances in the two arms of the meter bridge
has a resistance of 1 Q/cm. Calculate the unknown
are /? = 5 and S respectively. When the resistance
resistance X and the current drawn from the battery
urr
S is shunted with an equal resistance, the new

or
of negligible internal resistance.
balance length found to be 1-5 /j, where /j is the
FIGURE 3(Q).103
sf
initial balancing length. Calculate the value of S.
kks
Yo
ooo

X FIGURE 3(Q).106
R S
B

40 cm
re

A' B
ou
ad

6V G
YY

B
A C
[Ans. 4 0-66 A]
94. Using Kirchhoff’s rules, calculate the current
ndd

♦ (100-/2)cm *
Re

through the 40 £2 and 20 Q resistors in the


cm

following circuit. (CBSE 2019) I- <>


Fi

FIGURE 3(Q).104
80 V 20 Q
+
A VsAA/^ B
(CBSE 2019) [Ans. 10 £2]
97. A resistance /? = 2 £2 is connected to one of the
gaps in a meter bridge, which uses a wire of length
40 n
1 m. An unknown resistance X > 2 £2 is connected
D AA/vV C
to the other gap as shown in Fig. 3(Q).107. The
balance point is noticed at I cm from the positive
end of the battery. On interchanging R and X, it is
+ found that the balance point further shifts by 20
E I WW- F
40 V 10 £2
cm (away from end A). Neglecting the end
correction, calculate the value of unknown
resistance X used.
[Ans. 0 A, 4 A]
CURRENT ELECTRICITY 3/139

40 cm from the end A. When a resistance of 30


FIGURE 3(Q).107
X
Q is connected in series with X, the null point
R shifts by 10 cm. Find the position of the null point
when the 30 resistance is connected in series
with resistance Y instead of X. Determine the
values of resistances X and Y.

A
l£ B
FIGURE 3(Q).110
X Y
M—/cm—w- (100-/)cm ■M
rVvVi r^AV-i
B

&
(CBSE 2008) [Ans. 3 Q]
D
98. In the simple Wheatstone bridge circuit, where A C

the length AB of bridge wire is 1 m the resistors


X and Y have values 5 O and 2 respectively, <●>

w
Fig. 3(Q).108. When X is shunted by a length of
a wire, the balance point is found to be 0-625 m (CBSE 2009) [Ans. 33-3 cm ; 60 Q ; 90 ft]
from A. What is the resistance of the shunt ?

Flo
If the shunt wire is 0-75 m long and 0-25 mm in TYPE XIV. POTENTIOMETER

reeee
diameter, what is the resistivity of the material
of the wire ? 101. In a potentiometer, a standard cell of emf 5 V and
of negligible resistance maintains a steady current

FFr
FIGURE 3(Q).108
through the galvanometer wire of length 5 m. Two
X Y
WV primary cells of emfs £j and 62 are joined in series
5Q 2C1
for
with (i) same polarity and (ii) opposite polarity.
ur
The combination is connected through a
galvanometer and a jockey to the potentiometer.
kkss
A B The balancing lengths in the two cases are found
Yo
to be 350 cm and 50 cm respectively.
oo

(0 Draw the necessary circuit diagram.


eB

[Ans. 10 Q ; 6-54 x 10"’ £1 m] (/7) Find the value of emfs of the two cells.
99. In the melerbridge experimental set up, shown [CBSE 2004 (C)l [Ans. 2-0 V ; 1-50 V]
in Fig. 3(Q).109, the null point D is obtained at 102. A 10 metre long wire of uniform cross-section of
r
ou
ad

a distance of 40 cm from end A of the meter bridge 20 Q resistance is used as a potentiometer wire.
wire. If a resistance of 10 Q is connected in series This wire is connected in series with a battery of
YY

with /?!, null point is obtained at AD = 60 cm. FIGURE 3(Q).111


Calculate the values of and R^,.
ndd

5V
Re

480 Q
FIGURE 3(Q).109
h I
Ri R2
Fi

i'' V
A ,/K j
600 cm
B A' B
]
G

A
D
C 5 V, along with an external resistance of 480 Q. If
an unknown emf e is balanced at 600 cm of this
wire, calculate (0 the potential gradient of the
I—(●) potentiometer wire and (//) the value of the
K unknown emf e. (CBSE 2006)
(CBSE 2013) [Ans. 8 £2 and 12 Q] [Ans. (i) 2 X 10-^ V/cm ; (ii) 012 V]
100. Fig. 3(Q).I10 shows experimental set up of a 103. The potentiometer wire of length 200 cm has a
meter bridge. When the two unknown resistances resistance of 20 It is connected in series with a
X and Y are inserted, the null point D is obtained resistance 10 Q and an accumulator of emf
3/140 ^ Fundamental Physics (XII) PWTI
6 V having negligible internal resistance. A source 106. In an experiment of calibration of voltmeter, a
of 24 V is balanced against length / of the standard cell of emf M V is balanced against 440
potentiometer wire. Find the length /. cm of potentiometer wire. The potential difference
across the ends of a resistance is found to balance
FIGURE 3(Q).112
against 220 cm of the wire. The corresponding
R = 10f2 6V
reading of voltmeter is 0-5 volt. Find the error in
<●> the reading of voltmeter. [Ans. -0-05V]
K
TYPE XV. TYPICAL PROBLEMS
A B

107. 1 kg piece of copper is drawn into a wire 1 mm


thick, and another identical piece into a wire
2 mm thick. Compare the resistance of these
wires. [Ans. 16 : 1]
(CBSE 2003) [Ans. 120 cm]
108. An infinite ladder net work of resistance is
104. Potentiometer wire PQ of 1 m length is connected

w
constructed with 1 and 2 resistances, as
to a standard cell Ej. Another cell, E2 of emf
102 V is connected as shown in Fig. 3(Q).113 shown in Fig. 3(Q). 115. The 6 V battery between
with a resistance r and a switch S. With switch, S A and B has negligible internal resistance.

Flo
open, null position is obtained at a distance of FIGURE 3(Q).11S
51 cm from P. Calculate (0 potential gradient of

reeee
in ia 1Q
the potentiometer wire and (ii) emf of the cell

FFr
E]. (Hi) when switch S is closed, will null point
move towards P or towards Q ? Give reason for 2Q 2Q 20. 20
your answer ? (CBSE 2004)

for
ur
FIGURE 3(Q).113 6

(0 Show that the effective resistance between A


kkss
and B is 2 n.
Yo
oo

(//) What is the current that passes through the


2 Q resistance nearest to the battery ?
eB

P Q
[Ans. (ii) 1-5 A]
109. A cell of emf 2 V and internal resistance 0-1 Cl
r

supplies a current through a coil of resistance


ou
ad

11-9 The current is being measured by an


ammeter whose resistance is 6 Q. What reading
YY

^2 does it give ? What is the percentage difference


s
from the actual current, when the meter is not
ndd

[Ans. (0 0-02 V/cm (ii) 2 V (Hi) no shift]


Re

used ? [Ans. O'H A ; 33-3%]


105. In Fig. 3(Q).114,AB is 1 metre long uniform wire 110. You are given several identical resistances each
Fi

of 10 Q resistance. Other data are shown in the


of value /? = 10 O and each capable of carrying a
diagram. Calculate (i) potential gradient along
maximum current of one ampere. It is required
AB, (ii) length AO, when galvanometershows no to make a suitable combination of these
deflection.
resistances so as to have a resistance of 5 O
capable of carrying a current of 4 ampere. Find
the minimum number of resistances of the type
R that will be required for the job. [Ans. 8]
111. Calculate the relaxation time and mean free path
at room temperamre (i.e. 2TC), if the number of
free electrons per unit volume is 8-5 x 10^^/m^
and resistivity p = 1-7 x 10“^ i2-m. Given that
mass of electron = 91 x 10“^* kg,
i-6x 10-^^C and-t= 1-38 x KT^^JK"'.
[Ans. (1) 0-016 V cm“^ (ii) 31*2 cm] [Ans. 2-46 x s ; 28-7 A]
CURRENT ELECTRICITY 3/141

112. Twelve cells each having the same e.m.f. are 119. In a meter-bridge experiment with a resistance
connected in series and are kept in a closed box. /?j in left gap and a resistance X in a irght gap,
Some of the cells are connected in reverse order.
null point is obtained at 40 cm from the left end.
The battery is connected in series with an With a resistance R2 in the left gap and the same
ammeter, an external resistance R and two cells
resistance X in the irght gap, the null point is
of the same type as in the battery. The current obtained at 50 cm from left hand. Find the
when they aid each other is 3 ampere and current
is 2 ampere when the two oppose each other. How position of the null point if the right gap is
many cells are connected in reverse order ? containing X and the left gap is containing /?]
[Ans. 1] and /?2 (0 in series and (ii) in parallel.
113. A battery of emf E is connected with three [Ans. (0 62*5 cm (ji) 28-6 cm]

oww
resistances R,2 R and 3 /? in series. The voltage 120. A circuit has a section ABC as shown in Fig.
across 2 is measured with a voltmeter whose 3(Q).l 17. If the potentials at points A, B and C
resistance is 10 R. What is the percentage error ? are V,, V2 and Vj respectively, calculate the
[Ans. H-76%] potential at point 0.
114. Water boils in an electric kettle in 15 minutes

e
after switching on. If the length of the heating

FFrlo
re
wire is decreased to 2/3 of its initial value, then
in how much time the same amount of water will

rFee
boil with the same supply voltage.
[Ans. 10 minutes]

rF
115. An electric bulb rated for 500 watt and 100 volt
is used in a circuit having a 250 V supply.

fsoor
ouur
Calculate the resistance R that may be put in series
with bulb so that the bulb delivers 500 watt.
skf
[Ans. 30 Q]
ooko
Yo
116. The same mass of copper is drawn into two wires
Y
1 mm and 2 mm thick. These two wires are
Bo

connected in series to the source of current. What


reeB

^ ( —+
1 1 1
V.1 ^2
is the ratio of the heat produced in the wires ? Ans. ^
R R R R
[Ans. 16 : 1] ^2 3/ V'M 3/
ooY
uur

117. (/) A storage battery of emf 8 V, internal resistance 121. A Wheatstone bride is almost balanced [Fig.
ad

1 n is being charged by a 120 V d.c. source, using


a 15 Q resistor in series in the circuit. Calculate
3(Q).I18]., with point C grounded. Calculate
dY

the current in the circuit. («) terminal voltage (a) the potential of point B, (b) the potential of
across the battery during charging and point D. (c) If a galvanometer is connected
nind
Re

(Hi) chemical energy stored in the battery in 5 between B and D, what is the direction of current
minutes. (CBSE 2001, 2008) through it ? (d) For what value of the resistance
FFi

[Ans. (0 7 A (ii) 15 V (Hi) 16800 J] BC, would the bridge be in balanced state ?
118. In the net work shown in Fig. 3(Q).1I6 each
resistance is 2 H. Find the effective resistance FIGURE 3(Q).118
between A and B.

FIGURE 3(Q).116
2f2
■VvW

2Q 2n

A/WV WsAr
A B
20 20

[Ans. unci] [Ans. (a) 32 V, (b) 33 V, (c) D to B (rf) 22 Cl]


3/142 'a Fundamental Physics (XII)

For Difficult Questions

1. / = ev = {1-6 X 10-^^) X (6-8 x lO'^) 7. Total charge flowing in two minutes,

w
= 1-088 X 10“3 A ^ = /r = (20 X 10-^ A) X (2 X 60 s) = 24 x lO"^ C
2. / = + /q = (rtf^g + hq) e!t Charge flowing due to electrons
_ (6-0 X10^^ + 4-5 x 10^6) X1-6 x lO'*^ 24x10^
= 12xiO-^C

e
2 2
= 8-4 X 10-2 A = 8-4 mA No. of electrons flowing across a cross-section

e
wr
of a tube in 2 minutes is
ev
3. /=£
e

lloo
r
T iTirtv 2nr 12x10-^
n = = 7-5 X 10*5
1-6x10-’^

F
(1-6x10-*9)x(2-2x10^)

FFu
£
2x(22/7)x(5-0xl0"**) 8. I=nAev,i or — = nAev. (v /=£//?)

= M2x 10-2 A = 1-12


^ ,

rese
4. (a) No. of electrons in 0-6 mol, or nAev.
(p//A) d A
uro
A = 0-6 X (6 X 1022)

k
Fr
o
Charge flowing, or
e
— = ^
e
i.e., oc
1

^ = Ne = 0-6 X 6 X 10^3 X (1-6 X IQ-*^) C ^ nepl

foo
/

fr
= 5-76 X lO'* C
kso
V / V
Y
5-76x10^ d _ or V
' _ "d
(b) Current / = — = = 19-2 A
3/ 3
50x60
Y

/
B
Yo
5. Given, ^ = 8r-3r^-f5r2 9. If A is the area of cross-secUon of the wire, then
oo

number of electrons per unit volume of copper


eBr

(0Current / = — = 8-6r-i-I5f^ ...(0 wire is


e

dt
uru

2x102' -3
od

When r = 0, /=8A n = m

Ax(5xl0 2)
no

dl
ad

(/O -=-6-1-30/ I=nAevj


dt
Y

For / to be maximum or minimum 2x1Q2*


xA X (l-6x 10"*^) X (0-25 x 10"2)
di

dl ~ Ax(5x10"2)
Re
FF

— = 0 or-6-h30/ = 0or/ = 0-2s


in

dt = 1-6 A

(i70 Putting this value of t in (i), we get 10. No. of electrons per unit volume of silver,
/=8-6x0-2-t- 15(0-2)2 = 8- 1-2-h 0-6 N Nd
n =
= 7-4 A
M/d ~~M
As this value of I is less than that at r = 0, it will
6023x1026
be minimum value. So minimum value of current x(10-5xI02) = 5-8557 x 1028
= 7-4 A 108
I
6. (/) Amount of charge that flows in 10 s Then, vd ~
nAe
= area {OABD) under the 1-t graph 10
= Area of AOAE + area ABDE
5-8557xIo2*x314xl0"^xl-6xlO"‘^
= 3-4 X 10-^ ms-1
= ^-«-5x(10-4) =104-30 = 40C 11. Charge on proton = + e = 1-6 x 10"*^ C.
(i7) Amount of charge that flows in 14 s Surface area of earth = 4 n
= area OABC = Area OABD + area DBC = 4x3-14x(6-4x 106)2
= 40 + 5x(14 - 10)/2 = 50C = 5-14x 10*4 m2
CURRENT ELECTRICITY 3/143

Charge received by earth per second per 16. Here. / = 0-50 m ; Z> = 012 x KT^ m, /? = 4 0 Q.
= ne = lOx 16x 10-*^C /, = 1-5 m ; D, = 015 x lOr^ m, /?, = ?
So, current = rate of flow of charge
= n e X 4 7C R =
pi or /? oc
I

icD2/4
= (10 X 1-6 xl(Ti^)x 514x1014
= 0-82x10-3 A
. 5. = ix—=— (0-12x10-3)^ = 1-92Q
12. t =
/ /

UnAe
InAe
I
" R Df ”0-5 ^(0-15x10-3)2
17. (0 Let A, / be the original area of cross section
and length of the given wire and Aj, /j be the
1X (8 X 1q28 ) X (5 X IQ-2 ) X (l-6x 10-19) respective new values when the wire is drawn to
1
twice its original length. Then /| = 2 /. Using the
= 6-4 X 103 s relation A / = Aj /,, we have Ax = A III I

w
V = A mi- All. Therefore,
13. I=nAev^=nAe^E = nAepy = p /j/Aj = p 2 //(A/2)

Flo
= (8-5 X 1()28) X (10-3)2 X (1-6 X 10-19) = 4p//A=4/? = 4x 100 = 400a

e
X (4-5x10-^ )x — ({'0 When wire is drawn to half its radius, new

reee
0-2 area becomes l/4th. Length would become 4

FFr
= 1-22 A
times. .*. /?2 = ^ = 16 X 100 = 1600 ohm
14. Here, density, ^/=8-9 x 103 kg m“3; M = 63-5 kg; 18. As, under the same potential difference across
N =6-0(1 X 102^ per kg atom; /= 1-5 A, the two wires, the same current flows in them so

for
ur
D = 1-2 mm = 1-2 X 10-3 m. the resistance of two wires must be same.

Mass of 1 m3 of copper = 8-9 x lO^ kg pi .


kkss
Now, R =
7tr2 ’
Number of atoms per m3 of copper = N ^Nd
Yo
oo

M/d M
So /?cu =
Pcu^
and Rj=^
B

As 1 atom contributes one conduction electron,


nrr
therefore number of conduction electrons per m3 Cu /
re

of copper is
As Rcu-Rj so Pcu!=M
- or
PCu _ ^
nr?I
ou

Nd (6-02 xl026)x (8-9 x1Q3) Tcr/


ad

n = =
Cu P;
YY

M 63-5
2
= 8-44 X 1()2* m"3 Ecu _ _n.
■VlOxlO-’ "”■^ 5
or

') y p;
ndd

/
Re

V
^ nAe 19. Let I, A and d be the length, area of cross-section
Fi

and density of wire of mass m. Then


1-5
m= Aid or A = mAd

X (1-6x10-19)
7 4 Resisu«.cefi = £'-e^-e^
A m/ld m
Le., Rocfi
= 0-98 X 10"4 ms-l « 10-4
f,_ jo.f
15. Resistance of lamp, R = —
y 80
— = 8Q . { 100 J /
1 f 81 = 0-81
10 R /2 /2 10. 100
In order to use an arc lamp with operating voltage R' = Q-SIR
240 V, a resistance R' is to be connected in series
with it so that the current in the circuit should fR-R'\
% decrease in resistance = xlOO
not exceed 10 A. Then

V=I{R-^R() or 240 =10(8-«-/?') (R-0^1R^


XlOO = 19%
R' = 24-8 = 16Q
or
I R
3/144 'P>uuUe^'^ Fundamental Physics (XII) VOL.I

20. When a wire is stretched, its volume remains New resistance = 4/? = 4xl5 = 60Q
constant, hence
Resistance of each part = 3012
/] A] = /a^2 = V, the volume i.e. /?i = 30(2 and /?2 = 30ft
When they are connected in parallel, the effective
p/i p/jX/j plf
Now, = — , i.e., /?j oc If
A V resistance, = = 15CI
Rj+/?2 30 + 30

f, 5 Current, - = 0*2A

Hence ^=i=L_ 100 *) = M025


15

’ ‘f 1
25. p =
m
or x =
m
= 2*5xir^^s
ne^ X pe^n
or
R2=M025/?i
Mean Iftee path of free electron = x

w
/. % change in resistance
= 1-6 x 10-^ X 2-5 X 10-^^ = 4*0 x 10"** m
\

xlOO = -^-1 xlOO 26. Length of the wire used, / = 1000 7i D

Flo
V *1* J1
R
1 Ttr^ jcr
2

= (M025-l)x 100 = 10*25% Conductance G- —

ee
R pi pxlOOOjtZ)
21. Consider an imaginary cylinder of radius ;c and

Fr
thickness dx of the given cylinder. Its resistance r2 (0-4x10-3)2
is dR =
pdx 1000p D 1000X (49 X10-8) x (4 x 10"2)

for
ur
2kxI = 8*2 X 10n3 S
1 1
Conductivity o = —- _
ks
b
^ p 49x10-8
Yo
Total resistance, R = J f -
= 2*04xl0®Sm-^
oo

2nxl 2nl
a

27. Let D] and Z>2 ^ Oie diameter on the left hand


B

E Exlitl side and right hand side of constriction. Then


re

Current, / = —
R , D| = 20 mm = 2 x 10“3 m; = 1-3 x 10® Am“2
P'®8. v“/
a (0 Current flowing through the constriction
ou
ad
Y

22. R = ^ or A = pi (1-7 X10-8) X (5000) = (l-3xl0®)x—


7 4
A R 5
nd

= 4*09 A
Re

= 1-7 x 10-3 m2
Mass of copper required, (it) For a steady flow of current, 7| = I2
Fi

m = volume x density =Alxd


or
7| Aj — ^2 A2
= (1-7 X 10-3) X 5000 X (8-9 x Kp) = 756*5 kg %D?
1
or / = 37,
' 4 4
23. R =
4p/ or R oc
/
icl)2 d2 _ D. 2-Omm
= 1*15 mm

. = l f2f^415
■■ «l ^ 5 [3J V
28. G = -
R
= i. = ^ =
V 3-0
0*67S

or
«, = l«,=lxl5=4Q a = -
1 _ / _ /
p RA (Y/I)A
24. When a resistor is stretched to double its original
II 2-0x3
length, the new resistance becomes four times = 3*7xlO®Sm-l
its original resistance because Roc/2. VA 3-0x5-4x10-®
CURRENT ELECTRICITY 3/145

29. Here, G= 10^2-^/= 10cm = 0-10m, (Hi) Resistivity at 0°C


-1
a =
RqA _ RpXTtr^ _ 0-267x3-142xa0~^)^
m

1 / IG IG
Po = I I 0-50
o = — or r =

p Tzr'^R %r
2
Y KC = 1-68 X 10-^ Q m

0-10x10
Resistivity at 25°C, P25 = Po [1 + ct x 25]
= 1-78 X 10-3 m = 1-78 mm = 1-68 X 10-6 i^j 64 X 10-3 25]
~V314xI0^ = 1-948 X 10-6 Q m
30. Here, = 132 .Q, R^qq = ?,
a = 4-5 X 10-3 oQ-i 33. (a)Rf = 2Ro = Ro(l -t-a r)
1 1
Now /?, = /?o (1 + a t) or 2 = 1 + a f or t = — = 250“C
a 4x10-3
R 160 = Ro[l +(4-5 X 10-3) ^ 160]

w
or
132 =Ro[l-j-4-5x10-3x160] ...(/) 1
(b) As t = — , which is independent of
and R 400 = Rq[1 + 4-5 X 10-3 X 400] .(«●) a

Dividing («) by (/), we get dimensions of conductor, so the temperature


t (= 250”C) is valid for all copper conductors

FF loow
R
400 _ I+ 4-5x10-3x400 whatsoever may be their shape and size.

ee
132 1 + 4-5x10-3x160 34. At 100°C, for material A,

Fr
1 + 1-80 2-80
= 1-63 [1 + ttj (100-0)]
1 + 0-72 1-72
= 200 [1 +0-0031 X 100]

rer
ur
R40o= 132 X 1-63 = 215 n
31. Herea = 0-00125°C-’,
fofr Fo = 200x 1-31 = 262 n

At 100°C, for material B,


ks
/j = 300 K = 300 - 273 = 27°C
R' =/?2 [l + «2 (100-0)]
YYouro
s oo

R27 = 1 and R, = 2
= 100 [1 + 0-0068 X 100]
As R27 =Rq (1 + a X 27)
BBook

= 100 X 1-68= 168 a


and R, = Rq (1 + a X /)
r ee

Effective resistance in series combination of


or 1 = Rq (1 + a X 27) and 2 = Rg (1 + a f) resistances
oouru
ad

2_ 1 + ar or 1 + ar = 2 + 54a R = R'+R' =262 + 168 =430Q


I“l + ax27
Y

The number attached to 4,3,0 are yellow, orange


or t =
l+54a _ 1 + 54x0-00125
and black. The given carbon resistor is shown in
Ynd
Re

a 0-00125
Fig. 3(Q).119.
= 854 + 273 = 1127 K
FFindi

32. Here, r = 1 mm = 10“3 m ; / = 50 cm = 0-50 m ; FIGURE 3(Q).119


R, = 0-31 n, = 25°C, R2 = 0-51 Q, = 125°C Orange
(i) Temperature coefficient of resistance z
A
0-51-0-31
a =

Rj(r2-f,) 0-31(125-25) \1
0-20 Yellow Black
= 645x10'3'’C"^
0-31x100
35. Resistance = 4-7 /: £2 = 47 x 10^ £2. For numbers,
(li) Resistance at 0°C is 4,7 and 2, the colours are yellow, violet and red.
R 0-31 36. 1=5 mA = 5 X 10"3 A ; Resistance of carbon
1
= 0-267 £2 resistor, R = 01 x 10^ = 100 £2
l + of
1 1 + (6-4x10-3)x25
Voltage, V = /R = 5 X 10-3 X 100 = Q.5 y
3/146 Fundamental Physics (XII) VOL.I

37. Resistance of resistor = 0-34 Q = 34 x 10"^ £2 40. Here, 13 = 2 - (0-8 + 04) = 0-8 A
The colour of ±e third band of a coded resistor Potential difference across D and E
is due to multiplier 10“^ which is for silver. = 10 X I3 = 10 X 0-8 = 8 V
38. Resistance of each of the four parts of length //4 Potential difference across B and G = potential
16 difference across C and F = Potential difference
= 4Q.
*.= 4
IS across D and E

0-8R3 = 04/?2 = 8
When each part is stretched to length /, its volume

w
remains constant. So 8
or
«3 = = 10 £2
0-8
V = A,/,=A2/2
8
or ^ =i=^ =i and
/?2 = 04
= 20 £2

e
/ 4

roow
A,
Equivalent resistance between B and G, B! is

re
NowR2 = pVA or Roc l/A given
1 1 1 1 5 1
●. ^ = ixA = J_x4 = 16

reF
R'“l0'^20'^10“20“4

uFFll
R, /, Aj //4

e
Total resistance between A and H
or
/?2 = 16= 16 X 4 = 64 £2 = 6 + 4 = 10 £2

sFr
Thus, resistance of each stretched part = 64 £2.
41. Here, resistors 5£2 and 5£2 are in series. They

foro
When these four parts are connected in parallel,
ofk together are in parallel to each of the resistors
uor
the effective resistance R^ of the combination 30£2 and 15£2. Thus the effective resistance {R^^
is given by between A and B is
kos
L = _L + JL + i_ + i_ = 1 = 1.
YYo

R^ff 15 "^30'*'(5+ 5)
reeBB
oo

R 64 64 64 64 64 16
eff
18 1
or ^eff- ^
uurY

90 5
39. Given,
The equivalent circuit is as shown in Fig. 3(Q).120.
/I2 3 2
ad
doo

Effective e.nLf. of the circuit = 6 - 2 = 4V


When two wires are in series, the current through
nY

two wires is the same. As FIGURE 3(Q).120


6V 0-2
/ 1 B
nid
Re

I = nAe u j so, vd ~ or v^oc-


A
nAe
FFi

V 10£2
5£2

% \ 2 2V
R
When two wires are in parallel, the poL diff. Vis VWV
0-2
same across each wire. Then

V V 4
/ = 1 = nAev d or ●'d -
Current I =
5 + R + lO
= 0-2 A
R nAeR nAepUA
or 4 = 3+0-2/e or /e = (4-3)/0-2 = 5£2
V 1
or V or v.oc- Pot. diff. between B and A = 5 x 0-2 = 1*0 V
^ nepl d I
I
42. When key is closed and K2 is open, resistance
V of the circuit = /e + X. So current
_ ^2 _ 2
e
V
^2 ^ 1 L=I =
1
R+ X
.(0
CURRENT ELECTRICITY 3/147

When keys and K2 both are closed, S and X Now resistances in arm AC and CB are in series,
will be in parcel, then their effective resistance their equivalent resistance = 2ii + 2Q = 4Q,
IS which is in parallel with resistance of arm
AB (= 2 £2). Therefore equivalent resistance
SX between A and B is
R! =
S + X

Total resistance of circuit = R + R' = ±12 =1-1 ft


4+2 6"3
SX
= R+ 44. Here,e = 2-2V, V=l-8V;i? = 5£2;r=?
S+ X

oww
(2-2-18)
R = x5
I J
£ As r =
Total current, = 1-8
SX
R+
S+ X 10

Current through resistance X will be 9

ee
£ S 45. Heie£=12V,r = 2a/? = 4£2

FFrlo
/j = /i X X

r
S + X
R +
SX (S + X) Total resistance of circiuit-a+■+ R

F
S+ X = 2 + 4 = 6£2

ee
sS

Fr
12
ouru -(«)
Current in circuit, / =
£
— = 2A

sor r
(S + X)R + SX r +R 6

When voltmeter is placed across the cell, then


As
^2 2
1 eS
os kffo
terminal pot. diff.,
V = £-/r=12-2x2 = 8V
So [From (0 and (U)]
ook
Yo
Y
2 (S + X)R + SX When voltmeter is placed across the resistor R,
then pot diff. across R,
Bo

£ eS
reeB

or V' = //? = 2x4 = 8V.


(R + X)2 (S + X)R + SX
As V = V\ hence proved,
oouY

RS
ur

On simplifying, X= e
ad

R-S 46. Current in circuit, I =


R+ r
43. Here, two resistances along each side of triangle
d
Y

are in parallel, their equivalent resistance is


nidn

Case (i), 1 = — -(0


Re

4x4 4 + r
= 2Q
4 +4
FFi

The given network of resistances will be reduced Case (ii)y -(/O


9+r
to the equivalent circuit as shown in Fig.
3(Q).121. Solving (0 and («), we get
r=l£2 and £ = 5V
nGURE3(Q).121
47. Hete,£ = 3 V,r = 3£2,

FIGURE 3(Q).122
30

3V
I

9900
34148 P>uicUeft^'^ Fundamental Physics (XII) VOL.I

Resistance of voltmeter R = 990 The output power is maximum, when external


Refer to Fig. 3(Q).122, current in the circuit, resistance = internal resistance = 3 r
3e
€ 3V I
e
?- = lA
R+r (990+3)Ct”993 =302x10-3 A
max
3r + 3r 2r 2x1
51. Since galvanometer shows no deflection,
Terminal pot. diff. across the cell = pot. diff. potential difference across R = 2 W. Therefore,
across the voltmeter =V=IR
potential difference across SOO (ls:12-2=10V
= (3 02 X 10-3) X 990 = 2-99 V

w
Voltmeter used to measure emf of a cell will Current through ^0012 = ii
500
= JLA
50
record 2*99 V instead of 3 V. Hence the
percentage error is As same current flows through R,
e-V ^ v\ f. 2-99 \ .-. /? =
pot.difP. _ 2 = 100 f2

e
xl00= 1-- xl00= 1- xlOO
current (1/50)
l

row
8
30j

re
52. Here, /? = 500ar = 4a
= (1-0-9967) X 100 = 0*33%
48. EMF of the cell, 8 = /(/? + r) .-. Ae = e-V=/r= — xr

FFllo
F
R +r
In first case. 8 = 0-5 (12+ r) / \
Ae

u
r
In second case, 8 = 0-25 (25+ r)

ree
Percentage error = — x 100 = XlOO
0-5(12 + r) = 0-25(25 + r)
8 ^R + r ^

sFr
r 4 ^
On solving, r = 1 f2 X100 « 0-8%

kro
So 8 = 0-5 (12 + 1) = 6*5 V i,500 + 4 ;
uor
49. (a) When / = 0, total emf = terminal voltage offo 53. As, V=8-/r and 7 = -^
4e = 5-6V R+r
kos
or e = l*4V ZR 8X6
V = /R = 18 = .(i)
YYo
or
6 +r
eerBB

R+ r
From the graph when
oo

8X12
2-8 and 24 = .(«●)
rY

/ = 1-0 A, V = -^ = 0-7 V (for each cell) 12 + r


4
Solving (0 and (ii), we get
uu

(c) Internal resistance of one cell, r = 6I2 and 8 = 36V


8 1-4
When both the switches 5^ and S2 are closed,
ad
doo

r = = 0*712
I 2-0 then effective external resistance in the circuit
nY

max
will be
(b) The output power is maximum, when
6x12
nid

external resistance = total internal resistance R = = 4Q


Re

6+12
= 4r
FFi

zR 36x4
total emf 48 8 V = = 14*4 V
/ max R+ r 4+ 6
total resistance 4r + 4r 2r
54. In circuit (a).
1-4
= 1A Here two cells are connec ^d in opposition, so
2x0-7
Effective emf of the circuit, 8 = 9- 6 = 3V.
50. (fl) When 7 = 0, total emf = terminal voltage total resistance of circuit, /?=1 + 1 = 2I2
38 = 6-0V or 8 = 2*0V
current recorded by ideal ammeter.
(fc) When 7 =1-0 A,
3-0 /=1 = 1 = 1.SA
V = — = 1-0V (for each cell) R 2
3
As the current 7 flows fiom positive to negative
If r is the internal resistance of a cell, then terminal inside the cell of 6 V, i.e., the cell of 6 V
8 2-0 is being charged, so terminal potential difference
r = = 1-012 oftheceUis V=8 + 7r = 6+1-5 x 1 =7-5 V
7
max
2-0
Reading of voltmeter = 7*5 V
CURRENT ELECTRICITY 3/149

In circuit (6) Potential difference across R


Effective emf of the circuit, e = 9 + 6=15V = IR = 0-35 X 10 = 3-5 V
Total resistance of circuit, /?sl + ls2I2 57. Here, e= 1-5 V, r= 10 Q, /? = 30Q ; /= 1-5 A
Current recorded by ideal ammeter. As r is comparable to R, so cells are to be
e 15 connected in mixed grouping for maximum
/ = - — = 7-5 A current. Let there be n cells in series in each row
R 2
As the current I flows from -ve terminal to +ve and m such rows in parallel.
terminal inside the cell of 6 V, Le., the cell of 6 V Current in the mixed grouping of cells is
/wixl*5
is sending the current. The terminal potential mnz
.-. 1*5 =

oww
/ =
difference of the cell. mR + nr mx30 + «xl

V=e-/r = 6-7-5x 1=-1*5 V or 45 m+1*5 n= 1-5 mn .(0


Reading of voltmeter = -1*5 V nr
For maximum current, R^=
Reading of anmieter = 7*5 A m

ee
nxl
55. Refer to Fig. 3(Q).123(a), so, 30 = or n = 30 m .(w)

FFrlo
m

r
FIGURE 3(Q).123
Putting this value in (0, we have

rF
E=1-1V, r=0-5Q nv.O'sn i-iv,ra 45 m + 1-5 X 30 m = 1-5 m x 30 m

ee
or 45 m + 45 m = 45
ouru
rF
1. -‘I Iw 1-1 or 90 = 45 m or m = 2
From (ii), rt = 30 X 2 = 60

e
R = 0-5 n
o R=0-5 n
ffosor
.*. Total no. of cells required
os k = mn = 2x60 = 120
58. In a series combination of identical cells, each
M
of emf E, if one cell is coimected with erverse
ook
= HA
Yo
Y
R+r 0*5 + 0*5 polarity, it will reduce the effective emf of all
Bo

Refer to Fig. 3(Q).123(/?) cells by 2 E, but total internal resistance of cells


reeB

will remain unchanged.


M+M ^ 2-2 As two cells are connected with erverse polarity
oouY

0*5 + 0*5 + r 1+ r in the combination of cells, their el^ective emf


ur

= /i£-4£ = (/i-4)E.
ad

2-2
M = or 1 +r = 2 or r=lQ Total internal ersistance of the cells = nr.
Y
d

1+r
(«-4)£
56. Heree = 2V,r = 1*5 Q Current in the circuit, I =
nidn

nr
Re

Refer to Fig. 3(Q).124, n = 2, m = 2 Potential difference across A or £ is


FFi

mn£ 2x2x2 8 (n-4)£r


/
2^
/ = — = 0-35 A V = E + Ir = E + =2£ 1--
mR + nr 2x10 + 2x1-5 23 nr
V

59. Here,e = 2V,r=l-5Q,m = 2,n = 3,/?=10


FIGURE 3(Q).124
E, r 6, r mne 2x3x2 12
/ = = 049A
I h-
mR + nr 2x10 + 3x1-5 24-5

FIGURE 3(Q).125
e,r e,r
e.r e,r e.r
4 I I— a 4- I I
r-
h
E.r E,r e.r
-VSAr-
R = 10Q I
4- I
l2
|! I

wwv
As two branches are identical, current in each R
branch is, /j = /j = 7/2 = 0-35/2 = 0*175 A
3/150 "Pfuideefo-’^ Fundamental Physics (Xn)CQX9l
As two branches of cells are identical, the current 61. Here, /? = 20 «, /* = 50 kW = 50 x 1000 W
through each branch is = 5x lO^W, V= 5000 V
/, = /2 = 7/2 = 049/2 = 0*245 A Current supplied by generator.
Potential difference across R = IR = 049 x 10
5x10^
= 4*9V = 10A
V 5000
60. As the effective internal resistance of two cells
Power wasted as heat during transmission by cable
is less than the internal resistance T| of one cell,
so the cells are connected in parallel between the />'=/2/? = (10)2 X 20 = 2000 W = 2 kW
two points and 82. The circuit is shown in Power received by factory = P-P' = 50-2
Rg- 3(Q).126. Let V be the potential difference = 48 kW
between points and 82- The potential 62. Electric energy consumed per second = 1(X) J
difference between the terminals of first cell is
80
Heat produced per second = 80% = xlOO
100

ww
FIGURE 3(Q).126
80
= 80J = = 19*05 cal
4-2

FF loo
63. Total power of motor, P = V7 = 50 x 15 = 750 W
Mechanical power of motor, P' = 150 W

ee
Power dissipated across the winding of motor

ee r
= 750-150 = 600 W

rFrF
P' 150
XlOO
Efficiency of motor P
~ 750
rur
-V
ffoor
64. R2000 = vVP = (240)2/200 = 288 fit
= 20%

= e,-/,r, or /,=fl 'i


ks
Given P2000 = 10 ^
YYoou
then (X — (P2000 ~ ^o^'^^o ^ 2000
okos

The potential difference between the terminals 65. In series ; total resistance.
BBoo

of second cell is
R^ = R + R + R = 3R.
r ee

Z2-V V2
h Given V^IOR)^20 or
R
= 60
oouur
ad

Hence current in external resistance


In parallel, total resistance Rp = 72/3.
Y

-V
Power consumed = V^IRp = 3 V^IR
/ = /, + /2 = 3 X 60 = 180 watt
dnYd
Re

66. (a) Current drawn by heater,


/ \
1
P 1000 _ 50^
FFini

/.=-*-=
[n h] [n ^2,
1
V 220 “ 11^
/ \
^2 _ 60 ^ 3 ^
or / = !l+!i [vV = IR] (f>) Current drawn by bulb, ^2~~^ 220 11
^ 1 z J n ^2 J P^ 40 -
2
V Current drawn by radio, h~y~ ~
or I i+1 Current passing through the fuse line
ll '2JJ \ * ^2^ J
, r
= 7i + /o + 7-1 =
, 50 1 3 1—
2 = 5^ A^
^ ^ 11 11 11
or 7 Y2 + R(r2 + r^) _
67. Effective emf of the battery,
r^r2 e=10x2 = 20V

Total internal resistance of battery,


or / =
r=10x0-25 = 2-5i2
('iX'2) + *('2 + 'i)
CURRENT ELECTRICITY 3/1S1

Total resistance of circuit = r + /? = 2-5 + 47-5 When the voltage drops to K = 115 V, the total
= 500 a heat and light energy produced by the bulb in 20
min will be
As the battery emf opposes the charging voltage,
so effective emf
H = X (20 x 60) = 30,000 J
= y-8 = 220 - 20 = 200V R 529
Effective emf V -t = 30 kj
Charging current, I =
total resistance r + i? ^ , r 115 5 .
Current, I = — = = —A
200 R 529 23
= 4A
50 71. Volume of water in 4 cups,
(a) Power supplied by the mains V = 4 X 200 = 800 c.c.

P, = V7 = 220 x 4 = 880 W mass of water, m = volume x density

oww
(b) Power dissipated as heat = 800 X 1 = 800 gram.
P2 = /^(P + r) = 42x50 = 800W (a) Energy spent in boiling the water = ms dQ
= 800x 1 X (100 - 25) cal
(c) The difference of power -P\-P2
= 800 X 75 X 4-2 J
= 880-800 = 80W

e
800 x 75 x 4-2
It is stored in the battery in the form of chemical
kWh =0-07 kWh.

ree
3-6x10®
energy.

rFl
Fre
68. Here, e = l-6V, r = 0-10i2, 7 = 2 A Cost of boiling water = 0-07 x 3 = 0-21 Rs
= 21 paise

rr F
£ 1-6
As 7 = —^ or R + r = - = — =0-Sn
7? + r I 2 (b) Energy spent in boiling the water
= 800x lx(100-5)cal
ouur
or /? = 0-8-0-1= 0-70 Q
(O Voltage drop across R,
V=7/? = 2x0-70 = l-4V
sfoo = 800 X 95 X 4-2 J
800 x 95 x 4-2
kks
kWh = 0-09 kWh
3-6x10®
oooo
Yo
(ii) Rate of energy dissipation inside the resistor
= V7=l-4x2-0 = 2*8W Cost of boiling water = 0-09 x 3 = 0-27 Rs.
eBB

69. Let flj, /j be the area of cross-section and length = 27 paise


of wire A and 02, ^ be the area of cross-section 72. Equivalent resistance of 6 Q and 3 in parallel.
and length of wire B. As the two wires are of same 6x3
urr

= 2C1
material and mass, so their volume is same. Hence 6+3
You
ad

a. L 31 Total resistance of circuit, R = 2 -f 1 = 3 £2


^\h-^h — = Y = —I = 3ora|=3o2
dY

^2 Total current. I = — = 4A.


If e is the emf of the source, then the rate of 3£2
innd
Re

dissipation of heat in wire B is Current through 6 £2 resistor.


FFi

7,1 =7x
«2 = 4x
H — = 10 or = 10 6 +3 3
« /J,
Heat developed in it per minute
£2
= 10 or
£^^2 = 30
or
[v/2 = 3/] x6x60 =640J
p3//fl2 pi
Rate of heat dissipation in wire A is Current through 3 £2 resistor,
£^ £^ £^
72 =7x 6+3 3
R^ pUa^ pi 7?j +/?2
= 3x30 = 90W
Heat developed in it per minute

70. Resistance of the bulb R =


(230)2 = 529£2 9 rs
n2

100 = /|Rj/= - x3x60=i280J


3/152 Fundamental Physics (XII) VOL.I

Heat produced per minute in Ry (0 % increase in illuminating power of one bulb


= 72 =,(4)2x1 x60 = 960 J f n'
73. Here, V= 110 V;/ = 2A, xl00 = xlOO

Power supplied to motor = V7= 110 x 2 = 220 W


P {P /

Power loss in the form of heat = 10 cal s~^


= 10x4«2Js-l = 42Js-^ '(220)^/(20)^^_llxi00
(220)2/(21)2/?
Power used by motor = 220 - 42 = 178 W
power used by motor r2if -1 XlOO
Efficiency, Ti = XlOO
power supplied to motor 120 J

oww
178 441-400
XlOO = 80-9% XlOO = 10-25%
220 400

74. Resistance of aluminium wire. (I'O % increase in illumination of all the bulbs

e
' P** — P' < p/f >
_pl 29x10-^x150 xl00 = -1 XlOO

re
Ra! - = 0-29 a P

FFrlo
Al ^ 15x10"^ /

(220)^/20 i^.^xlOO
/
Current through main line,

rF
ee
/ =
130-124
= 20*69 A
(220)2/21P
ouru
rF
0*29
(21 A
200 -1 XlOO =5%
= 1*613 A
Current through one lamp, /[ = 120
124

ffosor
76. (a) Let P be the resistance of each bulb. Here
os k
20*69
.-. No. of lamps = 1*613 = 12*83 bulbs B and C are in parallel. Their effective
resistance
ook
YYo
Hence no. of lamps to be used = 12 PxP P
Power consumed = 12 x 200 = 2400 W
Pp = P + P” 2
Bo
reeB

= 2-4kW
75. Let R be the resistance of each bulb. Total These bulbs together are in series with bulb A.
Since in series combination of resistances,
oouY

resistance of all the 21 bulbs in series = 21 P


ur

.*. current, /=(220/21P); potential divides in proportion to resistance so


ad

niuminating power of each bulb.


dY

P + P/2 3 3
p = /^R =
r22of xP = '220 f 1
nidn

21P 21 , fi = i V = i X120 = 40V


Re

P + P/2 3 3
Illuminating power of all the bulbs,
FFi

Total power of bulb B and C,


P =
r220f x2IP = (220)^ P' = 60 + 60=120W
21P 21P
\ (b) Total power of all the three bulbs will be
When one bulb is fused, then current,
I = 220/20 R
1 = J_ J- = J- J_ = _L = JL
P ” P.A P' 60 120 120 40
Now illuminating power of each bulb,
or P = 40 watt
^2
"220f 1
/

P' =
220
xP = P^_^ = —A
77. Current in each bulb, / = — 3
20 P
.20J * V 220 11
9
Illuminating power of all the bulbs, No. of bulbs used, n = = 33
3/11

P" =
' 220 f x20P = (220)^ 78. Let i2 he the maximum allowed currents in
the two bulbs and Pp P2 their respective
[20RJ 20P
resistances
CURRENT ELECTRICITY 3/153

(220)2 For loop AfiEM


= 1936Q, 4(/ + /i) + 2/=12or 6/ + 4/i = 12 .(0
25
R
For loop BEDCB
25
= 0-114 A 4(/ + /,) + l/j=6 or 4/ + 5/,=6
220
Solving (i) and («). we get
(220)2
100
= 484Q, / =
and /2=-yA.
R Pot. diff. across R,
2 _ 100
= 0-454 A

V = (/ + /,)/? = |^^-|jx4 = :^V


220
Case (a). When bulbs are connected in series,
net resistance Pot diff. across each cell = pot diff. across R

ooww
/?, = /?!+ = 1936 + 484 = 2420 £l 48
Current in the circuit, = —V
7
._V' ^ 440 = 0-18 A Rate of energy dissipated in R = (7 + /,)2 R
R.s 2420
ri8 6f x4W = ri2f x4 = 11*75 W

e
It means a cunent of 0-18 A passes through both

ere
the bulbs connected in series. By comparing this V V

rFl
81. Let the currents in the various arms of the circuit
value of cunent with the value of i| and I2, we

Fre
note that 1 > /j but less than 12. It shows 25 W be as shown in Fig. 3(Q).128.

rrF
bulb will fuse.
Case (b). When bulbs are connected in parallel,

sffoo
ouur
let the cuirents through I and II bulbs be /j ^ and
kosk
12' ● In parallel combination, the voltage across
each bulb is the same.
YYo
oo

440 440
BB

h' = = 0-23 A
R^ “1936
rre

440 _ 440
and h' = = 0-91 A
R2 “1484
ouu
Y
ad

As ii > ij and > 1*2, hence in this case both According to KirchhofTs second rule, in the loop
the bulbs will fuse
dY

DCBD,
b^R1 3-l=3/,-fl/,-<-2(/,-/2)
79. As per question ; ^.
innd
Re

’ («,+r)2 («2+r)^ or
2 = 6/i-2/2 or 1=3/i-/2 .(0
In the loop BADB ;
Solve it for r in terms of R] and R2, we shall get
FFi

2-1=2/2+1/2-2(/j-/2)
r = ^RiR2 or
1 =-2/i + 5/2 .(*0
80. Refer to Fig. 3(Q). 127, applying Kirchhoff’sloop From (0 and (I'O, we have
law, we get
FIGURE 3(Q).127
3 7, - /j = - 2 /, 5 /2 or 5 7j = 6 /j or /, = I /2
Putting this value in (i*), we get
ei = 12V

1=3x|/,-/,=H,^ or /j=1a
A 4 —'SAAr-
I
ri=2fi
Lr
6 5
B
IM1
R = 4n
-VNAr-
... / =_x — = Aa
E
* 5 13 13
wll Pot diff. between B and D = (I 1^2
T2= 1 £1
c -VSA»— D
= (6 5 \ x2^ = —
2 V
E2 = 6V
[l3 13J 13
3/154 “Pnadee^'^, Fundamental Physics (XII) VOL.I

82. By Kiichhoff’s first rule. From Ohm’s law, z-lIR .(0


.(0 Applying Kirchhoff’s second rule to loop eABHKe
According to Kirchhoff’s second rule we have, £ = 5/ + 5/j + 5/ =10/ + 5/i ...(fi*)
For loop PRSP, 20+ 200 /2 = 5 For loop DFGCD^
For loop PRQP, 20 /j + 60 /^ = 4 .(«**) 5/i-5(/-/i)-5x2(/-/,)-5(/-/|) = 0
Solving (0, (k) and (iii), we get or 5/i-5/ + 5/i-10/+10/i-5/+5/i=0
39 11
A ;/2 = 215 A or 25/,-20/ = 0 or L=-I = -I .(«/)
860 ’ ^ 172 ^ ^ 25 5
83. Since galvanometer shows no deflection, hence
pot. diff. across R=2 volt. Now the circuit reduces From («) and (m), e
= 10/+5x|/ = 14/ ...(*v)

oww
to as shown in Fig. 3(Q).129.
From (i) and (iv), 2 //? = 14 / or R = 7Q
In closed loop ABCDEFA, we have
= 1a
500/ = -2+12= 10 or / = i°. = -LA
500 50
From(0 / = ^ = ,

ee
86. Refer Fig. 3(Q).131, for loop ABCFA
FIGURE 3(Q).129

FFrlo
r
soon B FIGURE 3(Q).131
C

rF
►-AA/W
ri =4Q Ii

ee
I
''I
H* —ViA^ ►—iB

-i-12V -i"2V h Ei=2V


ouru
rF
R
E2 = 1V
—vwv— % c

F E
ffosor
D P2 = 3(2
os k
E3 = 4V f3 = 2(2
In closed loop ABEFA, we have —^1 VAV—< D
I3 I3
ook
Yo
500/ + //?= 12 or 500x—+—/? = 12
Y
50 50
/l = +^2
Bo

R=ma
reeB

or
or 4/j-3/2 = -2+1=-1
84. According to Kirchhoff’s loop arle. or
4/i-3/2 = -1 .(i)
oouY

For the loop ABGH4, 5/i =-2 + 2 = 0 or /j=0


For loop FCDEF, /2 T2 + /3 T3 = - + £3
ur

For loop, BCFGB, 5/2~5/j =-2 + 2 = 0 3/2 + 2/3 = -1+4 = 3


ad

or

or 5 /2 = 5 /j = 5 X 0 = 0 or /2 = 0 or
3/2 + 2/3 = 3
dY

For loop CDEFC 5/3~5/2 = -2 + 2 = 0 At junction C, /3 = /| +12 .(«)


573 = 5/2 = 5x0 = 0 or /3 = 0
nidn

From (ii) and (iii), we have


Re

85. Let the current through the various branches of 3/2 + 2(/i+/2) = 3 or 2/1+5/2 = 3 . .(/V)
network be as shown in the Fig. 3(Q).130. Let R Solving (0 and (fv), we have,
FFi

be the equivalent resistance of skelton cube 2 7


between edges A and K. ^1 = 13 2 13
RGURE 3(Q).130 2 7 9 .
— + — = —A
C 2g-ii) G /3 = /l + /2 = 13 13 13
-Ii)/ (1-11 87. Follow example 87, the effective resistance
between two points, which are diagonally
opposite comers of a skelton cube of 12 wires
,rl = 5 r/6 = 5 X 6/6 = 5 Q
i .-. Pot diff developed = resistance x current
IX
A
K
w2I
= 5 X (40 X 10-3) = 0.2 V
88. The distribution of currents in the network,
21 E
4 following KirchhofTs first rule has been shown
in Fig. 3(Q).132.
CURRENT ELECTRICITY 3/155

According to Kirchhoff’s second rule in the In the closed circuit ABFEA ;


closed loop ABDA (/1 +/?3 + 12^2- ^2
or
11 /?3 + /2 (/?2 ^3) ~ ^2 ...(«)
FIGURE 3(Q).133
II El Ri
Di 4 I

If’ I1-
(I1+I2) R3
A —► ’WWV B

I2 I2-

oww
E2 h ^2
E — I 4 wwv—* F
-1+2=1 (/-/i)-2/i+(/-/i)2 Multiplying (i) by (7?2 + Bj) and (ii) by and
or 1=37-5/ 1 ...(0 subtracting we gel

ee
In the closed loop BCDB
£|7g2+£,7?3 ^2^
4-2=lx/+2x/+27, or 2 = 37+27, ...(»■) 7

FFrlo
I

r
Subtracting (i) and (»*). we get
Multiplying (i) by Ry and (n) by {7?, + R-^) and

rF
ee
1
subtracting, we get
1=7 7, or 7j=yA

rF
ouru 1
h ~
From(/), l=37-5x-
7?i7?2 + R2R2 ^ ^1^
3/ = I+= =
12 12
ffosor Potential difference across = (7, +12) /?3
or 7 =
os k
-A
E,7^2+£|/^3-^2^ . ^2^1 ■^^2^3“^!^'
or
7 7 7x3 7
(i) Potential difference between D and B
'r^R^Tr^R^Tr^ 7?,7?2+/?2^ + ^1^3
ook
Yo
1
Y
= 7.1 x2 = -x2 = = 0-29 V
(£i^2 + ^2^i)^3
Bo

7 7
reeB

(») Potential difference across the terminals of G 7{j7?2 + ^2^3 ^1^3


(receiving current for charging) 91. Refer to Fig. 3(Q).134(a)
oouY

, 4 ^ 22
ur

= e + 7r = 2+ -x2 = — = 3-143 V
7 7
ad

Potential difference across the terminals of 77


dY

(giving current)
4 24
nidn

= £' - 7r' = 4 — X1 = — = 3*43 V


Re

H
7 1

89. According to Kirchhoff’s first rule, current in arm


FFi

DC = 3 - 1 = 2 A
Pot. diff. across 2Q = 2x2 = 4V
As point A is at zero potential, so potential of
point C is Yq-2W
Now, = 4 or V£, - 2 = 4 or V£j = 6 V
Here, V£,-Vg = 4oryg=y^-4 = 6- 4 = 2V
90. Let the current through various arms as per
Kirchhoff’s first rule be as shown in Fig.
3(Q).133.
Using Kirchhoff’s voltage rule in the closed
circuit DABCD

(7, +72) T?3 + 7, 7?, - £,


or
7, (7?i + 7?3) + 72 7?3 = £, ...(0
3A156
^ Fundamental Physics (XU) VOL.I

R_ I 12 _ 30 30 3
FIGURE 3/Q).13S
5 ”(100-/)
or
S 100 - 30 “70 “7
80 V
84 20 a
+
or 84 = 3S or 5 = —= 28ft A B
3
When the Wheatone bridge is balanced the
P R '^ii
potential of B is same as that of D. So, — = — 40 n
Q S
P Q D C
or — = Which is the condition for balanced

Wheatstone bridge as shown in Fig. 3(Q). 144(c). ''h


It means potential of A is same as that of C. +
Therefore there will be no current flowing E' ♦ ¥
through galvanometer. h loa

w
40 V
(iiO When R and S are interchanged, then
S I 28 / Using Kirchhoff’s second rule

Flo
or
R (100-/) for closed mesh ABCDA ;.
12 (100-/)
80 = 2/, -40U2-IO = 607,-40/2

reee
7 /
or or / = 70cm or 4 = 3 7,-272 ...(i)
3 (100-/)

FFr
for closed mesh DCFE ;
P R
40 = 40 (72-7,)+ 10 72
92. For balanced Wheatstone bridge, ~q~~^
urr
= -407,+ 5072
Here P = 3i2,
3 3
Q = 3Q,R = 3a
or S = 3S2
or for
4 = -4 7,+5/2
From (0 and (i‘0, we have;
.(«■)
kkss
3 S
Therefore, the resistance of 4 O must be shunted 37i-27, = -47i + 572
Yo
ooo

with a reistance x so that the combined resistance


or
= 77, = 772 or 7, =72
of 4 Q and a; is equal to 3 So From (i) 4 = 3 7, - 2 7, = 7, or 7, = 4 A and 72=4 A
eB

1 i-i 1 1 1 1 Current through 40Q = 72-7, = 4-4 = 0 A


or AT = 12 £2
;c*^4“3 Cunent through 20 Q = 7, = 4 A
or — =
X 3 4 12
r

95. When no current flows tluough the arm AO, the


ou

93. When bridge is balanced, no current flows through


ad

galvanometer. So Wheatstone bridge is balanced. Then


YY

2 3
or X = -x6 =4Q 7=6
6 60 3
nndd

3
Total resistance of circuit,
Re

Total resistance of bridge wire AB = I x 100 (2 + 4)x(3 + 6)


R = + 24 = 6 £2
Fi

= 100Q
(2+4)+(3+6)
Total resistance of resistances X (= 4 Q) and 6 £2
connected in series = 4 + 6 = 10 £2
^ . . . » e 6 . .
Current m circuit, 7 = — = - = 1A
R 6
This resistance is in parallel with the resistance of
wire AB.
96. When bridge is balanced, then ^ ^ ...(i)
Effective resistance of circuit, S (100-/,)
10x100 100 When resistance S is shunted with resistance S,
£2
10 + 100“ 11 the effective resistance, S' =
SxS S

Current drawn finom the battery S +S 2

7 =
6
Then, R _ 1-5/,
= 0-66 A
100/11 S/2 (100-1-5/,)
94. Suppose the currents in the various branches of R 1-5/,
or
-(//)
the given circuit are as shown in Fig. 3(Q).135. S 2(100-1-5/,)
From (0 and (I'O, we have.
CURRENT ELECTRICITY 3/157

X+30 50 50
i ili = 1 ory=X+30...(«)
Y 100-50 50
(100-/() 2(100-1-5/,)
100 y = -y+30
On solving, cm. From (0 and (i/),
'-=1 3
or y/3 = 30
Putting values in (0, we get;
or y=90Q
5 (100/3) _ 100 = -1 or 5 = 5x2 = 10ft
S (100-100/3) 200 2 X =-x90 = 60n
3
2 / When 30 O resistance is connected in series with
97. .(0 /
X (100-/) X
y,then
When /? and X are interchanged, then y + 30 (100-/)

ww
X (/ + 20) _(/ + 20) 60 / 1 /
or or
2 (100-/-20) (80-/) 90 + 30 (100-/) 2 (100-/)
On solving we get, / = 33*3 cm
2 ^ (80-/) 101. (/) The circuit diagram is shown in Fig. 3(<5).136.

Flo
.(fO
x“(/ + 20)

ee
FIGURE 3(Q).136
C)n solving, from (0 and (ii), we shall get 8 = 5V

rere
/ = 40 cm and X = 3 O
<●>

rFF
V
98. Let R be the resistance of shunted wire, the
effective resistance of R and 5 O in parallel J
uur r
B
= (5xR)/(5 + R)
At balance point. foor
sks
h 1
5R/(5 + R)_ 0-625 0-625 _ 5
Yoo
Ei 82
oko

2 1-0-625 0-375 3

On solving we get, R = 10 fi (if) Potential gradient of potentiometer wire,


BBo

RA R%P- X = 5V ^ 5V _ 1 Vcm“*
re

Now, P = 5m 500cm 100


/ /
ouur

10 X (22/7) X (0-125 X 10~^)^ 1


ad

In first case, e, + 82 = X/, = X 350 = 3-50 ...(0


0-75 100
YY

= 6-54xir’Qm In second case, - £2 = Kl^


nndd

1
Re

99. Case(i) or R, = .(0 X 50 =0-50 .«(k)


* /?2 60~3 100
FFii

R+10 60 3 Solving (i) and (ii), we get


Case (ii);
”40 2 e,=2*0V and £2 = 1*50 V
102. Here, V=5Y,R- 480 Q, resistance of potentio
or
2 R, + 20 = 3 /?2 meter wire, r=20 Q. Length of potentiometer wire
L = 1000 cm

or 2x|Rj + 20 = 3R2 or 4R2 + 60 = 9Rj Potential gradient.

or R2 = 12Q ; From(1), R,=-xl2=8Q X = ' V yr


^R + rJ L
X —
^ 3
100. When null point is at D, the bridge is balanced. 5 20
= 2xlO^V/cm
X 40
^2=2orX=?y .(0 (480 + 20) 1000
y 100-40 60 3 3
Unknown emf e is balanced against length /
When resistance of 30 Q is connected in series = 600 cm. So emf e = X/ = 2 X 10^^x600 = 0*12 V
with X, then at the new null point position
3/158
T^n^ideefr ^ Fundamental Physics (XD) VOL.I

103. Here, L = 200 cm; r = 20 Q, /? = 10 Q, V = 6 V,


e = 24 V

Potential gradient, 7cZ)2/4 ct(10-3)2/4

K = '-1- ' 6 ^ 20 V/cm and = Pi__ P^2


^20+10 J2OO 7cD|/4 7c(2x10-3)2/4
24
so -5- = 16
c

Balancing length = — = 120 cm or


Ri = 16R2
E ao2
104. (a) Potential gradient,
108. Let .c be the equivalent resistance of infinite net

oww
work. Since the network is infinite, its total
= (M»2 V/cm
l 51 resistance x will not change with the addition of
one more unit of two resistances. Then the
(/?) emf of the cell
equivalent circuit of net work will be as shown
Ej = /: X 100 = 0 02 X 100 = 2 V

e
in Fig. 3(Q).137. Here, the total resistance of
(c) When switch5 is closed, no currentis drawn

re
from the cell E2, there is no shift in the position net work is x = l +
2xj: _ 2 + 3x

FFrllo
2+x 2+x
of null point as the position of null point depends

reF
upon the potential gradient along the or 2jc + jc^ = 2 + 3x or j(^-x-2 = 0

e
potentiometer wire [which depends upon the emf On solving, we get, x = 2 £1
ouru
of battery Ej and resistance of potentiometer wire
circuit and length of potentiometer] and emf of

osFr
FIGURE 3(Q).137
the cell E2 which does not change when switch S A
I C
jy/WW-

fkfor
is closed.
in
105. (0 Potential gradient along AB
kso
2n
= ' 4 ^ 10 = 0^16Vcm*i
x
T 6V
(15 + loJlOO
ooo
Yo
Y
BB

B
1-5
(ii) Current through 0-6 Q = = ^A- D

1-2+0-6 6 ’
Y
r ree

Cunrent in the circuit, / = - = - = 3A


|x0-6 = 0-5V
oouu

Pot. diff. across 0-6 Cl = X 2


6
ad

Across length AO, V = 0-3 V is to be balanced. voltage drop across l£2 = 3xl = 3V
Yd

Let / be the length AO, then /. Voltage drop across CandD = 6- 3 = 3V


Hence, current in arm CD = 3/2 = 1*5 A
nidn

0-5
0-5 = 0 016 X/ or 1 = = 31-2 cm 109. When ammeter is in the circuit, total resistance
Re

0016
of circuit = 01 + 11-9 + 6 = 18 O. Current in the
M
FFi

circuit, / = 2/18 = 1/9 = 0*11 A. When ammeter


106. Potential gradient, K = = 0 0025 V/cm.
440 is not in the circuit, total resistance of the circuit
Potential difference across R = 01+ 11-9= 12 a
= 0-0025 X 220 = 0-550 V Current in the circuit, f = 2/12 = 1/6 A
Error in voltmeter reading = reading in voltmeter I'-I
- reading of potentiometer % difference in current = xlOO
r
= 0-5 - 0-550 = -0-05 V
_ (1/6)-(1/9)
107. Let ^ be density of the materi^ of copper wire. XlOO = 33-3%
(1/6)
Let /j and I2 be lengths of copper wires of
diameter 1 mm and 2 mm respectively. As masg 110. Since each path can carry current of one ampere,
= volume X density = [(jc EP-fA) x[\d, so therefore to pass 4 A current, we need four paths
in parallel. Let r be the resistance of each path.
1 =
7c(10"3)2 Lxd =
7c(2x 10-3)2 The equivalent resistance of 4 parallel paths will
4 4
l^xd
be r/4. According to given problem,
or
li - 412 r/4 = 5 or r = 5x4 = 20Q.
CURRENT ELECTRICITY 3/159

In order to have 20 £1 resistance in each path, When a voltmeter of resistance 10 R is connected


two resistances each of resistance 10 £i be in parallel with 2 J?, their effective resistance
connected in series. Since there are four paths, the
total number of resistances required = 2x4 = 8 IRxlOR _5^
2R + 10R 3
m
111. Relaxation time, = Now total resistance of the circuit
ne^p
5R 17
= R+ + 3R = —R
91x10-3’ 3 3
(8-5 x 1()28) (1-6 X 10"’9 )3 x (1*7 x 10"®) Hence, current in the circuit.
= 2-46 X 10-14 s E 3E

(17/3)/? MR
3kT
From kinetic theory, w rms m So, voltage across Rp,

w
3x(1-38x10"33)x300 = M7 X 10^ ms-’ y\=hRp = MR 3 17

Flo
91x10-3’
This will be the reading of voltmeter.
Therefore, mean free path, X-v rms X X

reee
V-V.1
= M7 X 1()3 X 246 X 10-14 ^ 2-87 x KT’m .*. % error = xlOO
V

FFr
= 28*7 A
( VA
112. If one cell is connected in reverse order, then it 1 L XlOO
urr
will destroy one more cell. If n cells are connected
in reverseorder, then 2 n cells will be destroyed.
If £ is the emf of each cell, then effective emf. of
for y)
(5/17)£ XlOO = 11*76%
kkss
12 cells will be = (12 - 2 n) £. {E!3)
Yo
oooo

When this battery if aids two similar cells in


series, then it supplies a current of 3 A to 114. ^ = y\ _ y\ or A = iL .(0
eB

resistance R. Therefore, R, R^

^ (12-2w)£ + 2£ ^(14-2n)£ .i = i .(«●)


r

...(0 But /?« /, so


ou

R R
ad

When battery and two cells oppose each other,


YY

then the current in the external resistance is 2 A. Now, /2 = (2/3)/,


Hence,
From (0 and (ii);
nndd
Re

^ (12-2n)£-2£ (10-2w)£ ...(«*)


!l = !l or
3 15
— = — or r, = 10 minutes
2
Fi

R R h h
Dividing (0 by (/i), we get 115. Resistance of bulb, Rq = V^/P
= 100 X 100/500 = 20 £2
3 14-2n
or n = 1
2 " 10-2/1 Current in bulb, I = V/Rq = 100/20 = 5 A
For the same power dissipation in bulb, the
113. Total resistance of the circuit before connecting current in the circuit must be 5 A. When bulb is
the voltmeter = /? + 2/? + 3/? = 6/?.
operated on 200 V supply, then the safe
Hence the current, I = £/(6 R). resistance of the circuit (R') is given by
Voltage across the resistance 2 R is,
= 50Q
£ £ I 5
V = Ix2R = x2/? = —
6R 3
Resistance required to be put in series will be
This is the voltage to be measured. /? = /?'-20 = 50 - 20 = 30£2
3/160
“pfitzcCee^ '4. Fundamental Physics (XII)
The paths AEB and ACDB are in parallel, there
116. //=/2/e =/2-EL (m = Kr^ Id) fore, the effective resistance between A and B
Tcr^
-■
will be — = ^ 1 I 3 _ 4+3 _ 7 Sr
or R = —
= /2 px/M /2pw R Ir 8r 8r Sr 7

(Kr^)XKr~d r^^ d But r = 2Q, therefore, R = Sx2/l = 16/7 ohm

( 2X

So H CC
1 H
1= 1 f2^^ = 16:1
119.
X ~ 60 ~ 3
or R
* 3
...(0

^2 UlJ \ 1* /
/?2 50
117. Effective emf of the circuit, £'=120-8=112V X 50
or /?2 = X ...(«●)

Total resistance of circuit, /? = 1 + 15 = 16 Q {/) when /?j and /?2 are in series, the effective
e' 112 2X 5X
resistance = /?,+/?-, = — + X =
(i) Current / = - = — = 7A ^ 3

w
/e 16 3

5X/3 /
(ii) Terminal voltage during charging Then ; or5(100-/) = 3/
X (100-/)

Flo
y=£+/r = 8 + 7x1= 15 V
/ = 5 X 100/8 = 62-5 cm
(///) Chemical energy stored in the battery in 5 or

reeee
minutes = e // = 8 x 7 x (5 x 60) = 16800 J (//) When /?j and /?2 are in parallel, the effective
118. The distribution of currents in the circuit is shown resistance is

FFr
in the Fig. 3(Q).138(o). Due to symmetry, current
in arm AE is equal to current in the arm EB. Since 2X/3xX _2^
(2X/3 + X) ~ 5
the current in arm CE is equal to tlie current in
for
ur
the arm ED, therefore the resistance of the net 2X/5 I
Then, or 2(100-/) = 5/
work will not change if the wires CED and AEB X (100-/)
kkss
are disconnected at£ as shown in Fig. 3(Q).138(/>) 200
Yo
/ = = 28*6cm
Now resistance of path AEB = r + r = 2 r or
oo

120. According to Kirchhoff’s first rule at O.


eB

— — /j+/2 + /3 — 0 .(0
If Vq is the electric potential at 0,
r

Vj, V2 '^3 be the potential at A, B and C


ou
ad

respectively, then Vq~Vi=I]R^ ;


YY

Vq — V2 “ ^2 ^2 3tid Vq — V3 = /j R^
ndd
Re

Thus, /, =
Fi

and j

Putting values of /j, I2 and 73 in (/) we get


V.-V.
0 ^0-^2 .^0-^3 = 0
R
1

1 1 1 V
or V 1 1
^2 + ^
+^ ^3 = 0
0
R
(2r)xr 1 /?2 Rj, R
1 /?2 /?3
Resistance of path ACDB = r + + r
(2r) + r
1-1
8r
or Vq =
_ n . ^2 ^ r.L+j--1
3 L«i ^2 "^3. . ^1 ^2
CURRENT electricity 3/161

121. (a) (Current through arm ABC, or V|,-0 = 33 or Vo= + 33V


7 = 48/(10 + 20)= 1-6 A (c) As Wjy > Vq, so direction of current will be D
toB.
Pot. diff. across B and C = = 1*6 x 20
= 32 V id) The bridge will be balanced when
As Vc = 0 so V»=
B + 32V
ib) Current through arm ADC, /? "^11
r = 48/(5 + ll) = 3A
Pol diff. across D and C= = 3 x 11 = 33 or «=i«^=22n
5

ww
Flo
e
ree
Fr
rF
uurr
s for
kks
Yo
oooo
r eB
ou
ad
YY
nndd
Re
Fi
3/162
Fundamental Physics fXIIlPTSWl

WITH
SOLUTIONS

w
Q. 1. The storage battery of a car has an emf of 12 V. If the internal resistance of the battery is 0*4 Q,
what is the maximum current that can be drawn from the battery ?
Sol. Here, 8= 12 V, r=04Q

i=—

e
Maximum current, /max = 30A
r " 04

re
row
Q. 2. A battery of emf 10 V and internal resistance 3 £2 is connected to a resistor. If the current in the
circuit is 0-5 A, what is the resistance of the resistor ? What is the terminal voltage of the battery
when the circuit is closed ?

eeF
ullo
Sol. Here, 8= 10V;r = 3£2;/ = 0-5 A-;/f = ?; V=7

FF
8 10
/ =
or (/? + r) = j
= 20 or i? = 20 - 3 = 17 £2

srr
(J? + r) 0-5

roF
Terminal voltage, V = = 0-5 x 17 = 8-5 £2

k
Q. 3. At room temperature (27*0‘’C) the resistance of a heating element is 100 £2. What is the temperature
uor
ofof
of the element if the resistance is found to be 117 £2, given that the temperature co-efficient of the
material of the resistor is 1*70 x 10“^ “C"*.
kos
Sol. Here. i?27 = 100 a ;R,= 117 £2 ; r = ? ; a = 1-70 x I0-^“C~‘
YYo
B oo

^21 ^27
rY

We know that a = or r-27 =


eerB

/?27 0-27) /?2?xa


u

117-100
t = + 27 = + 27 = 1000 + 21 = 1027"C
oou
d

/?27xa 100x1-7x10^
ad
nY

Q. 4. A negligibly small current is passed through a wire of length 15 m and uniform cross-section
6*0 X 10“7 m^, and its resistance is measured to be 5*0 £2. What is the resistivity of the material at the
nid

temperature of the experiment ?


Re

Sol. Here, / = 15 m ; A = 6 0 x 10~7 m^ ; /? = 5 0 £2 ; p = ?


FFi

RA 5-0x6x10"7
P = = 2-0xl0*'^£2m
/ 15

Q. 5. A silver wire has a resistance of 2-1 Q at 27*5"C, and a resistance of 2-7 £2 at 100®C, Determine the
temperature coefficient of resistivity of silver.
Sol. Here, /?27-5 = 2-1 £2 ; /?,(,(, = 2-7 £2, a=?
^100 ~ ^27-5 2-7 - 2-1
a = = 0-0039"C"^
/e27.5>< (100-27-5) 2-1X (100-27-5)
Q. 6. A heating element using nichrome connected to a 230 V supply draws an initial current of 3-2 A
which settles after a few seconds to a steady value of 2-8 A. What is the steady temperature of the
heating element if the room temperature is 27-0®C ? Temperature coefficient of resistance of nichrome
averaged over the temperature range involved is 1*70 x 10^“C“i.
CURRENT ELECTRICITY 3/163

Sol. Here; /?27 = 230/3-2 = 2300/32 £2 ; = 230/2-8 = 2300/28 £2 ; a = 1-7 x lO"^


a =
~^7 or r-27 =
~^27
/?27x(r-27) /?27xa

or j- ^7 + 27 =
(2300/28)-(2300/32) + 27 = 840-2 + 27 = 867-2X.
/^7xa (2300/32) X 1-7x10-4
Q. 7. Detennme the current in each branch of the following
network: Fig. 3(N).l.

oww
Sol. The currents through the various arms of the circuit have
been shown in Fig. 3(N)-1-
According to Kirchhoff’s second law;
In a closed circuit EABCE ;
10 = 10 (jj +12) + 101| + 5 (/j —13)

e
10 = 25 I1 + 10/2-5 *3

re
or

● .(i)

FFrllo
or
2 = 5 Zj + 2 I2 ~ I3

reF
In a closed circuit ABDA ;

e
10 Zj + 5 13 — 5/2 = 0
or
ouru 2/i + /3-/2 = 0

osrF
or
h. = 2 z'l + Z3 m
In a closed circuit BCDB ;
5 (z’j — z'3) — 10 (z'2 + Z3) — 5/3 = 0
or 5 z'l - 10 z’2 - 20 Z3 = 0 or /j = 2 Z2 + 4 Z3
ffor
k (*w)
kso
From {ii) and {iii);
ooo
YYo
/j = 2 (2 /| + i'^ + 4 z'3 = 4 /j + 6 z’3 or 3 z'l =-6/3 or z'l = - 2 z‘3 .(iv)
BB

Putting this value in (z’z*) ; z’2 = 2 (- 2 z’3) + z’3 = - 3 z’3 .(V)


Putting values in (J); 2 = 5(-2/3) + 2(-3z’3)-z’3 or 2 = -17z’3 or z’3 = -2/17A
r ree
Y

From (z’v); z’, = -2(-2/17) = 4/17A;


oouu

From (v). z’2 =-3 (-2/17) = (6/17) A


ad

z’l + z’2 = (4/17) + (6/17) = (10/17) A


Yd

-13 = 4/17 - (_ 2/17) = (6/17) A


/2 + z*3 = 6/17 + (- 2/17) = 4/17 A
nidn

Q. 8. A storage battery of emf 8*0 V and internal resistance of 0*5 £2 is being charged by a 120 V d.c.
Re

supply using a series resistor of 15*5 Q. What is the terminal voltage of the battery during charging ?
What is the purpose of having a series resistor in the charging circuit ?
FFi

Sol. Here, e.m.f. of the battery = 8-0 V ; Voltage of d.c. supply = 120 V
Internal resistance of battery, r = 0-5 £2 ; External resistance, R - 15-5 £2.
Since a storage battery of e.m.f. 8 V is charged with a d.c. supply of 120 V, the effective e.m.f. in the
circuit is given by e=120-8=112V
Total resistance of the circuit = /? + r = 15-5 + 0-5 = 16-0 £2

e 112
= 7-0 A
Current in the circuit during charging is given by, / - A + r
,
10

Voltage across /? = //? = 7-0 x 15-5 = 108-5 V


During charging, the voltage of the d.c. supply in a circuit must be equal to the sum of the voltage drop
across R and terminal voltage of the battery
120 = 108-5 + V or V =120-108-5 = 11*5 V
The series resistor limits the current drawn from the external source of d.c. supply. In its absence the
current will be dangerouslyhigh.
3^64 ‘Pnadeefi^’4, Fundamental Physics (XII) VOL.I

Q.9. The namberdeiidty of free dectroDs in a copper conductor is estimated at 8*5 x 10^ How long
does an dectron take to drift from one end of a wire 3.0 m long to its other end ? The area of cross-
section die wire is 2.0 X lOr^m^and it is carrying a current of 3.0 A.
Sol. Here, n = 8.5xl028m-^;/=3.0m;A=:2.0xl0-®m2;/ = 3.0A,/ = ?
I
As, Is^nAevj nAe '

Now, / IjiAe 3.0x8.5xl0“x2.0xl(r^xl.6xl(r*’


3.0

s 2.72 X 10^ s =7 hour 33 minute.

w
Flo
e
reee
FFr
for
ur
kss
Yo
oo
B
re
ou
ad
YY
nd
Re
Fi
CURRENT ELECTRICITY 3/165

1 WITH ANSWERS,
HINTS AND SOLUTIONS

1
\
I IT siiivy

MULTIPLE CHOICE QUESTIONS-I

1. Consider a current carrying wire (current /) (a) He should measure /j more accurately
in the shape of a circle. Note that as the (b) He should change S to 1000 Q and repeat

ww
current progresses along the wire, the the experiment
direction of J (current density) changes in (c) He should change 5 to 3 and repeat the
experiment

FF loo
an exact manner, while the current I remains
unaffected. The agent that is essentially (d) He should give up hope of a more accurate
measurement with a meter bridge

ee
responsible for is
4. Two ceils of emfs approximately 5 V and 10 V

ee r
(a) source of emf
are to be accurately compared using a

rFrF
(b) electric field produced by charges accumu potentiometer of length 400 cm.
lated on the surface of wire
(a) The battery that runs the potentionmeter
rur
(c) the charges just behind a given segment of
wire which push them just the irght way by
ffoor
should have voltage of 8 V
(b) The battery of potentiometer can have a
ks
repulsion voltage of 15 V and R adjusted so that the
YYoou
(d) the charges ahead potential drop across the wire slightly
ookos

exceeds 10 V
2. Two batteries of emf £j and £2 (£2 > and
BBo

internal resistances rj and T2 respectively are (c) The first portion of 50 cm of wire itself should
have a potential drop of 10 V
re

connected in parallel as shown in Fig. 3(N).2.


(d) Potentiometer is usually used for comparing
resistances and not voltages
oouur
ad

5. A metal rod of the length 10 cm and a rec


Y

tangular cross-section of 1 cm x 'A cm is


connected to a battery across opposite faces.
dndY
Re

The resistance will be

(a) maximum when the battery is connected


FFini

across 1 cm x Vi cm faces
(a) The equivalent emf of the two cells is (b) maximum when the battery is connected
between £j and £2, i.e., £j < £^^ < £2 across 10 cm x I cm faces

(b) The equivalent emf £^^ is smaller than £j (c) maximum when the battery is connected
across 10 cm x ‘/a cm faces
(c) The is given by £^^ = Ej + £2 always
(d) same irrespective of the three faces
(d) £^^ is independent of internal resistances rj 6. Which of the following characteristics of
and T2
electrons determines the current in a
3. A resistance R is to be measured using a meter conductor ?
bridge. Student chooses the standard resis
(a) Drift velocity alone
tance 5 to be 100 fl. He flnds the null point at
/j = 2*9 cm. He is told to attempt to improve (b) Thermal velocity alone
the accuracy. Which of the following is a (c) Both drift velocity and thermal velocity
useful way ? {d) Neither drift nor thermal velocity
3/166 'Pn4^dc^fa. ^ Fundamental Physics (XII) orSTWI

MULTIPLE CHOICE QUESTIONS-II

7. Kirchhofl^’s junction rule is a reflection of


find R = h
(fl) conservation of current density vector R
i?3 = 10 Q within errors.
I
(/>) conservation of charge
(c) the fact that the momentum with which a
charged particle approahces a junction is
unchanged (as a vector) as the charged
particle leaves the junction
(rf) the fact that there is no accumulation of
charges at a junction
8. Consider a simple circuit shown in Fig. 3(N).3.

ww
stands for a variable rests- tance R'.

R' can vary from Rq to inflnity. r is internal


resistance of the battery (r « R « Rq).
(a) The errors of measurement of the two

Flo
FIGURE 3(N).3 students are the same

ee
{b) Errors of measurement do depend on the

rere
R' accuracy with which ^^d can be

rFF
measured
A B (c) If the student uses large values of /?2 and R 1’
uurr
R
the currents through the arms will be feeble.
1. .
foor This will make determination of null point
accurately more difficult
ks s
■W W
{d) Wheatstone bridge is a very accurate
Yoo
V
instrument and has no errors of measurement
ook

(a) Potential drop across AB is nearly constant 11. In a meter bridge the point D is a neutral point
BBo

as R' is varied (Fig. 3(N).5).


re

ib) Current through R' is nearly a constant as R' FIGURE 3(N).5


is varied R
ouur

r-’vVvVi
ad

(c) Current / depends sensitively on R' B

V
] [ [
YY

r + R
always A
A 100-/1 — C
nndd

9. Temperature dependence of resistivity p(7) of


Re

semiconductors, insulators and metals is


0 100
Fii

signiflcantly based on the following factors :


(a) number of charge carriers can change with
temperature T <●>
(b) time interval between two successive
collisions can depend on T (a) The meter bridge can have no other neutral
(c) length of material can be a function of T point for this set of resistances
id) mass of carriers is a function of T (b) When the jockey contacts a point on meter
10. The measurement of an unknown resistance wire left of D, current flows to B from the
wire
/? is to be carried out using Wheatstone bridge
(see Fig. 3(N).4). Two students perform an (c) When the jockey contacts a point on the meter
experiment in two ways. The first student wire to the irght of D, current flows from B
takes ^2 = 10 O and /f| = 5 £2. The other to the wire through galvanometer
student takes /?2=1000 £2 and i?i=500 £2. (d) When R is increased, the neutral point shifts
to left
In the standard arm, both take /?3 = 5 £2. Both
CURRENT ELECTRICITY 3/167

ANSWERS
1. (b) 2. (a) 3. (c) 4. (fc) 5. (a) 6. (a) 1. {b, d) 8. {a, d) 9. {a, b)
10. (b, c) 11, {a. b, c)

HINTS FOR DIFFICULT MULTIPLE CHOICE QUESTIONS

Multiple Choice Questions -1


1. The current density is a vector quantity. Its direction is given by the direction of flow of positive charge in
the circuit. The same is possible due to electric field produced by charges accumulated on the surface of
wire.

2. Refer Fig. 3(N).6 the equivalent internal resistance of two cells between A and B is

ww
'i'2 FIGURE 3(N).6
or r
eq
.40
‘'eq h h 'i + '2
If is the equivalent em/of the two cells in parallel between A and

Flo
B, then

ee
^eq _
[Refer Art. 3.26(b)]

rere
^eq rj T2 ''42

rFF
£
£j Kj + £2 q Xr
V2 ^42 '*'^2^1
uurr
eq eq
n^2 ''l''2 {^1 + ^2) (^] + '2)

£2>£i, so £i<€^^<£2.
foor
This shows that whatever may be the values of rj and r2, the value of £ eq is in between and £2- As
ks s
Yoo
2-9 R _ R 100x2-9
3. As meter-bridge is balanced, so
oook

or R = ^3Q
100-2-9 5 " 100 97-1
The meter-bridge (Le., Wheatstone bridge) is most sensitive and accurate when the resistance of all the
eBB

four arms of the bridge is of the same order. Therefore a student must use S = 3 i2 to improve the accuracy
of the measurement of resistance R. Hence, option (c) is correct
rr

4. In a potentiometer experiment, the em/of a cell can be measured if the potential drop along the potentiometer
ouu
ad

wire is more than the emf of the cell to be determined. As values of emfs of two cells are approximately 5
V and 10 V, therefore, the potential drop along the potentiometerwire must be more than 10 V. Hence,
YY

option (b) is correct.


nndd
Re

5. We know that R = —
A
(a) When the battery is connected across 1 cm x cm faces,
Fii

FIGURE 3{N).7
pxlO
then / = 10 cm ; A = 1 X '/2 cm^, R, = lx‘/2
= 20 pa
1
(b) When the battery is connected across 10 cm x 1 cm faces, -5-cm
2 —
I- ●10 cm
then
/ = 1/2 cm ; A = 10 X 1 cm^, /?2 = 10x1 20
(c) When the battery is connected across 10 cm x 1/2 cm faces,
pxl = ^a
then
/ = 1 cm ; A = 10 x 1/2 cm^, R^ = (10x‘/2) 5
6, Current I = n A ev^. i.e.,

Therefore current in a conductor is determined by drift velocity alone.


Multiple Choice Questions - II
7. Kirchhoff’s junction rule or Kirchhoff’s current rule is based on the law of conservation of charge and is a
reflection of the fact that there is no accumulation of charges at a junction.
3/168 Fundamental Physics (XII) VOL.I

V
8. Current, I = AsR'>>R, so R + R'^R' Hence, I = —

RR' r + /W?7/?' r+R


r +
R + R'

Thus, option id) is correct


RR'
Potential difference across A and B, Vj^g -Ix R + R'

VAB ~
V RR' VXRR' VxRR' _ VR [vR + R'^R']
r + iRR^/R + R') (R + R') r(R + R')+RR' rR'+RR'~ r + R
Vj^g is constant if /?' is varied. Hence, option (a) is a>rrect
Yab^ VR VR

w
Current through R', I' = R' (r + R) R' rR'+ RR'

Thus I' is not constant as R' is varied. The current I does not depend on the sensitively on R'.

Flo
m
9. Resistivity of a conductor is given by P = ne^x , where the various terms

e
rree
have their usual meanings. As temperature of semiconductor changes, n changes

r FF
and also x changes. Hence, p changes with temperature. Thus options (a) and
(ft) are true.
uurr
10. Refer Fig. 3(N).8.
For first student; ^2 =
For other student; R2 = 1000 Q,
^ ^3 = ^ ^
= 500 £2, R3 = 5 £2
for
kkss
According to Wheatstone bridge principle
Yo
oooo

R = RjX
eB

or

R Rj
Putting values in above relation, we note that the value of R for both the students is 10 Q.
r

The Wheatstone bridge is most sensitive and accurate if the resistance of all the four arms of bridge is of
ou
ad

the same value. Therefore, the errors of measurement of the two students depend on the accuracy and
YY

sensitivity of the bridge, which intum depends on the accuracy with which R2 and can be measured.
When /?2 large* Oie currents through the arms of bridge will be feeble. TTiis will make the
nndd

determination of null point accurately more difficult. Thus, options (ft) and (e) are correct.
Re

R li
Fi

11. When there is a neutral point at D in meter-bridge, then — =


S (100-/,)
For the given values of R and S, there will be only one value of /j for which we shall get the neutral point
on bridge wire. In this case - Vg = V4 - Vq or Vg = Therefore the galvanometer shows no deflection
when jockey contacts a point at D. There is no current in galvanometer arm.
When jockey contacts a point D, on meter-bridge wire left of D, the resistance of arm ADj becomes
smaller than previous value. Due to it, Vj^-Vg>Vj^~ or > V^. Therefore current flows to B
fi-om the wire through galvanometer.
When jockey contacts a point D2 on meter beidge wire right of D, the resistance of arm AD2 becomes more
than first value. Due to it, y-y^ > y. -y„ or y„ > y_
A A d d U2

Therefore the current will flow from B to the wire through galvanometer. When R is increased, the
neutral point will shift to the right instead of left on bridge wire. Thus, options (a), (ft) and (c) are correct
CURRENT ELECTRICITY 3/169

VERY SHORT ANSWER QUESTIONS

12. Is the motion of a charge across junction momentum conserving ? Why or why not ?
A ns. When motion of charge is taking place across a junction, then momentum of charge is not conserved.
Because when a charge (i.e., electron) approaches ajunction, in addition to a uniform electric field E that
it normally faces (which keeps the drift velocity fixed), there are accumulation of charges on the
surface of wires at the junction. Which also produce electric field, resulting the variation in the direction of
momentum of charge.
13. The relaxation time X is nearly independent of applied electric field E whereas it changes significantly

ooww
with temperature T. First fact Is (in part) responsible for Ohm’s law whereas the second fact leads to
variation of p with temperature. Elaborate why ?
eE
Ans. mVd
The drift velocity of electrons, vd ~
X, or x = or X «
m eE

e
re
Thus relaxation time depends on velocities of electrons and ions and electric field. The applied electric

rFFl
ree
field affects the velocities of electrons at the order of 1 mm/s, which is very very small and can be neglected.

F
This supports Ohm’s law. On the other hand, change in temperature{TK) affects velocitiesof electrons and

rF
m

ions at the order of l(r m/s, which affect x to great extent and hence to p, Le., resistivity as P

fsfoor
ouur
14. What are the advantages of the null-point method in a Wheatstone bridge ? What additional
measurements would be required to calculate /^unknown method ?
kosk
Ans. In a Wheatstone bridge, the null point method involves a balanced Wheatstone bridge, in which the ersistance
of galvanometer does not affect the balance point and there is no need to determine the currents in resistances
YYo
oo

and galvanometer.
BB

The unknown resistance can also be determined by using unbalanced Wheatstone bridge. But in this method
we require the additional accurate measurement of all the currents in resistors and galvanometer as well as
rre

the internal resistance of galvanometer.


oYuu

15. What is the advantages of using thick metallic strips to join wires in a potentiometer ?
ad

Ans. In potentiometer the number of straight wires each of length 1 metre are held parallel on a horizontal
dY

wooden board and Joined by thick metal strips in order to have long effective length of potentiometer wire
which is required for accurate measurement. Since the resistance of thick copper strips is very very small,
innd

can be ignored ; while measuring the potentiometer length for the null point. The potentiometer length for
Re

null deflection can be noted down directly from the metre scale provided on the wire, which leads to
FFi

accurate measurement.

16. For wiring in the home, one uses Cu wires or A1 wires. What considerations are involved in this ?
Ans. For wiring in a house, the two considerations are involved :

(i) cost of metal wire used. (//) Good conductivity of metal wire.
The silver wire has the best conductivity but it can not be used due to its high cost. After silver, the copper
and aluminium are the next best conductors. Their cost is also low than silver. Therefore copper and
aluminium are used for wiring in the house.
Conductivity of copper wire is more than that of aluminium wire and for a given area and length, the mass
of copper wire is more than that of aluminium wire. Therefore, if we require better conductivity without
considering the cost involved, then copper wire is preferred. If we keep in mind the cost factor involved in
the wiring, then aluminium wire is preferred.
17. Why are alloys used for making standard resistance coils ?
Ans. The allows (Le., manganin or constantan) are used for making the standard resistance coils as they have
low value of temperature coefficient of resistance and have high value of resistivity.
3/170 Fundamental Physics (XII) orSTW

cables having resistance


18. Power P is to be delivered to a device via transmission . If V is the voltage
across R and I the current through it, find the power wasted and how can it be reduced.
Ans. Power delivered, P = voltage x current = V7 or / = /W. If is the resistance of transmission line then
p2
power wasted, = R(^ = V
R
2 “C

In order to reduce V should be large. Therefore the power should be transmitted at high voltage.
19. AB is a potentiometer wire (Fig. 3(N).9). If the value of
FIGURE 3(N).9
R is increased, in which direction will the balance point
/ shift ? E

Ans. If R is increased, the current through the potentiometer wire K>


will decrease. Due to it, the potential gradient of
potentiometer wire will also decrease. As a result of it, the
balancing length will increase, i.e., the position of J will

w
A J B
shift on potentiometer wire towards B.
20. While doing an experiment with potentiometer (Fig.
3(N).10) it was found that the deflection is one sided and V

Flo
(i) the deflection decreased while moving from one end
A of the wire to the end B\ (ii) the deflection increased,

e
reee
while the jockey was moved towards the end B. E FIGURE 3(N).10
(a) Which terminal + or -ve of the cell £|, Is connected

Fr
at X in case (i) and how is £j related to £ ? K->
(b) Which terminal of the cell £| is connected at A in

for
ur
case (ii) ?
Ans. In the given potentiometer experiment, the deflection in the A B

galvanometer will be one sided if the emf of cell £j is greater E1


5
ks
than the emf of battery E because then emf Ej is greater X Y
Yo
than fall of potential across the potentiometer wire due to
oo

battery E.
B

(a) The deflection in galvanometer will decrease while moving from one end A of the poteniomter wire to
the end B, if positive of the cell £j is is connected to X and negative of the cell to Y, because in this
re

situation, the current in galvanometer decreases with the increase in length of potentiometer wire.
(b) The deflection in galvanometerwill increase while moving from end A of the poteniomter wire to the
ou
ad

end B if - ve of the cell £j is connected to X and + ve to Y. In this situation, the current in galvanometer
increases with the increase in the length of potentiometer wire.
Y

21. A cell of emf £ and internal resistance r is connected across an external resistance R. Plot a graph
showing the variation of P.D. across £, verses R.
nd
Re

Ans. Refer to solution of Conceptual Problem 24, Page 3/93.


Fi

SHORT ANSWER QUESTIONS

22^ First a set of n equal resistors of R each are connected in series to a battery of emf e and internal
resistance £. A current I is observed to flow. Then the n resistors are connected in parallel to the
same battery. It is observed that the current is increased 10 times. What is ‘n’ ?
e e
Ans. When n resistors are in series, then current in the circuit, I = ...(0
R + nR (n+l)R
When n resistors are in parallel, then effective resistance is Rg^=R/n.
e m
Current in circuit, V =
R + RIn R(n + l)

m lOe
As per question, /' = 10 / or n = 10
R(n + l) (n + l)R
CURRENT ELECTRICITY 3/171

23. Let there be n resistors with = max (Aj............. J{|,) and =min J?„}.
Show that when they are connected in parallel, the resultant resistance Rp < R^i„ and when they are
connected in series, the resultant resistance R^ > Interpret the result physically.
Ans. Let and R„,„ be the maximum value and minimum value of the given n unequal resistors. When n
resistors are connected in parallel, the effective resistance Rp is

-L=l+-L + ... + —
1
mm =
/e_:
mm
I ^min I I ^mi n

R
n Rl Rn

As Rp is less than the minimum resistance so ^>1 or R^in>^p or

oww
When n resistors are in series, then = +/?2 + + /? n

As R^ is greater than the max. resistance, R max R,>R max


In Fig. 3(N). 11(a), due to one resistor of resistance there is only one route for the current in circuit.
When n resistors are in parallel. Fig. 3(N).H(h), there are additional (n — 1) routes for the current in circuit.

e
Therefore > Rp or Rp <

re
In Fig. 3(N). 11(c) and id), due to one resistor of resistance the current in circuit is more than the

FFrllo
current due to n resistors in series of effective resistance R^. So

rF
ee
AAA/Sr- FIGURE 3(N).11
ouru A/SAAr

rF
^inra AAAA/- ^/nox

fosor
VWV ^AAf I—^ ^ ^ ^AAAr-^
AAAAt
os kf
^inin
I
ook
YYo
V

e o o
Bo
reeB

24. The circuit in Fig. 3(h0.12 shows two cells connected in opposition to each other. Cell Ei is of emf 6 V
oouY

and internal resistance 2 Q; the cell £2 of emf 4 V sand internal resistance 8 Q. Find the potential
ur

difference between the points A and B,


ad

FIGURE 3(N).13
Yd

FIGURE 3(N).12
6V, 2Q 4V, 8£2
B
i-4h
B
nidn
Re

El E2
FFi

Ans. Here, £1 = 6 V, ri = 2 Q, £2 = 4 V, rj = 8 ft
6-4 1 ^
Refer Fig. 3(N).13, current in circuit, I = = —A
2+8 5
Potential difference across A and B is the terminal voltage FIGURE 3(N).14
E E

of cell £j = - 7rj = 6-ix2 =5*6V I


5
25. Two cells of same emf E but internal resistance and T2 are
connected in series to an external resistor R (Fig. 3(N).14).
What should be the value of R so that the potential difference ■'VAAr
across the therminals of the first ceO becomes zero. R
3/172
4. Fundamental Physics (XII)
Ans. Refer Fig. 3(N).15, FIGURE 3{N),15
£+ £ IE E. '●l E.r2
Current in circuit, / =
Tj + r, + /? Tj + r2 + /?
As per question, let for the given value of £, terminal I"
potential difference across first cell be zero, i.e., E-Ir^-0
AAAAr
2E
or E = I,r = R
1
(rj + T2 + /?)
2r,
1 =
or or
Tj + T2 + /? = 2 Tj or
/? = rj-r2
(rj +T2+R)

w
26. Two conductors are made of the same material and have the same length. Conductor A is a solid wire
of diameter 1 mm. Conductor fi is a hollow tube of outer diameter 2 mm and inner diameter 1 mm.
Find the ratio of resistance to Rg.

Flo
Ans. For a solid wire of resistance /] =/, p, = p, £)] = I mm. A^=nD^/4 =7t(l)^/4mm^

e
reee
^A _Pi/|^ p/ _4p/

Fr
=
Aj :t(l)2/4 n

for
ur
For hollow tube of resistance Rg, ^2 = P2 = P- A^= n (D^ - D^)/4 =k (2^ - l^)/4 = 3 tc/4 mm^
= Ez!i^_pL- = M
ks
R ●●● = 3
B
A, (37t/4)
Yo
3n
oo

27. Suppose there is a circuit consisting of only resistances and batteries. Suppose one is to double (or
B

increase it to n-times) all voltages and all resistances. Show that currents are unaltered.
re

Ans. Let there be n batteries in series of the circuit.


Let R^^be the effective internal resistance of n resistors in
series and is the effective emf of n batteries in series and R is the external resistance in the circuit.
ou

Fig. 3(N)-16.
ad

FIGURE 3(N).16
Y

Current through R is I = ...(0 ^eff R


eff
R + R
I W/v
nd
Re

When all the voltages and resistances are increased n fold, then I” -I
Fi

enw pnew = nR
= m and = n R. R
eff > eff

t-nev/
nZ
Current, = eff eff ^eff = /
new new
R + R nR + nR R + R
eff eff eff

Thus, current remains unaltered when all voltages and resistors are increased to n folds.

LONG ANSWER QUESTIONS

28. Two cells of voltage 10 V and 2 V and internal resistances 10 Q and 5 Q respectively, are connected
in parallel with the positive end of 10 V battery connected to negative pole of 2 V battery (Fig.
3(N).17). Find the effective voltage and effective resistance of the combination.
CURRENT ELECTRICITY 3/173

FIGURE 3(N).17 FIGURE 3(N).18

R R I R
ww B ^WSf C ^vAA/v

h<
2V 2V wl
1
l2
■VSWs/—
50
A
r—«l
l2
AMAr
50
D

Ir' -Ii h-VSAA»—'


Re#
♦ I—vwv I E
o

ooww
I1
10V
10O o 10V
10O

Ans. In Fig. 3(N). 18(a), according to KirchhofTs first rule, at junction A,


/, = /+/2 or / = /,-/2 ...(0
According to Kiichhoff’s second rule, in closed loop ABCDA ; we have

e
ere
, IR + 2
7/?-/2x5=-2 or .(m)

rFl
Fre
10-//?
In closed loop BCDEFAB, IR + 1^x10= 10 h= m

rrF
or
10

j 10-//? IR 5+ 2 _\0-IR-2IR-4 6-3//?


sffoo
ouur
Putting values in (/), we get 10 10
10
6 _ 6/3
kosk
10/ = 6-3//? or /(10 + 3/?) = 6 or .(/V)
”l0 + 3/? (10/3) + /?
or
Yo
oo

If the 8^ and r^j^are the effective voltage and effective internal resistance of the combination, then from
Y
BB

/ = .(V)
Fig. 3(N).18(fr),
rre

Comparing (/v) and (v), we get, e^=6/3 = 2V ; rqgr=10/3Q.


ouu
Y

of 220 V. The wiring of the room consists of Cu of


ad

29. A room has AC run for 5 hours a day at a voltage


1 mm radius and a length of 10 m. Power consumption per day is 10 commercial units. What firaction
dY

of it goes in the joule heating in wires ? What would happen if the wiring is made of aluminium of the
same dimensions ? = 1*7 x 1(T® Qm, = 2*7 x 10^ Qm]
innd
Re

Ans. Energy consumed per day = 10 kWh


Time for using the energy per day = 5 h.
FFi

lOkWh P 2000 J/s


= 2 kW = 2000 J/s ; Current supplied, = 220 V »9A
Power consumed (P) =
5h

(17xl0~^)xl0 x9^
Power loss in copper wire = /?p„ = Esti
nr^
xl^ - (22/7) X (10-3)2
»4J/s

4
Fraction of power loss = — t— ; % of power loss = —xlOO =0-2%
2000 1000 1000

Power loss in A1 wire = /2 = (27x1Q-^)x10x92 = 64 J/s


Ttr^ (22/7) X (10-3)2
64
% of power loss in A1 wire = XlOO =0*32%
2000
3/174
^ Fundamental Physics (Xri) VOL.I

30. In an experiment with a potentiometer, Vg = 10 V. is adjusted FIGURE 3(N).19


to be 50 Q (Fig. 3(N).19). A student wanting to measure voltage
of a battery (approx. 8 V) finds no null point possible. He then P EliU q
dimmishes JR to 10 12 and is able to locate the null point on the
last (4*^) segment of the potentiometer. Find the resistance of the
potentiometer wire and potential drop per unit length across the Vb t
A 2
I
wire in the second case.
Ans. Let be the resistance of potentiometer wire. When /? = 50 Q in
series with potentiometer wire circuit, current through potentiometer I
/ =
V,B 10 Nlt V
wire IS
R + R' 50+R'
10
Potential drop across potentiometer wire = XR'
(50+ R')
Ki
As no null point is found on potentiometer wire while measuring

ww
voltage El (= 8 V), so
10/?'
<8 or 10/?'<400 + 8/?' or /?' < 200 Q
(50 + /?')

Flo
10/?'

ee
When /? = 1012, in series of potentiometer wire, then ;>8 or 2/?'>80 or /?'>40n
10+/?

rere
If no deflection in galvanometer is on the last third segment of the potentiometer wire, then

rFF
10x(3/4)/?'
<8 or 7-5 /?' < 80 + 8 /?' or - 0-5 /?' < 80 or /?' > 80/0-5 or /?' > 160 £2
uurr
10 + /?'
It means R' lies between 160 Q and 200 f2 foor
Therefore, any value of resistance of potentiometer wire between 160 f2 and 20012 will help us in achieving
ks s
the no deflection on fourth wire of potentiometer circuit.
Yoo
Here, potential drop across 4 m of wire > 8 V, potential drop across 3 m of wire < 8 V.
oook

If <|) is the potential gradient, then 4 <1) > 8 V ...(0 and 3<|)<8V .(a)
eBB

From (0, (l)>8V/4m = 2V/m or <l)>2V/m


8
From (ii\ 3<1)<8V or
$<- V/m or <|)<2-V/m FIGURE 3(N).20
rr
ouu

R = en
ad

31. (a) Consider circuit in Fig. 3(N).20. How much energy is absorbed by
electrons from the initial state of no current (ignore thermal motion) to
YY

the state of drift velocity ?


(b) Electrons give up energy at the rate of Rfi per second to the thermal
nndd
Re

energy. What time scale would one associate with energy in problem (a) ?
It = no of electron/volume = 10^/m^, length of circuit = 10 cm, cross-
Fii

section, A = (1 mm)^ V = 6V

I 1
V _ 6V
Ans. (a) Current / = —
1
= 1A, Drift velocity,
R~ 6Q.
=
nAe 1029 X (10-^) x(l-6x 10-^9) j^xlO-^m/s.
No. of electrons in the wire = nA l

1 1 \2.
1
KE of all the electrons = —m vhxnAl = -x(9-1x1(T31)x — xlO-^ x1029x(1(T^)x10"1
2 ® “
2 1-6 J
« 2 X 10-1’ j
(b) Power loss = P-R = l2 x 6 = 6 J/s
2xl0'*’j
All the KE of electrons would be lost in time = = 0-33 X 10-1’ s » 10-1’s
6 J/s
CURRENT ELECTRICITY 3/175

■I

NEET/JEE
SPECIAL

students should, refer;tQ ' I

ww
● MCQs In Physics for NEET >

Pradeep's Stellar Series.... * MCQs In Physics for JEE (Main)


I

●A sepa^telytavailable/fe^vthes^j^i^ThTnatio^s^^ t..

Floo
Multiple Choice Questions (with one correct Answer)

ee
reer
I. Electric current and drift velocity (a) 1-6 X 10^ S (b) 3-2 X lO^s

rFF
(c)64x lO^s {d) 8-0 X 10^ s
1. The electron of a hydrogen atom revolves around
uur r
II. Ohm’s law and Resistance
the proton in a circular nth orbit of radius
n2/i2 ^2
:5~ with a speed v„ = 2e^nh . The
ffoor
5. In an aluminium (Al) bar of square cross-section,
sks
Kme~ 0
a square hole is drilled and is filled with Iron (Fe)
YYoo
current due to circulating charge is proportional as shown in the Fig. 3(CF).l, The electrical
ooko

to resistivities of Al and Fe are 2-7 x 10"^ £2m and


10 X 10”^ iim, respectively. The electrical
eBB

(a) (b)e^
resistance between two faces P and Q of the
(c)^
composite bar is
[AMU (Med.) 2012]
r
ouur

2. The mean free path of electrons in a metal is


ad

4 X 10^ m. The electric field which can give on


Y

an average 2 eV energy to an electron in the metal


will be in the units V/m,
nddY

-11 -11
(a) 5 X 10 (b) 8 X 10
Re

(c) 5 X lO"^ (d)Bx 10”^


FFiin

(AIPMT 2009) 2475 1875


(a) pH ib)
3. The current flowing through wire depends on time 64 64
as, 7=3/2 + 2r + 5.
1875 2475
The charge flowing through the cross-section of (c) pI2 id) pH
49 132
the wire in time / = 0 to t = 2 second is
(JEE Advanced 2015)
ia) 22 C (b) 20 C
(c) 18 C (i/) 5 C (WB JEE 2009) 6. In order to quadruple the resistance of a wire of
uniform cross-section, a fraction of its length was
4. A uniform copper wire of length 1 m and cross- uniformly stretched till the final length of the
sectional area 0-5 mm^ carries a current of I A. entire wire was 1-5 times the initial length. The
Assuming that there are 8 x 10^^ free electrons fraction is
per cm^ in copper, how long will an electron take (a) 1/2 ib) 1/4
to drift from one end of the wire to the other end ?
(c) 1/6 id) 1/8
ANSWERS
1. (c) 2. (c) 3. (a) 4. (c) 5. (/;) 6. (d)
3/176
‘P'utdec^ '4. Fundamental Physics (XII) orsri
7. In the Fig. 3(CF).2, the potential at points B and 11. A 2 W carbon resistor is colour coded with green,
C, and the value of resistance R are black, red and gold respectively. The maximum
FIGURE 3(CF).2 current which can be passed through this resistor is
R 1A (a) 20 mA (b) 100 mA
C WAr B
1A (c) 04 mA (d) -63 mA
4Q

“no V (J£E Main 2019)

D A III. Resistivity, Electrical conductivity


and effect of temperature
(fl) 10 V, 4 V, 4 a (b) 6 V, 4 V. n
(c) 10 V, 4 V, 6 n (J) 6 V. 4 V, 4 ^ 12. In a conductor, if the number of conduction
8. A carbon resistor of (47 ± 4.7) kCl is to be marked electrons per unit volume is 8-5 x 10^ m“^ and
with rings of different colours for its mean free path is 25 fs (femio second), its
identification. The colour code sequence will be approximate resistivity is :

w
(a) Yellow - Green - Violet - Gold
K = 9-l X 10-3' jjg]
(b) Yellow - Violet - Orange - Silver (a) 10-3 (/>) 10-^ Om

Flo
(c) Violet - Yellow - Orange - Silver
(c) lO-'^Om id) 10-^ Om
(d) Green - Orange - Violet - Gold

e
(JEE Main 2019)

ree
(NEET 2018)
9. In the circuits shown in Fig. 3(CF).3(a) and (b), the 13. Copper and iron wires of same length and

FFr
readings of the voluneters and the ammeters will be diameter are in series and connected across a
battery. The resistivity of copper is about one sixth
urr
FIGURE 3(CF).3

100
VyA^r
I1
for of that of iron. If £j and £2 ^ die electric fields
in the copper and iron wires respectively, then
kkss
which one of the following is correct ?
(a) £, < £2 (/)) £j = £2 0
Yo
Vi
ooo

e Ai
(c) £j > £2 (d) £i=£^ = 0
B

14. V~I graph for a conductor (Platinum wire) at


temperamres Tj and r2 is as shown in Fig. 3(CF)-4.
re

10V
T2-T1 is proportional to
ou

10
I2
ad

WW
YY

ion
1-V/V-V2-'
o A2
ndd
Re
Fi

(a) cos 2 0 ib) sin 2 0


(a) V2 > Vj and /j = I2 (b) Vj = V2 and /j > I2
(c) cot 2 0 (d) tan 2 0.
(c) V, = V2 /, = I2 id) V2 > V] and /, > I2
(NEET 2019)
15. When 5 V potential difference is applied across a
wire of length 0-1 m, the drift speed of electrons
10. A copper wire is stretched to make it 0-5% longer. is 2-5 X 10"^ ms"'. If the electron density in the
The percentage change in its electric resistance, if wire is 8 x lO^^m"^. The resistivity of the material
its volume remains unchanged, is ; is close to
ia) 2-0% ib) 2-5% ia) 1-6 X 10-® Qm ib) 1-6 X 10-^ Clm
ic) 1-0% id) 0-5% ic) 1-6 X 10-^ nm id) 1-6 X 10-3 nm
(JEE Main 2019) (JEE Main 2015)

ANSWERS

7. (c) S.{b) 9. (c) 10. (c) II. (fl) 12. U/) 13. (a) 14. (c) IS. UO
CURRENT ELECTRICITY 3/177

16. At 0®C the electric resistance of a conductor B is 0.0 2c.a


r2
n times that of conductor A. The temperature (a) {b)
coefficient of resistance for conductors A and B 0,+02 0^ +O2
are ttj and respectively. The temperature Oj +O2 O, +O2
coefficient of resistance of a circuit segment (C) (d)
containing A and B in series is
20j O2 0,02
(RE-AIPMT 2015)
na, +na2 n Oj - n 02 20. A current of 5 A is flowing through magnesium
(a) ib)
1 +n l + n
wire. The current density is making an angle of
60° with area vector. Find the electric field.
0( +n02 02 +nO]
(c)
l +n
(d)
l +n
(Given area - 2 m^,
p = resistivity of magnesium = 11 x 10“^ SI units)
17. Following the above question, the temperature
coefficient of resistance of a circuit segment

ww
(a) 55 X 10-^ V/m - X10^ V/m
(b) 11
containing A and B in parallel is
no. +no
1 2 n Oj - n 02

Floo
ia) ib) — X10-^ V/m — X10^ V/m
l + n 1 + n
(O3 id)^-

ee
a., +no
Oj +n02 I
(JEE Main 2021)
ic) id)
l +n l +n

reer
rFF
IV. Resistances in series and Parallel
18. The region between two concentric spheres of
radii and is filled with a conductingmaterial,
21. The equivalent resistance between A and B for
uur r
of resistivity p. The potential of inner sphere is
and of outer sphere is where > V),. Due
to it, there is a current in radially outward
ffoor
the mesh shown in Fig. 3(CF).6 is :
sks
direction. The variation of electric field intensity
YYoo
ooko

£ as a function of distance r from centre of sphere


is given by
r eBB
ouur
ad
Y
dY
Re
nnd
FFii

(a) 4-812 (6) 7-2 Q


ic) 1612 id) 301:2
(NEET 2020)
22. The effective resistance of a parallel connection
that consists of four wires of equal legth, equal
area of cross-section and same material is 0-25 12.
What will be the effective resistance if they are
19. Two metal wires of identical dimensions are connected in series ?
connected in series. If <7j and O2 are the (a) 4 12 {b) 0-25 12
conductivities of the metal wires respectively, the (c) 0-5 12 (d) 1 12 (NEET 2020)
effective conductivity of the combination is ;
ANSWERS

16. (c) 17. (d) 18.(6) 19.(6) 20. (fl) 21. (c) 22. (a)
3/178
‘Pn.euUcft. ^ Fundamental Physics (XII) LV»1WI
23. The current /| (in A) flowing through 1 Cl resistor (a) 100, 50 (b) 50, 100
in the following circuit is (c) 0, 100 (d) 0, 50
FIGURE 3(CF).7 27. Six equal resistances are connected between
points .P Q and /?as shown in Fig. 3(CF). 10. Then
/l Ifi net resistance will be maximum between
■►A/WV—I 2Cl
^Q. ^WW-i

2Cl

1V

ww
(a) 0-5 ib) 0-2 (c) 0-25 (d) 04
(JEE Main 2020)
24. A set of ‘n’ equal resistors, of value 'R' each, are

Floo
connected in series to a battery of emf *£’ and
internal resistance */?’. The current drawn is I. {a) P and Q {b) Q and R

ee
Now, the ‘n’ resistors are connected in parallel to (c) P and R (d) any two points
the same battery. Then the current drawn from

eer
28. In the given circuit diagram, when the current

rFrF
battery becomes 101. The value of ‘/i’ is reaches steady state in the circuit, the charge on
id) 20 (b) 11 the capacitor of capacitance C will be
rur
(c) 10 (d) 9 (NEET2018) FIGURE 3(CF).11
25. Consider an infinite ladder of network shown in ffoor E r

Fig. 3(CF).8. A voltage is applied between points


s
-i AAAAt
osk
A and B. If the voltage is halved after each section.
YYoou
Find the ratio R-,!R^.
oook

'●l
eBB

FIGURE 3{CF),8 AAMr

Ri Ri Ri Ri Ri C
A<^AW-i-VW ■VW-i-VA-
r

R2 R2 R2 R2 R2 AWV
ouur
ad


'2
Y

id) CE (P) CE —L
ia)\:2 ib)2:\ (r + r2> (rj + r)
nd dY

(c) I : 1 id) 2: 3
Re

ic)CE id) CE a
26. In the circuit Fig. 3(CF).9, when key is closed,
('2 + r)
FFini

the ammeter reads Iq whether K2 is open or closed.


But when Xj is open, the ammeter reads /q/2 when (JEE Main 2017)
K2 is closed. Assuming that ammeter resistance 29. In Fig. 3(CF).I2, the resistance (in ohm) between
is much less than /?2, find the values of r and R^ the terminal points A and B of the given infinitely
(in ohms) long circuit will be

FIGURE 3(CF).9 FIGURE 3(CF).12


loon in in
A
Kl K2 “V/v AAA/

V/AV -vwaHX)-
R1
R2 = 100fr B
in

E, r

ANSWERS

23. (b) 24. (c) 25. ib) 26. id) 27. (n) 28. (a)
CURRENT ELECTRICITY 3/179

(fl) (b) 8 n, 4 n
(a) iS-V) (b) (i-Vi)
(c) 6 n, 9 a (d) 9 a, 6 n
ic) (l + ^l3) id) (2 + ^). (Odisha JEE 2010)
30. In the circuit shown, the cell is ideal with emf 34. If equivalent resistance of identical resistors in
= 15 V, each resistance is of 3 Q. The potential series combination is S and in parallel
difference across the capacitor is combination is P. If 5 = n P, then find the
FIGURE 3(CF).13 minimum possible value of n ?
3pF ia) 1 ib)2
R G
B H
(c)0 (d) 4 (JEE Main 2021)
R R
R R
A ■WW F V. Terminal potential difference
D and internal resistance of a cell
15V

ww
35. The internal resistance of primary cell is 4 It
generates a current of 0-2 A in an external resistance
ia) Zero (fc)9V of 21 The rate at which chemical energy is
(c) 12 V id) 15 V consumed in providing the current is

FF loo
(a) 0-42 J/s (b) 0-24 J/s
31. An electric current of 5 A is passing through a
circuit containing three wires arranged in para (c) 5 J/s id) 1 J/s (AIIMS 2013)

ee
llel. If the lengths and radii of the wires are in the 36. A student measures the terminal potential

ee r
ratio 2:3:4 and 3:4:5, then the ratio of currents difference (V) of a cell (of emf e and internal

rFrF
passing through wires should be resistance r) as a function of current (/) flowing
(a) 27 : 32 : 35 ib) 54 : 64 ; 75 through it. The slope and the intercept, of the
rur
(c) 9 : 16 : 25 id) 4:9: 16 graph between Fand 1, then respectively, equal
32. Fig. 3(CF).14 shows a network of eight resistors
each of resistance P (= 2 Q), connected to a 3 V
ffoor
(a) - r and e
(c) - e and r
ib) r and - e
id) e and - r
ks
battery of negligible internal resistance. The (AIPMT 2009)
YYoou
current I in the circuit is 37. In the circuit shown in Fig. 3(CF).16, cells are of
ookos

equal emf £ but of dilferent internal resistances,


BBo

FIGURE 3(CF).14
r, = 6 iQ and r2 = 4 Q. Reading of the ideal volt
re

3V meter connected across cell 1 is zero. The value

”I 1“
of the external resistance R in ohm is equal to
R B R
-VvVv—^
oouur
ad

D FIGURE 3(CF).16
R
Y

R R
R' R
dndY
Re

F R E
<v>
FFini

(ti) 0-25 A ib) 0-50 A


ic) 0-75 A id) 1-0 A E E '-2
l-VS/VSr
33. The equivalent resistance between points A and
B with switch S open and closed are respectively Cell 1 Cell 2

FIGURE 3(CF).15 (a) 10 a ib)4Q


12Q
■WvV ●B
ic)2Q id) 24 Q
S. 38. A battery consists of a variable number 'n' of
Q
identical cells (having internal resistance ‘r’ each)
en:
which are connected in series. The terminals of
●b.
the battery are short-circuited and the current I is
A— ■Wi/V
i2n
measured. Which of the graphs shows the correct
relationship between / and n ?
ANSWERS

29. (c) 30. (c) 31. ib) 31. id) 33. (b) 34. (d) 35. (d) 36. (a) 37. (c)
3/180
^>uxdeejb, 'a Fundamental Physics (XII) orsiBi

in series with an ammeter and two cells, identical


with the others. The current is 3 i4 with the cells
and battery aid with each other and 2 A when the
cells and battery oppose each other. How many
cells are wrongly connected.
ia)l
(c)3 (^4
42. Two identical cells each of emf 1-5 V are

connected in parallel across a parallel combination


of two resistors each of resistance 20 Q.
A voltmeter connected in the circuit measures
1 -2 V. The internal resistance of each cell is :
(a) 2-5 Q (b)4Q

ww
(NEET 2018)
(c)5 Q (d) 10 Q
VI. Combination of ceils (JEE Main 2022)

FF loo
39. In the following circuit, the current in each VII. Joule^s law of heatings
resistance is
Electric power and Electric energy

ee
FIGURE 3(CF).18
43. The charge flowing through a resistance R varies

ee r
2V 2V 2V
with time t as Q = at - b'P, where a and b are

rFrF
I-
positive constants. The total heat produced in R
IS:
rur
in in 1 n
ffoor {a)
a^R
3b
(b)
a^R
ks
2b
2V 2V 2V
YYoou
a^R a^R
ookos

{a) 0-5 A (fr)OA (0- (^0 (NEET 2016)


6b
BBo

(c)l A (d) 0-25 A


44. A rise of temperature of 4®C is observed in a
(JEE Main 2017)
re

conductor by passing a current. If the current is


40. Two cells with the same e.m.f. E and different tripled, the irse of temperature will be
oouur

internal resistance rj and r2 are connected in series


ad

ia) 8“C Q}) 12°C


to an external resistance R. A value of be selected
Y

such that the pot. diff. at the first cell should be (c) 16°C (d) 36°C
zero when {e) 24°C (Kerala PMT 2010)
dndY
Re

45. Three bulbs A, B and C are connected as shown


FIGURE 3(CF).19
in Fig. 3(CF).20. What changes occur in bright
FFini

ness of the bulbs when the switch is closed ?

FIGURE 3(CF) M

R I
AWW
B

(a) /? = r, + T2 {b) R-r^-r^


A
(c) R = r,/r2 id)R = r^ = r2-
41. 12 cells, each having same emf, are connected in
series and are kept in a closed box. Some of cells
are wrongly connected. This battery is connected
answer;

38. (c) 39.(6) 40.(6) 41. (a) 42. (c) 43. {d) 44. (d)
CURRENT ELECTRICITY 3/181

(a) Brightness of A increases, but that of C


FIGURE 3(CF).22
decreases.
A B
K ♦K ♦1
(b) Brightness of A remains the same but that of
C decreases,

(c) Brightness of both A and C decreases.


(d) Brightness of A increases but that of C remains (11
the same.

46. If two bulbs of wattage 25 W and 100 W respecti n


vely each rated at 220 V connected in series with
the supply of 440 V, which bulb will fuse ?
(a) 25 W bulb (i>) 100 W bulb

ww
E
(c) both of them (d) none of them.
(AlPMT 2012) 50. If power dissipated in the 9 O resistor in the circuit
47. In the circuit shown in Fig. 3(CF).21, the heat shown in Fig. 3(CF).23, is 36 watt, the potential

Flo
difference across the 2 resistor is
produced in 5 Q resistor due to current flowing

ee
through it is 10 calories per second. The heal FIGURE 3(CF).23

rere
produced in 6 resistor is 90

rFF
V/v-
FIGURE 3(CF).21
uur r
6Q
4Q en
—I foor
ks s
20
Yoo
A—► B
■vw
ook

50 V
-VvW
BBo

(a) 4 volt (b) 8 volt


re

(a) 1 calorie/sec (b) 2 calorie/sec (c) 10 volt (d) 2 volt


[AIPMT (Prelim) 2011]
ouur

(c) 3 calorie/sec id) 4 calorie/sec.


ad

51. The supply voltage to room is 120 V. The


48. How many lamps each of 50 W and 100 V can be
YY

resistance of the lead wires is 6 O. A 60 W bulb is


connected in parallel across a 120 V battery of already switched on. What is the decrease of
internal resistance 10 Q, so that each glows to voltage across the bulb, when a 240 W heater is
nndd
Re

full power ? switched on in parallel to the bulb ?


FFii

(a) 2 ib)4 (a) zero volt (b) 2-9 volt


(c) 13-3 volt (d) 10-04 volt
(c)6 id) 8.
[JEE Main 2012]
49. Six similar bulbs are connected as shown in theFig.
3(CF).22 with a dc source of emfE and zero internal
52. A d.c. voltage with appreciable ripple expressed
as V = Vj + V2 cos cor is applied to a resistor R. The
resistance. The ratio of power consumption by the
amount of heat generated per second is given by
bulbs when (0 all are glowing and (it) in the situation
when two from section A and one from section B are
(b)
2V^^+V}
(a)
glowing, will be : 2R 2R

ia) 4:9 ib) 9:4


(c) id) none of the above.
(c) 1 : 2 (<0 2: I (NEET 2019) 2R

ANSWERS

45. ia) 46. (a) 47.(0 4S. ib) 49. ib) 50. (c) 51. id) 52. ib)
3/182 U Fundamental Physics (XII) PiViWi
53. A heater boils 1 kg of water in time f j and another The heat capacity of the metal is :
heater boils the same water in time t2- If both the
heaters are connected in parallel, the combination (a)
4P(T,-Tq9 ib)
4P(T,-Tq)
will boil the water in time
1.1 1.1
(a)
I ‘2
(b)
1*2
(c)
4P(r,-rp)^ (d)
4P(t;-V^
t, +t
h 1 2

. .2 ,2_.2
M ^^2 (JEE Advanced 2019)
(c) (.d) J [2_
58. Two resistors of resistance, 100 Q and 200 Q are
connected in parallel in an electric circuit. The
54. In the above question, if the heaters are connected
in series, the combination will boil the same water ratio of the thermal energy developed in 100 Q to
in time that in 200 Q in a given time is
(a) 1 : 2 ib)2:\

ww
t, t
K2 1*2
(a) (b) (c) 1 : 4 id) 4:1 (MEET 2022)
/l /2 ^1 "^^2
VIII. Kirchhott’s laws
r2 + f2 .2_.2

FF loo
(c) id) il t2
^1+^2 0l -^2) 59. When the switch S, in the circuit shown in Fig.

ree
3(CF).25, is closed, then the value of current I will
55. A fuse wire with a circular cross-sectional radius
be
of 0-02 mm blows with a current of 5 ampere.

rreeF
id) 3 A ib) 5 A
For what current, another fuse wire made from
the same material with cross-sectional radius of ic) 4 A id) 2A
ur
0-04 mm will blow ?

id) 14-7 A ib)5A


fofroF FIGURE 3{CF).2S
(JEE MAIN 2019)

(c)3A
ks
id) 1-5 A
I2
56. Terminal voltage of cell (emf = 3 V and internal 20V*->—VVW—
C
WW-«-*iov
kos o
YYouor

resistance = r) is equal to 2-5 V and heat loss in R A 2Q ..j 4n B

is given by 0-5 watt. Find the power loss in internal


BBoo

resistance. 2Q
r ee

FIGURE 3(Q).24
ad
oouur

3 V
r
Y

AAAV

R ^ V = 0
d
Re

MA/V
dnY
FFini

(a) 0-3 W ib) 0-5 W 60. In the circuit shown, the current in the IQ resistor
is [Fig. 3(CF).26]
ic) 0-1 W id) 1 W
(JEE Main 2020)
FIGURE 3(CF).26
57. A current carrying wire heats a metal rod. The wire 6 V 20
provides a constant power (P) to the rod. The metal
rod is enclosed in an insulated container. It is
I P-A/W
observed that the temp>erature (7) in the metal rod
changes with time (r) as 1 o 9V

r, = To (1 + p/>/*>) 30 30
where P Is constant with appropriate dimension -vw -vw
while Tq is a constant with dimension of temperature. Q

ANSWERS
53. ib) 54. id) 55. (a) 56. ic) 57. (a) 58. ib) 59. (/;)
CURRENT ELECTRICITY 3/183

(a) 1-3 A from P toe .(b)0A R


1
(c) 0-13 A from Qto P {a) E ib) E
R
i ^2
(d) 0-13 A, from P toe [JEE Main 2015]
P
61. In the Fig. 3(CF).27, Pi = 10 P2 = 20 Q, (c) E (d) E
1

P3 = 40 Q, P4 = 80 a and = 5 V, Vg = 10 V, P, +Pj
V(-=20'W, 15 V. The current in the resistance 64. What would be the value of E so that the
Pi1 will be
galvanometer G in the circuit shown in Fig.
3(CF).30 shows no deflection.

FIGURE 3(CF) 30

5Q<
10a _
20 Vt — —©■ I
J

w
S5Q $10n
-4*

sa c >ioa
(a) 0-4 A towards O (b) 0-4 A away from O

Flo
10V
(c) 0-8 A towards O (d) 0-8 A away from O I J
0 0

reeee
62. In the given circuit the cells have zero internal 20 a sa

resistance. The current (in amperes) passing through


(fl)5V (b) 10 V

FFr
resistance P| and P2 respectively, are :
(c) 15 V (d) 20 V
FIGURE 3(CF).28
65. In the circuit shown in Fig. 3(CF).31, the current

for
ur
flowing through 25 V cell is
FIGURE 3(CF).31
kkss
*■
Rl S 20 R2 > 20 n
Yo
10V
5Vj, 20^ 30Vl
oo

25V
eB

+ +
1
5Q lOfi 5fi ,iin
10 V 10 V
r
ou
ad

(a) 1,2 (b) 2,2


(c) 0-5.0 (a) 7-2 A (b)6A
YY

(d) 0,1
(JEE Main 2019) (c) 12 A (d) 14-2 A
ndd

63. Fig. 3(CF).29 shows a circuit with known 66. A network of resistances, cell and capacitor
Re

resistances P, Pj and P2. Neglect the resistances C (= 2 |i.F) is shown in Fig. 3(CF).32. In steady
of the conducting wires and internal resistance
Fi

state condition, the charge on 2 )J.F capacitor is


of current sources. The magnitude of Q, while P is unknown resistance. Values of Q
electromotive force £j such that the current I and P are respectively
through the resistance P is zero will be
FIGURE 3(CF).32
FIGURE 3(CF).29
2V R an c
10V 4V
JVWW
R
A 1A B
2pF
Rl
2n
E-X R2

Ei-X

ANSWERS

60.(c) 61. (b) 62.(c) 63.(c) 64. (b) 65.(c)


3/184 ’P'teuUc^ Fundamental Physics (XII)
(a) 2 |X C and 2 Q {b)A\iC and 4 Q
FIGURE 3(CF),34
(c) 4 C and 10 il (d) 8 (i. C and 4
67. Two batteries with emf 12 V and 13 V are 10O
connected in parallel across a load resistor of rAAAn
10 The internal resistances of the two batteries S
are 1 Q and 2 Q respectively. The voltage across I
the load lies between A B

(a) 11.6Vand 11.7 V (/?) 11.5VandU.6V


(c) U.4Vandll.5V (d) 11.7 V and 11.8 V (a) 10-2 ohm ib) 10-6 ohm
(NEET 2018) (c) 10-8 ohm (d) il-i ohm
(JEE Main 2011)
IX. Wheatstone bridge/Meter bridge

w
71. In the circuit, Fig. 3(CF).35, the resistance
between A and C is
68. In the following circuit, the current through the
resistor /? (= 2 Q) is / Amperes. The value of / is

Flo
3(CF).35

ee
Fr
for
ur
ks
Yo
oo

(a) 7 r/5 {b) r


(c) 3 r/5 id) r/5.
eB

72. During an experiment with a meter bridge, the


(a) I A (.b)2A galvanometer shows a null point when the jockey
r

(c) 2-5 A id) 3-5 A is pressed at 40*0 cm using a standard resistance


ou
ad

(JEE Advanced 2015) of 90 as shown in Fig. 3(CF).36. The least count


of the scale used in the meter bridge is 1 mm. The
Y

69. A resistance wire connected in the left gap of a unknown resistance R is


metre bridge balances a 10 12 resistance in the
nd
Re

right gap at a point which divides the bridge wire


FIGURE 3(CF}.36
in the ratio 3 : 2. If the length of the resistance
Fi

wire is 1-5 m, then the length of 1 Q of the h

resistance wire is :
-1 R 90 n
(a) I-OxIO m (b) 1-5 X 10-' m 3-^VW-E 3-WA-E
(c) 1-5 X 10"^ m id) 1-0 X 10-2
(NEET 2020) f
70. A meter bridge is set-up as shown, to determine — 40.0 cm ♦I
an unknown resistance X using a standard 10 ohm
resistor. The galvanometer shows null point when
(a) 60 ±0-15 a (b) 135 ± 0-56 n
tapping-key is at 52 cm mark. The end-corrections
are 1 cm and 2 cm respectively for the ends A and (c) 60 ± 0-25 Q (d) 135 ±0-23 12
B. The determined value of .X is : (JEE Advanced 2014)

ANSWERS

66.(c) 67. (b) 68.(a) 69.(a) 70. (b) 71.(a) 72.(c)


CURRENT ELECTRICITY 3/185

73. On interchanging the resistances, the balance


FIGURE 3(CF).38
point of a meter bridge shifts to the left by 10 cm.
The resistance of their series combination is I Idi. + f
How much was the resistance on the left slot
-AWW- <●>
before interchanging the resistance ?
(a) 990 0 {b) 505 .O
(c) 550 0 {d) 910 0 A
1
I B

(JEE Main 2018)


2
74. A resistance of 2 O is connected across one gap 3
of a meter-bridge (the length of the wire is 100 cm) VM\W- VAVW/-
R X
and an unknown resistance, greater than 2 O, is
connected across the other gap. When these fflA
resistances are interchanged, the balance points

ww
<●>
shifts by 20 cm. Neglecting any correction, the
unknown resistance is
{a) K {I2 - /j) and KI2 (b) K and K (I2 - li)

Floo
(a) 3 0 (b)4a
(c)5 0 (d)6Q. (IIT 2007)
(c) K(i2~ 10 and Kl^ (d) Kli and KI2
(AIPMT 2010)

ee
75. A uniform wire of resistance R is stretched
77. A battery of emf 2 V is connected across a long

reer
uniformly to n times. It is then cut to form five

rFF
wires of equal length. These wires are arranged uniform wire AB of length 100 cm and resistance
0*02 O cm"'. Two cells of emfs I V and 2 V are
as shown in Fig. 3{CF).37. The effective
connected as shown in Fig. 3(CF).39. If the
uur r
resistance between A and C is

ffoor
galvanometer shows no deflection at point J, the
length of wire AJ is equal to
sks
YYoo
2V FIGURE 3(CF).39
ooko

20
■V'AAr
eBB

J
A&
r

2V t
ouur

20
ad

I
YY

c D
nR nR
nndd

■WA—'
Re

(0- 10
1V
FFii

X. Potentiometer
{a)0 (b) 25 cm
(c) 50 cm (d) 100 cm
76. A potentiometer circuit is set up as shown in Fig.
3(CF).38. The potential gradient, across the 78. A potentiometer wire is 100 cm long and a
potentiometer wire, is K volt/cm and the ammeter, constant potential difference is maintained across
present in the circuit, reads 1 -0 A when two way it. Two cells are connected in series first to support
key is switched off. The balance points, when the one another and then in opposite direction. The
key between the terminals (0 1 and 2 (i7) 1 and 3, balance points are obtained at 50 cm and 10 cm
from the positive end of the wire in the two cases.
is plugged in, are found to be at lengths /j cm and The ratio of the emf is :
I2 cm respectively. The magnitudes, of the
resistors R and X, in ohms, are then, equal, (a) 5 :4 {b)1:4
respectively, to (c) 3 : 2 ((/) 5 : 1 (NEET 2016)
ANSWERS

73. (c) 74. (rt) 75. {b) 76. (i>) 77. {a) 78. (c)
3/186 ’4. Fundamental Physics (XII) lV»lWi
79. A potentiometer wire AB having length L and FIGURE 3(CF).40
resistance 12 r is joined to a cell D of emf e and
internal resistance r, A cell C having emf zll and e, r
internal resistance 3 r is connected. The length AJ
+

D
at which the galvanometer as shown in Fig. 3(CF).40
shows no deflection is: J
A[3- ms

11 +
(fl) L («
12 24 G

.L.3r
2

(c) —L (d) ~L
24 12
(J£E Main 2019)

m ; Multiple

ww
Choice Questions (with One or More than One Correct Answers)

80. A resistance of 4 is connected across a cell. (d) if /?j and R2 are interchanged, magnitude of

Floo
Then it is replaced by another resistance of 1 the power dissipatedin will decreaseby a
factor of 9 (HT 2009)
It is found that power dissipated in resistance in

ee
both the cases is 16 watt. Then
FIGURE 3(CF).41

reer
(a) internal resistance of the cell is 2 Q

rFF
(b) emf of the cell is 12 V
(c) maximum power that can be dissipated in the
uur r
I- 2vn Rl
external resistance is 18 watt

(d) short circuit current from the cell is infinite


ffoor
sks
(National Standard Exam, in Physics 2012)
YYoo
ooko

81. The charge flowing in a conductor varies with 24 V-=■


6ka R3 .5kn
R2
time &sQ = at - bP’, The current:
eBB

{a) decreases linearly with time


{h) reaches maximum and then decreases
r

(c) falls to zero after t = al2b 84. For the circuit shown in the Fig. 3(CF).42
ouur
ad

{d) changes at a rate of - 2h


FIGURE 3(CF).42
YY

82. Two bulbs consume same power when operated A sn c'


B
at 200 V and 300 V respectively. When these bulbs ?—Ws/W
nndd
Re

are connected in series across a d.c. source of 500


volt, then 25 V 4Q
FFii

ion; 2n
(a) Ratio of potential difference across them is
3/2
i5n 2fi

(/>) Ratio of potential difference across them is I — E


AAAA/V
D
4/9

(c) Ratio of power consumed across them is 4/9 (a) potential difference between points B and E
{d) Ratio of power consumed across them is 2/3 is 5 V

83. For the circuit shown in the Fig. 3(CF).38 ib) potential difference between points A and B is
10 V
(tj) the current I through the battery is 7*5 mA
{b) the potential difference across R3 is 18 V (c) the current in the 4 £2 resistor is 0-25 A
(c) ratio of powers dissipated in Rj and /?2 3 id) the current in the 4 £2 resistor is 0-5 A

ANSWERS
79. (c) 80. {a.b.c) 81. (a,c,d) 82. (b.c) 83. ia.d) 84. (a.c)
CURRENT ELECTRICITY 3/187

85. Heater of an electric kettle is made of a wire of 88. If E denotes electric field in a uniform conductor,
length L and diameter d. It takes 4 minutes to raise / corresponding current through it,Ujdrift velocity
the temperature of 0-5 kg water by 40 K. This of electrons and P denotes thermal power
heater is replaced by a new heater having two produced in the conductor, then which of the
wires of the same material, each of length L and following graphs is/are correct ?
diameter 2 d. The way these wires are connected
is given in the options. How much time in minutes
will it take to raise the temperature of the same
amount of water by 40 K ?
(a) 4 if wires are in parallel
{b) 2 if wires are in series
(c) I if wire are in series
{d) 0-5 if wires are in parallel

ww
(JEE Advanced 2014, Paper 1)
86, An incandescent bulb has a thin filament of
tungsten that is heated to high temperature by

Flo
passing an electric current. The hot filament emits 89. Two ideal batteries of emf V^ and V2 and three

e
black body radiation. The filament is observed to
resistances R^, R-^ and R^ are connected as shown

reree
break up at random locations after a sufficiently
in Fig. 3(CF).44. The current in resistance /?2

r FF
long time of operation due to non-uniform would be zero if
evaporation of tungsten from the filament. If the
uurr
bulb is powered at constant voltage, which of the
following statement(s) is (are) true ?
(a) The temperature distribution over the filament
foor FIGURE 3(CF).44
ks s
R
is uniform. ViT R2
Yoo
ooook

—V/A—
(6) The resistance over small sections of the
filament decreases with time iV2
eBB

R3
(c) The filament emits more light at higher band ^-VWV-
of frequencies before it breaks up
rr

{d) The filament consumes less electrical power


(a) V| = and R^=R^ = R^
ouu
ad

towards the end of the life of the bulb 2


{b) Vj = V2 and = 2 /?2 =
YY

(JEE Advanced 2016)


87. Which of the following statements is/are correct ? (c) V^= 2 V2 and 2 /?, = 2 /?2 = R3
nndd
Re

(a) If n identical cells are connected in series and


(d) 2 V^ = V2 and 2 /?, = /?2 = /?3
then the battery thus formed, is short circuited
Fii

by a conduction wire, current through the wire (JEE Advanced 2014)


will be independent of n. 90. A uniform wire of resistance R is shaped into a
(h) If n identical cells are connected in parallel regular n-sided polygon (n is even). The
and then the battery thus formed, is short equivalent resistance between any two corners can
circuited by conducting wire, the current have :
through the wire will be proportional to n. (a) the maximum value R/4
(c) If n identical cells are connected in parallel
{b) the maximum value R/n
are then the battery thus formed, is short
circuited by a wire having a constant n-l
resistance, the current through the wire will (r) the minimum value R
increase as n increases.

(d) none of these. (d) the minimum value R/n


ANSWERS
85. (h,d) 86. (<●,</) 87. (a.h.c) 88. (a.h.d) 89. (a.h.d) 90. (a.c)
3/188 Fundamental Physics (XII) IV»1WI

91. For the circuit shown in Fig. 3(CF).45, (fl) the potential of point Z> is 40 V
{b) the currents in the sections AD, DB, DC are in
FIGURE 3(CF).45 Ei the ratio 3:2:1
Ri
I—WA—1 (c) the currents in the sections AD, DB, DC are in
the ratio of 1 : 2 : 3
^2 R2 (d) the net work draws a total power of 200 W.
I—ww
A B 93. For the resistance network shown in the Fig.
E3 3(CF).47, choose the correct option(j)
R3
I—WW-'
FIGURE 3(CF).47

p lo 2 Q O
(a) Equivalent internal resistance R is given by 2a 20.

1-_L _L _L

ww
10 1 n
R R
1 ^2 ^3
(b) Equivalent em/of the battery is
40
Q ■WVWtr
40 ~
4Q

Floo
£ = (£j + £2 + £3)/3. i.Il Iv

ee
£j/?2 + ^2^1
(C) If £3 = equivalent e.m..f of

reer
12V
+ /?2

rFF
the battery will be equal to £3. (a) The current through PQ is zero
uur r
ib)I^ = 3A
(d) Equivalent em/of the battery not only depends
upon values of £], £2and £3 but depends upon
ffoor
(c) The potential at S is less than that at Q
(d)l2 = 2 A (UT 2012)
values of /f]. /?2 ^3 ®lso.
sks
94. Two resistors having equal resistancesare joined
YYoo
92. In the net work shown in Fig. 3(CF).46, points A.
ooko

B and C are at potentials of 70 V, zero and 10 V inseries and current passed through the combi
respectively. Then nation. Neglect any variation in resistanceas the
eBB

temperature chagnes. In a given time intemval


(a) unequal amounts of thermal energy may be
produced
r
ouur
ad

(b) equal amounts of thermal energy must be


produced
YY

(c) the temperature may rise equally in the


resistances
nndd
Re

(d) the temperature must rise equally in the


resistors.
FFii

aa Multiple Choice Questions (Based on the given Passage/Comprehens ion)

Each comprehension given below is followed by some multiple choice questions. Each question has one
correct option. Choose the correct option.
S7npf5l!*?o'^5olll II
A cylindrical copper of a gas. The average thermal speed of the
conductor A£ of length /, area of cross-section free electrons in random motion at room
a has large number of free electrons, which temperature is of the order of 10^ ms"^ When
at room temperature move at random within a potential difference V is applied across the
the body of the conductor, like the molecules two ends of a given conductor, the free

ANSWERS

91. (a,c,d) 92. (a.b.d) 93. (a.b,c,d) 94. (b.c)


CURRENT ELECTRICITY 3/189

(a) drift velocity of electrons increases when


electrons in the conductor experience a force switch is closed.
and are accelerated towards the positive end (b) electrons are accelerated towards the positive
of the conductor. On their way, they suffer end of the conductor and their velocity
frequent collisions with the ions/atoms of the increases towards the other end of the
conductor and lose their gained kinetic conductor
energy. After each collision, the free electrons (c) the drifting of electrons takes place at the entire
are again accelerated due to electric field, length of the connecting wire. This electric
towards the positive end of the conductor and
effect propagates with the speed of light.
lose their gained kinetic energy in the next
collision with the ions/atoms of the conductor. (d) the electrons move towards the positive end
The average speed of the free electrons with and protons of conductor move towards
which they drift towards the positive end of negative end of the conductor.
the conductor under the effect of applied
(gaapdiMEflsoD

ww
electric field is called drift speed of the electrons. A wire of length 12 cm ;
resistance 12 £2 and of uniform area of
Answers the following questions crosssection is cut into twelve equal parts,

Floo
95. When a potential difference is applied across the which are connected to form a skelton cube.
two ends of the conductor, an electric field exists A cell of emf 2 V is connected across the two
(a) outside the conductor diagonally opposite corners of the cube.

ee
(b) inside the conductor Using kirchhofT’s laws of junction and loop.

reer
(c) both outside and inside the conductor

rFF
(d) no where Answers the following questions
96. The motion of electrons inbetween two successive 99. The effective resistance of the circuit is
uur r
collisions with the atoms/ions follows
(n) a straight path (b) circular path
(a) 4/5 Q
(c) 6/7 Q
ffoor (b) 5/6 Q
(d) 7/12 £2
sks
(c) elliptical path (d) curved path 100. The current drawn from the battery is
YYoo
97. The drift speed of the electrons depends on (a) 2-5 A {b) 24 A
ooko

{a) dimensions of the conductor (c) 2-3 A (d) 34 A


eBB

(b) number density of free electrons in the 101. The maximum current flowing in an arm of
conductor network is
(c) both (a) and (b) (d) none of these above (a) 04 A (b) 0-8 A (c) 1 -2 A (d) 24 A
r

98. Drift speed of electrons in a conductor is small


ouur

102. The minimum potential difference across an arm


ad

(= lO^ms"') when the switch is closed, the bulb of network is


YY

at a distance glows immediately. It is so because (a) 04 V (b) 0-8 V (c) 1 -2 V (d) 24 V


nndd

09
Re

Matching Type Questions


FFii

DIRECTIONS. In each of the following questions. Match column I and column II and select the
correct match out of the four given choices.

103. Column I Column II

(A) The standard resistance coil (p) of nickel-chromium alloy


(B) The best connecting wire (q) of tin-lead alloy
(C) The heating wire (r) of silver metal
(D) The fuse wire (s) of copper manganese nickel alloy

{a) A-p ; B-^ ; C-r ; D-5 (b) A-q ; B-r ; Cs ; D-p (c) A-r ; ; C-p ; D-q (d) As ; B-r ; C-p ; D-q

ANSWERS
95. ib) 96. id) 97. (d) 98.(c) 99. {b) 100.{b)
101. (6) 102. (a) 103. id)
3/190 Fundamental Physics rxiHCCTin
104. There are physical quantilies in column 1 and dimensional formulae in column II
Column 1 Column II
(A) Current density (p)
(B) Conductance iq) |M-' L-'-’ T'-’ A-J
(C) Electrical conductivity (r) IM‘'L-“T"A'1
(D) Specific resistance (.y) |M-'L-2t^a2j
(a) A-p ; ; C-r ; D-5 (b) A-q ; B-r ; C-^ ; D-p (c‘) A-r ; B-y ; C-p \ D-^ {<£) A-r; B-i ; C-q ; D-p

p q r s

Matrix-Match Type Questions r'-'i „,T

A
!© O o ©
DIRECTIONS. Each of the following questions contains statements given B
[0 O o ©

w
in two columns, which have to be matched. The answers to these questions
have to be appropriately bubbled. If the correct matches are A-p, A-q ;
B-r ; C-r, C-s and D-s, then the correctly bubbled matrix will look
c
1© r® oj m

Flo
like the one shown here. D
©©©0

reeee
105. There are physical quantities in column I and the statements are in column II :
Column 1 Column 11

FFr
(A) Current (p) Motion of electrons in a closed path
(B) EMF (q) Motion of electrons through a conductor without any collision
(C) Resistance (r)
for
Motion of electrons in a definite direction
ur
(D) Superconductor (.v) The collision of electrons while moving through the conductor
kkss
106. Column 1 Column II
Yo
(A) Max. potential difference between two points of a circuit (p) Voltmeter
oo

fB) EMF of the cell in a closed circuit (q) Potentiometer


eB

(C) The capacity of supplying the current in the circuit (r) Watt-hour
(D) The electric energy consumed in the circuit (.y) Ampere hour
r

VI.
ou
ad

Integer Type Questions A B C D


YY

DIRECTIONS. The answer to each of the following questions is a single 0) oTfoj (o


digit integer, ranging from 0 to 9. If the correct answers to the question
ndd

OOOQ
Re

numbers A, B, C and D (say) are 4, 0, 9 and 2 respectively, then the


correct darkeningof bubbles should be as shown on the side : © © @ ©
Fi

107. The length of a potentiometer wire


is 10 cm. A cell of emf £is balanced FIGURE 3(CF).48 5 a © @ @ @
at a length 10/3 cm from the AAAAAAr
0 © ®@
positive end of the wire. If the
length of the wire is increased by 5
+

7V T
® ®© ©
cm. at what distance (in cm) from
positive end will the same cell give
R — 12V @ © © @
a balance point ? ©00©
108. Refer Fig. 3(CF).48, for what value
of R (in ohm) will the current in ©: © © :<|)
galvanometer C be zero ? 9H9H9J>&9

ANSWERS
104. (</) 105. A-p. r ; B-p ; C-.y; D-q 106. A-p. q ; B-p. q \ C-s ; D-r
107. 5 lOH. 7
CURRENT ELECTRICITY 3/191

109. Total momentum of electrons in a straight wire of length 1000 m carrying a current of 70 ampere is
about X l(r^ Ns.

110. A battery of emf E and internal resistance r is connected to an FIGURE 3(CF).49


external resistance R. The maximum power in the external circuit
■*AAAAAr
is 9 W. The current (lowing in the circuit under the conditions is r
3 A. What is the value of E in volt ?
111. In the circuit. Fig. 3{CF).49, what should be the value of r in 20 40
ohm so that power developed in the resistor r will be maximum ? -WVWv WWW
112. When two identical batteries of internal resistance 1 each are
connected in .series across a resistor /?, the rate of heat produced
30
in R is 7|. When the same batteries are connected in parallel ●VAAAAr
across R, the rate is J2. If J\ = 2-25 J2, then the value of R in V
ohm is : (JEE 2010)

ww
VII. Assertion-Reason Type Questions

FF loo
FOR MEDICAL STUDENTS 116. Assertion. In a meter bridge experiment, null

ee
DIRECTIONS. Ineach of the following questions point is obtained at a point for an unknown
read the two statements and choose if resistance. When an unknown resistance is pul

eFr
A. both Assertion and Reason are true and the inside an enclosure maintained at a higher
temperature, the null point can be obtained at the

Frre
Reason is correct explanation of the Assertion.
r ur
same point as before by decreasing the value of
B. both Assertion and Reason are true, but Reason
is not the correct explanation of the Assertion.
fforo
the standard resistance.
ks
C. A.ssertion is true, but the Reason is fal.sc. Reason. Resistance of metal increases with
YYouo
D. both A.s.scrtion and Reason arc false. decrea.se in temperature.
koso

(n)A (h)Q (c-)C (rf)D


113. Assertion. The bending of an insulated wire
BBoo

increases the resistance of wire. (AIIMS 2015)


r ee

Reason. The drift velocity of electrons in bent 117. Assertion. The effective resi.stance of the network
wire decreases, 4
between P and Q is — r
ad
oouur

(a) A {h)B (c)C (d)D


Y

114. Assertion. In parallel combination of electrical


appliances, total power consumption is equal to FIGURE 3(CF).50
dnYd

the sum of the powers of the individual appliances.


Re

Reason. In parallel combination, the voltage


FFini

across each appliance is the same, as required for


Pi—WW -VWr—
the proper working of electrical appliance.
(a) A ih)B (c)C id)D
115. Assertion. Etich bulb in a frill of 20 bulbs in series
Q
when connected to supply voltage will emit more
light than each bulb in a frill of 19 bulbs In .series
when connected to same supply voltage. Reason. Symmetry can be applied to the network
Reason. Each bulb in a frill of 20 bulbs in series with respect to centre.
will get more voltage than that in a frill of 19 in) A (b)B (c) C (d)D
bulbs.
(AIIMS 2015)
(a) A (h)B {<-) C (^/)D

ANSWERS
109.4 110.6 111.2 112.4 113. (r/) 114. («) 115. (</) 116. (r/) 117. (r/)
3/192 “Pfutdee^ 'a Fundamental Physics (Xll) tTOWl

FOR ENGINEERING STUDENTS connections, whereas an electric bulb has a two


pin connection.
DIRECTIONS. In each of the following questions
read the two statements and choose if Statement-2. Three pin connections reduce
heating of connecting cables
A. Statement-1 is true ; Statement-2 is true ;
(a) A (b)B (c)C (d)D
Statement-2 is correct explanation of Statement-1.
B. Statement-1 is true ; Statement-2 is true ; 120. Statement-1. A laser beam 0-2 W power can drill
Statement-2 is not a correct explanation of holes through a metal sheet, whereas 1000 W
Statement-1. torch-light cannot.
C. Statement-1 is correct and Statement-2 is false. Statement-2. The frequency of laser light is much
D. Statement-1 is false and Statement-2 is true. higher than that of torch light,
(a) A (/j) B (c) C (d)D
118. Statement-1. When cells are connected in parallel
to the external load, the effective emf increases. 121. Statement-1. When the cell is in the open circuit,
Statement-2. All the cells will be sending the there is no force on a test charge inside the

ww
current to the external load in the same direction, electrolyte of the cell.
(a) A (/))B (c)C {d)D Statement-2. There is no field inside the cell,
119. Statement-1. Electric appliances with metallic when the cell is in open circuit.

Floo
body ; e.g. heaters, presses etc, have three pin (a) A (/))B (c)C (d)D

ee
reer
VIII. Multiple Choice Questions (Based on Experimental Skills)

rFF
uur r
122. Resistance of a given wire is obtained by connected to one gap of a metre bridge ; while a
measuring the current flowing in it and the voltage
difference applied across it. If the percentage
ffoor
resistance /?2 connected in the other gap. The
balance point is at 50 cm. Now when 10 O
sks
errors in the measurement of the current and the resistance is removed, the balance point shifts by
YYoo
voltage difference are 3% each, then error in the 10 cm. The value of R,1 is
ooko

value of resistance of the wire is (a) 10 Q (/>) 20 Q


eBB

(a) 6% (b) zero (c) 40 Q id) 60 n


(c) 1% (d) 3% 126. A uniform wire of resistance 20 12 having
resistance 1 f2 m"’ is bent in the form of a circle
r

123. Three unequal resistors in parallel are equivalent


ouur
ad

to a resistance 1 ohm. If two of them are in the as shown in Fig. 3(CF).51. If the equivalent
resistance bet- ween A and B is 1-8 £2, then length
Y

ratio of 1 : 2 and if no resistance value is frac


of the shorter section is
tional, the largest of three resistances in ohm is
nddY

(a) 4 (b)5
Re

FIGURE
(C)6 (^10
FFiin

124. In an experiment to measure the internal resistance


of a cell by a potentiometer, it is found that the
balance point is at a length 2 m, when the cell is
shunted by a 5 £2 resistance and at a length of
3 m, when the cell is shunted by a 10 £2 resistance.
The internal resistance of the cell is then
(a) 1 £2 (b) 1-5 £2
(c) 10 £2 (d) 15 £2
125. An unknown resistance is connected in series (a) 1-8 m (b) 2 m
with a resistance of 10 £2. This combination is
(c) 3-6 m (t/) 4 m

ANSWERS

118. id) 119. (c) 120. (c) 121. (c) 122. (a) 123. (c) 124. (c) 125. ib) 126. (b)
CURRENT ELECTRICITY 3/193

For Difficult Questions

El; Multiple Choice Questions ('"'‘f’ O"® Corroct^^^^


distance iKrn R,.xR 3x10-^x125x10-^
1. As, T = A1 Fe
R
speed V
n /?..+/?
A1 Fe
"3x10-5+125x10"^
2nGr.n^
0
h^xle^2 nh 0 1875x10-5 a 1875
iiCi
Kme^ xe^ me
4
64 64

me^ 6. Let / be the original length of the wire. A portion


, e exme^ of the length of wire is stretched so that the final
Current 4^2„3^3 “4^2„3;,3

ww
length becomes 1-5 /, Fig. 3(CF).52.
So /oc C®
^ FIGURE 3(CF).52
2. Average energy = workdone -eE\
A

Flo
2eV = e£x4x I0-* Ai
u

ee
E = ?-- = 5xloAin-‘ (l-x)

rere
or
4x10"^ m

r FF
/ = ^ = 3r2 + 2r + 5 A
3. Ai A2
uurr
dt

dQ = Ot^ + 2t + 5)dt foor \1


M—X *■
ks s
t=2s \2
Let /?j, /?2 be the resistances of the wire before
Yoo
(3i^+2t + 5)dt= —+ —+ 5r
ooook

Q = and after stretching. Let Aj be the area of cross-


' I 3 2 >0
/= 0 section of the original wire and A2 be the area of
eBB

= {P+r2+5,)^ =22C cross-section of the portion (1-5 /-x) of the wire.


4. Here, / = 1 m, A = 0-5 x 10"^ m^ = 5 x 10“^ m^ On stretching, the volume of the wire remains
n = 8 X 1Q22 cm"5 = 8 X K)28 m-^. / = 1 A
unchanged, so / Aj =xA^ + (1*5 /-x) A2
rr
ouu
ad

I or JLz^a 1
V
a-51-x)
YY

d ~
nAe

I InAe px p(l-5/-vC)
Also, R1 — and /?-, = —
nndd

Time, t — — “
Re

/ A A
As per question R2 = AR^
Fii

1x(8xI02^)x(5x10-’)x 1-6x10-^^
1 4pl _px p(l-5/-j:) _p;c p(l-5/~ j:)^
= 6-4 X 10^ s ■ A, A, A2 ^1 (l-x)A^
5. For Aluminium
X 7
On solving it we get, — = -
R
Pai^ _ 2-7x10-^x5Qx1Q-5 / 8
Fraction of wire stretched
A (49-4)x10-5
{l-x) x^ 7^1
= 3 X 10-5 a 1-- =1--
/ / 8 8
For Iron
7. Let the potential at points A, B, C and D be V^,
1-0x10-’x50x10"5 Vg, Vq and Vjy respectively.
R
Fe “ -6
= 125 X 10-5 n
4x10 Pot. diff. between the points B and A
Both will act as two resistances joined in paralle = V^-V^=10V
3/194 Fundamental Physics fxmprawn

13. Let /, A be the length and area of crosssection of


For Difficult Questions
each wire in series as shown in Fig. 3(CF).53.
As, earth potential is taken zero, so =0 FIGURE 3(CF).53
Vfi-0=10V or Vg=10V /

Pot. dilf. between the points C and D Cu Fe

= Vc- 1 x4 = 4V
Vc-0 = 4 or Vc = 4V V
Pot. diif. between the points B and C
= 10-4 = 6V 1
Given, Pc«=
Resistance R = - = 6ft
1 As V=7R = /£^
A
8. The value of given carbon resistor is

w
4*7 /
R = (47±47)kn = 47 kCl±
47
xl00%
Forcopper, ''cu = ^pQi ^ >
= 47x 103ft±10%

Flo
The colour code for 4, 7, 3 and 10% is yellow- I 6A

e
violet-oraiige-sflver. Thus option (6) is true.

reee
.''Ffe.^Ppe
9. For ideal voltmeter, the resistance is infinite and for
For iron, = ^Pr^ ; ^2 =

FFr
/ A
the ideal ammeter, resistance is zero.
1 _
1
10
V.I =/,xl0 = —xlO=IOV = -6 ; Hence, E| < E2
^2
for
ur
I IA
10

10 14. /?! =tan 6 = /?o(I t«^|)


V2=/2><10 = —xI0 = 10V .-. V, = V2
kss
10 and /?2 = cot e = /?Q (1 + a 72)
Yo
lOV lOV .*. cot 0 - tan 0 = /?o “ ^2) “ ^
oo

10 ft
= 1 A and /j = 10 ft
= 1 A
= /?oCt(r2-r|)
B

.-./,=/2 1
or T2-7; = (cot 0-tan 0)
re

10. R = p/ p/ p/^ aR^


A~ vn~ V
ou

1 /'cosO sinO' 2cos20


ad

dR ^ 2dl
YY

R I
= 2 X 0-5 = 1
a/?Q(^sin0 COS0 j a/?^sin20
dR = -^ cot 20 or T’2 - Tj oc cot2 0
nd

% change in re.sistance = —
Re

“«0
11. By colour coding, the resistance of carbon resistor 15. According to Ohm’s law
Fi

isR = 50x I02s1±5%.


I VA

As power, P=fi Rot / = (^= CL_ V = //? = /p-


A
or
P = 77
R ■ysoxio^ .
VA V
= 20x 10-^A = 20inA Nowz&J = Anevj so, p =
12. Here, n = 8-5 x I028 m"3 ; lAneVj IneVj
X = 25 fs = 25 X 10-'5 s 5

Resistivity, p =
m
^ O-l x8 X 1028x(1-6x 10-^9) X 2.5x10-^
ne^x = l-6xl0^ftm
(91xl0~^*) 16. Let /?| and /?2 be the resistances of A and B at 0“C
(8-5x1028)x(1-6x10-‘^)2x(25x10-‘5) and /?[ and be the resistances of A and B at
= 1 -67 X 10*^ ftm a* 10"* ftm /“C.Then
CURRENT ELECTRICITY 3/195

i^iiisinrL:^/ Epic b
JFor Difficult Questions

/?2=«/?i ; /?,' = /?, (1+a, f); /?J = /J2(l+a2^)


When resistances are connected in series, then

w
/?;=/?[ +=/?! (1 + a,/) +n/?i (l + 02 0
or/?'=/?! [(1 + n) + (tti + n 0t2)/] ...(/)

Let a_y be the temp. coeff. of resistance of .

e
combination of two resistances in ^ries.
Standard equation for series combination is

e
wr
■ V —V
. , Current, / si-S—t
/?;=(/?j + /?2)(l+a,/) = (/?j+n/?i)(l+a^f) ;

lloo
r
R ■ ‘
= /?i(l+n)(l+a,/) . The electric field intensity at the location of shell

F
= R^[(l+n) + (l+n)a,t) ...00

FFu
_ J 7/4 p/
Comparing (/) and (ii), we have IS . £ « - = — = —2
G 1/p
0|-Mia2
(a,+n02) = (l+/i)a, or a, = l+» where y is the current density and G is the

rese
electrical cohductiyity of the conducting material.
17. In parallel combination of resistances.
uro
k
Fr
/■

o
R^xnR^ nR^ £ = _fi-X P
410-2 [ R j 4ro;2 p r I
foo
R.1 +nRl 1+n

fr
fh
.. ^ J
kso
, _ R^xR^ _ R^ (l + a^t)xnR^(l + a2t)
Y
R
Ri+R^ Rl(l + a^t) + nRl(l + a2t) "TTl 1
Y
B
Yo
oo

«£j(l + aj/)(l + a2t)


«● ●
eBr

...O') As current I. is zero for r < and r > r^,, so


(l + a,f) + «(l + 0C2t)
e

. variation of £ and r will be represented by the


uru

If ttp is the temp, coeff. of resistance for parallel ' curve shown in option (fr).
od

combination of resistances, then


1$. Refer.to Pig. 3(GF).55 given below ;
o
ad

n/?,
n

R; = (l+«) [l+ttpO . FIGURE 3(CF).55


Y

...00
A"
di

A Oi 02
From 0) an<l (*0» we get
Re

-V7
FF

u
in

nR
1 n/?,(l + a,0(l + a20 K / —/—H
(1 + n) '■ (l+a^t)+n(l + a2t) Effective resistance of two wires iii series is
/
On solving, we get, l + «
= £, +/J,^ =-^ + -—
Oj A O2 A
21
18. Consider a spherical shell of radius r and
thickness dr as shown in Fig. 3(CF).54. o„A
eq ●

pdr
Resistance of this shell is, d£ =
27 1 I
4nr^
Total resistance.
gLa''
eq
a ®2. A0,02

2c^
R =
^ J Anr^
'-a
A%lr^ r^,j

A
3^196
Fundamental Ph3rsics (XII) VOL.I

[M liNtTLS^/ E X mm
When these four wires are connected in series as
For Difficult Questions
shown in Fig. 3(CF).58(!>), then the effective
resistance
20. Here, 7=5A,e = 60“,A = 2m2
p= 11 X KHsI units. /?, = /? + /? + /? + /? = 4/? = 4x 10 = 4*0a

5
Cunen. density, = = FIGURE 3(Q).S8
R
AM/V

w
5 R
= 5A/m2 AWV
2x1/2
A R B
Electric field, £ = yp = 5x(llx 1(H) VWV
= 55xl(Hv/tai O R

e
dVWV
21. Refer to Fig. 3(CF).56, the resistance between C

re
row
and E is. R R R R
A«—VWV—^VWV VWV VWV—*B
_(4 + 8)x6 O

eeF
=4n
"(4 + 8) + 6

ullo
CE ~

FF
23. In the upper branch of the circuit, equivalent

srr
Ixl
resistance, Req

roF
+ 2 =25 Q
1 + 1

k
Voltage across upper branch, V = 1 volt
uor
ofof Current in upper branch, / = —= — =0-4 A
V 1

R 2-5
kos
YYo
This current is equally divided between two 1 Cl
eerBB
oo

resistances each, so
rY

0-4

Refer to Fig, 3(CF).57, the resistance between A ^,=Y =0*2A


u

and B is. 24. When n resistors are connected in series to the


battery, then current in circuit is
oou
d
ad

^ab = 4 + 4 + 8 = 16Q
E
nY

/ =
FIGURE 3(CF).S7 nR + R ...(0

When n resistors are connected in parallel to


nid
Re

4C1 the battery, then current is


FFi

E
10 7 =
(R/n) + R ...(«■)
A B
Dividing (ii) by (i), we get
22. Since all the four wires are identical and of the
(n + l)R (n + 1)
same material, so each wire will be of the same 10 = = «
n
\

resistance (say R). -+1 R (1 + n)/n

When these four wires are coimected in parallel


So n = 10
Fig, 3(CF).58(a), then effective resistance Rp is
25. Let R be the equivalent resistance. Since the
network is infinite. The addition of one more unit
of two resistances Rj and /?2 will not alter the
total resistance of network, i.e., it should be R.
Given, Rp = 0-25 Q, so /? = 4 x 0-25 = 1 -0 G Fig. 3(CF),59.

A
CURRENT ELECTRICITY 3/197

Hi ISItTt sTe xigi^ I.O. M3l FIGURE 3(CF).60


For Difficult Questions
E r 1
WVV F—i/J
FIGURE 3(CF).59
R1 '*1
Wy P vwv

R2 R V/2

(/L-<I VWV—^/
Now /? = /?.+
R^R ...(0
E
1 / =
«2 + /? r+ T2
Since the voltage is halved after each section,

ooww
Potential difference across P and S,
R^R Vs-Vp = Ir2
^1 = ...00
Charge on the capacitor,
R^+R
Q = C{Vs-Vp) = CIr2
Putting the value of R^ in 0). we get
E CEr^

e
2RyR = Cx r, = ^

ree
R = or R —2 /?2 (r + rj) r + rj

rFl
R^+R 29. Let jc be the equivalent resistance of infinite net

Fre
Ri = R ...070 work. Since the net work is infinite, the addition

rrF
of one more unit of three resistances each of value
From equation (i7), R,I = ^^R ...Ov) 1 Q across the terminals will not alter, the total
2R 2
ouur
sffoo resistance of network, i.e., it should be x.
Therefore, the network would appear as shown
in Fig. 3(CF).61 and its total resistance should
okks
From (ii'O and (iv). remain x. Effective resistance between A and B is
Yo
R
ooo

1
Y

FIGURE 3(CF).61
BB

26. When is closed, the resistance i?j is ineffective.


in
E ^ VsA^
rr e

100+r 2(50+r)
ouu

or r =0 in X
ad
Y

When Ky is open, then


VW
dY

E E B

2 2(i2,+50) 2x100
innd
Re

IXJf
or = SO Q x = l + +1=2+
1 + x 1 + ;:
FFi

27. In the branch PR, the effective resistance = r/3.


In the branch QR, the effective resistance = r/2. On solving we get, x = (1 + V3)li
r r 5r
Resistance in the branch PRQ = - + “ = 30. A fully charged capacitor draws no current.
● 3 2 o Therefore, no current flows in arm GHF. So the
The ersistance of branch PRQ is in parallel to resistance R of arm HF is ineffective. The total
resistance of branch PQ. Thus effective resistance resistance of other ersistors in circuit is
^ _ rx5r/6 5r {R + R)xR 6x3
between P and Q = R' = + R = + 3=5Q
r + 5r/6 11 (/? + /?) + /? 6 +3

The equivalent resistance between Q and Rot R E 15


and P will be less than that between P and Q, Total current, / = *;^
R
= -t
5
= 3A
28. At steady state no current flows through Since in parallel circuit, the current divides in the
capacitor C. Thus the current through the inverse ratio of ersistance so current in arm ASGZ>
network will be as shown in Fig. 3(CF).60. Then = 1 A and current in arm AD = 2 A.


3/198
Fundamental Physics (XII)QSS]
nnncn
The equivalent resistance between A and B is
For Difficult Quentlons
12x6 6x12
R = 4 + 4 = 8(2
eg
So Pot. diff. between G and D 12 + 6 6+12

= Vc-V^=lx3 = 3V When switch S is closed, the equivalent circuit


Pot. diff. between D and F will be as shown in Fig. 3(CF).63(fl) and (b).
= V^-Vfr = 3x3 = 9V The equivalent resistance between A and B is

R
(4 + 4)x8
= 4Q
= 3 + 9= 12V eg
(4 + 4) + 8
Pot. diff. across capacitor = - Vyr = 12 V FIGURE 3(CF).63
31. Given, : /j : /3 = 2 : 3 : 4 = 2 /: 3 /: 4 /

oww
12 0 60
r, : /2 : =3 :4 :5 =3 r:4r: 5 r (— r-VvVv—I
A B
As, resistance of wire, R =
I—W/v—* I—vwv-*
60 120

ee
p2/ 2 p3/ 3
. R,1 =
= 9^; ^2 = o = —^ ● e AAAAr

FFrlo
7t(3r)“ 7c(4r)" 16

r
80

rF
ee
40 40
^ 7C(5r)2 25 A >vvw AAAAr B
ouru
rF
V V V
and ~ —
AA//V-
R
1 ^3 o
ffosor
os k 80
1
h'- h - h ~ 34. Let there be N resistors each of resistance r. When
/?. ^2 ^3 2R ' 3R'4R resistors are connected in series, their total
ook
resistance is, S = N r
Yo
9 16 25
Y
— = 54 : 64 : 75
2 3 4 When resistors are connected in parallel, then total
Bo

resistance is, P = r/N


reeB

32. Due to symmetry of network, there will no cun ent


in arms BF and CE. Therefore, the effective Given, S = n P N r = rix r/N
resistance of arms AB, BC and CD in series,
oouY

or = n or n = N^
ur

/?, = 2 + 2 + 2 = 6H.
Since N is an integer and combination of resistors
ad

The effective resistance of arms AF, FE and ED will not be possible if V= 1. So minimum value
dY

in senes. /?2 = 2 + 2 + 2 = 6n. of N =2, then, n = 2^ = 4


Between points A and D, /?, and R^ are in parallel, 35. Here,r = 4 W,/ = 0-2 A,F = 2I W
nidn

the effective resistance is


Re

If e is emf of the cell, then


/?! /?2 _ 6x6 £ = / (F + r) = 0-2 (21 + 4) = 5 volt.
FFi

R = = 3(2
R^+ Rry 6+6 Rate at which chemical energy is consumed
= e/=5xO-2=IJ/s
3V 36. Terminal pot. diff. V=e-/r = -/r+e ...(/)
Current, I = = 10A
3Q It shows as I increases, V decreases.

33. When switch S is opened the equivalent circuit The variation of V and I will be a straight line
will be as shown in Fig. 3(CF).62. with negative slope as shown in Fig. 3(CF).64.
VA FIGURE 3(CF),64
FIGURE 3(CF).62
12n 6n
Wv-n
A ●●B

I—VWV—l I—VvVv—*
6fi 120


CURRENT ELECTRICITY 3/201

51. Here,V=I20V;P = 60W


For Difficult Questions
(120)2
R = 240n
R 2K bulb p 60
R
“T 3 ~ 3 FIGURE 3(CF).69
£2 3E2 120 V 120 V
Power consumed, .PI = ...(0
R 2R
eff
240a 6Cl 240a 6Cl
When two bulbs from section A and one bulb
from section B are glowing, Fig. 3(CF).67, then 1 f
effective resistance of the circuit is
6on
FIGURE 3(CF).67
e O
R

w
I—V/vAf—1 R Refer to Fig. 3(CF). 69(a), /?^ = 240 + 6 = 246
V/vV 120
R x240= 117 07 V
Voltage across bulb, =
■VAV 246

FF loow
Refer to Fig. 3(CF). 69(Z»),

ee
(120)2 = 600.
Resistance of heater, /^heater “ 240

Fr
E
Equivalent resistance of circuit is
R 3R 240x60

rer
R' =6 + = 6 + 48=54 n
ur
R fr — — +/? —
eff 2

Power consumed, p. =
2

£2 2 £2
...(2)
fofr Fo
eq

Voltage across bulb,


240 + 60

=
120
x48 = 106-66 V
ks
f ~ 3£/2 " 3R 54
YYouro
Change in voltage = 117-07 - 106-66 = 10*04 V
s oo

P 1 3E2/2R _ 9 52. Heat generated per sec


2E2/3R ~ 4
v2 (Vj + V2coscor)2
BBook

H =
r ee

50. As, P = fiR or i = 4pTr R R


Current through 9 Q resistor in Fig. 3(CF).68 cos2 cot + 2 V, cos (or
oouru
ad

given below is
R
Y

A=Jf=^A
●7 1
Average value of cos'^ (or over a cycle = —
Ynd
Re

As resistors 9 O and 6 are connected in parallel, Average value of cos cot over a cycle = 0
therefore, potential difference Heat generated per second,
FFindi

- Vg = 9 /i = 6 /2 or /2 = 9 7,/6 =
9x2
= 3A v2+y2xl + 2V^V^xO 2Vf+V}
6 H =
» 2 2
R 2R
FIGURE 3(CF).68
53. Let £j, /?2 be the resistance of two heaters and H
is the heat required to boil 1 kg of water. If V is
A B
the voltage applied to heater, then
h 60
♦=VAW-1 V2 V2
(I1+I2) O1+I2) H = (I)
1 “
RI
■V//A—
20
V
or or A = ii Hi)
Current drawn from the battery = /j + 73
R
1 /?2 ^2
= 2+3 = 5 A. when two heaters are in parallel and t is time taken
Pot. diff. across 2 £2 resistor = (5 A) (2 £2) = 10 V to boil the given water, then
3/202 Fundamental Ph3reics fXinppnw
ri^^S^lE X K il^^NXP^ifyojS^
For Difficult Ouostions ' dQ
57. Power, P = rate of flow of heat = —
dt

„ (V^ V2> V2
P = ^ = ^imsT) = ms^
dt dt dt
[r, R^j ,R,' Heat capacity, 5 = mr
1 V-1 dT P
or r = -ifl
t
— +
●1. P=:SX^ or s =
dt dTIdt
*1 A *2 , iX^RylR^)
Given, r^=7’o(l + p/*'^) ...(0
=_iL_ =Jxlx. Blr
f=r,(0+pxjr
1 -3/4 3/4
) =
>+V'2 <,+<J 4

oww
p 4P
54. When the heaters are in series, then current 5=-
-3/4
through-each heater, J = V7(/?, + R2). If/ is the dTIdt pr^f
time taken to boil the water, then
4P
^3/4 = 4P £^4 ...(«)
// = L_t P L^O J
(/?l + /?2)/= ^
/.,

e
ree
7;

rFl From(0, -t =.l + p/‘/4

Fre
Z0
or ^ =iL

rr F
orp/‘/4 =5._,-ZLzZo
5)
ouur
or t =t
V
-J
'
2
/
1
R,
i;
sfoo
or. r3/4^ >,-rof
. Pto .
kks
/ = / fl+fl
A
Yo
oooo

or

. I VJ ‘ t,I . V+% ■ s-1L[ILiZoT=-1^ Pi-rof


^oPL P^o J P"r/
eBB

58. Let V be the potential difference applied across


the paj^lel combination of ersistors. Then thermal
urr

. energy developed in a resistor of ersistance R for


You
ad

x3/2 time / is
/r ^(K)4
55. /j = /, ^ = 5 = 14*7 A
[0^2)
dY

V2
● V 'i^ y £ =

56. Let / be the current in the circuit As per question,


innd
Re

25 = IR (0
__ Vh/Ri _R2^ 200 2
Fi

3 = /(/? +r)
and. 0-5 = fiR
,..(ii) V^t/1^" R^ 100 1
59. Refer to circuit shown in Fig. 3(CF).70. Let V be
Dividing («i) by (i)i we get the potential at point C. Then
/ = ^.iA FIGURE 3(CR.70
2-5 5

2-5 2-5 V *2
From(i), E = -j- — = 12-5D
1/5
20V«->-AAAA/
A 2C1 ^
VWv—*10V
4G B

3 3
From (if), ^ = y~ R = —-12-5
1/5
= 2*5 f2
5 2D
Power loss across internal resistance

= /2r- nf . v = o
i x2'5 = 0*1 watt
● [5)
CURRENT ELECTRICITY 3/203

O
FIGURE 3(CF).72
For Difficult Questions

l2
B 4 ¥ D
7 = /i + /2 .-.1 20-V ^ 10-V
V

2 4

On solving, V = 10 V RiS20n
R2<20n
. _ 20-10 _ —
10 = 5A
“ ‘ 2 2 .,di+l2)
+ +
A ♦ —4- E
r _ 10-10 = 0
10V
I, F l2 10V

oww
7=5+0=5A
10 = 20(7,+72) ...0)
60. The circuit can be redrawn as
In closed loop FCDEF, we have
FIGURE 3(CF).71 ...(«)
10 = 20 (7,+72)+ 20 72
lo 6V p 20 Solving (0 and (li), we get
-P—Wvr

e
R

FFrlo
7, = 0-5A;72 = 0

re
-Il-l2 -II
63. The distribution of current in the various arms as
30

rFee
10 T9V per Kirchhoff’s first rule, has been shown in Fig.
30 ^^II 3(CF).73.

rF
—VsAf—'s FIGURE 3(CF).73
R
\ 9 ,G
fsoor
?—vwv
ouur
In closed loop RPQSR, skf -II
27, + 1(7i-72) + 37, = 9
6 7,-72 = 9 ...(0 Rl
ooko
E R2
Yo
In closed loop PUTQP,
Y
El
372-1 (7,-72) = 6
Bo

hi
reeB

-7,+472 = 6 ...(«■)
B (I + Ii)
From (0 and (//).
/,-72 = -013 A Using KirchhofTs second rule in closed circuits
ooY
uur

This current is 0*13 A from QtoP. ABFGCDA and BFGCB, we have


ad

61. Let Vq be the potential at O and 7,, 72, and £=(7+7,)/?2 + 77?
dY

are the currents through /?,, /?2, R^ and /?4 and £, = (7 + 7i)7e2 + 7i/?i
respectively. Then Putting 7 = 0, we have ;
nind
Re

7,+ 72+ 73+ 74 = 0 E— -R2 and E^ — 7, (7J, + R^


Vn-V. , , Vo-Vp
FFi

= 0 E
R^ R2 Rj R^ («l + «2)

or Vq-S/o-IO, Vo-20, Vq-15 = 0 64. Refer Fig. 3(CF)-74, G will show no deflection
10 20 40 80 if points C and 7 are at the same potential.
On solving, we shall get. Vo = 9V
A B FIGURE 3(CF).74
Current in the resistance /?, is
/.=?izii=iz5
1
= 0*4 A
ir
20 V
^loa
sn
10V K
10 10V
I1
As Vo> so the current is flowing from O 5£1 ion
towards A Le. away from O. sn ion

62. Refer to circuit Fig. 3(CF).72. According to . ....


T10V
. I
■wvw
Kirchhoff’s second rule in closed loop AFCBA, we 0 2on 0 sn '■
have
3/204 ^ Fundamental Physics (XII)

For Difficult Questions


Pot. diff. across C and D, Vc' - Vq = 2 x 1 = 2 V
As. V^ = 0V, so Vc = 2V
20 Pot. diff. across capacitor = 4 - 2 = 2 V
In circuit ABCDM, current /j = = 1A
5 + 10 + 5 Charge on capacitor, j2 = 2Vx2|iF = 4|i,C
Pot. diff. between C and D = /jXl0=lxl0
FIGURE 3(CF).76
= 10 V

Since point D is a zero potential, therefore pot.


10V 2y R c *ww
4C1 4V

w
of point C = !0 V. A 1= 1A . -I 2mF
B

Hence, potential of point J = potential at point C 2Q


= 10 V
D
Since point M is at zero potential, pot. diff.

e
roow
between E and 7 = (£ - 10) V.

re
In circuit KLMJK, current I is 67. Refer to circuit given below
£-10 £-10 Using Kirchhoff’s voltage rule in loops
/ =

reF
5 + 10 + 5“ 20 ABCDE^A and ABCDE2A, we have

uFFll (/, +/2) IO + /1 X 1 = 12

e
Pot. diff. across 5 O between points £ and J is
or
11 /, + 10/2= 12 ...(0
£-10^^ _ £-10

sFr
£-10 = and (/i+/2)10 + /2x2=13
. 20 4

foro
or
10/( + 12/2= 13 ...{«)
ofk
uor
or 4£-40 = £-10or3£ = 30 or£=10V
65. Refer to Fig. 3(CF).75, using Kirchhoff’s voltage FIGURE 3{CF).77
rule to closed circuits AB JKA, ACHKA, ADGKA, 13V, 2Q
kos
AEFKA, we have
h h
Y
Yo
<■ <■
reeBB
oo

A E2 D
FIGURE 3{CF).75
12F, in
uurY

E D C B A
●4- ●4- h ●4-

10V 5V.i
20V. 30Vi *1 ^,ll+l2+l3 h El h
J3 +14
I4 I2 I1'
ad

sn ion
T 25V ion
doo

B -»■ WW C
nY

I4 ●^14) /1+/2
¥
G
>
H j— 'K
nid
Re

Solving (0 and (i7), we get;


30 + 25 = /, X 11 ...(/)
FFi

23
L=—A
- 20 + 25 = /2 X 5 ...(») 7.1 =
16 2 32
5 + 25 = /3 X 10 ..m
Voltage across load = (7, + 7^) x 10
-10 + 25=74x5 ...{iv)
Solving equations (f), (»). (“0 and (iv), we get 7 7 23
= —+ — xIO =11-56V
16 32
/, = 5 A, /2 = 1 A,
73 = 3 A, /4 = 3 A 68. Resistors above and below 8 n makes balanced

.●. Current through 25 V cell = 7, + 72 + 73 + 74 Wheatstone bridge. So circuit reduces to Fig.


= 5+ l+ 3 + 3 = 12A
3(CF).78
Now 1 n and 2 n are in series, total Resistance
66. In steady state, no current will flow through
= 1 +2 = 3Q
capacitor, Fig. 3(CF).76. Using Kirchhoff’s
voltage rule to the circuit ACDA, we have 2 n and 4 Q are also in series so, total Resistance
+ 10 + 2= 1 xR+ 1 x2 or 7? = 10 n =2+4=6n
CURRENT ELECTRICITY 3/205

69. As bridge is balanced,


For Difficult Questions

p ^ 3
SO, = l or P = -xI0 = 15a
10 2 2

Here, 15 ^2 resistance is of wire of length 1-5 m.


1-5
/. 1 resistance is of wire of length = — = 01 m
15

= 1-0 X 10"^ m
70. Using the condition for balanced Wheatstone
bridge, we get
X (52 + l)cm 53

w
10 (100-52+2) cm 50

53x10
Now this 3 f2 and 6 Q are in parallel, So their

Flo
or X = = 10-6 .Q
50
6x3
effective resistance Rp = = 20
71. Connect a battery of emf E across A and C. Let

ee
6 +3
the distribution of current in the various branches
Circuit reduces to Fig. 3(CF).79

Fr
of net work be as shown in Fig. 3(CF).81.
According to Kirchhoff’s second law in a closed
circuit, ABDA we have

for
ur
ks
Yo
oo
eB

Resistance above and below 10 O again forms


balanced Wheatstone bridge. Now 2 O and 4 O
r
ou
ad

are in series =2+4 = 60


6 O and 12 O are also in series = 6+12
Y

= 180
2r/i -r/2- r(/-/j) = 0
nd
Re

6x18
Effective resistance = = 4-5 0 or
3/,=/ + /2 ...(/)
6 + 18
Fi

In a closed circuit BCDB,


Circuit reduces to as shown in Fig. 3(CF).80.
r (/, + /2) - 2 r (/ - /j - /2) + /2 /● = 0
FIGURE 3(CF).80
or 3/| =2 7-4/2 ●●●(")
20 From (/) and {ii), 2 I -412 = I +12
AAAr or 5 /2 = / or l2 = 1/5
I
From (0, /| = (/ + /2V3 = [/ + (//5)]/3
-P 6-5 V 4-5 0

” 15 " 5
In a closed circuit EABCEA,
Total Resistance = 4-5 + 2 = 6-5 O
E = 2 r I^ + r (1] + I2)
6-5
^2/ I
Current, 1 =
6-5
= 1A
or£=(3 /i+/2)r = ^/r.,
5
.(«/)
3/206
T^n^ideeft. 'a. Fundamental Physics (XII)
●< j
or /2-10/= (100-0(110-/)
For Difficult Questions
= 11000 - 210/ + /2
or 200/= 11000 or / = 55cm
If R is the resistance of network between A and C
then E~IR ...(I'v) 55
R^- R^.y
7 7r (100-55)
From (Hi) and (/v), IR = -I r or R =—
5 45 9
or
R =R X— = R, X —
Alternate method. 2 ‘55 ' 11

Here, R^ = r ; /?2 = 2 r Given, /?j + /?2 = 1000 ^


The effective resistance of net work between A 1000x11
/?! = 1000 or /?j =

ooww
and C is 1
11 20
= S50Q
R)(Rf +3/?2) _ r(r + 3x2r) 7r

/?2 + 3 /?j 2r + 3r 5
74.
2 I
...(I)
5 (100-/)

e
72. A-£2 or /? = 60 il

ree
90~60 and - = (/ + 20) ^ (/ + 20)

rFl 2 (100-/)-20 ” (80-/)

Fre
If balancing length of bridge wire from zero end
2 _ (80-/)

rrF
R X
is X, then or ■Hi)
90 (100-^) S (/ + 20)

sffoo
ouur
/ (80-/)
Taking log of both the sides, we get From (/■) and (//),
(100-/) (/ + 20)
logg R - logg 90 = logg X - logg (100 -x)
oksk
Differentiating it, we get On solving, / = 40 cm.
Yo
oo

40 40
Y

AR_Ax A(IOO-x) From (/), — = - or 5 = 3


BB

R X (100-x) S (100 - 40) 60 3


rre

n 75. Here, R =
Error ●m /?,
D A/?
AD = 4.± —
Ax^A(IOO-x)
+ R When /2 = n /i
A
ouu

100-x
Y
ad

A2 = AA.-Ai-A
\
dY

Ar
/? = ± ^ + ^ x60 thenAj /j =A| (2 or
100-x ) 1^40 60 j h nl I n
innd

New resistance of the stretched wire


Re

= ± 0-25 Q
/? = 60±0-2Sn
R' = ^=
FFi

= tpR
73. Let /?J be the resistance on the left slot and /?2 b© A2 (Aj/n)
the resistance on the irght slot of meter bridge
Resistance of each part of wire, R^ = R'!5
before interchanging the resistances. Let / cm be
= r-P Rt5
the balancing length of balanced bridge. Then
These wires are arranged to form a balanced
R, I Wheatstone bridge. So
...(/)
/?2 (100-/) 2R,1 X2RI n^R
R AB ~ = R1 -
2R1 + 2R 1 5
On interchangingthe resistances
^2 _ (^-IQ) /?,I no-/ 76. When the key between terminals 1 and 2 is
or plugged in, R is in the circuit.
R,1 (110-/) ^2 /-lO
Vy=m^ \ R = Kl 1 (V /=1A)
/ no-/ When the key between terminals 1 and 3 is
●● (100-/) /-lO plugged in, R and X, both are in the circuit
CURRENT ELECTRICITY 3/207

For Difficult Questions - = 0 or a: = 0


Here. y^-V'y= 2
V2 = I(R + X)=\ (R + X) = Kl2 i.e., length of wire A/ = 0
V2-V^=R + X~R=X=K(l2-li) 78. Let e, and £2 be the emfs of two ceils and K be
77. Resistance of wire AB = 0 02 x 100 = 2 i2 the potential gradient of potentiometerwire.
Case (0, Ej +£2 = ATx50 -.(0
r 2 1
Current in wire AB, / = t—— = -A
Case {«), £j -£2 = ^x 10 ...(»)
2 +2 2

Let X be the resistance of length AJ of wire for Dividing (/) by (i7), we get
which galvanometer shows no deflection, then
ei+£2_50_^
Vj^-Vj = xl = xxin ...(/) £l -£2 10

Here, e, = 2V;e2=lV;r, = 2n;r2=in


or £j + £2 = 5 £|-5 £2 or 4ej=6e2

ww
Current in closed circuit having £j and £2 in series,
2 + 1 ^_6 _ 3
^'4 “2
or
h = 2+ 1
= 1A

Floo
Pot. diff. across C and D, i.e., 79. Let length AJ = I, of potentiometer wire. Current
flowing through the potentiometer wire is

ee
Vc-Vd = E,-/i r, = 2-lx2 = 0.
e e

eer
/ =
FIGURE 3(CF).82

rFrF
12r+r 13r
e = 2V
2Q
■WvV Potential gradient of potentiometer wire
rur
I
I
k =
s
L
ffoor
/ X I2r £ X 12r _ 12 £
13rxL~13L
osk
12e
YYoou
J
Afi- X/
£1 =2V2q t
oook

I3L
— h-WA-i
eBB

G
As galvanometer shows no deflection, so
C 1Q D £ 12 £ 13 L
—I —Wv'^—
^.ty = 2 or / =i or / =
r

13L 2 24
62= 1V
ouur
ad
Y

01 Multiple Choice Questions (with One or More than One Correct Answers)
nd dY
Re

( ^ f x4 (at-bt^) =a~2bt.
FFini

= xl 81. {a) I =
80. dt dt
4+ r I + r
i.e., / decreases with time
r = 2« (c) For l = Q,a-2bt = 0ort-a!2b
g {d) Rate of change of current
Current in circuit, / dl d
4 +2
— =— {a-2bt) = -2b
dt dt
e f x4
16 = or £=12 V 82. Here Vj = 200 V, ^2 = 300 V
4 + 2j
y2 y2
Power dissipated is maximum when external As P = so R =
R P
resistance is equal to internal resistance, i.e..
/? = r=2Q. For same power 'P\ R°< V^,
( 12 \2 x2 =18W R
I _
V.I f200f 4
Pmax So
2 +2
^2 ^2) 300 9
3/208 T^%etdee^ '4. Fundamental Physics (XII) LV»1WI
84. The resistances 2 O and 2 in series of arm CDE
For Difficult Questions
are in parallel to resistance 4 in arm CE. Their
For series, current is same, P = -fR, so Po: R 4x(2+2)
effective resistance = =: 2 n. This
and V = IR so V R 4 + (2+2)
resistance is in series with 8 Q of arm BC. The
YL = ^ = ^ and A = :?l = 1 effective resistance is 8 + 2 = 10 Q, which is in
^2 /?2 9 P, R, 9 parallel across 10 Q resistance of arm BE. The
83. Total resistance of circuit effective resistance between B and £,
6x1-5 10x10
= R,+
1
= 2+ R = = 50.
6 + 1-5
/?2 +/?3 10 + 10

= 2+ l-2 = 3-2*Ii Total resistance of circuit = 5 + 5+ 15 = 25 ft


(a) Current through the battery, 25 V

ww
Current in circuit, / = = 1A
24 V 24 25n
/ = =7-SmA
3-2kQ 3-2x1000 Potential difference across B and E = IR

FF loo
= I x5 = 5V
(b) Pot. diff. across R2 and R^,
Potential difference across A and B
V2 = (7-5 mA) X (1-2 k U) = 9 V.

ee
= 25 - 5 = 20 V
(c) Pot. diff. across /?j,

ee r
V, = 7-5mAx2kQ = 15 V

rFrF
Current in arm BE, /, = — = A
V.2
1 _ 152 ' 10 2
Power dissipated across P^ =
rur
/?! 2xl(P
ffoor ( 1
Current in arm BC is also A1 = — A
vj _ 92 2 J
ks
Power dissipated across /?2» ^2 “ 6x10^ Current in arm CE = current in arm CDE
^2
YYoou
okos

1
.PI 15^x6x10^
BBoo

=-A = 0-25 A
= 8-3 4
^2 2x10^x92
r ee

4pL
Power dissipated across R^, 85. Resistance of a wire, R =
nd^
oouur
ad

92
= 54 mW .
^3 = 1-5x1()3 when d = 2d, then R" = 4pZ. _R
Y

7C(2i/)2 4
{d) If /?j and /?2 interchanged, total resistance When two wires are in series, Then their effective
dndY
Re

of circuit is =6+
2x1-5 ^ 6 48, ^ ^ R R R
resistance, R. = — + — = —
2 + 1-5 7 7
FFini

4 4 2
24x7 When two wires are in parallel, their effective
Total current, / = = 3-5 mA
48x1000 (RJ4) (R/4) R
resistance, Rj =
6 (R/4) + R/4 8
Pot. diff. across = (3-5mA)x-kn = 3V
7
V2 V2 V2
Power dissipated across R-^, As per question H = x4 = ^1 = h
R {R/1) ' (/?/8)
32
- 6mW
1-5x103
On solving, = 2 min. and /2 = 0*5 min.
86. Since evaporation is given to be non-uniform,
As P^ < P3, so power dissipation in R^ will hence temperature must be non-uniform. Thus
decrease option (fl) is wrong. Due to evaporation, the cross-
sectional area of the filament at section decreases.
^3 54
— = 9 (decrease). Hence resistance of the given section increases
p; 6 as « 1/A. Thus option (b) is wrong.
CURRENT ELECTRICITY 3/209

H' II E>X Rlg^’


For Difficult Questions FIGURE 3{CF).83
B ¥ C

The portion of the filament of the bulb having I

highest evaporation rate would have rapidly


reduced area and would become break up area of T
Ri
cross-section. At break up, junction temperature R2
would be highest. Due to it, the light of highest A —V\A^“ D
band frequency would be emitted at those cross-
sections. Thus option (c) is true.
±_V2

oww
-a
Due to continuous heating of filament, the cross-
sectional area of a section of filament decreases.
Hence the resistance of the filament on the whole F*—?-^VW-^E
I I
R3
increases with time. As a result of it, the power
decreases because P = V^R or F « 1/F as V is

e
Since, current in R2 is zero R2 can have any value.
constant. Thus option (d) is true.

re
From («0, we note that, if Vj = ^2. then
87. Let F, r be the emf and internal resistance of each J?j — — R2 or — 2 R2 “ R^

FFrllo
cell.

reF
If Vl = 2 V2, /?i = 2/?3
nE E

e
In series, current I =
ouru If 2 Vi = V2.*3 = 2J«i = «2
nr r
Thus options (a, b and d) are true.
(independent of n) 90. The resistance of each side of polygon = R/n. For

osFr
maximum resistance between opposite comers,

fkfor
In parallel, current I - ; (i.e./ocii) we have two resistances each of R/2 in parallel.
r/n r

When n parallel cells are short circuited by a wire The maximum resistance= (R/2)x(RI2) _ R
kso
of constant ersistance R, the current. (RI2) + (RI2) “ 4
ooo
YYo
E nE For resistance between adjacent comers, we have
BB

{R + r/n) nR+r two resistances R/n and (n - 1) R/n in parallel.


Their effective resistance
As n increases r/n decreases, so / increases.
r ree
Y

_ (F/«)x[(w-l)/yw] jg(ii-l)
oouu

88. v^=—%
m
i.e.,VjocE " (R/n) + (n-l)R/n n
2
ad

91. As cells are in parallel, the equivalent internal


Yd

R R d^R i = JL J_ J_
nidn

resistance R is
R R^ R2 R^
Re

ButEocVj so On applying Kirchhoff’s voltage law in the loops,


FFi

P = VI = fiR, i.e., P«/2


^ '^2^1
89. Refer to Fig. 3(CF).83; using Kirchhoff’s voltage we can get ^3
law in closed circuit. ABCDA, we get
Equivalent emf of the battery depends upon the
= /Fj+0xR2 or 1=^ ...(0 values id Elf £2*^3 ^2 ^3*
92. Refer Fig. 3(CF).84, let V be the potential of
In closed circuit ABCDEFA, we get
point D.
...(«●)
Then 70-V=10/ 1 (0
Vi + V2 = IRi+IR2 or / =
R1+R3 and V-0 = 20/2 or V=20/2 .(*0
and V-10 = 30(/i-/2) ...(«)
From (0 and (I'O
Rj + R3 From (i) and (ii).
or
Vi/?i + V,R3=ViRi + V'2«1 70-20/2=10/1
or
ViR, = V2R^ ...(/«■) or
10/1+20/2=70 .(iv)
3/210 ‘pfutdeef.i ^ Fundamental Physics (XII) WMWI
through arms PQ and ST. The equivalent circuit
For Difficult Questions
will be as shown in Fig. 3(CF).85.

From (ii) and (/if).

w
The equivalent resistance of the circuit
20/2-10 = 30 /]-30/2
or
30 /j- 50 /2 = - 10 ...(V) R =
(2 + 2 + 2)x(4 + 4 + 4)
= 4Q

Flo
On solving, we get (2 + 2 + 2) + (4 + 4 + 4)

e
/i = 3A,/2=2A, (/j-/2)=1A Pot. diff. across A and S = 12 V.

reee
and F = 40 V 12

FFr
. /, = —= 3A
Total power consumed ^ 4
= (3)^ X 10 + (if X 20 + (1)2 X 30 12

for
^2 “ = 2A
ur
= 90 + 80 + 30 = 200 W 2 +2+2

93. If a plane mirror is placed along a line which is Since current is flowing from P to S, therefore
kkss
perpendicular bisector of line AB, then the left Vp > As Vp = Vq, so Vq > or Vg < Vq
Yo
part of the circuit is exact image of right part of
oo

94. Heat produced, H = Rt. Since /, R and t are


the circuit. So current in AP = current in SB and
same for the two resistors, hence equal amounts
B

current in AQ = current in TB. Therefore, in the of thermal energy must be produced. Also
re

given circuit the points P and Q are at the same H = me Q, therefore the temperature may irse
potential. Also the points S and T are also at the equally in the resistors if their sp-heat is same,
ou

same potential. Therefore, no current flows


ad

otherwise not.
YY

mi Multiple Choice Questions (Based on the given Passage/Comprehens ioii)


nd
Re

95. When a potential difference is applied across the 99. The effective resistance between diagonally
Fi

two ends of a conductor, the electric field exists opposite comers of a skelton cube is 5 r/6, where
inside the conductor. r is the resistance of each ann of skelton cube.
96. Here, r = 12/12 = 1 O/cm.
The free electrons moving towards the positive
Resistance between diagonally opposite
end of the conductor have two types of velocities
(/) thermal velocity (i7) velocity due to the acceler -xl = 5/6£l
comers of a skelton cube is = 7
ation by virtue of force on it due to electric field. 6

As a result of it, the electron follows a curved 100. Current drawn from the battery,
path in between two collisions in a conductor. e.m.f.
I = — = 2-4A
eE resistance 5/6
97. The drift speed of the electrons, = t.
m 101. The maximum current in an arm of the skelton
cube = 2 116 = lx 2-4/6 = 0-8 A
which is independent of dimensions and number
102. The minimum potental difference across an arm
density of free electrons in ±e conductor.
of network = / r/6 = 2-4 x 1/6 = 0*4 V
98. Knowledge based question.
3/211
CURRENT ELECTRICITY

T<?CG©
. r ■
I

For Difficult Questions

09 Matching Type Questions


\ AxAT
103. (A) The standard resistance coil is made of
(B) Conductance - —
R~ V~ W ~ Ml? T~^
manganin, i.e., an alloy of copper-
magnanese-nickel.
(B) The best connecting wire is of silver, which = [M-^L~^T^ A^]
offers least resistance.
(C) Electrical conductivity
(C) The heating wire is generally made of
= {M A^) \ = [AT^ 7^ A^]
I
nichrome, i.e., of nickel-chromium alloy. a =
Ra

w
(D) The fuse wire is made of tin-lead alloy.

104. (A) Current density = — = [AL (D) Specific resistance, P - -=[M^L^T~^A'-‘^].


c
a

Flo
e
ra

ree
Matrix-Match Type Questions

FFr
105. (A) Cunent is due to motion of electrons in a 106. (A) The maximum potential difference between
closed path and in a definite direction. two points of a circuit is measured by
urr
(B) EMF of the cell is the workdone in taking
the unit charge once around the closed path. for voltmeter and potentiometer.
(B) The emf of the cell in the closed circuit (i.e.,
kkss
(C) Resistance of a conductor is due to the terminal pot. diff.) is measured by voltmeter
collisions of the electrons with ions/atoms
Yo
and potentiometer.
ooo

while moving through the conductor.


(D) Superconductor is that conductor which (C) The capacity of supplying the current in the
B

offers no resistance to the electrons flowing circuit is in ampere hour.


re

through it, i.e., the electrons suffer no (D) The electric energy consumed in the circuit
collision with the ions/atoms while passing is taken in watt-hour.
ou

through the super-conductor.


ad
YY

VI. Integer Type Questions


ndd
Re

From (0 and (i/), we have


107. Let Eq be the pot. diff. applied across the total
Fi

length / (= 10 cm) of potentiometer wire. ^0 2E


0
— = xx
£o 3 3/
Potential gradient in the first case = /
.
I 10
— = 5cm
l(E,0 or X — —
£ 2 2
As per question, E--3 / ...(0
3
108. When the current through galvanometer is zero,
0 then pot. diff. across /? = 7 V
Potential gradient in second case = 31/2 3/ pot. diff. across 5n=12-7 = 5V
5
If X is the desired length of potentiometer to
balance the emf E of the cell, then current in 5 Q “ J ^ ^
E = xx
2£0
...(«) Resistance, R = l = 7ft
3/ 1
3/212
Fundaments Physics (XII) VOL.I

For Difficult Quosttons

109. Let a be the area of cross-section of wire and n


Using KirchhofiTs second law, in closed circuit
be the number density of electrons in the wire. If BCDEB, we have
N is the total no. of electrons crossing per second
through a cross-section of wire due to current / r/2-6/|=s0 or
flowing, then

N = -stnal or I finale -.(0

w
e
FIGURE 3fCF).87
D C
But
I ^ n a ev^ r

I nale BCl -d2


or = l E AMAr
ir
B

roow
e
nae nae
(II +12)'' - ^(Il +I2)

re
Total momentum of electrons in the straight wire 3Sl
-VWSr A
V
is = Nmv. =-xmxl

reF
uFFll
e

From (i),

e
_ 70x(9x10-3^)x1000
1-6x10“*® V = 9x^ _3/2
^(r + 2)

sFr
«4xi(r7Ns

foro
fk
uor
2V
110. Refer Fig. 3(CF).86, there will be maximum or
h-
power in the external circuit if /? = r. 3(r + 2)
okso
Power developed in ersistor r,
Y
FIGURE 3ICR.86
Yo
oo
reeBB

E.r 4V2

xr
9(r + 2)2
uurY

''1 = 3A
Power developed is maximum, when (r -i- 2)^ is
R minimuiTi
VWS^
(r+2)2 = 0
ad

or
doo

Maximum power consumed, or r2 + 4r-j-4 = 0


nY

or r2-4r + 4-»-8 r = 0
f—T £2
(r-2)2 + 8r=0
nid

P.max or
Re

xr = —= 9 . ...(0
,r + r. 4r
or r = 2Q
FFi

112. Given, = 2-25


Current, / = ^ =3
r + r
-liSf R = J,
^2
f 2E
an 1 R = J2
[2r*R)
or £ = 6r ...(lO 1
r + 2£
Solving (0 and (if), we get
/

E = 6\ ■ -<'-+2Rf J, = 2-25
111. The equivalent circuit with the distribution of (2r + R)^ /j
current is shown in Fig. 3(CF).87.
Using KiichhofiTs second law, in closed circuit or
r + 2R
= 1-5 = -
ABEFA, we have. 2r + £ 2

6/i + 3(/i + /2)-V=0 On solving, we get.


or
V=9/i+3/2 -.(0 /? = 4r=4x 1=4Q
CURRENT ELECTRICITY 3/213

For Difficult Questions

u
VII. Assertion-Reason Type Questions

F
FOR MEDICAL STUDENTS

owo
113. The bending of the insulating wire does not alter
the resistance of the wire. Hence assertion Is

s
wrong. Here reason Is also wrong.
114. In parallel combination of electrical appliances

o
k
r FF
l-J- J- + -L
R~ R^ Rj «3

r
ll FY
eeo
As voltage (V) across each appliance is same so

ro
y2 y2 y2
— or P-Pi + P2 + P^

o
YY
— = —— + —

R R,1 R^ R^

f
115. The resistance of 20 bulbs in series is more than 8r
B 1 _ 1 _1_ 2- = ll or R
that of 19 bulbs. The current in a frill of 20 bulbs ^e-15

or
R„^ r It 8r 8r
PQ
will be less than that of 19 bulbs. Since
okr
sf
od
ou
power
oc
(current)^, so the bulb in the frill of 19 Both Assertion and Reason are wrong.

bulbs will glow with more brightness than that of FOR ENGINEERING STUDENTS
u

20 bulbs frill. Thus both Assertion and Reason


118. When the identical cells are connected in parallel,
ee
urn

are false.
B

the effective e.m.f. is equal to e.m.f. of one cell.


116. Here both Assertion and Reason are wrong. Hence statement-l is wrong. Here statement-2
We know that with the increase in temperature,
oo

is correct
the value of unknown resistance of conductor 119. The electrical appliances with metallic body like
Yi
ad

increases. heater, press etc. have three pin connections. Two


pins are for supply line and third pin is for earth
FF

connection for safety purposes. Here statement-


In balanced meter bridge
’ X h 2 is wrong.
d
Re

120. A laser beam is a beam of light which is light


in

where R is standard known resistance and X is


amplification by stimulated emission of radiation.
unknown resistance. On heating, as X increases, The energy per unit area of the laser beam is very
so for the same value of R should be high as compared to the torch light. Here
increased and not decreased. statement-2 is wrong.
121. Here statement-1 is correct but statement-2 is
117. The equivalent circuit is shown in Fig.
wrong, because the electric field is there but the
3(CF).88(fl) and (b) net electric field is zero.

VIII. Multiple Choice Questions (Based on Experimental SkijIs)


Percentage error in measured value of
122. From /? = — resistance = 6%
/

AR
R
AV +. —
V
M
1 ioo
3 3
100
_ 6
100
123. Given
, R^
= ^2 or /?2 = 2/?,
3/214
IPftadeefr 4- Fundamental Ph}rsics (XII) VOL.I

For Difficult Questions £R


124. 1
= kl^ and = ^2
(Rl + r) (/^Z + r)
The equivalent resistance of parallel combination
. £/g^/(/g, + r) /, /?l(/g,+r) /,
is 1 = JL _L _L or

/f| /22 £/Z2/(/22 + r) /j /?2(/?,+r)


5(10+r)_2
I I 1 3 1 10(5 + r) 3
^ On solving, r=10Q
50

oww
1 1 3 1 3 125. For first case = 1
or — =
100-50
R 2R^ I 21?,
or
1?2= l?j + 10 .(0
31^ When 10 O resistance is removed, the balance
or
1 = 1^- 2R point will shift towards lower length.

ee
l
New balancing length = 50 - 10 = 40 cm.

FFrlo
r
For secondcase.

rF
or
1^=1+ 21?,

ee
^1 _ 40 _ 40 2
1 I*1
R^ 100 - 40 ”60 “3 ^~2

rF
Since no resistance value is fractional, hence.
ouru
From(0, |/?, =/?,+10 or /?, = 20O
fosor
minimum value of
/?, 3 126. Let the resistance of shorter part AB be x Since
skf
the total resistance is 2012, the resistance of longer
ooko
l?j=l+|x| = 1 + 1 = 212 AB part is (20 - jf). Between points A and B, the
Yo
So
Y
^ 2 3 two resistances jc and (20 - j:) are connected in
Bo

parallel, so their equivalent resistance is


reeB

and R^=^ = — 2
= 312 _ (20-jc)xj:
(20-x) + jc
= 1-8
ooY
uur

Maximum resistance value is of


Onsolvingget,x = 212
ad

/?2 = 2/?, = 2x3 = 6Q Thus, length of the shorter section = 2 x 1 = 2 m


dY
nind
Re
FFi
w
e
re
o
rw eF
ullo
FF
QJ[N1QT=®

srre
MOVING CHARGE'S
koF
fofr
uor
AUB magmetism: o
kos
YYo
rBB

CHAPTER 4.
oo
Y

MOVING CHARGES AND MAGNETISM


ur ee

CHAPTER 5.
Yd

MAGNETISM AND MATTER


oou
ad
idn
Re
in
FF
MOVING CHARGES

w
AND MAGNETISM

Flo
e
reee
4.1. INTRODUCTION

FFr
Both, electricity and magnetism have been known for more than 2000 years. However, their intimate
relationship was discovered only 200 years ago. Oersted discovered in the year 1820 that a straight wire
carrying current caused a noticeable deflection in a nearby magnetic compass needle. The wire carrying

for
ur
current must be associated with a magnetic field causing deflection in the compass needle.
When a wire carrying current is held perpendicular to the plane of paper, and a irng of compass needles
kss
shown in Fig. 4.1(a) when current emerges out of the plane
surrounds the wire the orientation of the needles is
Yo
of the paper. However, when current moves into the plane of the paper, the orientation of the needles is shown
oo

in Fig. 4.1(/j).
B

FIGURE 4.1
If we were to sprinkle iron filings around the wire
carrying current, they arrange themselves in concentric
re

circles with the wire as the centre, as shown in Fig.


^ ;
'
\ '
' >r \' \ '
4.1(c). Oersted thus concluded that moving charges or I 'i f
ou

I I I > 1 1
I I , » ●
ad

I I ‘
currents produce a magnetic field in the surrounding
\ \
/ y / I
YY

/
space.
o o o
In this unit, we shall study :
nd
Re

(i) How currents produce magnetic fields,


(«) How magnetic fields exert forces on moving charged particles like electrons, protons and current
Fi

carrying wires,
(Hi) How charged particles can be accelerated to very high energies in a cyclotron,
(iv) How currents and voltages are detected and measured by a galvanometer.

4.2. OERSTTD'S EXPERIMENT

Take a magnetic needle NS, which can rotate freely about a vertical axis in a horizontal plane. Hold a
conducting wire AS over the magnetic needle NS parallel to it. Complete the circuit by closing the key K
such that current flows from A to S.
It will be found that N-Pole of the magnetic needle gets deflected towards the west. Fig. 4.2(a). If
the direction of current in the wire is reversed (i.e. from B to A), the N-pole of magnetic needle gets
deflected towards east. Fig. 4.2(b). Since the magnetic needle can be deflected only by the interaction of
another magnetic field, therefore, the current in the wire must be producing a magnetic field in the
4/1
4/2 ‘P'tcuUef.i 'a Fundamental Physics (XII)
sunounding space. The direction of deflection of magnetic needle due to current in the wire is given by
Ampere^s swimming rule.
Ampere’s swimming rule. According to this rule, if we imagine a man swimming along the wire
in the direction of current with his face always turned towards the needle, so that the current
enters through his feet and leaves at his head, then the N-pole of the magnetic needle will be
deflected towards his left hand. This rule can be recollected with the help of the word SNOW,
where each letter has a deifnite sense. It means, if current flows from South to North, in a wire
held Over the magnetic needle, the north pole of the needle will be deflected towards West.

oww
FIGURE 4.2
W

- b-<-)n
Rh K Rh K S N

e
re
FFllro
reF
e
uoru
sFr
4.3. MAGNETIC FIELD
foro
fk
okso
Just as a static charge produces an electric field, a moving charge or current through a conductor produces
Y
Yo
a magnetic field.
oo
BB

The magnetic field is the space around a conductor carrying current or the space around a

magnet in which its magnetic effect can be felt.


rYree

Strength of magnetic field is also known as magnetic flux density or magnetic induction.
oouu

In order to define the magnetic field B , we deduce an expression for the force on a moving charge
ad

in a magnetic field.
Yd

FIGURE 4.3
Consider a positive charge q moving in a uniform magnetic field
Z
nidn

B (along OY) with a velocity v . Let the angle between v and B be 0, Fig.
Re

F
4.3. Due to interaction between the magnetic field produced due to moving B
FFi

V COS 0
charge (i.e. current) and magnetic field applied, the charge q experiences a force.
V sin 0,
Experimentally, it has been found that the magnitude of force F
experienced by the moving charge
(0 is directly proportional to the magnitude of the charge, i.e., F o: q X

(ii) is directly proportional to the component of velocity acting perpendicular to the direction of magnetic
field, i.e. F w sin 0

(Hi) is directly proportional to the magnitude of the magnetic field applied, i.e., F»c B
Combining the above factors, we get Foe qv sin 0 B or
F = kqv 5sin0
where A: is a constant of proportionality. Its value is found to be one i.e. k= 1.
F = qv B sin Q ...(1)
-> -» —»
-i
or
\F \ =q \ vxB I or
F=q(vxB) ...(2)
Note : F is called Magnetic Lorentz Force.
4/3
MOVING CHARGES AND MAGNETISM

Direction of F

The direction of F is given by the Right-Handed-Screw Rule or Right-Hand Rule.


FIGURE 4.4
If if and B are in the plane of paper, then * —> —> —>
—^
F =9 (u X B)
according to Right-Hand Rule, the direction of F
on positively charged panicle will be perpendicular
o o
to the plane of paper directed upward as shown
V/
in ■►B

Fig. 4.4(<2), and on negatively charged particle will V

be perpendicular to the plane of paper directed

oww
downwards, Fig. 4.4(b). ▼F = (v X B)

Definition of B

If n = 1, ^ = 1 and sin 9 = 1 or 9 = 90°, then from (1), F=lxlxfixl=fl

e
Thus, the magnetic field induction or magnetic flux density at a point in the magnetic field is

FFrlo
re
equal to the force experienced by a unit charge moving with a unit velocity perpendicular to the
direction of magnetic field at that point.

ree
F
SpecialCases. Case(i)Ife = 0°orl80“,thensine = 0. From(l), F^qv B(0) = 0

rF
It means, a charged particle is moving along or opposite to the direction of magnetic field, it does not
experience any force.

fsoor
ouur
Case (ii) If 9 = 90^ then sin 0 = 1 F^qv B(\) = qv B (Maximum)
skf
Case (iii) If 9 = 270®, then F = qv Bsin27(r = -^u B
ooko
Case (iv) If i> = 0, then. F-qv B sin 9 = 0
Yo
It means, if a charged particle is at rest in a magnetic field, it experiences no force and if a charged
Y

particle is moving along a line perpendicular to the direction of magnetic field, it experiences maximum
Bo
reB

force. The direction of this force can also be determined by Fleming’s Left Hand Rule, which can be stated
as follows ;
If we stretch the first finger, the central finger and the thumb of left hand in mutually
uur
ooY

perpendicular directions such that the first finger points to the direction of magnetic field, the
ad

centralfinger points to the direction of electric current (motion of the pos^ve charge), then the
dY

thumb represents the direction offorce experienced by the charged particle.


innd
Re

If 7 is along X-axis and B along Y-axis, then


FFi

F will be along Z-axis as illustrated in Fig. 4.5.


Unit of B . SI unit of B is tesla (T) or
weber/(metre)^, i.e., (Wb/m^) or Ns C"* m *
F
From (1), B =
^ u sin 9
If q = \ C,v ~ I ms"*,
0 = 90° or sin 9 = 1
and F= 1 N ; then

1
B = = 1T
ixlxl
4/4
Fundamental Physics (XII) VOL.I

Thus, the magnetic field induction or magnetic fiux density at a point is said to be one tesla if a
charge of one coulomb while moving at right angle to magnetic field, with a velocity of
a

1 ms~^ experiences a force of 1 newton, at that point.


IN IN
Hence, 1T = = 1 N A-l -1
ICxlms ^ 1 As X1 ms ^
m
(1 C = 1 ampere x 1 sec.)

cgs electromagnetic unit of magnetic field is gauss (denoted by G), where 1 G = 10^ T.
Dimensions of B .

F MLT-'^
As, B = Dimensions of B =
^usine = [A/A-> r-2]
Arar-i)

ww
Magnetic field has some properties similar to electric field

1. Magnetic field is a vector quantity like electric field.


2. Magnetic field has three dimensional relevance like electric

FF loo
field.
3. Magnetic field can depend on time like electric field.

ee
4. Magnetic field depends upon the permeability (p) of the material medium present in the field, whereas
electric field depends upon the permittivity (g) of the material medium present there. Infact both p and e are

ee r
rFrF
the measure of the extent to which the magnetic field and the electric field can penetrate the material medium.
5. The various magnetic fields and so on, at a point due to different sources in a region can
rur
ffoor
be added vectorially by superposition principle i.e., B=B^ + B^^ B^+.... whereas the various electric fields
ks
at a
point can be added vectorially by superposition principle i.e., E = ^ + + ^ +
YYoou
ookos

Retain in Memory
BBo

1. No force is experienced by a static charge in a magnetic field.


re

2. The force is experienced by a moving charge in a magnetic field only if the charge is not moving
parallel or antiparallel to the magnetic field.
oouur
ad

3. Magnetic force on the charge deflects the path of charge.


Y

4. Strength of magnetic field is generally referred to as simply “magnetic field”.


5. Earth’s magnetic field at its surface is about 3-5 x 10"^ T.
nd dY
Re

6. The static charge is a source of electric field but not of magnetic field whereas the moving charge
IS a source of electric field as well as of magnetic field.
FFini

ProbfemV An tt-partlcle of mass 6«65 x kg ISi travelling at right angles to a


magnetic field with a speed of 6 x 10^ ms ^ The strength of the magnetic field is 0*2 T. Calculate the
force on the a-particle and its acceleration.
Sol. Here, m = 6-65 x kg;^ = + 2e = + 2xl-6x IQ-’^ c ; u = 6 x 10^ ms
-1 .
; 5 = 0-2 T; 0 = 90°
Force on the a-particle in the magnetic field is
F=qv Bsin0 = (2x l-6x 10"‘9) x (6 x 10^) x 0-2 x sin 90° = 3-84 x 10-*^N
3'84xl0-’‘^
Acceleration of the a-particle = — = = 5-77 X 10^2 ms
-I
m 6-65x10 2'^
4.4. BIOT-SAVARTS UW

'Ti,- f experimental law predicted by Biot and Savart. It is also called Laplace’s law.
This law deals with the magnetic field or magnetic induction at a point due to a small current element
(i.e.y a small part of any conductor carrying current).
4/5
MOVING CHARGES AND MAGNETISM

Let us consider a small element AB (of length dl) of the conductor XY carrying a current /. Let 7 be the
position vector of the point/“from the current element /£ir* and e be the angle between dl and 7,Fig. 4.6.
According to Biot-Savart’s law, the magnitude of the magnetic field
induction dB (also called magnetic flux density) at a point P due to current
element depends upon the following factors as detailed below.
1

(/) dB I (ii) dB «* dl (iii) dB^ sin 0 (iv) ^2


Combining these factors, we get
I dl sin 9 I dl sin 0

w
dB oc or dB=K

where K is a constant of proportionality. Its value depends on the system of units chosen for the

Flo
measurement of the various quantities and also on the medium between point P and the current element.
When there is free space between current element and point, then

ee
^^0 and In cgs system, K= 1

Fr
In SI units, K =
4tc

where is absolute magnetic permeability of free space


(v 1T=1 Wbm-2)
for
Po = 4tcx 10“’Wb A"^ m"* =4tix 10"^T A“* m
ur
and

Ho Idl sin—9
ks
dB = -4J-X ...(3)
In SI units.
Yo
47t
oo
B

Idl sin 0
dB =
In cgs system.
re

r2

^0 I\dl X r \ Ho Hdlxr)
ou
ad

dB = .(4)
In vector form, we may write \dB\ = 4n
or
4ji
Y

Direction of dB . From (4), the direction of dB would obviously be the direction of the cross pr(^ucl
nd
Re

vector, dtx 7" . It is represented by the Right handed screw rule or Right Hand Rule. Here dB is
Fi

perpendicular to the plane containing dT and 7* and is directed inwards. If the point P is to the left of the
current element. dB will be perpendicular to the plane containing dl and r , directed outwards.
Magnetic field induction at point P due to current through entire wire is

B =
IIq Idlxr ..4(a)
J 4tc

IIq / dl sin 0
or
B=| 47t

♦Current element is the product of current (/) and length of very small segment of current carrying wire {dl).
current element I dl is a vector. Its direction is tangent to the element and is acting in the direction of
—»

The
current flow in the conductor.
4/6
^ Fundamental Physics (XII) VOL.I

Biot Savart’s law in terms of current density J, states that


Idl _ Idl
4;r -^dV A Adi ~ dV
where J = current density at any point on the current element, dV = volume of the element
Biot Savart’s law in terms of charge (q) and its velocity (u) is

dB dl
4ti r3 Idl=-^.dl =
dt
Biot Savart's law in terms of magnetising force or magnetising inteasity (H)* of the magnetic field is;
In SI or mks system

ww
dH =
dB \ Idl x7 1 Idtxr I I dl sin 0
and dH =
^^0 47t 4tc r2 4tc r~
In cgs em units

Flo
ee
I dlxr Idl sin Q
dH = dH =
and

rere
r3 r2

rFF
Some important features of Biot Savart's law
1. Biot Savart’s law is valid for a symmetrical current distribution.
uurr
foor
2. Biot Savart’s law is applicable only to very small length conductor carrying current.
3. This law can not be easily verified experimentally as the current carrying conductor of very small
ks s
length can not be obtained practically. ^
Yoo
4. This law is analogous to Coulomb’s law in electrostatics.
oook

5. The direction of dB is perpendicular to both I dT and "r


eBB

6. If 0 = 0*^ or 180°, i.e., the point P lies on the axis of the linear conductor
carrying current (or on the
wire carrying current) then dB 0
rr

= 0, which is minimum.
ouu

47t r2
ad

It means there is no magnetic ifeld induction at any point on the thin linear current carrying conducto.r
YY

7. If 0 = 90° i.e. the point P lies at a perpendicular position w.r.t. current element, then
nndd

\io Idl
Re

dB =
An ^2 > which is maximum. FIGURE 4.7
Fii

Sample Problem
An element, A I = Ax i is placed at
the origin as shown in Fig. 4.7 and carries a current 5 A. Find out
the magnetic field at a point P on the z-axis at a distance of 2-0 m
due to the element Ax = 1 cm. Give also, the direction of the I
magnetic field produced. X

Sol. Here, Idl = I Ax i, where 1 = 5 A


y^P
Ajt= 1 cm= lO'^m ;/-=2-0m; 7=2-0k
dB = 1^0 I dTx 7 Pq 7(AxOx(2-0it) = I0"'^x
5x10"2x2-0. «
4tc r3 4tc (-7)
r3 2^

- 1‘25 X 10“^ T along -y direction


*For detail of magnetising force or magnetising intensity refer to Art. 5.24.
4/7
MOVING CHARGES AND MAGNETISM

4.5. SIMILARITIES AND DIS-SIMILARITIES BETWEEN THE BIOT-SAVART'S LAW


FOR THE MAGNETIC FIELD AND COULOMB'S LAW FOR ELECTROSTATIC FIELD
Similarities

(0 Both the laws for fields are long range, since in both the laws, the field at a point varies inversely as
the square of the distance from the source to point of observation.
(ii) Both the ifelds obey superposition principle.
(Hi) The magnetic field is linearly related to its source, namely the current element / dl , and the
electric field is linearly related to its source, namely the electric charge q.
Dis-similarities
(i) The electrostatic field is produced by a scalar source namely, the electric charge q and the magnetic
field is produced by a vector source, a current element I dl .
(ii) The electrostatic field is acting along the displacement vector, i.e., line joining the source and
the field point. The magnetic field is acting perpendicular to the plane containing the current element

ww
I dl and displacement vector 7^, i.e., along the direction of (1 dl x r).
(Hi) The electrostatic field at a point due to a charge is independent of angle 0. whereas the magnetic

Floo
field at a point due to a current element is angle dependent. It means Coulomb lavp is independent of angle
whereas the Biot-Savart’s law is angle dependent.

ee
eer
4.6. MAGNETIC FIELD DUE TO A STRAIGHT WIRE CARRYING CURRENT

rFrF
Consider a straight wire conductor XY lying in the plane of paper, carrying current / in the direction X to
Y, Fig. 4.8(a). Let F be a point at a perpendicular distance a from the straight wire conductor. Clearly,
rur
ffoor
PC = a. Let the conductor be made of small current elements. Consider one such small current element 1 dl
s
of the straight wire conductor at O. Let r be the position vector of P w.r.t. current element and G be the angle
osk
YYoou
between I dl and r . Let CO — 1.
oook
eBB

According to Biot-Savart’s law, the magnetic field dB


(i.e., magnetic flux density or magnetic induction) at point P due to current
r
ouur

element I dl is given by
ad
Y

dB =
^0 I dlx r
nd dY

A% r^
Re
FFini

dB =
IAq X
I dl sin 0
or
4it r2 ,(5)

In It. angled A POC, 0 + (|> = 90® or 0 - 90® - ([)


sin 0 = sin (90® - <t>) = cos <t> ...(6)
a a
Also, cos (|) = — or r= ...(7)
cos(t>

/
And, tan ({) = — or / = a tan 4>
a

Differentiating it, we get dl = a sec^ ^ d^ ...(8)


Putting the values in (5) from (6), (7) and (8), we get
4/8
Fundamental Physics (XII) VOL.I

dB^ Mq /(gsec^(|)f/(|))cos({> _ Hq ~cos^d(^


^
4n a
2 ^ 4jt a <9)

COS^ (|)
->

The direction of dB , according to irght hand thumb rule, will be perpendicular to the plane of paper
directed inwards. As all the current elements of the conductor will also produce magnetic field in the same
direction, therefore, the total magnetic field at point P due to current through the whole straight conductor
can be obtained by integrating Eq. (9) within the limits - (|)j and + Thus

B =
1 dB =
4ti a J cos (|) </(|) =
4ju a -[sin 02 -sin(-0j)]

ww
lio I
B = —^-(sm0, +sin0.)
4tz a ' ^ ...(10)

Special cases. (/) When the conductor XY is of infinite length and the point P lies near the centre of

Flo
the conductor then 0j = 02 = 90* ''

e
^0^

ree
So, B = -[sin 90* +sin 90*] = or B =
4tc a 47t a 27CU dl)

Fr
rF
(«) When the conductor XY is of infinite length but the point P lies near the end Y (or X) then
uurr
0I - 90° and 02 = 0*.
So, B = 1^/—[sin 90° +sin 0*] =s
47t a 4n a
s i.e.,
for B =
4na
,..(lla)
kks
Yo
oooo

Thus, we note that the magnetic field due to an infinite long linear conductor carrying current
near its centre is twice than that near one of its ends.
eB

The variation of B with a for linear wire carrying current is shown in Fig. 4.8(b).
(ui) If length of conductor is finite, say L and point P lies on irght bisector of conductor, then
r
ou

L/2 L
ad

01 - 02 - 0 and sin0 =
YY

V^ +072)2
nndd

Then, B = [sin 0 +sin 0] = ^0 2/ L


sin0 =
Re

4na 4tc a 4tc a


■^4a^ +l}
Fi

(/v) When point P lies on the wire conductor, then d7 and 7 for each element of the straight wire
conductor are parallel. Therefore dl x r = 0 . So the magnetic field induction at P = 0.
Direction of magnetic field.The magnetic field lines due to straight FIGURE 4.9
conductor carrying current are in the form of concentric circles with the
conductor as centre, lying in a plane perpendicular to the straight B e
conductor. The direction of magnetic field lines is anticlockwise, if the
current flows from A to 5 in the straight conductor Fig. 4.9(a) and is
clockwise if the current flows from 5 to A in the straight conductor, Fig.
o
4.9(b). The direction of magnetic field lines is given by Right Hand A
o
A'
Thumb Rule or Maxwell’s cork screw rule.
4/9
MOVING CHARGES AND MAGNETISM

Right hand thumb rule. According to this rule, if we imagine the


linear wire conductor to be held in the grip of the right hand so FIGURE 4.10

that the thumb points in the direction of current, then the curvature
of the fingers around the conductor will represent the direction
of magnetic field lines, Fig. 4.10(a) and (b).
MaxwelVs cork screw rule. According to this rule, if we imagine a o

right handed screw placed along the current carrying linear wire
conductor, rotated such that the screw moves in the direction of
flow of current, then the direction of rotation of the thumb gives Q O G
the direction of magnetic lines offorce. This rule is illustrated in
Fig. 4.10(c)

'Sample Protoiom Q An infinitely long straight conductor carries a current of 100 A. At what
distance from the conductor is the magnetic field caused by the current equal to 0*5 x 10 T ?

w
Sol. Here, 7=100A;a = ?;5 = 0-5xl0^T
10-'^x2xl00
\^2l — 0*4 m

Flo
As, B = or a -
a 471 B 0-5 X10"^
FIGURE 4.11

reee
'sarnbie,' Problem W Find an expression for the magnetic 1 2a
field induction at the centre of a coil bent in the form of a square of Ar B

FFr
/
/

side 2 a, carrying current I, Fig. 4.11.


S
/
S
\

/
\

Sol. Here, (l)i = 45°;(J)2 = 45°



a N
/
urr
for
s

Total magnetic field induction at O due to current through all the la

sides of a square is four times that due to current through one arm of the a 45®
^ \
kkss
square and is given by
s
✓ \
a / \
✓ N
Yo
ooo

/ N
/ \

(sin 45“ +sin 45°) =4x


/ \

B = 4x C
4na[^ y[2 ^
D
4tc a na 2a I
eB

4.7. MAGNETIC FIELD AT THE CENTRE OF THE CIRCULAR COIL CARRYING CURRENT
r

Consider a circular coil of radius r with centre O, lying with its plane in the plane of paper. Let / be the
ou
ad

current flowing in the circular coil in the direction shown. Fig. 4.12. Suppose the circular coil is made of a
YY

large number of current elements each of length dl.


According to Biot-Savart’s law, the magnetic field at the centre of the circular coil due to the current
nndd
Re

—>

element / dl is given by
Fi

or dB^
Pq I dir sin 0 _ Pq / dl sin 0
4tc 4tc r3 4ti

where 7^ is the position vector of point O from the current element.


Since the angle between dl and r is 90° (i.e., 0 = 90°), therefore,

dB =
p(j /t//sin9Q° or dB = ^!JL ...(12)
An 47t

In this case, the direction of dB is perpendicular to the plane of the current loop and is directed
inwards. Since the current through all the elements of the circular coil will contribute to the magnetic field in
the same direction, therefore, the total magnetic field at point O due to current in the whole circular coil can
be obtained by integrating Eq. (12). Thus
4/10
'Pn*^<iee^ a Fundamental Physics (XII) VOL.I

B= dB= dl
J J4jt;-2 47Cr^-*
But jdl = total length of the circular coil = circumference of the current loop = 2nr

B = — .27Cr =
1^0 2tc/
4tc r- 4jc r

If the circular coil consists of n turns, then dl = 2nrn

B = Ho / ^ ^ 2nnl ^0 “X27cn
/
47T
^27irn = 4tc ,..(13)
4tc r

i.e.. B = -X angle subtended by coil at the centre.


4te r

ww
Direction of B

In the above case, the direction of magnetic field induction B (i.e., magnetic flux density) at point O
due to current through circular coil is perpendicular to the plane of the circular coil directed inwards.

Flo
In general, if current through a circular coil flows in clockwise direction,

e
then the direction of magnetic field at the centre of the circular coil is

reree
perpendicular to the plane of the coil, directed inwards. On the other hand, if

r FF
current through a circular coil flows in anticlockwise direction, then the direction
of magnetic field at the centre of the circular coil is perpendicular to the plane of
the coil directed outwards.
uurr
The magnetic lines of force due to circular coil
perpendicular to the plane of the wire loop and are
foor carrying current are
concentric circles near the
ks s
wire and practically straight near the centre of the wire loop. If the radius of the
Yoo
ooook

current loop is very large, the magnetic field near the centre of the current loop
is almost uniform. Fig. 4.13. The direction of magnetic field at the centre of
eBB

circular current loop is given by Right hand rule.


Right Hand Rule. According to this rule, if we hold the thumb of right hand mutually
rr

perpendicular to the gnp of the fingers such that the curvature of the fingers represents the
ouu
ad

direction of current in the wire loop, then the thumb of the irght hand will point in the direction
YY

of magnetic field near the centre of the current loop.


nndd

Retain in Memory
Re
Fii

Magnetic field lines around a current carrying wire are crowded up when the wire is bent into a
circular loop.

.Sa^ne'e Rrobie?^ In hydrogen atom, the electron is making 6-6 x revolutions per second
in a circular path of radius 0*53 A®. What is the magnetic field induction produced at the centre of the
orbit?

Sol. Here, v = 6-6 x 10*^ rps ; r = 0-53 x 10"^° m ;

Current due to orbital motion of electron is ;


/ = -! = ev = (1-6 X 10-19) X (6-6 X 10*5) = 1.056 x lO'^ A
T

B = Hq 2nnl __ IQ-'^ x2x(22/7)xlx(l-056xlQ-3) = 12-5 Wb/m2


471
(0-53xl0-'0)
MOVING CHARGES AND MAGNETISM 4/11

4.8. MAGNETIC FIELD AT A POINT ON THE AXIS


OF A CIRCULAR COIL CARRYING CURRENT
Consider a circular coil of radius a with centre O.
Let the plane of the coil be perpendicular to the plane
of the paper and current I be flowing in the coil in the
direction shown in Fig. 4.14. Suppose P is any point
on the axis of the circular coil at a distance x from its
centre O. Clearly, OP = x.
Consider two small elements of the coil each of
length dl, at C and D which are situated at diametrically
opposite edges. Then

ooww
PC =PD = r =

Let ZCPO = {(> = ZDPO


According to Biot Savart’s law, the magnitude of magnetic field induction (i.e. magnetic flux density) at

e
P due to current element I dl at C is given by

re
rFFl
lio /^/sin 90*^

ree
dB =

F
4tu r2 (●.● a is small, therefore 0 = 90°)

rF
Idl

4ti (a^+x^) ...(14)

fsfoor
ouur
The direction of dB is in the plane perpendicular to dl and r , and is directed as given by right
kosk
handed screw rule, Le. acting along PE X to CP.
Similarly, the magnitude of magnetic field induction at P due to current element of length dl at D is
YYo
oo

given by
BB

dB' _
= 1;^ /^/sin 90° _ )Xq/^/ ...(15)
rre

4tz +x'^)
Its direction is along PF X to D.P
ouu
Y
ad

From (14) and (15), dB = dB' =


dY

4n(a^ + x^)
Here, ZYPE = ZY'PF = ZCPO = ZDPO =
innd
Re

Resolving dB and dB' into two rectangular components, we have (i) dB cos <|) acts along PY and dB
FFi

sin (|) acts along PX, (i7) dB' cos <]) acts along PY and dB' sin acts along PX.
Since the components of the magnetic field induction acting along PY and PY' are equal and opposite,
they cancel out each other and the components of the magnetic field acting along PX (i. e. along the axis of the
coil) being in the same direction are added up. The same is true for all the diametrically opposite pairs of
current elements of the circular coil.

Thus total magnetic field induction {i.e. magnetic flux density) at P due to current through the whole
circular coil is given by
■ p-Q / i// sin 4>
B= dB sin ^ = — sin<|)J dl
47t(a^ +x^) 4n{a^ +x^)
a
Since, sin<|) = and
jdl = lKa
4/12 ‘Pn.eUee^’4. Fundamental Physics (XIOBEOB

E~ 2jia - ^0 27ca^/ along PX


4n{a^ +x^)Va^ +x^ 4?l +j^2)3/2
If there are n turns in the coil, then Jf// = 27can
B =
1^0 2jr«/g^ .(16)
47t(g2+;c^)3/2
Special Cases. 1. When point P lies at the centre of the circular coil, then j: = 0. From (16), we have
ti() 2nnla^ _ M^o 2rcn/
4tc 4n a

which is the same as given by Eq. (13) derived earlier.


2. W7*en point P lies far away from the centre of the coil, then x>> a. Now as can be

w
neglected in comparison to From (16), we have
^0 2nl A

Flo
B = ...(17)
4n 4;t

e
(where na^ = A = Area of current loop.)

reee
I^o 2M

FFr
or B = ...(18)
4tc

where, n IA = M = magnetic dipole moment of the current loop.


for
ur
Thus the current loop can be regarded as a magnetic dipole which produces its magnetic field and
magnetic dipole moment of the current loop is equal to the product of ampere turns and area of current
kss
loop.
Yo
S.I. unit of magnetic dipole moment is A-m\
oo

The polarity of the magnetic dipole due to current loop can be decided as follows :
B

Look at one face of the coil. If the direction of current through the coil
re

is clockwise, then that face has South polarity. Fig. 4.15(a). If the direction
of current is anticlockwise, then that face has North polarity. Fig. 4.15(b).
ou
ad

3. When the point lies on the axis of the coil at a distance equal to the
YY

radius of the coil Le. x = a.

IIq 2nnla} _ M-q


nd
Re

then, B =
4^(g2+^2j3/2 a
Fi

4. The variation of magnetic field induction (B) and distance (x) of a


point on the axis of a circular coil carrying current is represented by graph as
shown in Fig. 4.16.

electric current is flowing in a circular


coil of radius a. At what distance from the centre on the axis of the coil
will the magnetic field be l/8th of its value at the centre ?
Sol. Magnetic field induction at a point on the axis at a distance x from the centre of the circular coil

B.I =
Ho 2%nla^
carrying current is. 4jt (fl2 +^2j3/2
M-o 2n/t /
Magnetic field induction at the centre of the circular coil carrying current is, B2 =
47t
MOVING CHARGES AND MAGNETISM 4/13

As per question ; =
^2 so I^O Innla'? _ Mq 2nnl 1 or
a
2
1
8
4ti; +;c^)3/2 4j^ a 8 («2+a:2)3/2 8a

or 8 <3^ = (a^ + or 2 a = (a^ or 4 a^ = a^ + or x = Vs a

4.9. MAGNETIC FIELD DUE TO VARIOUS CURRENT CARRYING CONFIGURATIONS


1. The magnetic field at the centre O of the circular arc of FIGURE 4.17

radius r lying In the plane of paper as shown in Fig. 4.17 is P

B = -0
4tc r
o<;]e I

w
Its direction (according to Right Hand rule) is normal to the
plane of paper downwards. If the direction of current in the arc is Q

Flo
from Q to P, then the direction of B is normal to the plane of paper
FIGURE 4.16
upwards.

ee
2. Magnetic field at the centre O of two concentric coils

Fr
inclined at 90’’ Fig. 4.18 is

B =

for
I1
ur
If /j = /2 = / and ri = T2 = r ► 02
ks
Yo

^1/
Then 5 = V2R
oo

' 2r
B

where n is the number of turns in each coil.


re

3. If two concentric coils are inclined at an angle 9, Fig. 4.19,


then magnetic field at O is
ou
ad

B =-Jb[Vb[+2B^^ COS0
Y

4. The magnetic field at the common centre of two concentric


nd

coils of radii rj and T2 having turns nj and in which the same current
Re

I is flowing in anticlockwise direction, Fig. 4.20(a), is


Fi

B =
2r
^ ^ ^ 1^0 ”2 ^ _ 1^0 ^ 'h I ”2
I 2'2 2 jr

The direction of B will be normal to the plane


of paper upwards.
If current is flowing in two opposite directions,
Fig. 4.20(b), then

B = ^^0^ !h^!h
2 L'i '2 J
Its direction is normal to the plane of paper
upwards.
4/14 7^>uuie4ifi,'4, Fundamental Physics (XII) VOL.I

5. If a charged ring having chaise of radius r rotates with a frequency v then


current in the ring is, / = qv.
\^p2nl _\ipl _\^o
Magnetic field at the centre of the ring is, B = 4jc r 2r 2r

6. If an electron of charge e is revolving around the nucleus in a circular path of radius r with

speed V, then current


T 2jcr/v 2jtr

Magnetic field at the centre of that circle, i.e., at the nucleus is,

oww
B =
2r 2r V 2itr / 4jtr^
FIGURE 4.21
7. Magnetic field due to two parallel very long thin linear conductors 2nd 1st
carrying currents in the same direction, separated by distance r, Fig. 4.21.

e
(0 at a point P which is at a distance r/2 from both the conductors will

re
FFrllo
B =
be:
4k r 4k r

rF
4--X—M

(«*) at a point Q which is at a distance xfrom first conductor and (r + x) Q

ee
-I P -a

^0 1 1
ouru
from second conductor will be : B =

rF
A
4jc
-+
lx r + xA
r

Its direction is normal to the plane of paper downwards,

fosor
(ill*) at a point P which is at a distance xfivm first and (r- x)firom second conductor will be :
os kf
4 ^4--
1 FIGURE 4.22
B =
ook
Yo
4tc Ljc r-jcj
Y
Bo

The direction of B will be normal to the plane of paper upwards if B is


reeB

positive and downwards if B is negative. P


oouY

8. The magnetic field at the centre of separation P of two long linear


ur

conductors carrying currents in the opposite directions. Fig. 4.22, is :


ad

♦ B2
B=B+S mi 2/ 4q 2/ _|1q8/
Yd

* ^ 4jc(r/2) 4it(r/2) 4jc r


nidn

r
Re

[*.● Direction of Bj and is perpendicular to the plane of linear conductor directed inwards]
FFi

i.arwple]>l(^t;afe0sda1] FIGURE 4.23


There are two parallel current
carrying wires X and Y as shown in Fig. 4.23. Find the magnitude
and direction of the magnetic field at points, F, Q and B.

Sol. Here, 7j = 10 A;/2 = 15 A; r= 5 cm = 5 x 10“^m. Let n 10A-‘ ’'15A


be the unit vector perpendicular to the plane of paper unwards.
The magnetic field at F due to current in wire X is
P Q R
2x10 A _ A
n = 4xl0~^wr 5 cm 5cm 5cm 5 cm
^ 4tc r 5x10-2
The magnetic field at F due to current in wire Y is
2/j (-i;) = 10-2 2x15 (-«) = -2xl0-5nr
4— 10cm
2 4tc (r + r + r) 15x10-2
MOVING CHARGES AND MAGNETISM
4/15

The resultant magnetic field at P is

+ ^ =(4xlO-^)«-(2xlO-5)n = 2xIO-^rn
= 2 X 10“^ T acting normally outwards.
At point Q, both 5j and will be acting normally inwards. So

B _Ho 2x10 , A |i(, 2x15


Q - + ^2 = (-«)
471 5x10-2
20 30
= 10-2 X + 10-2x
5x10-2 5x10-2
(-n) =(4x10-5+6x10-5) (-H)

ww
= 10”^ T acting normally inwards.
f

At point R, will be acting normally inwards and will be acting normally outwards. So,

FF loo
2x10 2x20
B
= Sj +^2 =10-2x i-n) +10-2 X

ee
R
(5 + 5 + 5)x10-2 (5x10"^)

ee r
rFrF
- —xIO +8x10 5 (n) = / ^
j. acting normally outwards
rur
4.10. AMPERE'S CIRCUITAL LAW ffoor
In electrostatics, we have studied Gauss’s Law as an alternative form of Coulomb’s Law. Exactly in the
ks
s^e way, Ampere Circuital Law is an alternative form of Biot Savart Law in magnetostatics. Though Ampere’s
YYoou
ookos

circuital Law is quite general, we use this law to calculate magneUc field in symmetrical situations.
Consider an open surface with a boundary C, Fig. 4.24, and the current / is
BBo

FIGURE 4.24
passing through the surface. Let the boundary C be made of large number of small
re

line elements, each of length dl. The direction of dT of small line element under
oouur
ad

Study is acting tangentially to its length dl. Let Bj be the tangential component of J.
Y

the magnetic field induction at this element. Then B and dT are acting in the
dndY

same direction, angle between them is zero. We take the product of B, and dl for
Re

that element. 'Then


FFini

B^dl=B.dl
If length dl is very small and products for all elements of closed boundary are added together, then the

sum tends to be an integral around the closed path or loop {i.e., ^). Therefore, sum of B.dT over all
elements on a closed path B.dl - Line integral of B around the closed path or loop whose boundary
coincides with the closed path. Ampere’s circuital Law states that this integral (i.e., line integral of B
around the closed path) is equal to times the total current passing through the surface

I.e.,
9 B.dl =\i^ I ..(19)

where I is the total current threading the closed path or loop and is the absolute permeability of the space.
4/16 “Px<xdee^ ^ Fundamental Physics (XII) tywi

The relation (19) involves a sign convention ; given by the Right Hand Rule. According to this rule,
when fingers of right hand are curled in the sense, the boundary is traversed in the loop integral j) B.d I,
then the direction of the thumb gives the sense in which current / is regarded as positive.
According to sign convention, for the closed path shown in Fig. 4.25, /j is FIGURE 4.25

positive and I2 is negative. Then, according to Ampere’s circuital law

() B ,dl = 1^0 ~
h

where is the total current enclosed by the loop or closed path.

The relation (19) is independent of the size and shape of the closed path or

ww
loop enclosing the current.
Thus Ampere's circuital law states that the line integral of the magnetic field B around any

Floo
closed path in free space is equal to absolute permeability (P-q) times the net current (/) passing
through the surface enclosed by the closed path i.e.,

ee
j)B.d7-^Ql

reer
rFF
Proof of Ampere’s Circuital Law taking a circular closed path. Consider a long thin straight conductor
uur r
XY lying in the plane of paper, carrying current I flowing in the direction shown in Fig. 4.26(fl). The magneUc
ffoor
field produced around the conductor is shown by concentric circles perpendicular to the plane of paper, with
their centre lying on the conductor.
sks
The magnitude of magnetic field induction produced at a point P at a perpendicular distance r from the
YYoo
ooko

conductor is given by. B = 1^0 2/^Pq/


eBB

4tc r 2nr

The direction of B is along the tangent to the magnetic field line at that point.
r

at P. The direction of B and dl is the same, i.e.,


ouur

Take a small element of length dl of the closed path


ad

the angle between them is zero.


Y

Therefore, the line integral of B around the closed circular path of radius r is
dY
Re

r t t

O B .dl =(^ Bdlcos0°= B dl = 27cr x2nr = \iQl


nnd
FFii

where i)dl = 2Tzr = circumference of the circular path. This is Ampere circuital law.
MOVING CHARGES AND MAGNETISM
4/17

Proof of Ampere’s Circuital law taking any arbitrary Closed Path


Consider any arbitrary closed path perpendicular to the plane of paper around a long straight conductor
XY carrying current / from X to Y, lying in the plane of paper, Fig. 4.26(Z>).
Let the closed path be made of large number of small elements, where

AB ; BC ; CD=dl^
Let dQ^, r/02, dB^, be the angles subtended by the various small elements at point O through which
conductor is passing. Thenc/9j + dB2 + dB^ + = 2 tc
Suppose these small elements AB, BC, CD are small circular arcs of radii
r,, T2. /-3, respectively.
dl
Then dB^ =
1 .
; dB^= — ; dB^ =
‘“2.
'i '-2 ^3

ww
If 5j, B^, B3 are the magnetic field induction at a point along the small elements d^T, dl~^ ’^^3*

FF loo
B,1 =
then
47t r «2 = ; ^3 = 4tc ’
1

ree
Incase of each element, the magnetic field induction B and current element vector f// are in the same

ree F
direction.

Line integral of 5 around the arbitrary closed path is


ur
§B.dl = B^.dl^ + B^.dl^+ ^3
fofroF
(^/j) + B2 (dl2) + S3 (dl^) +.
ks
dl.1 + _ Pq 2 / fr/j dl2 dl^
soo

dl2 + 4k dl^ + + ....


YYouor
4n .r1 4k 4tc
. ^2 ^3
BBook

^0
2I[dB^ +f/02+t/e3+....] = 2/x27E — Pq/, which proves Ampere circuital law.
r ee

4;r 471

Obviously, this result is independent of r, i.e., size of the closed path.


oouru
ad

Note. If the point P lies inside the conductor carrying current which is at some distance from the axis of
Y

conductor, then according to Ampere circuital law ^ B _ (/7*= p p /


nY d
Re

5^2?^ relative magnetic permeability of the material of conductor. For its detail, refer to Art.
FFindi

4.11. APPLICATIONS OF AMPERE'S CIRCUITAL LAW


Ampere s circuital law is a simple tool for evaluating easily the magnetic field at a point due to a
current. While finding the magnetic field due to a current, using Ampere circuital law, we shall have to choose
a closed curve or loop (called an amperian loop) such that at each point of the loop either.
(0 B is tangential to the loop or B is non-zero constant or
—>

(«) B is normal to the loop or


{Hi) B vanishes.

If L IS the length of the loop for which B is tangential and is the total current enclosed by the loop,
dien according to Ampere’s circuital law

^B.dl = BL = \i^
4/18 “Pnadeefi. a Fundamental Physics (XII)

4.12. MAGNETIC FIELD DUE TO INFINITE LONG STRAIGHT WIRE CARRYING CURRENT
Consider an infinite long straight wire lying in the plane of paper. Let / be the current flowing through
it from X to y. A magnetic field is produced which has the same magnitude at all points that are at the same
distance from the wire, i.e. the magnetic field has cylindrical symmetry around the wire.

—i

w
Let P be a point at a perpendicular distance r from the straight wire and B FIGURE 4.27
be the magnetic field at P. It will be acting tangentially to the magnetic field line Y
passing through P. Consider an amperian loop as a circle of radius r, perpendicular
to the plane of paper with centre on wire such that point P lies on the loop, Fig.

e
4.27. The magnitude of magnetic field is same at all points on this loop. The

e
magnetic field B at P will be tangential to the circumference of the circular loop.

orw
r
P

We shall integrate the amperian path anticlockwise. Then B and dl are acting

F
ullo
—► ^

FF
in the same direction. The line integral of B around the closed loop is , J

e
sre
oB.dl = oBdlcosO° = B Odl=B2Kr

r
X

koF
As per sign convention, I is positive.

oofr
uor
Using Ampere’s circuital law. sf
j) B .dl = M-o /
ko
B2Tzr=\i^I
Y
or
rBB
Yo
oo

^0^ ^1^0 21
eY

or B = ...(21
2nr 4n r
re
u

The above result as given by (21) is of great importance from several points of view.
(0 It tells that the magnitude of magnetic field at every point on a circle of radius r with wire along th<
odu
o
ad

axis at the centre is same. It means the magnetic field due to current through infinite straight wire has;
Y
din

cylindrical symmetry.
(i'O The direction of magnetic field at every point on the circle is tangential to it and lines of constan
Re

magnitude of magnetic field form concentric circles. These circular lines are called magnetic field lines
FF
in

That is why we say that the magnetic field lines have no starting point or end point but form a closed patl
unlike the electrostatic field in which the electrostatic field lines have a starting point at positive charge an<
end point on negative charge.
{Hi) The expression for the magnetic field at a point due to current through a straight wire provides
theoretical support to Oersted’s experiment,
(iv) Though the wire carrying current is infinite, the magnetic field due to it at non-zero distance is nc
infinite. Since B oc
Mr, therefore, the magnitude of magnetic field increases as distance of point from the wir
decreases.

4.13. MAGNETIC FIELD DUE TO CURRENT THROUGH A VERY LONG CIRCULAR CYLINDER
Consider an infinite long cylinder of radius R with axis XY. Let I be the current passing through th
cylinder. A magnetic field is set up due to current through the cylinder in the form of circular magnetic line
of force, with their centres lying on the axis of cylinder. These lines of force are perpendicular to the length c
cylinder.
MOVING CHARGES AND MAGNETISM 4/19

Case I. Point P is lying outside the cylinder. Let r be the FIGURE 4.28
perpendicular distance of point P from the axis of cylinder, where I
—>

r > /?. Let B be the magnetic field induction at P. It is acting tangential


—>

to the magnetic line of force at P directed into the paper. Here B


I
I
—> J et B .
and dl are acting in the same direction. 1
Applying Ampere circuital law we have I
Q
—> —>
;p
(fB.dl=\iQl or ^5c//cos0‘’ = |1q/ I
I
1
'●4
R ^
or
jBdl = llaI or B2nr = \iQl
I
I
I

or 5 =
M , Le., B oc 1/r
I

ww
27tr

Case n. Point Q is lying inside cylinder. Here r<R. We may have two possibilities,
(i) If the current is only along the surface of cylinder which is so if the conductor is a cylindrical sheet

Flo
of metal, then current through the closed path L is zero. Using Ampere circutal law, we have, 5 = 0

e
(//) If the current is uniformly distributed throughout the cross-section of the conductor, then the current

reree
through closed path L is given by B FIGURE 4.29

r FF
i 2 /r2
/' =
nR^ p2
uurr
Applying Ampere’s circuital law, we have foor
kss
Yoo
ooook

= /' =
eBB

or or 5 = — i.e., Boer.
p2 2jcp2
If we plot a graph between magnetic field induction B and distance from the axis of cylinder for a
rr

current flowing through a solid cylinder, we get a curve of the type, as shown in Fig. 4.29.
ouu
ad

inducHon is maximum for a point on the surface of sdM


Y

cfU^erf0^ingfuri;^nti>and is zero for apoirtt on the axps of cylinder.


dY

A loug Straight solid metal rod of radius 4 cm carries a current 2 A,


Re
innd

uniformly distributed over its circular cross-section. Find the magnetic field induction at a distance 3
cm from the axis of the wire.
Fi

Sol. The point P is lying inside the rod at a perpendicular distance r (= 3 cm) from the axis of the wire.
Draw a circular closed path of radius r with centre on the axis of rod such that the point P lies on this closed path.

Current enclosed by the closed path V = —^


7CP2
XTir
R^
The magnetic field produced due to current flowing in the rod at every point over the closed path is
tangential to it and is equal in magnitude at all the points of the path.
-> Ir^
Line integral of B over the closed path is o B .dl =\LqI' or B2nr = \iQ R^

or 5 =
_ Ho 2/ r _ 10~'^x2x2x3xl0~^ = 7*5 X lOr^ T
2nrR^ 4ti R^ (4x10-2)2
4/20 ‘P'ladccfi. 4- Fundamental Physics (XII) W«Tl
Note. As the point lies inside the conductor, the magnetic field produced at that point will be times
the above value obtained, where |X^ is the relative permeability of the material of the conductor.
4.14. THE SOLENOID

A solenoid is a piece of equipment which is used for generating magnetic field. A long solenoid is one
whose length is very large, compared to its radius.
A solenoid consists of a long metallic insulated wire wound in the form of a helix, where the neighbouring,
turns are closely spaced. Each turn can be regarded as a circular loop.
A solenoid consists of an insulating long wire closely wound in the form of a helix. Its length is
very large as compared to its diameter.

oww
Magnetic field due to a solenoid
When current is passed through the solenoid, then each turn of the solenoid can be regarded as a circular
loop carrying current and thus will be producing a magnetic field. The total magnetic field is the vector sum
of the magnetic fields due to currents through all the turns in the solenoid. Fig. 4.30(«3) represents the magnetic

e
field lines due to a section of the solenoid in an enlarged manner.

FFrlo
re
FIGURE 4.30 4-

ree
F
4
4

rF
fsoor
ouur
skf
> 4
ooko
4
YYo
Bo

>
reB

4
4
uur
ooY

4
ad

e o
dY

At a point outside the solenoid, the magnetic fields due to neighbouring loops oppose each other and at
a point inside the solenoid, the magnetic fields are in the same direction. As a result of it, the effective
innd
Re

magnetic field outside the solenoid becomes weak, whereas the magnetic field in the interior of solenoid
becomes strong and uniform, acting along the axis of the solenoid. Fig. 4.30(b). When the solenoid is more
FFi

and more tightly packed, the magnetic field due to current through the solenoid will become zero for a point
outside near the middle of solenoid and magnetic field becomes strong and uniform at a point in the interior
of solenoid.

Let n be the number of turns per unit length of solenoid and I be the current flowing through the
solenoid.

Consider a rectangular amperian loop PQRS near S| 4—iR FIGURE 4.31

the middle of solenoid as shown in Fig. 4.31, where


PQ = L Let the magnetic field along the path PQ be B. innm

This field along RS is zero. As the paths QR and SP are


perpendicular to the axis of solenoid, the magnetic field
component along these paths is zero. Therefore, the
paths QR and SP will not contribute to the line integral
B
I §-
of magnetic field B .
MOVING CHARGES AND MAGNETISM 4/21

The total number of turns in length L = nL.

The line integral of magnetic field induction B over the closed path PQRS is
R P

^ B.dl = ( B.dl + B.dl + B.dl + B.dl ...(20)


PQRS p Q R

R R p
Q Q -i

Here, B.dl = BdlcosO°=BL and B.dl = fi^/cos90^ = 0= B.dl


Q Q 5
P P

Also, B.dl =0 (outside the solenoid, B = 0)

ooww
R

^ B.dT = BL + 0 + 0 + 0= BL .(21)
PQRS

e
From Ampere’s circuital law

ree
rFl
► —>

Fre
O B.dl =1^0^ total current through the rectangle BQB5

rrF
PQRS
DO YOU KNOW ?
= Pq X no. of turns in rectangle x current 1. A linear solenoid carrying
ouur
From (21) and (22), we have
sffoo ...(22) current is equivalent to a bar
magnet.
okks
2. The magnetic field lines due
...(23)
Yo
BL = \lQn LI or B = Po n / to current carrying solenoid
ooo

resemble exactly with those of


BB

If N is the total number of turns of the solenoid, then n = NIL


a bar magnet.
3. The magnetic field induction
rr e

then
B = Po«/= P-ojf at a point near the middle just
ouu

outside the curved face of the


ad

This relation gives the magnetic field induction at a point well


Y

solenoid carrying current is


inside the solenoid. At a point near the end of a solenoid, the
dY

zero.
magnetic field induction is found to be Po n HI = Pq A7/2 L, which
4. The net force on the magnetic
is half the magnetic field induction at the centre of the solenoid.
innd

dipole inside the solenoid


Re

From the above study we find that magnetic field lines due to carrying current is zero.
FFi

current carrying str^ght solenoid, is same as is due to a permanent


magnet.
If a magnetic material of permeability p is filled inside the FIGURE 4.32

solenoid, then magnetic field induction well inside the solenoid is 4B

\iN
5 = pn/=^/
L
The variation of magnetic field induction (B) of the solenoid I
I

with distance r from the centre of the solenoid along the axis of r-4-
I
>r
-L/2 O +L/2
solenoid is as shown in Fig. 4.32.

rsampfeteTogiSTnl A solenoid 50 cm long has 4 layers of windings of 350 turns each. The
radius of the lowest layer is 1-4 cm. If the current carried is 6-0 A, estimate the magnitude of magnetic
flux density (i) near the centre of the solenoid on its axis, (ii) near the ends on its axis, (iii) outside the
solenoid near its centre.
4/22
'a Fundamental Physics (XII)
Sol. Here, / = 50 cm = 0-50 m ; /● = 1-4 cm = 1-4 x 10“^m : / = 6-0 A ;
4x350 -1
No. of turns per unit length, n = = 2800 m
0-50

(i) The magnitude of B near the centre of solenoid on the axis is


fi = fion/ = 47uxl0-^x 2800 x 6-0 = 2-H x 10-2 j
(«) The magnitude of B near the end of solenoid on the axis is

jto«/ _ 2-11x10-2

w
B = = 1-05 X 10-2 T
2 2

(Hi) Outside the solenoid, the magnetic field is negligibly small as compared to that inside the solenoid.

roow
4.15. THE TOROID FIGURE 4.33

e
re
The toroid is a hollow
circular ring on which a large
number of insulated turns of a

reF
uFFll
metallic wire are closely wound. In /

e
I
1

fact, a toroid is an endless solenoid » *


t
4 I

in the form of a ring. Fig. 4.33(a).


I
I \

sFr
I
/

Magnetic held due to /

foro
current in ideal toroid X
uor
Let n be the number of turns
fk T
okso
per unit length of toroid and I be o
the current flowing through it. In
YYo
case of ideal toroid, the coil turns are circular and
oo

closely wound. A magnetic field of constant magnitude is


BB

set up inside the turns of toroid in the form of concentric circular magnetic field lines. The direction of the
magnetic field at a point is given by the tangent to the magnetic field line at that point. We draw three circular
rYree

amperian loops, 1, 2 and 3 of radii r^, r2 and r-^ to be traversed in clockwise direction as shown by dashed
oouu

circles in Fig. 4.33(/?), so that the points F, S and Q may lie on them. The circular area bounded by loops 2 and
3, both cut the toroid. Each turn of current carrying wire is cut once by the loop 2 and twice by the loop 3. Let
ad
Yd

Fj be the magnitude of magnetic field along loop 1. Line integral of magnetic field B^ along the loop 1 is
nidn

<p B^.dl = () i//cos0® = 5j 2 7C T] ...(0


Re

loop I loop 1
Loop 1 encloses no current.
FFi

According to Ampere’s circuital law

O B^.dl = |Xq X current enclosed by loop 1=Pqx0 = 0 or B^2k r^ = Q or S, =0


loop I
Let S3 be the magnitude of magnetic field along the loop 3. The line integral of magnetic field S3 along
' —^ ^ f
the loop 3 is 0 B^.dl = 0 B^dlcosQ°= B^2nr^
loopZ ioop'i

From the sectional cut as shown in Fig. 4.33(/?), we note that the current coming out of the plane of
paper is cancelled exactly by the current going into it. Therefore, the total current enclosed by loop 3 is zero.
According to Ampere’s circuital law
I —i >

T ^3 ● d/ = Pq X total current through loop 3 orS3 27ir3=pQx0 = 0 or


S3 = 0
loopi
MOVING CHARGES AND MAGNETISM 4/23

Let B the magnitude of magnetic field along the loop 2. Line integral of magnetic field along the
O B .dl =
loop 2 is
loop!

Current enclosed by the loop 2 = number of turns x current in each turn - l%r2nxl
According to Ampere’s circuital law

() 5.i//= jXq X total current or B 2 it r2 = ^ 2 ^''2'* B = )i^nl


hop!
This result is similar to that of solenoid.

The magnetic field at any point inside the empty space surrounded by toroid or outside tlie toroid is

ooww
zero.

hJ
The magnetic field at any point inside the toroid = JAq « / = JXq I
27tr
where N is the total number of turns in toroid and r is the mean radius of Toroid.

ee
Note that in reality, the turns of the toroid form a helix and not circle as is the case for ideal toroid. There

r
rFFl
is also a small magnetic field in the external portion of the toroid,

ree
F
isarnpielgt^'biem:; A coil wrapped around a toroid has inner radius of 10*0 cm and an outer

rF
radius of 12*0 cm. If the wire wrapped makes 1100 turns and carries a current of 5 A, find the magnetic
field (a) inside the core of the toroid (/>) outside the toroid at a distance 10 cm from the outer surface of

fsfoor
ouur
toroid.

Sol. Here, Tj = 10-0 cm ; T2 = 12-0 cm ; / = 5 A


kosk
10 + 12
Mean radius of toroid, r = = 11-0 cm = 11 X 10 ^ m = 011 m
YYo
oo

2
BB

Total length of toroid = circumference of toroid = 2 Tir = 2tc x 0-11 = 0-22 k m


total number of turns, N= 1100
rre

N _ 1100 -I
oYuu

Number of turns per unit length will be. 2nr 0-22 jc


m
ad

(a) Magnetic field induction inside the core of the toroid,


dY

1100
S= =(4jcx10 ^)x x5 = 0-01T
innd
Re

0-2271
FFi

(b) Magnetic field induction outside the toroid is zero because the field is only confined within the core
of the toroid on which winding has been made.

4.16. COMPARISON BETWEEN AMPERE'S CIRCUITAL LAW AND BIOT SAVARTS LAW
Similarities.

(1) Both relate the magnetic field with current.


(2) Both express the same physical consequences of a steady current.
Di^erences.

(1) Ampere’s law is applicable for symmetrical current distributions whereas the Biot Savart’s law is
applicable for asymmetrical current distributions.
_ —*
(2) Ampere’s law is for integral form of B and Biot Savart’s law is for differential form of B .
(3) Ampere’s law is based on the principle of electromagnetism and Biot Savart’s law is based on
magnetism.
4/24 A Fundamental Physics (XII) PZSTWl
4.17. MOTION OF A CHARGED PARTICLE IN A UNIFORM ELECTRIC FIELD
Suppose a particle of mass m, carrying charge
+ ^ is moving with velocity v along OX {i.e. along X-
axis). In the absence of an electric field, it meets the
screen PQ at point A, Fig. 4.34. Let the charged
particle be subjected to a uniform electric field of
strength E acting along OY{i.e. along Y-axis). Force
on charged particle due to electric field, F = qE
It is directed along E , i.e., along OY.
:. Acceleration of particle along OY is,

a =
qE

ww
m

This acceleration being perpendicular to the motion of charged particle, will not change the speed of the
charged particle along OX.

Floo
If X is the length of the region of electric field, then the time taken
by the charged particle to traverse it is given by DO YOU KNOW ?

ee
^ __ X If a charged particle is describing

eer
V a circular path of radius r under

rFrF
If y is the displacement of the charged particle along OY direction the effect of perpendicular
in the electric field during time t, then using the relation electric field, {e.g. motion of
rur
electron around the nucleus of an
S = «r + —
2
s ffoor atom), then
1. the force on the charged
osk
\ qE ^2 _ 1 qE (x
YYoou
particle is inversely proportional
We have. y = (0)r + --
oook

m 2 m to the square of radius of the


eBB

2mv^ 1
or x^ = circular path i.e.
r

2mv^ 2. the velocity of the charged


ouur
ad

or x^:=ky = k = a constant particle is inversely proportional


qE to the square root of the radius of
Y

This is an equadon of a parabola. Hence inside the electric field, the circular path I.e. ue«l/Vr
nd dY

the charged particle moves on a parabolic path OB and on leaving the


Re

field, it moves along a straight path BC, tangent to the curved path OB 3. the square of time period of
revolution of the charged particle
FFini

at B.
is directly proportional to the
Important points
cube of the radius of circular path
1. If a positively charged particle having charge q is at rest or moving I.e.

in the direction of electric field E , it experiences a force q E in the 4. A charged particle moving
—>
perpendicular to the direction of
direction of E and hence the particle is accelerated in the direction of
electric field gets accelerated
E . along or opposite to the direction
2. If a positively charged particle is moving opposite to the direction of electric field which causes the
change in speed, velocity,
of electric field E , if experiences a force ~ qE and henceit is retarded.
momentum and kinetic energy of
3. If the charged particle moves perpendicular to the direction of particle.
electric field E , it describes a parabolic path in the electric field.
MOVING CHARGES AND MAGNETISM 4/25

4.18. MOTION OF A CHARGED PARTICLE IN A UNIFORM MAGNETIC FIELD

Consider a uniform magnetic field of induction B directed along OX in the plane of the paper. Let a

rw
charged particle of mass m carrying charge q enter this field with a velocity v making an angle 0 with OX.
Resolving ^ into two rectangular components, we have : v cos 0 (= Vj) acting in the direction of the
magnetic field and v sin 0 (= v{) acting perpendicular to the direction of magnetic field.

r
wo
llou
F
FF
s
uro
ker
e
For
ro
For component velocity , the force acting on the charged particle due to magnetic field is

fof
o
Y
DO YOU KNOW ?
ooY
F = q (U2X B)
When a charged particle is
B
F= V2X?l=^t'2'Ssin90“ =^(t'sin0)B
ks
or ...(24) projected perpendicular to the
Yo
magnetic field,
r

The direction of this force F is given by Right Hand Rule and is (1) its path is circular in a plane
eB
e

perpendicular to the plane of


u
rd

perpendicular to the plane containing B and t>2 . As this force is magnetic field and direction of
motion of the charged particle.
perpendicular to V2 it cannot change the magnitude of velcity V2 ■ It
o
Yn
ou

(2) the speed and kinetic energy


ad

changes only the direction of motion of the particle. Due to this force,
of the particle remain constant.
the charged particle is made to move on a circular path in the magnetic
(3) the velocity of the charged
di

field, as shown in Fig. 4.35(i»).


particle changes only in direction.
FF

Here, magnetic field is shown perpendicular to the plane of paper


Re

Hence velocity and momentum of


in

directed inwards and particle is moving in the plane of paper. When the particle will charge.
particle is at points A, C and D the direction of magnetic force on the (4) the force acting on the charged
particle will be along AO, CO and DO respectively, i.e., directed towards
particle is independent of the
the centre O of the circular path. radius of circular path but
The force F on the charged particle due to magnetic field provides depends upon the speed of the
the required centripetal force = (mv^ I r) necessary for motion along charged particle i.e., F ^ (but
a circular path of radius r. F v).
(5) the time period of revolution
BqV2 =mv\l r or
V2 = B q r/m
of a charged particle in the
or V sinQ = B q r/m ...(25) magnetic field is independent of
The angular velocity of rotation of the particle in magnetic field velocity of particle and radius of
will be. the circular path i.e., T OC u®.
The time period depends upon the
(0 =
vs\x\^ _ B qr _ Bq ratio of charge/mass.
r m r m
4/26 4. Fundamental Physics (XII)
The frequency of rotation of the particle in magnetic field will be,

V -
CQ Bq
2lt 27tm ...(26)

The time period of revolution of the particle in the magnetic field will be,
1 27tm
7 = -
...(27)
V Bq
Thus time period is independent of the velocity of charged particle.

oww
From (26) and (27), we note that v and T do not depend upon velocity v of the particle. It means, all
the charged particles having the same specific charge (charge/mass) but moving with different velocities at a
point, will complete their circular paths due to component velocities perpendicular to the magnetic fields in
the same time.

e
For component velocity Uj (= v cos 0), there will be no force on the charged particle in the magnetic
—^ ^

re
field, because the angle between Uj and B is zero. Thus the charged particle covers the linear distance in

FFrllo
the direction of the magnetic field with a constant speed v cos 0.

rF
Therefore, under the combined effect of the two component velocities, the charged particle in magnetic

ee
field will cover linear path as well as circular path i.e. the path of the charged particle will be helical, whose
ouru
axis is parallel to the direction of magnetic field, Fig. 4.35(a).

rF
The linear distance covered by the charged particle in the magnetic field in time equal to one revolution

fosor
2k m
of its circular path (known as pitch of helix) will be, J = v^T = v cos 0
os kf Bq
Discussion
ook
Yo
Y
1. If a charged particle having charge q is at rest in a magnetic field B , it experiences no force; as
Bo

u = 0 and F = ^t;fisin6 = 0
reeB

2. If charged particle is moving parallel to the direction of , it also does not experience any force
oouY

because angle 0 between v and B is 0° or 180° and sin 0° = sin 180° = 0. Therefore, the charged particle in
ur

this situation will continue moving along the same path with the same velocity.
ad
Yd

3. If charged particle is moving perpendicular to the direction of 6 , it experiences maximum force,


nidn

which acts perpendicular to the direction of B as well as v . Hence this force will provide the required
Re

centripetal force and the charged particle will describe a circular path in the magnetic field of radius r, given
FFi

2
mv mv V
by = Bqv or r —

Bq {q/m)B
where (q/m) represents the specific charge of charged particle.
Thus, radius of the circular path is directly proportional to the speed and mass of the charged particle or
inversely proportional to specific charge of the charged particle.
4. From above relation it is clear that roc j_e., a slow moving charged particle describes a circular path
of small radius and fast moving charged particle describes a circular path of larger radius.
5. As electron and proton have same magnitude of charge but mass of proton is greater than that of
electron so specific charge of electron is greater than that of proton.
Therefore, if both electron and proton moving with same velocity enter a uniform magnetic field acting
perpendicular to the direction of motion of particles, then the electron describes a circular path of smaller
radius in magnetic field than that described by a proton because r ©c l/(q/m).
6. When two equal and oppositively charged particles (say electron and proton) having same momentum
enter a uniform perpendicular magnetic field, then the radii of their circular paths are equally curved.
MOVING CHARGES AND MAGNETISM 4/27

'Sj^iin'ipiej Problem D A beam of ions with velocity 2 x 10^ ms ^ enters normally into a uniform
magnetic Held of 0*04 tesla. If the specific charge of ion is 5 x 10^ C kg~^ find the radius of the circular

rw
path described.
Sol. Here, v = 2 x 10^ ms“* ; B = 0-04 T ; q/m = 5x10^ C/kg
mv V 2x10^
As qv B = m v^lr or r = = 0*1 m
qB {q! m) B (5xl0^)x0-04
Sample Problem Q An electron of mass 0*90 X 10"^ kg, under the action of magnetic field
moves in a circle of radius 2*0 cm at a speed of 3*0 x 10^ ms“‘. If a proton of mass 1*8 x 10"^^ kg were

r
wo
to move in a circle of the same radius in the same magnetic Held, Hnd its speed.

llou
Sol. Here, = 9 x IQ-^* kg ; = 0-02 m ; = 3 x 10^ ms"' ;

F
= 1-8 x kg ; =? ;
Kp - 0-02 m.
We know that qv B = m vVr or V - q r B/m ;

FF
s
V m
So L - P P
[as qp = qg-,rp = rJ

ro we q^rBIm^

er
m

k
e e p

e
rn^ _ 3x10^x9x10-'-^'

For
V = V X = 1*5 X 10^ ms-I

ro
p m 1-8x10“27
p

fof
o
YY
ooY
4.19. LORENT2 FORCE
ou
B
The force experienced by a charged particle moving in space where both electric and magnetic
ks
fields exist is called Lorentz force.
r

Force due to electric field. When a charged particle carrying charge +q is subjected to an electric field
eeB
u
rd

of strength E , it experiences a force given by


...(28)
Yn
oou
ad

The direction of is the same as that of £ .


di

Force due to magnetic field, if the charged particle is moving in a magnetic field B , with a velocity
FF
Re

V it experiences a force given by F^^^=q{vxB)


in

The direction of this force is in the direciton of vxB i.e. perpendicular to the plane containing v and
B as per Right hand screw rule.
Due to both, the electric and magnetic fields, the total force experienced by the charged particle is

F = F^+ F^ = q E+q (vx B) = q {E+ vx B) ^

F = q (£+ i»x B ) ...(29)


This is coh&d Lorentz force.
Special cases

Case I. When v, E and B , all the three are collinear. In this situation, the charged particle is
moving parallel or antiparallel to the fields, the magnetic force on the charged particle is zero. The electric

force on the charged particle will produce acceleration a —


m
4/28 Fundamental Physics (XII)QSm

along the direction of electric field. As a result of it, there will be change in the speed of charged particle
along the direction of the field. In this situation, there will be no change in the direction of motion of the
charged particle but, the speed, velocity, momentum and kinetic energy of charged particle will change.

Case II. When v, E and B are mutually perpendicular to each other. In this situation, if E and
—»

—> -4 —> —^ f
B are such that F == F + F =0, then acceleration of the particle, a = — = 0. It means the particle will
e m fjj

pass through the fields without any change in its velocity. Here, Fg = F^„ so qE = q v B or v = E/B.
This concept has been used in velocity-selector to get a charged beam having a definite velocity.
A A

^SampiejRrdbierhii An a-particle is moving in a magnetic field of (3r + 2y) tesla with a


velocity of (5 x 10® /) ms“^. What will be the magnetic force acting on the particle ?

ww
= (5x10® T):
—»

Sol. Here, B = (3/ + 2;) and V


q = 2 e.

Floo
F = q (v-xB) =^[(5x10® i)x(3?+2j)] = qx\0^ k-=2ex\0^ ic = 2 x 1-6 x 10“’^ x 10^ A
—>

ee
or I F I = 3*2 X 10"^® N, towards positive z-direction.

reer
rFF
4.20. VELOCITY FILTER

Velocity filter is an arrangement of cross electric and magnetic fields in a region which helps us
uur r
ffoor
to select from a beam, charged particles of the given velocity irrespective of their charge and
mass.
sks
YYoo
A velocity selector consists of two slits 5j and 52 held FIGURE 4.36
oooko

, Magnetic field
parallel to each other, with common axis, some distance apart.
acting Ir to the
In the region between the slits, uniform electric and magnetic
eBB

electric field

fields are applied, perpendicular to each other as well as to


the axis of slits, as shown in Fig. 4.36. When a beam of charged
r

particles of different charges and masses after passing through


ouur
ad

/ I
-4 I
\ S2
slit 5j enters the region of crossed electric field E and
r I
Y

Si \
I

A beam
magnetic field B , each particle experiences a force due to \±
V
nd dY

of + + +

these fields. Those particles which are moving with the velocity
Re

charged
particles
V, irrespective of their mass and charge, the force due to electric +
FFini

Electric
field (qE) is equal and opposite to the force due to magnetic field

field (qv B), then


q E= qv B or V = E/B

Such particles will go undeviated and filtered out of the region through the slit S2. Therefore, the par
ticles emerging from slit S2 will have the same velocity even though their charge and mass may be different.
The velocity filter is used in mass spectrograph which helps to find the mass and specific charge (charge/
mass) of the charged particle.

4.21. CYCLOTRON

A cyclotron is an equipment used for accelerating positively charged particles like proton, deutron,
alpha particles and heavier positive ions to desired velocities. It was developed by Lawrence and Livingstone.
Principle. The working of the cyclotron is based on the fact that a positively charged particle can be
accelerated to a sufficiently high energy with the help of smaller values of oscillating electric field by making
it to cross the same electric ifeld time and again, with the use of strong magnetic field.
MOVING CHARGES AND MAGNETISM 4/29

Construction. It consists of two £)-shaped hollow evacuated metal chambers Z>| and D2 called the
dees. These dees are placed horizontally with their diametric edges parallel and slightly separated from each
other. The dees are connected to high frequency oscillator which can produce a potential difference of the
order of 10^ volts at frequency ® 10^ Hz. The two dees are enclosed in an evacuated steel box and are well
insulated from it. The box is placed in a strong magnetic field produced by two pole pieces N,S of strong
electromagnets. The magnetic field is perpendicular to the plane of the dees. The ionic surface or the source
of positively charged particles is held at the centre P, as shown in Fig. 4.37.
Working and theory. At any instant, suppose Dj is at

w
negative potential and D2 is at positive potential. Therefore,
the positively charged ion produced at P, will be accelerated
towards Dj. On reaching inside Dj, the ion will be in a field
free space. Hence it moves with a constant speed in Z>j say v.

e
But due to perpendicular magnetic field of strength B, the ion

row
re
will describe a circular path of radius r (say) in D^, given by
2
mv

FFllo
F
Bqv =
r

where m and q are the mass and charge of the ion.

u
ree
mv

sFr
r —

Bq

kro
Time taken by ion to describe a semicircular path is given
uor
by, offo
kos
Ttr Tim 71
t =
YYo
eerBB

V Bq B{qlm)
oo

= a constant.
rY

This time is independent of both the speed of the ion and radius of the circular path. In case the time
uu

during which the positive ion describes a semicircular path is equal to the time during which half cycle of
electric oscillator is completed, then as the ion arrives in the gap between the two dees, the polarity of the two
ad
doo

dees is reversed i.e. Dj becomes positive and Dj negative. Then, the positive ion is accelerated towards D2
nY

and it enters D2 with greater speed which remains constant inside D2. The ion will describe a semicircular
path of greater radius due to perpendicular magnetic field and again will arrive in a gap between the two dees
exactly at the instant, the polarity of the two dees is reversed. Thus, the positive ion will go on accelerating
nid
Re

every time it comes into the gap between the dees and will go on describing circular path of greater and
FFi

greater radius with greater and greater speed and finally acquires a sufficiently high energy. The accelerated
ion can be removed out of the dees from window W, by applying the electric field across the deflecting plates
E and F.

Maximum Energy of positive ion


Let Uq, Tq = maximum velocity and maximum radius of the circular path followed by the positive ion in
cyclotron.
2

Then,
mv
0 Bqr^ Max.K.E.
1 7
=~m
1
= Bqv 0 or v
0 ”
m 2 ^ 2 m 2m
'b
Cyclotron Frequency
If T is the time period of oscillating electric field then T = 21 = 2 Tt mIBq
1 Bq
The cyclotron frequency is given by V = —
T 2nm
4/30 4- Fundamental Physics (XII)

It is also known as magnetic resonance frequency.


The cyclotron angular frequency is given by COp = 2 7C V = Bq/m
Note : In a given magnetic field, the two charged particles having the same cyclotron frequency, will
have the same value of specific charge (charge/mass).
Functions of Electric and Magnetic flelds in a Cyclotron
In cyclotron, the electric field is used to accelerate the heavy charged particle in a region between the
two dees. The magnetic field makes the heavy charged particle to describe the circular path inside the dees
and to reach in a gap between the two dees for further acceleration by electric field. Since in a magnetic field,
the charged particle describes a circular path so its motion is accelerated. Therefore, both the electric and the
magnetic fields in cyclotron are accelerating the charged particle.

oww
Limitations of cyclotron.
(1) When a positive ion is accelerated by the cyclotron, it moves with greater and greater speed. As the
speed of the ion becomes comparable with that of light, the mass of the ion increases according to the relation

e
FFrlo
re
1-
m = mQ
V

ree
F
where - the rest mass of the ion, m = the mass of the ion while moving with velocity v,
c = velocity of light.

rF
Km K
Now, the time taken by the ion to describe semicircular path is / =

fsoor
ouur
Bq Bq V
2
1-
skf
V C-2
ooko
It shows, that as v increases, t increases. It means, the positive ion will take longer time to describe
YYo
semicircular path than the time for half cycle of oscillating electric field. As a result of it, the ion will not
arrive in the gap between the two dees exactly at the instant, the polarity of the two dees is reversed and hence
Bo
reB

will not be accelerated further. Therefore, the ion cannot move with a speed beyond a certain limit in a
cyclotron.
(2) Cyclotron is suitable only for accelerating heavy particles like proton, deutron, a-particle etc.
uur
ooY

(3) The uncharged particles {e.g. neutrons) cannot be accelerated by cyclotron.


ad

(4) The lighter charged particle like electrons can not be accelerated by cyclotron because the mass of
dY

the electron is small and a small increase in energy of the electron makes the electrons move with a very high
speed. As a result of it, the electrons go quickly out of step with oscillating electric field.
innd
Re

Uses of cyclotron
1. The high energy particles produced in a cyclotron are used for artificial transmulation in order to
FFi

produce radio isotopes which are used for medical purposes in hospitals, to diagnose the chronic diseases and
for their treatment.

2. The high energy particles produced in a cyclotron are used to bombard the atomic nuclei and then to
study the resulting nuclear reaction which helps to investigate the nuclear structure.
3. It is used to add ions into the solids in order to improve the quality of solids.

4. The high energy particles produced in a cyclotron are used to produce other high energy particles
(like neutrons) through bombardment process, which are used in atomic reactors.
Sample; Problem D The magnetic flux density applied in a cyclotron is 3*5 T. What will be
the frequency of electric field that must be applied between the dees in order (a) to accelerate protons
(b) a-particles ? mass of proton 1*67 x 10“^^ kg.
Bq 3-5xl-6xI0-'^
Sol. (fl) v = = 5*35 X 10^ Hz.
27tm 2 X (22 / 7) X 1-67x10-27
MOVING CHARGES AND MAGNETISM 4/31

Bx2e 3-5x2xl-6xl0~‘^
{b) v = = 2-675 X 10’ Hz.
2TUx4m 2x(22/7)x4xI-67x10“2'^
Magnetic field applied on a cyclotron is 0*7 T and radius of its dees is
1-8 m. What will be the energy of the emergent protons in MeV ? Mass of proton = 1-67 x 10"^’ kg.
Sol. £ = (16x10-^9)2 X(0-7)2 X (1-8)2 _ (1-6x1Q-'9)2 x(Q.7)2x(1-8)2
J = MeV
2m 2x1-67x10-22 2x1-67x10-22 x1-6x10-*3
= 76 MeV

4.22. FORCE ON A CURRENT CARRYING CONDUCTOR PLACED IN A MAGNETIC FIELD


We know that current through a conductor is due to the motion of free electrons in the conductor in a
definite direction. When such a conductor is placed in a magnetic field, each electron moving in the magnetic

w
field experiences a force. Hence the current carrying conductor experiences a force when placed in the magnetic
field.

FF llowo
Expression for the force acting on the conductor carrying current placed In a magnetic field.
Consider a straight cylindrical conductor PQ of length /, area of cross-section A, carrying current I

ree
placed in a uniform magnetic field of induction, B . Let the conductor be placed along X-axis and magnetic
field be acting in XY plane making an angle 9 with X-axis. Suppose the current / flows thi'ough the conductor

rF
from the end P to Q, Fig. 4.38. Since the current in a conductor is due to motion of electrons, therefore, elec
trons are moving from the end Qxo P (along XX').
ur
FIGURE 4.38

Let, Vj drift velocity of electron, P


for fFreo Y B

Q
ks
-e = charge on each electron. 1 >9
YYour o
oo

Then magnetic Lorentz force on an electron e


o I X

is given by f = -e(v^xB) / ►
Bo Bks

If n is the number density of free electrons i.e. number of free electrons per unit volume of the conductor,
r ee

then total number of free electrons in the conductor will be given by N = n {Al) = nAl
oour u
ad

.-. Total force on the conductor is equal to the force acting on all the free electrons inside the conductor
Y

while moving in the magnetic field and is given by

F = N f =nAl[-e = ~n Al e {v^xB) ...(30)


nd
Re

We know that current through a conductor is related with drift


DO YOU KNOW ?
FFind Y i

velocity by the relation


I = nA e
11 = n A e Vj-1 The force on a wire carrying
current of any shape in a
We represent I I as current element vector. It acts in the direction uniform magnetic field is
of flow of current i.e. along OX. Since / T and have independent of its shape but
opposite
directions, hence we can write depends on the vector length
joining the beginning and
/ I =-n Alevj ...(31) ending of the current carrying
wire.
From (30) and (31), we have F=IIxB ...(32)

IFI = /I IxB 1

F= / /Ssin 9 ...(33)

where 9 is the smaller angle between / / and B .


4/32 'Pnadecfi- a Fundamental Physics (XII)

Special cases
Case I. If e = 0® or 180°, sin 0 = 0,
From (33), F = / /5 (0) = 0 (Minimum)
It means a linear conductor carrying a current when placed parallel to the direction of magnetic field,
experiences no force.
Case II. If 0 = 90°, sin0=l ;
From (33), F = //B X 1 = m (Maximum)
It means a linear conductor carrying current when placed perpendicular to the direction of magnetic
field, experiences maximum force, the direction of which is given by Right handed screw rule or Fleming’s

ooww
Left Hand Rule. Accordingly, the direction of F is perpendicular to the plane of paper directed upwards.

It is to be noted that the relation (33) is valid only if magnetic field B is uniform over the whole
length of the linear conductor.
Important Information

e
re
1. If the conductor carrying current / is not straight one and is placed in a uniform magnetic field B ,

rFFl
ree
then the magnetic force on the small element of this conductor of length dl is

F
rF
dF = l{d\.yE)
Total force experienced by the whole conductor is

fsfoor
ouur
F= dF = I{dl~xB)
kosk
\F\ = I ld/xBI = /I / IBsinO
Yo
oo
Y
BB

Here, 1 / I = magnitude of vector length of conductor ; which is the least distance between one end and
other end of current carrying conductor placed in a magnetic field.
rre

2. When a semicircular coil of radius r, carrying current / is placed in a uniform magnetic field B , then
ouu
Y
ad

1 M = 2r.
dY

Total magnetic force on semicircular coil is F = /(2r)Bsin 0.


3. When a circular coil or closed loop of any shape, carrying current / is placed in a uniform magnetic
innd
Re

field then I / ! = 0.
FFi

Total magnetic force on coil is zero as F = /xOxBsin0 = O

In this case, the different elementary parts of coil will experience different forces due to their different
orientations with the direction of magnetic field. The resultant force of all the forces on the various elementary
parts of coil is zero.
4. When a linear conductor is placed in a non-uniform magnetic field, then different elements of the
conductor will experience different forces.
fsg'mpieTRroBlg^ The horizontal component of earth’s magnetic field at a certain place is
3-0 X 10"^ T and the direction of the field is from the geographic south to the geographic north. A very
long straight conductor is carrying a steady current of 1 A. What is the force per unit length on it when
it is placed on a horizontal table and the direction of the current is (a) east to west (b) south to north ?
(NCERT Solved Example)
Sol. Here, B = 3-0 x 10"^ T ; / = 1 A ;/= 1 m
MOVING CHARGES AND MAGNETISM 4/33

(a) When current is flowing from east to west, 0 = 90®


F =// B sin 0 = 1 X 1 X 3-0 X 10-5 X sin ^qo _ 3.Q jq-s ^^,-1
(b) When current is flowing from south to north, 0 = 0°
/^ = / / B sin 0 = 1 X 1 X 3 0 X 10"5 x sin 0° = 0
Hence no force is acting per unit length on the conductor.

4.23. FORCE BETWEEN TWO PARALLEL LINEAR CONDUCTORS CARRYING CURRENT


In Fig. 4.39, Cj Dy and C2 D2 are two infinite long straight conductors carrying currents /j and I2 in the
same direction. They are held parallel to each other at a distance r apart, in the plane of paper. Magnetic field
is produced due to current through each conductor. Since each conductor is in the magnetic field produced by
the other, therefore, each conductor experiences a force.

oww
FIGURE 4.39

Dl D2 Di D2
■* <

e
B

ree
1

rFl
Fre
N

rr F
✓ N> s
t \
/
/■ \
' .F1
Q
N
i P I

\ S s /
s
ouur
sfoo
% ✓
V
kks
I1 i J2 -I2
Yo
oooo

1
B2
eBB

ci C2 C1 C2
e o
urr

Magnetic field induction at a point P on conductor C2 £>2 due to current /, passing through Cj Dy is
You
ad

B
1 ” ...(34)
dY

47t r
—}

According to right hand rule, the direction of magnetic field Bj is perpendicular to the plane of paper,
innd

directed inwards.
Re
Fi

As the current carrying conductor C2D2 lies in the magnetic field B, (produced by the current through CyDy),
therefore, the unit length of C2 D2 will experience a force given by FIGURE 4.39

£"2 = B) /2 X 1 = B| I2 Di ~r ► O2

Putting the value of B,, we have ...(35) 90“, ,1^


^ 4n r
Fi F2
4 ¥
According to Fleming's Left Hand Rule, force on conductor
C2 £>2 acts in the plane of the paper perpendicular to C2 £>2 , directed
towards Cy Dy. Similarly, it can be shown that the conductor Cy Dy 90*

also experiences a force given by (35), which acts in the plane of B2


^ ill 1
paper, perpendicular to Cy Dy and directed towards C2 £>2- Hence C, o
Dy and C2 £>2 attract each other. Ci C2
4/34 'PnaxUcp- '4- Fundamental Physics (XII) t-wm*
It means the two linear parallel conductors carrying currents in the same direction attract each
other. The force experienced by the unit length of each conductor is given by Eq. (35).
Further it can be shown that if the currents in conductors CjDjnnd C2D2are in opposite directions,
Fig. 4.39(c), they repel each other with the same force as given by Eq. (35).
The effective magnetic field due to two long linear parallel conductors carrying currents in the same
direction is shown in Fig. 4.40(a). When currents are in opposite directions, the resultant magnetic field is
shown in Fig. 4.40(/>). The pattern of the magnetic field indicates the force of aiuaction between two parallel
conductors carrying'current in the same direction and force of repulsion between two parallel conductors

w
carrying current in opposite directions.
FIGURE 4.40

e
✓ s
\
■* ✓
/ N S

' \ ' \ \\ 4

row
re

f ' ' ●V
s \
>
\ / f /
\ / '● ● \ \ \ > / f

▼ ▼ T T t I I T ▼ 4 ▲ 4 i 4 ( \ \
4
/ I I
I \ \ \ '● ✓ / \ '/III s
'K
.y // I \ V s 4 /
/ / S N 4
\ N \ / / / ✓

eeF
\ s V ✓ /

ullo
/
► /

FF
o o

srr
roF
Definition of ampere

k
Relation (13) is used to define 1 ampere.
uor
Let /, = /-, = 1 A ; r = 1 m. Then,
\Lq 2/^2
ofof 10-'^x2xlxl .. ^^0 = 10"'^
kos
F — — —
4tc r 1 4ti
YYo
eerBB
oo

= 2x 10"^ Nm
rY

Thus one ampere is that much current which when flowing through each of the two parallel
linear conductors placed in free space at a distance of one metre from each other will attract or
u

repel each other with a force of2x I0~^ N per metre of their length.
oou
ad
d
nY

4.24. EXPERIMENTAL DEMONSTRATION OF THE


MAGNETIC FORCE BETWEEN PARALLEL CURRENTS
nid
Re

Experiment I. Take two metallic wires A5 and CD.


FIGURE 4.41
Suspend them vertically parallel to each other. The lower
FFi

4-
ends of the wires are kept inside mercury in a trough. “
X_E
The current in the wires can be passed with the help of
X
battery E, and a key K in series. A, C A
c
I ●:k

In Fig. 4.41(a), when key K is closed, the vertical X


●:k
wires AB and CD bend towards each other and take the y
positions as shown by AB' and CD'. This shows that the r f
like currents attract each other.
©’ll)
In Fig. 4.41(/>), when key K is closed, the vertical
wires move away from each other and take the positions o o
as shown by AB' and CD'. This shows that the unlike
currents repei each other.
Experiment II: Roget’s Spiral
This apparatus is used to demonstrate the force of attraction between two parallel wires carrying currents
in the same direction. The apparatus used is shown in Fig. 4.42, where 5 is a soft metallic spring whose
MOVING CHARGES AND MAGNETISM 4/35

natural period of oscillation is about 0-5 s to 1 s. It is suspended vertically from a rigid support. The lower end
of the spring has a pointed tip B.D is a dish of brass having some mercury in it. £ is a source of direct current,
i.e., a battery which is connected between A and D through a key K.
Working. Adjust the spring so that the tip B of the spring just touches
the surface of mercury in dish. On closing key K, the tip of the spring
starts dancing up and down. It can be understood as follows ;
When tip of spring is just touching the mercury surface in dish
and the key K is closed, the circuit is completed. The current begins to
flow through the spring. The current through the spring will be equivalent
to parallel currents through the turns of .spring, flowing in the .same
direction. The turns of spring attract eachother, and the spring contracts.
The tip of the spring moves up and goes out of the surface of mercury in

ww
dish. Now the circuit is broken, the current stops flowing. The spring
relaxes and tries to comeback to its original position. The tip of spring
again touches mercury surface in dish, thereby completing the circuit.
The cycle continues with tick, tick, tick and the spring continues dancing.

Flo
Note. While doing Roget’s experiment, the students are advised to keep their faces away from mercury

e
vapours as they are poisonous. Students are also advised not to perform this experiment for a long time

ree
because in that situation they may inhale mercury vapours for long time which is very dangerous for health.

Fr
rF
Retain in Memory
uurr
1. If two linear current carrying conductors of unequal length are held parallel to each other, then

conductor and long conductor.


s for
the force on long conductor is due to magnetic field interaction due to current of short
kks
If /, £ = length of short and long conductor respectively
Yo
oooo

I], l2 = currents through short and long conductors


r = separation between these two parallel conductors
eB

^^0 ^2
(/) Then force on long conductor = force on short conductor = /
4tc
r
ou

(i7) The force on each conductor is same in magnitude and opposite in direction i.e. the forces on
ad
YY

two conductors obey Newton’s third law of motion.


2. When two currents approach a point or they go away from that point,
then they experience an attractive force. Fig. 4.43.
nndd
Re

3. When one current out of the two, approaches a point and another one
goes away from that point, then they experience a force of repulsion.
Fi

Fig. 4.44.
4. When two charges of values q ^ and q2 moving with velocities v j and V2 FIGURE 4.44
on parallel straight paths distance r apart, then the force acting between
them is given by
^ _ ^0 g| ^2 ^1 ^2
4tc
Proof. When a charge q moving with velocity v for time dt covers a
distancedl, it will behave as a current element / dl, where
dl
ldl = -^dl^ q- = qv
dt
If /j, I2 are the currents corresponding to motion of charges q^ and ^2 '''i^h velocity Vj and V2,
then =9, U| and I2 dl2 ~ q^ ^2"
4/36
^>uuUep. <*● Fundamental Physics (XII) P7STW

The magnitude of magnetic force acting between these two parallel current elements distance r

apart is given by _= Mo ^1^2 F = ^2


^d/jd/2
or
r2
m
4tc 4 71

5. A fixed horizontal wire A carrying current /j will support another FIGURE 4.45

horizontal wire B, carrying current I2, hanging in air at a distance r


below the wire A, if the direction of currents in the two wires is same. T
●A
ri F
Here, weight per unit length of wire B is equal to the force of attraction B
I2 img
Mq ^2

oww
acting per unit length on the wire B, i.e.. mg -
4k r

where m is the mass per unit length of wire B and g is the acceleration due to gravity.

e
^amplglB.saijSI Two straight wires A and B of lengths 10 m and 16 m and carrying currents

re
4-0 A and 5*0 A respectively in opposite directions, lie parallel to each other 4*0 cm apart. Compute the
force on a 10 cm long section of the wire B near its centre.

FFrllo
Sol. The magnitude of repulsive force per unit length on each wire is

reF
e
— = ilo. ^^1 ^2 = lO-"^ X
2x4-0x50
uoru I 4k (4x10-2)
r
= 10-^ Nm-^

osFr
Force on a 10 cm length of the wire B = 10“^ x 0-1 = 10“^ N

4.25. COMPARISON OF ELECTRICAL AND MAGNETIC FORCES


fkfor
okso
From Coulomb’s law, the magnitude of electrostatic force between two charges and ^2 separated by
distance r is given by
YYo
oo
BB

1 ^1
4 Tie
...(36)
0
r ree
Y

Consider two parallel conducting wires of length and d/2 carrying currents /j and I2 separated by
oouu

distance r.
The magnitude of the magnetic force acting between these two current elements is given by
ad
Yd

m
4k ...(37)
nidn

Let <?j and ^2 be the charges flowing for time t in wires for currents /j and I2. Then
Re
FFi

/jd/j = —-d/j
t
Vj
qj
and
t

F
.(38)
m
4k r2

Dividing (38) by (36) we get El m


5= V
1 U2 (\XqGq) ...(39)
F

Since left hand side is dimensionless, therefore, irght hand side of (39) must also be dimensionless. It
means the quantity Pq ^ 0 have the dimensions of (velocity)-^. Here v, and V2 are the drift velocities of
electrons in current elements. As drift velocities are of the order of 10"^ ms"^ therefore
U] V2 = 10-^ X 10-^ = 10-^ m2 s-2
MOVING CHARGES AND MAGNETISM 4/37

1
It is also found that c2 = or 1^0 ^o~ C-2 ...(39a*)
^0^0
-1
where c is the velocity of light in vacuum, i.e., c = 3 x 10^ ms
F 10-*
m
= 10-25
Putting values in (39), we get
F,
(3x10®)2
It shows that the magnetic forces are very much smaller than the electrostatic forces in current carrying
conductors. However, the electric forces never dominate the magnetic forces in current carrying conductors
because

(i) the matter is electrically neutral to very high degree of accuracy. The flowing charges do not disturb
the neutrality of the matter. Thus the Coulomb’s electric forces are very rarely in evidence.
(ii) In an electric current, the large number of electrons are drifting towards the same direction. This

w
adds up weak magnetic fields which become evident.
Note that if V\ and V2 were comparable to the speed of light, the magnetic and electric forces will be
comparable.

Flo
4.26. TORQUE ON A CURRENT CARRYING COIL IN A MAGNETIC FIELD

ee
Consider a rectangular coil PQRS

Fr
suspended in a uniform magnetic field
of induction B . Let PQ = RS = I and

for
QR-SP = b. Let I be the current flowing
ur
through the coil in the direction PQ RS
and 6 be the angle which plane of the
ks
coil makes with the direction of magnetic
Yo
field Fig. 4.46(a).
oo

^ ^ ^

Let Fj, F2, F3 and F^ be the


B

forces acting on the four current carrying


re

arms PQ, QR, RS and SP of the coil,


respectively.
ou
ad

The force on arm SP is given by,


Y

^ = / (^xB)
nd

F4 = / iSP) B sin (180® -Q) = IbB sin 0


Re

or

The direction of this force is in the direction of (SPxB) i.e. in the plane of coil directed upwards.
Fi

The force on the arm QR is given by, F2 = / (QR x B) or F2 = / (fiF) B sin 0 = / F sin 0
The direction of this force is in the plane of the coil directed downwards.

Since the forces ^ and are equal in magnitude and acting in opposite directions along the same
straight line, they cancel out each other i.e. their resultant effect on the coil is zero.
Now, the force on the arm PQ is given by
(V PQIB)
F^=l(PQxB) or F] =/(F0 Bsin 90" = //B
According to Fleming’s Left Hand Rule, the direction of this force is perpendicular to the plane of the
coil directed towards the reader.

1 1 1 11
^0= t^x4rt€«=10-2x
2
^ =c where c is the velocity of light.
4tc
0
9x10^ 9x10’5 (3x10*)2
4/38
Fundamental Physics (XII) VOL.I

And, force on the arm RS is given by


I {RSxB) or
F3 = /(P0Bsin (V RS LB)
The direction of this force, according to Fleming’s Left Hand Rule, is perpendicular to the plane of coil
and directed away from the reader.
The forces acting on the arms PQ and RS are equal, parallel and acting in opposite directions having
different lines of action. These forces form a couple, the effect of which is to rotate the coil in the anticlockwise
direction about the dotted line as axis.
The torque on the coil (equal to moment of couple) is given by t = either force x arm of the couple.

oww
The forces F, and F3 acting on the arms PQ and RS will be as shown in Fig. 4.46(/>) when seen from the
top. Arm of couple = ST = PS cos 0 = /> cos 6.
T = / /F X 6 cos 0 = IBA cos 0

(●.' l X b~A = area of coil PQRS) DO YOU KNOW ?

e
If the rectangular coil has n turns, then t = n / F A cos 0. 1. The net magnetic force on a

re
Note that if the normal drawn on the plane of the coil makes an current loop in a uniform

FFrllo
angle a with the direction of magnetic field, then magnetic field is zero but

rF
torque may or may not be zero.

ee
0 + a = 90“ or 0 = 90° - a ; and cos 0 - cos (90° - a) = sin a
Then torque becomes. 2. The net magnetic force on a
ouru current loop In a non-uniform

rF
X - nIBA sin a = MB sin a = \ Mx B I = I«/ Ax F I magnetic field is not zero but
torque may or may not be zero.

fosor
where, nIA = M = magnitude of the magnetic dipole moment of the 3. When a current loop is in the
os kf
rectangular current loop position of equilibrium in a
—*
uniform magnetic field, then
ook
Yo
X = Mx B = nl (AxB)
Y
the net magnetic force as well
This torque tends to rotate the coil about its own axis. Its value
Bo

as torque on current loop are


reeB

changes with angle between plane of coil and direction of magnetic field. zero.

Special cases 1. If the coil is set with its plane 4. Torque on the coil is maxi
oouY

parallel to the
direction of magnetic field B, then 0 = 0° and cos 0 = 1 mum. when coil is suspended
ur

in a radial magnetic field.


ad

Torque, x = nIBA (\) = nIBA (maximum)


This is the case with a radial field.
Yd

2. If the coil is set with its plane perpendicular to the direction of magnetic field B, then
nidn

0 = 90° and cos 0 = 0


Re

Torque, x = nIBA (0) = 0 (Minimum)


FFi

Sample Problem A circular coiI of 20 tums and radius 10 cm carries a current of 5 A. It Is


placed in a uniform magnetic field of 0-10 T. Find the torque acting on the coil when the magnetic field
is applied (a) normai to the plane of the coU (b) in the plane of coil. Also find out the total force acting
on the coll.

Sol. Here; n = 20 ; r = 0-10 m ; /= 5 A ; F = 0-10 T

22
Area of each turn of coil, A = 7i = 0 0314 m^

Torque acting on the coil, x = n IAB sin a


(a) Here, a = 0° so
X = 20 X 5 X (0-0314) x 0-1 x sin 0° = 0
(b) Here, a = 90° so
X = 20 X 5 X (0-0314) x 0-1 x sin 90° = 0*314 N-m.
Coil carrying cuitent acts as a magnetic dipole. The force acting on magnetic dipole in a uniform magnetic
field is zero.
MOVING CHARGES AND MAGNETISM 4/39

4.27. MOVING COIL GALVANOMETER FIGURE 4.47

Moving coil galvanometer is an instrument used for the


detection and measurement of small electric currents. It is also Phosphor
Bronze
called a dead beat galvanometer.
Strip
Principle. Its working is based on the fact that when a current
Ti .T2
carrying coil is placed in a uniform magnetic field, it experiences a
¥ ● IM

torque.
Si
Construction. It consists of a coil PQRSj having large number
of turns of in.sulated copper wire. Fig. 4.47. The coil is wound over a N S
L
non-magnetic metallic frame (usually brass) which may be rectangular
or circular in shape. The coil is suspended from a movable torsion
head H by means of a suspension fibre of phosphor bronze strip in a Q S' R

ww
C
uniform magnetic field produced by two strong cylindrical magnetic a

pole pieces N and S.


The lower end of the coil is connected to one end of a hair spring S' of

FF loo
quartz or phosphor bronze. The other end of this highly elastic spring* S' is
connected to a terminal 7’2- L is a .soft iron core which may be spherical if the

ee
coil is circular and cylindrical, if the coil is rectangular. It is so held within the

er
rFreF
coil, that the coil can rotate freely without touching the iron core and pole pieces.
This makes the magnetic field linked with coil to be radial field i.e. the plane of
rur
the coil in all positions remains parallel to the direction of magnetic field. Fig.
ffoor
4.48. M is concave mirror attached to the phosphor bronze strip. This helps us to
note the deflection of the coil using lamp and scale arrangement. The whole
ks
arrangement is enclosed in a non-metallic case to avoid disturbance due to air,
YYoou
okso

etc. The ca.se is provided with levelling .screws at the base. The torsion head is
connected to terminal T|. The galvanometer can be connected to the circuit
BBoo

through terminals and T2.


re

Theory.
Let, / = length PQ or /?5, of the coil, b = breadth QR or 5,F of the coil, n = number of turns in the coil.
oouur
ad

Area of each turn of the coil, A = I x b.


Y

Let, B = strength of the magnetic field in which coil is suspended.


I = current passing through the coil in the direction PQRS^ as shown in Fig. 4.47.
dnYd
Re

Let at any in.stant, a be the angle which the normal drawn on the plane of the coil makes with the
FFini

direction of magnetic field.


As already discussed, the rectangular coil carrying current when placed in the magnetic field experiences
a torque whose magnitude is given by t = nIBA sin a.
If the magnetic field is radial i.e. the plane of the coil is parallel to the direction of the magnetic field
then a = 90“* and sin a = 1. x = nIBA

Due to this torque, the coil rotates. The phosphor bronze strip gets twisted. As a result of it, a restoring
torque comes into play in the phosphor bronze strip, which would try to restore the coil back to its original
position.
Let 6 be the twist produced in the phosphor bronze strip due to rotation of the coil and k be the restoring
torque per unit twist of the phosphor bronze strip, then total restoring torque produced = kQ

*The spring S' does three jobs for us : (/) It provides passage of current for the coil PQRS, (ii) It keeps the
coil in position and (///) generates the restoring torque on the twisted coil.
4/40 ‘PfuuUe^ '4. Fundamental Physics (XII) WAWI
In equilibrium position of the coil, deflecting torque = restoring torque
nlBA = Jt 0 or 1 = — e or /=G0
nBA

where
nBA
a
constant for a galvanometer. It is known as galvanometer constant.
Hence, /« 0

It means, the deflection produced is proportional to the current flowing through the galvanometer. Such
a galvanometer has a linear scale.
Some important terms related to Moving Coil Galvanometer
1. Dead beat galvanometer. It is that galvanometer in which the coil in the deflected state comes to rest
atonce, after the passage of current through it and attains its initial position of rest atonce if current is switched
off. It means in dead beat galvanometer, the deflection can be noted in no time.

ww
A dead beat galvanometer can be achieved by winding the coil on a conducting frame. In such
galvanometer when coil rotates, the eddy currents are set up in metallic frame which produces damping effect
on the coil and coil comes to rest soon.

FF loo
2. Radial magnetic field. It is that magnetic field in which the plane of the coil in all the positions
is parallel to the direction of magnetic field.

ee
For radial magnetic field, the angle between the normal to the plane of the coil and direction of magnetic

ee r
rFrF
field is 90°, i.e., a = 90°.
Torque on the coil, X = nlBA sin 90° = nIBA
rur
which is maximum. In this situation, x «: /.
ffoor
Thus by using radial magnetic field in galvanometer, the deflection of the coil is proportional to the
current flowing through it. As a result of it, a linear scale can be used to determine the deflection of the coil
ks
in galvanometer.
YYoou
ookos

A radial magnetic field can be achieved in galvanometer by (i) properly cutting the magnetic pole
pieces in the shape of concave faces (n) using the soft iron core within the coil.
BBo

3. Properties of Suspension fibre used in galvanometer


re

(i) It should have high tensile strength.


(«) It should have high electrical conductivity.
oouur
ad

{Hi) It should be rustless with time,


Y

(/v) Its temperature coefficient of resistance should be small,


(v) Its restoring torque per unit twist should be small.
dndY
Re

4. Sensitivity of galvanometer
FFini

(a) Current sensitivity of a galvanometer is defined as the deflection produced in the galvanometer
when a unit current flows through it.
If 0 is the deflection in the galvanometer when current I is passed through it, then
0 nBA
Current sensitivity. I
I k nBA

The unit of current sensitivity is rad. A~^ or div. A~K


(b) Voltage sensitivity of a galvanometer is defined as the deflection produced in the galvanometer
when a unit voltage is applied across the two terminals of the galvanomete.r
Let,V = voltage applied across the two terminals of the galvanometer,
0 = deflection produced in the galvanometer.
Then, voltage sensitivity. v, = e/v

*0 = 0°, 0 + a = 90° implies a = 90°.


MOVING CHARGES AND MAGNETISM 4/41

If R = resistance of the galvanometer, / = current through it.


Then V=IR

Voltage sensitivity, ^nBA^ Is


^ m kR R
The unit of is rad or div.
Conditions for a sensitive galvanometer.
A galvanometer is said to be very sensitive if it shows large deflection even when a small current
is passed through it.
nBA
From the theory of galvanometer, 6 = I
k

For a given value of I, 0 will be large if nBA/k is large. It is so if

w
(a) n is large (b) B is large (c) A is large and (d) k is small.
(a) The value of n can not be increased beyond a certain limit because it results in an increase of the
resistance of the galvanometer and also makes the galvanometer bulky. This tends to decrease the sensitivity.

Flo
Hence n can not be increased beyond a limit.

e
(b) The value of B can be increased by using a strong horse shoe magnet,

rree
(c) The value of A can not be increased beyond a limit because in that case the coil will not be in a

r FF
uniform magnetic field. Moreover, it will make the galvanometer bulky and unmanageable.
(d) The value of k can be decreased. The value of k depends upon the nature of the material used as
uurr
galvanometer, quartz or phosphor bronze strip is used as a suspension strip.
Uses of moving coll galvanometer
for
suspension strip. The value of k is very small for quartz or phosphor bronze. That is why, in sensitive
kss
1. It is used to detect and measure a very small current.
ooook
Yo
2. It can be converted into ammeter by connecting a low resistance shunt in parallel to it.
3. It can be converted into voltmeter by connecting a high resistance in series of it.
eB

4. It can be converted into Avometer (i.e., ammeter, voltmeter and ohm meter) by using special
arrangement of resistance in it.
rr
ou
ad

[^arripieigr.obiernl A Current of 300 n A deflects the coil of a moving coil galvanometer through
60°. What should be the current to cause rotation through it/5 rad ? Determine the sensitivity of this
Y
dY

galvanometer.
It 180'
Re
innd

Sol. Here, I 1 = 300 pA ; 0] = 60° ; 5


Fi

k
/ = /oc 9 - a constant
As or
nBA nBA

k-h 02 36
so or
h ~ = /,1 = —x300
<f\ = 180 uA
^
/ 0 01
I

0. 36°
Current .sensitivity, I^ = 0-2°/pA
180 pA

4.28. WESTON TYPE GALVANOMETER


It is an electrical instrument which is used for the detection and measurement of current flowing through
the circuit. A weston type galvanometer is most commonly used in laboratory and is essentially a moving
coil, pivoted type dead beat galvanometer. It consists of a coil of many turns of fine insulated copper wire
wound over a metallic frame. The coil is mounted on two jewelled pivots and is symmetrically
4/42
Fundamental Physics (XII) or»Tl

placed between the cylindrical pole pieces of a strong permanent FIGURE 4.49
horse-shoe magnet. The motion of the coil is controlled by two
hair springs, one above and other below the coil, which are
soldered to the ends of the coil. These hair springs also serve as
leads for the current. An aluminium pointer is attached to its
moving part which moves on a scale with zero at its centre and Pointer

equal divisions (generally 25 or 30) on either side of central


zero. Fig. 4.49. Horse
Shoe
When current is passed through the galvanometer, its coil Magnet
N S
experiences a torque and gets rotated. As a result of it, the hair
springs get twisted, consequently the pointer shows a reading
on the scale. This galvanometer is very sensitive and gives full Coil
scale deflection for a small current. Hair Spring

w
4.29. SHUM

Flo
Shunt is a low resistance connected in parallel with the galvanometer or ammeter. It protects
the galvanometer or ammeter from the strong currents.

ee
If the current flowing in a circuit is strong, a galvanometer or a low range ammeter can not be put

Fr
directly in series with the circuit because that will damage the instrument. To overcome this difficulty, a low
resistance (i.e., shunt) is connected in parallel with this instrument. Then a major portion of the current passes
through this low resistance (i.e.. shunt) and only a small portion of current passes through the instrument. Due
for
ur
to it, the galvanometer or an ammeter remains safe.
To find the current through the galvanometer circuit, let
ks
/ = total current in the circuit, G = resistance of the galvanometer, S = resistance of the shunt,
Yo
Ig, f = currents through galvanometer and shunt respectively. It is clear that
oo

or
/, = /-/g or ...(40)
eB

FIGURE 4.50
According to ohm’s law. S

Potential difference between points A and B = l^G = f S ...(41)


r

I^G = (I-J^)S
or
'●s
ou
ad

S )
or Lg (G + S) = 1S or r =/ Ta r
Y

g
G + S

Putting the value of from (18) in (19), we get


nd
Re

G ^
(/-/,)G = /,5 or
IG = f(G + S) or
Fi

[G + S
Uses of shunt 1 ● A shunt is used to protect the galvanometer from the strong currents.
2. A shunt is used for converting a galvanometer into an ammeter.
3. A shunt may be used for increasing the range of ammeter.
Sample Problem A galvanometer has a resistance of 50 U. A resistance of 5 is connected
across its terminals. What part of the total current will flow through the galvanometer ?
(Hr. Board 2001)
Sol. Here, G = 50tl; S = 5n

5 1
As S = or
I-I I G +S 50 + 5 II
g

Fraction of total current through galvanometer is = L = 1 -

I 11
MOVING CHARGES AND MAGNETISM 4/43

4.30. AMMETER

An ammeter is a low resistance galvanometer. It is used to measure the current in a circuit.

A galvanometer can be converted into an ammeter by using a low resistance wire in parallel with the
galvanometer. The resistance of this wire (called the shunt wire) depends upon the range of the ammeter and
can be calculated as follows :
Let G = resistance of galvanometer, n = number of scale divisions in the galvanometer,
K = figure of merit or current for one scale deflection in the galvanometer.
Then current which produces full scale deflection in the galvanometer, = nK.
Let I be the maximum current to be measured by galvanometer.
FIGURE 4.51
To do so, a shunt of resistance 5 is connected in parallel with the
Ammeter
galvanometer so that out of the total current /, a part should pass z

ww
through the galvanometer and the remaining part (/ - /^) flows through s

the shunt, Fig. 4.51.


. ■(! -Ig) (I-lg)' ’
( I

Floo
8
G ...(42)
I-I » »

ee
s I I A Ig
Thus S can be calculated. '

eer
rFrF
If this value of shunt resistance S is connected in parallel with galvanometer, it works as an ammeter for
the range 0 to / ampere. Now the same scale of the galvanometer which was recording the maximum current
I before conversion into ammeter will record the maximum current /, after conversion into ammeter. It
rur
The effective resistance Rp of ammeter {i.e. shunted galvanometer) will be
s ffoor
means each division of the scale in ammeter will be showing higher* current than that of galvanometer.
osk
YYoou
GS
J___l_ I _ S+G or R =
oook

R ~ G S~ GS P G + S
p
eBB

As the shunt resistance is 4ow, the combined resistance of the galvanometer and the shunt is very low
and hence ammeter has a much lower resistance than galvanometer. An ideal ammeter has zero resistance.
r

Sample Problem An ammeter of resistance 0*80 Q can measure current upto 1-0 A.
ouur
ad

(a) What must be the value of shunt resistance to enable the ammeter to measure current upto 5 0 A ?
Y

(Jb) What is the combined resistance of the ammeter and the shunt ? (CBSE 2013)
nd dY

Sol. Here, G = 0-80 Q ; = 1 -0 A ; / = 5-0 A ; 5.


Re

IxO-8
FFini

(«3) S = = o-2n
5-1

GS 0-8 X 0-2
= 0-16 a
(6) Combined resistance of shunt and ammeter =
G + 5 ~ 0-8 + 0-2
4.31. VOLTMETER

A voltmeter is a high resistance galvanometer. It is used to measure the potential difference


between any two points of a circuit.
A galvanometer can be converted into a voltmeter by connecting a high resistance in series with the
galvanometer. The value of the resistance depends upon the range of voltmeter and can be calculated as
follows :

*Initial reading of each division of galvanometer to be used as ammeter is I^n and the reading of the .same
one division after conversion into ammeter is l/n.
4/44
4 Fundamental Physics (XII)
Let, G - resistance of galvanometer, n = number of scale divisions in the galvanometer,
K = figure of merit of galvanometer, t.e., current for one scale deflection of the galvanometer.
Current which produces full scale deflection in the galvanometer.
FIGURE 4.52
Let V be the potential difference to be measured by galvanometer. Voltmeter
To do so, a resistance R of such a value is connected in series r

with the galvanometer so that if a potential difference V is applied R


-VvVWv->
across the terminals A and 5, a current flows through the A 4 B
galvanometer. Fig. 4.52. V-

oww
Now, total resistance of voltmeter ~G + R
V V V
From Ohm’s law. I or G + R = — or R = G ...(43)
g G + R I I
g g

e
If this value of R is connected in series with galvanometer, it works as a voltmeter of the range 0 to V

re
volt. Now the same scale of the galvanometer which was recording the maximum potential G before

FFrllo
conversion will record the potential V after conversion into voltmeter. It means each division of the scale

rF
in voltmeter will show higher* potential than that of the galvanometer.

ee
Effective resistance of converted galvanometer into voltmeter is R^ = G + R
ouru
For voltmeter, a high resistance R is connected in series with the galvanometer, therefore, the resistance

rF
of voltmeter is very large as compared to that of galvanometer. The resistance of an ideal voltmeter is infinity.

fosor
Retain in Memory os kf
1.
The ammeter is connected in series and voltmeter inparallel in the circuit.
2. The range of ammeter can be increased but can not be decreased.
ook
YYo
3. The ammeter of lower range has higher resistance than the ammeter of higher range.
Bo

4. If the ammeter is connected in parallel to the circuit, the circuit resistance will reduce. The current in
reeB

the circuit will increase. Now ammeter will draw more amount of current due to its small resistance.
As a result of it may get damaged.
oouY

5. If the ammeter range is to be increased n times its original range, the value of shunt resistance should
ur

G
ad

be 5 =
n — I
Yd

6. The range of voltmeter can be increased or decreased.


nidn

7. The voltmeter of lower range has lower resistance than the voltmeter of higher range.
Re

8. If voltmeter is connected in series to the circuit, the resistance of circuit will become high and current
in circuit will reduce. Now the voltmeter will measure the emf of battery connected in circuit.
FFi

9. If the voltmeter range is to be increased to n times its original value, the series resistance to be
connectedshould be, /? = («- I) G.

.Sapipift Probiem A galvanometer with a coil of resistance 12 Q shows a full scale deflection
for a current of 2-5 mA. How will you convert it into a voltmeter of range 7-5 V ? Also, find the total
resistance of voltmeter formed.
Sol. A galvanometer can be converted into a voltmeter of given range by connecting a suitable resistance
7-5
R in series with galvanometer, which is given by, R = --G = -12 =2988fl
I
g
2-5x10-3
Total resistance of voltmeter formed = /? + G = 2988 + 12 = 3000 Q

♦Initial reading of each division of galvanometer to be used as voltmeter is / G/n and the reading of same
each division after conversion into voltmeter is /„ (G + R)ln.
o
^
MOVING CHARGES AND MAGNETISM 4/45

TYPE I. BIOT-SAVART’S LAW AND 10”^ X 5 X (0 01) X sin 45°


dB =
MAGNETIC FIELD DUE TO CURRENT
(1)2
THROUGH THE STRAIGHT CONDUCTOR
= 3-54xlO-^T
Formulae used Its direction is vertically downwards.
(0 Magnetic field induction dB at a point due to
current element is 0 ~> A
An element dl = dxi (where

1^0 I dl sinQ dx = 1 cm) is placed at the origin and carries a


dB =
4tc r2
large current i = 10 A. What is the magnetic field
on the F-axis at a distance of 0*5 m ?
where I dl is the current element and r is the distance (NCERT Solved Example)

w
of point from the current element. 9 is the angle Solution. Here, dl = dx~ I cm - 10“^ m ;
A
between dl and r .
/= 10 A ; r = 0-5 m ; r = rj

Flo
(i‘0 Magnetic field induction at a point due to linear
conductor carrying current I is,
lip Idl xT'^^o

ee
A

dB =
4tc 4n

Fr
B = (sin<j), +sin(j)^)
4710

where a is the perpendicular distance of point from P-Q /t/yg


for
4k
ur
the linear conductor. (J>i, <1)2 are the angles shown in
Fig. 4.8.
●●● dB = r k=4xlQ-^Tk
ks
(Hi) In case of infinite long conductor, B = (0-5)2
Yo
4k a
Example Q A current element 3 dl is at
oo

Units used. B is in tesla, I in ampere, o or r in metre. (0,0,0) alongy-axis. If d/ = 1 cm, find the magnetic
eB

field at a distance 20 cm on the x-axis.


^Example;a A current of 5 A is flowing Solution. Here, d/ = 1 cm = 10“2 m,
r

from south to north in a straight wire. Find the r = (20 cm) = (0-20 m) and 7^ = 0-2t
ou
ad

magnetic field due to a 1 cm piece of wire at a


/d7 = (3x10-2) J
Y

point 1 m north east from the piece of wire.


Solution. Refer to Fig. 4.53, P is the point at
nd
Re

which magnetic field is to be determined.


4tc r2
Fi

FIGURE 4.53
N 10-2 x(3x1Q-2 ^)x(0-20?)
.p
(0-20)3
W ●E
= 7-5x10-8(Jx?) =7-5xl(r*(-*)
= -(7-5x10-8
Thus magnitude of magnetic field is
7-5 X 10“® T and its direction is along negative
Here, / = 5 A, <// = 1 cm = 0-01 m ; z-axis.
r = 1 m, e =45°
●ExanipiajE] Two semi infinitely long
As, dB =
14p 7rf/sin9 straight current carrying conductors are held in
4ti r2 the form as shown in Fig. 4.54. One common end
4/46
Fundamental Physics (XII) VOL.I

of them is at the origin. If both the conductors Solution. The point (0, 0, - cl) lies on z-axis.
carry same current I, find the value of the Therefore the magnetic field induction at the given
magnetic field induction at a point (a, 6). point due to current along z-axis is zero.
The magnetic field induction due to current
Y; figure 4.S4
along jc-axis at the given point is B = 1^0. i. 7
47Cfl
a _ P(a,6)
The magnetic field induction due to current
along y-axis at the given point is

oww
--X
I ^ ^%a
Total magnetic field induction.
S<dution. Magnetic field induction at point P
due to current through the conductor along X-axis B = B+B =7^-(y-0
47Cfl

e
is

re
Ho/ It TWo parallel wires C and D
— sin0,I +sin —
placed perpendicular ot the plane of paper at a

FFrllo
4nb . 2
separation of r (= 8 cm), carry electric currents

reF
acting along negative Z-axis. = 6 A and/2 = 3 A in opptMsite directions as shown

e
in Fig. 4.55. Find the point on the line CD where
ouru = 1^0 / a
-H the resultant magnetic field is zero.

osFr
FIGURE 4.SS
Magnetic field induction at point P due to
current through the conductor along K-axis is fkfor D®
I2
kso
P
Hft / r K N r ►N ¥
ooo

B^=—— sinG.,+sin-
YYo
^ 4jCflL ^ 2J
BB

Solution. The required point on the line CD


acting along negative Z-axis. where the resultant magnetic field will be zero when
r ree

b the magnetic field at that point due to one wire must


Y

-fl
be equal and opposite to the magnetic field due to
oouu

another wire. This will be possible when the required


ad

The resultant magnetic field induction at P is point lies on the right side of wire D. Let this point
Yd

B = B, -I- B2 acting along negative Z-axis. be P at distance jc fix>m D.


nidn

^Hp./ a
+1 +
Ho / b
-fl
Magnetic field due to current /] at point P is
Re

I
FFi

4n{r + x)
.no^rri 1
4a: It is directed perpendicular to CP downwards.
Magnetic field due to current I2 at point P is
or B = Hq/ {a+b) ^ ^a^+b^
4ji: L ab ab ^ 4tc JC
It is directed perpendicular to DP upwards.
= ^^[(u+*)+Va^+*^]
4nab As resultant magnetic field at P is zero so
Ho 21 1 _ Hq 2/2
Same current I is flowing in B, -Bj or
47c(r-hjc) 4jc JC
three InHnitely long wires along x, y and z
directions. What is the magnetic field at point or =!i
(0,0,-a)? (r-l-jc) JC
MOVING CHARGES AND MAGNETISM 4/47

Fig. 4.57 shows a right-angled


or x =
12^ _3x(8xlQ-^) isosceles AP|0R having its base equal ton. A current
6-3
of 7 ampere is passing downwards along a thin
= 8x 10-2 m = 8 cm straight wire cutting the plane of paper normally
as shown at Q, Likewise a similar wire carries an
A current of 1*0 A flowing in equal curreut passing normally upwards at R.
the sides of an equilateral triangle of side Find the magnitude and directicm of the magnetic
4*5 X 10~2 m. Find the magnetic field at the field induction B at P, Assume the wires to be
centroid of the triangle. infinity long.
Solution. Here, / = 10 A, / = 4-5 x 10"2 m
Fig. 4.56, OF = 0(2 = OS
I
= -tan3(F= / ^4-5x10-2 m
2 iS ~ iS

w
Flo
reee
Solution. Here, =P/?=r. In rt angled APQP,

FFr
we have.
= ^+^=
urr
or for
r = al-yjl
kkss
Magnetic field at point P due to current through
wire at Q is
Yo
ooo

Magnetic field at the centroid O of the triangle


due to current through arm AB is given by
eB

' 4jt r 2jcr 27t(o/-^) ^na


B,1 =— — [sinO, +sin0-]
4tc a ‘ ^ It is acting along PR.
r
ou

Magnetic field at point P due to current through


ad

It is directed normally outwards the plane of


wire at R is
YY

paper

4*5x10-2
B , acting along PQ
Where, a = OP =
nndd

m
Re

2-S
and e, =02 = 60®
Fi

The magnetic field due to currents in the arms As ^ and ^ are acting perpendicular to each
AB, BC and CD is the same, in magnitude and other, so the resultant magnetic field at P is
direction, so the resultant magnetic field at O is
-- \2
t
B = 3B,i =3x ~ - [sin0, +sin0,]
4rt fl ' ^
P = ^P2+P2 = 1 -JlKa -Jlna
»V /

= 3x10-2 X
1x2-,^
[sin 60®+sin 60®]
4*5x10-2
■Jlna
xlO-5
4*5 2 Y =4xi(r5T
na
It is directed normally outwards, the plane
of paper. It acts towards the mid-point of QR,
4/48 'P.niidee^ ^ Fundamental Physics (XII)

TYPE II. MAGNETIC FIELD DUE TO V-o Idlx7 = 0


CURRENT THROUGH CIRCULAR COIL 4tc r3
ib) The magnetic field induction at O due to
Formulae used. (/) Magnetic field induction B at semicircular arc is
the centre of circular coil carrying current I is,

^^0 2nnf B =
1^0 7;t _^o / _ 4tex10~'^x12
B =
4jc r
where n = no. of turns in the coil 4k r 4 r 4x2x10"^
= 1-9x10-‘‘T
(ii) Magnetic field induction at a point on the axis
of the circular coil of radius r carrying current I is
The direction of B , according to Right hand
It
0 27in/r^
B = rule is normal to the plane of paper directed
4tu (r^ inwards. The magnetic field induction at the centre

ww
where x is the distance of point on the axis of circular of a circular loop of radius r carrying current I is,
coil from the centre of the circular coil.
(Hi) Magnetic field induction at the centre of an arc ^,_Pq 271/ _ltQ/_4Tixl0-'^xl2

FF loo
of radius r subtending an angle 0, carrying current / 4;t r 2r 2x2x10-2
= 3-8 X 1(H T

ee
IS S =
4tc r It is double than that due to semi-circular arc.

ee r
rFrF
The direction of magnetic field induction at the centre
Units used. B in tesla, / in ampere, r and x in metre. of circular arc is the same as that due to semicircular
arc if current in them is same and in same direction
rur
Example^ \ straight wire carrying a ffoor
(i.e., either clockwise or anticlockwise) and the
current of 12 A is bent into a semicircular arc of direction of magnetic field induction at the centre of
ks
radius 2*0 cm as shown in Fig. 4.58(a). Consider circular arc is different from that due to semicircular
YYoou
arc if current in them is in opposite directions,
ookos

the magnetic field B at the centre of the arc.


(c) When the wire is in the form shown in Fig.
(a) What is the magnetic field due to the
BBo

4.58(b), the magnetic field induction at O will remain


straight segments ? same in magnitude (= P9 x 10”'* T) but its direction
re

—>

(d) In what way the contribution to B from will be normal to the plane of paper directed
outwards.
oouur
ad

the semicircle differs from that of a circular loop


and in what way does it resemble ?
Y

A tightly wound 100 turn coil


(c) Would your answer be different if the wire
of radius 10 cm is carrying a current of 1 A. What
dndY

were bent into a semicircle arc of the same radius


Re

is the magnitude of the magnetic field at the centre


but in the opposite way as shown in Fig. 4.58(2^)? of the coil ? (NCERT Solved Example)
FFini

(NCERT Solved Example)


Solution. Here, n = 100, r = 10 cm = 0-10 m ;
FIGURE 4.58 /= 1 A.

Po 2nnl 10-7x27txl00xl
A E A D * E S =
4k
X
r 0-10

= 2 7C X 10-^ = 6-28 X 10-'* T


o
—* ^
■Exampiftlll Two conccntric coils X and Y
Solution, (a) For a point O, dl and r for of radii 16 cm and 10 cm respectively lie in the
each element of the straight segments AB and DE are same vertical plane containing the north- south
direction. Coil X has 20 turns and carries a current
parallel. Therefore, dl x r = 0. Hence, magnetic of 16 A ; coil Y has 25 turns and carries a current
field at O due to current through straight segments of 18 A. The sense of current in X is anti-clockwise
AB and DE is and in Y, clockwise, for an observer looking at the
MOVING CHARGES AND MAGNETISM 4/49

coil facing west, Fig. 4.59. Give the magnitude and Solution. Here, v = 2-18 x 10^ ms K
direction of the net magnetic field due to the coils r = 5-3 X 10-^^ m, e = 1-6 X 10-1^ C.
at their centre. (a) Time period of revolution of electron is given
by,
FIGURE 4.59
-11
2nr 27tx5-3xl0
7 = = 1-528 X 10-1^ s
V 2-18x10^
charge _ e
Equivalent current, / = —
time T

l-6xl0"^^
W
I
E -16
= 1*05 X ir^ A
L 1-528x10
I
Bx! I
IIq 271/ _ 10~'^ x2;rxl-05xl0~^

w
/
(b) B =
4tc r 5-3x10“*^
= 12-4 T

Flo
S Example m For a circular coil of radius
R and N turns carrying current /, the magnitude

ee
Solution. For coil X.
of the magnetic field at a point on its axis at a

Fr
r = 16 cm = 0-16 m, « = 20, /= 16 A distance x from its centre is given by
Magnetic field induction at the centre of the coil
X is given by

for
B =
ur
2(x2+J?2 )3/2
B _ Hq 2Tt»7 _ lQ-'^x27tx20xl6
4ti r 016 (a) Show that this reduces to the familiar
ks
result for field at the centre of the coil.
Yo
= 4tix 10-^T
oo

-i
(6) Consider two parallel coaxial circular
The direction of magnetic field induction coils of equal radius R, and number of turns Ny
eB

is towards east.
carrying equal currents in the same direction, and
separated by a distance R. Show that the field on
For coil .Y
the axis around the mid-point between the coils is
r
ou

/ = 18 A, « = 25. r = 10 cm = 0.10 m
ad

uniform over a distance that is small as compared


Magnetic field induction at the centre of coil Y to R and is given by
Y

is given by
B = 0-72 approximately.
nd

_ ttp Innl _ 10 ^ x2tcx25x18


Re

B _ _ _
R
>' 4k r
[Such as arrangement to produce a nearly
Fi

= 9t:x lO-^T uniform magnetic field over a small region is


known as Helmholtz coils.]
The direction of magnetic field induction By is
towards west IR^N
Net magnetic field = B^- B^ Solution, (a) Given, B =
= 9tcx 10^-4tcx 10-^ 2(;c2+i?2)3/2
= 5 7C X10”^ = 1'6 X 10“^ T (Towards West) When point is at the centre of circular coil.
x = 0.
Example m The electron in a hydrogen IN
atom circles around the proton with a speed of Then, B =
2-18xl0® ms"^ in an orbit of radius 5*3 x 10“'^ m. 2R^ 2R

Calculate (a) the equivalent current (b) magnetic Which is the same as the standard result.
field produced at the proton. Given charge on
(b) Here, a = R,n = N.LQtd be the distance of
electron is 1*6 x 10"*^ C and |1q = 4 7t x 10"’ TmA“^. point P from the mid-point O lying on the axis of the
4/50 'Pnadccp- 4. Fundamental Physics (XII) P75TWI
two circular coils C and D, where d << R. In Fig. ^-3/2
1 4d
4.60, X I-
2 x3/2
5R
5R-
\ 4 J
It acts along PO2.
Total magnetic field at P due to current through
the coils is given hy B = B^ + 82
-3/2 n-3/2
4d^ 4d
X 1 + + 1-
x3/2 ●
5R 5R

V 4 /

[Iq NIR^ ( ■ 1- 6d + 1 + 6d

w
R 2R^ 5J 5R 5R
For coil C, O.P = ,x. = — + d
I 1 ^
2 (From Binomial Theorem)

Flo
For coil D,
02P = ^=\-‘> = 0*72

reeee
V
5 y
R R
We know that magnetic field at a point on the
Example [|[] The Wire shown in Fig. 4.61.

FFr
axis of the circular coil of radius a, having n turns
and carrying current/, which is at a distance x from carries a current of 10 A. Determine the magnitude
the centre of the coil is given by of the magnetic field at the centre O. Given radius

for
ur
of the bent coil is 3 cm.

B =
M-0 2nnla^ Ho n I (Pb. Board 2001; AUMS 2013)
kkss
47C(x2+fl2)3/2 2(X^ +^2)3/2
Yo
Due to coil C, magnetic field at P is
oo

^0 NIR^
eB

B,1 =
2(x,2 +/?2)3/2
-i3/2

+ Rr
r

2
ou

L'^
ad

_\^qNIR^
YY

1
2 \3/2
/?2
+ d'^ + Rd + R^
ndd
Re

\ 4
As d^ can be neglected in comparison to R^,
Fi

therefore, Solution. Let n be the unit vector perpendicular


to the plane of wire loop directed outwards.
x-3/2
1 4d Here, I = 10A;r=3cm = 3x I0“^2 Angle
^1 = 2 n3/2
1 +
5R subtended by circular arc at O, 0 = 270° = (37t/2)
5/?2
rad. Magnetic field at O is
I 4
= 0+
HI A

It acts along P02- B^=B


0 ab
+ B
bed
+ B
de
-0(-n) + O
4k r
Similarly, due to coil D, magnetic field at P is
given by B
ab
and B.
de are zero because point O lies on
the wires ab and de.
[IqNIR^
«2 = -^^xLq =10-7 X 10 3
2(a:| / p
n3/2 B0 “
47t r 3x10-2
tX“
2
K

2 --d +R?-
\2 J = 1-57 X 10-4 T
MOVING CHARGES AND MAGNETISM 4/51

It is acting perpendicular to the plane of circular Solution. Magnetic field at the centre O due to
arc directed downwards. current through the whole current loop is
B = directed normally
Exsmple m Two coaxial circular loops L 1 downwards
and L2 of radii 3 cm and 4 cm are placed as shown
in Fig. 4.62. What should be the magnitude and = 0+ it —/ 9- + 0 + -(27C-9)
47t 4tc ,r
direction of the current in the loop L2 so that the 1

net magnetic field at the point O be zero ? 27t-9


(CBSE 2008)
471 Tj ''1
FIGURE 4.62
directed normally downwards
4cm
If 9 = 90° = (tu/2) rad, then
O i
—4cm—►K3em-
\^\t^ (27t-7C/2)1 _Ro/r 1 ^ 3'

ww
B =
II = 1A 47t 8
'i - r, _
Li
L2
(47cxl0-tx2fl00 300'

Floo
Solution. The resultant magnetic field at the
8 6 4 .
point O will be zero only if the direction of current

ee
= 2-88 X 10-® T
in loop L2 is opposite to that in loop L^. Then

reer
magnetic field at O due to current /j in loop L|

rFF
= magnetic field at O due to current I2 in loop L2 TYPE 111. AMPERE’S CIRCUITAL LAW
uur r
^^0 27t/j ^0 271/2
or

471 (fl2+j:2)3/2 471 +;c2)3/2 Formulae used.


ffoor
sks
where o represents the line integral over the
YYoo
or closed path, B is the magentic field induction at a
ooko

point on the closed path and / is the current threading


eBB

the closed path.


or
lx (003)2 /2X (0-04)2 Units used. B in tesla, I in ampere and r in metre.
[(0-03)2+(0-04)2 ]2/2 [(0-04)2+(0-03)2 ]2/2
r

Example m Fig. 4.64 shows a long


ouur
ad

lx (0-03)2 9
— = 0-56 A straight wire of circular crosssection (radius a)
h ~
Y

or
(0-04)2 16 carrying steady current /. The current I is
uniformly distributed across this crosssection.
dY

Example m The current loop abode


Re

Calculate the magnetic field in the region r < a


nnd

formed by two circular segments of radii and r > a. (NCERT Solved Example)
FFii

rj (= 4 cm) and T2 (= 6 cm) with common centre at


O, carries a current I (= 2 A) as shown in Fig. 4.63. FIGURE 4.64

What is the magnetic field at the common centre P

O ? What will be the value of magnetic field at O B*, r \2

when 9 = 90° ? i
-*►
R ●1 B ‘

Solution, (fl) Taking the case, when r < a. Let


the point be inside the wire at a distance r from
the axis of wire. Consider an Amperian loop labelled
1, which is a circle of radius r concentric with
crosssection such that point F] lies on this loop.
4/52
"Pnadec^ ^ Fundamental Physics (XII) P7S!Ml
Current enclosed by the loop, FIGURE 4.65
P
i 2 /r2
/' = fl/2
2
%a a
a/2
Q
Let B be the magnetic field induction at Pj. Line
integral of B over the Amperian loop is Vi
= =B (2nr) Magnetic field at point P at distance r (= 3 all)

w
from the axis of the wire is
According to Ampere’s circuital law,
* ^ ^
B -^^0 2/ _Mq/
y = )Xq X current threading the closed path ^ 471 r 4tc (3o/2) 37ta
The distance of point Q from the axis of wire

e
roow
r'= a-ail = all.

re
2
a

Ir^ Magnetic field at point Q at distance r (= all)


S27cr=P(, X — or B =
27Ca^
from the axis of the wire is

reF
uFFll
It means, B oc r (When r <a) B — x- = j!1o Z
a

e
Q
2jr In a^ 1 4;r a
(b) Taking the case when r>a. Take the point P

sFr
outside the wire at a distance r from the axis of wire.
Consider an Amperian loop labelled 2 which is a

oro
B
3 7t n }Iq / 3

k
circle concentric with crosssection of wire such that Q
uor
point P lies on this loop. Current enclosed by this
loop is 1.
off Magnetic field is maximum on the surface of
wire, where r = a. Hence,
kos
Let B be the magnetic field induction at P. Line
Y
_lio II _ Mq/
Yo
B
integral of B over the Amperian loop
reeBB
oo

max
47t r 2na

= j)B.dT=B (Inr) Example EE A wire of radius 0*8 cm


urY

According to Ampere’s circuital law carries a current of 100 A which is uniformly


distributed over its cross-section. Find the
oB.dl = Pq X current threading the closed magnetic field (a) at 0*2 cm from the axis o'" the
ou
ad
do

loop = 1 wire (&) at the surface of the wire and (c) at a point
nY

outside the wire 0*4 cm from the surface of the


^0^ wire. Neglect the permeability of the material of
5 2 7t r = Pq / OT B =
nid

Inr wire.
Re

1 Solution. Here, /? = 0-8 cm = 8 x 10"^ m ;


FFi

It means, B /= lOOA
(When r> a)
oc —

Example [0 ^ Straight thick long wire of (a) B.inside -h-llL


An
uniform cross-section of radius a is carrying a
steady current /. Calculate the ratio of magnetic
field at a point all above the surface of the wire to = 10-'^ X 2xl00x(0-2xl0-2)
that at a point all below its surface. What is the (8xI0~3)2
maximum value of the field of this wire ? = 6-25 X 10-^ T
(CBSE 2010)
2x100
Solution. Suppose the point P lies at distance {b) B surface = 10“^ X
all above the surface of the wire and point Q lies at 471 R 8x10-3
distance all below tlie surface of wire. Fig. 4.65. = 2-5 X 10-3 T
The distance of point P from the axis of wire (c) When point is outside the wire,
= a + o/2 = 3fl/2.
r = 0-8 + 0-4= 1-2 cm = l-2x lO'^ m.
MOVING CHARGES AND MAGNETISM 4/53

2x100
(a) B-\iQnl-4n.y x 200 x 2-5
B
outside
= iio = 6-28x10^1
4tc r 1-2x10“2
= 1-67 X T |Xq nl _ 47txI0~^ x200x2-5
ih) B = --^ 2

TYPE IV. MAGNETIC FIELD DUE = 3*14 X 10-^ T


TO SOLENOID AND TOROID
Example^ A copper wire having a
Formulae used, (i) Magnetic field induction B at resistance of 0*015 Q per metre is used to wind a
500 turns solenoid of radius 1*0 cm and length

w
a point well inside the long solenoid carrying current
22 cm. If the e.m.f. of the battery used is 6 V, then
N
find the magnetic field near the centre of the
/is.
S = Hon/=Ho7' solenoid.
where n = no. of turns per unit length of solenoid, Solution. Resistance of wire per metre

roow
e
N = no. of turns in the solenoid of length /. = 0-015 Qm
-1

(ii) Magnetic field induction S at a point at one end

re
No. of turns in solenoid, N = 500 ;
on the axis of a long solenoid carrying current I is
radius, r = 1 x 10"^m,
Po nl Po NI

reF
length of solenoid L-22 cm = 22 x 10“^ m,

uFFll
B =
2 21

e
e.m.f., e = 6 V
{Hi) Magnetic field induction B at a point inside the Length of wire used in solenoid

sFr
toroid carrying current I having n turns per unit
length is B = \iQtxI
yx(lxl0-2)x500m
foro
I - 2nrxN = 2x
(iv) Magnetic field induction 5 at a point outside
uor
the toroid is zero.
fk = 31-4 m
okso
Units used. B in tesla and I in ampere. Total resistance of wire used in solenoid,
Y
Yo
R = resistance per metre x /
Example ^ A solenoid of length 0*5 m has
oo
BB

= 0-015x314 a
a radius of 1 cm and is made up of 500 turns. It
Current in solenoid,
carries a current of 5 A. What is the magnitude of
rYree

the magnetic field inside the solenoid ? 6


/=^
oouu

= 12-73 A
(NCERT Solved Example) R 0-015x31-4
ad

Solution. Here, N = 500 ; / = 0-5 m ; / = 5 A ;


Magnetic field near the centre of solenoid
Yd

r = 1 cm = 10“^ m.
NI 500x12-73
nidn

I _ 0-5 = (4tcx10 ^)x


As = 50 so / > > r. (22x10-2)
Re

r ~ 10"2
= 3*64 X 10-2 X
Hence, we can use the formula of magnetic field
FFi

induction at a point inside the long solenoid Example ^ A toroid has a core of inner
NI 500 radius 20 cm and outer radius 22 cm around which
^ = 1^0 = 47txl0 ^x x5
4200 turns of a wire are wound. If the current in
I 0-5
the wire is 10 A, what is the magnetic field
= 6*28 X 10-3 T
(a) inside the core of toroid (b) outside the toroid
Example m A solenoid of length 50 cm, (c) in the empty space surrounded by toroid ?
having 100 turns carries a current of 2*5 A. Find Solution. Here, inner radius, rj = 20 cm.
the magnetic field, (a) in the interior of the Outer radius, r2 = 22 cm ; / = 10 A
solenoid, (b) at one end of the solenoid. .*. Mean radius of toroid.
Given, m, = 4 tc x 10“^ Wb A“* m“*.
Solution. Here, I = 2-5 A, r =
ri + r2 _ 20 + 22 = 21 cm = 0-21 m
n = 100/0-50 = 200 m-1 2 2
4/54
Fundamental Physics (XII) VOL.I

Total length of toroid = circumference of toroid 1


= 27ir = 27ix 0-21 Solution. Here, >» = - cm = 0-5 x 10“^ m ;

= 0-42 7C m
/ = 3 cm = 3 X 10"^ m ; i; = 3 X 10^ ms
-1 .

Total number of turns, N = 4200


d = 10 cm = 10-2 m ; V= 550 V
Number of turns per unit length will be,
Time to cross the tube.
4200 10000 -I
n —
042 7T
m
I 3xI0"2
n
= 10“^s
V 3x1Q2
(a) Magnetic field induction inside the core of
10000 1 1 Ee')
toroid, 5 X Pq n/ = 4tcx 10"2 1 (V/d)e
X xl0 = 0-04T y = ~at^ = =
/2
7t 2 m 2 m
(b) Magnetic field induction outside the toroid

ww
is zero, since the field is only confined inside the or
e _ 2yd _ 2x(0-5xl0 2)xl0~2
core of the toroid on which winding has been made, m Vf2 550x(10-^)2
(c) Magnetic field induction in the empty space

Floo
surrounded by toroid is also zero. = 1-8 X 10*1 Ckg-1
Example ^ A charged particle of charge

ee
TYPE V. MOTION OF CHARGED
2*0 pC moving along Ar-axis with a speed of 3 x 10^

reer
PARTICLE IN (;} ELECTRIC FIELD

rFF
, A A
AND (h) MAGNETIC FIELD ms
* enters a magnetic field, B = (0’3 j + 0‘4k)
Formulae used tesla acting in a space. What is the magnitude of
uur r
(0 Force on a charged particle of charge q in an
electric field of intensity E is
ffoor
magnetic force on the charged particle ?
Solution. Here, ^ = 2 0 pC = 2 x 10"^ C ;
sks
F = qE
= (3x10^ O -1
YYoo
V ms
(ii) Force on a charged particle of charge q moving
ooko

with velocity in a magnetic field B is fi = (0-3 y + 04^)r


eBB

F = q{vxB) or
F = qv B sin Q
Magnetic force on the charged particle is

where 0 is the smaller angle between if and B . F = q (vxB)


r
ouur
ad

(Hi) When magnetic field is acting perpendicular to = (2x10-6) [{3xl06)/x(0-3y* + 0 4y)]


the direction of motion of charged particle then
Y

F = qvB sin 90° =


mv
2
mv = (2x10-6)x106 [37xmj + 0-4k)]
dY

or r =
r
qB
Re

= 2 [(3x0-3) (/xy) + 3x04 (/xit)]


nnd

2nr 2nm
(iV) Time period of revolution, T =
FFii

V qB = 2[0-9^-l-27]=[l-8it-24i']
Units used. F in newton ; q in coulomb ; £ in V/m Magnitude of magnetic field is
or N/C, B in tesla, v in m/s and r in metre.
I FI = 7c1^)2+(-24)2
Example ^ A cathodc ray tube contains a = V3-24 + 5-76 = 3-0 N
pair of parallel metal plates 1*0 cm apart and
3*0 cm long. A narrow horizontal beam of elec
Example 03 A copper has 8‘0 x 102*
trons with velocity of 3 x 10^ ms~* is passed down electrons per cubic metre. A copper wire of length
the tube mid way between the two plates. When
the potential difference of 550 V is maintained 1 m and cross-sectional area 8*0 x KT** m2 carrying
across the plates, it is found that the electron beam a current and lying at right angle to a magnetic
is so deflected that it just strikes the end of one of field of strength 5 x 10~2 T experiences a force of
the plates. Find the specific charge of an electron. 8*0 X 10“2 N. Calculate the drift velocity of free
electrons in wire.
MOVING CHARGES AND MAGNETISM 4/55

Solution. Here, n = 8 0 x 10^^ m ^ = 1 m; when accelerated under a potential difference V, then


A = 8 X 10-6 m2 e = j.g ^ 10"*^ C, 1
eV = -mi>2
B = 5 X 10-2 T, F= g.Q X 10-2 N, 0 = 90” 2

Total charge contained in the wire is nI/2


(leV
q - Volume of wire x ne = Alne or V =
m
= (8 X 10-6) X 1 X (8 X 1028) X (1-6 X IQ-*^) .\n
= 1024X lO^C 2xl-6xl0"^^ x2-0x1Q2
If is the drift velocity of electrons, then 90x10
-31

F= g B sin 0
8x10^

ooww
F m/s
or V 3
^ qB sin 0 («) Force on electron due to transverse magnetic
(80x10-2) field is = Bev, which is perpendicular to B as well
(l-024xl05)x(5xl0-2)xsin 90° as t?. It will provide the required centripetal force

e
to the electron for its circular motion. Therefore, the

re
= 1*56 X 10-“* ms-^
trajectory of electron in magnetic field is circular.

rFFl
ree
^^ample What is the radius of the path

F
Its radius r can be given by
of an electron (mass 9 x lO"^^ kg and charge

rF
●)

1‘6 x 10“^® C) moving at a speed of 3 x 10^ ms"' in Bev =


mv~

a magnetic field of 6 x 10“^ T perpendicular to r

fsfoor
ouur
it ? What is its frequency ? Calculate its energy in mv 9x10-21 X (8x102/3)
keV. (NCERT Solved Example) or r =
kosk
Be 0-15 X1-6 X10-*^
Solution. Here, m = 9 x 10"2* kg ;
= 10 2 m=l mm
Yo
g = 1-6 X 10“^^ C ; u = 3 X 1Q2 ms
-1 .
oo
Y

B = 6x 10-^T (b) When electron makes an angle 0 with the


BB

mv (9xl0-2^)x(3xl02) direction of magnetic field, its component velocity


rre

= 0*28m perpendicular to the field.


qB (l-6xlb-'9) (6x10-^)
oYuu

Vi1 = v sin 30° = -xl02 X — = —x1Q2 m/s.


(6xl0-^)x(l-6xl0-‘^)
ad

V Bq 3 2 3
V =
27cr iKm -31
2x(22/7)x9xl0
dY

Due to this velocity, the force acting on the


= 1-7 X 102 Hz electron due to magnetic field will be providing the
innd

required centripetal force, hence the electron will


Re

1
F.
*
=-
2
mv 2 =ix(9xlO-2l)x(3xlo2)2j describe a circular path of radius The component
FFi

velocity along the magnetic field = v cos 30°


40-5x10-‘2
= 40-5x 10-^2 j = keV
8 S 4^I3
1-6x10-‘6 x1Q2 m/s.
3 2 3
= 2-53 keV
Due to this velocity, no force acts on the electron
Example RHl An electron emitted by a
in magnetic field, and electron moves without change
heated cathode and accelerated through a
in its speed along the magnetic field. Therefore, due
potential difference of 2*0 kV enters a region with to two perpendicular component velocities, the
a uniform magnetic field of 0*15 T. Determine the electron describes a helical path.
trajectory of the electron if the field (a) is The radius of the helical path,
transverse to its initial velocity (b) makes an angle
of 30° with the initial velocity. mv
1 9x10-2^x(4/3)x1Q2
Solution. Here, 2-0 kV = 2-0 x 10^ V ;
r =
Be 0-15X 1-6x10-’^
B = 0-15 T. Let e, m be the charge and mass of - 0-5 x IQ-2 m - 0*5 mm
the electron. If v is the velocity gained by electron
4/56 ‘Pnadce^ ^ Fundamental Physics (XII) PTSTMl
Example^ A proton, a deutron and moving perpendicular to the direction of magnetic
a-particle, whose kinetic energies are same, enter field due to current through straight wire. The force
perpendicularly a uniform magnetic field. on moving proton of charge q due to magnetic field
Compare the radii of their circular paths. is F = g u 5 sin 90°
Solution. Here, mass of proton, m=m \ mass = (1-6 X 10-^^) X (4 X 10^) X (4 X 10-6)
of deutron, = 2 m and mass or a-particle = 2-56 X 10-1* N
m
a = 4 m ; charge on proton, qp - e \ charge on According to Fleming’s Left Hand Rule, the
deutron, = e and charge on a-particle, q^-le. force acts in the plane of paper towards right
K.E. of charged particle, Example ED An electron moving

oww
E = ~ mv^ horizontally with a velocity of 4 x 10'* ms'i enters
2
or
V = ^2 E/m a region of uniform magnetic field of 10"* T, acting
Radius of the circular path of a charged particle vertically downwards as shown in Fig. 4.67. Draw
in the magnetic field is given by its tr^ectory and find out the time it takes to come
out of the region of magnetic field.

e
2E V2mE ■Jm

re
mv m
r = ; so
(CBSE (F) 2015)

FFrlo
Bq Bq m
Bq FIGURE 4.67

rF
ee
m
a X X X
r_ : r
p ● 'a “ _ ●~ ●“
ouru
rF
a e
X X X

Vw -^2 m ^ m

= 1:V2:1
ffosor X X X
e e le
os k
Example ^ A long Straight wire AB X X X
ook
carries a current of 4 A. A proton P travels at
Yo
Y
4 X lO® ms”^ parallel to the wire, 0*2 m from it
Bo

and in a direction opposite to the current as shown Solution. Here, u = 4 x 10^ ms“* ; B = 10“^ T
reeB

in Fig. 4.66. Calculate the force which the magnetic When a moving electron is subjected to a
field of current exerts on the proton. Also specify perpendicular magnetic field, it will discribe a
oouY

the direction of the force. semicircular trajectory inside the magnetic field and
ur

(CBSE 2002)
ad

FIGURE 4.66 comes out as shown in Fig. 4.68. Radius r, of the


circular path of electron is given by
Y
d

B
nidn

FIGURE 4.68
Re

0-2m
@P X X X
FFi

e
4*10®m/s X X

X / X
Solution. Here, / = 4 A ; u = 4 x 10^ ms -1. X


r= 0-2 m. e
X X X
Magnetic field induction at P is

B = ^ ^7 _ I0“^x2x4 == 4 X 10-6 T
47t r 0-2 2 mv
mv
= qvB or r =
The direction of B , according to Right Hand r qB
Thumb rule, is perpendicular to the plane of paper -31
directed inwards. r =
(9-1x10 )x(4xl0^)
Since proton is moving in opposite direction to (1-6x10"’^)xI0"6
the current carrying straight wire, hence the proton is = 22-75 X 10-3 m
MOVING CHARGES AND MAGNETISM 4/57

Time taken to come out of the region of (it) The frequency of revolution of electron
magnetic field is eB (1-6x10-'9)x(8-0x10-4)
V —
nr _ (22/7)x(22-75xl0~^) 27Cm 2x3-14x(9-lxl0-^')
V 4x10'* = 22x 10^ Hz =22 MHz
= 1-8 X 10-^ s {Hi) If the electron loses its energy in
successive collisions, then electron loses its speed
TYPE VI. (/) MOTION OF CHARGED progressively. If after collision, the velocity of
PARTICLE, BOTH IN ELECTRIC AND electron remains in the same plane of the initial

w
MAGNETIC FIELDS (//) CYCLOTRON circular orbit, the radius of the circular orbit will
**♦ ^ decrease in proportion to the decreased speed. If it is
Formulae used.(i) Force, F = q E+ q {vy. B) not so then the path of the electron will be helical
I between two collisions.
qB

e
roow
(if) Cyclotron frequency. V = —
T 2nm Example m A beam of electrons enters a

re
{in) Maximum K.E. of accelerated charged particle uniform magnetic field of 0*3 T with a velocity of
4 X 10® ms“* at an angle of 60" to the field. Find

reF
K
_

uFFll
IS, max.
2m the radius of the helical path taken by the beam.

e
Unit used. ^ in C, £ in N/C, B in tesla, Tq in m and Also find the pitch of the helix (distance travelled
mass m in kg. by a proton parallel to the magnetic field during

sFr
one period of rotation). Mass of proton is
1-67 X 10-2^ kg.

foro
Example ES In a chamber of a uniform
fk
Solution. Here, B = 0*3 T, u = 4 x 10® ms“*,
uor
magnetic field of 8*0 G [1 G = 10“^ T] is main 0 = 60".
okso
tained. An electron with a speed of 4*0 x 10^ ms“^ C,m= 1-67 x 10-2^ kg
,-19
enters the chamber in a direction normal to the ^ = l-6x 10
Y
Yo
field Components of the velocity of proton parallel
oo
reeBB

and perpendicular to the direction of magnetic field


(0 Describe the path of electron.
(«) What is the frequency of revolution of
are
U|| = V cos 0 = 4 X 10® X cos 60°
uurY

= (4 X 10®) x (1/2) = 2 X 10® ms


-1

electron ?
= u sin 0 = (4 X 10®) x sin 60°
(hi) What happens to the path of the
electron if it progressively loses its energy due to = 4 X 10® X (V3/2) = 3464 x 10® m/s.
ad
doo

collisions with the atoms or molecules of the


The proton moving with component velocity U|j
nY

environment ? (NCERT)
will move along the magnetic field and with
Solution. (0 Here, B = 8-0 G = 8-0 x 10^ T, component velocity Vj_ will describe a circular path
nid
Re

V = 4-0x 10® ms
-1
. e = 90°. in the magnetic field.
Hence, the path of proton in the magnetic field
FFi

If r is the radius of circular path, then


2 is helical path.
mv mv
B e u sin 90° = or r = The radius of the helical path is
r Be
mv.^1
_ (9-lxl0-®^)x(40xl0®) qv^B = r

" (8-0xl0^)x(l-6xl0“^^) mv
± _ (l-67xl0-2’)x(3464xl0®)
or r =
- 2*8 X 10“^ m = 2*8 cm qB (l-6xl0"'^)x0-3
The sense of rotation of electron in a circular - 1-2 X 10'^ m = 1‘2 cm
path can be predicted from the direction of Period of revolution of electron is
—> ^
centripetal force, F = - e (i> x B). If we see along Inr _ 2x3-14x(l-2xl0~^)
T =
the direction of B, the electron will be revolving in V
1 (3464x10®)
clockwise direction. = 2-175 X lO"’ s
4/58
Fundamental Physics (XII)
Pitch of the helix is
27Tvm
p = v^y X r=(2x 10^) X (2-175 X 10"^) Solution. ^ -
= 4*35 X 10“^ cm = 4-35 cm
_ ^ ^ ^ 22 (10 X10^) X (1-67 X10-2’) «0-66T
Example \ beam of alpha particle 7 1-6x10-^^
passes undeflected with a horizontal velocity u,
through a region of electric and magnetic fields, K
mutually perpendicular to each other and normal max.
2m
to the direction of the beam. If the magnitude of
the electric and magnetic fields are 120 kV m“* (0-66)2 X (P6 X 10-^9 )2 X (0-60)2
and 60 mT respectively, calculate (i) velocity of 2x1-67x10-2’
the beam (k) force with which it strikes a target
on a screen, if the alpha particle beam current is _ (0-66)2 X (1-6 x1Q-^^)2 X (0-60)2 MeV

w
equal to 1 mA ? Mass of proton = mass of neutron 2 X (1-67 X10-2’)x 1-6x10-12
= 1-675 X 10-2’ kg. = 7*5 MeV
Solution. Here, E= 120 kV m“^
Example^ If the maximum value of

Flo
= 120 X 10^ Vm-^ ;
accelerating potential provided by a radio

eeee
fi = 60 mT = 60 X T
frequency oscillator be 25 kV, find the number of
(/) For undeflected beam of alpha particle revolutions made by a proton in a cyclotron to

Fr
qv B=qE achieve one sixth of the speed of light. Mass of
E 120x102 proton = 1-67 x 10“2’ kg.

for
= 2 X 10^ ms”^
ur
or V = —
B 60x10-2 Solution. In a cyclotron, when a proton crosses
a region of potential difference V, the energy gained
(ii) Current carried by alpha particle beam
is eW. In cyclotron, in one revolution, the proton
ks
1 - 1 mA = 10-2 A
crosses the gap twice. So the energy gained by proton
Yo
Number of alpha particles striking the screen
oo

in each revolution = 2 eV.


per second Let the proton make n revolutions before emer
B

10-2 ging from the dees. The gain in its kinetic energy is
I J_
re

n= —

q 2e “ 2x1-6x10-'^ 1 T
E. =- mv^ = nx2eV
^ 2
ou
ad

= 3-125 X lO'^s-i
2
(c/6)2
Y

Mass of alpha particle, mv m


or n =
m
a
= 2mp + 2m„ = 4m^ 4eV 4eW
nd

= 4x 1-675 X 10-2’kg
Re

2
Force with which alpha particle beam strikes a me
Fi

or n =
target on the screen 4x36xeV
F= rtu = (4 X 1-675 x iO-2’)
1-67x10-2’X (3x10^)2
x(3-125x 1012)x2x 10^
= 419 X 10-5 isj 4x36x(1-6x10-i^)x(25x102)
= 261 revolutions

*s?a"ipie ^ A cyclotron oscillator freq


uency is 10 MHz. What should be the operating TYPE VII. force on CURRENT
magnetic field for accelerating protons ? If the CARRYING CONDUCTOR IN MAGNETIC FIELD
radius of its dees is 60 cm, what is the kinetic
Formulae used.
energy of the proton beam produced by the
accelerator in MeV ? —»

(/) F = / /5 sin 0 and {ii) F = I {lx B)


(e = 1*6 X 10-1® C, mp = 1-67 x ir2’ kg ; Units used. F in newton, I in ampere, / in metre and
1 MeV = 1-6 X 10-12 J) (CBSE 2005)
B in tesla.
(NCERT Solved Example)
MOVING CHARGES AND MAGNETISM 4/59

Example ES The horizontal component with its direction parallel to the axis along east to
of the earth’s magnetic field at a certain place is west. A wire carrying current of 7*0 A in the north
3*0 X 10“^ T and the direction of the field is from to south direction passes through this region. What

w
geographic south to the geographic north. A very is the magnitude and direction of the force on the
long straight conductor is carrying a steady wire if (a) the wire intersects the axes, (b) the wire is
current of 1 A. What is the force per unit length turned from N-S to north east-south west direction,
on it when it is placed on a horizontal table and (c) the wire in the N-S direction is lowered from the
the direction of current is (a) east to west axis by a distance 6*0 cm?
{b) south to north ? (NCERT) Solution. Here, 5 = 1-5 T, / = 7 0 A

wr
e
Solution. Here, S = 3-0 x 10"^ T, / = 1 A. {a) Length of the wire in a cylindrical magnetic
field, is equal to the diameter of the cylinderical region

r
Force on current carrying conductor in

oo
magnetic field is F = BIl sin 9 i.e. / = 20 cm = 0-20 m. Angle between current and

F
llu
magnetic field 0 = 90°
p
Force per unit length, f ~ ~ — BI sin 0 Force on the wire,
F = BIl sin 6 = 1-5 X 7 0 x 0-20 x sin 90° = 2*1 N

roFF
{a) When current flows from east to west, then The direction of this force is vertically

ees
0 = 90°
downwards, using Flemings’ left hand rule.'

rr
/=S/sin90° = (30x 10-5)x 1

Fk
(/>) When the wire is turned from N-S to northeast-
= 3 x ir^ Nm"*
northwest direction, the length of the wire in the

oo
{b) When current flows from south to north. cylindrical region of magnetic field is and 0 = 45°
then 0 = 0°
Force on the wire, Fj = B//j sin 45° = BIl
/=B/sin0° = 0
fofr
so [v /i sin 45° =/I
YY
ou
Example^ A Straight wire of mass
oY
or
Fj = 1-5 X 7-0 X 0-20 =2*1N
BrB

200 g and length 1*5 m carries a current of 2 A. It is The direction of this force is vertically
ok

suspended in mid air by a uniform horizontal downwards, using Fleming’s Left hand Rule,
(c) When the wire is lowered by 6-0 cm i.e. at
magnetic field B . What is the magnitude of the position CD in Fig. 4.69 (front view is taken), then
ruee

magnetic field ? (NCERT Solved Example) length of wire in magnetic field, /2 = 2 x


ooud

Solution. Here, m = 200 g = 0-200 kg, FIGURE 4.69


/ = 2A;/=i-5 m.
ad

axis of cyiindncal region


Yn

For arid air suspension, mg = BI I


0-200 X 9-8
di

mg
or B = ' = 0*65 T
II 2x1-5
Re
FF

\
in

pi®8S A long straight conductor C


i - H
Exam

6cm I
carrying a current of 3 A is placed parallel to a
short conductor D of length 5 cm, carrying a
I ^ .LD
current 4 A. The two conductors are 10 cm apart.
Find (i) the magnetic field due to C at D, («) The
approximate force on D. where, x x x = 4 x (10 + 6) or x = 8cm.
Solution. 0) Magnetic field due to C at D is /2 = 2 X 8 cm = 16 cm = 01 6 m.
Force on wire.
B = lio 2/ = 10"^ X 2x3 = 6 X 10-^ T F2 = BIl2 = 1-5x7x0-16
471 r 0-10
= 1*68 N vertically downwards.
(") Force on D, F = BI^l sin 0
= (6 X 10"^) X 4 X (5 X 10“2) X sin 90° Example ED A straight horizontal
= 1*2 X 10"^ N conducting rod of length 0*60 m and mass 60 g is
suspended by two vertical wires at its ends. A
Example EE A uniform magnetic field of current of 5*0 A is set up in the rod through the
1*5 T is in a cylindrical region of radius 10*0 cm
wire, (a) What magnetic field should be set up
4/60 "Pn^deefr U Fundamental Physics (XII) k’MWl
normal to the conductor in order that the tension Exai^pie^ES Fig. 4.70 shows a rectangular
in the wire is zero ? current-carrying loop placed 2 cm away from a
(b) What will be total tension in the wires if long, straight, current carrying conductor. What
the direction of current is reversed, keeping the is the direction and magnitude of the net force

r
acting on the loop ? (CBSE 2005)
magnetic field same as before (Ignore mass of the

n
Fll
wire), g = 10 ms"^. FIGURE 4.70
Solution. Here, / = 0-60 m ;
m ^ 60 g = 60 X 10-3 ; / = 5.0 A A 15A B
(o) Tension in the wire is zero if the force on A I '—! ,. 25A

f
the wire carrying current due to magnetic field is equal
and opposite to the weight of wire, i.e.,
o
3 < ►
2cm

u
BI l = mg

ow
C
(60x10~3)x10
—►
mg 10cm
or B = = 0-20 T
II 50XO-60

ori
The force on the conductor due to magnetic field Solution. Here, /j = 15 A ; /2= 25 A ;
will be upwards if the direction of magnetic field is = 2 X 10-2 m ; T2 = (2 + 10) x 10-2

ee
horizntal and normal to the conductor.
M-o
Force on BC, .F1 = X length BC

Fr
ib) When direction of current is reversed, B 11
4ti
and mg will act vertically downwards, the effective
ee
tension in the wires. 2x15x25
=10“2 X — X (25x10 *■)
ur
FF

T = B11 + mg (2x10-2)
sf
oo

= 0-2 X 5-0 X 0-60 + (60 x lO'^) x 10 = 9-375 X 10“^ (repulsive, away from XY)
= 1-2N
ok
^^0
Yo
Force on DA, F2 - 4n
X length DA
TYPE vm. FORCE BETWEEN 'a
Bo

TWO PARALLEL STRAIGHT


2x15x25
CONDUCTORS CARRYING CURRENTS = 10-2 X X 25 X 10-2
(2 +10) X10-2
Formulae used
ur

= 1-5625 X 10^ N (attractive towards XY)


(/) Force per unit length on each of the two long
ad

parallel conductors carrying currents /j and I2 placed Net force on the loop F = F1-F2
Y

distance r apart is = (9-375 - 1-5625) X 10^


= 7-8175 X 10^ N
21.1
F=^
d

f 2
Re

(respulsive, away from XY)


in

4tc r
Example ^ In Fig.4.71, the three parallel
(ii) Force on smaller conductor of length / carrying wires AB, CD and EF are long and have identical
current /], held parallel to infinite long linear con resistance. The separation between two
ductor carrying current I2, placed distance r apart is
neighbouring wires is 2-0 cm The wires AE and
BF have negligible resistance and ammeter reads
471 r 12 A. Calculate the magnetic force per unit length
{Hi) Force on long linear conductor carrying current of AB and CD.
/j, placed parallel to a small conductor of length /, FIGURE 4.71
carrying current I2 is A B

12A
47t r (A)—- *■
item ^
The direction of F' is opposite to F given in point E Ip
{ii).
Units used. /,, I2 in ampere, / and r in metre, and F Solution. Due to symmetry, the current through
in newton. each of wires AB, CD and EF is 4 A.
MOVING CHARGES AND MAGNETISM 4/61

Force per unit length on AB due to current in


CD is. TYPE IX. TORQUE ON A CURRENT LOOP

rw
2x4x4 Formulae used.
= lO-"^ X
* 4jc r (2x10-2)
= 16 X 10“^ Nm“^ directed downward
X -I A/x 5 I = MB sin a = (nIA) B sin a.
where M = magnetic dipole moment of the current
Force per unit length on AB due to current in
EF is.
loop (in Atyp-):
n - number of turns in the coil,
2x4x4 I - current in the coil,
= 10"’x

r
^ 4?i (r + r) (4x10"2) a = angle which a normal drawn on the plane of

wo
llou
= 8 X 10~^ Nm“^ directed downward coil makes with the direction of magnetic field.

F
Units used. I in ampere, A in m^, B in tesla, a in
Total force per unit length on AB is degrees
F = F] + ^2 = 16 X 10”^ + 8 X 10"^

FF
= 24 X 10“^ Nra“* directed downwards
Example^ A parallelogram shaped coil

s
Force per unit length on CD due to current in ABCD of sides 0*3 m and 0*2 m carries a current
AB is.
uro
ker
of 2*0 A as shown in Fig. 4.72. It is placed in a

e
2x4x4 —>
f:1 = lO-"^ X uniform magnetic field B = SOF parallel to AD.

For
(2x10-2)

ro
Find (i) forces on the sides of the coil and
= 16 X 10“^ Nm'^ directed upwards

fof
(k) torque on the coil.

o
Y
Force per unit length on CD due to current in
ooY
FIGURE 4.72
FFis,
B
A 2A
ks
2x4x4 B

F^ = 10-^ X 2x10“2
Yo
O'!
r

B
= 16 X 10"^ Nm"! directed downwards
eB
e

Da vC
u
rd

0-3 m

Total force per unit length of CD = ,F' + F2' = 0


Solution. Here, / = 0-3 m, 1? = 0-2 m ;
Example^ A wire AB is Carrying a Steady
o
Yn

7=2-0 A,B = 50T


ou
ad

current of 12 A and is lying on the table. Another —>

wire CD carrying current 5 A is held vertically (/) Since magnetic field B is parallel to sides
AD and BC, hence no force acts on these sides.
di

above AB at a height of 1 mm. Find the mass per


unit length of the wire CD so that it remains Force on the side AB is
FF
Re

suspended at the position when left free. Give the F = IIB sin 0 = 2-0 X 0-3 x 50 x sin 60°
in

direction of the current flowing in CD with respect = 2 X 0-3 X 50 X V3/2 = 25-98 N = 26 N


to that in AB. g = 10 ms~2. (C«SE 2013)
According to Flemings Left Hand rule the
Solution. Weight per unit length of upper wire
direction of the force is normally upwards. Similarly
= magnetic force per unit length force on the side DC will also be « 26 N.
Its direction is normally downwards.
I 4tc r (i/I Since force on sides AB and DC are equal,
Mass per unit length parallel and opposite, they form a couple which exerts
a torque. Therefore, torque on coil is
_m_\iQ 2/^/^ _lQ-'^x2xl2x5
X = F X AO sin 60° = 26 X 0-2 x -v^/2
/ 4tc rg (l0-3)xl0
= 4-5 Nm
= 1-2 X 10-3 kg m -1
The direction of current in CD must be opposite Example 03 A Circular coil of 20 turns and
to that of current in AB so that the force between the radius 10 cm is placed in a uniform magnetic field
two wires is repulsive. of 0.10 T normal to the plane of the coil. If the
4/62 ^ Fundamental Physics (XII)

current in the coil is 5.0 A, what is the (//) M = niA = ninr^


(a) total torque on the coil,
(b) total force on the coil, = 100x(3-2)x
y X (0-10)2
(c) average force on each electron in the coil
= 10 Am^
due to magnetic field ? The coil is made of copper
wire of cross-sectional area 10"® m^ and the free
iiii) T= \MxB l=MBsin0 where 0 is the
electron density in copper is given to be about
10^^ m"®. angle between M and B or between A and B .
Solution. Here, n = 20, r = 10 cm = 0.10 m, Initially, 0 = 0”, x = MB sin 0® = 0.
5 = 0.10T,a = 0°,/ = 5.0A
Finally, 0 = 90°, t = MB sin 90° = MB

ww
97
= 10 X 2 = 20 Nm
Area of the coil, A = nr^ = — x (0.10)2
7
dod , dti) dQ
(a) Now, X = n / B A sin a = n / S A sin 0° = 0 (,v) X = /a = / dt
= I —X —
dQ dt
(h) Total force on a planar current loop in a

Flo
magnetic field is always zero, dta

e
= / X 0) = MB sin 0.
(c) Given, 102^ m"^, A = 10"® m2

eree
d^
Force on an electron of charge e, moving with /co dco = MB sin QdQ.

FFr
drift velocity Vj in the magnetic field is given by Integrating it within the given conditions,
uurr
= Be-^

orr
71/2
F = BeVj NeA
(●.● / = NeAvJ) (S>

/ ©i/C0 = MB sin 0 dQ

F =
BI 0.10x5.0
= 5x10"2®N
sfo 0 0
kks
102^xl0"®
Yoo
NA
0)2
oooo

7C/2

Example m A 100 turn closely wound


I —
^ = MB(-cos9)”
eBB

circular coil of radius 10 cm carries a current of


ji
3*2 A. (0 What is the field at the centre of the coil ? — —MB cos cos 0° = MB
2
(<i) What is the magnetic moment of this
uurr

arrangement ? The coil is placed in a vertical plane


ad

n1/2 vl/2
and is free to rotate about a horizontal axis which 2MB 2x20
Yo

(0 =
dY

or
coincides with its diameter. A uniform magnetic / 0-1
field of 2 T in the horizontal direction exists such = 20 rad/s
innd

that initially the axis of the coil is in the direction


Re

Second Method for (iv)


of the field. The coil rotates through an angle of
Fi

90° under the influence of the magnetic field. Change in KE of rotation = work done in rotation
(«i) What are the magnitudes of the torques on 1
the coil in the initial and final positions ? (iv) What -lap- = MB (COS0J - cos@2)
is the angular speed acquired by the coil when it
has rotated by 90° ? The moment of inertia of the
where Oj = 0° ; 63 = 90°,/ = 0-1 kg m2 ;
MB = 20 Nm.
coil is 0*1 kg m2. (NCERT Solved Example)
1/2
Solution, it) Here, n = 100, r = 010 m, 2MB (COS0J - cos ©2)
1 = 3-2 A,B = 2T,/ = 0-l kg m2
0) -
/

B =
M-q 271«I oni/2
4ti r
2 X 20 X (cos 0° - cos 90
0-1

= ,^7 , —
10~2 x2x 22 X 100x3-2
= 2 X ir® T = 20 rad/s
7 010
MOVING CHARGES AND MAGNETISM 4/63

TYPE X. MOVING COIL GALVANOMETER Voltage sensitivity of meter A


Voltage sensitivity of meter B
Formulae used, (i) I = —0
= (Ratio of current sensitivity) x
nBA R
1
where, k = restoring torque per unit twist, = 1,
B = magnetic field in which coil of galvanometer is 14 20 ~ 28
suspended,
A = area of each turn of coil, 6 = deflection in Example ^ A Current of 300 ^lA deflects
galvanometer. a coil of a moving coil galvanometer through 30®.
A rt JiA What should be the current to cause the rotation
(ii) Current sensitivity, / = -I = k
(degree/A) through 7C/5 radians ? What is the current
(Hi) Voltage sensitivity, sensitivity of the galvanometer ? If resistance of
galvanometer is 50 £2, And its voltage sensitivity.

w
y =- = 1. = !1EA-L (degree/V) Solution. Here, /j = 300 Oj = 30®,
^ V IR kR ~ R
Here, R is the resistance of coil.
02 = I rad = 36®, Ij = ?

Flo
Units used. / in ampere, A in m^, B in tesla, 0 in
degrees, V in volt. For galvanometer.

reee
k k
^1 = — 0, and 1-) 0-,

FFr
Example^ Compare the current sensiti 04 1
nBA ^ nBA ^
vity and voltage sensitivity of the following moving
urr
coil galvanometers :
Meter A : n = 30, A = 1-5 x 10'^ m^
I
1
0

^2
for
1
kkss
B = 0-25 T, R = 20 Q 36°
or / = -^ X /, = x300^iA =360^iA
Meter B : /i = 35, A = 2-0 x 10“^ m^, 2 0, > 30®
Yo
ooo

B = 0-25 T, R = 30 Q. Current sensitivity.


eB

You are given that the springs in the two 0


1 _ 30®
metres have the same torsional constants. - 0-1 degree/|xA
/, 300 pA
Solution. For metre A, rij = 30,
r

Voltage sensitivity,
ou
ad

Ai = 1-5 X 10-3 ^2^ ^ 0-25 T, /?! = 20 Q.


_ 01 degree (M)"'
YY

For metre B, «2 = 35, Aj = 2-0 x 10~3 m^,


^2 = 0-25 T, = 30 a. ^ R 50Q
nndd
Re

. . . _ nBA
Current sensitivity of a meter = _ 0-1Qx(1Q-^)-'a
- 2000 degree/volt
Fi

50Q
Current sensitivity of meter A
Current sensitivity of meter B Example^ The Current sensitivity of a
moving coil galvanometer increases by 20% when
its resistance is increased by a factor 2. Calculate
k1 by what factor does the voltage sensitivity change ?
30x0-25x1-5x10~3 Solution. Given, /.,' = C + -^7 =1??/ ●
100'' 100-'’
S

35 X 0-25 x 2-Ox 10-3 R' = 2R

^_9_ Then, initial voltage sensitivity, = ~


70"l4
New voltage sensitivity,
nBA / ' 120 1
Now, voltage sensitivity of a meter = y/ = ^ I X = -V
kR R'
^100 2R 5 "
4/64
'a Fundamental Physics (Xll)tamJI

% decrease in voltage sensitivity Solution. In the given galvanometer, 10


divisions deflection is observed by 1 mA. So current
for full scale deflection is
^ 5 ^ xlOO =40%
^xl00 =
V
s /
g
= J-xl00mA = 10 mA = 10 X 10-3 A
10
Example m A current of 500 |iA deflects = 0-01 A
the coil of a moving coil galvanometer through As 2 divisions deflection is observed by 1 mV,
60°. What should be the current to cause the
so voltage for full scale deflection is
rotation through tc/5 radian ? What is the current

oww
sensitivity of galvanometer ? V = -xl00mV =50mV = 50x 10-3 V
Solution. Here, /j = 500 |i.A = 500 x 10“^ A, 2
= 0-05 V
7C 180°
e1 = 60°, A = ?, e. =-rad.= = 36° Galvanometer resistance.
^ 5 5

e
V 0-05
G = = 5Q

re
I 0-01
I — 6, and /- = — 02
^ nBA 3

FFrllo
g
1 “
nBA '
(/) For conversion into ammeter, / = 5 A,

rF
. ^2

ee
or
I 001
I 0 g
G = x5 =
1 I 5 =
ouru /-/ 5-0-01 499

rF
g

j 2 =^/,1 =—x500xl0-^
60
=300x10-* A Therefore, a shunt of (5/499) Q should be

fosor connected across the galvanometer to read 5 A for


0
os kf
36° full scale deflection.
Current sensitivity = -^ - iii) For conversion into voltmeter. For reading
^2 300x10-*
ook
Yo
1 div per volt, and there are 100 divisions, so the
Y
= 0-12 X 10* degree/A = 0-12 degree/|oA
voltage range should be 100 V.
Bo
reeB

TYPE XI. CONVERSION OF GALVANOMETER R = Z._G = —-5 = 10000 - 5 = 9995 Q


oouY

INTO (a) AMMETER AND (b) VOLTMETER I


g
0-01
ur

Formulae used. (/) Ig = nK Therefore, a resistance of 9995 C2 should


ad

I G be connected in series of the galvanometer to read


d

V
Y

g
{Hi) -G 1 div/volt.
(if) S =
(/-/J ^g
nidn

g
Example^ A resistance of 1980 Q is
Re

n = number of scale divisions in galvanometer,


connected in series with a voltmeter, after which
K = figure of merit of galvanometer,
FFi

the scale division becomes 100 times larger. Find


Ig = current for full scale deflection, the resistance of voltmeter.
/ = total current to be measured,
Solution. Let R be the resistance of voltmeter.
S = shunt resistance, G = galvanometer resistance,
Let n be the number of divisions in the voltmeter.
V = potential difference to be measured,
R = resistance to be connected in series The voltage (VO recorded by each division of
voltmeter when current ig flows through it is
Units used. /, Ig in A, V in volt, G, R in ohm. ig R/n^V ...(0
When resistance is connected in series with
Example A galvanometer with a scale voltmeter, then
divided into 100 equal divisions has a current ig (R + 1980)/n = 100 V ..(«)
sensitivity of 10 divisions per mA and a voltage Dividing (i7) by (i), we get
sensitivity of 2 divisions per mV. What adoptions /? + 1980= 100 R
are required to read (/) 5A for full scale and
or R = 1980/99 = 20 Q
(ii) 1 division per volt ?
MOVING CHARGES AND MAGNETISM
4/65

Example A galvanometer reads 5*0 V FIGURE 4.73


at full scale deflection and is graded according to « (A>
its resistance per volt at full scale deflection as

5000 Q V“^ (0 How will you convert it into a


3.00 n
voltmeter that reads 20 V at full scale deflection?
(ii) Will it still be graded 5000 Q ? (in) Will
you prefer this voltmeter to one that is graded 3.00 V
2000 a V"^ ? [CBSE 2001 (C)] Solution. (/■) Let G be the resistance of ammeter
Solution. Here, resistance per volt = 5000 {i.e. galvanometer). Then current in the circuit,
QV~^ It means for 1 volt pot. difference, the
resistance of galvanometer is 5000 Q. So current for e 3-00
I = = 0-048 A.
full scale deflection. R +G 300 + 6000
I (//) When the ammeter (Le., galvanometer) is
I
shunted with resistance S, its effective resistance,

w
8 A =0-2x 10-3 A
5000
, GS 60x002
(0 In order to convert it into a voltmeter of R 0-02 Q
^~G+S~60 + 0-02

Flo
range 0 to 20 V, let a resistance R be connected in
Current in the circuit
series with it. Then on applying an extra potential

e
difference = 20 V - 5 V = 15 V, the cuirent through e 3-00

rree
/ = = 0-99 A
it for full scale deflection is again 0-2 x 10"3 A i e R +R 300 +-02

r FF
/p = 0-2x 10-3 A. p

15 15
(in) For the ideal ammeter with zero resistance,
uurr
for
R L
8
= 15 or R = — = 75000 Q 3-00
I
8
0-2x10-3 current, / = = 1-00 A
3-00
Thus, to convert a given voltmeter (of range
kkss
0 to 5 V) into a voltmeter (of range 0 to 20 V) a I TYPE XII. TYPICAL EXAMPLES
I
Yo
resistance of 75000 H should be connected in series, I
oooo

with the given meter.


Example S3 In Fig. 4.74 bcdf is a circular
eB

(ii) Original resistance of voltmeter


coil of non-insulated thin uniform conductors, ab
= 5000 n V-’ X 5 V = 25000 Q
and de are very long straight parallel conductors,
rr

Total resistance after conversion tangential to the coil at the points b and d. If the
ou
ad

= 25000 + 75000 = 100,000 Q current 5 A enters the coil from a to b, find the
YY

Resistance per volt of new meter magnetic field induction at O, the centre of the
coil. The diameter of the coil is 10 cm.
100,000
nndd
Re

= 5000 a v-^ FIGURE 4.74


20
b 5A
Fi

i.e. it has the same grading as before. a

(Hi) If greater is the resistance per volt of a


c
/
meter, the lesser will be the current drawn from the O
circuit by it. Due to it, it works better. That is why 2. 5A
e

this meter (graded 5000 .Q V^) is more accurate d

that the one graded as 2000 Q V*. Solution. The current in the part bed of the coil
Example^ In the circult shown In Fig. is equal to the current in the part bfd of the coil, which
4.73, the current is to be measured. What is is equal to 2-5 A. Here,
the value of the current if the ammeter shown r =Ob = Oc = Od = Of=5cm = 5x IQ-^ m.

(i) is a galvanometer with a resistance Rq = 60H)0 Q. Magnetic field induction at O due to current
(ii) is a galvanometer described in (f) but converted through circular coil will be zero because the magnetic
to an ammeter by a shunt resistance = 0-02 £2 ; induction at O due to current through segment
(Hi) is an ideal ammeter with zero resistance ? of the coil is equal and opposite to that due to
(NCERT Solved Example) current through segment bfd of the coil.
4/66 pnadee^'4. Fundamental Physics (XII) VOL.I

Total resistance of the circuit


Magnetic field induction at O due to current
5000 15400
through long straight conductor ab is + 800 = 12
13 13
Bi = ^-(sin90“+sin0‘*)
4jc r
6 39
A
Current in the circuit, /j = 15400/13 7700
= 5 = 10t5t Potential difference across A. and B
41C r 5x10-2 39 5000 150

w
X = 1-95V
outward normally to the plane of paper. 7700 13 77
Magnetic field induction at O due to current Reading of voltmeter is 1*95 V
through long straight conductor de is iExaifnpi^Hil In the circuit shown in Fig.
-H:ol(sin9oo+sin0®)

e
4.76, a voltmeter erads 30 V when it is connected
^ 4jc r across 40012 resistance. Calculate what the same

e
o
voltmeter will read when connected across the

rw
= 10t5t

r
41C r 5x10-2 30012 resistance.

eF
outwards normally to the plane of paper FIGURE 4.76

ullo
Total magnetic field induction at O 30 V

FF
normal to plane of paper outwards. ^La/SA/VW B

rre
AAAA/W

s
300 a 40012
IexStoTi^PW Two resistors 400 Ci and

koF
800 Q are connected in series with a 6 V battery. , 60V
It is desired to measure the current in the circuit
fofr
uor
An ammeter of 10 Q resistance is used for this
purpose Fig. 4.75. What will be the reading in the o Solution. The circuit is shown in Fig. 4.29.
Let G be the resistance of voltmeter, connected
kos
ammeter ? Similarly, if a voltmeter of 10,000 Q
in parallel with 400 £2, their effective resistance
YYo
rBB

resistance is used to measure the potential


oo
Y

difference across the 40012 resistor, what will be 400xG


becomes.
the reading in the voltmeter ? G + 400
r ee

Current in the circuit,


u

FIGURE 4.75
^^00 n ^5
Yd

400 n 800 n
^Uaww-J^-www-i
oou

300 + 400G/(G + 400)


ad

WWW
400 n 80012
I1 It 60 (G + 400)
din

I
6V
I 300 (G + 400) + 400G
O Pot diff. across A and B = IRp = 30 V
Re
FF
in

400G
Solution. The circuits are shown in Figs. 4.75(a) or
60 (G + 400) = 30
and (b). 300 (G + 400)+ 400G (G + 400)
On solving. G= 1200 12
In Fig. 4.75(a), all the resistors are in series,
total resistance of the circuit, When this voltmeter is connected in parallel
with the resistance of 30012, the effective resistance
R = 400 + 800+10 = 12100
Hence, current in the circuit, , 1200x300 = 240 12
becomes.
1200 + 300
/=^:= ^ =0-00496 A Total resistance of the circuit
R 1210
= 240 + 400 = 64012
Reading in ammeter is 0*00496 A 60 3
= —A
In Fig. 4.7S(b)f the resistors of 400 12 and Current, =
640 32
10,00012 are in parallel, their effective resistance Rp Reading of voltmeter = pot. diff. across
400x 10,000 _ 5000 12
will be = /'R ' = —x240 =22*5V
400 + 10,000 13 V P 32
MOVING CHARGES AND MAGNETISM 4/67

1. Magnetic effect of current. It is a phenomenon by virtue of which a current produces, a magnetic field. A
straight current {i.e. linear conductor carrying current) produces a circular magnetic field and acircular current
(i.e. circular coil carrying current) produces a straight magnetic field at the centre of the circular coil.

2. Magnetic force on a charge q, moving with velocity in a magnetic field B is given by


\ F \ = q \ i^X B \ = qvBsmQ
\{q= l,v = 1, sin 0 = 1 or 0 = 90°, then F=B.
Thus strength of magnetic field at a point in the field is equal to the force acting on a unit charge moving
with a unit velocity, perpendicular to the direction of the magnetic field at that point.
Magnetic force on the charged particle in a magnetic field is maximum if the charged particle is moving

ww
perpendicular to the direction of magnetic field (i.e. 0 = 90°) and is minimum, if the charged particle is
moving along the direction of the magnetic field (Le. 0 = 0°).
3. SI Unit of magnetic field strength is tesla. If a charge of 1 coulomb, while moving with a velocity of

Floo
1 ms'* at a point, perpendicular to the direction of magnetic field, experiences a force of 1 newton, the
strength of magnetic field at that point is I tesla.

ee
I tesla = 1 N A * m '

reer
4. Dimensions of B = [MA r-2].
-1

rFF
5. Biot-Savart’s law. According to this law, the magnitude of magnetic field induction at a point due to a
current element of length dl, carrying current /, at a distance r from the element is given by
uur r
ffoor (in SI Units)
sks
47t r-'
YYoo
ooko

The direction of dB is given by Right Handed Screw Rule or Right Hand Rule.
eBB

6. Magnetic field induction at the centre of a circular coil of radius r carrying current I is given by
B = (in SI Units)
r

An r Ir
ouur
ad

The direction of this magnetic field induction is given by Right Hand palm rule.
Y

7. Magnetic field induction at a point, at perpendicular distance ‘a’ from the axis of long linear conductor
nddY

carrying current is given by,


Re

(in SI Units)
FFiin

B =
An a

The direction of the magnetic field lines is given by Right Hand thumb rule
8. Magnetic field induction at a point on the axis of a circular coil carrying current is given by

B =
4o Innla^
(in SI Units)
47C(fl2+_y2)3/2
where a is the radius of the coil, x is the distance of the point on the axis, from the centre of the coil, n is the
number of turns in the coil.

Magnetic dipole moment of the current loop is given by


M - nlA = nln a^.
where n is the number of turns in the loop, carrying current / and a is the radius of the loop. The SI unit of
M is ampere (metre)^ [A-m^]
4/68
“piunUe^’4. Fundamental Physics (XII) VOL.I

10. Ampere’s circuital law. It states that the line integral of magnetic field induction ~B around any closed
path in vacuum is equal to times the total current threading the closed path i.e.

11. A solenoid consists of an insulated long wire closely wound in the form of a helix. Its length is very large
as compared to its diameter.
Magnetic field induction at a point well inside the solenoid of length /, having N turns carrying current
/ is given by
fi = PoW//.
Magnetic field induction at a point on one end of the solenoid carrying current is 5 = Pq NUl I
12. Force on a charged particle in an electric field is

ww
F=qE.
This force acts in the direction of electric field. The charged particle describes a parabolic path if electric
field is acting perpendicular or at some angle with the initial direction of motion of the particle.

Flo
13. Magnetic force on a charged particle in a magnetic field is

ee
rere
\F\ = q \ vx B \ =qv Bsind.

r FF
This force acts perpendicular to the plane containing u and 5 , and is directed as given by Right Handed
uurr
screw rule. If the moving charged particle is subjected to a perpendicular magnetic field, it describes a
foor
circular path and if moving charged particle is subjected to a magnetic field, making some angle with its
initial direction of motion, it will describe a helical path.
ks s
Yoo
14. Lorentz force. The force experienced by a charged particle moving in space where both electric and magnetic
ooook

fields exist is called Lorentz force, i.e.


eBB

15. Cyclotron frequency. When the frequency of oscillations of a charged particle in cyclotron becomes equal
rr
ouu

to the frequency of the oscillating electric field applied, then the frequency of the particle is called cyclotron
ad

frequency, which is given by


YY

V =
Bq
nndd

2nm
Re

16. Force on a current carrying conductor in a magnetic field is given by


Fii

\Fl=I\TxB\==IlBsinQ
This force is maximum if sin 0 = I or 0 = 90“ and is minimum if sin 0 = 0 or 0 =r 0“ or 180“.
17. Two long parallel conductors carrying currents in the same direction attract each other and carrying
curents in the opposite direction, repel each other. The force acting per unit length of each conductor will be

4n r

where /j, /2 = currents in the two linear parallel conductors, placed distance r apart.
18. One ampere of current is that much current which when flowing through each of the two parallel long
linear conductors placed in free space at a distance of one metre from each other will attract or repel each
other with a force of 2 x 10"^ N per metre of their length.
MOVING CHARGES AND MAGNETISM

19. Torque on a coil of area A, having n turns, carrying current I, when suspended in a magnetic field of
strength B is given by
X = nIBA sin a

where a is the angle which a normal drawn on the plane of the coil makes with the direction of magnetic
field.

Torque is minimum if the plane of the coil is perpendicular to the direction of magnetic field, because a =
(T.

Torque is maximum if the plane of the coil is parallel to the direction of magnetic field because (X = 90®.
20. Radial magnetic field. It is that magnetic field, in which the plane of the coil in all positions remains
parallel to the direction of the magnetic field.
21. Current sensitivity of a galvanometer. It is defined as the deflection produced in the galvanometer, when
unit current flows through it

ww
Q nBA
Current sensitivity.
k

Floo
The unit of is rad. A~^ or div. A"'
22. Voltage sensitivity of a galvanometer. It is defined as the deflection produced in the galvanometer when

ee
a unit voltage is applied across the two terminals of the galvanometer.

eer
0 0 nBA fg

rFrF
V =-
Voltage sensitivity. s
IR~ kR R
rur
Unit of voltage sensitivity is rad. V~^ or div. V^.
ffoor
23. Shunt. It is a low resistance wire connected in parallel with the galvanometer or ammeter. It protects
s
the
galvanometer or ammeter from the strong currents.
osk
YYoou
24. Conversion of galvanometer into ammeter
oook

A galvanometer can be converted into an ammeter of given range by connecting a suitable low resistance S
eBB

in parallel to the given galvanometer, whose value is given by :


I
r

8
S = G
ouur
ad

/-/
Y

where Ig is the current for full scale deflection of galvanometer, / is the current to be measured by the
nd dY

galvanometer and G is the resistance of galvanometer.


Re

An ideal ammeter has zero resistance.


FFini

25. Conversion of galvanometer into voltmeter


A galvanometer can be converted into voltmeter of given range by connectintg a suitable resistance R in
series with the galvanometer, whose value is given by
V
R = -—G
I
8

where Vis the voltage to be measured, Ig is the current for full scale deflection of galvanometer and G is the
resistance of galvanometer.
The resistance of an ideal voltmeter is infinity.
4/70
Fundamental Physics (XII) VOL.I

'ilWB TT^ffBB QUESTIONS

Based on NCERT Book

I. Multiple Choice Questions (rt) 2 X 10“-’ T, acting downwards

w
(h) 2 X 10“^ T, acting upwards
1. Two wires carrying currents /, and /, lie , one
(c) 4 X 1(T-^ T, acting downwards
slightly above the other, in a horizontal plane us

shown in Fig. 4(Q).l. The region of vertically Ui) 4 X lO”*' T, acting upwards <CS«E 2020)

e
upward strongest magnetic field. 4. An electron and a proton are moving along the

e
same direction with the same kinetic energy. They

wr
FIGURE 4(Q).1
enter a uniform magnetic field acting perpendicular

lloo
r
to their velocities. The dependence of radius of
II
their paths on their masses is :

F
h

FFu
(a) r « w (h) r oc
/1
1
(c) r oc —
{(i) r oc

rese
(CHSE 2020)
uro 111 IV

k
Fr
o
5. A current of 5 A is flowing from east to west In a
long straight wire kept on a horizontal table. The

foo
fr
{a)\ (b) II
magnetic field developed at a distance of 10 cm
due south on the table is :
kso
Y
(c) III id) IV (CBSE 2022)
(rt) 1 X I0~-^ T acting downwards
2. A cunent / flows through a long straight conductor
Y
B

{b) \ X \ T acting upwards


Yo
which is bent into a circular loop of radius K in the
oo

middle as shown in the Fig. 4(Q).2. The magnitude (r) 2 X I0~^ T acting downwards
eBr

of the net magnetic field at point O will be : id) 2 X 10“-’ T acting upwards (CBSE 2020)
e
uru

FIGURE 4(Q).2
6. There are unifonn electric and magnetic fields in a
od

region pointing along X-axis. An a-particle is


projected along T-axis with a velocity v. The shape
no
ad

of the trajectory will be :


Y

Ui) circular in XZ plane


di

(b) circular in TL plane


Re
FF

(r) helical with its axis parallel to X-axis


in

Ul) helical with its axis parallel to K-axis


/ " /
(CBSE 2020)

7. A charged particle after being accelerated through


a potential difference V enters in a uniform
(a) zero (b) (l + TC) magnetic field and moves in a circle of radius r. If
2R
V Is doubled, the radius of the circle will
1 becomes:
(c) id) 1--
4tcR 2R { TCj (a) 2 r if’) -Jlr
(CBSE 2020)
3, A current 10 A is flowing from east to west in a ic)Ar id) r/VI 2020)
long straight wire kept on a horizontal table. The
magnetic field developed at a distance of 10 cm
8. Two concentric and coplanar circular loops P and
due north on the table is ; Q have their radii in the ratio 2; 3. Loop Q carries
a current 9 A in the anticlockwi.se direction. For
MOVING CHARGES AND MAGNETISM 4/71

the magnetic field to be zero at the common centre, (c) 0-6 (iT, vertically upward
loop P must carry id) 0-6 pT vertical downward (CBSE 2022)
(a) 3 A in clockwi.se direction -»

12. An electron is projected with velocity li along


(b) 9 A in clockwise direction the axis ofa current carrying long solenoid. Which
(c) 6 A in anticlockwise direction of the following .statements is true ?
(d) 6 A in the clockwise direction (a) The path of the electron will be circular about
(CBSE Sample Paper 2022*23) the axis

9. A long .straight wire of circular cross-section of (b) The electron will be accelerated along the axis
radius a carries a steady current 1. The current is (o) The path of the electron will be helical
uniformly distributed across its cross-section. The (d) The electron will continue to move with the
ratio of the magnitudes of magnetic field at a point
all above the surface of the wire to that at a point same velocity v along the axis of the

ww
distant «/2 below its surface is : solenoid (CBSE 2022)

(ft) 4 : I {b) 1 : 1 13. If the speed v of a charged particle moving in a


(c) 4; 3 id) 3 :4 magnetic field B (v is perpendicular to B) is

FF loo
(CBSE Sample Paper 2022-23) halved, then the radius of its path will :

ee
10. A current / is flowing through the loop as shown (fl) not change (h) become two times
in Fig. 4(Q).3 IMA = R,MB=2 R). The magnetic

er
(c) become one-fourth (d) become half

FrreF
(CBSE 2022)
field at the centre of the loop is times:
R ,-19
14. A test charge of 1-6 x 10 C is moving with a
rur
FIGURE 4(Q).3 fforo
velocity v = (4 i +3k) ms in a magnetic field
ks
B = Ok+ 4i )T. The force on this test charge
YYouo
okso

IS;
BBoo

(a) 24 j N (b) -24/ N


r ee

(<●) 24k N (^/)0


ad
oouur

15. Two identical thick wires and two identical thin


wires, all of the same material and the same length
Y

form a .square in three different ways .P Q and R


5
as shown in Fig. 4(Q).4. Due to the current on
dnYd
Re

(ii) —. into the plane of the paper


16 these loops, (he magnetic field at the centre of
the loop will be zero in ca.se of :
FFini

5
ib)
16
. out of the plane of the paper
FIGURE 4(Q).4

7
(c) —'
16
out of the plane of the paper

7
(d) 77
16
● into the plane of the paper
(CBSE 2022) (h) Q and R only
(rt) P and R only
II. A long straight wire in the horizontal plane carries (c) P and Q only (d) P Q and R
a current of 15 A in north to south direction. The
(CBSE 2022)
magnitude and direction of magnetic field at a
16. A circular current carrying coil produces a
point 2-5 m east of the wire respectively are :
magnetic field at its centre. The coil is rewound
(fl) 1-2 pT, vertically upward so as to have three turns and the same current is
(h) I -2 pT vertically downward
4/72
“Ptadce^ 4 Fundamental Physics (XII) VOL.I

passed through it, the new magnetic field at the R from the centre of loop is ^he ratio
centre is:
(S|/B2> is :
B
(a) 2V2 Q>) 1V2
(fl) 3 Bq ib)^
B
(c) -Jl (d}2 (CBSE 2022)

(0^ id)9BQ (CBSE 2022) 23. A current carrying wire kept in a uniform magnetic
field will experience a maximum force when it is
17. A long solenoid carrying current produces a
magnetic field B along its axis. If the number of (fl) perpendicular to the magnetic field
turns in the solenoid is halved and current in it is (b) parallel to the magnetic field
doubled the new magnetic field will be : (c) at an angle of 45° to the magnetic field
B (d) at an angle of 60° to the magnetic field
(a)~ (b)B (CBSE 2022)

w
(c)2B (d)4B (CBSE 2022) 24. A straight conducting rod of length / and mass m
is suspended in a horizontal plane by a pair of
18. Two moving coil galvanometers Gj and C2 have

Flo
the following particulars respectively. flexible strings in a magnetic field of magnitude
B. To remove the tension in the supporting strings,
/Vi = 30, A, = 3-6 X 10-3 m2, B^ = 0-25 T the magnitude of current in the wire is :

ee
N2 = 42, A2 = 1-8 X 10-3 m-,B2 = 0-50 T

Fr
mgB mgl
The spring constant is same for both the {a)
/ B
galvanometers. The ratio of current sensitivities
of G| and G2 is :
for
ur
mg IB
(a)5:7 {b)l-.5 (c) (^0
IB mg
(c) I : 4 (rf)l:l (CBSE 2022)
ks
(CBSE 2022)
19. A current carrying square loop is suspended in a
Yo
oo

uniform magnetic field acting in the plane of the 25. A proton and an alpha particle move in circular
orbits in a uniform magnetic field. Their speeds
eB

loop. If the force on one arm of the loop is F, are in the ratio of 9 : 4. The ratio of radii of their
the net force on the remaining three arms of the
r
loop will be
r

p
circular orbits
ou

IS :
ad

{a) 3F (b) ~3F _ ^alpha


Y

-*

(c) F 3 4
(d)-F (CBSE 2022)
w 4
nd
Re

20. Two parallel conductors carrying current of 4 0


A and 10 0 A are placed 2-5 cm apart in vacuum. 8
Fi

9
The force per unit length between them is (O9 .(d) r
8
(a) 6-4 X 10-^ Nm-' (b) 6-4 x lQ-2 Nm''
(CBSE 2022)
(c) 4-6 X 10"^ Nm-‘ (d) 3-2 x 10^ Nm-'
(CBSE 2022) 26. The current sensitivity of a galvanometer
increases by 20%. If its resistance also increases
21. If an ammeter is to be used in place of voltmeter, by 25%, the voltage sensitivity will
then we must connect with the ammeter a :
(a) decrease by 1 %
(a) low resistance in parallel
{b) increase by 5%
(b) low resistance in series
(c) increase by 10%
(c) high resistance in parallel
{d) decrease by 4%
(d) high resistance in series (CBSE 2022)
27. Three infinity long parallel straight current
22. The magnetic field at the centre of a current
carrying wires A, B and C are kept at equal
carrying circular loop of radius /? is S,. The distance from each other as shown in Fig. 4.(Q).5.
magnetic field at a point on its axis at a distance The wire C experiences net force F. The net force
MOVING CHARGES AND MAGNETISM 4/73

on wire C, when the current in wire A is reversed 31. A rectangular coil ABCD is placed near a long
will be straight current carrying straight wire as shown.
What is the net force on the rectangular coil ?
FIGURE 4(Q).5
FIGURE 4(Q).7

4-r r->
B C

27- h' /“ 2A“ “1A 1A” 15 cm

A B C ►A D
2 cm ^ -►
10 cm
(fl) zero {b) FH

ww
ic)F {d)lF (a) 25 X 10"^ N towards the wire
{b) 25 X 10"^ N Away from the wire
28. In a certain region of space, electric field E and

Floo
(c) 35 X 10“^ N, towards the wire
magnetic field S are perpendicular to each other. (d) 35 X 10"^ N away from the wire
An electron enters perpendicularly to both the field

ee
32. Two wires of same length are shaped into a square
and moves undeflected. The velocity of electron is

eer
and a circle if they carry same current, ratio of

rFrF
B magnetic moment is :
(a) ~ (« -
B (a) 2 : 7t (/?) 7t : 2
rur
(c) ExB

id) E B
(c) 7C: 4
ffoor (i7) 4:71
33. Current sensitivity of a galvanometer can be
s
29. A deuteron of kinetic energy 50 keV is describing increased by decreasing:
osk
YYoou
a circular orbits of radius 0-5 m in a plane (a) Magnetic field B
oook

—> ^ ^
(b) number of turns n
perpendicular to the magnetic field B . The kinetic
eBB

energy of the proton that describes a circular orbit (c) torsional constant k
(d) Area A
of same radius and inside same B is
r

34. An electric current passes through a long straight


ouur
ad

(a) 25 kev (b) 50kev


copper wire. At a perpendicular distance 5 cm from
(c) 200 kev (d) 100 keV
Y

the straight wire, the magnetic field is B. The


30. Two particles A and B with same charges and magnetic field at perpendicular distance 20 cm
nd dY

different masses and mg respectively) are from the straight wire would be
Re

moving in a plane inside uniform magnetic field B


B
which is perpendicular to the plane. The speed of
FFini

(a) - ib) -
the particles are and Vq respectively and the
trajectories are as shown in Fig. 4(Q).6. Then B B

W 3
35. A wire in the form of a circular loop, of one turn
carrying a current, produces magnetic induction B
at the centre. If the same wire is looped into a coil
of two turns and carries the same current, the new
value of magnetic induction at the centre is
(.a) B ib) 2B
{a) (b) Vq (c) 4B (d) ZB
(c) < mg and < Vg 36. A circular coil of radius a carries an electric current.
(d) nify = mg and < Vg The magnetic field due the coil at a point on the
4/74
Fundamental Physics (XII) VOL.I

axis of the coil located at a distance r from centre


42. Time period of a charged particle undergoing a
of the coil, such that r » a, varies circular motion in a uniform magnetic field is
I 1 independent of
(fi) -
r (a) speed of the particle
1 I (b) mass of the particle
<^>73 (cf)
,3/2 (c) charge of the particle
37. A solenoid has 1000 turns per metre length. If a (d) magnetic field
current of 5 A is flowing through it, then magnetic (CBSE Sample Paper 2020)
field inside the solenoid is
(a) 2nx I0~-''T (h) 2nx 10-5T
(c) 4 71X 10“^ T id) 2kx 10-5T II. Assertion-Reason Type Questions
38. Currents of 10 A and 2 A are flowing in opposite Direction. For question numbers 43 to 60, two
directions through two parallel wires A and B statements are given, one labelled Assertion (A)

w
respectively. If the wire A is infinitely long and wire and the other labelled Reason (R). Select the
is 2 m long, then force on wire B which is situated correctanswerto these questionsfrom the codes
(a), (b), (c) and (d) as given below :

Flo
at 10 cm from A, is
(a) 8 X 10-5 N ib) 6 X I0"5 N (a) Both A and R are true, and R is the correct

e
explanation of A.

rree
(c) 4x 10“5n id) 2x 10-5 jsj
(b) Both A and R are true, but R is not the correct

r FF
39. If distance between two curreni-carrying wires is
explanation of A.
doubled, then force between them is
(c) A is true but R is false.
uurr
(a) halved
(c) tripled
(b) doubled
id) quadrupled
40. The ratio of voltage sensitivity (V^) and current
for
id) A is false, and R is al.so false.
43. Assertion. When radius of a current carrying loop
kss
sensitivity (/^) of a moving coil galvanometer is is doubled, its magnetic moment becomes four
ooook
Yo
times.
1 1
ia) - ib) Reason. The magnetic moment of a current
eB

G
carrying loop is directly proportional to the area of
ic) G id) ep¬ the loop. (CBSE 2022)
rr

41. A 100 turns coil shown in the Fig. 4(Q).8 carries a 44. Assertion. Higher the range, lower is the resistance
ou
ad

current of 2 A in a magnetic field of 0-2 Wb - m -2■“. of an ammeter.


The torque acting on the coil is
Y

Reason. To increase the range of ammeter,


dY

additional shunt is added in series to it.


FIGURE 4(Q).8
Re

(CBSE 2022)
innd

8 cm
A B 45. Assertion. When a charged particle moves with a
Fi

©
Q
velocity in a magnetic field ~B (v \.~B), the
Q. " 10 cm . L
z
a.
force on the particle does not work.
Reason. The magnetic force is perpendicular to
C D the velocity of the particle.
46. Assertion. A proton and electron, with same
ia) 0-32 N-m tending to rotate the side AC into momenta, enter in a magnetic field in a direction
the page
at right angles to the lines of force. The radius of
ib) 0-32 N-m tending to rotate the side AC out of the paths followed by them will be the same.
the page
Reason. Electron has less mass than the proton.
(c) 0-64 N-m lending to rotate the side AC into
47. Assertion. A vertical wire carrying a current in the
the page
upward direction is placed in a horizontal magnetic
id) 0-64 N-m tending to rotate the side AC out of field directed towards north. The wire will
the page experience a force directed towards east.
MOVING CHARGES AND MAGNETISM 4/75

Reason. The direction of force on a wire carrying Reason. Magnetic field at a point on the axis of
current in a perpendicular magnetic field will be
wire at a distance r from one end is B =
given by Fleming’s Right Hand Rule. 4ti r
48. Assertion. On increasing the current sensitivity of
56. Assertion. Two infinitely long, thin, insulated,
a galvanometer by increasing the number of turns,
may not necessarily increase its voltage sensitivity. straight wires lie in the x-y plane along the x and
Reason. The resistance of the coil of the
y-axes respectively. Each wire carries a current I
in the positive x-direction and positive y-direction.
galvanometer increases on increasing the number
of turns. Fig- 4(Q).9. The magnetic field will be zero at all
points on the straight line, y.= x.
49. Assertion. To increase the range of ammeter, we

oww
must connect a suitable low resistance in parallel FIGURE 4(Q).9
to it.

Reason. The ammeter with increased range should ● 1

have high resistance.


50. Assertion. A magnetic field is not associated with

e
yA

a charge in uniform motion.

re
FFrlo
-~^X
Reason. A charge in a uniform motion produces J

rF
electric field only.

ee
A A

51. Assertion. If electron velocity is (2/+ 3 j) and Q

rF
ouru A A

is subjected to a magnetic field (2 / + 3 y), then Reason. The magnetic field at a point distance r
the magnitude of force on the electron is zero.

fosor
from a long linear current carrying wire is given
Reason. Force on the moving charged particle in
skf
, —* Un 21 A , A .
magnetic field is, F = q (v x B). by 5 =_5i.—n where n is a unit vector ±r to
ooko
4k r
YYo
52. Assertion. The maximum energy of deutron
coming out of a cyclotron accelerator is 20 MeV. the plane containing conductor and the point.
Bo

The maximum energy of a proton that can be


reeB

obtained from this accelerator is 10 MeV. 57. Assertion. A wire is bent in the form of a quadrant
Reason. Maximum energy of a charged particle of radius ras shown in Fig. 4(Q).10 The magnetic
ooY
uur

accelerated by cyclotron is directly proportional to


ad

field at the centre 0 due to current / is


mass of the particle. 8r
dY

53. Assertion. Electrons can not be accelerated to a

very high energy by means of cyclotron.


nind

Reason. The electron while accelerated in


Re

cyclotron goes quickly out of step with accelerating


FFi

electric field.

54. Assertion. A straight wire of diameter 0-5 mm


carrying a current of 1 A is replaced by another
wire of 1 mm diameter carrying the same current.
The strength of the magnetic field at distance 5 cm Reason. The magnetic field at the centre of an arc
from the axis of wire becomes one quarter of the of radius r, subtending an angle 0 at centre is
earlier value.

Reason. Magnetic field at a point due to straight B = l^'e.


4k r
current wire is inversely proportional to square of
the distance of point from the axis of wire. 58. Assertion. The magnetic field at the end of a very
55. Assertion. A current of 1 A is passed through a long current carrying solenoid is half of that at the
centre.
straight wire of length 2 0 m. The magnetic field
at a point in air at a distance of 4 m from one end Reason. For a sufficiently long solenoid carrying
^^0 current, the magnetic field within the solenoid is
of the wire lying on the axis of wire will be
8k uniform.
tesla.
4/76 4 Fundamental Physics (XII)

59. Assertion. A circular loop of radius R carrying a 60. Assertion. If an electron and a proton are projected
current / is placed in a uniform magnetic field with at right angles to a uniform magnetic field with
—>
the same linear momentum, the electron trajectory
its plane parallel to magnetic field B . The torque
will be less curved than the proton trajectory.
on the loop is zero.
Reason. Radius of the curved path of the charged
Reason. Torque on the current loop in the magnetic
particle moving in a perpendicular magnetic
field is BAJ sin 9, where 0 is the angle between
field is directly proportional to the mass of the
plane of loop and magnetic field.
particle.

ANSWERS

I. Multiple Choice Questions

ww
l.(&) 2. id) 3. (a) 4. (a) 5. (b) 6. (c) 7. (b) 8. (d) 9.(c) 10. (d)
11. (a) 12. (d) 13. (d) 14. (d) IS. (a) 16. id) 17. ib) 18. ia) 19. id) 20. (d)
21. (d) 22. (a) 23. (c) 24. (c) 25. (d) 26. id) 27.ia) 28. ia) 29. id) 30. (/?)

Flo
31, (a) 32. (c) 33. (c) 34. (b) 35.(c) 36. (c) 37. ia) 38. ia) 39. ia) 40.(a)

e
41. ib) 42.(a)

rree
II. Assertion-Reason Type Questions

r FF
43.ia) 44. ic) 45. ia) 46. ib) 47. id) 48. ia) 49. ic) 50. id) 51. ia) 52. id)
uurr
53.ia) 54. (^0 55. id) 56. ia) 57. ia) 58. ib)
for
59. id) 60. id)
kss
HINTS/EXPLANATIONS For Difficult Questions
ooook
Yo

I. Multiple Choice Questions


eB

FIGURE 4(Q).12
1. Let and B2 be the magnetic field due to the a
currents and I2 respectively. Then the magnetic
rr

fields in all the four regions is shown in the


ou
ad

I
Fig.4(Q).ll.
Y
dY

FIGURE 4(Q),11
b
Re

f
innd

II I
Fi

Bi (●). B2(.) Bi (.), B2(x)


/

h
> g

Bi (X). B2H Bi (X). B2(x)


III rv —» _ Rq 2 71/ nA +
^ ~ ^bcdef'^ ^abfg 4tc R

1 A
The region of vertically upward strongest magnetic 1 — n

field will be in region II.


2rV TtJ
A

2. Let n be the unit vector perpendicular to the plane


iVTi-i
—)

151 =
of paper downwards. The magnetic field B QiO 2R TtJ
due to current through the entire structure is
MOVING CHARGES AND MAGNETISM 4/77

3. Refer to Fig. 4(Q).13, the magnetic field at P due Force on charged particle due to magnetic field
to current in long straight wire = ^ V S, acting along-Z-axis. Under the effect of
these forces, the charged particle will describe a
FIGURE 4(Q).13 helical path with its axis parallel to 3f>axis
P
Fig. 4(Q).15.
X N
I
1
10 cm

± W E
10A

ww
B = ^q/ ^ (47txlQ-^)xl0 = 2 X 10“* T
2nR 2tcx (10x10-2)

Floo
According to Right Hand Thumb Rule, the

ee
direction of B is acting downwards.

eer
4. A charged particle of charge q, mass m while 7. Since, aV = — miP'

rFrF
2
moving with speed v enters a uniform magnetic
field B, then, force on charged particle due to 2qV
rur
magnetic field provides the required centripetal velocity acquired, v =
force. If r is the radius of the circular path traced
by charged particle in the uniform magnetic field,
ffoor m

Now, radius of the circular path in magnetic field,


s
osk
then 2qV .
YYoou
_ mv _ m i.e., r oc Vv
qB qB \
oook

m
2 2
mv mv
eBB

Bqv = or r = i.e., rocm


r Bq When V is doubled, then — = r V
= V2
S. Refer to Fig. 4(Q).14, / = 5 A, r = 0-10 m
r

or r' = lS
ouur
ad

FIGURE 4(Q).14 8. Refer to Fig. 4{Q).16, magnetic field at O due to


Y

N
current in loop Q
nd dY
Re

5A
W >E
FFini

!I 10 cm
I

I S
P

Magnetic field at P due to current in long straight


wire is

2x5
B = = 10-2
4ti r 0-10
2tcx9
B X
acting vertically
= 1 X 10"^ T, acting upwards, ^ 4tc r 47t 3r
according to Right Hand Thumb Rule. upwards.
6. Force on charged particle due to electric field, The magnetic field at O due to current in loop P
q E, acting along .^-axis. will neutralisethe magneticfield at O due to current
4/78 ‘Pta<Uefr 'a Fundamental Physics (XII)
in loop Q if the direction of current in F is 11. Refer to Fig. 4{Q).17, /= 15 A, r = 2-5 m.
clockwise. If Y is the current in loop F, then The magnetic field at point F is

^ ^ Po InV acting vertically downwards B =


^0 2/
^ 4tc 2r A% r upwards
The resultant magnetic field at O will be zero 2x15
= iO-"^ X

Itp ,, 27Ex9 ^ Itp 2%r


if Bq = Bp or 4ti
2-5
3r 4;t 2r
= -xlO-^T. vertically upwards
or r = 6A 5

oww
9. For a point F above the surface of wire, the distance = 1*2 jO.T, vertically upwards
of point F from the axis of wire,
a 3a FIGURE 4(Q).17
r=a+—= —
2 2 N

ee
Magnetic field at F due to current / in the wire,

FFrlo
r
B 2/^M /= 15A'-

rF
ee
^ 47C r 2nr
fV- E
*
Hq/ _Rq/ 2.5m ^

rF
ouru
iTiOall) '3% a
For a point Q, below the surface of wire, the
ffosor
os k
distance of point Q, from the axis of wire S I
/ - all = all
ook
Yo
Magnetic field at Q due to current 1 in the wire.
Y
12. In a current carrying solenoid, the magnetic field
Bo

_ Po 21/ Pq/(fl/2) Pp/ B is along the axis of solenoid, i.e., in the direction
reeB

47t 27tfl^ 47Cfl


of motion of electron./.e., angle between B and
oouY

Bp _ PQ//3itfl _ 4
ur

V IS zero.
ad

B
Q PQ//47ta 3 Force on electron while moving along the axis of
the solenoid is
dY

10. Let / be the unit vector out of the plane of the F = qv Bs,\n(f =0
nidn

paper. Magnetic field at M due to current in the Therefore, the electron will continue moving with
Re

loop will be
velocity ^ along the axis of the solenoid. Thus
FFi

^ ~ ^AB'^ ^BC'^ ^CD option id) is true.


13. Let q be the charge on the particle moving
p,) / 3ti a perpendicular to magnetic field and r be the radius
.-. B = — )+0
4ti F 2 of the circular path of the charged particle traced
in the magnetic field. Then
71 2

x-^(-/) + 0
mv
4n IR F ^ u sin 90® = ...d)
r

When speed of particle becomes u/2, then


F Ls I6j R 16 V m(v/2)2
F^-sin90“= ...(2)
r

into the plane of the paper. Dividing (1) by (2) we get


R 16
2 = 4//r or F = r/2
MOVING CHARGES AND MAGNETISM 4/79

14. Force, F = q(vxB) lipNiIj |1q(W2)x2/


I “
/ /

= l-6xl0"‘^ [(4i+3it)x(3ik+4/)]
= B
/
= 1-6x10“'^ [4x3(/ xA:) + 4x4(/ X /)

+3x3(ikxA:)-i-3x4(ikx/)]
18. Current sensitivity, =j =
For galvanomter G|,
= l-6x 10-‘9 [12(-y) +16x0 +9x0+12(7)]
= l-6x l(r‘9x0 = 0
/ ^ 30 X 0-25 X (3-6x10-3)
k k

15. The current will be divided equally in two halves For galvanometer G2,
of a loop if there is equal resistance of two halves

ww
of a loop. It will be so in case of loop P and R. ^ ^2^2^ _ 42 X 0-50 X (1-8x10-3)
"2 k ~ k
Now magnetic field at the centre of loop will be

FF loo
zero as the magnetic field at the centre of loop due ^^1 _ 30 x 0-25 x 3 6x10-3 30 5
to current in upper half of loop will be equal and
42 X 0-50 X1-8 X10-3 = ^ = 7

ee
opposite to the magnetic field at the centre of the
loop due to lower half of the loop. Thus option (a) 19. When a current loop is suspended in a uniform

er
Fr reF
is true. magnetic field, it will experience a torque but the
16. Let / be the total length of the circular coil and r be resultant force on the loop is zero. The loop will
rur
the radius of circular coil. Magnetic field at the come to rest with its plane perpendicular to
centre of circular coil due to current I is fforo
magnetic field, when the force on the remaining
ks
2rt/ three arms of the loop is -F.
YYouo
B.0 “ -0)
20. Here, /, = 4-0 A; /j = 10-0 A ;
okso

47c r 4jc H2n


r = 2-5 cm = 2-5 x 10“2 m.
BBoo

[v/ = 2icr]
r ee

Force per unit length, F =


When the coil is rewound to have three turns let, 47t r
Tj be the new radius of circular coil,
10-“^ X2x4-0x10-0
ad
oouur

then, / = 3 X 2 Jt rj or r j = //6 Jt = 3-2 X 1(H Nm-»


2-5x10-2
Y

Magnetic field at the centre of current carrying this


coil is 21. Ammeter can be converted into a voltmeter by
dnYd

using a high resistance in series of ammeter.


Re

B =
^0 2tc/x3 Itp 27t/x3 22. Magnetic field at the centre of current carrying
FFini

4jc 4ti //6jc circular loop is


_ Vip 2nl
A1 = -(/)
= 9x
\^p 2kI 4k R
4it 112k = 9B.
Magnetic field at a point on the axis of a circular
loop is
17. Magnetic field at a point on the axis well inside
the solenoid carrying current is -i^O 2tc//?2
^2 =
4tC (/?2+/f2)3/2
B =
I
\lp 2kIRP-
where N = no. of turns of the solenoid and / is the 4k R^ X (2)3/2
length of solenoid.
When, Wj = NI2 and /j = 2 /, then ^ 1^0 2kI
...(/I)
4k /?x (2)3/2
4/80 "pn^xdee^ ^ Fundamental Physics (XII) kTOTI
B1 27. Refer to three infinite long parallel straight wires
= (2)3^2 ^ 2-s^ carrying current shown in Fig. 4(Q). 18(a)
^2 The force per unit length on wire C due to current
23. Force experienced by a current carrying wire when in wire A is
placed in magnetic field is
E1 =
\iQ 2/x/
F = Bll sin 0 towards left hand side
4k 2r
where 0 is the angle which the wire makes with
the direction of magnetic field. The force will be The force per unit length on wire C due to current
in wire B is
maximum if sin 0 = 1 or 0 = 90®. i.e., the wire is
kept perpendicular to the magnetic field. IXI
towards left hand side
24. There will be no tension in the strings, if the force 4k r
on current carrying conducting rod due to magnetic

ww
field is equal and opposite to the weight of the rod.
FIGURE 4(Q).16
mg A B C
i.e., Bll ~ mg or / = r->
IB

Flo
e
25. Here !v^ - -■ r Ir = ?

eree
4' P -o 21" /> /-●
2/>. /i. /i.

FFr
- m / r
P P P or ri, = mpViJBqp 4
El
uurr A B C
and Bq^v^=m^vl!r^ or r^^m^v^lBq

orr
a
(fl) ih)
r
p _
m V sfo
kks
Yoo
r
a
m
a
V
a Net force on C per unit length
oooo

1 9 2 9 Ho
=-x-x-=- = F,+F2- — (1 + 1)
eBB

4 4 1 8 47t r

./ . 20 . _ , « r
26. New current sensitivity, ~ = 2x (Given)
uurr

4n r
When resistance of galvanometer is increased by
ad

Refer to Fig. 4(Q).18(/?),


Yo

25%, then
dY

Repulsive force per unit length on wire C due to


25 wire A is
G' = G + G = 1'25G
innd

100
Re

New voltage sensitivity, E'I = Ho 21 XI Up


4ji 2r 4k r
Fi

V' = K _ 1-2/. _120 V


24
Attractive force per unit length on wire C due to
" G' 1-25G 125 " 25 wire B is

% change = ^^^'^^xl00 ^2 =
_Hq 7x7
4ti r
lip
4k r

24
V -V xlOO Net force on wire C = Fj' -
_ X 25 " s

V
5
4k r 4k r
(24-25 2B. e E=ev B or u = E/B
XlOO
^ 25
1
K
29. Kinetic energy, K =
OC
1 or
xIOO =-4% 2 m m
25
MOVING CHARGES AND MAGNETISM 4/81

m
4 _ 2m,
or
B2 = 48j=4B
= 2
m m 2nnl ^o 27cn/
36. B =
4n(fl2 + r2)3/2“4jc
p p
or
Ap = 2^:j = 2 x 50 = 100keV
[As a^'¥p-»P’]
mt;2
30. qvB = — or mv = qvr or nwocr
B oc ~
r

As > rg, so > ntg Vg yj. Here, n = 1000m->; / = 5 A;


31. Here,/, =2A;/2=1 A,/ = 015m;
B = [Iq " ^ = ^ ^ 1000 X 5
r, = 0 02 m ; T2 = 0 02 + 0-10 = 012 m = 2jcx1(Ht
Force on arm AB of coil, FAB 38. Here, 7| = 10 A ; /2 = 2 A, ^2 = 2 m; r = 0-10 m
4n
towards the wire
F =
Ho 2/|/^t; 2x10x2x2
Force on arm CD of coil, 4jt r 0-10

ww
Rn 2/,/- . = 8x1(Hn
^CD = T^— away from the wire
4re Tj M’O I 1
39. B =

Floo
Boe-
Net force on the coil = F^g - F^/j, towards the wire 4n r r

1 1

ee
B
4jc 2/./i' L'i7-,"iJ £i=i=j: 1
or B- = —1-
B1 ^ 2
rj 2r 2

ee r
rFrF
1 1 nBA
= 10-2x2x2x 1 x015 40. Voltage sensitivity, =
La02 012J JfeG
rur
» 25 X10"'^ N otwards the wire
32. L = 4/ = 2jiror/ = Jir/2
ffoor
Current sensitivity, / =
^
ti

k
.H = l
ks
/. G
YYoou
^1 _ /^x/ _ /2 n^r^lA 41. Here, n = 100,/= 2 A;
ookos

M2 nr^xl nr^ nr^ 4 B = 0-2 Wb/m2, A = 008 x 010 m2


BBo

x = n/AB sin 90“= 100 X 2 X (008 x 010) x 0-2 x 1


= 0‘32Nm
re

1
33. Current sensitivity, I * = —^
k - ^“7 The direction of force on arm AC is peipendicular
oouur

to the arm AC upwards. The direction of force on


ad

As otrsional constant k decrease, /, increases.


arm BD is perpendicular to the arm BD downwards.
Y

1 As a ersult of it, the coil will tend to rotate with


34. B = or Bee-
4Jt r side AC moving out of the page,
nd dY

r
Re

42. Time period, T = it is independent of speed


●● £i = i = i i
Bq
FFini

20 4
^ ’2 of the particle.

B, B II. Assertion-Reason lype Questions


or Bn = “!● = “
2 4 4 43. Magnetic moment, M = IA = InP’ where r is the
35. Let r,, r2 be the radius of the circular loop of one radius of the current carrying loop, i.e„ M ec
When / = 2 r, then
turn and 2 turns of the given wire. Then
2 ic r, = 2 X 2 Jt T2 or r, = 2 r2 M'
M'ec {2 r)2 = 4 otMsAM
M
B =
Itp 2nln or B oc -
n
Thus both, Assertion and Reason are true and
An r r
Reason is the correct explanation of Assertion.
44. An ammeter of higher range should have very low
Bn = _2
-2. ru x-L
.r = -X
2 —2.
2r, = 4^ resistance. For an ideal ammeter the resistance must
1
B, «, rj ^2 be zero. But in practice, it is impossible to have
4/82 Fundamental Physics (XII) VOL.I

zero resistance. Therefore, to increase the range of Assertion and Reason are true and Reason is
ammeter, the additional shunt is added in parallel the correct explanation of Assertion.
to it, which reduces the resistance of ammeter. 52. Both i^ertion and Reason are \f?rong as energy
Hence Assertion is true but Reason is false. of a charged particle accelerated by cyclotron is
●- —> ^ ^ inversely proportional to mass of the particle.
45. Force on a particle, F = q{vxB), where F is 53. Both Assertion and Reason are true and Reason
perpendicular to both v as well as B. is the correct explanation of Assertion.
54. Magnetic field at a point due to linear wire carrying
Wprkdone for this force = F . ? = F j cos 90" = 0.
Hence, both Assertion and Reason are true and current at a distance r is, 5 = ^1, i.e., B « 1/r.
2k r
Reason is the correct explanation of Assertion.
46. When a moving charged particle enters a When r is constant, B remains unchanged. Thus
Both Assertion and Reason are wrong.

ooww
perpendicular magnetic field, then the force on
particle will make the particle to move on a circular 55. Both Assertion and Reason are false as the
path of radius r, given by magnetic field at a point on the axis of current
carrying wire is zero.
_ nnP- mv ^
Bq V = — or r = 56. Magnetic field at point P due to currents in wires
Bq Bq along X and y axes respectively will be

e
r

ree
[v mu = p = momentum of particle]

rFl
Since p and q are same for proton and electron so 1‘ = 4jc and

Fre
r is s^e, i.e., Assertion is true. Here Reason is
y ^ 4n X

rrF
also true but Reason is not the correct The resultant magnetic field at P will be zero if
explanation of Assertion. —> —>
i_i = 0
B^ + B2 =0, f.e.. 4tc y
ouur
47. The direction of force on wire carrying current in
a perpendicular magnetic field will be towards sffoo
west, according to Fleming’s left haind Rule, which
y x_

This is possible ifJ--i = 0ory = x


okks
will be applicable here. Thus, both Assertion and
YYo
y X
Reason are false.
ooo

Thus both Assertion and Reason are correct and


BB

ft l> A
the Reason is the correct explanation of
48. Current sensitivity, ^ Le.,.Ig oc n '
Assertion.
rr e

nBA_I, 57. For a quadrant of radius r, 0 = Jt/2 so


Voltage sensitivity, =
ouu

k<R R
ad
Y

B = . Thus both Assertion and


When we increase current sensitivity by increasing 4itr 2” Sr
dY

the number of turns, then the resistance of Reason are true and Reason is the correct
galvanometer coil also increases. But n/R increases
explanation of Assertion.
innd
Re

by different amounts depending upon the yalues 58. Both Assertion and Reason are true but Reason
of n and F. Due to it, voltage sensitivity may
is not the correct explanation of Assertion.
FFi

increase or decrease. Thus Assertion is true. Here


Reason is also true and is correct explanation 59. Both Assertion and Reason are wrong because
of Assertion. torque on the current loop in the magnetic field
—>
49. Here, Assertion is true but Reason is false, = FA/sin 0, where 0 is the angle between B and
because ammeter with increased range has low —>

resistance since a low resistance shunt is used in A . In the given Assertion, 0 = 90®, so torque on
the coil is = F Jt / sin 90® = F Ji / and not
parallel to increase the range of anuneter.
zero.
50. A charge in a uniform motion produces a steady
current, to which both the . electric and magnetic 60. Force on particle in magnetic field
fields are associated. Thus, both Assertion and mv
2
mv _ p
Reason are false. F =Bqv = or F =
F Bq Bq
51. Force, F = q(vxB). As angle between So, radius F is constant as p, F, q are same for
both electron and proton. Hence both Assertion
V and F is zero, so I FI =0. Thus, both and Reason are wrong.
4/83
MOVING CHARGES AND MAGNETISM

I K MS

I. Biot Savart’s law and magnetic field Sol. Here current element / dl is along z-direction
due to a linear current carrying wire
and position vector 'r is along }»-direction. So
1. How Is a magnetic field produced ? Idl = Idlk and r = rj

Sol. A magnetic field can be produced by any of the According to Biot Savart’s law, the magnetic
following ways : (i) by a magnet (//) by a current field at point P due to current element is
carrying conductor (m) by a moving charge and A A

(iv) by changing electric field. Ito^ {d]k.yr j\


2. How will you identify whether the magnetic

ww
4?t 4it
field at a point is due to the earth or due to
some current carrying conductor ?
Sol. We know that a freely suspended magnetic 4ti ^i-i)
needle always points to north-south direction

Flo
due to earth’s magnetic field. If a freely Hence, magnetic field at point P is directed

e
along negative x-direction.
suspended magnetic needle always rests in

ree
north-south direction, the magnetic field is due 5. Compare the electrostatic field given by

Fr
to earth, and if this magnetic needle turns to Coulomb’s law and magnetic field given by

rF
other direction and returns back to north- Biot Savart’s law.
some
uurr
south when the current is switched off, then the Sol. 1. Both the fields obey inverse square law, hence
magnetic field is due to a current carrying
conductor. for
both are of long range.
2. The electrostatic field is produced by an
s
kks
3. The net charge in a current-carrying electric charge which is a scalar. The magnetic
Yo
oooo

conductor is zero, even then it experiences a


force in a magnetic field. Why ? field is produced by a current element I dl ,
eB

which is a vector.
Sol. We know that a magnetic force acts on a charge
moving in a magnetic field. In a conductor 3. The electrostatic field due to a charge acts
carrying current, the free electrons move along or opposite to the line joining the source
r

towards the positive end of the conductor with of charge and the field point, depending upon
ou
ad

some drift velocity, hence magnetic force acts whether the field is due to positive charge or
YY

on them in a magnetic field. The positive ions negative charge.


of the conductor being stationary, do not
The magnetic field at a point due to current
nndd

experience any magnetic force.


Re

—^

4. A current / flows in a conductor placed element I dl is perpendicular to the plane


perpendicular to the plane of the paper.
Fi

Indicate the direction of the magnetic field


containing the displacement vector "r and the
current element I dl .
due to a small element dl at point P situated
4. The electrostatic field is linearly related with
at a distance r from the element as shown
the source of charge and magnetic field is
in the Fig. 4(Q).19. (CBSE 2009)
linearly related with source of magnetic field
FIGURE 4(Q).19 l.e. I dl , hence both the fields obey
superposition principle.
z*

5. The electrostatic field at a point due to a point


W/ charge is angle independent whereas the
p
♦y magnetic field at a point due to current element
depends on the angle between I dl and r .
4/84
Fundamental Physics (XII) VOL.J

6. Electrostatic field at a point due to source


charge decreases when a medium is inserted B = ^^0 InlA tdp IM
between the source charge and given point; as [From (18)]
4n 4tc
1
£ oc
where, n/A = M = Magnetic dipole moment of
the current loop.
The magnetic field at a point due to current Thus the current loop can be regarded as a
element increases when a medium is inserted
magnetic dipole which produces its magnetic
between the current element and the given field and magnetic dipole moment of the
point; as B MoMr current loop is equal to the product of
OC

6. A closed circuit is in the form of a regular ampere turns and area of current loop. SI
hexagon of side r. If the circuit carries current unit of magnetic dipole moment is A-m^.
/, what is the magnetic field induction at the 8. What is the magnetic moment of an electron
centre of the hexagon ? orbiting in a circular orbit of radius r with a

ww
Sol. Refer to Fig. 4(Q).20, AG = GB = HI speed V ?

OG = AG cox 30“ =
Sol. We know that orbiting electron behaves as a

current loop. The current due to orbital motion

Floo
of electron is

/=^
e ev

ee
T Inrlv 2nr

reer
Area of the current loop = n r

rFF
Magnetic moment of current loop
uur r
ev evr
= IA = xnr^ =
ffoor 2nr

9. A long wire carries a steady current, /. First


2
sks
it is bent into a circular coil of one turn when
YYoo
Magnetic field induction at 0 due to current in
ooko

the magnetic field induction at the centre is


arm AB is
Bq. Then the same wire is bent to form a
eBB

I circular coil of smaller radius but n turns


[sin30“+sin30“] then find the magnetic field induction at the
‘ 4;t^.V3/2) centre of this circular coil.
r

Sol. Let R be the radius of one turn circular coil.


ouur
ad

- Mq/
2jirV3 ’ perpendicular to the Then, _ ^0 27T/
Y

B0 ~
4k R
plane of closed circuit upwards.
nd dY

If r is the radius of circular coil of n turns, then


Re

The magnetic field induction at O due to currents


2Krn = 2nR or r = R/n
in arms BC, CD, DE, EF and FA is also equal to
FFiin

and acting in same direction. Therefore, total So, B = [^0 2nnl _ ^ 2kn I
magnetic field induction at O due to current in 4k r 4 k iWn)
closed current is
-
1^0 2tc/ .
/r

B = bB,1 =6x 4jt R


2nr^ Tzr -n^B 0
10. Magnetic field at the centre of a circular loop
II. Magnetic field due to a of area A carrying current / is B. What is the
circular coil carrying current magnetic moment of this loop ?
Sol. Let R be the radius of the circular loop. Then.
7. What do you understand by magnetic dipole A=kR^
moment of the current loop.
Sol. We know that magnetic field induction at any
or
R = ^^A/K
point on the axis of a circular coil of n turns Magnetic field induction at the centre of circular
each of area A, carrying current / is given by loop carrying current is
4/85
MOVING CHARGES AND MAGNETISM

^0 2tc/
4n R
IBR
or / =
^^0
2BR A, 2BA A
M^IA = =
^lo yn
^lo
11. Two small circular loops, marked (1) and (2),
carrying equal currents are placed with the

oww
geometrical axes perpendicular to each other
as shown in Fig. 4(Q).21. Find the magnitude Sol. Let /j and I2 be the lengths of the two circular
and direction of the net magnetic field arcs ACB and ADB respectively, p be the
produced at the point O. resistance per unit length of the circular

e
conductor.
FIGURE 4(Q).21

re
Resistance of circular arc ACB, = /jp
A-.-

FFrllo
—► o
Resistance of circular arc ADB, R2 - /2P

rF
“G f
Let Ii, I2 be the currents in arms ACB and ADB

ee
vy 1 respectively. As these two arcs are connected in
ouru parallel, so the potential difference across them

rF
must be equal i.e.,

fosor
os kf l\R\ ~ h^2
I
or /j/ip = /2^2P
or I\l\ — ...(0
ook
Sol. Magnetic field induction at O due to current
YYo
Magnetic field induction at O due to current in
loop 1 is arc ACB is.
Bo
reeB

B,1 =
2(x2+/?2)3/2’ 4jt r *
oouY

4n r r 4n
ur

acting towards left. It is acting upwards, perpendicular to the plane


ad

Magnetic field induction at O due to current loop of circular conductor.


2 is
Magnetic field induction at O due to current in
Yd

arc ADB is.


nidn

»2 =
2(x2+/?2)3/2
Re

acting vertically upwards. 4n r ^ 4n


FFi

Resultant magnetic field induction at O will be It is acting downwards, perpendicular to the


plane of circular conductor.
B = ^B} + bI =^B, (vBi = fi2) As 7| /j = I2 /2.

= V2x so fij B\=-Bi


2(x2+/f2)3/2 —> —>

.*. Resultant magnetic field at O = + S2


= (-Si)+S2 =0

12. As shown in Fig. 4(Q).22 a cell is connected 13. The circular loop PQRSTP formed by two
across two points A and of a uniform circular segments of radii If1 and Rit carries
circular conductor of radius r. Prove that the a current I. What is the magnetic field
induction at the centre O ? What will be the
magnetic field induction at its centre O will
be zero. magnetic field induction at O if a = 90® ?
4/86
'4. Fundamental Physics (XII) VQL.I

15. The magnetic field at a point near the


centre but outside a current carrying long
solenoid is zero. Explain why ?
Sol. When current is passed through a long straight
solenoid, then each turn of the solenoid can

be regarded as a circular loop carrying current


and thus will be producing a magnetic field.
The total magnetic field at a point is the vector
sum of the magnetic fields due to currents
Sol. Let n be the unit vector perpendicular to the through all the turns in the solenoid. At a point
near the centre outside the solenoid, the
plane of loop downwards. Total magnetic field
induction at 0 due to current through circular magnetic fields due to neighbouring loops are
equal in magnitude and opposite in direction.

ww
loop is
Hence the resultant magnetic field at a point
PQ STP near the centre outside the solenoid is zero.

FF loo
16. A hollow cylindrical conductor of inner
= 0+
^—
/?2
an + 0+^ — (27c-a)«
An R
radius a and outer radius b carries a current
I
I uniformly spread over its cross-section.

ree
_c^^(27i-a) A Find the magnetic field induction at a point
n
Inside the body of the conductor at a distance

rree F
An
l_/?2 /?.
r [where a < r < b] from the axis of the
a , (27c-a) cylinder.
ur
ov B =
4tc
L^2 R
I
Directed downwards
fofr oF
Sol. TTie Fig. 4(Q).24, shows the transverse cross-
section of hollow cylindrical conductor. Let P
be a point at distance r from the axis of the
ks
If a = 90° = nJ2 rad., then
YYouro
conductor well inside the conductor. Draw an
soo

imaginary cylinder of radius r whose axis


B = Kq/ n/2 ^ 3n/2 - 8
—+— coincing with the axis of hollow conductor.
An
1^2 L«2R
BBook

R 1
1
r ee

III. Ampere’s circuital law,


solenoid and magnetic field
oouru
ad

due to current in conducting rod


Y

14. Distinguish between Biot Savart’s law and


Ampere’s circuital law.
Ynd

Sol.
Re

S.
FFindi

Biot- Ampere’s
No. Savart's law circuital law

1. This law is based 1. This law is based


Current passing through the cylindrical shell of
inner radius a and outer radius r is
on the principle on the principle of
of magnetism. /(r^ -a^)
electromagnetism. = —TF—2T
2. This law is valid for 2. This law is valid for
K(b^ ~a^) ^ =
—^
asymmetrical current symmetrical current Let B be the magnetic field induction at P.
distribution. distributions. Using Ampere’s circuital law.
3. This law is the differ 3. This law is the
ential form of magnetic integral form of
field induction B or
B or H- or
B2nr = \i^^
magnetising force H .
4/87
MOVING CHARGES AND MAGNETISM

mv mv
or B = (iv) If = nB, then r' =
2Tt(b^ -a^)r eB' neB

IV. Force on charged particle 1 r


or r
due to electric and magnetic n n

fields and cyclotron


18. Describe qualitatively the path of a charged
17. A fine pencil of p-particles, moving with a particle moving in a region with uniform
speed V, enters a region (region I)> where a electrosatic and magnetic fields parallel to
uniform electric field and a uniform magnetic each other, with initial velocity (i) parallel
field are both present. These p-particles then (h) perpendicular (Hi) at an arbitrary angle

oww
move into region II where only the magnetic with the common direction of the fields.
field, (out of the two fields present in region I) Sol. (0 When a charged particle moves parallel to
exists. The path of the P-particles, in the two both electrostatic and magnetic fields then the
regions is as shown in the Fig. 4(Q).25 ; force on charged particle due to magnetic field
but due to electric field, the force on the

e
IS zero
FIGURE 4(Q).2S
charged particle will act along the direction of

re
FFrlo
+ + + + +
field and the charged particle will continue to
move along the original straight line path.

rF
ee
P
II (ii) When a charged particle moves
perpendicular to both electric and magnetic

rF
ouru
I fields, then due to electric field, the charged
particle describes a parabolic path and due to
(i) state the direction of magnetic field.
fosor magnetic field, it will describe a circular path.
skf
(«) state the relation between E and B in As a result of it, the particle will follow a
ooko
region I. resultant path in electric and magnetic fields.
YYo
(i«) Drive the expression for the radius of the If electric and magnetic fields are perpendicular
Bo

circular path of the p-particle in region II. to each other as well as perpendicular to the
reeB

(iv) If the magnitude of magnetic field, in motion of charged particle, the particle will
region II is changed to n times its earlier continue moving along the same path if force
particle due to electric field is equal and
ooY

value, (without changing the magnetic field on


uur

in region I) find the factor by which the opposite to the force on it due to magnetic field.
ad

radius of this circular path would change.


dY

(Hi) If we resolve v into two rectangular


(CBSE Sample Paper 2013) -» —>

components; along B and perpendicular to B.


nind

Sol. (i) As the p-particle goes undeviated while


Re

passing through the electric and magnetic fields, Then due to component velocity perpendicular
FFi

so the force on moving P-particle due to electric


to B, the particle will describe a circular path
field (-e E) must be equal and opposite to the
—* and due to component velocity along B ot
force due to magnetic field [~^ (v x B)]. Due
it gets accelerated or retarded due to E, along
to it, the direction of B must be
the same path.
perpendicular to plane of paper downwards. 19. A charged particle q moving in a straight line
(ii) In region I as (5-particle goes undeviated is accelerated by a potential difference V. It
enters a uniform magnetic field B perpendi
-eE = -e(vx B) or eE = evB or v-EIB cular to its path. Deduce in terms of V an
(Hi) If r is the radius of circular path of the (5- expression for the radius of the circular path
in which it travels. (CBSE 2000)
particle in region II, then
mv Sol. When charged particle is accelerated through a
evB - mv^r or r =
potential diff. V, then work done = gain in K.E.
eB
4/88
Fundamental Physics (XII) VOL.I

22. In what ways electric and magnetic fields are


or or v =
/ 2qV
m
different ?

Sol. (0 Electric field is due to charges at rest as well


The force on charged particle due to as in motion, whereas magnetic field is due to a
perpendicular magnetic field (=qvB) provides magnet or cuirent through a conductor or by a
the centripetal force (= mv^/r) so moving charge.
mv
2
(//) The strength of electric field at a point
qvB = decreases with the dielectric medium but the
strength of magnetic field increases when a

oww
permeable medium is inserted there,

i¥-J qB^
mv m 2mV
r =
or
qB qB (rii) The electric lines of force representing the
electric field do not form closed paths whereas
20. Both, the electric and magnetic fields the magnetic lines of force form closed paths.

e
can

deflect a moving electron. What is the 23. A neutron, a proton an electron and an a-

re
difference between these deflections ?

FFrllo
particle enter a region of constant mngn^tlf
Sol. In a perpendicular electric field, the force acts field with equal velocities. The magnetic field

rF
ee
on the electron in the opposite direction of is along the inward normal to the plane of
electric field. Hence the deflection is in the plane paper. The tracks of the particles are shown
ouru
rF
of electric field. In this case, the speed of the in Fig. 4(Q).26. Relate the tracks to the
electron changes and hence kinetic energy of particles.

fosor
the moving electron changes in the electric field.
os kf
In a perpendicular magnetic field, the direction
of force exerted on a moving electron is
ook
Yo
Y
perpendicular to the plane of its direction of
motion as well as of magnetic field. Hence the
Bo
reeB

deflection of the electron beam is perpendicular


to the plane of the magnetic field. In this case,
oouY

the work done by the force due to magnetic field


ur

acting on the moving electron is zero. Therefore,


ad

Sol. We know that force on a charged particle in the


the kinetic energy of the moving electron does magnetic field is
Yd

not change in the magnetic field.


nidn

^ ^

21. An electron beam is deflected in a given field. \ F \ — q \ vx B \ orF=(fuBsin0, so


Re

How will you detect whether the given field


(0 For neutral particle Le^ neutron, ^ = 0, hence
FFi

is a uniform magnetic field or a uniform


electric field ? F = 0. It means neutron will go undeflected Le.
track C corresponds to neutron.
Sol. We can detect the nature of field by observing
the trajectory and measuring the kinetic energy (I’O For negatively charged particle i.e. electron,
of electron. In a uniform electric field, the path the direction of force, according to Fleming’s
of electron beam moving perpendicular to field Left hand rule will be towards irght. So track
will be a parabola and the kinetic energy (and D corresponds to electron.
hence the speed) of the electron changes. In a («0 For positively charged particle, the direction
uniform magnetic field, the path of electron of force, according to Fleming’s left hand rule
beam perpendicular to field will be circular and will be towards left. So both tracks A and B
kinetic energy (and hence the speed) of the correspond to positively charged particles (i.e.
electron does not change. protons and a-particles).
4/89
MOVING CHARGES AND MAGNETISM

When a moving charged particle is subjected to 25. A charged particle enters a (non-uniform)
a perpendicular magnetic field, it describes a magnetic field varying from point to point,
circular path of radius r given by both in magnitude and direction, with a
certain initial velocity. What do you say about
m
r -
mV
or r ec — the final velocity of the particle when it leaves
Bq the field ?

Sol. The force on the moving charged particle in


m (2e 1

magnetic field is F = q(vxB). So F is


X X
m 4m e 2
^0. a

always perpendicular to the particle velocity v .


or r„ = 2r^ i.e.. a P
Hence the magnitude of velocity remains
i.e., track B corresponds to a-particle and
unchanged, only the direction changes. Thus the
track A to proton.
particle will leave the varying magnetic field

ww
24. Difl’erentiate between electric and magnetic with its initial velocity, but in a different
forces.
direction.
Sol.

FF loo
S.No. Electric force Magnetic force V. Force on current carrying wire in
magnetic field and force between two

ee
1. No magnetic force acts
1. Electric force acts on
parallel conductors carrying currents

er
a charged particle at on a charged particle at

rFreF
rest or in motion in rest in a magnetic field. 26. A straight horizontal conducting rod of
any direction in Magnetic force acts on length I and mass m is suspended by two
rur
electric field. a moving charged vertical wires at its ends. A current / is set up
particle only if it is not
moving parallel to the
ffoor
in the rod through the wires.
ks
direction of magnetic (i) How much magnetic field should be set
up normal to the conductor in order that the
YYoou
okso

field.
tension in the wires is zero ?
BBoo

2. Electric force always 2. Magnetic force always


acts on the charged acts in a direction (k) What w ill be the total tension in the wires
r ee

particle in the direc perpendicular to the if the direction of current is reversed, keeping
tion of electric field. direction of magnetic the magnetic field same as before ?
oouur
ad

field as well as the


direction of motion of Sol. (i) Tension in the supporting wires will be zero
Y

charged particle. if the weight of the rod acting vertically


downwards is balanced by the upward force on
3. The electric force on 3. The magnetic force on
dnYd
Re

a charged particle is, rod due to magnetic field, i.e..


a charged particle is,
FFini

—>
mg
F - q E , It depends F = q(vx B). It mg = IIB or B =
II
upon the charge on depends upon the
the particle and is charge and velocity of (ii) If the direction of current is reversed, the
independent of the particle. magnetic force on rod will act in the vertical
velocity of particle. downward direction. Now total tension in the
wires will be
4. Work is required to be 4. No work is done by the
done in displacing the magnetic field in T = mg + IIB = mg + mg =2 mg = 2 IIB
charged particle in an displacing the charged
electric field. particle because the 27. Two parallel wires carrying current in the
force and the displace same direction attract each other while two
ment are perpendicular beams of electrons travelling in the same
to each other. direction repel each other. Why ?
4/90
<t Fundamental Physics (XII) VOL.I

Sol. Two parallel wires carrying currents in the same velocity of the wire as a function of time,
direction attract each other due to magnetic assuming it to be at rest initially.
interaction between two wires carrying currents
because the current in a wire produces a FIGURE 4(Q),27
magnetic field and the magnetic interaction is I
of attractive nature when current in the two 2?
parallel wires is in the same direction.
d
The two beams of electrons travelling in the 1

same direction will be a source of both an

electric and magnetic fields. Due to magnetic I


interaction, there will be force of attraction
between the two moving electrons but due to Sol. The magnetic force on the wire of length d due
electrostaticinteraction,there will be a force of to magnetic field is F = I Bd sin 90° = IBd

oww
repulsion between them. If the beams of This force, according to Left Hand Rule, acts
electrons are moving slowly, the eletroslatic perpendicular to the wire in the plane of paper
force of repulsion between the electrons towards left hand side. Due to it, the wire moves
dominates the magnetic attraction between towards left with the acceleration.
them.

e
F IBd
28. A loosely wound helix made of stiff metal wire a - —

ree
rFl
is mounted vertically with the lower end just tn m

Fre
touching the mercury in a dish. When the Velocity of the wire at a time t will be
current from a battery is started in the metal

rr F
IBd
v = u + al = 0 +
wire through the mercury, the wire executes m
t

oscillatory motion with the lower end


ouur
jumping out of and into the mercury.
Explain.
or sfoo
V =
IBd

m
t towards left
kks
Sol. The windings of the metal helix carry currents
Yo
oooo

in the same direction, therefore they experience VI. Torque on coil in


attractive forces, pulling the lower end of metal magnetic field and galvanometer
eBB

wire out of mercury. When it happens, the circuit


breaks, the current disappears and hence the 31. A current carrying circular loop lies on a

smooth horizontal plane. Can a uniform


force of attraction disappears. As a result of it,
urr

magnetic field be set up in such a manner


the wire helix regains its original position, that the loop turns around itself (/.e., turns
You
ad

completing the circuit again. The process is about the vertical axis). (CBSE 2013)
repeated time and again. Due to it, the oscillatory
dY

motion is set up. Sol. No, because the loop can turn around itself if
the torque x is acting in the vertical direction.
29. Why should a solenoid tend to contract when
innd
Re

a current passes through it ? Since, torque, 'x~{MxB)=liA.yB) and


Sol. We know that two parallel conductors carrying
Fi

currents, in the same direction attract each other Af or A of the horizontal loop is in the vertical
and in opposite directions repel each other. direction, so the direction of t will be in the
Therefore, when current is passed through the plane of the loop for any direction of B .
coil of a solenoid, the parallel currents in the
various turns of solenoid flow in the same 32. A current carrying circular loop is located
direction. As a result of it, the various turns start in a uniform external magnetic field. If the
loop is free to turn, what is its orientation of
attracting one another and solenoid tends to
contract.
stable equilibrium ? Show that in this
30. A metal wire of mass m slides without friction
orientation, the flux of the total field (external
field + field produced by the loop) is
on two horizontal rails spaced distance maximum.
d-apart as shown in Fig.4(Q).27. The rails are
(CBSE 2013) (NCERT Solved Example)
situated in a uniform magnetic field
directed vertically upwards, and a battery Is Sol. The current carrying circular loop behaves as a
—)
sending a current / through them. Find the
magnetic dipole of magnetic moment M acting
MOVING CHARGES AND MAGNETISM 4/91

perpendicular to its plane. The torque on the iJlB .


sincr
current loop of magnetic dipole moment M in Ann
the magnetic field B is
Torque will be maximum if n is least (= 1) and
X = MB sin a = /A X S sin a, (V M = A[)
sin a has maximum value i.e., sin a = I
where a is the angle between M and B . The Thus torque will be maximum when n = 1
system will be in stable equilibrium if torque is
Maximum value of Torque, x max.
zero, which is so if a = 0". This is possible if —
An

B is parallel to A i.e. B is perpendicular to 35. A rectangular current carrying loop PQRS

oww
the plane of the loop. In this orientation, the is kept in a uniform magnetic field as shown
magnetic field produced by the loop is in the in the Fig. 4(Q).29.
same direction as that of external field, both
FIGURE 4(Q).29
normal to the plane of loop. It is due to this
fact, the magnetic flux due to total field is P Q

e
maximum.

re
N S
33. Three wires each of length 2-0 m are bent

FFrllo
into different rectangular loops and then R

reF
suspended in a magnetic field. Fig. 4(Q).28.
If the current in each of them be the same,

e
which loop shall be acted upon by largest {a) What is the direction of magnetic moment
uoru
torque ? If any of the wires be bent into of the current loop ?

sFr
circular loop, then ? {b) When is the torque acting on the loop

kforo
FIGURE 4(Q).28 (/) maximum, (ii) zero ? (CBSE 2005, 2009)
Sol. {a) Magnetic moment of the current loop,
sof
M = lA . Its direction is normal to the loop
ook
inwards the plane of coil.
Y
Yo
E E E
(b) Torque on the loop, x = MB sin a
09 CO to
BBo

o o o

0.5m (0 Torque is maximum if a = 90®, i.e., the plane


G of coil is parallel to the direction of magnetic
Y

0.2m 0.4m
r ree

o © field.
oouu

(ii) Torque is zero if a = 0°, i.e., when plane of


Sol. Torque (t) on a current loop suspended in a
ad

uniform magnetic field is given by the loop is perpendicular to the magnetic field.
Yd

X = I AB sin a i.e. t A. Since the area of


loop (c) = 0-5 m X 0-5 m is maximum; hence VII. Conversion of galvanometer
nidn

into ammeter and voltmeter


the largest torque will be acting on loop (c).
Re

When any wire is bent into a circular loop, the 36. While converting galvanometer into an
FFi

torque will be even more because for a given ammeter, how does the parallel low resistance
perimeter the area of the circle is maximum.
(shunt) bring the required changes in the
34. A length L of a wire carries a current I. Prove galvanometer ?
that if the wire is formed into a circular loop,
Sol. When a galvanometer is to be converted into an
the maximum torque in a given magnetic
ammeter of a given range, a suitable low
field will be developed if the coil has one turn
only. Find the value of the maximum torque resistance S = !gGI(l - /p is connected in parallel
acting on circular coil. with the galvanometer. In doing so, the limited
current of circuit flows through galvanometer
Sol. Let the circular coil formed has n turns, each of
radius r. Then L = 2.nrn ox r = L12 n n and remaining current of the circuit flows
through shunt. Due to it, a galvanometer works
nl? L2 as an ammeter of given range.
Area of the coil, A = 7t =
4n^n^ 4nn^ 37. A loop of irregular shape carrying current is
Torque on coil in magnetic field is located in an external magnetic field. If the
l} wire is flexible, why does it change to a circular
X = nIBA sin a = nIB X ^ sin a shape ? (NCERT Solved Example)
4nn^
4/92 7\adee^ '<*. Fundamental Physics (XII) vMmi

Sol. A loop of iiregular shape carrying current when 42. Why should the resistance of an ideal
placed in an external magnetic field, it assumes voltmeter be infinite and of ideal ammeter
circular shape with its plane normal to the field be zero ?
in order to maximise magnetic flux linked with Sol. A voltmeter is used to measure the potential
loop. This is because for a given perimeter, a difference across the two points of a circuit. For
circle encloses maximum area than any other it, the voltmeter is connected in parallel across
shape. those two points. The potential difference
38. A galvanometer gives full scale deflection measured by voltmeter will be accurate if no
with current Ig. Can it be converted into an current is drawn by voltmeter. It will be so if
ammeter of range I <Ig1 the resistance of voltmeter is infinite. Then the
Sol. A galvanometer can be converted into an voltmeter will be ideal voltmeter. Thus, ideal
ammeter of given range by using a suitable shunt voltmeter has infinite resistance.
resistance S in parallel with the galvanometer, An ammeter is used to measure the current in
given by

w
the circuit. For it, the ammeter is connected in
/_ xG series of the circuit. The current measured by
8
s = ammeter will be accurate if there is no change
I-l

Flo
8 in current of the circuit by the use of ammeter
in series of circuit. It is possible if resistance of
If / < Ig, then the value of S is negative. Hence,

ee
ammeter is zero. Then, the ammeter is called
a galvanometer can not be converted into an
ideal ammeter. Thus ideal ammeter has zero

Fr
ammeter of range / < Ig. resistance.
39. Can we increase or decrease the range of a
given ammeter ? 43. Can we decrease or increase the range of the

or
ur
Sol. We can increase the range of the given ammeter
sf given voltmeter ?
by using a suitable shunt in parallel with the Sol. We can increase the range of the given voltmeter
ammeter but we can not decrease the range of by putting a suitable high resistance in series
ok
the ammeter for the reason explained in the with the voltmeter.
Yo
above question. We can decrease the range of the given voltmeter
Bo

40. Write two reasons, why a galvanometer by putting a suitable resistance in parallel with
cannot be used as such to measure current the high resistance already in series with the
re

in a given circuit. (CBSE 2011)


galvanometer working as voltmeter, so that the
Sol. A galvanometer as such cannot be used to effective resistance of the voltmeter multiplied
ou
ad

measure the current in a circuit due to following


with the current (Ig) gives the required potential
reasons :
difference to be measured now.
Y

(i) Galvanometer is quite a sensitive device. It 44. Why is ammeter connected in series and
shows full scale deflection with a current of few voltmeter in parallel in the circuit ?
nd
Re

microampere, hence may get damaged if strong (CBSE 2016)


current is passed through it.
Fi

Sol. An ammeter is a low resistance galvanometer.


(ii) While measuring current, the galvanometer It is used to measure the current in ampere. To
whose resistance is large than ammeter has to measure the current of a circuit, the ammeter is
be connected in series of the circuit. Due to it, connected in series in tl e circuit so that the
there will be decrease in current of the circuit.
current to be measured must pass through it.
Thus, measurement of current will not be true. Since, the resistance of ammeter is low, so its
41. When a voltmeter is put across a part of the inclusion in series in the circuit does not change
circuit, does it record slightly less or more the resistance and hence the main current in the
than the actual voltage across that part ? circuit.
Explain. A voltmeter is a high resistance galvanometer.
Sol. When a voltmeter is put across a part of the It is used to measure potential difference
circuit, it draws some current from the circuit. between any two points of the circuit. To
Due to it, the potential difference across that measure the potential difference between the
part becomes less than actual. As a result of it, two points of a circuit, the voltmeter is
voltmeterwill record slightlyless than the actual connected in parallel in the circuit. The
potential difference across that part. voltmeter resistance being high, it draws
MOVING CHARGES AND MAGNETISM 4/93

minimum current from the main circuit and the 46. A galvanometer of resistance G can be
potential difference to be measured is not converted into a voltmeter of range 0 - V volts
affected materially. by connecting a resistance R in series with it.
45. By mistake, a voltmeter is connected in series How much resistance will be required to
and an ammeter is connected in parallel, with change its range from 0 to V/2 ?
a resistance in an electrical circuit. What will [CBSE 2015 (C)]
happen to the instruments ?
V V
Sol. Voltmeter resistance being very high, when used Sol. In first case, /? = G or 1 8
I R +G
in series with a resistance in an electrical circuit, g
the effective resistance of the circuit becomes
Let R' be the required resistance to change the
very high. Due to it, the current in the circuit range from 0 to V/2.
becomes very low. As voltmeter measures
V/2
potential difference between the two points, it In second case, I g =
will show the reading but will not be damaged. R' + G

w
The ammeter resistance is very low. When it is V V/2
connected in parallel with a resistance, the weak
R +G R'+G
current flowing through the circuit will pass

Flo
through the ammeter. Due to it, the ammeter will i? G
not show appreciable deflection. Therefore, the or 2 R + 2G = /? + G or
2

ee
ammeter also will not be damaged.

Fr
● Very Short Answer
P ● Short Answer

or
ur
sf * Long Answer

VERY SHORT ANSWER QUESTIONS Carrying 1 mark


ok
Yo
Bo

I. Biot Savart*s law and magnetic field


due to a linear current carrying wire
re

1. The north pole of a magnet is brought near a


stationary negatively charged conductor. Will
ou
ad

the pole experience any force ?


Ans. No, a stationary charge does not produce
Y

magnetic field.
2. What is the unit of magnetic field strength in
nd
Re

cgs system and SI ? State the relation between


them.
Fi

Ans. Unit of magnetic field in cgs system is gauss dB =


/^/sin9
and in SI is tesla or N A'^ m"* or weber m"^. 47t r2
1 tesla = lO'^ gauss. -»

3. Explain, how moving charge is a source of ^5 = ^0 xr)


magnetic field ? 4jt
Ans. A moving charge produces an electric current
which in turn causes the magnetic field. 6. A beam of a particles FIGURE 4{Q).31

4. What are the dimensions of p,q/4 k ? projected along + x-axis,


Ans. [M^ L^T-'^A-\ experiences a force due
5. State Biot-Savart’s law and express this law to a magnetic field along
in the vector form. (CBSE 2017) the +y-axis Fig. 4(Q).31. 'g-partlcle ^.
What is the direction of
Ans. Biot-Savart’s law states that the magnetic field magnetic field.
induction at a point due to current I in element
(CBSE 2010)
of length dl, at distance r from the element is
4/94 'P’KuUe^ ^ Fundamental Physics (XII)

Ans. According to Fleming's Left Hand rule, the B =


Mo 27in/ n
i.e., B oc
direction of magnetic field is along - z axis. 4n r r

7. State the rule that is used to find the direction B’ 2n


x- = 4 or 5' = 4B
of magnetic field acting at a point near a ●● B (r/2) n

current carrying straight conductor.


Change in magnetic field = B' - B = 4 B - B
Ans. The direction of magnetic field acting at a point
near a current carrying straight conductor can = 3B

be given by Right Hand thumb rule. 12. In Fig. 4(Q).32 is shown FIGURE 4(Q).32
According to this rule, if we imagine the linear a circular loop carrying
wire conductorto be held in the grip of the right current /. Show the
hand so that the thumb points in the direction direction of the magnetic
of current, then the curvature of the fingers field with the help of lines
around the conductorwill represent the direction of force.
of magnetic field lines.

w
(CBSE 2004)
8. Where is the magnetic field of a current
element (i) minimum and (ii) maximum ? Ans. The magnetic field lines of a circular loop
carrying current / are shown in Fig. 4.13.
Ans. (0 The magnetic field of a current element is

Flo
minimum (which is zero) along the axis of a III. Ampere’s circuital law,

e
current element. («) The magnetic field of a solenoid and magnetic field

reee
current element is maximum in a plane passing due to current in conducting rod

FFr
through the element and perpendicular to its
axis. 13. A current is set up in a long copper pipe. Is
9. What is the nature of the magnetic field there a magnetic field (a) inside (b) outside

or
the pipe ?
ur
associated with the current in a straight
Ans. Inside the pipe, the magnetic field is zero, but
conductor ?
ss f
there is a finite value of the magnetic field
Ans. The magnetic field lines due to straight
conductor carrying current are in the form of outside the pipe.
ok
Yo
concentric circles with the conductor as centre, 14. Compare Gauss’s law and Ampere’s law.
Ans. Gauss’s law corelates surface integral of electric
Bo

lying in a plane perpendicular to the straight


conductor.The directionof magnetic field lines field E with charge q over a surface [i.e.
re

is given by Right Hand thumb rule.


oE .ds = —], whereas Ampere’s law corelates
II. Magnetic field due to a ^0
ou
ad

circular coil carrying current the line integral of magnetic field B with
YY

10. How does a current loop behave like a bar current over a closed path [i.e. n B .dl = \Iq 1 ]●
(CBSE 2011)
magnet ? 15. Magnetic field lines can be entirely confined
nd
Re

Ans. A current loop behaves as a bar magnet because within the core of a toroid, but not within a
(CBSE 2009)
Fi

(0 one face of current loop behaves as a south straight solenoid. Why ?


pole and the other face as north pole. Ans. It is so because the magnetic field induction
outside the toroid is zero.
(ii) it possesses a magnetic dipole moment
(M = lA) and 16. How is the magnetic field inside a given
(fii) it experiences a torque in an external solenoid made strong ? -Oil)
magnetic field, which tends to align the axis of Ans. It is done by inserting laminated iron core inside
the solenoid.
the loop along the direction of magnetic field
17. What is the difference between solenoid and
as bar magnet does. (CBSE 2019)
toroid ?
11. What will be the change in the magnetic field
Ans. A solenoid consists of an insulating long wire
strength at the centre of the circular coil
closely wound in the form of helix. Its length is
carrying current, if the current through the very large as compared to its diameter.
coll is doubled and the radius of the coil is
A toroid is a hollow circular irng on which a
halved ?
large number of insulated turns of a metallic
Ans. Magnetic field induction at the centre of the wire are closely wound. Infact, toroid is an
circular coil carrying current is endless solenoid.
MOVING CHARGES AND MAGNETISM
4/95

18. In a solenoid carrying current, where is the magnetic field, acting perpendicular to their
magnetic field (/) maximum («) minimum and paths. Which of them wlU move in a circular
(Hi) half of the maximum value ?
path with higher frequency ? (CBSE 2018)
Ans. A solenoid carrying current ha.s (/) maximum Ans. Frequency of revolution of charged particle
value of magnetic field at a point will inside moving in a perpendicular uniform magnetic
the solenoid lying on the axis of solenoid. field is
(«) There is a minimum value of magnetic field
at a point outside the solenoid near the middle V =
2n m
i.e., V oc —

of solenoid and (»7) half of the maximum value m


m

oww
at a point near the end of the solenoid. As , so - > 1 or V, >

m
p
IV. Force on charged particle
25. Two protons of equal kinetic energies enter
due to electric and magnetic a region of uniform magnetic field. The first
fields and cyclotron proton enters normal to the field direction

e
while the second enters at 30" to the field
19. What is the value of absolute permeability

re
of free space ? Give Its units. direction. Name the trajectories followed by
**’«*"● [CBSE 2018 (C)]

FFllro
Ans. 4rcx lO-'^TmA"’ or Wb m-'
Ans. The first proton will describe a circular path

reF
in
20. Is the source of magnetic field analogue to a plane perpendicular to the direction of

e
the source of electric current. magnetic field. The second proton will describe
uoru
Ans. No, because the source of magnetic field is a a helical path with axis parallel to the direction

sFr
magnetic dipole, but the source of electric field of magnetic field.

foro
is an electric charge. 26. Derive an expression for the velocity of a
21. When a charged particle moving with a
fk positive ion passing undeflected through a
region where crossed and uniform electric
okso
velocity v is subjected to a magnetic field field E and magnetic field B are

simultaneously present.
Y
B, the force acting on it is non zero. Would
Yo
(CBSE Sample Paper 2019)
oo
BB

the particle gain any energy ? (CBSE 2013) Ans. The positive ion of charge q moving with
Ans. The magnetic force on a moving charged velocity v will go undeflected while passing
... —> —>

particle in a magnetic field is, F = q{vxB). through a region where two crossed uniform
rYree

electric and magnetic fields are present if the


oouu

Its direction is perpendicular to direction of force on ion due to electric field is equal and
motion of particle as well as perpendicular to
ad

opposite to the force on ion due to magnetic


Yd

the direction of magnetic field. Due to it, no field i.e..


work is done by the magnetic force on the Eq = qv B or v = EIB
nidn

charged particle. Therefore, the particle does not 27. A charged particle moving in a uniform
gain any kinetic energy.
Re

magnetic field penetrates a layer of lead and


22. Show that the kinetic energy of the particle thereby loses one half of its kinetic energy.
FFi

moving in a magnetic field remains constant. How does the time period of revolution of
(CBSE 2017) particle change.
Ans. A charged particle moving in a perpendicular Ans. The radius of the circular path, r=m vIqB. If
magnetic field describes a circular path, on E/( is the kinetic energy of the particle, then
which its speed v does not change. So its kinetic I
= —nnp-
( 1 K 9
or V
= 4'^E„/m
energy K = — mv^ remains unchanged.
V ^ ) m
radius, r =
23. An electron is moving vertically downwards. qB qB ’
If it passes through a magnetic field which is
directed from south to north in a horizontal 2nr 2nyj2m 2nm
plane, then in which direction the electron Time period =
would be deflected ?
V
qBj2Kf,/m qB
Ans. Towards west It means time period of revolution is inde
24. A proton and an electron travelling along pendent of K.E. This shows that if KE is halved,
parallel paths enter a region of uniform there is no change in time period of revolution.
4/96 “PnazUefi- Fundamental Physics (XII)

28. Establish analytically that the gain in kinetic to the direction of electric field but no force will
energy of the charged particle moving in a be acting on electron due to magnetic field.
magnetic field is zero. 33. What is meant by cyclotron frequency ?
Ans. Total gain in K.E. = work done Ans. It is the frequency of oscillation of a charged
t'S ^ '2 particle inbetween two dees of cyclotron, which
( dr , F -^dt is equal to the frequency of high frequency
F -dr = Fm —dt =
oscillator, creating electric field between two
m m
dt
'1
dees of cyclotron. Cyclotron frequency,
^ V = Bq!2 n m, which is independent of the radius
q(vxB)-vdt = q{vx v) Bdt =0 of the circular path and velocity of the charged

oww
'1 particle in the two dees of cyclotron.
29. A charged particle moving in a uniform 34. Looking downward, an electron appears
magnetic field penetrates a layer of lead and moving anticlock-wise on a horizontal circle
there by loses one-half of its kinetic energy. under a magnetic field. What is the direction

ee
How does the radius of curvature of its path of the magnetic field ?
change ? (CBSE 2010) Ads. The motion of electron in anticlock wise

FFrlo
r
2K
direction causes the current in clockwise
I

rF
Ans. Kinetic energy, ^ ~ —mv^
2 or v = direction. The magnetic force on the electron is

ee
along the radius towards the centre of circle. It
Radius of curvature, will be so if the direction of magnetic field is
ouru
rF
mv m 2K -j2mK vertically upwards.
r =
35. Which physical quantity has the unit
ffosor
qB qB m qB
Wb/m^ ? Is it a scalar or a vector quantity ?
os k
Therefore, r Vx, Ans. Wb/m^ is the SI unit of magnetic field induction
K/2 1 B, which is a vector quantity.
ook
Yo
So or r =-7sr
Y
r i K 41 V2
V. Force on current carrying wire in
Bo

30. State the principle of working of a cyclotron. magnetic field and force between two
reeB

(Pb. Board 2001)


parallel conductors carrying currents
Ans. The working of the cyclotron is based on the
oouY

36. Write expression for the force between two


ur

fact that a positively charged particle can be


short parallel wires carrying the currents
ad

accelerated to a sufficiently high energy with


the help of smaller values of oscillating electric when they are :
Y
d

field by making it to cross the same electric field (a) much separated
time and again with the use of strong magnetic
nidn

(b) very close


Re

field.
Po Po 2I,I^dl
Ans. (fl) ■ (b) F = -
FFi

31. An electron moves with velocity v =ak »n 471 4k r

A A

a magnetic field, B = b i + c j . Find the 37. What Is the force that a conductor dl
magnetic force on the electron. carrying a current / experiences, when placed
Ans. The force on electron in the magnetic field is
in a magnetic field B , when its length is
F = -e(ijx B) = -e[akxib i +c j)] making an angle of 30® with the direction of
field ?
A A r I
= -e[ab aci-i)] =ea[c i-bj\ Ans. I f/ ? I = / I dTx B I = / sin 30" = / dl B/2
32. A uniform magnetic field and a uniform
38. If the distance between two parallel current
electric field are produced, pointing in the
same direction. An electron is projected with carrying wires is doubled, what is the force
between them ?
its velocity pointed in the same direction.
What will be the effect on electron? Ans. The force acting on one wire due to currents
Ans. The electron velocity will decrease as it will
through two wires is inversely proportional to
the distance between them. Thus the force
experience force due to electric field opposite
MOVING CHARGES AND MAGNETISM 4/97

becomes 1/2 times if the distance between the Ans. Zero.


wires is doubled.
45. Two wires of equal lengths are bent in the
39. In which orientation is the force experienced form of two loops. One of the loops is square
by a current-carrying conductor placed in a shaped whereas the other loop is circular.
magnetic field (i) minimum («) maximum ? These are suspended in a uniform magnetic
Ans. Force on a current carrying conductor in field and the same current is passed through
magnetic field, F = IIB sin 6. them. Which loop will experience greater
(/) Force F will be minimum if sin 0 = 0 or torque ? Give reasons.
0 = 0“ or 180°, i.e., the linear conductor carrying Ans. For a wire of given length, the circular loop
current is parallel or antiparallel to the direction encloses greater area than the square loop. As,
of magnetic field. torque t = /AB, /. e., x « A, so the circular current
(ii) Force F will be maximum, if sin 0 = 1 or loop will experience greater torque in the
0 = 90°, i.e., the linear conductor carrying magnetic field.

ww
current is perpendicular to the direction of 46. On what interaction is the principle of
uniform magnetic field. galvanometer based ?
40. Two current elements are placed a certain Ans. The principle of galvanometer is based on the
distance apart but not parallel to each other.

FF loo
interaction of current and magnetic field.
Do they exert equal and opposite forces on 47. What is the value of net force acting on a
each other ?

ee
current carrying (i) rectangular coil and
Ans. Two current elements, placed a certain distance («) circular coil of same area placed in a

er
apart but not parallel to each other also exert uniform magnetic field ? What will be the

rFreF
forces on each other which are equal and torque acting in each case ?
opposite according to Newton’s third law of Ans. The net force acting on each current carrying
rur
action and reaction.
coil placed in magnetic field is zero. Torque on
41. Wires that carry equal but opposite currents
are often twisted together to reduce their
ffoor
each coil is same provided the plane of each
coil is equally inclined with the direction of
ks
magnetic effect at distant points. Why is this magnetic field.
YYoou
okso

effective ?
48. What is a dead beat galvanometer ?
Ans. It is so because the magnetic field B at a point
BBoo

Ans. Dead beat galvanometer is one in which the coil


is a function of distance and direction of current. comes to rest at once after the passage of current
re

Twisting of wires eliminates the magnetic through it. The deflection can be noted in no
time.
effects at a distant point because of the following
oouur
ad

reasons: (i) average distance is practically the 49. Why is the coil wrapped on a conducting
same and (//) current flows in the two wires in frame in a galvanometer ?
Y

opposite directions.
Ans. When a coil is rotated in the magnetic field, a
42. What is the ratio of electric and magnetic variable magnetic flux is linked with the
dnYd
Re

forces between two moving charges ? conducting frame of the coil, resulting the eddy
Ans. = 10^^. [Refer to Art. 4.25] currents in it, which opposes the cause
FFini

43. A horizontal wire placed perpendicular to a producing it. These eddy currents in conducting
magnetic field carries a current from left to frame help in stopping the coil soon i.e. in
right. The magnetic field is horizontal, making the galvanometerdead beat.
directed towards you. What is the direction 50. What is the function of soft iron cylinder
of magneticforce on the wire ? between the poles of a galvanometer ?
Ans. According to Fleming’s left hand rule, it is (CBSE 2017)
vertically downwards.
Ans. It concenuates the magnetic field and helps in
making the magnetic field radial and strong.
VI. Torque on coil in
51. Why are pole pieces of galvanometer made
magnetic field and galvanometer concave ?

44. A conducting circular loop of radius r carries Ans. To have a uniform, strong and radial magnetic
field.
a constant current i. It is placed in a uniform
magnetic field B such that B is perpendicular 52. What is the function of the radial magnetic
to the plane of loop. What is the magnetic field in the moving coil galvanometer ?
force acting on the loop ? (CBSP: 2019)
4/98 U Fundamental Physics (XII)

Ans. It helps the arm of the couple and hence the 57. Which has greater resistance
torque on coil always the same in all positions. (a) milliammeter or ammeter ?
This arrangement provides linear scale to the (b) milli voltmeter or voltmeter?
galvanometer. Ans. The resistance of milliammeter is greater than
53. Define the current sensitivity of a moving coil that of ammeter. The resistance of voltmeter is
galvanometer and state its SI unit ? greater than that of millivoltmeter.
[CBSE 2013, 2013 (C)J
58. Why should a voltmeter have a low current
Ans. The current sensitivity of a moving coil carrying capacity ?
galvanometer is defined as the deflection Ans. A voltmeter should have a low current carrying

oww
produced in the galvanometer when unit current capacity so that its resistance is high. It draws a
flows through it. very small current from the circuit. Due to it,
, 0 nBA
Current sensitivity, the potential difference {V = IR) to be measured
will not be much different from the actual value.
The SI unit of current sensitivity is rad A“*.

e
59. A galvanometer of resistance 50 £2 is shunted
54. Write two factors by which the current

re
by a resistance of 5 £2. What fraction of the
sensitivity of a moving coil galvanometer can main current passes through the galvano

FFllro
be increased ? meter? Through the shunt ?

reF
Ans. A current sensitivity of moving coil galvano- (Pb. Board 2011)

e
0 nBA
meter is / Ans. Fraction of current passing through galvano¬
uoru I k
i
5 1

sFr
Thus, current sensitivity can be increased (0 by meter - —
increasing the strength of magnetic field B or G+5 50 + 5 11

foro
I

increasing the number of turns in the coil.


fk
(/7) by decreasing the torsional constant k of its
Fraction of current passing through shunt
okso
suspension fibre. i
5 _
i -1 i
1-1?
I I / ll”ll
YYo
VII. Conversion of galvanometer
oo
BB

into ammeter and voltmeter 60. A current of 10“’ A produces 50 division


deflection in a galvanometer. Find its figure
55. Can a galvanometer as such be used for of merit.
rYree

measuring the current ? Explain. Ans. Figure of merit,


oouu

(CBSE 2017)
I 1^ = 2 X ir^ A/div
ad

Ans. No, a galvanometer as such can not be used for k=-


6 ^
Yd

measuring current as it is a very sensitive


instrument as it shows full scale deflection even 61. Can moving coil galvanometer be used to
nidn

for a very small current. detect an a.c. in a circuit ? Give reason.


Re

56. What is meant by figure of merit of a Ans. A moving coil galvanometer cannot be used to
galvanometer ? detect a.c. in a circuit, since it measures the
FFi

Ans. Figure of merit of galvanometer is defined as average value of current and the average value
the amount of current which produces one scale of a.c. over a complete cycle is zero.
deflection in the galvanometer.

SHORT ANSWER QUESTIONS Carrying 2 marks

O') maximum
I. Blot Savart^s law and magnetic field (CBSE 2007)
(u) minimum.
due to a linear current carrying wire —>

Ans. F =g(vxB)
1. Write the relation for the force F acting on
a charge carrier q moving with a velocity V or I F I = 1 vx 5 I =guBsin 0
through a magnetic field B in vector (0 F will be maximum, when sin 0 = 1 or
0 = 90°, i.e., the charged particle is moving
notation. Using this relation, deduce the
conditions under which this force will be perpendicular to the direction of magnetic field.
MOVtNG CHARGES AND MAGNETISM 4/99

(«) F will be minimum, When sin 0 = 0 or —»

0 = 0° or 180° le., the charged particle is


magnetic field due to a small element dl at
moving parallel to the direction of magnetic a point P at a distance r* from the element
field. (CBSE2009)
as shown in Fig. 4(Q).33.
2. What is magnetic flux density ? Define its
units and give its dimensions. FIGURE 4(Q).33
2a
Ans. Magnetic flux density at a point in a magnetic
field means magnetic field induction at that
point. It is defined as the force experienced by dl
.

a unit charge while moving with a unit velocity,


perpendicular to the direction of magnetic field 'o ^ p
at that point. Force experienced by the charged
particle having charge q moving with velocity X

V through a magnetic field B is given by

w
Ans. According to Biot-Savart’s law, the magnitude
I F I=^ 1 S I = ^j;6sin0 of the magnetic field induction at a point F, due
to current element of length dl, carrying current

Flo
F
or B = /, at a distance r from the element is given by
lyvsinO -»

ee
The SI unit of B is tesla, where I tesla is the \dB\ =
fio / r// sin 0 _ 1^0 /1 (// X r I
magnetic flux density at a point if 1 coulomb r3

Fr
47t 4jt
charge while moving with a velocity of 1 ms“’,
perpendicular to a magnetic field experiences a The direction of dB is perpendicular to the

or
force of 1 N at that point.
ur
—* —*

The dimensional formula of B plane containing dl and r and is directed


sf
along x-axis, which is given by Right Hand
[MLT--] Rule.
= A"']
ok
[AT]{LT~']
Yo
6. Fig. 4(Q).34 shows a cube made from twelve
3. In what respect does a wire carrying a uniform wires. Find the magnetic field at the
Bo

current differ from a wire, which carries no centre of the cube, if a battery Is connected
current ? between the points A and H.
re

Ans. A current carrying wire produces a magnetic


FIGURE 4(Q).34
field. When current is flowing through a wire,
ou
ad

the electrons move inside it along a definite


H
direction. On the other hand, if no current is
Y

flowing through a wire, the electrons inside this B


C.,
wire are in random motion, their average thermal
nd
Re

velocity is zero. Such a wire does not produce


any magnetic field.
Fi

4. A current of one ampere is passed through a I/A D


straight wire of length 2*0 metre. Find the
magnetic field at a point in air at a distance
3 metre from one end of wire but lying on Ans. We have the following current carrying
the axis of the wire. conducting pairs AB and EH, AF and CH, AD
Ans. When a point P lies on_^he axis of wire and GH, BG and DE. FG and DC. FE and BC,
conductor in air, then I dl and r for each for which each pair produce equal and opposite
element of the straight wire conductor are magnetic fields at the centre. As a result of it,
the resultant magnetic field at the centre will
parallel. Therefore, ldlxr=0, so the be zero.
magnetic field induction at given point is zero. 7. Find the expression for the magnetic field at
5. State Biot Savart’s law. A current I flows in a the centre O of a coil bent in the form of a
conductor placed perpendicular to the plane square of side 2a, carrying current /, Fig.
of paper. Indicate the direction of the 4(Q).35.
4/100 4- Fundamental Physics (XII)

10. Deduce the expression for the magnetic field


FIGURE 4(Q).35
induction at the centre of a circular electron
B C
orbit of radius r, and angular velocity of
I
orbiting electron to.
\ /

45'’/N Ans. The electron moving on a circular orbit acts like


E —-\-XO a current loop. Magnetic field induction in
\
magnitude, at the centre of the current loop is
I
4 B =
e
_ ItQgO)
A D
2a ♦I
471 r 47tr [T 2r 2k/(x> 47tr

11. What is magnetic dipole moment of a current


Ans. Here, OE = AE = a, ZAOE = ZBOE = 45® loop? Give its direction if any.
Magnetic field at O due to current in arm AB of Ans. Magnetic dipole moment of a current loop =
wire niA where n = no. of turns in a current loop;

w
i = current through the loop and A = area of
B.I _^/-
= — [sin45°+sin45®] = each turn of the loop. Magnetic dipole moment
47t a 47tfl
is a vector quantity. Its direction is perpendicular
It is acting vertically downwards. to the plane of loop directed outwards for

FF loow
The magnetic field at 0 due to currents in arms anticlockwise current in loop and is directed
inwards for clockwise current in loop.

ee
BC, CD and DA will also be of the same
12. A wire of length L metre carrying a current

Fr
magnitude and direction as g,. Therefore, of I ampere is bent in the form of a circle.
resultant magnetic field at O is Find its magneticmoment.
(Uttarakhand Board 2012)

rer
ur
B = 4B.=
1
47ta na fofr Fo
Ans. L = 2kR or R = LJ2n\
Magnetic moment = 1 A = / kR^
It is acting vertically downwards.
ks
\2
YYouro
= I -VIA 7C
s oo

II. Magnetic field due to a 271

circular coil carrying current 13. A circular coil of N 'Rirns and diameter d
BBook

carries a current /. It is unwound and


8. What is the magnetic effect of current ?
r ee

rewound to make another coil of diameter


Describe the nature of the magnetic field 2d, current I remaining the same. Calculate
related with the current in circular coil.
oouru

the ratio of the magnetic moments of the new


ad

Ans. When a current is passed through a conductor, coil and the original coil. (CBSE 2012)
Y

magnetic field is produced around the Ans. Magnetic moment of original coil,
conductor. It is called magnetic effect of current. Afj =: NUjtd^lA). Let be the number of turns
Ynd

The magnetic field is in the form of concentric


Re

of new wound coil, then


circular magnetic lines of force for a linear
NlK{dl2) = N^2n{2dl2) or N-^=N!2
FFindi

conductor carrying current. The magnetic field Magnetic moment of new coil,
is in the form of parallel straight lines at the
centre and concentric magnetic lines near the
circular coil carrying current. M2 = /Vi/TC[(2(i)2/4] =^lnd^
9. Plot a magnetic field due to a circular coil
carrying current and explain the law used to M1 ~ NlTuiy4 "
find the direction of magnetic field at the 14. Two identical circular loops, P and Q each of
centre of circular coil carrying current. radius r and carrying currents 7 and 2 7
Ans. The magnetic lines of force due to circular coil respectively are lying in parallel planes such
carrying current are perpendicular to the wire that they have a common axis. Their centres
loop and are circular near the wire and are distance 2 r apart. The direction of
practically straight near the centre of the wire current in both the loops Is clockwise as seen
from O which Is equidistant from both the
loop. Fig. 4.13. The direction of magnetic field
loops. Find the magnitude of the net magnetic
at the centre of circular current loop is given by Held at point O.
Right hand rule. For it refer to Art. 4.7.
MOVING CHARGES AND MAGNETISM 4/101

Ans. (0 At a point inside on the axis.


FIGURE 4(Q).36
2r
Magnetic field due to current i through 5i
♦ N
Sj = [Xq Mj I, along BA, and
> V2I Magnetic field due to current i through ^2
1 '
n
I ^2 = ^ong AB
o Net magnetic field = Bj - ^2
21
= m i (/ij - Wj), along BA
P Q (ii) At a point outside the combined system, net
magnetic field = Zero.
Ans. Magnetic field induction at 0 due to current in
coil P is IV. Force on charged particle
due to electric and magnetic
B
1 “
fields and cyclotron
2(^2 ^^2)3/2 4^^
16. Write the expression for Lorentz magnetic

w
pointings towards OCy force on a particle of charge q moving with
Magnetic field induction at 0 due to current in
coil Q is velocity v in a magnetic field B .Show that

Flo
no work is done by this force on the charged
p,,(2/)r^ lip/ (CBSE 2011)
^2 = particle.
2(r2 + r2)3/2 2^r

reeee
Ans. Lorentz magnetic force on a moving charged
pointing towards OC2 particle in the magnetic field is

Fr
Net magnetic field inducion at O
F =q(vxB)
This force is acting perpendicular to the plane
B = ^2 - Bj =
ijlr 4-^^r 4^2r
for
ur
containing v and B , and is directed as given
(pointing towards OCj) by Right Hand rule. Since work done,
kss
III. Ampere’s circuital law,
Yo
W = F. s = FscosQ,
oo

solenoid and magnetic field


due to current in conducting rod
where 5^ is displacement of charged particle
eB

in magnetic field. Here, angle 0 = 90°, so


15. Two long coaxial insulated solenoids 5^ and W= Fs cos 90° - 0.
S2 of equal lengths are wound one over the 17. A charged particle is moving on a circular
r

other as shown in Fig. 4(Q).37. A steady path of radius B in a uniform magnetic field
ou
ad

current ‘/’ flows through the inner solenoid under the Lorentz force F. How much work
YY

to the other end B, which is connected to is done by the force in one round ? Is the
the outer solenoid S2 through which the same momentum of the particle changing?
current flows in opposite direction so as to
nd
Re

(CBSE 2011)
come out at end A. If and »2 are the
Ans. When a charged particle moving on a circular
number of turns per unit length, find the
Fi

magnitude and direction of net magnetic field


path of radius B in a uniform magnetic field,
at a point (i) inside on the axis and
the Lorentz magnetic force F qv B) acting
on the particle, provides the required centripetal
(u) outside the combined system. force for its circular motion. It means the Lerentz
(CBSE 2014)
force acts along the radius towards the centre
of circular path. While moving on a circular

path, the small displacement dr of the charged


particle is always perpendicular to Lerentz force,
i.e., 8 = 90°, therefore work done

dW = F-d^ =F dr cos 90°= 0.


Since the velocity of the charged particle,
moving on a circular path is acting tangentially
to the path whose direction is changing
4/102
Fundamental Physics (XII) VOL.l

continuously in circular motion of the particle, field B, then it describe a circular path of radius
therefore the momentum of the particle is r given by
changing.
18- Two identical charged particles moving with Bqv =
m tr
or r =
mV _ p
same speed enter a region of uniform r
Bq Bq
magnetic field. If one of these enters normal where p ~ momentum of the panicle
to the field direction and the other enters
For the same values of p and B, I !q
along a direction at 30” with the field, what
would be the ratio of their angular
frequencies ? (CBSE Sample Paper 2008)
Ans. When a charged particle of charge q, mass m, 20. A deutron and a proton are accelerated by
moving with velocity v enters a uniform the cyclotron. Can both be accelerated with
magnetic field B, acting perpendicular to the the same oscillator frequency ? Give reason
direction of motion of charged particle, then the to justify your answer. (CBSE 2017)

ww
magnetic force on the particle provides
centripetal force. So Ans. Period of revolution of a charged particle In
qvB = tmP-lr' cyclotron is. T =
2rcm

Bq

Flo
or
vir - qB/m
The particle will describe a circular path of Frequency of revolution of a charged particle

ee
radius r. Angular frequency of particle,

rere
in cyclotron is, v = — =
V qB

rFF
(0= - = ^ T 2Km
r m
For deutron and a proton, charge q is same but
When the charged particle is moving, making ma.ss m is different, so frequency of revolution
uur r
an angle 30” with the direction of magnetic field,
then due to component velocity v cos 30” (along
foor of deutron and a proton in cyclotron will be
difterent. Hence they can not be accelerated with
sks
the direction of field) no force acts on the the same oscillator frequency.
Yoo
particle. Due to component velocity v sin 30”, 21. Show that the frequency of revolution, of a
ook

(acting normally to the direction of field) a force charged particle (in the X-Y plane), in a
BBo

is acting on the particle which provides the . A

required centripetal force for circular motion of uniform magnetic field B{=Bk) i
IS

independent of its speed.


re

particle. Thus,
Which pi-actical machine makes use of this
ouur

m(usin30”)- fact ?
ad

q (v sin 30”) B =
r What is the frequency of the alternating
YY

electric field used in this machine ?


or
usin30” _ qB Ans. The radius of the circular path traced by charged
nndd
Re

r r
particle in a perpendicular uniform magnetic
mv
field Is
FFii

Angular frequency (O =
usin30” _ qB
qB
r
Frequency of revolution,
Thus, the angular frequency of both the particles
is same. So ratio of their angular frequency V
1
u _ _ qB
= 1:1 T 2nr iKXtnv 2Km
19. An a-particle and a proton are moving In the which is independent of speed v.
plane of the paper in a region where there is A cyclotron makes use of this fact in which
—»

a uniform magnetic field B directed normal alternating electric field of frequency is


to the plane of the paper. If the two particles applied.
have equal linear momenta, what will be the 22. A proton and an alpha particle are projected
ratio of their trajectories in the field ? in a perpendicular magnetic field B which
(CBSE Sample Paper 2008) exists In a region of width <f. Fig. 4(Q) J8.
Find the ratio of the sines of angles of
Ans. When a charged particle of charge q moving deviation suffered by the proton and the
with velocity v is subjected to normal magnetic alpha particle if before entering the magnetic
MOVING CHARGES AND MAGNETISM 4/103

field both the particles (0 have the same


FIGURE 4(Q).39
momentum (ii) have the same kinetic energy.
z-axis z-axis

FIGURE 4(Q).38

O © @ @

© © V O,

w
V
V
® © 6
A -- q
c o ©
© ©^ x' X

© © © Path of
V
Ans. (0 The trajectory of charge q will be a circle in

e
© © © Particle

(/ - - > Y-Z plane as shown in Fig. 4(Q).39(/;).

e
(//) The .speed and kinetic energy of the charged

wr
lloo
particle remain constant, but velocity of the

r
(CBSE Sample Paper 2003)
charged particle changes in direction.
Ans. I f , Wq, are the mtisses of proton and a-particle

eF
then 24. A proton and a deutron having equal

u
momenta enter in a region of uniform
If
then

r FF
, q^ arc the charges of proton and a-particle,
^0 = 2 q^,.
When a charged particle de.scribes a circular path
magnetic Held at irght angles to the direction
of the field. Depict their trajectories In the
field. (CBSE 2013)

rrse
in the magnetic field, then radius of the path is
o Ans. As p = mv or V = plm. The radius of circular

koF
mv
p yj2 mK path traced by a charged particle in a
r =
perpendicular magnetic field is ;

ofr
Bq Bq Bq
where p is momentum and K is the kinetic
o mV _ p 1
sf
ou
r = or r —
ko
YY
energy of the particle. From figure, it is clear
that 0 is the angle of deviation (i.e., angle
B

r
P -^d ^
BY

between direction of final velocity and initial


oo

e
velocity) of the charged particle on leaving the ^d
r

magnetic field. Therefore, both the charged particles will


r ee

describe a trajectory of same radii as shown in


uu

In AOAC,
Fig. 4{Q).40.
d

sin0 = —
d Bqd Bqd Bqd
oo
ad

r mv P '2 m K FIGURE 4{Q).40


din

X X X X
Y

d
Ca.se (i). When p^ = p^,, then sin 0 O' ^ X X N X X

X X / X X

sin0,, ^ q p 1
Re

_ X X X X
FF
in

sin0 u q tt 2 X X \ X X

X X y X X

q X X X X

Case (ii). When - K^,, then sin 0 m


25. An a-particle and proton of the same kinetic
energy are in turn allowed to pass through a
1
1=1
m
X
a
= -X. - magnetic Held B, acting normal to the
sin0 a ‘la 2 \1 direction of motion of the particles. Calculate
the ratio of radii of the circular paths
23. A charge q moving along the X-axLs with a
described by them. (CBSE 2019)
velocity v is subjected to a uniform magnetic
Ans. Kinetic energy of a particle mass m moving with
field B acting along the Z-axis as it cros.ses
velocity v is
the origin O. Fig. 4(Q).39(a).
I
(i) TVace its trajectory, (ii) Does the charge K = — rmP'
gain kinetic energy as it enters the magnetic 2
field ? .lastify your aaswer. (CBSE 2009) or mv
= pmK
4/104
\ Fundamental Physics (XII)CZsXSI
Radius of the circular path described by the opposite to the current as shown in Fig. 4(Q).41.
charged particle in a perpendicular magnetic What is the force experienced by the proton
field is and what is its direction ? (CBSE 2010)

mv
V2mK ■'[m FIGURE 4(Q),41
r = or
B
Bq Bq I

r
a
m
— X— =
4m
— X—= 1
e
-"1 P Proton

'p Vp « m 2e
A

26. An a-particle and proton are accelerated


Ans. Magnetic field induction at P due to current I in

oww
through the same potential difference. Find
long straight wire AB is
the ratio (vJVp) of velocities acquired by two
particles. (CBSE Sample Paper 2020)
B-
Ans. Workdone on the charged particle when 4n d
accelerated under a potential difference V is

e
It acts perpendicular to the plane of paper

FFrlo
equal to gain in its kinetic energy. So

re
inwards, represented by n .
— mv^ or u = '2qV
1

ree
Lorentz force acting on proton at P can be given

F
or 1)«
m
by

rF
\

= ^x-£- = 2e j}^ _ 1
V
a
X F = q(vx B) = e n

V
V e 4m ^ 4nd

fsoor
m
ouur
p a

27. A charged particle moving with a uniform


skf or
F = -^-
ind
■ (vxn)
velocity v enters a region where uniform
ooko
YYo
electric and magnetic fields E and B are
2nd
Bo

present. It passes through the region without


reB

any change in its velocity. What can we acting in the plane of paper away from the
conclude about the (i) relative directions of wire.

29. Explain the main functions of electric and


uur
ooY

E, V and B ? (iV) magnitude of E and B ? magnetic fields in a cyclotron.


ad

(CBSE 2017) Ans. The oscillating electric field acting between the
dY

Ans. (i) A charged particle while passing through a two dees of the cyclotron will accelerate the
region goes undeflected (i.e. without any change charged particle when the particle passes
innd

through the gap between the two dees. The


Re

m velocity) if v, E and B are mutually magnetic field makes the charged particle to
FFi

perpendicularto each other, such that the forces describe a circular path inside the dee.
on charged particle due to electric field and 30. What is the basic principle of working of
magnetic field are equal and opposite. Due to cyclotron ? Write two uses of this machine.
it, they cancel out each others effect, Ans. The working of the cyclotron is based on the
(ii) The force on charged particle q due to fact that a heavy positively charged particle can
perpendicular magnetic field, be accelerated to a sufficiently high energy with
F^=qv Bsm9QP = qv B. the help of smaller values of oscillating electric
Force on charged particle due to perpendicular field, by making it to cross the same electric
electric field, F^ = q E. field time and again with the use of strong
As the charged particle goes undeflected, so magnetic field.
qv B = q E or V = E/B. A cyclotron is used (i) to bombard nuclei with
28. A long straight wire AB carries a current /. A high energy particles and to study the resulting
nuclear reaction (/i) to produce radioactive
proton P travels with a speed u, parallel to the
wire, at a distance d from it in a direction substances which may be used in hospitals for
diagnosing the diseases in the body.
MOVING CHARGES AND MAGNETISM 4/105

speeds, they will come closer to each other as


V. Force on current carrying wire in electrostatic attraction between them is stronger
magnetic field and force between two than magnetic repulsion between them.
parallel conductors carrying currents 35. A wire of length / metre carries a current I
ampere along the Y-axis. A magnetic field,
31. A body is suspended from the lower end of a
vertical spring. What shall be the effect on B = Bq (i + j + k) tesla exists m space. Find
the position of the body when a current is the magnitude of the force on the wire.
sent through the spring ? Does it depend upon Ans. As the wire carries currentI along the y-axis, so
the direction of current in the spring ? —> A

Ans. When the current is passed through a vertically I = Ij. Magnetic force on wire is

oww
suspended spring, the direction of currents in
two adjoining parallel turns of wire is the same.
F = I (7xB) = I [I jx[B^^U + j+i)]]
Hence a force of attraction will be acting there.
Due to it, the spring is compressed and the body
= llBQ[jxi + jxj^ jxk]
attached at the end of spring is lifted upwards. = //Bq [-it + O+n

ee
The similar observations are made if the
direction of current is reversed. Magnitude of the magnetic force is

FFrlo
F = //Sq [(1)2 + (- l)2]I/2 = iib^^N

r
32. Two long parallel wires are hanging freely.

rF
If they are connected to a battery (i) in series,

ee
36. Two long straight parallel conductors
(i{) in parallel, what would be the effect on carrying steady currents and along the
their positions ?

rF
same direction are separated by a distance
ouru
Ans. (i) When a battery is connected inseries to two d. How does one explain the force of attrac
long parallel wires, the currents in the two wires

fosor
tion between them ? If a third conductor
will be in opposite directions. Due to which a s kf carrying a current/(. in the opposite direction
force of repulsion will be acting between them is placed just in the middle of these
ooko
and they are moving further apart, (ii) When a conductors, find the resultant force acting on
Yo
battery is connected in parallel to two long the third conductor. [CBSE 2018 (C)]
Y
parallel wires, the currents in the two wires will Ans. Two straight parallel conductors carrying
Bo

be in same direction. Due to it, a force of


reeB

currents (i) in the same direction attract each


attraction will be acting between them and they other and (li) in the opposite directions repel
are coming closer to each other. each other. Refer to Fig. 4(Q).42, force on
ooY

33. A straight wire, of length L, carrying a


uur

conductor C due to conductor A is


ad

current /, stays suspended horizontally in II I


^^0
mid air in a region where there is a uniform I J =
a c
(acting towards right)
dY

CA
Ak dll
magnetic field B . The linear mass density Force on conductor C due to conductor B is
nidn

of the wire is X.. Obtain the magnitude of this


Re

magnetic Held. [CBSE 2004 (C)] I ^ I _ ^^0 '^Iblc


CB (acting towards left)
Ak dll
F
Fi

Ans. The wire remains suspended if weight of wire


FIGURE 4(Q).42
= force exerted by magnetic field, i.e.,
C
mg = ILB sin 90® or \Lg = ILB or B = Xg/l
The direction of B must be horizontal and
perpendicular to the length of wire. "Ic
34. A stream of protons is moving parallel to a PCB
stream of electrons. Will the two streams tend
Fca
to come closer or move apart ?
Ans. The behaviour of the two streams of protons
and electrons coming closer or moving apart
depends on their speed. If they have large
speeds, they will move apart because the
magnetic interaction is more repulsive than A 2 ^ B
electrostatic attraction. If they have small
4/106
Fundamental Physics (XlI)BZsI9]
Net force on conductor C will be The system will be in stable equilibrium if
torque is zero ; which is so if a = 0®. This is
IF..
net
1= —♦
An d An d possible if magnetic field B is perpendicular
to the plane of the coil.

nd (/^ - ) (towards irght) 40. Does the torque on planar current loop in a
magnetic field change when its shape is chan
37. A square loop of side a carrying a current /2 ged without changing its geometrical area.
is kept at a distance x from an infinitely long Ans. Torque on a planar current loop is given by
straight wire carrying a current 7j as shown T = niAB sin a, which is independent of shape
in Fig. 4(Q).43. Obtain the expression for the if area A is unchanged. Hence torque on a planar
re.sultant force acting on the loop. current loop in a magnetic field does not change

ooww
(CBSE 2019) when its shape is changed without changing its
area.
FIGURE 4{Q).43
41. State the underlying principle of working of
a moving coil galvanometer. Write two
reasons why a galvanometer can not be used

e
Il“ a

as such to measure current in a given circuit.

re
i>l2
(CBSE 2010, 2011)

rFFl
ree
a a

F
Ans. The principle of working of'a moving coil
galvanometer is ba.sed on the fact that if a current

rF
a

carrying coil is placed in a uniform radial field,


it experiences a torque.

fsfoor
ouur
Ans. The force on the near side of square loop is A galvanometer cannot be used as such to
repulsive and on far side is attractive. So mea,sure current in a circuit due to following
kosk
two reasons :
Resultant force on square loop is
YYo
(0 Galvanometer is very sensitive. It gives full
oo

^ _ Mq 2 V2 ^ ^2 acting in the scale deflection with a quite small current


BB

4jt X 4tc (x + a) (nearly few micro ampere).


plane of loop away from the long straight wire (;7) In order to measure the current, the
rre

galvanometer has to be connected in series of


1
■ 1 _ ^2 a
circuit. As its resistance is large, its presence in
oYuu

2tc ^ lx 2n X {x + a)
ad

x + a
the circuit will decrease the effective current in
the circuit
dY

VI. Torque on coil in 42. A rectangular coil of sides I and b carrying a


magnetic field and galvanometer current / is subjected to a uniform magnetic
innd
Re

38. A circular loop of radius 0*2 m carrying a field B acting at an angle 6 to its plane.
Write the expression for the torque acting on
FFi

current of 1A is placed in a uniform magnetic


field of 0*5 T. The magnetic field is it. In which orientation of the coil in the
perpendicular to the plane of the loop. What magnetic field, the torque is (/) minimum and
is the force experienced by the loop ? (u) maximum.
Ans. The current carrying loop is equivalent to a Ans. Torque on the coil, t = IIB b cos 6 = IlbB cos 6.
magnetic dipole. The magnetic dipole does not (0 Torque is minimum if cos 0 is minimum, i.e.,
cos 0 = 0 or 0 = 90°
experience any net force in a uniform magnetic
field. i.e., the plane of coil is set perpendicular to the
39. A small coil carrying current is held in a uni direction of magnetic field.
form magnetic field. How does the coil tend (//) Torque is maximum if cos 0 = I or 0 = 0°. It
to orient itself relative to magnetic field ? means the plane of coil is parallel to the direction
Ans. When a current carrying coil is held in a uniform of magnetic field.
magnetic field, the coil experiences a torque due 43. State properties of the material of the wire
to magnetic field given by used for suspension of the coil in a moving
t = MB sin a = ni AB sin a. coil galvanometer.
MOVING CHARGES AND MAGNETISM 4/107

0 / / 10 div/A
Ans. The properties of the material of the wire used As V = ~ ^ or R = s _

for suspension of the coil in a moving coil ^ V IR R V 20 div/V


galvanometer are as follows : = 500 a
1. It should have low torsional constant, i.e.,
48. The coils, in certain galvanometers, have a
restoring torque per unit twist should be fixed core made of a non-magnetic metallic
small.
material. Why does the oscillating coil come
2. It should have high tensile strength. to rest so quickly in such a case ?
3. It should be a non-magnetic substance. [CBSE 2008 (C)J
4. It should have a low temperature coefficient Ans. In a galvanometer the coil having a fixed core
of resistance.
made of non-magnetic metallic material, when
5. It should be a good conductor of electricity. oscillates, the eddy currents are set up in the
44. What is a radial magnetic field ? How has it metallic material which opposes the motion of
been achieved in moving coil galvanometer? the coil in the magnetic field. Due to it, the coil

ww
(CBSE 2010) comes to rest at once.

Ans. Radial magnetic field is that field, in which the 49. Increasing the current sensitivity of a gal
plane of the coil in all positions is parallel to vanometer may not necessarily increase its
the direction of the magnetic field. A radial voltage sensitivity. Justify this statement.

Flo
magnetic field has been achieved by (0 properly (CBSE 2011)

e
cutting the magnetic pole pieces in the shape of Ans. Current sensitivity of a galvanometer is given

eree
concave faces. (») using a soft iron core within nBA
by / = where n is the no. of turns in the

FFr
the coil.
k
45. Why is phosphor bronze alloy preferred for coil, A is the area of each turn of the coil, B
uurr
the suspension wire of a moving coil is the strength of uniform magnetic field and k

orr
galvanometer ? is the restoring torque per unit twist of
Ans. The suspiension wire of phosphor bronze alloy
is preferred in moving coil galvanometer
sfo
suspension fibre in galvanometer.
I nBA
kks
Voltage sensitivity =-^-
Yoo
because it has several advantages ;
oooo

(/) Its restoring torque per unit twist is small. can be increased by increasing n and A. But
Due to it, the galvanometer is very sensitive. increase of n increases the resistance R of
eBB

(/0 It has great tensile strength so that even if it galvanometer. Due to it, will decrease.
is thin, it will not break under the weight of the A can not be increased beyond a limit because
uurr

coil suspended from its end. in that case the entire coil will not be in a
uniform magnetic field. Moreover it will make
ad

{Hi) It is rust resisting. Hence it remains


Yo

unaffected by the weather conditions of air in the galvanometer bulky and unmanageable.
dY

which it is suspended.
46. What is the main function of a soft iron core
VII. Conversion of galvanometer
into ammeter and voltmeter
innd
Re

used in a moving coil galvanometer ?


Ans. (0 This makes the magnetic field radial. In 50. What information would you wish to have
Fi

such a magnetic field the plane of the coil is about the galvanometer before you convert
always parallel to the direction of magnetic field. the galvanometer into an ammeter or volt
Due to it, the galvanometer scale becomes linear. meter ?

(«) This increases the strength of magnetic Ans. Before converting the galvanometer into an
field due to the crowding of the magnetic lines ammeter or voltmeter we require the following
of force through the soft iron core, which in turn two informations.
increases the sensitiveness of the galvanometer. (0 The resistance of galvanometer and (//) the
47. The current sensitivity of a moving coil current for full scale deflection of galvanometer.
galvanometer is 10 div/mA and voltage 51. Why do we not use galvanometer as an
sensitivity is 20 div/V. Find the resistance of ammeter ?

the galvanometer. Ans. A galvanometer shows a full scale deflection


Ans, Here, = 10 div/mA = 10 div/10“^ A with a very small current. Hence a galvanometer
= 10^ div/A can m'ea.surc limited current. Therefore as such,
= 20 div/V a galvanometer can not be used as an ammeter,
which can measure the given large current.
4/108
‘P%^<U€^ ^ Fundamental Physics (XII)
52. Of the two identical galvanometers one is to r xG
g
be converted into an ammeter and another 5 =
into a milliammeter. Which of the shunts will I-I
g
be of larger resistance ? It shows that the shunt resistance to be used to
Ans. The shunt of milliammeter should have larger convert a galvanometer into a milliammeter is
resistance. It is so because the current measuring more than that needed to convert into an
range of milliammeter is less than that of ammeter. As the shunt is connected in parallel
ammeter. Therefore the larger portion of the with the galvanometer, so the effective
main current must pass through galvano resistance of converted galvanometer into
meter coil and less portion of current through GS
shunt for a converted galvanometer into ammeter or milliammeter is, R = . It
G + 5
ammeter/ milliammeter. It is only possible if the
resistance of shunt used is of larger value in shows that milliammeter will have a higher
milliammeter than in ammeter. resistance than that of ammeter.
57. Why is it that while using a moving coil gal

ww
53. Why should an ammeter have a low resis
tance and a high current carrying capacity ? vanometer as a voltmeter, a high resistance
Ans. An ammeter is used to measure the current, It in series is required whereas in an ammeter
can measure the current of the circuit if
a shunt is used 7 [CBSE 2010,2012,2013 (C)]

Flo
connected in series of the circuit. The ammeter Ans. A galvanometer can be converted into a
voltmeter of given range by connecting a high

ee
connected in series of the circuit can measure
the current and will not disturb the current of resistance of suitable value in series with it, so

rere
the circuit if its resistance is low and current that most of the voltage applied drops across it,

rFF
enabling the galvanometer to measure much
carrying capacity is high.
larger required voltage.
54. Why should a voltmeter have a high resis
uur r
A galvanometer can be converted into an
tance and a low current carrying capacity 7
Ans. Voltmeter is a high resistance galvanometer. It
foor ammeter of given range by connecting a low
resistance shunt in parallel with it, so that most
sks
is used to measure potential difference between of the current bypasses through the shunt
Yoo
two points of the circuit. To measure a potential resistance, enabling the galvanometer to
ook

difference between two points of a circuit, the measure much larger given current.
voltmeter is connected in parallel to the circuit
BBo

58. Compare a voltmeter and an ammeter.


across those two points. The potential difference
Ans. (0 Voltmeter is a high resistance device which
re

between those two points will not be affected if


is used to measure potential difference.
there is no change in the current flowing through
ouur

Ammeter is a low resistance device which is


the circuit between those two points, which will
ad

used to measure the current in electric circuit.


be so if practically no current flows through
YY

voltmeter. The same is possible if voltmeter has («■) Voltmeter is obtained by using a high resistance
a high resistance and a low current carrying of suitable value in series of the galvanometer.
nndd

capacity. That is why a voltmeter should have Ammeter is obtained by using a low resistance
Re

high resistance and low current capacity. shunt in parallel with a galvanometer.
FFii

55. What is an ammeter ? How is it used in an (Hi) Voltmeter is always connected in parallel
electric circuit 7 How does it differ from a to the conductor in circuit across which potential
voltmeter 7 difference is to be determined.

Ans. Ammeter is a low resistance galvanometer. The Ammeter is always used in series of circuit to
resistance of ammeter is low and that of measure the current,
voltmeter is high. Ammeter is connected in (jv) The range of voltmeter can be increased or
series and voltmeter in parallel in the circuit. decreased.

56. Which one of the two, an ammeter or a The range of ammeter can be increased but
milliammeter, has a higher resistance and cannot be decreased,
why 7 (v) Ideal voltmeter has infinite resistance.
Ans. An ammeter measures strong current and Ideal ammeter has zero resistance.
milliammeter measures small current. We know
59. The current flowing in the galvanometer G
that a galvanometer can be converted into an
when key K2 is kept open is /. On closing the
ammeter or milliammeter using a shunt
key K2, the current in the galvanometer
resistance S, given by
becomes //n, where n is an integer. Flg.4(Q).44.
MOVING CHARGES AND MAGNETISM 4/109

in-l)RS
FIGURE 4(Q).44 On solving, G =
R-in-l)S
S
K2 (n-l)RS
When R » S, G = = («-l)S
A/W R
R
60. A voltmeter of resistance R y and an ammeter
8 of resistance R^ are connected in a circuit to
measure a resistance R as shown In Fig.
h h <●> 4(Q).45. The ratio of the meter readings gives
Ki an apparent resistance R'. Show that R and
Obtain an expression for resistance G of the R' are related by the relation
galvanometer in terms of R, S and n. To what 1-J__ J-
form does this expression reduce when the (CBSE Sample Paper 2003)
value of R is very large as compared to S ?
R~ R' Ry

ww
(CBSE Sample Paper 2015) FIGURE 4(Q).45

Ans. When key K2 is open, current in the galvano


£

Flo
meter is I =
R + G

ee
When key K2 is closed, the equivalent resistance

rere
of the parallel combination of G and S is R

rFF
G5
R' =
G + S
uurr
Ans. Apparent resistance,
Total current in the circuit,
e foor R' =
voltmeter reading ___ V
ks s
R + GSHG-¥S) ammeter reading /
Yoo
oook

This current will divide between S and G in the


Total resistance of circuit, R = «A
inverse ratio of S and G.
R + Ry
eBB

Hence current through the galvanometer will be


RRy
5 £(G + 5) According to Ohm’s law.
h ~ R + Ry
rr

S + G (5 + G) /?(G + 5) + G5
ouu
ad

Se Also, e = IR, + V or - = R,.^-


^ I = R^ + R'
YY

RG + RS + GS
1 £ RRy RRy
= R^+R' or R' =
nndd

As per question, Iq — — or R^ +
Re

n n iR + G) R + Ry R + Ry
FFii

1 1 1
£ Se I
or — =
R + Ry _ 1 + —1 or
n(R + G) RG + RS+GS R' RRy Ry R R
R' Ry

SHORT ANSWER QUESTIONS Carrying 3 marks

1. Describe Oersted’s experiment and explain the 3. What is magnetic field ? State and explain Biot-
law by which direction of magnetic field Savart law.
produced can be accounted. [Art. 4.2] (Karnataka Board 2012, Jharkhand Board 2011)
2. What is the force acting on a moving charge in [Art. 4.3 and 4.4]
a uniform magnetic field ? Discuss the cases 4. Derive an expression for magnetic field strength
when the force is maximum and minimum and 5 at a point P due to current flowing through a
straight conductor using Biot Savart’s law.
define the unit of magnetic field B .
(Raj. Board 2011, Jharkhand Board 2011)
[Art. 4.3]
[Art. 4.6]
4/110 Fundamental Physics (XinPZSTW
5. State Biot-Savart’s law. Deduce the expression 13. Write the expression for the force acting on
for the magnetic field at a point on the axis of a a charged particle of charge q moving with a
current carrying circular loop of radius R, distant velocity ^ in the presence ofboth electric field
X from the centre. Hence write the magnetic field
at the centre of a loop. Draw the magnetic field E and magnetic field B. Obtain the condition
lines due to circular wire carrying current. under which the particle moves undeflected
[CBSE 2015, 2016, 2018 (C)] through the fields. (CBSE 2012)
[Refer to Art. 4.4 and 4.8] [Arts. 4.19 & 4.20]
6. State Ampere’s circuital law, expressing it in 14. (a) Write the expression for the force p acting
integral form. (CBSE 2014) [Art. 4.10] on a particle of mass m and charge q moving
7. A long straight wire of a circular cross-section
with velocity u in a magnetic field B. Under
of radius 'a' carries a steady current /. The
current is uniformly distributed across the cross- what conditions will it move in {a) a circular
path and iii) a helical path ?
section. Apply Ampere’s circuital law to

w
calculate the magnetic field at a point at distance (CBSE 2017) [Art 4.19]
'r' in the region for (i) r<a and («) r>a. 15. Write and explain the expression of Lorextz
force. What is the magnetic dipole moment of a
(CBSE 2010) [Art. 4.13]

Flo
current loop ? In what direction does it act.
8. A long solenoid with closely wound turns has n
(West Bengal Board 2012)

reeee
turns per unit of its length. A steady current /
[Arts. 4.19, 4.24]
flows through this solenoid. Use Ampere’s

FFr
circuital law to obtain an expression, for the 16. Explain the principle and working of a cyclotron
magnetic field, at a point on its axis and close with the help of a schematic diagram. Write the
to its mid point. expression for cyclotron frequency.

for
ur
(West Bengal Board 2012, CBSE 2019, 2011) (Kerala Board 2012, CBSE 2019, 2017, 2009)
[Art. 4.21]
[Art. 4.14]
kkss
9. State Ampere’s circuital law. Use this law to 17. Find the force on a current carrying conductor
placed in the uniform magnetic field. Name the
Yo
obtain the expression for the magnetic field
oo

inside an air cored toroid of average radius r


instrument in which this is used as a working
principle. [Art. 4.22]
eB

having n-tums per unit length and carrying a


steady current I. (CBSE 2015, 2018)
18. Find the force between two parallel conductors
carrying currents (/) in the same direction (//) in
[Arts. 4.10 and 4.15]
r

opposite directions and hence define one


ou

10. A long solenoid with air core has n turns per


ad

ampere.
unit length and carries a current /. Using
YY

(Karnataka Board 2012, CBSE 2011, 2009,


Ampere’s circuital law, derive an expression for
the magnetic field B at an interior point on its Pb. Board 2011) [Art. 4.23]
ndd

19. Two long straight parallel conductors carrying


Re

axis. Write an expression for magnetic field H


in the interior of the solenoid. currents /j and I2 are separated by a distance d.
Fi

(Sample Paper CBSE 2019) [Art. 4.14] If the currents are flowing in the same direction,
Ans. Magnetic field B at an interior point on the axis show how the magnetic field produced by one
of .solenoid is exerts an attractive force on the other. Obtain
the expression for the force and hence define I
B = \x^nl
But
ammpere. (CBSE Sample Paper 2022-23)
b = hh
[Art. 4.23]
So Of H= n I
20. Derive an expression for the maximum force
11. Di.scuss the motion of a charged particle when experienced by a straight conductor of length /,
subjected to a perpendicular uniform electric
field. carrying a current / and kept in a uniform
[Art. 4.17]
magnetic field B. (HP Board 2011) [Art. 4.22]
12. Discuss the motion of a charged particle when 21. Derive an expression for the force between two
subjected to a uniform magnetic field, when the
long parallel current carrying conductors. Use
direction of motion of the particle makes an
this expression to define SI unit of current.
angle (j) with the direction of magnetic field.
(CBSE 2012, 2010) (Art. 4.23]
tTamll Nadu Board 2012) [Art. 4.18]
MOVING CHARGES AND MAGNETISM 4/111

—»
22. A particle of mass m and charge q is moving at Sol. {/) The magnetic moment (w) due to a planar
speed V parallel to a long straight conductor square loop of side / carrying current I is
carrying current I as shown in Fig. 4(Q).46. Find
^ » ●>
magnitude and direction of electric field m = I A , where A = l~
required so that the particle goes undeflected.
(//) The currents in AB and EF are in the same
(Sample Paper CBSE 2019)
direction. So AB will be attracted towards EF
FIGURE 4(Q).46 with a force F|, given by
->

I—► V Py 2//| ^^0


F X (length AB) - I = 211
' 47t / 4it / 471
I.

oww
n

The currents in CD and EF are in opposite


directions. So CD will experience repulsion
away from EF, with a force F2, given by

ee
Ans. The magnetic field at a point with perpendicular ^^0
(length CD) = / = //,
distance r from linear conductor carrying current

FFrlo
2 47t 21 47t 21 Alt

r
lip 2tc/ _lio/ The forces on the portions BC and DA will

rF
IS B =

ee
4ti r 2t
cancel out each other’s effect. Therefore, net
It is acting perpendicular to the plane of paper force on loop is

rF
ouru
upwards.
The particle will go undeflectedif the force on ^^0 2//, - ^^0 ^^0
= ,F-F2 = II II

ffosor
1 I
particle due to magnetic field is equal and
os k 4;t 4ic 47C

opposite to the force on the particle due to towards EF


electric field.
As the area vector is parallel to the magnetic
ook
Yo
^^0 —/
Y
i.e., Bqv = Eq or E= Bv or E = V field, 8 = 0®, so torque on the square loop,
2 r
Bo

x = JAB sin Q = IAB sin 0® = 0. Also F, ^ F-,, so


reeB

It will be so if E is acting perpendicular to no torque acts on the loop.


the plane of paper downwards. 24. Find the torque acting on the rectangular current
oouY

loop suspended in a uniform magnetic field,


ur

23. (1) Write the expression for the magnetic


when it is inclined at an angle 6 with the
ad

moment (in) due to a planar square loop of direction of magnetic field.


dY

side ’/’ carrying a steady current / in a vector (CBSE 2019, 2013, Manipur Board 2011)
form.
nidn

[Art. 4.26]
Re

{//) In the Fig. 4(Q).47, this loop is placed in a


horizontal plane near a long straight conductor 25. Define current sensitivity and voltage sensitivity
FFi

of a moving coil galvanometer. How can a


carrying a steady current f at a distance / as
galvanometer be made more sensitive ?
shown. Give reasons to explain that the loop
will experience a net force but no torque. Write [Art. 4.27]
the expression for this force acting on the loop. 26. What is shunt ? State its uses. Find the relation
(CBSE 2010) for the current through shunt in terms of
resistance of galvanometer and shunt.
FIGURE 4(Q).47
[Art. 4.29]
I1
E F
27. What is an ammeter ? How can a galvanometer
be converted into an ammeter ? Explain.
B .
(Pb. Board 2011, HP Board 2011) 4 30^
/ 28. What is a voltmeter ? How can a galvanometer
be converted into a voltmeter ? Explain.
Dm—/—wD (HP Board 2013) j^rt. 4.31]
4/112 <t Fundamental Physics (XII) win

LONG ANSWER QUESTIONS Carrying 5 or more marks

1. Find the magnetic field induction (in magnitude Show that cyclotron frequency is independent
and direction) at a point due to current flowing of energy of the particle. Is there an upper limit
in a long straight conductor. Show the sketch on the energy acquired by the particle ? Give
of magnetic field produced. reason. (CBSE 2011, 2009) [Art. 4.21]
(Chhatisgarh Board 2011) (Art. 4.6] 7. Derive an expression for the force acting on a
2. State the Biot-Savart’s law for the magnetic field current carrying conductor placed in a uniform
due to a current carrying element. Use this law magnetic field and using this result to derive an
to obtain a formula for magnetic field at the expression for the torque acting on a cunent
centre of a circular loop of radius R carrying a carrying loop suspended in a uniform magnetic

oww
steady current /, sketch the magnetic field lines field. [Arts. 4.22 and 4.26]
for a current loop clearly indicating the direction 8. Discuss the principle, construction and theory
of the field. of a dead beat galvanometer. State two merits
(Bihar Board 2012, Uttarakhand Board 2012, of moving coil galvanometer. [Art. 4.27]
Raj. Board 2011, CBSE 2007) [Arts. 4.4, 7] 9. Draw a labelled diagram of a moving coil gal

e
3. Find the magnetic field induction at a point on vanometer. State the principle on which it works.

ree
rFl
the axis of a circular coil carrying current and Deduce an expression for the torque acting on

Fre
hence find the magnetic field at the centre of a rectangular current carrying loop kept in a

rrF
circular coil carrying current. [Art. 4.8] uniform magnetic field. Write two factors on
4. Derive an expression for the magnetic field which the current sensitivity of a moving coil
galvanometer depend. (Pb. Board 2011 ;
ouur
along the axis of an air-cored solenoid, using
Ampere’s circuital law. Sketch the magnetic
field lines for a finite solenoid. Explain why the
sffoo CBSE 2011, 2007) [Arts 4.27, 4.26]
10. Describe the working principle of a moving coil
kks
field at the exterior mid point is weak while at galvanometer. Why is it necessary to use (/) a
oooo
Yo
the interior it is uniform and strong. radial magnetic field and («) a cylindrical soft
iron core in a galvanometer ? Write the expre
eBB

(HP Board 2011) [Art. 4.14]


5. What is a cyclotron ? Discuss its construction, ssion for current sensitivity of the galvanometer.
working and theory. Explain cyclotron (CBSE Delhi 2017) [Arts 4.27]
urr

frequency. 11. Explain sensitivity of moving coil galvanometer.


You
ad

(Bihar Board 2012, .Iharkhand Board 2012, Show with the help of a circuit diagram that
Chhattisgarh Board 2011) [Art. 4.21] how a moving coil galvanometer can be
dY

6. With the help of labelled diagram, state the converted into an ammeter of given range. Write
underlying principle of a cyclotron. Explain necessary mathematical relation. What is the
innd
Re

resistance of ideal ammeter ?


clearly how it works to accelerate the charged
(Uttarakhand Board 2012) [Arts. 4.27, 4.30]
FFi

particles to high energies. (CBSE 2014)

CASE-BASED VERY SHORT/SHORT QUESTIONS

CASE 1. When a linear wire of vector length /, The direction of this force is given by Fleming’s
Left Hand Rule.
carrying current 7 is subjected to a uniform magnetic
For, 0 = 0° or 180°, sin 6 = 0, then
field B, the force experienced by wire is F = //Bx0 = 0 (Minimum)
The force on a wire carrying current of any shape
IFI = /1 / xB!=//Bsin6
in a uniform magnetic field is independent of its shape
where 8 is the angle between / and B. but depends on the vector length joining the beginning
For, 6 = 90°, F = 7 / B sin 90° = 7 / B (Maximum) and ending of the current wire.
MOVING CHARGES AND MAGNETISM 4/113

Read the above passage carefully and answer Read the above passage carefully and answer
the following very short and short questions. the following very short and short questions.
1. A horizontal wire placed perpendicular to a 5. What is the radial magnetic field ?
uniform magnetic field carries a current from left 6. What are the net magnetic force and torque on a
to irght. The magnetic field is horizontal, directed current loop in a non-uniform magnetic field ?
towards you. What is the direction of magnetic 7. What are the conditions under which the current
force on the wire ? loop is in the equilibrium position in a uniform
2. When a semicircular coil of radius r, carrying magnetic field ?
cmrent 1 is placed in a uniform magnetic field 8. A circular coil of 20 turns and radius 10 cm carries

B, find the force experienced by the coil. a current 5 A. If a magnetic field of 0-1T is applied
normal to the plane of the coil, find the torque
3. When a closed loop of any shape, carrying current acting on the coil.

ww
I is placed in a uniform magnetic field B, find CASE 3. There are two parallel coaxial circular
the force experienced by the coil. coils Cj and C2 of equal radius r and same number of
4. A horizontal straight conducting rod of length / turns N, carrying equal current I in the same direction
and mass m is suspended by two vertical wires at separated by a distance 2 R as shown in Fig. 4(Q).48.

Flo
its ends. A current / is set up in the rod through The direction of magnetic field at a point due to circular

e
the wires. How much magnetic field should be coil can be given by Right hand thumb rule.

ree
set up normal to the conductor in order that the

Fr
FIGURE 4(Q).48

rF
tension in the wires is zero ?
Ci
uurr
CASE 2. When a wire loop of n turns, each of

area A, carrying current I is suspended in a uniform


s for I-
kks
—>
0
magnetic field B, then the coil experiences a torque
Yo
oooo

given by
2 R
eB

T =nl (AxB)
Read the above passage carefully and answer
X = n I \ Ax B \ = n IAB sin 9
r

or
the following very short and short questions.
ou
ad

9. What is the direction of magnetic field at the


YY

where 6 is the angle between A andfi. This


centre Oy due to current in coil Cj ?
torque tends to rotate the coil about its axis of
10. What is the direction of the magnetic field at the
nndd

suspension till the angle 0 between A and B becomes centre Oy due to current in the coil C2 ?
Re

zero, i.e., the coil is set with its plane perpendicular to 11. What is the magnetic field at O due to current in
Fi

the direction of magnetic field.


If the coil is set with its plane parallel to the coil Cy ? Use = B
0
R
direction of magnetic field B, then 0 = 90®, so the torque,
t = n lAB sin 90° = n lAB (Maximum). In this situation, 12. What is the total magnetic field at 0 due to current
the magnetic field is called radial magnetic field. in coils Cj and C2 ?
ANSWERS
-*
1. According to Fleming’s Left Hand Rule, it is F = l\ I xBI =//5sin9 = /(2r)5sin0
vertically downwards.
2. When a semicircular coil of radius r, carrying 3. For a closed loop 1/1=0
current / is placed in a magnetic field, then I I I Force, F = // 5sin0 = /xOx5sin0=O
- 2 r. Now, the total force on the semicircular 4. Tension in the supporting wires will be zero if
coil is the weight of the rod acting vertically downwards
4/114 Fundamental Physics fXlUPZSTWl

is balanced by the upward force on rod due to


magnetic field, Le., mg = 11 B sin 90° = 11 B _[Iq 2kN1R^
B
i ~ acting along 0^ O2
4tc (/?2 + /?2)3/2
mg
or B =
II
_ Itp NIR^ _ }Iq/V/ _ B0
along 0^02
5. It is that magnetic field in which a coil carrying 2 2^2R^ ' 4V2/? “ 4V2
current when subjected to a magnetic field
experiences maximum torque. In this magnetic 12. Magnetic field at O due to cunent in coil Cj is
field, the plane of the coil is always parallel to B
0
the direction of the magnetic field. B,1 = _

along C>| O2
4V2 R 4V2
6. The net magnetic force on a current loop in a non-
uniform magnetic field is not zero but Torque may Magnetic field at O due to current in coil C2 is
and may not be zero.
_^Q 2kNIR^

w
7. In equilibrium position, the net magnetic force as acting along O ^ O2
well as Torque on the current loop are zero. 2 47t(/?2+/?2)3/2
8. Here, n = 20, r= 10 cm = 0-10 m ; / = 5 A,

Flo
B = 0-1 T, 9 = 0° NIR^ ^ |Xq NI
Torque, x = n IAB sin Q = nl (n i^) B sin 0° = 0 2 4V2 R

reeee
9. The direction of magnetic field at due to

FFr
B
0
current in coil C| is along O21 according to
- along Oj O2
Right Hand Thumb Rule.
10. The direction of magnetic field at the centre
for
ur
Resultant magnetic field at O is
due to current in coil C2 is along Oj Oj (according
to Right hand thumb Rule). - R + R - ^0 I ^0 -
kkss
along (?j O2
11. The magnetic field at O due to current in coil C] " ' 4V2 2V2
Yo
oo

IS
eB

CASE-BASED MCQs AND ASSERTION-REASON QUESTIONS

CASE 1. A charged particle of charge q moving


r

{d) can not be predicted.


ou
ad

with velocity y in an electric field £ experiences a


2. When a charged particle is moving perpendicular
YY

to the direction of magnetic field, its energy


force q ^ It is acting in the direction of £. A charged (fl) decreases {b) increases
ndd
Re

(c) remains unchanged


particle while moving with velocity t^ in a magnetic
{d) some time will decrease and sometimes will
Fi

field B. experiences a force F = q {v x S). This force increase.

is acting perpendicular to the plane containing For Question No. 3 and 4, we have given two
1) and B and is directed as given by Right Hand thumb statements each, one labelled as Assertion (A) and
other labelled as Reason (R).
Rule.
Choose the correct option out of the four options
Based on the above paragraph, answer given below :
questions no. 1 to 4 :
{a) Both A and R are true and R is the correct
1. The uniform electric and magnetic fields are explanation of A.
produced, pointing in the East direction. An
{b) Both A and R are true but R is not the correct
electron is projected with its velocity pointing east
direction. The electron velocity will explanation of A.
(a) increase (c) A is true, but R is false.
{b) decrease
{d) A is false and R is also false.
(c) remain unchanged
4/115
MOVING CHARGES AND MAGNETISM

3. Assertion. If a charged particle of charge q while 6. A galvanometer of resistance 30 shows full scale
deflection by current 600 \iA is to be converted
moving with velocity v i is subjected to a into an ammeter of range 0 to 5 A. The shunt
magnetic field B j, then the direction of force on resistance used in ohm is
the particle is along + z direction. (a) 3-0 milli ohm (b) 3-6 milli ohm
Reason. The direction of magnetic force on the (c) 3-2 milli ohm {d) 2-6 milli ohm.
charged particle is given by Right Hand thumb rule.
For Question No. 7 and 8, we have given two
4. Assertion. A charged particle moving perpendi
cular to a uniform electric field will go undeviated. statements each, one labelled as Assertion (A) and
other labelled as Reason (R).
Reason. There will be no effect of electric field
on the charged particle moving perpendicular to Choose the correct option out of the four options
the electric field. g?ven below:
(a) Both A and R are true and R is the correct
CASE 2. Galvanometer is a device to study the

ww
explanation of A.
current in the circuit. A galvanometer can be converted
into an ammeter of a given range by using a suitable (b) Both A and R are true and R is not the correct
low resistance in parallel of galvanometer. A explanation of A.

Flo
galvanometer can be converted into a voltmeter of given (c) A is true but R is false.
range by using a suitable high resistance in series of it.

ee
(d) A is false and R is also false.
The resistance of voltmeter is maximum, of ammeter is

rere
7. Assertion. Ammeter is used in series and voltmeter
least and of galvanometer is in between voltmeter and

r FF
ammeter. in parallel of the circuit.
Reason. Ammeter is used to measure the current
uurr
Based on the above paragraph, answer
questions no. 5 to 8 :
5. The resistance of a given milliammeter is
foor and voltmeter is used to measure the potential
difference.
ks s
8. Assertion. We can decrease the range of ammeter
Yoo
(a) less than that of ammeter
ooook

by using a resistance in series of ammter.


(b) more than that of micro-ammeter
Reason. The effective resistance in series
eBB

(c) more than that of ammeter combination of resistors will become less than the
(d) less than that of microammeter individual resistance.
rr
ouu
ad

ANSWERS
YY

1. {b) 2. (c) 3. (a) 4. id) 5. (c, d) 6.(b) l.{b) 8.{d)


nndd
Re

HINTS/EXPLANATIONS For Difficult Questions


Fii

1. The force on electron due to electric field E is 2. A charged particle, moving perpendicular to a mag
netic field experiences a force, F = qv B sin 90°
F^ = -eE, which is acting opposite to the = qv B, which acts perpendicular to ^ as well as
—>
direction of E. B. This force will provide the centripetal force,
The force on electron due to magnetic field is resulting the particle to move on circular path. As

^ = -c { V X S) = - c I u B sin 0“ I n = 0 F and displacement (7) on circular path are


Thus total force on electron.
perpendicular to each other, so magnitude of
velocity of the particle will not change, only
= -eE + 0 = -eE. direction of motion changes. Hence workdone on
F = F^+F m
—> —♦
the particle, W = F-s = F s COS 90° = 0.
Therefore, velocity of the electron will decrease.
4/116
4. Fundamental Physics (XII) VOL.I

Therefore, the speed of the charged particle remains IG


unchanged and hence energy of the charged 6. 5 =
g (600 X10-6)X30
particle in magnetic field remains unchanged. I-I
8 5-(600x10-6)
3. Both Assertion and Reason are true and the 600x10~6x30
Reason is the correct explanation of the 5
= 3-6 X 10-3
Assertion.
= 3*6 mini ohm
4. Both Assertion and Reason are false. As charged 7. Both Assertion and Reason are true but the
particle in the electric field will experience a force,
Reason is not the correct explanation of
F-qE, in the direction of E and will describe Assertion.

a parabolic path in the electric field. 8. We can not decrease the range of ammeter and the
5. The resistance of ammeter is least, more of effective resistance in series combination is greater
than the individual resistance. Thus both Assertion
milliammeterand maximum of microammeter. and Reason are false.

ww
FF loo
ee
TYPE I. BIOT-SAVART’S LAW 5. A rectangular loop of conductor of length a, breadth

ee r
AND MAGNETIC FIELD DUE b carrying current / as shown in Fig. 4(Q).49.

rFrF
TO CURRENT THROUGH Find the magnetic field induction at the centre O
THE STRAIGHT CONDUCTOR of the loop.
rur
1. A current of 10 A is flowing east to west in a
long wire kept in the east-west direction. Find
ffoor FIGURE 4(Q).49
Q
ks
I T
magnetic field in a horizontal plane at a distance
YYoou
I
of (i) 10 cm. north (ii) 20 cm south from the b
ookos

I
O
wire, and in a vertical plane at a distance of I
BBo

S
(Hi) 40 cm downwards, (iV) 50 cm upwards. ●mR

[Ans. (i) 2 X 10“^ T, vertically downwards


re

(ii) 10“® T, vertically upwards


(Hi) 5 X 10~6 T, horizontally pointing south
oouur
ad

Ans.
(iv) 4 X l()-6 T, horizontally pointing north.] Kab
Y

2. Calculate the magnetic field induction at the


centre of the coil in the form of a square of side 6. Two insulating infinite long conductors carrying
dndY

10 cm, carrying a current of 10 A. currents /, (= 2A) and Ij (= 4 A) lie mutually


Re

(Pb. Board 2001) [Ans. 1-13 x 10^ T] perpendicular to each other in the same plane, as
FFini

3. Two long parallel wires are placed at a distance shown in Fig. 4(Q).50. Find the magnetic field
of 12 cm from each other in air. Each wire has a at the point P (a, b), where a = 4 cm ; i? = 3 cm.
current of 3 A. Calculate the magnetic field at a FIGURE 4(Q).S0
point mid way between them when the currents
in them are : (0 in the same direction and (ii) in
opposite direction. 4 cm

[Ans. (/) Zero (ii) 2 x 10*® T, ~P(a. b)

normally downwards] 3 cm

4. A long straight wire carrying a current of 25 A is


placed in an external uniform magnetic field
3-0 X 10“^ T parallel to the current. Find the
magnitude of the resultant magnetic field at a
point 1-5 cm away from the wire.
[Ans. 4-48 x 10^ T] [Ans. 1-66 X 10 ® T ; normally outwards]
4/117
MOVING CHARGES AND MAGNETISM

7. Equal currents I-lAost flowing through the long


wires parallel to y-axis located at, a: = +1 m,x = +
2m,x = + 4m,A:= + 8m and so on but in opposite
directions as shown in Fig. 4(Q).51. What is the
magnetic field induction at the origin 0 ?
FIGURE 4(Q).51
■>’4

1 1 I I

O 1 2 4 8
[Ans. 5-2 X 10-^ T]
13. A current I (= 4A) flows along a thin wire PQRS
shaped as shown in Fig. 4(Q).53. The radius of

ww
[Ans. 1-33 X 10"^ T k ] the curved part of the wire is 10 0 cm. The angle
6 = 90°, Find the magnitude of the total magnetic
TYPE II. MAGNETIC FIELD field at the point O.

Flo
DUE TO CURRENT

ee
THROUGH CIRCULAR COIL

rere
8. A straight wire of length nil metre, is bent into a

rFF
circular shape. If the wire were to carry a current
of 5 A, calculate the magnetic field, due to it,
uurr
before bending, at a point distance 0-01 times
the radius of the circle formed from it. Also foor
ks s
calculate the magnetic field, at the centre of the
Yoo
circular loop formed, for the same value of
oook

current. [Ans. 4 x IQ-^ T ; 1-256 x ir^ T]


eBB

9. The electron of hydrogen atom moves along a [Ans. 2-68 x 10"^ T]


-10
circular path of radius 0-5 x 10 m (0 with a 14. Two identical loops P and 0 each of radius 5 cm
rr

speed of 4-0 x 10^ ms'^ (ii) with a frequency are lying in perpendicular planes such that they
ouu
ad

6-8 X 10*^ Hz. Calculate the magnetic field have a common centre as shown in the Fig.
produced at the centre of the circular path. 4(Q).54. Find the magnitude and direction of the
YY

(e= 1-6 X 10-19 C). [Ans. 25-6 T, 13-4 T] net magnetic field at the common centre of the
two coils, if they carry currents equal to 3 A and
nndd

10. The magnetic field due to a current-carrying


Re

4 A respectively. (CBSE 2017)


circular loop of radius 10 cm at its centre is
Fii

0-60 X 10^ T. Find the magnetic field due to this


loop at a point on the axis at a distance of 4 cm
from the centre. [Ans. 4 x 10“^ T]
11. An alpha particle moves along a circular path of
radius 2 A with a uniform speed of 2 x 10^ ms"*.
Calculate the m-agnetic field set up at the centre
of circular path. [Ans. 1-6 T]

12. A circular segment of radius 20 cm subtends an


angle of 60° at its centre. Fig. 4(Q).52. A current
of 10 A is flowing through it. Find the magnitude
and direction of the magnetic field produced at
[Ans. 6-28 x 10"® T ; Direction can not be deter
mined as direction of current is not given]
the centre.
4/118
Fundamental Physics (XII) VOL.I

15. Two identical circular coils, P and Q, carrying

currents 1 A and ^3 A respectively, are placed


concentrically and perpendicular to each other
lying in the AT and YZ planes. Find the magnitude
and direction of the net magnetic field at the
centre of the coils. (CBSE 2017)

Ans.
^0
T in X -2 plane
R

oww
16. In the net work shown in Fig. 4(Q).55, find the
magnetic field at the centre O of the coil.

ee
FIGURE 3(Q).S5

b 1 = 5A
5^0/e

FFrlo
a

r
Ans.
24jcr

rF
5cm
^2

ee
c e
O
TYPE III. AMPERE’S
ouru
rF
CIRCUITAL LAW
¥

ffosor
d USA os k 19. A long straight solid metal wire of radius R carries
a current i, uniformlydistributedover its circular
[Ans. 2 X 10 ® T normally out of cross-section. Find the magnetic field at a
the plane of paper]
ook
distance r from axis of wire (i) inside and
YYo
17. An electric current of / ampere is flowing in a {it) outside the wire.
Bo

long conductor CG as shown in the Fig. 4(Q).56.


reeB

Find the magnitude and direction of magnetic Ans. (i) («)


oouY

induction at the centre O of circular part. 2nR^ 4711 r


ur
ad

FIGURE 4(Q).56 20. A long straight solid conductor of radius 6 cm


Y
d

carries a current of 8 A, which uniformly


distributed over its circular cross-section. Find
nidn
Re

the magnetic field (a) at a distance of 3 cm from


the axis of the conductor (b) at a distance 10 cm
FFi

I
from the axis of the conductor.
c >
D F
G [Ans. (a) 1*33 x 10"^ T (6) 1-6 x 10"^ T]
I I
21. If the current density in a linear conductor of
radius 'a' varies with r according to relation
» where k isa constant and r is the distance
[Ans. — +1 acting ± r to loop upwards]
2r It of a point from the axis of conductor. Find the
magnetic field induction at a point distance r from
18. A conductor carrying current I is of the type as the axis, when (i) r<a and («) r>a.
shown in Fig. 4(Q).57. Find the magnetic field
induction at the common centre 0 of all the three
arcs.
Ans. (i) (H)
4 4r
MOVING CHARGES AND MAGNETISM 4/119

and pilch 24 cm. Find the components of


TYPE IV. MAGNETIC FIELD DUE
velocity of the proton along and perpendicular
TO SOLENOIDAND TOROID
to the magnetic field. Take the mass of the
22. A solenoid of length 0-20 m, having 120 turns proton = 1-6 X 10"^^ kg.
carries a current of 2-5 A. Find the magneticfield : [Ans. 1-2 X 10^ ms"^ 7-6 x lO"* ms"*]
(fl) in the interior of the solenoid, (b) at one end 29. An electron after being accelerated through a
of the solenoid. Given jig = ^^ potential difference of 100 V enters a uniform
[Ans. (a) 1-885 x ir^ T (6) 0-9425 x ir^ T] magnetic field of 0-004 T, perpendicular to its
23. A long wire carries a current of 20 A along the direction of motion. Calculate the radius of the
directed axis of a long solenoid. The field due to path described by the electron.
a solenoid is 4 mT. Find the resultant field at a [Ans. 8-4 mm]
point 3 mm from the solenoid axis.
30. A beam of protons enters a uniform magnetic
[Ans. 4-2 mT ; 18-4® with the directed axis] field of 0-3 T with a velocity of 4 x 10^ ms~^ at

ww
24. A copper wire having a resistance of 0-02 per an angle of 60® to the field. Find the radius of
metre is used to wind a 500 turns solenoid of
the helical path taken by the beam. Also find
radius 2-0 cm and length 30 cm. What should be the pilch of the helix (distance travelled by a

Flo
the emf of the battery which when connected proton parallel to the magnetic field during one
across the solenoid would produce a magnetic

e
field of 10"^ T, near the centre of the solenoid.
period of rotation). Mass of proton = 1-67 x

rree
10-2'' kg. [Ans. 1-2 cm ; 4-35 cm]
[Ans. 6 V]

r FF
31. An infinite long straight conductor XY is
25. A coil wrapped around a toroid has inner radius carrying a current of 10 A. An electron is moving
of 10 0 cm and an outer radius of 20-0 cm. If the
uurr
wire wrapped makes 600 turns and carries a
current of 10-0 A, what are the maximum and
for
with a speed of 10^ ms"^ parallel to the
conductor in air from point A to B, as shown in
Fig. 4(Q).58. The perpendicular distance
kss
minimum values of magnetic field within the
between the electron and the conductor XY is
ooook

toroid ? [Ans. 12 mT, 6 mT]


Yo
20 cm. Calculate the magnitude of the force
experienced by the electron. Write the direction
eBB

TYPE V. MOTION OF CHARGED


of this force.
PARTICLE IN (0 ELECTRIC FIELD
AND Hi) MAGNETIC FIELD
rr

FIGURE 4(Q).58
ou
ad

26. An electron enters electric field of 10^* V/m Electron


Y

●B
perpendicular to the field with a velocity of 10^
dY

I 20cm
m/s. Find the vertical displacement of electron
Re

after 2 milliseconds. Mass of electron = 9-1 x


innd

10A
10"^* kg ; charge on electron = 1-6 x 10"’^ C.
Fi

[Ans. 3-51 X 10^ m] [Ans. 1-6 X 10"” N, in the plane of


27. A chamber is maintained at a uniform magnetic paper away from AT]
field of 5 X 10“^ T. An electron with a speed of 32. A proton, a deutron and an alpha particle, after
5 X 10^ ms"* enters the chamber in a direction being accelerated through the same potential
normal to the field. Calculate (/) radius of the difference enter a region of uniform magnetic
path (ii) frequency of revolution of the
electron. field B, in a direction perpendicular to B.
Find the ratio of their kinetic energies. If the
Charge of electron = T6 x 10"*^ C,
radius of proton’s circular path is 7 cm, what
Mass of electron = 9-1 x 10"^* kg will be the radii of the paths of deutron and
[Ans. (/) 5-7 cm (h) 1-4 x 10* Hz] alpha particle ?
28. A proton projected in a magnetic field of 0-02
T travels along a helical path of radius 6 cm [Ans. 1 :1: 2, = 7-^2 cm, - 7-72cm]
4/120
^ Fundamental Physics (XII) VOL.I

TYPE VI. (i) MOTION OF


FIGURE 4(Q).S9
CHARGED PARTICLE,
BOTH IN ELECTRIC d
AND MAGNETIC FIELDS I ●
Hi) CYCLOTRON

33. A beam of proton passes undeflected with a


horizontal velocity v, through a region of
electric and magnetic fields, mutually perpendi I b
cular to each other and perpendicular to the
direction of the beam. If the magnitudes of the
electric and magnetic fields are 100 kV/m,
[Ans. 0*04 N on each arm ; force on arms ad
50 mT respectively, calculate the velocity of the

ww
beam v.
and be is towards right and on arms ab and
(CBSE 2008)
dc is down wards in plane of frame]
[Ans. 2 X 10^ ms"^]
38. A horizontal wire 0-1 m long having mass 3 g
34. If the maximum value of accelerating potential

Flo
carries a current of 5 A. Find the magnitude of
provided by a radio frequency oscillator be the magnetic field which must act at 30“ to the

ee
10 kV, calculate the number of revolutions length of the wire inorder to support its weight.

rere
made by an a-particle in a cyclotron to achieve [Ans. 0*1176 T]

rFF
one-tenth of the speed of light. Mass of 39. On a smooth inclined plane at 30® with the
proton = 1-67 X 10"^^ kg ; charge on proton horizontal a thin current carrying metallic rod
uur r
= 1-6 X 10-*^C.

35. A cyclotron has an oscillatory frequency of


foor
[Ans. 470 revolutions] is placed parallel to the horizontal ground. The
plane is located in a uniform magnetic field of
sks
10 M Hz and a dee radius of 60 cm. Calculate 0-15 tesla in the vertical direction. For what
Yoo
the magnetic field required to accelerate value of current can the rod remain stationary ?
ook

deutrons of mass 3-3 x lO’^”^ kg and charge 1-6 The mass per unit length of rod is 0-30 kg m-I‘.
BBo

X 10“*^ C. Find the energy of deutrons emerging [Ans. 11-3 A]


re

from the cyclotron. 40. Find the magnitude of the magnetic force on
[Ans. 1*3 T ; 14*74 MeV] the segment CD of length 30 cm placed in a
ouur
ad

magnetic field of 0-3 T and a current of 4 A


36. In a cyclotron, a magnetic field of 2-4 T is used
flows through it as shown in Fig. 4(Q).60.
YY

to accelerate protons. How rapidly should the


electric field between the dees be reversed ? The
nndd
Re

mass and the charge of protons are 1-67 x


10“^^ kg and 1-6 x 10“*^ C respectively.
FFii

[Ans. 1*37 X 10"* s, 3*64 x 10^ Hz]

TYPE VII. FORCE ON CURRENT


CARRYING CONDUCTOR
IN MAGNETIC FIELD

37. A current of 4A enters at the comer ‘a’ of a [Ans. 0*31 N]


square frame of side 10 cm and leaves at 41. Fig. 4(Q).61, shows an equilateral triangular
opposite comer ‘c\ A magnetic fild of 5 = loop CDE, carrying current I. Length of each
0-20 T acts in a direction perpendicular to the side of triangle is /. If a uniform magnetic field
plane of paper directed outwards Fig. 4(Q).59. exists parallel to side DE of loop, then find the
Find the magnitude and direction of the forces acting on the three wires CD, DE and
magentic forces on the four arms of the frame. EC separately.
MOVING CHARGES AND MAGNETISM 4/121

FIGURE 4(Q).63
P R O

- > ^2

M-
10 cm
[Ans. IIB V3/2, normally outwards, zero
[Ans. 10"^ N towards Q]
IIB V3/2, normally inwards]
45. Three long straight parallel wires are kept as

ww
TYPEVm. FORCE shown in Fig. 4(Q).64. The wire (3) carries a
current /.
BETWEEN TWO PARALLEL
STRAIGHT CONDUCTORS FIGURE 4(Q).64

Flo
CARRYING CURRENTS
(1) (2) (3)

e
42. Three long parallel wires AB. CD and EF of

ere
equal resistance are connected as shown in Fig. .. I2
I1-

FFr
4(Q).62. The separation between the neigh
bouring wires is 2 0 cm and wires AE and BF
uurr ■►X

orr
are of negligible resistance. The current is
supplied to the wires and ammeter reads 12 A.
Find the magnetic force per unit length
sfo I
z
kks
Yo
oooo

FIGURE 4(Q).62 >1


K a
E
eBB

(/) The direction of flow of current I in wire (3),


is such that the net force, on wire (1), due to
D
other two wires, is zero (ii) By reversing the
rr

A B direction of/, the net force, on the wire (2) due


ou
ad

to the other two wires, becomes zero.


YY

(a) of wire AB What will be the directions of current I, in the


(b) of wire CD. two cases ? Also obtain the relation between
nndd

[Ans. {a) 2*4 x 10“^ Nm"^ directed the magnitudes of current /j, /2 and /.
Re

(CBSE 2016 S)
upwards (b) Zero]
[Ans. (i) Downwards ; / = 2/2 («) upwards; / = /j]
Fi

43. A short conductor of length 5 0 cm is placed


parallel to a long conductor of length 1 -5 m near TYPE IX. TORQUE
its centre. The conductors carry currents 4 0 A
ON A CURRENT LOOP
and 3-0 A respectively in the same direction.
What is the total force experienced by the long 46. A circular coil of 200 turns and radius 10 cm is
conductor, when they are 3-0 cm apart? placed in a uniform magnetic field of 0-5 T,
[Ans. 4 X 10"* N ; attractive] normal to the plane of the coil. If the current in
the coil is 3-0 A, calculate the (a) total torque
44. Two very long, straight parallel wires P and Q
on the coil (b) total force on the coil (c) average
carry currents 5 A and 10 A respectively and are force on each electron in the coil, due to the
at a distance of 10 cm apart, as shown in Fig.
magnetic field. Assume that the area of cross-
4(Q).63. If a third wire R (length 10 cm) having section of the wire to be 10"^ and the free
a current of 5 A is placed in middle between them,
how much force will be acting on /? ? The
electron density is 10^^ m"^. (CBSE 2008)
-24
direction of current in all the wires is the same. [Ans. {a) zero {b) zero (c) 1-5 x 10 N]
4/122
'<t Fundamental Physics (XII) VOL.I

47. A circular coil of 200 turns, radius 5 cm carries 5x10 ^Nmdeg ^ find its current sensitivity i„ in

a current of 2*5 A. It is suspended vertically in degree per micro-ampere. (Ans. 0*36 deg/pA]
a uniform horizontal magnetic field of 0-25 T, 52. To increase the current sensitivity of a moving
with the plane of the coil making an angle of coil galvanometer by 50% Its resistance is
60° with the field lines. Calculate the magnitude increased so that the new resistance becomes
of the torque that must be applied on it to prevent twice its initial resistance. By what factor does
it from turning. [Ans. 0*49 Nm] its voltage sensitivity change ? (CBSE 2001)
48. A uniform magnetic field of 3000 G is [Ans. Decrease by 25%]
established along the positive Z direction. A 53. A galvanometerneeds 30 mV for a full scale
rectangular loop of sides 10 cm and 5 cm carries deflection of 30 divisions. Determine its voltage
a current of 12 A. It is placed in the magnetic sensitivity. What must be its resistance if its
field, (a) Calculate the maximum torque the loop current sensitivity is 2 div/pA ?
can experience, (b) Find the orientation of the [Ans. 10^ div/V ; 2000 Q]
loop for which the torque is zero, (c) In which

ww
54. A moving coil meter has the following
orientation the loop would be (i) in stable particulars : Number of turns, n = 24 ; Area of
equilibrium (i7) unstable equilibrium ? coil, A = 2-0 X 10“2 m^, magnetic field strength,

FF loo
[Ans. (fl) 1*8 X 10“^ Nm (b) Angle between B. =.0-20 T, Resisitance of the coil, /? = 14 Q,
area vector A and B is 0° or 180" (c) Loop (a) Indicate a simple way to increase the current

ree
will be in stable equilibrium if area vector sensitivity of the meter by 25%. (It is not easy
to change A or B). (b) If in so doing, the
A IS parallel to magnetic field B and in

rreeF
resistance of the coil increases by 7 £2, is the
unstable equilibrium when A is antiparallel voltage sensitivity of the modified meter greater
or lesser than the original meter ?
ur
to B ]
49. A coil in the shape of an equilateral triangle of
fofroF
[Ans. (a) n should be increased from 24 to 30
(b) Voltage sensitivity is lesser than original]
ks
side 0 02 m is suspended from a vertex such
that it is hanging in a vertical plane between
kos o

TYPE XI. CONVERSION


YYouor

the pole pieces of a permanent magnet OF GALVANOMETER


producing a horizontal magnetic field of 5 x
BBoo

INTO (a) AMMETER


lO*”^ T. Find the couple acting on the coil. When
AND (*) VOLTMETER
r ee

a current of 0-1 ampere is passed through it and


the magnetic field is parallel to its plane. 55. A galvanometer with a coil of resistance
ad
oouur

[Ans. 8*66 x 10"’ N-m] 12 0 Q shows full scale deflection for a current
Y

2-5 mA. How will you convert the meter into


TYPE X. MOVING COIL (/) an ammeter of range 0 to 7-5 A (b) a voltmeter
d

GALVANOMETER of range 0 to 10 V. (CBSE 2005)


Re
dnY

[Ans. (c) Shunt resistance4*0 x 10"^ D


50. The coil of a galvanometer is 0*02 x 0-8 m^. It
FFini

(b) series resistance 3988 O.]


consists of 200 turns of the wire and is in a
56. An ammeter gives full scale deflection with a
magnetic field of 0-20 T. The restoring torque current of 1 ampere. It is converted into an
constant of suspension fibre is 10“^ Nm/degree. ammeter of range 10 ampere. Find the ratio of
Assuming magnetic field to be radial (a) what the resistance of ammeter to the shunt resistance
is the maximum current that can be measured used.
[Ans. 9 : 10]
by this galvanometer if scale can accommodate
45° deflection ? (b) What is the smallest current 57. A galvanometer of resistance 50 Q gives full
deflection for a current of 0 05 A. Calculate the
that can be detected if minimum observed
length of shunt wire required to convert the
deflection is 01 degree ? [CBSE 2013 (ClJ
galvanometer into an ammeter of range 0 to
[Ans. (a) 4-69 x A (b) 1-56 x 1(T^ A] 5 A. The diameter of the shunt wire is 2mm and
51. The coil of a moving coil galvanometer has an its resistivity is 5 x 10~^ D m.
effective area 6 x 10“^ m^. It is suspended in a [Ans. 3T74 m in parallel]
magnetic field of 3 x 10"^ Wb m~^. If the 58. A galvanometer can be converted into a
torsional constant of the suspension fibre is voltmeter of certain range by connecting a
MOVING CHARGES AND MAGNETISM 4/123

resistance of 880 in series with it. When the large. Find the magnetic field induction at the
resistance of 420 Q is connected in series, the centre O.
range becomes half. Find the resistance of Ans. + 1
galvanometer. [Ans. 40 Q] 4jcrl 2 /.

59. The resistance of a pivoted type galvanometer


is 8 Q and current for full scale deflection on it 63. A current / = 1 A flowing through an infinite long
is 0*01 A. This galvanometer is to be converted wire A j divides between two infinitely long wires
into an ammeter of 5 A range. The only shunt Aj and A3 at junction B Fig. 4(Q).68. What is the
available is 0-02 SI. Find the value of R to be magnitude of the magnetic field at point 0 due
connected in series with the galvanometer coil. to currents through wires ?
Fig. 4(Q).65. FIGURE 4(Q).68

FIGURE 4(Q).65 I
I A2
>’a

ww
S -.1/2

B O. -►
z
R

FF loo
--W
I 1
T-m
2
[Ans. 1-98 fi] ▼1/2

ee
60. The voltmeter V in the Fig. 4(Q).66 reads 117 I

V and the ammeter A reads 0-13 A. The !As

ee r
00

rFrF
resistance of the voltmeter is 9000 Q and the
resistance of ammeter is 0-015 12. Compute (1) [Ans. Zero]
the resistance R, (ii) the power input to R.
rur
64. Two infinitely long straight current carrying
FIGURE 4(Q).66 ffoor
conductorsare held at right angles to each other
so that their common lies at the origin as shown
ks
<y> in Fig. 4(Q).69. If both the conductors cany the
YYoou
same current I as shown, find the magnetic field
ookos

at the point P (a, b).


-WAAV
BBo

R I
FIGURE 4(Q).69
re

[Ans. (/) 1000 n («) 13-689 watt]


P (a, b)
oouur

61. In a galvanometer there is a deflection of 10


ad

divisions per mA. The internal resistance of the


Y

r‘
galvanometer is 60 SI. If a shunt of 2-5 12 is
connected to the galvanometer and there are 50 4
dndY

0 I Q
Re

divisions in all on the scale of galvanometer


what maximum current can this galvanometer
FFini

read ? [CBSE 2001 (C)] [Ans. 125 mA] Ans.


47iab
[(fl+ft)+Va2+*2]
TYPE XII. TYPICAL PROBLEMS
65. A moving coil metre has the following
particulars. Number of turns, N = 20, Area of
62. A current I is flowing in a conductor shaped as the coil, A = 2-0 x 10~^ m^; Magnetic field
shown in Fig. 4(Q).67. The radius of the curved strength, B = 0-20 T, Resistance of the coil,
part is r and length of straight portion is very /?= 1212.

FIGURE 4(Q).67 {a) Indicate a simple way to increase the current


sensitivity of the metre by 20% (It is not easy to
change A or B).
D 1
◄- C (b) If in so doing, the resistance of the coil
inceases to 18 12, is the voltage sensitivity of
the modified meter greater or lesser than the
G
F 1
original meter ?
[Ans. (a) Increasing the turns by 20% (&) Lesser]
4/124 ‘P^indee^ '4. Fundamental Physics (XII) kTiVlII

66. In the Fig. 4(Q).70, a long wire AB is placed on 67. Find the force on the conductor carrying current
a table. A wire CD 10 cm long, is just above /} as shown in Fig. 4(Q).71.
AB. It can slide up and down on two vertical
wires. If the wires carry a current of 20 A, then FIGURE 4(Q).71
how much above AB, will the wire CD settle ? Q

The mass of the wire CD is 0-50 gram.


!/
I
FIGURE 4(Q).70 1
± Z

c D
*■ I

1^0 , (l+X^
B ●4 A Ans. 2njiog —

ww
47t
[Ans. 1*63 mm]

Floo
ee
For Difficult Questions

reer
rFF
1. Magnetic field at a point due to infinite long 10
linear conductor carrying current is = 10-'^ X ^ [sin 45°+sin 45”]
5x10
uur r
= 2-83 X 10-5 T
B =
4k r ffoor
Total magnetic field induction at 0
sks
B= = 4x2-83 X KT^
FIGURE 4(Q).72
YYoo
= l-13xlO^T
ooko

3. (0 Refer to Fig. 4(Q).74(a), the magnetic field


eBB

at point O due to current in wire I is


W ■4 E
10A
B -iio-x^ = 10"2x
2x3
= 10-5 j
r

1 -
4tc r 6x10
ouur
ad

Acting normally downwards


Y

S
Magnetic field at point O due to current in wire
dY

Its direction is given by Right hand thumb rule. 2x3


= 10-57
II is B^ = 10-2 X 6x10-2
Re

2. Refer to Fig. 4(Q).73,


nnd

OE = a = 5cm = 5x IQ-^ m. Acting normally upwards.


FFii

FIGURE 4(Q).73 FIGURE 4(Q).74


|4 10 cm >l I II I II
I I I I
^ D ●4 I
/
/
3A' ’
\0/ 6cm 6cm
10 cm 10 cm
¥ ■4-4* O o

^^45»i45
I
3A,. 3A,.
■ ‘ 3A
ei
E
¥
12 cm 12 cm
Magnetic field induction at the centre O due to o O
current in the arm AB of the wire is
Total magnetic field at O
B,1 = — — [sin 0, + sin 0, ] = fii -^2= 10"^- 10-5 =0
4n a ' ^
MOVING CHARGES AND MAGNETISM 4/125

(/O Refer to Fig. 4(Q).74(/»), total magnetic field


atO is
“ —[cos p + cos p]
4jc a
B = Ri + fij = + 10-^
= 2 X 10^ T acting normally downwards. b
4. Here,/=25A,r=l-5xl0-2m;B2 = 3xl(HT 4tc fl n a
Magnetic field due to straight current carrying —) —> —> —>
wire,
As ^d are acting in the same
2x25 direction, i.e., perpendicular to the plane of
^ 4ji r 05x10^ rectangular loop downwards,
B — B| + B2 + B3 + B4
This magnetic field will act perpendicular to
external magnetic field B2 (= 3 x 10"^ T). -}to / a b a b
Therefore, the magnitude of the resultant -y T+~+T+-

ww
magnetic field is
n
^|a^ + b^lb aba]
21

FF loo
B = = +(3xl0~*)^ n
●PT^lb a.
●V 3 y

ee
= 4-48x1(Ht _Vio2l^la^+l^

ee r
% ab

rFrF
5. Let Bj ,^2 * ^3 ^4 die magnetic fields at
O due to current in arms 1,2,3 and 4 respectively. 6. Magnetic field at P due to current /j is
rur
Fig. 4(Q).75. Hlh
The total magnetic field induction at O due to
current through rectangular loop is
ffoor
B,= 4jt fl ;’ directed normally inwards
ks
Magnetic field at P due to current I2 is
YYoou
B ssBj+B^+Bj+B^ B =iio£^.;
okos

directed normally outwards


^ An b
BBoo

RGURE4(Q).7S
Net magnetic field at point P is
r ee

P 1 Q
1
B = B2 - Bj; directed normally outwards.
oouur
ad

4^‘I <

4Jt lb
Y

a.

I
<■ 4 2
= 10-^x2
dndY

S
Re

3 R
a H .3x10-2 4x10-2.
FFini

= 1-56 X lOnS T
where, Bj = ^ = ikJL
4jc (b/2)
7. Mag. field induction at O due to current through
one wire is
X [sin (90® - a) + sin (90® - a)]
D
B we have
1 “
An r
- -^4-[cos
4tc b
a+cos a]
? = il!L2xl i
An Ll 2 4 8 16
= £!Llix2cosa
4jc b = liaix^^ n
=iia ( 1 1 Aa (\ 1 ^ A

2nv]*4~^T6*-
and B,2 sB.4 = ^ - ^ . Ho Ff 1
—]k]
/
4„(^) 1
2jc *-2 1
X [sin (90® - P) + sin (90® - P)]
l'-4j
4/126 “p>uuleefr’4. Fundamental Physics (XIl)EEai
A
11. Charge on a-paiticle, ^ = + 2 e, where e is itre
charge of an electron. Thus
2ic3 4jc 3
^ = 2 X 16 X 10rl9 C.
= 10-'^ x-Jk = 1-33X10"^ T* r = 2 X 10-10 m; u = 2 X IQO ms-1
3
Current l = S-~ q
T iTorh) 2nr
8. Given,2icr = Ji/2or r = - 0-25m ;/ = 5 A
4
Magnetic field at the centre of circular path
When wire is straight, the distance of point,
/ = 0 01 r = (0 01 X 0-25) m. 5 = Up 2nl
4jc r
_\^qI
2r
^\iof qoY\^qv
2r 2nr 4jc
This distance is very small as compared to length
(n/2) m of wire. Hence wire can be taken of
infinite length.
1(T7 X(2x16x10-19)x(2x1QO) = 1-6T
(2x10-10)2

ww
Magnetic field induction due to straight wire is
12. Let R be a unit vector perpendicular to the plane
B- 1^2/^ = 4xlO-^T of paper upwards. Then total magnetic ISeld at O
4n r' (MlxO-25)
due to current through the given structure is

Flo
Magnetic field induction at the centre of the
circular loop carrying current is = 0+^-0(-r) +0

ee
^ = ^ab-^^bc-^^cd 4jc r

rere
lip 2nl _ 10~^x2tcx5 = 1-256 xl0r«T
[As point O lies on the wire ab and cd, so

rFF
B =
4jc r (0-25)
uurr
foor
ev

9. (0 T = 2 jc riv and i = ^
10
2nr
. B=^!2.Z^e=io-2x n
X-
4jcr (20x10-2) 3
ks s
(iO i = ev = 5-2xl(r^T
Yoo
oook

13. Here, / = 4 A ; r = 10 0 cm = 010 m ;


Now use, B= ftp 2m ^ fiQ 2x ev
As = 90“ and PO = OS = r;
An r 4;r r
eBB

so Z0P5 = Z05F = 45“


10. Here, r = 10 cm = 010; P| = 0-50 x 10-^ T, Angle subtended at O by curved wire PQRS
jc = 4 cm = *04 m
rr

3ic
ouu

Magnetic field at the centre of a circular coil = — rad


ad

carrying current is
YY

Magnetic field at O due to current through portion


\Iq 271/ lip/ SP of wire is
B,=
nndd
Re

4ji r 2r .0 . e
sm — + sin -
1
Magnetic field due to circular cuirent loop at a 4tc r cos 0/2 L 2 2J
FFii

point on the axis of loop, distance x from the


centre of loop is
4jt r 2

(ip 2nlr'^ It is acting normally downwards.


"2”4;r (r2+x2)3/2"2(r2+;c2)3/2 Magnetic field at O due to current through portion
PQRS of wire is
B r3
'● B1 (r2 + x2)3/2 4tc r 4ic r 2

(0-10)3 It is acting normally downwards.


= 08
[(0-1)2 +(0-04)2 ]3'2 Total magnetic field at point O is

X 0-8 = (0-50 X 10-^) X 0-8 1^0 -


/ 2tan- + —
= 4xl(rST
B = Bj + = An r 2 2.
MOVING CHARGES AND MAGNETISM 4/127

Net magnetic field is


= 10-7 ^ _1 2 tan
90°
+ —
3k
01 2 2 ,
B =
+ =||t(l)2+(V3)2p
= 10-7x40 2tan45° + —
2 ^^0
= -- T {in X-Z plane)
R

= 10"7 x40 2xl + - 7C I 5 ,


L 2 16. Here ^ = /, = -A, Ob = r = 5 X 10-2 m
I 2 2 2
= 2*68x 10“^ T acting normally downwards. Total magnetic field induction at O due to
14. Refer to Fig. 4(Q).76, current in the net work is

FIGURE 4{Q).76 B — Bab + Bbcd + BbedA' Bdf

ww
n + ^nl_ A
i-n) +
Mo / ^
n
4;r r 4;r r 4;r r 4tc r
A

Flo
where n is a unit vector normally out of the
plane af paper.

ee
rere
A
. S=2x = 2xI0"7x—~— n

rFF
4n r 5x10-2
4A
= 2 X10“^, normally out of the plane of paper.
uurr
foor
17. Bo = Bcg + Boef

_ Ho 2/A_ Mo 27tl^n=—
Moi\\ +. .1
ks s
A

_ 1^0 ^1 (471X10 7)x3 nH n


Yoo
4;r r 4jt r 2r _ji
= 127txl0-^T
oook

2r
2x(5x10-2) A

where »i is a unit vector perpendicular to loop


eBB

B _ |4q^2 _ (47txl0~7)x4 upwards.


Q ” = 167TX 10-^ T
2r 2x(5x10“2) 18. We kn(>w that magnetic field induction at the
rr

centre of an arc subtending an angle 6 is


ouu

Net magnetic field


ad

B =
YY

B = 4tc r

Total magnetic field induction at O due to


nndd

= Va27i:xl 0-^)2 +(16tcx10-^)2


Re

current through the three arcs is


Fii

= 471X10“^ a/9+ 16 =(4tix10“®)x5 B=-^/ J 1 1


1 1 0^ = 5^o/e
4tc r 2r 3r 247cr
= 20 x 3-14 X 10^ T = 6-28 x KT® T
The direction of current I is not given, hence, 19. Fig. 4(Q).77, represents the section of solid metal
—^ wire of radius R, perpendicular to its length.
direction of B cannot be predicted.
15. Refer to figure of the hint of above question, we FIGURE 4(Q).77
have

_ _ Hq X 1 _ Itp
Bp = acting along Z-axis
2R 2R 2R

B _ ^0 ^2 _ 1^0 ^ M^O
^ 2R 2R 2 R
acting along X-axis
4/128 Fundamental Physics (Xn) VOL.I

(i) Let the point ? be lying inside the wire at a


perpendicular distance r from the axis of the wire.
Consider a circular path of radius r around the
/ = |i2)[r’</r = i^
axis of the wire. By symmetry, the magnetic field Using Ampere circuital law, we have
produced due to current flowing in the wire at
any point over this path is tangential to it and
equal in magnitude at all points on this path.
Current enclosed by the closed path nkr^
B2nr = Po^ — or B =
i 4
xjcr^ =ir^!R^
nR^ («*) If r > fl, then net current through the closed
path of radius r is.
Line integral of B over this closed path is
:-2

ww
ir

R^
(From Ampere’s circuital law) Using Ampere circuital law, we have

FF loo
or B2nr =
.*. B = 2
2itr/?2 2jc«2

ee
ee r
(k*) When point P is outside the wire, r > /? so B =

rFrF
4r
that the current enclosed by the closed path = i
According to Ampere’s circuital law NI
rur
22. (a) B = Poy;
j>B.dt =\lQi or F2Jir=Hfli ffoor
Here, / = 0-20 m ;iV= 120; /= 2-5 A
ks
or B =
YYoou
2jtr (h) S =
ookos

21
20. Here,i? = 6cm = 6x 10"2m;/ = 8A; 23. Refer to Fig. 4(Q).79, the magnetic field due to
BBo

(a) r = 3 cm = 3 X 10^^ m. Point is inside the


current through solenoid, is directed parallel
re

conductor
to the axis of wire where B^=4 mT. The magnetic
Po Ir (4itxlQ-7)^^8x(3xl(r^) field due to current through straight wire is in
oouur
ad

B =
2jc/?2 2jc (6x10-2)2 the form of concentric circular magnetic lines of
Y

= 1-33 X 10-® T force. On one circular magnetic line of force, the


magnetic field induction at all points is same in
(h) r = 10 cm = 0*10 m. Point is outside the
dndY

magnitude. The magnetic field induction due to


Re

conductor
current through straight wire at a perpendicular
FFini

distance r (= 3 mm) is
B = !!!-1£=,o-’x125
010
= 1*6 X KTS T
An r FIGURE 4(Q).79 ^

21. (0 Consider an

elementary ring of radius


r thickness dr whose
centre lies on the axis
i3mm^ 1

of the conductor. Area of


the elementary ring,
dA = 2 n r dr. Current
passing through the B = i^2/
elementary ring is
w
47t r ' It is acting ±r to B^
dl^JdA^ (kr^) (2n r dr) = k2nt^ dr 10-2x2x20
Total current passing through the closed path of = 1-33 X10-3 T _ 1.33
3x10-3
radius r is.
MOVING CHARGES AND MAGNETISM 4/129

The resultant magnetic field induction at P in (if) Frequency of revolution,


magnitude
eB _(16x10-^9)x(5x10-3)
B = ^42 + (1.33)2 ^ 4.2„x 2jcm 2x344x(9-1x10-31)
= 1*4x10* Hz
1-33
tanO = = tanl8-4‘’ 28. Here,B = 002T.r = 6xlO-2 m;
4

w
pitch = 24 X 10-2 m
or 6 = 18*4** with the directed axis
Let be the component of velocity of proton
24. Length of wire used, I = 2 tc r x no. of turns perpendicular to magnetic field. Then
= 2 7C X (2 X 10"2) X 500 m mv
_ rqB

e
1
Resistance per unit length = 0-02 O m"* r — or V
1 “
qB m
Total resistance of wire,

e
o
rw
/? = 2 7C X (2 X 10-2) X 500 X 0 02 = 0-4 jc Q _ (6x 10-2)X (1-6X 10-^9)X 0-02

r
V
1 “
No. of turns per unit length, 1-6x10“27

eF
= l*2xl0®ins-l

ullo
500 5000 -1

FF
n = m
30x10^2 3 Time period of revolution,

j _ 2icr _ 27tx6xl0~2

srre
1-2x10^ = 7ix 10^ s
As, B = \i^nlr=fj,Q~ V
1

oF
R

k
Let V2 be the component of velocity of proton
BR 10~2x(Q.4^) along the direction of magnetic field. Then

fofr
uor
So, e =
PqTi
25. Here inner radius of toroid,
(4 3T X10"’) X (5000/3)
= 6V

= 10 x 10"2
o V I* =
^
pitch
^

time period
24x10“2
^

jcxlO"^
kos
m
= 7*6 X 10^ msT^
YYo
rBB

Outer radius of toroid, r2 = 20 x 1(T2 m


oo

29. Here, V= 100 V;B = 0 004T;r=?


Y

B
= ^) N ^_(4tcx10-2)x600x10 Let V be the velocity acquired by electron when
r ee

max.
2 75^2 2tcx (10x10-2) accelerated through a pot. cliff. V. Then
u

/
= 12x Kr3T:^12mT eV =-mv^ 2eV
Yd

OT V =
oou

2
ad

^min. N ^_(47cx10-2)x600x10 As the electron describes a circular path of radius


id n

2 jcr2 2jcx (20x10-2) r in the perpendicular magnetic field, so


= 6x 10-2T = 6mT mt;2
Re

-evB
in
FF

1
1&^2
r

26. y = -crr2=:
-
2 m
or r =
mv _ m
eB eB V
j~2eV _ ^2meV
m eB
_ 10^ X (1-6 x1Q-*9)x (2x10-3)2
2x9-1x10-3* . V2x(9lxl0-2^)x(1.6xl0-^^)xl00
= 3*51xl0®m (1-6 X10"^^) X 0004
27. Here, 5 = 5 x 10~3 T, v = 5 x 10^ ms-*, = 84 X l()-2 m = 8*4 mm
e = l-6x l(r*® C, 30. Let Vj, V2 be the rectangular components of
m = 9-1 X 10-3* kg proton’s velocity perpendicular and parallel to
(0 Radius of the circular path, the direction of magnetic field. Then

mv _ (9-1X IQ-3* ) X (5 X 1Q2 ) Uj =usin60® =(4x10^)xV3/2


r =
= 3 464 X 10^ -1
Be ~ (5x10-3)x(1-6x10-*®) ms

^2 = V cos 60® = (4 X 10^) X 1/2


= 5-7 X 10-2 m = 5*7 cm
= 2 X 1()5 ms-*
4/130 7^n4ideefr ^ Fundamental Phjrsics (XII) VOL.I

The component velocity Uj makes the proton to


move along a circular path of radius r, given by For proton, = = 7cm
eB
mv}
K
Fordeutron, =
eB
mv (l-67xl0~^'^)x(3>464xl0^)
or r =
1 _
2x2mpXK
qB (1-6x10-19)x0-3 eB — =-j2r^ s^llxlcm
= 1-2 X 10-2 m = 1*2 cm
a
Time period of revolution of proton For a-particle, = 2eB

^_2nr 2x3a42xb2xl0~2
I

ww
2x4mpx2K
V3464x105
1
2eB
— = V2 s-Jlx 7cm
= 21-75 X 10-® s
33. Here, £ = 100 kV/m =100 x lO^ V/m = lO^ V/m,
The component velocity V2 of proton, makes the S = 50mT = 50xl0r3T

Flo
proton to move along the direction of magnetic

e
field.
_£ = _J5L_

eree
=2X ms-^
Therefore, due to velocity components v i and V2, B “50x10-3
the proton moves on the helical path.

FFr
34. Here,V=10kV=10xl03v=10^V,
Pitch of the helix, p = V2y-T
uurr Mass of a-particle,
= (2 X 105) X (21-75 X 10-*)

orr
= 4-35 X 10~2 m = 4*35 cm
m
a = 4 /lip = 4 X 1-67 X 10-22 kg .
sfo
Charge of a-particle,
,„ = 2,p = 2xl.6xlO-‘»C.
lip 2/ l(r2x2xlO_^Q_S.j.
kks
Yoo
31. B =
0-20 In a cyclotron, an a-particle gains energy ^
oooo

4jc r
when it crosses a region of potential difference
It is acting perpendicular to the plane of paper V. In one revolution, the a-particle crosses the
eBB

upwards. gj^ twice between dees, so the energy gained in


F-qvBsm 0 each revolution = 2 q^V
= 1-6 X 10-^® X 105 X 10^5 X sin 90®
uurr

Let a-particle makes n revolutions before


= l-6xlO-*^»N coming out of the dees. The gain in its kinetic
ad
Yo

energy will be
The direction of F [= q(v x B)] is in the plane
dY

2
of paper away from XY»
^m^vl=n2q^V or « =
innd

32. Charge on proton = e, charge on deutron = e.


Re

Charge on a-particle = 2 c.
c ^ 3x10*
Fi

Given, va = 3 X 102 ms-*


As = — mu2 = K (kinetic energy) 10 10
2

So, Ko^q n =
(4x1-67x10-22)x(3x1Q2)2
e: e:2e 4x(2x1-6x10"^9)x10^
-●- Kpi Kj.’ K^ = qp : q^: q^ =
=1:1:2 = 470 revolutions
Radius of circular path of a charged particle in a 35. Here, v = 10 x 10^ Hz = 10^ Hz ; r = 0-60 m
magnetic field is m = 3-3 X 10-22 kg ; ^ = 1-6 X lO-^® C
r -
mv ^2mK As v =
qB .
so, B=
2nmv

qB qB 2jim’ q

2 X 3-142 X (3-3 x10-22)x1Q2


/
1
V K = -mv2 ormv = .J2^ B = = 1*3T
2
(1-6x10-*®)
MOVING CHARGES AND MAGNETISM 4/131

Max. KE of the emerging deutron is 39. In equilibrium position. Fig. 4(Q).80

K
max
2m

(1-6x10-^^)^x(F3)^x(0-6)^ j
2x(3-3x10-2'^)

(l-6xl(ri^)^x(F3)^x(0-6)^ MeV Bil cos 0 = mg sin 0


” 2 X (3-3 X ) X (1-6 X IQ-)
or i = ^«S£8=f“Uuuie

oww
= 14-74 MeV BlcosQ yljB
36. Here, = 24 T, m = 1*67 x IQr^ kg ; 0-3 X 9-8 X tan 30”
= 11-3 A
^ = 1-6 X 10-19 C. 015
Time taken by proton to complete semicircular 40. Here, / = 30 cm = 0-30 m ; = 0-3 T, / = 4A,

e
path

FFrlo
0= 180”-120” = 60”

re
T _ 2nm _ nm Force, F = /B/ sin 0 = 4 x 0*3 x 0-3 x sin 60®

rFee
2~ 2Bq ~ Bq = 4x0-3 X 0-3 X S/2 =0-31N

rF
3-14xl-67xl0~^'l 41. For current in wire CD, the angle between /
ouru
24x1-6x10-19
and B is 120®, Le., 0 = 120®. Therefore, force
= 1-37 X 10t8 s
fsoor on wire CD is
skf
The direction of electric field should be erversed
Fcd =/[(^)xB] =//iBsinl2(y’ n
ooko
after every 1-37 x 10^ s.
Yo
Hme period of oscillation of electric field is = //fisinl20® = //Bx S/2
Y
or F,CD
Bo

r=2r = 2x 1-37X 10"*s Its direction according to Right Hand Rule is


reeB

Frequency of oscillation of applied electric field nonnally out of the plane of paper.
1 1
ooY

For arm DE, the angle between / and B is


uur

''“r” 2x1-37x10-* zero Le., 0 = 0®. Therefore, force on wire DE is


ad

= 3-64x10^ Hz //Bsin0® = 0
dY

37. Here, /=4A. This current divides equally in arms For arm EC, the angle between / and B is
nind

abc and adc, Le., the current through each arm


Re

120® Le., 0 = 120®. Therefore, force on wire EC


of square is, F = 2A. Length of square is
FFi

/ = 0-10 m; B = 0-20T; 0 = 90®.


Magnitude of the force on each arm of square Fc£/(CF)5sin 120® = //Bx S/2
frame is Its direction is along normal into the plane of
F = r/Bsin0 = 2x (0-10) x (0-20) x sin 90® paper.
= 04VtN 42. As three parallel wires are of equal resistance
and the total current supplied is 12 A, so current
According to Fleming’s left hand rule, the forces
on arms ad and be will be towards irght and on
through each wire is 4 A.
arms ab and dc will be downwards in the plane {a) Force per unit length of AB due to current in
CD is
of square firame.
38. F = Bil sin 0 = mg 2x4x4
F-Hq. =
' 4jc r (2x10-2)
or B =
mg (3x1Q-3)x9-8 = 0-1176 T
i7sin0 5 X 0-1X sin 30® = 1-6 X 10"^ Nm-^ directed upward
4/132
Fundamental Physics (XII) VOL.I

Force per unit length of due to current in EF (/i) When the direction of current in wire (3) is
2x4x4 reversed, i.e., in upwards direction. Fig. 4(Q).81,
is F^ = = 10-2 X then the net force on wire (2) will be zero if force
47t n
1 (4x10-2) on wire (2) due to wire (1) is equal and opposite
= 0’8 X 10"^ Nm-^ directed upward to the forcp on wire (2) due to wire (3), i.e..
Total force per unit length of AB is
F = Fj + F2 = 1-6 X 1(H + 0-8 X lOr^ FIGURE 4(Q).81

= 2*4 X 1(H Nm"^ directed upwards (1) (2) (3)


(b) Force per unit length of CD due to current in

oww
AB is
I1- i LI2
2x4x4
^ 4tc r (2x10-2)

e
= 1*6 X 10^ Nm~* directed downwards

re
Force per unit length of CD due to current in EF

FFrllo
is

reF
2x4x4

e
= 10-2 X
2 4jc r 2x10-2
ouru = 1*6 X 10"^ Nm-* directed upwards

osFr
^0 _ ^**0 ^^^2
= - 1-6 X 10"^ Nm“^ directed downwards 4jc a 4jc a
or / = /1

Total force per unit length of CD is


F' = Fj' -I- F2 = (1-6 X 10-^) -I- (-1-6 X 10-^) = 0 fkfor 46. Here, n = 200, r = 0-10 m ;
kso
A = jcr2 = jc (0-1)2 m2,
ooo

43. Force on long conductor is equal and opposite to S = 0-5T,e = 0“;/ = 3-0A
YYo
BB

the force on small conductor = (a) Torque = n lAB sin 0 = 200 x 3-0
4jt r X [7t X (0-1)2] ^ Q.5 gjjj Qo
r ree

Here, 7| = 4-0 A; /j = 3-0 A ; = 0


Y

r = 3-0 X 10"2 m; / = 5-0 X 10"2 (b) Magnetic forces on the two semi circular parts
oouu

ntL

44. Force on R due to wire P is of coil are equal and opposite acting in the same
ad

plane, hence the total force on the coil is Z',ro.


Yd

_ Pq 2/j I2 I _10-2x2x5x5x(0-10) (c) Here A = KT^ 1(P^ m'^


(5x10-2)
nidn

Force on each electron


= 10-^ N towards P
Re

Force on R due to wire Q is F = evB = e B


[NAe]
FFi

p _ ^0 _ IQ-2X2x10x5x(010)
^ Tj (5x10-2) V / = NAev or v =
NAe
= 2 X 10-^ N towards Q
Net force on F = F2 - Fj = 2 x 10“^ - 10"^ p-I^ _ 30x0-5 = 1-5 X 10-2^ N
= 10-® N towards Q NA 1029x10-5
45. (0 The net force on wire (1) will be zero if the 47. X = nIBA sin a ; Here, n = 200,7= 2-5 A ;
force on wire (1) due to wire (2) is equal and B = 0-25 T ; a = 90® - 60“ = 30“,
opposite to the force on wire (1) due to wire (3).
It is so when current direction in wire (3) is
A = crr2 =
downwards. Then
yx(0-05)2m2
F-q _ M-q 2/j7 48. Here,B = 3000G = 0-3T; 7= 12 A
4tc a 4jc (a-fa)
or
7=272 A = 10 X 5 = 50 cm2 - 50 x 10“^ m^.
4/133
MOVING CHARGES AND MAGNETISM

ia)x max. = MB sin 90“= 12 X (50 x 10“^) x 0-3 % decrease in voltage sensitivity
= 1-8 X 10-2 Nm / / \
V -V'
^-xl00= 1—^ xlOO
{b) Torque is zero when sin 0 = 0 i.e. 0 = 0“ or 180° Vs
(c) Loop will be in stable equilibrium if A is (
= 1-- XlOO = 25%
_> —>
parallel to B and in unstable equilibrium if A ^ ^ -
is antiparallel to B .
—>
53. Voltage needed. V = 30 mV = 30 x 10-^ V
Deflection of division, n = 30
49. Let the coil of triangular area A carrying current
30
I be suspended in a uniform magnetic field
5 (= 5 X 10“2 T). Let 0 be the angle between mag Voltage sensitivity, = ^ = 30x10"^
netic field and normal to the surface of the loop. = 10^ div/V
1

w
Here, area, A = — (base x height) Resistance of galvanometer is
2div/|iA 2x10^ div/A
base = 0-02 m;
i^"l(P div/V ” 1()3 div/V

Flo
height = 0*02 sin 60° = 0*07132 m.
= 2000 0
Then torque acting on the closed loop is

ee
X = BIA sin 0 0 nBA
54. (a) Current sensitivity h~~j~

Fr
k
= (5 X 10-2) X 0*1 X - X (0*02 X 0*07132)
X sin 90° Since it is easier to change n than k, A or B,
therefore the current sensitivity can be increased

for
ur
= 8-66xl(r7Nm
by increasing n. In order to increase the current
50. Here, A = 0*02 x 0*8 m^ ; n = 200 ; sensitivity by 25%, n should be increased by 25%.
B = 0*20 T; k - 10"^ Nm/degree Therefore new value of n should be
ks
Yo
(a) Imax = ?;0 = 45°;(fe)/„^. = ?;O = O*l° 25 125
-x24=30
oo

n' = n + n =
100 100 4
Use the relation, I = iLe
B

nBA (b) New resistance, R' = 14 + 7 = 21 S2. Voltage


51. Here, n = 1, A = 6 x KT^ m^, wBA ^24BA _.1*7BA
re

L
B = 3 X 10-2 wbm-2, cf = 5 X 10-® Nm deg"^ sensitivity =-^ = kR “Jtxl4~ k
ou
ad

fl nBA New voltage sensitivity.


Current sensitivity / =- = BA
n'BA _ 30BA
Y

^ I k
1*4
V= kR' cfx21 k
1x(3x10-2)x(6x1Q-2)
nd
Re

5x10-9 So,
V/ 14
= — <1 or y/<Vs
= 3*6 X l()5 deg A"* 1*7
Fi

3*6 xlO^ 55. (fl) For conversion into ammeter :


10®
deg/pA =0*36deg/pA G = 12*0 Q, /_ = 2*5 mA = 2*5 x 10"3 A,
/ = 7-5A

k-.«'=2R
150
52. Here,// = /^ + -Is
100 * = 100 S =
IG
g (25xlQ-^)xl2 2*5x12x10-2
I-I ~ 7*5-(2*5X10-2) - 74975
Initial voltage sensitivity. o

= 4-0 X 10^ Q in parallel


(b) For conversion into voltmeter
R
G = 12*0 £2, /„8 = 2*5 X 10"2 A ; V= 10 V
New voltage sensitivity.
V 10
/? = ^—G = -12
ll I 2*5x10-2
8
V' = —
s
R’ 2R 4
s
= 4000 - 12 = 3988 Q in series
4/134
7^n<ideefr '4, Fundamental Physics (XII) VOL.I

IG
56. S =
IxG G 61. Since the galvanometer has 50 divisions, so

current for full scale deflection is


/-/. 10-1 9
z
1
Resistance of ammeter formed
^g=J^x50mA =5mA = 5xl(r3A
K ^■^xg^(C//9)xG G/9 G G = 60Q,5 = 2-5 Q.
^ G+S G + G/9 10/9 “10 Let / be the maximum current which a
galvanometer can read when shunted with
_ G/10 ^ 9 resistance 5, then
S G/9 10
IG IS
I
57. S = 8 005 x 50 _ 0-05x50 _ 8
G + S
I-I 8
5 - 0-05 4-95 “99^

ww
Sjir^ 50.^22 X' (10~3)2_ or ^ _ /g(g + -y) (5x10-3) (60+2-5)
Now, / = — — X S 2-5
:
P 99 7 5x10-’
= 125 X 10-3 A = 125 mA
= 3174 m

Flo
58. Let G be the resistance of galvanometer. If a

e
resistance R is connected in series of galvano 62. Bq +Bjy^p+BpQ ; Here, Bqj^ = 0;

eree
meter it works as a voltmeter of range 0 to V
volt. The current for full scale deflection of

FFr
a
B _^0 4
DEF ~ n
4jt
uurr
voltmeter is I
V r

orr
8
R +G
^G = ^-Isin90°+sin0"lJ
sfo 4jr r
= if2.i.i
4„r
In firs, case,
kks
= 0 + |!Lx^/2+^'.5
Yoo
^0
oooo

V/2
4jr 2r 4;r r
V
In second case, / =
8
eBB

420-t-G 2(420-fG) + 1 n
A

4jrr^ 2
V V
uurr

880-fG 2(420-fG) where n is a unit vector perpendicular to the


plane of conductor outwards.
ad
Yo

or 880-HG = 840-t-2G or G = 40fi 63. Let Mag. fields at O due to current through wires
dY

59.
Ig(G + R) = (I-gS Ai,A2,A3be B2 and B^ erspectively. Then
innd
Re

or 0-01 (8 -I- /?) = (5 - 0-01) 0-02 = 4-99 x 0-02 B^ =0, V O lies on the axis of wire Aj
Fi

4-99x0-02 R
OT R = -8 = 9-98 - 8 =1-98Q [sin90'’+sin0'’] (-k)
0-01 2 4jt(l/2)
117 = l(r’ X 1 X 2 [1 -I- 0] i-k)
60. Current through voltmeter, /| = 9000
= 0-013 A
= -2x10-’ k
Current through /f, /j = / - /,
= 0-13-0-013 = 0-117 A [sin0®-fsin90“] k
^ 4tc(1/2)
117
(1) Resistance, R = = iooon
= 10-’xlx2[0+l] k =2x 10-’ k
0-117
Total magnetic field at O is
(I'O Power input to R = I^R -» -» -4 -»

B = B^+B2 + B^
= (0-117)2x1000
= 13*689 watt = 0 -<■ (- 2 -I-10-’ k ) -H (2 X 10-’ k) = 0
MOVING CHARGES AND MAGNETISM 4/135

64. PRefer to Fig. 4(Q).82, mapetic field at point


due to the conductor/ is
V'
Hence -^ = 5l* = “xH = l = o«3<I
It'll 18 20 6

or
V'/<V,
Therefore, the voUage sensitivity ofthe modified
meter is lesser than the original meter.
66. Let the wire CD settle at distance r above the
wire AB. The magnetic force on CD of length I

w
due to current in is F =
4jc r

Here, I = 10 cm. = 01 m, /| = I2 = 20 A

e
10-'^x2 x 20x 20 x0 1 8x10-^

re
row
a
F = N
&=—- sine.+sin— i-z r +1
^ b\_ ^ 2\ 47cfc \_^Ja^+b^
r r

Since the currents in AB and CD are in opposite

eeF
Its direction is normally into the plane of paper. directions, therefore force on CD will be

ullo
Magnetic field at point P due to conductor II is

FF
upwards. The gravitational force on CD acting
vertically downwards will be
-ikZ—● ● n X ● "1 b
«2 = sin0.,+sm— — ■ /-7 r +1 Fi = mg = 0-5 X 10"^ X 9-8 N

srr
4tc a 2 2j 4ita ya^+b^

roF
In the state of equilibrium, F = Fy

k
It is also acting normally into the plane of paper. 8x10"^
= 0-5x10"^ X 9-8
uor
Resultant magnetic field at .P
5 = Sj + B2
ofof or
r

8x10-®
kos
or r =
-7 = 1-63 X 10“^ m
1
f-+-l 0-5 X 9*8x10
YYo
a I w
eerBB

= ~ -+- +
oo

[^U^{b a) 67. Refer to Fig. 4(Q).83, the magnetic field


rY

induction at a point distance r from the long linear


a +b
conductor carrying current is
u

4n ab ab FIGURE 4(Q).83
oou

Q
d
ad

I
dr
= + b^ +(a + b)
nY

4nab L I i, t
r
nid
Re

e nBA
65. (0 Current sensitivity, / = — =
^ I k
FFi

I
As, it is not easy to change A or 5, hence
sensitivity can be increased either by increasing ^2/
B =
n or by decreasing k. As it is easier to change n, 4jc r
than k hence current sensitivity can be increased
by increasing n, /, can be increased by 20% if n Force on the current element of length dr carrying
is increased by 20% i.e. by increasing n ftom 20 current /j is
to 25. dr
^‘0
„ e nBA
dF =
4jt r
1
4jc 2//. 7
(ii) Voltage sensitivity, \-y~ Total force on the conductor of length / placed
Voltage sensitivity of modified meter will be perpendicularly is

n'BA
F =
v= kR' 4n ^ i r 4n 1 x
X
4/136
T^'tadecfi.'A Fundamental Physics (XIDCSSQ

WITH
SOLUTIONS

Q. 1. A circular coil of wire consisting of 100 turns, each of radius 8-0 cm carries a current of 0-40 A.
What is the magnitude of the magnetic field B at the centre of the coil ?
Sol. Here, n = 100 ; r = 8 0 x 10"^ m ; / = 0-40 A ; S = ?

B =
^0Innl = IO-’x2x^x'“xO'' = 3-1 X 1(H T
4tc 7 8x10"^

ww
Q. 2. A long straight wire carries a current of 35 A. What is the magnitude of the field Bata point 20 cm
from the wire ?
Sol. Here, / = 35 A, r = 20 cm = 20 x 10“^ m ; S = ?

Floo
B = = 2X35 = 3*5 X 10-5 T

ee
471 r 20x10*^

reer
Q. 3. A long straight wire in the horizontal plane carries a current of 50 A in north to south direction.

rFF
Give the magnitude and direction of B at a point 2*5 m east of the wire.
uur r
Sol. Here, / = 50 A ; r = 2-5 m ; B - ?

B =
ffoor
sks
4jc r
= 4 X 10-5 T
2-5
YYoo
ooko

It acts upwards, perpendicular to the plane of wire.


Q. 4. A horizontal overhead power lines carries a current of 90 A in east to west direction. What is the
eBB

magnitude and direction of the magnetic field due to the current 1*5 m below the line ?
Sol. Here, / = 90 A ; r= 1*5 m.
r
ouur

Rq 2/ _ 10-7x2x90
ad

Now, B = = 1-2 X 10-5 T


4ti r 1-5
YY

It acts towards south.

Q. 5. What is the magnitude of the magnetic force per unit length on a wire carrying a current of 8 A
nndd
Re

making an angle of 30® with the direction of a uniform magnetic field of 0*15 T ?
Sol. Here,/ = 8 A ; 0 = 30® ; B = 0-15 T ; F = ? ; / = 1 m.
FFii

F = B / / sin 0 = 0-15 X 8 X 1 X sin 30° = 0-15 x 8 x 1 x (1/2) = 0-6 Nm-^


Q. 6. A 3-0 cm wire carrying a current of 10 A is placed inside a solenoid perpendicular to its axis. The
magnetic field inside the solenoid is given to be 0-27 T. What is the magnetic force on the wire ?
Sol. Here,/= 3 0 cm = 3 x IQ-^m ; / = 10 A ;B = 0-27 T, 0- 90® ; F = ?
F = B / / sin 0 = 0-27 x 10 x (3 x 10“2) x sin 90® = 0-27 x 10 x 3 x 10“^ x I = 8*1 x 10'^ N
The direction of force is perpendicular to the direction of current as well as of magnetic field.
Q. 7. TVo long and parallel straight wires A and B carrying currents of 8 0 A and 5-0 A in the same
direction are separated by a distance of 4*0 cm. Estimate the force on a 10 cm section of wire A.
Sol. Here, /, = 8-0 A ; /2 = 5-0 A ; r = 4-0 x m ; / = 10 cm = 0-1 m ; F = ?
2x8x5x0-l
F = ltO_LJ./ = 10-7x = 2x10-5n
4;u r (4-Ox 10-2)
This force is attractive force normal to A towards B.
MOVING CHARGES AND MAGNETISM 4/137

Q. 8. A closely wound solenoid 80 cm long has 5 layers of windings of400 turns each. The diameter of the
solenoid is 1*8 cm. If the current carried is 8*0 A estimate the magnitude of B inside the solenoid
near its centre.
Sol. Here,/= 80 cm = 0*80 m;/V = 5 x 400 = 2000;/= 80 A ; Z)= 1-8 cm.
Magnitude of magnetic field induction at a point well inside the solenoid is

B = 10-7x2000 x 8.0 _ ^ v i»-2t


/ 0-80

Q. 9. A square coU of side 10 cm consists of 20 turns and carries a current of 12 A. The coil is suspended
vertically and normal to the plane of the coil makes an angle of 30® with the direction of a uniform

oww
horizontal magnetic field of magnitude 0*80 T. What is the magnitude of torque experienced by the
coil?

Sol. Here, /= 10cm = 0-10m; n = 20 ;/= 12 A ; a = 30®;= 0-80T; x = ?


Area, A = / x / = 0*10 x 010 = (O l)^ m^

e
X = n 5 / A sin a = 20 X 0-80 X 12 X (01)2 X sin 30® = 0-96 Nm

re
FFrllo
Q. 10. Two moving coil metres M| and M2 have the following particulars :
l?i = 10n;iVi = 30;Ai = 3-6xlO-3m2;lli= 0-25T;

rF
ee
«2 = 14 ft,iV2 = 42, Aj = 1*8 X l(Hm2; J82 = 0-50 T
(The spring constants are identical for the two metres). Determine the ratio of (a) current sensitivity
ouru
rF
and (^) voltage sensitivity of M2 and Mj.
Sol. For meter Mj, /?j = 1012 ; A/] = 30 ; Aj = 3*6 x 10“^ m^; = 0*25 T ;
fosor
For meter Mj^ R2 = 14 a, iVj = 42 ; Aj = 1*8 x 10"3 m^; Bj = 0-50 T ; *2 = cf.
os kf
As, current sensitivity = NB AJk.
ook
Yo
Y
. . „
(fl) So,
^^7 =
42X 0-50x 1-8 x 10"^ —
/Ik = 1-4
Bo

N^B^A^/ki 30x0-25x3-6xl0"^/fc
reeB
oouY

NBA
(b) Voltage sensitivity =
ur

kR
ad
Yd

So,
\ _ ^2^2^ /(*2^2) _ N2B2A2R1IC1 _ 42 x 0-50x(l-8xlQ-^)xl0x^ ^ ^

V^“ N^B^A^Kk^R^) "/V|B,AiB2&2 30x0-25x(3-6xl0"^)xl4x/:


nidn
Re

Q. 11. In a chamber, a uniform magnetic field of 6*5 G (IG = 10“^ T) is maintained. An electron is shot into
FFi

the field with a speed of 4*8 x 10^ ms~^ normal to the field, (i) Explain why the path of the electron
is a circle. Determine the radius of the circular orbit, (c = 1*6 x lO"*'^ C, = 9*1 x llH^ kg).
Sol. Here, B = 6-5 x 10-4 T, u = 4-8 x 10^ m/s, e = 1-6 x lO"^^ C ; 0 = 90® ; m = 91xl0-3lkg;r=?
(0 Force on the moving electron due to magnetic field will be. F = c u B sin 0.

The direction of this force is perpendicular to v as well as B therefore this force will only change the
direction of motion of the electron without affecting its velocity i.e. this force will provide the centripetal
force to the moving electron and hence, the electron will move on the circular path. If r is the radius of
circular path traced by electron, then
evB sin 90® = tmP-lr

mv (91x1Q-3>)x (4-8x10^) = 4*2 X 10"* m = 4*2 cm


or
Be ~ (6-5 X10-4 )x (1-6 xKT'9)
4/138
7>>ieideefi^ Fundamental Physics CXII) VOL.I

Q. 12. In Q.11, obtain the frequency of revolution of the electron in its circular orbit. Does the answer

depend on the speed of the electron ? Explain.


2
mv mv m, .
Sol. Bev = or Be = — =m(ii = m2nv
r r r

Be (6-5 XIQ-^ ) X (1-6 x 1Q~^^)


or V =
27im ,-31 = 1818x10^ Hz
2 X (22/7) X 9*1 X10
The relation for frequency is independent of speed of electron, hence the frequency of revolution of
electron is independent of speed of electron.

w
Q. 13. (a) A circularcoil of 30 turnsand radius8.0 cm. carrying a current of 6.0 A is suspended vertically
in a uniform horizontal magnetic field of magnitude 1.0 T. The field lines make on angio of 60** with
the normal to the coil. Calculate the magnitude of the counter torque that must be applied to prevent
the coil from turning.

e
roow
Would your answer change if the circular coU in (a) were erplaced by a planar coil of some

re
irregular shape that encloses the same area ? (All other particulars are also unaltered).
Sol. Here, n = 30, /= 6.0 A, 5 = 1.0T, a = 60®, r = 8.0cm = 8 x 10"2 m.

reF
22

uFFll
.-. Area of the coil, A = nr^ = — x (8 x lO"^ )^ = 2.01 x ICT^ ^2

e
(a) Now, X = nIBA sin a = 30 x 6.0 x 1.0 x (2.01 x 10“^) x sin 60®

sFr
= 30 X 6.0 X 2.01 x 10-2 X ^3 / 2 = 3 ^33

oro
(b) Since the torque on the planar loop does not depend upon the shape, in case the area of the loop is the

k
uor
same, the torque will ermain unchangad.
off
kos
Y
Yo
reeBB
oo
urY
ou
ad
do
nY
nid
Re
FFi
MOVING CHARGES AND MAGNETISM 4/139

I WITH ANSWERS,
HINTS AND SOLUTIONS

T IT [c.

MULTIPLE CHOICE QUESTIONS-I

1. Two charged particles traverse identical 4. An electron is projected with uniform velocity
helical paths in a completely opposite sense along the axis of a current carrying long
solenoid. Which of the following is true ?

ww
A

in a uniform magnetic field B = B^k - (a) The electron will be accelerated along the
(a) They have equal z-components of momenta axis
{b) They must have equal charges (b) The electron path will be circular about the

Flo
(c) They necessarily represent a particle-anti- axis

e
panicle pair (c) The electron will experience a force at 45° to

reree
(d) The charge to mass ratio satisfy : the axis and hence execute a helical path

r FF
e
-I-
e
= 0 (d) The electron will continue to move with
m h uniform velocity along the axis of the
uurr
2. Biot-Savart law indicates that the moving
electrons (velocity t/ ) produce a magnetic foor solenoid

5. In a cyclotron, a charged particle


ks s
field B such that (a) undergoes acceleration all the time.
Yoo
ooook

-» (b) speeds up between the dees because of the


(a) B X u {b) B W V magnetic field
eBB

(c) it obeys inverse cube law (c) speeds up in a dee


{d) it is along the line joining the electron and (d) slows down within a dee and speeds up
point of observation
rr

between dees
ouu

3. A current carrying circular loop of radius R


ad

6. A circular current loop of magnetic moment


is placed in the x-y plane with centre at the
M is in an arbitrary orientation In an external
YY

origin. Half of the loop with x > 0 is now bent


so that it now lies in the y-z plane, magnetic field B . The work doen to rotate
nndd
Re

(a) The magnitude of magnetic moment now the loop by 30° about an axis perpendicular
diminishes to its plane is
Fii

{b) The magnetic moment does not change rr MB

(c) The magnitude of B at (0,0,z), z » R (a) MB (M V3 —


increases
MB
{d) The magnitude of B at (0,0,z), z » B is (0- (d) zero
unchanged

MULTIPLE CHOICE QUESTIONS-II

7. The gyro>magnetic ratio of an electron in an 8. Consider a wire carrying a steady current, 7


H-atom, according to Bohr model, is placed in a uniform magnetic field B
(a) independent of which orbit it is in perpendicular to its length. Consider the
(b) negative (c) positive charges inside the wire. It is known that
(d) increases with the quantum number n magnetic forces do not work. This implies that
4/140 Pz^^/cc/: '4. Fundamental Physics (Xll)ESSIS
(a) motion of charges inside the conductor is (c) there may be a point on C where B and
unaffected by B since they do not absorb —i

dl are perpendicular
energy
(b) Some charges inside the wire move to the id) B vanishes everywhere on C
surface as a result of B 10. A cubical re^on of space is filled with some
uniform electric and magnetic fields. An
(c) if the wire moves under the influence of B , electron entei^the cube across one of its faces
no work is done by the force with velocity v and a positron enters via oppo
(d) if the wire moves under the influence of B ,
site face with velocity - . At this instant,
(a) the electric forces on both the particles cause
no work is done by the magnetic force on identical accelerations
the ions, assumed fixed within the wire
(b) the magentic forces on both the particles
9. Two identical current carrying coaxial loops, cause equal accelerations
carry current I in an opposite sense. A simple (c) both particlesgain or loose energyat the same

w
amperian loop passes through both of them rate

once. Calling the loop as C, id) the motion of the centre of mass (CM) is
determined by B alone
id) § B-dI=^^2/HQl

Flo
C 11. A charged particle would continue to move
with a constant velocity in a region wherein,
(f>) the value of ® B-dl is independent of sense

ee
c
id) E =0, B ^0 ib) E ?^0, B ^£0

Fr
of C
(c) £ 5*0, 5 =0 id)E=Q,B =0

or
ur
sf ANSWERS
l.(d) 2. ia) 3. ia) 4. id) 5. ia) 6. id) 7. ia,b) 8. ib.d)
9. ib,c) 10. ib,c,d) 11. ia,b,d)
ok
Yo
Bo

HINTS FOR DIFFICULT MULTIPLE CHOICE QUESTIONS


re

Multiple Choice Questions -1


1. In a uniform magnetic field, the two charged panicles will traverse identical helical paths in a completely
ou
ad

opposite sense if the charge/mass ratio of these two particles is same and charges on them are of opposite
character. In this situation (€/m)j + ie/m)2 = 0, holds good.
Y

2. According to Biot Savart’s law, the magnetic field B at a point distance r from a charge g moving with a
nd
Re

velocity v is given by. B = Po q (i^x r) or S = Pq usinG


Fi

4tc 4tc r2

The direction of £ is along (ux r*), i.e., perpendicular to the plane containing u*; nd . B at a point
obeys inverse square law and not inverse cube law.
3. For a circular loop of radius R, canying current I in x-y plane, the magnetic moment Af = / x 7t It acts
perpendicular to the loop along z-direction. When half of the current loop is bent in y-z plane, then magnetic
moment due to half current loop in x-y plane, Af 1 = / inR^H) acting along z-direction. Magnetic moment
due to half current loop in y-z plane, A/2 = / (tJ?^/2) along x-direction.
Effective magnetic moment due to entire bent current loop,

M' = ^M^+mI = kR^/l)'^ +il kR'^/2)'^


i.e., magnetic moment diminishes.
MOVING CHARGES AND MAGNETISM 4/141

The magnitude of B at a point on the axis of loop, distance z from the centre of current loop in x-y plane

is B =
Ho 2%IR^
4jc (/?2 + ^2)3/2

The magnitude of B at a point distance z from the centre of bent current loop, whose half part is in
x-y plane and half part is in y-z plane, is

n2
Ho jc/B^
B' =
\ 4jt (B^ + z2)3/2 + ^2)3/2 4n (/?2 + z2)3/2

4. There is a uniform magnetic field B inside the current carrying long solenoid acting along the axis of

ww
solenoid. The magnitude of force on the electron of charge (- e) moving with velocity in a magnetic
field B is IFI = -el uxBI = - e u B sin 6

Flo
-4
Here angle 0 between v and B is zero, i.e., 0 = 0® and sin 0 = 0. Therefore F = 0.
It means the electron will continue to move with a uniform velocity along the axis of the solenoid.

e
reree
5. In a cyclotron, a charged particle describes the circular paths inside the dees and is accelerated while going
from one dee to another due to electric field. During circular path, the charged particle has centripetal

r FF
acceleration which is provided by the magnetic force due to magnetic field. Therefore, a charged particle
undergoes acceleration all the time, inside the cyclotron.
uurr
foor
6. Let magnetic moment M of current loop be making an angle a with the direction of B. When a current
circular loop is rotated in a magnetic field by 30® about an axis perpendicular to its plane, there is no change in
ks s
Yoo
ooook

the angle a between magnetic moment M and magnetic field B. Therefore, 0] = a, 02 = a.


Workdone, W=MB (cos 0| - cos 02> = MB (cos a - cos a) = 0
eBB

Multiple Choice Questions -11


7. Consider the electron of mass m, charge e be revolving in nth orbit of hydrogen atom. According to Bohr’s
rr
ouu

theory, angular momentum of electron (L) = nh/lK.


ad

Magnetic moment of electron, M = nx Bohr magneton = n [eh/(4 n m)]


YY

magnetic moment _ neh/jAnm) e


Gyromagnetic ratio = — = a constant
nndd

angular momentum nh/(2n) 2m


Re

As value of e is negative, so gyromagnetic ratio is negative.


Fii

8. (a) Motion of charges inside the conductor is affected by magnetic field B, due to magnetic force F,

given by F =q(vxB)
(b) Due to magnetic force, some charges inside the wire move to the surface of wire.
—> -» ->
(c) The force on wire of length /, carrying current I when subjected to magnetic field B is, F = / (/ x B). It
—4 —>
acts perpendicular to the plane containing / and B and is directed as given by Right Hand rule. If the
—>

wire moves under the influence of B at an angle 0, where 0 ^ 90®, then work done, W = Fs cos 0, can not
be zero.

(d) When wire moves under the influence of B, then displacement of the ions is perpendicular to the

magnetic force F. Therefore work done is zero.


4/142 ‘P'uideefi. Fundamental Physics (XII) VOL-I

9. Consider a simple amperian loop passing once through both the identical current carrying coaxial loops.

(0 According to Ampere circuital law, J B .d / = jIq (/-/) = 0. Hence, option (a) is wrong.
C

rw
(lOAs ^ B.dl = 0, therefore B.dI is independent of sense of C. Thus option (b) is correct
C C

—> —)
(Hi) There will be a point on loop C, lying at the axis of two loops A and B, where B and dl are

perpendicular to each other. Thus option (c) is correct

e
—>

(iv) The value of B does not vanish on various points of C. Thus option (d) is wrong.

r
o
10. Acceleration of electron due to electric field is

llou
F
=-eElm.

w
Its direction is opposite to the direction of £.

FF
s
Acceleration of positron due to electric field is a'^=e Elm; Its direction is along the direction of £.
uro
Fk
Thus Hence, option (a) is wrong.

rere
oo
Magnetic force on electron, F ~~e(v.xB)
F e(vxB)
Acceleration of electron, ^ =—
fofr
o
Y
JTl
m m
ooY
Magnetic force on positron, F' = e {-vxB) = -e (vx B)
B
ks
Yo

Acceleration of positron, a'^=~ = —^ .ivxB) ^ ^


r

m - . Hence, option (h) is true.


m m
eB
e

As both the particles (electron and positron) are of same mass and same charge in magnitude having same
dru

acceleration due to magnetic field, hence they gain or loose the energy at the same rate. Thus option (c) is
true.
no
ou
ad

—> —>

Due to electric field, net electric force on electron-positron pair, = -e £+ e £ = 0.


iY

Net magnetic force on electron-positron pair =-e(vx B)+[-e(ux B)] = -2e(vx B)


Therefore, the motion of the centre of mass (CM) is determined by magnetic field alone. Thus option (d) is true.
nd
F
Re

11. A charged particle will move with a constant velocity in a region if the force on it due to electron field as
Fi

well as magnetic field is zero or force on charged particle due to electric field is equal and opposite to the
force on it due to magnetic field.
—> —¥

Force on charged particle due to electric field, F^=qE


—> —>

Force on charged particle due to magnetic field, F^=q (vxB) or F^ = qv BsinQ. m

Now, £g = 0 if £ = 0 and £„ = 0 if sin 0 = 0 or 0 = 0® or 180®


A chargedparticlewould continueto move with a constantvelocity in a region if the force due to electric
field and magnetic field is zero, Le.,
(0 £ = 0 and particle is moving along the direction of magnetic field (/.e., 0 = 0® or 180®). Hence,
B ^ 0. Option (a) is correct
(I'O £ = 0 and B = 0. Option (d) is correct

(i70 The resultant force, qE+q ( yx B) = 0. In this case E^O and B^O, i.c.. Option (B) is correct
(iv) If B = 0 but £ 0, then the char ged particle will be accelerated by electric field. Option (c) is wrong.
MOVING CHARGES AND MAGNETISM 4/143

VERY SHORT ANSWER QUESTIONS


12. Verify that the cyclotron frequency O) = eBIm has the correct dimensions of (T]“^
-5
mv- qB V
Ans. If a charged panicle is moving perpendicular to the magnetic field, then qvB = or — = CO
r m r

1
Dimensions of o) = qB _v
tn r

13. Show that a force that does no work must be a velocity dependent force.
—) ^
Ans.
Let a force F does no work for the displacement i// while moving with velocity iT for time c/t.
Then work done, dW ~ F.dl =0 or F.ifdt ~ 0 or F=() or Fv cos 0 = 0 or 0 = 90“.

ww
Therefore, for no work done, f must be velocity dependent if angle between F and if is 90°. If if
changes its direction, then F should also change its direction so that angle between F and v is 90”. It
is so when a charged particle is moving in a perpendicular uniform magnetic field.

Flo
14. The magnetic force depends on which depends on the inertial frame of reference. Does then the

e
V

magnetic force differ from inertial frame to frame ? Is it reasonable that the net acceleration has a

rree
different value in different frames of reference ?

r FF
Ans. Magnetic force on a charged particle, F = q (vx B) or F = ^ u fi sin 6.
uurr
15.
for
Thus the magnetic force is a velocity dependent. It differs from one inertial frame to another. The net
acceleration arising due to magnetic force is independent from frame to frame for inertial frames of reference.
Describe the motion of a charged particle in a cyclotron if the frequency of the radio frequency (rf)
kss
field were doubled.
ooook
Yo
27tm
Ans. Time period of revolution of charged particle in cyclotron, f =
Bq
eB

1 Bq
Frequency of revolution. V = —
T 27im
rr
ou
ad

1 1
If / = 2 V, then T' = — 1 27cm Km _ Km
2v 2^ Bq
= a constant.
Y

V'
Bq Bq
dY

It means, the charged particle will accelerate and decelerate alternately between the dees of cyclotron. So
the radius of path in the dees will remain unchanged.
Re
innd

16.
Two long wires carrying current /j and I2 are arranged
as shown in Fig. 4(N).l. The one carrying current /j is
Fi

along the Jtr-axis. The other carrying current /2 is along a


line parallel to they-axis given by x = 0 and z = d. Find the
force exerted at O2 because of the wire along the x>axis.
Ans.
Magnetic field at point O2 due to current /j in long wire along
x-axis is

B
^2 Ak d ' It will be acting along the y-axis. As second
wire is along y-axis, so the angle between the directions of
I2 / and Ft?2 is zero, i.e., 0 = 0”.

Force on wire lying along y-axis is,

/ = /2l \ = l2lBQ^&m(f = 0.
4/144 'a Fundamental Physics (XII) vA»imi

SHORT ANSWER QUESTIONS

17. A current carrying loop consists of 3 identical quarter circles of radius R, lying in the positive quadrants
of the AT-y, y-z and z-x planes with their centres at the origin, joined together. Find the direction and
magnitude to B at the origin.
\ ( ,, T ^ /

4- +
Ho /
— 7 =—
^ ^ r'>,
(i + j+k)
Ans. b-B +B +B 4n R 2-^ 4n R 2
xy yz zx 4n R 2 4n R 2
/ \

U|. / A A A
= ^ii
SR
+ J+k)
—»

18. A charged particle of charge e and mass m is moving in an electric field E and magnetic field B .

w
Construct dimensionless quantities and quantities of dimension [71"^
Ans. A dimensionless quantity cannot be constructed with e. m, E and B. A quantity of dimensions

Flo
[3-'] = (0 = [—1

e
reee
m

FFr
19. An electron enters with a velocity v=v^i into a cubical region (faces parallel to coordinate planes)
in which there are uniform electric and magnetic fields. The orbit of the electron is found to spiral

for
ur
down inside the cube in plane parallel to the x-y plane. Suggest a configuration of fields E and B
that can lead to it.
kss
Ans. As per question, the orbit of electron is spiral down inside the cube in a plane parallel to the x-y plane,
—♦ —» A . .
Yo
therefore, B must be along + z direction, i.e., B = B^k. Here the electric field should be along Ar-axis,
oo

i.e., £■ = Fy ?, where Eg > 0.


B

A
re

= dl i located at
20. Do magnetic forces obey Newton’s third law. Verify for two current elements d
-¥ ^
the origin and dl^ =dl J located at (0, R, 0). Both carry current I.
ou
ad
YY

Ans. Refer Fig. 4(N).2, magnetic field due to current element / </ /j at a FIGURE 4(N).2

Ho Idl^
nd

point on element d , B =
Re

47C R^ dll - J dl
Fi

= I- dl A; X —
\Iq ——
i dl k^ R
Force on dl^ >s, F = I dl jx B 4n R}
I

_Ho dl■^ = i dl
’471 ^ 47t /?2
It can not be zero.

Force on Jd\ due to magnetic field by current I dl2 is zero as magnetic field of / J /j has no interaction
with current element / d .

Therefore magnetic forces on current elements in this case do not obey Newton’s third law.
21. A multirange voltmeter can be constructed by using a galvanometer circuit as shown in Fig. 4{N)3.
We want to construct a voltmeter that can measure 2 V, 20 V and 200 V using a galvanometer of
resistance 10 and that produces maximum deflection for current of 1 mA. Find R|, Ri and R3 that
have to be used.
MOVING CHARGES AND MAGNETISM 4/145

Ans. Here, G = 10 Q,/^ = I mA = 10-3 A FIGURE 4<N).3


Case (0. y=2V,/?| =? Rl R2 R3
G ■WvrVSArrVsyVi

-10
= 1990Q = 2kn

Case (u), v=20v,(/f, +/e2) = ?


20
(/?i+/?2) = -10 -20,000- 10«20kQ
10-3
/?2 = 20 kn - 2 kQ = 18
6 0 6 6
Case (m), y=200V,(/?i +/?2 + «3) = ? 2V 20V 200V

200

w
/?! + /?2 + ^3 - -10 «200kQ /?3 = 200 kn - 20 kn « 180 kfl.
10-3
22. A long straight wire carrying current of 25 A rests on a table as shown in Fig. 4(N).4. Another wire

FF lowo
PQ of length 1 m, mass 2-5 g carries the same current but in the opposite direction. The wire PQ is
free to slide up and down. To what height will PQ rise ?
FIGURE 4(N).4
Ans. Here, /j = 25 A, /2 = 25 A, / = 1 m, m = 2-5 g = 2-5 x 10"3 kg

ee
In equilibrium position

Fr
h
P T*' Q
mg =
4n: h

rer
ur
OT h = Pq 2/^/^/
4te mg
10-7x2x25x25x1
(2'5xl0-3)x9-8
fofr Fo
= 51 xl0-^m = 0-51 cm
ks
YYouro
oo

LONG ANSWER QUESTIONS


BBo ks

23. A 100 turn rectangular coil ABCD (in XY plane) is hung from one arm FIGURE 4(N).S
r ee

of a balance (Fig. 4(N).5). A mass 500 g is added to the other arm to


balance the weight of the coil. A current 4*9 A passes through the coil
A
7^

A
oouru

and a constant magnetic field of 0*2 T acting inward (in xz plane) is


ad

switched on such that only arm CD of length 1 cm lies in the field. How
Y

much additional mass must be added to regain the balance ?


Ans. When the magnetic field is off and the weight added in one pan of a balance, aX
Ynd

B
Re

balances the rectangular coil in the other pan of balance. Then


Mgi = / or = Mg = 500 x 9-8 N
FFindi

AD
When current I is passed through the coil and the magnetic field is switched I xxlrxxx'
^.XX XX)i.'
on, let m be the mass added in a pan to balance the beam. Then
Mgl + mgl = I + IBL sin 90“ /
/BL 4-9x0-2x(1x10"2)
or mgl = IBL I or m = m —
9-8
= 10-5kg =ig
g
24. A rectangular conductii^ loop consists of two wires on two opposite sides of length / joined together by
rods of length d. The wires are each of the same material but with cross-sections differing by a factor of 2.
The thicker wire has a resistance R and the rods are of low resistance, which in turn are connected to a
constant voltage source Vq. The loop is placed in a uniform magnetic field B at 45" to its plane. Find t, the
torque exerted by the magnetic field on the loop about an axis through the centres of rods.
Ans. Refer to Fig. 4(N).6, force on arm AB due to magnetic field ,
y.
F, =I^IB = -^IB =
0
IB
R 2R
1
4/146 4^ Fundamental Physics (XII) VOL.I

It is acting Ir to the arm AB towards observer. RGURE4(N).6


Torque due to force Fj about axis EF is, li;l2
D
t f V ^ d 1 VJBd
0
= F,x-sin45‘’= IB E

1 2 2R j 2 ^ A^R I21
(anticlockwise) Ai=2A
T
Force on arm DC due to magnetic field. R2=R
' ll -p Vo
Ai=A I

F2=/2/5 = R
Ri = 2R
I
C
It is acting _Lr to arm DC towards observer. I T --►B
->

Torque due to force F2 about axis EF is I

oww
'd
d rVn 1 VABd >
x, = F,x-sin45‘’= -^IB = (clockwise) I1 + I2
222 yR j2 2^R
Net torque, x = 1:2 “ (clockwise)
Vp/g^r ll V^lRd VqAB

e
(V Ixd^A)
2-J2R L 2J 4^l2R A^R

ree
rFl
Fre
25. An electron and a positron are released from (0, 0, 0) and (0, 0,1*5 R) respectively, in a uniform

rr F
magneticfield B = , each with an equal momentum ofmagnitadep=cgR . Under what conditions
on the direction of momentum will the orbits be non-intersecting circles ?
ouur
Ans. Here, ?(=gp^) is acting along the jc-axis. For a circular orbit, sfoo
kks
the momentum of the electron and positron are in y-z plane.
oooo
Yo
Let P\ and P2 be the momentum of the electron and positron
respectively. Both of them due to same momentum (= e BR)
eBB

move on circular orbits, each of radius F, but in opposite sense.

Let P\ make an angle 0 with the y-axis and F2 must make


urr

the same angle 0 with y-axis. Refer to Fig. 4(N).7. The centres -►y
You
ad

of the respective circular orbits must be perpendicular to the


momenta at a distance R. Let the centre of the circular orbit of
dY

the electron be at Cg and of the positron be at Cp.


The coordinates of Q is (0, F sin 0, F cos 0)
innd
Re

/3 JC

The coordinates of Cp is 0, - Fsin 0, ^ ^ ^®


Fi

The two circular orbits will not intersect if the distance between their two centres is greater than 2 F.

Let ^ be the distance between Cp and Cg. Then


= [-F sin 0 - (F sin 0)]^ + -F-Fcos0 -(Fcos0)
U2
= 4F^ sin^ 0+-F^ -6F^ cos0+4F^ cos^ 0 = 4F^(sin^ 0 + cos^ 0)+^F^ -6F^ cos0
4 4
25
/●
F^-6f2cos0
4
The two circular orbits will not intersect each other if > 2 F or > 4 F^
25 9
^/?2_6/?2cos0>4/j2 or — -6cos0>4 or -->6cos0 or cos 0< 3/8
4 4 4
MOVING CHARGES AND MAGNETISM 4/147

26. A uniform conducting wire of length 12 a and resistance R is wound up as a current carrying coil in
the shape of (i) an equilateral triangle of side a ; (li) a square of sides a and, (iii) a regular hexagon of
sides a. The coil Is connected to a voltage source Vo- Find the magnetic moment of the coils in each
case.

FIGURE 4{N).8
Ans. (/) Refer to Fig. 4(N).8(a); No. of turns, /i = 4 a. a

a
Area of triangle A = -a X a sin 60® =a^J3l4
S

2
a a

(//) Refer to Fig. 4(N).8(^); a


/ N

= 3, area A=ay.a = (-P N

{in) Refer to Fig. 4(N).8(c); a a


a

3^|3 , h = 3 n = 2
n = 2, area A = 6 — x — tan 60® a~
2 2 2 o o

w
Magnetic moment, M = nlA
For(/), M,I =4xlxa~^/4 For (ii), A^2 = 3 X / X =3 /

Flo
M, = 2x/x
3a/3 2
= 3^l3a^I
For (Hi), a
3 2

reeee
27. Consider a circular current-carrying loop of radius R in the x-y plane with centre at origin. Consider

FFr
the line integral 3 {L) = B.dl taken along z-axis.
-L

for
ur
(a) Show that 3 {L) monotonically increases with L.
(b) Use an appropriate Amperian loop to show that 3 (<«) = Po where I Is the current In the wire,
kss
(c) Verify directly the above result,
coil of sides R carrying the same current I. What
Yo
(d) Suppose we replace the circular coil by a square
oo

can you say about 3 (L) and 3 («») ?


—^
eB

Ans. (fl) As circular loop carrying current is lying in A:-y plane, the magnetic field B is along the z-axis, in the
+L ●^L +L

direction of line integral. Hence, line integral, 3(L)= | B.dl Bd/cosO® = f Bdl = lBL.
r
ou
ad

-L -L -L

Therefore 3 (L) is monotonically increasing with L.


YY

{b) Consider a closed amperian path PQRP as shown in Fig. 4(N).9.


ndd

—^
Re

According to Ampere circuital law, line integral of B over the closed path PQRP is
+L
Fi

—* -4

B.dl = B.dl+ B.dl = B.dl +


FQR RF FQR -L
FQRP
FIGURE 4(N).9
Q
or 5.d/-l-3(L) = P()/ ...(/)
FQR
\

When L —> oo, then the value of B ^ 0 (as B oc


l/r') /
f

»p

B .dl = 0. From (/), 0 + 3 (««) = Po / or 3 («) = |i() / -L +L

FQR

(c) Magnetic field at a point on the axis of circular coil at a distance z from the centre of a circular coil of
●too +«■

radius R carrying current I is B , j = 2(z2+^2)3/2


dz
^ 2(z2 + /?2)3/2 ●
4/148 Fundamental Physics (XII) VOL.I

+«» „ I +JI/2
Putz = /?tan0,<fe = /?sec2erf0 /. J B^dz = ^ J cos0</0=m,7. -re/2

(rf) If circular coil is replaced by a square coil of side /?, carrying the same current /, then
^ (z)square ^ ^ (^)circular coil ^ (^)square ^ ^ (^)circular coil
Using arguments as in (b) 3 = 3 («»)circuiar = Ik)
FIGURE 4(N).10
28. A multirange current meter can be constructed by using a
S3
galvanometer circuit as shown in Fig. 4(N).10. We want a current
meter that can measure 10 mA, 100 mA and 1 A using a

oww
S1 $2
galvanometer of resistance 10 Q and that produces maximnm WSr-i-VW-
deflection for current of 1 mA. Find 5,, 52 and Sj that have to be
used.

Ans. Here, G = 10 (2, /^ = 1 mA = 10"^ A.

ee
To convert a galvanometer into an ammeter of given range of current

FFrlo
/.G

r
g
0 to /, the shunt resistance required is 5 =

rF
I-I

ee
8 6 0 6 O
10mA 100mA 1A
Case (i), / = 10 mA = 10 x KT^ A, 5 = 5j + S2 + Sy

rF
ouru
ImAxlOfi 10 _
5, + 53 + 53 = ...(0
(10-1) mA 9 “
fosor
Case (u), When I = 100 mA, 8 = 82 + 8^, galvanometer resistance = G + 5|
s kf
ooko
+ _ 1 mAx(10+5,) 10+5,
YYo
52 + 53 =
/-/ (100-1) mA 99
8
Bo
reeB

Case (iw), When / = 1 A, 5 = 53, galvanometer resistance = (G + 5^ + 82)


e _^g(^ + ‘^l + ‘^2)_l*nA[10 + 5,+52]_10 + 5,+52
ooY
uur

...m
^ I-I 8 (1000-1) mA “ 999
ad

Putting the value of (ii) in (1), we get


dY

10+5, 10 n 10 10 100
nidn

5.+ or S. l+_ = = _
Re

\ 99 j 9 99 99
1
99 9
FFi

5.x
1
100^100 or 5, =1Q
99 99

1 1
From (h).
^2 + ^3 =-(10+1) = ^ ...(iV)

From (m’O. „ 10 + 1 + 5, 11 + 5, ^2 _ 11
53 = 999
2. = 999
2 or
^3- 999 “ 999 ....(V)

Subtracting (v) from (iv), we get.

5 +A = l_iL = 12? or
5,xl000 i(X) 1
or 5, = — = 0*1 Q
2 999 9 999 999 999 “999 ^ 10
1 1 1 1 1
From (tv), or Sj = -- = — or 5a = 0*011 O
9 10 90 ^
MOVING CHARGES AND MAGNETISM 4/149

29. Five long wires A, B, C, D and E, each carrying current I are arranged to form edges of a pentagonal
prism as shown in Fig. 4(N).ll. Each carries current out of the plane of paper,
(a) What will be magnetic induction at a point on the axis O ? Axis is at a distance R from each wire,
(b) What will be the field if current in one of the wires (say A) is switched off ?
(c) What if current in one of the wire (say) A is reversed ?
Ans. (a) Let the five wires A, B. C, D and £ be perpendicular to the plane of paper at
locations as shown in Fig. 4(N).ll.
Resultant magnetic field induction at O due to current through all the
five wires is = 0 (as magnetic field induction due to five wires will be

oww
represented by various sides of a closed pentagon in one order, lying
in the plane of paper).
(b) Total Magnetic field induction at O due to currents through the
wires B, C, D and E is equal and opposite to the magnetic field

e
induction at O due to current through wire A.

re
Magnetic field induction at O due to current through wire A is

FFrllo
B = ^ ^ acting Ir ot AO towards right.

reF
e
H 21
ouru
.’. Magnetic field induction at O due to currents through the wires B, C, D and £ is = 4jc R
acting±r

osrF
to AO otwards left
(c) If current in wire A is reversed, then ottal magnetic field induction at O

ffor
k
= mag. field induction due to wire A + mag. field induction due to wires B, C, D and £
kso
(acting-Lr AO towards left) + An (acting -Lr AO towards left)
R
ooo
An R
Yo
Y
BB

acting Jj* AO otwards left


nR
r ree
Y
oouu
ad
Yd
nidn
Re
F
Fi
4/150
Fundamental Physics (XII) voL.r

SPECIAL

ultimate preparation brthis'uniTf^comp^^titive'^^cafnmatlt^st-st ude'nts should refer to

w
● ● * j iW.’-’,n V.— tr, -.W ; ;
T^r
PradeepS MCQs In Physics for NEET
MCQs in Physics for JEE (Main) . ■
.r.
separatelyfavailabielfontheselexaminationslfe^yfe^

Flo
u Multiplv^

e
reee
(with One Correct Answer)

FFr
I. Biot Saviiri s lu'- ■ LI i.'H
FIGURE 4{CF).2
due to :● »■ n ill nu r lu ■'> 1; s D

or
ur
1. The magnetic field intensity due to a thin wire
ss f C E
carrying current I shown in Fig. 4{CF). 1is
ok
Yo
●O
Bo

B F
I
re

^0^
ou
ad

(a) (b)
3V3 Ka
YY

Tta

Ho' Li„/
nd

(C) (d)
Re

(a) [jc - a + tan a]


2kR Tca ■>/5na
Fi

3. The current I ampere is flowing in an equilateral


(b) [7c-a] triangle of side a the magnetic field induction at
2nR the centroid will be

Mo^ ia)
5V2Pq/
(c) [;t + a] 2na
(b)
2nR 3jta

3po/
M ic) id)
id)
2tzR
[jE + a-tana] (DPMT 2006) 2%a 3^na ■
4. Current / flows through a long conducting wire
2. A current I is flowing in a hexagonal coil of side bent at right angles as shown in the Fig. 4(CF).3.
a Fig. 4(CF).2. The magnetic field induction at The magnetic field at a point P and the right bi
the centre O of the coil will be sector of the angle COD at a distance r from O is
MOVING CHARGES AND MAGNETISM 4/151

(a) 1/2 {b) 1


I FIGURE 4(CF).3
(c) 2/3 id) 2.
7. Two long parallel wires are at a distance 2 d apart.
They carry steady equal currents flowing out of
the plane 'of paper as shown in Fig. 4(CF).5. The
variation of the magnetic field B along the line
XX' is given by

w
C FIGURE 4(CF).5
i
I
2^0 /
(«) (b) I
nr jcr I
I V

e
X <:>—X'
(V2+I) (V2+I)

row
ic) 47cr id) 4n r

re
5. Two identical long conducting wires AOB and I I
I I

COD are placed at right angle to each other, with

FFllo
F
d
one above other such that O is their common point
©

u
for the two. The wires carry /[ and currents,

ree
respectively. Point P is lying at distance d from 0
along a direction perpendicular to the plane

sFr
B

containing the wires. The magnetic field at the

kro
point P will be X X'
uor
id
fk)
'2nd V I-y^
h
ib) 2nd
Po offo
(/1+/2)
I
I
I
I
I

I
I
kos
1 I

Po Ik)
ul-ih
YYo
d
(c) id)
eerBB
oo

2nd 2nd
o
(AIPMT 2014)
rY

6. An infinitely long conductor PQR is bent to form a


8. A non-planar loop of conducting wire carrying a
right angle as shown in Fig. 4(CF).4. A current I
uu

current / is placed as shown in the Fig. 4(CF).6.


flows through PQR. The magnetic field due to this
Each of the straight sections of the loop is of
current at the point Af is Hj. Now another infinitely
ad
doo

length 2 a. The magnetic field due to this loop at


long straight conductor QS is connected at Q so that
the point P (<2, 0. a) points in the direction
nY

the current is HI in QR as well as in QS, the current


in PQ remaining unchanged. The magnetic field at
nid
Re

M is now H2. The ratio //1///2 given by


FFi

FIGURE 4(CF).4
M

I 90° I 1/2
— CD- --S
Q 90°

' '1/2

R
! A A A
1 A A

I (a) i-j+k) ib) i~j+k + i)


+ 'oo V3

A
4/152
^%adee^'4. Fundamental Physics fXTHPTSTi
1 A A A 1 A ^
(c) —a+j+k) (d) -^U + k)
magnitude of the resultant magnetic field at the
3^ 42 centre of the current loop is
9. A long straight wire along z-axis carries a current
I in the negative z-direction. The magnetic field
vector 5 at a point having coordinates (x, y) in
the z = 0 plane is

(a) (b)
\i(jHx?+yj)
2n(x^ +y^) 2tc(x^ + y^)

(.c)
\i-oHxj-yt) (d)
\^ol(xj+yt)
2n(x^ + y^) 2ti{x^ + y^) («)
10. Infinite number of straight wires, each carrying Tir 7tr

ww
current I are equally spaced as shown in Fig. ^]IqI
4(CF).7. The adjacent wires have current in (c) (d)
3nr 37cr
opposite directions. Net magnetic field at 0 is

FF loo
13. Two parallel beams of protons and electrons,
FIGURE 4(CF).7
carrying equal currents are fixed at a separation

ee
■►00
d. The protons and electrons move in opposite
directions. There is a point P on the straight

ee r
rFrF
I
I perpendicular line joining the two beams at a
distance x from one beam. The magnetic field at
this point is B. If B is plotted against x, it can be
rur
I I
z ffoor represented by the curve.
ks
3
YYoou
4'^
ookos

3a
► 5'4
6
BBo
re

(a)Zero .(b)
8tc
oouur
ad

(c) ^ lo^ (d)


Y

4tc rV3 2jt

11. One of the two identical conducting wires of length


dndY
Re

L is bent in the form of a circular loop and the


other one into a circular coil of N identical turns.
FFini

If the same current is passed in both, the ratio of


the magnetic field at the central of the loop {B[) to
that at the centre of the coil (5^) i.e., BJBq will
be:

(a) N (b) 1/N


14. Two parallel wires in the plane of paper are
(c) (d) 1/n2 distance xq apart. A point charge is moving with
(JEE Main 2019) speed u between the wires in the same plane at a
12. Fig. 4(CF).8 shows a square loop /*Q.ST with edge distance X| from one of the wires. When the wires
length r. The resistance of the wire PQS is R and carry currentof magnitudeI in the same direction,
that of PTS is 2 R. If the wire is uniform, the the radius of curvature of the path of the point
charge is /?]. In contrast if the currents I in the
ANSWERS

8. (d) 9. [a) 10. id) 11. id) 12. (c) 13. (h)
MOVING CHARGES AND MAGNETISM 4/153

two wires have directions opposite to each other. id) Jc2/8V2


(c)7t2/16
The value of curvature of the path is /?2-
x^Jx^ = 3, the value of /?|//?2 : (JEE Main 2016)
{a) 1 (b) 2 (c) 3 (^0 4 18. Charge q is uniformly distributed on a disc of
(JEE Advanced Paper 1, 2014) radius r. If the disc is rotated with a frequency v,
15. An infinitely long straight conductor carries a then magnetic field induction at the centre will
be
current of 5 A as shown in Fig. 4(CF).lO. An
electron is moving with a speed of 10^ m/s parallel ib)
(a)
to the conductor. The perpendicular distance

w
vr r

between the electron and the conductor is 20 cm


at an instant. Calculate the magnitude of the force ic) id)
experienced by the electron at that instant. qvr

19. Two circular coils C and D have equal number of

e
FIGURE 4(CF).10

row
turns and carry equal currents in the same

re
-1
Electron u“10^ms direction in the same sense and subtend same solid
angle at point O. Fig. 4(CF). 11. The smaller coil

FFllo
F
C is midway between O and D. If we represent
20 cm mag- netic field induction due to bigger coil and

u
smaller coil C at 0 as and respectively,

ree
then BjJBq is

sFr
p 5A Q

kro
,-20 N -20
uor
(«) 8 X 10 {b) 4 X 10 N

(c) 8 7C X 10
-20
N (d) 4 7t X 10
-20 offo
N

(MEET 2021)
kos
Y
Yo
eerBB

II. Magnetic field due to a


oo

circular coll carrying current


(c) 1 ; 2 (d) 1 : 4
rY

(a) 1 : 1 ib)2: I
16. The dipole moment of a circular loop carrying a 20. A long wire bent as shown in Fig. 4(CF).12,
u

current I, is m and the magnetic field at the centre carries a current/. If the radius of the semi-circular
of the loop is Bj. When the dipole moment is portion is r, the magnetic field induction at the
ou
ad
do

doubled by keeping the current constant, the centre P is


nY

magnetic field at the centre of the loop is B2- The


FIGURE 4(CF).12
● ●

ratio — IS
nid
Re

«2
F

I
Fi

(a) 2 ib) V3 (c) .S id)

(JEE Main 2018)


17. Two identical wires A and B, each of length /
carry the same current /. Wire A is bent into a
circle of radius R and wire B is bent to form a
square of side a. If B^ and Bg are the values of
magnetic field at the centres of the circle and
square respectively, then B^/Bg is :
(a) 7U^/8 ib) 7r^/16->/2
ANSWERS

14. (f) 15. (ti) 16. (f) 17. (d) I8.(/;) 19. (f)

&
4/154
T^n^tdee^'4^ Fundamental Physics (XlI)BZsMl
25. A current / flows in an infinitely long wire with
(a) 4/- ib) 47tr cross section in the form of a semicircular ring of
radius R. The magnitude of the magnetic induction
along its axis is
-4
(c) 4r 2nr {d) 4nr
id) ib)
ItO-R 2%R
(AIPMT 2015)
21. Two similar coils of radius/? and number of turns
Nare lying concentrically with their planes at an ic) id)
4t{^R n^R
angle 60" to each other. The currents flowing in
them are / and 2 / respectively. The resultant (AIEEE 2011)
magnetic induction at the centre will be 26. A loop carrying current I lies in the x-y plane as
A

Si^NI shown in the Fig. 4{CF).I3. The unit vector k is

ww
ia) ib)
R 2R coming out of the plane of the paper. The magnetic
moment of the current loop is

Flo
ic) id) FIGURE 4fCF).13
2R 2R

e
reree
22. A coil of n number of turns is wound tightly in
the form of a spiral with inner and outer radii

r FF
a

and h respectively. When a current of strength /


is pa.ssed through the coil, the magnetic field at
uurr
its centre is

1^0 d I
foor
ks s
, d ●►x
ib)
Yoo
2(b~a)
ooook
eBB

b
ic)
b 2{b~a)
l«g^ -
a

(IIT 2011, Karnataka CET 2009)


rr
ouu

^ A I TZ 1 T ^
ad

23. The magnetic field at the centre of a circular coil ia) (b) - + 1 a^lk
of radius r, due to current / flowing through it, is J
YY

B. The magnetic field at a point along the axis at a


distance r/2 from the centre is (f) + (d) (2Jt+I) a^Ik
nndd
Re

2
(a) BI2 ih)2B ^ (IIT 2012)
Fii

27. A charge Q is uniformly distributed over the


ic) -j= B B surface of non-conducting di.sc of radius R. The
Vs VJJ disc rotates about an axis perpendicular to its
(JEE Main 2022) plane and passing through its centre with an
angular velocity co. As a result of this rotation a
24. Magnetic fields at the two points on the axis of a
circular coil at a distance of 0-05 m and 0-2 m from magnetic field of induction B is obtained at the
the centre are in the ratio 8:1. The radius of the centre of the di.sc. If we keep both the amount of
coil is charge placed on the disc and its angular velocity
ia) 0-15 m
to be constant and vary the radius of the disc
ib) 0-2 m
then the variation of the magnetic induction at
(c) 0-1 m id) 1-0 m
the centre of the disc will be repre.senled by the
(JEE Main 2021) figure
ANSWERS

20. ib) 21. id) 22. id) 23. (c) 24. (c) 25. (d) 26. ib)
MOVING CHARGES AND MAGNETISM 4/155

field at the central of the loop (5^ ) to that at the


centre of the coil i.e.. will be

(a) ib) N-
N

1
(c) N id).

(JEE Main 2020)

III. Ampere circuital law,


Solenoid and Toroid

30. A long straight wire of radius a carries a steady

ww
current i. The current is uniformly distributed
across its cross-section. The ratio of the magnetic
field at all and 2 a is

FF loo
(a) 1/2 ib) 1/4 (c) 4 (d) I

ee
(NEET 2016, AIEEE 2007)
31. An infinitely long hollow conducting cylinder

ee r
(AIEEE 2012)

rFrF
28. Refer to Fig. 4(CF). 15, where c = 3a and a<b< with inner radius — and outer radius R carries a
2
rur
c, the magnetic field induction at point O will be uniform current density along its length. The
zero due to current / in the various portion of the
network when
ffoor magnitude of the magnetic field, 1^1 as a

ks
function of the radial distance r from the axis is
YYoou
best represented by
ookos
BBo

FIGURE 4(CF).16
re
oouur
ad

|Bl
1
Y

I
I

1
R/2 R r R/2 R r
dndY
Re

o o
FFini

la |B| IBI
ia) b = —a («b = -
7
>
9a \3a R/2 R r

(c) /> = — (d)b = o


13 15

29. One of the two identical wires of length L is bent (IIT 2012)
inform of a circular loop and the other one into a
circular coil of N identical turns. If the same 32. A long straight metal rod has a very long hole of
current is passed in both, the ratio of the magnetic radius r, made parallel to the rod axis as shown
ANSWERS

27. (a) 28. (c) 29. (d) 30. (d) 3U (d)


4/156
‘P^tuUe^ ^ Fundamental Physics (XII) VOL.I D

in Fig. 4(CF). 17. Here, OC = r. If 7 is the current


flowing through the rod, then the value of mag IV. Force on charged particle
netic field induction on the axis of the hole is in electric and magnetic flelds
35. An electron falls from rest through a vertical
distance A in a uniform and vertically upwards
directed electric field E. The direction of electric
field is now reversed, keeping its magnitude the
same. A proton is allowed to fall from rest in it
through the same vertical distance h. The lime of
fall of the electron, in comparison to the time of
fall of the proton is
(«) 10 times greater {b) 5 times greater
(c) Smaller {d) Equal

ww
(NEET 2018)
36. An electron, a proton and an alpha particle having
(a) {b) the same kinetic energy are moving in circular

FF loo
2n{r^-r'^) orbits of radii r^.r^, respectively in a uniform
magnetic field B. The relation between '●/>’ '■a

ee
IS
(C) (^0

er
67i(r2^-rj2) («) r,>rj, = ^r {b) r, < = r„

Fr reF
a

33. A long solenoid of 50 cm length having 100 turns (c) r^<r^<r, a


(J) r^<r„<r^
rur
carries a current of 2-5 A. The magnetic field at (JEE Main 2018)
the centre of the solenoid is ;
[)io = 4nx 10“'^ Tm A“*]
fforo
37. A conductor lies along the z-axis at
- 1-5 ^ z < 1-5 m and carries a fixed current of
ks
(a) 3-14 X lO-'^T (b) 6-28 X 10-5 T
YYouo
10 0 A in ~a. direction in Fig. 4(CF).18. For a
okso

(c) 314x 10-5 T id) 6-28 X lO-'^T


field B=3 0xI0“^e‘*2-*-a^T, find the power
BBoo

(NEET 2020)
34. From Ampere’s circuital law for a long straight wire required to move the conductor at constant speed
r ee

of circular cross-section carrying a steady current, to.r = 2-0m,y = 0m in 5 x 10“^ s. Assume parallel
the variation of magnetic field in the inside and motion along the A:-axis.
ad
oouur

outside region of the wire is


Y

(a) Uniform and remains constant for both the


regions.
dnYd
Re

(b) A linearly increasing function of distance upto


the boundary of the wire and then linearly
FFini

decreasing for the outside region,


(c) A linearly increasing function of distance r
upto the boundary of the wire and then
1
decreasing one with - dependence for the
outside region. (a) 14-85 W ib) 29-7 W

id) A linearly decreasing function of distance upto (c) 1-57 W id) 2-97 W
the boundary of the wire and then a linearly (JEE Main 2014)
increasing one for the outside region. 38. A particle having the same charge as that of
(NEET 2022) electron moves in a circular path of radius 0-5 cm
ANSWERS

32. ih) 33. id) 34. (c) 35. (c) 36. (h) 37. id)
MOVING CHARGES AND MAGNETISM 4/157

under the influence of a magnetic field of 0-5 T. qbB qib-a)B


If an electric field of 100 V/m makes it to move (a) ib)
m m
in a straight path, then the mass of the particle is
(Given charge of electron = 1-6 x 10”^^ C) qaB qib+a)B
(a) 9-1 X 10-3' kg 0) 1-6 X 10-27 kg (c) id) 2m
m

(c) 1-6 X 10-'® kg (d) 2-0 X 10*2'' kg 42. An electron moving with a speed u along the
(JEE Main 2019)
positive x-axis at y = 0 enters a region of uniform
39. A charged particle of mass m and charge q is —> A

projected into a uniform magnetic field of magnetic field B^-B^k which exists to the
induction B with speed u which is perpendicular right of y-axis, Fig. 4(CF).20. The electron exits
from the region after some time with speed v at
to B . The width of the magnetic field is d. The coordinate y, then
impulse imparted to the particle by the field is

ww
id « nwlqB) FIGURE 4(CF).20

FIGURE 4(CF).19
X X X X

Flo
q V
¥ B
X X X X

ee
m

rere
u X X X X

rFF
-e
X X X X
uur r
d foor
ks s
(a) V > u, y < 0 (b)v = u,y>0
(b) qBv/d
Yoo
(a) qBd (d)v = u,y<0
(c) V > u, y > 0
ook

(c) mvfiqBd) id) mv^liqBd)


43. A charged particle with charge q enters a region
BBo

40. A proton of mass 1-67 x 10"27 kg and charge of constant, uniform and mutually orthogonal
1-6 X 10"'® C is projected with a speed of
re

2 X 10^ m/s at an angle of 60” to the X-axis. If a fields £ and B with a velocity "v perpendicular
uniform magnetic field of 0104 tesla is applied
ouur

-i

to both £ and 5 , and comes out without any


ad

along y-axis, the path of proton is


change in magnitude or direction of if. Then
YY

(a) a circle of radius 0-2 m and time period


7C X I0"7 s
(a) if=j5x£/£2 ib) 1} = ExBlB^
nndd
Re

ib) a circle of radius = 0-1 m and time period


2 7C X 10“7 s
(c) v = BxE/B^ id) v=ExBIE^
FFii

(c) a helix of radius = 0-1 m and time period


2 7t X 10"7 s (AIEEE 2007)

id) a helix of radius = 0-2 m and lime period 44. Two large vertical and parallel metal plates having
4 7C X 10"7 s. a separation of 1 cm are connected to a DC voltage
source of potential difference X. A proton is
41. A particle of mass m and charge q moves with a released at rest midway between the two plates.
constant velocity v along the positive x-direction. It is found to move at 45” to the vertical just after
It enters a region containing a uniform magnetic release. Then X is nearly [use g = 10 ms-2]
field B directed along the negative z-direction,
extending from x - a to x = b. The mininmum ia) 1 X 10-5 V ib) 1 X 10-7 V
value of V required so that the particle can just ic) I X 10-® V id) 1 X I0-'°V
enter the region x> bis (UT 2012)
ANSWERS

38. id) 39. («) 40. (f) 41. (/>) 42. 43. (h) 44. (c)

I
4/158
<t Fundamental Physics (XII)
45. TNvo long current carrying thin wires, both with , A A A

current / are held by insulating threads of length and F =4/-20j+l2k


L and are in equilibrium as shown in Fig.
4(CF).21, with threads making an angle 0 with What will be the complete expression for ^ ?
the vertical. If wires have mass X per unit length,
. A A A
(a) 6/ + 6j-8k (b) -8/-8 j-6k
then the value of / is : (g = gravitational
acceleration) . ^
(c) -6/-6j-8k
A A A , A A A
(d) 8/+8 j-6k
(NEET 2021)
48. A particle of mass m and charge q is released from
rest in uniform electric field. If there is no other
force on the particle, the dependence of its speed
V on the distancex travelled by it is correctly given
by (graphs are schematic and not drawn to scale)

ww
FIGURE 4(CF).22

Floo
^4 VA

TzXgL ttXg L
(n) .sin 6 {b) 2sin6

ee
Y|X„cos6 cos 6 («) {b)

reer
rFF
(c) 2
^KgL 'nXg L
tan 6 (d) tan 0
1^0 h) VA VA
uur r
46. When a proton is relea.sed from rest in a room,
(JEE Main 2015)
it
(c)
ffoor /
(d)
sks
starts with an initial acceleration Gq towards west.
YYoo
When it is projected towards north with a speed
ooko

-A-X ^X

Vy it moves with an initial acceleration 3 a 0


eBB

towards west. The electric and magnetic fields in


the room are (JEE Main 2020)
49. Abeamofprotonswithspeed4x 10^ms“* enters
r

ma 3 mo
ouur

0 0
a uniform magnetic field of 0-3 T at an angle of
ad

(a) east. down


e ev
0 60'’ to the magnetic field. The pitch of the resulting
Y

helical path of protons is close to ;


2ma
(Mass of the proton = 1 -67 x 10"^^ kg, charge of
dY

nui
0 0
ib) west. up
Re

e ev the proton = 1-67 x 10“'^ C)


nnd

0
(a) 4 cm {b) 2 cm
FFii

ma 2tna (c) 12 cm
0 0 (d) 5 cm
(f) west. down
e ev
0 (JEE Main 2020)
50. A proton, a deutron and an a-particle are moving
3ma
with the same momentum in a uniform magnetic
ma
0 0
id) eiist. up (AIPMT 2013)
e ev
0 field. The ratio of magnetic forces acting on them
is
and their speeds are in the ratio ,
47. In the product. (a) 2 : I : 1 and 4 : 2 ; 1

F = ^(ux x(fi?+ it) (i) 1 ; 2 ; 4 and 2 : 1 : 1


(c) 1 : 2 : 4 and 1:1:2
A A A ●,

For ^ = I and u = 2 / + 4; + 6it (^0 4 : 2 : 1 and 2 : 1 : 1 (JEE Main 2021)


ANSWERI

45. (b) 46. (c) 47. (c) 48. (h) 49. (a) 50.(a)

\
MOVING CHARGES AND MAGNETISM 4/159

51. Ionised hydrogen atoms and a-particles with same 55. A horizontal rod of mass 10 g and length 10 cm
momenta enter perpendicular to a constant is placed on a smooth plane inclined at an angle
magnetic field B. The ratio of their radii of their of 60" with the horizontal with the length of the
paths will be : rod parallel to the edge of the inclined plane. A
uniform magnetic field induction B is applied
(a) 2 : 1 ib) 1:2
vertically downwards. If the current through the
(c) 4 ; 1 (</) 1 : 4 (NEET 2019) rod is 1-73 ampere, the value of B for which the
rod remains stationary on the inclined plane is
V. Force on current carrying Ig = 10 m/s“J
conductor in magnetic field {a) 0-5 T (b) l-OT
-1 (c) 1-5T id) 1-7 T
52. A metallic rod of mass per unit length 0.5 kg m
56. An arrangement of three parallel straight wires
is lying horizontally on a smooth inclined plane
placed perpendicular to plane of paper carry same
which makes an angle of 30° with the horizontal.
current 7’ along the same direction as shown in

ww
The rod is not allowed to slide down by flowing
Fig. 4(CF).24. Magnitude of force per unit length
a current through it when a magnetic field of on middle wire ‘B' is given by
induction 0.25 T is acting on it in the vertical

FF loo
direction. The current flowing in the rod to keep FIGURE 4(CF).24

it stationary is B. - d C
( 4)

ee
(a) 14.76 4 ib) 5.98 4 7

ee r
●' 90
(c) 7.14 4 id) 11.32 4

rFrF
(NEET 2018) d
rur
53. In the Fig. 4(CF).23, a coil of single turn is wound
on a sphere of radius R and mass m. The plane of
the coil is parallel to the plane and lies in the
ffoor AK
ks
equatorial plane of the sphere. Current in the coil
YYoou
is I. The value of B if the sphere is in equilibrium ib)
okos

ia)
is 2nd K d
BBoo

.n

1^0'“
re

(c) id)
%d Snd
(NEET 2017)
oouur
ad

57. Three very long straight current carrying


Y

conductors are placed parallel to each other as


shown in 4(CF).25. The conductors 4 and C arc
fixed whereas conductor B is free to move. If (he
dnYd
Re

conductor B is pulled towards irght through a vei^


(a) mg cos 0/(71 IR) ib) mg/n IR small distance r, then what is the net force acting
FFini

(c) mg tan 0/tc IR id) mg sin 0/tc IR on it and the angular frequency of the resulting
54. An electric current runs counter clockwise in a oscillation?

rectangular loop around the outside edge of this FIGURE 4(CF).25


page, which lies flat on your table. A uniform A B C
magnetic field is then turned on, directed parallel
to the page from top to bottom, the magnetic force
on the page will cause
-●1 id
(a) the bottom edge to lift
ib) the top edge to lift
(c) the left edge to lift
id) the irght edge to lift d d

51. {(i) 52. (d) 53. ib) 54. ib) 55. (/>) 56. (it)
4/160 7^'uuUc/b. 4. Fundamental Physics (XII) LWi
A

(a) -8/ {h) -161'


ia) ■1^0 ^^0 I M-q .(b)
Kd^ ’ V Kmd^ 4jtr^ y Inmd^ (c) 8? id) 16?
60. A rectangular loop of sides 10 cm and 5 cm
(c) carrying a current / of 12 A is placed in different
2;rr^ ’ 4jimd orientations as shown in the figures below :

1^0^^0 (d + r)-(d-r) I FIGURE 4(CF),28 ^


id)
2tz
’ y TtnuP- z

A- / B
58. A thin flexible wire of length L is connected to B 4
I
adjacent fixed points and carries a current I in the "I .d ●►y
>
clockwise direction, as shown in Fig. 4(CF).26.

ww
I /
When the system is put in a uniform magnetic X‘
(0 Hi)
field of strength B going into the plane of the
paper, the wire takes the shape of a circle. The

Flo
B B
tension in the wire is :
"I

ee
/“ I
FIGURE 4(CF).26 >7^

rere
I

rFF
X
(Hi) (iv)
uur r
foor If there is a uniform magnetic field of 0-3 T in the
positive z-direction, in which orientations the loop
would be in (/) stable equilibrium and (ii) unstable
ks s
equilibrium,
Yoo
ook

(a) (i) and («) respectively


BBo

IBL (b) (0 and (Hi) respectively


(a) IBL (b) (c) (//) and (iv) respectively
re

71

IBL IBL (d) (it) and (Hi) respectively (JEE Main 2015)
ouur

(c) (^0 (IIT 2010)


ad

2tc 4tc ■
VI. Torque on coll carrying current
YY

59. A conducting wire bent in the form of a parabola in magnetic field and galvanometer
= 2 X, carries a current / = 2 A as shown in Fig.
nndd

4(CF).27. The wire is subjected to a uniform 61. A coil in the shape of an equilateral triangle of
Re

magnetic field B =-2k tesla. The magnetic side I is suspended between the pole pieces of
FFii

—t
force on the wire in (newton) is permanent magnet. Such that fi is in plane of
FIGURE 1.27
the coil. If due to a current I in the triangle, a
torque x acts on it, the side / of the triangle is :
n1/2
X
(a) (b) 2
■J3{BI Sbi
*'X(m)

(d)
J-fjL"'
■^[bi
(AIPMT 2005)

ANSWERS

57. id) 58.(c) 59. id) 50. {(■) 61. ih)


MOVING CHARGES AND MAGNETISM 4/161

62. A conducting wire of length / is turned In the form Apg ApB


of a circular coil and a current / is passed through {a) sin0 = {b) cos 6 =
IB 2 IB
it. For the torque, due to magnetic field produced
at its centre, to be maximum, the numberof turns
2Apg 2Apg
in the coil will be (c) tan 0 = id) cot 0 =
IB IB
(a) one ib) two
(c)three (d) more than three. 65. A conducting ring of mass 100 gram and radius
0-5 m is placed on a smooth horizontal plane. The
63. A circular loop of mass m and radius r in X-Y
ring carries a current of / = 4 A. A horizontal
plane on a horizontal table as shown in Fig. magnetic field B =: 10 T is switched on at time
4(CF).29. A uniform magnetic field B is applied r = 0 as shown in Fig. 4(CF).31. The initial angular
parallel to X-axis. The current / in the loop, so acceleration of the ring is
that its one edge just lifts from the table is

ww
FIGURE 4<CF).31
FIGURE 4(CF).29
>
y

Floo
7 >6

ee
eer
>x

rFrF
rur
(a) 200 71 rad/s^ (b) 400 n rad/s^

mg mg
s ffoor
(c) 800 Tt rad/s^ id) 1200 7C rad/s^
ia) ib) VII. Conversion of gaivanometer
osk
YYoou
nrB InrB
into ammeter and voltmeter
oook

mg 2nrB
eBB

ic) (d) 66. In the circuit given below, a voltmeter reads 20 V


Ttr^ B mg
when it is connected across 400 resistance.
64. A wire of cross-section A, density p forms three Calculate what the same voltmeter will erad when
r

sides of a square of each length a. It is free to connected across the 200 Q resistance.
ouur
ad

rotate about axis CD. It is_held vertically in a


Y

uniform magnetic field B acting vertically FIGURE 4(CF).32


upwards. When current I is passed through it, the 20V
nd dY
Re

structure is deflected by an angle 0 from the


vertical, then the value of 0 is given by
FFini

^Wv B
200 a ^ 400 n
FIGURE 4(CF).30
—>
B
I h M
D C
50V

id) 100 V ib) 50 V


(c) 30 V id) 10 V
67. A galvanometer of resistance 25 Q is connected
to a battery of 2 volt along with a ersistance in
E series. When the value of this resistance is 3000 Q,

ANSWERS

62. ia) 63. ib) 64. id) 65. (c) 66. (d)
4/162
Fundamental Physics fXinPTSTWl
a full scale deflection of 30 units is obtained in
70. When a current of 5 mA is passed through a

the galvanometer. In order to reduce this galvanometer having a coil of resistance 15 3^, it
deflection to 20 units, the resistance in series will shows full scale deflection. The value of the
be
resistance to be put in series with the
(a) 4514 n {b) 5413 n galvanometer to convert it into to voltmeter of
(c) 2000 n id) 6000 il. range 0- 10 V is

68. In an ammeter 0-2% of main current passes (a) 2-535 X 10^12 ib) 4-005 X 10^ £2
through the galvanometer. If resistance of (c) 1-985 X 10^ Q id) 2-045 X 10^ Q
galvanometer is G, the resistance of ammeter will
(JEE Main 2017)
be
71. The current sensitivity of a moving coil
G 499 galvanometer increases by 35%, when its
(fl) ib) G
499 500 resistance is increased by a factor 3. The voltage

ww
sensitivity of galvanometer changes by a factor
C 500 G
(c) id) (a) 35% (b) 45%
500

Floo
499
(c) 55% (d) none of the above
(AIPMT 2014)
72. A galvanometer having a resistance of 20 £2 and

ee
69. A galvanometer has a sensitivity of 60 30 divisions on both sides has figure of merit

reer
division/ampere. When a shunt is used its 0-005 ampere/division. The resistance that should

rFF
sensitivity becomes 10 divisions/ampere. What is be connected in series such that it can be used as
the value of shunt used if the resistance of the a voltmeter upto 15 volt is
uur r
galvanometer is 20 £2 ?
(a) 2 £2 (/>)3£2 ffoor
(a) 100 £2
ic) 80 £2
ib) 125 £2
id) 120 £2
sks
(c)4£2 (rf)6£2
YYoo
(JEE Main 2019)
ooko


eBB

Multiple Choice Questions (with One or More than One Correct Answers)

73. A proton moving with a constant velocity passes id) Both, electric and magnetic fields may be
r
ouur

through a region of space without any change in present between t and At, but electric field is
ad

its velocity. If £ and B represents the electric fields definitely present.


YY

. and magnetic fields respectively, then this region 75. He'*' and all having same kinetic energy
of space may have : pass through a region in which there is a uniform
nndd
Re

(a)£=0,£ = 0 ib)E = 0,B^0 magnetic field perpendicular to their velocity. The


(c) £ 0, £ = 0 id)E:fi0,B^0 masses of H"*", He'*’ and O^’*’ are 1 amu, 4 amu and
FFii

74. The K.E. iK) of a charged particle is zero at time 16 amu respectively. Then
t, 2 K at time 4t and 4k at time 5c and then it (a) H"* will be deflected most
remains constant upto time 8/. Choose the correct ib) will be deflected most
statement/statements out of the following : (c) He’’’ and 0“’’’ will be deflected equally
(fl) The electric field is the only field present id) all will be defiected equally
between t and 4t
76. A particle of charge + q and mass m moving under
ib) Between 5t and St only magnetic field is
present the influence of a uniform electric field e7 and
A

(c) Both, electric and magnetic fields may be uniform magnetic field Bk follows a trajectory
present from £ to Q as shown in 4(CF).33.The velocities
ANSWERS

67. («) 68. ic) 69. ic) 70. ic) 71. ic) 11. (c) 73. iu,b,d)
74. ic.d) IS. (a.c)
MOVING CHARGES AND MAGNETISM 4/163

A (cO They will come out at different times.


at Pand Q^t vi and -2v j respectively which (IIT 2011)
of following statementfs) is/are correct ?
79. Consider the motion of a positive point charge in
a region where there are simultaneous uniform A
FIGURE 4(CF).33
electric and magnetic fields E=EqJ and
Si
P B = BqJ .At time t = 0, this charge has velocity
~v in the jc-y plane, making an angle 6 with the
a
©BA x-'dxis. Which of the following option(s) is (are)
correct for time / > 0 ?
(a) If 6 = 0®, the charge moves in a circular path
Q
in the x-z plane
2a
-2vj (b) If 0 = 0®, the charge undergoes helical motion

w
with constant pitch along the y-axis
(c) If 0 = 10®, the charge undergoes helical motion

Flo
3 in v~
■) >
with its pitch increasing with time, along the
(a) E = - y-axis

e
4 qa
(cO If 0 = 90®, the charge undergoes linear but

rree
(b) Rate of work done by the electric field at P is accelerated motion along the y-axis (IIT 2012)

r FF
3 mv
3 80. Two long parallel wires, AB and CD, carry equal
currents in opposite directions. They lie in the
uurr
4 a

(c) Rate of work done by the electric field at


for
x-y plane, parallel to the x-axis, and pass through
the points (0, - a, 0) and (0, a, 0) respectively,
kss
P = 0
Fig. 4(CF).34. The resultant magnetic field is :
ooook
Yo
(f/) Rate of work done by both fields at Q is
zero
eB

77. For a charged particle moving in a magnetic field,


the velocity vector and acceleration vector at some
rr

A A A

instant are t; = 3i + 4j and a = 2i + x j


ou
ad

respectively. Select the correct alternative (s):


Y
dY

(a)x = - 1-5
(b)x = 3
Re
innd

(c) magnetic field is along z-direclion


Fi

(d) K.E. of particle is constant


78. An electron and a proton are moving on straight
parallel paths with same velocity. They enter a {a) zero on the x-axis
semi-infinite region of uniform magnetic field (b) maximum on the x-axis
perpendicular to the velocity. Which of the (c) directed along the z-axis at the origin, but not
following statement(s) is/are true ? at other points on the z-axis
(a) They will never come out of the magnetic field (d) directed along the z-axis at all points on the
region z-axis.
(b) They will come out travelling along parallel 81. A beam of electrons starts to accelerate from rest
paths due to a uniform electric field in vacuum, the
(c) They will come out at the same time moving electrons :

ANSWERS

lit. Ui.h.(l) 77. Ui.c,(l) 78. ih.ii) 79. U\(i) 80. {h,<D
4/164
“Pn^teUefr’^ Fundamental Physics (XIOESDD
(a) initially experience a force of mtual repulsion
vE.
(b) experience a force of mutual attraction after (c) By = By+ vE^
travelling a certain distance
(c) will continue to diverge due to electrostatic 86. A particle of mass m and charge q moving with
repulsion velocity v enters region II normal to the boundary
(d) will follow parallel lines because there is no as shown in Fig. 4(CF).35. Region II has a uniform
force of attraction between them. magnetic field B perpendicular to the plane of the
paper. The length of the region II is /. Choose the
82. Current flows through a straight cylindrical correct choice (j).
conductor of radius r. The current is distributed
uniformly over its cross-section. The magnetic FIGURE 4(CF).35
field at a distance jc from the axis of the conductor Region I Region II Region III
has magnitude B: X X X X

ww
(a) B = 0 at the axis X X X X

(/?) fi oc X for 0 < X < r V


X X X X

(c)B oc I/x for X > r X X X X

Flo
(d) B is maximum for x = r X X X X
●—/—►
83. A microammeter has a resistance of 100 Q and a

ee
full scale range of 50 [l A. It can be used as a (a) The particle enters region III only if its velocity

rere
voltmeter or a higher range ammeter provided a

rFF
V >qlB/m
resistance is added to it. Pick the correct range
and resistance combination(s).
(b) The particle enters region III only if its velocity
uur r
V <qlBlm
(a) 50 V range with 10 A: ^2 resistance in series
(b) 10 V range with 200 k Q resistance in series
ffoor (c) Path length of the particle in region II is
sks
q! B
(c) 5 mA range with 1 Q resistance in parallel maximum when velocity ^ -
Yoo
m
oko

(d) 10 mA range with I Q resistance in parallel


(d) Time spent in region II is same for any velocity
84. Two ions have equal masses but one is singly
BBo

V as long as the particle returns to region I


ionized and the other is doubly ionized. They are
re

(irr 2008)
projected from the same place in a uniform mag
netic field with the same velocity perpendicular 87. In cyclotron, an ion is made to travel successively
ouur
ad

to the field, along semicircles of increasing radius under the


action of a magnetic field. The angular velocity
YY

(a) the two circles touch each other


(b) the two circles do not touch each other of the ion is independent of
(a) speed of ion
nndd

(b) radius of the circle


Re

(c) the circle described by the singly-ionized


charge will have a radius double that of the (c) mass of the ion (d) charge of the ion
FFii

other circle (National Standard Exam in Physics 2012)


(d) both the ions will go along circles of equal 88. A conductor shown in Fig. 4(CF).36 carrying
radii. constant current / is kept in the x-y plane in a
85. Let E and B denote electric and magnetic fields uniform magnetic field B. If F is the magnitude
of the total magnetic force acting on the
in a frame S and E' and B' in another frame S' conductor, then the correct statement(s) is (are)
moving with respect to S at a velocity v . Two of FIGURE 4(CF).36
the followinge quations are wrong. Identify them
R R
vB, I 7T/4
(a) Ey'^Ey- c2 - (b)Ey'^Ey+vB^
Hi
¥ *-X
M L R-^W-R->H L H

ANSWERS

81. (a.b) 82. ia.b.c,d) 83. (b.c) 84. {ax) 85. (ax) 86. (a.c.d) 87. {a.b)
MOVING CHARGES AND MAGNETISM 4/165

(a) If B is along z, (L + /?) (c) If B is along y, F°^(L + R)

(b) If B is along x, F = Q (d) If 5 is along z, F = 0


(JEE Advanced 2015)

mi Multiple Choice Questions (Based on the given Passage/Compreh ension));


Each comprehension given below is followed hy some multiple choice questions. Each question has one
correct option. Choose the correct option.

D In a moving coil galvano Fig. 4(CF).37 shows a


meter, there is a coil of copper having number circular loop of radius a with two long
of insulated turns N, each of area A. The coil parallel wires (numbered 1 and 2) all in plane

ww
is suspended in a radial magnetic field B. The of paper. The distance of each wire from the
centre of the loop is d. The loop and the wires
moment of inertia of the coil about its
are carrying the same current I.

Floo
rotational axis is I. The scale divisions in the
The current in the loop is in the counter
galvanometer are n and resistance of the coil clockwise direction if seen from the above.

ee
is R.
FIGURE 4(CF).37

reer
Answer the following questons i5

rFF
Q I 1
I
d
89. If a current ig in the coil produces a deflection of 4
I

7c/3 radian to the pointer of galvanometer, the


uur r
I

value of torsional constant of the spring is


ffoor CNJ

£
sks
V
NBAi0
(b) t I
YYoo
(a) 2n
I
3n
ooko

I
I
I
P ■ R
eBB

NBALk 3NBAI0
0
(0 — id)
K
Answer the following questions :
r

90. The voltage sensitivity of the galvanometer (in 92. When d-a, but wires are not touching the loop.
ouur
ad

rad/volt) is It is found that the net magnetic field on the axis


of the loop is zero at height h above the loop. In
Y

K 3LR
0 that case
ia) 3R ib)
nddY

K
id) current in wire 1 and wire 2 is in the direction
Re

PQ and RS, respectively and h = a


FFiin

n K
ib) current in wire 1 and wire 2 is in the direction
3LR id)
0 PQ and SR, respectively and h = a
(C) current in wire 1 and wire 2 is in the direction
91. If the charge Q is passed almost instantaneously
PQ and SR, respectively and ft = 1 -2 a
through the coil, the angular speed of the coil
id) current in wire I and wire 2 is in the direction
immediately after this is
PQ and RS, respectively and ft = 1 -2 a
NBAQ 2 NBAQ (JEE Advanced 2014)
id) ib)
I I 93. Consider d»a and the loop is rotated about its
diameter parallel to the wires by 30® from the
3 NBAQ NBAQ^ position in the Fig. 4(CF).45. If the currents in
ic) id) the wires are in the opposite directions, the torque
I I
on the loop at its new position will be (consider
ANSWERS

88. (a,b.c) 89. (d) 90. (c) 91. (a) 92. (c)
4/166
Fundamental Physics (XII) VOL.I

that the net field due to the wires is constant over


^/3Mo/
2 _2
V3(lo/
2 _2
the loop)
a a
(c) {<£)
d Id

(a) (b) (JEE Advanced 2014)


d Id

09 Matching Type Questions


DIRECTIONS. In each of the following questions, match column I and column II and select the correct
match out of the four given choices.
94. Column 1 Column II
(A) Ampere Swimming rule ip) Direction of induced current in a conductor
(B) Fleming’s Left hand rule {q) Direction of magnetic field lines due to

ww
current through circular coil
(C) Fleming’s Right hand rule (r) Direction of deflection of magnetic needle
due to current in a straight conductor

FF loo
(D) Right hand thumb rule (s) Direction of force on the current carrying
conductor due to magnetic field

ee
(a) A-p ; B-^ ; C-r ; D-s {b) A-q ; B-r ; C-^ ; D-p (c) A-r, B-s ; C-p ; D-^ {d) A-j ; B-p ; C-q ; D-r

er
rFreF
K9 Matrix-Match Type Questions
rur
p q s
T I

DIRECTIONS. Each of the following questions contains ffoor statements


A
o o Ol
ks
given in two columns, which have to be matched. The answers to these
questions have to be appropriately bubbled. If the correct matches are
B
0 ©O©
YYoou
okso

A-p, A-g ; B-^, B-r; C-r, C-s and D-s, then the correctly bubbled matrix
will look like the one shown here.
c
© © Ol r©
BBoo

D
© ©1 © m
r ee

95.
There are some regions having electric or/and magnetic fields in column I and the possible trajectories
oouur
ad

of the positive lest charge in rest/molion in column II are given :


Y

Column I Column II
dnYd

(A) A positively charged particle is projected with (P)


Re

Straight line
some velocity in a region with uniform electric field
FFini

(B) A positively charged particle is projected with (q) Parabola

some velocity in a region with a uniform magnetic field.


(C) A positively charged particle is projected with a velocity (r) Circle

lying in a plane normal to a region containing, parallel


. uniform electric and magnetic fields.
(D) A positively charged particle is released from rest in (s) Helix

a region containing perpendicular uniform electric and


magnetic fields.

ANSWERS
93. (h) 94. (c) 95. A-p, q ; B-p, q, r ; C-.r; D-p, s
MOVING CHARGES AND MAGNETISM 4/167

VI. Integer Type Questions A B C D

©®@©
DIRECTIONS. The answer to each of the following questions is a single digit integer,
ranging from 0 to 9. If the correct answers to the question numbers A, B, C and D
©0©o
(say) are 4, 0, 9 and 2 respectively, then the correct darkening of bubbles should be 0© ®©
as shown on the side :
©©©©
96. There are two infinite long parallel straight current carrying wires, A and B separated
by a distance r. Fig. 4(CF).38. The current in each wire is I. The ratio of magnitude
©©®®
of magnetic field at points P and Q when points P and Q lie in the plane of wires is : ©© ®©
FIGURE 4(CF).38 ©©©®
0 ©0 0
A B

ww
●I
©©©®
--P 4 MQ © :© ® ©

Flo
e
-I

reree
4 r ►

r FF
97. A length of wire carries a steady current /. It is bent first to form a circular plane coil of one turn. The same
uurr
length is now bent more sharply to give double loop of smaller radius. If the same current I is passed, the
foor
ratio of the magnitude of magnetic field at the centre with its first value is :
98. A current 1 amp is flowing in the sides of an equilateral triangle of side 4-5 x 10"^ m. The magnetic field at
kss
the centroid of the triangle in the units of (10”^ T) is :
Yoo
ooook

99. A cylindrical cavity of diameter a exists inside a cylinder of diameter 2 a as shown in the Fig. 4(CF).39.
eBB

Both the cylinder and the cavity are infinitely long. A uniform current density J flows along the length. If the
(IIT 2012)
magnitude of the magnetic field at the point P is given by — a 7 , then the value of N is
rr

12
ouu
ad
Y
dY
Re
innd
Fi

100. A galvanometer gives full scale deflection with 0-006 A current. By connecting it to a 4990 W resistance,
2/1
it can be converted into a voltmeter of range 0-30 V. If connected to a ^ resistance, it becomes an
249

ammeter of range 0 - 1 -5 A. The value of /i is : (JEE Advanced 2014)

ANSWERS

96.(8) 97.(4) 98.(4) 99.(5) 100.(5)


4/168
“Ptadee^'^. Fundamental Physics (XII)LV»1BI

VII.
Assertion-Reason Type Questions
FOR MEDICAL STUDENTS (A) Statement-I is true ; Statement-2 is true ; State
ment-2 is a correct explanation of Statement-I.
DIRECTIONS. In each of the following questions
read the two statements and choose if: (B) Statement-I is true ; Statement-2 is true ;
(A) both Assertion and Reason are true and the Statement-2 is not a correct explanation of
Statement-1.
Reason is the correct explanation of the
Assertion. (C) tatement-1 is true ; Statement-2 is false.
(B) both Assertion and Reason are true but Reason (D) Statement-1 is false ; Statement-2 is true.
is not a correct explanation of the Assertion. 104. Statement-1. If an electron and proton enter a
(C) Assertion is true but the Reason is false.
perpendicular magnetic field with equal
(D) both Assertion and Reason are false. momentum, then path of electron is more curved

ww
than that of proton.
101. Assertion. If a proton and an a-particlc enter a
Statement-2. Electron is lighter particle.
uniform magnetic field perpendicularly, with the
same speed, then time period of revolution of the (a) A (b)B (c)C {i/)D

Floo
a-particle is double than that of proton. 105. Statement-1. Higher the range, greater is the
resistance of ammeter.
Reason. In a magnetic field, the time period of

ee
revolution of a charged particle is directly Statement-2. To increase the range of ammeter,

eer
proportional to mass of particle and is inversely additional shunt needs to be used across it.

rFrF
proportional to charge of particle, (a) A {b)B (c)C (cOD
(a) A (b)h (c)C (d)D (JEE Main 2013)
rur
102. Assertion. If two long parallel wires, hanging
freely are connected to a battery in series, they
ffoor
106. Statement-1. A proton and an a-particle are
projected with the same kinetic energy at right
s
come closer to each other. angles to a uniform magnetic field, then the a-
osk
YYoou
Reason. Force of attraction acts between the two particle will move along a circular path of smaller
oook

parallel wires in series carrying current, radius than the proton.


eBB

(a) A (b)B (c)C (i/)D mv


2

103. Assertion. When a magnetic dipole is placed in a Statement-2. Bqv-


r
non uniform magnetic field, only a torque acts
r

(a)A ib)B (c)C (i/)D


on the dipole.
ouur
ad

107. Statement-1. The sensitivity of a moving coil


Reason. Force would also act on dipole if galvanometer is increased by using a soft iron
Y

magnetic field were uniform.


cylinderas a core inside the coil.
(fl)A (b)B (c)C (</)D
nd dY

Statement-2. Soft iron has a high magnetic


Re

FOR ENGINEERING STUDENTS permeability and cannot be easily magnetised or


demagnetised,
FFini

DIRECTIONS. In each of the following questions (a) A (b)B (c)C (d)D


read the two statements and choose if. (UT 2008)

VIII.
U Multiple Choice Questions (Based on
ExperimentaliiBltillsi)||
108. A galvanometer with resistance 1000 O gives full (c) 0-10 Q in series (d) 0-10 in parallel
scale deflection at 0-1 mA. What value of
resistance should be added to 1000 n to increase
109. A galvanometer of resistance 3663 Q gives full
its current range 10 A. scale deflection for a certain current 1^. The value
of the resistance of the shunt which when joined
(a) 0 01 in series (b) 0-01 in parallel to the galvanometer coil will result in 1/34 of the
ANSWERS

101. (a) 102. (J) 103. (d) 104. (d) 105. {d) 106. (d) 107. (c) 108. (b)
MOVING CHARGES AND MAGNETISM 4/169

total current passing through the galvanometer scale deflection. The figure of merit of the
IS galvanometer in microampere/division is
(cj) 100 n ib) 200 n (a) 330 ib) 660
(c) 300 O (d) none of the above (c) 500 (d) 600
110. In the circuit, shown in Fig. 4(CF).40, 112. A resistance of 980 .Q is connected in series with
R = 5000 n. If key is closed, galvanometer a voltmeter, after which the scale division
shows a deflection of 30 scale division. On
becomes 50 times larger. The resistance of
closing key K2 and making S = 20 fi, the voltmeter is
deflection of galvanometer reduces to 15 division.
The resistance of galvanometer is (a) ion (b) 20 n
(c) 100 a (d) 200 a
FIGURE 4(CF).40
113. In the circuit shown in Fig. 4(CF).41 ammeter A
E = 2V Ki
is a galvanometer of resistance 60 n shunted with
I—c>

ww
resistance 0-02 Q. What is the current recorded
by ammeter ?
■R
FIGURE 4(CF).41

FF loo
4IH>^

ee
K2 30

ee r
rFrF
ia) 50 a ib) 30 a 3V

(c) 20 n id) 15 O
rur
111. A galvanometer having 20 division scale and 50 O
resistance is connected in series to a cell of e.m.f.
ffoor
ia) 0-5 A ib) 1-0 A
ks
1*5 V through a resistance of 100 O, shows full (c) 2-0 A id) 10 A
YYoou
ookos
BBo
re

For Difficult Questions


ouur
ad

Multiple Choice Questions (with one correct Ans^^


Y

1.
dndY

(2 sin a) (as OC = R cos a)


Re

4 71 /? cos a
FFini

or B __V tanct
BA ~
iTtR

It is acting vertically downwards.

B
ADB ~ (27t-2a) (71-a)
AnR 271/?

It is acting vertically downwards.


Magnetic field due to current in wire BA, Total magnetic field, B - Bgj^ +
_

(7t-a + tana)
B BA “
[sin a + sin a] InR
4tc iOC)
ANSWERS

109. id) no. ic) 111. (c) 112. ib) 113. ib)
4/170
Fundamental Physics (XII) VOL.I

[M IlNtllg/iE X R ^
For Difficult Quoatlons = 3x
HI — x2x3
4k a
= 9^
iTia
2. The edge of every side of hexagonal will make 4. Refer Fig. 4(CF).45. Magnetic field at P due to
an
current through CO is same as that due to current
angle 60“ with the line joining the edge to the through OD.
centre O. So <j), = 30“ and (j)2 = 30“.
Total magnetic field induction at O due to current I

through all the six sides will be

B = 6x-^i2. ^ (sin <{>] + sin


4k OG

oww
e
Total magnetic field at P due to currents

re
FFrlo
through CO and OD is
^ “ ^co + ^OD = 2 B CO [v ^CO-^OD^

rF
ee
= 2x
H /
[sin 45“ + sin 90“]

rF
4jc ir/-j2)
ouru
fosor
skf 1

Here, OG = flcos30“= V5o/2 4k r LV2 J 4jt r


ooko
5. Refer to Fig. 4(CF).46
YYo
B = I M ^ 1
4n i^a/2)[2'^2)~ FIGURE 4(CF).46
Bo

Ka
reeB

D
3. Refer to Fig. 4(CF).44. P Ba

-2 cot60“=-x-^
d
InAOBD, r =
ooY

2 ^
uur

B-
G
E
ad

Magnetic Held induction at O due to current A O B


through all the three sides of a triangle will be
dY

I2
nind
Re

C
FFi

B\=^^
2;r a
acting in the plane of paper downwards
B.I1 acting in the plane of paper towards right.
2n d

/. B
iKd

B = 3x
HI
- (sin 60“ + sin 60“) 6. As per question, it is clear that current through
4k r QS should be 7/2 in the direction Q to S.
Magnetic field induction at a distance r from a
= 3x long straight conductor carrying current I is
4jtr_£_^[ 2 2
,2^3 J B =
47ir
(sin 4>, + sin <1)2)
' ^
MOVING CHARGES AND MAGNETISM 4/171

tfllNiTLSv; E-X yp
magnetic field induction is along the ersultant of
For Difficult Queotiono
A ^ A . 1 A A
I and * i.e., (,-+jt)
As the point M is at the end side of a long straight V2
conductor, so (|>| = 90® and <j>2 = 0®. Therefore
magnetic field induction at M due to current
through PQ is

B = (sin 90® + sin 0®)


471 r

“»
4nr acting- perpendicular
- - to the plane of
paper outwards.
The magnetic field induction at M due to current

ww
in QR will be zero. As per question

h.^^L
1 9. Refer to Fig. 4(CF).48,
47t r -* A ^

Flo
-► ^ r XI+VI
When straight conductor QS is connected at Q, r = xi + yj; and r = - = , . ~ ~
+7

e
reree
u,e„ Magnetic field induction at P,
^ 4n r 4k r 4ti(^2 r j

r FF
So H^ _ HQ//(47cr) _ 2 47T r 2tc + y2
uurr
4k[2 foorIt acts perpendicular to position vector.
FIGURE 4(CF).48
ks s
7. Magnetic field induction due to a straight long
Yoo
ooook

conductor at a distance r away from it is given by


P(x.y)
eBB

B = ^
4ti r
. Its direction is given by Right hand
thumb rule. The magnetic field induction due to
rr
ouu

current in the two wires in the space between them


ad

is in opposite direction, therefore, the net magnetic


YY

fig 21A fiQ 21 4


field is B =
4jt X ^ 4k (d — x)^
nndd
Re

At mid point, ;c=d!2, the magnetic field B is zero.


Fii

Unit vector perpendicular to the position vector


Atx< d/2, the magnetic field B is along j . At A A A A A
A_A^A jxi+yj)xk_ yi-xj
> d/2 the magnetic field is along - j . When x
is negative, then the value of B is negative. When
x>d, then the value of B is positive. Therefore, ●* ●* A
[L^Kyi-xJ)
B=\B\n =
the correct option is as shown in Fig. 4(CF).5(ft).
2tc (x^ + y2)
8. Refer to Fig. 4(CF).47, the direction of magnetic
field induction at a point (a, 0, a) for the current 10. Refer to Fig. 4(CF).49, the magnetic field at O
due to portion ABCD is pointing along z-axis i.e. due to straight wire carrying current is

represented by k ● The direction of magnetic field 5=l^i.[sin30®+sin30®]ik


4ti a
induction at the given point due to current in
portion DEFA is pointing +x axis i.e. represented = l i 1
A

by i . Therefore, the effective direction of


4ic a 12^2. 47C a
4A172 Fundamental Ph3^ics (XII) VOL.I

acting perpendicular to the plane of loop


For Difficult Questions downwards.
Magnetic field induction at O due to current
FIGURE 4(CF).49 through FT or er is
a

HZf . n . jc

^PT-^- - sm-+sin-

4jcr/2\^ 44; 67cr


acting perpendicular to the plane of loop upwards.

Hie resultant magnetic field induction at O is
04r B = BpQ + Bq^ - Bpj-B 7S
'"J3(
S
_ >^Hn/,
3jcr 37ir
'l^yLgl
6jtr 6nr
- J

ww
3nr

Tbtal magnetic field at O due to current through 13. The two parallel beams of protons and electrons

Flo
all the straight parallel wires carrying currents will be equivalent to two linear parallel line wires

e
canying currents in the same direction. Refer Fig.
B=>B^k + B^i-k) + F3 ik) + F4 (-ik)+... 4<CP).S0, the resultant magnetic field at P is

ere
FFr
B — RjB2 + Fj — F4 + .... B
FIGURE 4(Cn.80
D
uurr
orr
4?t fl L
sfo
4;r -log^(l
a + l) =
kks
Yo
oooo

X P
Here, cos30®= —r or a = rcos 30® = r-y/312
(d-x)^
eBB

Li iil

4jc (rVfT^ ^ InTJs ^


B =
rr

A ♦ C
11. L = 2 JC r = 2 It r| N or r=1/2 7t and rj = 1/2 n N d
ou
ad
YY

jto/_ Up/ 1^0 2/ Ho 2/


2r “ 2(L/2ic) J F* ^^21 i- 1

4jc ;c 4jc(</-x) 4jc Ljc


nndd

B _PqA^/ _ {ipNI . B 1 If = dtl, B^Q.\Sx< dll, B is positive and if


Re

2r. "2(I/2jcN) ‘ ' Be" x>dH then B is negative. Thus variation between
Fi

F and ;c will be represented by option (F).


12. Let /] and I2 be the currents in arms PQS and 14. Refer to Fig. 4(CF).51, when currents in the two
PTS respectively. Then / = /j + /j. wires are in the same direction, then magnetic
As resistance of arm PQS = R field induction at O is
and resistance of arm PTS = 2 R
FIGURE 4(CF).51
r / 2/ . /
Therefore, ^ and ^2 3
Magnetic field induction at 0 due to current
through PQ or QS is
19.
F = F
Ho A r ● JC - It 4- Xi -►K—
PQ QS = . sm- + sm-
4JC//2L 4 4J
Li iil

= ii!LMx4.=:^
4jc rl2 V2 3jtr
Xo
MOVING CHARGES AND MAGNETISM 4/173

When the magnetic dipole moment is doubled


For Difficult Queationa
{le. ni -2 m).
Let be the radius of circular loop, then
Bi = m'=^2m=^InR^ or InR^-2xIxnR^
2nxi 2n(xo-xi)
1 ■ _pp/r .<0-2x1' or R' = ^l2R
= M 1_
2k[xi 2jc Laci(A^-xi) lto27c/ Up/
When the currents in the two wires are in opposite Magnetic field By = 4ic R 2R
directions, then
and
=iiaZ
1 «2 =
2x(Sr)
2jiLjci (;c^“X|)J 2n Lx|(jqo-;ci)
In magnetic field, the radius of curvature of curved
path of the moving charged particle is related to Bz

ww
B as
17. For A, l = 2nRorR = U2n
1

Floo
^ 2R 2xU2n I
or
^ _ XQ _ Xq/Xi _ 3
~(xo-2xi) “(V^i-2) 3-2 “ For B, Refer to Fig. 4(CF)-53

ee
15. Refer to Fig. 4(CF).52, the magnetic field at point FIGURE 4(CF).53

reer
A due to current in long wire PQ is

rFF
a
R
¥
FIGURE 4(CF).S2 I
uur r
At
V = 10^ ms,-i
ffoor 45Ys y
a

''I
sks
YYoo
20 cm
ooko

P 5A Q <■
> P S
eBB

FIGURE 4.13 / = 4fl or a-1/4


OT^a/2 = l/S
r
ouur

— =ixlO"*T Magnetic field at O due to current through a


ad

B =
4jt r (20x10-2) 2 square is
Y

Its direction is upwards, perpendicular to the plane 1^0 /


B„ =4x [sin 45® + sin 45®]
dY

of paper. B
4n (OT)
Re

Now magnitude of force acting on the electron


nnd

due to this field is 1


FFii

= 4x
4ii l/B LV2 V2j“ n^l
IFI 9 I V X 0 I = ^vBsinO
B
A _ (IIqWM) _ V2w2 _
Here, v
= 105|ra-‘, B = ^xl0->T, Bb " (16Hp
18. Consider a small element of the disc of radius
“ 16 ” 8^2
x
?= 1-6 xl<r'»C. 6 = 907 width dx. Fig. 4(CF).54. Face area of the small
element of disc -2 nxdx
IFI=(1-6x10-^^)x105x V24x10'^ xsin90® Charge on the elementary part of disc,
= 0-8 X KT'® N dq = -^x2nxdx
nr^
= 8xl(r®>N
2qxdx
16. Let R be the radius of the circular loop or dq =
r2
Magnetic dipole moment, m = / x jt r2
4/174
4 Fundamental Physics (XII)

20. Total magnetic field induction at P is


For Ptfficult Questions

B - Bah+Bi,c+

-(-;) + (“*) +
4tc r 4ti r

4k r 4tc r

nl/2
^2 / \2
I B\ = 'HoH + HoZLZ.
, 4tc r 4k r

ww
Magnetic field induction at 0 due to rotation of
elementary ring with frequency v is
4n r
dB =
M^O '^.Kdqv
('.● dl = dqx v) 21. Refer to Fig. 4(CF).56.

Flo
4n X
Magnetic field induction at centre O due to coil
1^0 2qxdx

e
X

reree
XV =
lx r2 carrying current / is b^- 2R

r FF
Total magnetic field conduction at O due to Magnelic field induction due to other coil at O,
rotation of whole disc with frequency v is carrying current 2 / is
uurr
B =
r 1^0^"
0
r- foor
r
kss
19. Since the two coils subtend the same solid angle
Yoo
ooook

at O, hence area of coil D = 4 x area of coil C,


area
eBB

I because solid angle =


(-Lr distance)^
Therefore, radius of coil £) = 2 x radius of coil C
rr
ouu

_^o 27r/(2r)2
ad

● B
D ~
47c[(2r)2 + a2]3/2
Y
dY

fiQNll fXQNI
52 = = 25 1
Re
innd

2R R
Fi

Angle between S| and 82 is 9 = 60°.


Resultant magnetic field induction at O is

5 = 1/5,^ + 52^+2 Si 52 cos 60°


= + (2 5i )2 + 2 5, X 2 5i XY2
2nl
«C =
4;i[r2+(a/2)2]3/2 = ^/7 5l = 2R
-i3/2
4r2 + fl2 22. Take an element of thickness dr at a distance r
● ^= 4 4 1

Be {4r2+a2)3/2^ 4 8 " 2 from the centre of spiral coil as shown in Fig.


4(CF).57.

i
MOVING CHARGES AND MAGNETISM 4/175

24. Magnetic field at a point on the axis of circular


For Difficult Questions coil is

B =
fio 2nNIR^
4n (/?2 + a;2)3/2 2(/?2 + x2)3/2

At AT] = 0-05 m, Bj = 2[/?2 + (0-05)2p

w
iipN/R^
At;t2 “ ^2 ~
2 [/?2 +(02)2^2

e
B, (R^ + 004)^^^

re
5, (i?2+00025)^^"

row
B 8
As per question, —L =

eeF
ullo
1
^2

FF
No. of turns per unit thickness of spiral coil
N 8 _ (/?^ +0-04)2^2
{b-a) I ~ (/?2 + 00025)3/2

srr
roF
No. of turns in the element of thickness dr is
^8^2/3 (/?2+0-04)

k
or
N
dr 1 r} + 0-0025
uor
ofof
n =

(b - a)
Magnetic field induction at the centre of the spiral or
4 _ /?2 + Q.04
1 " )?2 + 0-0025
Y
kos
coil due to current in element of thickness dr is
Yo
4 {R^ + 0-0025) = R- + 0-04
B

or

2nnl _ lip f Ndr ^ 1 _ PpNI ^


oo

dB = 4/?2 + o-OIOO = /?2 + o-04


rY

or
4?t r 2 b—a r 2{b-a) r
erB

or 3/?2 = 0-04-0-01 =0-03


e

Total magnetic field induction at the centre of


or R- = 0-0\ or /?=0*lm
u

spiral wire due to current through the wire is


25. Consider two elements on semicircular cro.ss-
ou
d

dr ]X.gNI ‘
o

dr
ad

B = section of long wire at C and D, each of length <//


2{b-a) r 2{b-a) ^ r symmetrically situated with respect to point O
nY

a a
lying on the axis of wire, having semicircular
cross-section as shown in Fig. 4(CF).58.
nid
Re

2(b-a) a
F

23. Magnetic field at the centre of a circular coil of


Fi

radius r.due tocurrent/is B =


Pq 2nl _ PqI
4n r 2r

Magnetic field at a point on the axis of a circular


coil at a distance x (= r/2) from the centre of the
coil is

B' _
= ^^0 iKlr^ Pplr^
4n (r2+jc2)3/2 2(^2 + ;c2j3/2
Current through each element,
4\3/2
= B
dl = — dl and dl = r dQ ,■
2[r2+(r/2)2]3/2 2r(5/4) 3/2 5 jcr

dl = -rd^ = -d^
sT nr n
4/176 4 Fundamental Physics (XII) P/«TI
,A. vfS 9
Time period of rotation of disc, 7=2 7t/o)
For Difficult Questions
Current / associated with rotating charge dq is
Magnitude of magnetic field induction at 0 due j_ 2Qrdrl Qa>rdr
to current dl in element at C or D is T 2jc/(0
fiQ 2dl flQdl
dB = Magnetic field induction dB at the centre 0 of
4ji r 2jtr
the elementary irng due to current I is
Net magnetic field induction at 0 due to both the
elements carrying current dl is dB = ^2^^^ _\lQ^^Q(!jrdr ^ji^Qaidr
4n r 2r 2r nR^ 2nR^
= 2 {dB) cos 6 = 2 X COSP
2n r Total magnetic field induction at 0 due to entire
rotating disc is
= -^xf-dd)cos0 =^COS0(/0
TC'^r
R
PQgO)dr_PogCO/g _^og0)

w
Ttr \7t I
B =
Total magnetic field induction at 0 due to current i0 2%r} 2nR^ 2kR
through the entire wire is 1

Flo
B oc
(V Q and O) are unchanged)
7C R
cosQdQ =-~ sin — sinO
K- rl 2 Hence, variation of B and R should be a rectangu

ee
0
lar hyperbola as represented in option (a).

Fr
26. Area enclosed by the given current loop is
28. Magnetic field induction at 0 will be
A = Area of 4 semisphere each of radius {a/2) plus
ii() I 3 71A Pq I 3k , A Mf)/o
-*
area of a square of each side a B = ^-x—n + ^ {-n) +^-2n{n)

or
ur
2 4na 2 4nb 2 4ti:c
Ka 2 r-
TC , 2
= -[4K{a/2)^] + a^ = + =
sf +1
As per question, 5 = 0.
2 2 2

—> f jr ^ « A lio
ok
0 =
Yo
Area vector, A = — +1 a k 47cn 2 4Kb 2 4kc
2
Bo

Magnetic moment of the current loop, or 0 = ^^0 Ik 3 3_ +, ~2


4k 2a 2b c
^ A a^k
2^ - - + 1 a^Ik
re

-4
M-I A^I -+1
12 J 12 J or
3 3
(as c = 3 a)
ou

27. Face area of disc -kR^ 2a 2b 3a


ad

Q
Charge per unit area of disc = 3_
Y

kR} or
2a 3a~ 2b
Consider an elementary ring of disc of radius r
nd

On solving, we get,
Re

and thickness dr, as shown in Fig. 4(CF).59. Face


area of elementary ring -2Krdr 9a
Fi

b =
Charge on elementary irng, 13
Q 2Qrdr 29. L-2Kr\ = A 2 7tr2
dq =
kR
yX27ir</r = r}
L ^ L
and r, = -——
* 2tc ^ 2tiN

B,= Pq/^ Pq/


2r.1 2{U2k) L

A^Pq/_ APq/ aVqTc/


5c =
2rj 2{U2kN) L

Be
MOVING CHARGES AND MAGNETISM 4/177

For Difficult Quostlorts 5 = ^iQ /[r--(/?/2)-j _ 2^lo/ /f-


2jtr 3/?^/4 4r
30. Since uniform current is flowing through a
straight wire, the current enclosed by the first _ 2Ho
I Kr^ I r,“ 371/?
jjK[{R^-(R/2f] r- 4r
amperian path is I =
TtR? R'^ [V / = 77t(/?2-/?2/4)|
Using, Ampere circuital law,
= H<l£ r- , i.e., Boc(r-/?2/4r)
j> B-dl'=\i^i 2 4r

When point P is outside the cylinde,r i.e., r> R,

oww
B 2 7C rj = Pfl /
then

Ho' Ho Ir?I Ho^ , i.e., B 1


B =
or
2k 2k
X
R- 2kR-
B2Kr-p^I or B= 27tr
OC —>

Therefore, option (d) is true.


Magnetic field induction at a distance r2 is

e
rFFlo
32. Draw a cylindrical gaussian surface whose axis

re
B ^rr^ ^ (a/2) 2a ^ ^ lies at the axis of hole as shown in Fig. 4(CF).61.

ree
-2
B' /?2

F
4tc t2 a

rF
31. Let I be the current through the long hollow
conducting cylinder, as shown in Fig. 4(CF).60.

fsfoor
When point P is in.side the hollow cylinder (i.e.,
ouur
r < R/2), then
kosk
, —> —>

(.) B-dl = [ifl X current threading the closed path


Yo
oo

B27tr = )iflX0 = 0or B = 0


Y

When point P is inbetween inner and outer part


BB

of conductingcylinders, (i.e., R/2 < r<R)


rre

/7c[r^-(/?/2)2]
B27cr = Po
oYuu

n{R^-(R/2)^]
ad

Let r be the radius of this gaussian surface.


dY

FIGURE 4(CF).6Q
Area of crosssection of this gaussian surface
4
innd

= Kr
Re

R/2
Current threading the gaussian surface
Fi
F

Ixnr^ /r2
/' =
i4 K(r^-r^) (r}~r^)
B Magnetic field induction at point C is

B =
Ho 27' 2 Ir^ liplr
I
I
4k r 4nr(r^-r^) 27C(r2^-rj^)
4—♦
I R/2 33. Here, / = 50 cm = 0-50 m. /V = 100, / = 2-5 A
I
Magnetic field at the centre of the solenoid Is

B =
yj^NJ (47txlO~^)xlOOx2-5
I 0-50
!i
= 6-28 X 10-* T
4/178 T^^uuUc^ '4. Fundamental Physics (XII) LWI
o
As mass of electron (m^) is least so is least,
For Difficult Questions
but rp = So r. < r_ = r
e p a

34. For a long straight wire of radius R, carrying 37. Refer to Fig. 4(CF).62,
current /, the magnetic field at a distance r (< R), Here,/= 1-5 + 1-5 = 3-0 m ;/= 10 0 A,
from the axis of the wire for a point inside the work
wire, according to Ampere’s circuital law is Average Power =
time

B = HJs i.e., B r FIGURE 4(CF).62


2tc
O
For a point outside the wire, at distance r from I
I / = 3 m

oww
the axis of the wire, ” I = 10A z

B = Hi i.e.,
; B oc

27t r r ▼ X

Thus, option (c) is true. 2

e
FFrlo
35. Acceleration of electron while falling in the W= Fdx= Blldx

re
eE 0 0
electric field, a =

ree
F
m
e

3-0xl0‘^e‘^“-"xl0x3i/x

rF
Time taken by electron to fall height h is 0

h = -afl

fsoor
ouur
2
2
= 9xl0"^x \e-^^^dx
skf
2h 2h '2hm^ 0
ooko
or / =
V e Ehttg e E
YYo
a
-0-2x -^●2x2
-I
As charge on proton = charge on electron = e, so = 9x10-^ ^ = 9x10-3 ^
Bo

L -0-2 Jo L -0-2 J
reB

time taken by proton to fall through height h


when field is reversed is
9x10-3 9x10-3
x[l-.^^] = xO-33
uur
ooY

0-2 0-2
/' =
ad

] e E
2-97x10-3 j
dY

m 0-2
^>1 or t'>f
innd
Re

/ 2-97x10-3
P = = 2-97 W
(0-2) X 5x10-3
FFi

It means electron will take smaller time to fall.

mv^ mv
mv^
36. F ~ qvB = or r = Bqr
r qB 38. Bqv = or i^ =
r m

1
Kinetic energy, K = -itw ^ when particle goes straight under the combined
or mv = ^J2^nK
effect of electric and magnetic field, then
J2mK -Tm Bqr
r = — or r oc B qv = Eq or E = Bv= B
qB \ m J

m
a or m =
qB'^r
e p .a ■ £

(1-6 X10-'^) X (aS)2 X (0-5 X10-2)


100
e e 2e e e e = 2 X 10-2^ kg
MOVING CHARGES AND MAGNETISM 4/179

1*67 X X10^
For Difficult Questiona
_ = 01m
q B~ 16x10->9x0104
39. Force on the charged particle in magnetic field is
2kk _ 2nx0-l
F = qv B, acting in the direction as shown in Now T =
Fig. 4(CF).63. It provides the required centripetal
force. Due to it, the charged particle describes a = 2 7C X l(r^ s
circular path AC of radius r with centre at O. Then
41. Refer to Fig. 4(CF).65,
mv^ mv V qB
qvB = or r =s or
r qB r m m
= qvB
r

mv
or = qB
b-a

ww
qBib-a)
or V =

Floo
m

42. Given u and B are ir to each other. Force on

ee
reer
electron due to magnetic field is, F ^-eiux B).

rFF
Since this force acts perpendicular to if,
uur r
therefore, it does not do any work. Due to it, the
ffoor
magnitude of velocity of electron remains
constant. Due to the perpendicular m^netic field
sks
In Fig. 4(CF).63, d=rsin0 acting pn the moving electron, the electron will
YYoo
ooko

Whend«r, sin0»0 = d/r describe a circular path in the magnetic field in


clockwise direction. As a result of it, the condition
eBB

Change in velocity of charged particle fiom A to (d) Le,y v=u and < 0 is satisfied.
C is, Au = V sin 0 = u 0 (forsmall 0), acting along
CE 43. When charged particle goes undefected, then
r

qE = qv B or v~ E/B
ouur

Thus impulse imparted = change in momentum


ad

The two forces acting on moving charged particle


= Ap = mAu = mt;0 = mt» {(Ur) = md (qBIm)
Y

due to electric and magnetic fields oppose each


= qBd
dY

other if V is along Ex B
40. Since the proton is entering the magnetic field at
Re
nnd

some angle other than 90”, its path is helix. EB £5 sin 90’’ l£x£l
Now, t; = —;r = —
FFii

Component velocity of proton along X-axis, Fig. £2 £2


4(CF).64
Vx = V cos 60” = 2 X 10^ X 1/2 = 10^ ms"*. ->
V =
(Ex B)
B^
y FIGURE 4(CF).64
44. Refer to Fig. 4(CF).66,
X - potential difference applied between two large
vertical plates held distance d = 1 cm = 10~^ m
B
V apart.
Electric field intensity in a region between the
■Q two plates, E =
pot.diff. _ X V/m
X
distance 10“^
When proton is released at O, the force on it due
Due to component velocity the radius of the to electric field = q E acting along the direction
helix is described and is given by the relation of field, Le., along OA.
4/180
Pta^iUe^'4. Fundamental Physics fXinrosTWl
46. In this question, the proton moves from rest
For Difficult Questions
towards west. It is due to a force on proton by
virtue of electric field along west.
FIGURE 4(CF).66
Acceleration of proton due to electric field
10"2 m
-►
eE ma
0
= a
0 or £■ = west
m e

q qE When proton is projected towards north with a


0
mg
speed Vq, it moves with an acceleration 3 Oq
towards west, shows that the proton is
B experiencing forces due to electric field along
west and magnetic field acting vertically
downwards. Therefore, the acceleration of proton
due to magnetic field = 3 Oq - czq = 2 iSq.

ww
X
Force on proton due to magnetic field

Weight of proton = mg acting vertically m (2 £3!q) = e Dq 5

Floo
downwards, i.e., along OB. Ima
0
or B = downwards
Since proton moves at 45“ to the vertical just after

ee
ev
0
the release, so tan 45“ = qE/mg

eer
Choice (c) is correct.

rFrF
X
or
q E = mg or q— = rng
d 47. Given, q= \, ^=2i +4j +6k,
rur
mgd _ (l-6xlO-^'^)xlQxlQ-^
or X =
1-6x10-^^
s ffoor F = 4/-20;-+I2A, B=Bi+BUsJ
= 1 X 10-® V
osk
YYoou
45. Refer to Fig. 4(CF).67, let / be the length of a (vx B) = (2i +4j +6k)^(B'i +B j + B^k)
oook

wire under study. In equilibrium state


eBB

T cos0 / j k
T cos 6 = ffig = X / g or / = ...(0 2 4 6
Xg
B B .B0
r
ouur
ad

= i (45o-65) + y (6B-2BQ) + it(2S-4S)


Y

= (4Bo-6B)? + (6S-2Bo) ]-IBk


nd dY
Re

—* »

Given, F = q{vxB)
FFini

(4/-20j'+12ife)

= l[(4So-6S)j-+(6B-2B(j)/-2fijt]
Comparing the coefficients of j , / and A on both
the sides, we have
TsinG = F =
4ti r 4 = 4Bq-6B or 2Bq-3B = 2 ...(O
- 20 = 6 B-2 Bn0 or Bn-3B=10 ...(«)
-iio /X/
X
T cos 9
12 = -2B or B=-6 ...m
2k (2Lsin6) Xg
From («), Bq = 10 + 3 B = 10 + 3 X (- 6) = - 8
or I =2sin0
iiLXg A A A
Hence,
^iiQCose B = -6i-6j-Sk
4/181
MOVING CHARGES AND MAGNETISM

For Difficult Questions

48. Force on the particle in the electric field, F^qE


Work done on the particle by electric field when
particle moves through distance x = q Ex
Gain in KE of particle = - mv^
Using work-energy theorem, we have
1 2 2 _ X
qEx=~ mv^ or V
S3. Refer to Fig. 4(CF).69, Magnetic dipole moment
^ 2 m

of the coil carrying current, M = I % R^- Its


which is an equation of parabola.

ww
direction is perpendicular to the plane upwards.
Thus option (b) is true.
1 27tm
Resolving ^ into two rectangular components
49. Pitch = V cos 60" x T = ux-x we have, B cos 0 acts perpendicular to the plane
2 qB downwards and B sin 6 acts along the plane

Flo
downwards.

e
= (4xl0^)x-I (2x314)x(l-67xlQ-^^)

eree
(l-67xl0-'^)x0-3

FFr
4x3-14
xlO"^ 4x 10”^ m = 4 cm
uurr
0-3

orr
50. Momentum, p = nw or v = phn
Force on moving charged particle in magnetic
field, F = qv B = q (p/m) B = qp Bfm
sfo
kks
Yoo
Vp = P'f^p
oooo

^d = plf»d
eBB

a’ v^ = p/m a

. Pd . _ P . g ■
.‘.Fp-.F,:F^ =
uurr

m 2m 4m
p ^d
m m
a
ad
Yo

1 1
dY

= 1:-: - = 2:1:1
2 2

J_ J . 1 _ 1 . 1 . 1 For rotational equilibrium of sphere about point


innd
Re

Vp-Vd-Va = m 2m 4m P, (mg sin Q) R = M (B sin 9)


Fi

mg
1 1 or mg /? sin 0 = / 7C B sin 6 or B =
= 1:-:- =4:2:1 nlR
2 4
54. The force on a linear conductor carrying current
2 1
mv mt> _ IS F = / / B sin 0
51. Bqv = or r = r « —
r Bq Bq P
where 9 is the smaller angle between / / and B .
Hence, — In Fig. 4(CF).70, for arm PS and B(2. 6 = 0" and
''a
/ 1 180°, and sin 0 = 0, so no force acts on them.
The force on arm QP = I I B sin 90° acting
52. Refer to Fig. 4(CF).68, as the rod is just in
perpendicular to the plane of paper upwards.
equilibrium state, so The force on arm 5B = / / B sin 90° acting
mg sin 30° = B11 cos 30°
perpendicular to the plane of paper downwards.
0-5 X 9-8 1
or 7 =
mg
tan 30 = X —= = 11-32 A Therefore, the top edge QP of loop will be lifted
Bl 0-25x1 Vs up.
4/182
^nad€€^'<i. Fundamental Physics rxinwrsTW
For Difficult Quobtioos. F _Mq2/x/ _4„/^
' 471: d 2nd along BA
FIGURE 4(CF).70 Magnetic force acting per unit length on wire B
/ due to currents in wires B and C is
P Q

along BC
2nd

B /"
\ B ● ●/ 8
Bis
Net magnetic force per unit length on the wire

S R
f-' = = /-I V2 (●●● = /'V
/

w
55. Refer Fig.4(CF).71, the forces acting on the rod 2nd ■Jlnd
are
57. The net force acting on the conductor after it gets
FIGURE 4(CF).71 displaced through a very small distance r is given

Flo
by

e
reee
FIGURE 4<CF).73
B/ B///COS 60*

FFr
A B C
D,
*■811
c ' 'h

for
ur
-●/
Fl
f^2
kkss
mg
60*
Yo
oo

< -

(/) rng acting veitically downwards . d d


B

(//> B! I acting horizontally. The rod will be in


equilibrium if the resolved components of the
re

F=IF|-F2l =
forces parallel to the inclined plane are equal and InUl - r) 2n{d + r)
opposite ie.,
ou
ad

mg sin 60“ = /?/ / cos 60° //q/Zq \{d+r)-{d-r)


YY

2n
or B =
mg
tan 60“ =
lO'- xl0xV3
= IT
II I-73XOI
nd
Re

56. Refer to Fig. 4(CF).72, AB = BC = d As it is given that r « d, so F -


nd-
Fi

Magnetic force acting per unit length of wire B


due to currents in wires A and B is ■> Ml) ^Z()
Now mr(sr = => 0)- = Y
FIGURE 4(CF).72 nd- nmd~

f^2
Q^' 0) =

Y Jimd^
90“
58. Consider a small element of length dL of the wire
F,- of radius R, as shown in Fig. 4(CF).74
Force on the element : BI (dL) = 2 Tsin 0
BI(R2Q) = 2TQ (v 0 is small)

L
or T=BIR =BI (V F=2tcF)
2tc
MOVING CHARGES AND MAGNETISM 4/183

4t
For Difficult Questions
X = / — /2 B SO /^ =
4 Sbi
FIGURE 4(CF).74 xl/2
/
X .
So / = 2

e 62. Let n be the no. of turns formed of circular coil


Tslne ' / Tslne of radius r, in length / of the wire. Then
I
/ = (2 It r) n or r =
2Tsine 2nn

oww
Maximum torque, x max. = nIAB-nI B

/2/B
59. When ,r = 2 m ;
1 / It X (//2 It n)^ B =
= n
4it«

)>2 = 2x2 = 4orjf = 2m

e
X will be maximum if /i is minimum. If will be so

re
FFrlo
length>li5 = 2 + 2 = 4m if n = 1.

This bent conducting wire will be of vector length 63. Here, area of the coil A = it The direction of

rF
ee
/= AB=4(-J) = -4j A is normal to the plane of coil, making an angle
90“ with the field direction. Therefore, the torque

rF
ouru
Force on wire, F = I I xB on the coil is 'i = BIA = BInf^
Since only one edge of circular coil lifts from the

fosor
= 2[(-4;)x(-2A)l = 16f skf
60. Torque on the coil, x = lAB sin 0 = MB sin 0 table, the torque required is =
ooko
where 0 is the angle between M and B . For
Yo
mg
Y
... or/ = 2%rB
equilibrium, torque is zero.
Bo
reeB

In case of stable equilibrium, M is parallel to 64. Mass of each side of square, m = Aap A

Magnetic moment of current loop, M = I ;


B and in unstable equilibrium M is antiparallel
ooY

acting perpendicular to the plane of current loop.


uur

—>

to B . The same is true for fig. (iO and fig. (/V)


ad

In the deflected position of current loop. Fig.


respectively. 4<CF).76, the perpendicular distance of C or t)
dY

61. x = /ABsin90“ = /AB

I Angle between A and B is 90“1 from axis of rotation =—sin 0.


nind
Re

2
I
Area, A = - X base x height
FFi

. I
= -x/x/sin60" =
2 4
4/184
'4. Fundamental Physics (XII)

For Difficult Quostlons


Pot. difference between A and B = IRp = 20
50(400+ G) 400G
Perpendicular distance of weight mg of wire EF -20
from axis of rotation CD-a sin 0
200(400+ G) + 400G''400 +G
Torque on current square loop in magnetic field or
50
X400G = 20
= m sin (90° ~0) = MB cos 0 200(400+ 3G]
= / B cos 0
100 G = 8000 + 60 G
Torque on loop due to gravity or G = 200n
= lmg {all) sin 0 + mg o sin 0 When the voltmeter is connected in parallel to
= 2 mg <3 sin 0 =s 2 A a p g X a sin 0 the resistance 200 fi, Net resistance becomes
= 2A p g sin 0 / 200x200
R = ioon
In deflected position, the above two torques are

ww
P 200+200
equal and opposite, so Total resistance of the circuit = 100 + 400
/ <3^ B cos 0 s: 2 A p g sin 0 = 500n
2Apg
or cot 0 = Current /' = — -—A

Flo
IB
500 ” 10

e
65. Here, mass of ring, m = 100 g = 0-1 kg ; Reading of voltmeter = Potential difference across

eree
radius of irng, B = 0-5 m ; / = 4 A, B = 10 T R^
Magnetic moment of irng, M = / (717?^) = /'/?!P = —X100 = 10V

FFr
10
acting perpendicular to the plane of irng.
uurr 67. Current through galvanometer,
Torque on the irng due to current

orr
T = A/Bsin 90° = InR^B V 2

Moment of inertia of irng, about an axis of rotation


tangent to the irng
sfo /
g
B + G 3000+25
Where k is figure of merit of galvanometer. The
= *x30 ...(0
kks
Yoo
1 current coresponding to 20 unit deflection of
oooo

V = -mR-?
2 galvanometer,
eBB

If a is the initial angular acceleration of the ring,


then initial torque on ring i=-^x20 = -/ = -x — A
30 3« 3^3025^
x' = /'a = imB^xa
uurr

If B' is the resistance to be used in series of


2
galvanometer, then
ad
Yo

-mB^a = /7tB^B
dY

V 2 2 2
2 i = or -X
B' + G 3 3025 B' + 25'
2InB 2x4x7txl0
innd

- 800 n rad/s^ On solving, we get, B' = 4514 n


Re

or a =
m 01
66. Let G be the resistance of voltmeter connected in 68. Refer to Fig. 4(CF).77, Fractional current passing
Fi

through galvanometer,
parallel to 400 resistance. Net resistance
400 G FIGURE 4{CF),77
Rp = 400 + G Is
Now, Resistance 200 Q and Rp are in series. Total
effective resistance
I
400G ^ +■

B^ =200 + /
^400 + G^
V
Now, current in the circuit I = — I
8__ _§ 0-2 1
or G + S - 500 S .(0
I G + 5 100 500
50 50(400+ G)
/ =
GS _ CS G
200 + 400 G ^ 200(400+ G) +400 G Resistance of ammeter =
G+S 5005 500
400 + G
MOVING CHARGES AND MAGNETISM 4/185

For Difficult Question* New voltage sensitivity, ' =


10
69. Here, / « 60 and 1^
oc

H35 1

- ; G == 20 100 "j 3/?


/ 60 6

/ = —y
g _ ^ 20 ^
As, -j G + 5
% decrease in voltage sensitivity
1 s

6 20 + 5 -V n- —y
20 "

ww
s
^ xl00 = xlOO
On solving, we get 5 = 4Q V„.V
70. Here, /g = 5x lO'^ A ; G= 15 Q, y= 10 V, = 55%
10
72. Current for full scale deflection,

Flo
R = — ~G = -15
5x10-3

e
/^ = 30x0005 = 0-150 A

eree
= 2000 - 15 = 1-985 X 10^ n A galvanometer can be converted into a voltmeter
of given range by connecting a resistance R in

FFr
135
71. Given // = /_ + il/ = series, given by
s
uurr
^ 100 ^ 100

orr
15
R = --G = -20 =80n

Initial voltage sensitivity, V = —


^
I

R
sfo 0-15
kks
Yoo
oooo

D9[lMultlple Choice Questions (with One or More than One Correct Ansyeigjl
eBB

73. When £ 0, the velocity of proton changes as ■/m


urr

i.e., roc [for same K and B]


Fg = qE when B 0, velocity of proton may be <?
ad

parallel or antiparallel to that of field as such


Yo

= 0 and velocity of proton does not change. ^16 74


dY

and r OC

Thus, : (a) When £ = 0, S = 0, = 0,


OC
oc 2+
o 2 1
1 ’ 1
velocity doesn’t change
innd
Re

(i?) When E = 0, B ^0, proton moves along B and ^q2+ are equal. So He'*' and will
Fi

and velocity doesn’t change. be deflected equally.


(d) When E^O, B ^0, Fg and F^ may balance H VI/1 _ 1
each other, (a) Since
~V4/1 2’
74. (a) Since K.E. is increasing from t to 4/, electric
field is definitely present but magnetic field may I 1
also be present. Hence incorrect statement. r ^ = — He"*" = - r j.
2 2 0^*
(b) Since K.E. is constant from 5f to either will be deflected most.
only magnetic field is present or both fields are
76. (fl) Increase in K.E. = work done by electric field
present but cancel out each other hence incorrect
statement.
m (2v)^ - “ mv^ ~ qE (2a)
Thus, ‘c* and ‘d’ are correct statements.

mV 1 J2mK 3
r = J or £ = T
75. (c) As r =
qB and ^ = 2 ’ qB 4 ,
4/186
Fundamental Physics (XII)BZSDO

For Difficult Questions


79, Here, E = £y7 and .
(h) Rate of work done by electric field at P Let m. </ be the mass and charge of the positive
- Electric force x Velocity at P point charge which is moving with velocity V
3 mv^ 3 mv^ in the electric and magnetic fields acting along Y-
= qExv- q V axis.
4 qa 4 a

{ci) At Q, force acting on particle due to electric


field is zero (as 2v j is perpendicular to both
A A

E i and B k) and hence rate of work done by


both fields is zero.

77. (a) Since magnetic force (F^) is always

ww
perpendicular to the velocity vector if , i.e.,
-* -*
Fm L V or a j. V or a-v = 0 ●,T

Flo
or (2/'+.ry)-(3/ + 4j')=0,-r = -l-5.

ee
rere
(c) Since B is ± to the plane containing the

rFF
-*

velocity vector, in this case (XY-planc), B is Resolving v into two rectangular components
along Z-directlon. we have, v cos 0 acts along X-axis and v sin 0
uur r
id) Since work done by magnetic force is zero.
So K.E. of particle remains constant. foor acts along K-axis.
Due to component velocity v sin 0, the charged
ks s
particle is accelerated due to electric field with
78. Both will travel in semicircular path as shown in
Yoo
acceleration, = Eq qlm along K-axis.
Fig. 4(CF).78, and they will come out of
ook

The particle does not experience any force due to


magnetic field travelling along parallel paths. magnetic field.
BBo

Time spent by the charged particle to describe Due to component velocity v cos 0, the particle
semicircular path in the magnetic field is
re

is accelerated due to electric field with


acceleration, a^. = fy qlm along K-axis.
ouur

FIGURE 4(CF).78
ad

The particle experiences maximum force due to


X X X X X
magnetic field and describes a circular path with
YY

X X X X X
time period, T = 2 tc mIBq which is independent
of /● and V.
nndd
Re

X X X X X
Due to combined electric and magnetic fields
along K-axis, when 0 = 10", the particle will
FFii

X X X
p—* describe a helical path who.se pitch increases with
4
X X X X X time along the K-axis iue., option (c) is true).
X X X X X
When 0 = 90”, the particle does not experience
any force due to magnetic field. Hence it is
X X X X X accelerated along y-axis due to electric field alone.
V
X X
Thus option (d) is true.
X
-e
80. The magnetic field at a point on .r-axis due to
X X X X X
currents through AB and CD will be maximum,
given by
71W
1 = or i m ^0^
Bq B = B^+B^ = 4n a 4n a a

As mass (m) of electron is different from that of acting along z-axis


proton, hence time for their semicircular paths The resultant magnetic field will be directed along
will be different. the z-axis at all points on the z-axis.
MOVING CHARGES AND MAGNETISM 4/187

85. Check the accuracy of each relation by the method


For Difficult Que»tions of dimensions.
86. In region 11, the magnetic field is uniform and
81. The beam of electrons consists of number of
velocity of the particle in region II is normal to
electrons. In moving beam of electrons two types the magnetic field, so the particle will describe a
of forces play their part. (/) Electrostatic mv

respulsion due to like charges on electrons and circular path of radius r given by, r =
Bq
(n) Magnetic attraction, as the two moving
electrons in the same direction behave as two For particle to enter region III, Fig. 4(CF).80(a)
parallel linear conductors carrying current in the FIGURE 4(CF).8Q
same direction. Initially electrostatic respulsion
II II
is more effective than the magnetic attraction as X X X X X X X X

speed of electrons is small. When speed of


X X X X
electrons becomes high, the magnetic attraction X X X X I II

ww
III
becomes more prominent than electrostatic X X X X

X X^X X III J
repulsion. X X X ,x

82. Magnetic field induction at a point inside the X X X X X X X X

FF loo
4- >
cylindrical conductor is S .r
Magnetic field induction at a point outside the © o

ee
cylindrical conductor is B \!x
oc

ee r
Bql

rFrF
Magnetic field induction at a point on the axis of r> I or
mv
> I or u >
cylindrical conductor is zero. Bq m
rur
For X = r, 5 is maximum. For path length of particle in region II to be
83. Here, G = 100 Q ; = 50 x 10-^ A = 5 x 10“^ A ffoor
maximum, / = r or v =
Bql
ks
For a voltmeter of range 50 V, m
YYoou
50
-100 = 10^- 102 = 9-999x \0^Q.
ookos

R = The time of revolution of charged particle in


5x10"^
BBo

Tir kI Tirn
For a voltmeter of range 10 V, region II is t= which is
re

10
V V Bq
R = -100 = 199,900 n
5x10"^ independent of v. Therefore, all those particles
ouur
ad

» 200 in series moving with any velocity, which return to region


I, spend the same time in region II.
Y

For an ammeter of range 5 inA,


87. Here, as we know in cyclotron centripetal force
5xl0~^ xlOO
on charge is equal to magnetic force =F
dndY

5 = 1 Q in parallel m
Re

5x10"^ - 5x10-5
For an ammeter of range 10 inA rnv^ V qB
FFini

So = qvB ^ —=
r m
5x10-5x100
r

S = = 0-5 in parallel
10xl0“5 -5x10-5 V
As, i; = rco. So — = to
84. When a charged particle is subjected to a perpen r

dicular magnetic field, it will describe a circular


path of radius, r = mv/B q i.e. r \l{qlm). Thus
OC

qB
option (c) is true. As time period of revolution, (0 = {i.e.. Independent of u, r]
m
2 Tim
T = is independent of v and r, so the 88. While finding the magnetic force on a current
Bq
carrying wire in magnetic field, the vector length
panicle will return to its initial position after one of the wire is taken. Here, the vector length of the
revolution, hence the two particles of different
wire is
charges will return to their initial positions after
one revolution and their paths vrill touch each I = iL+R + R + L)7 = 2(L + RU
other there.
4/188
it Fundamental Physics fXinPTSTW

For Difficult Questions ^ ^ A


When B is along;c-axis i.e., B = Bi, then
Force on conductor of length I, carrying current /
?=/[2(L+fl)/x5?] = 0 (v/xi' = 0)
when placed in a magnetic field b= (Bk) is
“* ^ A

F = I[TxB]=I [2(L + R)'ixBk] When B is along >>-0x18 i.e., B = Bj, then

= 21B{L + R) (-j) F = I [2(L + R)7xBj] =2I{L + R)Bk


Foc(L + /?) So, F oe (L + i?)

oww
mi: Multiple
7^^ Choice Questions (Based on the given Pas8age/Comprehen8 lon)|

89. As I - ^e;

e
NBA

FFrlo
re
, NBAi NBAL0 3NBAi 0

ree
F
so k = =
e 7C/3 7t

rF
= 0 _ 71/3
K
90. Voltage sensitivity,

fsoor
iR LR 3/„F
ouur
0 skf 0

91. Corresponding to charge Q, if the current i flows


for time dt, then
ooko
YYo
Torque, T = NBA i or xdt = NBA idl = NBAQ
Bo

It CO is the angular speed of the coil produced when


If net magnetic field at O is zero, then
reB

charge Q passes through the coil, then, change in


angular momentum
= ^2 or Pp/g^ _ l^/g
= I(H = xdt=NBAQ
uur
ooY

2(g^+/j^)^^^ K(a^+h^)
ad

NBAQ
dY

or CO = a 1
I or

2yla^ + h^ n

92. Let O be a point at a height h above the loop.


innd
Re

Refer to Fig. 4(CF).81, the magnetic field 2


induction at O due to current loop
g
FFi

or
[v 7c2= 10]
4(a^+h^) K^~10
27l/g^ Pp/g^
Bi -1^0
- or 10a^ = 4g2+4/i^ or /i^ = 3 g^/2
4tC (g2 + *2)3/2 2(a'^ + h^f^ or h = h2a
Magnetic field induction at 0 due to currents in 93. When d » a, then magnetic field at 0 due to
the wires 1 and 2. currents in wires I and 2 will be

B2=2x
1^0 21
X
d S = 2xii2-li = HoI
4k d nd

Torque on the loop = BIA sin 30°


Po /</ _ Pq/^ _lto/ ●? 1
(v d=a) X/X(Kg‘^)X- _
K(d^ + /j^) n{a^ + h^) nd 2 ” 2d
MOVING CHARGES AND MAGNETISM 4/189

For Difficult Questions

09 Matching Type Questions

94. Ampere swimming rule tells the direction of deflection of the magnetic needle due to current in a straight
conductor, when magnetic needle is held below the conductor.
Fleming’s left hand rule tells the direction of force on the current carrying conductor placed perpendicular
to a uniform magnetic field.
Fleming’s Right hand rule tells the direction of induced current,

oww
Right hand thumb rule tells the direction of magnetic field lines due to current through circular coil.

m Matrix-Match Type Questions

ee
95. A positively charged particle is projected with magnetic field is acting at some angle with the
some velocity in a region with a uniform electric direction of motion of charged particle.

FFrlo
r
field, it will describe a straight line if field is acting A positively charged particle is projected with a

rF
parallel to the motion of charged particle and velocity lying in a plane normal to a region

ee
parabolic path if field is acting at some angle or containing parallel uniform electric and magnetic
perpendicular to the motion of charged particle. fields, the particle will describe helical path of
ouru
rF
increasing radius.
A positively charged particle is projected with
some velocity in a uniform magnetic field, it will A positively charged particle is projected in a

ffosor region containing perpendicular uniform electric


describe straight line if field is parallel to the
os k
motion of charged particle since no force will be and magnetic fields will describe straight line if
force due to electric field is equal and opposite to
acting due to magnetic field. It will describe
ook
that due to magnetic field. If the force on charged
Yo
circular path if magnetic field is acting
Y
particle due to two fields is not same, then the
perpendicular to the direction of motion of
Bo

particle will describe a helical path.


reeB

charged particle. It will describe helix if the


oouY

VI. Integer Type Questions


ur
ad

96. Magnetic field at P due to currents in two wires Magnetic field induction at the centre of circular
dY

coil due to current I is


will be acting perpendicular to the plane of wires,
nidn

upwards and is given by !^0 2Ttn/ _ Itp 2jcx1x/ _


Re

B =
4n r 4ti {//2tu) I
B -^^0 2/ ^ ^0 21 ^2\LqI
FFi

^ 47C(r/2) 4it(r/2) Kr When wire is taken in the form of double loop.


I
Magnetic field at Q due to current in A is then / = 2 X 2 7C r, or .r=-~ and n = 2
* ' 4k
perpendicular to the plane of wire upwards and
due to current in B is perpendicular to the plane .*. Magnetic field induction at the centre of the
of wire downwardsand is given by circular coil,

B2 47i2r
^ n„2/
4jir
n„/
47ir B,=
Jip iKXnxI _ Mp 2kx2x/
4k 'i “4k (//4k)
Bp (2p^//Kr) = 8 nl

Bq (Pp//4Kr) = 4xpp I
97. When wire is taken in the form of one turn circular
B

coil, then length, l = 2nrorr = -J—,n=\


2k
B
4/190
Fundamental Physics (XiniTTSTWl

current through the cylindrical cavity of radius


For Difficult Questions
all is

98. Refer Fig. 4(CF).82, the magnetic Held induction /


at the centroid O due to current / through one side /' =
KO
rXjrU/2)2=1
^ 4
HC of the triangle will be
Due to current in cylindrical cavity, the magnetic
B, = ——(sin0, +sin0T) field induction at P is
4jc r ‘ ^

It will be acting perpendicular to the plane of _Mu2 (//4)


triangle upwards. 471 {a + afl)
Total magnetic field induction at O due to current
through all the three sides of the triangle will be 2/
= iill.
3|.l, / 4tc 4x(3«/2)

w
// = 3/?. = —^-[sin0,
4tc /● '
+.sin0,^ I
_ Fq2 ina^J) _

Flo
FIGURE 4(CF).82 4ti 6</ 12

reeee
Net magnetic field induction at P is

FFr
// = « -B
I
2 2 12

for
ur
kkss
2 6 12
Yo
oo

As per question, =
eB

12 12

N =5
r

100. Here, = 0-006 KR = 4990 LI, V = 30 V. Let G


Here./= I A. 0, =6O" = 02
ou
ad

be the resi.stance of galvanometer, then


BD all
YY

anil r = OO -
tan 60® ^ V=l^(G + R)
ndd

c=^ 30
Re

___ 4-5x10-2
or R = -4990= 10
a
m
h 0-006
2■^/3 ~ 2>/3
Fi

When galvanometer works ixa an ammeter, then


I
// = 3xI0“2x shunt resistance required is
(4-5xI0-2/2>/3)
I..G 0-006x10
X (sin 60® + sin 60®| S =
X
= —

On solving, B~4x 10~® T l~l^ (F.S--006) 149-4


99. Due to current in complete cylinder, the magnetic As per question.
field induction at P is
2n 6

249 149-4
4tc a 4na

6x249
or n = = 5
2 2x149-4
MOVING CHARGES AND MAGNETISM 4/191

For Diflicuit Qiiubtioiiu

VII. Assertion-Reason Type Questions

FOR MEDICAL STUDENTS


106. E
2
imr or v = ■yllJJ m

iKtll . Ill
101. Time period, T = i.e. T OC

V = wrf'“/r
Be,
Bolh ihe Assertion and Reason are correct and

I
or
mv m IE ■^2mE
Reason is the correct explanation of Assertion.
Bq Bq m
Bq
102. When two long parallel wires, arc connected to u

w
battery in series. They carry currents in oppr>site ■Jm
directions, hence they repel each other. i.e.. r OC

Flo
Both Assertion and Reason are false.

e
103. In a non uniform magnetic Held, both a torque
!isi- II x- = I
r m -)

ree
P P
and a net force act on the dipole. X

FFr
m
If magnetic field were uniform, net force on dipole a

would he xero.
i.e., radius of both the particles should he
urr
Both Assertion and Reason are false.

for
the same. Hence statement-1 is wrong and
statement-2 is correct
FOR ENGINEERING STUDENTS
kkss
107. In a moving coil galvanometer.
104. Momentum, /> = mv or v = /)///;. In a magnetic
Yo
ooo

k nBA
field, qv B = mv-h- or r = mvlq B = filq B / = e or e = - /
iiHA k
I.e. \!q
B

~ U‘i^U\ - = I● For the given value of current /, 0 increases il B


re

Thus statement-1 is false and slalenieiit-2 is increases. The use of in)n core increases the mag
true. netic field, so the deflection 0 increases for Ihe
ou
ad

same value of current 1 making the galvanometer


105. In order to increase the range of ammeter. Its
YY

resistance should be decreased, i.e., additional more sensitive. Hence, statement-1 is correct.

shunt is connected in parallel. Thus statement-1 We know that soft iron can be easily magneti.scd
ndd

is false. Stalement-2 is true. or demagnetised, hence statement-2 is wnmg.


Re

VIII.
Fi

Multiple Choice Questions (Based on Experimental Skills)


I G / . X 3663
108. Here,G= KKK) fi,/^, = 01 niA = hH A; Now, S = = Ulil

/=I0A

I G
Itr'^ x !(MK) no. G =
RS _50(X)x20 = 2on
.S’ =
-4 R-S~
10-10

E
- 0*01 A in parallel. 111. / -Ilk =
R +G
100. If / is the total current, then
/„ = //34 E
or k
t>r I - 34 /i{/? + G)
4/192 'Ptadeefr^'^ Fundamental Physics (XII) VOL.I

For Difficult Questions

1-5 Dividing (iO by (i), we have


= 0-5xl0“3A/div /? + 980 = 50/?
20(100 + 50)
or /; = 980/49 = 20 a
= 500 X 10^ A/div = 500 ji A/div 113. Resistance of anuneter,
112. Let i? be the resistance of voltmeter and n be the
number of divisions in the voltmeter. When GS _ 60x002
G' =
current flows through voltmeter, the voltage G + 5 ""60 + 0 02

oww
recorded by each division of voltmeter is
E
Current / =
/ _^ = V (0 R + G'
n

30

e
When resistance 980 Cl is connected in series, then = 10A
'60x002'

re
3-0 +
j 8 (/?+980)

FFrllo
= 50V .(«●) ,60 + 002,
n

rF
ee
ouru
rF
fosor
os kf
ook
YYo
Bo
reeB
oouY
ur
ad
Yd
nidn
Re
FFi
w
MAGNETISM AND MATTER

Flo
e
5.1. INTRODUCTION

reee
In the last chapter, we studied magnetic effects of current. We learnt that electricity and magnetism are

FFr
not independent phenomena. Infact, moving electric charges or electric currents produce magnetic fields.
In this chapter, we trace the origin of the science of magnetism, and its subsequent growth. A Greek
Philosopher, Thales of Miletus had observed as long back as 600 B.C, that a naturally occuring ore of iron

for
ur
attracted small pieces of iron towards it. This ore was found in the district of Magnesia in Asia Minor in
Greece. Hence the ore was named magnetite. The phenomenon of attraction of small bits of iron, steel,
kss
cobalt, nickel, etc. towards the ore was called magnetism. The iron ore showing this effect was called a
Yo
natural magnet. The structural formula of this natural ore was
oo

The Chinese discovered that a piece of magnetite, when suspended freely, always points out roughly in
B

the north south direction. Thus a natural magnet has attractive and directive properties. A magnetic compass
based on directive property of magnets was used by navigators to find their way in steering the ships.
re

That is why magnetite was also called ‘lode stone’ in the sense of leading stone.
ou

The natural magnets have often irregular shape and they are weak. It is found that a piece of iron or steel
ad

can acquire magnetic properties, on rubbing with a magnet. Such magnets made out of iron and steel are
YY

called artificial magnets. Artificial magnets can have desired shape and desired strength. A bar magnet, a
horse shoe magnet, magnetic needle, compass needle etc. all are artificial magnets.
nd
Re

5.2. THE BAR MAGNET


Fi

It is the most commonly used form of an artificial magnet.


When we hold a sheet of glass over a short bar magent and sprinkle
some iron filings on the sheet, the iron filings rearrange themselves as
shown in Fig. 5.1. The pattern suggests that attraction is maximum at the
two ends of the bar magnet. These ends are called poles of the magnet.

5.3. SOME BASICS OF MAGNETISM

Some of the commonly known ideas of magnetism are :


1. The earth behaves as a magnet. This is as if a huge bar magnet is held at the centre of earth with its
north pole pointing approximately towards geographic south. The south pole of the imaginary bar magnet
would point approximately towards geographic south.
5/1
5/2 A Fundamental Physics fXIH PTSTMi

2. Every magnet attracts small pieces of magnetic substances like iron, cobalt, nickel and steel toward
it. The attraction is maximum at the two ends of themagnet, as shown in Fig. 5.1. These ends are called poles
of the magnet.
3. When a magnet is suspended freely with the help ofan unspun silken thread, it comes to rest along
the north south direction. If it is turned slightly from this direction and left, it again returns to this direction.
The pole of the magnet which points towards geographic north is called north pole of the magnet (iV) and the
pole which points towards geographic south is called south pole (5), Fig. 5.2(a).
Note that poles exist always in pairs; two poles of a magnet are always unlike but of equal strength.
Further, poles N and S are situated a little inwards from the geometrical ends A and B of the magnet.
The magnetic length {NS) of magnet is roughly 6/7 of its geometric length {AB). We represent NS by 21*
{and not I).
The straight line XX' passing through north and south FIGURE 5.2

poles of a magnet is called axial line of the magnet. A line YY'

ww
passing through centre of the magnet in a direction
perpendicular to the length of the magnet is called equatorial
line of the magnet.
Geog. s N Geog.

FF loo
The straight line joining north and south poles of a freely South r~ Til. _-North ©
suspended magnet represents magnetic N-S direction. A B I A

ree
H- 2/ -H
vertical plane PQ passing through N-S line of a freely
Y
suspended magnet is called magnetic meridian. These are Pr-

rree F
shown in Fig. 5.2{b).
Magnetic
4. Like poles repel each other and unlike poles attract ■S N
Meridian
ur
each other.

To show this, we suspend a bar magnet with the help of a


thread. When we bring N pole of another magnet near the N
fofroF X’- -X O
ks
pole of suspended magnet, we observe repulsion. Fig. 5.3.
soo

Similarly, south pole of one magnet repels south pole of the


YYouor
I
—'Q
Y’
other, Fig. 5.4. However, when S pole of one is brought near N
BBook

pole of suspended magnet, there is attraction, Fig. 5.5.


r ee

FIGURE 5.4
FIGURE 5.5
oouru
ad
Y
nY d
Re

REPULSION
FFindi

5. The force ofattraction or repulsion F between two magnetic poles of strengths m | and mj separated
by a distance r in space is directly proportional to the product ofpole strengths and inversely proportional
to the square of the distance between their centres, i.e.,
num., m^nij
F oc — or F = K , where K is magnetic force constant.
r2 r2

*Conventionally, this is done for simplification of calculations, as distance of each pole from the centre
would be /.
MAGNETISM AND MATTER 5/3

^^0 == 10-^ WbA-' m -1


In SI units, K =
4n
where )Jq is absolute magnetic permeability of free space (air/vacuum).

^ _ Pq ...(1)
4n r-

This is called Coulomb’s law of magnetic force*. However, in cgs system, the value of K= 1.
SI Unit of magnetic pole strength
IX 1
Suppose m] = m2 = 1 Am, r = I m. Then from (1), F = 10“^ x ^5- = 10~^ N.
Thus, strength of a magnetic pole is said to be one ampere-metre, if it repels an equal and
similar pole, when placed in vacuum (or air) at a distance of one metre from it, with a force of

ww
10-’ N.

6. The magnetic poles always exist in pairs. The poles of a magnet FIGURE 5.6

Floo
can never be separated Le. magnetic monopoles do not exist. In an
attempt to separate the magnetic poles, if we break a magnet, we find

ee
new poles formed at the broken ends. If the two pieces are broken again,
we find the broken ends contain new poles. Fig. 5.6. Thus each piece,

reer
rFF
howsoever small, is a complete magnet in itself. Even if a magnet is
broken into molecules, each molecule shall be a complete magnet.Thus,
uur r
we cannot isolate the north or south pole of a magnet. Unlike electric charges, isolated magnetic north and
south poles known as magnetic monopoles do not exist. ffoor
7. Inductive Property. When a piece of a magnetic material like soft iron, cobalt, nickel etc. is placed
sks
near a bar magnet, it acquires magnetism. The magnetism so acquired is called induced magnetism and this
YYoo
ooko

property of magnetism is called inductive property.


8. Repulsion is the sure test of magnetism. Suppose we have two identical pieces, one is iron and other
eBB

is a magnet. To distinguish which is which, we take another bar magnet. This bar magnet would attract the
iron piece and also one end of this bar magnet would attract the opposite pole of the given magnet. However,
r

one end of this bar magnet would repel only like pole of the given magnet.
ouur
ad

Hence when repulsion occurs between the testing bar magnet and one of the two given pieces, that piece
must be a magnet. That is why we say that repulsion is sure test of magnetism.
YY

9. We can make artificial magnets out of iron and its alloys.


nndd
Re

● sVmpi5! lenrH Two unlike magnetic poles of strength 10 A - m each are held in air at a
distance of 0*5 m from eachother.What is the magneticforce of attraction between them ?
FFii

Sol. Here, m^=m2 = 10 A~m, r = 0-5 m, F = ?


According to Coulomb’s Law of magnetic force,
10x10
F = ^^ = 10-7x = 4 X 10-^ N
4 71 r2 (0-5)2
5.4. THE MAGNETIC FIELD LINES

We know that Magnetic Field is the space around a magnet or the space around a conductor carrying
current, in which its magnetic effect can be felt.
In order to visualise a magnetic field graphically, Faraday introduced the concept of the magnetic field
lines. These lines give us a visual and intuitive realisation of the magnetic field.

* This correspondsto Coulomb’slaw in electrostatics.


5/4 “Pn<zeUe^ ^ Fundamental Physics (XII) PZilBl
According to Faraday,
Magnetic ifeld line is an imaginary curve, the tangent to which at any point gives us the direction
of magnetic field B at that point.
If we imagine a number of small compass needles around a magnet, each compass needle experiences a
torque due to the field of the magnet. The torque acting on a compass needle aligns it in the direction of the
magnetic field.

w
The path along which the compass needles are aligned is known as
magnetic field line.
In Fig.5.7, XY represents a magnetic field line. At any point P on the line, we

e
have drawn a tangent PA to the curve XY. This tangent PA represents the direction of

re
—♦

row
magnetic field B at .P
In a region, magnetic field may be uniform or non-uniform.

eeF
Magnetic field in a region is said to be uniform if

ullo
FF
the field has same strength and same direction at all the FIGURE 5.8
points in the region. Uniform magnetic field in the X X X X
plane of paper is represented by a set of equidistant

srr
X X X X
parallel field lines as shown in Fig. 5. 8(a).

roF
X X X X

Fig. 5.S(b) represents a uniform magnetic field

k
X X X X

perpendicular to the plane of the paper and inwards. X X X X


uor
Fig. 5.8(c) represents a uniform magnetic field
perpendicular to the plane of paper and outwards. '
o ofof o o
Y
kos
Magnetic field in a region is said to be non-uniform if it has FIGURE 5.9
Yo
eerBB

different strengths at different points in the region. A non- uniform


oo

magnetic field is represented by unequally spaced field lines, which


rY

may be parallel or converging or diverging as shown in Fig. 5.9 -► >

A strong magnetic field is represented by closely spaced


u

field lines, and a weak magnetic field is represented by widely


spaced field lines.
ou
d
o
ad

In Fig. 5.10(a), we have shown magnetic field lines due to a bar magnet NS. Fig. 5.10(/?) shows magnetic
nY

field lines due to a finite solenoid carrying current. For the sake of comparison, we have shown electric field
lines due to an electric dipole in Fig. 5.10(c).
nid
Re
F

FIGURE 5.10
Fi

V \ t
/ ✓

''4k''
V \ / ✓

if
/'f '
✓ N ✓ N / ! I ' \ '
/ /● ●V / / / I
\ I » '
/ t \ \ / / \
\ III I I I )

V
I
\
/
V
-
\
/ /
I
/
I i
\
r
\ /
1
/
j + til
s s / s

y
" ‘ !.v/
/
/ / /; \ \ N

✓ ^ / 4. ' s / \
✓ / N S / /
1 \ s
I
\

Magnetic field lines due to Magnetic field lines due to Electric filed lines due to an
a bar magnet a solenoid carrying current electric dipole
O o o
MAGNETISM AND MATTER 5/5

We find that magnetic field lines of a bar magnet and a current carrying solenoid resemble very closely.
However, the two are different from electric field lines due to an electric dipole shown in Fig. 5.10(c).

5.5. PROPERTIES OF MAGNETIC FIELD LINES


1. The magnetic field lines of a magnet (or of a solenoid carrying current) form closed continuous
loops. We imagine magnetic field lines to be extending through the body of the magnet/solenoid from south
pole to north pole, as shown in Figs. 5.10(a) and (b).
This is unlike electric field lines, which begin from a positive charge
DO YOU KNOW ?
andendonthenegativechargeorescapetoinfinity,asshowninFig.5.10(c) . Magnetic field inside a long
Note that electric field lines are discontinuous only in the sense that solenoid carrying current is a
uniform magnetic field. On the
no such lines exist inside a charged body. However, from a positively
contrary, magnetic field of a bar
charged body to a negatively charged body, there is no discontinuity in magnet is a non-uniform
the electric field lines. That is why at very far off points, field lines due tomagnetic field.

ww
an electric dipole and a magnetic dipole appear identical.
2. Outside the body of the magnet, the direction of magnetic field lines is from north pole to south pole.
3. At any given point, tangent to the magnetic field line represents the direction of net magnetic field

Flo
(5) at that point.

e
4. The magnitude of magnetic field at any point is represented by the number of magnetic field lines

reree
passing normally through unit area around that point. Therefore, crowded lines represent a strong magnetic

r FF
field and lines which are not so crowded represent a weak magnetic field.
5. No two magnetic field lines can intersect eachothe.r This is
uurr
because as shown in Fig. 5.11, at the point of intersection, say P, we can
foor
draw two tangents PA and PC to the two magnetic field lines. This would
mean two directions of magnetic field at the same point P. This is not possible.
kss
6. Magnetic field lines have a tendency to contract longitudinally
Yoo
ooook

indicating attracton between unUke magnetic poles. The lines also have a
tendency to dilate laterally, indicating repulsion between like magnetic poles.
eBB

Note that (i) In a particular region, where there is no magnetic field, there would be no magnetic field lines.
(ii) Though magnetic field lines are imaginary, they can be mapped using a small compass needle. This
process can be compared to photographing a virtual object.
rr
ouu

(Hi) Magnetic field lines exist inside every magnetised material.


ad

Note. Magnetic field lines due to a bar magnet can be plotted on a sheet of paper using a small magnetic
Y

compass. This will be only a two dimensional plot. In actual practice, field lines exist all around the magnet
dY

in three dimensions.
Re
innd

Retain in Memory
Fi

Basic Difference between electricity and magnetism


1. The simplest source of electricity is a point charge. It can be positive or negative. The simplest
source of magnetism is a magnetic dipole as magnetic monopoles are not known to exist.
2. Electric charge is scalar. It experiences a force in an electric field. The magnetic dipole has a
dipole moment, which is a vector. It experiences a torque in a magnetic field.
3. The direction of magnetic dipole moment is from south pole to north pole. This is analogous to
the direction of electric dipole moment of an electric dipole from negative charge to positive
charge.

5.6. MAGNETIC DIPOLE

A magnetic dipole consists of two unlike poles of equal strength and separated by a small distance.
For example, a bar magnet, a compass needle etc. are magnetic dipoles. We shall show that a current
loop behaves as a magnetic dipole. An atom of a magnetic material behaves as a dipole due to electrons
revolving around the nucleus.
5/6
Fundamental Physics (XlDPTSTWri
The two poles of a magnetic dipole (or a magnet), called nortli pole and south pole are always of equal
strength, and of opposite nature. Further such two magnetic poles exist always in pairs and cannot be separated
from each other.

The distance between the two poles of a bar magnet Is called the magnetic length of the magnet. It ISi: i
a

vector directed from 5-pole of magnet to its A/-pole, and is represented by 2 / .


Magnetic dipole moment is the product of strength of either pole (m) and the magnetic length
(2 1 ) of the magnet.

It is represented by M .
FIGURE 5.12
Magnetic dipole moment = strength of either pole x magnetic length
m m
M =m{2l) ...(2) ●S N*

w
Magnetic dipole moment is a vector quantity directed from South to North H-
2/
-M

pole of the magnet, as shown in Fig. 5.12.

Flo
The SI unit of M is joule/tesla or ampere metre^. .*. SI unit of pole strength is Am.
The dimensional formula of magnetic moment is [M^ LrT /], and the dimensional formula of pole

reeee
strength is [Af ^ L /].

FFr
Retain in Memory
When a magnet is cut along its length (longitudinally) into two equal parts, the pole strength of each
for
ur
half becomes half. Also, dipole moment of each half becomes half. However, when the magnet IS i
cut perpendicular to its length (/.e., transversally), the pole strength of each half remains the same.
kkss
But dipole moment of each half becomes half.
Yo
oo

5.7. MAGNETIC FIELD STRENGTH AT A POINT DUE TO BAR MAGNET


eB

The strength of magnetic field at any point is defined as the force experienced by a hypothetical
unit north pole placed at that point. It is a vector quantity. The direction of magnetic field (b)
r

is the direction along which hypothetical unit north pole would tend to move iffree to do so.
ou
ad
YY

Note that we have used the word hypothetical unit north pole in the above definition. This is because an
isolated magnetic pole does not exist. The simplest magnetic structure that exists is a magnetic dipole.
(a) When point lies on axial line of bar magnet
ndd
Re

Let 21 be the magnetic length of a bar magnet with pole strength m and centre O. M is the magnetic
Fi

dipole moment of the magnet, where, M = m x 2 /; OP = fr, is the distance of the point Pon the axial line from
the centre of the magnet, Fig. 5.13. Magnetic field strength at P due to N i>oIe of magnet is
, along NP produced. FIGURE 5.13
4k nP- 4n(d-l)^ m O m

Magnetic field strength at P due to 5 pole of magnet,


B' = ^0 ^ m P
, along PS SW 2/ WN B' B
4k SP^ 4K(d + l)^ d ■I

Magnetic field strength at P due to the bar magnet


P, =B-B' =
in m
.^0 m
I

4jl(P-/)2 4K(d + l)- 4k [id-l)~ (d+l)~^


B. =
m f id + l)^-id~l)^ \i^^m.4ld _\if^(mx2l)2d
I
4tc (^2-/2)2 4K(d~-r~)- ~ 4Kid-~l-)-
MAGNETISM AND MATTER 5/7

B
^0 2Md
I “ ...(3)
471 -/2)2
B
When the magnet is short, !^<<d^ 1 “
4tc 4tu
...(4)

The direction of B^ is along NP produced.


(b) When point lies on equatorial line of bar magnet
In Fig. 5.14. the point P is shown on equatorial line of the same bar magnet, where OP = d. Magnetic
field strength at P due to N pole of magnet,
B =
1^0 wxl _ Pq m
, along NP produced.
4k NP^ 47t(j2+/2)
It is represented by PA.

ww
Magnetic field strength at P due to S pole of magnet, B^ = Pq /n X1 _ Pq
m
along PS.
47t SP'^ ~4tc(^/2+/2)’
It is represented by PC.

Floo
As B = B' in magnitude, their components B sin 0 along OP produced and
B' sin 0 along PO will cancel out. However, components along PX parallel to NS

ee
will add up. Therefore,

reer
rFF
Magnetic field strength at P due to the bar magnet,
82 = B cos Q+ S' cos Q = 2 B' cos 0, along PX
uur r
«2 = '2—
^^0
^
m
^X“r=
47t(f/2+/2) ^ + /2
/
ffoor mx2/

4tc (J2+/2)3/2
sks
Yoo
M
^2 = ...(5)
oko

471 (J2+/2).V2
BBo

.^0 M
If the magnet is short, -P<<d^ «2 = .46)
re

47C(i/2j3/2 4k d^
ouur

The direction of B2 is along PX, a line parallel to NS, as shown in Fig. 5.14.
ad

B
YY

I
Dividing (4) by (6), we get = 2 or = 2 B2
nndd
Re

Hence, magnetic field due to a short bar magnet at any point on axial line of magnet is twice the
FFii

magnetic field at a point at the same distance on the equatorial line of the magnet.

Retain in Memory
(/) When point lies on a.xUil line of magnetic dipole, it is known as end-on position or tangent A position.
(/7) When point lies on equatorial line of magnetic dipole, it is known as broadside on position or
tangent B position.
(Hi) At any point on axial line of a dipole, magnetic field strength B^ is along M .
—>

(iv) At any point on equatorial line of a dipole, magnetic field strength B2 is in a direction opposite to
-♦

M .

‘Sample Problem Q A magnet 2 cm long has a pole .strength of 60 Am. Find the magnitude of
magnetic field strength B at a point on its axis at a distance of 20 cm from it. What would be the value
of A, if the point were to lie at the .same distance on equatorial line of magnet ?
5/8
‘Pxadccf:- Fundamental Physics (XII) P7awi
Sol. Here, 2 / = 2 cm = 0-02 m ; m = 60 Am, B = ?, d = 20 cm = 0-2 m
On axial line, as / < d.

B- ^ 2M _ 2xmx2/ _10~'^x2x60x0-02 = 3-0 x 10-^ T


4n 4tu (0-2)3

At the same distance, on equatorial line, ^2 =“><3-0xl0 ^ T = 1-5 X 10-5 T

Calculate the magnetic field due to a bar magnet 2 cm long and having
pole strength of 100 A-m at a point 10 cm from each pole.
Sol. Here, B = ?2/ = 2cm = 2x 10"^ m

1
m = 100 Am, ((f + = 10 cm = — m

w
On the equatorial line of bar magnet.

B =
^^0 M
^^0 (mx2/) _ 10-^x100x2x10-2
= 2 X 10-^ T

Flo
4tC ((^2 +;2j3/2 ^^2 ^;2^3/2 (1/10)3

reee
5.8. CURRENT LOOP AS A MAGNETIC DIPOLE
Consider a plane loop of wire carrying current. Fig.

FFr
5.15. Looking at the upper face, current is anticlockwise.
Therefore, it has a north polarity. Looking at the lower
uurr
face of the loop, current is clockwise. Therefore, it has a
south polarity. The current carrying loop thus behaves as a for
kkss
system of two equal and opposite magnetic poles and hence
the current carrying loop behaves as a magnetic dipole.
Yo
oooo

The magnetic dipole moment of the current loop (M)


eB

is directly proportional to (0 strength of current (/) through


the loop and (//) area (A) enclosed by the loop.
i.e., A/ e« / and M A
r
ou
ad

M = KIA ...(7)
YY

where AT is a constant of proportionality.


If we define unit magnetic dipole moment as that of a small one turn loop of unit area carrying unit
nndd
Re

current, then from (7), l=Kxlxl or K- 1

from (7), M-IA


Fi

For such turns. M=^NIA ...(8)

The SI unit of M is ampere metre^.


One ampere metre^ is the magnetic moment of one turn loop of area one square metre carrying
a current of one ampere.

In vector form, we can rewrite Eqn. (8) as M = NIAn ...(9)


A .
where n is unit vector perpendicular to the plane of the loop in a direction given by right handed screw
rule.
The factor Nl is called ampere turns of the circular current loop. Thus magnetic dipole moment of
current loop = ampere turns x area of the loop.
Note that eqn. (9) is valid for planar current loop of any shape and size.
MAGNETISM AND MATTER 5/9

fsampielRl^giemli A wire of length L metre carrying current I ampere is bent in the form of
a circle. Find its magnetic moment. (Uttarakhand Board 2012)

Sol. If r is radius of the circle, then lnr~L or r = -^


27C
\2
( L V I l}
M-lA = Inr^ = In — =
271 4ti

5.9. BAR MAGNET AS AN EQUIVALENT SOLENOID


We know that a current loop acts as a magnetic dipole. According to Ampere’s hypothesis, all magnetic
phenomena can be explained in terms of circulating currents. FIGURE 5.16
In Figs. 5.10(a) and {b), magnetic field lines for a bar magnet and a S N
current carrying solenoid resemble very closely. Therefore, a bar magnet

w
can be thought of as a large number of circulating currents in analogy S N I S N

with a solenoid. Cutting a bar magnet is like cutting a solenoid. We get two N
smaller solenoids with weaker magnetic properties. The magnetic field lines

Flo
remain continuous, emerging from one face of one solenoid and entering
into other face of other solenoid, Fig. 5.16. If we were to move a small snyfMninMHTiN

ee
N ! S
compass needle in the neighbourhood of a bar magnet and a current carrying

Fr
solenoid, we would find that the deflections of the needle are similar in both
cases.

To demonstrate the similarity of a current carrying solenoid to a bar magnet, let us calculate axial
for
ur
field of a finite solenoid carrying current. FIGURE 5.17

In Fig. 5.17, suppose a = radius of solenoid,


ks
dx
2 / = length of solenoid with centre O

Ri
A
Yo
n = number of turns per unit length of solenoid,
oo

i = strength of current passed through the solenoid -p


eB

\7
i 0
We have to calculate magnetic field at any point P
on the axis of solenoid, where OP = r. Consider a small
r

element of width dx of the solenoid, at a distance x M---- / —


ou
ad

N- -H
from O. r
Y

Number of turns in the element = n dx.


Using eqn. (16) on page 4/12, magnitude of magnetic field at P due to this current element is
nd
Re

dB =
Pq i (n dx) ...(10)
Fi

2[(r-x)2+a2p^2
If P lies at a very large distance from O, i.e., r»a and r » x, then [(r - x)^ + a^]^^ ®
dB =
Pq I a^ n dx ...(11)
2r^
As range of variation of jr is from x^ — ltox=i + l, therefore the magnitude of total magnetic field at P
due to current carrying solenoid
● 2 x=+l
B =
Po nia dx - Pq nia^
2r^ x=-l 2r^

Ho 2ni2l)ina^
B^ ...(12)
2 4tc r3
5/10
'4. Fundamental Physics (XII)
If M is magnetic moment of the solenoid, then
M = total no. of turns x current x area of cross section
M = « (2 /) X / X (ji (p-)

B =
fLo 2M
4tc P ...(13)

This is the expression for magnetic field on the axial line of a short bar magnet. For details, see Art. 5.7.
Thus, the axial field of a finite solenoid carrying current is .same as that of a bar magnet. Hence, for all
practical purposes, a finite solenoid carrying current is equivalent to a bar magnet.

oww
Retain in Memory
1. Magnetic dipole moment of a current carrying solenoid is FIGURE 5.18
determined by right hand rhumb rule. According to this rule,
when fingers of right hand curl in the direction of current, the

ee
B 8
thumb of right hand gives the direction of magnetic dipole

FFrlo
moment of the solenoid Fig. 5.18

r
rF
2. When two small magnets of magnetic moments A/j and M-y are

ee
© ©
placed distance r apart, then the force acting between them A A'
ouru Pq 6A/,A/^

rF
(0 in the end on position is, F =
471

ffosor
(«) in the broad side on position is, F = Pq 3
os k M2
4n
ook
Yo
Y
5.10. TORQUE ON A BAR MAGNET IN A MAGNETIC FIELD
Bo

In Fig. 5.19, a uniform magnetic field B is


reeB

FIGURE 5.19
represented by equidistant parallel lines. NS is a bar
oouY

>-
magnet of length 2/ and strength of each pole = m. The
ur

■►mB
magnet is held at Z0 with the direction of B .
ad

■►
dY

0
Force on N-po\e = mB, along B ■►B
N S
nidn

Force on 5-pole - mB, opposite to B


Re

>
S/<Y0
These forces being equal, unlike and parallel will mB A
F
Fi

form a couple, which tends to rotate the magnet ■►

clockwise so as to align it along 5 .

Draw NA perpendicular to B and SA II B


.'. Torque acting on the bar magnet
X = moment of the couple = force x perp. distance = mB x NA ..(14)

In ANAS, sin 0 = NA _ NA
NA = 21 sin 0
NS ~ 21
Put in (14), X = mB X 21 sin 0. As M = m.y 21 X = MB sin 0 ...(15)

In vector form, we can rewrite this equation as z = M xB


MAGNETISM AND MATTER 5/11

The direction of 7 is perpendicu!;^- to the plane containing and B , and is given by irght handed
screw rule.
When B = 1 and e = 90^ then from (15), t = A/ x 1 sin 90“ = A/ or Af = x
Hence we may define

Magnetic dipole moment is the torque acting on a magnetic dipole held perpendicular to a
uniform magnetic field of unit strength.
Unit of Af is unit of x divided by unit of B. Therefore, the SI unit of M is joule per tesla (JT” ).

5.11. DETERMINATION OF MAGNrrUDE OF


MAGNETIC FIELD USING A COMPASS NEEDLE

oww
In Fig. 5.20 equidistant parallel lines represent a uniform magnetic field
B in the plane of the paper. NS is a small compass needle of magnetic moment
M, held at an angle 0 with B.
Torque acting on the compass needle, x = MB sin 0. This is restoring

e
ree
—»

rFl
torque as it tries to align the compass along the direction of B..

Fre
But X = / ot, where I is moment of inertia of the compass needle and ot is its angular acceleration. When

rrF
0 is small, sin 0 = 0.
-MB
0 or a « 0 (Minus sign for restoring torque)
ouur
X = - MB 0 = / a or a =

The motion of compass needle is simple harmonic. Time


/
sffoo
period of oscillation of compass needle is
kks
0
oooo
Yo
T = 271
\ a
eBB

/
T = ln
V MB
urr

Squaring both sides, we gel


You
ad

I 4 71^/
T- =47t^ or B =
dY

MB MT'^
Knowing M, / and T, we can calculate B.
innd
Re

5.12. POTENTIAL ENERGY OF A MAGNETIC DIPOLE IN A MAGNETIC FIELD


FFi

Potential energy of a magnetic dipole in a magnetic field is the energy possessed by the dipole
due to its particular position in the field.
When a magnetic dipole of moment M is held at an angle 0 with the direction of a uniform magnetic
field B , the magnitude of the torque acting on the dipole is
X = MB sin 0 ...(16)

This torque tends to align the dipole in the direction of the field. Work has to be done in rotating the
dipole against the action of the torque. This work done is stored in the magnetic dipole as potential energy of
the dipole.
Now, small amount of work done in rotating the dipole through a small angle dd against the restoring
torque is
dW =zdQ = MB sin 0 dQ
s/12
'4. Fundamental Physics (XII) VOL.I

Total work done in rotating the dipole from 0 = 6, to 0 = 63 is


6-7
? 0,
MB sm 0 d0 = M5 [- cos 0] = - MB [cos 02 - cos 0]]
e1 6
1

w
Potential energy of the dipole is (/ = W = - MB (cos 02 - cos 0|) (17)

When 0j = 90°, and @2 = 0, then t/ := W = - MB (cos 0 - cos 90°)


C/=VK=:-Mflcos0 ...(18)

e
In vector notation, we may rewrite (18) as

e
wr
...(19)

lloo
U =~M.B

r
Particular Cases

eF
1. When 0 = 90°

u
r FF £/ = - MB cos 0 = - cos 90° = 0

i.e, when the dipole is perpendicular to magnetic field its potential energy is zero.

rrse
o
koF
Hence to calculate potential energy of dipole at any position making angle 0 with B. , we use

U -~MB (cos 02- cos 0])

oofr
and
take 0j = 90° and 02 = 0. Therefore,
sf
ou
ko
YY
V -- MB (cos 0 - cos 90°) = - MB cos 0
B

2. When 0 = 0°
BY
oo

— MB cos 0° = - MB, which is minimum.


r

This is the position of stable equilibrium, i.e., when the magnetic dipole is aligned along the magnetic
uur ee

field, it is in stable equilibrium having minimum .PE.


d

3. When 0 = 180°
oo
ad

U-- MB cos 180° = MB, which is maximum. This is the position of unstable equilibrium.
din
Y

A magnet having a magnetic moment of 4-0 x lO'* J/T is free to rotate mi:_
a horizontal plane where a magnetic field 4 x 10"^ T exists. Find the work done in rotating the magnet
Re
FF
in

slowly from a direction parallel to the field to a direction 60° from the field.
Sol. Here, M = 4-0 x J/T, B = 4 x 10"^ T, W=l, 0j = 0°, @3 = 60°
W = -MB (cos 02 - cos 0j) = - 4-0 X lO'^ X 4 X 10"^ (cos 60° - cos 0°)

= -1-6 —1 =0-8J
.2

Calculate the magnitude of torque required to hold a bar magnet of


magnetic moment 200 Am^ along a direction making an angle of 30° with the direction of uniform
magnetic field of 0*36 G.
Sol. Here, x = ?, M = 200 Am^, e = 30°. B = 0-36 G = 0-36 x 10"^ T
T = MB sin 0 = 200 x 0-36 x 10^ sin 30° = 3*6 x 10“^ N -m
MAGNETISM AND MATTER 5/13

5.13. THE ELECTROSTATIC ANALOGUE OF MAGNETISM -*

We have studied that two unlike magnetic poles of equal strength (m) separated by a distance (2 1)
form a magnetic dipole. It has a magnetic moment M ~ m (2 I), which is directed from south to north
through the magnet.
The magnetic dipole is analogous to an electric dipole, which consists of two equal charges of opposite
sign (± g) separated by a certain distance (2 a). It has an electric dipole moment (2 a), which is
directed from - ^ to + ^ through the dipole.
The equations for magnetic field B due to a magnetic dipole (or bar magnet) can be obtained from the
equations of electric field E due to an electric dipole by making the following changes :
^^0

ww
■>
E >B ’ P ’ 5^ 0 4tc

Therefore, for any point on axial line of short bar magnet at a distance (d) from the centre of magnet, we
_ 1^0 2A/

Flo
wnte B
1 “

e
4?:

ree
And for any point on equatorial line of short bar magnet at a distance (d) from the centre of magnet, we

Fr
rF
write
^2 = 4tc d^
uurr
This corresponds to torque on electric dipole in an for
Further, torque on magnetic dipole in an external magnetic field B is, x = MxB
external electric field E as, x = pxE
s
kks
^ ^ ^

Similarly, the potential energy of magnetic dipole in an external magnetic field B is, U ~~M .B
Yo
oooo

This corresponds to potential energy of an electric dipole in an external electric field E as, U = - p.E
eB

The analogy between magnetism and electrostatics is summarised in the following table :
Magnetism Electrostatics
S. No. Physical Quantity
r
ou
ad

—»
YY

1. Dipole moment M =mx2 I p = qx2 a


^
X = M xB X = p xE
nndd

2. Torque in external field


Re

3. Potential energy of dipole u = -M.B u = - p.E


Fi

IIq (2M I 2p
4. Field intensity on axial line ^1 = ^1 = 47te 0 V r3
4n d^
1 P
5. Field intensity on equatorial line £2 = £2 =
4tc [d'^ 4jie
0 V r3

Retain in Memory
There is one fundamental difference between electricity and magnetism. Whereas in electricity,
an isolated charge can exist; in magnetism, an isolated magnetic pole does not exist. The simplest
magnetic structure that can exist is only a magnetic dipole or current loop characterized by magnetic
dipole moment M . This is unlike a scalar charge q which characterizes electricity.

1
5/14
^>uKtec^'^ Fundamental Physics (XII)BSISI

Retain in Memory
In an external magnetic field, the magnetic dipole experiences a torque (unlike the force experienced
by a charge q in an external electric field). The effect of torque is to align the dipole along the
external magnetic field. The directive property of a magnet is attributed to the torque acting on
the magnetic dipole in the magnetic field of earth.

5.14. GAUSS'S LAW FOR MAGNETISM

According to Gauss *s lawfor magnetism, the net magnetic flux ) through any closed surface

oww
is always zero.

The law implies that the number of magnetic field lines leaving any closed surface is always equal to the
number of magnetic field lines entering it.
Suppose a closed surface S is held in a uniform magnetic

e
re
field B . Consider a small vector area element A ? of this

FFrllo
surface, Fig. 5.21. Magnetic flux through this area element is

rF
—> j

ee
defined as = B .A s . Summing over all small area
ouru
elements of the surface, we obtain according to Gauss’s law for

rF
magnetism,

^B = 'L^h='L b.a7 = o
fosor ...(20)
all
os kf
all

If the area elements are really small, we can rewrite this


ook
YYo
equation as
Bo

= () B .ds = 0
reeB

(21)
s
oouY

Compare this equation with Gauss’s law in electrostatics, i.e., electric flux through a closed surface S is
ur

given by
ad
Yd

f n

“ T £ . c/5 = — (22)
^0
nidn
Re

where q is electric charge enclosed by the closed surface. If an electric dipole were enclosed by the
FFi

surface equal and opposite charges in the dipole add upto zero. Therefore, tjtg would be zero. The fact that <|)5
— 0 indicates that the simplest magnetic element is a dipole or current loop. The isolated magnetic poles,
called magnetic monopoles are not known to exist. All magnetic phenomena can be explained in terms of an
arrangement of magnetic dipoles and/or current loops.
Gauss’s theorem in magnetism establishes that:
(/) isolated magnetic poles called magnetic monopoles do not exist, or
(//■) in magnetism, there is no counterpart of isolated free charge in electricity, or
(///) magnetic poles always e.xist in unlike pairs of equal strength.
Thus corresponding to eqn. (22) of Gauss’s theorem in electrostatics, we can visualize eqn. (21) as
f —* —»
4)^ = J 5 . i/5 = |0q (m) + Pq (-m) = 0
s

where m is strength of N-pole and ~ m is strength of 5-pole of same magnet; )Iq is absolute magnetic permeability
of free space.
MAGNETISM AND MATTER 5/15

5.15. MAGNETIC FIELD OF EARTH

Sir William Gilbert was the first to suggest in the year 1600, that earth itself is a huge magnet His
statement was based on the following evidence :
(/) A magnet suspended from a thread and free to rotate in a DO YOU KNOW ?
horizontal plane comes to rest along the north-south direction. On
disturbing, the magnet returns quickly to its north-south direction again. ● Though there are large deposius
This is as if a huge bar magnet is placed approximately along the axis of iron inside the earth, it is not
of rotation of earth and deep in its interior. The north pole of this at all likely that a large solid
fictitious magnet must be towards geographic south so as to attract soutli block of iron stretches from

pole of the suspended magnet and vice-versa. We must add here that north pole to south pole.
fictitious magnet is just a conceptual device adopted to explain the
earth’s magnetic field and its peculiarities. In reality, there is no such

ww
bar magnet in the interior of the earth,
(ii) When a soft iron piece is buried under the surface of earth in the north south direction, it is found to
acquire the properties of a magnet after some time.

FF loo
(in) When we draw field lines of a magnet, we come across neutral points. At these points, magnetic
field due to the magnet is neutralized or cancelled exactly by the magnetic field of earth. If earth had no

ree
magnetism of its own, we would never observe neutral points.

rreeF
The branch of Physics which deals with the study of magnetism of earth is called terrestrial
magnetism or geomagnetism.
ur
fofroF
It has been established that earth’s magnetic field is fairly uniform. The strength of this field at the
surface of earth is approximately 0-1 gauss or 10“^ tesla. The field is not confined only to earth’s surface. It
ks
extends uplo a height nearly 5 times the radius of earth (= 5 x 6400 km = 32000 km).
kos o
YYouor

5.16. CAUSE OF EARTH'S MAGNETISM


BBoo

The exact cause of earth’s magnetism is not yet known fully. However, some important postulates in this
r ee

respect are as follows :


(fl) As stated earlier, William Gilbert suggested in the year 1600 that the earth itself is a huge magnet.
ad

(b) According to Prof. Brackett, earth's magnetism may be due to rotation of earth about its axis. This
oouur

is because every substance is made up of charged particles (protons and electrons). Therefore, a substance
Y

rotating about an axis is equivalent to circulating currents, which are responsible for the magnetisation,
(c) In the outer layers of earth’s atmosphere, gases are in the ionised state, primarily on account of
nYd
Re

cosmic rays. As earth rotates, strong electric currents are set up due to movement of (charged) ions. These
FFinid

currents might be magnetising the earth.


(d) The earth’s core is extremely hot and molten. Circulating ions of iron and nickel in the highly
conducting liquid region of the earth's core might be forming current loops and producing earth’s magnetic
ifeld. This hypothesis is supported by the following facts :
(/) Moon has no molten core and hence no magnetic field, (ii) Venus has a slower rate of rotation and
hence a weaker magnetic field. (Hi) Amongst the planets, Jupiter has the fastest rotation rate. Therefore, it has
a fairly strong magnetic field.

5.17. GENERAL FEATURES OF EARTH'S MAGNETIC FIELD

The earth’s magnetic field is approximately like that of a giant magnetic dipole, whose axis is inclined
roughly 11-3° west of the axis of rotation of earth, which is along geographic north and geographic south
S „ Fig. 5.22. The south pole of this giant magnetic dipole is towards geographic north and north pole of this
di^le is towards geographic south. The strength of earth’s magnetic field at its surface is of the order of
10“^ tesla. For example, at Delhi, earth’s field is 0-35 gauss = 3-5 x 10"^ T.
5/16 'P’undee^'it. Fundamental Physics (XII)QSB]
As the north pole of a freely suspended
magnet points approximately towards N^,
therefore, earth’s magnetic pole near A/^ is called
north magnetic pole N^. Similarly, the magnetic
pole near Sg is called south magnetic pole S m'

Note that and are points on earth’s


surface and not in Ae interior of earth. Therefore,
we can specify their location exactly in terms of
longitude and latitude. At present, is located
at a place in North Canada, longitude 71-8® W
and latitude 79-74® N. Obviously, will be at a
place diametrically opposite to Le., longitude
108-2° E and latitude 79-74“ S. The magnetic

ww
poles are approximately 2000 km away from
geographic poles.
As is clear from Fig. 5.22, magnetic equator

Flo
divides the earth’s surface into two (magnetic)

e
hemi spheres. The magnetic field lines enter into

ree
the hemisphere containing Ng and come out of

Fr
the hemisphere containing Sg.

rF
Note that the straight line passing through
uurr
the geographic north and south poles of earth is
called the geographic axis. It is the axis of
rotation of earth.
s for
The straight line passing through magnetic north and south poles of earth is called magnetic axis. The
kks
Yo
great circle on the earth perpendicular to the magnetic axis is called magnetic equator.
oooo

The vertical plane passing through geographic north and south poles is called geographic meridian.
eB

And the vertical plane passing through the magnetic axis of a freely suspended magnet is called magnetic
meridian.
r

5.18. MAGNETIC ELEMENTS


ou
ad
YY

Magnetic elements of earth at a place are the quantities which describe completely in magnitude
as well as direction, the magnetic field of earth at that place.
nndd
Re

Following are the three magnetic elements of earth :

1. Magnetic declination (6),


Fi

2. Magnetic inclination or Magnetic dip (5),


3. Horizontal component {H) of earth’s magnetic field.
1. Magnetic declination. FIGURE 5.23
When a magnet is suspended freely by an unspun silken thread, the
ends of the magnet do not point out exactly along geographic north south
direction, as shown in Fig. 5.23. As stated already, the line joining the north
and south poles of a freely suspended magnet is called magnetic axis. The
axis about which earth rotates is the geographic axis. ^9.
The small angle between magnetic axis and geographic axis at
a place is defined as the magnetic declination at that place. It is sl»)— N

represented by 0, Fig. 5.23.


" N
g
MAGNETISM AND MATTER 5/17

In Fig. 5.24, CD represents the magnetic axis and


CD represents the geographic axis. The vertical plane
CDAB passing through magnetic axis CD is the magnetic
meridian and the vertical plane CDAB' is the geographic
meridian. By definition, ./CDC - .ZBAB' = 6 is the
magnetic declination. Hence, we may also define
Magnetic declination at a place is the angle
between magnetic meridian and geographic
meridian at that place.

Magnetic declination arises because the axis of the


fictitious bar magnet responsible for earth’s magnetism
does not coincide with the axis of rotation of earth (which

ww
is the line joining the geographic north Ng and geographic
south Sg). Infact, the imaginary magnet lies at an angle
of about 11-3® west of the polar axis as shown in Fig. 5.22.

Floo
This is the order of magnetic declination at the place.
2. Magnetic Dip or Magnetic Inclination

ee
Magnetic dip or Magnetic inclination at a place is defined as the angle which the direction of

reer
total strength of earth’s magnetic field makes with a horizontal line in magnetic meridian.

rFF
It is the angle by which total intensity of earth’s magnetic field dips or comes up out of the horizontal
uur r
plane. It is represented by 5. The value of magnetic dip at a place can be measured using an instrument called
‘Dip circle’. ffoor
sks
The essential parts of a dip circle are shown in Fig. 5.25. A
YYoo
magnetic needle NS is pivoted on a horizontal axle at the centre of a
ooko

circular vertical scale V. The ends of magnetic needle are free to move
over this scale graduated in degrees marked 0° - 0® in the horizontal
eBB

direction and 90® - 90® in the vertical direction. The needle and the
scale are enclosed in a rectangular box B with glass windows. The
r

box is mounted on a vertical pillar P on horizontal base HB provided


ouur
ad

with levelling screws L&L'.A circular scale C (0® to 360°) is engraved


Y

on the HB. The box can be rotated over the horizontal base and its
position can be read on the circular scale C with the help of a vernier
nd dY

scale V' which can slide over circular scale.


Re

To use the dip circle, (i) adjust the levelling screws till the base
FFini

is horizontal and scale V inside ±e box is vertical.


(h) rotate the box B till the ends of magnetic needle NS read
90® - 90° on the vertical scale.
Obviously, the needle in this position is aligned with the vertical
component of earth’s magnetic field. The horizontal component of
earth’s field being perpendicular to this plane does not affect the
needle.

The reading of the vernier V' is noted on horizontal circular scale C.


(in) rotate the box exactly through 90° from this position. The plane of vertical scale V is now in magnetic
meridian. The total field strength R of earth’s magnetic field is in the same plane and hence magnetic needle
rests along R . Angle which R makes with the horizontal direction in magnetic meridian is the angle of dip.
As horizontal line is marked 0-0, the readings of the ends of magnetic needle on the vertical scale give us
directly the angle of dip (6), Fig. 5.25.
5/18
4 Fundamental Physics (XII) W»Tl
At different places on the surface of earth, 5 is different. For example, at the two magnetic poles, the
magnetic needle rests vertically. Therefore, angle of dip at the two poles is 90°. Obviously in the northern
hemisphere, where south pole of ficitious magnet lies, the north pole of the mounted magnetic needle will dip
down and vice-versa. On the magnetic equator, the angle of dip is zero as the needle would rest horizontally
at the magnetic equator. At other places, the value of 6 lies between 0° and 90°. The dip angle at Delhi is about
42° below the horizontal.
3. Horizontal Component

It is the component of total field strength of earth’s magnetic field in the horizontal direction in
magnetic meridian. It is represented by H.

In Fig. 5.24, AK represents the total field strength of earth’s magnetic field, .ZBAK = 6. The resultant

oww
field R along AK is resolved into two rectangular components:
Horizontal component along AB is At = // = /? cos 6 ...(23)
Vertical component along AD is AM - K= /? sin 6 ...(24)
Square (23) and (24), and add g ^ gjj^2 gj _ p.2

e
FFrlo
re
R = ...(25)

ree
F
Dividing (24) by (23), we gel

rF
/? sin 6 _ V V
tan 5 = —
or
...(26)
R cos 6 H H

ffsoor
ouur
The value of horizontal component H = R cos 5 is different at different places. At the magnetic poles,
kosk
8 = 90 //= V? cos 90° = zero
At the magnetic equator, 8 = 0‘ // = /? cos 0° = /? = maximum.
YYo
oo

Horizontal component {H) can be measured using both, a vibration magnetometer and a deflection
BB

magnetometer.
The value of // at a place on the surface of earth is of the order of 3-2 x 10“^ tesla.
re

Retain in Memory
uur
ooY
ad

Note that the direction of horizontal component H of earth’s magnetic field is from geographic
south to geographic north above the surface of earth, (if we ignore declination).
dY

More about angle of dip S


innd
Re

(/) At a place on the magnetic poles, total earth’s magnetic field is perpendicular to the surface of earth
i.e. vertical.
FFi

R = V As V = /? sin 8 sin 8 = 1 ; 8 = 90°


As
H = Rcosb-Rcos 90° = 0 therefore, at magnetic poles, 8 = 90° and H = 0
i.e., earth always has a horizontal component except at magnetic poles. A freely suspended magnet at
poles will stand vertical with its north pole pointing towards earth’s geographical north pole (which is magnetic
south), and vice-versa.
(«) At a place on the equator, total earth’s magnetic field is parallel to the surface of earth i.e. horizontal.
R = H
As H = R cos 8 cos 8 = 1 ; 8 = 0°
As
V = /? sin 8 = /? sin 0° = 0. Therefore, at the equator, 8 = 0° and V = 0
i.e., earth always has a vertical component except at equator. A freely suspended magnet at equator will
stand horizontal.

(m) In a vertical plane at an angle 0 with the magnetic meridian.


H' = H cos e and V'= V
MAGNETISM AND MATTER 5/19

Therefore, observed angle of dip 6' in a vertical plane making Z9 with magnetic meridian is given by
r V tan 6 tan 6
tan 6' = I.e., tan 8' = ...(27)
H' Hcosd COS0’ COS0

Here, 5 is true value of dip and 5' is apparent value of dip.


For a vertical plane other than magnetic meridian, 0 > 0°, cos 0 < 1
/. 6' > 5 i.e., angle of dip increases,
tan 6
when 0 = 90°, tan 5' = r: oo

cos 90°

6' = 90° i.e., in a plane perpendicular to magnetic meridian, dip needle will stand vertical.
(iV) If 6j and §2 are observed angles of dip in two arbitrary vertical planes-which are perpendicular to
eachother, then the true angle of dip 5 is given by the relation :

ww
*cot^ 8 = cot^ 8j + cot^ &2 ...(28)

Floo
(f) If X is magnetic latitude at a place, then it can be shown that the angle of dip 8 at that place is given by

ee
tan 8 = 2 tan X, ...(29)

reer
rFF
Retain in Memory
uur r
ffoor
A magnet suspended freely at equator will set itself parallel to the surface of earth while one suspended
at pole will stand vertical.
sks
YYoo
ooko

iSarripTefR'iioBfem^ At a place, the horizontal component of earth’s magnetic field is B and


eBB

angle of dip is 60°. What is the value of horizontal component of earth’s magnetic field at equator ?
(CBSE 2017)

Sol. Here, // = S and 8 = 60°.


r
ouur

H = R cos 8
ad

From
B = R cos 60° = R/2 R = 2B
Y

At equator, 8 = 0 H — R cos 0° = 2 B cos 0° = 2 B


nd dY

'sample;Rro The horizontal component of earth’s magnetic field at a place is


Re

0*4 X 1(H T. If angle of dip is 45°, what are the values of vertical component and total field strength of
FFini

earth’s field ?
Sol. Here, // = 0-4 X 10-^T, 8 = 45°, //= ? R = ?
V
As — = tan o .-. V = /ftan5 = 0-4x 10“^ tan 45° = 0-4 x IQ-^T
H

H 0-4xl0"^
From H = R cos 8, R = = 04V2xI0-^T =0*56xl0-^T
cos 8 cos 45°

5.19. VARIATION OF EARTH'S MAGNETIC FIELD


(ff) Global Variation (i.e. from place to place).
Strength of terrestrial magnetism as we know, is small (» 4 x 10“^ T). Smaller still is the back ground
field of our galaxy, the Milky way (= 2 x 10“'^ T).
*For proof, see Q. 25 of Problems for Practice, Page 5/73.
5/20
4^ Fundamental Physics (XII) WSTMl
At about 30000 km above the surface of earth, the earth’s field falls below 10“^ T. Beyond this height,
solar wind disturbs the dipole pattern. The solar wind consists of streams of charged particles that emerge
continuously from the sun. These particles get trapped near the magnetic poles of earth. They ionise the
atmosphere above these poles causing a spectacular display of light.Magnetic maps showing the variation of
magnetic elements from place to place have been drawn.
On these magnetic maps :
(i) The lines joining places of equal declination are called isogonic lines,
(it) The lines joining places of zero declination are called agonic lines.
(Hi) The lines joining places of equal dip/inclination are called isoclinic lines,
(tv) The lines joining places of zero dip/inclination are called aclinic lines. Similarly,
(v) The lines joining places of equal values of horizontal component (H) of earth’s magnetic field are
called Isodynamiclines.

w
(b) Temporal Variation g from time to time)
The magnetic field of earth is found to change with time. These changes may be characterised as short
term and long term changes.

FF loow
In short term changes, the magnetic poles of earth S,„ and keep on shifting their position. For

ee
example, magnetic south pole (in northern arctic region of Canada) has been found to move in north-westerly

Fr
direction at a rate of 10 km per year. Detailed maps of earth’s magnetic field are maintained and revised from
time to time.

The long term changes are on geological time scale =10^ to 10^ years. It appears that the direction of

rer
ur
fofr Fo
earth s magnetic field has reversed itself every million year or so. It is believed that once in a million years or
so, the currents in the earth’s core slow down, come to halt and then pick up in the reverse direction.
ks
5.20. NEUTRAL POINTS
YYouro
s oo

When we trace magnetic lines of force around a magnet using a compass needle, what we obtain is the
resultant of the magnetic field of magnet and that of the earth. As earth’s magnetic field is fixed, the resultant
BBook

magnetic field would depend on the direction in which the magnet is placed. In the plot of the
r ee

resultant magnetic field, we come across points at which field


(B) due to the magnet becomes equal and opposite to the
oouru
ad

horizontal component (H) of earth’s field i.e. B = H.


Y

Therefore, the net magnetic field at these points will be


zero. These points where net magnetic field due to the
Ynd
Re

magnet and magnetic field due to the earth is zero are

called Neutral points, a small compass needle placed at a


FFindi

neutral point shall experience no force/torque. Therefore, it


can set itself in any direction, which may be different from
the usual NS direction.

We can show that (/) When a magnet is placed with its


north pole towards geographic south, neutral points P and
Q lie on axial line of the magnet as shown in Fig. 5.26.
At each neutral point,

B _Mo 2Md
= H
I ~
(i/2_/2)2
If the magnet is short /« d.

B,I = = H
471 d'^
MAGNETISM AND MATTER 5/21

(li) When a magnet is placed with its north pole


towards geographic north, neutral points P and Q lie
on equatorial line of the magnet as shown in Fig.
5.27.

At each neutral point,


M
R =^- = H
2 4tC

If the magnet is short, /« d

B. = = H
4tc i/3

ww
In both the formulae, symbols have their usual meaning.
Note. Comparison of Figs. 5.26 and 5.27 shows that when north pole of bar magnet is turned
through 180", neutral points turn in the same direction through 180"/2 = 90".

Flo
'Sampje Problem D A short bar magnet is placed with its north pole pointing south. The

e
neutral point is 10 cm away from the centre of the magnet. If H = 0*4 gauss, calculate magnetic moment

reree
of the magnet.

r FF
Sol. Here, i/ = 10 cm = 0-1 m, // = 04 gauss = 0-4 x 10“^ T, Af = ?
Mo 2iVr = H
Neutral points in this case, lie on axial line of magnet, such that
uurr
4n d^
lO'^x
2M

(0-1)
3
= 04x10"^ or foor
A/ = 0‘2 A m2
kss
Yoo
ooook

■sample Problem Q A short bar magnet is placed with its north pole pointing north. If
magnetic moment of magnet is 04 Am^ and horizontal component of earth’s magnetic field is 04
eBB

gauss, what is the distance of neutral point from the centre of the magnet ?
Sol. Here, M = 0-4 Am^, H = 0 4 gauss = 0-4 x 10^ T , = ?
rr
ouu
ad

When north pole is pointing north, b = = H


47t c/3
Y
dY

0-4
10“2x = 04x10“^
c/3
Re
innd

i/3= 10-3, c/= lO"* m=10cm


Fi

5.21. THE TANGENT LAW IN MAGNETISM

According to the tangent law in magnetism, when a


magnet is suspended under the combined action of
two uniform magnetic fields of strengths F and H,
acting at 90° to each other, the magnet comes to rest
making an angle 0 with the direction of H, such that
F = HtanQ ...(30)

In Fig. 5.28, NS is a bar magnet of length 21, strength of each


pole = m and dipole moment M = m .y 21. Under the combined
action of two mutually perpendicular fields F and H, the magnet
comes to rest at an angle 0 with H.
5/22
Fundamental Physics (XII) ESSO
A couple acts on the magnet due to each magnetic field. In equilibrium, the moments of the two couples
balance each other.

Draw NA parallel to H and SA parallel to F. Let them meet at A.


Moment of the couple due to field H
Xj - mH X SA
It is in the anticlockwise direction.
Moment of the couple due to field F
^2 - mF X NA
It is in the clockwise direction. In equilibrium, the moments of two couples must be equal and opposite
l.e.y
T2 = Tj
SA
mF xNA = mH x SA

ww
or F = Hx = H tan 9, i.e. F = //tane ...(31)
NA

which proves the tangent law.

Floo
Usually, H is horizontal component of earth’s field and F is field due to a magnetic dipole or current
loop.

ee
5.22. TANGENT GALVANOMETER

reer
rFF
It is an instrument which is used for detection
and measurement of low electric currents.
uur r
Principle. It is based on tangent law in
magnetism. ffoor
sks
Construction. A tangent galvanometer consists
YYoo
of circular frame AT of a non magnetic material like
ooko

wood, ebonite or brass. It is mounted vertically on a


eBB

horizontal base called turn table (7), provided with


levelling screws 5j and 52 at the base. The frame can
be rotated about a vertical axis passing through its
r
ouur

centre. Fig. 5.29.


ad

Three coils of fine insulated copper wire having


YY

2,50 and 500 number of turns are wound on the circular


frame. The coils are well insulated from one another.
nndd
Re

One end of each of the three coils is connected to a


common terminal C and their other ends are connected
FFii

SCREWS
to terminals on the turn table.
A circular magnetometer box of non-magnelic material is held at the centre of circular frame. This box
has a small magnetic needle ns pivoted at its centre on a vertical axis with a long thin aluminium pointer p
fixed at 90" to the magnetic needle. Both, the magnetic needle and the pointer are free to move as one body.
The ends of the pointer lie over a horizontal circular scale graduated in degrees and divided into four quadrants
of 0° - 90" each. A small plane mirror is fixed at the base under the pointer. This would remove error due to
parallax in reading the position of the pointer on the scale.
Setting : (/) Adjust the levelling screws so that the turn table is horizontal and the frame is exactly
vertical.

{//) Rotate the vertical frame so that the plane of the frame lies exactly over the magnetic needle in the
box. This sets the frame in magnetic meridian.
(Hi) Hold the vertical frame firmly with hand and turn the magnetometer box, so that ends of the aluminium
pointer lie on zero zero line, on the scale.
MAGNETISM AND MATTER 5/23

Theory and Working. In Fig. 5.30, XY is cross section of the FIGURE 5.30

circular frame perpendicular to the plane of the paper. X


As the frame is set already in magnetic meridian, therefore, [XX]
earth’s horizontal field H is along YX, {i.e., along south north H) k
direction). When a current I is passed through any one coil* of W E

tangent galvanometer, a magnetic field F develops along the axis of


the circular coil and perpendicular to the plane of the coil i.e. along
east west direction. The magnetic needle mounted in the Ps 0 s
n

magnetometer box is under the combined action of two mutually


perpendicular fields F and H. The needle gets deflected and comes > F
to rest making an angle 0 with the direction of H. s

The pointer also turns through the same angle 0. As pointer
was already set on 0-0 line, therefore angle 0 is read from the scale

ww
under the ends of the pointer.
According to tangent law in magnetism,
F = // tan 0 ...(32) XX

Flo
y
^ Mo 27m/**

ee
But r —
4k r

rere
where n is number of turns in the coil, r is radius of the coil and |iQ is magnetic permeability of free space.

rFF
^0 27m/ = H tan Q 47Tr // tan 0
.'. From (32), or I =
uur r
4tc r

2rH
foor H
)Xq .27m
sks
/ = tan 0 = tan0
...(33)
Yoo
or
«^0 njj,Q/2r
ook
BBo

n\iQ H
Now = G . the galvanometer constant. / = — tan 0
re

2r G

H H 2rH
ouur

Put = K , another constant.


ad

G w|lo/2r npo
YY

K is called reduction factor of tangent galvanometer.


...(34)
nndd

From (33), / = ^:tan0


Re

By measuring 0, and knowing K, current I can be calculated.


FFii

Reduction factor of Tangent Galvanometer


When 0 = 45“, from (34) ; / = /i: tan 45° = Kxl or K= I
Hence reduction factor of a tangent galvanometer is equal to current, required to be passed
through tangent galvanometer to produce a deflection of 45°. K is measured in ampere.
The value of K depends on number of turns («), radius of coil (r) and horizontal component of earth’s
magnetic field (//) at the place. Obviously, reduction factor of a tangent galvanometer increases with increase
in radius of the coil, and decreases with increase in number of turns of the coil.
Note that a tangent galvanometer is also called moving magnet type galvanometer, because, here the
coil is stationary and the magnetic needle moves. This is contrary to moving coil galvanometer. Art. 4.27.
*The coil is chosen according to the strength of current I to be measured. For low currents, coil with larger
number of turns is used. This is because from (33), / x n tan 0
Refer to Art. 4.7, B is replaced by F here.
5/24 priade^ ^ Fundamental Physics (XII) orafi

'.Sample Problem D The radius of the coil of a tangent galvanometer is 16 cm. Ho>v many
turns of the wire should be used if a current of 40 mA is to produce a deflection of 45“. Given, horizontal
component of earth’s field is 0*36 x 10"^ T.
Sol, Here, r = 16 cm = 016 m, « = ?, / = 40 mA = 40 x 10"3 A, 0 = 45^ H = 0-36 x IQ-^ T

As
2r// tan 0 2xO-16xO-36xlO“^ tan 45°
= //tan0 n — = 229
2r 471x10 ^ x(40xl0

Problem Q Two short magnets having magnetic moments in the ratio 8 : 1, when
placed on opposite sides of a deflection magnetometer produce no deflection. If distance of stronger
magnet from the centre of deflection magnetometer is 6 cm, what is the distance of weaker magnet
from the centre ?
M

w
Sol. Here, - = -,£/[= 6 cm ^^2 “ ^
^2 1

Flo
M ( d
1 _ ^
For short magnets, In the null method t = (2)3

e
^2 1

reee
d d

FFr
I 6 -
= 2; d,2 =-!2 — = 3 cm
^2 2

for
ur
5.23. BOHR MAGNETON OR MAGNETIC DIPOLE MOMENT
OF AN ATOM DUE TO REVOLVING ELECTRON
kkss
In every atom, electrons revolve around the nucleus. A revolving electron is like a loop of current, which
Yo
has a definite magnetic dipole moment. When electron revolves in anticlockwise direction, the equivalent
oo

current is clockwise, Fig. 5.31(a). Therefore, upper face of the electron loop acts as south pole and lower face
acts as north pole, Fig. 5.31{/7). Hence an atom behaves as a magnetic dipole.
B

If e is the charge on an electron revolving in an orbit of radius r with a uniform angular velocity ©, then
re

equivalent current
ou

. _ charge _ e
ad
YY

time T

where, T = the period of revolution of electron = 2 tc/©


nd
Re

e ©e

2ti/© 2ti: ...(35)


Fi

Area of the orbit A = 7i

Magnetic moment of the atom is given by


(Oe 1
M = i A = nr^ OT M = —e ©r
2
271 ...(36)
2

According to Bohr’s theory, an electron in an atom can revolve only in certain stationary orbitSy in
which angular momentum of electron (m v r) is an integral multiple of hH 7t, where h is Planck’s constant.
nh
Thus, mvr = where n= 1, 2, 3 denotes the number of the orbit. Using v = r©, we get
2tc’
nh nh
m(r©)r = or ©r^ = ...(37)
2tu 27tW
MAGNETISM AND MATTER 5/25

1 nh eh
M =-e. = n
...(38)
Put in (36), 2 27rm Anm

where = eh!A Km ...(39)

From (38), it is clear that the magnetic moment of an atom is quantised having values 1 2 |ig, 3
eh
and so on, where is the least value of atomic dipole moment, - . This is called Bohr Magneton.
4jim
It serves as the unit of atomic magnetic dipole moment. We can calculate its value.
eh 1.6x10-^9x6.6x10-^
...(40)
47un
4x —x9x10-3‘
7

|4g = 9.27 X 10
-24
ampere metre^. We may define

ww
Bohr magneton os the minimum magnetic dipole moment associated with an atom due to orbital
motion of an electron in the first stationary orbit of the atom.

Flo
1
— ecdr^

ee
Again, we define, Gyromagnetic ratio of electron = magnetic moment _ 2

rere
2
angular momentum mcor

r FF
ehlAnm e 1-6x10-‘9
uurr
= 8-8xI0^<^ C/kg
Win
foor 2m 2x9x10-^*
ks s
5.24. SOME IMPORTANT TERMS USED IN MAGNETISM
Yoo
ooook

To describe the magnetic properties of materials, we define the following few terms, which should be
clearly understood;
eBB

1. Magnetic field strength ; Magnetic field Induction or Magnetic flux density {B)
To understand this term, we make use of Lorentz force equation.
rr

When a positive test charge qQ is fired with a velocity v through a point P and the moving charge
ouu
ad

experiences a sideways force F, we assert that a magnetic field is present at P. The magnetic induction B of
YY

this field is a vector satisfying Lorentz force equation

...(41)
nndd

F = qf^(vxB)
Re
Fii

Magnitude of this force is, F = ^0 ^ sin 9 ...(42)


where 0 is smaller angle between v and B .

If, ^0 = ^ ^ ^ ® B = F. We may define


Magnetic field strength or magnetic field induction or flux density of magnetic field being
equal to the force experienced by a unit positive charge moving with unit velocity in a direction
perpendicular to the magnetic field.

We may also deifne Magnetic field strength or Magnetic field induction or Magnetic flux density (B) as
the number of magnetic field lines passing normally through unit area of the substance.
If <j) is the magnetic flux passing normally through an area A, then

(j) = B. A = BA cos 0, where 0 is the angle between magnetic field strength B and area vector A.
5/26 Fundamental Physics (XII) IWi

When magnetic field lines pass normally through the area,


0 = 0®, <f> = BA cos 0° = BA or B = 0/A
SI unit of magnetic flux density is tesla (7) or Weber/metre^

w
The CGS unit of B is gauss, where 1 gauss = 10“^ T

2. Magnetising force or Magnetising Intensity (//)


The degree to which a magnetic field can magnetise a material is represented in terms of magnetising

wr
e
force or magnetising intensity (H).

r
Let us consider a toroidal solenoid with n turns per unit length carrying a current I wound round a irng

oo
of a magnetic material. The magnetic induction of the field produced in the material of the toroidal solenoid

F
llu
IS

B = [Lnl ...(43)

FF
The product (n/) is called the magnetising force or magnetisingintensityH. i.e.

ees
rr
uro H = nl ...(44)

ooFk
so that from (43), B = \iH Hence, ...(45)

fofr
The magnitude of magnetising force may be defined as the number of ampere turns flowing
so
Y
round unit length of toroidal solenoid to produce the magnetic inductionB, in the solenoid.
oY
B

If inside the toroidal solenoid, there is free space, then magnetic induction
ok
Yo
Bq = Pq/7 ...(46)
eBr
rue

where |iQ is magnetic permeability of free space. // is a vector quantity.


oud

The SI units of H are ampere tum/metre i.e. Am"^


The dimensional formula of 7/is [M^ L~^ T^A^].
no
ad

3. Magnetic Permeability (ji)


iY

It is the ability of a material to permit the passage of magnetic lines offorce through it i.e. the degree
or extent to which magnetic field can penetrate or permeate a material is called magnetic permeability of the
nd
Re
F

material. It is represented by |i.


Magnetic permeability of a material is equal to the ratio of the magnitude of magnetic induction (B) to
Fi

the magnetising force or magnetising intensity (H). It is represented by [i.


B
I.e.
ft = — or B = [lH.
H

Unit of B
SI unit of magnetic permeability =
Unit ofH

T -1
r = Tm A
Am"’

Weber -1
l^ = — ● = Wb m~^ A *
m
MAGNETISM AND MATTER 5/27

As Wh A ^ = henry {H), therefore, SI unit of |a is also H m K


The dimensional formula of magnetic permeability is [MLT~^ A~^].
Relative magnetic permeability of a material is defined as the ratio of the number of magnetic
field lines per unit area in that material (Le.fiux density B in the material) to the number of
magnetic field lines per unit area that would be present, if the medium were replaced by vacuum.
(Le. flux density Bq in vacuum).

w
B
i.e., ...(47)
B
0

has no dimensions. Its value for vacuum is one.

e
Relative magnetic permeability of a material may also be defined as the ratio of magnetic

re
row
permeability of the material (|i) and magnetic permeability of free space (|1q)

eeF
P-

ullo
or ...(48)

FF
^^0

We know that pg = 4 x 10 ^ weber/amp-metre (Wb A ^ m *) or henry/metre (Hm *).

srr
roF
4. Intensity of Magnetisation (/)

k
It represents the extent to which a specimen is magnetised, when placed in a magnetising field.
uor
Quantitatively,
ofof
The intensity of magnetisation of a magnetic material is defined as the magnetic moment per
kos
unit volume of the material placed in the magnetising field.
YYo
eerBB
oo

^ _ magnetic moment _ M
rY

...(49)
volume V

If a = uniform area of cross-section of the magnetised specimen (a rectangular bar)


u

2 I = magnetic length of the specimen, m = strength of each pole of the specimen.


oou
d
ad

mx2l m
/ =
nY

From (49), <2x2/ a Hence, ...(50)


nid

Intensity of magnetistation of a magnetic material is also defined as the pole strength per unit
Re

area of cross-section of the material, placed in the magnetisingfield.


FFi

As ^ _ magnetic moment . Its SI unit is I =


Amp. metre^ = Am ’.
volume metre^
The SI unit of I are the same as SI units of H.

Intensity of magnetisation is a vector quantity. Its dimensional formula is [M T A].


5. Magnetic susceptibility (%„,)
It is a property which determines how easily a specimen can be magnetised when placed in the
magnetising field. Quantitatively,
Susceptibility of a magnetic materialis defined as the ratio of the intensity of magnetisation (I)
induced in the material to the magnetising force (H) applied on it. Magnetic susceptibility is
represented by
I
Thus Xm ...(51)
H
5/28 ‘pfuxtUe^ Fundamental Physics (XII) iyx»iwi
Since by definition, / is magnetic moment per unit volume, is usually called volume susceptibility of
the material.

As units of // and / are same (A m“^), therefore (x^„) has no units and no dimension. It is only a number.

rw
5.25. RELATION BETWEEN MAGNEHC PERMEABILITY AND MAGNEHC SUSCEPTIBILITY

When a magnetic material is placed in a magnetising field of magnetising intensity ff, the material gets
magnetised. The total magnetic induction B in the material is the sum of the magnetic induction Bq in vacuum
produced by the magnetising intensity and magnetic induction B^^, due to magnetisation of the material,
Fig. 5.32.
FIGURE $.32
Therefore, B=Bq+B ...(52)

r
m

wo
But,Bo=PoH andB^ = Po/, nmmmm Bq =

llou
F
where / is the intensity of magnetisation induced in the
magnetic material. Therefore, from (52)
...(53) n3^^fflSfflS£p B = Bo + B m

FF
s
I
Now as

ro H

ker
e
From (53), B =
11

For
But B = \iH .-. or = 1+X or
lir=^ + Xm ...(54)

ro
m

^0

fof
This is the relation between relative magnetic permeability and magnetic susceptibility of the material.

o
YY
ooY
Note. We know that the cgs unit of magnetic induction B is gauss which is 10^ tesla.
ou
The cgs unit of magnetising intensity H is the oersted.
B
ks
1 gauss 10-4 T
From (46), H -Bq/\Xq 1 oersted =
r

47txl0"’TA-^ m
eeB
u

1 oersted = 80 Am ^
rd

Table 5.1 gives a summary of important terms used in magnetism.


Yn
oou
ad

TABLE 5.1. Important Terms used in Magnetism


di

Physical Quantity Symbol Nature Dimensions Units Remarks


FF
Re

-*

1. Magnetic field induction, Magnetic [MT-2 T 1 T = 10“* gauss


in

B vector

flux density or Magnetic field strength


2. Magnetising intensity or
-*
vector [L-* /I] Am~^
H
Magnetising force
-1 M
3. Intensity of magnetisation / vector [Z,-' A] Am I =
V
4. Magnetic moment vector [i^A] Am^
M

5. Magnetic Permeability scalar A"^] TmA~●1

^*0 = 10"^
6. Magnetic Permeability of free space ^0 scalar [MLT-2 ^-2i TmA~^
4n

7. Relative magnetic scalar No No

permeability
8. Magnetic Susceptibility Xm scalar No No Rr = 1 + Xm
MAGNETISM AND MATTER 5/29

●Sam^o^fobjom't|J A bar magnet made of steel has a magnetic moment of 2*5 Am^ and a
mass of 6‘6 g. If density of steel is 7*9 x 1(P kg/m^, what is the Intensity of magnetisation ?
Sol. Here, M = 2-5 Am^, m = 6-6 g = 6-6 x 10“^ kg
p = 7-9 X 10^ kg/m^, 1 = 1
= _2-5x7-9xlCP = 2‘99 x 10® Am-*
V mjp m
6-6x10-3

w
I Samplia^Prbia'Xaiia ■ An iron rod of 0*2 cm^ cross-sectional area is subjected to a magnetising
field of 1200 Am“*. The suscaptibility of iron is 599. Find the permeability and the magnetic flux
produced.
Sol. Here, A = 0-2 cm^ = 0-2 x 10^

e
H= 1200 Am-*. = 599, p = ? (|) = ?

row
re
From
p = Po (1 + Xm) = 471 X lO-"^ (1 + 599) = 7*536 x 10^ Tm A"*
^ = BA = i]iH) A = 7-536 x lO"^ x 1200 x 0-2 x 10^ = 1*81 x 10"^ Wb

FFllo
F
5.26. CLASSIFICATION OF MAGNETIC MATERIALS

u
ree
There is a large variety of elements and compounds on earth. Some new elements, alloys and compounds
have been synthesized in the laboratory. Faraday classified these substances on the basis of their magnetic

sFr
properties, into the following three categories :

kro
(i) Diamagnetic substances,
uor
(«) Paramagnetic substances, and
(hi) Ferromagnetic substances.
Their main characteristics are discussed below :
offo
kos
YYo
(a) Diamagnetic Substances
eerBB
oo

The diamagnetic substances are those in which the individual atoms/molecules/ions do not
rY

possess any net magnetic moment on their own. When such substances are placed in an external
magnetising field, they get feebly magnetised in a direction opposite to the magnetising field.
uu

Examples of some diamagnetic materials are: Antimony, Bismuth, Copper, Lead, Gold, Silver, Diamond,
ad
doo

Zinc, Quartz ; Water, Alcohol, Mercury ; Air, Hydrogen, Nitrogen and dl inert gases like Helium, Neon,
nY

Argon, etc.
Some of the other important properties of diamagnetic substances are :
nid
Re

(/) When suspended in a uniform magnetic field, they


set their longest axis at right angles to the direction of the FIGURE 5.33
FFi

V
field, i.e., the shortest axis is along the direction of the field. y V

This is shown in Fig. 5.33.


(ii) When placed in a non-uniform magnetic field,
these substances have a tendency to move from stronger
parts of the field to the weaker parts. For example, consider N s
z
a diamagnetic liquid put in a watch glass placed on two pole
pieces of an electromagnet held close to each other
(« 2 mm or so). When the current is switched on, the liquid
level acquires the shape shown in Fig. 5.34 (a). The liquid FIGURE 5.34
accumulates on the sides where field is weaker. The
depression in the middle is due to stronger field at the centre.
If the distance between the poles is increased, the effect is N S N S
reversed, as in this case, the field is weaker at the centre.
This is shown in Fig. 5.34 (b). o o
5/30 '4- Fundamental Physics (XII)

(Hi) The level of a diamagnetic liquid in U tube is depressed, in FIGURE 5.35

the limb which is between the poles of a strong magnet as shown in


Fig. 5.35. It confirais movement of diamagnetics from stronger parts of
the field to weaker parts. This property implies that whereas a magnet
attracts metals like iron, it would repel a diamagnetic substance like
coppe.r
N S t
(iv) When a specimen of a diamagnetic material is placed in a
magnetising field, the magnetic field lines prefer not to pass through
I
the specimen, Fig. 5.36. It implies that the magnetic field lines are repelled
or expelled and the field inside the material is reduced. This reduction is
slight, being one part in 10^.
(v) Relative magnetic permeability of diamagnetic substances
is always less than unity.

ww
FIGURE 5.36
B
and B < Bn0 < 1
B

Floo
0

(w) From the relation = (1 + Xm)’ \^r < Xm negative.


N
Bo m S

ee
Hence susceptibility of diamagnetic substances has a small negative B < B0
value.

eer
rFrF
(v/j) Susceptibility of diamagnetics does not change with temperature. Bismuth at low temperature is
an exception to this general property.
rur
Values of Magnetic susceptibility of some diamagnetic substances at room temperature (« 300 K) are
given in Table 5.2.
s ffoor
TABLE 5.2. Magnetic susceptibility of some diamagnetic substances at 300 K
osk
YYoou
oook

S. No. Diamagnetic substance Xm


eBB

1, Copper -9-6x 10-^


2. Lead - 1-6 X 10-5
r

3. Diamond -2-2 X 10-5


ouur
ad

4. Gold - 3-6 X 10-5


Y

5. Water -91 X 10-^


6. Mercury - 3-2 X 10-5
nd dY
Re

7. Hydrogen -2-1 X 10-^


8. Nitrogen -50x 10-^
FFini

9. Bismuth -l-6x 10-^


10. Silver -2-6x 10-5
11. Silicon - 4-2 X 10-5

(v«0 Intensity of magnetisation (/) of a diamagnetic .substance is FIGURE 5.37


negative, small and directly proportional to the applied magnetising
—>

force {H) as shown in Fig. 5.37.


Explanation of Diamagnetism
We know that each electron in an atom is revolvingin an orbit around
the nucleus. The revolving electron is equivalent to a tiny loop of current.
Therefore, it possesses some orbital magnetic dipole moment
M,/ = current x area of the loop.
MAGNETISM AND MATTER 5/31

In addition to the orbital motion, every electron is assumed to have a spin motion around its axis.
Therefore, another dipole magnetic moment called spin magnetic moment is also associated with election.
—> —> —>
The vector sum of and provides the net magnetic dipole moment M to the atom, i.e.,

M =M^+M^
In a diamagnetic substance, M^ and cancel each other for every atom so that the atom has no net
magnetic dipole moment.
Therefore,motionof all the electronsin an atom of a diamagnetic material is assumed to be reduced to
motion of two electrons revolving with same angular velocity in a circular orbit ofsame radius, but in opposite sense.
The magnetic moment of the two being equal and opposite, cancel each other (in the absence of any

ooww
external magnetic field) so that net magnetic moment of each atom is zero. Figs. 5.38 and 5.39.
e“ FIGURE S.38
<■

e
ree
o.

rFl I 'n N.

Fre
V
I

rrF
e~
e“
ouur
M
sffoo
okks
YYo
Let a uniform external magnetic field of induction
oo

be applied perpendicular to the plane of rotation


of electron and directed away from the reader. Each electron experiences a magnetic Lorentz force F = Be v.
BB

According to Fleming’s left hand rule, the Lorentz force F acts radially outwards on the electron revolving
rre

anti-clockwise. It tends to decrease the centripetal force and hence decrease the velocity of electron to
(V- A t?). As a result, the magnetic moment of electron decreases to
ouu

(M - A M), Fig. 5.40. On the electron


Y
ad

revolving clockwise, the Lorentz force acts radially inwards, tending to increase the centripetal force and
dY

hOTce tlw velocity of electron to (i?+A i/). Therefore, the magnetic moment of electron increases to
(M+A Af), Rg. 5.41. The vector addition of these two magnetic moments gives rise to a net dipole magnetic
innd
Re

moment 2 A Af, directed towards the reader i.e. opposite to external field B .
FFi

RGURE 540
e~
4

o. (iJ-At?)
I '"x
X
I X
X
I

e“ e“

F
B

(M-AM)
5/32 “Ptcideefr 4^ Fundamental Physics (XII)

The net magnetic moment 2AAf so developed is called induced magnetic moment. The magnetic
moments induced in different atoms add vectorially to give a net magnetisation of the material in a direction

opposite to B . This accounts for the diamagnetic behaviour of materials.


Further, as the appearance of induced magnetic moment in atoms is not affected by the thermal motion
of the atoms, therefore, magnetic susceptibility of such materials does not depend on temperature of the
materials.

w
Retain in Memory
1. Superconductors are the most exotic diamagnetic
materials. These are metals cooled to very low temperatures,
which have zero resistance, i.e., perfect conductivity is

e
row
associated with perfect diamagnetism. The magnetic field

re
lines are completely expelled from super- conductors, as
shown in Fig. 5.42.

FFllo
F
B
= 0

u
B.0

ree
i.e., relative magnetic permeability of super-conductors is zero

sFr
As

kro
Xm — " 1
uor
offo
i.e., magnetic susceptibility of superconductors is minus one.
2. A superconductor repels a magnet and in turn, is repelled by the magnet.
kos
3. The phenomenon of perfect diamagnetism in superconductors is called Meissner effect.
YYo
Superconducting magnets have been used for running magnetically leviated superfast trains.
eerBB
oo

(b) Paramagnetic substances


rY

Paramagnetic substances are those in which each individual atomJmolecule/ion has some net
uu

non zero magnetic moment of its own. When such substances are placed in an external
magnetising field, they get feebly magnetised in the direction of the magnetising field.
ad
doo
nY

Examples of some of the paramagnetic substances are Aluminium,Chromium, Manganese, Lithium,


Magnesium, Platinum, Tungsten, Sodium, Potasium, Copper chloride, liquid Oxygen, etc.
nid
Re

Some of the other important properties of paramagnetic substances are :


(/) When suspended in a uniform magnetic field, they
FFi

FIGURE 5.43
rotate so as to bring their longest axis along the direction
of the field. This is shown in Fig. 5.43.
(ii) When placed in a non-uniform magnetic field,
they tend to move from weaker parts of the field to the
stronger parts. It implies that paramagnetic substances get N S
weakly attracted to a magnet. For example, consider a
paramagnetic powder or Liquid put in a watch glass placed
on two pole pieces of an electromagnet held close to each
other (® 2 mm or so). As the current is switched on, the liquid FIGURE 5.44

level acquires a shape shown in Fig. 5.44(a), accumulating


at the centre, where the field is the strongest. If the distance
N S
between the poles is increased, the effect is reversed, as in N S

that case, the field is weaker at the centre. This is shown in e ©


Fig. 5.44(6).
MAGNETISM AND MATTER 5/33

(in) When a magnetic field is applied to the level of a paramagnetic liquid in one limb of a U-tube,
the liquid level rises as shown in Fig. 5.45. It confirms that paramagnetic substances move from weaker to
stronger parts of the magnetic field.

w
FIGURE 5.45

FIGURE 5.46

N S
1

wr
e
t

r
oo
F
llu
roFF
ees
(iv) When a specimen of a paramagnetic substance is placed in a magnetising field, the magnetic

rr
field lines prefer to pass through the specimen rather than through air, as shown in Fig. 5.46. Thus

Fk
magnetic induction B inside the sample is more than the magnetic induction Bq outside the sample, />.,

ro
B>Bq This enhancement is slight, being one part in 10^.

ofo
(v) Relative magnetic permeability of paramagnetic substances is always more than unity.
YY
B
sof
ou
As „ and B>B.0 p,>l.
oY
BrB
ok
It means paramagnetic substances have a tendency to pull in magnetic field lines when placed in a
magnetising field,
(vi) From the SI relation, =1+ as > 1, therefore, Xm inusi be positive. Hence, susceptibility of
ruee

paramagnetic substances is positive, though small.


ooud

Values of magnetic susceptibility of some paramagnetic substances at room temperature (= 300 K) are
given in Table 5.3.
ad
Yn

TABLE 5.3.
Magnetic susceptibility of some paramagnetic substances at 300 K
di

S. No. Paramagnetic substances Xm


Re
FF
in

1. Aluminium 2-3 X 10-5


2. Calcium 1-9 X 10-5
3. Chromium 2-7 X 10-^
4. Lithium 21 X 10-5
5. Magnesium 1-2 X 10-5
6. Niobium 2-6 X 10-5
7. Oxygen (STP) 2-1 X 10-^
8. Platinum 2-9 X 10-^
9. Tungsten 6-8 X 10-5

(vii) Susceptibility of paramagnetic substances varies inversely as the temperature of the substance.
1
i-e., X m OC
— ,/.e.,they lose their magnetic character with rise in temperature.

The graph between temperature T and (l/Xm) is a straight line as shown in Fig. 5.47.
5/34 "Pn^ieUe^ Fundamental Physics (XII)

(vUi) The intensity of magnetisation (/) of a paramagnetic substance is positive, small and directly

w
proportional to the applied magnetising force (H) as shown in Fig. 5.48.
Explanation of paramagnetism
In paramagnetic materials, every atom has some permanent magnetic dipole moment. In the absence of

Flo
an external magnetic field, the atomic dipoles are randomly oriented so that the average magnetic moment
per unit volume of the material is zero. Therefore, a paramagnetic material does not behave as a magnet in the

reeee
absence of an external magnetic field, Fig. 5.49.

FFr
FIGURE 5.49 FIGURE 5.50

for
ur
H> «=0
kkss
s
Yo
oo

H> F=i> =>


eB

H=0 H
r

When an external magnetic field is applied, it tries to align the atomic magnetic dipoles in the direction
ou
ad

of the field. Fig. 5.50. That is why the specimen gets magnetised weakly in the direction of the field. This is
YY

paramagnetism.
When we raise the temperature of the material, the atomic magnetic dipoles acquire some kinetic energy.
ndd
Re

This tends to disorient the dipoles. That is why magnetic susceptibility of the paramagnetic materials decreases
with irse in temperature.
Fi

(c) Ferromagnetic substances

Ferromagnetic substances are those in which each individual atom/molecule/ion has a non
zero magnetic moment, as in a paramagnetic substance.
When such substances are placed in an external magnetising field, they get strongly magnetised
in the direction of the field.

Examples of ferromagnetic materials are Iron, Cobalt, Nickel, Gadolinium and a number of their
alloys.
The ferromagnetic materials show all the properties of paramagnetic substances, but to a much
greater degree. For example,
(0 ThQ' are stron^y magnetised in the direction oK external magnetising fidd in whidi they are placed.
(//) They have a strong tendency to move from a region of weak magnetic field to the region of strong
magnetic field, i.e., they get strongly attracted to a magnet.
MAGNETISM AND MATTER 5/35

{Hi) Relative magnetic permeability of ferromagnetic materials is very large (* 10^ to 10^)
(tv) The susceptibility of ferromagnetic materials is also very large.
That is why they can be magnetised easily and strongly,
(v) With rise in temperature, susceptibility of ferromagnetics
decreases. At a certain temperature, ferromagnetics change over to
paramagnelics. This transition temperature is called Curie

w
temperature. For example, curie temperature of iron is about 1000 K.
—>

The intensity of magnetisation ( /) of a ferromagnetic substance is


positive, large and varies non linearly with the applied magnetising force H max

e
(//) as shown in Fig. 5.51.

e
o
Explanation of Ferromagnetism

rw
r
Ferromagnetism has been explained by Weiss on the basis of domain theory in addition to the

eF
usual electron theory.

ullo
Like paramagnetic substances, each atom of a ferromagnetic substance is a tiny magnetic dipole having

FF
permanant dipole moment. However, in ferromagnetic materials, atoms fonn a very large number of small
effective regions called domains. Each domain has a linear dimension » 1000 A° and contains about 10^®

srre
atoms. Within each domain, a special interaction called exchange coupling renders dipole moments of all the

oF
atoms in a particular direction. Thus each domain is a strong magnet without any external magnetic field.

k
Inspite of this, a ferromagnetic substance does not behave as a magnet, because in the absence of external

fofr
uor
magnetic field, the magnetic moments of different domains are randomly oriented so that their resultant
magnetic moment in any direction is zero, Fig. 5.52. o
Y
When an external magnetising field is applied on the ferromagnetic substance, it gets strongly magnetised.
kos
Yo
rBB

This can be understood in terms of


oo

(/) displacement ofboundaries of the domains i.e. domains which are oriented in the direction of applied
eY

field increase in size and the domains which are oriented opposite to the field decrease in size. Fig. 5.53.
ure

FIGURE 5.52 FIGURE 5.53 FIGURE 5.54


Yd
ou

Z
o

X
ad

S
S
¥ z
id n

V /*
7f SIZ
zrs
Re

z s:
in
FF

z
z
z
z

(//) rotation of the domains i.e. the domains rotate till their magnetic moments are aligned in the direction
of the applied magnetic field. Fig. 5.54. This would happen only when the magnetic field applied is very
strong. FIGURE 5.55

Fig. 5.55(a) shows the state when the domains have partially
aligned along B and amalgamated to form a single giant domain.
This is how ferromagnetic material gets strongly magnetised in -►

the direction of the applied field. In contrast. Fig. 5.55(b) shows


B B = 0
the state when B = 0 .
o o
5/36 Fundamental Physics (XII)QSm

When external magnetic field is removed, magnetization disappears in some of the materials like soft
iron. These materials are called soft ferromagnets. However, in materials like Alnico (an alloy of iron,
aluminium, nickel, cobalt and copper), the magnetization persists even after removal of external magnetic
field. Such materials are called hard ferromagnets.
Further, when a ferromagnetic material is heated, its magnetization decreases gradually with rise in
temperature. At a particular temperature, ferromagnetic material changes over to paramagnetic material. This
is because the domain structure disintegrates with rise in temperature. The temperature of transition from
ferromagnetism to paramagnetism is called the Curie temperature. It is different for different ferromagnetic
materials as shown in Table 5.4.

TABLE 5.4. Curie temperature of some ferromagnetic materials

Material Curie Temperature

w
Cobalt 1394 K

Flo
Iron 1043 K

Fc203 893 K

ee
Nickel 631 K

Fr
Gadolinium 317 K

Table 5.5 displays the values of susceptibility (x„), relative magnetic permeability (p^) and absolute
for
ur
magnetic permeability (p) of the three types of magnetic substances,
ks
TABLE 5.5. p^ and p of magnetic substances
Yo
oo

Substance Xm
B

1. Diamagnetic 0 S Pr < 1 ^<^■0


re

2. Paramagnetic 1 <P^<(1 +€) P>P(J


3. Ferromagnetic Xm>>! P,>> 1
ou
ad
Y

5.27. CURIE LAW IN MAGNETISM


nd
Re

According to Curie law,


Fi

Intensity of magnetisation (I) of a magnetic material is (0 directly proportional to magnetic


field induction (B), and (ii) inversely proportional to the temperature (T) of the material.

I
i.e., I oc B, and I oe —
T

B
Combining these factors, we get / oc —
T

As B Of H, magnetising intensity
H I 1
/« — or OC

T H T

*Here € is a small positive number


MAGNETISM AND MATTER 5/37

I 1 C
But — = Xm Xm oc or Xm ...(56)
H T T

where C is a constant of proportionality and is called Curie constant.


Thus for a paramagnetic material, both Xm depend not only
on the material, but also on the sample temperature.
The law is physically reasonable as increasing H tends to align the
elementary dipoles in the specimen increasing /, whereas increasing T
tends to interfere with this alignment, decreasing I.
The variation of / with H/T is shown in Fig. 5.56.
At very high fields or at very low temperatures, the magnetisation
approaches its maximum value, when all atomic dipole moments are
aligned. This is called the saturation magnetisation value, as is DO YOU KNOW ?

ww
indicated by the portion of graph parallel to X-axis. The susceptibility of ferro
Note that magnetic thermometers are based on Curie law. These magnetics, above the Curie
thermometers are used to measure very low temperatures (< 1 K) by temperature i.e. in the para

Floo
measuring the susceptibility of a paramagnetic substance. magnetic phase is given by
Curie’s law is well verified experimentally provided the ratio

ee
C
H/T is not too large. Xm

eer
T-T

rFrF
The variation of l/Xm with temperature (7^ for a paramagnetic
material is a straight line as shown already in Fig. 5.47. where is Curie temperature and
sample temperature T > T^. This
rur
Sami;^ .Problem ^ Effective dipole moment of a paramag
netic salt is 4*5 joule/tesla at 4*2 K under magnetic field of 0 8 T.
Find magnetic moment at 2*8 K, when magnetic field is 1T. Assume
s ffoor
is the form Curie law assumes for
ferromagnetic materials above the
Curie temperature.
osk
YYoou
Curie’s Law.

Mi = 4-5 J/T, Ti = 4-2 K, = 0-8 T


oook

Sol. Here,
M2 = ? Tz = 2-8 K,
eBB

According to Curie’s Law


M2 M
r

1
xr.1
XT2 -
ouur
ad

B
l
Y

M. T 4-5x4-2xl-0
1
M2 = x-J-xB,
T 2 = = 8-437 JT
-1
nd dY

B 0-8 X 2-8
1
Re

5.28. HYSTERISIS CURVE


FFini

The hysterisis curve represents the relation between


—» —*

magnetic field induction B (or intensity of magnetization / )


of a ferromagnetic material with magnetizing force or
—>

magnetic intensity H ● The shape of the hysterisis curve is


shown in Fig. 5.57. It represents ^e behaviour of the material
as it is taken through a cycle of magnetization.
Suppose the material is unmagnetised initially, i.e.,
5=0 and H = 0 . This state is represented by the origin
O. We place the material in a solenoid and increase the
current through the solenoid gradually. The magnetising force
H increases. The magnetic field induction B in the
5/38
"Ptadee^ 4 Fundamental Physics (XII) W»TI
material increases and saturates as depicted in the curve oa. This behaviour represents alignment and merger
of the domains of ferromagnetic material until no further enhancement in 5 is possible. Therefore, there is
no use of increasing solenoid current and hence magnetising intensity H beyond this. The point a represents
the saturation point.

Next, we decrease the solenoid current and hence magnetising intensity H till it reduces to zero. The
curve follows the path ab showing that when H =0, B ^0. Thus, some magnetism is left in the specimen.

The value of magnetic induction B left in the specimen when the magnetising force is reduced
to zero is called Retentity or Remanence or Residual magnetism of the material.
It shows that the domains are not completely randomised even when the magnetising force is removed.

ww
Next, the current in the solenoid is reversed and increased slowly. Certain domains are flipped until the
net magnetic field induction B inside is reduced to zero. This is represented by the curve be. It means to
reduce the residual magnetism or retentivity to zero, we have to apply a magnetisingforce = OC in opposite

Flo
direction. This value of magnetising force is called coercivity of the material

ee
As the reverse current in solenoid is increased in magnitude, we once again obtain.saturation in the

rere
reverse direction at d. The variation is represented by the curve cd. Next, the solenoid current is reduced

r FF
(curve de), reversed and increased (curve ea). The cycle repeats itself. From Fig. 5.57, we find that saturated
magnetic induction B^ is of the order of 1 -5 T and coercivity is of the order of - 90 Am"'.
uurr
foor
From the above discussion, it is clear that when a specimen of a magnetic material is taken through a
cycle of magnetisation, the intensity of magnetisation (I) and magnetic ifeld induction (B) lag behind the
magnetising force (H). Thus, even if the magnetising force H is made zero, the values of I and B do not
ks s
reduce to zero ie.. the specimen tends to retain the magnetic properties.
Yoo
ooook

This phenomenon of lagging of I or B behind H when a specimen of a magnetic material is


eBB

subjected to a cycle of magnetisation is called hysteresis. The curve abedea is called hysteresis
loop.
rr

From Fig. 5.57, we make the following observations :


ouu
ad

(/) For a given value of H ; value of B or I is not unique, but depends on previous history of the sample.
YY

(i7) No segment O a, ab, etc. of the hysterisis curve is linear. Therefore, B or I is not proportional to H
over any appreciable range.
nndd
Re

Retain in Memory
Fii

1. We can plot I-H loop in the same way as we have plotted B~H loop of the ferromagnetic material.
The shapes of I-H loop and B-H loop for a given material are identical.
B B-
2. Note that BH = B has the dimensions of eneigy per unit volume. That is why area

within the BH loop represents energy dissipated per unit volume in the material per cycle of
magnetisation.

5.29. ENERGY DISSIPATION DUE TO HYSTERESIS


When a magnetic material is taken through a cycle of magnetisation, some energy is spent in the process.
The energy cannot be recovered and is dissipated in the form of heat in the specimen. Infact, when a specimen
is taken through a cycle of magnetisation, the molecular magnets (magnetic dipoles) in the specimen are
oriented and reoriented time and again. This molecular motion within the sp>ecimen results in the production
MAGNETISM AND MATTER 5/39

of heal. It can be shown that the loss of energy per unit volume of specimen per cvde of magnetisation in cgs
system is equal to the area of the I-H loop of the specimen and in SI, it is equal to the area of the B-H loop.
Tlae exact shape and size of the 1-H and B-H loops depend upon the nature of the material and history of the
specimen. By studying the hysteresis loops of various magnetic materials, we can study the difference in their
properties e.g., retentivity, coercivity, permeability, susceptibility and energy loss, etc. Such studies enable us
to select suitable materials for different purposes e.g. for electromagnets, for transformer cores, for permanent
magnets etc.
For example, hysteresis loop for soft iron is narrow and large,
whereas the hysteresis loop for steel is wide and short, [Fig. 5.58].
The hysterisis loops of soft iron and steel reveal that
(j) The refentivity of soft iron is greater than the retentivity of steel,
{ii) Soft iron is more strongly magnetised than steel,
(Hi) Coercivity of soft iron is less than coercivity of steel. It means

ww
soft iron loses its magnetism more rapidly than steel does,
(iv) As area of I-H loop for soft iron is smaller than the area of
I-H loop for steel, therefore, hysterisis loss in case ofsoft iron is smaller

Floo
than the hysterisis loss in case of steel.
of 12 kg is 300 J m“^ cycle”*. If density of

ee
^Sa'mpjetRlioiSrerng The hysterisis loss for a sample
iron is 7500 kg/m^, calculate energy loss per hour at 50 cydes/sec-

eer
rFrF
Sol. Here, energy loss/volume, per cycle = 300 Jm”^ cycle"',
mass 12 12 _ 12
m^ ; Total energy loss/cycle -300x 7500 ’ 25
rur
Volume of sample -
density
12
7500

Energy loss/sec = — x 50 = 24 J/sec ;


ffoor
Energy loss/hour = 24 x 60 x 60 J = 8*64 x lO'* J
s
osk
25
YYoou
oook

530. USES OF FERROMAGNETICS


eBB

Important properties of ferromagnetic materials are studied from their hysteresis curves. These properties
help us in the proper selection of materials for various practical applications as detailed below :
r

(a) Permanent Magnets


DO YOU KNOW ?
ouur
ad

Permanent magnets are the materials which retain at room


temperature, their ferromagnetic properties for a long time. The material That ferromagnetic materials
Y

chosen should have have been divided into two types ;


1. Soft magnetic materials,
nd dY

(/) high retentivity so that the magnet is strong,


Re

which have low retentivity, low


(//) high coercivity so that the magnetisation is not erased by stray
magnetic fields, temperature changes or mechanical damage due coercivity and low hysterisis loss.
FFini

Soft iron, mp. metal and stalloy


to rough handling etc.
are some of the examples of these
(m) high permeability so that it can be magnetised easily. materials, used primarily for
Steel is preferred for making permanent magnets. Though reten- electromagnets, cores of trans
tiviiy of steel is slightly smaller than that of soft iron, yet its coercivity formers, motors and generators.
is much larger than that of soft iron. Other suitable materials for 2. Hard magnetic materials,
permanent magnets are (/) cobalt steel, (52% iron, 36% cobalt, which have high retentivity, high
7% tungsten, 4-0% chromium, 0-5% manganese and 0-5% carbon) coercivity and large hysterisis
(//) carbon steel (98% iron ; 0-86% carbon and 0-9% manganese), loss. Steel, alnico, alcomax and
(m) the alloy alnico (55% iron, 10% aluminium, 17% nickel, 12% ticonal are some of the examples
cobalt and 6% copper), (/v) Ticonal containing tin, cobalt, nickel and of these materials, which have
aluminium.
been used for making permanent
An efficient way to make a permanent magnet is to place a magnets of electric metres and
ferromagnetic rod in a solenoid and pass current through the solenoid. loud speakers, etc.
The magnetic field of the solenoid magnetises the rod.
5/40 ^ Fundamental Physics (XII) IWl
(b) Electromagnets
The core of electromagnets is made of ferromagnetic materials, FIGURE 5.59 Solenoid
which have high permeability, low coercivity and low retentivity. Soft
iron is a suitable material for this purpose. When a soft iron rod is placed ]—Soft
in a solenoid and current is passed through the solenoid as shown in iron

Fig. 5.59, magnetism of the solenoid is increased by a thousand fold.


When the solenoid current is switched off, the magnetism is removed
instantly as coercivity of soft iron is very low. Electromagnets are used in
electric bells, loudspeakers and telephone diaphragms. Giant
electromagnets are used in cranes to lift machinery, etc.
Factors affecting the strength of an electromagnet
DO YOU KNOW ?
As the strength of magnetic field around a current carrying wire is
proportional to the current in the wire, therefore, the strength of the field exciting application of

ww
of an electromagnet is also proportional to current flowing through the superconducting electromagnets
coil. The magnetic field produced by each loop of the coil is same as that in making a magnet trans
produced by any other loop. As these fields are in the same direction, portation train which can move

FF loo
therefore, increasing the number of loops in an electromagnet increases with high speed. While moving,
the strength of the magnetic field. this train floats several inches

ee
Further, the strength of an electromagnet can be increased by placing above the rail lines and its speed

Fr
an iron rod or core inside the coil. The field inside the coil magnetizes is limited only by air friction and
the core by induction. The magnetic strength of the core adds to that of passanger comfort.
ur
the coil to produce a much stronger electromagnet.

rre
fofr oF
Hence, we conclude that the factors affecting the strength of electromagnets are : (0 Number of loops in
the coil, (ii) Strength of current passed, (Hi) Nature of the core of electromagnet, (tv) Temperature. As
ks
magnetism is lost at high temperature, therefore, to retain magnetism is a given material, it must be maintained
YYouro
soo

at low temperature,
(c) Transformercores
BBook

The materials used for making transformer cores, chokes, diaphragms of telephones etc. are subjected
r ee

to ac cycle of magnetisation for a long period. Therefore, low hysterisis loss is the first essential conditionfor
such materials. The hysterisis curve of such materials must be narrow. The energy dissipated and therefore,
oouru
ad

heating of the material will be small. The material chosen must also have high resistivity to decrease energy
losses due to eddy currents. Soft iron is more suitable for transformer cores. More effective alloys have also
Y

been developed for this purpose. Some of them are perm alloys, mu metal (76% nickel, 17% iron, 5% copper
and 2% chromium) and radio metal, etc.
Ynd
Re
FFindi

Curiosity Question
f Q. What are some of the Important technological applications of magnetism ?
Ans. The actual list of technological applications of magnetism is really long. Some of these are :
(0 Magnets are used In meters, motors, electric generators, loud speakers etc.
(//) Large electromagnets are used to pick up heavy loads.
(Ill) Large electromagnets are being used to leviate modern trains. The so-called maglev trains
are faster and provide a much smoother ride than the ordinary track system, due to absence of
friction between the train and the track,
(Iv) Magnetic tapes are used in sound and video recording equipment.
{)/) Magnetic recording material is used on computer discs.
(vl) Superconducting magnets are used to contain the plasmas used in controlled nuclear fusion
research.
J
MAGNETISM AND MATTER 5/41

I I 2jc2 = (1-a:)2 ^x = \-x


TYPE I. MAGNETIC DIPOLE MOMENT or

AND FORCE BETWEEN MAGNETIC POLES


1
= 0-414 m
Formulae used.
1. Magnetic (dipole) moment M = m{2l) ExarnpiaiQ Two identical thin bar
2. Force between two magnetic poles in air, magnets, each of length L and pole strength m are
placed at right angles to each other, with the N
4jt pole of one touching the S-pole of the other. Find
Units used. M is in JT
-I
or A m^, distance (r), the magnetic moment of the system.

ww
Solution. As is clear from Fig. 5.61, the system
magnetic length (2 1) in metre ; m^, m2 are pole
strengths in Am.
behaves as a magnet whose pole strength is m and
length is A/2'^i
^ = 10"^ Wb A“‘ -1

Flo
Standard Value. m FIGURE 5.61
4tu

e
T N2 «

ree
N

Exampiefl \ magnetised steel wire

Fr
rF
31.4 cm long has a pole strength of 0-2 Am. It is L
uurr
bent in the form of a semicircle. Calculate its

for
\

magnetic moment.
90'’
Solution. Here, / = 31-4 cm, m - 0-2 Am, s S2
A/ = ?
kks
Let r be radius of semicircle.
Yo
Ni Si
oooo

L
, / 31-4 = 10 cm = 0-1 m
l = nr, r = — =
eB

71 3-14
Now,
M = m X 2 r = 0-2 X 2 X 0-1 = 0*04 Am^
= 4l?nF =
r

Exampio@ Two Similar magnetic poles,


ou
ad

Magnetic moment of the system


YY

having pole strengths in the ratio 1: 2 are placed


M = pole strength x N2S\
1 m apart. Find the point where a unit pole
experiences no net force due to the two poles. M =m.L^ = ^mL
nndd
Re

Solution. Let mj = w and m2 =* 2 m, r = 1 m. Exa rnpi«E! Calculate the force acting


Let unit pole placed at P, at a distance x from
Fi

between two magnets of length 15 cm each and


W] experience no net force, Fig. 5.60. pole strength 80 Am each when the separation
FIGURE 5.60 between their north poles is 10 cm and that
P m2
between south poles is 40 cm.
+
Solution. The given situation is shown in Fig.
\4- X->t< {^-x) >l 5.62.
1 m ♦I FIGURE 5.62

40 cm

^0 ^ nr^xl Si Ni N2 S2
4ti 4it I (l-x)2
►W ►H
15 cm 10 cm 15 cm
1
ai-
(1 - 2 Here, mj = m2 = 80 Am
5/42
4- Fundamental Physics (XII) LWI
Force of attraction between Nx and
Exampte § The e» maonetic field at
80x80 the equator is approximately 0*4 G. Estimate the
' 47t earth’s dipole moment
(0-25)2 (NCERT Solved Example)
= 0-01024 N Solution. If we assume that earth’s magnetic
Force of attraction between N2 and 5| field is due to a bar magnet at the centre of earth,
m, m
held along the polar axis of earth, then the equatorial
1
F
r2 - =F, = 0-01024 N

w
4ti magnetic field is B =
4ti
Force of repulsion between and N2
where d-R = radius of earth = 6-4x10^ m
f^3 “-hi 80x80
m, m
1
^ = 10-’x Using 5 = 0-4 G = 0-4 x 10"^ T,
= 0-064 N
4tc r2 (0-1)2

e
roow
and = 10"2Wb A"' -1

re
m
, we get
Force of repulsion between and S2 4k

F,=^^
^ 4tc r2 = 10-’x (0-40)2
80x80 M
0-4x10"^ = 10"’

F
= 0-004 N

uFFll
(6-4x10^)3

ree
Resultant force between the two magnets
M =
04x10^(6-4x10^)^
F=F,:^F2-F,-F^

sFr
10-’
= 0-01024 + 0-01024 - 0-064 - 0-004 = 1*05 X 1023 Am2

kro
= -0-04752 N
uor
F = - 0*048 N (minus sign for repulsion). offo Example 0 What is the magnitude of the
equatorial and axial fields due to a bar magnet of

I
kos
TYPE II. MAGNETIC FIELD length 5*0 cm at a distance of 50 cm from its mid
Y
Yo
STRENGTH DUE TO A BAR MAGNET point ? The magnetic moment of the bar magnet
eerBB
oo

is 0*40 Am2. (NCERT Solved Example)


Formulae used. 1. Magnetic field strength at a
urY

point on axial line of a bar magnet. Solution. Here, B2 = ?, ^1= ?, 2 / = 5-0 cm,
IMd
/ = 2-5 cm = 2-5 x 10“2 m.
B.I =
_^o
i/ = 50 cm = 0-5 m, Af = 0-40 Am2
ou

47C(d2_/2)2
ad
do

B - ^ ^
nY

Hq 2M 0:l«d)
When the magnet is short, -P« d^, Sj = 4k d^ 2 4jt (i/2+/2j3/2 4k d^
nid
Re

040
2. Magnetic field strength at a point on equatorial = I0~2 X = 3*2 X10"^ T
F

line of a bar magnet. (0-5)2


Fi

_Ho W _^o 2Md _ lip 2M


B,= B.I = (v l«d)
4tI(^/2+/2)3/2 4jt(f/2_/2)2 “47c ^3
2x0-40
If the magnet is short, P « cP B.I =10-2 X = 6*4xl0"^T
4ti
(0-5)3
B
3. For a short bar magnet, —L = 2 or B^^ = 2 82- Example B Two short magnets P and Q
^2
Units used. B,, So are in tesla ; d, / in metre and M
are placed one over another with their magnetic
in Am2. axes mutually perpendicular to eachother. It is
Standard Value. found that resultant magnetic field at a point on
the prolongation of magnetic axis of P is inclined
^^0 -1 at 30® with this axis. Compare the magnetic
m
or lO-^TmA-*.
4k moments of the two magnets.
MAGNETISM AND MATTER 5/43

Solution. In Fig. 5.63, at any point A, distant r Solution. Here, /V = 50, / = 10 A,


from the centre of two magnets. 10
r- — cm = 5x10-2 m
2M
^0 1 2
magnetic field due to P, =
4rt r3 M = 2 From M-NIA = NI{ni^)
A/ = 50x lOx 3-14(5 X 10-2)2
magnetic field due to Q, $2 - M = 3-925 Am2
4tc
According to irght hand thumb rule, direction
FIGURE —>

of M is perpendicular to the plane of loop and away


Q from observer.
S2

TYPE IV. TORQUE ACTING ON A


MAGNETIC DIPOLE AND POTENTIAL

w
P ♦ Ml
N1
ENERGY OF DIPOLE IN A MAGNETIC FIELD
S1
r

N2 Formulae used. 1. Torque, x = MB sin 6

Flo
M2
2. Potential energy, (/ = - MB (cos 02 - cos 6^)
As resultant field is inclined at 30° with Bj, Units used. B in tesla, M in or Am^, x in

ee
therefore. N-m, U in joule ; 0p 02 in degrees.

Fr
B, _ M2 1
tan p - = tan 30° =
Exsmple m A short bar magnet placed
B
1
2M
1 V3
with its axis at 30° to a uniform magnetic field of

for
ur
0-2 T experiences a torque of 0-06 N-m. Calculate
2 magnetic moment of the magnet What orientation
ks
of magnet corresponds to its stable equilibrium in
Yo
the magnetic field ? (CBSE 20021
oo

TYPE III. MAGNETIC MOMENT


OF A CURRENT LOOP Solution. Here, 0 = 30°, S = 0-2 T,
B

X = 0-06 N-m, M = ?
re

Formulae used.
From, X = MB sin 0,
Magnetic moment of a current loop, M = NIA
ou
ad

Units used. N is a number with no units, I in X 006


M = - 0-6 Am^
ampere, A in m^, M in Am^. 0-2 sin 30°
Y

Bsin 0

Example Q An electron in an atom Potential energy of magnetic dipole


nd
Re

t/ = - MB cos 0
revolves around the nucleus in an orbit of radius
In stable equilibrium, P.E. is minimum .
Fi

0*5 A. Calculate the equivalent magnetic moment


if the frequency of revolution of electron is cos 0 = 1 or 0 = 0°
10'® MHz. (CBSE 2000 (C)l
i.e., bar magnet will be in stable equilibrium.
Solution. Revolving electron is equivalent to a ->

current loop. Its magnetic moment is when its magnetic moment M is parallel to
M = NIA - 1 ■ (ev) 7U
magneticfield B.
= (1-6 X 10-'^ X 10*^) 3-14 (0-5 X 10“'^
= 1-256 X 10-23
Example m An electron moves around the
nucleus in a hydrogen atom of radius 0-51 A with
Example A loop of wire having 50 turns a velocity of 2 x 10^ m/s. Calculate (i) the equi
carries a current of 10 A in anticlockwise direction. valent current due to orbital motion. (11) magnetic
If diameter of loop is 10 cm, what is the magnitude field produced at the centre of nucleus. (Hi) mag-
and direction of magnetic moment of current netic moment associated with electron.
loop ? (CBSE 2008)
5/44 Fundamental Physics rxinPTSWl
Solution. Here, r = 0-51 A = 0-51 x 10"^® m, («)lV=?e2=180°
D = 2 X 10^ m/s (opposite to magnetic field)
e ev
W-~ MB (cos 02 - cos 0|)
(/)/ = - = - 6 X 0-44 (cos 180® - cos 60®)
T Inr/v 2nr
= - 2-64 (- 1 - 0-5) = 3-96 J
1-6x10-^5x2x10^ (/))t=: 70 = 02= 180®
= 10-3 A T = Mfi sin 0 = 6 X 0-44 sin 180® = Zero
2x3-14x0-51x10-^0
A circular coil of 100 turns

(»■) B =
Hq/ 47txlO-2xlQ-3 = 12*3 T and having an effective radius of 5 cm carries a
2r " 2x0-51x10-^0 current of O'l ampere. How much work is required
to turn it in an external magnetic field of 1*5
(«0 A/ = /A =
ev
XTcr^ weber/m^ through 180® about an axis

ww
27ir perpendicular to magnetic field. The plane of the
= 10-3x3-14 (0-51 X 10-^0)2 coil is initially perpendicular to the magnetic field.
Solution. Here, A'^=100, r =5 cm = 0-05 m,
= 8*16 X 10-24 Am2

Floo
7 = 0-1 amp., W=7, 1-5 Wb/m^,
ExamplejQ A short bar magnet of 0^ =0°, 02= 180®

ee
magnetic moment 0*9 JT"^ is placed with its axis Now, area A = 7i r^ = 3-14 (0-05)^ sq m

reer
at 30® to a uniform magnetic field. It experiences

rFF
W = ~MB (cos 02“ cos 0|)
a torque of 0*063 J. Calculate the magnitude of = - (AY A) B (cos 02 - cos 0|)
magnetic field. In which orientation will the bar
uur r
= - 100 X 0-1 X 3-14 (0-05)2
magnet be in stable equilibrium in the magnetic
field ? (CBSE 2012)
ffoor X 1-5 [cos 180® - cos 0°]
sks
= -31-4 (0-05 )2x1-5 (-1-1)
Solution. Here, M = 0-9 JITK 0 = 30®,
YYoo
= 0*24 J
ooko

x = 0-063 J, B = ?
Example m A current of 5 A is flowing
eBB

0-063
X = MB sin 0, 5 = —^— through a 10 turns circular coil of radius 7 cm.
Msin0 0-9 X sin 30® The coil lies in x-y plane. What is the magnitude
r

= 0*14 T and direction of dipole moment associated with it ?


ouur
ad

For stable equilibrium, the magnetic moment If this coll were placed in a uniform external
magnetic field directed along the j:-axis, in which
YY

M of magnet must be parallel to magnetic field B. plane would the coil lie in equilibrium ? 7t = 22/7.
(CBSE Sample Paper 2003)
nndd

A bar magnet of magnetic


Re

moment 6 J/T Is aligned at 60® with a uniform Solution. Here, I = 5A, = 10, r = 7 x 10"^ m
FFii

external magnetic field of 0*44 T. Calculate M-NIA = NI (n r^)


(a) the work done in turning the magnet to align 22
its magnetic moment (/) normal to the magnetic
field (//) opposite to the magnetic field ; and
= 10x5xy (7x10-2)2 =0*77 Am^
—>

(A) the torque on the magnet in the final orientation The direction of M is perpendicular to the
in case (i7). (CBSE 2018) plane of the coil, along Z-axls.
Solution. Here, Af = 6 J/T, 0j = 60®, Torque on the current loop, x = MB sin 0
B = 0-44 T
where 0 is angle between A7 and B .
(a) (0 W=?02 = 9O®
For stable equilibrium, x = 0 0 = 0®.
(normal to magnetic field)
As W = - MB (cos 02 - cos 0j) Hence B should be perpendicular to plane of
W = - 6 X 0-44 (cos 90® - cos 60®) the coil. Hence in the condition of stable
= -2-64 (0 - 0-5) = 1*32 J equilibrium, coil will be in F-Z plane.
MAGNETISM AND MATTER 5/45

.Example. m A current of 7‘0 A is flowing Solution. Here, M = 60 Am^


in a plane circular coil of radius 1*0 cm having // = 40 |X Wb/m2 = 40 X 10"® Wb/m^
100 turns. The coil is placed in a uniform T = 1-2 X 10“^ Nm. Declination, 9 = ?
magnetic field of 0*2 Wb/m^. If the coil is free to As is known, in stable equilibrium, a compass

rw
rotate, what orientation would correspond to its needle points along magnetic N-S and experiences
(0 stable equilibrium and (u) unstable equilibrium ? no torque. But when compass needle is turned through
Calculate potential energy of the coil in the two angle of declination (9), it points towards geographic
cases. (CBSE 2000) N-S and experiences torque,
T = MB sin 9
Solution.Here,/ = 7-0 A, r- 1-0cm= lO'^m,

e
N = 100, B = 0-2 Wb/m^. sm 9 = —
X 1-2x10~3 _ 1

r
M = NIA =NInr^ MB 60x40x10"^ 2

o
llou
22

F
= 100x7-0x— (10-2)2 =o.22Am2 lExampie.fEI jhe value of dip at a place is

w
7

(0 Stable equilibrium corresponds to 9 = 0® 45“. The plane of the dip circle is turned through

FF
60*’ from the magnetic meridian. Find the apparent

s
i.e. M parallel to B. value of dip.
Solution. Here, 5 = 45®, 9 = 60®, 5' = ?
uro cos 0® = 0-22 X 0-2 X 1

ok
rere
= » 0-044 J tan 5 tan 45® I
tan 5' = = 2

roF
cos 9 cos 60° 1/ 2
(«■) unstable equilibrium corresponds to M
8' = tan-J (2) = 63-4®

fof
—>

antiparallel to B. RE. is maximum


o 'Exam plja' [El The magnetic field at a point
Y
ooY
u = -MB cos 180® = - 0-22 x 0-2 x (- 1)
max.

= 0 *044 J
on magnetic equator is 3-1 x 10“® T. If radius of
B
earth is taken as 6400 km, what is magnetic moment
ks
of the assumed dipole at the centre of earth ?
Yo
L TYPEV. MAGNETIC ELEMENTS OF EARTH
I
r

Solution. As is clear from Fig. 5.64, any point


on the magnetic equator lies on equatorial line of the
eB
e
ru

assumed magnetic dipole at the centre of earth.


d

Formulae used. 1. Declination, 9 = angle between


magnetic meridian and geographic meridian
2. Horizontal component, H = Rcos 8,
o
ou
Yn
ad

vertical component, V = /? sin 8


Dipole
where 8 is the angle of dip at the place.
di

3. R = V//2 + y2 4. tan 8 = VIH Equator


FF
Re
in

5. When vertical plane carrying magnetic needle in


magnetic meridian is turned through 9 degree,
apparent value of dip (8') is given by :
V V tan 8
tan 8' = —●
H' //cos0 COS0 B
equQ 47t /?2
Units used. All angles 0, 8 in degrees, resultant
4tc
field of earth, R and its components H, V in tesla. M - B X/?2
equa
^^0
Exam
pla m A compass needle whose = 10^ (3-1 X 10-5) (6400 X 10^)2
magnetic moment is 60 Am2 pointing geographic M=8-1x1()22 Am2
north at a certain place where horizontal compo
nent of earth’s magnetic field is 40 |iWb/m2 Exampie'‘Rn] A ship Is to reach a place 10®
south of west. In what direction should it be
experiences a torque of 1-2 x 10^ Nm. What Is
the declination of the place ? steered if declination at the place is 17® west ?
5/46
A Fundamental Physics (XII) tTOWl

Solution. As the ship is to reach a place 10® Example^ A long Straight horizontal
south of west i.e. along OA, Fig. 5.65, therefore, it cable carries a current of 2.5 amp. in the
should be steered west of (magnetic) north at an angle direction 10® south of west to 10® north of east.
of (90-17+10) = 83®
Fig. 5.66. The magnetic meridian of the place
happens to be 10® west of the geographic
meridian. The earth’s magnetic field at the
location is 033 G and the angle of dip is zero.
Locate the line of neutral points (Ignore the

w
thickness of the cable). [At neutral points,
magnetic field due to a current cable is equal and
opposite to the horizontal component of earth’s
GE magnetic Held.]

e
row
FIGURE 5.66

re
MN GN

FFllo
F
u
ree
10®
Example m A magnet suspended at 30®

sFr
with magnetic meridian makes an angle of 45® with GW
10®

kro
the horizontal. What shall be the actual value of
the angle of dip ?
uor
(Uttarakhand Board 2012)
Solution. Here, 0 = 30°; Apparent value of dip,
5| = 45°; Actual value of dip, 5 = ?
offo
kos
If H is horizontal component of earth’s magnetic
YYo
eerBB

MS
oo

GS
V
field in magnetic meridian, then tan 5 = —
rY

H Solution. Here, i = 2.5 amp.;


Let be component of H at 30® to magnetic R = 0.33 G = 0.33 x 10^ T ; 5 = 0°
uu

meridian, then Horizontal component of earth’s field


ad
doo

V V tan 6 H = R cos 6 - 0.33 x 10^ cos 0°


tan5| =
nY

H
I
H cos 0 cos 0 = 0.33 X 10-^ tesla

or tan 5 = tan 6j x cos 0 = tan 45® x cos 30® Let the neutral points lie at a distance r from
nid
Re

the cable.
V3 1-732 Strength of magnetic field on this line due to
FFi

= lx = 0-866
2 2
current in the cable =
^^0 I

8 = tan-' (0-866) = 40*9® 27cr

Example^ In the magnetic meridian of ^0 I

a certain place, the horizontal component of the At neutral point = H


2kt
earth’s magnetic field is 0*26 G and dip angle is
60®. What is the magnetic field of earth at this
location ?
ii-pi _ 471X10-^x2.5 = 1.5 X 10-2
(NCERT Solved Example) 2nH ~ 2jtx0.33xl0-^
m

Solution. Here, H = 0-26 G, 6 = 60°, R = 7


- 1.5 cm
As H = R cos 5
Hence neutral points lie on a straight line
R =
H _ 0-26 0-26 parallel to the cable at a perpendicular distance of
= 0-52 G
cos 8 cos 60° (1 / 2) 1.5 cm above the plane of the paper.
MAGNETISM AND MATTER 5/47

Example ^ A telephonic cable at a place =3.19x 10-5-2 X


has four long straight horizontal wires carrying a = 1.19 X 10“5 tesla
current of 1.0 amp. in the same direction east to
Hence,
west The earth’s magnetic field at the place is 0.39
G and the angle of dip is SS''. The magnetic
declination is almost zero. What are the resultant = -^/(l .19x10-5^ + (2^2 X10-5 )2
magnetic fields at points 4*0 cm below and above = 2.5 X 10"5 tesla

w
the cable ?
At point P, 4 cm above the wire, horizontal
Solution. Here, No. of wires, ;i = 4, / = 1.0 amp. ;
component of earth’s field and field due to current
earth’s field, R = 0.39 G = 0.39 x 10"^ T are in the same direction. Fig. 5.67,
dip, 6 = 35° ; declination 0 = 0° H2 = H+B=3.\9x 10"5 + 2x 10-5

e
row
= 5.19x 10-5 T

re
R^=7,R^=7-
r = 4 cm each = 4 x 10“^ m

F
FFllo
Magnetic field at 4 cm due to currents in 4 wires

u
^0 47txl0-'^xl = V(5-19 X10-5 ^ (2-2 X10-5 y

ree
I
B = 4x = 4x = 2x10 5 tesla
2jtr 2tcx4x10 ^ = 5‘54 X 10“5 tesla

sFr
Horizontal component of earth’s field

kro
// = /? cos 5 = 0.39 X 10^ cos 35°
I TYPE VI. NEUTRAL POINTS
I
uor
= 0.39x 10-4x0.8192 offo Formulae used. 1. When magnet is placed with its
= 3.19 X 10'5 tesla
kos
N pole towards 5 of earth, neutral points lie on axial
Y
Yo
Vertical component of earth’s field line of magnet, such that
eerBB
oo

V=/?sin5=0.39x 10^ sin 35° _ Pq 2Md


rY

B = H
I “
= 0.39 X 10-4 X 0.5736 4TC(rf2_/2j2
= 2.2x 10-5 tesla
u

2. When magnet is placed with its N pole towards


At point Q, 4 cm below the wire, horizontal N of eirth, neutral points lie on equatorial line of
ou
ad
do

component due to earth’s field and field due to current magnet, such that
nY

are in opposite directions. Fig. 5.67, so that M


D _iio = W
2 47l(d2+/2)3/2
nid

1
Re

Units used. M is in Am^ or JT"* ; d and / are in


FFi

metre, 6], ^2 ^ tesla.

^^0 = 10“'^WbA-’m
Standard Value.
4k

Example PS A Small bar magnet has a


W ●4 E magnetic moment 5 Am^. The neutral point is
CURRENT
obtained on axial line when it is placed in magnetic
meridian with its N pole pointing S of earth and
neutral point is obtained on equatorial Iine» when
it is placed with its N pole pointing towards north
of earth. If horizontal component of earth’s field
is 0*38 G, find the position of neutral points in the
two cases.
5/48 ‘P'tadee^’A Fundamental Physics (XII)ISQIBI
Solution. Here, M- 5 Am^,
= //
if = 0-38 G = 0-38 xKHT. As B2 =-

If dj is distance of neutral point when iVpole of


magnet is pointing S of earth, then
- 47C^r
n _ ^^0 2M =H
1”4tc d?1

2x5
^ 3 ^ 10^_xl^
4x10-5 ^ 04x 10-2 m3
d.3=ii2-—= 10-’x = 04 X 10“2 X 10^ cm3
^ 47t H a38xio-^
= 4000 cm3
= 26-3 X 10-3 m

ww
= (26-3 X 10-3)1/3 m ^ 2-97 x lO-i m ^2 = (4000)1/3 = 15-87 cm
= 29‘7 cm
A magnet placed in the

Flo
magnetic meridian with its north pole pointing
In the second case, - north of the earth produces a neutral point at a
47Cd3

e
distance of 0*15 m from either pole. It is then

rree
broken into two equal parts and one such piece is

r FF
ILtp M_ 10"'/x5
d|= 47tH = 13-15 X l(r3 placed in a similar position. Find the position of
0-38x10“^ the neutral point
uurr
^2 = (13-15 X 10-3)1/3 m = 2-36 x IQ-l m
= 23*6 cm
for
Solution. As N-pole of magnet is pointing
north, neutral points lie on equatorial line of magnet
kss
at a distance x from each pole.
riix:'qmplbj|S|3 The magnetic moment of a
ooook
Yo
M
short bar magnet is 1*6 A m^. It is placed in the H = B .(0
e<pi.
4tu 7?
eB

magnetic meridian with north pole pointing south.


The neutral point is obtained at 20 cm from the
When the magnet is broken into two equal
centre of the magnet. Calculate the horizontal
rr

parts, pole strength remains unchanged, but


component H of earth’s field. If magnet be
ou
ad

magnetic moment is reduced to half


reversed i.e. north pole pointing north, find the
Y

i.e.. Af' = M2
position of the neutral point
dY

Solution. Here, M = 1-6 A m^, If y is distance of neutral point from each pole
Re
innd

of the part, then


1
d,1 = 20 cm = -m,if=?
Fi

.(«)
4 71
When north pole of magnet is pointing south,
neutral points lie on axial line of magnet. At neutral From (0 and (u')>
point.

A =
_ ^0 2M 4tt jc'3 471
1
4tc d 1 3

10-'/X 2x1-6 x'3 =;c3


or = 4 X 10~5 tesla M 2
(1/5)3
0-15
When magnet is reversed i.e. north pole is
21/3 1.26
pointing north, neutral points lie on equatorial line
of the magnet. We have to calculate ^2- = 0*119 m from each pole
MAGNETISM AND MATTER 5/49

I I
TYPE VII. TANGENT LAW, TYPE VIII. ATOM AS A MAGNETIC DIPOLE
TANGENT GALVANOMETER

Formulae used. Formula used. 1. Magnetic moment of an atom


due to revolving electron
1. F = // tan 0, where 0 is angle with H
^ eh \
M = n
H H IrH
2.1 = K tan 0, where K = — Aizm
G \iQn/2r eh
Units used. The field intensities F and H are in where Itg = 47um
= Bohr magneton
tesla. Cuixent / is in ampere, radius r is in metre. = 9-27x 10“^ Am^.
Magnetic moment M is in Am^. Time period T in
sec. Frequency v is in hertz.
Units used. M and |ig in Am^ ; e in coulomb ;

ww
m in kg and h in Js.
Example ^ Two tangent galvanometers Standard Values, e = 1-6 x KT'^C,
A and B have coils of radii 8 cm and 16 cm
m = 9-1 X 10-3' kg, ft = 6-6 X 10-34 J.s
respectively and resistance 8 ohm each. They are

Flo
connected in parallel to a cell of emf 4 V and negli Example EE In a hydrogen atom, the

e
electron moves in an orbit of radius 0*5 A, making

ree
gible internal resistance. The deflections produced
are 30° and 60° respectively. A has 2 turns. What lO^'^rps. Calculate the magnetic moment

Fr
rF
is the number of turns in B ? [CBSE 2007 (C)j associated with the orbital motion of electron.
Solution. Here, r = 0-5 A = 0-5 x 10“'^ m ;
uurr
Solution. Here, rj = 8 cm, K2 = 16 cm,
Fj = = 8 ohm
F = 4 V, 01 = 30°, 02 = 60°, = 2, «2 = ? for v = 10*^rps,M = ?
Charge on electron, e = 1-6 x IQ-'^ C
s
kks
2r,H 2/*2//
,=i=
Yo
1 e
oooo

tan0 I ” tan ©2
Equivalent current, ^
= ev
R
^0 "2
eB

Area of the orbit, A = Kr^


n
T2 tan Oj As M = IA
2 ~
r:1 tan0 1
.*. M = ev(7C/^) = l*6xlO-'®xlO'®
r
ou
ad

X 3-14 X (0*5 X lO-'V


^2xl6tan60°^4.>^
YY

= 12 = 1*256 xl0-“ Am^


8 tan 30° U-Js
nndd

Example^ An electron moves around


Example ^ The coil in a tangent galvano
Re

the nucleus in a hydrogen atom of radius 0*51 A


meter is 16 cm in radius. How many turns of the
Fi

wire should be wound on it if a current of 20 mA is


with a velocity of 2 x 10^ m/s. Calculate the
following :
to produce a deflection of 45° ? Take horizontal
component of earth’s field = 0.36 x 10"^ T. (i) the equivalent current due to orbital
motion of electron, (ii) magnetic field produced at
Solution. Here, r = 16 cm = 16 x 10-^ m, the centre of nucleus, (iii) the magnetic moment
n = ?;/ = 20mA = 20x 10-3 associated with the electron. (CBSE 2008)
0 = 45°: // = 0.36 X 10^ tesla Solution. Here, r = 0*51 A = 0-51 x 10"'® m
2rH
u = 2 X 10® m/s
I = tan0
From e ev

2nr/v 2itr
2rH 2x16x10-2 X 0.36x10-4
n = tan 0 = F6xl0-'^x2xl0®
AnxlO-'^ X20x10-3 = 10^ A
2x3-14x0-51xl0-'®
= 458 turns
5/50 ^●teitUei^'4- Fundamental Physics (XIl)KSSIM]

lip / _ 47cx1Q-'^x10^ Example ^ jhe coercivity of a certain


(») B = = 1-23 T
2r 2xO-51xlO-'0 permanent magnet is 4-0 x lO"* Am“*. This magnet
is placed inside a solenoid 15 cm long and having
ev 2 ^
(Hi) M = IA = X7ur = — evr
600 turns. Calculate the current required to
2nr 2
demagnetise the magnet completely.
I
= -xl-6xl0-‘9x2xl0^x0-5lxl0-'0 Solution. Here, coercivity = 4-0 x 10"^ Am~* of
2
magnet implies that magnetising intensity
= 8-16 X 10-25 Am2 H = 4 0 X 10'^ Am“' is required to be applied in
opposite direction to demagnetise the magnet.
TYPE IX. MAGNETIC n = number of turns per unit length
PROPERTIES OF MATERIALS
600
- 4000 tums/m
15x10-2

ww
B
Formulae used. 1. ft = T7 As H ~ nl
H
I H 4-0x10'*

FF loo
= 10 A
2- ~»
H
where / = magnetic moment/volume n 4000

ree
li
3. \^r=~ Example EH A toroidal solenoid has 3000
^*0 turns and a mean radius of 10 cm. It has a soft

rreeF
4. B = |ip(//+/) iron core of relative magnetic permeability 2000.
5. 1 -I-X,,,. Find the magnitude of the magnetic field in the
ur
Units used. / and // are in Am”', B in tesla, p in T
A~‘ m, M in Am^, Volume V in m^, and have fofroF
core when a current of 1.0 amp. is passed through
the solenoid.
ks
no units. 3000
Solution. Here, n = , r = 10 cm = 0.1 m.
Standard value. Pp = 4 tc x 10”2 Wb A"* m
-I
kos o

2iir
YYouor

p^ - 2000, / = 1 amp., B = ?
BBoo

Example The core of a toroid having 3000


B = PpPr/i /=4tcx10-2 x2000x 2tcx0-1 xl
r ee

3000 turns has inner and outer radii of 11 cm and


= 12 tesla
12 cm respectively. The magnetic field in the core
for a current of 0*70 A is 2*5 T. Calculate relative
ad
oouur

permeability of the core ?


Example]^ A magnetising field of 1600
Y

Am“^ produces a magnetic flux of 2*4 x 10“5 weber


Solution. Here, N = 3000, r| = 11 cm, in a bar of iron of cross section 0*2 cm^. Calculate
nYd

r2=12cm, / = 0-70 A,S = 2-5T,p^=? permeability and susceptibility of the bar.


Re

Solution. Here, H = 1600 Am"*,


n + r^ _ 11 + 12
FFinid

Mean radius = = 11-5 cm = 2-4 x 10-5


2 2
= 11-5 X 10-2 m
a = 0-2 cm2 ^ 0-2 X 10^ m^ p = ?, X m
— 9

Number of turns per unit length, <S? _ 2-4x10-5


= 1-2 weber/m2
N 3000
a ~ 0-2x10’'*
n —
B 1-2
27ir 2x314xll-5xl0’2 i^ = — - I'S X 10-^ TA-‘ m
H 1600
As B = \inl
As
B 2-5x2x3-14x11-5x10’2
7-5 xlO-'*
ni 3000x0-7 Xm -1 = -1
^0 4JTX10-2
= 8-6 X 10^
8-6x10^ 7-5x105
-1 = 597-1-1 =596*1
= 684*4
4tc
Pq 4tcx10 2
MAGNETISM AND MATTER 5/51

A solenoid of 500 turns per Example^ An ideal solenoid having


metre is carrying a current of 3 A. Its core is made 2000 turns per meter has an iron core of
of iron, which has a relative permeability of 5000. permeability 500 and carries a current of 1*0 A.
Determine the magnitudes of magnetic intensity, Calculate (i) magnetising intensity (H) at the
intensity of magnetisation and magnetic field centre of solenoid (ii) magnetic permeability of
inside the core.
iron (Hi) magnetic susceptibility of iron
Solution. Here, n = 500 tums/m ; (iv) intensity of magnetisation.
i = 3A ; 11^=5000. Solution. Here, n = 2000, / = 1-0 A
Magnetic intensity, (/) Magnetising intensity, H = nl

oww
// = « I = 500 m“* X 3 A = 1500 Am“' = 2000 X 1 = 2000 Am-'
As m (ii) Magnetic permeability
= 11, - 1 = 5000 - 1 = 4999 = 5000 ii = Pfl = 4 rt X lO-”^ X 500
= 6-28 X 10-^ Tm A“'.

e
11
Also, |i, - = 5000
11 = 5000 llg ("'0 Xn, = 11, - 1 = 500 - 1 = 499.

re
^0

FFrllo
Intensity of magnetisation, (/v) l=x„H = 499x 2000 = 9-98 x 10® Am"’

rF
1 = X^H= 5000 X 1500 = 7-5 x 10^ Am"' Example An iron atom has a dipole

ee
Magnetic field inside the core, moment of 9 x 10“^ Am^. Density of iron is
ouru
B = ^H = 5000\IqH= 5000 (4tt x lO""^) x 1500 8 g/cm^ and molecular mass is 55 g mole”'. If

rF
domain size of iron is a cube of side 10“® m find
fi = 37i = 3x—= 9-4T (/) number of atoms in the domain (ii) maximum

fosor
7
dipole moment (Hi) intensity of magnetisation.
Example IS A solenoid has a core of a
os kf
Solution. Dipole moment of each atom
material with relative permeability 400. The = 9x I0"2‘'Am2
ook
YYo
windings of the solenoid are insulated from the
Density of iron = 8 g/cc = 8 x 10^ kg/m^
core and carry a current of 2 A. If the number of
Bo

Molecular mass = 55 g = 55 x 10“^ kg


reeB

turns is 1000 per metre, calculate (i) H (ii) B


Volume of domain, = (I x 10“^ m)^ = m^
(Hi) Intensity of magnetisation /, and the magne
oouY

tising current. (NCERT Solved Example) Mass of domain = Vol. x density


ur

Solution. Here, ii^ = 400, = 10"'^ X 8 X 10^ = 8 X 10"'5 kg


ad

Number of atoms in the domain


r = 2 A, n = 1000 per metre
Yd

(/) H = nr^ 1000 X 2 = 2 X 10^ Am"' _ mass of domain x Avogadro number


nidn

(iV) B = li// = mjii^// molecular mass


Re

= 4 7t X 10-’ X 400 (2 X 10^) = 1-0 T 8xl0-^^x6-023xl023


(/■//) From B = ^(H + T), where I is intensity of
FFi

magnetisation, 55x10-3
B 1-0 = 8-76x lO'O
/ = // =
4tuxI0 y-2xl03 Maximum dipole moment
= 9x10-2^x8-76x10^0
= 7-95x 10®-0-02 X 10®
M = 7-88x 10-'3 Am2
= 7-93x105Am-'
Intensity of magnetisation,
(i I') The magnetising current is the additional
7-88x10“'3
current that needs to be passed through the windings / =^=
of the solenoid in the absence of the core, which would V 10-18
produce a B value as in the presence of the core. Thus, = 7-88 X 10® Am-'
5 = Pon(/'-f/,„)
1-0 = 4 71X 10"2x 1000 (2 + Exsmple EE Find the percentage increase
10
in the magnetic field B when the space within a
current carrying toroid is filled with aluminium
=796 - 2 = 794 A
of susceptibility 2-1 x 10-®.
5/52 ‘P’uidee^ Fundamental Physics (XII)

Solution. In the absence of aluminium, If £ is hysterisis loss (energy/vol./cycle), then


= Mo energy lost/hour = £x Vx50x60x60
= 8-64 X
In the presence of aluminium,
£x lO-^x 18 X 10^ = 8-64 X 10^
B = ^// = Po(l
Increase in magnetic field induction 8-64x10'^
£ =
18x10
- 4‘8 X 10^ J/m^/cycle
B-Br.0
Percentage increase = xlOO
B Exar^piieFin Find curie constant for a
0
sample from % versus VT graph shovm in Fig. 5.68.
XlOO Solution. From Fig. 5.68,
Mo^
for X = 0-5, ^ = 10x10-3K"' = 10-2 K-^

w
= 2-1 X 10"^ X 100 = 2-1 X 10-^%
C
Example ED A bar of 5 x 10~^ area is c = x-r = o-5x
1
= 50K
As X = J

Flo
subjected to a field of 600 Am. If susceptibility of 10-2
material is 499, calculate (i) permeability («) flux

reeee
density and (Hi) magnetic flux.
Solution. Here, A = 5 x 10”^ m^, H = 600 Am

Fr
X,„ = 499,^ = ?£ = ?(|> = ?
It = Mo (1 + ^ f
for
ur
= 6-28 X 10^ T A'* m
B = \iH = 6-28 X ICr* X 600 = 0-377 T
kss
(J) = 6A = 0-377 X 5 X 10“^
Yo
oo

= 1-885 X 10-3
eB

I TYPE X. CURIE LAW AND HYSTERISIS LOSS


I Example ^ Hysterisis loss for a sample
r

of 120 kg is 300 J m-3/cycle. Frequency of supply


ou

C
ad

Formulae used.l. X m = — , where C is a constant. is 50 cycle/s. If density of iron is 7500 kg/m^, find
YY

energy loss in 60 minutes.


2. Energy loss/volume/cycle = area of IH loop
Solution. Here, m= 120 kg
1
nd

Hysterisis loss = 300 J m“3/cycle.


Re

X area of BH loop
4tc
Frequency = 50 c/s
Fi

Units used. Tis in K. has no units. Energy is in


joule.
Density of iron = 7500 kg/m^
Total time = 60 mins = 60 x 60 s

Example ^ A Specimen of iron weighing Volume of specimen -


mass 120 m^
3
6 kg loses 8-64 x 10"* J of energy per hour when density 7500
subjected to 50 c/s a.c. If density of iron is 120 360
6000 km/m3, what is the hysterisis loss per unit Loss of energy/cycle = 300 x
7500 ~ 75
volume per cycle ?
Solution. Here, m = 6 kg 360
Loss of energy/sec = x50
Energy lost/hour - 8-64 x lO'^ J/hr. 75
V = 50 c/s, p = 6000 kg/m^ ; 360
hysterisis loss = ? Total loss of energy = x50x60x60
75

volume, V = —
m
^ ~ 10"3 m3 = 240 X 3600
p 6000 = 86-4 X 10^ J
MAGNETISM AND MATTER 5/53

'Exampio A sample of paramagnetic mentioned in (b) ? (d) The bar magnet is replaced
salt contains 2 x atomic dipoles, each of by a solenoid of cross sectional area 2 x 1(H m^
moment 1.5 x 10"^^ JT~^. The sample is placed and 1000 turns, but the same magnetic moment.
under a homogeneous magnetic field of 0.64 T and Determine the current flowing through the
cooled to a temperature of 4.2 K. The degree of solenoid. (NCERT Solved Example)
magnetic saturation achieved is equal to 15%. Solution. («) Here, 0 = 30", B = 800 x lO"^ T,
What is the total dipole moment of the sample for T = 0-016 Nm,Af = ?
a magnetic field of 0.98 T and a temperature of As t = MB sin 0
2.8 K. (Assume Curie’s la>v).
T 0-016
Solution. Here, no. of dipoles, n = 2 x 10^ M =
Magnetic moment of each dipole, 5sin0 800x10^ xsin 30"
M'={.5x 1(T23 = 0*40 Am^

ww
Total dipole moment of sample = n x Af (^) For most stable position, 0 = 0® and for most
= 2x102“^ X 1.5 X 10-23 unstable position, 0 = 180°.
= 30 J'r* work done by external force

Flo
As saturation achieved is 15%, therefore, - - MB (cos 180® - cos 0°) = 2 MB

ee
effective dipole moment = 2 X 0-40 X 800 X 10^ = 0-064 J
(c) Work done by external mag. field

rere
15

rFF
M
1
x30 = 4.5JT“^ = - 0-064 J
100
(^0 AsAf = A7A
= 0.64T, =4.2K
uur r
.-. 0-40= 1000x7x2 X 10“^;
M2 = ? B2 = 0.98 T,
According to Curie’s law,
ffoor
T2 = 2.8 K
/ =
0-40
= 2A
sks
0-2
Yoo
£.~L
oko

Xm
T~ H Example m A magnetic needle has
BBo

magnetic moment of 6-7 x 10"2 Am^ and moment


or
of Inertia of 7-5 x 10“^ kg m^. It performs 10
re

T
complete oscillations in 6-70 s. What is the
As I oe M and H oc B
ouur

magnitude of the magnetic field ?


ad

CB (NCERT Solved Example)


YY

M oc

T M
^ ^1 'T'l Solution. Here, Af = 6-7 x 10“2 Am^,
7=7-5x 10-^kg m2
nndd
Re

or
_B2T\M 1
=
0.98x4.2x4.5
Time for one oscillation.
T^B, 2.8x0.64 ’
FFii

6-70
T = = 0-67 s ; fi = ?
= 10.34 10

I
I
TYPE XI. TYPICAL EXAMPLES From T* = 2n
Vms
4n-l 4x(22/7)2x7-5x10-^
Example ^ \ short bar magnet placed B = = 0-01T
A7T2 6-7x10-2 (0-67)2
with its axis at 30® experiences a torque of
0-016 N-m in an external field of 800 G.
(a) What is the magnetic moment of the
Example^ A domain in ferromagnetic
magnet ? (&) What is the work done by an external iron is in the form of a cube of side length 10“^ m.
Estimate the number of iron atoms in the domain
force in moving it from its most stable to most
unstable position ? (c) What is the work done by and the maximum possible dipole moment and
force due to external magnetic field in the process magnetisationof the domain. The molecularmass

*For derivation of this formula, refer to Art. 5.11.


5/54 7\euUeft Fundamental Physics (XII) iWilMl

of iron is 55 g/mole, and its density is Number of atoms in the domain


7*9 g/cm^. Assume that each iron atom has a dipole 6-023x1023x7-9x10-^
moment of 9*27 x 10"^ Am^.
55
(NCERT Solved Example) 16
= 8-65 X 10 atoms.
Solution. Here, length of cubic domain
Maximum possible dipole moment is achieved
= /= 10~^ m when all the atomic dipole moments are perfectly
Volume of domain = V = P = (10^)^ aligned (which of course is unrealistic).
- 10“^^ m^ - 10“^ cm^
M max = (8-65x 10‘^)x9-27x
= 8 0 X 10-’ Am^
Mass of domain = Volume x density
Max. Intensity of magnetisation
- 10“^ X 7-9 gram. M 8-0x10"'^

ww
max
/
It is given that 55 gram of iron contain 6-023 x max
Volume of domain 10
-12

10^^ iron atoms (Avogadro’s number) = 8 X 10^ Am ^

FF loo
ee
ee r
rFrF
1. According to molecular theory, every molecule of a magnetic substance (whether magnetised or not), is a
complete magnet in itself.
rur
In an unmagnetised substance, the molecular magnets are randomly oriented such that they form closed
ffoor
chains. That is why resultant magnetism of the unmagnetised specimen is zero. On magnetising the substance,
the molecular magnets are realigned in the direction of the field.
ks
2. Magnetic field lines are hypothetical lines which enable us to understand certain phenomena in magnetism.
YYoou
okos

Tangent to field line at a point gives us the direction of magnetic field B at that point. No two magnetic
BBoo

field lines can intersect each other.


Magnetic field lines are continuous curves from north to south, outside the body of the magnet and from
r ee

south to north inside the body of the magnet. A current carrying solenoid from outside resembles a bar
magnet. Inside the solenoid, there is a strong magnetic field, which can magnetise a specimen. The solenoid
oouur
ad

is hollow from inside, whereas the bar magnet is solid.


Y

3. A magnetic dipole consists of two unlike poles of equal strength (m) and separated by a small distance
(2 /). It has a magnetic dipole moment, M =m (2 0, directed from south to north. The SI units of M are
dnYd
Re

joule/tesla or ampere metre^


FFini

When a bar magnet of magnetic moment M is held at Z0 with the direction of a uniform magnetic field B ,
a torque x = MB sin 0 acts on the magnet. It tries to align the dipole in the direction of the field. Work is
required to be done in rotating the dipole against the action of the torque. This work is stored as potential
energy of the dipole. .PE. = W=- MB (cos &2 ~
—*

Here, 6j = initial angle & 02 is final angle of bar magnet with B .


Usually, 0j is taken as 90°, which is the position of zero P.E.
4. A loop of current acts as a dipole of magnetic moment M = lAn where / is strength of current through the
loop and A is area enclosed by the loop, n is unit vector along M .
5. According to Gauss’s theorem in magnetism, surface integral of magnetic field induction over a surface
f —> —»
(closed or open) is always zero, i.e. w B .ds = 0
S

This theorem establishes that magnetic poles always exist in equal and unlike pairs.
MAGNETISM AND MATTER
5/55

6. Te^estrial magnetism deals with the magnetic field of earth. It is described in terms of three quantities,
which are called magnetic elements of earth. These are :

(i) Magnetic declination (0) («) Magnetic inclination or Dip (6) (Hi) Horizontal component (H) of earth’s
magnetic field.
Magnetic decUnation at a place is the angle between magnetic axis and the geographic axis. It is also equal
to the angle between magnetic meridian and geographic meridian at that place.
Magnetic Inclination or dip at a place is defined as the angle, which the direction of total earth’s magnetic
field makes with a horizontal line in magnetic meridian.
Horizontal component of earth’s magnetic field is the component of total earth’s magnetic field (/?) in
the horizontal direction in magnetic meridian. Obviously.
H = R cos 6 ; V=/? sin 8

=R and — = tanS

w
H
At magnetic poles, 8 = 90® and at magnetic equator, 8 = 0®
7. Magnetic Field strength 5| due to a bar magnet at any point on the axial line of the magnet is

Flo
^1 _ Ho IMd
47C(f/2_/2j2

ee
where d is distance of the point from the centre of the magnet. The direction of is along SNproduced.

Fr
Magnetic Field strength B2 due to a bar magnet at any pt. on equatorial line of the bar magnet is
M

for
B
ur
^ 47T(rf2+/2)3/2
The direction of B2 is along a line parallel to NS.
ks
8. Neutral points are the points where the resultant magnetic field due to the field of the bar magnet and due
Yo
oo

to field of earth is zero. When a magnet is placed with its N pole towards geographic north, neutral points
eB

lie on equatorial line of the magnet. At each neutral point, B2 = 1^0 M


= H
4jC(</2 +/2)3/2
r

where H is horizontal component of earth’s magnetic field.


ou
ad

When a bar magnet is placed with its //pole towards geographic south, the neutral points lie on axial line of
Y

2Md
the magnet. At each neutral point. B,= = H
471(^2 _/2)2
nd
Re

Tangent Law. When a magnet is suspended freely under the combined action of two uniform magnetic
Fi

fields of strengths F and H acting at 90® to each other and the magnet comes to rest making an angle 0 with
the direction of H, then F= tan 0
This is tangent law. A tangent galvanometer is based on tangent law.
2rH H
In a tangent galvanometer, / = tan0 = —tan0 = A:tan0
1^0 « G

H
where K = — = reduction factor of tangent galvanometer and G = \Iq nllr is galvanometer constant
10.
Every atom behaves as a magnetic dipole, because electrons in the atom revolve around the nucleus. The
neh
magnetic moment M associated with an atomic dipole is ^ =
47CTO
where n = 1, 2, 3,....denotes the number of orbit
eh
and = = least value of dipole moment of atom = 9.27 x Am2.
471OT is called Bohr magneton.
5/56 Fundamental Physics (XIDESSSU

11. Some important terms used in magnetism are :


(/) Magnetic field strength, Magnetic field induction or Magnetic flux density (B) is defined from
—* ♦

Lorentz forceeqn. F = q^{v x B)


SI unit of B is tesla and cgs unit of B is gauss, where 1 tesla = 10^ gauss,
(if) Magnetising force or magnetising intensity (H) = number of ampere turns flowing round unit length
of toroidal solenoid to produce magnetic field induction B in the solenoid. H = n LSI unit of H is Am"*.

B u
(hi) Relative Magnetic permeability ^0 ^

ww
Relative magnetic permeability has no units.
M
(jv) Intensity of magnetisation I = — — . SI unit of I is Am * ; same as that of H.

Flo
e
(v) Magnetic susceptibility (x^) = — . It has no units.

ere
H

FFr
Important relations governing these terms are :
uurr B = \LQiH + l) ; ^,= 1+X m

orr
12. Classification of magnetic materials.
sfo
On the basis of magnetic properties, different materials have been classified into three categories :
kks
Diamagnetic, Paramagnetic and Ferromagnetic.
Yo
oooo

Diamagnetic substances are those in which the individual atoms/molecules/ions do not possess any net
magnetic moment on their own. These are feebly magnetised in a direction opposite to that of the magnetising
eBB

field in which they are placed. For example, bismuth, antimony, copper, gold, quartz, mercury, water, alcohol,
air, hydrogen etc. The permeability (p) of diamagnetic substances is less than one and their susceptibility is
rr

negative, which does not change with temperature.


ou

Paramagnetic substances are those in which each individual atom/molecule/ion has a net non zero magnetic
ad
YY

moment of its own. These are weakly magnetised in the direction of the magnetising field, in which they are
placed. For example, aluminium, platinum, chromium, manganese, crown glass, oxygen etc. The permeability
nndd

|i of paramagnetic substances is greater than one, and their susceptibility is positive. The susceptibility (X^)
Re

of such materials decreases with rise in temperature.


Fi

Ferromagnetic substances are those in which each individual atom/molecule/ion has a non zero megnetic
moment on its own. They are magnetised strongly in the direction of the magnetising field in which they are
placed. For example, iron, cobalt, nickel and a number of their alloys. Ferromagnetic materials show all the
properties of paramagnetic substances to a much greater degree.
At a certain temperature, ferromagnetics pass over to paramagnetics. This transition temperature is called
Curie temperature. For soft iron. Curie temp, is 1000 K.
The three kinds of behaviour (dia, para, ferro) has been explained on the basis of electron theory ofmagnetism.
13. Hysteresis is the phenomenon of lagging of magnetic induction (B) and intensity of magnetisation (/)
behind the magnetising field (H), when a specimen is taken through a cycle of magnetisation.
From the hysteresis loop of a material, we can study about retentivlty, coercivity etc. of the material. The
loss of energy per unit volume of the specimen per cycle of magnetisation is equal to area of the I—H loop
(in cgs system) or the area of B-H loop (in SI units) of the specimen. The study of these characteristics
enables us to select suitable materials for different purposes.
MAGNETISM AND MATTER 5/57

QUESTIONS

Based on NCERT Book

1. Multiple Choice Questions (c) High mechanically hardness, all over


1. If the magnetizing field on a ferromagnetic material (d) Mechanically hard surface
is increased, its permeability 8. The area of B-H loop for soft iron, as compared to
that for steel is ;
(a) decreases {b) increases
(c) remains unchanged (a) More (b) Less
(d) first decreases and then increases (c) Equal (d) Zero

(CBSE Sample Paper 2022-23) 9. A stationary magnet does not interact with :
(a) iron rod (b) moving charge
2. The horizontal component of earth’s magnetic field

w
at a place is 0-2 G, whereas its total magnetic field (c) magnet (d) stationary charge
is 04 G. The angle of dip at the place is 10. The value of the magnetic susceptibility for a super
(fl) 30° (b) 45° conductor is :

Flo
(c) 60° (d) 90° (CBSE 2022) (a) zero (b) Infinity
(c) highly positive

e
(d) highly negative

reee
3. A bar magnet has magnetic dipole moment M. Its 11. A bar magnet AB with magnetic moment M is cut
initial position is parallel to the direction of uniform

FFr
into two equal parts perpendicular to its axis. One
magnetic field B. In this position, the magnitudes part is kept over the other so that end B is exactly
of torque and force acting on it are ; over A. What will be the magnetic moment of the

for
ur
combination so formed ?
(a) 0 and MB {b) zero and zero
(a) Zero ib) MIA
kkss
(c) MB and MB {d) \ M xB \ and zero (c) M {d) 3 M/4
Yo
(CBSE 2022) 12. Two unlike magnetic poles of strength 10 A-m each
oo

are held in air at a distance of O-IO m from each


4. Which of the following statements is correct ?
B

other. What is the magnetic force of attraction


id) Magnetic field lines do not form closed loops between them ?
re

ib) Magnetic field lines start from A^-pole and end id) 3 X 10-3 N ib) 2-5 X 10-3 N
at S-pole of a magnet
(c) 1-5 X 10-3 N id) 1 X 10-3 N
ou
ad

(c) Two magnetic field lines may intersect each 13. S.I. unit of magnetic pole strength is
YY

than
id) Ampere/meter ib) Ampere-meter
id) Tangent at a point on every field line gives the (c) volt/meter id) Ampere/meter^
nd

direction of magnetic field at that point


Re

14. Which of the following is an example for


(CBSE 2022) diamagnetic substances ?
Fi

5. Which of the following is weakly repelled by a (a) copper ib) nickel


magnetic field :
(c) aluminum id) iron
id) Iron ib) Cobalt
15. At a certain place, the horizontal component of
(c) Steel id) Copper earth’s magnetic field is Bq and angle of dip is 45°.
6. If a diamagnetic material is placed in a magnetic The total intensity of the field at that place is
field, the magnetic field inside the material
(a) Bo ib) ,B0
compared to that outside will be
id) Slightly less ib) Slightly more (c)2Bo
(c) Very high id) Same 16. The susceptibility of a ferromagnetic material is %
7. The permanent magnetic material is characterised at 27°C. At what temperature will its susceptibility
by: be 0-5 %.
id) Narrow hysteresis loop id) 54°C ib) 327°C
ib) Broad hysteresis loop (c) 600°C id) 237°C
5/58 Fundamental Physics (XIOBSm

17. Horizontal component of earth’s magnetic field at (b) A moving charge


a place is 3*2 x 10"^ T and angle of dip is 60°. The (c) A stationary charge
resultant intensity of earth’s magnetic field at the (d) A rectangular current loop with its plane
place is parallel to the field (CBSE 2022)
(a) 3-2 X 10-^ T (b) 6-4 X 10-5 j 25. What is the angle of dip at a place, where horizontal
(c) 1-6 X 10-5 T (d) 12-8 X 10-5 j and vertical components of earth’s magnetic field
18. If the distance between two similar poles of equal are equal ?
strength is doubled, to get the same repulsive force, (a) 30° (b) 60°
the pole strength of each pole should be (c)45° (d) 90° (CBSE 2021)

(a) increased by ^2 times 26. When a magnetic needle is kept in a non-uniform


magnetic field, if experiences
(b) increased by 4 times
(fl) a force and a torque

w
(c) halved
(b) only a force
(d) doubled
(c) only a torque
19. The SI unit of magnetic field intensity is

Flo
(d) neither a force nor a torque
(a) Am N
-1
(b) NA-' m-’
27. Which of the following statements is incorrect
(c) NA“^ m-^ (d) NA"' m-^

ee
about hysteresis ?
20. Two wires of the same length are shaped into a
(a) This effect is common to all ferromagnetic

Fr
square of side ‘a’ and a circle with radius r. If they substances
carry same current, the ratio of their magnetic
moment is (b) The hysteresis loop area is proportional to the

for
ur
thermal energy developed per unit volume of
(a) 2 : Tc ib)n:2
the material,
(c) 7E: 4 id)4 :n
ks
(c) The hysteresis loop area is independent of the
A 4y
Yo
21. A magnet of magnetic moment 501 Am'^ is thermal energy developed per unit volume of
oo

the substance.
placed along the jc-axis in a magnetic field,
B

id) The shape of the hysteresis loop is


B = (0-5/ -I- 3 0 j) T. The torque acting on the characteristic of the material.
re

magnet is
28. Relative permeability of iron is 5500. Its magnetic
susceptibility is :
ou

(fl) 175 k Nm (b) 75 i Nm


ad

(a) 5500 X 10-"^ ib) 5500 X lO"^


Y

ic) 150 cf Nm id) 25^kNm (c) 5499 (i/) 5501


22. A current i flows in a circular coil of radius r. If 29. Retentivity is maximum for
nd
Re

the coil is placed in a uniform magnetic field B (a) steel ib) soft iron
with its plane parallel to the field, magnitude of
Fi

(c) copper (d) silver


the torque that acts on the coil is
ia) zero ib)2nriB II. Assertion-Reason Type Questions
(c) iB id)2Kr^iB Direction. For question numbers 30 to 38, two
23. The vertical component of earth’s magnetic field statements are given, one labelled Assertion (A)
1
and the other labelled Reason (R). Select the
at a place is times the horizontal component. correct answer to these questions from the codes
(a), (6), (c) and (</) as given below :
The angle of dip at the place is id) Both, A and R are true, and R is correct
(t^)0° ib) 30° explanation of A.
(c) 45° {^0 60° (CBSE 2022) ib) Both, A and R are true, but R is not the correct
24. Which of the following is not affected by the explanation of A.
presence of a magnetic field ? (c) A is true, but R is false.
ia) Current carrying conductor id) A is false, and R is also false.
5/59
MAGNETISM AND MATTER

30. Assertion. A circular coil carrying current behaves 35. Assertion. Three similar bar magnets, each of
as a magnetic dipole. magnetic moment M are placed to form an
Reason. A magnetic dipole possesses magnetic
equilateral triangle as shown in Fig. 5(Q).l. The
magnetic moment of the system is zero.

north and south poles of equal strength a very large


distance apart.
31. Assertion. At the magnetic poles of the earth, a
compass needle will be vertical.
Reason. At the magnetic poles of the earth, the
magnetic field lines enter or leave the surface of
earth normally.
32. Assertion. Soft iron is most suitable as a core of
the transformer,

w
Reason. The retentivity and coercivity of soft iron
are very small.
33. Assertion. A bar magnet of magnetic moment m is

Flo
divided into two equal parts by cutting it
Reason. The given equilateral structure of three
perpendicular to its length, the magnetic moment

reeee
magnets do not have two free poles as required for
of either piece is also m/4.
magnets.

FFr
Reason. Magnetic moment of bar magnet is the 36. Assertion. Magnetic lines of force exert
product of pole strength and semi-length of the longitudinal tension.
magnet.
for
ur
Reason. There is a force of attraction between N-
34. Assertion. The magnetic field strength due to a
pole and S-pole of magnets.
short bar magnet on its axial line at distance r is
kkss
double than at the equitorial line at the same 37. Assertion. The magnetic field lines of a magnet
Yo
(or of a solenoid carrying current) are not closed
oo

distance.
continuous loop.
eB

Reason. Magnetic field strength at a point on axial


line. Reason. The magnetic field lines start from north
pole and end on south pole.
r

38. Assertion. The magnetic lines of force exert lateral


ou
ad

B
47t pressure.
YY

Reason. The magnetic line of force is a straight or


and on equitorial line
curved path in the magnetic field, tangent to it at a
ndd
Re

B
point gives the direction of magnetic field at that
eq ^ ● point.
Fi

ANSWERS

I. Multiple Choice Questions


5. id) 6. {a) 7. ib) 8. ib) 9. id) 10. id)
l.(fl) 2.(c) 3. ib) 4. id)
15. ib) 16. {b) 17.{b) 18. id) 19. {b) 20. (c)
11. ia) 12. id) 13. (fc) 14. (a)
24. (c) 25.(c) 26.(a) 27. (c) 28. ic) 29.ib)
21, (c) 22. (c) 23. Q>)

II. Assertion-Reason Type Questions


34.(fl) 35.id) 36. (a) 37. (d) 38. ib)
30. (c) 31. (a) 32. (c) 33. (t/)
5/60
7>>ut(ieefi,'4, Fundamental Physics (XII) VOL.I

HINTS/EXPLANATIONS For Difficult Questions


L Multiple Choice Questions
10x10
12. f = =
1. When magnetising field on a ferromagnetic 4n (0-10)2
material is increased, its permeability decreases.
= 1x1(HN
As, ^=1+X^=I + .I ; tt
M
So as ^ increases, the value of p decreases. 13. Pole strength,
21
2. Here, ^ = 0-2G
and i? = 04G,5=? AnP-
From H^Rcos 5 SI unit of m = -Am
m

0-2 1
15. Here, J^/ = 5q ; 8 = 45*

ww
cos 0 = — = — , 8 = 60*
/? 0-4 2
Total intensity of earth’s magnetic field is
3. Here, 0 = 0*
R = H/cos 8 = Pq/cos 45* = Bq V2

Flo
T = MP sin 0 = m sin 0* = Zero
F = m-MB = Ztro
16. As magnetic susceptibility, X ** ^

e
reree
4. Tangent at a point on a magnetic field line gives us
the direction of magnetic field at that point.

r FF
Magnetic lines of force form closed loops. They
.-. 5^ = 5.
go from ^-pole to S-pole out side of a magnet and Xl ^2
uurr
5-pole to N-pole inside the magnet. The two foor
magnetic lines of force do not intersect each other or
0-5 x_ 27 + 273
or r = 327*C
ks s
as at the point of intersection two tangents can be X ~ / + 273
Yoo
drawn which gives two directions of magnetic field
ooook

at that point, which is impossible. H _ 3-2x10-^ 3-2x10-5


eBB

17. R^
5. Copper is a diamagnetic which is weakly repelled COS0 cos60* 1/2
by a magnetic field.
= 6*4xlO^T
6. The diamagnetic materials when placed in a
rr
ouu

18. Let m be the pole strength of each pole and r be


ad

magnetic field, the magnetic lines of force prefer


not to pass through the specimen. the distance between them. The force of repulsion
YY

between them is
7. Knowledge based question.
nndd

8. The area of B-H loop measures energy dissipated


Re

lip ffixm
per unit volume in the material per cycle of 4jc r2 .(1)
Fii

magnetisation, which is less for soft iron than steel.


9. A magnetic can interact with a magnet but can not When / = 2randm = m',
interact with a stationary charge. A moving charge then as per question
also has a magnetic field around it which can be
interacted by a magnet. M-q m X m
10. Superconductor behaves as a good diamagnetic 4tc (2r)2 ~4jc r2
material. Its magnetic susceptibility is highly
negatives. .*. (i«0^ = 4»*2
11. Magnetic moment of each part of the magnet or m' = 2 m

= M12 19. From F-BIIsinQ

Total magnetic moment of the combination of two F N


B = = NA-lm-l
//sin0 Amxl
parts M = (M/2) + (-M/2) = 0
5/61
MAGNETISM AND MATTER

20. Here, 4 a = 2 ic r or 4a/2 n 28. Here, |i^ = 5500 ; we know that

M1
_ i\ _ n
or = 1=5500-1=5499

n
(4a'f * 29. Retentivity is maximum for soft iron because
2n during hysteresis loop, when magnetising field is
reduced to zero, the intensity of magnetisation left
in the material is maximum (related to retentivity)
21. Torque, x = M xB for soft iron as compared to other materials,

= (50/)x(0-5i+3-0y) n. Assertion-Reason Type Questions


30. Here, Assertion is true but Reason is false as a
= 50x 0-5 (? X O + (50x 30) (i X j) magnetic dipole has north and south pole of equal
strength a little distance apart.

w
= 25(0) + 150ife =150iNm 31. Both Assertion and Reason are true and Reason
22. Area of the circular coil, A = 7C A The direction of is the correct explanation of Assertion because
a freely suspended magnetic needle comes to rest

Flo
A is perpendicular to the plane of area. Thus angle with its axis along the direction of magnetic field

e
and tangent at a point on the magnetic line of force

ree
between A and magnetic field B is 90®, i.e.,
tells the direction of magnetic field at that point
0 = 90®. Hence, magnitude of torque on the coil is

FFr
32. Here, Assertion is true but Reason is false as the
x = /A5sin0
soft iron has high retentivity but small coercivity.
urr
for
=/Xw X 5 X sin 90®
33. Here, Both Assertion and Reason are false, as
the magnetic moment of a bar magnet is equal to
kkss
the product of pole strength and magnetic length
1 of the magnet On dividing a bar magnet into two
Yo
ooo

23. V=^H equal parts, each part will act as a magnet of same
pole strength but with a magnetic length half of
B

. . y y 1 n the previous value. Thus the magnetic moment of


re

= tan — each piece of bar magnet will be M/2.


H vV3 S 6
34. Both Assertion and Reason are true and Reason
ou
ad

8 = 11/6 = 30® is the correct explanation of Assertion.


YY

35. Both Assertion and Reason are false as the given


24. A current carrying conductor experiences a force structure behaves as two magnets [one as a
in the magnetic field, F = B I LA moving charge
ndd

combination of magnets A and B and other magnet


Re

experiences a force in the magnetic field, C] with magnetic moment M each placed on each
F=^ V B sin 0. For a stationary charge u = 0, then
Fi

other.
force, F = 0. A rectangular current loop with its
plane parallel to the field, will have a magnetic
moment M, perpendicular to the plane of the loop
and thus experiences a couple =5 MB sin90®=M B.
Due to it, the current loop will be affected.

25. tanS = —=
H ^
H = 1 .-.8 = 45"

26. Knowledge based question.


27. Option (c) is incorrect because hysteresis loop
area depends on the thermal energy developed per
unit volume of the substance.
5/62
Fundamental Physics (XII) VOL.I

Refer to Fig. 5(Q).3, the magnetic moment of As


M^=Mb,
magnet A, = M, represented by OR The —*

magnetic moment of magnet B, MB = M, so Mb will bisect the angle POQ


represented by OQ.
i.e., ZPOT = 60" which is parallel to the magnetic
moment of magnet C with like poles in the same
direction.

w
Hence total magnetic moment of the system
= Mb + Mq
~M+M

e
= 2M

row
re
36. Both Assertion and Reason are true and Reason
is the correct explanation of Assertion.

eeF
ullo
FF
ZPOQ = 120". The resultant magnetic moment of 37. Both Assertion and Reason are false because
magnets A and B is magnetic field lines are continuous closed loops.
They emerge from A^-pole and enter into magnet at

srr
roF
Mr = ^|Ml+Ml+2M^MBCos 120" 5-pole and inside the magnet, they go from 5-pole

k
to A^-poIe of magnet.
uor
= ^Im^+M^ + 2M xMx(-l/2) ofof
= M 38. Both Assertion and Reason are true but Reason
is not the true explanation of Assertion.
kos
Y
Yo
eerBB
oo

PROBLEMS
urY

I. Magnetic moment and


3. What is meant by magnetic screening or
force between magnetic poles
ou

shielding ?
o
ad
d

1. Why is a current loop considered a magnetic Sol. Magnetic screening/shielding is the


nY

dipole ? phenomenon of protection of a region against


nid

Sol. Like a bar magnet/magnetic dipole, a current any external magnetic effects. For example,
Re

loop possesses magnetic moment M ~ NIA. In when a soft iron ring is placed in a magnetic
FFi

an external magnetic field, the current loop field, most of the lines are found to pass through
experiences a torque and aligns its axis parallel the ring and no lines pass through the space
to magnetic field. inside the ring. Thus space inside the ring is
2. Does the length of an iron bar change when shielded. Fig. 5(Q).4(a).
it is magnetised ?
FIGURE 5(Q).4
Sol. Yes, on magnetisation, the molecular magnets
are aligned parallel to the field. Therefore, the
length of the bar in the direction of B=0 B=0
magnetisation increases. This effect is called
magnetostriction effect. This effect is used for
producing ultrasonic waves. o
MAGNETISM AND MATTER 5/63

Superconductors also provide perfect magnetic screening as no magnetic lines of force pass through the
superconductor. Fig. 5(Q).4(b).
Magnetic screening has been used to protect cosily wrist watches from external magnetic fields by enclosing
them in a soft iron core.

4. Many of the Figs. 5(Q).5 show magnetic field lines wrongly (thick lines in the figs). Point out what is
wrong with them. Some of them may describe electrostatic field lines correctly. Point out which
ones . Remember we are talking of only static electric or magnetic field. (NCERT Solved Example)
Sol. (a) Wrong, because magnetic lines of force never emanate from a single point as shown. The field lines

oww
shown represent electric field of a single positive charge.
(b) Wrong, because magnetic field lines do not cross one another,
(c) Correct, magnetic lines of force are confined within a toroid. Field lines form closed loops as each loop
encloses a region across which a current passes.

e
(d) Wrong, field lines cannot be completely straight. The lines should curve out at the edges and form

re
closed loops,
(e) Correct, around both, the N-pole and S*pole, the net flux of the field Is zero.

FFllro
reF
(f) Wrong, these lines cannot represent a magnetic field because all the field lines cannot emanate from the

e
upper shaded (plate).
uoru
The given field lines infact show the electrostatic field lines between positively charged upper plate and

osFr
negatively charged lower plate,
(g) Wrong, magnetic field lines between two poles cannot be precisely straight at the ends. Some deviation

fkfor
of lines is inevitable, otherwise. Ampere’s law is violated. This is true al.so for electric field lines.
okso
Y
Yo
oo
BB
Y
r ree
oouu
ad
Yd
nidn
Re
FFi
5/64
^ Fundamental Physics (XII) VOL.I

5. (a) Magnetic field lines show the directions If monopoles existed, the magnetic flux would
(at every point) which a small magnetised no longer be zero, but equal to Pq times the pole
needle takes up (at that point). Do the strength enclosed by the surface, i.e...
magnetic field lines also represent the lines
of force of a moving charged particle at every y B .ds~ Pq/w
point ?
(</) No, there is no force or torque on an element
(b) Magnetic field lines can be entirely due to the field produced by that element itself.
confined within the core of a toroid, but not But there is a force (or torque) on an element of
within a straight solenoid. Why ?
the same wire. However, for the special case of
(c) If magnetic monopoles existed, how would a straight wire, this force is zero,
Gauss’s law of magnetism be modified ? (e) Yes, a system can have magnetic moment
(d) Does a bar magnet exert a torque on itself even if its net charge is zero. For example, every

w
due to its own field ? Does one element of a atom of para and ferromagnetic materials has a
current carrying wire exert a force on magnetic moment, though every atom is
another element of the same wire ? electrically neutral. Again, a neutron has no

Flo
(e) Magnetic field arises due to charges in charge, but it does have some magnetic moment.

e
motion. Can a system have magnetic moment (j) An iron nail is made up of a large number of

rree
even though its net charge is zero ? atoms, in which so many electronic charges are

r FF
(f) Magnetic force is always normal to the in motion. All these charges in motion experi
velocity of a charge and therefore does no
ence a magnetic force when held near a magnet.
uurr
work. An iron nail held near a magnet, when
released, increases its kinetic energy as it for
The magnetic forces do not change speed of the
charges, but they do change their velocity. The
velocity of centre of mass may increase at the
kkss
moves to cling to the magnet. What agency is
expense of nail’s internal energy. Thus internal
responsible for this increase in kinetic energy
Yo
oooo

if not the magnetic field ? energy of the nail is responsible for increase in
kinetic energy of the nail, as a whole.
eB

(NCERT Solved Example)


Sol. {a) We know, force on a charge q moving with U. Magnetic field due to bar magnet
r

avel, V in a uniform magnetic field of strength 6. What is meant by a magnetic field ? How is
ou
ad

—♦ it produced ?
B is
YY

Sol. A magnetic field is the space around a magnet


or the space around a wire carrying current. In
F = q{vxB)
nndd

which its magnetic effect can be felt.


Re

Magnetic force is always normal to B A magnetic field may be produced in many


Fi

ways. For example, (/) by a magnet (i7) by a


Magnetic field lines of 5 cannot represent the
current carrying conductor {Hi) by a moving
lines of force of moving charged particle.
charge (/v) by a varying electric field,
(6) Magnetic field lines can be entirely confined (displacementcurrent)
to the core of a toroid because toroid has no
ends. It can confine the field within its core. A III. Torque on magnet in
straight solenoid has two ends. If the entire magnetic field and its P.E.
magnetic flux were confined between these
7. A bar magnet is placed in a uniform magnetic
ends, the magnetic field lines will no longer be
continuous, field with its magnetic moment making an
angle 0 with the field. Write expressions for
(c) According to Gauss’s law in magnetism, torque on the bar magnet and potential
magnetic flux over any surface (closed or open) energy of magnet in this orientation. When
is always zero,/.e., (f, ~B .7s = 0 Is this energy minimum ?
(CBSE Sample Paper 2003)
MAGNETISM AND MATTER 5/65

SoK T = MB sin 0 original magnet but magnetic moment is halved


P.E. = - MB cos 6 when 0 = 0, because length is halved.
P.E. SI - MB cos 0°--MB = minimum When we cut along the length, pole strength of
Thus, P.E of magnet is minimum when its each new magnet is half the pole strength of
—♦ original magnet ; magnetic moment is also
magnetic moment M is aligned parallel to the halved, as length remains the same.
field B. (b) On melting, iron bar magnet loses its
8. A bar magnet of magnetic moment M is magnetism to some extent. This is because its
temperature exceeds Curie temperature
aligned parallel to the direction of a uniform
(= 750“C) for iron,
magnetic field B. Calculate work done to
align the magnetic moment (/) opposite to the (c) When a magnetised needle is put in a uniform
Held (u) normal to Geld direction ? magnetic field, forces on north and south poles
(CBSE 2001) of the needle are equal and unlike. Therefore,
net force is zero. But these forces form a torque

w
Sol. Work done in turning a dipole from orientation which aligns the magnetic needle in the direction
0j to 02 is given by of the field.
W -- MB (cos 02 - cos 0j) An iron nail is unmagneiised. It experiences

Flo
force of attraction; gets magnetised, then
(0 In aligning m opposite to magnetic field, experiences a torque and gets aligned along the

ee
01 =0, 02= 180° field.

Fr
IV = - MB (cos 180° - cos 0°) (d) No, it is not necessery that every magnetic
= -A/5(-1-1) = 2MB field configuration must have a north pole and
a south pole. The poles exist only when the source

for
ur
(/i) In aligning M perpendicular to magnetic has some net magnetic moment. For example, in
field, 0, =O°.02 = 9O° case of a toroid and infinite straight conductor
carrying cunent, there are no poles, as net magnetic
ks
W=-MB (cos 90°- cos 0°) = + MB
moment in both the cases is zero,
Yo
9. (a) What happens if a bar magnet is cut into
oo

two pieces (i) transverse to its length (e) Magnetic poles always exist in pairs.
However, one can imagine magnetic field
(ii) along its length ? (Pb. Board 2011)
B

configuration with three poles-when north poles


(b) What happens if an iron bar magnet is of two magnets are glued together or south poles
re

melted ? Does It retain its magnetism ? of two magnets are glued together to provide a
(c) A magnetised needle in a uniform three pole field configuration.
ou
ad

magnetic Geld experiences a torque but no (/) Place bar B horizontally on a table. Take bar
Y

net force. However, an iron nail near a bar A. Touch any one end of A at the middle of B. If
magnet experiences a force of attraction in A experiences no force then B is magnetised and
nd

addition to a torque, explain, A is unmagnetised.


Re

(d) Must every magnetic Geld conGguration However, if A experiences the same force at the
Fi

have a north pole and a south pole ? What middle of B as at the ends of B, then A is
about the Geld due to a toroid ? magnetised and B is not magnetised.
(e) Can you think of magnetic field
IV. Magnetism of Earth, Magnetic
conGguration with three poles ? elements and Neutral Points
(f) Two identical looking iron bars A and B
are given, one of which is deGnitely known 10. The angle of dip at a location in southern
to be magnetised. How would one ascertain India is about 18°. Would you expect a greater
whether or not both are magnetised ? If only or lesser dip angle in Britain ?
one is magnetised how does one ascertain Sol. Britain is located close to geographic north pole
which one? Use nothing else but the bars A of earth. Therefore, dip angle would be much
and B. (NCERT Solved Example) greater in Britain than the angle of dip in South
Sol. (fl) In both the cases, we get two magnets each India.
having north and south poles. 11. How many neutral points on a horizontal
When we cut transverse to the length, pole board are there when a magnet is held
strength of each new magnet is same as that of vertically on the board ?
5/66
4- Fundamental Physics (XII) VOL.I

Sol. There will be only one neutral point on the 18. The susceptibility of a magnetic material is
horizontal board. This is because field of earth - 4*2 X ItH. What type of material does it
is from south to north; and the field of magnetic represent ? (CBSE 2010)
pole on the board is radially outwards. At any Sol. As susceptibility is negative, the material must
point towards south of magnetic pole, field of be diamagnetic.
earth and field of magnetic pole will cancel out 19. Two substances A and B have relative
to give a neutral point.
permeabilities slightly greater and less than
V. Tangent law and Tangent unity respectively. What is their magnetic
nature ? (ICSE 2001)
Galvanometer
Sol. )i^= 1 +x or x = Hr- 1
12. A compass needle, pivoted about the horizontal For A, \ir> 1
axis and forced to move in the magnetic X js small and positive.
meridian is observed to point along (/) vertical
direction at place A, (Jo) horizontal direction at It must be paramagnetic.

ww
For B,
place B. What are the angles of dip at the two
places ? (CBSE Sample Paper 2004) X small and negative.
Sol. At place A, 8 = 90° and at place 6 = 0° Hence, B must be diamagnetic.

Flo
13. A compass box and a dip circle were taken to 20. How does the intensity of magnetisation of a
magnetic north pole of earth. What would paramagnetic material vary with increasing

e
reree
one observe with regard to the direction of applied magnetic field ? (CBSE 2006)
their needles and why ? Sol. At low field values, intensity of magnetisation

r FF
Sol. At north pole of earth, horizontal component increases almost linearly with field. At
of earth’s magnetic field is zero. Therefore, sufficiently high field values, intensity of
uurr
compass needle may point out in any random
foor
direction. The axis of dip needle would become
vertical as earth’s magnetic field is along the
magnetisation gets saturated becoming
independent of field.
ks s
vertical.
21. Why do magnetic lines of force prefer to pass
Yoo
through ferromagnetic materials ?
ooook

VI. Magnetic properties of materials (CBSE 2011)


eBB

Sol. This is because permeability (p) and sus-


14. Mention two characteristic properties of the ceptibility (Xm) such materials are very high.
material suitable for making core of 22. What is the basic use of hysteresis curve ?
transformer.
rr

(CBSE 2012)
Sol. The basic use of hysteresis curve lies in the
ouu
ad

Sol. Two core material of transformer must have selection of suitable materials for different
(/) high permeability (i7) low hysterisis loss.
YY

purposes like core of transformer/generator,


IS. Give two essential characteristics of a
electromagnets, permanent magnets etc. The
material used for preparing an electro* choice is made on the basis of properties like
nndd
Re

magnet. (CBSE 2004) retentivity, coercivity, energy loss etc. which are
Sol. The material used for preparing electromagnets revealed by the hysteresis loop.
Fii

must have (/) high permeability (n) low 23. A uniform magnetic field gets modified as
retentivity.
shown in Fig. 5(Q).6, when two specimens are
16. Why do we prefer to use the alloy alnico for
making permanent magnets ? (CBSE 2004) FIGURE 5(Q).6

Sol. This is because this alloy has high retentivity


and high coercivity. The only disadvantage is >
that alnico is brittle.
>

Vn. Classification of magnetic


materials
(a)
17. The susceptibility of a magnetic material ¥
is 1*9 X 10“^. What type of material does it
represent ? (CBSE 2011)
Sol. As susceptibility is positive, the material must (*)
be paramagnetic.
MAGNETISM AND MATTER 5/67

placed in it. (i) Identify the specimen A and 25. The susceptibility of a magnetic material is
B («) How is the magnetic susceptibility of 0*9853. Identify the type of magnetic
specimen A different from that of specimen material. Draw the modification of the field
B. (CBSE Sample Paper 2022-23) pattern on keeping a piece of this material in
Sol. (/■) In Fig. 5(Q).6(a), the specimen is a uniform magnetic Held. (CBSE 2018)

diamagnetic as magnetic lines of force prefer Sol. Here, Xm - 0‘9853. As magnetic su.sceptibility
not to pass through it. of the material is positive, though small, the
In Fig. 5{Q).6{b), the specimen is paramagnetic magnetic material must be paramagnetic.
as magnetic lines of force prefer its pass through When a piece of this material is placed in a
it. uniform magnetic field, the magnetic field lines
prefer to pass through the specimen (as shown
(//) Magnetic susceptibility of (a) is small in Fig. 5(Q).7 rather than through air. The
negative and that of (b) is small positive. magnetic induction B inside the sample is more

w
24. An iron bar magnet is heated to lOOC’C and than the magnetic induction Bq outside the
then cooled in a magnetic field free space. sample i.e., B> Bq. However, this enhancement
Will it retain magnetism ? (Raj. Board 2012) is small, being one part in 10^.

Flo
Sol. No, it will not retain magnetism. Curie FIGURE 5(Q).7
temperature for iron is about 750°C. At this Bo

ee
temperature, it will become paramagnetic and
lose its magnetism on heating further. Hence it

Fr
N S
will not retain magnetism on cooling in a
magnetic field free space. B > Bq

for
ur
● Very Short Answer
● Short Answer
ks
● Long Answer
Yo
oo

VERY SHORT ANSWER QUESTIONS Carrying 1 mark


eB

I. Magnetic moment and force 4. Magnetic lines of force are endless. Comment
between magnetic poles Ans. True. This is because magnetic lines of force
r

are closed continuous loops. These lines exist


ou

1. What are the SI units of pole strength and


ad

even inside the magnet.


magnetic moment ? (CBSE 2003)
Y

5. What is the significance of Gauss’s Law in


Ans. SI unit of pole strength is ampere-metre and SI
magnetism ?
unit of magnetic moment is ampere metre^.
nd

Ans. Magnetic poles exist in unlike pairs of equal


Re

2. Write the formula for magnetic moment of a


current loop. (CBSE 2002) strengths.
Fi

Ans. Af = NIA, when N is number of turns /, the 6. Give two examples of magnetic dipole.
current and A = area of cross-section of loop. Ans. Every atom of para and ferro magnetic
3. Distinguish between a magnetic dipole and substances; a loop of current are magnetic
an electric dipole. dipoles.
Ans.
7. On what factors does the pole strength of a
magnet depend ?
Magnetic Dipole Electric Dipole
Ans. The pole strength of a magnet may depend on
1. It consists of two its cross section ; nature and state of
1. It consists of two equal
equal and unlike and opposite charges magnetisation.
magnetic poles which which have a separate 8. Give one important point of distinction
do not have any existence. between magnetism and electricity.
separate existence.
Ans. Whereas in electricity, an isolated charge exists;
2. Poles of a magnetic 2. Charges of an electric in magnetism, an isolated magnetic pole does
dipole do not move. dipole can move. not exist.
5/68 7\txdcc^ <t Fundamental Physics (XII)
17. What is the approximate distance upto which
II. Magnetic field due to bar magnet earth’s magnetic field extends ?
9. Write an expression for magnetic field inten Ans. The magnetic field of earth extends to nearly
five times the radius of the earth \.e.
sity at any point an axial line of a bar magnet.
5 X (64 X 10^) km = 3-2 X 10** km.
2Md
Ans. B,1 - =— ; along SN 18. What are isogonic, isoclinic and isodynamic
4tu(^/2_/2)2 lines ?

where symbols have usual meaning. Ans. Isogonic lines are the lines joining places of
equal declination.
10. On equatorial line of bar magnet, what is the
Isoclinic lines are the lines joining places of
formula for magnetic field strength ?
equal dip/inclination.
M Isodynamic lines are the lines joining places of

ww
Ans. parallel to NS.
47t (^/2+/2)3/2 equal horizontal component of earth’s field.
19. What is the maximum value of angle of dip ?
III. Torque on magnet in magnetic At what place does it occur ?

FF loo
field and its P.E. Ans. 9(f, at earth’s magnetic poles.
20. What is the angle of dip at a place where

ee
11. A magnetic dipole is situated in the direcion horizontal and vertical components of earth’s

ee r
of a magnetic field. What is its potential held are equal ?

rFrF
energy ? If it is rotated by 180°, then what V
amount of work will be done ? Ans. 45°. This is because tan8=—=1 ; 6 = 45°.
rur
H
Ans. P.E. of dipole = - MB cos 0° = - MB
Work done = MB (cos 0° - cos 180°)
ffoor
21. The angles of dip at two places are
respectively 0° and 90°. W’here are these
ks
= MB(l + 1) values on earth ? (CBSE 2011)
YYoou
= 2 MB.
Ans. At magnetic equator, angle of dip = 0° and at
ookos

12. What is the strength of earth’s magnetic field earth’s magnetic poles, angle of dip = 90°.
BBo

at the surface of earth ?


22. In the northern hemisphere, do magnetic
re

Ans. It is of the order of 10”^ tesla. lines of force due to earth’s field point to
13. What is the torque acting on a magnet of wards or away from earth ? (Hr. Board 2002)
oouur
ad

magnetic moment M held at an angle 6 with Ans. Towards the earth because the artifical magnet
Y

the direction of magnetic field B ? inside the earth has south pole on that side.
Ans. X = MB sin 0 23. Name the elements or parameters of earth’s
dndY

magnetic field. (Hr. Board 2002)


Re

14. What does the torque do ?


Ans. The torque tries to align the magnet along the Ans. The three parameters of earth’s magnetic field
FFini

direction of magnetic field (6 —> Zero). are : Declination, Dip and Horizontal
component of earth’s magnetic field.
IV. Magnetism of Earth, Magnetic
elements and Neutral Points
V. Tangent law and Tangent
Galvanometer

15. Where is the vertical component of earth’s 24. What is a reduction factor of tangent gal
magnetic field zero ? vanometer and its unit of measurement ?
Ans. At earth’s magnetic equator. Ans. Reduction factor of a tangent galvanometer is
16. A magnetic needle placed on a piece of cork equal to current, required to be passed through
is floating on the calm surface of a lake in galvanometer to produce a deflection of 45°.
the northern hemisphere. Would the Reduction factor is measured in ampere.
magnetic needle alongwith the cork move 25. Before using tangent galvanometer for the
towards north ? measurementof current, why is the plane of
Ans. No, the magnetic needle will come to rest along coil of tangent galvanometer set in the
N-S direction. magnetic meridian ?
MAGNETISM AND MATTER
5/69

Ans. When plane of the coil of tangent galvanometer


is set in the magnetic meridian and current is Vll. Classification of magnetic
materials
passed through the coil, then the magnetic field
produced due to current in coil is perpendicular 35. Which of the following substances are
to the plane of coil in East-Westdirection. The diamagnetic ? (CBSE 2013)
horizontal component of earth is in South-North Copper, Aluminium, Sodium, Bismuth
direction. Then the magnetic needle of tangent Ans. Copper and Bismuth are diamagnetic.
galvanometer will obey tangent law, and current 36. Which of the following substances are

w
(/) can be measured by noting the deflection 0. paramagnetic ? fCBSE 2013 (C)}
AI, Bi, Cu, Ca, Pb, Ni.
VI. Magnetic properties of materials Ans. Aluminium and calcium are paramagnetic.
26. What is the order of magnetic moment of an 37. The permeability of Bismuth is 0*9983. To

e
roow
atom ? which class of magnetic material Bi belongs.

re
(CBSE 2011)
Ans. Magnetic moment of an atom is of the order of
10“^^ Am^. Ans. Bismuth is diamagnetic as its permeability is
less than one.

reF
uFFll
27. What are the SI units of magnetising force 38. What is the basic difference between the

e
or magnetising intensity ? atom/molecule of a diamagnetic and
paramagnetic material ?
Ans. SI units of magnetising force or magnetising

sFr
intensity are Ampere/metre. (CBSE Sample Paper 2008)

foro
Ans. The atoms/molecules of a diamagnetic
28. Name the cgs unit of magnetising Intensity.
ofk
uor
substance do not possess any net magnetic
How is it related to SI unit of intensity.
moment on their own. In paramagnetic
Ans. The cgs unit of magnetising intensity H is substances, each individual atom/molecule has
kos
oersted.
non zero magnetic moment of its own.
Y
Yo
1 oersted = 80 Am“* 39. Why are electromagnets made of soft iron ?
reeBB
oo

29. What is relative magnetic permeability of (Kerala Board 2012)


Ans. This is because coercivity of soft iron is small.
uurY

superconductors ?
Ans. For superconductors, = 0. 40. Magnetisation and demagnetisation of soft
iron is easier/more difficult as compared to
30. What is magnetic susceptibility of super steel. Why ?
ad

conductors.
doo

Ans. Magnetisation and demagnetisation of soft iron


nY

Ans. As [1^ = I + = 0, for superconductors, is easier compared to steel, because coercivity,


■ ● ~~ 1 of soft iron is smaller than the coercivity of steel.
nid

41. Classify the following into dia and para


Re

31. What is Meissner effect ?


magnetic substances : aluminium, copper,
F
Fi

Ans. It is the phenomenon of perfect diamagnetism water, mercury, oxygen, hydrogen.


of superconductors. Ans. Dia : copper, water, mercury, hydrogen.
32. What are units of magnetic permeability ? Para : aluminium, oxygen.
Ans. Tesla metre/ampere. (Tm A"*) 42. Write the names of three ferromagnetic
substances. (Kerala Board 2012)
33. How does the magnetic susceptibility of a Ans. Iron, cobalt and nickel.
paramagnetic material change with
43. Magnetic moment of atoms of certain
temperature ?
materials is zero. Name such materials.
Ans. The magnetic suscetibility of a paramagnetic Ans. Diamagnetic materials, like copper, bismuth etc.
material varies inversely with temperature, i.e., 44. Which materials have relative magnetic
Xm OC MT.
permeability > 1 ?
34. What are SI units of magnetic susceptibility ? Ans. Para and ferromagnetic materials.
-1 45. Can there be a material, which is non
/ Am
Ans. X in =1 X/n 1*0 units. magnetic ?
-1
H A m
Ans. No, every substance is atleast diamagnetic.
5/70 ‘P'uztiee^'A Fundamental Physics (XIDCSIS

SHORT ANSWER QUESTIONS Carrying 2 marks

I. Magnetic moment and force III. Torque on magnet in magnetic


field and its P.E.
between magnetic poles
1. How is magnetic force between two poles 6. Under what situation a magnet suspended in
affected when strength of each pole is dou> a uniform magnetic Held will be (a) in stable
bled and distance between them is halved ? equilibrium and {b) in unstable equilibrium ?
Ans. Force becomes 16 times. This is because Ans. When axis of the magnet of magnetic moment
M makes an angle 0 with the direction of
m^m:^ fiQ (2m,)(2m2) magnetic field B, then torque on magnet is
F — o—
4ji 471 (r/2)2 X = MB sin 6

ww
= 16F and its PE, U = - MB cos 0.
2. No two magnetic lines of force can intersect. (a) The magnet will be in stable equilibrium if
1 = 0 and U is minimum. It will be so if the
Why ?

FF loo
Ans. This is because at the point of intersection, there magnetic moment of magnet acts in the direction
will be two tangents to the two lines of force, of magnetic field, i.e., 0 = 0°.

ee
which would mean two directions of magnetic ib) The magnet will be in unstable equilibrium
if X = 0 and U is maximum. It will be so if the

ee r
field intensity at that point. This is not possible.

rFrF
3. Can ever there be a magnet (a) with no pole magnetic moment of magnet acts in opposite
(b) with two similar poles (c) with three direction to that of magnetic field.
7. When will a magnet in an external magnetic
rur
poles ?
Ans. (a) Yes, there can be a magnet with no pole e.g.
in case of a toroid carrying current.
ffoor field be in unstable equilibrium ?
Ans. The magnet is in unstable equilibrium, when
ks
(b) Yes, there can be a magnet with two similar fii and B are directed opposite to each other
YYoou
ookos

poles when like poles of two magnets are glued i.e., when 0 = 180°. Torque = MB sin 180° = zero
together, and P.E. - - MB cos 180° = MB = maximum.
BBo

(c) Yes, there can be a magnet with three poles,


again when like poles of two magnets are glued
re

IV. Magnetism of Earth, Magnetic


together.
elements and Neutral Points
oouur

4. Define unit pole from Coulomb’s law of


ad

magnetism. 8. What is the proof of earth’s magnetism ?


Y

Ans. From Coulomb’s law of magnetism. Ans. It is based on the followingevidences ;


(z) A magnet suspended from a thread and free
dndY

F =
Re

4n to rotate in a horizontal plane comes to rest along


If z/ij = m2 = zti; r = 1 m and F= 10"^ N. the north-south direction,
FFini

mxni (zz) When a soft iron piece is buried under the earth
Then 10-'^=10-^
l2 in the north-south direction, it is found to acquire
or m^ = 1 or zn = ± 1 the properties of a magnet after some time.
Hence, unit magnetic pole is that pole which {Hi) When we plot magnetic field lines of a
when placed in free space at a distance of 1 m magnet, neutral points are obtained.
from a like pole of same strength would repel it 9. Is there a strong’ magnet inside the earth
with a force of 10“^ N. responsible for earth’s magnetism ? If there
is a magnet, what is its inclination w.r.t to
II. Magnetic field due to bar magnet north-south direction ?

5. For a short magnetic dipole, intensity at any Ans. Actually there is no permanent strong magnet
point an axial line is same as intensity at same inside the earth responsible for earth’s
distance on equatorial line. Is it true ? magnetism. However to explain the earth’s
Ans. No, the statement is not true. Infact, at a given magnetism. Dr. Gilbert considered a fictitious
distance from the centre of magnet, intensity on strong magnet inside the earth, whose magnetic
axial line is twice the intensity on equatorial line. axis is inclined roughly 11-3° west of axis of
MAGNETISM AND MATTER 5/71

rotation of earth, which is along geographic


north and geographic south. FIGURE 5(Q).8

10. At what positions, the neutral points will lie


Cl I C2
for a bar magnet when magnetic axis of
A B
magnet is lying in the magnetic meridian
(/) with N-pole of magnet pointing North
(h) with S-pole of magnet pointing North ?
Ans. (0 The neutral points will lie on equatorial line Net magnetic moment of combination
of bar magnet of magnetic moment M and = MI2 - MU
magnetic length 2 I, where = 0.
M
H = FIGURE S(Q).9

ooww
47C(iy2 + /2^3/2 A
S N Ci
where d is the perpendicular distance of neutral
N - S C2
point from the centre of magnet. B

(i7) The neutral points will be on axial line of


15. Two circular loops of radii r and 2 r have
bar magnet, where

e
currents I and HI flowing through them in

ree
IIq 2M d

rFl
H = clockwise and anticlockwise sense respec*

Fre
47l(f/2_/2j2 lively. If their equivalent magnetic moments
are Mj and M2, what is the relation between

rrF
11. How does a magnetic compass behave at a
neutral point ? Af| and M2 ? (CBSE Sample Paper 2011)
Ans. A/, =/(7Cr^)
ouur
Ans. At a neutral point, net magnetic field due to
magnet and e<irth is zero. Therefore, a compass
needle may point out in any random direction.
sffoo M2 = ^ (n4r^) = I (2jtr^)
okks
Yo
V. Tangent law and Tangent
ooo
Y

Galvanometer
BB

12. Write mathematical form of tangent law in


rr e

magnetism. The direction of M^ is opposite to the direction


ouu

Ans. F=H tan 0, where F and H are strengths of two


of M2.
ad
Y

magnetic fields acting perpendicular to each


dY

other and 0 is the angle which freely suspended 16. State two methods to destroy the magnetism
magnet makes with the direction of//. of a magnet.
innd

13. To work with a tangent galvanometer, how Ans. (i) By heating the magnet.
Re

do you set its coil ? (i7) By applying magnetic field in the reverse
Fi
F

Ans. The coil of tangent galvanometer is set in direction.

magnetic meridian by making it parallel to the 17. Define the term : magnetic dipole moment of
magnetic needle in the magnetometer box. a current loop. Write the expression for the
magnetic moment when an electron revolves
VI. Magnetic properties of materials at a speed v around an orbit of radius r in
hydrogen atom. (CBSE 2008)
14. A bar magnet AB is cut into two equal parts
one part is kept over the other so that pole Ans. A current carrying loop behaves as a system of
C2 is above Cj. If M is magnetic moment of two equal and opposite magnetic poles
original magnet, what will be the magnetic separated by a distance. Hence it behaves as a
moment of the combination so formed ? magnetic dipole. Magnetic dipole moment of
(CBSE Sample Paper 2008) current loop is the product of current / and area
A enclosed by the loop of current, i.e.
Ans. As shown in Fig. 5(Q).8, each part has magnetic
moment M/2. These are aligned in opposite M = !A = —xjtr^= —-— X 7C =
evr

directions, Fig. 5(Q).9. T 2kt/v 2


5/72 “Pn^ndee^ a Fundamental Physics (XII)

In a hyrdogen atom, when an electron revolves FIGURE 5(Q).11


at a speed v around an orbit of radius r, the
magnetic moment is given by
>

f eh Nickel
M =n
47im Antimony
>

where e is charge on electron, m is mass of


Aluminium
electron ; n denotes the number of orbit and h
is Flack’s constant. 22. What is the susceptbillty and permeability
of a perfectly diamagnetic substance ?
VII. Classification of magnetic (Kerala Board 2012)
materials
Ans. For a perfectly diamagnetic substance,

w
18. A uniform magnetic field gets modified as B = po(//+/) = 0 I = ~H.
I
shown in Fig. 5(Q).10. When two specimens Therefore, Xm = -l
X and Y are placed in it. Identify the H

Flo
specimens X and Y. (CBSE 2013) Also, = 1 + = 1 - 1 =0
^ = Po = Zero.

ee
FIGURE 5(Q).10
23. Commenton the state of magnetisationof a

Fr
substance whose atoms contain odd number
X
of electrons.

for
Ans. An atom which contains odd number of
ur
electrons will have atleast one electron left
Y unpaired. So it has some permanent magnetic
ks
m dipole moment. Hence the atom must have a
Yo
paramagnetic character.
oo

24. Which material is used to make electro


B

Ans. X is diamagnetic as it expels magnetic lines magnets and why ? (CBSE 2017, 2010)
re

Y is paramagnetic or ferromagnetic as it pulls Ans. Soft iron is used to make electromagnets,


the magnetic lines into it. because hysteresis loop for soft iron is narrow.
ou
ad

19. How are wrist watches protected from Therefore, energy loss/volume/cycle is small.
powerful magnets ? 25. Which materialis used in making permanent
Y

Ans. The machine parts of a wrist watch are magnets and why ? (CBSE 2017, 2010)
nd

surrounded by an iron ring, which acts as a Ans. For making permanent magnets, a material
Re

screen against external magnetic fields. having high coercivity is used. Steel is a better
Fi

20. Why does the magnetisation of a choice compared to soft iron.


paramagnetic salt increase on cooling ? 26. What is the basic difference between the
Ans. At lower temperatures, tendency to disrupt the atom/molecule of a diamagnetic and a
alignment of dipoles with applied magnetic field paramagnetic material ? Why are elements
arising from random thermal motion is reduced. with even atomic number more likely to be
21. Three identical specimens of magnetic diamagnetic ?
materials nickel, antimony and aluminium Ans. Atoms/molecules of diamagnetic substances
are kept in a non-uniform magnetic field. have electrons that can be grouped in pairs with
Draw the modiHcations In the field lines in spins in opposite sense. In paramagnetic
each case. Justify your answer. (CBSE 2010) materials, atoms/molecules have excess
Ans. The modifications in field lines are as shown in electrons which spin in the same sense.
Fig. 5(Q).ll. This is because. Nickel is Elements with even atomic number have
ferromagnetic, Antimony is diamagnetic and electrons which can be grouped into pairs with
Aluminium is paramagnetic. spins in opposite sense. When external field is
MAGNETISM AND MATTER 5/73

applied, the electrons in each pair get equal Ans. For material A, > 1. Therefoe, ‘A’ must be
magnetic moments opposite to each other, paramagnetic.
making the resultant magnetic moment zero. For material S, < 1. Therefore, 'B' must be
Therefore, such elements are more likely to be diamagnetic.
diamagnetic.
As =
27. Suppose you have two bars of identical
Susceptibility of ‘A’ is positive and
dimensions, one made of paramagnetic
substance and the other of diamagnetic susceptibility of 'B' is negative.
substance. If you place these bars along a uni 29. (a) Define the term magnetic susceptibility
form magnetic field, show diagramatically and write its relation in terms of relative
what modification in the field pattern would magnetic permeability.
take place in each case. [CBSE 2017 (C)] {b) Two magnetic materials A and B have
Ans. Inside a paramagnetic bar, field concentrates relative magnetic permeabilities of 0*96 and
slightly in the bar, Fig. 5(Q). 12(a). Inside a 500. Identify the magnetic materials A and B.

w
diamagnetic bar, magnetic field lines avoid to [CBSE 2018 (C)]
enter. This is shown in Fig. 5(Q).12(/?).
Ans. (fl) Magnetic susceptibility is a property which

Flo
FIGURE S(Q).12 determines how easily a specimen can be
magnetised, when placed in the magnetising

reeee
field.

FFr
o >●
Magnetic susceptibility of a material is defined
Paramagnetic
^ ^ ^ ^ as the ratio of intensity of magnetisation (/)
induced in the material to the magnetising force

for
ur
(//) applied on it. It is represented by % m'

_--r- I
kkss
i.e., Xm
o H
Yo
We can show that relative magnetic permeability
oo

Diamagnetic (p,,) is related to (x^) by the relation


eB

28. Out of the two magnetic materials. ‘A’ has l^.= l+X m

relative permeability slightly greater than (/>) For magnetic material A, p^ = 0-96 < I
r

unity while ‘B’ has less than unity. Identify .●. A must be diamagnetic in nature.
ou
ad

the nature of materials ‘A’ and ‘B’. Will their


For magnetic material B, p^= 500 which is high.
susceptibilities be positive or negative ?
YY

Therefore, B must be a ferromagnetic material.


(CBSE 2014)
ndd
Re

SHORT ANSWER QUESTIONS Carrying 3 marks


Fi

1. Discuss the basic properties of a magnet. 5. Draw the magnetic field lines due to a circular
(Pb. Board 2011) [Art. 5.3]
loop of area A carrying current/. Show that it
2. What are magnetic lines of force ? Give their
acts as a bar magnet of magnetic moment
important properties. [Arts. 5.4 & 5]
3. What is the magnetic dipole moment of a current M =IA (CBSE 2015) [Art. 5.8]
loop ? In what direction does it act ? 6. Derive an expression for the magnetic field due
(West Bengal Board 2012, Karnataka Board 2012) to a solenoid of length 2 /, radius a, having n
[Art. 5.6] number of turns per unit length and carrying a
4. What is magnetic dipole moment ? Calculate steady current I, at a point on the axial line,
distant r from the centre of the solenoid. How
the magnetic dipole moment of a revolving
electron. (Jharkhand Board 2011) does this expression compare with the axial
[Arts. 5.6 & 5.8]
magnetic field due to a bar magnet of magnetic
moment M ? (CBSE 2015) [Art. 5. 9]
5/74 pftatUefi. ^ Fundamental Physics (XII) EE3MI
7. In a current carrying solenoid, current is flowing 15. Deduce an expression for the magnetic dipole
clockwisefrom left end. Draw the magneticfield moment of an electron orbiting around the
produced in it. (Raj. Board 2011) [Art. 5.9] central nucleus.

8. Define strength of magnetic field. Obtain an (Jharkhand Board 2011, Raj. Board 2011,
expression for strength of magnetic field on the CBSE 2010, 2009)
axis of a bar magnet.
[Art. 5.23]
(Chhatisgarh Board 2011) [Art 5.7]
9. Derive an expression for torque acting on a bar 16. Explain magnetic permeability, magnetising
magnet held at an angle with the direction of a force and magnetic field induction. State the
uniform magnetic field. relation between them. [Refer to Art 5.24]
(AP Board 2012, Jharkhand Board 2012) 17. Write three points of differences between para-,
[Art 5.10] dia- and ferro-magnetic materials, giving one
(CBSE 2019) [Art. 5.26]

ww
10. Calculate potential energy of a magnetic dipole example of each.
in a magnetic field. 18. Explain paramagnetism on the basis of electron
(Jharkhand Board 2012) [Art. 5.12] theory. (AP Board 2012) [Art. 5.26]
11. Find expression for work done in rotating a bar 19. State and explain Curie law in magnetism.

Flo
magnet in a uniform magnetic field. [Art. 5.27]

e
(Chhatisgarh Board 2012) 20. Define the term retentiviiy and coercivity.

eree
[Arts. 5.12] (Hr. Board 2011) [Art. 5.28]

FFr
12. What are magnetic elements at a place ? Define 21. What is meant by hysteresis loss ? Where is the
them. (Uttarakhand Board 2012) study of hysteresis used ?
uurr
[Art. 5.18]

orr
[Arts. 5.28, 5.30]
13. (a) State Gauss’s law for magnetism. Explain its
significance.
sfo
22. Explain magnetic properties of soft iron and
steel. (UttarakhandBoard 2012) [Art. 5.29]
kks
Yoo
(b) Write the four important properties of the 23. Discuss briefly the uses of ferromagnetics.
oooo

magnetic field lines due to a bar magnet. [Art 5.30]


(CBSE 2019} [Art. 5.14, 5.5] 24. Write any two differences between electro
eBB

14. State tangent law in magnetism. How will the magnet and permanant magnet.
reduction factor of a tangent galvanometer (Bihar Board 2012)
uurr

change when the number of turns of its coil is [Art. 5.30]


doubled ? Explain.
ad

25. Mention two characteristic properties of the


Yo

(AP Board 2012, Chhatisgarh Board 2012) material suitable for making core of a
dY

[Arts. 5.21 & 5.22] transformer. (CBSE 2012) [Art 5.30]


innd
Re

LONG ANSWER QUESTIONS Carrying 5 or more marks


Fi

1. Depict the field line pattern due to a current 4. State and prove tangent law in magnetism. Give
carrying solenoid of finite length, two factors by which tangent galvanometer can
be made more sensitive. [Arts. 5.21 & 22]
(0 In what way do these lines differ from those
due to an electric dipole, 5. Describe the principle, construction, theory and
(/i) Why can’t two magnetic field lines intersect working of a tangent galvanometer.
each other ? (CBSE 2011) [Art 5.4] [Art 5.22]
2. What are the magnetic elements of earth ? 6. Define magnetising field, magnetic induction,
Explain them briefly. (Raj. Board 2012) permeability, intensity of magnetisation and
[Art. 5.18] susceptibility. Establish relation between
permeability and susceptibility.
3. Explain what is meant by neutral points ? How
[Arts. 5.24 & 25]
will you calculate magnetic moment of a bar
magnet by locating the neutral points ? 7. Distinguish between the magnetic properties of
[Art 5.20] dia, para and ferromagnetic substances in terms
of
5/75
MAGNETISM AND MATTER

(0 susceptibility 8. What are dia, para and ferro magnetic


materials ? Discuss their important properties.
(h) magnetic permeability and
(HP Board 2011) [Art. 5.26]
(i/7) Coercivily. Give one example of each of 9. Discuss briefly electron theory of magnetism.
these materials.
How does it account for the three kinds of
Draw the field lines due to an external magnetic behaviour of magnetic substances ?
field near a [Art. 5.26]
(i) diamagnetic 10. What is meant by hysteresis ? Discuss briefly
(//■) paramagnetic substance. the dissipation of energy due to hysteresis. Draw
(CBSE2007) [Art 5.26] hysteresis curves of soft iron and steel.
(Raj. Board 2011) [Arts. 5.28 & 291

CASE-BASED VERY SHORT/SHORT QUESTIONS

ww
CASE 1. According to molecular Theory, every through the loop, A is area enclosed by the loop, n is
—^

molecule of a magnetic substance (whether magnetised unit vector along M .


or not) is a complete magnet in itself. In an unmagnetised

Flo
Based on the above paragraph, answer
substance, molecular magnets are randomly oriented

ee
questions no. 1 to 4 :
such that they form closed chains. That is why the
1. What is the magnitude and direction of magnetic

rere
resultant magnetism of the unmagnetised specimen is

rFF
zero. On magnetising the specimen, the molecular dipole moment of current loop ?
magnets are realigned in tlie direction of the field. 2. Which face of current loop behaves as a 5-pole
uur r
of magnetic dipole ?
Magnetic field lines are hypothetical lines, which
enable us to understand certain phenomena in
magnetism. Tangent to field line at a point gives us the
foor3. When a bar magnet of magnetic moment M is
placed along the direction of a uniform magnetic
ks s
field B, then what are the total force and torque
Yoo
ook

direction of magnetic field B at that point. No two on the bar magnet ?


magnetic field lines intersect eachother. 4. What is the work done in rotating the magnetic
BBo

A magnetic dipole consists of two unlike poles dipole from equilibrium position to 180° in a
re

of equal strength(m) separated by a small distance uniform magnetic field ?


ouur

(2 /). It has a magnetic dipole moment M = m(2 /) CASE 2. According to Gauss’s theorem in
ad

directed from south pole to north pole of dipole magnetism, surface integral of magnetic field over a
YY

through dipole. surface, closed or open is always zero, i.e..


nndd

S.I. units of magnetic moment M are


Re

() B d 5=0
ampere meter^ or joule/tesla.
FFii

When a bar magnet of magnetic moment M is held This theorem establishes that magnetic poles
at Z9 with the direction of uniform magnetic field B , always exist in equal and unlike pairs.
a torque x = MB sin 0 acts on the magnet. It tries to Magnetism of earth is called terrestrial
align the dipole along the field. Work (W) is required to magnetism. It is described in terms of three quantities,
be done in rotating the dipole against the action of the called magnetic elements of earth.
torque. This work is stored as potential energy of the (/) Magnetic declination (0) at a place is the angle
dcpole. between magnetic axis (or magnetic meridian) and
P.E. = W = -MB (cos 02 - cos 0i) geographic axis (or geographic meridian).
Usually. 01 = 90°, which is the position of zero («*) Magnetic inclination or dip (5) at a place is
potential energy. the angle, which direction of total earth’s magnetic field
A loop of current acts as a dipole of magnetic makes with a horizontal line in magnetic meridian.
moment M =IAn , where / is the strength of current
(kO Horizontal Component of earth’s magnetic
field (H). It is the component of earth’s total magnetic
5/76
Fundamental Physics (XII) VOL.I

field (R) in horizontal direction in magnetic meridian, CASE 3. Neutral Points are the points where the
clearly,
resultant magnetic field due to field of tlie bar magnet
H = RcosS, V = /?sin 6 and the field of earth is zero.
V
= R and — = tan 8 When a magnet is placed with its north pole
H
towards geographic south, the neutral points lie on axial
At magnetic equator, 5 = 0® line of the magnet. At each neutral point,
and at magnetic poles, 5 = 90®.
_^o 2Md
Magnetic field strength (Bj) due to a bar magnet B
1
4n
-H
at any point on axial line of magnet.

B,1 =
2Md Here, H is horizontal component of earth’s
; along SN produced. magnetic field.
471 (£/2_;2j2
Magnetic field strength (Bj) due to a bar magnet When a bar magnet is placed with its north pole

ww
at any point on equatorial line of bar magnet. towards geographic north, neutral points lie on

equatorial line of the magnet. At each neutral point


M
B —

Flo
^ 471 +/2^3/2 .^0 M
= H
4tc (d2+/2)3/2

e
The direction of 5^ is along a line parallel to NS.

rree
Based on the above paragraph, answer

r FF
Based on the above paragraph, answer questions no. 9 to 12 :
questions no. 5 to 8 :
9. What are neutral points ?
uurr
5. With usual notation, o ^ds
does it represent ?
= zero. What
for
10. To obtain neutral points on axial line of a bar
magnet, how will you place the magnet ?
kss
6. What are magnetic elements of earth? 11. On axial line or equatorial line of a bar magnet,
ooook
Yo
how does the field due to magnet vary ?
7. What are the values of magnetic inclination at
poles and at equator ? 12. What are the directions of magnetic fields due to
eB

bar magnet and Earth at neutral points when


8. What is the ratio of magnetic field strength at
any point on axial line and on equatorial line of (/) point is on the axial line of bar magnet
rr

a short bar magnet ? («) point is on the equatorial line of bar magnet.
ou
ad
Y

ANSWERS
dY

1. Magnitude of magnetic dipole moment of current 4.


Workdone, V/=-MB (cos 02 - cos 0j)
Re
innd

loop, M = nIA, where n is the number of turns in


Here, 01 =0®
the current loop, / is the current in loop and A is
Fi

and
the area of each turn of the loop. 02 = 180®, so
—> W = MB [cos 180“ - cos 0®]
Direction of M is from 5-pole to N-pole through
the dipole.
= 1MB
2. That face of current loop will show south polarity 5.
where the direction of current in the loop is It represents Gauss’s theorem in magnetism.
clockwise. 6. Three magnetic elements of earth are

3. Let m be the pole strength of each pole and M be (/) Magnetic declination (0).
the magnetic moment of the bar magnet. («) Magnetic inclination or magnetic dip (5).
Total force on the bar magnet in magnetic field. {Hi) Horizontal component of earth’s magnetic
= mB - mB = 0 field {H).
7.
Torque on the bar magnet in the magnetic field At magnetic poles, 5 = 90® and at magnetic equator,
= MB sin 0® = 0. 5 = 0®.
5/77
MAGNETISM AND MATTER

8. For a short bar magnet d>>l> so magnetic field 10. To obtain neutral points on axial line of a bar
strength due to short bar magnet at any point on magnet, the magnet must be placed with its north
the axial line of short bar magnet is pole towards geographic south.
11. On axial line or equatorial line of a short bar
Itp 2Md liplMd _ lip 2M magnet, magnetic field due to the magnet varies
^1 = Md^ where d is distance of the point from the
471 ((/2-/2)2 “ 47t 4tc d^ as

centre of the magnet.


Magnetic field strength at any point on equatorial 12. (/) When neutral points are on the axial line of a
line of short bar magnet is bar magnet, then the north pole of bar magnet is
towards geographic south, then at neutral points,
^^0 M ^^0 M
Ho ^ the direction of magnetic field due to bar magnet
4tc {d^f^ 4ti d^ is north to south direction and the direction of
earth’s magnetic field is from south to north
direction.
B
-J- = 2 («) When neutral points are on the equatorial line

ww
of bar magnet, then the north pole of magnet is
towards geographic north, then at neutral points,
9. Neutral points are the positions where resultant the direction of magnetic field due to bar magnet

Floo
of magnetic field due to the bar magnet and due is north to south direction and the direction of
to earth cancel out so that net magnetic field is earth’s magnetic field is from south to north

ee
zero. direction.

reer
rFF
CASE-BASED MCQs AND ASSERTION-REASON QUESTIOj^

neucleus, therefore, every atom behaves as magnetic


uur r
CASE 1. As electrons in an atom revolve around the

diple. Atomic dipole moment is M =


n(eh)
4nm
ffoor
sks
where n = I, 2, 3 denotes the number of orbit
YYoo
ooko

eh
and - = least value of dipole moment of atom = 9-27 x 10 Am^ — Bohr magneton.
eBB

Aitm

Some important terms used in magnetism are :


(0 Magnetic field strength or magnetic field induction or magnetic flux density (B). It is measured in tesla
r
ouur

or gauss, where 1 tesla = lO** gauss.


ad

(h) Magnetising force or Magnetising intensity {H) = n /. It is measured in Am *.


YY

(m) Relative Magnetic permeability 14^= —


® =iL . It has no units.
nndd

B,0 ^^0
Re

M
FFii

(iv) Intensity of magnetisation, I = — =


It is measured in Am"*, same as the unit of magnetising force.

(v) Magnetic susceptibility (X-) = 77 . It has no unit.


ti

Important relations are, B = |Iq (// + /); P-r = ^ Xm-


Based on the above paragraph, answer questions no. 1 to 4 :
1. Least value of dipole moment of an atom is called Bohr magneton, whose value is
(a) 9-27 X lO^'* Am^ ib) 9-27 X 10-24 Am^
(c) 9-27 X 10"*^ Am^ {d) 9-27 X 1023 Am2
2. Tesla is SI unit of

(a) magnetic permeability ib) magnetic field strength


(c) intensity of megnetisation {d) magnetic susceptibility
5/78
^ Fundamental Physics (XII) VOL.I

For (^esUon No. 3 and 4, we have given two 3. Ferromagnetic substances are those in which
statements each, one labelled as Assertion (A) each individual atom/molecule/ionhas some net
and other labelled as Reason (R). magnetic moment of its own. They are magnetised
Cho(»e the correct option out of the four options strongly in the direction of the magnetising field in
given below: which they are placed. For example, iron, cobalt, nickel
(a) Both, A and R are true and R is correct and a number of their alloys. Ferromagnetic materials
explanation of A
exhibit all the properties of paramagnetic materials to a
much greater degree. At a certain temperature,
{b) Both, A and R are true, but R is not a correct ferromagnetics pass over to paramagnetics. This
explanation of A temperature is known as Curie Temperature. For soft
(c) A is true but R is false iron, curie temperature is 1000 K.
(d) Both, A and R are false. The three kinds of behaviour has been explained
on the basis of electron theory of magnetism.
3. Assertion (A). Relative magnetic permeability Based on the above paragraph, answer
has no units.
questions no. 5 to 8 :

w
Reason (R). This is beacuse = —, where 5. Which of the following is not diamagnetic ?
symbols have standard meaning. ^0 (a) Bismuth (b) Antimony

Flo
4. Assertion (A). Magnetising intensity (H) and (c) Quartz (d) Iron
intensity of magnetisation (/) have same units. 6. Which of the following is not magnetised

reeee
Reason (R). The two represent absolutely same strongly in the direction of magnetising field ?

FFr
quantities. (a) cobalt (b) Nickel
(c) Platinum (d) Gold
CASK 2. On the basis of magnetic properties,
different materials have been classified into three
for
For Question No. 7 and 8, we have given two
ur
categories : Diamagnetic ; Paramagnetic and statements each, one labelled as Assertion (A)
Ferromagnetic. and other labelled as Reason (R).
kkss
1. Diamagnetic Substances are those in which
Choose the correct option out of the four options
Yo
given below:
oo

individual atoms/molecules/ions donot possess any net (a) Both, A and R are true, and R is correct
eB

magnetic moment on their own. These are feebly explanation of A.


magnetised in a direction opposite to the magnetising (b) Both, A and R are true, but R is not a correct
field in which they are placed. For example : Bi, Sb, explanation of A.
r

Au, Cu, quartz, water, alcohol, air, hydrogen etc.


ou
ad

(c) A is true, but R is false.


Permeability (p) of diamagnetic substances is less (d) Both A and R are false.
YY

than one and their susceptibility is negative, which does 7. Assertion. Different materials have been
not change with temperature. classified into three categories on the basis of
ndd
Re

2. Paramagnetic substances are those in which their magnetic properties.


each individual atom/molecule/ion has some net
Fi

Reason. Magnetic properties exhibited by them


magnetic moment of its own. These substances are are distinct.
weakly magnetised in the direction of the field in which
8. Assertion. At a certain temperature, ferro
they are placed. For example, aluminium, platinum, magnetics pass over to paramagnetic.s. This
chromium, mangane.se, crown glass, oxygen etc. The temperature is called curie temperature.
su.sceptibility of paramagnetic substances is positive and Reason. For all ferromagnetic materials, curie
it decreases with rise in temperature. temperature is the same.
ANSWERS

2. (b) 3. (a) 4. (c) S.(d) 6.(d) 7. (a) 8.(c)


HINTS/EXPLANATIONS For Difficult Questions
1. One Bohr magneton = 9-27 x 10“^“^ Am^. 3. Both, A and R are true, and R is correct
2. Tesla is SI unit of magnetic field strength. explanation of A.
MAGNETISM AND MATTER 5/79

4. Magnetising intensity H = n i. Its unit is Ampere Thus Assertion is true but Reason is false.
turns per metre /.e.,Am"^ 5. Iron is not diamagnetic.
and intensity of magnetisation 6. Gold is not magnetised strongly in the direction
of magnetising field.
^ _ magnetic moment
volume 7. Both, A and R are true and R Is correct
Am^ explanation of A.
Its unit is — = Am *. 8. Assertion is true, but the Reason Is false.
m

oww
TYPE I. MAGNETIC DIPOLE
6. The magnetic field at a point on the magnetic
MOMENT AND FORCE
equator is found to be 3T x 10“^ T. Taking the
earth’s radius to be 6400 km, calculate the
BETWEEN MAGNETIC POLES magnetic moment of the assumed dipole at the
earth’s centre. [Ans. 8*1 x 10^^ Am^]

e
1. A steel wire of length / has a magnetic moment

ree
rFl
M. It is bent into a semicircular arc. What is the

Fre
TYPE III. MAGNETIC
new magnetic moment ? [Ans. 2 Af/jcl
MOMENT OF A CURRENT LOOP

rrF
2. Two thin bar magnets of pole strengths 25 Am
and 48 Am respectively and lengths 0-20 m and 7. A current of 5 A is flowing through a 10 turn
ouur
0-25 m respectively are placed at irght angles to
each other with the N-pole of first touching the
sffoo
circular coil of radius 7 cm. The coil lies in XY
plane. What is the magnitude and direction of
magnetic dipole moment associated with it ?
kks
S-pole of the second. Find the magnetic moment
oooo
Yo
of the system. [Ans. 13 Am^l (CliSE Sample Paper 2003)
3. Two identical magnets with a length 10 cm and [Ans. 0*77 Am^’ along Z-axis]
eBB

weight 50 gf each are arranged freely with their 8. The magnetic dipole moment of earth is
like poles facing in a vertical glass tube. The 6-4 X 10^* A m^. If we consider it to be due to a
urr

upper magnet hangs in air above the lower one current loop wound around the magnetic equator
You

so that the distance between the nearest poles of of the earth, then what should be the magnitude
ad

the magnets is 3 mm. Determine the pole strength of the current ? Take earth to be a sphere of radius
dY

of the poles of these magnets. [Ans. 6*64 Am]


6400 km. [Ans. 5 X lO’ A]
innd
Re

TYPE II. MAGNETIC TYPE IV. TORQUE ACTING


ON A MAGNETIC DIPOLE
FFi

FIELD STRENGTH
AND POTENTIAL ENERGY OF
DUE TO A BAR MAGNET
DIPOLE IN A MAGNETIC FIELD
4. A bar magnet of length 10 cm has a pole strength
of 10 Am. Calculate the magnetic field at a 9. A magnetised needle of magnetic moment
distance of 0-2 m from its centre at a point on its 4-8 X 10~- JT"' is placed at 30® with the direction
(i) axial line («) equatorial line. of uniform magnetic field of magnitude 3 x
[Ans. 2-84 x ir^ T ; 1-14 x IQ-^ T] 10”^ T. What is the torque acting on the needle ?
5. Two small magnets are placed horizontally (CBSE 2001) [Ans. 7-2 x lO"^ N-m]
perpendicular to magnetic meridian. Their north 10. A bar magnet placed in a uniform magnetic field
poles are at 30 cm east and 20 cm west from a of strength 0-3 T with its axis at 30® to the field
compass needle. Compare the magnetic moments experiences a torque of 0-06 N-m. What is the
of the magnets, if compass needle remains magnetic moment of the bar magnet ?
undeflected. [Ans. 8:27] (ICSE 2006) [Ans. 0-4 Am^]
5/80
'<i. Fundamental Physics (XII)
11. A circular coil of 100 turns and having a radius is 04 G. Determine the magnitude of earth’s
of 0-05 m carries a current of 0-1 A. Calculate magnetic field at the place.
the work required to turn the coil in an external
(CBSE 2011) [Ans. 0*8 G]
magnetic field of 1 -5 T through 180° about an
20. The declination at a place is 20° west. In what
axis perpendicular to the magnetic field. The
direction should a ship be steered so that it
plane of the coil is initially at right angles to the
magnetic field. reaches geographic north east ?
[Ans. 0-24 J]
[Ans. 65° east of magnetic north]
12. A magnetic needle lying parallel to a magnetic
field needs 2 J work to turn it through 60°. 21. The true dip at a place is 30°. In what plane is the
Calculate the torque required to maintain the dip apparently 60° ? [Ans. 70*5°]
needle in this position. [Ans. 3*46 N-m] 22. A ship sails due east according to compass. If
13. A bar magnet with poles -Ts cm apart and pole declination at that place is 19° east of north. What
strength 144 Am rests with its centre on a is the true direction of ship ?

w
frictionless pivot. The magnet is held in [Ans. 19° south of east]
equilibrium at an angle of 60° to a uniform
magnetic field of intensity 0-20 T by applying a TYPE VI. NEUTRAL POINTS

Flo
force F at irght angles to its axis, 01 m from its
pivot. What is the value of force F ? 23. A bar magnet of length 10 cm is placed in the

reeee
magnetic meridian with its north pole pointing
[Ans. 0*432 N]
towards the geographic north. A neutral point is

FFr
obtained at a distance of 12 cm from the cenu-e
TYPEV. MAGNETIC OF EARTH
of the magnet. Find the magnetic moment of the
14. Vertical component of earth’s magnetic field at a
for
magnet, when H = 0-34 gauss.
ur
[Ans. 0*747
place is ^ times its horizontal component. If
kkss
24. A very short bar magnet has magnetic moment
total intensity of earth’s magnetic field at the
14175 J T“^ It is placed (i) with its north pole
Yo
place is 04 Gauss, find (/) angle of dip
oo

pointing towards geographic north (ii) with its


(ii) horizontal component of earth’s magnetic
north pole pointing towards geographic south. If
eB

field. (CBSE 2010) [Ans. 60° ; 0*2 G]


horizontal component of earth’s field at the place
15. The declination at a place is 15° west of north. In is 042 gauss, calculate the distance of the neutral
which direction should a ship be steered so that
r

points from the magnet.


ou
ad

it reaches a place due east ? [Ans. 105° east]


[Ans. 15 cm ; 18*9 cm]
16. If the horizontal component of earth’s magnetic
YY

25. A neutral point is found on the axis of a bar


field at a place is 04 x 10“* T where dip is 60°, magnet at a distance of 10 cm from its one end.
what are the values of vertical component and
ndd

If the length of the magnet be 10 cm, and


Re

resultant magnetic field ?


H = 0-3 G. Find the magnetic moment of the
[Ans. 0*69 X 10^ T ; 0-8 x 10^ T]
Fi

magnet. [Ans. 0*506 Am^]


17. A dip circle shows an apparent dip of 60° at a
place where true dip is 45°. If dip circle is rotated TYPEVn. TANGENT LAW
through 90°, what apparent value of dip will it AND TANGENT GALVANOMETER
show ? [Ans. 51°]
26. Two tangent galvanometers. A and B have their
18. A compass needle whose magnetic moment is number of turns in the ratio 3 : 1 and diameters
60 Am^ points geographic north at a certain place of coils in the ratio 4:1. Explain with reason
where H = 40 pT. The compass needle (0 Which galvanometer has greater reduction
experiences a torque of 1-2 x 10“^ N-m. What is factor ? (ii) Which galvanometer shows greater
the declination at the place ? [Ans. 30°] deflection, when both are connected in series to
19. A magnetic needle free to rotate in a vertical plane a d.c. source ? [Ans. (/) A, (ii) B]
parallel to magnetic meridian has its north tip 27. Two tangent galvanometers differ only in the
down at 60° with the horizontal. The horizontal
matter of number of turns in the coil. On passing
component of earth’s magnetic field at that place current through the two joined in series, the first
MAGNETISM AND MATTER 5/81

shows a deflection of 35® and the other shows 36. The magnetic field B and the magnetic intensity
45® deflection. Compute the ratio of their number // in a material are found to be 1-6 T and
of turns. Take tan 35® = 0-7. [Ans. 7:10] 1000 A/m respectively. Calculate the relative
28. A 2 turn coil of radius 10 cm is placed with its
permeability and the susceptibility Xm
material. [Ans. 1-3 x 10^ each]
plane in magnetic meridian. A small magnetic
37. The maximum value of permeability of )i-metal
needle is suspended at the centre of the coil by a
torsion free silk thread. On passing a current
{77% Ni, 16% Fe, 5% Cu, 2% Cr) is 0-126 T-m/A
Find the maximum relative permeability and
through the coil, the needle is deflected through [Ans. 1‘00 X 10^ each]
susceptbility.
45®. Calculate the strength of the current if
horizontal component of earth’s field is 38. Calculate permeability and susceptibility of a
1-6 X 10“^ T [Ans. 1*27 A] magnetic bar of cross-section 0-1 cm^ having
magnetic flux of 2-41 x 10"^ Wb due to
TYPE VIII. ATOM AS A
magnetic intensity of 3200 Am"^

w
MAGNETIC DIPOLE
[Ans. 7-53 x 1(H Tm A“^ ; 596-1]

29. An electron is revolving in a hydrogen atom in a TYPEX. CURIE LAW

Flo
circular orbit of radius 4 A making lO’^ rps. What AND HYSTERISIS LOSS
is the magnetic moment associated with this

eeee
39. The magnetic susceptibility of magnesium at
electron ? [Ans. 8-038 x 10"^ Am^] 300 K is 1-2 X 10^. At what temperature will its

Fr
30. The dipole moment of a coil of 200 turns carrying magnetic susceptibility become 1-44 x 10^ ?
a current of 3 A is 9-24 A m^. What is the diameter (CBSE2019) [Ans. 250 K]
of the coil ? [Ans. 14 cm]

for
40. An iron sample having mass 8-4 kg is repeatedly
ur
taken over cycles of magnetisation and
TYPE IX. MAGNETIC
demagnetisation at a frequency of 50 c/s. It is
ks
PROPERTIES OF MATERIALS found that 3-2 x lO'^ J of energy is dissipated as
Yo
heat in the sample in 30 min. If density of iron is
oo

31. A magnet of magnetic moment 2-5 Am^ weighs 7200 km/m^, what is the hysterisis loss ?
66 g. If density of material of the magnet is
[Ans. 304-8 Jm"^ cycle"^]
B

7500 kg/m^, find the intensity of magnetisation.


41. For a magnetising field of intensity 2 x 10^
re

[Ans. 2-84 x 10^ Am”’]


Am"’, aluminium at 280 K acquires intensity of
32. Assume that each iron atom has a permanent magnetisation of 4-8 x 10"^ Am”’. Find the
ou
ad

magnetic moment = 1-85 x 10"^^ Am^. The susceptibility of aluminium at 280 K. If the
number density of atoms in iron is 8-52 x
Y

temperature of the metal is raised to 320 K, what


10^* m”^ Find maximum magnetisation and will be its susceptibility and intensity of magne
maximum magnetic induction in the iron bar. tisation ?
nd
Re

[Ans. 1-58 X 10^ Am"' ; 1-985 T] [Ans. 2-4 X 10”® ; 2-1 x 10"^, 4-2 x 10”2 Am”']
Fi

33. A magnetising field of 1500 A/m produces a 42. Fig. 5(Q).13 shows the variation of susceptibility
flux of 2-4 X 10"^ weber in a bar of iron of cross- of a material with in', where Tis the lemperanire.
sectional area 0-5 cm^. Calculate the permeability Calculate curie constant of the material.
and susceptibility of the iron bar used.
(CBSE 2008) [Ans. 255 ; 254]
34. An iron rod of 0-2 cm^ area of cross-section is
subjected to a magnetising field of 1200 A m”’.
If susceptibility of iron is 599, calculate
(0 permeability, (/i) magnetic flux produced.
[Ans. 7-54 x lO"^ TA”^ m ; 1-81 x IB"® Wb]
35. Calculate the magnetic field intensity at a distance
of 20 cm. from a pole of strength 40 Am in air.
Find the magnetic induction at the same point.
[Ans. 62-5 K]
[Ans. 79-54 Am"*; lO"^ Wb/m^]
5/82
Fundamental Physics fXinrosTW

46. A closely wound solenoid of 2000 turns and


TYPE XL TYPICAL PROBLEMS
area of cross section 1-6 x 10~^ carrying a
43. A long straight horizontal cable carries a current current of 4-0 A is suspended through its centre
of 3-3 Ain the direction 10® .south of west to 10° allowing it to turn in a horizontal plane. What is
north of east. The magnetic meridian of the place the magnetic moment associated with the
happens to be 10® west of the geographic solenoid ? What are the force and torque on the
meridian. The earth’s magnetic field at the solenoid if a uniform horizontal field of 7 5 x
location is 0-33 G and the angle of dip is zero 10“^ T is set up at an angle of 30° with the axis
degree. Locate the positions of neutral points ? of the solenoid ? (CBSE 2015)
{Ans. A set of neutral points parallel to and
above the cable are located at distance of [Ans. 1-28 Am^ ;F = 0,x^ 0-048 Nm]
2 cm from cable] 47. An iron rod of volume 10“^ m^ and relative
44. A magnet 15 cm long with poles of strength 250 permeability 1000 Is placed inside a long solenoid

ww
Am lies on a table. Find the magnitude of the wound with 5 tums/cm. If a current of 0-5 A is
magnetic intensity S at a point P, 20 cm directly passed through the solenoid, find the magnetic
above the north pole of the magnet. moment of the rod. [Ans. 25 Am^j

Floo
[Ans. 388-1 p T] 48. A bar magnet has pole strength 4-5 Am, magnetic
45. Two identical short bar magnets each of magnetic length 12 cm and cross sectional area 0-9 cm-.

ee
moment 12-5 Am^ are placed at a separation of Find (a) intensity of magnetisation (/)

reer
10 cm between their centres, such that their axes
(b) magnetising intensity (H) at the centre and

rFF
are perpendicular to eachother. Find the magnetic (c) magnetic induction (B) at the centre of the
field at a point midway between the two magnets.
magnet. [Ans. (a) 5 x 10** Am~*, (b) 199 Am“^
uur r
[Ans. 2-24 x T, 8 = 26-6°]
ffoor (c) 6-26 x ir^ T]
sks
YYoo
ooko

For Difficult Ouestions


eBB

M
1. Here, pole strength, m= —
r

FIGURE 5(Q1.14
ouur
ad

If r is radius of semicircle, then M2


rnNT
Y

/
nr = I,
nddY

7U
Re

New magnetic moment M' -m.y 2 r 0-25 m


FFiin

M 21 2M
I n n
S
M1
2. Here, m| = 25 Am ; 2 /j = 0-20 m S N
mj = 48 Am ; 2 /2 = 0-25 m 0-20m

A/, =m|(2/i) = 25 x0-20 = 5-0Am2


3. Let m, = m2 = m, r = 3 mm = 3 x 10"^ m
= m2 (2 Ij) = 48 X 0-25 = 12 0 Am^
For hanging in air in balanced position,
As is clear from Fig. 5(Q).14,
F = 50g/=50xl0-^x9-8N
Magnetic moment of the system As, F = h.!!h!!!2
4tc r~
M = m.m
50 X 10“^ X 9-8= 10-^
(3x10-3)2
V(5-0)-+(12-0)- = 13 Am^ m3 = 9x5x9-8 x 10“* , m - 6-64 Am.
MAGNETISM AND MATTER 5/83

4. Here, 2 / = 10 cm, / = 5 cm = 0-05 m, The direction of magnetic dipole moment is


m=\0 Am perpendicular to the plane of the coil, along Z-
axis.

Af = mx2/ = 10 X 0-1 = 1 Am- M 6-4x102'


8. / = = 5x10’A
2Md lQ-^x2x 1x0-2 icr2 (22/7)x(6-4x 10^)2
B,1 =Baxial
“4;r " (0-2^-0-052)2 9. Here, M = 4-8 x 10-2 JT-', 0 = 30",
0-4x10-2 5 = 3x 10-2t,x = ?
= 2-84 X KT® T X = MB sin 0 = 4-8 X 10“2 x 3 x 10“2 x sin 30®
“ (0-0375)2
= 7*2xlO-^N-m
^2 ~ ^equatorial
10. Here, 8 = 0-3 T, 0 = 30®,
10-2x1

ww
M
x = 0-06N-m,M=?
4jt (^/2+/2)3/2 (0.22+0-052)3/2
ao6
10-2 10-2 As X = MB sin 0, M = —-
= ^= 5^ = M4xl(r«T

Flo
Bsin0 0-3sin30
(0-0425)3^2 8.76x10-3 = (HAm2

e
5. The compass needle at C lies on axial line of

reree
11. Here, n = 100, r = 0-05 m, / = 0-1 A
two magnets. Fig. 5(Q).15. As it remains

r FF
undeflected, therefore, fields due to two 1V=?, B=1-5T,
magnets must be equal and opposite. 0, = 0®, (When plane of coil is at 90® to E)
uurr
N FIGURE 5(Q).1S
foor 02= 180®
M = n/A = n/(Jif2)
kss
W E = 100 X 0-1 (3-14) (0-05)2
Yoo
= 7-85 X 10-2 Am2
ooook

S W=- MB (cos 02 - cos Oj)


eBB

sii = - 7-85 X 10-2 X J.5 (cos J80O_ cos 0®)


w ►w— H W=0-24J
20cm 30cm
rr

12. Here, Oj = 0®, W= 2 J, 02 = 60®, X = ?


ouu
ad

B,=B2 W = -MB (cos 02 - cos 0j)


Y

2 =-MB (cos 60®-cos 0®)


dY

4jt dj3 4^ ^3
Re
innd

; 2
L= i. /20f
Fi

M MB = 2x2 = 4
= 8:27
M, d^ 30 Torque required, X = MB sin 0 = 4 sin 60®
6. A point on magnetic equator is in broadside-on = 4x
V3 = 3-46Nm
position of the earth’s assumed dipole. Therefore, 2

Brf3 13. Here, 2l = ^|3cm =Sxl0~'^ m


B = or M =
4nd} Po/4tc m = 14-4 Am, 0 = 60®, B = 0-20 T
F = ? r = 0-l m
M = 3-1x10^5(6-4x10^
10-2 Now, X = r F sin 90® = MB sin 0

= 8*lxl022Am2 MB sin 0 m (2/) B sin 0


F =
7. Here,/=5A,A/= 10, r = 7 cm = 7 x 10-2 m, r r

M = WM =M(JI /2) = 10 X 5 X 3-14 (7 X 10-2)2 ^ 14-4 X V3 X 10"2 X 0-2 sin 60®—


F = = 0-432 N
= 0-77 Am2 0-1
5/84 ^ Fundamental Physics (XII)
14. V = Sh, /? = 0-4G,8=?//=? 19. Here, 8 = 60°; // = 04 G ; /? = ?

,tan oj. = —
V Va// = Va R = H _ 04 04 = 0-8G
8 = 60' cos 8 005 60° 1/2
H H

1
20. As is clear from Fig. 5(Q).17, is 20° west
H = R cos 8 - 04 cos 60° = 0 4 x — = 0-2 G
2
of Ng Sg. To reach geographic NE, the ship must
IS. In Fig. 5(Q).16, Magnetic NS is at 15° west of
Geographic N-S. The ship is to go along OP
towards geographic east. Therefore, the ship must
be steered 90° + 15° = 105° east of the compass

ooww
needle (Mag. N-S).

FIGURE 5(Q).16 Wg
MN GN

ee
'15'West
It

r
rFFl
ree
90'+15'

F
\ =105'
1
GW
O
GE
be steered at (20° + 45°) = 65° east of magnetic

rF
P
north.
Ship
21. Here, 8 = 30° and 8' = 60°, 0 = ?

fsfoor
ouur
V V V
tan 8 = — and tan 8' = —
kosk
GS MS H ff' //cos0
Yo
16. Here, // = 04 x lO"^ T, 8 = 60°, V =?,/? = ? tan 8'
oo

tan 8' =
Y
V = // tan 8 = 04 x 10“^ tan 60° COS0
BB

= 04-y^xlO^T =0‘69xl0-^T cos 9 = tan8 _ tan 30° _ 1 _ 1


rre

tan 8' tan 60° Vs.Vs 3


H 04x10“^
oYuu

cos 0 = 0-3333 8 = 70-5°


R = = 0-8 X 10-^ T
ad

cos 8 cos 60° 22. In Fig. 5(Q).18, compass needle is represented


dY

17. Here. 8] =60°, 8 = 45°, 82 = ? by MN-MS at 19° east of north. As the ship is
As cot^ 8j + cot^ 82 = cot^ 8 sailing due east according to the campass,
innd

therefore the ship is actually sailing at 19° south


Re

cot^ 82 = cot^ 8 - cot^ 8] of east as shown.


= (cot 45°)^ - (cot 60°)^
FFi

n2 FIGURE 5(Q).18
1
= 1- - = 0-67 GN

iV3 3 ,MN

cotS2 =V^ = 0-816


82 = cor^ (0-816) = 51° 19'

18. When compass needle points along magnetic


N-S, no torque acts on it. However, when it points W
9'
E

towards geographic N-S, torque on it is


T = MH sin 0 Ship

1-2x10-^ 1
sin 0 = —
MH 60x40x10-^” 2
0=30' MS GS
MAGNETISM AND MATTER 5/85

23. Here, 2 / = 10 cm, / = 5 cm = 5 x IQr^ m,


●.
r. /to
—A = _d.x —= -x- = ->l
4 1 4 ,
rf=12cm, // = 034G = 0-34xKHt KB 1 3 3
"a
As N-pole of magnet is towards geographic north,
therefore neutral points lie on equatorial line of i.e. Kj^ > Kg
the magnet. (ii) When both the tangent galvanometer are
connected in series, there will be same current in
M
H = h them. Then
4n (rf2+/2)3/2 /= tan 0^ = tan 0g

oww
M = — or
tan0 B Ka 4
= 10"^ [12^ + 5^]^^ (10-^)3/2 X 0-34 X 10^ Le. then 0^ < tan 0^ or 0^ < 0||
= 10^ X 13 X 13 X 13 X 10-^ X 0-34 x 10-^

e
27. As the two tangent galvanometers are connected
M = 0-747 JT-l

re
in series, they carry the same current.

FFrllo
\^M ^1 =
2rH
tan 0,1 = f ^ 2rH tan 02

rF
24. (0 H =
^ 1^0 ”2

ee
4jc

!h.= tan 0, tan 35" _ 0-7 _ 7


ouru 1/3
_ 10-7x14175
-d/3

rF
or d-
4ii H ”L 0-42x10-4 "2 tan 02 tan 45* 1 10

fosor
= 0-15 m = 15 cm
28. Here, /»= 2, r = 10 cm = 01 m, 0 = 45*, / = ?
//=1-6x10-5t
os kf
00 H = ^2^ 2rH
4 OT d^
ook
As / = tan0
YYo
1/3
Bo

nl/3
Ho 2M 10-7x2x14175
reeB

j 2x0-1x1-6x10-^ tan 45*


or d =
4to H ”L 0-42x10-4 = 1*27 A
4tox10-7x2
oouY

= 0-189 m
ur

= 18*9 cm 29. Heie,r=4A = 4x 1(T^®m,v = 10^^rps,M = ?


ad

25. Here,2/= 10cm,H = 0-3 G,M = ? Af = /A = (ev) OT r2


Yd

r = 10 + / = 10 + 5 = 15 cm = 15 X 10-2 m = (1-6 X 10-*® X 10*5) 3.14 (4 X 10-10)2


= 8-038 X IQT^ Am2
nidn

As neutral point is on the axis of bar magnet,


Re

therefore, 30. Here, AT= 200, /= 3 A,M = 9-24 A m^?


If r is radius of coil, then
FFi

2xM
H =
4to r3 (15x10-2)3 M = NIA =NIii —
4

M =
(15x10-2)3 g
4x9-24
2x10-7
3-14x 200 x3
3375x10-^x0-3x10-4
= 0-14 m= 14 cm
2x10-7
31. Here, A/= 2-5 Am2,
= 506*25 X 10^ = 0*506 Am2
mass, /n = 66 g = 66 X 10~3 kg,
26. Given, = 11 and ^ t1 or — = -1 p = 7500kg/m3,/=?
2r, 0?
n
B B _ 2-5x7500
2rH .
V m/p m 66x10-3
(0 Reduction factor, K = ue., Koc- = 2*84xl05Am-i
n
5/86
Fundamental Physics fXinrosnw
32. Here, magnetic moment of each atom 38. Here, A = 0-1 cm2 = 0-1 x lO"^ m2, \l = 1,%^ = 7
= 1-85 X 10-23 Am2 (|) = 2-41 X 10-3 Wb,
Number of atoms/volume = 8-52 x m“3 //=3200Am-^
Max. magnetisation,
241x10-3
M ^ =
/ = _ = 1-85 X 10-23 X 8-52 X 1028 H H AH 01x10-^x3200
= 1*58 X 10<^ Am-l = 7-53 X 1(H TmA-l

Max. mag. inductionB = \Iq{H + I) 7-53x10-^


-1
= 4 7t X 10-2 (0 + i-58 X 10^) = 1-985 T ^^0 4jcx10“2
33. Here, H = 1500 A/m, <j) = 2*4 x 10-^ weber

ww
a = 0-5 cm2 _ q.^ jq-4 ^2 X„ = 597 1 =596-1
39. Here. Tj = 300 K, %, = 12 x 103.
72 = ?, = 1-44 X 103
({>_ 2 4x10-^

Flo
Now B = — = 4-8x10-‘T
a 0-5x10-^ As

e
A ^2

ree
B _4-8xlQ-» = 3-2x10^

Fr
H ~ 1500 1-2x103
T*2 =^T -=

rF
or ~ x300 = 250K
3-2x10^ Z2 1-44x103
uurr
= 0-255x103 = 255

As
IIq 4jtx10-2
Xm = ^ir-l=255-l=254
s for
40. Here, m = 8-4. kg, v = 50 c/s,
/ = 30 min = 30 x 60 s
Energy dissipated, £ = 3-2 x 10^ J,
kks
34. Here, A = 0-2 cm2 _ q.2 ^ lo*^ m2 ;
Yo
p = 7200 kg/m3
oooo

//= 1200 Am-‘;X;„ = 599


p^=l+X„=l ■(■599 = 600 Volume, V = — 84 m-^3
eB

p = Mo p^ = 4 jux.10-2x600 p 7200
= 7-54xlO-^TmA-l
32x10^x7200
r

Magnetic flux ([) = m A Energy dissipated/volume =


ou
ad

84
= (7-54 X 10-^) X 1200 X (0-2 x 10^)
YY

= 1-81 X 10^ Wb Hysterisis loss = Energy dissipated/volume/cycle


35. Here, = 20 cm = 1/5 m, m = 40 Am, _ 3-2x10^x7200
nndd

= 304-8 Jm'^ cyde"^


Re

//=?,5 = ? 84x30x60x50
41. Here, H = 2x\0^ Am-*, 7= 280 K.
Fi

1 mxl 1x7 40
H = =: X = 79-54 Am-l 7 = 4-8x10-2 Am-*
4tc 4x22 (1/5)2
I 4-8x10-2
B = Mo^ = 4tcx10-2x 79-54 = IQ-^ Wbm-2 = 2-4 X 10^5
B 1-6
H 2x103
36. = 1-6 X 10-3 Tm A-* r = 320 K, X' = ?
H 1000 m

According to Curie’s law.


X'm T
_ M _ 1-6x10-3 = 1-3x103 X r
’’ Mo 4tcx10-2
m
7 280
y! m X 24x10-3
320
X;„ = Mr-l = l-3xl03-l =1-3x103
= 2-1x10-3
37. M;. -iL = 0-126
=
Mo 4jcxl0
^ = 1-00x103 As X' m
r
H

Susceptbility, X^ = 1 = 100 10^ - * /' = X'„// = 2-l X 10-3x2X 103


= 1-00 X 10® = 4-2x 10“2Am-l
MAGNETISM AND MATTER 5/87

42. From the graph in Fig. 5(Q).13, Horizontal components of is


15
when x,„ = 0-5, -=
- 8 0x10-3 K"' ^
sin 0 = 400x— = 240 uT
25 ^
According to Curie’s Law Resultant vertical component = 625 - 320
C = 305)J.T
Xm
Resultant field at B = -^(305)^ + (240)^
T

1 1000
or C = x„,^T =0-5x = 0-5x = 388-1 M-T
80x10"3 8-0
45. Here, Mj = Mj~ 12-5 Am^.
= 62-5 K
Ol 02 = 10 cm, Fig. 5(Q).20,
43. Here, I = 3-3 A , B = 0-33 G = 0-33 x 10^ T, P is midway between and O2

ww
6 = 0®. Horizontal component of earth’s magnetic d=Oy P = 02P=5cm = 5x 10-2 m

field // = B cos 6 = 0-33 x 10^ cos 0° = 0-33 x


IQ-^ T. The neutral points will be parallel and FIGURE 5(Q).20 S2

FF loo
above the cable. Let r be the distance of neutral
point from the cable. Then

ee
O2

er
H = or Si Oi Ni
4n H

rFreF
4n r

2x3-3 N2
r =10-2 ^ = 2xl0 2ni = 2cm
rur
0-33x10-^
As P lies an axial line of Nj 5j,
Thus the set of neutral points will lie parallel ffoor 10-2 x2xl2-5
ks
to cable, and above the cable at distance of
^1 =
2 cm from cable. 471 (J)3 (5x10-2)3
YYoou
okso

44. As is clear from Fig. 5(Q).19, = 0 02 T, along 0| PO2


BBoo

Again. P lies on equatorial line of N2 S2


re

^^0 ^2 _ 10-2 X12-5


^2 = = 0-01,1BO.,
4n (5x10-2)3
oouur
ad
Y

Resultant magnetic field strength at P

B= = ^{0-02)2 +(0-01)2
dnYd
Re

= 2-24 X 10-2 j
FFini

If 0 is the angle which S makes with B^, then

tan 0 -
^2 001
= 0-5 : 0 = 26-6‘
B _ Pg m,xwi2 _ 10-2x250x1 B
1
0-02
N ~
4k (NP)^ (0-2)2 46. Here, N = 2000, A = 1-6 x 10^ m2, / = 4-0 A
= 625x 10"^T = 625pT M = N1A = 2000 X 4-0 X i-6 X 10^
= 1-28 Am2
Now SP = ^(0-2)2+(015)2 ^ 0-25 m Net force experienced by the magnetic dipole in
10-2x250x1 uniform horizontal field, F = 0
^5 = = 400 p T ; The magnitude of torque exerted by the
(0-25)2 —^

Vertical component of B^ is magnetic field B on the solenoid is


20 T = MB sin 0 = 1-28 x 7-5 x 10-2 30“
Bj cos 0 = 400 X — = 320 p T, opposite to B^^ = 0-048 Nm
5/88 Fundamental Physics (XII) VOL.i

The torque tends to align the axis of the m 4-5


—>
ia) I = - .
solenoid, i.e., its magnetic moment M along a 0-9x10^
—¥
= 5xlO^Am-lfh)mStoN
the field B.
(h) Magnetic intensity at the centre, due to its
47. Here, n = S tums/cm = 500 tums/m iV-pole
y = 1(H m^, |ii^ = 1000, i = 0-5 A m 4-5
We know that B = (// + /) «.=
47tr2"4,t(0-06)2 where r = 2W
B
-H = 99-5 Am"^ towards 5-pole
Magnetic intensity at the centre due to its 5 pole

ww
m 4-5
For a solenoid of n turns per unit length carrying »2 =
4nr^ 4jc(0-06)2
current i;H = ni.

Floo
/ = (n^-l)m. = 99-5 Am“^ towards 5-pole.
/= (1000-l)x 500 x 0-5 Resultant magnetic intensity at the centre

ee
/ = 2-5xl05Am-l

eer
= 99-5 + 99-5
As magnetic moment, M = / x V

rFrF
/. M=2-5xl05xl0-^ = 25Am2 = 199 Am“^, towards 5-pole
ur r
48. Here, m = 4*5 Am., (c)Now,B = Po(i? + /)
2/ = 12 cm = 0-12 m
a = 0-9 cm^ = 0-9 x 1(H m^
ffoor
s = 4jcx 10-'7(-199 + 5x 10^)
= 6*26 X ICr^ T, towards N pole
osk
YYoou
oook
r eBB
ouur
ad
Y
nd dY
Re
FFini
MAGNETISM AND MATTER 5/89

^IF
1
m* -Ar- '1*
'^^Arik
f li
iL
' 6 hi*
● i
WITH
Y SOLUTIONS
V -● I. - ●
11.

Q. 1. A short bar magnet placed with its axis at 30” with a uniform external magnetic fleld of 0*25 T
experiences a torque of magnitude equal to 4*5 x 10~^ J. What is the magnitude of magnetic moment

oww
of the magnet ?
Sol. Here, 6 = 30”. B = 0-25 T, t = 4-5 x 10"2 J, M = ?

T 4-5x10"2
As, x = M B sin 9 M = = 0*36 JT-^

e
fisinG 0-25 sin 30”

re
FFrlo
Q. 2. A short bar magnet of moment 0*32 JT~^ is placed in a uniform external magnetic fleld of 0*15 T. If
the bar is free to rotate in the plane of the fleld, which orientations would correspond to its, (i)

rF
ee
stable and (ii) unstable equilibrium ? What is the potential energy of the magnet in each case ?

rF
Sol. Here, M = 0-32 JT"*, B = 0-15 T
ouru
(0 In stable equilibrium, the bar magnet is aligned along the magnetic field, i.e. 0 = 0°

fosor
Potential Energy = - MB cos 0” = - 0-32 x 015 x 0 = - 4*8 x 10“^ J
skf
(//) In unstable equilibrium, the magnet is so oriented that magnetic moment is at 180” to the magnetic
ooko
field i.e.Q= 180”
Yo
1) = 4*8 x 10 ^ J.
Y
Potential energy = - MB cos 180” = - 0-32 x 0-15 (-
Bo

Q. 3. A closely wound solenoid of 800 turns and area of cross section 2*5 x 10^ carries a current of 3*0 A.
reeB

Explain the sense in which the solenoid acts like a bar magnet What is its associated magnetic moment ?
Sol. Here, n = 800, A = 2-5 x lO"^ m^. / = 3 0 A
ooY
uur

A magnetic field develops along the axis of the solenoid. Therefore, the current carrying solenoid behaves
ad

like a bar magnet.


dY

M = n 1A = 800 X 3 0 x 2*5 x 10“^ = 0*6 JT“^, along the axis of solenoid.


Q. 4. If the solenoid in the above question is free to turn about the vertical direction, and a uniform
nind
Re

horizontal magnetic fleld of 0*25 T is applied, what is the magnitude of the torque on the solenoid
F
Fi

when its axis makes on angle of 30” with the direction of the applied fleld ?
Sol. Here, M = 0-6 (from Ques. 5) B - 0-25 T ; x = ? 8 = 30”

As. X = Af B sin 6 T = 0-6 X 0-25 sin 30” = 0*075 N.m.

Q. 5. A bar magnet of magnetic moment 1.5 JT~^ lies aligned with the direction of a uniform magnetic
field of 0.22 T.

(a) What 1s the amount of work done to turn the magnet so as to align its magnetic moment
(1) normal to the fleld direction, (ii) opposite to the field direction ?
(b) What is the torque on the magnet in cases (i) and (ii) ?
Sol. Here,M= 1.5 JT"', B = 0.22 T , W=?
(a) (/) Here, 0] = 0” (along the field), 02 = 90° (1 to the field)
As W = - MB (cos 02 - cos 9[)
VV= - 1.5 X 0.22 (cos 90° - cos 0”) = - 0.33 (0 - 1) = 0.33 J
5/90 Fundamental Physics (XII) VOL.I

(//) Here, = (T, 62 = 180“


W = -L5x 0.22 (cos 180“ - cos 0“) = - 0.33 (- 1 - 1) = 0.66 J
(b) Torque x = MBsin 0

(OHere, 0 = 90“, T = 1.5 X 0.22 sin 90“ = 033 Nm

(11) Here, 0 = 180“, x= 1.5x0.22 sin 180“ = 0

Q. 6. A closely wound solenoid of 2000 turns and area of cross section 1.6 x 10~^ carrying a current
of. 4 amp. is suspended through its centre allowing it to turn in a horizontal plane :
(a) What is the magnetic moment associated with the solenoid ?
(b) What are the force and torque on the solenoid if a uniform horizontal magnetic field of

ww
7*5 X 10~^ T is set up at an an^e of 30“ with the axis of the solenoid ?
Sol. (fl) N = 2000, A = 1.6 X 10-^ m2, / = 4 amp., M = ?, B = 7-5 x 10"2 T.
As, M = NIA .*. A/ = 2000 x4x 1.6 X 10-^ = 13811^*

Flo
(b) Net force on the solenoid = 0

e
ere
Torque, x = Af B sin 0 = 1.28 x 7.5 x 10"2 sin 30“ = 1.28 x 7.5 x 10“2 x ^ = 4.8 x 10r2 Nm.

FFr
Q. 7. A short bar magnet has a magnetic moment of 0*48 JT~*. Give the direction and magnitude of the
uurr
orr
magnetic field produced by the magnet at a distance of 10 cm from the centre of the magnet on

Sol. Here, M = 048 JT"', B = ?; d- Id cm = 0-1 m


sfo
(i) the axis (ii) the equatorial line (normal bisector) of the magnet
kks
Yo
oooo

_ ^0 2M =10-2 X 2x048
(0 On the axis of the magnet, B = = 0*96 X 10^ T along S-N direction.
An (0-1)3
eBB

(ii) On the equatorial line of the magnet


rr

048
5 = ^x^ = 10"2x (O-I) j = 0*48 X lOr^ T, along N-S dilution.
ou
ad

An d^
YY

Q. 8. A short bar magnet placed in a horizontal plane has its axis aligned along the magnetic north sooth
nndd

direction. Null points are found on the axis of the magnet at 14 cm from the .centre of the magnet
Re

The earth’s magnetic field at the place is 0*36 G and the angle of dip is zero. What is the total
Fi

magneto field on the normal bisector of the magnet at the same distance as the null points (ue.
14 cm) from the centre of the magnet? (At null points, field doe to a magnet is equal and opposite to
the horizontal component of earth’s magnetic field).
Sol. As null points are on the axis of the magnet, therefore

^lo 2A/
^1 = An d} = H

On the equatorial line of magnet at same distance (d), field due to the magnet is

B =iloJl=£L=.^
3 4n</3 2 2
.*. Total magnetic field at this point on equatorial line is

B = 77 + ^2 = // + - = -// =-x0-36 = 0*54G


2 2 2
MAGNETISM AND MATTER 5/91

Q. 9. If the bar magnet in the above problem is tamed around by 180®, where will the new null points be
located ?

Sol. When the bar magnet is turned through 180®, neutral points would lie on equatorial line, so that

.40
^ 47t

a1 =
_^0 2M ...(/O
In the previous question.
4jc d}1

Itp M _ 1^0 2M d} (14)3

ww
From (0 and (I'O,
^T^dl 4tc “i-i 2

Floo
14
= IMcm
^2

ee
reer
rFF
uur r
ffoor
sks
YYoo
ooko
r eBB
ouur
ad
Y
dY
Re
nnd
FFii
5/92
4. Fundamental Physics (XII) VOL.I

I ●:
WITH ANSWERS,
■.sx.

*;
/ .
^ HINTS AND SOLUTIONS
●) . T
7TT"

mm ir--- s
il-

MULTIPLE CHOICE QUESTIONS-I

1. A toroid of n turns, mean radius R and cross-


(c) domains are partially aligned

ww
sectional radius a carries current I. It is placed (d) domains are all perfectly aligned
on a horizontal table taken as x-y plane. Its
4. Consider the two idealized systems : (/) a
magnetic moment M parallel plate capacitor with large plates and

Flo
(a) is non-zero and points in the z-direction by small separation and (a) a long solenoid of
symmetry length L » R, radius of cross-section. In

e
rree
—^

{b) points along the axis of the toroid (A/ = A/ J) (/) E is ideally treated as a constant between

r FF
(c) is zero, otherwise there would be a field plates and zero outside. In (a) magnetic field
is constant inside the solenoid and zero
falling as at large distances outside the outside. These idealised assumptions,
uurr
torioid ^
(d) is pointing radially outwards for
however, contradict fundamental laws
below :
as
kss
2. The magnetic field of Earth can be modelled (a) case (/) contradicts Gauss’s law for electro
by that of a point dipole placed at the centre
ooook

static fields
Yo

of the Earth. The dipole axis makes an angle (b) case (a) contradicts Gauss’s law for magnetic
of 11-3“ with the axis of Earth. At Mumbai, fields,
eB

declination is nearly zero. Then,


(a) the declination varies between II-3° W to (c) case (i) agrees with <£E-dT=0
rr

11-3“ E
(d) case (a) contradicts dH-dT= Ien
ou
ad

(b) the least declination is 0” J

(c) the plane defined by dipole axis and Earth


Y

5. A paramagnetic sample shows a net magneti


dY

axis passes through Greenwich sation of 8 Am"^ when placed in an external


(d) declination averaged over Earth must be magnetic field of 0*6 T at a temperature of
Re
innd

always negative 4 K. When the same sample is placed in an


3. In a permanent magnet at room temperature external magnetic field of 0*2 T at a temp
Fi

(a) magnetic moment of each molecule is zero erature of 16 K, the magnetisation will be
{b) the individual molecules have non-zero 32
id) — Am ‘
{b) I Am
-1
magnetic moment which are all perfectly
aligned
(c) 6 Am"^ id) 24 Am~*.

MULTIPLE CHOICE QUESTIONS-II

6. S is the surface of a lump of magnetic (c) Lines of H are necessarily continuous across
material.
S
(a) Lines of B are necessarily continuous across id) Lines of H can not all be continuous across
S
ib) Some lines of B must be discontinuous 7. The primary origin(s) of magnetism lies in
across S
id) atomic currents
MAGNETISM AND MATTER 5/93

(fc) Pauli exclusion principle (a) electrostatic field lines can end on charges
(c) polar nature of molecules and conductors have free charges
(d) intrinsic spin of electron (b) lines of B can also end but conductors can
not end them
8. A long solenoid has 1000 turns per metre and
carries a current of 1 A. It has a soft iron core (c) lines of B can not end on any material and
of = 1000. The core is heated beyond the perfect shielding is not possible
Curie temperature, Tg, (d) shells of high permeability materials can be

w
(a) The H field in the solenoid is (nearly) un- used to divert lines of B from the interior
—>

changed but the B field decreases drastically region


—» —*
10. Let the magnetic field on earth be modelled
{b) The H and B fields in the solenoid are

e
by that of a point magnetic dipole at the centre
nearly unchanged

row
of earth. The angle of dip at a point on the

re
(c) The magnetisation in the core reverses direction geographical equator
(d) The magnetisation in the core diminishes by (a) is always zero

FFllo
F
a factor of about 10®
(b) can be zero at specific points
9. Essential difference between electrosatic

u
(c) can be positive or negative

ree
shielding by a conducting shell and magneto
static shielding is due to (d) is bounded

sFr
kro
ANSWERS
uor
2. (a) 3. (c) 4. (b) 5. ib) 6. (a, d) 7. {a, d) 8. (fl, d)
1. ic)
9. (a, c, d) 10. (b, c, d)
offo
kos
YYo
HINTS FOR DIFFICULT MULTIPLE CHOICE QUESTIONS
eerBB
oo
rY

Multiple Choice Questions -1

1. In case of a toroid, the magnetic field is only confined inside the


uu

FIGURE 5(N).1 FIGURE 5(N).2


body of toroid in the form of concentric magnetic lines of force
ad
doo

and there is no magnetic field outside the body of toroid. Thus the
magnetic moment of toroid is zero. Hence, option (c) is correct.
nY

E
2. Since the axis of the magnetic dipole placed at the centre of earth
nid

makes an angle 11 -3® with the axis of earth, two possibilities arises
Re

as shown in Fig. 5(N).l and 5(N).2. Hence the declination varies S


FFi

between ll-3®Wto

3. In a permanent magnet at room temperature domains of a magnet are partially alingned due to thermal
agitation,
f >

4. According to Gauss’s law for electrostatic field, y E .ds = Q/


S

It does not contradict for electrostatic fields as the electric field lines do not form a continuous closed path.
() E.ds = 0
According to Gauss’s law in magnetic field,

It contradicts for magnetic field because there is a magnetic field inside the solenoid and no field outside
the solenoid carrying current but the magnetic field lines form the closed paths.
5. Here, Ii = 8 Am"' ; = 0-6 T, rj = 4 K ; /2 = ?, ^2 = T, ^2 = 16 K
B (magnetic field induction)
According to Curie law, / OC

r( temp in kelvin)
5/94
‘Pnadet^’^ Fundamental Physics (XU)ESm

h _ ^2 ^1
— -1--L I 8 2 -1
x~ =
0-6^16 "I2
or /- =/,x—= = —Am
/
I
B
1 '2 ^ * 12 12 3

Multiple Choice Questions — II

6. The lines of magnetic field induction B are necessarily continuous across the surface 5 of a lump of
magnetic material. Outside the lump of magnetic material, H = a/p^ and inside the lump of magnetic
material, H = B!\i^ \i^ where is the relative permeability of material. Thus the lines of H cannot all be
continuous across surface S.

7. The primary origin of magnetism lies in the fact that the electrons are revolving and spinning about nucleus
of an atom, which gives irse to currents and hence to magnetism. Thus, options (a) and (c) are true.
8. In the solenoid, H = n/=a constant and S = p^ p^ n / changes due to variation in p^ When temperature

ww
of
the iron core of solenoid (which is ferromagnetic material) is rai.sed beyond curie temperature, then soft
iron core behaves as para magnetic material. We know that (Xn,)pard ~ (XHi)Fe!ro “

Flo
^X„,)FerTo 10^ = 10*
10-5

ee
rere
Thus magnetisation of the core diminishes by a factor 10^. Hence, options (a) and (d) are correct

r FF
9. Knowledge based question.
10. The angle of dip at a point on the geographical equator satisfies the optioas (b), (c) and (d).
uurr
foor
VERY SHORT ANSWER QUESTIONS
ks s
Yoo
ooook

II. A proton has spin and magnetic moment just like an electron. Why then its effect is neglected in
i
magnetism of materials ?
eBB

eh 1
Ans. Magnetic dipole moment due to spinning proton or electron, M = or M oe
rr

m
ouu
ad

M m
P _
YY

e
«1
Mg m
p minxg 1837 Mp « Mg
nndd
Re

12. A permanent magnet in the shape of a thin cylinder of length 10 cm has M = 10^ A/m. Calculate the
magnetisation current 7^. (Hert; M is the intensity of magnetisation).
FFii

Ans. Here, / = 10 cm = 0-1 m ; A7= 10^ A/m


(magnetisation current)
As M =
/(length)
/^ = A7x/= 10^x01 = 105 A

13. Explain quantitatively the order of magnitude difference between the diamagnetic susceptibility of
N2 (~ 5 X ir^ (at STP) and Cu (~ lO’^).

Ans. Density of nitrogen. _ 28g ^ 28g


22-4 litre 22400cc

Wc know, density of copper, = 8 g/cc

X-= 1-6x10-4 ●
Pc II 22400 8
MAGNETISM AND MATTER 5/95

X
^2 5x10“^ -4
Given, = 5xi0 -(/)
Xci 10"^

intensity of magnetisation (/) _ I _ magnetic moment (M)/volume(V)


We know that x =
magnetising force (H) H H

M M
Mp
HV //[mass(m)/density(p)] Hm

X OC
p for the given value of MIHm.

oww
X P
N,
-4
Hence, - = 1-6x10 ...Hi)
^Cu Pcu
From (0 and (//), we note that the major difference in susceptibility of N2 and Cu is due to their density.

e
FFrlo
14. From molecular view point, discuss the temperaturedependence of susceptibility for diamagnetism,

re
paramagnetism and ferromagnetism.

ree
F
Ans. Susceptibility of magnetic material, x = //// where / is the intensity of magnetisation induced in the material
and H is the magnetising force.

rF
Diamagnetism is due to orbital motion of electrons in an atom developing magnetic moments opposite to
applied field. Due to it, the resultant magnetic moment of the diamagnetic material Is zero. Hence,

fsoor
ouur
susceptibility x of diamagnetic material is not much affected by temperature.
Paramagnetism and ferromagnetism is due to alignments of atomic magnetic moments in the direction of
skf
the applied field. As temperature is raised, this alignment is disturbed, resulting decrease in susceptibility
ooko
of both, with increase in temperature.
Yo
15. A ball of superconducting material is dipped in liquid nitrogen and placed near a bar magnet, (i) In
Y
Bo

which direction will it move ? (ii) What will be the direction of it’s magnetic moment ?
reB

Ans. A superconducting material and nitrogen are diamagnetic in nature. When a ball of superconducting material
is dipped in liquid nitrogen, it behaves as a diamagnetic material. When it is placed near a bar magnet, it
will be feebly magnetised opposite to the direction of magnetising field. Due to it, (1) it will be repelled,
uur
ooY

i.e., moving away from magnet (//) its direction of magnetic moment will be opposite to the direction of
ad

magnetic field of magnet.


dY

SHORT ANSWER QUESTIONS


innd
Re

16. Verify the Causes law for magnetic Held of a point dipole of FIGURE 5(N).3
Fi
F

dipole moment M at the origin for the surface which is a sphere


of radius R.

Ans. According to Gauss’s law in magnetism, () B .ds = 0 ● To explain


it as per question, refer Fig. 5(N).3, magnetic moment of dipole at
—> A

origin O, M = M k . Let F be a point at distance r from O and OP


makes an angle G with z-axis. Component of M along
OP - M cos 0.
The point P is on the axial line of dipole of magnetic moment
M cos 9. The magnetic field induction at P due to dipole of moment
».# c- it ^o2A/cos9a
A/cosGis 5 = — r
4tc r'
Consider a spherical surface of radius r with centre at O lying in v-z plane. Take an elementary area ds of
the surface at P. Then
dT = r(rs\n6dQ)r = sinGc/Gr
5/96 Vtttdeefr'4. Fundamental Physics (Xn) VOL.I

2JI
f ^ f Ho COS0 A . 2 ● A JA A.
®B.£fa=(b-i^ r.(r'^sme</er) = —— f 2sin6cos6^6
3 / 4jj ^ 47t r J
2ji \2ji
= f sin20J6 = Hq M('-cos2e = — [cos4k-cos0] = —[1-1] = 0.
4«r J 4ji r 2 JO 4jc2r 4jc2r

17. Three identical bar magnets are rivetted together at FIGURE 5(N).4 FIGURE 5(N).S
centre in the same plane as shown in Fig. 5(N).4. This
system is placed at rest in a slowly varying magnetic
field. It is found that the system of magnets does not
show any motion. The north-south poles of one magnet
are shown in the Fig. 5(N).4. Determine the poles of the

w
remaining two.
Ans. The system will be in stable equilibrium if the net force on

Flo
the system is zero and net torque on the system is also
zero. It will be so if the poles of the remaining two magnets

reeee
are as shown in Fig. 5(N).5.
18. Suppose we want to verify the analogy between electrostatic and magnetostatic by an explicit

FFr
experiment Consider the motion of (i) electric dipole p in an electrostatic field £ and (ii) magnetic

for
dipole M in a magnetic field B . Write down a set of conditions on E,B, p,M so that the two
ur
motions are verified to be identical. (Assume identical initial conditions).
kkss
Ans. Let 0 be the angle between p and E . Torque on electric dipole of moment P in electric field
Yo
oo

E, X = p£sin0. Let 6 be the angle between M andB . Torque on magnetic dipole moment M in
eB

magnetic field B , x' = MB sin 0.


l\vo motions will be identical if p £ sin 0 = MB sin 0 or pE = MB
r

M
ou
ad

But £ = c£ pcB = MB or p = —
c
YY

19. A bar magnet of magnetic moment M and moment of inertia I (about centre, perpendicular to length)
is cut into two equal pieces, perpendicular to length. Let T be the period of oscillations of the original
ndd
Re

magnet about an axis through the mid point, perpendicular to length, in a magnetic field B . What
would be the similar period T* for each piece ?
Fi

Ans. The moment of inertia of a bar magnet of mass m, length / about an axis passing through its centre and
m/2
perpendicularto its length is / = . Let Af be the magnetic moment of the magnet, B is the uniform
12
magnetic field in which the magnet is oscillating, then time period of oscillation is

r = 2jt ...(0

When magnet is cut into two equal pieces, perpendicular to length, then moment of inertia of each piece of
magnet about an axis perpendicular to length passing through its centre is

/' =
m
(//2)2 _ m/2 1^7
2 12 “ 12 ^8 8
Magnetic dipole moment, M' - MU

Its time period of oscillation is r' = 27C


V
= 271 7/8 _2tc [7
{MI2)B ■" 2 Va» ~ 2
MAGNETISM AND MATTER 5/97

20. Use (i) the Ampere’s law for H and (ii) continuity of lines of B , to conclude that inside a bar

magnet,(a) lines of H run from the N pole to S pole, while (b) lines of B must run from the S pole
to N pole. FIGURE 5(N).6

Ans. Consider a magnetic field line of B through the bar magnet as shown in
Fig. 5(N).6. It must be a closed loop. Let C be the amperian loop. Then

= J ^.dT>o
P

H.dl {i.e., positive)

oww
Q Q

It is so because the angle between B and dl is less than 90° inside the bar magnet. So B .dl is positive.

Hence, the lines of ^ must run from S pole to N pole

ee
inside the bar magnet.

FFrlo
Q
<t) H.dl = H.dT +

r
According to Ampere’s law, O H.dT =0 H.dl =0

rF
ee
PQP PQP p Q
P Q
ouru
rF
As H.dl >0. so H.dl <0 (i-e.. negative)
Q P

ffosor
It will be so if angle between H and dl is more than 90°, so that cos 9 is negative. It means the line of H
os k
must run from N pole to S pole inside the bar magnet.
ook
YYo
LONG ANSWER QUESTIONS
Bo
reeB

21. Verify the Ampere’s law^for magnetic field of a point dipole of FIGURE 5(N).7
dipole moment M = M k . Take C as the closed curve running
oouY

clockwise along (1) the z>axis from z = c>0 to (ii) along the
ur

quarter circle of radius R and centre at the origin, in the first


ad

quadrant of x-z plane ; (iii) along the x-axis from x - R to x - a,


and (iv) along the quarter circle of radius a and centre at the
dY

origin in the first quadrant of x-z plane.


nidn

Ans. Refer Fig. 5(N).7, From P to Q, every point on the z-axis lies at the
Re

axial line of magnetic dipole of moment M . Magnetic field induction


FFi

—> A

at a point distance z from the magnetic dipole of moment M (= M k)

IS
4tc 2nz^
(i) Along z-axis from P to Q.
Q Q R

B.dl =
Si//cos0°=j Bdz
P P a

R
iV_L 1 1 '
2
a
2k 2k a
4ti R^ ^
(ii) Along the quarter circle QS of radius R. Refer Fig. 5(N).8, point A lies on the equatorial line of the
magnetic dipole of moment M sin 9.
5/98 Fundament^ Physics (Xn)D319l
Magnetic field at point i4 on the circular arc is

B =
Mo Msin9
; dl = RdQ
4n /?3
rt/2
Ho ^ sin 6-RdO
J B.dl = BdlcosO= j 4k R^
circular arc QS 0

Ji/2

4kr^ (-cose); 4k

(in) Along jr-axis over the path ST. Fig. 5(N).9, every point lies on
the equatorial line of magnetic dipole. Magnetic field inductionat a

w
point distance x from the dipole is
z FieURE5(Nl.a

4kx^

Flo
e
ree
.dl =0

Fr
rF
—> —>
M
[v angle between (-M) and dl is9Cn L.-I
uurr
R

for
(/v) Along the quarter circle TP of radius a. Refer Fig. 5(N).10.
a

Refer case (ii) ; Line integral of B along the quarter circle TP of


s
radius a is
kks
y
Yo
oooo

J bJ- j 4k 4k or
f sine^e
J
eB

cifcular arc TP re/2 re/2

=^^[-cosefi
r

re/2
ou

4k
ad
YY

^ ^ ^ ^ ^ ^
B.dl =J B.dl +J B.dl +J B.dl +j B.dl
nndd
Re

QST P Q S T
Fi

.HpAff 1 L +iio ^ = 0
4k Lfl2 ^2 4tC/?2 I ‘‘“‘-'J
22. What are the dimensions of %y fbe magnetic susceptibility ? Consider an H-atom. Guess an expression
for X upto a constant by constructing a quantity of dimensions of Xt Put of parameters
of the atom : e, m, v, R and Here, m is the electronic mass, v is electronic velocity, R is Bohr
radius. Estimate the number so obtained and compare with the value of I x I ~ for many solid
materials.

I (intensity of magnetisation)
Ans. Magnetic susceptibility, % nt
H (magnetising force)
As / ^d H have same units and dimensions, hence % no dimensions.
In this question, % is to be related with e, m, v, R and iip. We know that dimensions of = [MLQr^]
[From Biot Savart’s law, dB = Ho / dl sin 9
4tc /●2
MAGNETISM AND MATTER 5/99

AnP-dB _ 4nr^ F
dB =
F
or
^H) = ^usinO'
I dl sin d I dl sin 0 qvsin S'

l2x(MLT-2)
Dimensions of = = [MLj2“^] where Q is for charge]
(er-*)(L)xix(G)ar-*)x(i)
As X is dimensionless, it should have no involvement of charge Q in its dimensional formula. It will be so
if |Jq and e together should have the value |1q as e has the dimensions of charge.

w
Let X= m°v^ EF ...(0
where a, c are the powers of m, v and R such that relation (0 is satisfied.
Dimensional equation of (/) is L® T® 0® = (M* O Q~^) x (Q^) (M") x (LT~^)^ x (LY
„ ^l+a ^l+6+c qO

e
roow
Equating the powers of M, L and T, we get

re
0=l+fl or a = -l 0=1+b+c ...(«)
0 = -b or b = 0 From(«), 0=l+0 + c or c = -l

F
Putting values in (0, we get

uFFll
ree
mR

sFr
Here, |Lio = 4jtx lO-'^TmA-' ;e= l-6x 10-*^C,

oro
m = 91 X 10-3> kg ; /? = KT'® m

k
uor
off
^ (47Cx10-^)x(L6x10-»9)2 ^
(9-1X10-31 )xlO-‘0
kos
Y
Yo
10“^
eerBB
oo

X
= 10
10-5
5C(given solid)
urY

23. Assume the dipole model for earth^s magnetic Held B which is given by By = vertical component of
lip 2MCOS0 . M-q sinOAf .
magnetic field =
r3
; Bg = Horizontal component of mangetic field = 4n r3
ou

4n
ad
do

0 = 90** - latitude as measured from magnetic equator.


nY

Find loci of points for which (i) I H I is minimum ; (ii) dip angle is zero ; and (iii) dip angle is ± 45^
nid
Re

Itp 2Af cos6


Ans. (0 Given, vertical component of earth’s magnetic field. By = 4n r3
FFi

M sin 0
B
Horizontal component of earth’s magnetic field H ~
47C r3

B = Jb}+B^ = —■^V4cos^0 + sin2 0 = —^■^3cos^ 0 + 1 ...(0

From (/), we note that B is minimum if cos 0 = 0 or ® ~ which is so, at magnetic equator. Thus, B is
minimum at magnetic equator which is the loci of points.
By _ 2cos0
(ii) If 5 is the angle of dip, then tan 6 = BH sin0
= 2cot0.

Angle of dip 8 is zero if cot 0 = 0. It will be so if 0 = n/2. So, the angle of dip is zero on magnetic equator
which is the loci of points.
(hi) If 8 is 45®, then tan 45® = 2 cot 0 = 2/tan 0or tan0 = 2or0 = tan"l (2) is the loci of points.
5/100 "Pfutdee^'^, Fundamental Physics fXinroTWI
24. Consider the plane 5 formed by the dipole axis and the axis of earth.
Let P be point on the magnetic equator and In 5. Let Q be the point of
intersection of the geographical and magnetic equators Obtain the
declination and dip angles at P and Q.
Ans. Refer Fig. 5(N).ll ;
(0 The point P is on the magnetic equator and in 5, so at P,
declination = 0 and angle of dip = 0
(n) The point Q is at the intersection of geographical equator (G£) and
magnetic equator (ME), so dip = 0 and declination = 11‘3®
25. There are two current carrying planar coils made each from identical
wires of length L. Cj is circular (radius R) and Ci is square (side a).

oww
They are so constructed that they have same frequency of oscillation
when they are placed in the same uniform B and carry the same
current i. Find a in terms of R.

Magnetic moment, Mj =/ij i Aj = 2tiR


^ xi'xtc/?^ =
UR
Ans. For circular coil Ct, n, = — ,

e
rFFlo
* ^ 2tiR 2

re
ree
2 - Li a

F
For square coil Cj, «2 ” ~
4a
Magnetic moment, T I A, = —xixa
^ ^ 4a 4

rF
Moment of inertia of circular coil about the diameter
massx(radius)^ mR^
as axis, /j =

fsfoor
ouur
2 2
Moment of inertia of square coil about an axis passing through its centre parallel to breadth
kosk
2
ma
YYo
12
oo

Time period of oscillation of the magnet in magnetic field is given by


BB

/ 271 MB
T = 2n
rre

and to =
MB T I
oYuu
ad

M. B
2 _ M^B
and CO
2 “
dY

^2
M
L1RI2 _ Llal4
innd
Re

Given, 0^2=0^, so
h h
or
mR^n nuP-m
FFi

On solving, a = 3 R
MAGNETISM AND MATTER 5/101

NEET/JEE
SPECIAL

rr
o

ww
● MCQs in Physics for MEET
y.iv-
i
●●
iPradeep’s Stellar Series.... ● MCQs in Physics for JEE (Main)

Floo
ee
HJ Multiple Choice Questions one correct Answer]

eer
rFrF
1. Force» torque and potential (c) >/3J
energy of a magnet in magnetic field
rur
[AIPMT (Main) 2012]
1. A magnet with moment M is given. If it is bent
into a semicircular form, its new magnetic
ffoor
4. An insulating rod of length L carries a charge Q
s
distributed uniformly on it. The rod is pivoted at
osk
YYoou
moment will be :
its mid-point and is rotated at a frequency v about
oook

(a) M/n (b) M/2 a fixed axis perpendicular to the length of rod
eBB

{c)M {d) 2 M/k passing through the pivot. The magnetic moment
(DPMI 2002) of the rod system is
2. An insulating thin rod of length I has a linear nQvI? nQvI?
r

(a) (b)
ouur
ad

X
2 3
charge density = Pq y on it. The rod is rotated nQvl}
nQvl}
Y

about an axis passing through the origin (x - 0) (c) id)


12
6
and perpendicular to the rod. If the rod makes n
nd dY

5. A bar magnet of length and magnetic dipole


Re

rotations per second, then the time averaged


moment ‘M’ is bent in the form of an arc as shown
magnetic moment of the rod is :
FFini

71
in Fig 5(CF).l. The new magnetic dipole moment
(a) 7C n p ib) — no /3 will be
3 ^
71
(c) -np /3 id) n p
4
(JEE Main 2019)
3. A magnetic needle suspended parallel to a
magnetic field requires Vs J of work to turn it
through 60". The torque needed to maintain the
needle in this position will be :
(a) 2V3J (fc)3J
ANSWERS

1. id) 2. ib) 3. ib) 4. id)


5/102 “P>t<uUcfa. \ Fundamental Physics (XII) P75T1
M I nl} 111}
Am^
(«) y {b) M (c)
4A
Am“ id)
7T

(NEET 2020)
ic) Im (d)
K Tt 10. A uniform conducting wire of length 12 a and
(AIPMT 2013) resistance 'R' is wound up as a current carrying
coil in the shape of,
6. A 250 turn rectangular coil of length 2-1 cm and
width 1-25 cm carries a current of 85 nA and (0 an equilateral triangle of side 'a'
subjected to a magnetic field of strength 0-85 T. («) a square of side ‘a’
Work done for rotating the coil by 180" against The magnetic dipole moments of the coil in each
the torque is case respectively are
(a) 9-1 pJ (b) 4-55 pJ
(fl) 4 la^ and 3 la^ (b) -s/3 la^ and 3 la^

ww
(c) 2-3 pJ (d) M5pJ
(NEET 2017) (c) 3 la^ and la^ {d) 3 la^ and 4 la^
7. Following figures show the arrangement of bar (NEET 2021)

FF loo
magnets in different configurations. Each magnet 11. A wire of length L metre carrying a current of /
has magnetic dipole moment (m). Which ampere is bent in the form of a circle of two turns.

ee
configuration has highest value of magnetic Its magnetic moment is

ee r
dipole moment ?

rFrF
(c) Il}lA K Am^ ib) IL^/S n Am2
5(CF) (c)/7tZ,2/8 Am^ (d) 4 II}Ik Am^
rur
II. Magnetic field due to magnet;
ffoor neutral points and vibration
ks
magnetometer
YYoou
ookos

12. Two identical magnetic dipoles of magnetic


moment 1-0 Am^ each, placed at a separation of
BBo

2 m with their axes perpendicular to each other.


re

The resultant magnetic field at a point midway


between the dipoles is
ouur
ad

(a) 5 X KT'^T {b) ->/5x10-’T


Y

(c) 10-^ T (J)2x KT’T


(AIPMT 2014) 13. Two identical magnetic dipoles of magnetic
dndY
Re

8. A magnetic needle of magnetic moment 6-7 x moments 2 Am^ are placed at a separation of 2 m
lO*"^ Am^ and moment of inenia 7-5 x 10“^ kg with their axes perpendicular to each other in air.
FFini

m^ is performing simple harmonic oscillations in The resultant magnetic field at a mid point
a magnetic field of 0 01 T. Time taken for 10 between the dipoles is
complete oscillations is (a) 4->/5xlO^^T Qj) 2V5xl0“^T
(a) 6-98 s {b) 8-76 s
(c) 6-65 s id) 8-89 s (c) 4-s/5x10"''T id) 2V5xl0-'^T
(JEE Main 2017) (e) 4>/2xlO^T (Kerala PET 2011)
9. A wire of length L metre carrying a current of I
14. A vibration magnetometer placed in magnetic
ampere is bent in the form of a circle. Its magnetic
moment is:
meridian has a small bar magnet. The magnet
executes oscillations with a time period of 2 sec
11} 11} in earth’s horizontal magnetic field of 24 micro
ia) Am^ {b) — Am^ tesla. When a horizontal field of 18 microtesla is
4tc 4
ANSWERS
5. (< ) 6. {<0 7. (c) 8. (f) 9. («) 10. (/j) II. (/>) 12. {/)) 1.3. (</)
MAGNETISM AND MATTER 5/103

produced opposite to the earth’s field by placing 19. An iron rod of suscentibility 599 is subjected to a
a current carrying wire, the new time period of magnetising field of 1200 A m"^ The permeability
magnet will be of the material of the rod is :

(«) 1 s ib)2s (|iQ = 4 7C X 10"^ T m A'^)


(c)3s (d)4s (AIPMT 2010) (a) 8-Ox 10-5 Tm A-'
15. A magnet is cut in three equal parts by cutting it (b) 2-4 Ttx 10-5 TmA-*
perpendicular to its length. The time period of (c) 2-4 K X 10-’ T m A”*
original magnet is Tq in a uniform magnetic field (d) 2-4 71X 10-^Tm A-' {NEET 2020)
B. Then, the time period of each part in the same 20. Magnetic materials used for making permanent
magnetic field is magnets (P) and magnets in a transformer (T) have
T T different properties. Of the following, which
(i-) -f property best matches for the type of magnet
required ?

ww
(d) none of these (ii) T: Large retentivity ; Small coercivity
(b) P : Small retentivity ; Large coercivity
(AIIMS 2014) (c) T; Large retentivity ; Large coercivity

Floo
16. A small bar magnet placed with its axis at 30° {d) P : Large retentivity ; Large coercivity
with an external field of 0-6 T experiences a torque

ee
(JEE Main 2020)
of 0*018 Nm. The minimum work required to 21. An iron rod of volume IQ-^ m^ and relative

reer
rotate it from its stable to unstable equilibrium

rFF
permeability 1000 is placed as core in a solenoid
position is with 10 tums/cm. if a current of 0-5 A is passed
(a) 9-2 X 10-5 j (b) 6 4 X 10-5 j
uur r
through the solenoid, then the magnetic moment
(c) ll*7x 10-5 J (d) 1-2 X 10-5 J
(JEE Main 2020)
ffoor
of the rod will be

(a) 0-5 X 10^ Ara“ {b) 50 X 105 Am5


sks
(c) 5 X 10“ Am^ {d) 500 X 105 Am5
YYoo
ooko

III. Magnetic elements at a place (JEE Main 2020)


eBB

17. If 0 J and 62 be the apparent angles of dip observed IV. Oia, Para and Ferromagnetic
materials
in two vertical planes at right angles to each other,
then the true angle of dip 6 is given by
r

22. A perfectly diamagnetic sphere has a small


ouur
ad

{a) cot5 9 =s cot5 6j -f cot5 62 spherical cavity at its centre, which is filled with
ib) tan5 0 = ian5 0| + tan5 02 a paramagnetic substance. The whole system is
Y

(c) COt5 9 = COt5 0, - COt5 02 placed in a uniform magnetic field B . Then the
dY

field inside the paramagnetic substance is


Re

{d) tan5 0 = tan5 0, - tan5 02 (NEET 2017)


nnd

18. At a point A on the earth’s surface the angle of


FFii

FIGURE 5(CF).3
dip, 5 = + 25°. At a point B on the earth’s surface
the angle of dip, 5 = - 25°. We can interpret that:
(a) A and B are both located in the northern
hemisphere
{b) A is located in the southern hemisphere and -»

B is located in the northern hemisphere (a) much larger than I B I and parallel to B
(c) A is located in the northern hemisphere and
{b) zero (c) B
B is located in the southern hemisphere
{d) A and B are both located in the southern {d) much larger than I B I but opposite to B
hemisphere (NEET 2019) (JEE Main 2020)
ANSWERS
14. (r/) IS.ih) 16. (</) 17. («) 18. (c) 10. (</) 20. ((/) 2I.(< ) 22. ih)
5/104 pfuieUe^’4. Fundamental Physics fXinpJSTWl
23. Hysteresis loop for two magnetic materials A and 27. Ferro magnetic materials used in transformer must
B are given below : have

(fl) low permeability and high hysteresis loss


(b) high permeability and low hysteresis loss
(c) high permeability and high hysteresis loss
(d) low permeability and low hysteresis loss
(Karntataka GET 2009)
28. If the magnetising field on a ferromagnetic
material is increased, its permeability is
(a) decreased (b) increased
(c) is unaffected
(d) may be increased or decreased
These materials are used to make magnets for
electric generators, transformer core and electro (AlIMS 2015)

ww
magnetic core. Then it is proper to use : 29. A 250 turn rectangular coil of length 2-1 cm and
width 1-25 cm carries a current of 85 )iA and
(a) A for electric generators and transformers
subjected to a magnetic field of strength 0-85 T.
(b) A for electromagnets and B for electric Work done for rotating the coil by 180** against

Floo
generators the torque is
(c) A for transformers and B for electric (a) 9-1 pJ (b) 4-55 pJ

ee
generators (c) 2-3 fiJ (d) 1-15 pJ

reer
rFF
(d) B for electromagnets and transformers. (NEET 2017)
(JEE Main 2016) 30. The variation of magnetic susceptibility % with
the temperature Tof a ferromagnetic material can
uur r
24. For which of the following magnetic materials.
5 = pW is valid, where B is the magnetic induction
and H is the magnetising field and p is nearly a
ffoor
be plotted as
sks
constant for a wide range of H?
YYoo
(a) Diamagnetic and ferromagnetic materials only
ooko

(b) Diamagnetic and paramagnetic materials only


eBB

(c) Paramagnetic and ferromagnetic materials only


(d) Diamagnetic, paramagnetic and ferromagnetic
materials
r

(US 2010)
ouur
ad

25. Needles N^, N2 and are made of a ferromag


netic, a paramagnetic and a diamagnetic substance
Y

respectively. A magnet when close to them, will


nddY

(a) attract N^ strongly, but repel N2 and weekly


Re

(b) attract all three of them


FFiin

(c) attract and N2 strongly but repel N3


(d) attract Nj strongly, N2 weakly and repel A^3
weakly (AIEEE 2006)
26. If the magnetic dipole moment of an atom of (AIIMS 2015)
diamagnetic material, paramagnetic material and
31. A bar magnet is demagnetised by inserting it
ferromagnetic material are denoted by p^, p^ and inside a solenoid of length 0-2 m, 100 turns and
respectively, then :
carrying a current of 5-2 A. The coercivity of the
((3) Pp = 0 and Py 56 0 bar magnet is
ib) and p^ = 0 (a) 285 A/m (b) 2600 A/m
(c)pj;t0andpy9i0 (c) 520 A/m id) 1200 A/ra
(f/) Prf = 0 and Pp 0 (JEE Main 2019)
ANSWERS

23. (£0 24. (d) 25. (d) 26. (d) 27. (b) 28. (a) 29. (a) 30. (b) 31. (b)
MAGNETISM AND MATTER 5/105

32. The coercivity of a small magnet where the 34. A thin diamagnetic rod is placed vertically
ferromagnet gets demagnetized is 3 x 10^ Am“*. between the poles of an electromagnet. When the
The current required to be passed in a solenoid of current in the electromagnet is switched on, then
length 10 cm and number of turns 100, so that the diamagnetic rod is pushed up, out of the
the magnet gets demagnetized when inside the horizontal magnetic field. Hence, the rod gains
solenoid is : gravitational potential energy. The work required
to do this comes from
(a) 3 A (b) 6 A
(a) The lattice structure of the material of the rod
(c) 30 mA (d) 60 mA {b) The magnetic field
(JEE Main 2014) (c) The current source
33. A superconductor has T^. (0) = 100 K. When a (d) The induced electric field due to the changing
magnetic field of 7-5 tesla is applied, its magnetic field (NEET 2018)
decreases to 75 K. For this material, one can 35. A bar magnet is osciallating in the earth’s
definitely say that when magnetic field with a period T. What happens to

ww
(a) 5 = 5 tesla, (B) = 80 K its period of motion if its mass is quad
rupled without changing its dimensions ?
(b) B = 5 tesla, 75 K < (B) < 100 K
(fl) Motion remains SHM with period = T/2

FF loo
(c) = 10 tesla, 75 K < r^(B)< 100 K (b) Motion remains SHM with period = 2 T
(i/) B = 10 tesla. (B) = 70 K (c) Motion remains SHM with period = 4 T

ee
(AIPMT 2010) (d) Motion is not SHM, but period = T

ee r
rFrF
tlBl MuHipIo CholCO Questions (with One or More than One Correct Answers)
rur
36. A magnetic dipole is placed north south. Pj, P2, ffoor FIGURE 5(CF).6
ks
Qj, Q2 are four points at the same distance from
the dipole towards north, south, east and west
YYoou
ookos

respectively. The directions of magnetic fields due


to the dipole are same at
BBo

(a) and P2 (b) <21 and 02


re

(c) P| and Q, (d) P2 and 02-


37. A horizontal circular loop carries a current that
oouur
ad

looks anticlockwise when viewed from above. It


Y

is replaced by an equivalent magnetic dipole NS, 39. An infinite current carrying wire passes through
which of the following is true ? point O and is perpendicular to the plane con
taining a current carrying loop ABCD as shown
dndY

(ij) The line NS should be along a diameter of the


Re

loop. in Fig. 5(CF).7. Choose the correct option (s).


FFini

(b) The line NS should be perpendicular to the


FIGURE 5(CF).7
plane of the loop,
(c) South pole should be below the loop.
(d) North pole should be below the loop.
38. In the given diagram, a line of force of a particular
field is shown in Fig. 5(CF).6. Out of the -●O'

following options, it can never represent o

(a) an electric field


(b) a magnetic field
(c) a gravitational field of a mass at rest
(d) an induced electric field

ANSWERS

32. (a) 33. (b) 34. (c) 35. (b) 36. (o.b) 37. (b.c) 38. (a.c)
5/106 Fundamental Physics fXTTIPZSWl
(a) Net force on the loon is zero, \OOnM
(d) tiic iiiagniluac of magiicLic uciu is
(fo) Net torque on the loop is zero. units 3(2
(c) As seen from O, the loop rotates clockwise. (JEE Advanced 2013)
{d) As seen from O, the loop rotates anticlockwise.
41. A steady current I flows along an infinitely long
40. A particle of mass M and positive charge Q hollow cylindrical conductor of radius R. This
-1 cylinder is placed coaxially inside an infinite
moving with a constant velocity u I = 4 f ms ' solenoid of radius 2 R. The solenoid has n turns
enters in a region of uniform static magnetic field, per unit length and carries a steady current /.
normal to the x-y plane. The region of the Consider a point P at a distance r from the
magneticfield extends from = 0 to jc = L for all common axis. The correct statement(s) is (are)
values of y. After passing through this region, the (a) In the region 6 < r < /?, the magnetic field is
particle emerges on the otherside after 10 milli non-zero

seconds with a velocity = 2(%^?^-i-y)ms -1 (b) In the region /? < r < 2 /?, the magnetic field is

ww
The correct statement(s) is (are) along the common axis

(a) the direction of magnetic field is - z direction (c) In the region P < r < 2 /?, the magnetic field is

FF loo
tangent to the circle of radius r, centred on
(b) the direction of magnetic field is + z direction the axis
50jtM

ee
(c) the magnitude of magnetic field is (d) In the region r> 2 R, the magnetic field is
units 3(2

Fe r
non-zero
(JEE Advanced 2013)

Ql"Multiple

Frre
Choice Questions (Based on the given Passage/Comprehen slon)
rur
correct option. Choose the correct option.
fforo
Each comprehension given below is followed by some multiple choice questions. Each question has one
ks
D
YYouo
koso

The earth’s magnetic Answer the following questons


field is approximately like that of a giant 42. The intensity of earth’s magnetic field at the
BBoo

magnetic dipole, whose axis is inclined surface of earth is of the order of


roughly 20“ west of the axis of rotation of
r ee

earth which is the geographic N-S direction. (a) 10-^ T (b) 10-^ G
The strength of earth’s magnetic field is of (c) 10"'-* T (d) lO-^G
ad
oouur

the order of 10”^ tesla. Three magnetic


43. The vertical component of earth’s magnetic field
Y

elements of earth are : Magnetic declination


(6), magnetic inclination or magnetic dip (6) is zero at a place where angle of dip is
dnYd

and horizontal component (H) of earth’s


Re

(a) 45“ (b) 60“


magnetic field. Magnetic declination is the (c) 90“ id) O'
FFini

small angle between magnetic axis and


geographic axis at a place. Magnetic dip at a 44. At magnetic poles, a compass needle points out
place is the angle, which the direction of total (u) along N-S
strength of earth’s magnetic field (R) makes (b) in any direction
with a horizontal line in magnetic meridian.
If // is horizontalcomponentand V is vertical (c) along
componentof earth’s magneticfield, then H (d) at 45“ to N-S direction.

= /? cos 5, V = /? sin 5 and R = . 45. Magnetic declination is


At a place on the surface of earth. H is of the (a) 20“ west (b) 20“ south
order of 3*2 x 10"^ tesla.
(c) 20“ east (d) 20“ north

ANSWERS

39. Ui.v) M). Ui.i ) 41. Ui,(l) 42. (u) 43. UD 44. (/;) 45. iti)
MAGNETISM AND MATTER 5/107

09 Matching Type Questions


DIRECTIONS. In each of the following questions, match column I and column II and select the correct
match out of the fovir given choices.
46. There are physical quuniities in column I and demensional formulae in column II
Column I Column II

(A) Permeability (p) [AfL^T^A^]


(B) Magnetic induction ig)
(C) Susceptibility (r) [MLT-^A~^]
(D) Intensity of magnetisation (s)

(a) A-p; B-(? ; C-r; D-j ib) A-<? ; B-r; C-^ ; D*p (c) A-r; ; C-p ; D-^ (d) As ,B~p \ C-q ; D-r

w
Flo
Matrix-Match Type Questions
p <»
I— 1

ee
DIRECTIONS. Each of the following questions contains statements
given in two columns, which have to be matched. The answers to these
A!
© Oo©

Fr
questions have to be appropriately bubbled. If the correct matches are © © lD 0
A-Pj A-q; B-r; C-r, C-s and D-s, then the correctly bubbled matrix
Cj 0 ©O©
for
will look like the one shown here.
ur
47. Column I Column II
D
© ©©©
ks
Yo
(A) Paramagnetic material ip) They move from weaker to stronger part of the magnetic field.
oo

(B) Ferromagnetic material iq) The value of susceptibilityis positive.


eB

(C) Hard magnetic material ir) Permanent magnet


(D) Soft magnetic material is) Core of transformer
r
ou
ad

48. Column I Column II A B C D


Y

(A) Magnetic moment (P) scalar

000,0
nd

(B) Magnetic induction (9)


Re

vector

(C) Permeability ir) NIA-m


© © @ ©
Fi

Intensity of magnetization Nm^/Wb


(D) is)
® ® ® ®
VI. Integer Type Questions 0 @ 0 0
® ® © ©
DIRECTIONS. The answer to each of the following questions is a single digit
integer, ranging from 0 to 9. If the correct answers to the question numbers A, © © © ©
B, C and D (say) are 4, 0, 9 and 2 respectively,then the correct darkening of ® ® © ©
bubbles should be as shown on the side :

49. The net magnetic moment of the system of magnetic poles shown in Fig.
! I
(9)i9H9)t91
>( V7!
5(CF).8 is n ma where n is nearly :

ANSWERS

46.(r) 47. A-p, q ; li-p, q, r, .v; C-p, q, r ; D-/j, q, s 48. A-^/, .V; B-</, r ; C-p ; D-q
5/108 "P%etd€e^ ^ Fundamental Physics (XII) P7STW1

FIGURE S(CF),9
Sl Ni
P
I-

(i) K 0.3 m >■

50. The coercivity of a bar magnet is 120 A/m. It is to be


demagnetised by placing it inside a solenoid of length 120 cm
0.4 m

ww
and number of turns 72. The current (in A) flowing through
the solenoid is :
N2
51. Two short magnets (1) and (2) of magnetic moments 2 Am^
and 5 Am^ respectively are placed along two lines drawn at

Flo
X

90® to each other as shown in Fig. 5(CF).9. At the point of

e
intersection of their axes, the magnitude of magnetic field is

reree
nx 10-5 T. What is n? $2

r FF
VII.
Assertion-Reason Type Questions
uurr
FOR MEDICAL STUDENTS foor Reason. Charge is responsible for dipole moment.
(a) A (b)B (c)C (^OD
kss
DIRECTIONS. In each of the following questions 55. Assertion. The magnetic moment (|x) of an
Yoo
ooook

read the two statements and choose if:


electron revolving around the nucleus decreases
(A) both Assertion and Reason are true and the with increasing principal quantum number (n).
eBB

Reason is the correct explanation of the Reason. Magnetic moment of the revolving
Assertion. electron, ja«« «.
(B) both Assertion and Reason are true but Reason (a) A {b)B (c)C (d)D
rr
ouu

is not a correct explanation of the Assertion. (AIIMS 2015)


ad

(C) Assertion is true but the Reason is false. 56. Assertion. Above Curie temperature, a
Y

(D) both Assertion and Reason are false. frerromagnetic material becomes paramagnetic.
dY

Reason. When a magnetic material is heated to


52. Assertion. Poles of a magnet can never be very high temperature, it loses its magnetic
Re
innd

separated. properties,
Reason. Since each atom of a magnetic material (a) A (b)B (c)C (^/)D
Fi

is a magnet in itself. (AIIMS 2015)


(a) A (b)B (c) C (d)D FOR ENGINEERING STUDENTS
53. Assertion. Magnetic moment is a vector quantity,
DIRECTIONS. In each of the following questions
whose direction inside the magnet is, from South read the two statements and choose if,
to North.
(A) Statement-1 is true ; Statement-2 is true ; State-
Reason. Magnetic lines of force emanate from ment-2 is a correct explanation of Statement-1.
A'-pole and enter into the S-pole. (B) Statement-1 is true ; Statement-2 is true ;
(a) A (/7)B (c)C (d)D Statement-2 is not a correct explanation of
54. Assertion. An atom of a magnetic material Statement-1.
behaves as a dipole because of positive charge (C) Statement-] is true ; Statement-2 is false.
on the nucleus. (D) Statement-1 is false ; Statement-2 is true.
ANSWERS

49.(2) 50.(2) 51.(2) 52. (a) 53. (n) 54. (d) 55. (d) 56. (a)

$
MAGNETISM AND MATTER 5/109

57. Statement-1. A current I flows along the length 58. Statement-1. Magnetic field strength at a point
of an infinitely long straight and thin walled pipe. on axial line of a bar magnet is along South to
Then the magnetic field at any point inside the North pole of magnet.
pipe is zero, Statement-2. The magnetic field strength can
f —>
Statement-2, o = never be along North to South pole of magnet.
(fl)A (fc)B (c)C id)D
(a) A .(b)h (c)C (d)D

A A 9

For Difficult Questions

H Multiple Choice Questions (with one correct Ansvyer)j

ww
1. A/ - magnetic moment, m - pole strength .*. ■yfi -- blB (cos 60® - cos 0®)

FF loo
2 I - length of magnet 1
Af = ~MB --1
...(0 12

ee
As M= mx 2 I, m= —
21
MB = 2S

ee r
or
When it is bent into semi-circular form, then

rFrF
21 X =MB sin 0 = 2V3 sin 60®
7cr = 2 /, r = —
rur
TC
= 2V3x —
Distance between two poles, 21' = 2 r ~ —
4/
ffoor 2
= 3J
ks
71
4. Refer to Fig. 5(CF).10, consider a small element
4/ of rod of length dx at distance x from the axis of
YYoou
New magnetic moment M' = m x2 I' = mx —
ookos

TC rotation.

Putting value of (i)


BBo

FIGURE 5(CF).10
Axis of rotation
re

M ^ 4/_ 2M
21 71 n
ouur
ad

2. Charge on the element of rod of length dx at <-


Y

distance x from axis of rotation. X

dq^p dx Charge on small element dx,


dndY

Current developed on element,


Re

Q
dJ~dqxn = pdxn dQ=-dx
FFini

Magnetic moment developed in the element due Current in the element due to rotation of rod
to rotation of rod, dM = d!xn7?- = {p dxn)xn:)?-
Q
= np n 7?- dx dl = vdQ = v — dx
Total magnetic moment developed due to rotation
of rod is Magnetic moment of the small element
f vO \ 0
M =
rl
2 . TC p rt dM ={dl)xnx^ = —dx nx^
Tip n X dx = —t- \ ^
0 3
2 j
3. Hei«,e, =O®,02=6O®, -——x^dx
L
W=^J ,x=l Total magnetic moment of rod system due to
As W=-MB (cos 02-cos 0i) rotation of the whole rod is

ANSWERS

57. (a) 58. (c)

i
5/110 Fundamental Physics (XIDESSBl
In case (c),
For Dlffficuit Queations

+1/2
Afj = +m^+ 2mm cos 30*
M ^ f x^dx = nvQl}
L L 12
= +nP-^ =m-^2 + ^
-1/2
= m V3-732 =1-93 m
M
5. Before bending, strength of each pole,
On bending, as shown in Fig. 5(CF).11, distance
60“
between poles = AB = lrs\n — = r

FIGURE S(CR.11

w
/
I
IN B
Y
\ /
/

FF loow
\
\ /
\ t
/

ee
r\ /'●
N30^i30“,

Fr
'●/
V
O

rer
ur
New magnetic moment, M' = mxr fofr Fo
ks
M / ZM
YYouro
/ n/3 n
s oo

Choice (c) is correct


BBook

6. Here N = 250;/ = 21 x lOr^ m;


r ee

Z>= 1*25 X 10-2m;


/ = 85x 10-^ A,
oouru
ad

B = 0-85T
Y

Magnetic moment of coil due to current


In case (d),
M = NIA=NIlxb
Ynd
Re

= 250 X (85 X 10-^) X (2-1 X 1-25) x lO"^ M^ = + m2 + 2m.m cos 60“


FFindi

Work done, W = MB (cos 0| - cos 62) = = 1732 m


= MB (cos 0“ - cos 180“) = 2 MB
Choice (c) is the correct answer.
= 2 X 250 X (85 X 10“^) x (21 x 125) 8. Here, M = 6-7 x 10-2 ^m2
X 10-4 X 0-85
/ = 7-5xl0-^kgm2
= 91 X 10^J = 9*ljiJ B = 001T;/=?
7. As is known, the direction of magnetic moment If T is time period of vibration of needle, then
of a magnet is along SN
In case (a). From Fig. 5(CF). 12(a) Resultant 7-5x10-6
T = 2k
magnetic moment 6-7x10-2x0-01
r= 0-665 s
M
I = V/n2+m2 =mV2 =1-414 m
For 10 complete oscillations,
In case (b), M2 = m - m = 0 / = 10 T = 10 X 0-665 s = 6*65 s
MAGNETISM AND MATTER 5/111

For Difficult Questions L


11. Here, L = 2x2n r orr =
4tc
9. Let r be the radius of the circle formed.
M = nIA=2xIxKn
L
Then, L = 2nr or r =—
2tc H?
= 2 / X 71 X (L/4 71)- = — Am^
Magnetic moment = !a = I (Tcr^) 87T

= Inx(U2nf 12. As the axes are perpendicular, mid |X)int lies on


axial line of one magnet and on equatorial line of
Am^ other magnet.
47U

V l()-^x2xl
B,=
10. Current / = — is same for both the loops.

ww
R 4ti I'’
= 2x 10-7 T

FF loo
10'7 xl
and ^7 - - 10 7 T
4tu d^ I--’

ee
As B,I IB2’

Fr
.*. Resultant field = + =s/5xHr7T

rree
ur
13. The point P lies on the axial line of magnet /Vj Sj
fofr oF
at distance rj (= 1 m) and on equatorial line of
magnet A/2 *^2 Fig. .5(CF).14.
ks
YYouro
soo

Total length of the conducting wire = 12 a FIGURE S(CF).14


N2
(0 In case of triangle loop, of each side a,
BBook

length of wire = 3a P Bi I

E
r ee

Number of turns in the triangular loop of the wire M2


Si N1
oouru
ad

I2n
A? = = 4 B
3a B2 S2
Y

1
Area of triangle, A = -basexheight _Hq2M|
Ynd

= 4x10"7t
Re

.-. B. =
3
471
1
FFindi

a
= —oXflsin 60° =
2 4
B^7 =-^^
4tc
= 10'7x^
|5
= 2xI0-7t
Magnetic dipole moment of triangular loop
a
Resultant magnetic field at P,
M =NIA = 4Ix = ^la^
4

(ii) Total length of four sides of a square of each


B =
^Bf + Bl =V('^xI0-7)2+(2x!0-7)2
side a = 4 a. Number of turns in square loop, = 2-75x10-71'
N = 1^=3 14. Here, T, = 2 s, T2 = ?//,=// = 24 x IQ-^ 7’
4a
//2 = /7-F=24- 18 = 6^1 r=6x 10-^ T
Area of square. A' =
T, H 24xlQ-*^
.●. Magnetic moment of square loop As
T.1 Hi \ 6x10"*^
M' = I^ lA'
M' = 3Ia^ r2 = 2 7'|=2x2 = 4s
5/112 “PnacCee^ ^ Fundamental Physics (XII) tviwi
iH I MiTISVEXiBiyAN!dTii:ONSl
For Difficult Questions
19. Here, x,„ = 599, /= 1200 Am'’. ^ = ?
H = Ho(l +%J = 4tcx I0-'^(1 +599)
15, For original magnet, = 2-4 JE X ir** T mA-l
20. The best choice is {d), because for permanent
/
r„0 = 27c ...(/) magnet P, retentivity should be as large as possible
\MB
and coercivity should also be as large as possible.
When a magnet is cut into three equal parts, 21. Here, y= A/= 10”^ ; n=10tums/cm
perpendicular to its length, then magnetic moment = 1000 lurns/m
of each part is M' = M/3. Moment of inertia of

oww
W = rt X / = 1000 X/,
each part is
7 = 0-5 A, p^= 1000
r = ~ (Zy3)^ __L 'llL-1.
m
M = NIA(\i^-\) = (nl) M (p^- 1)
3^ 12 ~ n ~ 11 = n/y(p,-l)

ee
= 1000 X 0-5 X 10-3 (1000-1)
r
New time period, T' = 2k = 499-5 = 500 = 5 X 10^ Am^

FFrlo
r
yAf'B 22, Magnetic field lines do not tend to pass through

rF
diamagnetic substance. Therefore, magnetic field

ee
nil
= 2ti inside the paramagnetic substance at the centre
ouru {M/3)B of sphere is zero as shown in Fig. 5(CF).15.

rF
T.0

ffosor
1 I
= -2n. os k
3 3

16. Here 0 = 30°, = 0*6 T, X = 0-018 Nm


ook
YYo
From, X = MB sin 0
Bo

X 0-018
= 0-06 Am^ 23. We know that area of the hysteresis loop tells the
reeB

M =
BsinO 0-6 X1/2
net energy absorbed per unit volume by the
oouY

For stable equilibrium, 0 = 0° material per cycle of magnetisation.


ur

For unstable equilibrium, 0 = 180° We also know that for electromagnets and trans
ad

formers, the energy loss should be small. Which


IniUal P.E. = (/,- = - MB,
Y
d

is for Fig. 5(CF).4(/>). Thus option (</) is true.


Final P.E. = MB
24. The relation = pi/ is valid for all Diamagnetic,
nidn

Work done = MS - MB) = 1MB


Re

paramagnetic and ferromagnetic materials.


= 2 X 0-06 X 0-6
25. As ferromagnetics tend to move towards stronger
FFi

= 7-2 X 10-2 j magnetic field, so, Wj will get attracted strongly,


V paramagnetic also get attracted to stronger
17. Here, tan Sj = -~
H
and tan 0.,2 = —
r» magnetic field upto some extent. So will also
1
get weakly attracted, but diamagnetic, i.e.,N^ tend
y
and tan0 = — to move away from magnetic field, hence will
H get weakly repelled.
.*. H^ = VcotQ^, //2=Vcot02, 7/=Vcot0 26. According to electron theory of magnetism, an
//2=//2 + t/2
atom of a diamagnetic material has no intrinsic
As
dipole moment, whereas atom of a para-magnetic
.’. cot^ 0 = y2 cot^ 0] + cot^ 02 material has some intrinsic dipole moment
Hence, cot^ 0 = cot^ 0j + cot^ ©2 i.e., Prf = 0 and
18. Angle of dip is zero at equator, positive in 27. A ferromagnetic materials core used in
northern hemisphere and negative in southern transfarmer must have high permeability so that
hemisphere. it may show high value of magnetic induction for
MAGNETISM AND MATTER 5/113

100
For Difficult Questions
.●. n / = Ho // or H = n I = x5-2
0-2

the given magnetising field. It should have low = 2600 A/m

hysteresis loss in order to decrease the energy 32. Here, coercivity of given magnet
losses.
B
28. Magnetic permeability of a ferromagnetic — = 3 X 10^ Am-'
1
^^0
substance, p, = B/H or H«
H BL (3xl0^)x0-l
As B = or / = = 3A
where H is the magnetising field. If H increases, 100
L 1^0^
then H decreases.

oww
29. Here Af=250;/ = 2-I x 10^2 m ; 33. Given, for the magnetic field 0 to 7-5 tesla, the
ft = 1 -25 X 10-2 m ;/= 85 X 10“^ A, value of critical temperature for given
superconductor lies in between 100 K to 75 K.
B=0-85T
Therefore, for B = 5 tesla, critical temperature of
Magnetic moment of coil due to current superconductor is more than 75 K but less than

e
M = NIA =NIlxb
100 K. Hence, option (ft) is correct.

FFrlo
re
= 250 X (85 X 10-^) X (2-1 X 1-25) x IQ-^ 34. The gravitational potential energy gained by the
Work done, W = MB (cos 0] - cos 62) diamagnetic rod comes from the energy of

ree
F
= MB(cos0°-cos 180°) current source.

= 2 MB

rF
35. r=2 7t V//M//
= 2 X 250 X (85 X lO"^) x (2-1 x 1-25)
where / = moment of inertia of magnet about the
X 10“^ X 0-85

fsoor
ouur
= 9-1 X l(r^J = 9-lHJ (/2+ft2)
skf suspension fibre = mass
30. The magnetic susceptibility (%) of a ferromagnetic 12
ooko
substance decreases with increase in temperature when mass is quadrupled. I becomes 4 times.
Yo
i.e. M(T - T^). Hence, option (ft) is true.
Y
Therefore, T becomes ^ times i.e. 2 T the
Bo

31. Coercivity = magnetising force, H


motion continues to be simple harmonic.
reB

As, B = |iQ // and B = Ho ” ^


uur
ooY

Choice Questions (with One or More than One Correct Arisyirfersj^


ad
dY

36. As is clear from Fig. 5(CF).I6, the directions of 37. The magnetic field due to the horizontal circular
magnetic fields due to dipole are same at (Bj, P-^) loop is perpendicular to the plane of the loop and
innd
Re

and at (fii. Q^)- upwards. Therefore, the line NS of dipole should


FFi

be perpendicular to the plane of the loop and north


FIGURE S(CF).16
pole should be upwards or south pole should be
i .N
downwards.

P1 38. A current carrying circular coil does not have an


electric field and a gravitational field. But it has
n
magnetic field and induced electric field if the
magnetic field due to current in the coil is
Q2 Q1
W E changing with time.
39. There is no force on the portion of wire AB and
S
CD as current in them is parallel to the direction
of magnetic field. The magnetic force on wire BC
is perpendicular to the loop directed outwards and
P2
on wire OA, the magnetic force would be
s
5/114 “PnaeUe^ k Fundamental Physics (XII)

For Difficult Questions 0 _ 7t/6 _ Mn


<0 ~ QBIM ~ 6QB
perpendicular to the loop directed inwards. Both
these forces will be equal in magnitude. Therefore, M n M K
or B =
the net force on the wire loop is zero and torque 6Qt 6(2x(10xl0--^)
on the loop would be along the clockwise
direction as seen from O. 50 A/71

40. Refer to Fig. 5(CF).17, component of final 3e


velocity of particle is in positive y-direction. The
centre of circular path of particle in magnetic field 41. Let B(- and B^ be the magnetic feld due to current

ooww
is present on positive y-direction. So magnetic
in cylinderC and solenoid S respectivelyat point
P, distance r from the common axis.
field is present in negative Z'direction.

e
re
rFFl
ree
F
rF
fsfoor
ouur
kosk
YYo
oo
BB

If 0 is the angle of deviation of the particle with


rre

x-axis while emerging from magnetic field, then


oYuu
ad

2 I 71
tan 0 = = tan — (n) For Q<r<R,
2V3 ^ 6
dY

B = B^=|XQn/9!:0 hence correct.


(b) For R<r<2R
innd

7t
Re

or 0 = -
6
B = ^Bj + b}.
FFi

Angular velocity of rotation of particle in


it is not acting along the axis of cylinder, hence
QB wrong,
magnetic field, (o =
M
(c) For R < r<2 R, B is not in the plane of circle,
Time taken by particle to cross the magnetic field hence wrong.
IS
(d) For r>2R,B^0, hence correct.

DU MultipiG Choice Questions (Based on the given Passage/Comprehens ion)

42. At the surface of earth, magnetic field intensity 44. A compass needle can point out in any direction
is of the order of 10" “* tesla. at magnetic poles, because horizontal component
43. As V=/esin5 = 0 of earth’s magnetic field is zero there.
sin 8 = 0, 8 = 0“. 45. Magnetic declination is 20® west.
MAGNETISM AND MATTER 5/115

Kor Difficult Questions

09 Matching Type Questions


46. Penneabillly has the dimensional formula has no dimensions and intensity of magnetisation
[MLT~^A~^]. The dimensional formula of has the dimensions [M^L~^T^A\ and same units
magnetic induction is Susceptibility Am~K

w
K9 Matrix-Match Type Questions
47. Knowledge based questions.. Magnetic induction is a vector, having the units
48. Magnetic moment is a vector and its units are N/A-m.

e
row
Nm-Vwb. Permeability is scalar and intensity of

re
magnetization is a vector.
VI. Integer Type Questions

FFllo
F
u
49. The arrangement of magnetic poles as shown in

ree
FIGURE 5<CF).20
Fig. 5(CF).l9 is equivalent to two magnets of

sFr
magnetic moments A/, =A^2= (wx«) inclined to
each other at 60®, Fig. 5(CF).19.

kro
uor
FIGURE 5(CF).19 offo
kos
YYo
eerBB
oo
rY
uu

51. Here,M^-2 Am-,- 0*3 m


ad
doo

-m
Ml = ma
A/t = 5 Am^, ^2 = O' 4ni
nY

The point of intersection P lies on axial line of


Net magnetic moment is the two magnets. Fig. 5(CF).9.
nid
Re

2M 2x2
M =
+A^| +2Af, Af2cos60' 0
X
I
= I0"‘^x
FFi

471 (J? (0-3)^


= , (ma)- +(ma)^ + 2(iiui)(ma)- = 1-48 X I0--‘^TalongS| N,
1 2
2x5
«2 = =10“^ x
M = ma -yfi = 1-732(;h«), 4tc
4 (0-4)
.3

which can be taken as 2 {ma) = 1-56X 10-5 T along


50. As is clear from Fig. 5(CF).20, As and B-, are perpendicular to each other,
coercivity OA = H = ni = 120 A/m. therefore resultant magnetic field at P is
numberof turns 72 B = r + B;
Here, « = = 60
length l-2(m)
= 7(1-48x10-5)2+(1.56x10-5)2
H 120
= 2A
= 2-15x 10-5t = /(X I0-5t
n 60 where n = 2-15, which can be taken as 2
5/116 ‘P'utdee^ Fundamental Physics (XII)

For Difficult Queations

VII. Assertion-Reason Type Questions

FOR MEDICAL STUDENTS Thus Assertion is wrong. Here Reason though


52. Both the assertion and reason are true and latter
correct but cannot explain the Assertion.
56. Here both Assertion and Reason are true and
is correct explanation of the former.
53. Both, the assertion and reason are true and the Reason is the correct explanation of Assertion
latter is correct explanation of the former. because when ferromagnetic material is healed
54. Both, the assertion and reason are false.
above curve temperature the domains in

ww
ferromagnetic material are broken and material
55. Magnetic moment of revolving electron around behaves as a paramagnetic material.
the nucleus is.
FOR ENGINEERING STUDENTS
eo) 2 _ evr
M = IA = — nr^ -

Flo
nr
T 27t 2 57. Here, both statement-1 and statement-2 are

e
correct and statement-2 is the correct

eree
1 explanation of statement-1.
But V OC
and r «*:
58. Statement-1 is true, but the statement-2 is false.

FFr
n

When the point lies on equatorial line of bar


uurr e{I/n)x/i2 magnet, magnetic field strength is parallel to

orr
Therefore, M OC or M OC n
2 North to South pole of magnet.
sfo
kks
Yoo
oooo
eBB
uurr
ad
Yo
dY
innd
Re
Fi
w
e
e
o
wr
r
eF
[LDIMDT=41

ullo
r FF
Bl,10rEOMAi^£TlC

rsre
oF
uo
k
IMDPCTION iWD
MjmiiiiiNG cuimiiTS oofr
sf
ko
BB
YYo
Y
oo
r

CHAPTER 6.
uur ee

ELECTROMAGNETIC INDUCTION
d
oo
ad

CHAPTER 7.
din
Y

ALTERNATING CURRENTS
Re
FF
in
ww
ELECTROMAGNETIC

Flo
e
INDUCTION

ree
Fr
rF
uurr
6.1. INTRODUCTION
s for
In unit 3. we have seen that moving charges produce magnetic field. The question arises, as to whether
moving magnets produce electric currents, i.e., does nature permit such a symmetrical relation between
kks
electricity and magnetism ? The answer is a resounding Yes ! The experiments of Michael Faraday in U.K.
Yo
oooo

and Joseph Henry in U.S.A. established around the year 1830 that electric currents were induced in closed
coils, when subjected to changing magnetic Helds.
eB

This phenomenon of generating current/ e.mf. in a conducting circuit by a change in strength,


position or orientation ofan external magneticfield is called ^^^*^^l>^if*ognetic Induction (KMl).
r

The e.m.f so developed is called induced e.mf. If the conductor is in the form of a closed
ou
ad

circuit, a current flows in the circuit. This is called induced current.


YY

This phenomenon of EMI is the basis of working of power generators, dynamos, transformers etc. We
nndd

cannot imagine a world where there is no electricity, no lights, no train.s, no telephones, no personal computers
Re

and nothing el.se that works on electricity. Therefore, the phenomenon of EMI is not merely of theoretical or
Fi

academic interest. The present day civilisation owes a great deal to the discovery of the phenomenon of
electromagnetic induction.

6.2. MAGNETIC FLUX

The magnetic flux 0 through any surface of area A held in a magnetic field B is measured by
the total number of magnetic lines of force crossing the surface normally. It is equal to dot
—^ —>

product of B and A .

For a uniform magnetic field B crossing the plane of area A at an angle 0 with the normal to the
plane, magnetic flux <t> is given by

6/1
6/2 ‘P>tAd€*^ 'a Fundamental Physics (XII) P/HWI

(}) = B. A = flA cos 0 ...(1)

where 0 is smaller angle between B and A , i.e., 0 is the angle,


which normal to the surface area makes with B . This is shown in

Fig. 6.1. Here « represents unit vector along the outdrawn normal to
the area element.

Magnetic Flux associated with variable magnetic field

If magnetic field B is variable, we divide the surface into a


number of area elements. Each area element is smallenough to have
constant magnetic field through it.

w
If 5,, — .., B^ be the magnetic fields at the area elements
●—> ^

Flo
A Aj, A A2, A A^ , AA^ respectively, then total magnetic flux
through the surface is

reeee
.AA,+B2-AA2 + fl„-AA^

FFr
all

If A A tends to zero, we can write the summation in the form of integral.

for
ur
( ) = jB.rfA
kkss
The integration is over the entire surface area.
Yo
When the magnetic field is touching the surface tangentially, Fig. 6.2,
oo

0 = 90°
eB

(}i = fiA cos 90“ = 0


r

FIGURE 6.2
FIGURE 6.3(8)
ou
ad

2
YY

B
t

- r n
B
ndd

¥
Re

>●

>●
Fi

>
0 = 0°;
9 = 90°: <t> = 0 <j) = max.

When the magnetic field is normal to the surface, Fig. 6.3(a), 0 = 0“,
(J) = 5A cos 0“ = BA = max. value.

This means B - — , i.e., magneticfield strength B is magneticflux per unit area and is called magnetic
A

flux density or magnetic Induction.


Thus denseness of magnetic lines of force is associated with the strength of magnetic field. For
example, in Fig. 6.3(B), we have shown a point source, which may be a charge or a magnetic pole. As many
as 16 field lines are shown to flow from this source. If Area 1 is taken as 1 sq. unit, Area 2 is 4 square
units.
ELECTROMAGNETIC INDUCTION 6/3

16
flux density at Area 2 ~ ^ field lines per square unit,

16
and flux density at Area 1 = — = 16 field lines per square unit.
1

w
Flo
ee
Fr
Therefore, magnetic field strength or magnetic flux density at Area 1 is 4 times the magnetic field
strength or magnetic flux density at Area 2.

for
ur
Units of Magnetic Flux

The SI unit of magnetic flux is weber (Wb). One weber is the amount of magnetic flux over an
ks
Yo
area of 1 square metre held normal to a uniform magnetic field of one tesla. Thus
oo

1 weber = 1 tesla x 1 m^
eB

The c.g.s. unit o/<j> is maxwell (A/r), where I weber = 10® maxwell.
It should be clearly understood that magnetic flux is a scalar quantity (represented by the dot product
r
ou
ad

of two vectors B and A ).


Y

The dimensional formula of ^ can be deduced from ({> = £A cos 6


from F= Bqv, B = F!q v
nd
Re
Fi

(j) = — A cos 0 = — = [ML^ .(2)


qv {AT){LT~^)
In this dimensional formula, A stands for ampere, SI unit of current.
As [ML^ T'^] corresponds to energy, SI unit of magnetic flux will be

weber =
joule _ joule joule-sec = volt-sec.
ampere coulomb/sec coulomb

fSampieTRroiJiernl What Is the magnetic flux linked with a coil of N turns of area of cross
section A held with its plane parallel to the field ? (Kerala PET 2012)

Sol. ^ = N BA cos 0
As plane of area of coil is parallel to the field, normal to the area makes an angle of 90® with the field.
Therefore, BA cos 90° = Zero
6/4 Fundamental Physics (XII)QSm
6.3. THE EXPERIMENTS OF FARADAY AND HENRY

The understanding of electromagnetic induction is based on a scries of experiments carried out by


Faraday and Henry. Some of the important experiments are described below :
Experiment 1. Current induced by a magnet
Fig. 6.4 shows a coil or loop C of a few turns of con- ducting
material insulated from one another. It is connected to a sensitive

galvanometer G. Faraday and Henry observed that


(i) When north pole of a bar magnet is pushed towards the
coil, the galvanometer shows a sudden deflection,
indicatingthat current is induced in the coil,
(ii) The galvanometerdeflection is temporary.It lasts as long

ww
as the bar magnet is in motion.
There is no deflection when the bar magnet is held

FF loo
stationary, anywhere, even within the coil,
(ill) When the magnet is moved away from the coil, the galvanometer shows deflection in the opposite

ree
direction, indicating reversal in the direction of induced current,
(iv) When south pole of bar magnet is moved towards or away from the coil, the galvanometer deflections

eF
tu’e opposite to those observed with the north pole for simiUu’ movements,

Frre
r ur
(v) The galvanometerdeflection(and hence induced current) is found to be larger when magnet is pushed
towards or pulled away from the coil faster, fforo
ks
(vi) When the bsu’ magnet is held stationiU'y and coil C is moved towards or away from the magnet, the
YYouo
same effects are observed. It shows that relative motio

You might also like